Page 1 :
M.K.G, CA EDUCATION, 9811429230 / 9212011367, WWW.MKGEDUCATION.COM, , INCOME TAX, (Volume – 1), , , , , , , , , , , , , , COMPUTATION OF TOTAL INCOME AND TAX LIABILITY, TAXABILITY OF GIFT, ADVANCE PAYMENT OF TAX, RESIDENTIAL STATUS & SCOPE OF TOTAL INCOME, INCOME UNDER THE HEAD HOUSE PROPERTY, DEDUCTION FROM GROSS TOTAL INCOME, AGRICULTURAL INCOME, CLUBBING OF INCOME, INCOME UNDER THE HEAD OTHER SOURCES, DEDUCTION OF TAX AT SOURCE, MISCELLANEOUS TOPICS, INSTRUCTIONS TO EXAMINEES, , 14-88, 89-103, 104-116, 117-158, 159-212, 213-267, 268-286, 287-312, 313-331, 332-373, 374-380, 381-386, , Including EXAMINATION QUESTIONS/RTP/MTP, After the book has been published, some error/mistake etc may be detected/or there, may be some amendments etc, all such corrections/amendments shall be uploaded at, end of the book given on our website. Students are requested to visit our website in, order to update the book, , 47th Edition, CA (INTER), , MAY-2022/NOV-2022, P.Y. 2021-22, A.Y. 2022-23, F.A. – 2021, , Author, This Book is the result of combined efforts of, Chartered Accountants/ company executives /, other professionals / feedback of our thousands of students, ₹750
Page 3 :
3, PAPER – 4: TAXATION, (One paper ─ Three hours –100 Marks), Objective: To develop an understanding of the provisions of income-tax law and goods and services tax law, and to acquire the ability to apply such knowledge to make computations and address basic application, oriented issues., SECTION A: INCOME TAX LAW (60 MARKS), Contents:, 1. Basic Concepts, (i) Income-tax law: An introduction, (ii) Important definitions in the Income-tax Act, 1961, (iii) Concept of previous year and assessment year, (iv) Basis of Charge and Rates of Tax, 2. Residential status and scope of total income, (i) Residential status, (ii) Scope of total income, 3. Incomes which do not form part of total income (other than charitable trusts and institutions,, political parties and electoral trusts), (i) Incomes not included in total income, (ii) Tax holiday for newly established units in Special Economic Zones, 4. Heads of income and the provisions governing computation of income under different heads, (i) Salaries, (ii) Income from house property, (iii) Profits and gains of business or profession, (iv) Capital gains, (v) Income from other sources, 5. Income of other persons included in assessee's total income, (i) Clubbing of income: An introduction, (ii) Transfer of income without transfer of assets, (iii) Income arising from revocable transfer of assets, (iv) Clubbing of income of income arising to spouse, minor child and son’s wife in certain cases, (v) Conversion of self-acquired property into property of HUF, 6. Aggregation of income; Set-off, or carry forward and set-off of losses, (i) Aggregation of income, (ii) Concept of set-off and carry forward and set-off of losses, (iii) Provisions governing set-off and carry forward and set-off of losses under different heads of income, (iv) Order of set-off of losses, 7. Deductions from gross total income, (i) General provisions, (ii) Deductions in respect of certain payments, (iii) Specific deductions in respect of certain income, (iv) Deductions in respect of other income, (v) Other deductions, 8. Computation of total income and tax liability of individuals, (i) Income to be considered while computing total income of individuals, (ii) Procedure for computation of total income and tax liability of individuals, 9. Advance tax, tax deduction at source and introduction to tax collection at source, (i) Introduction, (ii) Direct Payment, (iii) Provisions concerning deduction of tax at source, (iv) Advance payment of tax
Page 4 :
4, (v) Interest for defaults in payment of advance tax and deferment of advance tax, (vi) Tax collection at source – Basic concept, (vii) Tax deduction and collection account number, 10. Provisions for filing return of income and self-assessment, (i) Return of Income, (ii) Compulsory filing of return of income, (iii) Fee and Interest for default in furnishing return of income, (iv) Return of loss, (v) Provisions relating to belated return, revised return etc., (vi) Permanent account number, (vii) Persons authorized to verify return of income, (viii) Self-assessment, SECTION B – INDIRECT TAXES (40 MARKS), Contents:, 1. Concept of indirect taxes, (i) Concept and features of indirect taxes, (ii) Principal indirect taxes, 2. Goods and Services Tax (GST) Laws, (i) GST Laws: An introduction including Constitutional aspects, (ii) Levy and collection of CGST and IGST, a) Application of CGST/IGST law, b) Concept of supply including composite and mixed supplies, c) Charge of tax, d) Exemption from tax, e) Composition levy, (iii) Basic concepts of time and value of supply, (iv) Input tax credit, (v) Computation of GST liability, (vi) Registration, (vii) Tax invoice; Credit and Debit Notes; Electronic way bill, (viii) Returns, (ix) Payment of tax including reverse charge, Note – If any new legislation(s) is enacted in place of an existing legislation(s), the syllabus will accordingly, include the corresponding provisions of such new legislation(s) in place of the existing legislation(s) with, effect from the date to be notified by the Institute. Similarly, if any existing legislation ceases to have effect,, the syllabus will accordingly exclude such legislation with effect from the date to be notified by the Institute., Students shall not be examined with reference to any particular State GST Law., Consequential/corresponding amendments made in the provisions of the Income-tax law and Goods and, Services Tax laws covered in the syllabus of this paper which arise out of the amendments made in the, provisions not covered in the syllabus will not form part of the syllabus. Further, the specific, inclusions/exclusions in the various topics covered in the syllabus will be effected every year by way of, Study Guidelines.
Page 5 :
5, , WEIGHTAGE, Intermediate Course Paper 4: Taxation (100 Marks), Section A: Income-tax Law (60 Marks), I. (5%-10%), 1. Basic Concepts, (i) Income-tax law: An introduction, (ii) Important definitions in the Income-tax Act, 1961, (iii) Concept of previous year and assessment year, (iv) Basis of Charge and Rates of Tax, II. (10%-15%), 2. Residential status and scope of total income, (i) Residential status, (ii) Scope of total income, III. (25%-30%), 3. Incomes which do not form part of total income (other than charitable trusts and institutions,, political parties and electoral trusts), (i) Incomes not included in total income, (ii) Tax holiday for newly established units in Special Economic Zones, 4. Heads of income and the provisions governing computation of income under different heads, (i) Salaries, (ii) Income from house property, (iii) Profits and gains of business or profession, (iv) Capital gains, (v) Income from other sources, IV (15%-20%), 5. Income of other persons included in assessee's total income, (i) Clubbing of income: An introduction, (ii) Transfer of income without transfer of assets, (iii) Income arising from revocable transfer of assets, (iv) Clubbing of income of income arising to spouse, minor child and son’s wife in certain cases, (v) Conversion of self-acquired property into property of HUF, 6. Aggregation of income; Set-off, or carry forward and set-off of losses, (i) Aggregation of income, (ii) Concept of set-off and carry forward and set-off of losses, (iii) Provisions governing set-off and carry forward and set-off of losses under different heads of income, (iv) Order of set-off of losses, 7. Deductions from gross total income, (i) General provisions, (ii) Deductions in respect of certain payments, (iii) Specific deductions in respect of certain income, (iv) Deductions in respect of other incomes, (v) Other deductions, V. (20%-25%), 8. Computation of total income and tax liability of individuals, (i) Income to be considered while computing total income of individuals, (ii) Procedure for computation of total income and tax liability of individuals
Page 6 :
6, VI. (10%-15%), 9. Advance tax, tax deduction at source, (i) Introduction, (ii) Direct Payment, (iii) Provisions concerning deduction of tax at source, (iv) Advance payment of tax, (v) Interest for defaults in payment of advance tax and deferment of advance tax, 10 Introduction to tax collection at source, (i) Tax collection at source – Basic concept, (ii) Tax deduction and collection account number, 11. Provisions for filing return of income and self-assessment, (i) Return of Income, (ii) Compulsory filing of return of income, (iii) Fee and Interest for default in furnishing return of income, (iv) Return of loss, (v) Provisions relating to belated return, revised return etc., (vi) Permanent account number, (vii) Persons authorized to verify return of income, (viii) Self-assessment, Section B: Indirect Taxes (40 Marks), I (30%-50% ), 1. Levy and collection of CGST and IGST, (a) Application of CGST/IGST law, (b) Concept of supply including composite and mixed supplies, (c) Charge of tax (including reverse charge), (d) Exemption from tax, (e) Composition levy, 2. Basic concepts of time and value of supply, 3. Input tax credit, II (20%-30%), 1. Computation of GST liability, III (25%-40%), 1. Registration, 2. Tax invoice; Credit and Debit Notes; Electronic way bill, 3. Returns, 4. Payment of tax, IV (0 - 5%), 1. Concept of indirect taxes - Concept and features of indirect taxes; Principal indirect taxes, 2. GST Laws: An introduction including Constitutional aspects
Page 7 :
7, , ETI AGARWAL, ALL INDIA TOPPER OF CA-IPC (NOV-13), ROLL NO. - 366539, MARKS IN TAXATION:89%, , (HIGHEST MARKS IN TAXATION ALL OVER INDIA), (AGGREGATE MARKS 79.71%), , (FEEDBACK), A man for whom teaching is neither a business nor a profession, rather a passion for doing good,, great and unique in the field of teaching is none other than MK Gupta Sir., Sir"s unmatchable style of teaching coupled with his patience and calmness in dealing with students, is simply excellent., The structure of learning pattern, regular mock tests, motivational cash prizes and student friendly, study material covering practical illustrations, past year questions and bare act.. all contributed to, making this journey easy and building up the confidence needed for IPCC., Moreover, the vast knowledge and experience of the faculty assisted in making the concepts crystal, clear and handling each n every doubt of students., The administration and management stands second to none., MK GUPTA classes is a place which can change the word impossible 2 I M POSSIBLE. It made, me a better person both personally n professionally., I think 4 success 4 elements are necessary-desire, dedication, direction and discipline...and all the 4, i got from Sir.., THANK YOU so much Sir.., In the end i would just like to say MK GUPTA SIR NOT ONLY MAKES CA. HE MAKES, HUMANS!!, , ETI AGARWAL
Page 8 :
8, , AKSHAY JAIN, ALL INDIA TOPPER OF CA-IPC (NOV-13), ROLL NO.- 368162, MARKS IN TAXATION : 87%, (SECOND HIGHEST MARKS IN TAXATION ALL OVER INDIA), , (AGGREGATE MARKS 79.71%), , (FEEDBACK), Experience of those four months with M.K. GUPTA SIR was out of the world., As a teacher, M.K. GUPTA SIR is just like a sea of knowledge & you get each and everything from, very beginning to end from him., Sir is really a nice person. He is very motivational and his words of motivation can influence, anybody to work hard & make their parents proud., M.K. GUPTA CA EDUCATION is the only place where the provisions of tax laws are combined, with the practical knowledge. Study material provided is excellent and it contains numerous, problems covering all aspects and such type of problems are not available anywhere. Sir is not, giving any home work rather home work is done in the class itself and students are invited to solve, the problem before the entire class., Be honest towards your studies & Sir will show you the way of success. The way, Sir is making, students ready for the professional world is praiseworthy. Exposure given by sir to face interview of, Big four CA Firms is excellent., The test Series conducted by the Sir in all the subjects of IPC is very nice Scheme to score such, good marks and exam are conducted in the similar manner as it is conducted by ICAI., I would like to express my gratitude to Sir because it was only his efforts that helped me reach this, position., Sir its your Success., A Message to all : “COME & HAVE A TIME THAT YOU WILL CHERISH THROUGHOUT YOUR LIFE”., , AKSHAY JAIN
Page 9 :
9, , VIJENDER AGGARWAL, ALL INDIA TOPPER OF CA-IPCC (NOV-10), ROLL NO. - 174639, MARKS IN TAXATION:92%, , (HIGHEST MARKS IN TAXATION ALL OVER INDIA), (AGGREGATE MARKS 83.71%), , (FEEDBACK), A person who possesses such vast knowledge in the field of taxation, that we people can only dream, of, is none other than M. K. Gupta Sir., He possesses the rare ability to teach this procedural subject with utmost ease, enabling his students, to grasp all the provisions without any confusion., The quality of study material provided is such that a good study of it helped me score 92 marks. The, variety and complexity of practical problems covered in the books are not available anywhere else., One can find many places where taxation is being taught but it is hardly possible to find a better, place where tax laws are combined with their practical applicability to ensure that all concepts are, crystal clear., Sir is extremely generous. Money-making doesn’t appear to be his priority and it is clearly reflected, in his classes, where the infrastructure and administration stands second to none and students are, awarded handsome cash-prizes not only in classes but also in tests, which are regularly conducted., Thanking Sir for all what he has done would be an insult since it was only his efforts that helped me, reach this position. Sir, its your success. The relationship between us started in CPT only and, continued in IPCC and I hope it will continue forever., , VIJENDER AGGARWAL
Page 10 :
10, , PRACHI JAIN, ALL INDIA TOPPER OF CA-PCC (MAY-10), ROLL NO. - 66312, MARKS IN TAXATION:88%, , (HIGHEST MARKS IN TAXATION ALL OVER INDIA), (AGGREGATE MARKS 77.67%), , (FEEDBACK), M. K. Gupta Sir is an outstanding teacher. He is not only a good teacher but a good person by heart., His way of teaching is excellent. There are many provisions in tax but Sir repeats every provision, atleast two times. This helps in understanding those provisions easily., His books are very good. Everything from theory to PRACTICAL ILLUSTRATION,, EXAMINATION QUESTIONS and BARE ACT is covered in his books., Sir’s staff and management is also very good. Everything is handled in a systematic manner and on, time. Overall it was a good experience., Thanks Sir !! :-, , PRACHI JAIN
Page 11 :
11, , RESULTS, (CA-Intermediate), NO OTHER TEACHER OF TAXATION IN INDIA HAS BETTER RESULT THAN OURS, , OPINION OF OUR STUDENTS, 1. PRASHANT YADAV (Roll No.354233), 92 Marks, M.K. Gupta Sir is an outstanding teacher. He possesses very vast knowledge of taxation. Sir repeats every, concept atleast three times which makes all concepts crystal clear. Study material provided is very good, it, covers everything from illustration to examination problem and from theory to Bare Act. Staff and, infrastructure facilities of MKG Classes is incomparable. Thank you Sir for your love and support., , 2. MOHIT SHARMA (Roll No.353392), 89 Marks, A brilliant personality in my life who has motivated the student to a good path. He is very different from, others. Sir concentrates not only on the Marks but also on the overall development of the student., I am truly glad that I studied from Sir. He taught me how to compete in life. Every student get very good, marks with a little effort, if he is a student of M.K. Gupta CA Education., 3. MANISHA BHAMBRI (Roll No.456626), 89 Marks, M.K. Gupta Sir is the best teacher I have ever met. His study material being the best helped me a lot in my, exams. He is the most sincere teacher who never waste a single moment and gives his best towards his, profession. He teaches not only the theoretical portion but the practical approach too. He teaches us how to, be a good human being and how to live life happily. Thank you Sir for your support every time I needed., 4. ANISH SHRESTHA (Roll No.344028), 88 Marks, M.K. Gupta Sir is a very excellent teacher. The way he is dedicated towards teaching make us to be, dedicated towards our study. Every concept and every doubt of taxation whichever do I had, he has make, clear. The best thing about Sir is, he use to revise the concept more than 3 times which makes student very, easy for preparing their exam., You will have all the sufficient material for study and lots of questions with answers for practice a, systematically designed materials., Thank a lot to Sir for being so much helpful and lot of love., 5. KAPIL KHANNA (Roll No.341539), 85 Marks, Sir ‘M.K. Gupta’ is the best teacher for Taxation. I feel fortunate to be his student, the amount of knowledge, he imparts is fantastic and uncomparable. He is a person who burns himself up like a candle to light the path, of his dearest students to the road of success. I wish Sir teaches all the subjects of IPCC, since he is simply, the best. Thank you for everything Sir. It you and only you who can guide students like us to reach the, zenith., 6. JITENDRA (Roll No.337780), 85 Marks, Before joining CA, I was so much scary about the “Taxation” but after joining M.K. Gupta CA Education, for taking taxation class my scary converted into my strength now. This is just because of Sir’s knowledge, & teaching style with practicality. Study material provided by Sir is also awesome for study., 7. PUNEET WASAN (Roll No.368537), 84 Marks, M.K. Gupta Sir has a vast knowledge in the subject. The topics taken in the class are very well planned.
Page 12 :
12, I found the book really very good. Infact, I practiced all the previous attempts questions of each chapter and, every small question was covered in the chapter. I recommend the students to be thorough with book and, one will score undoubtedly high marks in tax. All the best!! Thank you so much Sir., 8. ISHA MALIK (Roll No.339842), 84 Marks, I do not have words to express my greatfulness for M.K. Gupta Sir. He really possesses vast knowledge and, rich experience in taxation. Study material provided by Sir is also very good which covers everything for, getting through the exam. There is no doubt that due to excellent coaching given by Sir, I have been able to, secure good marks. I pray to God for his long, happy and prospective life. I wish him to continue give, coaching to the prospective students for a longer period. I appeal to all the students who qualify CPT to take, coaching from Gupta Sir for getting sure success. Thank you Sir., 9. RUPAL GARG(Roll No.393844), 84 Marks, M.K. Gupta Sir is, as I believe, the best teacher for Taxation. He is so knowledgeable that I was totally awe, inspired by him. Every day in the class was exciting as he explains everything with real examples and full, depth. The books are superb with lots of practical questions. Thank you Sir., 10. AKANSHA GOEL (Roll No.336693), 84 Marks, It was a great experience studying from M.K. Gupta Sir. He has a vast pool of the knowledge of the subject., The book is a comprehensive one too., 11. PRASIT SHARMA (Roll No.344702), 84 Marks, Taking about the coaching, the teaching style of M.K. Gupta Sir is too much excellent. He has good dealing, with student in every situation. If anyone asked about the taxes coaching, I prefer M.K. Gupta Sir because he, is the one & only best teacher in Taxation., 12. MANSI BAJAJ (Roll No.354329), 83 Marks, Sir teaches so well and clarifies all our queries. He makes us understand the whole concept very clearly. He, is an amazing teacher and the best teacher in the field of Taxation., 13. RASHI GUPTA (Roll No.337864), 83 Marks, M.K. Gupta Sir is a very friendly and helping teacher. He always answered my queries well. His coaching, classes are very knowledgeable and books are also very good., 14. HARSH AGARWAL (Roll No.491097) 83 Marks, M.K. Gupta Sir is a good teacher. He teaches all aspects of Taxation whether it is practical knowledge or, theoretical knowledge. He teaches every point for 2-3 times and it gets learn in class only. His practical, knowledge about the subject is very good., 15. RAHUL ARORA (Roll No.337403), 83 Marks, M.K. Gupta Sir is a great mentor. Sir has excellent knowledge about the subject. He makes every concept, crystal clear. Every concept is explained atleast twice in the class. He connect every topic with practical life., Study material is excellent. Bare Act is covered in the study material. Three months experience with M.K., Gupta Sir is memorable moments of my life. Thank you Sir, for your guidance and encouragement., 16. SEJAL MEHTA (Roll No.353096), 83 Marks, Coaching for Taxation was an enriching experience in terms of the conceptual clarity which I gained on, each and every topic. Learning tax became so easy with the simplified notes provided., Also, the kind of knowledge that Sir shares with the students is very commendable and useful in, understanding the practical aspects of Taxation. Attending the coaching is worth the time spent.
Page 13 :
13, 17. ANU SETHI (Roll No.353491), 83 Marks, I have never seen teacher like M.K. Gupta Sir. His way to teaching, knowledge and experience is awesome, i.e. brilliant. Overall regards for such marks is only M.K. Gupta Sir., 18. ASHISH GUPTA (Roll No.353575), 82 Marks, M.K. Gupta Sir is a very good teacher and he has a very vast knowledge of taxation. He gives his best to, every student in a class. The atmosphere of the class when he was teaching in a class is very awesome. I am, giving all my credit to M.K. Gupta Sir for securing marks in Taxation., 19. RAGHAV GUPTA (Roll No.491122), 82 Marks, M.K. Gupta Sir is an outstanding teacher. He possesses a very vast knowledge about the subject. His way of, teaching is fabulous. Every concept is explained with help of an example. Study material is all exhaustive, that he provides. Also, queries are taken up promptly. Thank you Sir for your guidance., 20. SHREYA MALIK (Roll No.340228), 82 Marks, M.K. Gupta Sir is the best teacher I have ever come across. His level of knowledge is tremendous. The way, he teaches, with so much patience and willingness, keeps every student motivated. The marks I have scored, in tax is all because of him. Thank you so much Sir. I am a student of video class and I have never met Sir in, person. I would be grateful if I would be given a chance to meet him in person., 21. PRABHAW KUMAR AGARWALLA (Roll No.369428) 82 Marks, Teaching was excellent and queries handled were excellent. Teaching methodology was really excellent and, helped a lot to me., 22. PRABHAT RANJAN (Roll No.347926), 81 Marks, M.K. Gupta Sir has a very deep knowledge about the subject and his practical approach towards the subject., Sir repeats every provision atleast twice. This helps in understanding those provision easily., The books notes and all the management is done very properly and in a smooth manner. All in all the best, way to study tax., 23. ANKIT KHEMKA (Roll No.338055), 81 Marks, M.K. Gupta Sir is excellent teacher of Tax. He repeats the provision two to three times and doubts are also, taken by the faculty. His books are also very good. Bare Act is covered in his books for more understanding, about the Act. Sir also provide regular test and prize also given by him motivates the student to work hard., Environment provided by M.K. Gupta Classes is also very good to study., 24. ARTI SRIVASTAVA (Roll No.347859), 80 Marks, Sir’s unmatchable style of teaching. Regular mock test, also help in to achieve good marks in Taxation. Sir’s, books contain illustration. Past year question also help to achiever to good marks. Sir’s build confidence in, every student to achieve success in life. Thank you, so much Sir., 25. SHIVANGI GUPTA (Roll No.337956), 80 Marks, M.K. Gupta Sir is an amazing teacher. The tax subject is all about provisions so many sections but Sir makes, it simpler for us out of all the subjects, I found Taxation to be the most interesting one., Sir’s study material and notes are sufficient. Study material covers all the past year exam questions, practice, questions with solutions. His practical experiences help our understanding level to reach new heights. Thank, you Sir for everything.
Page 14 :
Computation of Total Income And Tax Liability, , 14, , COMPUTATION OF TOTAL INCOME, AND TAX LIABILITY, BASIS OF CHARGE AND RATES OF TAXES, SECTIONS PARTICULARS, 15 to 17, Salary, 22 to 27, House Property, 28 to 44DB, Business/Profession, 45 to 55A, Capital Gains, 56 to 59, Other Sources, 288A, Rounding off of Total Income, 288B, Rounding off of Tax, 87A, Rebate from Income Tax, 2(24)(ix), Meaning of Casual Income, 115BB, Rate of tax on Casual Income, 112, Long Term Capital Gains, 112A, Long Term Capital Gains when STT has been paid, 111A, Short Term Capital Gains when STT has been paid, 2(31), Meaning of Person, 10(1), Agricultural Income, Question 1: Write a note on Computation of Total Income., Answer: Computation of Total Income, If the income is taxable, it will be further divided into five different categories of income which are called, heads of income i.e. if the income is received from the employer, it will be considered to be income under, the head salary; if the income is in connection with letting out of house property, income is taxable under the, head house property; if the income is from any business or profession, it is taxable under the head profits, and gains of business/profession; if any capital asset (gold, land, house etc) has been transferred, income is, taxable under the head capital gains; if there is any other income like interest or winnings from a lottery etc,, it is covered under the head other sources., Income shall be computed under each head i.e. expenses incurred shall be deducted from the gross receipt as, per the provisions of the relevant head., Income computed under each head shall be added up to compute the gross total income., Certain concessions are allowed from the gross total income which are called deduction from gross total, income under section 80C to 80U., After permitting the deductions, remaining income is called total income., Computation of total income can be shown mathematically in the manner given below:, Total Income of an assessee shall be computed in the following steps:, Compute the income of the assessee under all the five heads, permitting exemption/deductions of each head., ₹, (i) Income from Salaries (Section 15 to 17), ……….., (ii) Income from House Property (Section 22 to 27), ……….., (iii) Profits and gains of Business or Profession (Section 28 to 44DB), ……….., (iv) Capital Gains (Section 45 to 55A), ……….., (v) Income from Other Sources (Section 56 to 59), ……….., Gross Total Income, ……….., _______, Deductions from gross total income [Section 80C to 80U], ……….., _______, Total Income, ……….., _______
Page 15 :
Computation of Total Income And Tax Liability, , 15, , Total Income shall be rounded off u/s 288A in the multiples of 10 and for this purpose, any paisa shall be, ignored and if the last digit is 5 or more, it will be rounded off to the higher multiple otherwise it will be, rounded off to the lower multiple., Example, (i) ₹6,28,456 shall be rounded off as 6,28,460, (ii) ₹6,28,455 shall be rounded off as 6,28,460, (iii) ₹6,28,454 shall be rounded off as 6,28,450, (iv) ₹6,28,455.99 shall be rounded off as 6,28,460, (v) ₹6,28,454.99 shall be rounded off as 6,28,450, Question 2: Write a note on Computation of Tax Liability of individual., Answer: Computation of Tax Liability, Tax liability of an individual shall be computed at the slab rates given in the relevant Finance Act i.e., Finance Act, 2021 and the rates are as given below:, • Resident individual of the age of 60 years or more at any time upto the end of relevant previous year, but less than eighty years (senior citizen), If total income is upto ₹3,00,000, NIL, On next ₹2,00,000, 5%, On next ₹5,00,000, 20%, On Balance amount, 30%, “Senior Citizen” means an individual resident in India who is of the age of 60 years or more at any, time during the relevant previous year. If date of birth is 1st April, 1962, 60 year will be completed on, 31-03-22 and assessee shall be senior citizen in previous year 2021-22., • Resident individual of the age of 80 years or more at any time upto the end of relevant previous year, (Very senior citizen), If total income is upto ₹5,00,000, NIL, On next ₹5,00,000, 20%, On Balance amount, 30%, “Very Senior Citizen” means an individual resident in India who is of the age of 80 years or more at, any time during the relevant previous year. If date of birth is 1st April, 1942, 80 year will be completed, on 31-03-22 and assessee shall be very senior citizen in previous year 2021-22., • Any other Individual, Income shall be taxable at the slab rates given below:, If total Income upto ₹2,50,000, NIL, On next ₹2,50,000, 5%, On next ₹5,00,000, 20%, On Balance amount, 30%, Example, (i) Mr. X has total income of ₹6,00,000, (ii) Mr. X has total income of ₹8,00,000, (iii) Mr. X has total income of ₹10,00,000, (iv) Mr. X has total income of ₹12,00,000, (v) Mr. X has total income of ₹20,00,000, Solution:, (i) Total income, On first ₹2,50,000, On next ₹2,50,000 @ 5%, On balance ₹1,00,000 @ 20%, Tax before health and education cess, Add: health & education cess @ 4%, Tax Liability, , 6,00,000, Nil, 12,500, 20,000, 32,500, 1,300, 33,800
Page 16 :
Computation of Total Income And Tax Liability, , 16, , (ii) Total income, On first ₹2,50,000, On next ₹2,50,000 @ 5%, On balance ₹3,00,000 @ 20%, Tax before health and education cess, Add: health & education cess @ 4%, Tax Liability, , 8,00,000, Nil, 12,500, 60,000, 72,500, 2,900, 75,400, , (iii) Total income, On first ₹2,50,000, On next ₹2,50,000 @ 5%, On balance ₹5,00,000 @ 20%, Tax before health and education cess, Add: health & education cess @ 4%, Tax Liability, , 10,00,000, Nil, 12,500, 1,00,000, 1,12,500, 4,500, 1,17,000, , (iv) Total income, On first ₹2,50,000, On next ₹2,50,000 @ 5%, On balance ₹5,00,000 @ 20%, On balance ₹2,00,000 @ 30%, Tax before health and education cess, Add: health & education cess @ 4%, Tax Liability, , 12,00,000, Nil, 12,500, 1,00,000, 60,000, 1,72,500, 6,900, 1,79,400, , (v) Total income, On first ₹2,50,000, On next ₹2,50,000 @ 5%, On balance ₹5,00,000 @ 20%, On balance ₹10,00,000 @ 30%, Tax before health and education cess, Add: health & education cess @ 4%, Tax Liability, , 20,00,000, Nil, 12,500, 1,00,000, 3,00,000, 4,12,500, 16,500, 4,29,000, , Question 3: Explain Health and Education Cess, Answer: Health and Education Cess, If any tax is charged for any specific purpose, it is called Cess. Health and Education Cess shall be charged, @ 4% on the amount of income tax., Rounding off of Tax Section 288B, Any amount payable, and the amount of refund due, shall be rounded off in the multiples of ₹10 in the, similar manner as in case of total income under section 288A., Question 4: Explain Previous Year and Assessment Year, Answer: Every person has to pay tax on Income of a particular financial year and such year is called, previous year. Further computation of income and tax liability is computed in the subsequent year and it is, called assessment year, eg. if income is to be computed for financial year 2021-22, it will be called previous, year and subsequent year i.e. 2022-23 shall be called assessment year. The term previous year is defined u/s, 3 and assessment year is defined u/s 2(9).
Page 17 :
Computation of Total Income And Tax Liability, , 17, , Question 5: Explain Budget / Finance Bill / Finance Act, Answer: Budget / Finance Bill / Finance Act, Every year budget is presented in general in February and all the amendments are given in general in the, budget e.g. Budget presented in 2021 shall be called budget 2021 and subsequently it will be called Finance, Bill 2021 and after it has been passed by the parliament and signed by the President, it will be called Finance, Act 2021 and its provisions shall be applicable from previous year 2021-22/ assessment year 2022-23., For the students appearing in May/Nov-2022, previous year shall be 2021-22 and assessment year shall be, 2022-23., Illustration 1:, Mr. X has income as given below:, Income under the head Salary, Income under the head House Property, Income under the head Business/Profession, Deductions allowed under section 80C to 80U are ₹1,10,000., Compute the income the tax liability for previous year 2021-22., , 4,00,000, 5,00,000, 6,30,253, , Solution:, Computation of Total Income of Mr. X, Previous Year 2021-22, Assessment Year 2022-23, Income under the head Salary, Income under the head House Property, Income under the Business/Profession, Gross Total Income, Less: Deduction u/s 80C to 80U, Total Income, Rounded off u/s 288A, Computation of Tax Liability, Total Income, First 2,50,000, On next 2,50,000 x 5%, On next 5,00,000 x 20%, On balance 4,20,250 x 30% Add: HEC @ 4%, Tax Liability, Rounded off u/s 288B, , ₹, 4,00,000.00, 5,00,000.00, 6,30,253.00, 15,30,253.00, (1,10,000.00), 14,20,253.00, 14,20,250.00, 14,20,250.00, Nil, 12,500.00, 1,00,000.00, 1,26,075.00, 2,38,575.00, 9,543.00, 2,48,118.00, 2,48,120.00, , (b) Presume he has completed age of 60 years as on 31.03.2022., Solution:, Computation of Total Income of Mr. X, Previous Year 2021-22, Assessment Year 2022-23, Income under the head Salary, Income under the head House Property, Income under the Business/Profession, Gross Total Income, Less: Deduction u/s 80C to 80U, Total Income, Rounded off u/s 288A, , ₹, 4,00,000.00, 5,00,000.00, 6,30,253.00, 15,30,253.00, (1,10,000.00), 14,20,253.00, 14,20,250.00
Page 18 :
Computation of Total Income And Tax Liability, Computation of Tax Liability, Total Income, First 3,00,000, On next 2,00,000 x 5%, On next 5,00,000 x 20%, On balance 4,20,250 x 30% Add: HEC @ 4%, Tax Liability, Rounded off u/s 288B, , 18, 14,20,250.00, Nil, 10,000.00, 1,00,000.00, 1,26,075.00, 2,36,075.00, 9,443.00, 2,45,518.00, 2,45,520.00, , (c) Presume he has completed age of 80 years as on 31.03.2022., Solution:, Computation of Total Income of Mr. X, Previous Year 2021-22, Assessment Year 2022-23, Income under the head Salary, Income under the head House Property, Income under the Business/Profession, Gross Total Income, Less: Deduction u/s 80C to 80U, Total Income, Rounded off u/s 288A, Computation of Tax Liability, Total Income, First 5,00,000, On next 5,00,000 x 20%, On balance 4,20,250 x 30% Add: HEC @ 4%, Tax Liability, Rounded off u/s 288B, Illustration 2:, Compute tax liability in the following cases for the assessment year 2022-23., (i), Mr. X (resident) has total income of ₹15,00,000, (ii), Mr. X (non-resident) has total income of ₹15,00,000, (iii) Mrs. X (resident) has total income of ₹15,00,000, (iv), Mrs. X (non-resident) has total income of ₹15,00,000, (v), Mr. X (resident), aged 60 years has total income of ₹15,00,000, (vi), Mrs. X (resident), aged 60 years has total income of ₹15,00,000, (vii) Mr. X (non-resident), aged 60 years has total income of ₹15,00,000, (viii) Mrs. X (non-resident), aged 60 years has total income of ₹15,00,000, (ix), Mr. X (resident), aged 80 years has total income of ₹15,00,000, (x), Mrs. X (resident), aged 80 years has total income of ₹15,00,000, (xi), Mr. X (non-resident), aged 80 years has total income of ₹15,00,000, (xii) Mrs. X (non-resident), aged 80 years has total income of ₹15,00,000, Solution:, (i) Computation of Tax Liability, Total Income, First 2,50,000, On next 2,50,000 x 5%, -, , ₹, 4,00,000.00, 5,00,000.00, 6,30,253.00, 15,30,253.00, (1,10,000.00), 14,20,253.00, 14,20,250.00, 14,20,250.00, Nil, 1,00,000.00, 1,26,075.00, 2,26,075.00, 9,043.00, 2,35,118.00, 2,35,120.00, , ₹, 15,00,000.00, Nil, 12,500.00
Page 19 :
Computation of Total Income And Tax Liability, On next 5,00,000 x 20% On balance 5,00,000 x 30%Tax before health and education cess, Add: HEC @ 4%, Tax Liability, (ii) Computation of Tax Liability, Total Income, First 2,50,000, On next 2,50,000 x 5%, On next 5,00,000 x 20% On balance 5,00,000 x 30%Tax before health and education cess, Add: HEC @ 4%, Tax Liability, (iii) Computation of Tax Liability, Total Income, First 2,50,000, On next 2,50,000 x 5%, On next 5,00,000 x 20% On balance 5,00,000 x 30%Tax before health and education cess, Add: HEC @ 4%, Tax Liability, (iv) Computation of Tax Liability, Total Income, First 2,50,000, On next 2,50,000 x 5%, On next 5,00,000 x 20% On balance 5,00,000 x 30%Tax before health and education cess, Add: HEC @ 4%, Tax Liability, (v) Computation of Tax Liability, Total Income, First 3,00,000, On next 2,00,000 x 5%, On next 5,00,000 x 20% On balance 5,00,000 x 30%Tax before health and education cess, Add: HEC @ 4%, Tax Liability, (vi) Computation of Tax Liability, Total Income, First 3,00,000, On next 2,00,000 x 5%, On next 5,00,000 x 20% On balance 5,00,000 x 30%Tax before health and education cess, Add: HEC @ 4%, Tax Liability, (vii) Computation of Tax Liability, Total Income, , 19, 1,00,000.00, 1,50,000.00, 2,62,500.00, 10,500.00, 2,73,000.00, 15,00,000.00, Nil, 12,500.00, 1,00,000.00, 1,50,000.00, 2,62,500.00, 10,500.00, 2,73,000.00, 15,00,000.00, Nil, 12,500.00, 1,00,000.00, 1,50,000.00, 2,62,500.00, 10,500.00, 2,73,000.00, 15,00,000.00, Nil, 12,500.00, 1,00,000.00, 1,50,000.00, 2,62,500.00, 10,500.00, 2,73,000.00, 15,00,000.00, Nil, 10,000.00, 1,00,000.00, 1,50,000.00, 2,60,000.00, 10,400.00, 2,70,400.00, 15,00,000.00, Nil, 10,000.00, 1,00,000.00, 1,50,000.00, 2,60,000.00, 10,400.00, 2,70,400.00, 15,00,000.00
Page 20 :
Computation of Total Income And Tax Liability, First 2,50,000, On next 2,50,000 x 5%, On next 5,00,000 x 20% On balance 5,00,000 x 30%Tax before health and education cess, Add: HEC @ 4%, Tax Liability, (viii) Computation of Tax Liability, Total Income, First 2,50,000, On next 2,50,000 x 5%, On next 5,00,000 x 20% On balance 5,00,000 x 30%Tax before health and education cess, Add: HEC @ 4%, Tax Liability, (ix) Computation of Tax Liability, Total Income, First 5,00,000, On next 5,00,000 x 20% On balance 5,00,000 x 30%Tax before health and education cess, Add: HEC @ 4%, Tax Liability, (x) Computation of Tax Liability, Total Income, First 5,00,000, On next 5,00,000 x 20% On balance 5,00,000 x 30%Tax before health and education cess, Add: HEC @ 4%, Tax Liability, (xi) Computation of Tax Liability, Total Income, First 2,50,000, On next 2,50,000 x 5%, On next 5,00,000 x 20% On balance 5,00,000 x 30%Tax before health and education cess, Add: HEC @ 4%, Tax Liability, (xii) Computation of Tax Liability, Total Income, First 2,50,000, On next 2,50,000 x 5%, On next 5,00,000 x 20% On balance 5,00,000 x 30%Tax before health and education cess, Add: HEC @ 4%, Tax Liability, , 20, Nil, 12,500.00, 1,00,000.00, 1,50,000.00, 2,62,500.00, 10,500.00, 2,73,000.00, 15,00,000.00, Nil, 12,500.00, 1,00,000.00, 1,50,000.00, 2,62,500.00, 10,500.00, 2,73,000.00, 15,00,000.00, Nil, 1,00,000.00, 1,50,000.00, 2,50,000.00, 10,000.00, 2,60,000.00, 15,00,000.00, Nil, 1,00,000.00, 1,50,000.00, 2,50,000.00, 10,000.00, 2,60,000.00, 15,00,000.00, Nil, 12,500.00, 1,00,000.00, 1,50,000.00, 2,62,500.00, 10,500.00, 2,73,000.00, 15,00,000.00, Nil, 12,500.00, 1,00,000.00, 1,50,000.00, 2,62,500.00, 10,500.00, 2,73,000.00
Page 21 :
Computation of Total Income And Tax Liability, , 21, , Question 6: Explain surcharge in case of individual., Answer: Surcharge shall be applicable, - @ 10% provided total income is exceeding ₹ 50 lakhs but it is upto ₹ 100 lakhs., - @ 15% provided total income is exceeding ₹ 100 lakhs but it is upto ₹ 200 lakh., - @ 25% provided total income is exceeding ₹ 200 lakhs but it is upto ₹ 500 lakh., - @ 37% provided total income is exceeding ₹ 500 Lakhs., Health & education cess shall be charged on the total of tax plus surcharge., e.g. Mr. X has total income of ₹550,00,000, in this case his tax liability shall be, Total Income, Tax at slab rate, Add: Surcharge @ 37%, Tax before health and education cess, Add: HEC @ 4%, Tax Liability, , 550,00,000.00, 1,63,12,500.00, 60,35,625.00, 2,23,48,125.00, 8,93,925.00, 2,32,42,050.00, , e.g. Mr. X has total income of ₹220,00,000, in this case his tax liability shall be, Total Income, Tax at slab rate, Add: Surcharge @ 25%, Tax before health and education cess, Add: HEC @ 4%, Tax Liability, , 220,00,000.00, 64,12,500.00, 16,03,125.00, 80,15,625.00, 3,20,625.00, 83,36,250.00, , e.g. Mr. X has total income of ₹110,00,000, in this case his tax liability shall be, Total Income, Tax at slab rate, Add: Surcharge @ 15%, Tax before health and education cess, Add: HEC @ 4%, Tax Liability, , 110,00,000.00, 31,12,500.00, 4,66,875.00, 35,79,375.00, 1,43,175.00, 37,22,550.00, , If Mr. X has total income of ₹100,00,000, his tax liability shall be, Total Income, Tax at slab rate, Add: Surcharge @ 10%, Tax before health and education cess, Add: HEC @ 4%, Tax Liability, , 100,00,000.00, 28,12,500.00, 2,81,250.00, 30,93,750.00, 1,23,750.00, 32,17,500.00, , If Mr. X has total income of ₹50,00,000, his tax liability shall be, Total Income, Tax at slab rate, Add: HEC @ 4%, Tax Liability, , 50,00,000.00, 13,12,500.00, 52,500.00, 13,65,000.00, , Marginal Relief, If there is marginal increase in income over ₹50 lakhs/ ₹100 lakhs/ ₹200 lakhs/ ₹500 lakhs, surcharge is, applicable on entire amount of income tax and as a result increase in tax is more than the increase in income., In order to remove this defect, assessee shall be allowed relief to the extent increase in tax is more than the, increase in income and it is called marginal relief and it can be shown in the manner given below:, e.g. If Mr. X has total income of ₹51,00,000, his tax liability shall be computed in the manner given below:, Total Income, 51,00,000.00
Page 22 :
Computation of Total Income And Tax Liability, , 22, , Tax on ₹51,00,000 at slab rate, 13,42,500.00, Add: Surcharge @ 10%, 1,34,250.00, Tax before marginal relief, 14,76,750.00, Less: Marginal Relief, (64,250.00), Working Note:, Tax + surcharge @10% on income of ₹51,00,000, 14,76,750, Tax on income of ₹50,00,000, (13,12,500), Increase in tax, 1,64,250, Increase in income, 1,00,000, Marginal Relief (1,64,250 – 1,00,000), 64,250, Tax after marginal relief, 14,12,500.00, Add: HEC @ 4%, 56,500.00, Tax Liability, 14,69,000.00, e.g. If Mr. X has total income of ₹102,00,000, his tax liability shall be computed in the manner given below:, Total Income, 102,00,000.00, Tax on ₹102,00,000 at slab rate, 28,72,500.00, Add: Surcharge @ 15%, 4,30,875.00, Tax before marginal relief, 33,03,375.00, Less: Marginal Relief, (9,625.00), Working Note:, Tax + surcharge @15% on income of ₹102,00,000, 33,03,375, Tax + surcharge @10% on income of ₹100,00,000 (30,93,750), Increase in tax, 2,09,625, Increase in income, 2,00,000, Marginal Relief (2,09,625 – 2,00,000), 9,625, Tax after marginal relief, 32,93,750.00, Add: HEC @ 4%, 1,31,750.00, Tax Liability, 34,25,500.00, e.g. If Mr. X has total income of ₹101,80,000, his tax liability shall be computed in the manner given below:, Total Income, 101,80,000.00, Tax on ₹101,80,000 at slab rate, 28,66,500.00, Add: Surcharge @ 15%, 4,29,975.00, Tax before marginal relief, 32,96,475.00, Less: Marginal Relief, (22,725.00), Working Note:, Tax + surcharge @15% on income of ₹101,80,000, 32,96,475, Tax + surcharge @10% on income of ₹100,00,000 (30,93,750), Increase in tax, 2,02,725, Increase in income, 1,80,000, Marginal Relief (2,02,725 – 1,80,000), 22,725, Tax after marginal relief, 32,73,750.00, Add: HEC @ 4%, 1,30,950.00, Tax Liability, 34,04,700.00, A person having total income from ₹50 lakhs to ₹100 lakhs shall be eligible for marginal relief upto total, income of ₹51,95,890 and afterwards he will not be eligible for marginal relief when slab of ₹2,50,000 is, applicable. (for slab rate of ₹3,00,000, such income shall be ₹51,95,520 and for slab of ₹5,00,000, such, income shall be ₹51,94,030), A person having total income from ₹100 lakhs to ₹200 lakhs shall be eligible for marginal relief upto total, income of ₹102,14,690 and afterwards he will not be eligible for marginal relief when slab of ₹2,50,000 is, applicable. (for slab rate of ₹3,00,000, such income shall be ₹102,14,500 and for slab of ₹5,00,000, such, income shall be ₹102,13,740)
Page 23 :
Computation of Total Income And Tax Liability, , 23, , A person having total income from ₹200 lakhs to ₹500 lakhs shall be eligible for marginal relief upto total, income of ₹209,30,000 and afterwards he will not be eligible for marginal relief when slab of ₹2,50,000 is, applicable. (for slab rate of ₹3,00,000, such income shall be ₹209,29,600 and for slab of ₹5,00,000, such, income shall be ₹209,28,000), A person having total income above ₹500 lakhs shall be eligible for marginal relief upto total income of, ₹530,17,830 and afterwards he will not be eligible for marginal relief when slab of ₹2,50,000 is, applicable. (for slab rate of ₹3,00,000, such income shall be ₹530,17,320 and for slab of ₹5,00,000, such, income shall be ₹530,15,280), Example 1: Mr. X has income as given below:, Income under the head PGBP, LTCG 112A, STCG 111A, Compute his tax liability for A.Y. 2022-23., Solution:, Computation of Total Income and Tax Liability of Mr. X, Income under the head PGBP, LTCG 112A, STCG 111A, Gross Total Income, Less: Deductions u/s 80C to 80U, Total Income, Computation of Tax Liability, Tax on LTCG 100,00,000 (101,00,000-1,00,000) @ 10% u/s 112A, Tax on STCG 100,00,000 @ 15% u/s 111A, Tax on 5,05,00,000 at slab rate, Add: Surcharge on PGBP Income (1,49,62,500) X 37%, Add: Surcharge @ 15% on 25,00,000, Less: Marginal Relief, Working Note:, Tax + surcharge @ 37% on income of ₹505,00,000 204,98,625, Tax + surcharge @25% on income of ₹500,00,000 (185,15,625), Increase in tax, 19,83,000, Increase in income, 5,00,000, Marginal Relief (19,83,000 – 5,00,000), 14,83,000, Tax Before cess, Add: Health and education cess @ 4%, Tax Liability, Example 2: Mr. X has income as given below:, Income under the head PGBP, Dividend Income, Compute his tax liability for A.Y. 2022-23., Solution:, Computation of Total Income and Tax Liability of Mr. X, Income under the head PGBP, Dividend Income, Gross Total Income, Less: Deductions u/s 80C to 80U, Total Income, , ₹505,00,000, ₹101,00,000, ₹100,00,000, , ₹, 505,00,000, 101,00,000, 100,00,000, 706,00,000, Nil, 706,00,000, 10,00,000.00, 15,00,000.00, 1,49,62,500.00, 55,36,125.00, 3,75,000.00, (14,83,000), , 218,90,625.00, 8,75,625.00, 227,66,250.00, ₹202,00,000, ₹100,00,000, , ₹, 202,00,000, 100,00,000, 302,00,000, Nil, 302,00,000
Page 24 :
Computation of Total Income And Tax Liability, Option 1: Taking Dividend tax at slab rate for surcharge, Computation of Tax Liability, Tax on Business Income 2,02,00,000 at 30%, Tax on Dividend Income 1,00,00,000 at slab rate, Tax Before Surcharge, Add: Surcharge on Dividend Income (28,12,500) X 15%, Add: Surcharge on PGBP Income (60,60,000) X 25%, Less: Marginal Relief, Working Note:, Tax + surcharge @ 25% on income of ₹202,00,000 75,75,000, Tax + surcharge @15% on income of ₹200,00,000 (69,00,000), Increase in tax, 6,75,000, Increase in income, 2,00,000, Marginal Relief (6,75,000 – 2,00,000), 4,75,000, Tax Before cess, Add: Health and education cess @ 4%, Tax Liability, Option 2: Taking Dividend tax @ 30% for surcharge and PGBP income on slab rate, Computation of Tax Liability, Tax on Business Income 2,02,00,000 at slab rate, Tax on Dividend Income 1,00,00,000 at 30%, Tax Before Surcharge, Add: Surcharge on Dividend Income (30,00,000) X 15%, Add: Surcharge on PGBP Income at slab (58,72,500) X 25%, Less: Marginal Relief, Working Note:, Tax + surcharge @ 25% on income of ₹202,00,000 73,40,625, Tax + surcharge @15% on income of ₹200,00,000 (66,84,375), Increase in tax, 6,56,250, Increase in income, 2,00,000, Marginal Relief (6,56,250 – 2,00,000), 4,56,250, Tax Before cess, Add: Health and education cess @ 4%, Tax Liability, Option 3: Taking Dividend tax on average basis for surcharge, Computation of Tax Liability, Tax on 3,02,00,000 at slab rate, Add: Surcharge on Dividend Income, (88,72,500/3,02,00,000 x 1,00,00,000 = 29,37,914) X 15%, Add: Surcharge on PGBP Income, (88,72,500/3,02,00,000 x 2,02,00,000 = 59,34,586) X 25%, Less: Marginal Relief, Working Note:, Tax + surcharge @ 25% on income of ₹202,00,000 74,18,233, Tax + surcharge @15% on income of ₹200,00,000 (67,57,202), (88,72,500/302,00,000 x 2,00,00,000) x 115%, Increase in tax, 6,61,031, Increase in income, 2,00,000, Marginal Relief (6,61,031 – 2,00,000), 4,41,031, Tax Before cess, , 24, , 60,60,000.00, 28,12,500.00, 88,72,500.00, 4,21,875.00, 15,15,000.00, (4,75,000.00), , 103,34,375.00, 4,13,375.00, 107,47,750.00, , 58,72,500.00, 30,00,000.00, 88,72,500.00, 4,50,000.00, 14,68,125.00, (4,56,250.00), , 103,34,375.00, 4,13,375.00, 107,47,750.00, , 88,72,500.00, 4,40,687.10, 14,83,646.52, (4,41,031.00), , 103,55,802.62
Page 25 :
Computation of Total Income And Tax Liability, Add: Health and education cess @ 4%, Tax Liability, Rounded off u/s 288B, , 25, 4,14,232.10, 107,70,034.72, 107,70,030.00, , Illustration 3:, Compute tax liability in the following cases for the assessment year 2022-23., (i), Mr. X (resident) has total income of ₹50,05,000, (ii) Mr. X (non-resident) has total income of ₹52,00,000, (iii) Mrs. X (resident) has total income of ₹101,00,000, (iv) Mrs. X (non-resident) has total income of ₹85,00,000, (v) Mr. X (resident), aged 60 years has total income of ₹106,00,000, (vi) Mrs. X (resident), aged 60 years has total income of ₹57,00,000, (vii) Mr. X (non-resident), aged 60 years has total income of ₹108,00,000, (viii) Mrs. X (non-resident), aged 60 years has total income of ₹101,50,000, (ix) Mr. X (resident), aged 80 years has total income of ₹54,25,000, (x) Mrs. X (resident), aged 80 years has total income of ₹102,00,000, (xi) Mr. X (non-resident), aged 80 years has total income of ₹55,22,380, (xii) Mrs. X (non-resident), aged 80 years has total income of ₹45,00,000, (xiii) Mr. X (resident) has total income of ₹201,00,000, (xiv) Mr. X (resident) has total income of ₹205,00,000, (xv) Mr. X (resident) has total income of ₹501,00,000, (xvi) Mr. X (resident) has total income of ₹505,00,000, (xvii) Mr. X (non - resident) has total income of ₹300,00,000, Solution:, (i) Computation of Tax Liability, Total Income, Tax on ₹50,05,000 at slab rate, Add: Surcharge @ 10%, Tax before marginal relief, Less: Marginal Relief, Working Note:, Tax + surcharge on income of ₹50,05,000, Tax on income of ₹50,00,000, Increase in tax, Increase in income, Marginal Relief (1,32,900 – 5,000), Tax after marginal relief, Add: HEC @ 4%, Tax Liability, (ii) Computation of Tax Liability, Total Income, Tax on ₹52,00,000 at slab rate, Add: Surcharge @ 10%, Tax before health & education cess, Add: HEC @ 4%, Tax Liability, (iii) Computation of Tax Liability, Total Income, Tax on ₹101,00,000 at slab rate, Add: Surcharge @ 15%, Tax before marginal relief, , ₹, 50,05,000, 13,14,000, 1,31,400, 14,45,400, (1,27,900), 14,45,400, (13,12,500), 1,32,900, 5,000, 1,27,900, 13,17,500, 52,700, 13,70,200, 52,00,000.00, 13,72,500.00, 1,37,250.00, 15,09,750.00, 60,390.00, 15,70,140.00, 101,00,000.00, 28,42,500.00, 4,26,375.00, 32,68,875.00
Page 26 :
Computation of Total Income And Tax Liability, Less: Marginal Relief, Working Note:, Tax + surcharge @15% on income of ₹101,00,000, Tax + surcharge @10% on income of ₹100,00,000, Increase in tax, Increase in income, Marginal Relief (1,75,125 – 1,00,000), Tax after marginal relief, Add: HEC @ 4%, Tax Liability, , 26, (75,125.00), , 32,68,875, (30,93,750), 1,75,125, 1,00,000, 75,125, 31,93,750.00, 1,27,750.00, 33,21,500.00, , (iv) Computation of Tax Liability, Total Income, Tax on ₹85,00,000 at slab rate, Add: Surcharge @ 10%, Tax before health & education cess, Add: HEC @ 4%, Tax Liability, , 85,00,000.00, 23,62,500.00, 2,36,250.00, 25,98,750.00, 1,03,950.00, 27,02,700.00, , (v) Computation of Tax Liability, Total Income, Tax on ₹106,00,000 at slab rate, Add: Surcharge @ 15%, Tax before health & education cess, Add: HEC @ 4%, Tax Liability, , 106,00,000, 29,90,000, 4,48,500, 34,38,500.00, 1,37,540.00, 35,76,040.00, , (vi) Computation of Tax Liability, Total Income, Tax on ₹57,00,000 at slab rate, Add: Surcharge @ 10%, Tax before health & education cess, Add: HEC @ 4%, Tax Liability, , 57,00,000, 15,20,000, 1,52,000, 16,72,000.00, 66,880.00, 17,38,880.00, , (vii) Computation of Tax Liability, Total Income, Tax on ₹108,00,000 at slab rate, Add: Surcharge @ 15%, Tax before health & education cess, Add: HEC @ 4%, Tax Liability, , 108,00,000.00, 30,52,500.00, 4,57,875.00, 35,10,375.00, 1,40,415.00, 36,50,790.00, , (viii) Computation of Tax Liability, Total Income, Tax on ₹101,50,000 at slab rate, Add: Surcharge @ 15%, Tax before marginal relief, Less: Marginal Relief, Working Note:, Tax + surcharge @15% on income of ₹101,50,000, Tax + surcharge @10% on income of ₹100,00,000, Increase in tax, , 101,50,000.00, 28,57,500.00, 4,28,625.00, 32,86,125.00, (42,375.00), 32,86,125, (30,93,750), 1,92,375
Page 27 :
Computation of Total Income And Tax Liability, Increase in income, Marginal Relief (1,92,375 – 1,50,000), Tax after marginal relief, Add: HEC @ 4%, Tax Liability, , 1,50,000, 42,375, 32,43,750.00, 1,29,750.00, 33,73,500.00, , (ix) Computation of Tax Liability, Total Income, Tax on ₹54,25,000 at slab rate, Add: Surcharge @ 10%, Tax before health & education cess, Add: HEC @ 4%, Tax Liability, (x) Computation of Tax Liability, Total Income, Tax on ₹102,00,000 at slab rate, Add: Surcharge @ 15%, Tax before marginal relief, Less: Marginal Relief, Working Note:, Tax + surcharge @15% on income of ₹102,00,000, Tax + surcharge @10% on income of ₹100,00,000, Increase in tax, Increase in income, Marginal Relief (2,09,000 – 2,00,000), Tax after marginal relief, Add: HEC @ 4%, Tax Liability, , 27, , 54,25,000.00, 14,27,500.00, 1,42,750.00, 15,70,250.00, 62,810.00, 16,33,060.00, 102,00,000, 28,60,000, 4,29,000, 32,89,000, (9,000), 32,89,000, (30,80,000), 2,09,000, 2,00,000, 9,000, 32,80,000, 1,31,200, 34,11,200, , (xi) Computation of Tax Liability, Total Income, Tax on ₹55,22,380 at slab rate, Add: Surcharge @ 10%, Tax before health & education cess, Add: HEC @ 4%, Tax Liability, Rounded off u/s 288B, , 55,22,380.00, 14,69,214.00, 1,46,921.40, 16,16,135.40, 64,645.42, 16,80,780.82, 16,80,780.00, , (xii) Computation of Tax Liability, Total Income, Tax on ₹45,00,000 at slab rate, Add: Surcharge, Tax before health & education cess, Add: HEC @ 4%, Tax Liability, , 45,00,000.00, 11,62,500.00, Nil, 11,62,500.00, 46,500.00, 12,09,000.00, , (xiii) Computation of Tax Liability, Total Income, Tax on ₹201,00,000 at slab rate, Add: Surcharge @ 25%, Tax before marginal relief, Less: Marginal Relief, , 201,00,000, 58,42,500, 14,60,625, 73,03,125, (5,18,750)
Page 28 :
Computation of Total Income And Tax Liability, Working Note:, Tax + surcharge @25% on income of ₹201,00,000, Tax + surcharge @15% on income of ₹200,00,000, Increase in tax, Increase in income, Marginal Relief (6,18,750 – 1,00,000), Tax after marginal relief, Add: HEC @ 4%, Tax Liability, (xiv) Computation of Tax Liability, Total Income, Tax on ₹205,00,000 at slab rate, Add: Surcharge @ 25%, Tax before marginal relief, Less: Marginal Relief, Working Note:, Tax + surcharge @25% on income of ₹205,00,000, Tax + surcharge @15% on income of ₹200,00,000, Increase in tax, Increase in income, Marginal Relief (7,68,750 – 5,00,000), Tax after marginal relief, Add: HEC @ 4%, Tax Liability, , 28, , 73,03,125, (66,84,375), 6,18,750, 1,00,000, 5,18,750, 67,84,375, 2,71,375, 70,55,750, 205,00,000, 59,62,500, 14,90,625, 74,53,125, (2,68,750), 74,53,125, (66,84,375), 7,68,750, 5,00,000, 2,68,750, , (xv) Computation of Tax Liability, Total Income, Tax on ₹501,00,000 at slab rate, Add: Surcharge @ 37%, Tax before marginal relief, Less: Marginal Relief, Working Note:, Tax + surcharge @37% on income of ₹501,00,000 203,34,225, Tax + surcharge @25% on income of ₹500,00,000 (1,85,15,625), Increase in tax, 18,18,600, Increase in income, 1,00,000, Marginal Relief (18,18,600 – 1,00,000), 17,18,600, Tax after marginal relief, Add: HEC @ 4%, Tax Liability, (xvi) Computation of Tax Liability, Total Income, Tax on ₹505,00,000 at slab rate, Add: Surcharge @ 37%, Tax before marginal relief, Less: Marginal Relief, Working Note:, Tax + surcharge @37% on income of ₹505,00,000 2,04,98,625, Tax + surcharge @25% on income of ₹500,00,000 (1,85,15,625), Increase in tax, 19,83,000, Increase in income, 5,00,000, , 71,84,375, 2,87,375, 74,71,750, 501,00,000, 1,48,42,500, 54,91,725, 2,03,34,225, (17,18,600), , 1,86,15,625, 7,44,625, 1,93,60,250, 505,00,000, 1,49,62,500, 55,36,125, 2,04,98,625, (14,83,000)
Page 29 :
Computation of Total Income And Tax Liability, Marginal Relief (19,83,000 – 5,00,000), Tax after marginal relief, Add: HEC @ 4%, Tax Liability, , 29, , 14,83,000, , (xvii) Computation of Tax Liability, Total Income, Tax on ₹300,00,000 at slab rate, Add: Surcharge @ 25%, Tax before HEC, Add: HEC @ 4%, Tax Liability, , 1,90,15,625, 7,60,625, 1,97,76,250, 300,00,000.00, 88,12,500.00, 22,03,125.00, 110,15,625.00, 4,40,625.00, 114,56,250.00, , Question 7: Write a note on Rebate under section 87A., Answer: Rebate in case of Resident Individual Section 87A, Rebate i.e. concession from income tax shall be allowed only to RESIDENT INDIVIDUAL (not to, non-resident individual or any other person)., Rebate shall be allowed only if total income is not exceeding ₹5,00,000, Rebate shall be allowed upto ₹12,500., Health & education cess shall be applied only after permitting rebate under section 87A., Rebate shall be allowed even from tax on LTCG or STCG under section 111A or Casual Income., No Rebate shall be allowed from LTCG u/s 112A., E.g., Mr. X has total income of ₹5,00,000, his tax liability shall be, Computation of Tax Liability, Total Income, 5,00,000.00, Tax on ₹5,00,000 at slab rate, 12,500.00, Less: Rebate u/s 87A, (12,500.00), Tax before health & education cess, Nil, Add: HEC @ 4%, Nil, Tax Liability, Nil, If in the above case total income is ₹4,96,000, tax liability shall be, Computation of Tax Liability, Total Income, Tax on ₹4,96,000 at slab rate, Less: Rebate u/s 87A, Tax before health & education cess, Add: HEC @ 4%, Tax Liability, , 4,96,000, 12,300.00, (12,300.00), Nil, Nil, Nil, , If in the above case total income is ₹5,01,000, tax liability shall be, Computation of Tax Liability, Total Income, Tax on ₹5,01,000 at slab rate, Less: Rebate u/s 87A, Tax before health & education cess, Add: HEC @ 4%, Tax Liability, Rounded off u/s 288B, , 5,01,000.00, 12,700.00, Nil, 12,700.00, 508.00, 13,208.00, 13,210.00, , Illustration 4:, Mr. X has gross total income ₹5,60,000 and deduction allowed under section 80C to 80U are ₹60,000., Compute his tax liability previous year 2021-22, assessment year 2022-23.
Page 30 :
Computation of Total Income And Tax Liability, , 30, , Solution:, Computation of Tax Liability, Gross Total Income, Less: Deduction u/s 80C to 80U, Total Income, Tax on ₹5,00,000 at slab rate, Less: Rebate u/s 87A, Tax before health & education cess, Add: HEC @ 4%, Tax Liability, , 5,60,000, (60,000), 5,00,000, 12,500, (12,500), Nil, Nil, Nil, , (b) Presume he is a resident and is aged 62 years., Solution:, Computation of Tax Liability, Gross Total Income, Less: Deduction u/s 80C to 80U, Total Income, Tax on ₹5,00,000 at slab rate, Less: Rebate u/s 87A, Tax before health & education cess, Add: HEC @ 4%, Tax Liability, , 5,60,000, (60,000), 5,00,000, 10,000, (10,000), Nil, Nil, Nil, , (c) Presume he is a non-resident and is aged 62 years., Solution:, Computation of Tax Liability, Gross Total Income, Less: Deduction u/s 80C to 80U, Total Income, Tax on ₹5,00,000 at slab rate, Less: Rebate u/s 87A, Tax before health & education cess, Add: HEC @ 4%, Tax Liability, , 5,60,000, (60,000), 5,00,000, 12,500, Nil, 12,500, 500, 13,000, , (d) Presume he is a non-resident and is aged 82 years., Solution:, Computation of Tax Liability, Gross Total Income, Less: Deduction u/s 80C to 80U, Total Income, Tax on ₹5,00,000 at slab rate, Less: Rebate u/s 87A, Tax before health & education cess, Add: HEC @ 4%, Tax Liability, , 5,60,000, (60,000), 5,00,000, 12,500, Nil, 12,500, 500, 13,000, , Illustration 5:, Compute tax liability in the following cases for the assessment year 2022-23., (i), Mr. X (resident) has total income of ₹4,70,000, (ii), Mr. X (non-resident) has total income of ₹5,00,000, (iii), Mrs. X (resident) has total income of ₹4,95,000, (iv), Mrs. X (non-resident) has total income of ₹4,70,000
Page 31 :
Computation of Total Income And Tax Liability, (v), Mr. X (resident), aged 60 years has total income of ₹4,90,000, (vi), Mrs. X (resident), aged 60 years has total income of ₹4,90,000, (vii) Mr. X (non-resident), aged 60 years has total income of ₹4,90,000, (viii) Mrs. X (non-resident), aged 60 years has total income of ₹4,90,000, (ix), Mr. X (resident), aged 80 years has total income of ₹6,00,000, (x), Mrs. X (resident), aged 80 years has total income of ₹6,00,000, (xi), Mr. X (non-resident), aged 80 years has total income of ₹5,00,000, (xii) Mrs. X (non-resident), aged 80 years has total income of ₹4,90,000, Solution:, (i) Computation of Tax Liability, Total Income, Tax on ₹4,70,000 at slab rate, Less: Rebate u/s 87A, Tax before health & education cess, Add: HEC @ 4%, Tax Liability, (ii) Computation of Tax Liability, Total Income, Tax on ₹5,00,000 at slab rate, Less: Rebate u/s 87A, Tax before health & education cess, Add: HEC @ 4%, Tax Liability, Note: Rebate under section 87A is not allowed to non-resident., (iii) Computation of Tax Liability, Total Income, Tax on ₹4,95,000 at slab rate, Less: Rebate u/s 87A, Tax before health & education cess, Add: HEC @ 4%, Tax Liability, (iv) Computation of Tax Liability, Total Income, Tax on ₹4,70,000 at slab rate, Add: HEC @ 4%, Tax Liability, Note: Rebate under section 87A is not allowed to non-resident., , 31, , ₹, 4,70,000, 11,000, (11,000), Nil, Nil, Nil, 5,00,000, 12,500, Nil, 12,500, 500, 13,000, , 4,95,000, 12,250, (12,250), Nil, Nil, Nil, 4,70,000, 11,000, 440, 11,440, , (v) Computation of Tax Liability, Total Income, Tax on ₹4,90,000 at slab rate, Less: Rebate u/s 87A, Tax before health & education cess, Add: HEC @ 4%, Tax Liability, , 4,90,000, 9,500, (9,500), Nil, Nil, Nil, , (vi) Computation of Tax Liability, Total Income, Tax on ₹4,90,000 at slab rate, Less: Rebate u/s 87A, , 4,90,000, 9,500, (9,500)
Page 32 :
Computation of Total Income And Tax Liability, Tax before health & education cess, Add: HEC @ 4%, Tax Liability, , 32, Nil, Nil, Nil, , (vii) Computation of Tax Liability, Total Income, Tax on ₹4,90,000 at slab rate, Add: HEC @ 4%, Tax Liability, Note: Rebate under section 87A is not allowed to non-resident., , 4,90,000, 12,000, 480, 12,480, , (viii) Computation of Tax Liability, Total Income, Tax on ₹4,90,000 at slab rate, Add: HEC @ 4%, Tax Liability, Note: Rebate under section 87A is not allowed to non-resident., , 4,90,000, 12,000, 480, 12,480, , (ix) Computation of Tax Liability, Total Income, Tax on ₹6,00,000 at slab rate, Add: HEC @ 4%, Tax Liability, , 6,00,000, 20,000, 800, 20,800, , (x) Computation of Tax Liability, Total Income, Tax on ₹6,00,000 at slab rate, Add: HEC @ 4%, Tax Liability, , 6,00,000, 20,000, 800, 20,800, , (xi) Computation of Tax Liability, Total Income, Tax on ₹5,00,000 at slab rate, Add: HEC @ 4%, Tax Liability, Note: Rebate under section 87A is not allowed to non-resident., , 5,00,000, 12,500, 500, 13,000, , (xii) Computation of Tax Liability, Total Income, Tax on ₹4,90,000 at slab rate, Add: HEC @ 4%, Tax Liability, Note: Rebate under section 87A is not allowed to non-resident., , 4,90,000, 12,000, 480, 12,480, , Illustration 6:, Compute tax liability in the following cases for the assessment year 2022-23., (i), Mr. X (resident) has total income of ₹5,05,000, (ii) Mr. X (non-resident) has total income of ₹4,20,000, (iii) Mrs. X (resident) has total income of ₹4,58,000, (iv) Mrs. X (non-resident) has total income of ₹12,00,000, (v) Mr. X (resident), aged 60 years has total income of ₹22,00,000, (vi) Mrs. X (resident), aged 60 years has total income of ₹105,00,000, (vii) Mr. X (non-resident), aged 60 years has total income of ₹70,00,000
Page 33 :
Computation of Total Income And Tax Liability, (viii) Mrs. X (non-resident), aged 60 years has total income of ₹3,00,000, (ix) Mr. X (resident), aged 80 years has total income of ₹3,99,000, (x) Mrs. X (resident), aged 80 years has total income of ₹103,00,000, (xi) Mr. X (non-resident), aged 80 years has total income of ₹12,00,000, (xii) Mrs. X (non-resident), aged 80 years has total income of ₹9,00,000, Solution:, (i) Computation of Tax Liability, Total Income, Tax on ₹5,05,000 at slab rate, Add: HEC @ 4%, Tax Liability, (ii) Computation of Tax Liability, Total Income, Tax on ₹4,20,000 at slab rate, Add: HEC @ 4%, Tax Liability, Note: Rebate under section 87A is not allowed to non-resident., (iii) Computation of Tax Liability, Total Income, Tax on ₹4,58,000 at slab rate, Less: Rebate u/s 87A, Tax before health & education cess, Add: HEC @ 4%, Tax Liability, , 33, , ₹, 5,05,000.00, 13,500.00, 540.00, 14,040.00, 4,20,000, 8,500, 340, 8,840, , 4,58,000, 10,400, (10,400), Nil, Nil, Nil, , (iv) Computation of Tax Liability, Total Income, Tax on ₹12,00,000 at slab rate, Add: HEC @ 4%, Tax Liability, , 12,00,000, 1,72,500, 6,900, 1,79,400, , (v) Computation of Tax Liability, Total Income, Tax on ₹22,00,000 at slab rate, Add: HEC @ 4%, Tax Liability, , 22,00,000, 4,70,000, 18,800, 4,88,800, , (vi) Computation of Tax Liability, Total Income, Tax on ₹105,00,000 at slab rate, Add: Surcharge @ 15%, Tax before health & education cess, Add: HEC @ 4%, Tax Liability, , 105,00,000, 29,60,000, 4,44,000, 34,04,000, 1,36,160, 35,40,160, , (vii) Computation of Tax Liability, Total Income, Tax on ₹70,00,000 at slab rate, Add: Surcharge @ 10%, Tax before health & education cess, Add: HEC @ 4%, , 70,00,000, 19,12,500, 1,91,250, 21,03,750, 84,150
Page 34 :
Computation of Total Income And Tax Liability, Tax Liability, (viii) Computation of Tax Liability, Total Income, Tax on ₹3,00,000 at slab rate, Add: HEC @ 4%, Tax Liability, Note: Rebate under section 87A is not allowed for non-resident., (ix) Computation of Tax Liability, Total Income, Tax on ₹3,99,000 at slab rate, Tax Liability, (x) Computation of Tax Liability, Total Income, Tax on ₹103,00,000 at slab rate, Add: Surcharge @ 15%, Tax before health & education cess, Add: HEC @ 4%, Tax Liability, , 34, 21,87,900, 3,00,000, 2,500, 100, 2,600, , 3,99,000, Nil, Nil, 103,00,000, 28,90,000, 4,33,500, 33,23,500, 1,32,940, 34,56,440, , (xi) Computation of Tax Liability, Total Income, Tax on ₹12,00,000 at slab rate, Add: HEC @ 4%, Tax Liability, , 12,00,000, 1,72,500, 6,900, 1,79,400, , (xii) Computation of Tax Liability, Total Income, Tax on ₹9,00,000 at slab rate, Add: HEC @ 4%, Tax Liability, , 9,00,000, 92,500, 3,700, 96,200, , Question 8: Explain taxability of Casual Income., Answer: As per section 115BB, casual income shall be taxable @ 30%., As per section 2(24)(ix), casual income means any winnings from lotteries, crossword puzzles, races, including horse races, card games and other games of any sort or from gambling or betting of any form or, nature whatsoever., Lottery includes winnings from prizes awarded to any person by draw of lots or by chance or in any other, manner whatsoever, under any scheme or arrangement by whatever name called., Card game and other game of any sort includes any game show, an entertainment programme on television, or electronic mode, in which people compete to win prizes or any other similar game., Casual income shall be taxable under the head Other Sources and it will be included in the gross total, income and also total income but while computing tax liability, casual income shall be separated from total, income and shall be taxable @ 30%., If assessee has incurred any expenditure in connection with earning of casual income, such expenditure shall, not be allowed to be deducted, eg. Mr. X purchased lottery tickets of ₹10,000 and he had a winning of ₹, 1,00,000, in this case expenditure of ₹10,000 shall not be allowed to be deducted and income of ₹1,00,000, shall be taxable @ 30%.
Page 35 :
Computation of Total Income And Tax Liability, , 35, , As per section 58(4), deduction under section 80C to 80U shall not be allowed from casual income however, as per section 87A, rebate shall be allowed., Illustration 7:, Mr. X has income under the head Salary ₹5,00,000 and casual income ₹3,00,000 and deduction under, section 80C to 80U ₹2,00,000, in this case his tax liability shall be, Computation of Total Income of Mr. X, Previous Year 2021-22, Assessment Year 2022-23, ₹, Income under the head Salary, 5,00,000.00, Income under the Other Sources (Casual income), 3,00,000.00, Gross Total Income, 8,00,000.00, Less: Deduction u/s 80C to 80U, (2,00,000.00), Total Income, 6,00,000.00, Computation of Tax Liability, Tax on casual income ₹3,00,000 @ 30%, Tax on normal income ₹3,00,000 at slab rate, Tax before health & education cess, Add: HEC @ 4%, Tax Liability, , 90,000.00, 2,500.00, 92,500.00, 3,700.00, 96,200.00, , (b) If in the above case deduction allowed under section 80C to 80U is ₹6,00,000, tax liability shall be, Computation of Total Income of Mr. X, Previous Year 2021-22, Assessment Year 2022-23, Income under the head Salary, Income under the Other Sources (Casual income), Gross Total Income, Less: Deduction u/s 80C to 80U, Total Income, Computation of Tax Liability, Tax on casual income ₹3,00,000 @ 30%, Less: Rebate u/s 87A, Tax before health & education cess, Add: HEC @ 4%, Tax Liability, , ₹, 5,00,000.00, 3,00,000.00, 8,00,000.00, (5,00,000.00), 3,00,000.00, 90,000.00, (12,500.00), 77,500.00, 3,100.00, 80,600.00, , Illustration 8: Mr. X has casual income of ₹102,00,000 and deduction allowed under section 80C to 80U, are ₹5,00,000, in this case his tax liability shall be, Computation of Total Income of Mr. X, Previous Year 2021-22, Assessment Year 2022-23, Income under the Other Sources (Casual income), Gross Total Income, Less: Deduction u/s 80C to 80U, Total Income, Computation of Tax Liability, Tax on casual income ₹102,00,000 @ 30%, Add: Surcharge @ 15%, Tax before marginal relief, Less: Marginal Relief, , ₹, 102,00,000.00, 102,00,000.00, Nil, 102,00,000.00, 30,60,000.00, 4,59,000.00, 35,19,000.00, (19,000.00)
Page 36 :
Computation of Total Income And Tax Liability, Working Note:, Tax + surcharge on income of ₹102,00,000, Tax + surcharge on income of ₹100,00,000, Increase in tax, Increase in income, Marginal Relief (2,19,000 – 2,00,000), Tax after marginal relief, Add: HEC @ 4%, Tax Liability, , 36, , 35,19,000, (33,00,000), 2,19,000, 2,00,000, 19,000, 35,00,000.00, 1,40,000.00, 36,40,000.00, , Illustration 9: Mr. X has income under the head Salary ₹70,000 and casual income ₹2,50,000 and deduction, u/s 80C to 80U ₹40,000. Compute his tax liability assessment year 2022-23., Solution:, Computation of Total Income of Mr. X, Previous Year 2021-22, Assessment Year 2022-23, ₹, Income under the head Salary, 70,000.00, Income under the Other Sources (Casual income), 2,50,000.00, Gross Total Income, 3,20,000.00, Less: Deduction u/s 80C to 80U, (40,000.00), Total Income, 2,80,000.00, Computation of Tax Liability, Tax on casual income ₹2,50,000 @ 30%, 75,000.00, Tax on normal income ₹30,000 at slab rate, Nil, Less: Rebate u/s 87A, (12,500.00), Tax before health & education cess, 62,500.00, Add: HEC @ 4%, 2,500.00, Tax Liability, 65,000.00, (b) Presume he is non-resident., Solution:, Computation of Total Income of Mr. X, Previous Year 2021-22, Assessment Year 2022-23, Income under the head Salary, Income under the Other Sources (Casual income), Gross Total Income, Less: Deduction u/s 80C to 80U, Total Income, Computation of Tax Liability, Tax on casual income ₹2,50,000 @ 30%, Tax on normal income ₹30,000 at slab rate, Less: Rebate u/s 87A, Tax before health & education cess, Add: HEC @ 4%, Tax Liability, , ₹, 70,000.00, 2,50,000.00, 3,20,000.00, (40,000.00), 2,80,000.00, 75,000.00, Nil, Nil, 75,000.00, 3,000.00, 78,000.00, , A person having Total Income from ₹50 lakhs to ₹100 lakhs shall be eligible for marginal relief upto total, income of ₹52,23,880 in case of casual income., A person having Total Income from ₹100 lakhs to ₹200 lakhs shall be eligible for marginal relief upto, total income of ₹102,29,000 in case of casual income., A person having Total Income from ₹200 lakhs to ₹500 lakhs shall be eligible for marginal relief upto, total income of ₹209,60,000 in case of casual income.
Page 37 :
Computation of Total Income And Tax Liability, , 37, , A person having Total Income above ₹500 lakhs shall be eligible for marginal relief upto total income of, ₹530,56,030 in case of casual income., Illustration 10: Mr. X has casual income ₹ 100 lakh, in this case his Tax Liability for P.Y. 2021-22, A.Y. 2022-23 shall beComputation of Tax Liability, Tax on casual income ₹100,00,000 @ 30%, 30,00,000.00, Add: Surcharge @ 10%, 3,00,000.00, Tax before health & education cess, 33,00,000.00, Add: HEC @ 4%, 1,32,000.00, Tax Liability, 34,32,000.00, (b) Mr. X has casual income ₹ 101 lakh, in this case his Tax Liability for P.Y. 2021-22 A.Y. 2022-23, shall beComputation of Tax Liability, Tax on casual income ₹101,00,000 @ 30%, 30,30,000.00, Add: Surcharge @ 15%, 4,54,500.00, Tax before marginal relief, 34,84,500.00, Less: Marginal Relief, (84,500.00), Working Note:, Tax + surcharge on income of ₹101,00,000, 34,84,500, Tax + surcharge on income of ₹100,00,000, (33,00,000), Increase in tax, 1,84,500, Increase in income, 1,00,000, Marginal Relief (1,84,500 – 1,00,000), 84,500, Tax after marginal relief, 34,00,000.00, Add: HEC @ 4%, 1,36,000.00, Tax Liability, 35,36,000.00, (c) Mr. X has casual income ₹ 102 lakh, in this case his Tax Liability for P.Y. 2021-22 A.Y. 2022-23, shall beComputation of Tax Liability, Tax on casual income ₹102,00,000 @ 30%, 30,60,000.00, Add: Surcharge @ 15%, 4,59,000.00, Tax before marginal relief, 35,19,000.00, Less: Marginal Relief, (19,000.00), Working Note:, Tax + surcharge on income of ₹102,00,000, 35,19,000, Tax + surcharge on income of ₹100,00,000, (33,00,000), Increase in tax, 2,19,000, Increase in income, 2,00,000, Marginal Relief (2,19,000 – 2,00,000), 19,000, Tax after marginal relief, 35,00,000.00, Add: HEC @ 4%, 1,40,000.00, Tax Liability, 36,40,000.00, (d) Mr. X has casual income ₹103 lakh, in this case his Tax Liability for P.Y. 2021-22 A.Y. 2022-23, shall beComputation of Tax Liability, Tax on casual income ₹103,00,000 @ 30%, 30,90,000.00, Add: Surcharge @ 15%, 4,63,500.00, Tax before health & education cess, 35,53,500.00, Add: HEC @ 4%, 1,42,140.00
Page 38 :
Computation of Total Income And Tax Liability, Tax Liability, , 38, 36,95,640.00, , Question 9: Explain taxability of Capital Gains., Answer: If any capital asset has been transferred like land, building, gold etc. profit shall be called capital, gains and if the asset has been transferred within a period of three years, capital gains shall be short term and, shall be taxable at the normal rate and if asset is sold after 3 years, it will be long term capital gain and as, per section 112, it shall be taxable @ 20% and also deductions under section 80C to 80U i.e. Chapter VI-A,, shall not be allowed from long term capital gains., In case of listed shares or units of equity oriented mutual fund etc., period of three years shall be taken as, one year., If any person has transferred listed equity shares or listed units of equity oriented mutual funds and has paid, securities transaction tax, in such cases long term capital gain shall be taxable @ 10% u/s 112A but only, amount in excess of ₹1,00,000 and short term capital gains shall be covered under section 111A and shall be, taxable @ 15% and deductions under section 80C to 80U i.e. Chapter VI-A, shall not be allowed from such, long term or short term capital gains., Rebate u/s 87A shall be allowed from tax on LTCG or STCG 111A. (No Rebate u/s 87A from LTCG 112A), Special provision for resident individual, In case of a resident individual if total income excluding long term capital gains 112/112A and short term, capital gain covered under section 111A and casual income, is below the amount which is exempt from, income tax (i.e.2,50,000/3,00,000/5,00,000), in such cases deficiency in the exemption shall be allowed, from long term capital gains 112 or short term capital gain under section 111A or long term capital gains, under section 112A, in that order. Such benefit is not allowed to a non-resident., Illustration 11:, Mr. X has incomes as given below:, • Income under the head salary, • Income under the head house property, • Income under the head business/profession, • Long term capital gains, • Long term capital gains u/s 112A, • Short term capital gains, • Short term capital gains u/s 111A, • Casual Income (winnings of lottery), • Other income, Deductions allowed under section 80C to 80U, Compute his tax liability for the assessment year 2022-23., Solution:, Computation of Total Income, • Income under the head salary, • Income under the head house property, • Income under the head business/profession, • Long term capital gains, • Long term capital gains u/s 112A, • Short term capital gains, • Short term capital gains u/s 111A, • Casual Income (winnings of lottery), • Other income, Gross Total Income, Less: Deduction u/s 80C to 80U, Total Income, , 35,000, 45,000, 30,000, 1,10,000, 5,00,000, 25,000, 7,00,000, 55,000, 3,000, 2,00,000, ₹, 35,000, 45,000, 30,000, 1,10,000, 5,00,000, 25,000, 7,00,000, 55,000, 3,000, 15,03,000, (1,38,000), 13,65,000
Page 39 :
Computation of Total Income And Tax Liability, Computation of Tax Liability, Tax on Long term capital gains (₹1,10,000 – 1,10,000)@ 20% u/s 112, Tax on Short term capital gains (₹7,00,000 – 1,40,000)@ 15% u/s 111A, Tax on Long term capital gains 4,00,000 (5,00,000-1,00,000) @ 10% u/s 112A, Tax on Casual Income ₹55,000 @ 30% u/s 115BB, Tax on normal income, Tax before health & education cess, Add: HEC @ 4%, Tax Liability, (b) Presume he is non-resident., Solution:, Computation of Total Income, • Income under the head salary, • Income under the head house property, • Income under the head business/profession, • Long term capital gains, • Long term capital gains u/s 112A, • Short term capital gains, • Short term capital gains u/s 111A, • Casual Income (winnings of lottery), • Other income, Gross Total Income, Less: Deduction u/s 80C to 80U, Total Income, Computation of Tax Liability, Tax on Long term capital gains ₹1,10,000 @ 20% u/s 112, Tax on Short term capital gains ₹7,00,000 @ 15% u/s 111A, Tax on Long term capital gains ₹4,00,000 (5,00,000-1,00,000) @ 10% u/s 112A, Tax on Casual Income ₹55,000 @ 30% u/s 115BB, Tax on normal income, Tax before health & education cess, Add: HEC @ 4%, Tax Liability, , 39, , Nil, 84,000, 40,000, 16,500, Nil, 1,40,500, 5,620, 1,46,120, ₹, 35,000, 45,000, 30,000, 1,10,000, 5,00,000, 25,000, 7,00,000, 55,000, 3,000, 15,03,000, (1,38,000), 13,65,000, 22,000, 1,05,000, 40,000, 16,500, Nil, 1,83,500, 7,340, 1,90,840, , (c) Presume he is resident and is aged 62 years., Solution:, Computation of Total Income, • Income under the head salary, • Income under the head house property, • Income under the head business/profession, • Long term capital gains, • Long term capital gains u/s 112A, • Short term capital gains, • Short term capital gains u/s 111A, • Casual Income (winnings of lottery), • Other income, Gross Total Income, Less: Deduction u/s 80C to 80U, Total Income, , ₹, 35,000, 45,000, 30,000, 1,10,000, 5,00,000, 25,000, 7,00,000, 55,000, 3,000, 15,03,000, (1,38,000), 13,65,000
Page 40 :
Computation of Total Income And Tax Liability, Computation of Tax Liability, Tax on Long term capital gains (₹1,10,000 – 1,10,000)@ 20% u/s 112, Tax on Short term capital gains (₹7,00,000 – 1,90,000)@ 15% u/s 111A, Tax on Long term capital gains ₹4,00,000 (5,00,000-1,00,000) @ 10% u/s 112A, Tax on Casual Income ₹55,000 @ 30% u/s 115BB, Tax on normal income, Tax before health & education cess, Add: HEC @ 4%, Tax Liability, , 40, Nil, 76,500, 40,000, 16,500, Nil, 1,33,000, 5,320, 1,38,320, , (d) Presume he is resident and is aged 82 years., Solution:, Computation of Total Income, • Income under the head salary, • Income under the head house property, • Income under the head business/profession, • Long term capital gains, • Long term capital gains u/s 112A, • Short term capital gains, • Short term capital gains u/s 111A, • Casual Income (winnings of lottery), • Other income, Gross Total Income, Less: Deduction u/s 80C to 80U, Total Income, , ₹, 35,000, 45,000, 30,000, 1,10,000, 5,00,000, 25,000, 7,00,000, 55,000, 3,000, 15,03,000, (1,38,000), 13,65,000, , Computation of Tax Liability, Tax on Long term capital gains (₹1,10,000 – 1,10,000)@ 20% u/s 112, Nil, Tax on Short term capital gains (₹7,00,000 – 3,90,000)@ 15% u/s 111A, 46,500, Tax on Long term capital gains ₹4,00,000 (5,00,000-1,00,000) @ 10% u/s 112A, 40,000, Tax on Casual Income ₹55,000 @ 30% u/s 115BB, 16,500, Tax on normal income, Nil, Tax before health & education cess, 1,03,000, Add: HEC @ 4%, 4,120, Tax Liability, 1,07,120, Illustration 11A:, (i) Mr X has income under the head house property ₹5,00,000 and LTCG 112A ₹1,00,000. Compute, tax payable., Answer:, Computation of Total Income, Income under the head House Property, 5,00,000.00, Income under the head Capital Gains, 1,00,000.00, Gross Total Income, 6,00,000.00, Less: Deduction u/s 80C, Nil, Total Income, 6,00,000.00, Computation of Tax Liability, Tax on ₹5,00,000 at slab rate, Tax on Nil (₹1,00,000- ₹1,00,000) u/s LTCG 112A, Less: Rebate u/s 87A, Tax before health & education cess, Add: HEC @ 4%, , 12,500.00, Nil, Nil, 12,500.00, 500.00
Page 41 :
Computation of Total Income And Tax Liability, Tax Liability, , 41, 13,000.00, , Note: Since total income is exceeding ₹5,00,000, rebate is not allowed., (ii) Mr X has LTCG 112A ₹50,50,000. Compute tax payable., Answer:, Computation of Total Income, Income under the head Capital Gains, Gross Total Income, Less: Deduction u/s 80C, Total Income, Computation of Tax Liability, Tax on ₹47,00,000 (50,50,000-1,00,000-2,50,000) u/s LTCG 112A, Add: Surcharge @ 10%, Less: Marginal Relief, Working Note:, Tax + surcharge on income of ₹50,50,000, 5,17,000, Tax on income of ₹50,00,000, (4,65,000), ((50,00,000-1,00,000-2,50,000) x 10%), Increase in tax, 52,000, Increase in income, 50,000, Marginal Relief (52,000 – 50,000), 2,000, Tax before health & education cess, Add: HEC @ 4%, Tax Liability, (iii) Mr X has LTCG 112A ₹51,00,000. Compute tax payable., Answer:, Computation of Total Income, Income under the head Capital Gains, Gross Total Income, Less: Deduction u/s 80C, Total Income, Computation of Tax Liability, Tax on ₹47,50,000 (51,00,000-1,00,000-2,50,000) u/s LTCG 112A, Add: Surcharge @ 10%, Less: Marginal Relief, Working Note:, Tax + surcharge on income of ₹51,00,000, 5,22,500, Tax on income of ₹50,00,000, (4,65,000), ((50,00,000-1,00,000-2,50,000) x 10%), Increase in tax, 57,500, Increase in income, 1,00,000, Marginal Relief (57,500 – 1,00,000), Nil, Tax before health & education cess, Add: HEC @ 4%, Tax Liability, , 50,50,000.00, 50,50,000.00, Nil, 50,50,000.00, 4,70,000.00, 47,000.00, (2,000), , 5,15,000.00, 20,600.00, 5,35,600.00, , 51,00,000.00, 51,00,000.00, Nil, 51,00,000.00, 4,75,000.00, 47,500.00, Nil, , 5,22,500.00, 20,900.00, 5,43,400.00, , Illustration 12: Compute tax liability for the assessment year 2022-23 in the following situations:, (i) Mr. X is resident in India and has income under the head house property ₹40,000 and income under the, head salary ₹30,000 and long term capital gains ₹4,80,000., (ii) Presume in the above situation the assessee is Mrs. X.
Page 42 :
Computation of Total Income And Tax Liability, (iii) Presume in the above situation the assessee is Mrs. X and she is aged about 70 years., (iv) Presume in the above situation the assessee is Mr. X and he is aged about 70 years., (v) Presume in the above situation the assessee is Mrs. X and she is aged about 85 years., (vi) Presume in the above situation the assessee is Mr. X and he is aged about 85 years., (vii) Presume in all the above situations, the assessee is non-resident in India., Solution:, (i), Computation of Total Income, Income under the head Salary, Income under the head House Property, Income under the head Capital Gains (LTCG), Gross Total Income, Less: Deduction u/s 80C to 80U, Total Income, Computation of Tax Liability, Tax on LTCG ₹3,00,000 (4,80,000 – 1,80,000) @ 20% u/s 112, Tax on ₹70,000 at slab rate, Tax before health & education cess, Add: HEC @ 4%, Tax Liability, (ii), Total Income, Computation of Tax Liability, Tax on LTCG ₹3,00,000 (4,80,000 – 1,80,000) @ 20% u/s 112, Tax on ₹70,000 at slab rate, Tax before health & education cess, Add: HEC @ 4%, Tax Liability, (iii), Total Income, Computation of Tax Liability, Tax on LTCG ₹2,50,000 (4,80,000 – 2,30,000) @ 20% u/s 112, Tax on ₹70,000 at slab rate, Tax before health & education cess, Add: HEC @ 4%, Tax Liability, (iv), Total Income, Computation of Tax Liability, Tax on LTCG ₹2,50,000 (4,80,000 – 2,30,000) @ 20% u/s 112, Tax on ₹70,000 at slab rate, Tax before health & education cess, Add: HEC @ 4%, Tax Liability, (v), Total Income, Computation of Tax Liability, Tax on LTCG ₹50,000 (4,80,000 – 4,30,000) @ 20% u/s 112, Tax on ₹70,000 at slab rate, , 42, , ₹, 30,000, 40,000, 4,80,000, 5,50,000, Nil, 5,50,000, 60,000, Nil, 60,000, 2,400, 62,400, 5,50,000, 60,000, Nil, 60,000, 2,400, 62,400, 5,50,000, 50,000, Nil, 50,000, 2,000, 52,000, 5,50,000, 50,000, Nil, 50,000, 2,000, 52,000, 5,50,000, 10,000, Nil
Page 43 :
Computation of Total Income And Tax Liability, Tax before health & education cess, Add: HEC @ 4%, Tax Liability, (vi), Total Income, Computation of Tax Liability, Tax on LTCG ₹50,000 (4,80,000 – 4,30,000) @ 20% u/s 112, Tax on ₹70,000 at slab rate, Tax before health & education cess, Add: HEC @ 4%, Tax Liability, (vii), In situation (i), Total Income, Computation of Tax Liability, Tax on LTCG ₹4,80,000 @ 20% u/s 112, Tax on ₹70,000 at slab rate, Tax before health & education cess, Add: HEC @ 4%, Tax Liability, In situation (ii), Total Income, Computation of Tax Liability, Tax on LTCG ₹4,80,000 @ 20% u/s 112, Tax on ₹70,000 at slab rate, Tax before health & education cess, Add: HEC @ 4%, Tax Liability, In situation (iii), Total Income, Computation of Tax Liability, Tax on LTCG ₹4,80,000 @ 20% u/s 112, Tax on ₹70,000 at slab rate, Tax before health & education cess, Add: HEC @ 4%, Tax Liability, In situation (iv), Total Income, Computation of Tax Liability, Tax on LTCG ₹4,80,000 @ 20% u/s 112, Tax on ₹70,000 at slab rate, Tax before health & education cess, Add: HEC @ 4%, Tax Liability, In situation (v), Total Income, Computation of Tax Liability, Tax on LTCG ₹4,80,000 @ 20% u/s 112, , 43, 10,000, 400, 10,400, 5,50,000, 10,000, Nil, 10,000, 400, 10,400, , 5,50,000, 96,000, Nil, 96,000, 3,840, 99,840, 5,50,000, 96,000, Nil, 96,000, 3,840, 99,840, 5,50,000, 96,000, Nil, 96,000, 3,840, 99,840, 5,50,000, 96,000, Nil, 96,000, 3,840, 99,840, 5,50,000, 96,000
Page 44 :
Computation of Total Income And Tax Liability, Tax on ₹70,000 at slab rate, Tax before health & education cess, Add: HEC @ 4%, Tax Liability, In situation (vi), Total Income, Computation of Tax Liability, Tax on LTCG ₹4,80,000 @ 20% u/s 112, Tax on ₹70,000 at slab rate, Tax before health & education cess, Add: HEC @ 4%, Tax Liability, Illustration 13: Compute tax liability in the following cases, (i), Mr. X a resident has long term capital gains ₹3,50,000., (ii), Mr. X a resident has casual income ₹3,50,000., (iii) Mr. X a resident has short term capital gains u/s 111A ₹3,50,000., (iv), Mr. X a non-resident has long term capital gains ₹3,50,000., (v), Mr. X a non-resident has casual income ₹3,50,000., (vi), Mr. X a non-resident has short term capital gains u/s 111A ₹3,50,000., (vii) Mr. X a non-resident aged 61 years has long term capital gains ₹3,50,000., (viii) Mr. X a non-resident aged 61 years has casual income ₹3,50,000., (ix), Mr. X a non-resident aged 61 years has short term capital gains u/s 111A ₹3,50,000., Solution:, (i), Income under the head capital gains (LTCG), Total Income, Computation of Tax Liability, Tax on Long term capital gains ₹1,00,000 (₹3,50,000 – ₹2,50,000)@ 20% u/s 112, Less: Rebate u/s 87A, Tax before health & education cess, Add: HEC @ 4%, Tax Liability, (ii), Income under the head other sources (casual income), Total Income, Computation of Tax Liability, Tax on casual income ₹3,50,000 @ 30% u/s 115BB, Less: Rebate u/s 87A, Tax before health & education cess, Add: HEC @ 4%, Tax Liability, (iii), Income under the head capital gains (STCG u/s 111A), Total Income, Computation of Tax Liability, Tax on Short term capital gains ₹1,00,000 (₹3,50,000 – ₹2,50,000)@ 15% u/s 111A, Less: Rebate u/s 87A, Tax before health & education cess, Add: HEC @ 4%, Tax Liability, , 44, Nil, 96,000, 3,840, 99,840, 5,50,000, 96,000, Nil, 96,000, 3,840, 99,840, , ₹, 3,50,000, 3,50,000, 20,000, (12,500), 7,500, 300, 7,800, 3,50,000, 3,50,000, 1,05,000, (12,500), 92,500, 3,700, 96,200, 3,50,000, 3,50,000, 15,000, (12,500), 2,500, 100, 2,600
Page 45 :
Computation of Total Income And Tax Liability, (iv), Income under the head capital gains (LTCG), Total Income, Computation of Tax Liability, Tax on Long term capital gains ₹3,50,000 @ 20% u/s 112, Add: HEC @ 4%, Tax Liability, (v), Income under the head other sources (casual income), Total Income, Computation of Tax Liability, Tax on casual income ₹3,50,000 @ 30% u/s 115BB, Add: HEC @ 4%, Tax Liability, (vi), Income under the head capital gains (STCG u/s 111A), Total Income, Computation of Tax Liability, Tax on Short term capital gains ₹3,50,000 @ 15% u/s 111A, Add: HEC @ 4%, Tax Liability, (vii), Income under the head capital gains (LTCG), Total Income, Computation of Tax Liability, Tax on Long term capital gains ₹3,50,000 @ 20% u/s 112, Add: HEC @ 4%, Tax Liability, (viii), Income under the head other sources (casual income), Total Income, Computation of Tax Liability, Tax on casual income ₹3,50,000 @ 30% u/s 115BB, Add: HEC @ 4%, Tax Liability, (ix), Income under the head capital gains (STCG u/s 111A), Total Income, Computation of Tax Liability, Tax on Short term capital gains ₹3,50,000 @ 15% u/s 111A, Add: HEC @ 4%, Tax Liability, Illustration 14: Compute tax liability for the assessment year 2022-23 in the following situations:, (i) Mr. X is resident in India and his incomes are as follows:, (a) Income under the head Salary ₹1,20,000, (b) Income under the head House Property ₹60,000, , 45, 3,50,000, 3,50,000, 70,000, 2,800, 72,800, 3,50,000, 3,50,000, 1,05,000, 4,200, 1,09,200, 3,50,000, 3,50,000, 52,500, 2,100, 54,600, 3,50,000, 3,50,000, 70,000, 2,800, 72,800, 3,50,000, 3,50,000, 1,05,000, 4,200, 1,09,200, 3,50,000, 3,50,000, 52,500, 2,100, 54,600
Page 46 :
Computation of Total Income And Tax Liability, , 46, , (c) Long term capital gains ₹2,20,000, (d) Short term capital gain under section 111A ₹1,10,000, (e) Casual Income ₹90,000, (f) Deduction under section 80C to 80U ₹2,00,000 ., (ii) Presume in the above situation the assessee is Mrs. X., (iii) Presume in the above situation the assessee is Mrs. X and she is aged about 70 years., (iv) Presume in the above situation the assessee is Mr. X and he is aged about 70 years., (v) Presume in the above situation the assessee is Mrs. X and she is aged about 83 years., (vi) Presume in the above situation the assessee is Mr. X and he is aged about 83 years., (vii) Presume in the above situation the assessee is Mr. X and he is aged about 70 years and he is nonresident., (viii) Presume in the above situation the assessee is Mr. X and he is aged about 83 years old and he is nonresident., Solution:, (i), ₹, ₹, Computation of Total Income, Income under the head Salary, 1,20,000, Income under the head House Property, 60,000, Income under the head Capital Gains, Long term capital gains, 2,20,000, Short term capital gains u/s 111A, 1,10,000, 3,30,000, Income under the head Other Sources (Casual Income), 90,000, Gross Total Income, 6,00,000, Less: Deduction u/s 80C to 80U, (1,80,000), Total Income, 4,20,000, Computation of Tax Liability, Tax on LTCG (2,20,000 – 2,20,000) @ 20% u/s 112, Nil, Tax on STCG ₹80,000 (₹1,10,000 – 30,000) @ 15% u/s 111A, 12,000, Tax on Casual income ₹90,000 @ 30% u/s 115BB, 27,000, Tax on normal income at slab rate, Nil, Tax before Rebate u/s 87A, 39,000, Less: Rebate u/s 87A, (12,500), Tax before health & education cess, 26,500, Add: HEC @ 4%, 1,060, Tax Liability, 27,560, (ii), Total Income, Computation of Tax Liability, Tax on LTCG (2,20,000 – 2,20,000) @ 20% u/s 112, Tax on STCG ₹80,000 (₹1,10,000 – 30,000) @ 15% u/s 111A, Tax on Casual income ₹90,000 @ 30% u/s 115BB, Tax on normal income at slab rate, Tax before Rebate u/s 87A, Less: Rebate u/s 87A, Tax before health & education cess, Add: HEC @ 4%, Tax Liability, (iii), Total Income, Computation of Tax Liability, Tax on LTCG (2,20,000 – 2,20,000) @ 20% u/s 112, , 4,20,000, Nil, 12,000, 27,000, Nil, 39,000, (12,500), 26,500, 1,060, 27,560, 4,20,000, Nil
Page 47 :
Computation of Total Income And Tax Liability, Tax on STCG ₹30,000 (1,10,000 – 80,000) @ 15% u/s 111A, Tax on Casual Income ₹90,000 @ 30% u/s 115BB, Tax on normal income at slab rate, Tax before Rebate u/s 87A, Less: Rebate u/s 87A, Tax before health & education cess, Add: HEC @ 4%, Tax Liability, (iv), Total Income, Computation of Tax Liability, Tax on LTCG (2,20,000 – 2,20,000) @ 20% u/s 112, Tax on STCG ₹30,000 (1,10,000 – 80,000) @ 15% u/s 111A, Tax on Casual Income ₹90,000 @ 30% u/s 115BB, Tax on normal income at slab rate, Tax before Rebate u/s 87A, Less: Rebate u/s 87A, Tax before health & education cess, Add: HEC @ 4%, Tax Liability, (v), Total Income, Computation of Tax Liability, Tax on LTCG (2,20,000 – 2,20,000) @ 20% u/s 112, Tax on STCG (1,10,000 – 1,10,000) @ 15% u/s 111A, Tax on Casual Income ₹90,000 @ 30% u/s 115BB, Tax on normal income at slab rate, Tax before Rebate u/s 87A, Less: Rebate u/s 87A, Tax before health & education cess, Add: HEC @ 4%, Tax Liability, (vi), Total Income, Computation of Tax Liability, Tax on LTCG (2,20,000 – 2,20,000) @ 20% u/s 112, Tax on STCG (1,10,000 – 1,10,000) @ 15% u/s 111A, Tax on Casual Income ₹90,000 @ 30% u/s 115BB, Tax on normal income at slab rate, Tax before Rebate u/s 87A, Less: Rebate u/s 87A, Tax before health & education cess, Add: HEC @ 4%, Tax Liability, (vii), Total Income, Computation of Tax Liability, Tax on LTCG ₹2,20,000 @ 20% u/s 112, Tax on STCG ₹1,10,000 @ 15% u/s 111A, , 47, 4,500, 27,000, Nil, 31,500, (12,500), 19,000, 760, 19,760, 4,20,000, Nil, 4,500, 27,000, Nil, 31,500, (12,500), 19,000, 760, 19,760, 4,20,000, Nil, Nil, 27,000, Nil, 27,000, (12,500), 14,500, 580, 15,080, 4,20,000, Nil, Nil, 27,000, Nil, 27,000, (12,500), 14,500, 580, 15,080, 4,20,000, 44,000, 16,500
Page 48 :
Computation of Total Income And Tax Liability, Tax on Casual income ₹90,000 @ 30% u/s 115BB, Tax on normal income at slab rate, Tax before Rebate u/s 87A, Less: Rebate u/s 87A, Tax before health & education cess, Add: HEC @ 4%, Tax Liability, (viii), Total Income, Computation of Tax Liability, Tax on LTCG ₹2,20,000 @ 20% u/s 112, Tax on STCG ₹1,10,000 @ 15% u/s 111A, Tax on Casual income ₹90,000 @ 30% u/s 115BB, Tax on normal income at slab rate, Tax before Rebate u/s 87A, Less: Rebate u/s 87A, Tax before health & education cess, Add: HEC @ 4%, Tax Liability, , 48, 27,000, Nil, 87,500, Nil, 87,500, 3,500, 91,000, 4,20,000, 44,000, 16,500, 27,000, Nil, 87,500, Nil, 87,500, 3,500, 91,000, , Illustration 15:, Mr. X has long term capital gain ₹31 lakh and normal income ₹70 lakh, in this case his tax liability shall be, Total income, 101,00,000.00, LTCG ₹31,00,000 x 20%, 6,20,000.00, Normal income at slab rate, 19,12,500.00, Tax before surcharge, 25,32,500.00, Add: Surcharge @ 15%, 3,79,875.00, Tax before marginal relief, 29,12,375.00, Less: Marginal relief, (48,625.00), Tax + Surcharge on ₹101 lakhs, 29,12,375, Tax + Surcharge on ₹100 lakhs, (₹100 lakhs can be normal income ₹70 lakhs + LTCG ₹30 lakhs or, normal income ₹69 lakhs and LTCG ₹31 lakhs . It is not given in, the Act what combination should be taken. Hence it is a question, of law and any of the combination can be taken and it will be, correct), If first combination is taken, income tax shall be, Normal income ₹70 lakhs, 19,12,500, LTCG ₹30 lakhs, 6,00,000, Total, 25,12,500, Add: Surcharge @ 10%, 2,51,250, Total, 27,63,750, Increase in tax (29,12,375 – 27,63,750), 1,48,625, Marginal relief (1,48,625 – 1,00,000), 48,625, Tax before health & education cess, 28,63,750.00, Add: HEC @ 4%, 1,14,550.00, Tax Liability, 29,78,300.00, Second option: Normal income ₹69 lakhs and LTCG ₹31 lakhs, Total income, LTCG ₹31,00,000 x 20%, Normal income at slab rate, , 101,00,000.00, 6,20,000.00, 19,12,500.00
Page 49 :
Computation of Total Income And Tax Liability, Tax before surcharge, Add: Surcharge @ 15%, Tax before marginal relief, Less: Marginal relief, Tax + Surcharge on ₹101 lakhs, 29,12,375, Tax + Surcharge on ₹100 lakhs, (₹100 lakhs can be normal income ₹70 lakhs + LTCG ₹30 lakhs or, normal income ₹69 lakhs and LTCG ₹31 lakhs . It is not given in, the Act what combination should be taken. Hence it is a question, of law and any of the combination can be taken and it will be, correct), If second combination is taken, income tax shall be, Normal income ₹69 lakhs, 18,82,500, LTCG ₹31 lakhs, 6,20,000, Total, 25,02,500, Add: Surcharge @ 10%, 2,50,250, Total, 27,52,750, Increase in tax (29,12,375 – 27,52,750), 1,59,625, Marginal relief (1,59,625 – 1,00,000), 59,625, Tax before health & education cess, Add: HEC @ 4%, Tax Liability, , 49, 25,32,500.00, 3,79,875.00, 29,12,375.00, (59,625.00), , 28,52,750.00, 1,14,110.00, 29,66,860.00, , The rates of surcharge applicable for A.Y.2022-23 are as follows:, Individual/HUF/AOP/BOI/Artificial juridical person, Income-tax computed on normal income or section 111A or section 112 or section 112A or section, 115BAC would be increased by surcharge given under the following table –, S., No., , Particulars, , Rate of, surcharge, on income-, , (i), , Where the total income, (including, dividend, income and capital gains, chargeable to tax u/s, 111A and 112A) > ₹ 50, lakhs but ≤ ₹ 1 crore, , 10%, , (ii), , (iii), , Where total income, (including, dividend, income and capital gains, chargeable to tax u/s, 111A and 112A) > ₹ 1, crore but ≤ ₹ 2 crore, Where total income, , 15%, , 25%, , Example, Components of total, Applicable rate of, income, surcharge, Example 1, • Dividend ₹ 10 lakhs;, Surcharge would be, • STCG u/s 111A ₹ 20 lakhs; levied, @10%, on, • LTCG u/s 112A ₹ 25 lakhs; income-tax computed, on total income of ₹, and, 95 lakhs., • Other income ₹ 40 lakhs, Example 2, • Dividend ₹ 10 lakhs;, Surcharge would be, @15%, on, • STCG u/s 111A ₹ 60 lakhs; levied, • LTCG u/s 112A ₹ 65 lakhs; income-tax computed, on total income of ₹, and, 1.85 crores., • Other income ₹ 50 lakhs, Example 3
Page 50 :
Computation of Total Income And Tax Liability, (excluding, dividend, income and capital gains, chargeable to tax u/s, 111A and 112A) > ₹ 2, crore but ≤ ₹ 5 crore, , The rate of surcharge on, the income-tax payable, on the portion of, dividend income and, capital gains chargeable, to tax u/s 111A and 112A, , • Dividend income ₹ 60, lakhs;, • STCG u/s 111A ₹ 54 lakh;, • LTCG u/s 112A ₹ 55 lakh;, and, • Other income ₹ 3 crores, , 37%, , Rate of surcharge on the, income-tax payable on, the portion of dividend, income and capital gains, chargeable to tax u/s, 111A and 112A, , Not, exceeding, 15%, , Example 4, • Dividend income ₹ 60, lakhs;, • STCG u/s 111A ₹ 50 lakhs;, • LTCG u/s 112A ₹ 65 lakhs;, and, • Other income ₹ 6 crore, , (iv), , (v), , Where total income, (including, dividend, income and capital gains, chargeable to tax u/s, 111A and 112A) > ₹ 2, crore in cases not, covered under (iii) and, (iv) above, , 15%, , Surcharge @ 15%, would be levied on, income-tax on:, • Dividend income, of ₹ 60 lakhs;, • STCG of ₹ 54, lakhs chargeable to, tax u/s 111A; and, • LTCG of ₹ 55, lakhs chargeable to, tax u/s 112A., Surcharge @ 25%, would be leviable, on, income-tax, computed on other, income of ₹ 3, crores included in, total income., , Not, exceeding, 15%, , Where total income, (excluding, dividend, income and capital gains, chargeable to tax u/s, 111A and 112A) > ₹ 5, crore, , 50, , Surcharge @ 15%, would be levied on, income-tax on:, • Dividend income, of ₹ 60 lakhs;, • STCG of ₹ 50, lakhs chargeable to, tax u/s 111A; and, • LTCG of ₹ 65, lakhs chargeable to, tax u/s 112A., Surcharge @ 37%, would be leviable, on, income-tax, computed on other, income of ₹ 6, crores included in, total income., , Example 5, • Dividend income ₹ 55, lakhs;, • STCG u/s 111A ₹ 60 lakhs;, • LTCG u/s 112A ₹ 55 lakhs;, and, • Other income ₹ 1.10 crore, , Surcharge would be, levied @ 15% on, income-tax computed, on total income of ₹, 2.80 crore., , Special Provision of Surcharge for short term 111A , Long term 112A and Dividend Income, Surcharge @ 25% or 37% shall never be applicable on short term capital gain 111A , Long term capital, gains 112A and dividend income i.e. surcharge of 25% or 37% shall be applicable only if total income, excluding short term capital gain under section 111A and long term capital gain under section 112A, is, exceeding ₹ 200 Lakhs or ₹ 500 Lakhs.
Page 51 :
Computation of Total Income And Tax Liability, The calculations shall be done in the manner given below:, Example 1: Mr. X has income as given below:, Income under the head PGBP, LTCG 112A, STCG 111A, Dividend from domestic company, Compute his tax liability for A.Y. 2022-23., (b) Suppose income under the head PGBP ₹600,00,000, (c) Suppose income under the head PGBP ₹90,00,000, Solution:, Computation of Total Income and Tax Liability of Mr. X, Income under the head PGBP, LTCG 112A, STCG 111A, Dividend from domestic company, Gross Total Income, Less: Deductions u/s 80C to 80U, Total Income, , 51, , ₹300,00,000, ₹51,00,000, ₹50,00,000, ₹100,00,000, , ₹, 300,00,000, 51,00,000, 50,00,000, 100,00,000, 501,00,000, Nil, 501,00,000, , Option 1: Taking Dividend tax at slab rate for surcharge, Computation of Tax Liability, Tax on LTCG 50,00,000 (51,00,000-1,00,000) @ 10% u/s 112A, Tax on STCG 50,00,000 @15% u/s 111A, Tax on 4,00,00,000 at slab rate, Tax Before Surcharge, Add: Surcharge @ 15% on 12,50,000, Add: Surcharge on Dividend Income (slab basis) (28,12,500) X 15%, Add: Surcharge on PGBP Income (3,00,00,000 x 30% = 90,00,000) X 25%, Tax Before cess, Add: Health and education cess @ 4%, Tax Liability, , 5,00,000.00, 7,50,000.00, 118,12,500.00, 1,30,62,500.00, 1,87,500.00, 4,21,875.00, 22,50,000.00, 159,21,875.00, 6,36,875.00, 165,58,750.00, , Option 2: Taking Dividend tax @ 30% for surcharge and PGBP income on slab rate, Computation of Tax Liability, Tax on LTCG 50,00,000 (51,00,000-1,00,000) @ 10% u/s 112A, Tax on STCG 50,00,000 @15% u/s 111A, Tax on 4,00,00,000 at slab rate, Tax Before Surcharge, Add: Surcharge @ 15% on 12,50,000, Add: Surcharge on Dividend Income (100,00,000 x 30% = 30,00,000) X 15%, Add: Surcharge on PGBP Income at slab (88,12,500) X 25%, Tax Before cess, Add: Health and education cess @ 4%, Tax Liability, , 5,00,000.00, 7,50,000.00, 118,12,500.00, 1,30,62,500.00, 1,87,500.00, 4,50,000.00, 22,03,125.00, 159,03,125.00, 6,36,125.00, 165,39,250.00, , Option 3: Taking Dividend tax on average basis for surcharge, Computation of Tax Liability, Tax on LTCG 50,00,000 (51,00,000-1,00,000) @ 10% u/s 112A, Tax on STCG 50,00,000 @15% u/s 111A, Tax on 4,00,00,000 at slab rate, Tax Before Surcharge, Add: Surcharge @ 15% on 12,50,000, Add: Surcharge on Dividend Income, , 5,00,000.00, 7,50,000.00, 118,12,500.00, 1,30,62,500.00, 1,87,500.00
Page 52 :
Computation of Total Income And Tax Liability, (1,18,12,500/4,00,00,000 x 1,00,00,000 = 29,53,125) X 15%, Add: Surcharge on PGBP Income, (1,18,12,500/4,00,00,000 x 3,00,00,000 = 88,59,375) X 25%, Tax Before cess, Add: Health and education cess @ 4%, Tax Liability, Rounded off u/s 288B, , 52, 4,42,968.75, 22,14,843.75, 159,07,812.50, 6,36,312.50, 165,44,125.00, 165,44,130.00, , Note: Calculation for surcharge on tax on Dividend is not discussed in the act when it includes other, income also. As per our view we have discussed 3 methods above. Student can solve the solution by, taking any of the above methods., (b), Solution:, Computation of Total Income and Tax Liability of Mr. X, ₹, Income under the head PGBP, 600,00,000, LTCG 112A, 51,00,000, STCG 111A, 50,00,000, Dividend from domestic company, 100,00,000, Gross Total Income, 801,00,000, Less: Deductions u/s 80C to 80U, Nil, Total Income, 801,00,000, Option 1: Taking Dividend tax at slab rate for surcharge, Computation of Tax Liability, Tax on LTCG 50,00,000 (51,00,000-1,00,000) @ 10% u/s 112A, Tax on STCG 50,00,000 @15% u/s 111A, Tax on 7,00,00,000 at slab rate, Tax Before Surcharge, Add: Surcharge @ 15% on 12,50,000, Add: Surcharge on Dividend Income (slab basis) (28,12,500) X 15%, Add: Surcharge on PGBP Income (6,00,00,000 x 30%= 180,00,000) X 37%, Tax Before cess, Add: Health and education cess @ 4%, Tax Liability, , 5,00,000.00, 7,50,000.00, 208,12,500.00, 220,62,500.00, 1,87,500.00, 4,21,875.00, 66,60,000.00, 293,31,875.00, 11,73,275.00, 305,05,150.00, , Option 2: Taking Dividend tax @ 30% for surcharge and PGBP income on slab rate, Computation of Tax Liability, Tax on LTCG 50,00,000 (51,00,000-1,00,000) @ 10% u/s 112A, Tax on STCG 50,00,000 @15% u/s 111A, Tax on 7,00,00,000 at slab rate, Tax Before Surcharge, Add: Surcharge @ 15% on 12,50,000, Add: Surcharge on Dividend Income (100,00,000 x 30% = 30,00,000) X 15%, Add: Surcharge on PGBP Income at slab (178,12,500) X 37%, Tax Before cess, Add: Health and education cess @ 4%, Tax Liability, , 5,00,000.00, 7,50,000.00, 208,12,500.00, 220,62,500.00, 1,87,500.00, 4,50,000.00, 65,90,625.00, 292,92,625.00, 11,71,625.00, 304,64,250.00, , Option 3: Taking Dividend tax on average basis for surcharge, Computation of Tax Liability, Tax on LTCG 50,00,000 (51,00,000-1,00,000) @ 10% u/s 112A, Tax on STCG 50,00,000 @15% u/s 111A, Tax on 7,00,00,000 at slab rate, , 5,00,000.00, 7,50,000.00, 208,12,500.00
Page 53 :
Computation of Total Income And Tax Liability, Tax Before Surcharge, Add: Surcharge @ 15% on 12,50,000, Add: Surcharge on Dividend Income, (208,12,500/7,00,00,000 x 1,00,00,000=29,73,214.29) X 15%, Add: Surcharge on PGBP Income, (208,12,500/7,00,00,000 x 6,00,00,000 = 178,39,285.71) X 37%, Tax Before cess, Add: Health and education cess @ 4%, Tax Liability, Rounded off u/s 288B, (c), Solution:, Computation of Total Income and Tax Liability of Mr. X, Income under the head PGBP, LTCG 112A, STCG 111A, Dividend from domestic company, Gross Total Income, Less: Deductions u/s 80C to 80U, Total Income, Computation of Tax Liability, Tax on LTCG 50,00,000 (51,00,000-1,00,000) @ 10% u/s 112A, Tax on STCG 50,00,000 @15% u/s 111A, Tax on 1,90,00,000 at slab rate, Tax Before Surcharge, Add: Surcharge @ 15%, Tax Before cess, Add: Health and education cess @ 4%, Tax Liability, Example 2: Mr. X has income as given below:, Income under the head Salary, LTCG 112A, STCG 111A, Dividend from domestic company, Compute his tax liability for A.Y. 2022-23., (b) Suppose income under the head Salary ₹300,00,000, (c) Suppose income under the head Salary ₹500,00,000, Solution:, Computation of Total Income and Tax Liability of Mr. X, Income under the head Salary, LTCG 112A, STCG 111A, Dividend from domestic company, Gross Total Income, Less: Deductions u/s 80C to 80U, Total Income, Computation of Tax Liability, Tax on LTCG 20,00,000 (21,00,000-1,00,000) @ 10% u/s 112A, Tax on STCG 10,00,000 @15% u/s 111A, Tax on 1,80,00,000 at slab rate, Tax Before Surcharge, , 53, 220,62,500.00, 1,87,500.00, 4,45,982.14, 66,00,535.71, 292,96,517.85, 11,71,860.71, 304,68,378.56, 304,68,380.00, , ₹, 90,00,000, 51,00,000, 50,00,000, 100,00,000, 201,00,000, Nil, 201,00,000, 5,00,000.00, 7,50,000.00, 55,12,500.00, 67,62,500.00, 10,14,375.00, 77,76,875.00, 3,11,075.00, 80,87,950.00, ₹150,00,000, ₹21,00,000, ₹10,00,000, ₹30,00,000, , ₹, 150,00,000, 21,00,000, 10,00,000, 30,00,000, 211,00,000, Nil, 211,00,000, 2,00,000.00, 1,50,000.00, 52,12,500.00, 55,62,500.00
Page 54 :
Computation of Total Income And Tax Liability, Add: Surcharge @ 15%, Tax Before cess, Add: Health and education cess @ 4%, Tax Liability, , 54, 8,34,375.00, 63,96,875.00, 2,55,875.00, 66,52,750.00, , (b) Computation of Total Income and Tax Liability of Mr. X, Income under the head Salary, LTCG 112A, STCG 111A, Dividend from domestic company, Gross Total Income, Less: Deductions u/s 80C to 80U, Total Income, Taking Dividend tax on average basis for surcharge, Computation of Tax Liability, Tax on LTCG 20,00,000 (21,00,000-1,00,000) @ 10% u/s 112A, Tax on STCG 10,00,000 @15% u/s 111A, Tax on 3,30,00,000 at slab rate, Tax Before Surcharge, Add: Surcharge @ 15% on 3,50,000, Add: Surcharge on Dividend Income, (97,12,500/3,30,00,000 x 30,00,000= 8,82,954.55) X 15%, Add: Surcharge on salary Income, (97,12,500/330,00,000 x 300,00,000 =88,29,545.45) X 25%, Tax Before cess, Add: Health and education cess @ 4%, Tax Liability, Rounded off u/s 288B, , 22,07,386.36, 124,54,829.54, 4,98,193.18, 129,53,022.72, 129,53,020.00, , (c) Computation of Total Income and Tax Liability of Mr. X, Income under the head Salary, LTCG 112A, STCG 111A, Dividend from domestic company, Gross Total Income, Less: Deductions u/s 80C to 80U, Total Income, , ₹, 500,00,000, 21,00,000, 10,00,000, 30,00,000, 561,00,000, Nil, 561,00,000, , Taking Dividend tax on average basis for surcharge, Computation of Tax Liability, Tax on LTCG 20,00,000 (21,00,000-1,00,000) @ 10% u/s 112A, Tax on STCG 10,00,000 @15% u/s 111A, Tax on 5,30,00,000 at slab rate, Tax Before Surcharge, Add: Surcharge @ 15% on 3,50,000, Add: Surcharge on Dividend Income, (157,12,500/5,30,00,000 x 30,00,000 = 8,89,386.79) X 15%, Add: Surcharge on Salary Income, (157,12,500/530,00,000 x 500,00,000 = 148,23,113.21) X 25%, Tax Before cess, Add: Health and education cess @ 4%, Tax Liability, Rounded off u/s 288B, , ₹, 300,00,000, 21,00,000, 10,00,000, 30,00,000, 361,00,000, Nil, 361,00,000, 2,00,000.00, 1,50,000.00, 97,12,500.00, 100,62,500.00, 52,500.00, 1,32,443.18, , 2,00,000.00, 1,50,000.00, 157,12,500.00, 160,62,500.00, 52,500.00, 1,33,408.02, 37,05,778.30, 199,54,186.32, 7,98,167.45, 207,52,353.77, 207,52,350.00
Page 55 :
Computation of Total Income And Tax Liability, , 55, , Note: The above solution is calculated on following average basis. Students can solve through other, methods also, Example 3: Mr. X has income from business ₹ 203 lakhs and short term capital gain under section, 111A ₹ 30 lakhs and long term capital gains under section 112A ₹ 41 lakhs, in this case tax liability, shall be computed in the manner given below, Solution:, Income under the head business/profession from business, 203,00,000, Income under the head capital gains, Short term capital gains under section 111A, 30,00,000, Long term capital gains under section 112A, 41,00,000, Gross total income/total income, 274,00,000, Tax liability on 203,00,000 at slab rate, 2,50,000, Nil, 2,50,000, 12,500, 5,00,000, 1,00,000, 193,00,000, 57,90,000, Total, Add: Surcharge @ 25%, Total, Marginal Relief, Tax on ₹ 203 lakhs at slab rate + surcharge 25%, Tax on ₹ 200 lakhs at slab rate + surcharge 15%, Increase in Tax Liability, Increase in income, Marginal Relief, , 59,02,500, 14,75,625, 73,78,125, 73,78,125, 66,84,375, 6,93,750, 3,00,000, , Tax on short term capital gain under section 111A 30,00,000 X 15%, Add: Surcharge @ 15%, Tax on long term capital gain under section 112A (41,00,000 – 1,00,000) X 10%, Add: Surcharge @ 15%, Add: HEC @ 4%, Tax liability, , (3,93,750), 69,84,375, 4,50,000, 67,500, 4,00,000, 60,000, 79,61,875, 3,18,475, 82,80,350, , Example 4: Mr. X has income from business ₹ 501 lakhs and short term capital gain under section, 111A ₹ 30 lakhs and long term capital gains under section 112A ₹ 41 lakhs, in this case tax liability, shall be computed in the manner given below, Solution:, Income under the head business/profession from business, 501,00,000, Income under the head capital gains, Short term capital gains under section 111A, 30,00,000, Long term capital gains under section 112A, 41,00,000, Gross total income/total income, 572,00,000, Computation of Tax Liability, Total Income, Tax on ₹501,00,000 at slab rate, Add: Surcharge @ 37%, Tax before marginal relief, Less: Marginal Relief, , 501,00,000, 1,48,42,500, 54,91,725, 203,34,225, (17,18,600)
Page 56 :
Computation of Total Income And Tax Liability, , 56, , Working Note:, Tax + surcharge @37% on income of ₹501,00,000 203,34,225, Tax + surcharge @25% on income of ₹500,00,000 (1,85,15,625), Increase in tax, 18,18,600, Increase in income, 1,00,000, Marginal Relief (18,18,600 – 1,00,000), 17,18,600, Tax after marginal relief, Tax on short term capital gain under section 111A 30,00,000 X 15%, Add: Surcharge @ 15%, Tax on long term capital gain under section 112A (41,00,000 – 1,00,000) X 10%, Add: Surcharge @ 15%, Tax Liability, Add: HEC @ 4%, Tax Liability, , 186,15,625, 4,50,000, 67,500, 4,00,000, 60,000, 195,93,125, 7,83,725, 203,76,850, , Example 5: Mr. X has income from business ₹ 505 lakhs and short term capital gain under section, 111A ₹ 30 lakhs and long term capital gains under section 112A ₹ 41 lakhs, in this case tax liability, shall be computed in the manner given below, Solution:, Income under the head business/profession from business, 505,00,000, Income under the head capital gains, Short term capital gains under section 111A, 30,00,000, Long term capital gains under section 112A, 41,00,000, Gross total income/total income, 576,00,000, Computation of Tax Liability, Total Income, Tax on ₹505,00,000 at slab rate, Add: Surcharge @ 37%, Tax before marginal relief, Less: Marginal Relief, , 505,00,000, 149,62,500, 55,36,125, 204,98,625, (14,83,000), , Working Note:, Tax + surcharge @37% on income of ₹505,00,000 2,04,98,625, Tax + surcharge @25% on income of ₹500,00,000 (1,85,15,625), Increase in tax, 19,83,000, Increase in income, 5,00,000, Marginal Relief (19,83,000 – 5,00,000), 14,83,000, Tax after marginal relief, Tax on short term capital gain under section 111A 30,00,000 X 15%, Add: Surcharge @ 15%, Tax on long term capital gain under section 112A (41,00,000 – 1,00,000) X 10%, Add: Surcharge @ 15%, Tax liability before HEC, Add: HEC @ 4%, Tax Liability, , 190,15,625, 4,50,000, 67,500, 4,00,000, 60,000, 199,93,125, 7,99,725, 207,92,850
Page 57 :
Computation of Total Income And Tax Liability, , 57, , Example 6: Mr. X has income from business ₹ 550 lakhs and short term capital gain under section, 111A ₹ 30 lakhs and long term capital gains under section 112A ₹ 41 lakhs, in this case tax liability, shall be computed in the manner given below, Solution:, Income under the head business/profession from business, 550,00,000, Income under the head capital gains, Short term capital gains under section 111A, 30,00,000, Long term capital gains under section 112A, 41,00,000, Gross total income/total income, 621,00,000, Computation of Tax Liability, Total Income, Tax on ₹550,00,000 at slab rate, Add: Surcharge @ 37%, Tax on normal income, Tax on short term capital gain under section 111A 30,00,000 X 15%, Add: Surcharge @ 15%, Tax on long term capital gain under section 112A (41,00,000 – 1,00,000) X 10%, Add: Surcharge @ 15%, Tax liability before HEC, Add: HEC @ 4%, Tax Liability, , 550,00,000, 163,12,500, 60,35,625, 223,48,125, 4,50,000, 67,500, 4,00,000, 60,000, 233,25,625, 9,33,025, 242,58,650, , Question 10: Write a note on taxability of income of Partnership Firm/Limited Liability Partnership, Firm., Answer: Partnership firm/LLP, Long term capital gains are taxable @ 20%, STCG u/s 111A shall be taxable @ 15% , LTCG u/s 112A, shall be taxable in excess of 1,00,000 @ 10% and casual income @ 30% and other incomes are also taxable, @ 30%., Surcharge shall be applicable @ 12% provided total income is exceeding ₹ 1 crore., Marginal Relief, Marginal relief shall be allowed if income has exceeded ₹100 lakhs., Health & education cess is applicable @ 4%, Deductions under section 80C to 80U shall be allowed in the normal manner., Partnership firm is regulated through Partnership Act,1932 and Limited Liability Partnership firm is, regulated through Limited Liability Partnership Act, 2008., Question 11: Write a note on taxability of income of domestic company., Answer: Domestic Company, Long term capital gains are taxable @ 20%, STCG u/s 111A shall be taxable @ 15% , LTCG u/s 112A, shall be taxable in excess of 1,00,000 @ 10% and casual income @ 30% and other incomes are also taxable, @ 30%., Surcharge shall be applicable, - @ 7% provided total income is exceeding ₹100 lakhs but it is upto ₹1000 lakhs, - @ 12% provided total income is exceeding ₹1000 lakhs., Marginal relief shall be allowed if income has exceeded ₹100 lakhs / 1000 lakhs, Health & education cess is applicable @ 4%, Deductions under section 80C to 80U shall be allowed in the normal manner., (If total turnover or gross receipts in P.Y. 2018-19 does not exceed 400 crores, tax rate shall be 25% instead
Page 58 :
Computation of Total Income And Tax Liability, , 58, , of 30%)., As per section 115BAA, In case of domestic companies tax shall be payable @ 22% provided company, shall not avail any exemption and deductions. Rate of surcharge shall be 10% irrespective of income. HEC, @ 4%., As per section 115BAB, In case of domestic companies which has been setup and registered on or after, 01.10.2019 and is a manufacturing concern, tax shall be payable @ 15% provided company shall not avail, any exemption and deductions. Rate of surcharge shall be 10% irrespective of income. HEC @ 4%., Example, Compute the tax liability of X Ltd., a domestic company, assuming that the total income of X Ltd. is, ₹1,01,00,000 and the total income does not include any income in the nature of capital gains., Answer, Total income, 1,01,00,000, Tax on @ 30%, 30,30,000, Add: Surcharge @ 7%, 2,12,100, Tax before marginal relief, 32,42,100, Less: Marginal Relief, (1,42,100), Working Note:, Tax + surcharge on income of ₹101,00,000, 32,42,100, Tax on income of ₹100,00,000, (30,00,000), Increase in tax, 2,42,100, Increase in income, 1,00,000, Marginal Relief (2,42,100 – 1,00,000), 1,42,100, Tax after marginal relief, 31,00,000, Add: HEC @ 4%, 1,24,000, Tax Liability, 32,24,000, Example, Compute the tax liability of X Ltd., a domestic company, assuming that the total income of X Ltd. is, ₹10,01,00,000 and the total income does not include any income in the nature of capital gains., Answer:, Total income, 10,01,00,000, Tax on @ 30%, 300,30,000, Add: Surcharge @ 12%, 36,03,600, Tax before marginal relief, 336,33,600, Less: Marginal Relief, (14,33,600), Working Note:, Tax + surcharge @ 12% on income of ₹10,01,00,000, 336,33,600, Tax + surcharge @ 7% on income of ₹1000,00,000, (321,00,000), Increase in tax, 15,33,600, Increase in income, 1,00,000, Marginal Relief (15,33,600 – 1,00,000), 14,33,600, Tax after marginal relief, 322,00,000, Add: HEC @ 4%, 12,88,000, Tax Liability, 334,88,000, Question 12: Write a note on taxability of income of Foreign company., Answer: Foreign Company, Long term capital gains are taxable @ 20%, STCG u/s 111A shall be taxable @ 15%, LTCG u/s 112A, shall be taxable in excess of 1,00,000 @ 10% and casual income @ 30% and other incomes are taxable @, 40%., Surcharge shall be applicable, - @ 2% provided total income is exceeding ₹100 lakhs but it is upto ₹1000 lakhs.
Page 59 :
Computation of Total Income And Tax Liability, -, , 59, , @ 5% provided total income is exceeding ₹1000 lakhs, , Marginal relief shall be allowed if income has exceeded ₹100 lakhs / 1000 lakhs, Health & education cess is applicable @ 4%, Deductions under section 80C to 80U shall be allowed in the normal manner., Question 13: Explain meaning of domestic company., Answer: Meaning of domestic company, As per section 2(22A), "Domestic company" means an Indian company, or any other company which, in, respect of its income liable to tax under this Act, has made the prescribed arrangements for the declaration, and payment, within India, of the dividends (including dividends on preference shares) payable out of such, income., Prescribed arrangements for declaration and payment of dividends within India. Rule 27, The arrangements referred to in [sections 194 and 236] to be made by a company for the declaration and, payment of dividends (including dividends on preference shares) within India shall be as follows :, (1) The share-register of the company for all shareholders shall be regularly maintained at its principal place, of business within India, in respect of any assessment year from a date not later than the 1st day of April of, such year., (2) The general meeting for passing the accounts of the previous year relevant to the assessment year and for, declaring any dividends in respect thereof shall be held only at a place within India., (3) The dividends declared, if any, shall be payable only within India to all shareholders., If any foreign company has complied with all the above conditions, it will be considered to be domestic, company otherwise it will be considered to be foreign company., Illustration 16 (A): Compute tax liability of ABC Ltd. a domestic company in the following situations:, (i) The company has income under the head Business/Profession ₹50,000., (ii) The company has income under the head Business/Profession ₹1,00,000., (iii) The company has income under the head Business/Profession ₹500,00,000., (iv) The company has income under the head Business/Profession ₹100,00,000., (v) The company has long term capital gains of ₹50,000., (vi) The company has long term capital gains of ₹200,00,000., (vii) The company has long term capital gains of ₹5,00,000., (viii) The company has long term capital gains of ₹10,20,000., (ix) The company has income under the head Business/Profession ₹11 crore., Solution:, ₹, (i) Computation of Tax Liability, Income under the head Business/Profession, 50,000, Total Income, 50,000, Tax on ₹50,000 @ 30%, 15,000, Add: HEC @ 4%, 600, Tax Liability, 15,600, (ii) Computation of Tax Liability, Income under the head Business/Profession, Total Income, Tax on ₹1,00,000 @ 30%, Add: HEC @ 4%, Tax Liability, , 1,00,000, 1,00,000, 30,000, 1,200, 31,200, , (iii) Computation of Tax Liability, Income under the head Business/Profession, Total Income, Tax on ₹500,00,000 @ 30%, , 500,00,000, 500,00,000, 150,00,000
Page 60 :
Computation of Total Income And Tax Liability, Add: Surcharge @ 7%, Add: HEC @ 4%, Tax Liability, (iv) Computation of Tax Liability, Income under the head Business/Profession, Total Income, Tax on ₹100,00,000 @ 30%, Add: HEC @ 4%, Tax Liability, , 60, 10,50,000, 6,42,000, 166,92,000, 100,00,000, 100,00,000, 30,00,000, 1,20,000, 31,20,000, , (v) Computation of Tax Liability, Income under the head Capital Gains (long term capital gains), Total Income, Tax on ₹50,000 @ 20%, Add: HEC @ 4%, Tax Liability, , 50,000, 50,000, 10,000, 400, 10,400, , (vi) Computation of Tax Liability, Income under the head Capital Gains (long term capital gains), Total Income, Tax on ₹200,00,000 @ 20%, Add: Surcharge @ 7%, Add: HEC @ 4%, Tax Liability, , 200,00,000, 200,00,000, 40,00,000, 2,80,000, 1,71,200, 44,51,200, , (vii) Computation of Tax Liability, Income under the head Capital Gains (long term capital gains), Total Income, Tax on ₹5,00,000 @ 20%, Add: HEC @ 4%, Tax Liability, , 5,00,000, 5,00,000, 1,00,000, 4,000, 1,04,000, , (viii) Computation of Tax Liability, Income under the head Capital Gains (long term capital gains), Total Income, Tax on ₹10,20,000 @ 20%, Add: HEC @ 4%, Tax Liability, , 10,20,000, 10,20,000, 2,04,000, 8,160, 2,12,160, , (ix) Computation of Tax Liability, Income under the head Business/profession, Total Income, Tax on ₹11,00,00,000 @ 30%, Add: Surcharge @ 12%, Tax before health & education cess, Add: HEC @ 4%, Tax Liability, Illustration 16(B): Presume in all the above situations the assessee is a partnership firm ., Solution:, (i) Computation of Tax Liability, Income under the head Business/Profession, Total Income, , 11,00,00,000, 11,00,00,000, 330,00,000, 39,60,000, 369,60,000, 14,78,400, 384,38,400, ₹, 50,000, 50,000
Page 61 :
Computation of Total Income And Tax Liability, Tax on ₹50,000 @ 30%, Add: HEC @ 4%, Tax Liability, , 61, 15,000, 600, 15,600, , (ii) Computation of Tax Liability, Income under the head Business/Profession, Total Income, Tax on ₹1,00,000 @ 30%, Add: HEC @ 4%, Tax Liability, (iii) Computation of Tax Liability, Income under the head Business/Profession, Total Income, Tax on ₹500,00,000 @ 30%, Add: Surcharge @ 12%, Tax before health & education cess, Add: HEC @ 4%, Tax Liability, , 500,00,000, 500,00,000, 150,00,000, 18,00,000, 168,00,000, 6,72,000, 174,72,000, , (iv) Computation of Tax Liability, Income under the head Business/Profession, Total Income, Tax on ₹100,00,000 @ 30%, Add: HEC @ 4%, Tax Liability, , 100,00,000, 100,00,000, 30,00,000, 1,20,000, 31,20,000, , (v) Computation of Tax Liability, Income under the head Capital Gains (long term capital gains), Total Income, Tax on ₹50,000 @ 20%, Add: HEC @ 4%, Tax Liability, (vi) Computation of Tax Liability, Income under the head Capital Gains (long term capital gains), Total Income, Tax on ₹200,00,000 @ 20%, Add: Surcharge @ 12%, Tax before health & education cess, Add: HEC @ 4%, Tax Liability, (vii) Computation of Tax Liability, Income under the head Capital Gains (long term capital gains), Total Income, Tax on ₹5,00,000 @ 20%, Add: HEC @ 4%, Tax Liability, (viii) Computation of Tax Liability, Income under the head Capital Gains (long term capital gains), Total Income, Tax on ₹10,20,000 @ 20%, Add: HEC @ 4%, , 1,00,000, 1,00,000, 30,000, 1,200, 31,200, , 50,000, 50,000, 10,000, 400, 10,400, 200,00,000, 200,00,000, 40,00,000, 4,80,000, 44,80,000, 1,79,200, 46,59,200, 5,00,000, 5,00,000, 1,00,000, 4,000, 1,04,000, 10,20,000, 10,20,000, 2,04,000, 8,160
Page 62 :
Computation of Total Income And Tax Liability, Tax Liability, (ix) Computation of Tax Liability, Income under the head Business/profession, Total Income, Tax on ₹11,00,00,000 @ 30%, Add: Surcharge @ 12%, Tax before health & education cess, Add: HEC @ 4%, Tax Liability, Illustration 16(C): Presume in all the above situations the assessee is a foreign company., Solution:, (i) Computation of Tax Liability, Income under the head Business/Profession, Total Income, Tax on ₹50,000 @ 40%, Add: HEC @ 4%, Tax Liability, , 62, 2,12,160, 11,00,00,000, 11,00,00,000, 330,00,000, 39,60,000, 369,60,000, 14,78,400, 384,38,400, ₹, 50,000, 50,000, 20,000, 800, 20,800, , (ii) Computation of Tax Liability, Income under the head Business/Profession, Total Income, Tax on ₹1,00,000 @ 40%, Add: HEC @ 4%, Tax Liability, , 1,00,000, 1,00,000, 40,000, 1,600, 41,600, , (iii) Computation of Tax Liability, Income under the head Business/Profession, Total Income, Tax on ₹500,00,000 @ 40%, Add: Surcharge @ 2%, Add: HEC @ 4%, Tax Liability, , 500,00,000, 500,00,000, 200,00,000, 4,00,000, 8,16,000, 212,16,000, , (iv) Computation of Tax Liability, Income under the head Business/Profession, Total Income, Tax on ₹100,00,000 @ 40%, Add: HEC @ 4%, Tax Liability, (v) Computation of Tax Liability, Income under the head Capital Gains (long term capital gains), Total Income, Tax on ₹50,000 @ 20%, Add: HEC @ 4%, Tax Liability, (vi) Computation of Tax Liability, Income under the head Capital Gains (long term capital gains), Total Income, Tax on ₹200,00,000 @ 20%, Add: Surcharge @ 2%, , 100,00,000, 100,00,000, 40,00,000, 1,60,000, 41,60,000, 50,000, 50,000, 10,000, 400, 10,400, 200,00,000, 200,00,000, 40,00,000, 80,000
Page 63 :
Computation of Total Income And Tax Liability, Add: HEC @ 4%, Tax Liability, , 63, 1,63,200, 42,43,200, , (vii) Computation of Tax Liability, Income under the head Capital Gains (long term capital gains), Total Income, Tax on ₹5,00,000 @ 20%, Add: HEC @ 4%, Tax Liability, , 5,00,000, 5,00,000, 1,00,000, 4,000, 1,04,000, , (viii) Computation of Tax Liability, Income under the head Capital Gains (long term capital gains), Total Income, Tax on ₹10,20,000 @ 20%, Add: HEC @ 4%, Tax Liability, , 10,20,000, 10,20,000, 2,04,000, 8,160, 2,12,160, , (ix) Computation of Tax Liability, Income under the head Business/profession, Total Income, Tax on ₹11,00,00,000 @ 40%, Add: Surcharge @ 5%, Tax before health & education cess, Add: HEC @ 4%, Tax Liability, Illustration 17: X (HUF) has incomes as given below:, 1. Income under the head Business/Profession ₹5,00,000, 2. Income under the head House Property ₹3,00,000, 3. Long term capital gains ₹4,00,000, 4. Short term capital gains under section 111A ₹3,00,000, 5. Casual Income ₹2,00,000, 6. Deductions allowed under section 80C to 80U ₹35,000, Compute tax liability of HUF for the assessment year 2022-23., Solution:, Computation of Total Income, Income under the head Business/Profession, Income under the head House Property, Income under the head Capital Gains, Long term capital gains, Short term capital gains u/s 111A, Income under the head Other Sources (Casual Income), Gross Total Income, Less: Deduction u/s 80C to 80U, Total Income, Computation of Tax Liability, Tax on LTCG ₹4,00,000 @ 20% u/s 112, Tax on STCG ₹3,00,000 @ 15% u/s 111A, Tax on Casual income ₹2,00,000 @ 30% u/s 115BB, Tax on ₹7,65,000 at slab rate, Tax before health & education cess, Add: HEC @ 4%, Tax Liability, , 11,00,00,000, 11,00,00,000, 440,00,000, 22,00,000, 462,00,000, 18,48,000, 480,48,000, , ₹, , ₹, 5,00,000.00, 3,00,000.00, , 4,00,000, 3,00,000, , 7,00,000.00, 2,00,000.00, 17,00,000.00, (35,000.00), 16,65,000.00, 80,000.00, 45,000.00, 60,000.00, 65,500.00, 2,50,500.00, 10,020.00, 2,60,520.00
Page 64 :
Computation of Total Income And Tax Liability, , 64, , Question 14: Write a note on Computation of Tax Liability of HUF., Answer: Tax liability of Hindu undivided family, Hindu undivided family means any family which is Hindu by religion and its senior most male member is, called karta and karta is responsible for control and management of HUF. Parental property / business etc, received by karta shall be considered to be common property and taxability shall be as given below:, Normal income of Hindu undivided family shall be computed at the normal slab rate as given below:, Income shall be taxable at the slab rates given below:, If total Income upto ₹2,50,000, NIL, On next ₹2,50,000, 5%, On next ₹5,00,000, 20%, On Balance amount, 30%, Slab rate of senior citizen is not applicable to HUF even if age of Karta or its member is exceeding 60 years, or 80 years., Surcharge shall be applicable, @ 10% if total income has exceeded ₹50 lakhs but upto ₹100 lakhs., @ 15% if total income has exceeded ₹100 lakhs but upto ₹200 lakhs., @ 25% if total income has exceeded ₹200 lakhs but upto ₹500 lakhs., @ 37% if total income has exceeded ₹500 lakhs., Surcharge of 25% or 37% shall be applicable only if total income excluding short term capital gain under, section 111A and long term capital gain under section 112A, is exceeding ₹ 200 Lakhs or ₹ 500 Lakhs, All other provisions shall be similar to individual but rebate under section 87A is not allowed. Tax rates for, LTCG /LTCG 112A/ STCG u/s 111A and casual income are the same for all the persons., If normal income of resident HUF is less than the exemption limit, the difference of the amount shall be, allowed to be deducted from long term capital gain and if long term capital gains are not sufficient, it will be, allowed to be adjusted from short term capital gains under section 111A or long term capital gains u/s 112A, but it will not be allowed to be adjusted from casual income., (What is HUF is given in the Hindu Law and it is not covered in the syllabus), Example, XY HUF has income under the head business/profession ₹20 lakhs and its Karta Mr. X has individual, income ₹12 lakhs, in this case tax liability of HUF and that of Karta shall be, Tax liability of HUF ₹20 lakhs at slab rate, 4,12,500, Add: HEC @ 4%, 16,500, Tax Liability, 4,29,000, Tax Liability of Karta ₹12 lakhs at slab rate, 1,72,500, Add: HEC @ 4%, 6,900, Tax Liability, 1,79,400, Question 15: Write a note on Computation of Tax Liability of Body of Individuals/Association of, Persons., Answer: Tax liability of BOI/AOP, Body of individual means a group of individuals which is neither a company nor a partnership firm. If it is, registered in some other Act, it will be called incorporated BOI. E.g. cooperative society or Trust etc. If such, a group includes persons other than individual also, it will be called AOP., In general normal income shall be taxable at normal slab rate but rate may change as per provisions of, section 167B. (NOT COVERED IN SYLLABUS), Surcharge shall be applicable, @ 10% if total income has exceeded ₹50 lakhs but upto ₹100 lakhs., @ 15% if total income has exceeded ₹100 lakhs but upto ₹200 lakhs., @ 25% if total income has exceeded ₹200 lakhs but upto ₹500 lakhs., @ 37% if total income has exceeded ₹500 lakhs.
Page 65 :
Computation of Total Income And Tax Liability, , 65, , Surcharge of 25% or 37% shall be applicable only if total income excluding short term capital gain under, section 111A and long term capital gain under section 112A, is exceeding ₹ 200 Lakhs or ₹ 500 Lakhs, Deductions under section 80C to 80U shall be allowed in the normal manner., Question 16: Write a note on Computation of Tax Liability of Local Authority., Answer: Tax liability of local authority, In order to maintain any town or city, there is always some authority responsible and such authority is called, local authority e.g. MCD in Delhi. Such authority is allowed to collect house tax with regard to every type of, house property and also some other tax are collected by such authority. In general income of such authority, is exempt from income tax under section 10(20) but if such authority is doing any business, its income is, taxable just like a partnership firm. Deductions under section 80C to 80U shall be allowed in the normal, manner., Question 17: Explain meaning of Person Section 2(31)., Answer: Meaning of Person Section 2(31), “Person” includes—, (i) an individual,, (ii) a Hindu undivided family,, (iii) a company,, (iv) a firm,, (v) an association of persons or a body of individuals, whether incorporated or not,, (vi) a local authority,, (vii) every artificial juridical person, not covered above and income is taxable as slab rate (juridical means, legal) e.g. ICAI or Delhi University etc., Question 18 [V. Imp.]: Discuss Partial Integration of Agricultural Income?, Or, Discuss Indirect Taxing of Agricultural Income?, Or, Under the Constitution, the power to levy a tax on agricultural income vests in the States. However,, Parliament has also levied a tax on such income. Explain how this has been achieved?, Answer:, Agricultural Income Section 10(1), Under section 10(1), any agricultural income in India is fully exempt from income tax but if the, agricultural income is from outside India, it is chargeable to tax. (As per entry no. 82 of Union List, Central, Government has the power to levy income tax on income except agricultural income and power to levy tax, on agricultural income has been given to the State Government vide entry no. 46 of State List), Indirect taxing of agricultural income or partial integration of agricultural income (Under the, constitution, the power to levy a tax on agricultural income vests in the states. However, parliament, has also levied a tax on such income. Explain how this has been achieved?), If any person has agricultural income as well as non-agricultural income, his tax liability shall be computed, in the manner given below:, 1. Compute tax on the total of agricultural income and non- agricultural income considering it to be total, income of the assessee., 2. Compute tax on exemption limit (₹2,50,000 / 3,00,000 / 5,00,000) and agricultural income considering, it to be total income., 3., Deduct tax computed under Step 2 from Step 1 and apply surcharge if any and allow rebate if any and, health & education cess., 4., Long term capital gain, casual income and short term capital gain u/s 111A shall not be taken into, consideration for the purpose of partial integration
Page 66 :
Computation of Total Income And Tax Liability, 5., 6., , 66, , If Agricultural income is upto ₹5,000, or non-agricultural income is upto the limit not chargeable to tax, (₹2,50,000/3,00,000/5,00,000), partial integration is not applicable., Partial integration is not applicable in case of a partnership firm or a company., , Power to levy taxes, The Constitution of India, in Article 265 lays down that “No tax shall be levied or collected except by, authority of law.” Accordingly for levy of any tax, a law needs to be framed by the government., Constitution of India gives the power to levy and collect taxes, whether direct or indirect, to the Central and, State Government. The Parliament and State Legislatures are empowered to make laws on the matters, enumerated in the Seventh Schedule by virtue of Article 246 of the Constitution of India., Seventh Schedule to Article 246 contains three lists which enumerate the matters under which the, Parliament and the State Legislatures have the authority to make laws for the purpose of levy of taxes., The following are the lists:, (i) Union List: Parliament has the exclusive power to make laws on the matters contained in Union List., (ii) State List: The Legislatures of any State has the exclusive power to make laws on the matters contained, in the State List., (iii) Concurrent List: Both Parliament and State Legislatures have the power to make laws on the matters, contained in the Concurrent list., Income-tax is the most significant direct tax. Entry 82 of the Union List i.e., List I in the Seventh Schedule, to Article 246 of the Constitution of India has given the power to the Parliament to make laws on taxes on, income other than agricultural income., Illustration 18: Compute tax liability in the following cases:, (i) Mr. X has income under the head business/profession ₹ 5,50,000 and agricultural Income ₹ 2,50,000., (ii) Mr. Y has income under the head business/profession ₹ 10,00,000 and agricultural Income ₹ 2,50,000, (iii) Mr. Z has income under the head business/profession ₹ 10,00,000 and agricultural Income ₹ 2,50,000., Deductions allowed under section 80C to 80U ₹ 1,00,000., (iv) Mr. A has income under the head business/profession ₹ 5,50,000 and agricultural Income ₹ 10,00,000., (v) Mr. A has income under the head business/profession ₹ 2,00,000 and agricultural Income ₹ 10,00,000., (vi) Mr. A has LTCG ₹ 10,00,000 and agricultural Income ₹ 10,00,000., (vii) Mr. B, a Non - Resident has LTCG ₹ 10,00,000 and agricultural Income ₹ 10,00,000., (viii) Mr. A has Casual Income ₹ 10,00,000 and agricultural Income ₹ 10,00,000., Solution:, ₹, (i) Computation of Total Income, Income under the head Business/Profession, 5,50,000, Gross Total Income, 5,50,000, Less: Deduction u/s 80C, Nil, Total Income, 5,50,000, Agricultural Income, 2,50,000, Computation of Tax Liability, Normal income 5,50,000, Step 1. Tax on (5,50,000 + 2,50,000) at slab rate, 72,500.00, Step 2. Tax on (₹2,50,000 + 2,50,000) at slab rates, (12,500.00), Step 3. Deduct Tax at Step 2 from Tax at Step 1, 60,000.00, Tax before health & education cess, 60,000.00, Add: HEC @ 4%, 2,400.00, Tax Liability, 62,400.00, (ii) Computation of Total Income, Income under the head Business/Profession, , 10,00,000
Page 67 :
Computation of Total Income And Tax Liability, Gross Total Income, Less: Deduction u/s 80C, Total Income, Agricultural Income, Computation of Tax Liability, Normal income 10,00,000, Step 1. Tax on (10,00,000 + 2,50,000) at slab rate, Step 2. Tax on (₹2,50,000 + 2,50,000) at slab rates, Step 3. Deduct Tax at Step 2 from Tax at Step 1, Tax before health & education cess, Add: HEC @ 4%, Tax Liability, (iii) Computation of Total Income, Income under the head Business/Profession, Gross Total Income, Less: Deduction u/s 80C, Total Income, Agricultural Income, Computation of Tax Liability, Normal income 9,00,000, Step 1. Tax on (9,00,000 + 2,50,000) at slab rate, Step 2. Tax on (₹2,50,000 + 2,50,000) at slab rates, Step 3. Deduct Tax at Step 2 from Tax at Step 1, Tax before health & education cess, Add: HEC @ 4%, Tax Liability, (iv) Computation of Total Income, Income under the head Business/Profession, Gross Total Income, Less: Deduction u/s 80C, Total Income, Agricultural Income, Computation of Tax Liability, Normal income 5,50,000, Step 1. Tax on (5,50,000 + 10,00,000) at slab rate, Step 2. Tax on (₹2,50,000 + 10,00,000) at slab rates, Step 3. Deduct Tax at Step 2 from Tax at Step 1, Tax before health & education cess, Add: HEC @ 4%, Tax Liability, , 67, 10,00,000, Nil, 10,00,000, 2,50,000, 1,87,500.00, (12,500.00), 1,75,000.00, 1,75,000.00, 7,000.00, 1,82,000.00, 10,00,000, 10,00,000, (1,00,000), 9,00,000, 2,50,000, 1,57,500.00, (12,500.00), 1,45,000.00, 1,45,000.00, 5,800.00, 1,50,800.00, 5,50,000, 5,50,000, Nil, 5,50,000, 10,00,000, 2,77,500.00, (1,87,500.00), 90,000.00, 90,000.00, 3,600.00, 93,600.00, , (v) There will be no partial Integration as normal income is less than the exemption limit and Tax Liability, is Nil., (vi) There will be no partial Integration as normal income is Nil, Computation of Total Income, Income under the head Capital Gains (LTCG), Gross Total Income, Less: Deduction u/s 80C, Total Income, , 10,00,000, 10,00,000, Nil, 10,00,000
Page 68 :
Computation of Total Income And Tax Liability, Computation of Tax Liability, Tax on LTCG ₹ 7,50,000 (₹10,00,000- ₹ 2,50,000) @ 20% u/s 112, Add: HEC @ 4%, Tax Liability, (vii) There will be no partial Integration as normal income is Nil, Computation of Total Income, Income under the head Capital Gains (LTCG), Gross Total Income, Less: Deduction u/s 80C, Total Income, Computation of Tax Liability, Tax on LTCG ₹10,00,000 @ 20% u/s 112, Add: HEC @ 4%, Tax Liability, (viii) There will be no partial Integration as normal income is Nil, Computation of Total Income, Income under the head Other Sources (Casual Income), Gross Total Income, Less: Deduction u/s 80C, Total Income, Computation of Tax Liability, Tax on Casual Income ₹10,00,000 @ 30%, Add: HEC @ 4%, Tax Liability, , 68, , 1,50,000, 6,000, 1,56,000, , 10,00,000, 10,00,000, Nil, 10,00,000, 2,00,000, 8,000, 2,08,000, , 10,00,000, 10,00,000, Nil, 10,00,000, 3,00,000, 12,000, 3,12,000, , Illustration 19: (a) Mr. X, aged 68 years, has income under the head House Property ₹5,25,000, agricultural, income of ₹1,00,000, Long term capital gain amounting to ₹45,000 and casual income ₹35,000. He is, eligible for deduction under section 80C ₹20,000., Compute tax liability of Mr. X for assessment year 2022-23., Solution:, ₹, Computation of Total Income, Income under the head House Property, 5,25,000, Income under the head Capital Gains (Long term capital gain), 45,000, Income under the head Other Sources (Casual Income), 35,000, Gross Total Income, 6,05,000, Less: Deduction u/s 80C, (20,000), Total Income, 5,85,000, Agricultural Income, 1,00,000, Computation of Tax Liability, Tax on casual income ₹35,000 @ 30% u/s 115BB, 10,500.00, Tax on long term capital gain ₹45,000 @ 20% u/s 112, 9,000.00, Normal income 5,05,000, Step 1. Tax on (5,05,000 + 1,00,000) at slab rate, 31,000.00, Step 2. Tax on (₹3,00,000 + 1,00,000) at slab rates, (5,000.00), Step 3. Deduct Tax at Step 2 from Tax at Step 1, 26,000.00, Tax before health & education cess, 45,500.00, Add: HEC @ 4%, 1,820.00, Tax Liability, 47,320.00
Page 69 :
Computation of Total Income And Tax Liability, (b) Presume in the above question, Mr. X is Non Resident., Solution:, Computation of Total Income, Income under the head House Property, Income under the head Capital Gains (LTCG), Income under the head Other Sources (Casual Income), Gross Total Income, Less: Deduction u/s 80C, Total Income, Agricultural Income, Computation of Tax Liability, Tax on casual income ₹35,000 @ 30% u/s 115BB, Tax on long term capital gain ₹45,000 @ 20% u/s 112, Normal income 5,05,000, Step 1. Tax on (5,05,000 + 1,00,000), Step 2. Tax on (₹2,50,000 + 1,00,000) at slab rates, Step 3. Deduct Tax at Step 2 from Tax at Step 1, Tax before health & education cess, Add: HEC @ 4%, Tax Liability, , 69, , ₹, 5,25,000, 45,000, 35,000, 6,05,000, (20,000), 5,85,000, 1,00,000, 10,500.00, 9,000.00, 33,500.00, (5,000.00), 28,500.00, 48,000.00, 1,920.00, 49,920.00, , (c) Presume in the above question, Mr. X is resident and do not have any income from house property., Solution:, ₹, There will be no partial integration., Computation of Total Income, Income under the head Capital Gains (LTCG), 45,000, Income under the head Other Sources (casual income), 35,000, Gross Total Income, 80,000, Less: Deduction u/s 80C to 80U, NIL, Total Income, 80,000, Agricultural Income, 1,00,000, Computation of Tax Liability, Tax on casual income ₹35,000 @ 30% u/s 115BB, 10,500, Tax on long term capital gain (₹45,000- 45,000), Nil, Normal income Nil, Tax on normal income, Nil, Less: Rebate u/s 87A, (10,500), Tax before health & education cess, Nil, Add: HEC @ 4%, Nil, Tax Liability, Nil, Illustration 20: Mrs. X has income asunder –, Income under the head Salary, Income under the head Capital Gains, Long term capital gain, Short term capital gain, Income under the head Other Sources (lottery), Agricultural income, Deduction allowed under section 80C to 80U, Compute her tax liability for the assessment year 2022-23 in two situations –, (i) She is resident, , 1,55,000, 27,000, 1,09,000, 7,000, 1,05,000, 7,000
Page 70 :
Computation of Total Income And Tax Liability, (ii) She is non-resident., Solution:, (i) She is resident, Computation of Total Income, Income under the head Salary, Income under the head Capital Gains, Long term capital gain, Short term capital gain, Income under the head Other Sources (lottery), Gross Total Income, Less: Deduction u/s 80C to 80U, Total Income, Agricultural income, Computation of Tax Liability, Tax on casual income ₹7,000 @ 30% u/s 115BB, Tax on long term capital gain ₹27,000 @ 20% u/s 112, Normal income ₹2,57,000, Step 1. Tax on (2,57,000 + 1,05,000), Step 2. Tax on (₹2,50,000 + 1,05,000) at slab rates, Step 3. Deduct Tax at Step 2 from tax at Step 1, Less: Rebate u/s 87A, Tax before health & education cess, Add: HEC @ 4%, Tax Liability, , 70, ₹, 1,55,000, 27,000, 1,09,000, 7,000, 2,98,000, (7,000), 2,91,000, 1,05,000, 2,100.00, 5,400.00, 5,600.00, (5,250.00), 350.00, (7,850.00), Nil, Nil, Nil, , (ii) She is non-resident, Computation of Total Income, Income under the head Salary, Income under the head Capital Gains, Long term capital gain, Short term capital gain, Income under the head Other Sources (Casual Income), Gross Total Income, Less: Deduction u/s 80C to 80U, Total Income, Agricultural income, Computation of Tax Liability, Tax on casual income ₹7,000 @ 30% u/s 115BB, Tax on long term capital gain ₹27,000 @ 20% u/s 112, Normal income ₹2,57,000, Step 1. Tax on (2,57,000 + 1,05,000), Step 2. Tax on (₹2,50,000 + 1,05,000) at slab rates, Step 3. Deduct Tax at Step 2 from tax at Step 1, Tax before health & education cess, Add: HEC @ 4%, Tax Liability, Rounded off u/s 288B, Note: Rebate under section 87A is not allowed to non-resident., , 1,55,000, 27,000, 1,09,000, 7,000, 2,98,000, (7,000), 2,91,000, 1,05,000, 2,100.00, 5,400.00, 5,600.00, (5,250.00), 350.00, 7,850.00, 314.00, 8,164.00, 8,160.00, , Illustration 21: Mr. X has agricultural income of ₹4,900 and non-agricultural income of ₹2,65,000., Compute his tax liability for the assessment year 2022-23., Solution:, , ₹
Page 71 :
Computation of Total Income And Tax Liability, , 71, , Agricultural Income, 4,900, Non Agricultural income, 2,65,000, In this case, Agricultural income is upto ₹5000/-, thereby, partial integration shall not be applicable., Computation of Tax Liability, Tax on ₹2,65,000 at slab rate, 750, Less: Rebate u/s 87A, (750), Tax Liability, Nil, Illustration 22: Mr. X has agricultural income of ₹5,00,000 and non-agricultural income of ₹2,50,000., Compute his tax liability for the assessment year 2022-23., Solution:, His tax liability shall be nil, since his non-agricultural income is ₹2,50,000 and partial integration is not, applicable., Illustration 23: Mr. X (aged 70 years) has agricultural income of ₹3,80,000 and non-agricultural income of, ₹3,00,000., Compute his tax liability for the assessment year 2022-23., Solution:, His tax liability shall be nil, since his non-agricultural income is ₹3,00,000 and partial integration is not, applicable., Illustration 24: Mrs. X has agricultural income of ₹1,00,000, income under the head salary amounting to ₹, 3,05,000, long term capital gain of ₹10,00,000 and casual income of ₹1,00,000 (winnings of a game show on, TV)., Compute her tax liability for the assessment year 2022-23. Her date of birth is 01.04.1962., Solution:, ₹, Computation of Total Income, Income under the head Salary, 3,05,000.00, Income under the head Capital Gains (LTCG), 10,00,000.00, Income under the head Other Sources (Casual Income), 1,00,000.00, Gross Total Income, 14,05,000.00, Less: Deduction u/s 80C to 80U, Nil, Total Income, 14,05,000.00, Agricultural Income, 1,00,000.00, Computation of Tax Liability, Tax on casual income ₹1,00,000 @ 30% u/s 115BB, 30,000.00, Tax on long term capital gain of ₹10,00,000 @ 20% u/s 112, 2,00,000.00, Normal income ₹3,05,000, Step 1. Tax on (3,05,000 + 1,00,000), 5,250.00, Step 2. Tax on (₹3,00,000 + 1,00,000) at slab rates, (5,000.00), Step 3. Deduct Tax at Step 2 from Tax at Step 1, 250.00, Tax before health & education cess, 2,30,250.00, Add: HEC @ 4%, 9,210.00, Tax Liability, 2,39,460.00, Note: 1. Casual income shall include all the activities as per section 2(24)(ix)., Illustration 25: Mr. X has income from business ₹102,00,000 and agricultural income ₹10,00,000., Compute his tax liability, Solution:, Total income, 102,00,000, Step 1. Tax on (agricultural + non agricultural income), i.e. Tax on ₹ 112,00,000/- at slab rates, 31,72,500, Step 2. Tax on (₹2,50,000 + agricultural income) at slab rates, (1,87,500), Step 3. Deduct Tax at Step 2 from Tax at Step 1, 29,85,000
Page 72 :
Computation of Total Income And Tax Liability, Add: Surcharge @ 15%, Tax + surcharge before HEC, Marginal relief, Working Note:, Tax + surcharge on income ₹102,00,000, Tax + surcharge if total income was ₹100 lakhs, 100,00,000 + 10,00,000 = 110,00,000, 31,12,500, 2,50,000 + 10,00,000 = 12,50,000, (1,87,500), Tax, 29,25,000, Tax + surcharge @10% on ₹100 lakhs, Increase in tax, Increase in income 102,00,000 – 100,00,000 =, Marginal relief (2,15,250 – 2,00,000), Tax before health & education cess, Add: HEC @ 4%, Tax Liability, , 72, 4,47,750, 34,32,750, (15,250), , 34,32,750, , (32,17,500), 2,15,250, (2,00,000), 15,250, 34,17,500, 1,36,700, 35,54,200, , Question 19: Write a note on taxability as per section 115BAC., Answer: An Individual or a Hindu undivided family shall be allowed to compute tax liability at the rates, given below. But in that case many of the exemptions and deductions listed u/s 115BAC shall not be, allowed., at the option of such person be computed at the rate of tax given below:, If total Income upto ₹2,50,000, Nil, On next ₹2,50,000, 5%, On next ₹2,50,000, 10%, On next ₹2,50,000, 15%, On next ₹2,50,000, 20%, On next ₹2,50,000, 25%, On Balance amount, 30%, Example, (i) Mr. X has total income of ₹6,00,000, (ii) Mr. X has total income of ₹8,00,000, (iii) Mr. X has total income of ₹10,00,000, (iv) Mr. X has total income of ₹12,00,000, (v) Mr. X has total income of ₹20,00,000, (b) Presume in above all cases Mr. X has opted for section 115BAC., Solution:, (i) Total income, On first ₹2,50,000, On next ₹2,50,000 @ 5%, On balance ₹1,00,000 @ 20%, Tax before health and education cess, Add: health & education cess @ 4%, Tax Liability, , 6,00,000, Nil, 12,500, 20,000, 32,500, 1,300, 33,800, , (b) Tax liability as per section 115BAC, Total income, On first ₹2,50,000, On next ₹2,50,000 @ 5%, On balance ₹1,00,000 @ 10%, Tax before health and education cess, Add: health & education cess @ 4%, Tax Liability, , 6,00,000, Nil, 12,500, 10,000, 22,500, 900, 23,400
Page 73 :
Computation of Total Income And Tax Liability, , 73, , (ii) Total income, On first ₹2,50,000, On next ₹2,50,000 @ 5%, On balance ₹3,00,000 @ 20%, Tax before health and education cess, Add: health & education cess @ 4%, Tax Liability, , 8,00,000, Nil, 12,500, 60,000, 72,500, 2,900, 75,400, , (b) Tax liability as per section 115BAC, Total income, On first ₹2,50,000, On next ₹2,50,000 @ 5%, On next ₹2,50,000 @ 10%, On balance ₹50,000 @ 15%, Tax before health and education cess, Add: health & education cess @ 4%, Tax Liability, , 8,00,000, Nil, 12,500, 25,000, 7,500, 45,000, 1,800, 46,800, , (iii) Total income, On first ₹2,50,000, On next ₹2,50,000 @ 5%, On balance ₹5,00,000 @ 20%, Tax before health and education cess, Add: health & education cess @ 4%, Tax Liability, , 10,00,000, Nil, 12,500, 1,00,000, 1,12,500, 4,500, 1,17,000, , (b) Tax liability as per section 115BAC, Total income, On first ₹2,50,000, On next ₹2,50,000 @ 5%, On next ₹2,50,000 @ 10%, On balance ₹2,50,000 @ 15%, Tax before health and education cess, Add: health & education cess @ 4%, Tax Liability, , 10,00,000, Nil, 12,500, 25,000, 37,500, 75,000, 3,000, 78,000, , (iv) Total income, On first ₹2,50,000, On next ₹2,50,000 @ 5%, On balance ₹5,00,000 @ 20%, On balance ₹2,00,000 @ 30%, Tax before health and education cess, Add: health & education cess @ 4%, Tax Liability, (b) Tax liability as per section 115BAC, Total income, On first ₹2,50,000, On next ₹2,50,000 @ 5%, On next ₹2,50,000 @ 10%, On next ₹2,50,000 @ 15%, On balance ₹2,00,000 @ 20%, Tax before health and education cess, , 12,00,000, Nil, 12,500, 1,00,000, 60,000, 1,72,500, 6,900, 1,79,400, 12,00,000, Nil, 12,500, 25,000, 37,500, 40,000, 1,15,000
Page 74 :
Computation of Total Income And Tax Liability, , 74, , Add: health & education cess @ 4%, Tax Liability, , 4,600, 1,19,600, , (v) Total income, On first ₹2,50,000, On next ₹2,50,000 @ 5%, On balance ₹5,00,000 @ 20%, On balance ₹10,00,000 @ 30%, Tax before health and education cess, Add: health & education cess @ 4%, Tax Liability, , 20,00,000, Nil, 12,500, 1,00,000, 3,00,000, 4,12,500, 16,500, 4,29,000, , (b) Tax liability as per section 115BAC, Total income, On first ₹2,50,000, On next ₹2,50,000 @ 5%, On next ₹2,50,000 @ 10%, On next ₹2,50,000 @ 15%, On next ₹2,50,000 @ 20%, On next ₹2,50,000 @ 25%, On balance ₹5,00,000 @ 30%, Tax before health and education cess, Add: health & education cess @ 4%, Tax Liability, , 20,00,000, Nil, 12,500, 25,000, 37,500, 50,000, 62,500, 1,50,000, 3,37,500, 13,500, 3,51,000, , Illustration 26:, Mr. X has income as given below:, Income under the head Salary, Income under the head House Property, Income under the head Business/Profession, Deductions allowed under section 80C to 80U are ₹1,10,000., Compute the income and the tax liability as per section 115BAC for previous year 2021-22., Solution:, Computation of Total Income of Mr. X, Previous Year 2021-22, Assessment Year 2022-23, Income under the head Salary, Income under the head House Property, Income under the Business/Profession, Gross Total Income, Less: Deduction u/s 80C to 80U (Not Allowed), Total Income, Rounded off u/s 288A, Computation of Tax Liability as per section 115BAC, On first ₹2,50,000, On next ₹2,50,000 @ 5%, On next ₹2,50,000 @ 10%, On next ₹2,50,000 @ 15%, On next ₹2,50,000 @ 20%, On next ₹2,50,000 @ 25%, On balance ₹30,250 @ 30%, Tax before health and education cess, Add: health & education cess @ 4%, Tax Liability, , 4,00,000, 5,00,000, 6,30,253, , ₹, 4,00,000.00, 5,00,000.00, 6,30,253.00, 15,30,253.00, Nil, 15,30,253.00, 15,30,250.00, Nil, 12,500, 25,000, 37,500, 50,000, 62,500, 9,075, 1,96,575, 7,863, 2,04,438
Page 75 :
Computation of Total Income And Tax Liability, Rounded off u/s 288B, , 75, 2,04,440, , Illustration 27:, Mr. X has income as given below:, Income under the head Salary, Income under the head House Property, Income under the head Business/Profession, Deductions allowed under section 80C to 80U are ₹1,10,000., Compute the income the tax liability as per section 115BAC for previous year 2021-22., Solution:, Computation of Total Income of Mr. X, Previous Year 2021-22, Assessment Year 2022-23, , 5,00,000, 6,00,000, 3,30,500, , ₹, 5,00,000.00, 6,00,000.00, 3,30,500.00, 14,30,500.00, Nil, 14,30,500.00, , Income under the head Salary, Income under the head House Property, Income under the Business/Profession, Gross Total Income, Less: Deduction u/s 80C to 80U (Not Allowed), Total Income, Computation of Tax Liability as per section 115BAC, On first ₹2,50,000, On next ₹2,50,000 @ 5%, On next ₹2,50,000 @ 10%, On next ₹2,50,000 @ 15%, On next ₹2,50,000 @ 20%, On balance ₹1,80,500 @ 25%, Tax before health and education cess, Add: health & education cess @ 4%, Tax Liability, , Nil, 12,500, 25,000, 37,500, 50,000, 45,125, 1,70,125, 6,805, 1,76,930, , Co-operative society, (i) Where the total income does not exceed ₹ 10,000, , 10% of the total income, , (ii) Where the total income exceeds ₹ 10,000 but does not, exceed ₹ 20,000, , ₹1,000 plus 20% of the amount by which, the total income exceeds ₹ 10,000, , (iii) Where the total income exceeds ₹ 20,000, , ₹3,000 plus 30% of the amount by which, the total income exceeds ₹ 20,000, , Note - Co-operative society, resident in India, can opt for concessional rate of tax @ 25.168% (i.e., tax @, 22% plus surcharge@10% plus health and education cess (HEC) @ 4%) under section 115BAD in respect, of its total income computed without giving effect to deduction under section 10AA, 32AD, 35AD, 35CCC,, additional depreciation under section 32(1) (iia), deductions under Chapter VI-A (other than section, 80JJAA) etc. and set off of loss and depreciation brought forward from earlier years relating to the above, deductions. The provisions of alternate minimum tax under section 115JC would not be applicable to cooperative society opting for section 115BAD., This section will be dealt with in detail at Final level.
Page 76 :
Computation of Total Income And Tax Liability, , 76, , MULTIPLE CHOICE QUESTIONS, 1. The basic source of income-tax law is (a) Income-tax Act, 1961, (b) Income-tax Rules, 1962, (c) Circulars/Notifications issued by CBDT, (d) Judgments of Courts, 2. A domestic company means (a) Only an Indian company, (b) Both Indian company and a foreign company having a branch in India, (c) Both Indian company and a foreign company having business connection in India, (d) Both Indian company and a foreign company which has made the prescribed arrangement for declaration, and payment of dividends in India out of the income chargeable to tax in India, 3. The rates of income-tax are mentioned in (a) The Income-tax Act, 1961 only, (b) Both Income-tax Act, 1961 and Income-tax Rules, 1962, (c) The First Schedule to the Annual Finance Act, (d) Both Income-tax Act, 1961 and the First Schedule to the Annual Finance Act, 4. The surcharge applicable in the case of an individual is (a) 10% of tax payable if total income exceeds ₹50 lakhs but does not exceed ₹1 crore, (b) 10% of tax payable if total income exceeds ₹1 crore but does not exceed ₹2 crore, (c) 15% of tax payable if total income exceeds ₹1 crore but does not exceed ₹2 crore, (d) Both (a) and (c), as the case may be., 5. In respect of a non-resident assessee, who is of the age of 60 years or more but less than 80 years at, any time during the previous year 2021-22, (a) Basic exemption of ₹2,50,000 is available, (b) Basic exemption of ₹3,00,000 is available, (c) Basic exemption of ₹5,00,000 is available, (d) No basic exemption limit would be available, 6. In case of a domestic company whose gross receipts for the P.Y. 2018-19 is upto ₹400 crores, the, rate of tax applicable is (a) 29% , (b) 25% , (c) 30% , (d) None of the above, 7. The surcharge applicable to a domestic company for A.Y. 2022-23 is (a) 5%, if total income exceeds ₹1 crore., (b) 10%, if the total income exceeds ₹1 crore, (c) 7%, if the total income exceeds ₹1 crore but does not exceed ₹10 crore, and 15%, if the total income, exceeds ₹10 crore., (d) 7%, if the total income exceeds ₹1 crore but does not exceed ₹10 crore, and 12%, if the total income, exceeds ₹10 crore., 8. The surcharge applicable to a foreign company for A.Y. 2022-23 is (a) 5%, if the total income exceeds ₹1 crore., (b) 10%, if the total income exceeds ₹1 crore., (c) 2%, if the total income exceeds ₹1 crore but does not exceed ₹10 crore and 5% if the total income, exceeds ₹10 crore., (d) 2%, if the total income exceeds ₹10 crore., 9. The rate of tax applicable to a partnership firm for A.Y. 2022-23 is (a) 25%, (b) 30%, (c) 35%, (d) 40%, 10. Where the total income of an artificial juridical person is ₹3,10,000, the income-tax before cess, payable is ₹............... and surcharge payable is ₹.............., (a) ₹3,000; surcharge – nil.
Page 77 :
Computation of Total Income And Tax Liability, , 77, , (b) ₹6,000; surcharge – nil., (c) ₹500; surcharge – nil, (d) ₹93,000; surcharge – ₹4650, 11. What is the basic exemption limit for a woman assessee for A.Y. 2022-23, who turned 60 years on, 31.03.2022?, (a) ₹2,00,000, (b) ₹3,00,000, (c) ₹2,50,000, (d) ₹5,00,000, 12. What is the rate of surcharge applicable to individuals having total income exceeding ₹1 crore but, does not exceed ₹2 crore?, (a) 15%, (b) 12% , (c) 10% , (d) 2%, 13. What is the basic exemption limit for Mrs. X, a resident individual who is of the age of 80 years as, on 31.3.2022?, (a) ₹5,00,000, (b) ₹2,40,000, (c) ₹3,00,000, (d) ₹2,50,000, 14. Tax Liability of a resident individual having LTCG 3.5 Lakh shall be, (a) ₹7,800, (b) ₹72,800, (c) ₹18,200, (d) ₹20,800, 15. The maximum amount of rebate allowable under section 87A for A.Y. 2022-23 is (a) ₹2,000, if the total income does not exceed ₹5 lakh, (b) ₹5,000, if the total income does not exceed ₹5 lakh, (c) ₹12,500, if the total income does not exceed ₹5 lakh, (d) ₹2,500,if the total income does not exceed ₹3.5 lakh, 16. If Mr. Y’s total income for A.Y. 2022-23 is ₹52 Lakhs, surcharge is payable at the rate of (a) 15%, (b) 12%, (c) 10%, (d) 2%, 17. Unexhausted basic exemption limit of a non-resident individual can be adjusted against –, (a) only LTCG taxable @20% u/s 112, (b) only STCG taxable @15% u/s 111A, (c) only LTCG taxable @ 10% u/s 112A, (d) casual income taxable @30% u/s 115BB, (e) none of these, 18. Unexhausted basic exemption limit of a resident individual can be adjusted against –, (a) only LTCG taxable @20% u/s 112, (b) only STCG taxable @15% u/s 111A, (c) only LTCG taxable @ 10% u/s 112A, (d) From (a) or (b) or (c), (e) Casual income taxable @ 30% u/s 115BB, 19. The concept of partial integration of agricultural income with non-agricultural income is, applicable to (a) only individuals & HUF, (b) only firms and companies, (c) Individuals, HUF, AOPs/BOIs & Artificial juridical persons, (d) All persons
Page 78 :
Computation of Total Income And Tax Liability, , 78, , 20. What is the basic exemption limit for Mr. X, a resident individual who has completed the age of, 60 years as on 31.03.2023?, (a) ₹5,00,000, (b) ₹2,40,000, (c) ₹3,00,000, (d) ₹2,50,000, 21. Rebate u/s 87A shall be allowed to, (a) all persons, (b) only individual, (c) only resident individual, (d) resident individual & HUF, 22. Rebate u/s 87A shall be allowed if total income is, (a) less than ₹5,00,000, (b) less than ₹3,50,000, (c) upto ₹5,00,000, (d) upto ₹3,50,000, 23. Marginal relief shall be allowed to, (a) all persons, (b) only individual, (c) individual & HUF, (d) non -resident, 24. Mr. X has agricultural Income of ₹4,900 and non – agricultural income of ₹2,65,000. Tax Payable, shall be, (a) Nil, (b) ₹780, (c) ₹1,030, (d) ₹14,030, 25. Mr. X has agricultural Income of ₹1,00,000 and non – agricultural income of ₹2,45,000. Tax, Payable shall be, (a) Nil, (b) ₹4,940, (c) ₹2,340, (d) ₹17,940, 26. Tax Liability of a resident individual having only STCG 111A 3.5 Lakh shall be, (a) ₹2,600, (b) ₹15,600, (c) ₹54,600, (d) ₹13,000, 27. Tax Liability of a resident individual having only casual income 3.5 Lakh shall be, (a) ₹96,200, (b) ₹18,200, (c) ₹31,200, (d) ₹1,09,200, 28. Tax Liability of a non - resident individual having only STCG 111A 3.5 Lakh shall be, (a) ₹15,600, (b) ₹2,600, (c) ₹54,600, (d) ₹52,000, 29. The rate of tax applicable to a limited liability partnership (LLP) for A.Y. 2022- 23 is (a) 25%, (b) 30%, (c) 40%, (d) at slab rate
Page 79 :
Computation of Total Income And Tax Liability, , 79, , 30. Mr. Devansh has agricultural income of ₹2,30,000 and business income of ₹2,45,000. Which of the, following statements are correct?, (a) Agricultural income has to be aggregated with business income for tax rate purposes, (b) No aggregation is required since agricultural income is less than basic exemption limit., (c) No aggregation is required since business income is less than basic exemption limit., (d) Agricultural income is exempt under section 10(1) but the same has to be aggregated with business, income, since it exceeds ₹5,000, , Check Solution given on our website www.mkgeducation.com
Page 80 :
Computation of Total Income And Tax Liability, , 80, , PRACTICE PROBLEMS, TOTAL PROBLEMS 18, Problem 1., Compute tax liability in the following cases for the assessment year 2022-23., (i), Mr. X (resident) has total income of ₹22,00,000, (ii), Mr. X (non-resident) has total income of ₹22,00,000, (iii), Mrs. X (resident) has total income of ₹22,00,000, (iv), Mrs. X (non-resident) has total income of ₹22,00,000, (v), Mr. X (resident), aged 60 years has total income of ₹22,00,000, (vi), Mrs. X (resident), aged 60 years has total income of ₹22,00,000, (vii) Mr. X (non-resident), aged 60 years has total income of ₹22,00,000, (viii) Mrs. X (non-resident), aged 60 years has total income of ₹22,00,000, (ix), Mr. X (resident), aged 80 years has total income of ₹22,00,000, (x), Mrs. X (resident), aged 80 years has total income of ₹22,00,000, (xi), Mr. X (non-resident), aged 80 years has total income of ₹22,00,000, (xii) Mrs. X (non-resident), aged 80 years has total income of ₹22,00,000, Answer = (i) Tax Liability: ₹4,91,400; (ii) ₹4,91,400; (iii) ₹4,91,400; (iv) ₹4,91,400; (v) ₹4,88,800; (vi), ₹4,88,800; (vii) ₹4,91,400; (viii) ₹4,91,400; (ix) ₹4,78,400; (x) ₹4,78,400; (xi) ₹4,91,400; (xii) ₹4,91,400, Problem 2., Compute tax liability in the following cases for the assessment year 2022-23., (i), Mr. X (resident) has total income of ₹100,05,000, (ii), Mr. X (non-resident) has total income of ₹102,00,000, (iii) Mrs. X (resident) has total income of ₹90,00,000, (iv) Mrs. X (non-resident) has total income of ₹4,98,000, (v), Mr. X (resident), aged 60 years has total income of ₹4,05,000, (vi) Mrs. X (resident), aged 60 years has total income of ₹102,05,000, (vii) Mr. X (non-resident), aged 60 years has total income of ₹25,00,000, (viii) Mrs. X (non-resident), aged 60 years has total income of ₹4,50,000, (ix) Mr. X (resident), aged 80 years has total income of ₹3,80,000, (x), Mrs. X (resident), aged 80 years has total income of ₹110,00,000, (xi) Mr. X (non-resident), aged 80 years has total income of ₹99,99,000, (xii) Mrs. X (non-resident), aged 80 years has total income of ₹12,00,000, (xiii) Mr. X (resident) has total income of ₹202,00,000, (xiv) Mr. X (resident) has total income of ₹502,00,000, Answer = (i) Tax Liability: ₹32,22,700; (ii) ₹34,25,500; (iii) ₹28,74,300; (iv) ₹12,900; (v) Nil (vi), ₹34,27,840; (vii) ₹5,85,000; (viii) ₹10,400; (ix) Nil ; (x) ₹37,07,600; (xi) ₹32,17,160; (xii) ₹1,79,400; (xiii), ₹71,59,750; (xiv) ₹1,94,64,250, Problem 3., Compute tax liability in the following cases for the assessment year 2022-23., (i) Mr. X (resident) has total income of ₹50,20,000, (ii) Mr. X (non-resident) has total income of ₹53,00,000, (iii) Mrs. X (resident), aged 60 years has total income of ₹51,00,000, (iv) Mr. X (resident), aged 80 years has total income of ₹54,00,000, (v) Mr. X (non-resident), aged 80 years has total income of ₹51,22,000, (vi) Mrs. X (non-resident), has total income of ₹50,80,000, Answer = (i) Tax Liability: ₹13,85,800; (ii) ₹16,04,460; (iii) ₹14,66,400; (iv) ₹16,24,480; (v) ₹14,91,880;, (vi) ₹14,48,200
Page 81 :
Computation of Total Income And Tax Liability, , 81, , Problem 4., Compute tax liability in the following cases for the assessment year 2022-23., (i), Mr. X (resident) has total income of ₹4,90,000, (ii), Mr. X (non-resident) has total income of ₹4,90,000, (iii), Mrs. X (resident) has total income of ₹4,90,000, (iv), Mrs. X (non-resident) has total income of ₹4,90,000, (v), Mr. X (resident), aged 60 years has total income of ₹4,90,000, (vi), Mrs. X (resident), aged 60 years has total income of ₹4,90,000, (vii) Mr. X (non-resident), aged 60 years has total income of ₹4,90,000, (viii) Mrs. X (non-resident), aged 60 years has total income of ₹4,90,000, (ix), Mr. X (resident), aged 80 years has total income of ₹4,90,000, (x), Mrs. X (resident), aged 80 years has total income of ₹4,90,000, (xi), Mr. X (non-resident), aged 80 years has total income of ₹4,90,000, (xii) Mrs. X (non-resident), aged 80 years has total income of ₹4,90,000, Answer = (i) Tax Liability: Nil; (ii) ₹12,480; (iii) Nil; (iv) ₹12,480; (v) Nil; (vi) Nil; (vii) ₹12,480; (viii), ₹12,480; (ix) Nil; (x) Nil; (xi) ₹12,480; (xii) ₹12,480, Problem 5., Mr. X has income asunder:, • Income under the head salary, • Income under the head house property, • Income under the head business/profession, • Long term capital gains, • Short term capital gains, • Casual income (winnings of lottery), Deductions allowed under section 80C to 80U, Compute his tax liability for the assessment year 2022-23., Answer = Tax Liability: ₹1,02,960, , ₹, 2,40,000, 1,55,000, 3,30,000, 1,20,000, 35,000, 65,000, 45,000, , (b) Presume the assessee is Mrs. X., Answer = Tax Liability: ₹1,02,960, (c) Presume the assessee is Mr. X, aged 65 years., Answer = Tax Liability: ₹1,00,360, (d) Presume the assessee is Mrs. X (non-resident)., Answer = Tax Liability: ₹1,02,960, (e) Presume the assessee is Mr. X (non-resident) aged 65 years., Answer = Tax Liability: ₹1,02,960, (f) Presume the assessee is Mr. X, aged 85 years., Answer = Tax Liability: ₹89,960, (g) Presume the assessee is Mr. X (non-resident) aged 85 years., Answer = Tax Liability: ₹1,02,960, Problem 6., Compute tax liability for the assessment year 2022-23 in the following situations:, (i) Mr. X is resident in India and has income under the head house property ₹50,000 and income under the, head salary ₹30,000 and long term capital gains ₹8,00,000., (ii) Presume in the above situation the assessee is Mrs. X., (iii) Presume in the above situation the assessee is Mrs. X and she is aged about 70 years., (iv) Presume in the above situation the assessee is Mr. X and he is aged about 70 years., (v) Presume in the above situation the assessee is Mrs. X and she is aged about 87 years., (vi) Presume in the above situation the assessee is Mr. X and he is aged about 87 years.
Page 82 :
Computation of Total Income And Tax Liability, , 82, , (vii) Presume in all the above situations, the assessee is non-resident in India., Answer = (i) ₹1,31,040; (ii) ₹1,31,040; (iii) ₹1,20,640; (iv) ₹1,20,640; (v) ₹79,040; (vi) ₹79,040; (vii), Situation (i): ₹1,66,400; Situation (ii): ₹1,66,400; Situation (iii): ₹1,66,400; Situation (iv): ₹1,66,400; (v):, ₹1,66,400; Situation (vi): ₹1,66,400, Problem 7., Compute tax liability for the assessment year 2022-23 in the following situations:, (i) Mr. X is resident in India and his incomes are as follows:, (a) Income under the head Salary ₹90,000, (b) Income under the head House Property ₹60,000, (c) Long term capital gains ₹2,30,000, (d) Short term capital gain under section 111A ₹2,40,000, (e) Casual Income ₹70,000, (f) Deduction under section 80C to 80U ₹2,00,000 ., (ii) Presume in the above situation the assessee is Mrs. X., (iii) Presume in the above situation the assessee is Mrs. X and she is aged about 70 years., (iv) Presume in the above situation the assessee is Mr. X and he is aged about 70 years., (v) Presume in the above situation the assessee is Mr. X and he is aged about 70 years old and he is nonresident., (vi) Presume in the above situation the assessee is Mrs. X and she is aged about 82 years., (vii) Presume in the above situation the assessee is Mr. X and he is aged about 82 years., (viii) Presume in the above situation the assessee is Mr. X and he is aged about 82 years old and he is nonresident., Answer = (i) ₹56,160; (ii) ₹56,160; (iii) ₹48,360; (iv) ₹48,360; (v) ₹1,07,120; (vi) ₹21,840; (vii) ₹21,840;, (viii) ₹1,07,120, Problem 8, Compute tax liability in the following situations:, (i), Mrs. X has income under the head House Property ₹8,42,324., (ii), Mr. X has income under the head Business Profession ₹14,42,336., (iii) Mr. X aged 66 years has long term capital gains ₹11,35,335., (iv), Mr. X has long term capital gains of ₹13,35,334.90., (v), Mrs. X has short term capital gains under section 111A ₹10,20,335., (vi), Mrs. X, non-resident, has long term capital gains ₹5,40,337., (vii) Mr. X, non-resident, aged about 66 years has winning of a lottery ₹7,20,000., (viii) Mr. X aged 86 years has long term capital gains ₹15,65,385., (ix), Mr. X, non-resident, aged about 90 years has winning of a lottery ₹10,20,000., Answer = (i) ₹84,200; (ii) ₹2,55,010; (iii) ₹1,73,750; (iv) ₹2,25,750; (v) ₹1,20,170; (vi) ₹1,12,390; (vii), ₹2,24,640; (viii) ₹2,21,600; (ix) ₹3,18,240, Problem 9., Compute tax liability of ABC Ltd. a domestic company in the following situations for assessment year, 2022-23:, (i) The company has income under the head Business/Profession ₹70,000., (ii) The company has income under the head Business/Profession ₹150,00,000., (iii) The company has income under the head Business/Profession ₹6,00,000., (iv) The company has income under the head Business/Profession ₹10,30,000., (v) The company has long term capital gains of ₹700,00,000., (vi) The company has long term capital gains of ₹1,50,000., (vii) The company has long term capital gains of ₹6,00,000., (viii) The company has long term capital gains of ₹10,30,000., (ix) The company has casual income ₹400,00,000., Answer = (i) Tax Liability: ₹21,840; (ii) ₹50,07,600; (iii) ₹1,87,200; (iv) ₹3,21,360; (v) ₹155,79,200; (vi)
Page 83 :
Computation of Total Income And Tax Liability, , 83, , ₹31,200; (vii) ₹1,24,800; (viii) ₹2,14,240; (ix) ₹133,53,600, (b) Presume all the above situations the company is a foreign company., Answer = (i) Tax Liability: ₹29,120; (ii) ₹63,64,800; (iii) ₹2,49,600; (iv) ₹4,28,480; (v) ₹148,51,200; (vi), ₹31,200; (vii) ₹1,24,800; (viii) ₹2,14,240; (ix) ₹127,29,600, Problem 10., ABC (HUF) has incomes as given below:, 1. Income under the head Business/Profession ₹6,00,000, 2. Income under the head House Property ₹4,00,000, 3. Long term capital gains ₹4,50,000, 4. Short term capital gains under section 111A ₹3,50,000, 5. Casual Income ₹3,50,000, 6. Deductions allowed under section 80C to 80U ₹1,25,000, Compute tax liability of HUF for the assessment year 2022-23., Answer = Tax Liability: ₹3,48,400, Problem 11., Mr. X has income under the head salary ₹3,00,000 and income under the head house property ₹16,00,000, and long term capital gain ₹1,00,000 and agricultural income ₹4,00,000. Deductions allowed under section, 80C to 80U ₹60,000., Compute his income and tax liability for Assessment Year 2022-23., Answer = Total Income: ₹19,40,000; Tax Liability: ₹4,80,480, (b) Presume assessee is Mrs. X and is aged 64 years., Answer = Total Income: ₹19,40,000; Tax Liability: ₹4,70,080, Problem 12., Mr. X has income under the head house property ₹3,00,000 and long term capital gain ₹5,00,000 and, agricultural income ₹3,00,000. Deductions under section 80C to 80U ₹1,00,000., Compute his income and tax liability for Assessment Year 2022-23., Answer = Total Income: ₹7,00,000; Tax Liability: ₹93,600, (b) Presume Mr. X is aged 82 years, Answer = Total Income: ₹7,00,000; Tax Liability: ₹41,600, Problem 13., Mrs. X has casual income ₹5,00,000 and short term capital gain under section 111A ₹7,00,000 and, agricultural income ₹3,00,000., Compute her tax liability for Assessment Year 2022-23., Answer = Tax Liability: ₹2,26,200, (b) Presume she is non-resident and is aged 63 years., Answer = Tax Liability: ₹2,65,200, Problem 14., Mr. X has agricultural income ₹10,00,000 and income from business ₹12,00,000 and casual income, ₹5,00,000 and he has completed the age of 80 years on 31.03.2022. Compute his tax liability Assessment, Year 2022-23., Answer = Tax Liability: ₹3,74,400
Page 84 :
Computation of Total Income And Tax Liability, Problem 15., Mrs. X has income as given below:, Income under the head Salary, Income under the head House Property, Short Term Capital Gain, Short Term Capital Gain111A, Long Term Capital Gain, Casual Income, Deduction u/s 80C to 80U, Agricultural Income, Compute her Tax Liability for the A.Y.2022-23, Answer = Tax Liability: ₹1,02,960, , 84, ₹, 3,00,000, 1,00,000, 50,000, 2,00,000, 1,50,000, 70,000, 1,10,000, 5,00,000, , (b) Presume in above she is aged 81 years., Answer = Tax Liability: ₹51,480, (c) Presume in (a) above she is Non Resident and deduction u/s 80C-80U is ₹3,00,000., Answer = Tax Liability: ₹84,240, Problem 16., Mrs. X has income under the head house property ₹2,00,000 and long term capital gain ₹10,00,000 and, agricultural income ₹7,00,000. Deduction under section 80C to 80U ₹60,000., Compute her income and tax liability for Assessment Year 2022-23., Answer = Total Income: ₹11,40,000; Tax Liability: ₹1,85,120, (b) Presume Mrs. X is aged 79 years and income under the head house property is ₹10,00,000, Answer = Total Income: ₹19,40,000; Tax Liability: ₹4,07,680, Problem 17., ABC (HUF) has incomes as given below:, 1. Income under the head Business/Profession ₹6,00,000, 2. Income under the head House Property ₹4,00,000, 3. Long term capital gains ₹4,50,000, 4. Short term capital gains under section 111A ₹3,50,000, 5. Casual Income ₹3,50,000, 6. Deductions allowed under section 80C to 80U ₹1,25,000, Compute tax liability of HUF for the assessment year 2022-23 as per section 115BAC, Answer = Tax Liability: 3,35,400, Problem 18., Mrs. X has income as given below:, Income under the head Salary, Income under the head House Property, Income under the head business profession, Deduction u/s 80C to 80U, Compute her Tax Liability as per section 115BAC for the A.Y.2022-23, Answer = Tax Liability: ₹1,43,000, , ₹, 3,00,000, 1,00,000, 9,00,000, 1,00,000, , Check Detailed Solution given on our website www.mkgeducation.com
Page 85 :
Computation of Total Income And Tax Liability, , 85, , EXAMINATION QUESTIONS, MAY – 2019 (OLD COURSE), Question 5 (a), Marks 3, Miss Himanshi (68 years) is a resident individual. During the assessment year 2022-23, she has income from, Long-term capital gain on transfer of equity shares ₹3,80,000 (Securities transaction has been paid on, acquisition and transfer of the said shares) and income from Other sources ₹ 2,75,000., Compute her tax liability for Assessment year 2022-23., Solution:, Computation of Total Income, Long term capital gains u/s 112A, Income from other sources, Gross Total Income, Less: Deduction u/s 80C to 80U, Total Income, , ₹, 3,80,000, 2,75,000, 6,55,000, Nil, 6,55,000, , Computation of Tax Liability, Tax on Loan term capital gains 2,55,000 (3,80,000-1,00,000-25,000) @ 10% u/s 112A, Tax on normal income Nil (2,75,000-2,75,000), Tax before health & education cess, Add: HEC @ 4%, Tax Liability, , 25,500, Nil, 25,500, 1,020, 26,520, , Note:, 1. Rebate u/s 87A shall not be allowed since income is more than ₹5,00,000., 2. Basic exemption allowed is ₹3,00,000 as Miss Himanshi is 68 years old., NOV – 2018 (NEW COURSE), Question 6 (c), Marks 5, Mr. Rajat Saini, aged 32 years, furnishes the following details of his total income for the A.Y. 2022-23:, Income under the head Salary, 27,88,000, Income under the head House Property, 15,80,000, Income under the head Other sources, 7,22,000, He has not claimed any deduction under chapter VIA. You are required to compute tax liability of Mr. Rajat, Saini as per the provisions of Income Tax Act, 1961., Solution:, Computation of Total Income, ₹, Income under the head Salary, 27,88,000, Income under the head House Property, 15,80,000, Income under the head Other sources, 7,22,000, Gross Total Income, 50,90,000, Less: Deduction u/s 80C to 80U, Nil, Total Income, 50,90,000, Computation of Tax Liability, Total Income, 50,90,000, Tax on ₹50,90,000 at slab rate, 13,39,500, Add: Surcharge @ 10%, 1,33,950, Tax before marginal relief, 14,73,450
Page 86 :
Computation of Total Income And Tax Liability, Less: Marginal Relief, Working Note:, Tax + surcharge on income of ₹50,90,000, Tax on income of ₹50,00,000, Increase in tax, Increase in income, Marginal Relief (1,60,950 –90,000), Tax after marginal relief, Add: HEC @ 4%, Tax Liability, , 86, (70,950), , 14,73,450, (13,12,500), 1,60,950, 90,000, 70,950, 14,02,500, 56,100, 14,58,600, , NOV – 2011, Question 7, (4 Marks), Discuss the taxability of agricultural income under the Income Tax Act, 1961. How will income be, computed where an individual derives agricultural and non-agricultural income?, Answer: Under section 10(1), any agricultural income in India is fully exempt from income tax but if the, agricultural income is from outside India, it is chargeable to tax., Indirect taxing of agricultural income or partial integration of agricultural income (Under the, constitution, the power to levy a tax on agricultural income vests in the states. However, parliament, has also levied a tax on such income. Explain how this has been achieved?), If any person has agricultural income as well as non-agricultural income, his tax liability shall be computed, in the manner given below:, 1. Compute tax on the total of agricultural income and non- agricultural income considering it to be total, income of the assessee., 2. Compute tax on exemption limit (₹2,50,000 / 3,00,000 / 5,00,000) and agricultural income considering, it to be total income., 3. Deduct tax computed under Step 2 from Step 1 and apply health & education cess., 4. Long term capital gain, casual income and short term capital gain u/s 111A shall not be taken into, consideration for the purpose of partial integration, 5. If Agricultural income is upto ₹5,000, or non-agricultural income is upto the limit not chargeable to tax, (₹2,50,000/3,00,000/5,00,000), partial integration is not applicable., 6. Partial integration is not applicable in case of a partnership firm or a company., MAY – 2007, Question 3, The broad break-up of tax and allied details of Mrs. X, born on 31st March, 1962 are as under:, Long-term capital gains on sale of house, Short-term capital gains on sale of shares in B Ltd. (STT paid), Prize winning from a T.V. show, Business income, Net agricultural income, Deduction allowed under section 80C to 80U, Compute the tax payable by Mrs. X for the assessment year 2022-23., Answer., Computation of Total Income, Business Income, Long term capital gain on sale of house, Short-term capital gains on sale of shares in B Ltd. (STT paid), Casual Income (Prize winning from a T.V. show), Gross Total Income, Less: Deduction u/s 80C to 80U, Total Income, , (4 Marks), ₹, 2,00,000, 30,000, 20,000, 2,90,000, 4,40,000, 60,000, 2,90,000, 2,00,000, 30,000, 20,000, 5,40,000, (60,000), 4,80,000
Page 87 :
Computation of Total Income And Tax Liability, Computation of tax payable by Mrs. X for the A.Y. 2022-23, Particulars, (i) Tax on long-term capital gain of ₹1,30,000 (2,00,000 – 70,000) @ 20%, (ii) Tax on short term capital gain of ₹30,000 @ 15%, (iii) Tax on winnings of ₹20,000 from a T.V. show @ 30%, (iv) Tax on balance income of ₹2,30,000 at slab rate, Deficiency of ₹70,000 has been allowed from LTCG, Amount of tax before Rebate, Less: Rebate u/s 87A, Tax before HEC, Add: HEC @ 4%, Tax payable by Mrs. X, , 87, ₹, 26,000, 4,500, 6,000, Nil, 36,500, (12,500), 24,000, 960, 24,960, , (i) Mrs. X has completed 60 years of age on 31st March, 2022 i.e. she has completed the age of 60 years on, during the previous year. Therefore, she is entitled to the higher basic exemption limit of ₹3,00,000., (ii) Partial integration is not applicable because her non-agricultural income is not exceeding the exemption, limit of ₹3,00,000., (b) Presume income from business is ₹5,00,000., Computation of Gross Total Income, Business Income, 5,00,000, Long term capital gain on sale of house, 2,00,000, Short-term capital gains on sale of shares in B Ltd. (STT paid), 30,000, Casual Income (Prize winning from a T.V. show), 20,000, Gross Total Income, 7,50,000, Less: Deduction u/s 80C to 80U, (60,000), Total Income, 6,90,000, Computation of tax payable by Mrs. X for the A.Y. 2022-23, ₹, (i) Tax on long-term capital gain of ₹2,00,000 @ 20%, 40,000, (ii) Tax on short term capital gain of ₹30,000 @ 15%, 4,500, (iii) Tax on winnings of ₹20,000 from a T.V. show @ 30%, 6,000, (iv) Tax on balance income of ₹4,40,000, 1. Tax on 4,40,000 + 4,40,000 = 8,80,000, 86,000, 2. Tax on 3,00,000 + 4,40,000 = 7,40,000, (58,000), Tax at step no.1 minus tax at step no.2, 28,000, Amount of tax before HEC, 78,500, Add: HEC @ 4%, 3,140, Tax payable by Mrs. X, 81,640, Mrs. X has completed 60 years of age on 31st March, 2022 i.e. she has completed the age of 60 years during, the previous year. Therefore, she is entitled to the higher basic exemption limit of ₹3,00,000., MAY – 2006, Question 2, (3 Marks), (a) The total income of Mrs. X computed for the assessment year 2022-23 is ₹3,80,000, which includes the, following:, ₹, Long-term capital gains, 30,000, Winnings from lotteries, 20,000, Short-term capital gains covered by Sec. 111A, 10,000, Agricultural income earned by her was ₹50,000., Compute the tax payable by Mrs. X., Solution:, Computation of tax payable by Mrs. X for the A.Y. 2022-23, ₹, Tax on lottery income of ₹20,000 @ 30%, 6,000, Tax on long-term capital gain of ₹30,000 @ 20%, 6,000, Tax on short-term capital gain covered u/s 111A @ 15% of ₹10,000, 1,500
Page 88 :
Computation of Total Income And Tax Liability, Normal income ₹3,20,000, Tax on (₹3,20,000 + ₹50,000) at slab rate, Tax on (₹2,50,000 + ₹50,000) at slab rate, Tax on normal income (6,000 – 2,500), Tax before rebate, Less: Rebate u/s 87A, Tax before HEC, Add: HEC @ 4%, Tax payable by Mrs. X, , 88, 6,000, (2,500), 3,500, 17,000, (12,500), 4,500, 180, 4,680
Page 89 :
Taxability of Gift, , 89, , TAXABILITY OF GIFT, SECTION 56, TAXABILITY OF GIFT, SECTION 56(2)(x), Question 1: Explain taxability of gift., Answer:, Gift received by any person shall be taxable and the gifts shall be divided into 3 parts., 1. Gift of sum of money, 2. Gift of any property other than immovable property, 3. Gift of immovable property, Taxability is as given below:, 1. Gift of sum of money, If any person has received any sum of money from one or more persons without consideration and the, aggregate value of all such gifts received during the year exceeds fifty thousand rupees, the whole of the, aggregate value of such sum shall be taxable under the head Other Sources but if the aggregate value is upto, ₹50,000, entire amount shall be exempt from income tax. E.g. Mr. X has received 3 gifts of ₹15,000 each, from his 3 friends, entire amount of ₹45,000 is exempt from income tax but if he has received 3 gifts of, ₹20,000 each, entire amount of ₹60,000 shall be taxable. Further it will be considered to be normal income., 2. Gift of any property other than immovable property, If any person has received gift of any property other than immovable property without consideration and the, aggregate fair market value of such properties received during a particular year exceeds ₹50,000, it will be, taxable under the head Other Sources but if aggregate value of all such properties is upto ₹50,000, it will be, exempt from income tax., If the consideration is less than the aggregate fair market value of such properties by an amount exceeding, ₹50,000, aggregate fair market value as exceeds such consideration shall be taxable under the head Other, Sources. Further it will be considered to be normal income., 3. Gift of immovable property, If any person has received any immovable property without consideration, it will be exempt if stamp duty, value is upto ₹50,000 but if the stamp duty value exceeds fifty thousand rupees, entire stamp duty value, shall be taxable under the head Other Sources. Value of individual immovable property shall be taken into, consideration instead of aggregate value of all such properties., (If any person is selling immovable property, its Conveyance Deed shall be prepared in the office of, Registrar and some tax has to be paid to the State Government for transferring the property and it is called, stamp duty and the value on which such duty is charged is called stamp duty value (also called circle rate)., A person may not disclose the right value hence the value is determined by State Government.), If immovable property has been received for a consideration which is less than the stamp duty value of the, property by an amount exceeding fifty thousand rupees and also stamp duty value is exceeding by more than, 10% of the actual consideration, in such cases taxable amount shall be the stamp duty value of such, property as exceeds such consideration., Example, (i) Mr. X purchased immovable property for ₹3,00,000 but stamp duty value is ₹5,00,000, taxable amount, shall be ₹2,00,000, (ii) Mr. X has sold immovable property to Mr. Y for ₹100,00,000 but stamp duty value is ₹110,00,000, in, this taxable amount shall be Nil because stamp duty value is not exceeding the actual consideration by more
Page 90 :
Taxability of Gift, , 90, , than 10% but if stamp duty value is ₹ 111,00,000, taxable amount shall be ₹ 11,00,000 because stamp duty, is exceeding by more than 10% of actual consideration., If the date of the agreement fixing the amount of consideration for the transfer of immovable property and, the date of registration are not the same and in such cases, the stamp duty value on the date of the agreement, shall be taken into consideration but part of consideration should have been paid by account payee cheque,, an account payee bank draft or by use of electronic clearing system through a bank account or through, such other electronic modes as may be prescribed. (Other electronic mode means Credit Card, Debit, Card, Net Banking, IMPS (Immediate Payment Service), UPI (Unified Payment Interface), RTGS (Real, Time Gross Settlement), NEFT (National Electronic Funds Transfer), and BHIM (Bharat Interface for, Money) Aadhaar Pay) on or before the date of agreement. E.g. Mr. X has entered into agreement with a, builder ABC Limited on 01.07.2016 for purchase of one building for ₹20,00,000 but stamp duty value was, ₹27,00,000 and advance of ₹3,00,000 was given by cheque but property was transferred in his name on, 01.07.2021 and on that date stamp duty value was ₹35,00,000, in this case amount of gift shall be ₹7,00,000, (27,00,000 – 20,00,000). (Difference amount is more than ₹50,000 and more than 10% of the consideration)., Similarly, it will also be considered to be normal income., The gift is exempt in the following cases, (a) If any individual has received any gift from any of his relative, it will be exempt from income tax. The, term relative shall include, (a) spouse of the individual;, (b) brother or sister of the individual;, (c) brother or sister of the spouse of the individual;, (d) brother or sister of either of the parents of the individual;, (e) any lineal ascendant or descendant of the individual; (ascendant means mother/ father/ grand mother /, grand father and so on: Descendant means son / daughter / grand son / grand daughter etc., (f) any lineal ascendant or descendant of the spouse of the individual;, (g) spouse of the person referred to in items (b) to (f), Whether mother’s parents shall be included in lineal ascended is a question of law., (b) If any individual has received any gift from any person of any amount on the occasion of his/her, marriage. If gift is received by the parents of such individual, in that case it will be taxable. If any individual, has received gift on the occasion of anniversary, it will be taxable., (c) If any person has received any gift under a will/ inheritance, it will be exempt from income tax., (d) in contemplation of death of the payer or donor (Contemplation of Death means the apprehension of an, individual that his life will end in the immediate future by a particular illness etc.), (e) from any local authority or charitable hospital or charitable educational institution or charitable trust or, other similar organisation., Question 2: Explain meaning of property., Answer:, "PROPERTY" means the following capital asset of the assessee, namely:—, (i) immovable property being land or building or both;, (ii) shares and securities;, (iii) jewellery;, (iv) archaeological collections (relating to past/ ancient), (v) drawings (a picture or diagram made with a pencil, pen, or crayon without paint.), (vi) paintings;, (vii) sculptures;, (viii) any work of art; or, (ix) bullion (Gold and silver in the form of biscuits / bricks / bars), If any person has received gift of any other property, it will not be taxable e.g. motor car or plant and, machinery or a watch or a mobile phone etc., E.g. Mr. X received a mobile phone valued ₹70,000 from his friend, in this case, it will be exempt from
Page 91 :
Taxability of Gift, , 91, , income tax., Question 3: Write a note on Taxability of gift received by HUF from its members., Answer:, If any Hindu undivided family has received any gift from any of its members, it will be exempt from, income tax. E.g. One HUF has received cash gift of ₹10,00,000 from one of its members, it will be exempt, from income tax., If HUF has given gift to its member, it will be taxable., Question 4: Write a note on Taxability of stock-in-trade., Answer:, If any person has received any asset as stock-in-trade, it will not be taxable as gift e.g. Mr. X is a dealer in, gold and he has purchased gold for ₹20 lakhs but market value is ₹ 27 lakhs, in this case it will not be, taxable as gift (because cost will be shown in the books as ₹20 lakhs and entire profit on sale shall be, taxable under the head business/profession.), MTP NOV -2020, 2. From the following transactions relating to Mrs. Natasha, determine the amount chargeable to tax in her, hands for the A.Y. 2022-23. Your answer should be supported by reason:, (i) On 1-1-2022, being her birthday, she received a gift of ₹ 40,000 by means of cheque from her, father's maternal uncle., (ii) On 12-2-2022, she acquired a vacant site from her friend for ₹1,32,000. The State stamp, valuation authority fixed the value of site at ₹ 2,00,000 for stamp duty purpose., (iii) She bought 50 equity shares of a private company from another friend for ₹ 75,000. The fair, market value of such shares on the date of purchase was ₹ 1,33,000., (3 Marks), Answer:, Computation of amount chargeable to tax in hands of Mrs. Natasha for A.Y. 2022-23, (i), , (ii), , Particulars, Even though father’s maternal uncle does not fall within the definition of “relative”, under section 56(2)(x), gift of ₹40,000 received from him by cheque is not chargeable, to tax since the aggregate sum of money received by Mrs. Natasha without, consideration from non-relatives (other than on the occasion of marriage) during the, previous year 2021-22 does not exceed ₹ 50,000, Purchase of vacant site for inadequate consideration on 12.2.2022 would attract the, provisions of section 56(2)(x). Where any immovable property is received for a, consideration which is less than the stamp duty value of the property by an amount, and the difference between stamp duty value and consideration is more the higher of, ₹50,000 and 10% of consideration, the difference between the stamp duty value and, consideration is chargeable to tax in the hands of Individual., , ₹, Nil, , 68,000, , Therefore, in the given case ₹ 68,000 (₹ 2,00,000 - ₹ 1,32,000) is taxable in the hands, of Mrs. Natasha, since the difference exceeds ₹50,000, being the higher of ₹50,000, and 10% of consideration., (iii), , Since shares are included in the definition of “property” and difference between the, purchase value and fair market value of shares is ₹ 58,000 (₹ 1,33,000 - ₹ 75,000) i.e., it exceeds ₹ 50,000, the difference would be taxable under section 56(2)(x)., Amount chargeable to tax, , 58,000, 1,26,000, , Illustration 1: Mr. X, a dealer in shares, received the following without consideration during the P.Y.202122 from his friend Mr. Y, -
Page 92 :
Taxability of Gift, , 92, , (1) Cash gift of ₹ 75,000 on his anniversary, 15th April, 2021., (2) Bullion, the fair market value of which was ₹ 60,000, on his birthday, 19th June, 2021., (3) A plot of land at Faridabad on 1st July, 2021, the stamp value of which is ₹ 5 lakh on that date. Mr. Y had, purchased the land in April, 2015., Mr. X purchased from his friend Z, who is also a dealer in shares, 1000 shares of X Ltd. @ ₹ 400 each on, 19th June, 2021, the fair market value of which was ₹ 600 each on that date., (4) Mr. X sold these shares in the course of his business on 23rd June, 2021., (5) On 1st November, 2021, Mr. X took possession of property (building) booked by him two years back at ₹, 20 lakh. The stamp duty value of the property as on 1st November, 2021 was ₹ 32 lakh and on the date of, booking was ₹ 23 lakh. He had paid ₹ 1 lakh by account payee cheque as down payment on the date of, booking., Compute the income of Mr. X chargeable under the head “Income from other sources” for A.Y.2022-23., Solution:, Particulars, ₹, 75,000, (1) Cash gift is taxable, 60,000, (2) Since bullion is included in the definition of property, it is taxable., 5,00,000, (3) Stamp duty value of plot of land at Faridabad is taxable., (4) Difference of ₹ 2 lakh in the value of shares of X Ltd. purchased from Mr. Z, a dealer in, shares, is not taxable as it represents the stock-in-trade of Mr. X. Since Mr. X is a dealer in, shares and it has been mentioned that the shares were subsequently sold in the course of, his business, such shares represent the stock-in-trade of Mr. X., (5) Difference between the stamp duty value of ₹23 lakh on the date of booking and the actual 3,00,000, consideration of ₹20 lakh paid is taxable. (Difference amount is more than ₹50,000 and, more than 10% of the consideration), Income from Other Sources, 9,35,000, Illustration 2: Discuss the taxability or otherwise of the following in the hands of the recipient under, section 56(2)(x) the Income-tax Act, 1961 (i) X HUF received ₹ 75,000 in cash from niece of Mr. X (i.e., daughter of Mr. X’s sister). Mr. X is the, Karta of the HUF., (ii) Miss. X, a member of her father’s HUF, transferred a house property to the HUF without consideration., The stamp duty value of the house property is ₹ 9,00,000., (iii) Mr. X received 100 shares of A Ltd. from his friend as a gift on occasion of his 25th marriage, anniversary. The fair market value on that date was ₹ 100 per share. He also received jewellery worth, ₹45,000 (FMV) from his nephew on the same day., (iv) X HUF gifted a car to son of Karta for achieving good marks in XII board examination. The fair market, value of the car is ₹ 5,25,000., Solution:, Reason, Taxable/, Amount, Non-taxable liable to, tax (₹), (i) Taxable, 75,000, Sum of money exceeding ₹50,000 received without consideration from a, non-relative is taxable under section 56(2)(x). Daughter of Mr. X’s sister is, not a relative of X HUF, since she is not a member of X HUF., (ii) Non-taxable Nil, Immovable property received without consideration by a HUF from its, relative is not taxable under section 56(2)(x). Since Miss. X is a member of, the HUF, she is a relative of the HUF., (iii) Taxable, 55,000, As per provisions of section 56(2)(x), in case the aggregate fair market, value of property, other than immovable property, received without, consideration exceeds ₹50,000, the whole of the aggregate value shall be, taxable. In this case, the aggregate fair market value of shares (₹10,000), and jewellery (₹45,000) exceeds ₹50,000. Hence, the entire amount of, ₹55,000 shall be taxable.
Page 93 :
Taxability of Gift, (iv) Non-taxable, , Nil, , 93, , Car is not included in the definition of property for the purpose of section, 56(2)(x), therefore, the same shall not be taxable., , Illustration 3: Discuss taxability in the following cases:, (i) Mr. X has received three gifts from his three friends, (a) ₹55,000 in cash, (b) Land with market value ₹5,00,000 but the value for the purpose of charging stamp duty ₹4,00,000., (c) Jewellery with market value ₹3,00,000, In this case, taxable amount shall be 55,000 + 4,00,000 + 3,00,000 = 7,55,000, (ii) Mr. X has received gift of ₹50,000 in cash from his friend, in this case it will not be considered to be his, income., (iii) Mr. X has received gift of ₹1,50,000 in cash from his brother, in this case it will not be considered to be, his income., (iv) Mr. X has received gift of ₹1,50,000 in cash from his mother’s sister, in this case it will not be, considered to be his income., (v) Mr. X has received gift of ₹1,50,000 in cash from his father’s brother, in this case it will not be, considered to be his income., (vi) Mr. X has received gift of ₹1,50,000 in cash from his cousin, in this case it will be chargeable to tax., (vii) Mr. X has received gift of ₹1,50,000 in cash from brother of his spouse, in this case it will not be, considered to be his income., (viii) Mr. X has received gift of ₹1,50,000 in cash from his grand father, in this case it will not be considered, to be his income., (ix) Mr. X has received gift of ₹1,50,000 in cash from spouse of his brother, in this case it will not be, considered to be his income., (x) Mr. X has received gift of ₹1,50,000 in cash from husband of his sister, in this case it will not be, considered to be his income., (xi) Mr. X has received gift of ₹1,50,000 in cash from sister of his brother’s wife, in this case it will be, considered to be his income., (xii) Mr. X has received gift of ₹1,50,000 in cash from the sister of his spouse, in this case it will not be, considered to be his income., (xiii) Mr. X has received gift of ₹5,000 in cash on his birthday from each of his eleven friends, in this case it, will be considered to be his income because the total amount is exceeding ₹50,000., (xiv) Mr. X has received gift of property valued ₹1,50,000 from his friend, in this case it will be considered, to be his income., (xv) Mr. X has received gift of ₹1,50,000 in cash from his friend on the occasion of his marriage, in this case, it will not be considered to be his income., (xvi) Mr. X has received gift of ₹75,000 in cash and property ₹75,000 from his fiancee, in this case gift in, cash will be considered to be his income and the gift as property shall also be considered to be his income., Question 5: Explain taxability of gift received from employer., Answer: Gifts to the Employees, Section 17(2)(viii) Rule 3(7)(iv), Gift given by the employer in kind upto ₹5,000 in aggregate during a particular year is exempt and excess, over it is taxable. If the employer has given any voucher or token in lieu of which such gift may be received,, it will also be exempt in the similar manner., Gifts in cash or gifts convertible into cash i.e. gift cheques etc. shall be fully chargeable to tax., E.g. Mr. X is employed in ABC Ltd. and he has received a cash gift of ₹11,000 from his employer, in this, case taxable amount shall be ₹11,000 and it will be income under the head Salary and shall be taxable at the, normal rate but if Mr. X has received one wrist watch of ₹11,000 from his employer, taxable amount shall, be ₹6,000., Question 6: Explain Taxability of gift received in connection with business/profession., Answer: Gifts or Perquisites from Clients, Section 28, The value of any benefit or perquisite, whether convertible into money or not, arising from business or the, exercise of a profession, shall be taxable under the head business profession.
Page 94 :
Taxability of Gift, , 94, , If any person has received any gift or perquisite or benefit either in cash or in kind from any of his clients, it, will be considered to be business receipt and shall be taken into consideration while computing income, under the head business/profession., Example: A Doctor has received a gift of ₹ 40,000 from one of his clients, in this case it will be considered, to be income under the head business/profession., Question 7: Explain taxability of scholarship/ award / reward., Answer: Scholarship Section 10(16), Any scholarship received by a person for meeting the cost of education shall be exempt from income tax., Award/ Reward, Section 10(17A), Any award or reward whether in cash or in kind instituted by the Central Government or the State, Government shall be exempt from income tax. Similarly any private award or reward shall be exempt from, income tax if approved by the Central Government.
Page 95 :
Taxability of Gift, , 95, , MULTIPLE CHOICE QUESTIONS, 1. Gift of sum of money is exempt if, (a) aggregate value during particular year is less ₹50,000, (b) aggregate value during particular year is upto ₹50,000, (c) if value of individual gift is upto ₹50,000, (d) aggregate value during particular year is less ₹1,00,000, 2. In case of gift of immovable property, value to be taken into consideration shall be, (a) market value of individual property, (b) market value of all the properties, (c) stamp duty value of individual property, (d) stamp duty value of all the properties, 3. For the purpose of gift, the term relative shall include, (a) grand father of individual, (b) bother’s son of individual, (c) brother of father of spouse, (d) all the above, 4. Property for the purpose of gift shall include, (a) shares and securities, (b) jewellery, (c) mobile phone, (d) work of art, (e) (a), (b) and (d), 5. Which of the statement is correct, (a) gift received by an employee is exempt upto ₹10,000, (b) cash gift received by an employee upto ₹5,000 is exempt, (c) gift in kind received by an employee upto ₹5,000 is exempt, (d) gift in kind received by an employee upto ₹50,000 is exempt, 6. Which of the statement is correct, (a) scholarship received by any person is exempt u/s 10(17A), (b) award or reward of central government is taxable u/s 28, (c) gift received by a professional from his client is taxable as per section 28, (d) gift received by an employee exceeding ₹5,000 is taxable u/h other sources, 7. which of the statement is false, (a) gift received by HUF from its member is exempt, (b) gift received from sister of spouse is exempt, (c) gift of motor car received from a friend is exempt, (d) cash gift upto ₹5,000 received from employer is exempt, 8. The term relative do not include, (a) lineal ascendant or descendant of individual, (b) lineal ascendant or descendant of spouse of individual, (c) lineal ascendant or descendant of brother of individual, (d) none of the above, 9. If any person has purchased immovable property for ₹20 lakh but stamp duty value is ₹23 lakhs, in, this case taxable amount of gift shall be, (a) 3 lakh, (b) 1 lakh, (c) Nil, (d) 23 lakh, 10. Which of the following gift is taxable, (a) Mr. X received cash gift ₹51,000 from his friend, (b) Mr. Y received cash gift ₹51,000 from his fiancée
Page 96 :
Taxability of Gift, , 96, , (c) Mr. Z received cash gift ₹51,000 from his friend’s father, (d) all the above, (e) none of the above, 11. Mr. Kashyap has acquired a building from his friend on 10.10.2021 for ₹15,00,000. The stamp, duty value of the building on the date of purchase is ₹15,70,000. Income chargeable to tax in the hands, of Mr. Kashyap is, (a) ₹ 70,000, (b) ₹ 50,000, (c) Nil, (d) ₹ 20,000, 12. Ganesh received ₹60,000 from his friend on the occasion of his birthday, (a) The entire amount of ₹60,000 is taxable, (b) ₹50,000 is taxable, (c) The entire amount is exempt, (d) ₹10,000 is taxable, 13. Mr. Y has received a sum of ₹51,000 on 24.10.2021 from relatives on the occasion of his marriage., (a) Entire ₹51,000 is chargeable to tax., (b) Only ₹ 1,000 is chargeable to tax, (c) Entire ₹ 51,000 is exempt from tax, (d) Only 50% i.e., ₹ 25,500 is chargeable to tax, 14. Mr. Mayank has received a sum of ₹ 75,000 on 24.10.2021 from his friend on the occasion of his, marriage anniversary., (a) Entire ₹ 75,000 is chargeable to tax., (b) Entire ₹75,000 is exempt from tax, (c) Only ₹ 25,000 is chargeable to tax, (d) Only 50% i.e., ₹ 37,500 is chargeable to tax, 15. Ashok took possession of property on 31st August 2021 booked by him three years back at ₹25, lakhs, The Stamp Duty Value (SDV) of the property as on 31st August 2021 was ₹31 lakh and on date, of booking it was ₹29 lakh. He had paid ₹2 lakh by A/c payee cheque as down payment on date of, booking. Which of the following will be considered as income, if any, and in which previous year, (a) ₹4 lakhs in P.Y. 2021-22, (b) ₹4 lakhs in P.Y. 2018-19, (c) ₹6 lakhs in P.Y. 2020-21, (d) No income shall be taxable, since down payment was paid by A/c cheque while booking the property, 16. Mr. Kishore celebrated his 50th marriage anniversary. On this occasion, his wife received a, diamond necklace worth ₹5,00,000 from Kishore's brother. Kishore's son gifted him a luxurious car, worth ₹15,00,000, His grandchildren gifted them a new furniture set worth ₹3,00,000. Also, he, received cash gifts from his friends amounting collectively to ₹80,000. Which of them the following, statements stand true on taxability., (a) Neither Mr. Kishore nor Mrs. Kishore will be liable for tax for any gifts since they have been received on, occasion of marriage anniversary, (b) Mr. Kishore & Mrs. Kishore will jointly share the tax liability on all the gifts, (c) Mrs. Kishore will be liable to pay tax on diamond set and Mr. Kishore will bear tax for the cash gifts, received, (d) Mr. Kishore will be liable for tax on cash gifts only., 17. Sujata, aged 16 years, received scholarship of ₹50,000 during the previous year 2021-22. Which of, the following statements are true regarding taxability of such income:, (a) Such income shall be assessed in hands of Sujata, (b) Such income to be included with the income of parent whose income before such clubbing is higher, (c) Such income is completely exempt from tax, (d) Such income to be clubbed with father's income
Page 97 :
Taxability of Gift, , 97, , 18. Mr. X received cash gift ₹ 51,000 and gift of jewelry valued ₹ 49,000, in this case taxable amount, shall be, (a) ₹ 51,000, (b) ₹ 49,000, (c) ₹ 1,00,000, (d) Nil, (e) none of these, 19. Mr. X received cash gift ₹ 40,000, gift of land stamp duty value ₹ 40,000 and gift of building stamp, duty value ₹ 40,000, in this case taxable amount shall be, (a) ₹ 40,000, (b) ₹ 80,000, (c) ₹ 1,20,000, (d) Nil, (e) none of these, 20. Mr. X purchased one house property for ₹ 3,00,000 market value ₹ 7,00,000 stamp duty value ₹, 3,40,000, in this case taxable amount shall be, (a) ₹ 4,00,000, (b) ₹ 40,000, (c) Nil, (d) ₹ 3,40,000, (e) none of these, , Check Solution given on our website www.mkgeducation.com
Page 98 :
Taxability of Gift, , 98, , PRACTICE PROBLEMS, TOTAL PROBLEMS 5, Problem 1., Discuss taxability in the following cases:, (i) Mr. X has received gift of ₹ 50,000 in cash from his friend., (ii) Mr. X has received gift of ₹ 2,50,000 in cash from his brother., (iii) Mr. X has received gift of ₹ 2,50,000 in cash from his mother’s sister., (iv) Mr. X has received gift of ₹ 2,50,000 in cash from his father’s brother., (v) Mr. X has received gift of ₹ 2,50,000 in cash from his cousin., (vi) Mr. X has received gift of ₹ 2,50,000 in cash from brother of his spouse., (vii) Mr. X has received gift of ₹ 2,50,000 in cash from his grand father., (viii) Mr. X has received gift of ₹ 2,50,000 in cash from spouse of his brother., (ix) Mr. X has received gift of ₹ 2,50,000 in cash from husband of his sister., (x) Mr. X has received gift of ₹ 2,50,000 in cash from sister of his brother’s wife., (xi) Mr. X has received gift of ₹ 2,50,000 in cash from the sister of his spouse., (xii) Mr. X has received gift of ₹6,000 in cash on his birthday from each of his eleven friends., (xiii) Mr. X has received gift of ₹ 2,50,000 as property from his friend., (xiv) Mr. X has received gift of ₹2,50,000 in cash from his friend on the occasion of his marriage., (xv) Mr. X has received gift of ₹1,00,000 in cash and ₹1,00,000 as property from his fiancée., Problem 2., Mr. X submits the particulars for the previous year 2021-22 as given below:, 1. He has received a gift of ₹27,000 from one of his friend on 01.09.2021., 2. He has received a gift of ₹11,000 on 01.10.2021 from his wife Mrs. X., 3. He has received a gift of ₹29,000 from his step daughter on 01.01.2022., 4. He has received a gift of ₹27,000 from grand mother of Mrs. X on 07.01.2022., 5. He has received a gift of ₹70,000 in kind from his employer on 01.03.2022., 6. He has received gold as gift from his friend on 01.12.2021 with value ₹2,00,000., 7. He has received ₹27,000 as gift from his maternal aunt (mother’s sister) on 10.12.2021., 8. He has received two gifts of ₹30,000 each from his neighbours on 01.06.2021., Compute his tax liability for assessment year 2022-23., Answer = Tax Liability: Nil, Problem 3., Mr. X received gift in cash ₹5,00,000 from son of his father’s brother and gift of ₹1,00,000 in cash from, brother of father of Mrs. X. He has agricultural income ₹5,00,000., Compute his tax liability for Assessment Year 2022-23., Answer = Tax Liability: ₹83,200, (b) He is aged 81 years., Answer = Tax Liability: ₹31,200, (c) He is non-resident and he has completed age of 80 years as on 31.03.2022., Answer = Tax Liability: ₹83,200, Problem 4., Mr. X received jewellery valued ₹8,00,000 from brother of his grand father and his agricultural income is, ₹1,00,000., Compute his income and tax liability for Assessment Year 2022-23., Answer = Total Income: ₹8,00,000; Tax Liability: ₹91,000
Page 99 :
Taxability of Gift, , 99, , Problem 5., Following gifts are received by Mrs. X, who is carrying on jewellery business, during the previous year, 2021-22:, (i) On the occasion of her marriage on 07.09.2021, she has received ₹1,20,000 as gift out of which ₹85,000, are from relatives and balance from friends., (ii) On 03.10.2021, she has received cash gift of ₹2,50,000 from cousin of her mother., (iii) A mobile phone worth ₹15,000 is gifted by her friend on 21.09.2021., (iv) She gets a cash gift of ₹2,40,000 from the elder brother of her husband's grandfather on 10.12.2021., (v) She has received a cash gift of ₹6,00,000 from her friend on 27.01.2022., (vi) She has received bullion, the fair market value of which was ₹4,75,000 on her birthday,19.01.2022., Mrs. X purchased from her friend, who is also carrying jewellery business, jewellery at ₹ 2,50,000 on, 25.01.2022, the fair market value of which was ₹5,00,000 on that date., Compute total income and tax liability of Mrs. X for A.Y.2022-23., Answer = Total Income: ₹15,65,000; Tax Liability: ₹2,93,280, , Check Detailed Solution given on our website www.mkgeducation.com
Page 100 :
Taxability of Gift, , 100, , EXAMINATION QUESTIONS, MAY – 2018 (NEW COURSE), Question 5 (a), Marks 3, Discuss the taxability of the following receipts in the hands of Mr. Sanjay Kamboj under the Income Tax, Act, 1961 for A.Y.2022-23:, (i) ₹51,000 received from his sister living in US on 1-6-2021., (ii) Received a car from his friend on payment of ₹2,50,000 the FMV of which was ₹5,50,000., Provisions of taxability or Non-taxability must be discussed., Answer:, (i) As per section 56(2), Gift received from relative is not taxable. In the given case, Sister is covered, under the definition of relative and Gift received from her is not taxable., (ii) As per section 56(2), Gift in Kind exceeding ₹50,000 received from non - relative is taxable but in, the given case Car is not covered under the definition of Gift in Kind hence Car received from, non-relative is not taxable., , MAY – 2016, Question 4(a), (2 x 2 = 4 Marks), Discuss the taxability or otherwise in the hands of the recipients, as per the provisions of the Income-tax, Act, 1961:, (i), Mr. N, a member of his father's HUF, transferred a house property to the HUF without, consideration. The value of the house is ₹10 lacs as per the Registrar of stamp duty., (ii), Mr. Kumar gifted a car to his sister's son (Sunil) for achieving good marks in CA Final exam. The, fair market value of the car is ₹5,00,000., Answer:, (i) Non-Taxable: As per sec 56(2)(x), if HUF has received any Gift from its member, it will be exempt from, Income tax. In the given case, HUF has received a Gift of house property from its member Mr. N hence it, will be exempt from income tax and what is the value of house property shall not matter., (ii) Non-Taxable: If any person has received a gift from brother of mother, it will be covered in the, definition of relative and shall be exempt from income tax further if a gift is taxable it should be covered in, the definition of property as given u/s 56(2)(x). In the given case gift is from relative and further gift is of, motor car which is not covered in the definition of property hence it will be exempt from Income Tax., , MAY – 2012, Question 1, (1 Marks), State whether the following are chargeable to tax and the amount liable to tax., A sum of ₹1,20,000 was received as gift from non-relatives by Mr. X on the occasion of the marriage of his, son Mr. Y., Answer: As per section 56(2)(x), if any gift has been received on the occasion of marriage, it will be exempt, from income tax but if gift has been received by the parents of the person getting married, such gift shall be, taxable hence in this case gift received by Mr. X is taxable because marriage is that of his son Mr. Y., , MAY – 2011, Question 7, (4 Marks), The following details have been furnished by Mrs. X, pertaining to the year ended 31.03.2022:, (i) Cash gift of ₹51,000 received from her friend on the occasion of her “Shastiaptha Poorthi”, a wedding, function celebrated on her husband completing 60 years of age. This was also her 25th wedding anniversary.
Page 101 :
Taxability of Gift, , 101, , (ii) On the above occasion, a diamond necklace worth ₹2 lacs was presented by her sister living in Dubai., (iii) When she celebrated her daughter’s wedding on 21.02.2022, her friend assigned in Mrs. X’s favour, a, fixed deposit held by the said friend in a scheduled bank; the value of the fixed deposit and the accrued, interest on the said date was ₹51,000., (iv) She has short term capital gains under section 111A ₹10 lakhs., Compute total income and tax liability for assessment year 2022-23., Answer: (i) Any sum of money received by an individual on the occasion of the marriage of the individual, is exempt. This provision is, however, not applicable to a cash gift received during a wedding function, celebrated on completion of 60 years of age., The gift of ₹51,000 received from a non-relative is, therefore, chargeable to tax under section 56(2)(x) in the, hands of Mrs. X., (ii) The provisions of section 56(2)(x) are not attracted in respect of any sum of money or property received, from a relative. Thus, the gift of diamond necklace received from her sister is not taxable under section, 56(2)(x), even though jewellery falls within the definition of “property”., (iii) To be exempt from applicability of section 56(2)(x), the property should be received on the occasion of, the marriage of the individual, not that of the individual’s son or daughter. Therefore, this exemption, provision is not attracted in this case., Any sum of money received without consideration by an individual is chargeable to tax under section, 56(2)(x), if the aggregate value exceeds ₹50,000 in a year. “Sum of money” has, however, not been defined, under section 56(2)(x)., Therefore, there are two possible views in respect of the value of fixed deposit assigned in favour of Mrs., X–, (1)The first view is that fixed deposit does not fall within the meaning of “sum of money” and therefore, the, provisions of section 56(2)(x) are not attracted. Fixed deposit is also not included in the definition of, “property”., (2) However, another possible view is that fixed deposit assigned in favour of Mrs. X falls within the, meaning of “sum of money” received., Income assessable as “Income from other sources”, If the first view is taken, the total amount chargeable to tax as “Income from other sources” would be, ₹51,000, being cash gift received from a friend on her Shastiaptha Poorthi., As per the second view, the provisions of section 56(2)(x) would be attracted in respect of the fixed deposit, assigned and the “Income from other sources” of Mrs. X would be ₹1,02,000 (₹51,000 + ₹51,000)., Tax liability as per first view, Income under the head Other Sources, 51,000.00, Income under the head Capital Gains (STCG u/s 111A), 10,00,000.00, Gross Total Income, 10,51,000.00, Less: Deduction u/s 80C to 80U, Nil, Total Income, 10,51,000.00, Computation of Tax Liability, Tax on ₹51,000 at slab rate, Nil, Tax on ₹8,01,000 (₹10,00,000 – ₹1,99,000) @ 15%, 1,20,150.00, Add: HEC @ 4%, 4,806.00, Tax Liability, 1,24,956.00, Rounded off u/s 288B, 1,24,960.00, Tax liability as per second view, Income under the head Other Sources, 1,02,000.00, Income under the head Capital Gains (STCG u/s 111A), 10,00,000.00, Gross Total Income, 11,02,000.00, Less: Deduction u/s 80C to 80U, Nil, Total Income, 11,02,000.00, Computation of Tax Liability, Tax on ₹1,02,000 at slab rate, Tax on ₹8,52,000 (₹10,00,000 – ₹1,48,000) @ 15%, , Nil, 1,27,800.00
Page 102 :
Taxability of Gift, Add: HEC @ 4%, Tax Liability, Rounded off u/s 288B, , 102, 5,112.00, 1,32,912.00, 1,32,910.00, , NOV – 2008, Question 2, (4 Marks), Mrs. X has received the following gifts during previous year 2021-22., (i) On the occasion of her marriage on 14.08.2021, she has received ₹90,000 as gift out of which ₹70,000 are, from relatives and balance from friends., (ii) On 12.09.2021, she has received gift of ₹18,000 from cousin of her mother., (iii) A cell phone of ₹71,000 is gifted by her employer on 15.08.2021., (iv) She gets a gift of ₹25,000 from the elder brother of her husband's grandfather on 25.10.2021., (v) She has received a gift of ₹2,000 from her friend on 14.04.2021., (vi) She has won ₹4 lakh from a game show on electronic media., Compute her tax liability for assessment year 2022-23., Answer:, Computation of taxable income of Mrs. X from gifts for A.Y. 2022-23, Particulars, Taxable amount, Reason for taxability or, ₹, otherwise of each gift, • Relatives and friends, Nil, Gifts received on the occasion of, marriage are not taxable., • Cousin of Mrs. X’s mother, 18,000, Cousin of Mrs. X’s mother is, not a relative. Hence, the gift is taxable., • Elder brother of husband’s grandfather, 25,000, Brother of husband’s grandfather is, not a relative. Hence, the gift is taxable., • Friend, 2,000, Gift from friend is taxable., Aggregate value of gifts, 45,000, Since the aggregate value of gifts received by Mrs. X during the previous year 2021-22 does not exceed, ₹50,000, the same is not chargeable to tax under section 56(2)(x) of the Income-Tax Act, 1961., Gift received from the employer in kind upto ₹5,000 is exempt from income tax but excess over it is taxable, hence in this case taxable amount of gift shall be ₹66,000 (71,000 – 5,000) and it will be taxable under the, head Salary., Gross Salary, 66,000, Less: Standard deduction u/s 16(ia), (50,000), Income under the head Salary, 16,000, Income under the head Other Sources, Gross Total Income, Less: Deduction u/s 80C to 80U, Total Income, Computation of Tax Liability, Tax on ₹16,000 at slab rate, Tax on ₹4,00,000 @ 30%, Less: Rebate u/s 87A, Tax before HEC, Add: HEC @ 4%, Tax Liability, , 4,00,000, 4,16,000, Nil, 4,16,000, Nil, 1,20,000, (12,500), 1,07,500, 4,300, 1,11,800, , MAY – 2008, Question 3, Choose the correct answer with reference to the provisions of the Income-tax Act, 1961:, Mr. X received ₹70,000 from his friend on the occasion of his birthday., , (1 Marks)
Page 103 :
Taxability of Gift, , 103, , (a) The entire amount of ₹70,000 is taxable, (b) ₹25,000 is taxable, (c) The entire amount is exempt, (d) None of the above., Answer:, (a) The entire amount of ₹70,000 is taxable., , MAY – 2005, Question 1, (1 Marks), Gift of ₹5,00,000 received on 10th July, 2021 through account payee cheque from a non-relative regularly, assessed to income-tax, is, (a) A capital receipt not chargeable to tax, (b) Chargeable to tax as income from other sources, (c) Chargeable to tax as business income, (d) Exempt upto ₹50,000 and balance chargeable to tax as income from other sources., Answer:, (b) Chargeable to tax as income from other sources
Page 104 :
Advance Payment Of Tax, , 104, , ADVANCE PAYMENT OF TAX, OR, , PAY AS YOU EARN SCHEME, SECTION 207 TO 219, SECTIONS, 207, 208, 211, 234A, 234B, 234C, Rule 119A, , PARTICULARS, Liability for payment of advance tax, Conditions of liability to pay advance tax, Instalments of advance tax and due dates, Computation of interest in case of payment of tax after last date of filing of ROI, Computation of interest in case of payment of tax after end of previous year, Computation of interest in case of default in payment of advance tax, Rounded off of the amount on which interest is to be computed, , Question 1: Write a note on advance payment of income tax., Answer: As per section 207, every person shall pay tax in advance as per the provisions of advance tax i.e., in general every person should estimate his income and pay tax however exact amount of income tax shall, be calculated at the end of the year., A senior citizen who do not have income under the head business/profession shall be exempt from payment, of advance tax. In this case, interest u/s 234B & 234C shall not be payable but Interest u/s 234A shall, be payable., As per section 208, advance tax shall be payable during a financial year in every case where the amount of, such tax payable by the assessee during that year, is ten thousand rupees or more., An assessee has to estimate his current income and pay advance tax thereon. He need not submit any, estimate or statement of income to the Assessing Officer., As per section 211, all assessee have to pay advance tax in the manner given below:, Due date of installment, Amount payable, th, Upto 15 June of P.Y., 15% of tax payable, Upto 15th September of P.Y., 45% of tax payable, Upto 15th December of P.Y., 75% of tax payable, Upto 15th March of P.Y., 100% of tax payable, Example, For the previous year 2021-22, ABC Ltd. has estimated its tax payable to be `2,00,000, in this case advance, tax shall be paid by the company as given below:, Upto 15.06.2021, 30,000, Upto 15.09.2021, 90,000, Upto 15.12.2021, 1,50,000, Upto 15.03.2022, 2,00,000, If the last day for payment of any installment of advance tax is a day on which the receiving bank is closed,, the assessee can make the payment on the next immediately following working day.
Page 105 :
Advance Payment Of Tax, , 105, , Question 2: Write a note on payment of interest for late payment of income tax, Answer: As per section 234C, if any person has defaulted in payment of advance tax, interest shall be, charged @ 1% per month for a period of 3 months on the amount of default in each installment, but for the, last installment, interest shall be charged only for one month., Income tax paid upto 31st March of previous year is also called advance tax., As per section 234B, if advance tax paid is less than 90% of actual tax liability, assessee shall be required to, pay interest @ 1% per month or part of a month from 1st April of assessment year upto the date of payment., If advance tax paid is 90% or more of actual tax liability, no interest is payable., As per section 234A, if any person has paid income tax after expiry of the last date of filing of return of, income, interest shall be payable @ 1% p.m. or part of the month for the period subsequent to the last date, of filing of return of income., Illustration 1: ABC Ltd. has estimated its tax liability for assessment year 2022-23 `4,40,000 and has paid, advance tax accordingly but actual tax liability was found to be `10,00,000., The company has paid balance amount on 02.01.2023 and filed return of income on the same date., Compute interest payable under section 234A, 234B, and 234C., Solution:, Estimated Tax = 4,40,000, Actual Tax = 10,00,000, Interest under section 234C shall be computed in the manner given below:, Tax Payable, Tax Paid, Default, 15.06.2021, 1,50,000, 66,000, 84,000, Interest u/s 234C =, 84000 x 1% x 3 = 2,520, 15.09.2021, 4,50,000, 1,98,000, 2,52,000, Interest u/s 234C =, 2,52,000 x 1% x 3 = 7,560, 15.12.2021, 7,50,000, 3,30,000, 4,20,000, Interest u/s 234C =, 4,20,000 x 1% x 3 = 12,600, 15.03.2022, 10,00,000, 4,40,000, 5,60,000, Interest u/s 234C =, 5,60,000 x 1% x 1 = 5,600, Total interest payable u/s 234C, , 28,280, , Interest under section 234B shall be computed from 01.04.2022 to 02.01.2023 and is as given below:, 10,00,000 – 4,40,000 = 5,60,000 x 1% x 10 =, 56,000, Interest under section 234A shall be computed from 01.11.2022 to 02.01.2023 and is as given below:, 5,60,000 x 1% x 3 =, 16,800, Total interest payable (28,280 + 56,000 + 16,800), 1,01,080, Illustration 2: ABC Ltd. has tax liability of `7,00,000 for the previous year 2021-22 and the company has, not paid any advance tax and entire tax amount was paid by the company on 31.12.2022. In this case,, interest shall be calculated in the manner given below:, 1. Interest u/s 234C, `, 15.06.2021, 1,05,000 x 1% x 3, =, 3,150, 15.09.2021, 3,15,000 x 1% x 3, =, 9,450, 15.12.2021, 5,25,000 x 1% x 3, =, 15,750, 15.03.2022, 7,00,000 x 1% x 1, =, 7,000, Total interest payable, 35,350, 2. Interest u/s 234B (01-04-2022 to 31-12-2022), 7,00,000 x 1% x 9 =, 3. Interest u/s 234A (01-11-2022 to 31-12-2022), , 63,000
Page 106 :
Advance Payment Of Tax, 7,00,000 x 1% x 2 =, Total Interest Payable, , 106, 14,000, 1,12,350, , Question 3: Explain Special Provision for all assessee., Answer: Special Provision for all assessee, If the advance tax paid by the assessee upto 15th June is 12% of the tax payable and upto 15th September,, is 36% of the tax payable, in such cases no interest shall be charged for default in such instalment., Illustration 3: ABC Ltd. has estimated its tax payable to be `5,00,000 for previous year 2021-22 and has, paid advance tax accordingly but actual tax liability of the company was found to be `5,50,000 and, difference of tax amount was paid on 10.12.2022. Compute interest under section 234A, 234B and 234C., Solution:, Interest under section 234C shall be computed in the manner given below:, Tax Payable, Tax Paid, Default, 15.06.2021, 82,500, 75,000, 7,500, Interest u/s 234C =, Nil (because advance tax paid is at least 12%), 15.09.2021, 2,47,500, 2,25,000, 22,500, Interest u/s 234C =, Nil (because advance tax paid is at least 36%), 15.12.2021, 4,12,500, 3,75,000, 37,500, Interest u/s 234C = 37,500 x 1% x 3 = 1,125, 15.03.2022, 5,50,000, 5,00,000, 50,000, Interest u/s 234C = 50,000 x 1% x 1 = 500, Total interest payable u/s 234C, 1,625, Interest under section 234B, Advance tax paid is more than 90% of actual tax liability, no interest is payable, Interest under section 234A shall be computed from 01.11.2022 to 10.12.2022 and is as given below:, 50,000 x 1% x 2 =, 1,000, Total interest payable (1,625 + 1,000), 2,625, Question 4: Explain Payment of advance tax in case of capital gains/casual income/ newly setup, business/ profession/dividend income., Answer: Payment of advance tax in case of capital gains/casual income/newly setup business/, profession/ dividend income Section 234C, In case of capital gains, casual income and dividend income, no advance tax is payable on estimated basis, but if there is actual accrual of casual income or capital gains or dividend income, advance tax is to be paid, in the subsequent installments and if such accrual is after 15th March, advance tax is to be paid upto 31st, March of previous year otherwise interest shall be charged under section 234C., Illustration 4: Mr. X has paid advance tax as given below:, Upto 15.06.2021, ` 15,000, Upto 15.09.2021, ` 45,000, Upto 15.12.2021, ` 95,000, Upto 15.03.2022, `1,70,000, He had long term capital gains of `3,00,000 on 01.01.2022 and his income under the head, business/Profession is `11,00,000, He has filed return of income on 10.12.2022 and has paid difference of the tax on 10.12.2022., Last date for filing of return is 31.07.2022., Compute interest payable under section 234A, 234B and 234C., Solution:, Computation of Tax Liability, `, Normal Income, 11,00,000, Long term capital gains, 3,00,000, Total Income, 14,00,000, Tax on `11,00,000 at slab rate, 1,42,500
Page 107 :
Advance Payment Of Tax, , 107, , Tax on `3,00,000 @ 20%, 60,000, Add: HEC @ 4%, 8,100, Tax Liability, 2,10,600, (Tax liability excluding capital gains i.e. `14,00,000 - `3,00,000 = `11,00,000 at slab rate + HEC @ 4%, 1,48,200), Interest u/s 234C, Since capital gains arises on 1st January 2022, installments for 15th June, 15th September and 15th December, shall be checked without including tax on capital gain and shall be as given below:, Amount payable, Amount actually paid, Shortfall, `, `, `, Upto 15.06.2021 (1,48,200 x 15%), 22,230.00, 15,000, 7,230.00, Rounded off under rule 119A = 7,200, Interest u/s 234C = 7,200 x 1% x 3 = 216, Upto 15.09.2021 (1,48,200 x 45%), 66,690.00, Interest u/s 234C = 21,600 x 1% x 3 = 648, , 45,000, , 21,690.00, , Upto 15.12.2021 (1,48,200 x 75%), 1,11,150.00, Rounded off under rule 119A = 16,100, Interest u/s 234C = 16,100 x 1% x 3 = 483, , 95,000, , 16,150.00, , Installment for 15th March shall be including tax on capital gains and is as given below:, Upto 15.03.2022 (2,10,600 x 100%) 2,10,600, 1,70,000, 40,600, Interest u/s 234C = 40,600 x 1% x 1 = 406, Total Interest u/s 234C, , `1,756, , Interest u/s 234B (01-04-2022 to 10-12-2022), 40,600 x 1% x 9, , `3,654, , Interest u/s 234A (01-08-2022 to 10-12-2022), 40,600 x 1% x 5, , `2,030, , Similar provision shall be applicable in case of a newly setup Business/Profession, If any Assessee has started Business/Profession in the current year, assessee shall be exempt from payment, of advance tax prior to commencement of Business/Profession i.e. advance tax has to be paid in installments, subsequent to commencement of Business/Profession., If Business/Profession has been started after 15th March, advance tax should be paid upto 31st March, otherwise Interest shall be charged under section 234C for one month @ 1%., Example: Mr. X started his business on 01.10.2021 and had income `10,00,000 upto 31.03.2022, In this, case, he will be required to pay advance tax in the manner given below:, `, Income under the head Business/ Profession, 10,00,000, Gross Total Income/Total Income, 10,00,000, Computation of Tax Liability, Tax on 10,00,000 at slab rate, 1,12,500, Add: HEC @ 4%, 4,500, Tax Liability, 1,17,000, Advance Tax Payment, 15.06.2021, 15.09.2021, 15.12.2021 (1,17,000 x 75%), 15.03.2022 (1,17,000 x 100%), , Nil, Nil, 87,750.00, 1,17,000.00, , Illustration 5: ABC Ltd. started his business on 01.10.2021 and had earning from business from 01.10.2021, to 31.03.2022 `20,00,000, in this case company need not pay advance tax upto 15.09.2021 but advance tax
Page 108 :
Advance Payment Of Tax, , 108, , is to be paid in subsequent installments. Company should pay advance tax on 15.12.2021 equal to, (20,00,000 x 30% + HEC) x 75% = 4,68,000 and company should pay total advance tax on 15.03.2022 equal, to (20,00,000 x 30% + HEC) x 100% = 6,24,000., Illustration 6: A partnership firm made the following payments of advance tax during the financial year, 2022-23:, `, Upto June 15, 2021, 4,15,000, Upto September 15, 2021, 8,25,000, Upto December 15, 2021, 16,64,000, Upto March 15, 2022, 26,23,000, Return of income filed by the firm is `88,00,000 under the head “profits and gains of business or profession”, and `9,50,000 by way of long term capital gains on sale of a property effected on December 1, 2021. What, is the interest payable by the assessee under section 234B and section 234C for assessment year 2022-23?, Assume that the return of income was filed on 31.10.2022 i.e. the due date and tax was fully paid on self, assessment., Solution:, Computation of Tax Liability, `, Business income, 88,00,000, Long term capital gains, 9,50,000, Total Income, 97,50,000, Tax on `88,00,000 @ 30%, 26,40,000, Tax on `9,50,000 @ 20%, 1,90,000, Add: HEC @ 4%, 1,13,200, Tax Liability, 29,43,200, (Tax liability excluding capital gains `97,50,000 - `9,50,000 = `88,00,000 x 30% + HEC@ 4%, 27,45,600), Interest u/s 234C, Since capital gains arises on 1st December 2021, installment for 15th June and 15th September shall be, checked without including tax on capital gain and shall be as given below:, Amount payable, Amount actually paid, Shortfall, `, `, `, Upto 15.06 2021 (27,45,600 x 15%) 4,11,840, 4,15,000, NIL, Interest u/s 234C = Nil, Upto 15.09 2021 (27,45,600 x 45%) 12,35,520, Rounded off under rule 119A = 4,10,500, Interest u/s 234C = 4,10,500 x 1% x 3 month = 12,315, , 8,25,000, , 4,10,520, , Installments for 15th December and 15th March shall be including tax on capital gains and is as given below:, Upto 15.12.2021 (29,43,200 x 75%) 22,07,400, 16,64,000, 5,43,400, Interest u/s 234C = 5,43,400 x 1% x 3 month = 16,302, Upto 15.03.2022 (29,43,200 x 100%) 29,43,200, Interest u/s 234C = 3,20,200 x 1% x 1 month = 3,202, , 26,23,000, , 3,20,200, , Total Interest u/s 234C, , `31,819, , Interest u/s 234B (01-04-2022 to 31-10-2022), 3,20,200 x 1% x 7, , `22,414, , Rounding off for the purpose of calculating Interest Rule 119A, As per rule 119A, the principal amount shall be rounded off in the multiples of `100 and for this purpose
Page 109 :
Advance Payment Of Tax, , 109, , any fraction of `100 shall be ignored. E.g. `1,60,275 shall be rounded off as 1,60,200., Illustration 7: ABC Ltd. has paid advance tax for the previous year 2021-22 as given below:, 1. Upto 15.06.2021, ` 50,000, 2. Upto 15.09.2021, `1,50,000, 3. Upto 15.12.2021, `3,00,000, 4. Upto 15.03.2022, `6,00,000, Actual tax liability was found to be `7,00,000 and balance tax was paid on 10.12.2022. Compute interest, payable under section 234A, 234B, 234C., Solution:, Interest under section 234C shall be computed in the manner given below:, Tax Payable, Tax Paid, Shortfall, 15.06.2021, 1,05,000, 50,000, 55,000, Interest u/s 234C = 55,000 x 1% x 3 =, 1,650, 15.09.2021, 3,15,000, 1,50,000, 1,65,000, Interest u/s 234C = 1,65,000 x 1% x 3 =, 4,950, 15.12.2021, 5,25,000, 3,00,000, 2,25,000, Interest u/s 234C = 2,25,000 x 1% x 3 =, 6,750, 15.03.2022, 7,00,000, 6,00,000, 1,00,000, Interest u/s 234C = 1,00,000 x 1% x 1 =, 1,000, Total interest payable u/s 234C, 14,350, Interest under section 234B shall be computed from 01.04.2022 to 10.12.2022 and is as given below:, 7,00,000 – 6,00,000 = 1,00,000 x 1% x 9 =, 9,000, Interest under section 234A shall be computed from 01.11.2022 to 10.12.2022 and is as given below:, 1,00,000 x 1% x 2 =, 2,000, Total interest payable (14,350 + 9,000 + 2,000), 25,350, (b) Presume actual tax liability is `6,50,000., Solution:, Interest under section 234C shall be computed in the manner given below:, Tax Payable, Tax Paid, Shortfall, 15.06.2021, 97,500, 50,000, 47,500, Interest u/s 234C = 47,500 x 1% x 3 = 1,425, 15.09.2021, 2,92,500, 1,50,000, 1,42,500, Interest u/s 234C = 1,42,500 x 1% x 3 = 4,275, 15.12.2021, 4,87,500, 3,00,000, 1,87,500, Interest u/s 234C = 1,87,500 x 1% x 3 = 5,625, 15.03.2022, 6,50,000, 6,00,000, 50,000, Interest u/s 234C = 50,000 x 1% x 1 =, 500, Total interest payable u/s 234C, 11,825, Interest under section 234B, Advance tax paid is more than 90% of actual tax liability, no interest is payable, Interest under section 234A shall be computed from 01.11.2022 to 10.12.2022 and is as given below:, 50,000 x 1% x 2 =, 1,000, Total interest payable (11,825 + 1,000), 12,825, Due date for filing the return of income Section 139(1), Return is to be filed in general upto 31st July of the assessment year, however, in the following cases, the last, date shall be 31st October of the assessment year., 1. Every company assessee, Example, For the previous year 2021-22, ABC Ltd. has to file its return of income upto 31.10.2022.
Page 110 :
Advance Payment Of Tax, , 110, , 2. Any other person who is required to get his accounts audited either under Income Tax Act or under any, other Act., Example, Mr. X has his own business and his turnover for previous year 2021-22 is `102 lakhs. In this case, the last, date of filing the return of income shall be 31.10.2022, but if turnover is `97 lakhs, the last date shall be, 31.07.2022., Similarly if a partnership firm XY has turnover of its business ` 65 lakhs for previous year 2021-22, in this, case, the last date of filing of return of income shall be 31.07.2022., Audit u/s 44AB, 1. As per Section 44AB, Audit is compulsory for every person carrying on business and sales turnover or, gross receipts exceeds ` 1 crore during the previous year and every person carrying on profession and gross, receipts is exceeding `50 lakh during the previous year., 2. Every person carrying on business, if his total sales turnover or gross receipts, in business exceeds `10, Crore during the previous year provided that in the case of a person whose––, (a) aggregate of all amounts received including amount received for sales, turnover or gross receipts, during the previous year, in cash, does not exceed five per cent. of the said amount. and, (b) aggregate of all payments made including amount incurred for expenditure, in cash, during the, previous year does not exceed five per cent. of the said payment, If tax liability is less than ` 10,000 then interest u/s 234B & 234C shall not be payable but Interest u/s 234A, shall be payable., Eg. Mr. X has Income under LTCG 112A is ` 4 Lakh and he paid income tax on 10/12/2022. Compute, Interest u/s 234A,234B & 234C. In this case, His Tax Liability shall be :, `, Income under the head Capital Gains, 4,00,000, Gross Total Income/Total Income, 4,00,000, Computation of Tax Liability, Tax on `50,000 (4,00,000-1,00,000-2,50,000) @ 10% u/s 112A, 5,000, Add: HEC @ 4%, 200, Tax Liability, 5,200, Since Tax Liability is less than ` 10,000 hence Interest u/s 234B & 234C is not payable., Interest u/s 234A = 5,200 x 1% x 5 = `260, Question 5: Explain Powers of Assessing officer to direct the Assessee to pay Advance Tax., Answer: Powers of Assessing Officer to direct the Assessee to pay Advance Tax Section 209, 210, If any person has not paid advance tax and Assessing Officer is of the opinion that such person has to pay, advance tax, in such cases Assessing Officer may issue him a notice in Form No.28 directing such person to, pay advance tax but notice can be given only if such person has already been assessed through regular, assessment in any of the earlier years. Regular assessment shall include scrutiny assessment under section, 143(3) or Best judgement assessment under section 144. If Assessing Officer has issue notice, estimated, income for such year shall be the income of the latest previous year in respect of which the assessee has, been assessed by way of regular assessment but if assessee has filed any return subsequently and income, reported in such return is higher than the income selected above, in that case income reported by the, assessee shall be considered to be estimated income of current year., If the assessee do not pay the advance tax even after receiving such notice, he will be considered to be, assessee in-default as per section 218 and penalty can be imposed equal to the amount not paid as per, section 221., If the assessee finds that his tax liability shall be less than the amount computed by the Assessing Officer,, assessee may give a reply in Form No.28A and can pay tax as per his own estimate., Example
Page 111 :
Advance Payment Of Tax, , 111, , For the previous year 2021-22, ABC Ltd. has not paid any advance tax till 10.10.2021 and in the earlier, years the company was assessed in the manner given below:, 2018-19, 143(3) (Scrutiny Assessment), 7,00,000, 2019-20, 144 (Best Judgement Assessment), 10,00,000, 2020-21, ROI, 8,00,000, In this case Assessing officer shall have the powers to give notice to the assessee and its estimated income, shall be considered to be `10,00,000. If any assessee has received a notice in form no. 28 but he finds that, his tax liability shall be less than the amount computed by the Assessing Officer, in that case he can give a, reply in form no. 28A and can pay tax as per his own estimate.
Page 112 :
Advance Payment Of Tax, , 112, , MULTIPLE CHOICE QUESTIONS, 1. Which of the following statement is not correct., (a) advance tax is payable only if tax payable during a particular year is `10,000 or more, (b) a senior citizen is always exempt from payment of advance tax, (c) upto 15th Dec of previous year, advance tax payable is atleast 75%, (d) in case of default in payment of advance tax interest shall be charged @ 1% p.m., 2. Which of the following statement is not correct., (a) if atleast 12% of actual tax has been paid upto 15th June, interest u/s 234C is not payable, (b) if atleast 24% of actual tax has been paid upto 15th Sept, interest u/s 234C is not payable, (c) if atleast 75% of actual tax has been paid upto 15th Dec, interest u/s 234C is not payable, (d) last date for Filing of ROI in case of a company is 31st Oct of the assessment year, 3. Which of the following statement is not correct., (a) advance tax payable upto 15th June of the previous year is atleast 15%, (b) advance tax payable upto 15th Sept of the previous year is atleast 45%, (c) advance tax payable upto 15th Dec of the previous year is atleast 70%, (d) advance tax payable upto 15th March of the previous year is atleast 100%, 4. In case of default in payment of advance tax, interest shall be charged, (a) 2% p.m. u/s 234C, (b) 1% p.m. u/s 234B, (c) 1.5% p.m. u/s 234A, (d) 1% p.m. u/s 234C, 5. In case of default in payment of Self assessment tax, interest shall be charged, (a) 2% p.m. u/s 234C , (b) 1% p.m. u/s 234B ,(c) 1.5% p.m. u/s 234A , (d) 1% p.m. u/s 234C, 6. In case of default in payment of self assessment tax after last date of filing of ROI, interest shall be, charged, (a) 2% p.m. u/s 234C, (b) 1% p.m. u/s 234B, (c) 1% p.m. u/s 234A + 1% p.m. u/s 234B, (d) 1% p.m. u/s 234C, 7. Advance tax is payable by a senior citizen only when, (a) he has income u/h capital gains, (b) he has income u/h house property, (c) he has income u/h business/profession, (d) he has income u/h casual income, 8. Which of the statement is correct, (a) a company has to pay advance tax irrespective of tax payable, (b) a partnership firm has to pay advance tax if tax payable is `1,00,000 or more, (c) all the assessees are required to pay advance tax if tax payable is `10,000 or above, (d) an individual is exempt from payment of advance tax, 9. Audit is required, (a) if turnover of business exceeding `100 lakhs, (b) if turnover of profession exceeding `50 lakhs, (c) (a) or (b), (d) if income of business is exceeding `10 lakhs, 10. Assessing Officer can issue a notice for payment of advance tax, (a) to any assessee, (b) any assessee who has been assessed trough regular assessment, (c) any assessee who paid tax of `10 lakh or more in the earlier years, (d) any assessee who has not paid advance tax for last three year, , Check Solution given on our website www.mkgeducation.com
Page 113 :
Advance Payment Of Tax, , 113, , PRACTICE PROBLEMS, TOTAL PROBLEMS 3, Problem 1., Mr. X has incomes as given below:, 1. Income under the head house property `15,00,000, 2. Gift of a painting from a friend with market value `2,00,000, 3. Gift of shares and securities from Mrs. X valued `3,00,000, 4. Agricultural income `3,00,000, He has paid advance tax as given below:, `15,000, Upto 15th June 2021, th, `30,000, Upto 15 Sept 2021, `50,000, Upto 15th Dec 2021, `60,000, Upto 15th March 2022, Balance amount of tax was paid and return of income was filed on 10th Sept 2022., Compute his tax liability for the Assessment Year 2022-23 and also interest under section 234A, 234B and, 234C., Answer = Tax Liability: `4,05,600; Interest under section 234A: `6,912; Interest under section 234B:, `20,736; Interest under section 234C: `17,031, Problem 2., ABC Ltd. (an Indian Company) has income as given below:, 1. Income from Business `20,00,000, 2. Income under the head House Property `7,00,000, The company has paid advance tax as given below:, Upto 15th June 2021, ` 20,000, ` 30,000, Upto 15th Sept 2021, ` 80,000, Upto 15th Dec 2021, `1,00,000, Upto 15th March 2022, Balance amount of tax was paid and return of income was filed on 10th Dec 2022., Due date for filing of ROI in case of ABC Ltd. is 31.10.2022., Compute tax liability for the Assessment Year 2022-23 and also interest under section 234A, 234B and, 234C., Answer = Tax Liability: `8,42,400; Interest under section 234A: `14,848; Interest under section 234B:, `66,816; Interest under section 234C: `37,637, Problem 3., Mrs. X has income under the head house property `18,00,000 and she has received gift of `3,00,000 in cash, from her husband’s sister and `1,00,000 from her sister’s husband and `1,20,000 from sister of her mother, in law. She has agricultural income of `4,00,000. She has paid advance tax as given below:, ` 15,000, Upto 15th June 2021, ` 45,000, Upto 15th Sept 2021, ` 75,000, Upto 15th Dec 2021, th, `1,00,000, Upto 15 March 2022, Balance amount of tax was paid on 10th Dec 2022 and return of income filed on the same date and due date, for filing return of income is 31.07.2022., Compute her tax liability for the Assessment Year 2022-23 and also interest under section 234A, 234B and, 234C., Answer = Tax Liability: `4,84,640; Interest under section 234A: `19,230; Interest under section 234B:, `34,614; Interest under section 234C: `19,416, , Check Detailed Solution given on our website www.mkgeducation.com
Page 114 :
Advance Payment Of Tax, , 114, , EXAMINATION QUESTIONS, NOV – 2017, Question 5(a) (i), (Marks 2), Under section 208, obligation to pay advance tax arises in every case where the advance tax payable is `, 10,000 or more, State exception to this rule., Answer: As per Section 208, Advance Tax shall be payable during a financial year in every case where the, amount of such tax payable by the assessee is `10,000 or more however as per section 207, a senior citizen, who do not have income under the head business/profession shall be exempt from payment of advance tax,, eg. Mr. X is a senior citizen and he has income under the head business/ profession and also under the head, house property, in this case he will be required to pay advance tax but if he do not have income under the, head business / profession, he will be exempt from payment of advance tax., , NOV – 2016, Question 6(a), (2 Marks), Discuss the provision under Income Tax Act for Payment of Advance Tax in case of Capital Gain., Answer: Refer Answer given in the book, , MAY – 2016, Question 7(a)(ii)., (4 Marks), Briefly discuss the provisions of Section 234B of the Income-tax Act,1961 for the short - payment or nonpayment of advance tax., Refer Answer given in the Chapter, , NOV – 2015, Question 7(a)., Explain briefly the provisions of advance tax on capital gains and casual income., Refer Answer given in the Chapter, , (4 Marks), , MAY – 2014, (2 x 2 = 4 Marks), Question. Who is liable to pay Advance Tax? What is the procedure to compute the Advance Tax payable?, Solution: As per section 207, every person is liable to pay advance tax however a senior citizen is exempt, from payment of advance tax provided he do not have any income under the head Business/Profession. As, per section 208, every person in whose case tax payable is less than `10,000 shall also be exempt from, payment of advance tax. For this purpose, the assessee himself shall estimate his tax liability / tax payable, and shall pay advance tax in installments given under section 211. An assessee is exempt from payment of, advance tax with regard to capital gains and casual income but advance tax should be paid after accrual of, such income in subsequent installments. Net agricultural income is also to be considered for the purpose of, computing advance tax., A person may increase or reduce the amount of advance tax payable in subsequent installment(s) in, accordance with his estimate of current income., , MAY – 2013, Question 7(a), (4 Marks), Briefly discuss the provisions relating to payment of advance tax in case of capital gains and casual income., Solution: Refer answer given in the chapter
Page 115 :
Advance Payment Of Tax, , 115, , JUNE – 2009, Question 5, Enlist the installments of advance tax and due dates thereon in case of companies., Answer. Refer answer given in the chapter, , (4 Marks), , MAY – 2008, Question 5, (4 x 2 = 8 Marks), (a) Briefly discuss about the interest chargeable under Section 234A for delay or default in furnishing return, of income., (b) What are the due dates of instalments and the quantum of advance tax payable by companies?, Answer 5(a)/5(b) . Refer answer given in the chapter, , NOV – 2007, Question 5, (4 Marks), Briefly discuss the provisions relating to payment of advance tax on income arising from capital gains and, casual income., Answer. Refer answer given in the chapter
Page 116 :
Advance Payment Of Tax, , CHALLAN, NO./, ITNS 280, , 116, , Tax Applicable (Tick One)*, (0020) INCOME-TAX ON COMPANIES, (CORPORATION TAX), (0021) INCOME TAX (OTHER THAN, COMPANIES), , Assessment Year, -, , Permanent Account, Number, Full Name, Complete Address with City & State, , Tel. No., , Pin, Type Of Payment (Tick One), , Advance Tax (100), Self Assessment Tax (300), Tax on Regular Assessment (400), DETAILS OF PAYMENTS, , Tax on Distributed Profits of Domestic Companies (106), Tax on Distributed Income to Unit Holders (107), Amount (in Rs. Only), , FOR USE IN RECEIVING BANK, , Debit to A/c / Cheque credited on, , Income Tax, Surcharge, Health & education cess, Interest, Penalty, Others, Total, Total (in words), CRORES, , LACS, , D D, , M, , M, , SPACE FOR BANK SEAL, , THOUSANDS, , HUNDREDS, , Paid in Cash/Debit to A/c /Cheque No., Drawn on, (Name of the Bank and Branch), Date:, , TENS, , UNITS, , Dated, , Rs., Signature of person making payment, , Taxpayers Counterfoil (To be filled up by tax payer), , SPACE FOR BANK SEAL, , PAN, Received from, (Name), Cash/ Debit to A/c /Cheque No., For Rs., Rs. (in words), Drawn on, (Name of the Bank and Branch), on account of, Companies/Other than Companies/Tax, Income Tax on, Type of Payment, for the Assessment Year, , (Strike out whichever is not, applicable), (To be filled up by person making the, payment), , -, , Rs., , Y, , Y
Page 117 :
Residential Status & Scope of Total Income, , 117, , RESIDENTIAL STATUS, &, , SCOPE OF TOTAL INCOME, SECTION 5 TO 9, PARTICULARS, Scope of total income/incidence of tax, Rules for determining the residential status of an individual, Rules for determining the residential status of an Hindu Undivided Family, Rules for Determining the residential status of Firm/ Association of person/Body of, individual, Rules for determining the residential status of a company, Rules for determining the residential status of any other person, Incomes deemed to be received, Income deemed to accrue or arise in India, , SECTIONS, 5, 6(1), 6(6)(a), 6(2), 6(6)(b), 6(2), 6(3), 6(4), 7, 9, , Residential Status Section 6, Whether a particular income shall be taxed or not shall depend on the residential status and the type of, income. Residential status infact explains connection of the person with the country and types of income, explains the connection of the income with the country. If the person do not have any connection and also, the incomes do not have any connection with the country, the income shall not be taxable but if either the, person or the income has any connection, the income is taxable., For this purpose, taxpayers are classified into three broad categories on the basis of their residential status., (1) Resident and ordinarily resident, (2) Resident but not ordinarily resident, (3) Non-resident, The residential status of an assessee is ascertained with reference to each previous year., Question 1: How to determine Residential status of individuals Section 6(1)/6(6)(a), Answer: Under section 6(1), an individual is said to be resident in India in any previous year, if he satisfies, any one of the following conditions:, (i) He stays in India for 182 days or more during the relevant previous year, (ii) He stays in India for 60 days or more and also for 365 days or more during 4 years preceding the, relevant previous year., If the individual satisfies any one of the conditions mentioned above, he is a resident, otherwise the, individual is a non-resident., e.g. Mr. X stayed in India for 200 days in previous year 2021-22, in this case he will be considered to be, resident in India., If Mr. X stayed in India for 100 days in previous year 2021-22 and also for 365 days during 4 years, preceding the previous year 2021-22, he will be considered to be resident in India., If Mr. X stayed in India during previous year 2021-22 for 59 days, he will be considered to be non-resident, in previous year 2021-22., The term “stay in India” includes stay in the territorial waters of India (i.e. 12 nautical miles into the sea, from the Indian coastline). Even the stay in a ship or boat in the territorial waters of India shall be, considered to be stay in India. (1 nautical mile = 1.1515 miles = 1.852 Kms).
Page 118 :
Residential Status & Scope of Total Income, , 118, , It is not necessary that the period of stay must be continuous nor is it essential that the stay should be at the, usual place of residence, business or employment of the individual., For the purpose of counting the number of days stayed in India, both the date of departure as well as the date, of arrival are considered to be in India., Meaning of Not-ordinarily resident Section 6(6)(a), An individual who is resident of India shall be considered to be NOR if he has complied with at least one of, the conditions given below:, (i), If such individual has during the 7 previous years preceding the relevant previous year been in India, for a period of 729 days or less or, (ii), If such individual has been non-resident in India in 9 years out of 10 previous years preceding the, relevant previous year, If he has not complied with even a single condition, he will be considered to be ROR., Income Tax Act has defined NOR but we can define ROR in the manner given below:, An individual is said to be a resident and ordinarily resident if he satisfies both the following conditions:, (i) He is a resident in any 2 out of the last 10 years preceding the relevant previous year, and, (ii) His total stay in India in the last 7 years preceding the relevant previous year is 730 days or more., If the individual satisfies both the conditions mentioned above, he is a resident and ordinarily resident but if, only one or none of the conditions are satisfied, the individual is a resident but not ordinarily resident., Illustration 1: Determine residential status of Mr. X for the assessment year 2022-23, who stays in India, during various financial years asunder:, Previous Years, Stay, 2014-15, 91, 2021-22, 100, 2013-14, 90, 2020-21, 200, 2012-13, 88, 2019-20, 91, 2011-12, 89, 2018-19, 90, 2010-11, 86, 2017-18, 2009-10, 87, 89, 87, 2008-09, 2016-17, 89, 2015-16, 82, 90, 2007-08, Solution:, As per section 6(1), Stay in India is 60 days plus 365 days during 4 years preceding the relevant previous, year, hence he is resident., His stay during 7 years is 730 hence he is not able to comply with first condition of section 6(6)(a). He is, able to comply with second condition of 6(6)(a) i.e. he is non-resident in atleast 9 years out of 10 years, preceding the relevant previous year hence he is NOR., Years, Status, 2016-17, Non-resident, 2021-22, Resident, 2015-16, Non-resident, 2020-21, Resident, 2014-15, Non-resident, 2019-20, Non-resident, 2013-14, Non-resident, 2018-19, Non-resident, 2012-13, Non-resident, 2017-18, 2011-12, Non-resident, Non-resident, Illustration 2: Determine residential status of Mr. X for the assessment year 2022-23, who stays in India, during various financial years asunder:, Previous Years, Stay, 2014-15, 80, 2021-22, 75, 2013-14, 91, 2020-21, 197, 2012-13, 86, 2019-20, 94, 2011-12, 85, 2018-19, 89, 2010-11, 89, 2017-18, 2009-10, 90, 72
Page 119 :
Residential Status & Scope of Total Income, , 119, , 89, 2008-09, 69, 2016-17, 2015-16, 91, 92, 2007-08, Solution:, As per section 6(1), Stay in India is 60 days plus 365 days during 4 years preceding the relevant previous, year, hence he is resident., His stay during 7 years is 730 hence he is not able to comply with first condition of section 6(6)(a). He is, able to comply with second condition of 6(6)(a) i.e. he is non-resident in atleast 9 years out of 10 years, preceding the relevant previous year hence he is NOR., Years, Status, 2016-17, Non-resident, 2021-22, Resident, 2015-16, Non-resident, 2020-21, Resident, 2014-15, Non-resident, 2019-20, Non-resident, 2013-14, Non-resident, 2018-19, Non-resident, 2012-13, Non-resident, 2017-18, 2011-12, Non-resident, Non-resident, Illustration 3: Mr. X an American citizen has come to India for the first time on 10.07.2017, as an, employee of a multinational company. The particulars of his arrival and departure are as given below:, Date of arrival, Date of departure, 10.07.2017, 07.08.2018, 07.10.2018, 27.11.2019, 01.03.2020, 01.02.2021, 10.05.2021, 30.03.2022, Not yet returned, Determine his residential status for previous year 2017-18 to 2021-22., Solution:, Previous Year 2017-18, {July – 22, August – 31, September 30, October 31, November 30, December 31, January 31, February – 28,, March – 31}, Days of stay in India are 265., As per section 6(1), Stay in India is 182 days or more during the relevant previous year, hence he is resident., and his stay during 7 years is Nil hence he is able to comply with first condition of section 6(6)(a). hence he, is NOR., Previous Year 2018-19, {April – 30, May – 31, June – 30, July – 31,August – 7,October – 25, November – 30, December – 31,, January – 31, February – 28, March – 31}, Days of stay in India are 305., As per section 6(1), Stay in India is 182 days or more during the relevant previous year, hence he is resident., and his stay during 7 years is 265 days hence he is able to comply with first condition of section 6(6)(a)., hence he is NOR., Previous Year 2019-20, {April – 30, May – 31, June – 30, July – 31,August – 31,September – 30 , October – 31, November – 27,, March – 31}, Days of stay in India are 272, so Mr. X is Resident., As per section 6(1), Stay in India is 182 days or more during the relevant previous year, hence he is resident., and his stay during 7 years is 570 days hence he is able to comply with first condition of section 6(6)(a)., hence he is NOR., Previous Year 2020-21, {April – 30, May – 31, June – 30, July – 31,August – 31, September – 30, October – 31, November – 30,, December – 31, January – 31, February – 1}, Days of stay in India are 307., As per section 6(1), Stay in India is 182 days or more during the relevant previous year, hence he is resident., and his stay during 7 years is more than 729 days hence he is not able to comply with first condition of
Page 120 :
Residential Status & Scope of Total Income, , 120, , section 6(6)(a) and he is not able to comply second condition also i.e. non-resident in atleast 9 years out of, 10 years preceding the relevant previous year. hence he is ROR., Previous Year 2021-22, {May – 22, June – 30, July – 31,August – 31, September – 30, October – 31, November – 30, December –, 31, January – 31, February – 28, March - 30}, Days of stay in India are 325., As per section 6(1), Stay in India is 182 days or more during the relevant previous year, hence he is resident., and his stay during 7 years is more than 729 days hence he is not able to comply with first condition of, section 6(6)(a) and he is not able to comply second condition of section 6(6)(a) also i.e. non-resident in, atleast 9 years out of 10 years preceding the relevant previous year. hence he is ROR., Illustration 4: Mr. X an American citizen has come to India for the first time on 01.07.2018 as an executive, of a multinational company. His employer has allowed him to visit USA every year and for this purpose he, will be leaving India every year on 1st November and shall come back on 31st December, besides that he has, visited Hong Kong on several occasions in connection with the official work, because he is looking after the, employer’s operations in Hong Kong also, with details asunder:, Date of leaving India, Date of arriving in India, 10.09.2018, 30.09.2018, 07.02.2019, 08.05.2019, 11.07.2019, 21.10.2019, 10.02.2020, 23.07.2020, 11.02.2021, 12.06.2021, 01.02.2022, 10.04.2022, Determine his residential status for the previous years 2018-19 to 2021-22., Solution:, Previous Year 2018-19, {July – 31, August – 31, September – 11, October – 31, November – 1, December – 1, January – 31,, February – 7}, Days of stay in India are 144, As per section 6(1), Stay in India is less than 182 days but more than 60 days during the relevant previous, year but Stay in India is less than 365 days during 4 years preceding the relevant previous year, hence he is, not complying even a single condition of section 6(1) hence he is Non – resident., Previous Year 2019-20, {May – 24, June – 30, July – 11, October – 11, November – 1, December – 1, January – 31, February – 10}, Days of stay in India are 119., As per section 6(1), Stay in India is less than 182 days but more than 60 days during the relevant previous, year but Stay in India is less than 365 days during 4 years preceding the relevant previous year, hence he is, not complying even a single condition of section 6(1) hence he is Non – resident., Previous Year 2020-21, {July – 9, August – 31, September – 30, October – 31, November – 1, December – 1, January – 31, February, – 11}, Days of stay in India are 145, As per section 6(1), Stay in India is less than 182 days but more than 60 days during the relevant previous, year but Stay in India is less than 365 days during 4 years preceding the relevant previous year, hence he is, not complying even a single condition of section 6(1) hence he is Non – resident., Previous Year 2021-22, {June – 19, July – 31, August – 31, September – 30, October – 31, November – 1, December – 1, January –, 31, February – 1}, Days of stay in India are 176. During the preceding 4 years, his stay is for 365 days or more so he is, resident. His stay during 7 years is 729 days or less, hence he is resident but not ordinarily resident., Illustration 5: Mr. X, the Australian cricketer comes to India for 105 days every year. Find out his, residential status for the A.Y. 2022-23.
Page 121 :
Residential Status & Scope of Total Income, , 121, , Solution: He has complied with condition of 60 + 365 days hence he is resident further stay in 7 years is, more than 729 days and also condition of non-resident in 9 years out of 10 years is not complied with hence, he is ROR., Illustration 6: Mr. X, a Canadian citizen, comes to India for the first time during the P.Y.2017-18. During, the financial years 2017-18, 2018-19, 2019-20, 2020-21 and 2021-22 he was in India for 55 days, 60 days,, 90 days, 150 days and 70 days respectively. Determine his residential status for the A.Y.2022-23., Solution:, During the previous year 2021-22, Mr. B was in India for 70 days and during the 4 years preceding the, previous year 2021-22, he was in India for 355 days (i.e. 55 + 60 + 90 + 150 days)., Thus, he does not satisfy section 6(1). Therefore, he is a non-resident for the previous year 2021-22., Illustration 7: On 01.06.2019 Mr. X, a Malaysian citizen leaves India after stay of 10 years. During the, financial year 2020-21 he comes to India for a period of 46 days. Later, he returns to India for one year on, 10.10.2021., Determine Mr. X’s residential status for the assessment year 2022-23., Solution:, No. of days of stay in India, P.Y. 2021-22, 173 Days, {22 + 30 + 31 + 31 + 28 + 31}, P.Y. 2020-21, 46 Days, P.Y. 2019-20, 62 Days, {30 + 31 + 1}, P.Y. 2018-19, 365 Days, P.Y. 2017-18, 365 Days, P.Y. 2016-17, 365 Days, P.Y. 2015-16, 366 Days, P.Y. 2014-15, 365 Days, P.Y. 2013-14, 365 Days, P.Y. 2012-13, 365 Days, P.Y. 2011-12, 366 Days, The person is resident and ordinarily resident. Mr. X was in India for 60 days in 2021-22 and for 365 days, or more in the 4 years immediately preceding the relevant previous year and he does not satisfy even a, single condition of section 6(6)(a)., NOV – 2011 (2 Marks), Brett Lee, an Australian cricket player visits India for 100 days in every financial year. This has been his, practice for the past 10 financial years. Find out his residential status for the assessment year 2022-23., Answer: An individual is said to be resident in India in any previous year, if he complies with at least one of, the following conditions:(a) He is in India in that year for a period amounting in all to 182 days or more, or, (b) He is in India in that year for a period amounting in all to 60 days or more and also for 365 days or more, during four years preceding the relevant previous year., Since, Brett Lee has complied with the second condition hence he is resident., Further more, an individual shall be considered to be not ordinarily resident in India in case his stay in India, is 729 days or less during preceding seven years or he is non-resident in atleast 9 years during preceding 10, years., Since stay of Brett Lee during preceding seven years is 700 days. Hence, he is NOR., MAY – 1998 (4 Marks), Mr. Nixon, an American citizen, is appointed by a multi-national company to its branch in New Delhi in, 2018. Mr. Nixon has never been to India before this appointment. He arrives in Bombay on 15th April, 2018, and joins the New Delhi office on 20th April, 2018. His wife and children join him in India on 20th October,, 2018. The company allotted him a leased residence for purposes of his stay. This residence is occupied by, him from the beginning of October, 2018.
Page 122 :
Residential Status & Scope of Total Income, , 122, , On 10th February, 2019, he is transferred by his employer, on deputation basis, to be the Regional Chief of, his employer’s operations in South East Asia having headquarters in Hongkong. He leaves New Delhi on, 11th February and arrives in Hongkong on 12th February, 2019. Mr. Nixon leaves behind his wife and, children in India till 14th August, 2020, when they leave along with him for Hongkong. Mr. Nixon had come, to India earlier on 15th June, 2020, on two months’ leave. The members of the family occupied the residence, till date of departure to Hongkong., At the end of the period of deputation, Mr. Nixon is reposted to India and joins the New Delhi office of his, employer as Chief of Indian operations on 2nd February, 2022., In what residential status Mr. Nixon will be assessable, for the various years, to income tax in India?, Answer:, The period of stay of Mr. Nixon for various years is given below:, P.Y. 2018-19/ A.Y. 2019-20, Period of stay, 303 days, (April – 16, May -31, June – 30, July – 31, August – 31, September – 30, October – 31, November – 30,, December – 31, January – 31, February – 11), P.Y. 2019-20/ A.Y. 2020-21, Period of stay, Nil, P.Y. 2020-21/ A.Y. 2021-22, Period of stay, 61 days, (June – 16, July – 31, August – 14), P.Y. 2021-22/ A.Y. 2022-23, Period of stay, 58 days, (February – 27, March – 31), Under section 6(1) of the Act, an individual is said to be resident in India in any previous year if he satisfies, one of the following basic conditions:, (i) is in India in the previous year for a period of 182 days or more ;, (ii) is in India for a period of 60 days or more in the previous year and 365 days or more during the four, years preceding the previous year., A person will be considered to be ‘not ordinarily resident’ if he satisfies any of the following two conditions, viz;, (i) he has been in India for a period of 729 days or less in 7 previous years preceding the relevant previous, year., (ii) he has been a non resident in India in 9 out of 10 previous years preceding the relevant previous year ; or, Maintenance of a residence in India or the stay of the wife and children in India are not relevant for, determining the residential status of Mr. Nixon., In the above background, Mr. Nixon’s case will be decided as under:, (i) P.Y. 2018-19/ A.Y. 2019-20: has been in India for 303 days. He will be a resident under the basic, conditions. Since his stay in seven years preceding the relevant previous year is Nil i.e. 729 days or less,, hence he will be NOR, (ii) P.Y. 2019-20/A. Y. 2020-21: has not been in India at all ; though his wife and children continue to reside, in New Delhi, he will be a non-resident for this year., (iii) P.Y. 2020-21/A.Y. 2021-22: has been in India for 61 days and for 303 days in 4 years preceding the, relevant previous year hence he will be non-resident., (iv) P.Y. 2021-22/A.Y. 2022-23: has been in India for 58 days, he will be non-resident., Question 2: Write a note on determination of residential status of Individual covered in special, category., Answer: Special category: explanation to section 6(1), Certain individuals are covered in the special category and they will be considered to be resident only if they, stay in India for 182 days or more i.e. second condition of 60 plus 365 days shall not be applicable and such, individuals are:, 1. Any individual who is a citizen of India and has left India for taking up any business or profession or, employment outside India e.g. Mr. X is a citizen of India and has left India on 01.09.2021 for taking up an, employment in Germany, in this case he will be covered in the special category and his status shall be nonresident. If any such person is employed in India and he has been transferred outside India, he will also be, covered in the special category. E.g. Mr. X is employed in Punjab National Bank in India and he has been, transferred to the London branch, in this case he will be covered in the special category. If any person has
Page 123 :
Residential Status & Scope of Total Income, , 123, , business or profession in India and he is going out of India in connection with business or profession, he will, not be covered in special category., 2. Any individual who is a citizen of India or is a person of Indian origin and is having, business/profession/employment outside India and has come to India on a visit shall also be covered in the, special category e.g. Mr. X is a citizen of India and is settled as a doctor in USA and has come to India on a, visit for 181 days, he will be covered in the special category and his status shall be non-resident., A person is said to be of Indian origin if he or either of his parents or either of his grandparents (including, parents of mother) were born in undivided India. e.g. Mr. X has taken birth in UK and is a citizen of UK but, his grand father has taken birth in India in 1942, in this case Mr. X will be considered to be person of Indian, origin., Further in case of citizen of India or person of Indian origin having total income other than income from, foreign sources, exceeding 15 lakhs during the year, such individual shall be resident in India if he stays, for 120 days during the year and also for 365 days during 4 years preceding the relevant previous year., Further as per section 6 (6)(c), he will be considered to be NOR, if his stay is maximum upto 181 days. If, stay is 182 days or more, he will be ROR/NOR as per section 6(6)(a) i.e. such person can stay in India for, maximum 119 days to maintain his status of Non Resident. e.g. Mr. X is a citizen of India and is settled, outside India. (he has total income other than income from foreign sources exceeding 15 lakhs) He visits, India 120 days and earlier 4 years, he was in India for 365 days, now he will be NOR but earlier he was NR, and impact shall be his income accruing / arising abroad and received abroad but from a business controlled, from India or from a profession setup in India shall be taxable. If his total income other than income from, foreign sources is upto 15 lakhs, he will be non resident., 3. Any individual who is a citizen of India and has left India as a member of crew of an Indian ship, shall, also be covered in special category. The time period mentioned in Continuous Discharge Certificate shall be, considered to be the period of stay outside India and remaining time period shall be considered to be stay in, India., 4. Deemed Resident Section 6(1A), As per section 6(1A), If any individual is a citizen of India and has total income other than the income from, foreign sources, exceeding 15 lakhs during the previous year, he shall be considered to be resident in India if he is, not liable to tax in any other country or territory by reason of his domicile or residence or any other criteria of, similar nature. Further as per section 6 (6)(d), he will be NOR. This clause shall not apply in case of an individual, who is resident in India in the previous year under section 6(1) E.g. Mr. X is a citizen of India and is settled in a, country where he is not liable to tax (he has total income other than income from foreign sources exceeding 15 lakhs), and he has not visited India during the current year, in this case he will be considered to be resident and further as per, section 6(6)(d), he will be considered to be NOR. (purpose is to tax stateless persons in India provided they are Indian, citizens) If his total income other than income from foreign sources is upto 15 lakhs, he will be non resident., , "income from foreign sources" means income which accrues or arises outside India (except income, derived from a business controlled in or a profession set up in India) and which is not deemed to accrue, arise in India. E.g. Mr. X has income accruing/arising abroad ₹ 25 lakh, out of which income from a, business controlled from India or profession set up in India is ₹ 18 lakh. He has income accruing/arising, in India 13 lakh, in this case income from foreign sources is ₹ 7 lakh (25-18) and income other than, income from foreign sources is ₹ 31 lakh (18+13), Example, Mr. Anand is an Indian citizen and a member of the crew of a Singapore bound Indian ship engaged in, international traffic departing from Chennai port on 6th June, 2021. From the following details for the, P.Y.2021-22, determine the residential status of Mr. Anand for A.Y.2022-23, assuming that his stay in India, in the last 4 previous years (preceding P.Y.2021-22) is 400 days and last seven previous years (preceding, P.Y.2021-22) is 750 days:
Page 124 :
Residential Status & Scope of Total Income, , 124, , Particulars, Date, Date entered into the Continuous Discharge Certificate in respect of joining the, 6th June, 2021, ship by Mr. Anand, Date entered into the Continuous Discharge Certificate in respect of signing off the 9th December, 2021, ship by Mr. Anand, Answer., In this case, the voyage is undertaken by an Indian ship engaged in international traffic, originating from a, port in India (i.e., the Chennai port) and having its destination at a port outside India (i.e., the Singapore, port). Hence, the voyage is an eligible voyage for the purposes of section 6(1). Therefore, the period, beginning from 6th June, 2021 and ending on 9th December, 2021, being the dates entered into the, Continuous Discharge Certificate in respect of joining the ship and signing off from the ship by Mr. Anand,, an Indian citizen who is a member of the crew of the ship, has to be excluded for computing the period of, his stay in India. Accordingly, 187 days [25+31+31+30+31+30+9] have to be excluded from the period of, his stay in India. Consequently, Mr. Anand’s period of stay in India during the P.Y.2021-22 would be 178, days [i.e., 365 days – 187 days]. Since his period of stay in India during the P.Y.2021-22 is less than 182, days, he is a non-resident for A.Y.2022-23., Note - Since the residential status of Mr. Anand is “non-resident” for A.Y.2022-23 consequent to his, number of days of stay in P.Y.2021-22 being less than 182 days, his period of stay in the earlier previous, years become irrelevant., RTP NOV -2020, 1. You are required to determine the residential status of Mr. Dinesh, a citizen of India, for the previous, year 2021-22., Mr. Dinesh is a member of crew of a Singapore bound Indian ship, carrying passengers in the, international waters, which left Kochi port in Kerala, on 16th August, 2021., Following details are made available to you for the previous year 2021-22:, Particulars, Date, Date entered into the Continuous Discharge Certificate in respect of joining 16th August, 2021, the ship by Mr. Dinesh, Date entered into the Continuous Discharge Certificate in respect of, 21st January, 2022, signing off the ship by Mr. Dinesh, In June, 2021, he had gone out of India to Dubai on a private tour for a continuous period of 27 days., During the last four years preceding the previous year 2021-22, he was present in India for 425 days., During the last seven previous years preceding the previous year 2021 -22, he was present in India for, 830 days., Answer:, As per Explanation 1 to section 6(1), where an Indian citizen leaves India as a member of crew of an Indian, ship, he will be resident in India only if he stayed in India for 182 days during the relevant previous year., As per Explanation 2 to section 6(1), in case of an individual, being a citizen of India and a member of the, crew of a foreign bound ship leaving India, the period or periods of stay in India shall, in respect of an, eligible voyage, not include the period commencing from the date entered into the Continuous Discharge, Certificate in respect of joining of ship by the said individual for the eligible voyage and ending on the date, entered into the Continuous Discharge Certificate in respect of signing off by that individual from the ship in, respect of such voyage., Eligible voyage includes a voyage undertaken by an Indian ship engaged in the carriage of passengers in, international traffic, originating from any port in India and having its destination at a port outside India., In this case, voyage is undertaken by a foreign bound Indian ship engaged in the carriage of passengers in, international traffic, originating from a port in India (i.e., the Kochi port) and having its destination at a port, outside India (i.e., the Singapore port). Hence, the voyage is an eligible voyage., Therefore, the period from 16th August, 2021 and ending on 21st January, 2022 has to be excluded for
Page 125 :
Residential Status & Scope of Total Income, , 125, , computing the period of stay of Mr. Dinesh in India. Accordingly, the period of 159 days, [16+30+31+30+31+21] has to be excluded for computing the period of his stay in India during the, P.Y.2021-22., Further, since Mr. Dinesh had also gone out of India to Dubai on a private tour for a continuous period of 27, days in June, 2021, such period has also to be excluded for computing his period of stay in India during the, P.Y.2021-22., Consequently, the period of stay in India during the P.Y. 2021-22 would be 179 days [i.e., 365 days – 159, days – 27 days], which is less than 182 days., Thus, Mr. Dinesh would be a non-resident for A.Y. 2022-23., Since the residential status of Mr. Dinesh is “non-resident” for A.Y. 2022-23 consequent to his number of, days of stay in India in P.Y. 2021-22, being less than 182 days, his period of stay in India in the earlier, previous years become irrelevant, MTP NOV -2020, 1. Mr. Square, an Indian citizen, currently resides in Dubai. He came to India on a visit and his total, stay in India during the F.Y.2021-22 was 135 days. He has no source of Income in India. Following, is his details of stay in India in the preceding previous years:, Financial Year, 2020-21, 2019-20, 2018-19, 2017-18, 2016-17, 2015-16, 2014-15, , Days of Stay in India, 125, 106, 83, 78, 37, 40, 35, , You are his tax consultant. Advise him on his residential status for the P.Y. 2021-22., (a) Resident but Not Ordinary Resident (RNOR), (b) Resident and Ordinary Resident, (c) Non-Resident, (d) Resident but information incomplete to know whether resident but not ordinarily resident or, resident and ordinarily resident, (2 Marks), Answer: (c), As per section 6(1),Mr. Square is covered under special category and second condition i.e 60 + 365 days is, not applicable. It means stay of Mr. Square in the current year must be 182 days or more. But in the given, case his stay is 135 days hence he is considered to be Non-resident, Illustration 8: Mr. X, an Indian citizen, leaves India on 22.09.2021 for the first time, to work as an officer, of a company in France. Determine his residential status for the A.Y. 2022-23., Solution:, During the previous year 2021-22, Mr. X, an Indian citizen, was in India for 175 days (i.e. 30 + 31+ 30 + 31, + 31 + 22 days). He does not satisfy the minimum criteria of 182 days. Also, since he is an Indian citizen, leaving India for the purposes of employment, the second condition under section 6(1) is not applicable to, him. Therefore, Mr. X is a non-resident for the A.Y.2022-23., Illustration 9: Mr. X and Mrs. X are settled outside India for the purpose of employment and they came to, India on 15.10.2021 on a visit for 7 months. Both of them are Indian citizens. In the earlier years they were, in India as follows:
Page 126 :
Residential Status & Scope of Total Income, , 126, , Year, Mr. X, Mrs. X, 2020 – 2021, 235 Days, 365 Days, 2019 – 2020, 330 Days, 30 Days, 2018 – 2019, Nil, 28 Days, 2017 – 2018, 118 Days, 120 Days, Find out the residential status of Mr. X and Mrs. X for the assessment year 2022-23., Solution:, Both are NR for the assessment year 2022-23, Stay of Mr. X in India, Previous Year 2021-22, 168 Days, {17 + 30 + 31 + 31 + 28 + 31}, Stay of Mrs. X in India, Previous Year 2021-22, 168 Days, {17 + 30 + 31 + 31 + 28 + 31}, Since they are covered in special category they will be resident only if their stay in India in relevant previous, year is 182 days or more, hence they are non–resident., NOV – 2010 (3 Marks), Mr. Ram an Indian citizen left India on 22.09.2021 for the first time to work as an officer of a company in, Germany., Determine the residential status of Ram for the assessment year 2022-23 and explain the conditions to be, fulfilled for the same under the Income-tax Act, 1961., Answer: Under section 6(1), an individual is said to be resident in India in any previous year if he satisfies, any one of the following conditions –, (i) He has been in India during the previous year for a total period of 182 days or more, or, (ii) He has been in India for a period of 60 days or more during the relevant previous year and also for 365, days or more during 4 years preceding the relevant previous year., In the case of Indian citizens leaving India for employment, the period of stay during the previous year must, be 182 days to become resident., During the previous year 2021-22, Mr. Ram, an Indian citizen, was in India for 175 days only (i.e. 30 + 31 +, 30 + 31 + 31 + 22 days). Thereafter, he left India for employment purposes. Since he does not satisfy the, minimum criteria of 182 days, he is a non-resident for the A.Y. 2022-23., NOV – 2002 (3 Marks), In the year P.Y. 2021-2022, a sailor has remained on ship as a crew member of an Indian Ship as follows:, (1) Outside the territorial waters of India for 183 days., (2) Inside the territorial waters of India for 182 days., Is he considered to be resident or not for the Assessment Year 2022-23. Comment., Answer: As per section 6(1) he is covered in the special category and he will be considered to be resident if, he stays in India for 182 days or more., In the given case, period of stay in India is 182 days. Therefore, the sailor is treated as a resident in India., Question 3: Explain how to determine residential status of HUF Section 6(2)/6(6)(b)., Answer: As per section 6(2), an HUF would be resident in India if the control and management of its, affairs is situated wholly or partly in India. If the control and management of the affairs is situated wholly, outside India it will be considered to be non-resident. Since control and management of HUF is in the hands, of its Karta hence place of stay of Karta shall be taken into consideration i.e. if Karta is out of India, throughout the year, HUF shall be Non-resident but if Karta has come to India for a few days, HUF shall be, resident., The expression ‘control and management’ refers to the central control and management and not to the, carrying on of day-to-day business by servants, employees or agents.
Page 127 :
Residential Status & Scope of Total Income, , 127, , Meaning of Not-ordinarily resident Section 6(6)(b), If an HUF is resident, as per section 6(6)(b), it will be considered to be NOR if its Karta has complied with, at least one of the conditions given below:, (i), If the karta is in India during the 7 previous years preceding the relevant previous year for a period of, 729 days or less., (ii), If the karta is non-resident in India in any 9 out of the 10 previous years preceding the relevant, previous year, or, If karta has not complied with even a single condition, HUF shall be ROR., Illustration 10: Karta of one Hindu Undivided Family comes to India every year for minimum 60 days and, maximum 91 days. Determine residential status of the Hindu Undivided Family and also that of the Karta, for the assessment year 2022-23., Solution:, Hindu Undivided Family is resident since the Karta has come to India for at least 60 days but the stay of, Karta during seven years can be maximum 637 days hence Hindu Undivided Family shall be considered to, be resident but not ordinarily resident., Karta in his individual capacity is non-resident because he cannot comply with even one of the two, conditions given under section 6(1)., Illustration 11: One Hindu Undivided Family is being managed partly from Mumbai and partly from, Nepal. Mr. X (a foreign citizen), Karta of Hindu Undivided Family, comes on a visit to India every year, since 1982 in month of April for 105 days., Determine residential status of the Hindu Undivided Family and also that of the Karta in his individual, capacity for the assessment year 2022-23., Solution:, For the previous year 2021-22, the control and management of the affairs of Hindu Undivided Family is, being partly managed from India. Hence Hindu Undivided Family is resident but Mr. X cannot comply with, any of the conditions of section 6(6)(b), hence Hindu Undivided Family is resident and ordinarily resident., Karta shall be considered to be resident and ordinarily resident because his stay during 7 years is 735 days., Also, he will not be non-resident in nine years out of ten years preceding the relevant previous year., JUNE – 2009 (2 Marks), State with reason, whether the following statements are True or False:, Mr. X, Karta of HUF, claims that the HUF is non-resident as the business of HUF is transacted from UK and, all the policy decisions are taken there., Answer. True, A HUF is considered to be a non-resident where the control and management of its affairs, are situated wholly outside India. In the given case, since all the policy decisions of HUF are taken from, UK, the HUF is a non-resident., Question 4: Explain how to determine residential status of partnership firm or Body of Individual or, Association of Persons., Answer:, Firms and Association of Persons Section 6(2), A firm and an AOP would be resident in India if the control and management of its affairs is situated wholly, or partly in India. Where the control and management of the affairs is situated wholly outside India, the firm, and AOP would become a non-resident. There are no ROR or NOR in case of persons other than individual, or HUF. E.g. XY partnership firm has two partners Mr. X and Mr. Y and Mr. X is working partner and is in, USA throughout the year and Mr. Y is a dormant partner and is in India throughout the year, in this case, partnership firm shall be non-resident but if Mr. X has come to India for a few days, partnership firm shall, be resident.
Page 128 :
Residential Status & Scope of Total Income, , 128, , Question 5: Explain how to determine residential status of a Company., Answer:, Companies Section 6(3), An Indian company is always resident in India even if its control and management is outside India or its, business is outside India., A foreign company shall be resident in India if its place of effective management, at any time in that year, is, in India., “Place of effective management” means a place where key management and commercial decisions are made, for the conduct of the business of an entity., e.g. Micromax Informatics Ltd. was incorporated in India and it has business in many countries outside, India, in this case company shall be considered to be resident., e.g. HCL Technologies Ltd. was incorporated in India and it has its control and management outside India, also, in this case company shall be considered to be resident., e.g. ABC Ltd. was incorporated outside India and place of effective management is in India, in this case, company shall be considered to be resident., e.g. Videocon Industries Ltd. was incorporated in India, in this case company shall be considered to be, resident., e.g. Samsung Electronics Co., Ltd. was incorporated in South Korea and place of effective management is, also in South Korea , in this case company shall be considered to be non-resident., e.g. BlackBerry Ltd. was incorporated in Canada and place of effective management is also in Canada, in, this case company shall be considered to be non-resident., Illustration 12: ABC Inc., a Swedish company headquartered at Stockholm, not having a permanent, establishment in India, has set up a liaison office in Mumbai in April, 2021 in compliance with RBI, guidelines to look after its day to day business operations in India, spread awareness about the company’s, products and explore further opportunities. The liaison office takes decisions relating to day to day routine, operations and performs support functions that are preparatory and auxiliary in nature. The significant, management and commercial decisions are, however, in substance made by the Board of Directors at, Sweden. Determine the residential status of ABC Inc. for A.Y.2022-23., Answer., As per Section 6(3), a company would be resident in India in any previous year, if(i) it is an Indian company; or, (ii) its place of effective management, in that year, is in India ., In this case, ABC Inc. is a foreign company. Therefore, it would be resident in India for P.Y.2021-22 only if, its place of effective management, in that year, is in India., Explanation to section 6(3) defines “place of effective management” to mean a place where key, management and commercial decisions that are necessary for the conduct of the business of an entity as a, whole are, in substance made. In the case of ABC Inc., its place of effective management for P.Y.2021-22 is, not in India, since the significant management and commercial decisions are, in substance, made by the, Board of Directors outside India in Sweden., ABC Inc. has only a liaison office in India through which it looks after its routine day to day business, operations in India. The place where decisions relating to day to day routine operations are taken and, support functions that are preparatory or auxiliary in nature are performed are not relevant in determining, the place of effective management., Hence, ABC Inc., being a foreign company is a non-resident for A.Y.2022-23, since its place of effective, management is outside India in the P.Y.2021-22., Illustration 13: Wipro Ltd. an Indian company has most of its business outside India. Determine its, residential status., Solution:, An Indian company shall always be considered to be resident in India., Illustration 14: Afcons Infrastructure International Ltd. is incorporated in Mauritius and its place of, effective management is in Mauritius. Determine its residential status for the assessment year 2022-23.
Page 129 :
Residential Status & Scope of Total Income, , 129, , Solution:, Foreign company shall be resident in India only if its place of effective management, at any time in that, year, is in India. Hence, Afcons Infrastructure International Ltd. is a non-resident company., Illustration 15: Bista Ltd., a foreign company and it carries on majority of its operations and decision, making activities from Calcutta and Assam but some part of operational activities and few decisions are, being taken from the place at which registered office of Bista Ltd. is located, i.e. Dhaka., Determine its residential status for the assessment year 2022-23., Solution:, Bista Ltd. is a foreign company and its place of effective management is in India. Hence Bista Ltd. is, resident in India for the assessment year 2022-23., Question 6: Explain how to determine residential status of other persons., Answer:, Local Authorities and Artificial Juridical Persons Section 6(4), Local authorities and artificial juridical persons would be resident in India if the control and management of, its affairs is situated wholly or partly in India. Where the control and management of the affairs is situated, wholly outside India, they would become non-residents.
Page 130 :
Residential Status & Scope of Total Income, , 130, , TAX INCIDENCE / SCOPE OF TOTAL INCOME, Question 7: Write a note on scope of total income or tax incidence., Answer: As per section 5, scope of total income or tax incidence in various status shall be as given, below:, (1) Resident and ordinarily resident – In case of ROR, the following incomes shall be taxable., (i), income accruing / arising in India., (ii), income received or deemed to be received in India even if accruing /arising abroad., (iii) income accruing / arising aboard and received aboard., In simpler terms, ROR has to pay tax on his world income in India., Meaning of income received in India, Income shall be considered to be received in India if it has been received directly in India from its source i.e., if the income has been received outside India and after that it was transferred to India, it will not be, considered to be income received in India rather it is income received abroad., Therefore, when once an amount is received as income, remittance or transmission of that amount from one, place or person to another does not constitute receipt of income in the hands of the subsequent recipient or at, the place of subsequent receipt., Example, Mr. X has one house in USA and rent has been received directly in India. It will be considered to be income, received in India and it is chargeable to tax in case of all the three status, but if Mr. X has one bank account, with Bank of America, New York and rent has been deposited in that account and subsequently the bank has, transferred the amount to Mr. X in India, it will be considered to be income received outside India, because, income has already been received outside India and subsequently it was remitted to India., Similarly, if Mr. X has income in Nepal and it was deposited in the branch of an Indian bank in Nepal,, subsequently the amount was remitted in India, it will be considered to be income received outside India., (2) Resident but not ordinarily resident – The following incomes shall be taxable., (i), income accruing / arising in India., (ii), income received or deemed to be received in India even if accruing /arising abroad., (iii) income accruing / arising aboard and received aboard but from a business controlled from India or, from a profession which was set up in India., Meaning of profession setup in India, Profession set up in India means that it was originally setup in India and subsequently there was an, expansion outside India. E.g. Mr. X started his profession of an advocate in Delhi and subsequently he, opened his branch outside India, it will be called profession setup in India., (3) Non-resident –The following incomes shall be taxable., (i), income accruing /arising in India., (ii), income received or deemed to be received in India even if accruing /arising abroad., NOV – 2020 EXAM (NEW COURSE), Question 4 (a), 5 Marks, Mr. Thomas, a non-resident and citizen of Japan entered into following transactions during the previous year, ended 31.03.2022. Examine the tax implications in the hands of Mr. Thomas for the Assessment Year 202223 as per Income Tax Act, 1961. (Give brief reasoning), (l) Interest received from Mr. Marshal, a non-resident outside India (The borrowed fund is used by Mr., Marshal for investing in Indian company's debt fund for earning interest), (2) Received `10 lakhs in Japan from a business enterprise in India for granting license for computer, software (not hardware Specific)., (3) He is also engaged in the business of running news agency and earned income of `10 lakhs from, collection of news and views in India for transmission outside India.
Page 131 :
Residential Status & Scope of Total Income, , 131, , (4) He entered into an agreement with SKK & Co., a partnership firm for transfer of technical documents, and design and for providing. services relating thereto, to set up a Denim Jeans manufacturing plant, in, Surat (India). He charged `10 lakhs for these services from SKK & Co., Solution:, (1) As per section 9, If loan has been taken by a non-resident, interest income shall be accruing / arising in, India only if loan amount has been utilised in India in business/profession but if loan amount is utilised in, any other source in India or it has been used outside India, interest income shall be accruing / arising abroad., In the given case, loan amount is used for investing in Indian company debt fund for earning interest and, not for business purpose hence interest income shall not be considered to be accruing arising from India and, shall not be taxable in India., (2) As per section 9, If any income is accruing and arising in India relating to royalty or technical fees etc., it, will be taxable in India even if the person receiving income is non-resident and even if such non-resident do, not have any Territorial Nexus with India i.e. such non-resident do not have a residence or place of business, or business connection in India and also the non-resident has not rendered services in India. In the given, case, income received for granting licence for computer software shall be deemed to be income accruing, arising in India and shall be taxable in India., (3) As per section 9, If any non-resident has the business of running a news agency or of publishing, newspapers, magazines or journals etc. outside India, no income shall be deemed to accrue or arise in, India to him from activities which are confined to the collection of news and views in India for transmission, out of India but if newspaper etc. is being sold in India, there will be business connection or if there is, telecasting or broadcasting of such news/views etc. in India, there will be business connection and income, shall be taxable to that extent., In the given case, income is from transmission outside India hence income shall not be deemed to accrue, arise in India and shall not be taxable in India., (4) As per section 9, income by way of fees for technical services payable by a person who is a nonresident, where the fees are payable in respect of services utilised in a business or profession carried on by, such person in India or for the purposes of making or earning any income from any source in India., In the given case, services utilized in a business in India hence income shall be accruing arising from India, and same shall be taxable in India., RTP NOV -2020, 2. Mr. Nishant, a resident but not ordinarily resident for the previous year 2020-21 and resident and, ordinarily resident for the previous year 2021-22 has received rent from property in Canada amounting to, `1,00,000 during the P.Y.2020-21. He has deposited the same in a bank in Canada. During the financial year, 2021-22, he remitted this amount to India through approved banking channels. Is such rent taxable in India,, and if so, how much and in which year?, (a) Yes; ` 70,000 was taxable in India during the previous year 2020-21., (b) Yes; ` 1,00,000 was taxable in India during the previous year 2020-21., (c) Yes; ` 70,000 was taxable in India during the previous year 2021-22., (d) No; such rent is not taxable in India either during the previous year 2020-21 or during the previous, year 2021-22., Answer: (d), Hint: Since Mr. Nishant is a resident but not ordinarily resident, amount received outside India from, rental income outside India is not taxable in India, Illustration 16: Mr. X has income asunder:, 1. He has income from a business in Germany amounting to `3,00,000 and half of it was received in, India., 2. He has interest income of `1,00,000 from UK Development Bond and entire interest income was, credited to a bank account in UK. Subsequently, the amount was transferred in India., 3. He has a business in Bombay and entire income of `3,00,000 was received in UK.
Page 132 :
Residential Status & Scope of Total Income, , 132, , 4. He has one house property in Ghaziabad and income of `5,00,000 was received in UK., 5. He has received salary income of `5,00,000 (computed) in India and half of the services were, rendered in UK and half in India., (Presume all the above incomes are computed incomes), Compute his income presuming that he is NOR, NR and ROR., Solution:, ROR, NOR, NR, 1. Income received in India, 1,50,000, 1,50,000, 1,50,000, Income accruing/arising abroad and received abroad, 1,50,000, xxxxx, xxxxx, 2. Income accruing/arising abroad and received abroad, 1,00,000, xxxxx, xxxxx, 3. Income accruing/arising in India, 3,00,000, 3,00,000, 3,00,000, 4. Income accruing/arising in India, 5,00,000, 5,00,000, 5,00,000, 5. Income received in India, 5,00,000, 5,00,000, 5,00,000, Total, 17,00,000, 14,50,000, 14,50,000, Illustration 17: ABC partnership firm has an income of `3 lakhs in India and income accruing/arising, abroad and also received abroad `23 lakhs. It consists of two partners. Mr. X who is an active partner, is, staying outside India throughout the year. Mr. Y is a dormant partner and is staying in India throughout the, year., Compute tax liability of the partnership firm in India for the assessment year 2022-23., (b) Also compute tax liability of the firm if Mr. Y is also an active partner., Solution:, `, (a) Partnership firm is non-resident, Income from business/profession in India, 3,00,000, Gross Total Income, 3,00,000, Less: Deduction u/s 80C to 80U, Nil, Total Income, 3,00,000, Tax @ 30% + HEC @ 4%, 93,600, (b) Partnership firm is resident, Income from business/profession, 26,00,000, Gross Total Income, 26,00,000, Less: Deduction u/s 80C to 80U, Nil, Total Income, 26,00,000, Tax @ 30% + HEC @ 4%, 8,11,200, Illustration 18: Mr. X had following income during the previous year ended 31st March, 2022:, (1) Salary received in India for three months (being computed income), (2) Income from house property in India, (3) Interest on savings bank deposit in SBI, in India, (4) Amount brought into India out of the past-untaxed profits earned in Germany, (5) Income from business in Bangladesh, being controlled from India, You are required to compute his gross total income for the assessment year 2022-23, if he is a, (a) resident and ordinarily resident;, (b) not ordinarily resident; and, (c) non-resident., Presume all the above income is computed income., Solution:, ROR, NOR, (1) Salary received in India, 25,000 25,000, • Taxable on receipt basis, (2) Income from house property in India, 18,000 18,000, • Income accruing/arising in India, , `, 25,000, 18,000, 4,000, 20,500, 12,542, , NR, 25,000, 18,000
Page 133 :
Residential Status & Scope of Total Income, (3) Interest on savings bank deposit in SBI, in India, • Income accruing/arising in India, (4) Past untaxed profits brought in India, • Not an income of the previous year 2020-21 hence not taxable, (5) Income from business in Bangladesh being controlled from India, • Not taxable in case of non resident, Gross Total Income, , 133, 4,000, , 4,000, , 4,000, , —, , —, , —, , 12,542, , 12,542, , —, , 59,542, , 59,542, , 47,000, , Illustration 19: Mr. X earns the following income during the financial year 2021-22:, `, (1) Income from house property in London, received in India, 60,000, (2) Profits from business in Japan and managed from there (received in Japan), 9,00,000, (3) Profits from business in Kenya, controlled from India, Profits received in Kenya, 3,00,000, (4) Profits from business in Delhi, managed from Japan, 7,00,000, (5) Capital gains on transfer of shares of Indian companies, sold in USA and gains were, received there, 2,00,000, (6) Pension from former employer in India, received in Japan, 50,000, (7) Profits from business in Pakistan, deposited in bank there, 20,000, (8) Profit on sale of asset in India but received in London, 8,000, (9) Past untaxed profits of UK business of 2018-19 brought into India in 2021-22, 90,000, (10) Interest on Government securities accrued in India but received in Paris, 80,000, (11) Interest on USA Government securities, received in India, 20,000, (12) Salary earned in Bombay, but received in UK, 60,000, (13) Income from property in Paris, received there, 1,00,000, (Presume all the above incomes are computed incomes), Determine the gross total income of Mr. X if he is (i) resident and ordinarily resident, resident but not, ordinarily resident, non-resident in India during the financial year 2021-22., Solution:, ROR, NOR, NR, (1) Income received in India, 60,000, 60,000, 60,000, (2) Income accruing/arising and received outside India, 9,00,000, —, —, (3) Income accruing/arising and received outside India, but, 3,00,000, 3,00,000, —, business controlled from India, (4) Income accruing/arising in India, 7,00,000, 7,00,000, 7,00,000, (5) Income accruing/arising in India, 2,00,000, 2,00,000, 2,00,000, (6) Income accruing/arising in India, 50,000, 50,000, 50,000, (7) Income accruing/arising and received outside India, 20,000, —, —, (8) Income accruing/arising in India, 8,000, 8,000, 8,000, (9) Past untaxed profits, —, —, —, (10) Income accruing/arising in India, 80,000, 80,000, 80,000, (11) Income received in India, 20,000, 20,000, 20,000, (12) Income accruing/arising in India, 60,000, 60,000, 60,000, (13) Income accruing/arising and received outside India, 1,00,000, —, —, Gross Total Income, 24,98,000 14,78,000, 11,78,000, Illustration 20: ABC Pvt. Ltd., an Indian company has an income of `30 lakhs from a business in India., This company has a business income of `12 lakhs from outside India. Out of which 7 lakhs were received in, India and balance outside India., Compute tax liability of the Indian company for the assessment year 2022-23., Solution:, `, Income from business in India, 30,00,000, Income from outside India, 12,00,000
Page 134 :
Residential Status & Scope of Total Income, , 134, , Income under the head Business/Profession, 42,00,000, Gross Total Income, 42,00,000, Less: Deductions u/s 80C to 80U, Nil, Total Income, 42,00,000, Computation of Tax Liability, Tax on `42,00,000 @ 30%, 12,60,000, Add: HEC @ 4%, 50,400, Tax Liability, 13,10,400, Note: Indian company is always considered to be resident in India and its incomes even earned and received, outside India shall be chargeable to tax in India., Illustration 21: Mr. X, a foreign citizen (not being a person of Indian origin) came to India for the first time, on 2nd December, 2021 for a visit of 210 days. Mr. X had the following income during the previous year, `, ended 31st March, 2022:, (1) Salary (computed) received in India for three months, 1,00,000, (2) Income from house property in London (received there), 2,75,200, (3) Amount brought into India out of the past-untaxed profits earned in Germany, 80,000, (4) Income from agriculture in Sri Lanka, received and invested there, 12,300, (5) Income from business in Nepal, being controlled from India, 35,000, (6) Income from house property in USA received in USA, (`76,000 is used in Canada for meeting the educational expenses of Mr. X’s daughter and, ` 10,000 is later on remitted in India), 86,000, You are required to compute his total income for the assessment year 2022-23., Solution:, Mr. X is a foreign citizen. He was in India during the previous year 2021-22 for 120 (30 + 31 + 28 + 31), days. Thus, he does not satisfy the first condition of 182 days. The second condition is also not satisfied as, Mr. X came to India for first time during the previous year 2021-22., Mr. X is therefore non–resident in India. The total income of Mr. X for the assessment year 2022-23 will be:, `, (1) Salary (computed) received in India for three months, • Taxable on receipt basis, 1,00,000, (2) Income from house property in London (received there), • Not taxable as income is accruing & arising outside India and is also received outside India, —, (3) Amount brought in India out of the past untaxed-profits earned in Germany, • Not taxable as it is not income, —, (4) Income from agriculture in Sri Lanka being invested there, • Income accrued and received outside India, —, (5) Income from business in Nepal, being controlled from India, • Not taxable in the case of non- resident, —, (6) Income from house property in USA received in USA, (` 76,000 is used in Canada or meeting the educational expenses of Mr. X’s daughter and, ` 10,000 is later on received in India), • Income accrued and received outside India, —, Gross Total Income, 1,00,000, Less: Deduction u/s 80C to 80U, Nil, Total Income, 1,00,000, Illustration 22: Mr. X earns the following incomes during the financial year 2021-22., (1) Profits from a business in Japan, controlled from India,, (half of the profits received in India), (2) Income from property in Bombay, received in UK, (3) Income from a property in USA, received there but subsequently remitted to India, (4) Income from property in USA, received there (`50,000 remitted in India), , `, 40,000, 70,000, 2,00,000, 80,000
Page 135 :
Residential Status & Scope of Total Income, , 135, , (5) Salary received in India for services rendered in USA, 50,000, (6) Income from profession in Paris, which was set up in India, received in Paris, 80,000, (7) Interest from deposit with an Indian company, received in Japan, 9,000, (8) Income from profession in Bombay received in Paris, 30,000, (9) Profits of business in Iran, deposited in a bank there, business controlled from India, (out of `4,00,000, ` 1,00,000 is remitted in India), 4,00,000, (10) Interest on German development bonds, half of which is received in India, 10,000, (11) Income from property in Canada, one-fifth is received in India, 50,000, (Presume all the above incomes are computed income i.e. all the exemptions and deductions have already, been allowed), Determine the gross total income of Mr. X if he is (i) resident and ordinarily resident, (ii) resident but not, ordinarily resident, (iii) non-resident in India during the financial year 2021-22., Solution:, ROR, NOR, NR, (1) Income accruing/arising outside India from a business controlled in, 40,000, 40,000, 20,000, India, half of the income received in India, (2) Income accruing/arising in India, 70,000, 70,000, 70,000, (3) Income accruing/arising outside India and received outside India, 2,00,000, —, —, (4) Income accruing/arising outside India and received outside India, 80,000, —, —, (5) Income received in India, 50,000, 50,000, 50,000, (6) Income accruing/arising and received outside India, but profession, 80,000, 80,000, —, set up in India, (7) Income accruing/arising in India, 9,000, 9,000, 9,000, (8) Income accruing/arising in India, 30,000, 30,000, 30,000, (9) Income accruing/arising outside India and received outside India,, but business controlled from India, 4,00,000 4,00,000, —, (10) Income accruing/arising outside India, half received outside India, 10,000, 5,000, 5,000, and half in India, 50,000, 10,000, 10,000, (11) Income accruing/arising outside India, 4/5th received outside India, and 1/5th in India, Gross Total Income, , 10,19,000, , 6,94,000, , 1,94,000, , Illustration 23: Mr. X is a citizen of India and is employed in ABC Limited and getting salary `1,00,000, p.m. and he was transferred out of India on 01.09.2021 and he left India for first time from 01.09.2021 and, he visiting in India from 26.01.2022 to 15.02.2022 and salary for January 2022 was received in India and at, the time of departure he received 3 gifts `20,000 from 3 friends each and also a phone of `70,000., He has agricultural income in India `4,00,000, Compute his tax liability for assessment year 2022-23., Solution:, Mr. X shall be covered in special category so therefore his status shall be non-resident., `, (30 + 31 + 30 + 31 + 31 + 1 + 6 + 15) 175 days., Income under the head Salary accruing / arising in India, 1,00,000 x 5, 5,00,000, Income received in India, 1,00,000 x 1, 1,00,000, Gross Salary, 6,00,000, Less: Deduction u/s 16(ia), (50,000), Income under the head salary, 5,50,000, Income under the head Other Sources, Gross Total Income, , 60,000, 6,10,000
Page 136 :
Residential Status & Scope of Total Income, Less: Deduction 80C to 80U, Total Income, Agricultural Income, Computation of Tax Liability, Step 1. Tax on (6,10,000 + 4,00,000 = 10,10,000) at slab rate, Step 2. Tax on (4,00,000 + 2,50,000 = 6,50,000) at slab rate, Deduct step 2 from step 1, Add: HEC 4%, Tax Liability, , 136, Nil, 6,10,000, 4,00,000, 1,15,500, (42,500), 73,000, 2,920, 75,920, , Question 8: Explain meaning of income deemed to accrue or arise in India Section 9., Answer:, Income deemed to the accruing/arising in India shall be taxable in all the three status i.e. ROR/NOR/NR., As per section 9, the following incomes shall be deemed to be accruing / arising in India., 1. If any income has its source in India, such income shall be considered to be accruing / arising in India i.e., employment/house property/business/profession/capital asset or any other source of income is in India but if, source is partly in India and partly outside India, income shall be accruing / arising in India only to the, extent the source is in India e.g. Mr. X is employed in Punjab National Bank and is posted in Delhi branch, on a salary of `1,00,000 p.m. In this case, his income shall be deemed to be accruing/arising in India but if, he is transferred to the London branch w.e.f 01.01.2022, his income accruing/arising in India shall be, `9,00,000 i.e. salary upto 31.12.2021 and the income which is accruing/arising abroad shall be `3,00,000, (i.e. salary from 01.01.2022 to 31.03.2022), Business connection, If any person has business in India as well as outside India, it will be called business connection and in case, of such business, the income of the business deemed to accrue or arise in India shall be only such part of the, income as is reasonably attributable to the operations carried out in India and as per rule 10, assessing, officer shall have the powers to determine the extent upto which income is accruing/arising in India., There will be a business connection if any non-resident has business outside India but has agent in India who, (a) habitually secures orders in India, for the non-resident., (b) habitually maintains in India a stock of goods from which he regularly delivers goods on behalf of the, non-resident or, (c) habitually concludes contracts on behalf of the non-resident or plays the principal role leading to, conclusion of contracts and the contracts are in the name of non - resident or the contracts are for the transfer, of ownership or for granting of right to use property owned by that non- resident or the provision of services, by the non - resident., There is no business connection in the following three cases:, (a) If any non-resident has business outside India but such person is purchasing goods from India and do not, have any other activity in India, in this case there is no business connection but if such person has any other, activity in India, it will be considered to be business connection. e.g. Mr. X a non-resident has one shop in, New York for selling Indian goods and all these goods are purchased from India. In this case, there is no, business connection. However, if assessee is carrying out any other activity in India, it will be considered to, be business connection., If in the above case the assessee has manufacturing unit in India, it will be considered to be a business, connection., (b) If any non-resident has the business of running a news agency or of publishing newspapers,, magazines or journals etc. outside India, no income shall be deemed to accrue or arise in India to him, from activities which are confined to the collection of news and views in India for transmission out of India, but if newspaper etc. is being sold in India, there will be business connection or if there is telecasting or, broadcasting of such news/views etc. in India, there will be business connection and income shall be taxable, to that extent., (c) If any non-resident is doing shooting of any cinematograph film in India, there is no business
Page 137 :
Residential Status & Scope of Total Income, , 137, , connection but if such film is being shown in India, there will be business connection., “Explanation 3A.–– The income attributable to the operations carried out in India for the purpose of, business connection, shall include income from––, (i) such advertisement which targets a customer who resides in India or a customer who accesses the, advertisement through internet protocol address located in India;, (ii) sale of data collected from a person who resides in India or from a person who uses internet, protocol address located in India; and, (iii) sale of goods or services using data collected from a person who resides in India or from a person, who uses internet protocol address located in India.”, , SIGNIFICANT ECONOMIC PRESENCE (SEP), Significant economic presence of a non-resident in India shall also be considered to be business, connection and SEP means (a) transaction in respect of any goods, services or property carried out by a non-resident with in India, including provision of download of data or software in India, if the aggregate of payments arising from, such transaction or transactions during the previous year exceeds such amount as may be prescribed; or, (b) systematic and continuous soliciting of business activities or engaging in interaction with such, number of users as may be prescribed, in India through digital means:, Provided that the transactions or activities shall constitute significant economic presence in India,, whether or not,—, (i) the agreement for such transactions or activities is entered in India;, (ii) the non-resident has a residence or place of business in India; or, (iii) the non-resident renders services in India:, Provided further that only so much of income as is attributable to the transactions or activities referred, above shall be deemed to accrue or arise in India.’, 2. If any person is holding shares of any Indian company, any capital gain on transfer of such shares shall be, considered to be income accruing/arising in India even if shares were sold outside India., In case of shares of a foreign company, capital gains shall be accruing / arising in India if the value of the, shares is because of the assets located in India or because of business in India (the amendment is to overrule, the judgment in Vodafone case)., 3. If any individual is a citizen of India and is an employee of the government and is posted outside India,, his salary income shall be accruing / arising in India e.g. Mr. X is citizen of India and is an IFS. He is posted, in Indian embassy in USA, in this case, his salary income shall be accruing/arising in India., 4. If any loan has been taken by the government from outside India, interest paid by the government shall be, considered to be income of the person who has received such interest and it is accruing / arising in India and, it do not matter whether loan was used in India or outside India. e.g. If Central Government has taken a loan, from an agency in USA, equivalent to Indian `1,000 lakh @ 10%, in this case, interest of `100 lakhs paid by, the Government to such agency shall be considered to be the income of such agency accruing/arising in, India., If such loan has been taken by a person who is resident in India, interest income shall be accruing / arising in, India only if loan amount has been used in India but if loan amount has been utilized outside India it will be, accruing / arising abroad. E.g. ABC Ltd. an Indian company has taken a loan from an agency in USA and, the amount was utilized in USA. In this case, interest income shall be accruing/arising in USA but if loan, amount is used in India in any source, it will be accruing / arising in India, If such loan has been taken by a non-resident, interest income shall be accruing / arising in India only if loan, amount has been utilised in India in business/profession but if loan amount is utilised in any other source in, India or it has been used outside India, interest income shall be accruing / arising abroad. E.g. X Ltd. a nonresident company has taken a loan from outside India and loan amount was utilized in India in house, property. In this case, interest paid by the company shall be income of the recipient accruing/arising abroad, but if loan amount was utilised in India in business/profession, interest income shall be considered to be, accruing/arising in India. The person receiving interest shall be liable to pay income tax on such income, even if such person do not have any Territorial Nexus with India i.e. such non-resident do not have a
Page 138 :
Residential Status & Scope of Total Income, , 138, , residence or place of business or business connection in India, MAY – 2006 (2 Marks), Mr. X, left for USA on 01.05.2021. He has not visited India thereafter. Mr. X borrows money from his, friend Mr. Y, who left India one week before Mr. X's departure, to the extent of `10 lakhs and buys shares in, X Ltd., an Indian company. Discuss the taxability of the interest charged @ 10% in Mr. Y's hands where the, same has been received in New York., Answer: Stay of Mr. X and Mr. Y during the previous year 2021-22 is less than 60 days hence both of them, are non-residents as per section 6(1)., As per section 9, if any non-resident has taken loan from outside India and the loan was utilized in India in, any source other than business or profession, interest received by the person who has given the loan shall not, be considered to be accruing/arising in India and is not taxable in India. In the given case, loan amount was, invested in the shares of an Indian company hence interest received by Mr. Y shall not be considered to be, income accruing/arising in India., 5. If government has taken any patent right or any technical services from outside India and has paid royalty, or technical fee for such patent right etc., it will be considered to be income of the person who has received, it and it is accruing / arising in India even if the patent right etc. has been used outside India., If such payment is being given by any resident or non-resident, it will be income of the recipient accruing /, arising in India only if such patent right etc. has been used in India otherwise it will be accruing / arising, abroad., Royalty means amount payable in connection with patent, invention, model, design, formula, process, trade, marks etc., Fees for Technical Services means any consideration for the rendering of Managerial, Technical or, Consultancy Services., If any income is accruing and arising in India relating to royalty or technical fees etc., it will be taxable in, India even if the person receiving income is non-resident and even if such non-resident do not have any, Territorial Nexus with India i.e. such non-resident do not have a residence or place of business or business, connection in India and also the non-resident has not rendered services in India., 6. If any person has received pension, it will be deemed to be accruing/arising in India if the employer is in, India. E.g. Mr. X is settled in Canada and is getting a pension of `30,000 p.m. from Punjab National Bank,, in this case his pension income shall be accruing/arising in India., 7. If any resident has gifted any amount or has gifted any property situated in India to any person outside, India, it will be considered to be income accruing/arising from India., MAY-2012 (3 Marks), Discuss the correctness or otherwise of the statement – “Income deemed to accrue or arise in India to a nonresident by way of interest, royalty and fees for technical services is to be taxed irrespective of territorial, nexus”., Answer: As per section 9, if any non-resident has provided any patent right or any managerial, technical, services and such patent right etc was used in India, in such cases any royalty or fee received by nonresident shall be considered to be income accruing/arising in India and shall be taxable and it do not matter, that the non-resident do not have residence or place of business or business connection in India i.e. there is, no territorial nexus or non-resident has not rendered services in India. E.g. If Suzuki Incorporation of Japan, a non-resident company has provided technical know-how in Japan to Maruti Udyog Limited for use in, India and has received `300,00,000 in this case, such income is deemed to be accruing/arising in India and is, taxable in India even if Suzuki Incorporation do not have any Territorial Nexus with India i.e. the company, do not have place of residence or place of business in India. Similarly if any loan was given by a nonresident to some other non-resident and such other non-resident has utilized loan amount in India in, business/profession, interest received by the non-resident shall be considered to be his income, accruing/arising in India even if such non-resident do not have any territorial nexus with India., MAY – 2011 (3 Marks), Miss Vivitha paid a sum of 5000 USD to Mr. Kulasekhara, a management consultant practicing in Colombo,, specializing in project financing. The payment was made in Colombo. Mr. Kulasekhara is a non-resident.
Page 139 :
Residential Status & Scope of Total Income, , 139, , The consultancy related to a project in India with possible Ceylonese collaboration. Is this payment, chargeable to tax in India in the hands of Mr. Kulasekhara, since the services were used in India?, Answer: As per section 9, if any non-resident has provided any patent right or any managerial, technical, services and such patent right etc was used in India, in such cases any royalty or fee received by nonresident shall be considered to be income accruing/arising in India and shall be taxable and it do not matter, that the non-resident do not have residence or place of business or business connection in India i.e. there is, no territorial nexus or non-resident has not rendered services in India. In the instant case, since the services, were utilized in India, the payment received by Mr. Kulasekhara, a non-resident, in Colombo is chargeable, to tax in his hands in India, as it is deemed to accrue or arise in India., NOV – 2009 (4 Marks), Determine the taxability of income of US based company ABC Ltd., in India on entering following, transactions during the financial year 2021-22:, (i) `5 lacs received from an Indian domestic company for providing technical know how in India., (ii) `6 lacs from an Indian firm for conducting the feasibility study for the new project in Finland., (iii) `4 lacs from a non-resident for use of patent for a business in India., (iv) `8 lacs from a non-resident Indian for use of know how for a business in Singapore., (v) `10 lacs for supply of manuals and designs for the business to be established in Singapore., Explain the rate of tax applicable on taxable income for US based company, ABC Ltd., in India., Answer: A non resident is chargeable to tax in India in respect of following incomes:, (i) Income received or deemed to be received in India., (ii) Income accruing or arising or deemed to accrue or arise in India., In view of the above provisions, taxability of income is determined in following manner:, S. No. Transaction details, Amount (`), (i), Amount received from an Indian domestic company for providing technical, 5 Lacs, know how in India is taxable in India, (ii), Conducting the feasibility study for the new project in Finland for the Indian firm, Nil, is not taxable in India as it is for the business outside India., (iii), Money received from a non resident for use of patent for a business in India is, 4 Lacs, taxable in India, (iv), Money received from a non resident Indian for use of know-how for a business in, Nil, Singapore is for the business outside India, therefore not taxable in India., Nil, (v), Payment received for supply of manuals and designs for the business to be, established in Singapore is not taxable in India., Total Income in India, 9 Lacs, The basic normal rate applicable for the US based company who is a foreign company is 40% In case the, taxable income is more than 1 crore but upto `10 crore in the previous year, the surcharge @ 2% is, applicable. The HEC is payable @ 4%., Question 9: Explain income deemed to be received in India., Answer: Such incomes is taxable in all the three status. Under section 7, employer’s contribution to, Recognised Provident Fund in excess of 12% of salary of the employee shall be considered to be income, deemed to be received in India. Similarly, interest on the provident fund balance in excess of 9.5% p.a. shall, be considered to be income deemed to be received in India., As per provisions of employees provident fund and miscellaneous provisions Act,1952, every employee and, employer shall contribute to Recognised provident fund @ 12% of salary of employee and for this purpose, account shall be opened in the name of the employee with provident fund commissioner. Interest earned on, such amount shall be credited to the RPF. Employer’s contribution in excess of 12% of salary of employee, shall be taxable under the head salary. Interest on employer’s contribution in excess of 9.5% per annum shall, be taxable under the head salary. Interest on employees contribution in excess of 9.5% per annum shall be, taxable under the head other sources., Deduction shall be allowed under section 80C from gross total income for employees contribution but, maximum deduction allowed shall be ` 1,50,000 (Details given under section 80C)
Page 140 :
Residential Status & Scope of Total Income, , 140, , If any Seafarer (crew member of ship) is Non-resident and Income is accruing/arising abroad and his, income has been received directly in his bank account in India, such income shall not be taxable., Illustration 24: Mrs. X is a citizen of India and is employed in ABC Ltd. in India and is getting salary of, `60,000 p.m. and she was transferred out of India w.e.f 01.09.2021 and for this purpose she left India on, 01.09.2021 for the first time and she visited India from 27.12.2021 to 07.01.2022 and her salary for the, month of Dec’ 2021 was received in India. Employer and employee both have contributed @ 13% (each) of, salary to the recognized provident fund and during the year interest of `50,000 was credited to the, recognized provident fund @ 10% p.a., Compute her total income and tax liability in India for assessment year 2022-23., (b) Presume she was transferred w.e.f 01.11.2021 and she left India on 01.11.2021 for the first time., Solution:, As per section 6(1), in this case, Mrs. X is covered in special category and her stay in India is less than 182, days hence she will be non-resident and her incomes taxable in India shall be, `, Income accruing/arising in India, 3,00,000.00, 60,000 x 5, Income received in India, 60,000.00, 60,000 x 1, Income deemed to be received in India, Employer contribution, 7,200.00, (60,000 x 12) x 1% (13% - 12%), Interest in excess of 9.5%, 50,000 /10% x 0.5% = 2,500, Interest on employer contribution, 1,250.00, 2,500 /2, (Interest on employee contribution i.e. `1,250, shall be taxable under the head Other Sources), Gross Salary, 3,68,450.00, Less: Standard Deduction u/s 16(ia), (50,000.00), Income under the head Salary, 3,18,450.00, Income under the head Other Sources, 1,250.00, Gross Total Income, 3,19,700.00, Less: Deduction u/s 80C, (93,600.00), Contribution to recognized provident fund, (60,000 x 12) x 13%, Total Income, 2,26,100.00, Computation of Tax Liability, Tax on `2,26,100 at slab rate, Tax Liability, , Nil, Nil, , Solution (b):, In this case, Mrs. X is covered in special category and her stay in India is more than 182 days hence she will, be ROR and her incomes taxable in India shall be, `, Income accruing/arising in India, 4,20,000.00, 60,000 x 7, Income received in India, 60,000.00, 60,000 x 1, Income accruing/arising abroad / received abroad, 2,40,000.00, 60,000 x 4, Income deemed to be received in India, Employer contribution, 7,200.00
Page 141 :
Residential Status & Scope of Total Income, (60,000 x 12) x 1% (13% - 12%), Interest in excess of 9.5%, 50,000 /10% x 0.5% = 2,500, Interest on employer contribution, 2,500 /2, (Interest on employee contribution i.e. `1,250, shall be taxable under the head Other Sources), Gross Salary, Less: Deduction u/s 16(ia), Income under the head Salary, Income under the head Other Sources, Gross Total Income, Less: Deduction u/s 80C, Contribution to recognized provident fund, (60,000 x 12) x 13%, Total Income, , 141, , 1,250.00, , 7,28,450.00, (50,000.00), 6,78,450.00, 1,250.00, 6,79,700.00, (93,600.00), 5,86,100.00, , Computation of Tax Liability, Tax on `5,86,100 at slab rate, 29,720.00, Add: HEC @ 4%, 1,188.80, Tax Liability, 30,908.80, Rounded off u/s 288B, 30,910.00, Any past untaxed profits shall not be considered to be the income of the current year in any status i.e., ROR, NOR, NR., Example, Mr. X had income of `3,00,000 in the year 2018-19 but he has not disclosed the income. It was detected in, the previous year 2021-22. In this case, it will not be considered to be income of 2021-22 in any status,, rather it will be considered to be income of the year 2018-19., Illustration 25: Determine the taxability of the following incomes in the hands of a resident and ordinarily, resident, resident but not ordinarily resident, and non-resident for the A.Y. 2022-23 –, Particulars, Amount (` ), (1) Interest on UK Development Bonds, 50% of interest received in India, 10,000, (2) Income from a business in Chennai (50% is received in India), 20,000, (3) Profits on sale of shares of an Indian company received in London, 20,000, (4) Profits on sale of plant at Germany 50% of profits are received in India, 40,000, 70,000, (5) Income earned from business in Germany which is controlled from Delhi (`40,000 is, received in India), (6) Profits from a business in Delhi but managed entirely from London, 15,000, (7) Income from property in London deposited in a Indian Bank at London, brought to, 50,000, India, (8) Interest for debentures in an Indian company received in London., 12,000, (9) Fees for technical services rendered in India but received in London, 8,000, (10) Profits from a business in Bombay managed from London, 26,000, (11) Pension for services rendered in India but received in Burma, 4,000, (12) Income from property situated in Pakistan received there, 16,000, (13) Past foreign untaxed income brought to India during the previous year, 5,000, (14) Income from agricultural land in Nepal received there and then brought to India, 18,000, (15) Income from profession in Kenya which was set up in India, received there but spent, 5,000, in India, (16) Gift received on the occasion of his wedding, 20,000, (17) Interest on savings bank deposit in State Bank of India, 10,000, (18) Income from a business in Russia, controlled from Russia, 20,000
Page 142 :
Residential Status & Scope of Total Income, (19) Agricultural income from a land in Rajasthan, Computation of Gross Total Income for the A.Y.2022-23, Particulars, Resident and, Resident but not, ordinarily resident ordinarily resident, `, `, (1) Interest on UK Development Bonds, 50% of, 10,000, 5,000, interest received in India, 20,000, 20,000, (2) Income from a business in Chennai (50% is, received in India), (3) Profits on sale of shares of an Indian company, 20,000, 20,000, received in London, (4) Profits on sale of plant at Germany 50% of, 40,000, 20,000, profits are received in India, (5) Income earned from business in Germany, 70,000, 70,000, which is controlled from Delhi, out of which, `40,000 is received in India, (6) Profits from a business in Delhi but managed, 15,000, 15,000, entirely from London, (7) Income from property in London deposited in, 50,000, a Bank at London, later on remitted to India, (8) Interest for debentures in an Indian company, 12,000, 12,000, received in London., (9) Fees for technical services rendered in India, 8,000, 8,000, but received in London, (10) Profits from a business in Bombay managed, 26,000, 26,000, from London, (11) Pension for services rendered in India but, 4,000, 4,000, received in Burma, (12) Income from property situated in Pakistan, 16,000, received there, (13) Past foreign untaxed income brought to India, during the previous year, (14) Income from agricultural land in Nepal, 18,000, received there and then brought to India, (15) Income from profession in Kenya which was, 5,000, 5,000, set up in India, received there but spent in India, (16) Gift received on the occasion of his wedding, [not an income], (17) Interest on savings bank deposit in State, 10,000, 10,000, Bank of India, (18) Income from a business in Russia, controlled, 20,000, from Russia, (19) Agricultural income from a land in Rajasthan, [it is exempt u/s 10(1)], Gross Total Income, 3,44,000, 2,15,000, , 142, 15,000, , Solution:, , Non, resident, `, 5,000, 20,000, 20,000, 20,000, 40,000, 15,000, 12,000, 8,000, 26,000, 4,000, 10,000, 1,80,000
Page 143 :
Residential Status & Scope of Total Income, , 143, , MULTIPLE CHOICE QUESTIONS, 1. If Anirudh has stayed in India in the P.Y. 2021-22 for 181 days, and he is non-resident in 9 out of 10, years immediately preceding the current previous year and he has stayed in India for 365 days in all, in the 4 years immediately preceding the current previous year and 420 days in all in the 7 years, immediately preceding the current previous year, his residential status for the A.Y.2022-23 would be(a) Resident and ordinarily resident, (b) Resident but not ordinarily resident, (c) Non-resident, (d) Cannot be ascertained with the given information, 2. Raman was employed in Hindustan Lever Ltd. He received a salary of `40,000 p.m. from 1.4.2021, to 27.9.2021. He resigned and left for Dubai for the first time on 1.10.2021 and got salary of rupee, equivalent of `80,000 p.m. from 1.10.2021 to 31.3.2022. His salary for October to December 2021 was, credited in his Dubai bank account and the salary for January to March 2022 was credited in his, Bombay account directly. He is liable to tax in respect of (a) Income received in India from Hindustan Lever Ltd;, (b) Income received in India and in Dubai;, (c) Income received in India from Hindustan Lever Ltd. and income directly credited in India;, (d) Income received in Dubai, 3. A company would be a resident in India for the P.Y. 2021-22, if, (a) it is an Indian company, (b) during the year, majority of its directors are resident in India, (c) during the year, its Place of Effective Management is in India, (d) both (a) and (c), 4. Income accruing in London and received there is taxable in India in the case of(a) resident and ordinarily resident only, (b) both resident and ordinarily resident and resident but not ordinarily resident, (c) both resident and non-resident, (d) non-resident, 5. Incomes which accrue or arise outside India but received directly in India are taxable in case of(a) resident and ordinarily resident only, (b) both resident and ordinarily resident and resident but not ordinarily resident, (c) non-resident, (d) All the above, 6. Income earned from a contract negotiated by an agent in India in the name of a non-resident and, approved by such non-resident shall:, (a) be taxable in India as such income is deemed to accrue or arise in India, (b) not be taxable in India as there is no business connection in India, (c) be taxable in India only if it is received in India, (d) not taxable in India as such income accrues or arises outside India, 7. Fees for technical services paid by the Central Government will be taxable in case of –, (a) resident and ordinarily resident only, (b) both resident and ordinarily resident and resident but not ordinarily resident, (c) non-resident, (d) All the above, 8. Short term capital gains on sale of shares of an Indian company received in Australia is taxable in, case of–, (a) resident and ordinarily resident only, (b) both resident and ordinarily resident and resident but not ordinarily resident, (c) non-resident only, (d) All the above, 9. Income from a business in Canada, controlled from Canada is taxable in case of –, (a) resident and ordinarily resident only
Page 144 :
Residential Status & Scope of Total Income, , 144, , (b) both resident and ordinarily resident and resident but not ordinarily resident, (c) non-resident, (d) All the above, 10. Income from Australian company received in Australia in the year 2019, brought to India during, the previous year 2021-22 is taxable in case of –, (a) resident and ordinarily resident only, (b) resident but not ordinarily resident, (c) non-resident, (d) None of the above, 11. If Mr. Akash has stayed in India in the P.Y. 2021-22 for 100 days, and he is non-resident in 9 out, of 10 years immediately preceding the current previous year and he has stayed in India for 365 days, in all in the 4 years immediately preceding the current previous year and 730 days in all in the 7 years, immediately preceding the current previous year, his residential status for the A.Y.2022- 23 would be(a) Resident and ordinarily resident, (b) Resident but not ordinarily resident, (c) Non-resident, (d) Cannot be ascertained with the given information, 12. If Mr. A has stayed in India in the P.Y. 2021-22 for 100 days, and he is non-resident in 8 out of 10, years immediately preceding the current previous year and he has stayed in India for 365 days in all, in the 4 years immediately preceding the current previous year and 710 days in all in the 7 years, immediately preceding the current previous year, his residential status for the A.Y.2022- 23 would be(a) Resident and ordinarily resident, (b) Resident but not ordinarily resident, (c) Non-resident, (d) Cannot be ascertained with the given information, 13. Mr. A, a Canadian citizen, comes to India for the first time during the P.Y. 2017-18. He was in, India during 2017-18- 55 days, 2018-19 – 60 days, 2019-20 – 90 days, 2020-21 – 150 days, 2021-22 – 70, days. Residential status for the previous year 2021-22 shall be, (a) Resident and ordinarily resident, (b) Resident but not ordinarily resident, (c) Non-resident, (d) Cannot be ascertained with the given information, 14. Karta of one HUF comes to India every year for minimum 10 days and maximum 104 days,, residential status of HUF shall be, (a) Resident and ordinarily resident, (b) Resident but not ordinarily resident, (c) Non-resident, (d) Cannot be ascertained with the given information, 15. ABC Ltd. an Indian company has most of its business outside India and also control and, management outside India. Residential status of company shall be, (a) Resident, (b) Resident but not ordinarily resident, (c) Non-resident, (d) None of the above, 16. A Korean company received `20 lakhs from a non – resident for use of patent for a business in, India is _____________, (a) taxable in India, (b) not taxable in India, (c) None of the above, 17. A non-resident received `15 lakhs from a Foreign Company outside India, it is_____, (a) taxable in India, (b) not taxable in India, (c) partly taxable in India
Page 145 :
Residential Status & Scope of Total Income, , 145, , (d) None of the above, 18. John is a foreign citizen born in USA. His father was born in Delhi in 1960 and his grand-father, was born in Lahore in 1935 but his mother was born in UK in 1963. John came to India for the first, time on 1st June, 2021 and stayed in India for 183 days and then left for USA. His residential status, for the A.Y. 2022-23 shall be :, (a) Resident and ordinarily resident, (b) Resident but not ordinarily resident, (c) Non-resident, (d) Foreign National, 19. The following income of Ms. Nargis who is a non-resident shall be included in her total income:, (i) Salary for 2 months received in Delhi `40,000., (ii) Interest on Savings Bank Account in Mumbai `2,100., (iii) Agricultural income in Bangladesh and Invested in shares in Bangladesh., (iv) Amount brought into India out of past non-taxed profits earned in USA., (a) (i), (iii) and (iv), (b) (i) and (ii), (c) (i), (ii) and (iv), (d) All the four above, 20. The income earned during the previous year is subject to tax under the Act on the basis of, residential status of an assessee. However, the residential status of an assessee ........................ every, year., (a) will not change, (b) will certainly change, (c) may change, (d) None of the above, 21. Mr. Rajiv, born and brought up in India left for employment in Belgium on 15-10-2021. He has, never gone out of India, previously. What is his residential status for the assessment year 2022-23?, (a) Non-resident, (b) Not ordinarily resident, (c) Resident and ordinarily resident in India, (d) Indian citizen, 22. Mr. Ramji (age 55) is Karta of HUF doing textile business at Nagur. Mr. Ramji is residing in, Dubai for the past 10 years and visited India for 20 days every year for filing the income tax return of, HUF. His two major sons take care of the day to day affairs of the business in India. The residential, status of HUF for the assessment year 2022-23 is :, (a) Non-resident, (b) Resident and ordinarily resident in India, (c) Not ordinarily resident, (d) None of the above, 23. Past untaxed profit of the previous year 2016-17 brought to India in 2021-22 is chargeable to tax in, the assessment year 2022-23 in the hands of, (a) All the assessees, (b) Resident and ordinarily resident in India, (c) Non-resident in India, (d) None of the above., 24. Total income of a person is determined on the basis of his, (a) Residential status in India, (b) Citizenship in India, (c) Both (a) and (b) above, (d) None of the above.
Page 146 :
Residential Status & Scope of Total Income, , 146, , 25. Alpha Ltd. is an Indian company. It carries its business in Delhi and London. Total control and, management of the company is situated in London. More than 85% of its business income is from the, business in England. If so, its residential status will be —, (a) Resident, (b) Non-resident, (c) Not ordinarily resident, (d) Foreign company., , Check Solution given on our website www.mkgeducation.com
Page 147 :
Residential Status & Scope of Total Income, , 147, , PRACTICE PROBLEMS, TOTAL PROBLEMS 24, Problem 1 TO 10, Determine residential status of Mr. X for the assessment year 2022-23, who stays in India during various, financial years asunder:, Previous, 1, 2, 3, 4, 5, 6, 7, 8, 9, 10, Years, 2021-22, 65, 183, 181, 69, 300, 70, 72, 95, 180, 93, 2020-21, 91, 90, 87, 110, 97, 99, 94, 92, 91, 90, 2019-20, 190, 78, 98, 91, 103, 104, 101, 100, 99, 80, 2018-19, 89, 120, 189, 196, 110, 98, 97, 96, 95, 90, 2017-18, 87, 91, 92, 93, 94, 95, 94, 93, 92, 100, 2016-17, 86, 99, 92, 95, 99, 100, 101, 100, 99, 90, 2015-16, 84, 66, 93, 94, 365, 210, 209, 208, 207, 80, 2014-15, 105, 210, 91, 93, —, 0, 91, 92, 91, 90, 2013-14, 110, 110, 92, 92, 362, 300, 200, 100, —, 100, 2012-13, 112, 94, 93, 91, 10, 99, 88, 77, 66, 110, 2011-12, 100, 96, 91, 90, 310, 100, 99, 92, 94, 120, 2010-11, 91, 199, 90, 89, 210, 92, 94, 96, 98, 130, 2009-10, 94, 81, 89, 8, 92, 80, 70, 60, 50, 100, 2008-09, 97, 82, 88, 87, 88, 55, 65, 75, 85, 80, 2007-08, 99, 83, 87, 86, 84, 40, 50, 60, 70, 60, Answer = (1) ROR; (2) ROR; (3) ROR; (4) ROR; (5) ROR; (6) NOR; (7) ROR; (8) ROR; (9) ROR; (10), NR, Problem 11., Mr. X, a citizen of USA, has come to India for the first time on 01.07.2017. The particulars of his arrival and, departure are as given below:, Date of arrival, Date of departure, 01.07.2017, 11.12.2017, 27.03.2018, 21.07.2018, 10.09.2019, 01.03.2019, 01.01.2020, 23.09.2020, 01.02.2021, 01.07.2021, 11.02.2022, ——, Determine his residential status for various years., Answer = 2017-18 – Non-Resident (NR), 2018-19 – Resident but not ordinarily resident (NOR), 2019-20 – Resident but not ordinarily resident (NOR), 2020-21 – Resident but not ordinarily resident (NOR), 2021-22– Resident and ordinarily resident (ROR), Problem 12., Mr. X, a citizen of U.K., has come to India for the first time on 01.07.2017. The particulars of his arrival and, departure are as given below:, Date of arrival, Date of departure, 01.07.2017, 07.09.2017, 01.01.2018, 08.03.2018, 11.07.2018, 20.09.2018
Page 148 :
Residential Status & Scope of Total Income, 10.02.2019, 01.01.2020, 11.03.2021, 27.03.2022, Determine his residential status for various years., Answer = 2017-18 – Non-Resident (NR), 2018-19 – Non-Resident (NR), 2019-20 – Non-Resident (NR), 2020-21 – Resident but not ordinarily resident (NOR), 2021-22 – Resident but not ordinarily resident (NOR), , 148, 09.05.2019, 20.05.2020, 21.06.2021, ——, , Problem 13., Mr. X goes out of India every year for 274 days., Determine his residential status for the previous year 2021-22., Answer = Resident but not ordinarily resident (NOR), Problem 14., Mr. X, a citizen of Japan, has come to India for the first time on 01.10.2021 for 200 days., Determine his residential status for the assessment year 2022-23., Answer = Resident but not ordinarily resident (NOR), Problem 15., Mr. X, a citizen of U.K. came to India for the first time on 01.07.2011 in connection with his employment., He left India on 01.11.2020 for taking up a job in USA. He again came to India on 01.01.2022 on a visit and, left India on 01.03.2022., Determine his residential status for the assessment year 2022-23., Answer = Resident and ordinarily resident (ROR), Problem 16., Mr. X, a German citizen, came to India on 23.05.2020 and left India on 30.05.2021., Determine his residential status for the assessment year 2021-22, 2022-23., Answer =, Assessment Year 2021-22: Resident but not ordinarily resident (NOR), Assessment Year 2022-23: Non- Resident (NR), Problem 17., Mr. X, a citizen of India, is employed in Soliton Technologies, an Indian company. His employer has, transferred him to his branch in Japan. Mr. X left India on 29.09.2021 for his new posting in Japan., Determine his residential status for the assessment year 2022-23., Prior to this, Mr. X was posted outside India for 11 months in the previous year 2016-17 and for 10.5, months in the year 2012-13., Answer = Resident and ordinarily resident (ROR), Problem 18., Dr. Reddy’s Labs is an Indian company and has borrowed funds from Bank of America, New York for, investing it in one of its projects in USA. In this case, interest paid by Dr. Reddy’s Labs to Bank of America, shall be accruing/arising __________., Answer = Outside India, Problem 19., Calculate taxable income of an individual on the basis of the following informations, for the assessment year, 2022-23, if he is:, (a) Ordinarily Resident, (b) Not Ordinarily Resident; and
Page 149 :
Residential Status & Scope of Total Income, , 149, , (c) Non-Resident, `, (i) Profit from business in Japan received in India., 10,000, (ii) Income from agriculture in Pakistan – it is all spent on the education of children there, 5,000, (iii) Income accrued in India but received in England, 10,000, (iv) Income from house property in Pakistan deposited in a bank there, 2,000, (v) Profits of business in America deposited in a bank there. This business is controlled from India 50,000, (vi) Profits earned from business in Meerut, 12,000, (vii) Past untaxed foreign income brought into India during the previous year, 10,000, (Presume that all the incomes are computed incomes), Answer:, Taxable Income: Resident and ordinarily resident (ROR): `89,000;, Resident but not ordinarily resident (NOR) : `82,000;, Non-Resident (NR) : `32,000, Problem 20., Mr. X earns the following income during the previous year 2021-22., Compute his gross total income for assessment year 2022-23 if he is, (i) resident and ordinarily resident., (ii) resident but not ordinarily resident., (iii) non-resident., (1) Income from agricultural land in Bhutan received there and remitted to India later on, (2) Pension for service rendered in India, but received in Paris, (3) Past untaxed profits of 2020-21 brought into India in 2021-22, (4) Profits from business in Paris, deposited in bank there, (5) Profits from business in Canada, controlled from India, profits received there, (6) Interest on saving bank deposit in Punjab National Bank, in India, (7) Capital gain on sale of a house in Delhi, amount received in Paris, Answer:, Resident and ordinarily resident (ROR): `5,50,000, Resident but not ordinarily resident (NOR): `4,10,000, Non-Resident (NR): `2,35,000, Problem 21., Mr. X earns the following income during the previous year 2021-22., Compute his Gross total income for assessment year 2022-23 if he is, (i) resident and ordinarily resident., (ii) resident but not ordinarily resident., (iii) non-resident., (1) Profit on sale of machinery in India, but received in Japan, (2) Profits from business in Bombay, managed from Japan, (3) Profits from business in Japan, managed from there, received there, (4) Income from house property in India, (5) Income from property in Japan and received there, (6) Income from agriculture in Japan being invested there, (7) Fees for technical services rendered in India but received in Japan, (8) Interest on Government securities accrued in India but received in Japan, (9) Interest on Japan Government securities, received in India, (Presume that all the incomes are computed incomes), Answer:, Resident and ordinarily resident (ROR): `10,50,000, Resident but not ordinarily resident (NOR): `6,80,000, Non-Resident (NR): `6,80,000, , `, 40,000, 15,000, 50,000, 1,00,000, 1,75,000, 20,000, 2,00,000, , `, 1,20,000, 2,25,000, 1,45,000, 1,50,000, 1,50,000, 75,000, 65,000, 80,000, 40,000
Page 150 :
Residential Status & Scope of Total Income, , 150, , Problem 22., Mr. X earns the following incomes during the financial year 2021-22., `, (1) Profits from a business in Japan, controlled from India, half of the profits received in India, 60,000, (2) Income from agriculture in Nepal, brought to India, 10,000, (3) Income u/h house property in Bombay, received in UK, 1,70,000, (4) Income u/h house property in USA, received there but subsequently remitted to India, 2,20,000, (5) Income u/h house property in USA, received there (`50,000 remitted in India), 1,00,000, (6) Salary received in India for services rendered in USA, 60,000, (7) Income from profession in Paris, which was set up in India, received in Paris, 90,000, (8) Interest from deposit with an Indian company, received in Japan, 19,000, (9) Income from profession in Bombay received in Paris, 39,000, (10) Profits of business in Iran, deposited in a bank there, business controlled from India, (out of `4,80,000, ` 1,00,000 is remitted in India), 4,80,000, (11) Interest on German development bonds, half of which is received in India, 12,000, (12) Income under the head house property in Canada, one-fifth is received in India, 50,000, (Presume all the above incomes are computed income i.e. all the exemptions and deductions have already, been allowed), Determine the gross total income of Mr. X if he is, (i) resident and ordinarily resident,, (ii) resident but not ordinarily resident,, (iii) non-resident in India during the financial year 2021-22., Answer: Resident and ordinarily resident (ROR): `13,10,000, Resident but not ordinarily resident (NOR): `9,34,000, Non-Resident (NR): `3,34,000, Problem 23., Mr. X is a citizen of India and is employed in ABC Ltd and is getting a salary of `60,000 p.m. He purchased, one building in India on 1st May, 2021 for `10,00,000 and its market value is `22,00,000 and value for the, purpose of charging stamp duty is `13,00,000. He purchased gold for `8,00,000 and its market value is, `11,00,000. He was transferred out of India w.e.f. 1st Sept, 2021 and he left India on 1st Sept, 2021 and one, of his friend gifted him one colour TV on this occasion, market value `1,00,000., He has gone out of India in earlier years also., P.Y. 2020-21, 100 days, P.Y. 2019-20, 200 days, He visited India from 01.02.2022 to 14.02.2022 and salary for January, 2022 was received in India., He has purchased one house property in USA in December 2021 and sold in March 2022 and there were, short term capital gain of `6,00,000 and the amount was received in USA., Compute his tax liability for the A.Y.2022-23., Answer: Tax Liability: `98,280, Problem 24., Mrs. X is employed in ABC Ltd in India and she is an American citizen and is getting a salary of `2,00,000, p.m., She received gift of one painting in India from her friend on 01.07.2021 and its market value is `49,000 and, she also received gift in cash of `49,000 from the same friend and gift of immovable property with value for, the purpose of charging stamp duty is `51,000 from the same friend., She purchased UK Development bond and interest equivalent of `2,00,000 was received in USA., She visited USA for 182 days during P.Y.2021-22., In the earlier year her stay in India was, P.Y. 2020-21, 110 days, P.Y. 2019-20, 120 days
Page 151 :
Residential Status & Scope of Total Income, , 151, , P.Y. 2018-19, 300 days, P.Y. 2017-18, 182 days, P.Y. 2016-17, 185 days, P.Y. 2015-16, 200 days, P.Y. 2014-15, 300 days, Compute her tax liability in India for the A.Y. 2022-23., Answer: Tax Liability: `6,16,510, , Check Detailed Solution given on our website www.mkgeducation.com
Page 152 :
Residential Status & Scope of Total Income, , 152, , EXAMINATION QUESTIONS, Solutions of all Examinations Questions given on our website, www.mkgeducation.com, MAY – 2019 (NEW COURSE), Question 2 (a), , Marks 7, , The following are the income of Shri Subhash Chandra, a citizen of India for the previous year 2021-22:, (i), (ii), (iii), (iv), (v), (vi), , Income from business in India ` 2,00,000. The business is controlled from London and, ` 60,000 were remitted to London., Profits from business earned in Japan ` 70,000 of which ` 20,000 were received in India., This business is controlled from India., Untaxed income of ` 1,30,000 for the year 2018-19 of a business in England which was, brought in India on 3rd March, 2022., Royalty of ` 4,00,000 received from Shri Ramesh, a resident for technical service provided to, run a business outside India., Agricultural income ` 90,000 in Bhutan., Income of ` 73,000 from house property in Dubai, which was deposited in bank at Dubai., , Compute Gross total income of Shri Subhash Chandra for the A.Y. 2022-23, if he is –, (1), A resident and Ordinarily Resident, and, (2), A resident and Not Ordinarily Resident, , MAY – 2019 (OLD COURSE), Question 2(a), Mr. Bachhan has provided the following details of his income for the year ended 31-3-2022., Particulars, (1) Short term capital gains on sale of shares in Indian company received in Japan., , Marks 4, `, 85,000, , (2) Rent from property in Bangladesh deposited in a bank at Dhaka, later on remitted to, India through approved banking channels., , 96,000, , Compute his total income for the Assessment Year 2022-23 in case of he is:, (i), Resident and ordinarily resident;, (ii), Resident but not ordinarily resident; or, (iii) Non-resident, , NOV – 2018 (NEW COURSE), Question 2(a), Following incomes are derived by Mr. Krishna Kumar during the year ended 31-3-2022:, Pension received from the US Government, Agricultural income from lands in Malaysia, Rent received from let out property in Colombo, Sri Lanka, Discuss the taxability of the above items where the assessee is (i) Resident, (ii) Non-Resident, , Marks 6, 3,20,000, 2,70,000, 4,20,000
Page 153 :
Residential Status & Scope of Total Income, , 153, , NOV – 2018 (OLD COURSE), Question 2(a), Marks 5, Mr. Surya, an Indian citizen, travelled frequently out of India for his business trip as well as for his outings., He left India from Mumbai airport on 15th May 2021 as stamped in the passport. He has been in India for, less than 365 days during the 4 years immediately preceding the previous year and has not been in India for, at least 60 days in the previous year., Determine :, (i) Residential status of Mr. Surya and, (ii) Total income for the assessment year 2022-23 from the following information:, (1) Short term capital gain on the sale of shares of Trena India Ltd. a listed Indian company amounting, to `35,000. The sale proceeds were credited to his swiss bank account., (2) Interest on fixed deposit with State Bank of India (Mumbai) amounting to `8,000 was credited to, his saving account., , MAY – 2018 (NEW COURSE), Question 3, Compute the Gross Total Income in the hands of an individual, if he is, (a) a resident and ordinary resident ; and, (b) a non – resident for the A.Y. 2022-23, S.No., Particulars, (i), Interest from German Derivatives Bonds (1/3 received in India), (ii) Income from agriculture land situated in Malaysia ,remitted to India, (iii) Income earned from business in Dubai, Controlled from India (`20,000 received in, India., (iv) Profit from business in Mumbai, controlled from Australia, (v), Interest received from Mr. Ashok (NRI) on loan provided to him for business in India, (vi) Profit from business in Canada controlled from Mumbai (60% of profits deposited in, a bank in Canada and 40% remitted to India), (vii) Amount received from an NRI for the use of know-how for his business in Singapore, (viii) Past years untaxed foreign income brought to India, , Marks 10, , Amount `, 21,000, 51,000, 75,000, 1,75,000, 35,000, 60,000, 8,00,000, 50,000, , MAY – 2018 (OLD COURSE), Question 3 (a), Marks 5, Mrs. Karuna kapoor, is a Hollywood actress. Her passport reveals the Following information about her stay, in India., 2021-22, From April 3rd, to, July 11th, nd, 2020-21, From June 22, to, July 11th, 2019-20, From Feb 10th, to, March 26th, th, 2018-19, From Sept 7, to, March 26th, th, 2017-18, From May 17, to, September 30th, rd, 2016-17, From April 3, to, July 11th, 2015-16, From April 3rd, to, July 11th, rd, 2014-15, From April 3, to, July 11th, rd, 2013-14, From April 3, to, July 11th, Find out her residential status for all the assessment years.
Page 154 :
Residential Status & Scope of Total Income, , 154, , NOV – 2017, Question 2(a), (5 Marks), DAISY Ltd., a foreign company, incorporated in USA and engaged in the Manufacturing and distribution of, diamonds, set up a branch office in India in June 2021.The branch office was required to purchase uncut and, unassorted diamonds from the dealers of Mumbai and export them to USA., Out of 20 shareholders of DAISY Ltd., 12 shareholders are non-resident in India. All the major decisions, were taken through Board Meetings held at USA., (i) Determine the residential status of DAISY Ltd. for the Assessment Year 2022-23., (ii) Discuss the tax treatment of profit from export business., , MAY – 2017, Question 2(a) (i), (4 Marks), During the last four years preceding the financial year 2021-22, Mr. Damodhar, a citizen of India, was, present in India for 430 days. During the last seven previous years preceding the previous year 2021-22, he, was present in India for 830 days., Mr. Damodhar is a member of crew of a Dubai bound Indian ship in the international waters, which left, Kochi port in Kerala, on 12th August, 2021., Following details are made available to you for the previous year 2021-22:, Particulars, , Date, , Date entered into the continuous discharge certificate in respect of joining the, ship by Mr. Damodhar., , 12th August,2021, , Date entered into the continuous discharge certificate in respect of signing off, the ship by Mr. Damodhar., , 21st January,2022, , In May, 2021 he had gone out of India to Singapore and Malaysia on a private tour for a continuous period, of 29 days., You are required to determine the residential status of Mr. Damodhar for the previous year 2021-22., Question 4(a), (4 Marks), A Korean Company Damjung Ltd. entered in to the following transactions during the financial year 202122:, (a) Received `20 lakhs from a non-resident for use of patent for a business in India., (b) Received `15 lakhs from a non-resident Indian for use of know-how for a business in Sri Lanka and this, amount was received in Japan. [Assume that the above amount is converted/stated in Indian Rupees]., (c) Received `7 lakhs from RR Co. Ltd., an Indian company for providing technical know-how in India., (d) Received `5 lakhs from R & Co. Mumbai for conducting the feasibility study for a new project in Nepal, and the payment was made in Nepal., Explain briefly, whether the above receipts are chargeable to tax in India., , NOV – 2016, Question 4(a), (2 Marks), State with reasons whether the following receipts are taxable or not under the provisions of Income-tax Act,, 1961?, Mr. Frdie, a non-resident residing in Sweden, has received rent from Mr. Nadal, also a non-resident residing, in France in respect of a property taken on lease at Mumbai. Since this income is received outside India from, a non-resident, Frdie claims that his income is not chargeable to Tax in India.
Page 155 :
Residential Status & Scope of Total Income, , 155, , MAY – 2016, Question 2(a)(i), (4 Marks), How is the residential status of a Company determined for the purpose of Income - Tax Act, 1961, for the, Assessment Year 2022-23?, Answer: Refer answer given in the book, , NOV – 2015, Question 2(a)., (4 Marks), Mr. X, an Indian Citizen left India on 20.04.2019 for the first time to setup a software firm in Singapore. On, 10.04.2021, he entered into an agreement with XYZ Limited, an Indian Company for the transfer of, technical documents and designs to setup an automobile factory in Faridabad. He reached India along with, his team to render the requisite services on 15.05.2021 and was able to complete his assignment on, 20.08.2021. He left for Singapore on 21.08.2021. He charged `50 Lakhs for his services from XYZ Limited., Determine the residential status of Mr. X for the Assessment Year 2022-23 and explain as to the taxability, of the fees charged from XYZ Limited as per Income Tax Act, 1961., , MAY – 2015, Question 2(a)(i)., (4 Marks), Explain with reasons whether the following transactions attract income-tax in India in the hands of, recipients?, (a) Salary paid to Mr. David, a citizen of India `15,00,000 by the Central Government for the services, rendered in Canada., (b) Legal charges of `7,50,000 paid to Mr. Johnson, a lawyer of London, who visited India to represent a, case at the Supreme Court., (c) Royalty paid to Rajeev, a non-resident by, a resident for a business carried on in Sri Lanka., (d) Interest received of `1,00,000, on money borrowed from France, by Ms. Dyana, a non-resident for the, business at Bangalore., Question 2(a)(ii)., (4 Marks), Ms. Bindu, a non-resident, residing in New York since 2000, came back to India on 19.02.2020 for, permanent settlement in India. Explain her residential status of Ms. Bindu for the Assessment Year 202223. In accordance with the various provision of Indian Income tax Act., , NOV – 2014, Question 2(a)., (5 Marks), Mrs. X and Mrs. Y are sisters and they earned the following income during the Financial Year 2021-22., Mrs. X is settled in Malaysia since 2016 and visits India for a month every year. Mrs. Y is settled in Indore, since her marriage in 2016. Compute the total income of Mrs. X and Mrs. Y for the assessment year 202223:, Sl., Particulars, Mrs. X Mrs. Y, No., `, `, (i), Income from Profession in Malaysia, (set up in India) received there, 15,000, (ii), Profit from business in Delhi, but managed directly from Malaysia, 40,000, (iii), Rent (computed) from property in Malaysia deposited in a Bank at Malaysia, 1,20,000, later on remitted to India through approved banking channels., (iv), Cash gift received from a friend on Mrs. Y’s 50th birthday, - 51,000, (v), Agricultural income from land in Maharashtra, 7,500, 4,000, (vi), Past foreign untaxed income brought to India, 5,000, (vii), Fees for technical services rendered in India received in Malaysia, 25,000, (viii) Income from a business in Pune (Mrs. X receives 50% of the income in India), 12,000 15,000, (ix), Interest on debentures in an Indian company (Mrs. X received the same in, 18,500 14,000
Page 156 :
Residential Status & Scope of Total Income, (x), (xi), , Malaysia), Short-term capital gain on sale of shares of an Indian company, Interest on Fixed Deposit with SBI in India, , 156, 15,000, 12,000, , 25,500, 8,000, , MAY – 2013, Question 2(a)., (4 Marks), Mr. X and Mr. Y are brothers and they earned the following incomes during the financial year 2021-22., Mr. X settled in America in the year 2016 and Mr. Y settled in Mumbai. Mr. X visits India for 20 days, every year. Mr. Y also visits America every year for a month. Compute their total income for the, Assessment year 2022-23 from the following information., Sl., No., 1., 2., 3., 4., 5., 6., 7., , Particulars, , Interest on American Development bonds, 50% of interest received in, India., Short term capital gains on sale of shares of an Indian company, received in India., Profit from a business in Mumbai, but managed directly from America., Income from a business in Mumbai., Fees for technical services rendered in America and received in America., The services were, however, utilized in India., Interest on fixed deposit with State Bank of India, Mumbai., Income from house property at Mumbai., , Mr. X, `, , Mr. Y, `, , 46,000, , 18,000, , 45,000, , 75,000, , 10,000, 32,000, 1,50,000, , 28,000, -, , 4,500, 67,200, , 12,000, 38,500, , MAY – 2012, Question 2, (5 Marks), Mr. X & Mr. Y are brothers and they earned the following incomes during the financial year 2021-22. Mr. X, settled in Canada in the year 2016 and Mr. Y settled in Delhi. Compute the total income for the assessment, year 2022-23., Sl., Particulars, Mr. X, Mr. Y, No., 1., Interest on Canada Development Bond, (only 50% of interest, 35,000, 40,000, received in India), 2., Profit from a business in Nagpur, but managed directly from, 1,00,000, 1,40,000, London, 3., Short term capital gain on sale of shares of an Indian, 60,000, 90,000, company received in India, 4., Income from a business in Chennai, 80,000, 70,000, 5., Fees for technical services rendered in India, but received in, 1,00,000, ----Canada, 6., Interest on fixed deposit in UCO Bank, Delhi, 7,000, 12,000, 7., Agricultural income from a land situated in Andhra Pradesh, 55,000, 45,000, 8., Income under the head house property at Bhopal, 1,00,000, 60,000, , NOV – 2011, Question 5, (3 Marks), Mr. X (Citizen of India) a Government employee serving in the Ministry of External Affairs left India for, the first time on 31.03.2021 due to his transfer to Indian High Commission in Canada. He did not visit any, time during previous year 2021-22. He has received the following income for the Previous Year 2021-22., `, (i) Income under the head Salary (Computed), 5,00,000
Page 157 :
Residential Status & Scope of Total Income, (ii) Interest on fixed deposit from bank in India, (iii) Income from agriculture in Pakistan, (iv) Income from house property in Pakistan, Compute his gross total income for Assessment Year 2022-23., , 157, 1,00,000, 2,00,000, 2,50,000, , MAY – 2010, Question 3, (4 Marks), From the following particulars of Income furnished by Mr. X pertaining to the year ended 31.03.2022,, compute the total income for the assessment year 2022-23, if he is:, (i) Resident and ordinary resident;, (ii) Resident but not ordinary resident;, (iii) Non-resident:, Particulars, Amount (`), (a) Profit on sale of shares in Indian Company received in Germany, 15,000, (b) Income from business in London deposited in a bank in London,, later on remitted to India through approved banking channels, 75,000, (c) Agricultural income from lands in Gujarat, 25,000, , MAY – 2010, Question 1, (2 Marks each), Answer the following with reasons having regard to the provisions of the Income-Tax Act, 1961 for the, Assessment Year 2022-23:, (i) State the Scope of total income in the case of an individual, whose residential status is ‘non-resident’ with, reference to section 5., (ii) Mr. X a citizen of India received salary from the Government of India for the services rendered outside, India. Is the salary income chargeable to tax?, , NOV – 2009, Question 1, (4 Marks), Mr. X and Mr. Y are brothers and they earned the following incomes during the financial year 2021-22. Mr., X settled in U.K. in the year 2017 and Mr. Y settled in Surat. Compute the gross total income for the, Assessment Year 2022-23., Sr. No. Particulars, Mr. X, Mr. Y, 1., Interest on U.K. development bonds, 50% of interest received in India, 25,000, 20,000, 2., Profit from a business in Mumbai, but managed directly from London, 10,000, 12,000, 3., Profit on sale of shares of an Indian company received in India, 50,000, 80,000, 4., Income from a business in Delhi, 20,000, 20,000, 5., Fees for technical services rendered in India, but received in London, 1,00,000, 6., Interest on fixed deposit in SBI, Bangalore, 5,000, 15,000, 7., Agricultural income from a land situated in Rajasthan, 25,000, 25,000, 8., Income under the head House Property at Bangalore, 50,400, 33,600, , MAY – 2008, Question 3, Choose the correct answer with reference to the provisions of the Income-tax Act, 1961:, Income accruing in Japan and received there is taxable in India in the case of –, (a) Resident and ordinarily resident only, (b) Both resident and ordinarily resident and resident but not ordinarily resident, (c) Both resident and non-resident, (d) Non-resident, , (1 Marks)
Page 158 :
Residential Status & Scope of Total Income, , 158, , Answer: (a) Resident and ordinarily resident only, , MAY – 2007, Question 1, (2 Marks), State with reasons, whether the following statement is true or false, with regard to the provisions of the, Income-tax Act, 1961:, Only individuals and HUFs can be resident, but not ordinarily resident in India; firms can be either a, resident or non-resident.
Page 159 :
Income Under The Head House Property, , 159, , INCOME UNDER THE HEAD, , HOUSE PROPERTY, SECTION 22 TO 27, PARTICULARS, Income from house property—Chargeability and meaning of house property, Annual value—, House let out throughout the year/partly let out/partly vacant, One house which is self-occupied, House partly self-occupied/partly let out, may or may not be vacant, More than one house which are self-occupied, Municipal tax, Treatment of unrealised rent, Statutory Deduction/Standard Deduction, Interest on capital borrowed, Interest in case of loan taken from outside India, Recovery of unrealised rent / Arrears of rent, Property owned by co-owners, Deemed ownership, Meaning of unrealised rent, , SECTIONS, 22, 23(1), 23(2), 23(3), 23(4), Proviso to sec 23(1), Explanation to 23(1), 24(a), 24(b), 25, 25A, 26, 27, Rule 4, , Chargeability of income under the head house property Section 22, Income from letting out of house property is chargeable to tax under the head House Property. If the income, is from sale or purchase of house property, it will be taxable under the head Capital Gains, however if the, sale or purchase is part of a business, income is taxable under the head Business/Profession., House property shall include all types of house properties i.e. residential houses, shops, godowns, cinema, building, workshop building, hotel buildings etc., The term house property shall include not only the buildings but also the lands appurtenant thereto i.e. the, term house property shall include even any open land which is part and parcel of the building. E.g. Mr. X, has one big house and it includes vast open area within its boundaries. The house has been let out at a rent of, `1,00,000 p.m., out of which rent of `25,000 p.m. is attributable to the open land. In this case, entire rental, income is taxable under the head house property., If any person has let out only land, which is not essential part of any building, income is taxable under the, head other sources. E.g. Mr. X has one big piece of land which is let out for arranging exhibitions or for the, purpose of marriage parties etc., rent received or receivable is taxable under the head other sources (It is also, called vacant site lease rent). If any person has business of letting out of open land, income shall be taxable, under the head business profession, Income from property held as stock-in-trade/ from business of letting out house property, If any person is holding house property as stock-in-trade i.e. for sale/purchase of house property, income, shall be taxable under the head Business/Profession. Similarly if any person has business of letting out of, house property, income shall be taxable under the head Business/Profession. E.g. ABC Ltd. is holding 500, flats for the purpose of letting out, income shall be taxable under the head business/profession.
Page 160 :
Income Under The Head House Property, , 160, , If any person is holding house property for the purpose of sale/purchase but it has been let out for some, time, income shall be taxable under the head House Property., Computation of Income under the head House Property, Gross Annual Value (GAV), Less: Municipal Taxes, Net Annual Value (NAV), Less:, Deduction allowed under section 24, - Statutory deduction / standard deduction @ 30% of NAV [Section 24(a)], - Interest on borrowed capital for construction etc. [Section 24(b)], Income under the head “House Property”, , `……………., `.……………, `.….……….., `……..………, `…………….., `…………….., , Question 1: Write a note on computation of income of a house property which is let out throughout, the year., Answer: As per section 23(1)(a)/(b), gross annual value i.e. reasonable rental value shall be computed in, the manner given below:, 1. Compare Fair Rent and Municipal Valuation and select the higher., 2. Compare the rent so selected with Standard Rent and the lower of the two shall be considered to be, Expected Rent. (It is also called Annual Letting Value), 3. Compare Expected Rent with Rent Received or Receivable and the higher shall be considered to be Gross, Annual Value., Fair rent i.e. the rent of similar types of buildings in the same locality., Municipal valuation i.e. rental value determined by the municipality for the purpose of charging municipal, tax. It is also called rateable value., Standard rent i.e. the highest possible rent as per Rent Control Act., Rent received or receivable, Illustration 1: Mr. X has one house property which is let out @ `80,000 p.m. Fair rent ` 90,000 p.m.,, Municipal Valuation `70,000 p.m., Standard Rent `81,000 p.m. Municipal tax paid `60,000 and interest, paid on loan for construction of house property is ` 50,000., Compute his Income Tax Liability for A.Y 2022-23., Solution:, `, Computation of income under the head House Property, Gross Annual Value, 9,72,000.00, Working Note:, `, (a) Fair Rent (90,000 x 12), 10,80,000, (b) Municipal Value (70,000 x 12), 8,40,000, (c) Higher of (a) or (b), 10,80,000, (d) Standard Rent (81,000 x 12), 9,72,000, (e) Expected Rent {Lower of c or d}, 9,72,000, (f) Rent received /receivable (80,000 x 12), 9,60,000, GAV shall be higher of (e) or (f), 9,72,000, Less: Municipal Tax, (60,000.00), Net Annual Value, 9,12,000.00, Less: 30% of NAV u/s 24(a), (2,73,600.00), Less: Interest on capital borrowed u/s 24(b), (50,000.00), Income under the head House Property, 5,88,400.00, Gross Total Income, 5,88,400.00, Less: Deduction u/s 80C to 80U, NIL, Total Income, 5,88,400.00, Computation of Tax Liability, Tax on `5,88,400 at slab rate, 30,180.00
Page 161 :
Income Under The Head House Property, Add: HEC @ 4%, Tax Liability, Rounded off u/s 288B, , 161, 1,207.20, 31,387.20, 31,390.00, , Illustration 2: Mrs. X has let out one House property @ `62,000 p.m., Municipal Valuation `72,000 p.m.,, Fair Rent `90,000 p.m., Standard Rent `1,00,000 p.m., Municipal Tax paid `40,000 and Interest on loan, taken for construction `60,000, She has completed the age of 60 years on 31.03.2023., Compute Income Tax Liability for the A.Y 2022-23., Solution:, `, Computation of income under the head House Property, Gross Annual Value, 10,80,000.00, Working Note:, `, (a) Fair Rent (90,000 x 12), 10,80,000, (b) Municipal Value (72,000 x 12), 8,64,000, (c) Higher of (a) or (b), 10,80,000, (d) Standard Rent (1,00,000 x 12), 12,00,000, (e) Expected Rent {Lower of c or d}, 10,80,000, (f) Rent received /receivable (62,000 x 12), 7,44,000, GAV shall be higher of (e) or (f), 10,80,000, Less: Municipal Tax, (40,000.00), Net Annual Value, 10,40,000.00, Less: 30% of NAV u/s 24(a), (3,12,000.00), Less: Interest on capital borrowed u/s 24(b), (60,000.00), Income from house property, 6,68,000.00, Gross Total Income, 6,68,000.00, Less: Deduction u/s 80C to 80U, NIL, Total Income, 6,68,000.00, Computation of Tax Liability, Tax on `6,68,000 at slab rate, 46,100.00, Add: HEC @ 4%, 1,844.00, Tax Liability, 47,944.00, Rounded off u/s 288B, 47,940.00, Illustration 3: Mr. X owns five houses in Chennai, all of which are let-out. Compute the GAV of each, house from the information given below –, Particulars, House I, House II, House III, House IV, House V, Municipal Value, 80,000, 55,000, 65,000, 24,000, 75,000, Fair Rent, 90,000, 60,000, 65,000, 25,000, 80,000, Standard Rent, N.A., 75,000, 58,000, N.A., 78,000, Actual rent received/ receivable, 72,000, 72,000, 60,000, 30,000, 72,000, Solution:, GAV, 90,000, 72,000, 60,000, 30,000, 78,000, MAY – 2012 (5 Marks), Mr. X owns five houses at Cochin. Compute the gross annual value of each house from the information, given below:, House-I, House-II, House-III, House-IV, House –V, Municipal value, 1,20,000, 2,40,000, 1,10,000, 90,000, 75,000, Fair rent, 1,50,000, 2,40,000, 1,14,000, 84,000, 80,000, Standard rent, 1,08,000, N.A., 1,44,000, N.A., 78,000, Actual rent received/, 1,80,000, 2,10,000, 1,20,000, 1,08,000, 72,000
Page 162 :
Income Under The Head House Property, receivable, Answer:, House I, Computation of Gross Annual Value, (a) Fair Rent, (b) Municipal Valuation, (c) Higher of (a) or (b), (d) Standard Rent, (e) Expected Rent {Lower of (c) or (d)}, (f) Rent Received/Receivable, (g) Higher of (e) or (f) shall be GAV, House II, Computation of Gross Annual Value, (a) Fair Rent, (b) Municipal Valuation, (c) Higher of (a) or (b), (d) Standard Rent, (e) Expected Rent {Lower of (c) or (d)}, (f) Rent Received/Receivable, (g) Higher of (e) or (f) shall be GAV, House III, Computation of Gross Annual Value, (a) Fair Rent, (b) Municipal Valuation, (c) Higher of (a) or (b), (d) Standard Rent, (e) Expected Rent {Lower of (c) or (d)}, (f) Rent Received/Receivable, (g) Higher of (e) or (f) shall be GAV, House IV, Computation of Gross Annual Value, (a) Fair Rent, (b) Municipal Valuation, (c) Higher of (a) or (b), (d) Standard Rent, (e) Expected Rent {Lower of (c) or (d)}, (f) Rent Received/Receivable, (g) Higher of (e) or (f) shall be GAV, House V, Computation of Gross Annual Value, (a) Fair Rent, (b) Municipal Valuation, (c) Higher of (a) or (b), (d) Standard Rent, (e) Expected Rent {Lower of (c) or (d)}, (f) Rent Received/Receivable, (g) Higher of (e) or (f) shall be GAV, , 162, , `, 1,50,000, 1,20,000, 1,50,000, 1,08,000, 1,08,000, 1,80,000, 1,80,000, `, 2,40,000, 2,40,000, 2,40,000, N.A, 2,40,000, 2,10,000, 2,40,000, `, 1,14,000, 1,10,000, 1,14,000, 1,44,000, 1,14,000, 1,20,000, 1,20,000, `, 84,000, 90,000, 90,000, N.A, 90,000, 1,08,000, 1,08,000, `, 80,000, 75,000, 80,000, 78,000, 78,000, 72,000, 78,000, , Question 2: Explain deductibility of property taxes (municipal taxes)., Answer: Property Taxes (municipal taxes) Proviso to Section 23(1), In order to maintain any particular town or city, there is always some authority and it is called local authority, e.g. MCD in Delhi and such authority is allowed to charge certain tax in connection with house property and
Page 163 :
Income Under The Head House Property, , 163, , such tax are called municipal tax or house tax or property tax. If an assessee has paid such tax, deduction, shall be allowed for the tax so paid from GAV but if tax is due but not paid, deduction is not allowed., If tax has been paid by the tenant, in that case tax shall not be allowed to be deducted., Example, During the previous year 2021-22 municipality has levied taxes `20,000, but the assessee has paid `15,000., In this case, amount allowed to be deducted is `15,000. In the next year, municipality has levied taxes of, `45,000 but the assessee has paid ` 55,000 which includes `5,000 for the earlier year and `5,000 for the, subsequent year. In this case, amount allowed to be deducted in previous year 2022-23 shall be `55,000., Question 3: Write a note on set off and carry forward of losses under the head house property., Answer: Set off and carry forward of losses under the head house property Section 70/71/71B, Inter Source adjustment Section 70, As per section 70, if any person has loss from any house property, such loss can be set off from income of, any other house property and it is called inter-source adjustment or intra-head adjustment. E.g. Mr. X has, two houses: there is loss of `5,00,000 from one house and income of `8,00,000 from the other house, in this, case, loss of one source (house) can be set off from income of the other source (house)., Inter Head adjustment Section 71, As per section 71, unadjusted loss can be set off from incomes of other heads but as per section 58(4), such, loss can not be set off from casual income and it is called inter-head adjustment. E.g. Mr. X has loss from, house property `1,50,000 and income from business/profession `5,00,000, in this case, loss is allowed to be, set off but if he has any casual income, loss can not be set off from casual income., Where in respect of any assessment year, the net result of the computation under the head “Income from, house property” is a loss and the assessee has income assessable under any other head of income, the, assessee shall not be entitled to set off such loss, to the extent the amount of the loss exceeds `2,00,000,, against income under the other head.’., Carry Forward and Set Off Section 71B, As per section 71B, unadjusted loss is allowed to be carried forward to the subsequent years but for a, maximum period of 8 years starting from the year subsequent to the year in which the loss was incurred and, in the subsequent years, loss can be set off only from income under the head house property. E.g. Mr. X has, incurred loss under the head house property in the previous year 2021-22/assessment year 2022-23 and it, could not be set off in the same year, it can be carried forward upto Previous Year 2029-30/Assessment Year, 2030-31 (as shown below), Year 1, Previous year 2022-23, Assessment Year 2023-24, Year 2, Previous year 2023-24, Assessment Year 2024-25, Year 3, Previous year 2024-25, Assessment Year 2025-26, Year 4, Previous year 2025-26, Assessment Year 2026-27, Year 5, Previous year 2026-27, Assessment Year 2027-28, Year 6, Previous year 2027-28, Assessment Year 2028-29, Year 7, Previous year 2028-29, Assessment Year 2029-30, Year 8, Previous year 2029-30, Assessment Year 2030-31, E.g. Mr. X has loss under the head house property of the previous year 2013-14/assessment year 2014-15, `5,00,000 and income under the head house property `5,00,000 in previous year 2021-22/assessment year, 2022-23, in this case, loss shall be allowed to be set off because it will be allowed to be carried forward upto, a period of 8 years starting from Previous Year 2014-15/Assessment Year 2015-16 and is as shown below:, Year 1, Previous year 2014-15, Assessment Year 2015-16, Year 2, Previous year 2015-16, Assessment Year 2016-17, Year 3, Previous year 2016-17, Assessment Year 2017-18, Year 4, Previous year 2017-18, Assessment Year 2018-19, Year 5, Previous year 2018-19, Assessment Year 2019-20, Year 6, Previous year 2019-20, Assessment Year 2020-21, Year 7, Previous year 2020-21, Assessment Year 2021-22
Page 164 :
Income Under The Head House Property, Year 8, , Previous year 2021-22, , 164, , Assessment Year 2022-23, , Additional Points, 1. If the loss can be set off, it has to be set off compulsorily i.e. it is not voluntary. E.g. Mr. X has loss under, the head house property `1,50,000 in previous year 2021-22/assessment year 2022-23 and income under the, head business/profession `1,50,000 in the same year, in this case loss has to be set off., 2. Any loss has to be set off first within the same head and after that under some other heads and after that, carry forward is allowed., 3. Loss of current year shall be set off first and only after that brought forward losses can be adjusted, eg., Mr. X has income from one house ` 10,00,000 and loss from other house ` 10,00,000 in P.Y. 2021-22 and, also unadjusted loss of ` 10,00,000 under the head house property of P.Y. 2013-14, in this case loss of, current year is to be adjusted first., Illustration 4: Mr. X has Loss under the head House Property `13,00,000 and income under the head Salary, `8,00,000 and income under the head Business/Profession `6,00,000 and LTCG `20,00,000 and Casual, income `5,00,000. Compute his tax liability for A.Y. 2022-23., Solution:, In this case, Mr. X has the option to set off the loss under the head House Property either from normal, income or from LTCG and tax liability in two options shall be:, Option-1: Set off from normal income:, Computation of Total Income, `, Income under the head Salary, 8,00,000, Less: Loss under the head House Property, (2,00,000), Income under the head Salary, 6,00,000, Income under the head Business/Profession, 6,00,000, Long term capital gain, 20,00,000, Casual income, 5,00,000, Gross Total Income, 37,00,000, Less: Deduction u/s 80C to 80U, Nil, Total Income, 37,00,000, Computation of Tax Liability, Tax on Casual income `5,00,000 @ 30% u/s 115BB, 1,50,000, Tax on LTCG `20,00,000 @ 20% u/s 112, 4,00,000, Tax on normal income `12,00,000 at slab rate, 1,72,500, Tax before HEC, 7,22,500, Add: HEC @ 4%, 28,900, Tax Liability, 7,51,400, Loss under the head house property of `11,00,000 shall be carried forward., Note: As per section 71, Maximum loss of `2,00,000 is allowed to be set off from other heads., Option-2: Set off from LTCG:, Computation of Total Income, `, Income under the head Salary, 8,00,000, Income under the head Business/Profession, 6,00,000, Long term capital gain, 20,00,000, Less: Loss under the head House Property, (2,00,000), Long term capital gain, 18,00,000, Casual income, 5,00,000, Gross Total Income, 37,00,000, Less: Deduction u/s 80C to 80U, Nil, Total Income, 37,00,000, Computation of Tax Liability, Tax on Casual income `5,00,000 @ 30% u/s 115BB, 1,50,000, Tax on LTCG `18,00,000 @ 20% u/s 112, 3,60,000
Page 165 :
Income Under The Head House Property, Tax on normal income `14,00,000 at slab rate, Tax before HEC, Add: HEC @ 4%, Tax Liability, Loss under the head house property of `11,00,000 shall be carried forward., Note: As per section 71, Maximum loss of `2,00,000 is allowed to be set off from other heads., Option-1 is better since Total Tax Liability is lower in this option., , 165, 2,32,500, 7,42,500, 29,700, 7,72,200, , Illustration 5: Mr. X has Loss under the head House Property ` 20,00,000 and income under the head, Salary ` 10,00,000 and income under the head Business/Profession ` 11,00,000 and LTCG ` 10,00,000 and, deduction u/s 80C to 80U is ` 2,00,000. Compute his tax liability for A.Y. 2022-23., Solution:, In this case, Mr. X has the option to set off the loss under the head House Property either from normal, income or from LTCG and tax liability in two options shall be:, Option-1: Set off from normal income:, Computation of Total Income, `, Income under the head Salary, 10,00,000, Less: Loss under the head House Property, (2,00,000), Income under the head Salary, 8,00,000, Income under the head Business/Profession, 11,00,000, Long term capital gain, 10,00,000, Gross Total Income, 29,00,000, Less: Deduction u/s 80C to 80U, (2,00,000), Total Income, 27,00,000, Computation of Tax Liability, Tax on LTCG `10,00,000 @ 20% u/s 112, 2,00,000, Tax on normal income 17,00,000 at slab rate, 3,22,500, Tax before HEC, 5,22,500, Add: HEC @ 4%, 20,900, Tax Liability, 5,43,400, Loss under the head house property of `18,00,000 shall be carried forward., Note: As per section 71, Maximum loss of `2,00,000 is allowed to be set off from other heads., Option-2: Set off from LTCG:, Computation of Total Income, Income under the head Salary, Income under the head Business/Profession, Long term capital gain, Less: Loss under the head House Property, Long term capital gain, Gross Total Income, Less: Deduction u/s 80C to 80U, Total Income, Computation of Tax Liability, Tax on LTCG `8,00,000 @ 20% u/s 112, Tax on normal income `19,00,000 at slab rate, Tax before HEC, Add: HEC @ 4%, Tax Liability, Loss under the head house property of `18,00,000 shall be carried forward., Note: As per section 71, Maximum loss of `2,00,000 is allowed to be set off from other heads., Option-1 is better since Total Tax Liability is lower in this option., , `, 10,00,000, 11,00,000, 10,00,000, (2,00,000), 8,00,000, 29,00,000, (2,00,000), 27,00,000, 1,60,000, 3,82,500, 5,42,500, 21,700, 5,64,200
Page 166 :
Income Under The Head House Property, , 166, , Question 4: Write a note on computation of income of house lying vacant for some period., Answer: House lying vacant for some period Section 23(1)(c), If the house is partly let out and partly vacant, in such cases expected rent shall be computed for 12 months, but while computing rent received /receivable, rent for the period for which the house was vacant shall be, excluded and GAV shall be higher of expected rent and rent received/receivable but if the rent, received/receivable is less than the expected rent owing to vacancy, in that case rent received/receivable, shall be gross annual value. e.g. If expected rent is `20,000 p.m. and rent received/receivable is `15,000, p.m. and there is vacancy for 5 months, in this case GAV shall be the expected rent because even if there, was no vacancy, still rent received/receivable was less than expected rent., If in this case rent received/receivable is `25,000 p.m. and it is vacant for 5 months, gross annual value shall, be the rent received/receivable because if there was no vacancy, rent R/R would have been higher than, expected rent accordingly in the given case, R/R is lower than expected rent owing to vacancy., Illustration 6: Compute gross annual value in the following cases for the assessment year 2022-23:, Particulars, Situation 1, Situation 2, Situation 3, Situation 4, Fair Rent (p.m.), 9,000, 13,000, 16,000, 12,000, Municipal Valuation (p.m.), 10,000, 9,000, 18,000, 9,000, Standard Rent (p.m.), 12,000, 11,000, 16,000, 7,000, Rent received/ receivable, 7,000, 11,500, 16,000, 20,000, (p.m.), Vacancy, 1 month, 1 month, 2 months, 2 month, Solution:, Situation 1, `, Computation of Gross Annual Value, (a) Fair Rent, 1,08,000, (9,000 x 12), (b) Municipal Valuation, 1,20,000, (10,000 x 12), (c) Higher of (a) or (b), 1,20,000, (d) Standard Rent, 1,44,000, (12,000 x 12), (e) Expected Rent {Lower of (c) or (d)}, 1,20,000, (f) Rent Received/Receivable, 77,000, (7,000 x 11), If there was no vacancy, in that case rent received/receivable would have been `7000 x 12 = `84,000, and it is still less than expected rent, therefore GAV shall be expected rent., Gross Annual Value, 1,20,000, Situation 2, `, Computation of Gross Annual Value, (a) Fair Rent, 1,56,000, (13,000 x 12), (b) Municipal Valuation, 1,08,000, (9,000 x 12), (c) Higher of (a) or (b), 1,56,000, (d) Standard Rent, 1,32,000, (11,000 x 12), (e) Expected Rent {Lower of (c) or (d)}, 1,32,000, (f) Rent Received/Receivable, 1,26,500, (11,500 x 11), In this case, if there was no vacancy, rent received/receivable would have been `11500 x 12 = `1,38,000, hence rent received/receivable is lower in this case due to vacancy, therefore GAV, shall be the rent received/receivable.
Page 167 :
Income Under The Head House Property, , 167, , Gross Annual Value, 1,26,500, Situation 3, `, Computation of Gross Annual Value, (a) Fair Rent, 1,92,000, (16,000 x 12), (b) Municipal Valuation, 2,16,000, (18,000 x 12), (c) Higher of (a) or (b), 2,16,000, (d) Standard Rent, 1,92,000, (16,000 x 12), (e) Expected Rent {Lower of (c) or (d)}, 1,92,000, (f) Rent Received/Receivable, 1,60,000, (16,000 x 10), In this case, if there was no vacancy, rent received/receivable would have been `16,000 x 12 = `1,92,000, hence rent received/receivable is lower in this case owing to vacancy, therefore GAV, shall be the rent received/receivable., Gross Annual Value, 1,60,000, Situation 4, `, Computation of Gross Annual Value, (a) Fair Rent, 1,44,000, (12,000 x 12), (b) Municipal Valuation, 1,08,000, (9,000 x 12), (c) Higher of (a) or (b), 1,44,000, (d) Standard Rent, 84,000, (7,000 x 12), (e) Expected Rent {Lower of (c) or (d)}, 84,000, (f) Rent Received/Receivable, 2,00,000, (20,000 x 10), In this case, rent R/R is higher than the expected rent, GAV shall be Rent R/R, Gross Annual Value, 2,00,000, Question 5: Write a note on House lying vacant for full year., Answer: As per section 23(1)(c), if any House Property is lying vacant throughout the year, it will be, considered to be deemed to be let out and income shall be computed in the similar manner as in case of a let, out house. Expected Rent shall be considered to be Gross annual value., As per section 23 (5), Where the property consisting of any building or land appurtenant thereto is held as, stock-in trade and the property or any part of the property is not let during the whole of the previous year,, the annual value of such property or part of the property, for the period up to two year from the end of the, financial year in which the certificate of completion of construction of the property is obtained from the, competent authority, shall be taken to be nil., Question 6: Write a note on unrealized rent., Answer: Treatment of unrealised rent Explanation to Section 23(1)/Rule 4, Unrealised rent means such rent which is irrecoverable and is considered to be lost i.e. bad debt and in such, cases, expected rent shall be computed for full year and while computing rent received or receivable, such, unrealised rent shall be excluded and GAV shall be higher of expected rent and rent received/receivable (no, special treatment like vacancy)., e.g. Mr. X has let out one house `50,000 p.m. , fair rent `45,000 p.m., municipal valuation `40,000 p.m., standard rent `70,000 p.m. and there was unrealized rent for 3 months, in this case GAV of the house shall, be, Expected rent (45,000 x 12), 5,40,000
Page 168 :
Income Under The Head House Property, , 168, , Rent received /receivable (50,000 x 9), 4,50,000, GAV, 5,40,000, Rent shall be considered to be unrealised rent only if all the conditions of Rule 4 have been complied with, and such conditions are:, (a) the defaulting tenant has vacated, or steps have been taken to compel him to vacate the property;, (b) the defaulting tenant is not in occupation of any other property of the assessee;, (c) the assessee has taken all reasonable steps to institute legal proceedings for the recovery of the unpaid, rent or satisfies the Assessing Officer that legal proceedings would be useless., (d) the tenancy is bona fide (genuine), Recovery of unrealised rent Section 25A, If any assessee has recovered unrealized rent in subsequent years, rent so recovered shall be considered to be, income of the assessee under the head house property and it do not matter whether the assessee has any, house property in his name in that year or not. If assessee has received any interest, it will be considered to, be income of the assessee under the head other sources. If assessee has incurred any expenses on legal, proceedings, it will not be allowed to be deducted., A sum equal to thirty per cent of the unrealised rent shall be allowed as deduction., Illustration 7: Mr. X has let out one house at `70,000 per month fair rent `80,000 per month municipal, valuation `60,000 per month, Standard Rent ` 65,000 per month. Municipal tax paid `40,000, Interest u/s 24, (b) `50,000. Assessee has recovered unrealized rent of `60,000 plus interest `7,000. He has incurred legal, expenses `12,000 compute his Income and Tax Liability A.Y.2022-23., Solution:, Computation of income under the head house property, `, Gross Annual Value, 8,40,000.00, Working Note:, `, (a) Fair Rent (80,000 x 12), 9,60,000, (b) Municipal Valuation (60,000 x 12), 7,20,000, (c) Higher of (a) or (b), 9,60,000, (d) Standard Rent (65,000 x 12), 7,80,000, (e) Expected Rent {Lower of (c) or (d)}, 7,80,000, (f) Rent received /receivable (70,000 x 12), 8,40,000, (g) Higher of (e) or (f) shall be GAV, 8,40,000, Less: Municipal Tax, (40,000.00), Net Annual Value, 8,00,000.00, Less: 30% of NAV u/s 24(a), (2,40,000.00), Less: Interest on capital borrowed u/s 24(b), (50,000.00), Income under the head House Property, 5,10,000.00, Add: Recovery of Unrealised rent u/s 25A, 60,000.00, Less: Deduction @ 30%, (18,000.00), 42,000.00, Income under the head House Property, 5,52,000.00, Income under the head other sources, Interest from unrealized rent, Gross Total Income, Less: Deduction u/s 80C to 80U, Total Income, Computation of Tax Liability, Tax on ` 5,59,000 at slab rate, Add: HEC @ 4%, Tax Liability, Rounded off u/s 288B, , 7,000.00, 5,59,000.00, NIL, 5,59,000.00, 24,300.00, 972.00, 25,272.00, 25,270.00
Page 169 :
Income Under The Head House Property, , 169, , Illustration 8: Compute gross annual value in the following cases for the assessment year 2022-23:, Particulars, Situation 1, Situation 2, Situation 3, Situation 4, Fair Rent (p.m.), 11,000, 13,000, 16,000, 14,000, Municipal Valuation (p.m.), 12,000, 11,000, 18,000, 9,000, Standard Rent (p.m.), 13,000, 12,000, 17,000, 8,000, Rent received/ receivable, 8,000, 12,500, 17,000, 21,000, (p.m.), Vacancy, 2 months, 3 month, 1 month, Unrealised rent, 1 month, 1 month, 3 month, Solution:, Situation 1, `, Computation of Gross Annual Value, (a) Fair Rent, 1,32,000, (11,000 x 12), (b) Municipal Valuation, 1,44,000, (12,000 x 12), (c) Higher of (a) or (b), 1,44,000, (d) Standard Rent, 1,56,000, (13,000 x 12), (e) Expected Rent {Lower of (c) or (d)}, 1,44,000, (f) Rent Received/Receivable, 88,000, (8,000 x 11), GAV = Higher of (e) or (f), 1,44,000, Gross Annual Value, 1,44,000, Situation 2, Computation of Gross Annual Value, (a) Fair Rent, 1,56,000, (13,000 x 12), (b) Municipal Valuation, 1,32,000, (11,000 x 12), (c) Higher of (a) or (b), 1,56,000, (d) Standard Rent, 1,44,000, (12,000 x 12), (e) Expected Rent {Lower of (c) or (d)}, 1,44,000, (f) Rent Received/Receivable, 1,25,000, (12,500 x 10), In this case, if there was no vacancy, rent received/receivable would have been `12,500 x 12 = `1,50,000, hence rent received/receivable is lower in this case due to vacancy, therefore GAV, shall be the rent received/receivable., Gross Annual Value, 1,25,000, Situation 3, Computation of Gross Annual Value, (a) Fair Rent, 1,92,000, (16,000 x 12), (b) Municipal Valuation, 2,16,000, (18,000 x 12), (c) Higher of (a) or (b), 2,16,000, (d) Standard Rent, 2,04,000, (17,000 x 12), (e) Expected Rent {Lower of (c) or (d)}, 2,04,000, (f) Rent Received/Receivable, 1,36,000, (17,000 x 8)
Page 170 :
Income Under The Head House Property, , 170, , If there was no vacancy, in that case rent received/receivable would have been `17,000 x 11= `1,87,000, and It was still less than expected rent, therefore GAV shall be expected rent., Gross Annual Value, 2,04,000, Situation 4, Computation of Gross Annual Value, (a) Fair Rent, 1,68,000, (14,000 x 12), (b) Municipal Valuation, 1,08,000, (9,000 x 12), (c) Higher of (a) or (b), 1,68,000, (d) Standard Rent, 96,000, (8,000 x 12), (e) Expected Rent {Lower of (c) or (d)}, 96,000, (f) Rent Received/Receivable, 1,68,000, (21,000 x 8), In this case, rent R/R is higher than the expected rent, GAV shall be Rent R/R, Gross Annual Value, 1,68,000, Illustration 9: Mr. X has let out one house property to Mr. Y @ ` 80,000 p.m. Fair rent `90,000 p.m., Municipal valuation `80,000 p.m. and Standard rent of the house `76,000 p.m. The house remained vacant, for 2 months and there was unrealised rent for 3 months. Mr. X has paid municipal tax of `60,000 and, interest on loan for construction of house property is `69,000., Compute his Income Tax Liability for A.Y.2022-23., Solution:, `, Computation of income under the head house property, Gross Annual Value, 9,12,000.00, Working Note:, `, (a) Fair Rent (90,000 x 12), 10,80,000, (b) Municipal Valuation (80,000 x 12), 9,60,000, (c) Higher of (a) or (b), 10,80,000, (d) Standard Rent (76,000 x 12), 9,12,000, (e) Expected Rent {Lower of (c) or (d)}, 9,12,000, (f) Rent received /receivable (80,000 x 7), 5,60,000, If there was no vacancy, in that case rent received receivable would have been, `7,20,000 and it was still less than expected rent ,therefore GAV shall be expected rent, GAV, 9,12,000, Less: Municipal Tax, (60,000.00), Net Annual Value, 8,52,000.00, Less: 30% of NAV u/s 24(a), (2,55,600.00), Less: Interest on capital borrowed u/s 24(b), (69,000.00), Income under the head House Property, 5,27,400.00, Gross Total Income, 5,27,400.00, Less: Deduction u/s 80C to 80U, NIL, Total Income, 5,27,400.00, Computation of Tax Liability, Tax on ` 5,27,400 at slab rate, 17,980.00, Add: HEC @ 4%, 719.20, Tax Liability, 18,699.20, Rounded off u/s 288B, 18,700.00, Illustration 10: Mr. X has a property whose municipal valuation is ` 1,30,000 p.a. The fair rent is `1,10,000, p.a. and the standard rent fixed by the Rent Control Act is ` 1,20,000 p.a. The property was let out for a rent, of ` 11,000 p.m. throughout the previous year. Unrealised rent was ` 11,000 and all conditions prescribed by
Page 171 :
Income Under The Head House Property, , 171, , Rule 4 are satisfied. He paid municipal taxes @ 10% of municipal valuation. Interest on borrowed capital, was ` 2,00,000 for the year. He has LTCG `6,00,000. Compute his tax liability for A.Y. 2022-23., Solution:, Computation of Income from house property of Mr. X for A.Y. 2022-23, Gross Annual Value, 1,21,000.00, Working Note:, `, (a) Fair Rent, 1,10,000, (b) Municipal Valuation, 1,30,000, (c) Higher of (a) or (b), 1,30,000, (d) Standard Rent, 1,20,000, (e) Expected Rent {Lower of (c) or (d)}, 1,20,000, (f) Rent received /receivable (11,000 x 11), 1,21,000, GAV, 1,21,000, Less: Municipal Tax ( 10% of `1,30,000), (13,000.00), Net Annual Value, 1,08,000.00, Less: 30% of NAV u/s 24(a), (32,400.00), Less: Interest on capital borrowed u/s 24(b), (2,00,000.00), Loss under the head House Property, (1,24,400.00), Loss shall be adjusted from LTCG and balance LTCG shall be (6,00,000 – 1,24,400), 4,75,600.00, Total Income, 4,75,600.00, Income Tax (`4,75,600 – 2,50,000) x 20%, 45,120.00, Less: Rebate u/s 87A, (12,500.00), Tax before Health & Education cess, 32,620.00, Add: HEC @ 4%, 1,304.80, Tax Liability, 33,924.80, Rounded off u/s 288B, 33,920.00, Illustration 11: Mr. X has a property whose municipal valuation is ` 2,50,000 p.a. The fair rent is `2,00,000, p.a. and the standard rent fixed by the Rent Control Act is ` 2,10,000 p.a. The property was let out for a rent, of ` 20,000 p.m. However, the tenant vacated the property on 31.01.2022. Unrealised rent was `20,000 and, all conditions prescribed by Rule 4 are satisfied. He paid municipal taxes @ 8% of municipal valuation., Interest on borrowed capital was `1,65,000 for the year. He has casual income `3,00,000. Compute his tax, liability for A.Y. 2022-23., (b) Presume he is non-resident and his date of birth 31.03.1962., Solution:, Computation of income from house property of Mr. X for A.Y. 2022-23, Gross Annual Value, 1,80,000.00, Working Note:, `, (a) Fair Rent, 2,00,000, (b) Municipal Valuation, 2,50,000, (c) Higher of (a) or (b), 2,50,000, (d) Standard Rent, 2,10,000, (e) Expected Rent {Lower of (c) or (d)}, 2,10,000, (f) Rent received /receivable (20,000 x 9), 1,80,000, If there was no vacancy, in that case rent received receivable would have been, (20,000 x 11) `2,20,000 which is exceeding expected rent hence GAV shall be rent, received /receivable i.e. `1,80,000, GAV, 1,80,000, Less: Municipal Tax (8% of `2,50,000), (20,000.00), Net Annual Value, 1,60,000.00, Less: 30% of NAV u/s 24(a), (48,000.00), Less: Interest on capital borrowed u/s 24(b), (1,65,000.00), Loss under the head House Property, (53,000.00)
Page 172 :
Income Under The Head House Property, , 172, , Loss shall be carried forward and as per section 58(4), loss shall not be adjusted from casual income, Casual income, 3,00,000.00, Total Income, 3,00,000.00, Income Tax `3,00,000 x 30%, 90,000.00, Less: Rebate u/s 87A, (12,500.00), Tax before health & education cess, 77,500.00, Add: HEC @ 4%, 3,100.00, Tax Liability, 80,600.00, (b), Income Tax `3,00,000 x 30%, 90,000.00, Add: HEC @ 4%, 3,600.00, Tax Liability, 93,600.00, Note: Rebate under section 87A is not allowed to non-resident., MAY – 2012 (4 Marks), Explain the treatment of unrealized rent and its recovery in subsequent years under the provisions of Income, Tax Act, 1961., Answer: Refer answer given in the book, Question 7: Write a note on Statutory Deduction or Standard Deduction., Answer: Statutory Deduction or Standard Deduction Section 24(a), Under section 24(a), every assessee shall be allowed a notional expenditure equal to thirty per cent of the, net annual value of the house for the various expenditures incurred by him., Actual expenditure incurred by the assessee shall not be taken into consideration., Example, Net annual value of one house is `3,00,000 and actual expenditure incurred on repairs are `75,000,, deduction allowed under section 24(a) shall be `90,000., Question 8: Write a note on deduction for interest on the capital borrowed., Answer: Interest on borrowed capital is allowed as deduction under section 24(b), If any assessee has taken a loan or advance for purchase/ construction / renovation / addition / alteration /, substitution or repair etc. of the house property, interest on such loan shall be allowed to be deducted under, section 24(b) from NAV and interest is allowed on due basis but only simple interest is allowed i.e. interest, on interest is not allowed. The assessee can take any number of loan. Interest for the year for which income, is being computed shall be allowed in the same year and shall be called current period interest. Interest for, the period prior to the year in which the house was purchased or constructed shall be called prior period, interest and such interest shall be allowed in 5 annual equal instalments starting from the year in which the, house was purchased or constructed. E.g. If Mr. X had taken a loan of `5,00,000 for construction of property, on 01.10.2020 and interest is payable @ 10% p.a. and the construction was completed on 30.06.2021, in this, case interest allowed under section 24(b) shall be:, Interest for the year (01.04.2021 to 31.03.2022) = 10% of ` 5,00,000 = ` 50,000, Prior period interest =10% of ` 5,00,000 for 6 months (from 01.10.2020 to 31.03.2021)=` 25,000, Prior period interest to be allowed in 5 equal annual installments of ` 5,000 from the year of completion of, construction i.e. in this case, P.Y.2021-22., Therefore, total interest deduction under section 24(b) = 50,000 + 5000 = ` 55,000., If any assessee has taken a new loan to repay the original loan, in such cases interest for such new loan shall, be allowed in the similar manner., Unpaid purchase price would be considered as capital borrowed:, Where a buyer enters into an arrangement with a seller to pay the sale price in installments along with, interest due thereon, the seller becomes the lender in relation to the unpaid purchase price and the buyer, becomes the borrower. In such a case, unpaid purchase price can be treated as capital borrowed for acquiring, property and interest paid thereon can be allowed as deduction under section 24.
Page 173 :
Income Under The Head House Property, , 173, , Illustration 12: Mr. X has taken a loan of `15,00,000 on 01.07.2017 from State Bank of India @ 12% p.a., for construction of one house which was completed on 01.05.2021 and was let out @ `90,000 p.m. w.e.f, 01.07.2021 and Fair rent is `1,25,000 p.m. and the assessee has paid municipal tax of `30,000 in P.Y. 202122 and the assessee has repaid the loan amount in annual instalment of `1,00,000 starting from 01.01.2020., Compute his income tax liability for the assessment year 2022-23., Solution:, `, Computation of income under the head House Property, Gross Annual Value, 13,75,000.00, Working Note:, `, (a) Fair Rent (1,25,000 x 11), 13,75,000, (b) Expected Rent, 13,75,000, (c) Rent received /receivable (90,000 x 9), 8,10,000, If there was no vacancy, in that case rent received/receivable would have been, `9,90,000 and it was still less than expected rent ,therefore GAV shall be expected rent, GAV, 13,75,000, Less: Municipal Tax, (30,000.00), Net Annual Value, 13,45,000.00, Less: 30% of NAV u/s 24(a), (4,03,500.00), Less: Interest on capital borrowed u/s 24(b), (2,84,400.00), Working Note:, `, Current period Interest, From 01.04.2021 to 31.03.2022, (13,00,000 x 12% x 9/12) + (12,00,000 x 12% x 3 /12) = 1,53,000, Prior period interest, From 01.07.2017 to 31.03.2021, 15,00,000 x 12% x 30/12 = 4,50,000, 14,00,000 x 12% x 12/12 = 1,68,000, 13,00,000 x 12% x 3/12 = 39,000, Instalment = 6,57,000 / 5 =, 1,31,400, Total Interest = `1,53,000 + `1,31,400=, 2,84,400, Income under the head house property, 6,57,100.00, Gross Total Income, 6,57,100.00, Less: Deduction u/s 80C, (1,00,000.00), Total Income, 5,57,100.00, Computation of Tax Liability, Tax on normal income `5,57,100 at slab rate, 23,920.00, Add: HEC @ 4%, 956.80, Tax Liability, 24,876.80, Rounded off u/s 288B, 24,880.00, Illustration 13: Mr. X has taken a loan of `15,00,000 on 01.07.2017 from State Bank of India @ 12% p.a., for construction of one house which was completed on 01.04.2020 and was let out @ `90,000 p.m. w.e.f, 01.05.2020 and Fair rent is `1,00,000 p.m. and the assessee has paid municipal tax of `30,000 in P.Y. 202122 and the assessee has repaid the loan amount in annual instalment of `1,00,000 starting from 01.01.2020., Compute his income tax liability for the assessment year 2022-23., Solution:, `, Computation of income under the head House Property, Gross Annual Value, 12,00,000.00, Working Note:, `, (a) Fair Rent (1,00,000 x 12), 12,00,000, (b) Expected Rent, 12,00,000, (c) Rent received /receivable (90,000 x 12), 10,80,000
Page 174 :
Income Under The Head House Property, GAV, Less: Municipal Tax, Net Annual Value, Less: 30% of NAV u/s 24(a), Less: Interest on capital borrowed u/s 24(b), Working Note:, Current period Interest, From 01.04.2021 to 31.03.2022, (13,00,000 x 12% x 9/12) + (12,00,000 x 12% x 3 /12) = 1,53,000, Prior period interest, From 01.07.2017 to 31.03.2020, 15,00,000 x 12% x 30/12 = 4,50,000, 14,00,000 x 12% x 3/12 = 42,000, Instalment = 4,92,000 / 5 =, Total Interest = `1,53,000 + `98,400=, Income under the head house property, Gross Total Income, Less: Deduction u/s 80C, Total Income, Computation of Tax Liability, Tax on normal income `4,67,600 at slab rate, Less: Rebate u/s 87A, Tax Liability, , 174, 12,00,000, (30,000.00), 11,70,000.00, (3,51,000.00), (2,51,400.00), `, , 98,400, 2,51,400, 5,67,600.00, 5,67,600.00, (1,00,000.00), 4,67,600.00, 10,880.00, (10,880.00), Nil, , Illustration 14: Mr. X took a loan of `5,00,000 on 01.10.2018 @ 10% p.a. for construction of house which, was completed on 31.03.2022., Compute interest on capital borrowed for the previous year 2021-22., Solution:, Prior period interest, From 01.10.2018 to 31.03.2021, = 5,00,000 x 10% x 30/12 = `1,25,000, Installment = `1,25,000/5 = `25,000, Current year interest, From 01.04.2021 to 31.03.2022, = 5,00,000 x 10% = `50,000, Total Interest = `25,000 + `50,000 = `75,000, Illustration 15: Mr. X has taken a loan of `10,00,000 from SBI on 01/04/2019 @ 10% p.a. for construction, of one house which was completed on 01/07/2021 and was let out at a rent of `30,000 per month paid, municipal taxes `40,000. He has taken loan of `10,00,000 from PNB on 01/10/2021 @ 12% p.a. to repay, the original loan compute his income and tax liability for Assessment year 2022-23., Solution:, `, Computation of income under the head House Property, Gross Annual Value (30,000 X 9), 2,70,000.00, Less: Municipal Tax, (40,000.00), Net Annual Value, 2,30,000.00, Less: 30% of NAV u/s 24(a), (69,000.00), Less: Interest on capital borrowed u/s 24(b), (1,50,000.00), Working Note:, `, Current period Interest, From 01.04.2021 to 31.03.2022, (10,00,000 x 10% x 6/12) + (10,00,000 x 12% x 6 /12) = 1,10,000, Prior period interest
Page 175 :
Income Under The Head House Property, From 01.04.2019 to 31.03.2021, 10,00,000 x 10% x 2 = 2,00,000, Instalment = 2,00,000 / 5 =, Total Interest = `1,10,000 + `40,000=, Income under the head house property, Gross Total Income, Less: Deduction u/s 80C, Total Income, , 175, , 40,000, 1,50,000, 11,000.00, 11,000.00, Nil, 11,000.00, , Computation of Tax Liability, Tax Liability, , Nil, , Note: In this case whether deduction u/s 80C shall be allowed or not is not clear. In our opinion no, deduction shall be allowed., Illustration 16: Mr. X has taken a loan of `10,00,000 from SBI on 01/04/2018 @ 10% p.a. for construction, of one house. The Assessee has taken a loan of 6,00,000 from PNB on 01/10/2020 @ 12% p.a. to repay loan, to SBI. House was completed on 01/07/2021 and was let out at a rent of `1,00,000 per month paid, municipal taxes `10,000. Compute his income and tax liability for Assessment year 2022-23., Solution:, `, Computation of income under the head House Property, Gross Annual Value (100,000 X 9), 9,00,000.00, Less: Municipal Tax, (10,000.00), Net Annual Value, 8,90,000.00, Less: 30% of NAV u/s 24(a), (2,67,000.00), Less: Interest on capital borrowed u/s 24(b), (1,73,200.00), Working Note:, `, Current period Interest, From 01.04.2021 to 31.03.2022, (4,00,000 x 10% ) + (6,00,000 x 12%) = 1,12,000, Prior period interest, From 01.04.2018 to 31.03.2021, 10,00,000 x 10% x 30/12 = 2,50,000, 4,00,000 x 10% x 6/12 =, 20,000, 6,00,000 x 12% x 6/12 =, 36,000, Instalment = 3,06,000 / 5 =, Total Interest = `1,12,000 + `61,200=, Income under the head house property, Gross Total Income, Less: Deduction u/s 80C, Total Income, Computation of Tax Liability, Tax on `4,49,800 at slab rate, Less: Rebate u/s 87A, Tax Liability, , 61,200, 1,73,200, 4,49,800.00, 4,49,800.00, Nil, 4,49,800.00, 9,990.00, (9,990.00), Nil, , Loan from outside India, As per section 25 if loan is taken from outside India, in that case also interest is allowed but the person, making payment of interest should deduct tax at source or the person receiving interest should have an agent, in India., Illustration 17: Mr. X has constructed one house on 01.09.2021 and it was let out @ `1,25,000 p.m. and, municipal taxes paid are `35,000. The house was constructed after taking a loan from outside India and
Page 176 :
Income Under The Head House Property, , 176, , interest allowed under section 24(b) is `2,10,000, but the assessee has not deducted tax at source., Compute assessee’s tax liability for assessment year 2022-23., Solution:, `, Gross Annual Value (1,25,000 x 7), 8,75,000.00, Less: Municipal Taxes, (35,000.00), Net Annual Value, 8,40,000.00, Less: 30% of NAV u/s 24(a), (2,52,000.00), Less: Interest on capital borrowed u/s 24(b), Nil, Income under the head House Property, 5,88,000.00, Computation of Tax Liability, Tax on `5,88,000 at slab rate, 30,100.00, Add: HEC @ 4%, 1,204.00, Tax Liability, 31,304.00, Rounded off u/s 288B, 31,300.00, (b) Presume in the above question, the person who has given the loan has one agent in India as per section, 163., Compute tax liability for the assessment year 2022-23., Solution:, `, Gross Annual Value (1,25,000 x 7), 8,75,000.00, Less: Municipal Taxes, (35,000.00), Net Annual Value, 8,40,000.00, Less: 30% of NAV u/s 24(a), (2,52,000.00), Less: Interest on capital borrowed u/s 24(b), (2,10,000.00), Income under the head House Property, 3,78,000.00, Computation of Tax Liability, Tax on `3,78,000 at slab rate, 6,400.00, Less: Rebate u/s 87A, (6,400.00), Tax Liability, Nil, (c) Presume in the above question, the assessee has deducted tax at source., Compute tax liability for the assessment year 2022-23., Solution:, Gross Annual Value (1,25,000 x 7), Less: Municipal Taxes, Net Annual Value, Less: 30% of NAV u/s 24(a), Less: Interest on capital borrowed u/s 24(b), Income under the head House Property, Computation of Tax Liability, Tax on `3,78,000 at slab rate, Less: Rebate u/s 87A, Tax Liability, , `, 8,75,000.00, (35,000.00), 8,40,000.00, (2,52,000.00), (2,10,000.00), 3,78,000.00, 6,400.00, (6,400.00), Nil, , MAY – 2007 (6 Marks), Miss Charlie, an American national, got married to Mr. Radhey of India in USA on 02.03.2021 and came to, India for the first time on 16.03.2021. She left for USA on 23.09.2021., She returned to India again on 27.03.2022. While in India, she had purchased a show room in Mumbai on, 22.04.2021, which was leased out to a company on a rent of `25,000 p.m. from 01.05.2021. She had taken, loan from a bank for purchase of this show room on which bank had charged interest of `97,500 upto, 31.03.2022., She had received the following gifts from her relatives and friends during 01.04.2021 to 30.06.2021:, - From parents of husband, ` 51,000, - From married sister of husband, ` 11,000
Page 177 :
Income Under The Head House Property, , 177, , - From two very close friends of her husband, `1,51,000 and `21,000, `1,72,000, Determine her residential status and compute the total income chargeable to tax alongwith the amount of tax, payable on such income for the Assessment Year 2022-23., Answer., As per section 6(1), an individual is considered to be resident in India if he stays in India for 182 days or, more or he stays in India for 60 days or more during the relevant previous year and also for 365 days or, more during 4 years preceding the relevant previous year., Since Miss Charlie is not able to comply with any of the condition mentioned above, she is non-resident in, previous year 2020-21., Her stay in India during the previous year 2021-22 and in the preceding four years is as under:P.Y. 2021-22, 01.04.2021 to 23.09.2021, - 176 days, 27.03.2022 to 31.03.2022, - 5 days, Total, 181 days, Four preceding previous years, P.Y. 2020- 2021 [16.03.2021 to 31.03.2021], P.Y. 2019- 2020 [01.04.2019 to 31.03.2020], P.Y. 2018- 2019 [01.04.2018 to 31.03.2019], P.Y. 2017- 2018 [01.04.2017 to 31.03.2018], Total, , - 16 days, Nil, Nil, Nil, 16 days, , Computation of total income of Miss. Charlie for the A.Y. 2022-23, `, Income from house property, Gross Annual Value [25,000 x 11], 2,75,000, Less: Municipal taxes, Nil, Net Annual Value, 2,75,000, Less: Standard deduction 30% of NAV u/s 24(a), (82,500), Less: Interest on loan 24(b), (97,500), Income under the Head House Property, 95,000, Income from other sources, - `51,000 received from parents of husband would be exempt, Nil, - `11,000 received from married sister of husband is exempt, Nil, - From two friends of husband `1,51,000 and `21,000, aggregating to `1,72,000., 1,72,000, Income under the head Other Sources, 1,72,000, Total Income, 2,67,000, Computation of tax payable by Miss. Charlie for the A.Y. 2022-23, Tax on total income of `2,67,000, 850.00, Add: HEC @ 4%, 34.00, Tax Liability, 884.00, Rounded off u/s 288B, 880.00, Notes –, 1. Actual rent received has been taken as the gross annual value in the absence of other information (i.e., Municipal value, fair rental value and standard rent) in the question., 2. Rebate under section 87A is not allowed to non-resident., Question 9: Write a note on house which is self-occupied., Answer: House which is self-occupied Section 23(2)(a), If any person has house which is self-occupied (maximum two house), its GAV shall be nil and municipal, tax are not allowed to be deducted and NAV shall also be nil and deduction under section 24(a) is not, allowed but deduction under section 24(b) is allowed but maximum `30,000 however it will be maximum, `2,00,000 if loan has been taken w.e.f 01st April 1999 onwards for purchase or construction and house has
Page 178 :
Income Under The Head House Property, , 178, , been purchased or constructed within 5 years from the end of the year in which the assessee has taken loan, and also assessee should submit a certificate from the lender certifying the amount of interest. If loan is, taken for repairs / renovations etc. maximum interest allowed shall be `30,000., If the house is self occupied as well as vacant, its income shall be computed as if it is self occupied house., E.g. Mr. X has one house which is vacant for 3 months and self occupied for 9 months, its income shall be, computed considering it to be self occupied house., The ceiling prescribed for two self-occupied property as above in respect of interest on loan borrowed does, not apply to a deemed let-out property., Illustration 18: Mrs. X has one house property at Indira Nagar in Bangalore. She stays with her family in, the house. The rent of similar property in the neighbourhood is `25,000 p.m. The municipal valuation is, `23,000 p.m. Municipal taxes paid is `8,000. The loan of `20,00,000 was taken on 01.01.2015 from SBI, Housing Finance Ltd. The construction was completed on 30.11.2017. The accumulated interest up to, 31.03.2017 is `3,00,000. During the previous year 2021-22, Mrs. X paid ` 1,88,000 which included, `1,44,000 as interest. Compute Mrs. X’s income from house property for A.Y. 2022-23. All the conditions, for higher deduction of interest in case of self-occupied property is satisfied., Solution:, Computation of income from house property of Mrs. X for A.Y. 2022-23, Particulars, Amount, GAV, Nil, Less: Municipal Tax, Nil, NAV, Nil, Less: Statutory Deduction u/s 24(a), Nil, Less: Interest on capital u/s 24(b), Current period interest `1,44,000, Prior period installment `3,00,000 / 5 = `60,000, Maximum allowed, (2,00,000), Loss from house property, (2,00,000), Illustration 19(a): Mr. X has taken a loan of `5,00,000 on 01.10.1998 @ 10% p.a. for construction of a, house which was completed on 01.10.2020 and the house remained self-occupied throughout the previous, year 2021-22., The assessee has income under the head salary `4,00,000., Compute tax liability for assessment year 2022-23., Solution:, `, Net Annual Value, Nil, Less: Interest on capital borrowed u/s 24(b), (30,000), Working Note:, `, Current period Interest, From 01.04.2021 to 31.03.2022, 5,00,000 x 10% =, 50,000, Prior period interest, From 01.10.1998 to 31.03.2020, 5,00,000 x 10% x 258/12 =, 10,75,000, Instalment = 10,75,000 / 5 =, 2,15,000, Total Interest = `50,000 + `2,15000 =, 2,65,000, Subject to maximum `30,000, Loss under the head House Property, (30,000), Income under the head Salary, 4,00,000, Gross Total Income, 3,70,000, Less: Deduction u/s 80C to 80U, Nil, Total Income, 3,70,000
Page 179 :
Income Under The Head House Property, Computation of Tax Liability, Tax on `3,70,000 at slab rate, Less: Rebate u/s 87A, Tax Liability, , 179, 6,000, (6,000), Nil, , Illustration 19(b): Presume in above question, the loan was taken on 01.10.2017. The assessee has, submitted a certificate confirming the amount of interest., Solution:, `, Net Annual Value, Nil, Less: Interest on capital borrowed u/s 24(b), (75,000), Working Note:, `, Current period Interest, From 01.04.2021 to 31.03.2022, 5,00,000 x 10% =, 50,000, Prior period interest, From 01.10.2017 to 31.03.2020, 5,00,000 x 10% x 30/12 =, 1,25,000, Instalment = 1,25,000 / 5 =, 25,000, Total Interest = `50,000 + `25,000 =, 75,000, Loss under the head House Property, (75,000), Income under the head Salary, 4,00,000, Gross Total Income, 3,25,000, Less: Deduction u/s 80C to 80U, Nil, Total Income, 3,25,000, Computation of Tax Liability, Tax on `3,25,000 at slab rate, 3,750.00, Less: Rebate u/s 87A, (3,750.00), Tax Liability, Nil, NOV – 2013 (5 Marks), Mr. X owns a residential house in Delhi. The house is having two identical units. First unit of the house is, self-occupied by Mr. X and another unit is rented for `55,000 p.m. The rented unit was vacant for three, months during the year. The particulars of the house for the previous year 2021-22 are as under:, Standard Rent, `11,20,000 p.a., Municipal Valuation, `10,44,000 p.a., Fair Rent, `11,35,000 p.a., Municipal tax paid by Mr. X, 12% of the Municipal Valuation, Light and water charges, `800 p.m., Interest on borrowed capital, `2,000 p.m., Insurance charges, `3,500 p.a., Painting expenses, `16,000 p.a., Compute his income and tax liability of Mr. X for the assessment year 2022-23., Answer:, Computation of Income from house property of Mr. X for A.Y. 2022-23, Rented unit (50% of total area), Gross Annual Value (GAV), 4,95,000, (a) Municipal valuation (` 10,44,000 x ½), 5,22,000, (b) Fair rent (` 11,35,000 x ½), 5,67,500, (c) Higher of (a) or (b), 5,67,500, (d) Standard rent (` 11,20,000 x ½), 5,60,000, (e) Expected rent lower of (c) or (d), 5,60,000, (f) Rent receivable for the whole year (` 55,000 x 9), 4,95,000
Page 180 :
Income Under The Head House Property, If there was no vacancy, rent received/receivable would have been 55,000 x 12 =, 6,60,000, which is higher than Expected rent, hence GAV shall be rent, received/receivable, Less: Municipal taxes (12% of ` 5,22,000), Net Annual Value (NAV), Less : Deductions, (a) 30% of NAV under section 24(a), (b) Interest on borrowed capital (` 1,000 x 12) u/s 24(b), Taxable income from let out portion, Self occupied unit (50% of total area), Annual value, Less : Deduction under section 24(b):, Interest on borrowed capital (` 1,000 x 12), Income from house property, Gross Total Income, Less: Deduction u/s 80C to 80U, Total Income (Rounded off u/s 288A), , 180, , (62,640), 4,32,360, (1,29,708), (12,000), 2,90,652, Nil, (12,000), 2,78,652, 2,78,652, Nil, 2,78,650, , Computation of Tax Liability, Tax on `2,78,650 at slab rate, 1,432.50, Less: Rebate u/s 87A, (1,432.50), Tax Liability, Nil, Note: No deduction will be allowed separately for light and water charges, insurance charges and painting, expenses., NOV – 2008 (5 Marks), Mr. X owns one residential house in Mumbai. The house is having two units. First unit of the house is self, occupied by Mr. X and another unit is rented for `55,000 p.m. The rented unit was vacant for 2 months, during the year., The particulars of the house for the previous year 2021-22 are as under:, Standard rent, ` 10,62,000 p.a., Municipal valuation, ` 8,90,000 p.a., Fair rent, ` 10,85,000 p. a, Municipal tax, 15% of municipal valuation, Light and water charges paid by the tenant, ` 500 p.m., Interest on borrowed capital, ` 1,500 p.m., Insurance charges paid by Mr. X, ` 3,000 p.a., Repairs, ` 12,000 p.a., Compute income from house property of Mr. X and tax liability for the A.Y. 2022-23., Answer., Computation of Income from house property for A.Y. 2022-23, (A) Rented unit (50% of total area), `, Gross Annual Value, 5,50,000, Working note:, `, (a) Fair rent (`10,85,000 x ½), 5,42,500, (b) Municipal valuation (`8,90,000 x ½), 4,45,000, (c) Higher of (a) or (b), 5,42,500, (d) Standard rent (`10,62,000 x ½), 5,31,000, (e) Expected rent (lower of (c) or (d), 5,31,000, (f) Rent received or receivable (`55,000 x 10), 5,50,000, Since, the actual rent received is higher than the annual letting value, the actual, rent received is the Gross Annual value i.e. `5,50,000
Page 181 :
Income Under The Head House Property, Less: Municipal taxes (15% of `4,45,000), Net Annual value, Less: Deductions under section 24, (i) 30% of net annual value u/s 24(a), (ii) Interest on borrowed capital (`750 x 12) u/s 24(b), Taxable income from let out portion, (B) Self occupied unit (50% of total area), Annual value, Less: Deduction under section 24, Interest on borrowed capital (`750 x 12) u/s 24(b), Income from House property, Income under the head House Property, Gross Total Income, Less: Deduction u/s 80C to 80U, Total Income (rounded off u/s 288A), Computation of Tax Liability, Tax on `3,20,280 at slab rate, Less: Rebate u/s 87A, Tax Liability, , 181, (66,750), 4,83,250, 1,44,975, 9,000, , (1,53,975), 3,29,275, , Nil, 9,000, , (9,000), 3,20,275, 3,20,275, 3,20,275, Nil, 3,20,280, 3,514.00, (3,514.00), Nil, , Notes:, (i) It is assumed that both the units are of identical size. Therefore, the rented unit would represent 50% of, total area and the self-occupied unit would represent 50% of total area., (ii) No deduction will be allowed separately for light and water charges, insurance charges and repairs., MAY – 2000 (3 Marks), Mr. X commenced construction of a residential house intended exclusively for his residence, on 01.11.2020., He raised a loan from PNB of `5,00,000 at 16 per cent interest for the purpose of construction on, 01.11.2020. Finding that there was an over-run in the cost of construction he raised a further loan of, `8,00,000 at the same rate of interest on 01.10.2021. The assessee has submitted a certificate confirming the, amount of interest. What is the interest allowable under section 24, assuming that the construction was, completed by 31.03.2022?, Answer:, Since the house was for self-occupation only, the annual value of the property would be ‘nil’ under section, 23(2). The interest allowable for the current year has to be considered with respect to both the loans., Interest on loan borrowed after 01.04.1999 is eligible for deduction subject to a maximum of `2,00,000 in, the case of self occupied property., Prior period interest (upto 31.03.2021), (5,00,000 x 16% x 5 / 12 ), This is to be allowed over 5 years beginning with assessment year 2022-23. Amount, allowable for each year, Interest eligible for deduction for the assessment year 2022-23, Prior period interest : One-fifth of `33,333, Interest on first loan : Current year interest : 5,00,000 @ 16%, Interest on second loan : 8,00,000 @ 16% x 6/12, Total interest, Therefore, interest allowable under section 24 would be `1,50,667, , 33,333, 6,667, 6,667, 80,000, 64,000, 1,50,667
Page 182 :
Income Under The Head House Property, , 182, , Question 10: Write a note on more than two house which are self-occupied (deemed to be let out, property)., Answer: More than two house which are self-occupied (deemed to be let out property) Section 23(4), If any assessee has more than two house which are self-occupied, in such cases only two of these houses, shall be considered to be self-occupied and income shall be computed under section 23(2) and all other, houses shall be deemed to be let out and income shall be computed in the similar manner as in case of let out, house. Expected rent shall be considered to be GAV of the house., Illustration 20: Mr. X has 3 houses which are self occupied and the details of these houses is as under., Particulars, House I, House II, House III, (In `), (In `), (In `), Fair rent, 11,00,000, 12,00,000, 11,50,000, Municipal valuation, 11,24,000, 11,78,000, 11,25,000, Standard rent, 13,00,000, 12,50,000, 11,40,000, Municipal taxes paid, 1,00,000, 80,000, 90,000, Interest on capital borrowed on, 3,20,000, 2,90,000, 1,90,000, 01.04.2016 and all the necessary, conditions are complied with to avail, higher amount of interest., Repair charges, 10,000, 3,000, 8,000, Date of completion of house, 01.10.2018, 01.10.2018, 01.10.2018, Compute income under the head house property., Solution:, Option I, House I & II is Self Occupied, Loss, House III is deemed to be Let Out, Gross Annual Value, Working Note:, (a) Fair rent, (b) Municipal valuation, (c) Higher of (a) or (b), (d) Standard rent, (e) Expected rent {Lower of (c) or (d)}, GAV = Expected rent, Less: Municipal Taxes, Net Annual Value, Less: 30% of NAV u/s 24(a), Less: Interest on capital borrowed u/s 24(b), Income, Income under Option I [(2,00,000) + 5,45,000], Option II, House II & III is Self Occupied, Loss, House I is deemed to be Let Out, Gross Annual Value, Working Note:, (a) Fair rent, (b) Municipal valuation, (c) Higher of (a) or (b), , `, (2,00,000), 11,40,000, `, 11,50,000, 11,25,000, 11,50,000, 11,40,000, 11,40,000, 11,40,000, (90,000), 10,50,000, (3,15,000), (1,90,000), 5,45,000, 3,45,000, (2,00,000), 11,24,000, `, 11,00,000, 11,24,000, 11,24,000
Page 183 :
Income Under The Head House Property, (d) Standard rent, (e) Expected rent {Lower of (c) or (d)}, GAV = Expected rent, Less: Municipal Taxes, Net Annual Value, Less: 30% of NAV u/s 24(a), Less: Interest on capital borrowed u/s 24(b), Income, Income under Option II [3,96,800 + (2,00,000)], Option III, House I & III is Self Occupied, Loss, House II is deemed to be Let Out, Gross Annual Value, Working Note:, (a) Fair rent, (b) Municipal valuation, (c) Higher of (a) or (b), (d) Standard rent, (e) Expected rent {Lower of (c) or (d)}, GAV = Expected rent, Less: Municipal Taxes, Net Annual Value, Less: 30% of NAV u/s 24(a), Less: Interest on capital borrowed u/s 24(b), Income, Income under Option III [4,94,000 + (2,00,000)], Second Option is the best, Income under the head House Property, , 183, 13,00,000, 11,24,000, 11,24,000, (1,00,000), 10,24,000, (3,07,200), (3,20,000), 3,96,800, 1,96,800, , (2,00,000), 12,00,000, `, 12,00,000, 11,78,000, 12,00,000, 12,50,000, 12,00,000, 12,00,000, (80,000), 11,20,000, (3,36,000), (2,90,000), 4,94,000, 2,94,000, 1,96,800, , NOV – 1999 (6 Marks), Mr. X occupied two flats for his residential purposes, particulars of which are as follows:, Particulars, Flat I(in `), Flat II(in `), Municipal Valuation, 90,000, 45,000, Fair Rent, 1,20,000, 40,000, Fair rent under Rent Control Act, (i.e. Standard Rent), 80,000, Not available, Municipal taxes paid, 10% of municipal valuation 10% of municipal valuation, Fire insurance paid, 1,000, 600, Interest payable on capital borrowed, for purchase of flat, 40,000, Nil, Income of Mr. X from his Proprietary business– XYZ Warehousing Corporation is `6,50,000., Determine the taxable income and tax liability for the assessment year 2022-23. You are informed that Mr., X could not occupy flat for 2 months commencing from December 1st, 2021 and that he has attained the age, of 60 on 23.08.2021., Answer:, `, As per the amendments, two house shall be treated as self-occupied instead of one house but the aggregate, amount of interest shall not exceed `30,000/`2,00,000 as the case may be., Income shall be computed as per Section 23(2), Flat I & II is Self Occupied Sec 23(2), Income, (30,000.00)
Page 184 :
Income Under The Head House Property, Computation of Gross Total Income, Income under the head House Property, Income under the head Business/Profession, Gross Total Income, Less: Deduction u/s 80C to 80U, Total Income, , 184, (30,000.00), 6,50,000.00, 6,20,000.00, Nil, 6,20,000.00, , Computation of Tax Liability, Tax on `6,20,000 at slab rate, Add: HEC @ 4%, Tax Liability, , 34,000.00, 1,360.00, 35,360.00, , MAY – 2014 (5 Marks), Mrs. X has two houses, both of which are self occupied. The particulars of these are given below:, Particulars, (Value in `), House — I, House — II, Municipal Valuation per annum, 1,20,000, 1,15,000, Fair Rent per annum, 1,50,000, 1,75,000, Standard rent per annum, 1,00,000, 1,65,000, Date of completion, 31-03-1999, 31-03-2001, Municipal taxes payable during the year (paid for House II only), 12%, 8%, Interest on money borrowed for repair of property during current year, 55,000, Compute Mrs. X’s income from the House Property for the Assessment Year 2022-23., Solution:, In this case, Mrs. X has more than one house property for self-occupation. As per section 23(2), Mrs. X can, avail the benefit of self-occupation (i.e., benefit of “Nil” Annual Value) in respect of both the house, properties but aggregate amount of interest shall not exceed `30,000/`2,00,000 as the case may be., House I & II – Self-occupied, Income, Loss under the head house property, , `, (30,000), (30,000), RTP NOV -2020, , 1. Ms. Pihu has three houses, all of which are self-occupied. The particulars of these houses are given, below:, (Value in `), Particulars, House-I, House-II, House-III, 1,20,000, 1,20,000, 1,30,000, Municipal Valuation per annum, 1,45,000, 1,85,000, 1,10,000, Fair Rent per annum, 1,30,000, 1,90,000, 1,00,000, Standard rent per annum, 31.5.2011, 31-07-2008, 30-01-2005, Date of completion, 10%, 9%, 12%, Municipal taxes payable during the year, (paid for House II & III only), –, 75,000, –, Interest on money borrowed for repair of property, during current year, You are required to compute Pihu’s income from house property for the Assessment Year 2022-23 and, suggest which houses should be opted by Pihu to be assessed as self- occupied so that her tax liability is, minimum.
Page 185 :
Income Under The Head House Property, , 185, , Answer:, In this case, Pihu has more than two house properties for self-occupation. As per section 23(4), Pihu can, avail the benefit of self-occupation (i.e., benefit of “Nil” Annual Value) only in respect of any two of the, house properties, at her option. The other house property would be treated as “deemed let-out” property,, in respect of which the Expected rent would be the gross annual value. Pihu should, therefore, consider, the most beneficial option while deciding which house properties should be treated by her as selfoccupied., OPTION 1 [House I & II – Self-occupied and House III- Deemed to be let out], If House I and II are opted to be self-occupied, Pihu’s income from house property for A.Y.2022-23, would be –, Particulars, Amount in `, House I (Self-occupied) [Annual value is Nil], Nil, House II (Self-occupied) [Annual value is Nil, but interest deduction would be available,, (30,000), subject to a maximum of ` 30,000. In case of money borrowed for repair of selfoccupied property, the interest deduction would be restricted to `30,000, irrespective of, the date of borrowal]., 82,600, House III (Deemed to be let-out) [See Working Note below], Income from house property, 52,600, OPTION 2 [House I & III – Self-occupied and House II- Deemed to be let out], If House I and III are opted to be self-occupied, Pihu’s income from house property for A.Y.2022-23, would be –, Particulars, Amount in `, House I (Self-occupied) [Annual value is Nil], Nil, House II (Deemed to be let-out) [See Working Note below], 46,940, House III (Self-occupied) [Annual value is Nil], Nil, Income from house property, 46,940, OPTION 3 [House I – Deemed to be let out and House II & III – Self-occupied], If House II and III are opted to be self-occupied, Pihu’s income from house property for A.Y.2022-23, would be –, Particulars, Amount in `, House I (Deemed to be let-out) [See Working Note below], 70,000, House II (Self-occupied) [Annual value is Nil, but interest deduction would be available,, subject to a maximum of ` 30,000. In case of money borrowed for repair of self(30,000), occupied property, the interest deduction would be restricted to `30,000, irrespective of, the date of borrowal]., Nil, House III (Self-occupied) [Annual value is Nil], Income from house property, 40,000, Since Option 3 is more beneficial, Pihu should opt to treat House – II & III as Self- occupied and House, I as Deemed to be let out, in which case, her income from house property would be ` 40,000 for the A.Y., 2022-23., Working Note:, Computation of income from House I, II and House III assuming that all are deemed to be let out, Amount in Rupees, Particulars, House-I House-II House-III, Gross Annual Value (GAV), Expected rent is the GAV of house property, Expected rent= Higher of Municipal Value and Fair Rent but, 100,000, 1,85,000, 1,30,000, restricted to Standard Rent
Page 186 :
Income Under The Head House Property, Less: Municipal taxes (paid by the owner during the previous, year), Net Annual Value (NAV), Less: Deductions under section 24, (a) 30% of NAV, (b) Interest on borrowed capital (allowed in full in case of, deemed let out property), Income from deemed to be let-out house property, , 186, (Nil), , (10,800), , (12,000), , 1,00,000, , 1,74,200, , 1,18,000, , (30,000), –, , (52,260), (75,000), , (35,400), –, , 70,000, , 46,940, , 82,600, , Question 11: Write a note on computation of Income of Unoccupied House., Answer: Income of unoccupied house section 23(2)(b), As per section 23(2)(b),if any house cannot actually be occupied by the owner by reason of the fact that, owing to his employment, business or profession carried on at any other place, he has to reside at that other, place in a building not belonging to him, in such cases assessee shall have the option to compute income of, such house as if it is self-occupied i.e. it will not be considered to be deemed to be let out. Benefit u/s, 23(2)(a) or 23(2)(b) shall be allowed for maximum two houses., Question 12: Write a note on house property which is divided into different portions/units., Answer: If any house property is divided into different portions, every portion shall be considered to be a, separate house and income shall be computed accordingly. There is no need to treat the whole property as a, single unit for computation of income from house property., Municipal valuation/fair rent/standard rent, if not given separately, shall be apportioned between the let-out, portion and self-occupied portion either on plinth area or built-up floor space or on such other reasonable, basis., Property taxes, if given on a consolidated basis can be bifurcated as attributable to each portion or floor on a, reasonable basis., Illustration 21: Mr. X owns a house in Madras. During the previous year 2021-22, 2/3rd portion of the, house was self-occupied and 1/3rd portion was let out for residential purposes at a rent of ` 8,000 p.m., Municipal value of the property is `3,00,000 p.a., fair rent is `2,70,000 p.a. and standard rent is ` 3,30,000, p.a. He paid municipal taxes @ 10% of municipal value during the year. A loan of `25,00,000 was taken by, him during the year 2017 for acquiring the property. Interest on loan paid during the previous year 2021-22, was `1,20,000. Compute Mr. X’s income from house property for the A.Y. 2022-23. All the conditions for, higher deduction of interest in case of self-occupied property is satisfied., Solution:, There are two units of the house. Unit I with 2/3rd area is used by Mr. X for self-occupation throughout the, year and no benefit is derived from that unit, hence it will be treated as self-occupied and its annual value, will be nil. Unit 2 with 1/3rd area is let-out through out the previous year and its annual value has to be, determined as per section 23(1)., Computation of income from house property of Mr. X for A.Y. 2022-23, Particulars, `, Unit I (2/3rd area – self-occupied), Annual Value, Nil, Less: Deduction under section 24(b) 2/3rd of ` 1,20,000, (80,000), Loss from Unit I (self-occupied), (80,000), Unit II (1/3rd area – let out), Gross Annual Value, 1,00,000, Working Note:, `, (a) Fair rent, 90,000, (b) Municipal valuation, 1,00,000, (c) Higher of (a) or (b), 1,00,000, (d) Standard rent, 1,10,000, (e) Expected rent {Lower of (c) or (d)}, 1,00,000
Page 187 :
Income Under The Head House Property, (f) Rent received/ receivable (8,000 x 12), GAV = Expected rent, Less: Municipal Taxes, Net Annual Value, Less: 30% of NAV u/s 24(a), Less: Interest on capital borrowed u/s 24(b), Income under the head House Property, Net Loss, , 187, 96,000, 1,00,000, (10,000), 90,000, (27,000), (40,000), 23,000, (57,000), , Question 13: Write a note on house property owned by the assessee and used for own, business/profession., Answer: House property owned by the assessee and used for own business/profession Section, 22/section 30, If any person owns any house property and it is being used by him in his own business/profession, income of, such building shall not be computed under the head house property rather income shall be computed under, the head business/profession and as per section 30, for this purpose, while computing the income under the, head business/profession, no rent shall be allowed to be debited to the profit & loss account in connection, with such building. All the expenses of the house property shall be debited to the profit and loss account and, deduction under section 24(a) is not allowed rather actual expenditure shall be debited to the profit and loss, account. Such expenditure may be municipal tax, repairs, depreciation, land revenue, ground rent etc., Illustration 22: Mrs. X aged 62 years is engaged in a business in her own building and furnishes the, following information., Market rent of the building is `1,00,000 p.m. and expenses incurred on repairs are `37,000 and interest on, loan taken for construction of the building is `65,000 and depreciation `30,000 and municipal tax paid, `30,000 and land revenue paid `10,000 and premium paid for insurance of the house `7,000. ground rent, paid `8,000., Income from business before debiting any expense of house property is `16,00,000., Compute her income tax liability for Assessment Year 2022-23., Solution:, `, Income from business before debiting any expense of house property, 16,00,000.00, Less: Repair of Building, (37,000.00), Less: Interest on loan taken for construction of building, (65,000.00), Less: Depreciation, (30,000.00), Less: Municipal Taxes, (30,000.00), Less: Land revenue, (10,000.00), Less: Insurance premium of the house, (7,000.00), Less: Ground rent, (8,000.00), Income under the head Business/Profession, 14,13,000.00, Gross Total Income, 14,13,000.00, Less: Deduction u/s 80C to 80U, Nil, Total Income, 14,13,000.00, Computation of Tax Liability, Tax on `14,13,000 at slab rate, 2,33,900.00, Add: HEC @ 4%, 9,356.00, Tax Liability, 2,43,256.00, Rounded off u/s 288B, 2,43,260.00, Question 14: Write a note on a house property which is let-out for part of the year and self-occupied, for part of the year and may or may not be vacant., Answer: A house property which is let-out for part of the year and self-occupied for part of the year, and may or may not be vacant Section 23(3)
Page 188 :
Income Under The Head House Property, , 188, , If any house property is let out as well as self-occupied, in such cases expected rent (also called annual, letting value) shall be computed for full year but Rent received/receivable shall be only for the period the, house was let out and GAV shall be the higher. There will not be any such adjustment as in case of vacancy., Illustration 23: Mr. X constructed one house in 2020 and it is let out for 4 months and self occupied for 6, months and vacant for 2 months during previous year 2021-22. Municipal valuation of the house is `40,000, p.m. and fair rent `30,000 p.m. Standard rent of the house is `38,000 p.m. It was let out @ `32,000 p.m., Municipal tax levied is `6,000 out of which `2,000 was paid by the tenant and `2,000 by the assessee and, balance `2,000 yet to be paid., Interest on the capital borrowed for construction of the house is `30,000., Long Term Capital Gains is `2,10,000, Compute his income and tax Liability for the assessment year 2022-23., Solution:, Computation of income from House Property of Mr. X, Gross Annual Value, Working Note:, `, (a) Fair Rent (30,000 x 12), 3,60,000, (b) Municipal Valuation (40,000 x 12), 4,80,000, (c) Higher of (a) or (b), 4,80,000, (d) Standard Rent (38,000 x 12), 4,56,000, (e) Expected Rent {Lower of (c) or (d)}, 4,56,000, (f) Rent Received/Receivable (32,000 x 4), 1,28,000, If there was no vacancy, in that case rent received/receivable would have been, `1,92,000 and it was still less than expected rent, therefore GAV shall be expected, rent., GAV, 4,56,000, Less: Municipal Taxes, Net Annual Value, Less: 30% of NAV u/s 24(a), Less: Interest on capital borrowed u/s 24(b), Income under the head House Property, Long Term Capital Gains, Gross Total Income, Less: Deduction u/s 80C to 80U, Total Income, Computation of Tax Liability, Tax on `2,10,000 @ 20% u/s 112, Tax on `2,87,800 at slab rate, Less: Rebate u/s 87A, Tax before health & education cess, Add: HEC @ 4%, Tax Liability, Rounded off u/s 288B, , `, 4,56,000, , (2,000), 4,54,000, (1,36,200), (30,000), 2,87,800, 2,10,000, 4,97,800, Nil, 4,97,800, 42,000.00, 1,890.00, (12,500.00), 31,390.00, 1,255.60, 32,645.60, 32,650.00, , Illustration 24: Mrs. X owns a house property at Adyar in Chennai. The municipal value of the property is, `5,00,000, fair rent is ` 4,20,000 and standard rent is ` 4,80,000. The property was let-out for `50,000 p.m., up to December 2021. Thereafter, the tenant vacated the property and Mrs. X used the house for selfoccupation. Rent for the months of November and December 2021 could not be realised in spite of the, owner’s efforts. All the conditions prescribed under Rule 4 are satisfied. She paid municipal taxes @ 12%, during the year. She had paid interest of ` 25,000 during the year for amount borrowed for repairs for the, house property. She has LTCG `110,00,000. She has completed age of 80 years as on 31.03.2022. Compute, her tax liability for the A.Y. 2022-23.
Page 189 :
Income Under The Head House Property, , 189, , Solution:, Computation of income from house property of Mrs. X for the A.Y.2022-23, Gross Annual Value, 4,80,000.00, Working Note:, `, (a) Fair rent, 4,20,000, (b) Municipal valuation, 5,00,000, (c) Higher of (a) or (b), 5,00,000, (d) Standard rent, 4,80,000, (e) Expected rent {Lower of (c) or (d)}, 4,80,000, (f) Rent received/ receivable (50,000 x 7), 3,50,000, (unrealised rent 2 months and self occupied 3 months), GAV = Expected rent, 4,80,000, Less: Municipal Taxes, (60,000.00), Net Annual Value, 4,20,000.00, Less: 30% of NAV u/s 24(a), (1,26,000.00), Less: Interest on capital borrowed u/s 24(b), (25,000.00), Income under the head House Property, 2,69,000.00, LTCG, 110,00,000.00, Total Income, 112,69,000.00, Normal income, 2,69,000.00, LTCG (110,00,000 – 2,31,000) x 20%, 21,53,800.00, Add: Surcharge @ 15%, 3,23,070.00, Tax before health & education cess, 24,76,870.00, Add: HEC @ 4%, 99,074.80, Tax Liability, 25,75,944.48, Rounded off u/s 288B, 25,75,940.00, NOV – 2001 (4 Marks), From the following particulars furnished by Mr. X for the previous year ending 31.03.2022. Compute the, taxable income and tax liability for assessment year 2022-23:, (i) He owns a house property at metro city. The fair rental value per annum is ` 27,000 and the municipal, value is ` 24,000., (ii) The house was let out from 01.04.2021 to 31.08.2021 @ `2,100 per month. From 01.09.2021 Mr. X, occupies the house for his residence., (iii) Expenditure incurred on property and paid:, (a) Municipal tax, `4,000, (b) Fire insurance, `2,500, (c) Land revenue, `4,600, (d) Repairs, `1,000, (iv) Interest paid on borrowings for construction:, (a) For the current year, `21,600, (b) Instalment of prior period, `12,960, He has long term capital gains of `5,00,000., Answer:, Computation of income under the head House Property, `, Gross Annual Value, 27,000.00, Working Note:, `, (a) Fair Rent, 27,000, (b) Municipal Valuation, 24,000, (c) Higher of (a) or (b), 27,000, (d) Expected rent, 27,000, (e) Rent Received/Receivable (2,100 x 5), 10,500
Page 190 :
Income Under The Head House Property, GAV = Higher of (d) or (e), 27,000, Less: Municipal taxes, Net Annual Value, Less: 30% of NAV u/s 24(a), Less: Interest on capital borrowed u/s 24(b) (21,600 +12,960), Loss from house property, Income under the head Capital Gains (LTCG), Income under the head Capital gains after adjusting loss from house property, Gross Total Income, Less: Deduction u/s 80C to 80U, Total Income, Computation of Tax Liability, Tax on `2,31,540 (`4,81,540 – `2,50,000) @ 20% u/s 112, Less: Rebate u/s 87A, Tax before health and education cess, Add: HEC @ 4%, Tax Liability, Rounded off u/s 288B, , 190, , (4,000.00), 23,000.00, (6,900.00), (34,560.00), (18,460.00), 5,00,000.00, 4,81,540.00, 4,81,540.00, Nil, 4,81,540.00, 46,308.00, (12,500.00), 33,808.00, 1,352.32, 35,160.32, 35,160.00, , Illustration 25: Mr. X has one big house. 25% of it is being used by the assessee in his own, business/profession and 50% of the house is let out @ `10,000 p.m. However, it remained vacant for one, month and there is unrealised rent for 1½ month. Remaining 25% is self occupied throughout the year. Fair, rent of the entire house is `25,000 p.m., municipal valuation `22,000 p.m. and municipal tax paid is, `22,000. Insurance premium paid is `6,000, repair charges `8,000, land revenue paid `4,000, ground rent is, `3,000 and depreciation of the house is `12,000. Assessee’s income under the head business/profession, before charging expenditure relating to house property is `2,00,000., Compute his total income and tax liability for assessment year 2022-23., Solution:, `, Computation of income under the head House Property, Income from self occupied portion, Income of self occupied portion, Nil, Income of let out portion, Gross Annual Value, 1,50,000.00, Working Note:, `, (a) Fair Rent (12,500 x 12), 1,50,000, (b) Municipal Valuation (11,000 x 12), 1,32,000, (c) Expected rent {Higher of (a) or (b)}, 1,50,000, (d) Rent Received/Receivable (10,000 x 9.5), 95,000, If there was no vacancy, in that case rent received/receivable would have been, `1,05,000 and it was still less than expected rent, therefore GAV shall be expected, rent, GAV, 1,50,000, Less: Municipal taxes, (11,000.00), Net Annual Value, 1,39,000.00, Less: 30% of NAV u/s 24(a), (41,700.00), Less: Interest on capital borrowed u/s 24(b), Nil, Income under the head House Property, 97,300.00, Computation of income under the head Business/Profession, Income before debiting any expense of the house property, 2,00,000.00, Less: Municipal taxes, (5,500.00), Less: Insurance premium, (1,500.00), Less: Repairs charges, (2,000.00), Less: Land revenue, (1,000.00)
Page 191 :
Income Under The Head House Property, Less: Ground Rent, Less: Depreciation, Income under the head Business/Profession, Computation of Total Income, Income under the head House Property, Income under the head Business/Profession, Gross Total Income, Less: Deductions u/s 80C to 80U, Total Income, Computation of Tax Liability, Tax on `2,83,550 at slab rate, Less: Rebate u/s 87A, Tax Liability, , 191, (750.00), (3,000.00), 1,86,250.00, 97,300.00, 1,86,250.00, 2,83,550.00, Nil, 2,83,550.00, 1,677.50, (1,677.50), Nil, , NOV – 2003 (5 Marks), Mr. A and B constructed their houses on a piece of land purchased by them at New Delhi. The built up area, of each house was 1,000 sq. ft. ground floor and an equal area in the first floor., A started construction on 01.04.2020 and completed on 31.03.2021. A occupied the entire house on, 01.04.2021. A has availed a housing loan of `20 lakhs @ 12% p.a. on 01.04.2020 and has also submitted a, certificate from the lender certifying the amount of interest., B started construction on 01.07.2020 and completed on 01.07.2021. B occupied the ground floor on, 01.07.2021 and let out the first floor for a rent of `15,000 per month. However, the tenant vacated the house, on 31.12.2021 and B occupied the entire house during the period 01.01.2022 to 31.03.2022. B has availed a, housing loan of `12 lakhs @ 10% p.a. on 01.07.2020 and has also submitted a certificate from the lender, certifying the amount of interest., Following are the other information:, `, (i) Fair rental value of each unit, 6,00,000 per annum, (Ground floor / First floor), (ii) Municipal value of each unit, 3,00,000 per annum, (Ground floor / First floor), (iii) Municipal taxes paid by, A, 8,000, B, 8,000, (iv) Repair and maintenance charges paid by, A, 28,000, B, 30,000, No repayment was made by either of them till 31.03.2022., Compute tax liability for the assessment year 2022-23., Answer:, Computation of income from House Property of Mr. A, `, Net Annual Value, Nil, Less: 30% of NAV 24(a), Nil, Less: Interest on capital borrowed u/s 24(b), (2,00,000), Interest paid on borrowed capital, = 20,00,000 @ 12 % = `2,40,000, Interest deduction restricted to `2,00,000, There is no prior period interest, Loss under the head “House Property”, (2,00,000), Tax Liability, Nil, Computation of income from House Property of Mr. B, Ground floor (self occupied), Net Annual Value, Nil, Less: 30% of NAV u/s 24(a), Nil, Less: Interest on capital borrowed u/s 24(b), (69,000)
Page 192 :
Income Under The Head House Property, Working Note:, Prior period interest, From 01.07.2020 to 31.03.2021, = 12,00,000 x 10% x 9 / 12 = 90,000, 90,000 allowed in 5 equal instalments, = 90,000 / 5 = ` 18,000 per annum, = 18,000 / 2 = `9,000, Current period interest, From 01.04.2021 to 31.03.2022, = 12,00,000 x 10% x 1/2 = `60,000, Total Interest = `60,000 + ` 9,000 = `69,000, Loss from House Property, First floor (Let out), Gross Annual Value, Working Note:, (a) Fair Rent (6,00,000 x 9/12), (b) Municipal Valuation (3,00,000 x 9/12), (c) Higher of (a) or (b), (d) Expected Rent, (e) Rent Received/Receivable (15,000 x 6), GAV = Higher of (d) or (e), Less: Municipal taxes, Net Annual Value, Less: 30% of NAV u/s 24(a), Less: Interest on capital borrowed u/s 24(b), Working Note:, Prior period interest, From 01.07.2020 to 31.03.2021, = 12,00,000 x 10% x 9 / 12 = 90,000, 90,000 allowed in 5 equal instalments, = 90,000 / 5 = ` 18,000 per annum, = 18,000 / 2 = `9,000, Current period interest, From 01.04.2021 to 31.03.2022, = 12,00,000 x 10% x 1/2 = `60,000, Total Interest = `60,000 + ` 9,000 = `69,000, Income from House Property, Income under the head “Income from House Property” of Mr. B, (Both ground floor and first floor), Tax Liability, , 192, , (69,000), 4,50,000, `, 4,50,000, 2,25,000, 4,50,000, 4,50,000, 90,000, 4,50,000, (4,000), 4,46,000, (1,33,800), (69,000), , 2,43,200, 1,74,200, Nil, , Question 15: Write a note on composite rent., Answer: Composite Rent, A person may let out a house property alongwith some facilities like generator or security etc. and rent may, be charged combined for the house property as well as facility, such rent is called composite rent and in such, cases rent for house property shall be taxable under the head house property and rent for facilities shall be, taxable under the head other sources but after deducting all the expenses relating to such facility., If there are so many facilities and bifurcation is not possible, income shall be taxable under the head, business/profession or other sources e.g. in case of hotel business/paying guest accommodation/warehousing, or auditorium etc., income is taxable under the head business/profession.
Page 193 :
Income Under The Head House Property, , 193, , Illustration 26: Mr. X has let out one house alongwith generator facility and has charged a sum of `40,000, p.m. as rent, out of which `3,000 p.m. is attributable to the generator. He has paid `2,300 and the tenant has, paid `900 towards municipal taxes. The interest on the capital borrowed for construction of the house is, `7,000. Mr. X has paid repair charge of the generator `3,400, fuel charges `5,600 and operator’s salary `300, p.m., Compute the tax liability of Mr. X for assessment year 2022-23., Solution:, `, Computation of income under the head House Property, Gross Annual Value (37,000 x 12), 4,44,000.00, Less: Municipal Taxes, (2,300.00), Net Annual Value, 4,41,700.00, Less: 30% of NAV u/s 24(a), (1,32,510.00), Less: Interest on capital borrowed u/s 24(b), (7,000.00), Income under the head House Property, 3,02,190.00, Computation of income under the head Other Sources, Income from generator, (3,000 x 12), Less: Repair charges, Less: Fuel charges, Less: Operator Salary, (300 x 12), Income under the head Other Sources, Computation of Total Income, Income under the head House Property, Income under the head Other Sources, Gross Total Income, Less: Deduction u/s 80C to 80U, Total Income, Computation of Tax Liability, Tax on `3,25,590 at slab rate, Less: Rebate u/s 87A, Tax before health & education cess, Tax Liability, , 36,000.00, (3,400.00), (5,600.00), (3,600.00), 23,400.00, 3,02,190.00, 23,400.00, 3,25,590.00, Nil, 3,25,590.00, 3,779.50, (3,779.50), Nil, Nil, , Question 16: Write a note on letting out of building which is supplementary to the business., Answer: Letting out of building which is supplementary to the business, If any person has let out any house property for any purpose which is supplementary to the business of the, assessee, in such cases rental income shall be taxable under the head business/profession and all expenses of, such house property shall be debited to the profit and loss account. E.g. If a Public school has let out a part, of its building to a Bank, in this case rent received shall be considered to be income under the head, Business/Profession and all expenses of such house property shall be debited to profit and loss account., Similarly, if any company has constructed houses for the employees in their premises and it is let out to the, employees, rental income is taxable under the head Business/Profession., Question 17: Write a note on tax liability in respect of arrears of rent., Answer: Tax liability in respect of arrears of rent Section 25A, Sometimes rent of a house property may be increased from retrospective effect i.e. from back date and the, assessee may receive arrear of rent, such arrears are taxable in the year in which arrears have been received,, however deduction shall be allowed @ 30% of such arrears and only the balance amount shall be taxable. It, do not matter that the assessee do not have any house property in his name in that year., There is no specific provision given in the Income Tax Act relating to advance payment of rent.
Page 194 :
Income Under The Head House Property, , 194, , Illustration 27: Mr. X has let out his house to State Bank @ `20,000 p.m. The bank has increased the rent, on 1st July, 2021 to `27,000 p.m. retrospectively w.e.f. 01.11.2020. The assessee has paid municipal taxes of, `7,000 during the previous year 2021-22., Compute income under the head House Property for assessment year 2022-23., Solution:, Computation of income under the head House Property, `, `, Gross Annual Value (27,000 x 12), 3,24,000, Less: Municipal Taxes, (7,000), Net Annual Value, 3,17,000, Less: 30% of NAV u/s 24(a), (95,100), Less: Interest on capital borrowed u/s 24(b), Nil, 2,21,900, Add: Arrears of rent (Sec 25A) (7,000 x 5), 35,000, Less: 30% of `35,000, (10,500), 24,500, Income under the House Property, 2,46,400, MAY – 2002 (4 Marks), Mr. X owns a house property which is let out. During the previous year ending 31.03.2022 he receives (i), arrears of rent of `30,000 and (ii) unrealised rent of `20,000., You are requested to, (a), state, how they should be dealt with as per the provisions of the Act, and, (b), compute the income chargeable under the head “Income from house property”., Answer:, (a) As per provisions of section 25A, arrears of rent will be charged to tax as income from house property in, the previous year in which such rent is received, after deducting a sum equal to 30% of such amount. The, taxability shall be there whether Mr. X remains as the owner of the property in the concerned year or not. In, this case, it shall be taxed as income from house property in the year of receipt of such arrear rent., (b) As per the provisions of section 25A, the unrealised rent when received, it shall be deemed to be the, income chargeable under the head “Income from house property” and shall be charged to tax in the year of, receipt, after deducting a sum equal to 30% of such amount. In this case also, the taxability shall be there,, irrespective of the fact whether Mr. X is the owner of property or not in the year of receipt., Computation of income from house property, `, Arrears of rent, 30,000, (9,000), Less : Deduction @ 30% of `30,000/- u/s 25A, 21,000, Add : Unrealised rent received, 20,000, (6,000), Less : Deduction @ 30% of `20,000/- u/s 25A, 14,000, Income from house property, 35,000, Question 18: Write a note on sub-letting of house property., Answer: Sub-letting of house property Section 56, If any person has let out any house property and the tenant has further given the same house property on, rent, it is called sub-letting and in this case, income of the person who has sub-let the house property, shall, be computed under the head Other Sources and income shall be equal to gross rent received – expenses, incurred for such house property., Example, Mr. X has let out one house to Mr. Y at a rent of `1,00,000 p.m. and has paid municipal tax of `1,00,000., Mr. Y has sub-let 50% of the house property to Mr. Z at a rent of `70,000 p.m., in this case income of Mr. X, and Mr. Y shall be computed in the manner given below:, Income of Mr. X shall be computed under the head House Property, GAV (1,00,000 x 12), 12,00,000
Page 195 :
Income Under The Head House Property, Less: Municipal Tax, NAV, Less: Deduction u/s 24(a), Income under the head House Property, Income of Mr. Y shall be computed under the head Other Sources, Gross Rent received (70,000 x 12), Less: Rent paid by him (1,00,000 x 50% x 12), Income under the head Other Sources, , 195, (1,00,000), 11,00,000, (3,30,000), 7,70,000, 8,40,000, (6,00,000), 2,40,000, , Question 19: Write a note on computation of income from house property situated outside India., Answer:, Income of house property situated outside India shall be computed in the similar manner as in case of house, property situated in India and such income shall be taxable in the case of ROR. In case of NR or NOR such, income is exempt provided income is received outside India i.e. if income is received in India, it will be, taxable in case of NR/NOR also., Illustration 28: Mr. X, a British national, is a resident and ordinarily resident in India during the P.Y.202122. He owns a house in London, which he has let out at £ 10,000 p.m. The municipal taxes paid to the, Municipal Corporation of London is £ 8,000 during the P.Y.2021-22. The value of one £ in Indian rupee to, be taken at ` 100. Compute Mr. X’s taxable income for the A.Y. 2022-23., Solution:, For the P.Y.2021-22, Mr. X, a British national, is resident and ordinarily resident in India. Therefore,, income received by him by way of rent of the house property located in London is to be included in the total, income in India. Municipal taxes paid in London is be to allowed as deduction from the gross annual value., Computation of Income from house property of Mr. X for A.Y. 2022-23, Particulars, `, 1,20,00,000, Gross Annual Value (£ 10,000 × 12 × 100), Less: Municipal taxes paid (£ 8,000 × 100), Net Annual Value (NAV), Less: Deduction under section 24, (a) 30% of NAV, Income from house property, , (8,00,000), 1,12,00,000, (33,60,000), 78,40,000, , NOV – 2009 (4 Marks), Mrs. X, a resident and ordinarily resident individual, owns a house in U.S.A. She receives rent @ $ 2,000, per month. She paid municipal taxes of $ 1,500 during the financial year 2021-22., She also owns a two storied house in Mumbai, ground floor is used for her residence and first floor is let out, at a monthly rent of `10,000., Municipal taxes paid for the house amounts to `7,500. Mrs. X had constructed the house by taking a loan, from a nationalized bank on 20.06.2018. She repaid the loan of `54,000 including interest of `24,000 in the, current year., The value of one dollar is to be taken as `45., Compute total income from house property and also tax liability of Mrs. X for assessment year 2022-23., Answer., Computation of Income from House Property of Mrs. X for the Assessment Year 2022-23, `, GAV of the house in USA, ($2000 p.m. x `45 per USD x 12 months), 10,80,000.00, Less: Municipal taxes paid ($1500 x `45 per USD), (67,500.00), Net Annual Value, 10,12,500.00, Less: Statutory deduction under section 24(a) @ 30% of NAV, (3,03,750.00), Income from House property, 7,08,750.00, GAV of house at Mumbai (let out portion)- Ist Floor
Page 196 :
Income Under The Head House Property, Gross Annual Value (10,000 x 12), Less: Municipal taxes paid (1/2 of `7,500), Net Annual Value (NAV), Less: Statutory deduction under section 24(a) @ 30% of NAV, Less: Interest on Housing loan (1/2 of `24,000) 24(b), Income from House property, GAV of house at Mumbai (self occupied portion) – Ground Floor, Less: Municipal taxes, Net Annual Value, Less: Statutory deduction under section 24(a) @ 30% of NAV, Less: Interest on Housing Loan (1/2 of `24,000) 24(b), Loss from House property, Income from House property, Gross Total Income, Less: Deduction u/s 80C (Repayment of housing loan), Total Income (rounded off u/s 288A), Computation of Tax Liability, Tax on `7,36,130 at slab rate, Add: HEC @ 4%, Tax Liability, Rounded off u/s 288B, , 196, 1,20,000.00, (3,750.00), 1,16,250.00, (34,875), (12,000), 69,375.00, Nil, Nil, Nil, Nil, (12,000.00), (12,000.00), 7,66,125.00, 7,66,125.00, (30,000.00), 7,36,130.00, 59,726.00, 2,389.04, 62,115.04, 62,120.00, , Question 20: Write a note on treatment of income from co-owned property., Answer: Treatment of income from co-owned property Section 26, If any house property is owned by co-owners and their shares are given, in such cases each such co-owner, has to pay tax on his share in the income of house property but if shares are not given, it will be considered, to be income of co-owners (BOI/AOP) e.g. Mr. X and Mr. Y are co-owners of a house property and their, shares are not given and income is `20 lakhs, in this case it will be taxable as income of co-owners but if, share of each one is 50%, Mr. X will pay tax on income of `10 lakh and Mr. Y will pay tax on income of, `10 lakh., Where the house property owned by co-owners is self occupied by each of the co-owners, the annual value, of the property of each co-owner will be Nil and each co-owner shall be entitled to a deduction of `30,000 /, ` 2,00,000, as the case may be, under section 24(b) on account of interest on borrowed capital e.g. Mr. X, and Mr. Y are co-owners of a particular house property and it is self occupied by each one of them and each, one has share of 50% and interest on loan taken for construction of a house is `5 lakh and loan was taken, w.e.f 01.04.1999 onwards and house was completed within 5 years and a certificate has been submitted, confirming the amount of interest, in this case loss of Mr. X shall be `2 lakh and loss of Mr. Y shall be `2, lakh., If share of individual, co-owner is not given, it will be considered to be loss of BOI and loss can be, maximum ` 2,00,000., If any house property is owned by a partnership firm or company, it will be considered to be income of, partnership firm or company and not that of partners or shareholders., NOV – 2009 (4 Marks), Mr. X is a co-owner of a house property alongwith his brother., Municipal value of the Property, 1,60,000, Fair Rent, 1,50,000, Standard Rent under the Rent Control Act, 1,70,000, Rent received, 15,000 p.m., The loan for the construction of this property is jointly taken and the interest charged by the bank is `25,000, out of which `21,000 have been paid. Interest on the unpaid interest is `450. To repay this loan, X and his, brother have taken a fresh loan and interest charged on this loan is `5,000., The Municipal taxes of `5,100 have been paid by the tenant.
Page 197 :
Income Under The Head House Property, Mr. X has 50% share in the house property., Mr. X has income from Other Sources `2,60,000., Compute the income from this property chargeable in the hands of Mr. X and tax liability for A.Y., Answer. Computation of income from house property of Mr. X for A.Y. 2022-23, `, Gross Annual Value, Working Note:, (a) Municipal value of property, 1,60,000, (b) Fair rent, 1,50,000, (c) Higher of (a) and (b), 1,60,000, (d) Standard rent, 1,70,000, (e) Annual Letting Value / Expected Rent [lower of (c) and (d)], 1,60,000, (f) Actual rent [15,000 x 12], 1,80,000, (g) Gross Annual Value [higher of (e) and (f)], 1,80,000, Less: Municipal taxes – paid by the tenant, hence not deductible, Net Annual Value (NAV), Less: Standard deduction 30% of NAV u/s 24(a), Less: Interest on housing loan u/s 24(b), Interest on loan taken from bank, Interest on fresh loan to repay old loan for this property, Income under the head house property, 50% share taxable in the hands of Mr. X, Income under the head House Property, Income under the head Other Sources, Gross Total Income, Less: Deduction u/s 80C to 80U, Total Income, Computation of Tax Liability, Tax on `3,08,000 at slab rate, Less: Rebate u/s 87A, Tax Liability, , 197, , 2022-23., `, 1,80,000, , Nil, 1,80,000, (54,000), (25,000), (5,000), 96,000, 48,000, 48,000, 2,60,000, 3,08,000, Nil, 3,08,000, 2,900, (2,900), Nil, , Note: Interest on housing loan is allowable as a deduction under section 24 on accrual basis. Further,, interest on fresh loan taken to repay old loan is also allowable as deduction. However, interest on unpaid, interest is not allowable as deduction under section 24., Question 21: Write a note on owner / deemed owner., Answer: Owner / deemed owner Section 22 / 27, As per section 22, the owner of house property shall be liable to pay income tax and if the title of the, ownership of the property is under dispute in a court of law, the decision as to who will be the owner, chargeable to income-tax under section 22 will be of the Income-tax Department till the court gives its, decision to the suit filed in respect of such property. E.g. Ownership of a house property is disputed among, two brothers Mr. X and Mr. Y and rent is being received by Mr. X, in this case Mr. X shall be considered to, be the owner of the house property till decision is given by the Court and after that amount of tax shall be, adjusted as per court decision., As per section 27, the following persons, are deemed to be the owners., (i) Transfer of a house property to spouse – In case of transfer of house property by an individual to his or, her spouse otherwise than for adequate consideration, the transferor is deemed to be the owner of the, transferred property. In case of transfer to spouse in connection with an agreement to live apart, the, transferor will not be deemed to be the owner. The transferee will be the owner of the house property. In, case of transfer of house property by an individual to his or her minor child, the transferor is deemed to be, the owner of the house property transferred. In case of transfer to a minor married daughter, the transferor is, not deemed to be the owner. E.g. Mr. X has two house property each having income of `10 lakh and Mr. X
Page 198 :
Income Under The Head House Property, , 198, , has gifted one house property to Mrs. X, in this case income from such house property shall be taxable in the, hands of Mr. X but if Mr. X has sold the house property to Mrs. X and has taken full payment, in that case, income from house property shall be taxable in the hands of Mrs. X., (ii) Person in possession of a property – If any person has given possession of house property and has, taken full payment but ownership in documents has not yet been transferred, in such cases the proposed, buyer is the deemed owner and shall be liable to pay income tax and it is called part performance of a, contract of the nature referred to in section 53A of the Transfer of Property Act. E.g. Mr. X has sold his, house property to Mr. Y for `50 lakhs and has taken full payment and possession has been given to Mr. Y, but conveyance deed is not prepared in the name of Mr. Y, in this case Mr. Y is the deemed owner., (iii) Member of a co-operative society etc. – A member of a co-operative society, company or other, association of persons to whom a building or part thereof is allotted or leased under a House Building, Scheme of a society/company/association, shall be deemed to be owner of that building or part thereof, allotted to him although the co-operative society/company/ association is the legal owner of that building., (iv) Holder of an impartible estate – The impartible estate is a property which is not divisible. The holder, of an impartible estate shall be deemed to be the individual owner of all properties comprised in the estate., (v) Person having right in a property for a period not less than 12 years – A person who acquires any, rights in or with respect to any building or part thereof, by virtue of any transaction as is referred to in, section 269UA(f) i.e. transfer by way of lease for not less than 12 years, shall be deemed to be the owner of, that building or part thereof., NOV – 2010 (2 Marks each), , Explain briefly the applicability of section 22 for chargeability of income tax for:, (i) House property situated in foreign country and, (ii) House property with disputed ownership., Answer:, Applicability of section 22 for chargeability of income-tax for –, (i) House property situated in foreign country, A resident and ordinarily assessee is taxable under section 22 in respect of a house property situated in, foreign country. A resident but not ordinarily resident or a non resident is taxable in respect of income from, such property if the income is received in India during the previous year., (ii) House property with disputed ownership, If the title of ownership of the house property is under dispute in a court of law, the decision about who is, the owner lies with the Court but till then income tax shall be required from the person who is the beneficial, owner of the house property., Illustration 29: Mr. Anand sold his residential house property in March, 2021. In June, 2021, he recovered, rent of `10,000 from Mr. Gaurav, to whom he had let out his house for two years from April 2015 to March, 2017. He could not realise two months rent of `20,000 from him and to that extent his actual rent was, reduced while computing income from house property for A.Y.2017-18., Further, he had let out his property from April, 2017 to February, 2021 to Mr. Satish. In April, 2019, he had, increased the rent from `12,000 to `15,000 per month and the same was a subject matter of dispute. In, September, 2021, the matter was finally settled and Mr. Anand received `69,000 as arrears of rent for the, period April 2019 to February, 2021. Would the recovery of unrealised rent and arrears of rent be taxable in, the hands of Mr. Anand, and if so in which year?, Solution:, Since the unrealised rent was recovered in the P.Y.2021-22, the same would be taxable in the A.Y.2022-23, under section 25A, irrespective of the fact that Mr. Anand was not the owner of the house in that year., Further, the arrears of rent was also received in the P.Y.2021-22, and hence the same would be taxable in the, A.Y.2022-23 under section 25A, even though Mr. Anand was not the owner of the house in that year. A, deduction of 30% of unrealised rent recovered and arrears of rent would be allowed while computing, income from house property of Mr. Anand for A.Y.2022-23., Computation of income from house property of Mr. Anand for A.Y.2022-23, Recovery of Unrealised Rent, 10,000
Page 199 :
Income Under The Head House Property, Add: Arrear of Rent Received, Total, Less: Deduction @ 30%, Income under the head House Property, , 199, 69,000, 79,000, (23,700), 55,300, , Illustration 30: Mrs. Rohini Ravi, a citizen of the U.S.A., is a resident and ordinarily resident in India, during the financial year 2020-21. She owns a house property at Los Angeles, U.S.A., which is used as her, residence. The annual value of the house is $20,000. The value of one USD ($) may be taken as ` 65., She took ownership and possession of a flat in Chennai on 1.7.2021, which is used for self-occupation,, while she is in India. The flat was used by her for 7 months only during the year ended 31.3.2022. The, municipal valuation is `32,000 p.m. and the fair rent is `4,20,000 p.a., She paid the following to Corporation of Chennai: Property Tax `16,200 Sewerage Tax `1,800, She had taken a loan from Standard Chartered Bank in June, 2019 for purchasing this flat. Interest on loan, was as under:, Period prior to 1.4.2021, `49,200, 1.4.2021 to 30.6.2021, `50,800, 1.7.2021 to 31.3.2022, `1,31,300, Certificate confirming the amount of Interest has been deposited., She had a house property in Bangalore, which was sold in March, 2017. In respect of this house, she, received arrears of rent of `60,000 in March, 2022. This amount has not been charged to tax earlier., Compute the income chargeable from house property of Mrs. Rohini Ravi for the assessment year 2022-23., Solution: Since the assessee is a resident and ordinarily resident in India, her global income would form part, of her total income i.e., income earned in India as well as outside India will form part of her total income., She possesses a self-occupied house at Los Angeles as well as at Chennai. She can take the benefit of “Nil”, Annual Value in respect of both the house properties. As regards the Bangalore house, arrears of rent will be, chargeable to tax as income from house property in the year of receipt under section 25A., It is not essential that the assessee should continue to be the owner. 30% of the arrears of rent shall be, allowed as deduction. Accordingly, the income from house property of Mrs. Rohini Ravi will be calculated, as under:, Self - occupied house at Los Angeles, Gross Annual Value, Nil, Less: Municipal taxes, Nil, Net Annual Value, Nil, Less: Statutory deduction under section 24(a) @ 30% of NAV, Nil, Less: Interest on Housing Loan u/s 24(b), Nil, Loss from House property, Nil, Self - occupied property at Chennai, Gross Annual Value, Nil, Less: Municipal taxes, Nil, Net Annual Value, Nil, Less: Statutory deduction under section 24(a) @ 30% of NAV, Nil, Less: Interest on Housing Loan u/s 24(b) (Note), (1,91,940), Loss from House property, (1,91,940), Arrears in respect of Bangalore Property (Section 25A), Arrears of rent received, 60,000.00, Less: Deduction under section 25A @ 30%, (18,000.00), Income from House property, 42,000.00, Loss under the head House property (1,91,940 - 42,000), 1,49,940.00, Note: Calculation of Interest, Interest for the current year (50,800+1,31,300), 1,82,100.00, Add: 1/5th of Prior period interest (49,200 x 1/5), 9,840.00, Interest deduction allowed u/s 24 (b), 1,91,940.00
Page 200 :
Income Under The Head House Property, , 200, , MULTIPLE CHOICE QUESTIONS, 1. Vacant site lease rent is taxable as, (a) Income from house property, (b) Business income or income from house property, as the case may be, (c) Income from other sources or business income, as the case may be, (d) Income from other sources or income from house property, as the case may be, 2. Treatment of unrealized rent for determining income from house property, (a) To be deducted from expected rent, (b) To be deducted from actual rent, (c) To be deducted under section 24 from annual value, (d) To be deducted from both expected rent and actual rent, 3. Municipal taxes to be deducted from GAV should be, (a) Paid by the tenant during the previous year, (b) Paid by the owner during the previous year, (c) Accrued during the previous year, (d) Paid during the previous year either by tenant or owner, 4. Deduction under section 24(a) is, (a) 1/3rd of NAV, (b) repairs actually incurred by the owner, (c) 30% of NAV, (d) Interest on borrowed capital, 5. Interest on borrowed capital accrued up to the end of the previous year prior to the year of, completion of construction is, (a) allowed as a deduction in the year of completion of construction, (b) allowed in 5 equal annual installments from the year of completion of construction, (c) allowed in the respective year in which the interest accrues, (d) not allowed, 6. The ceiling limit of deduction under section 24(b) in respect of interest on loan taken on 1.4.2019 for, repairs of a self-occupied house is, (a) `30,000 p.a., (b) `1,50,000 p.a., (c) `2,00,000 p.a., (d) No limit, 7. Where an assessee has two house properties for self-occupation, the benefit of nil annual value will, be available in respect of (a) Both the properties, (b) The property which has been acquired/constructed first, (c) Any one of the properties, at the option of the assessee, (d) Any one of the properties and once option is exercised cannot be changed in subsequent years, 8. Leena received `30,000 as arrears of rent during the P.Y. 2021-22. The amount taxable under, section 25A would be (a) `30,000, (b) `21,000, (c) `20,000, (d) `15,000, 9. Vidya received `90,000 in May, 2021 towards recovery of unrealised rent, which was deducted from, actual rent during the P.Y. 2019-20 for determining annual value. Legal expense incurred in relation, to unrealized rent is `20,000. The amount taxable under section 25A for A.Y.2022-23 would be (a) `70,000, (b) `63,000, (c) `60,000, (d) `49,000
Page 201 :
Income Under The Head House Property, , 201, , 10. Ganesh and Rajesh are co-owners of a self-occupied property. They own 50% share each. The, interest paid by each co-owner during the previous year on loan (taken for acquisition of property, during the year 2004) is `2,05,000. The amount of allowable deduction in respect of each co-owner is –, (a) `2,05,000, (b) `1,02,500, (c) `2,00,000, (d) `1,00,000, 11. In respect of loss from house property, which of the following statements are correct?, (a) While computing income from any house property, the maximum interest deduction allowable under, section 24 is `2 lakhs, (b) Loss from house property relating to a particular year can be set-off against income under any other head, during that year only to the extent of `2 lakhs, (c) The loss in excess of `2 lakh, which is not set-off during the year, can be carried forward for set-off, against any head of income in the succeeding year(s), (d) All of the above, 12. Mr. X purchased in October, 2021, a flat in Chennai, to be used for his own residential purposes, with the financial assistance of housing loan taken from PNB Housing Finance Ltd. He has paid, interest on such loan till March, 2022 of `1,78,780. The amount of interest paid on such loan allowed, u/s 24 is (all conditions for claiming higher interest have been complied with):, (a) `1,25,000, (b) `1,78,780, (c) `1,50,000, (d) None of the above, 13. Mr. Zen owns a flat in Mumbai which was let out by him in the previous year 2021 – 22 on a rent, of `20,000 p.m. upto December, 2021 and for `30,000 p.m. thereafter. The annual municipal value is, of `3,00,000, Fair Rent is `2,50,000 and Standard Rent is `2,90,000. The Gross Annual Value of the, flat shall be taken as:, (a) ` 2,70,000, (b) ` 3,00,000, (c) ` 2,50,000, (d) ` 2,90,000, 14. In case of inter source adjustment the loss derived from a house property can be set off during the, year against:, (a) the income of any other house property, (b) the capital gain, (c) Business/ Profession, (d) (b) and (c) above, 15. Mr. Ahmed acquired a property in April, 2021 for self-residential use. The loan interest payable to, State Bank of India for the financial year 2021-22 amounts to ` 2,10,000. The amount eligible for, deduction under section 24 is (all conditions for claiming higher interest have been complied with):, (a) ` 30,000, (b) ` 2,00,000, (c) ` 2,10,000, (d) ` 1,50,000, 16. A borrowed `5,00,000 @ 12% p.a. on 1-4-2017 for construction of house property which was, completed on 15-3-2021. The amount is still unpaid. The deduction of interest for previous year 202122 shall be :, (a) `60,000, (b) `96,000, (c) `1,80,000, (d) `2,40,000
Page 202 :
Income Under The Head House Property, , 202, , 17. Mr. Shahu has loss from house property for the assessment year 2022-23. He can carry forward, such loss for subsequent ............... assessment years., (a) 4, (b) Nil, (c) 8, (d) Indefinite, 18. Ms. Padmaja let out a property for `20,000 per month during the year 2021-22. The municipal tax, on the let-out property was enhanced retrospectively. Hence, she paid `60,000 as municipal tax which, included arrears of municipal tax of `45,000. Her income from house property is —, (a) `1,80,000, (b) `1,57,500, (c) `1,26,000, (d) `1,36,500, 19. The construction of a house was completed on 31st January, 2022. The owner of the house took a, loan of `20,00,000 @ 6% p.a. on 1st May, 2021. In this case the deduction allowable for the previous, year 2021-22 towards interest on borrowings is —, (a) 22,000, (b) 24,000, (c) 1,10,000, (d) None of the above., 20. The brought forward loss from a house property can be set off during the year against:, (a) the income of any other house property, (b) the capital gain, (c) the income under other sources, (d) all the above, 21. The current year loss under the head house property can be set off from other heads maximum, upto_____________., (a) ` 1,50,000, (b) ` 2,00,000, (c) ` 2,50,000, (d) ` 3,00,000, 22. Standard Deduction u/s 24(a) shall be, (a) 25% of NAV, (b) 30% of NAV, (c) 25% of GAV, (d) 30% of GAV, 23. GAV shall be, (a) Higher of expected rent and rent received/receivable, (b) Lower of expected rent and rent received/receivable, (c) Higher of municipal value and fair rent, (d) NAV minus municipal taxes, 24. Expected rent shall be, (a) Higher of municipal value and fair rent but restricted to Standard rent, (a) Lower of municipal value and fair rent but maximum to Standard rent, (c) Higher of municipal value and fair rent, (d) Lower of municipal value and fair rent, 25. Prior Period Interest shall be allowed in, (a) 5 annual equal installments, (a) 4 annual equal installments, (c) 3 annual equal installments, (d) 2 annual equal installments
Page 203 :
Income Under The Head House Property, , 203, , 26. The Ceiling limit of deduction u/s 24(b) in respect of interest on loan taken for let out property, shall be, (a) ` 30,000 p.a., (a) ` 1,50,000 p.a., (c) ` 2,00,000 p.a., (d) No limit, 27. Recovery of unrealized rent shall be taxable under the head, (a) House Property, (b) Business/Profession, (c) Other sources the, (d) None of the above, 28. If any person is engaged in the business of letting out of house property, income shall be taxable, under the head, (a) Business/profession, (b) House property, (c) Other sources, (d) Capital gains, (e) None of these, 29. Loss from house property is not allowed to be set off from, (a) Income under the head Salary, (b) Income under the head Business/Profession, (c) Income under the head Capital Gains, (d) Casual Income, 30. If loan has been taken on 01.04.2020 and house was completed on 31.03.2021, in this case, (a) no prior period installment in p.y. 2021-22, (b) there is prior period installment in p.y. 2021-22, , Check Solution given on our website www.mkgeducation.com
Page 204 :
Income Under The Head House Property, , 204, , PRACTICE PROBLEMS, TOTAL PROBLEMS 30, Problem 1:, Mr. X has let out one building @ ` 90,000 p.m. and fair rent is ` 80,000 p.m. standard rent ` 1,00,000 p.m., Municipal valuation ` 81,000 p.m., Municipal Tax paid ` 70,000 p.a., Interest on loan for construction of, house property ` 82,000., Compute his tax liability for assessment year 2022-23., Answer: Tax Liability: `39,000, Problem 2:, X Ltd. has let out one building to ABC Ltd. @ `3,00,000 p.m. and X Ltd. has paid municipal tax of, `6,00,000 p.a. X Ltd. has paid interest of `3,00,000 on loan taken for construction of building. Fair rent of, the building is `2,50,000 p.m. and Municipal Valuation is `2,75,000 p.m. and Standard Rent is `2,80,000, p.m., Compute Income Tax Liability for assessment year 2022-23., Answer: Income Tax Liability: `5,61,600, Problem 3:, XYZ Ltd. has let out one building to ABC Ltd. @ `2,00,000 p.m. Fair rent is `1,80,000 p.m. and standard, rent `2,20,000 p.m. The company paid municipal tax of `6,00,000 during the year., Compute income tax Liability of XYZ Ltd., Answer: Income Tax Liability: `3,93,120, Problem 4:, Mr. X has let out one house at a rent of `50,000 p.m. Fair rent `55,000 p.m. Municipal Valuation `52,000, p.m., standard rent `60,000 p.m.. The house remain vacant for 3 months. The assessee paid municipal tax, `30,000. Interest on loan u/s 24(b) is `20,000.Compute Income and Tax Liability A.Y. 2022-23., Answer: Income Tax Liability: Nil, (b) Presume it is let out at a rent of `60,000 P.m., Answer: Income Tax Liability: Nil, (c) Presume it is let out at a rent of `55,000 P.m., Answer: Income Tax Liability: Nil, (d) Presume it is let out at a rent of `1,00,000 P.m., Answer: Income Tax Liability: `31,510, Problem 5:, Mr. X has let out one house at a rent of `15,00,000 p.m. Fair rent `15,50,000 p.m. Municipal Valuation, `15,20,000 p.m., standard rent `16,00,000 p.m.. The house remain vacant for 3 months. The assessee paid, municipal tax `13,00,000. Interest on loan u/s 24(b) is `12,00,000.Compute Income and Tax Liability A.Y., 2022-23., Answer: Income Tax Liability: `36,90,260, (b) Presume it is let out at a rent of `16,00,000 P.m., Answer: Income Tax Liability: `25,20,800, (c) Presume it is let out at a rent of `15,60,000 P.m., Answer: Income Tax Liability: `24,34,320, (d) Presume it is let out at a rent of `20,00,000 P.m., Answer: Income Tax Liability: `35,39,560, Problem 6., Mr. X has loss under the head house property `3,00,000 and LTCG `4,00,000 and casual income `1,00,000, Compute his income and tax liability for A.Y. 2022-23., Answer: Income Tax Liability: `18,200
Page 205 :
Income Under The Head House Property, , 205, , Problem 7., Mr. X has income under the head house property `3,00,000 and LTCG `4,00,000 and casual income `, 1,00,000. He has loss under the head house property `4,00,000 of P.Y. 2014-15 and it could not be adjusted., Compute his income and tax liability for A.Y. 2022-23., Answer: Income Tax Liability: `49,400, Problem 8., Mr. X has income under the head house property `3,00,000 and income under the head business/profession, `5,00,000 and LTCG `4,00,000 and casual income `1,00,000. He has loss under the head house property `, 4,00,000 of P.Y. 2014-15 and it could not be adjusted. Compute his income and tax liability for A.Y. 202223., Answer: Income Tax Liability: `1,27,400, Problem 9., Mr. X has income from house - 1 `8,00,000 and loss from house - 2 `3,00,000 and STCG 111A ` 1,50,000, and casual income `1,00,000 and he received Dividend of `2,00,000 from Domestic Company and he has, Agriculture Income from India `2,00,000. He has loss under the head house property `4,00,000 of P.Y., 2014-15 and it could not be adjusted. Compute his income and tax liability for A.Y. 2022-23., Answer: Income Tax Liability: `57,200, Problem 10., Compute gross annual value in the following cases for the assessment year 2022-23:, Particulars, Situation 1, Situation 2, Situation 3, Situation 4, Fair Rent (p.m.), 10,000, 12,000, 13,000, 15,000, Municipal Valuation (p.m.), 11,000, 10,000, 8,000, 17,000, Standard Rent (p.m.), 12,000, 11,000, 7,000, 16,000, Rent received/ receivable (p.m.), 7,000, 11,500, 20,000, 16,000, Vacancy, 2 months, 1 month, 3 month, Unrealised rent, 1 month, 3 month, 1 month, Answer = Gross Annual Value: Situation 1: `1,32,000; Situation 2: `1,15,000; Situation 3: `1,60,000;, Situation 4: `1,92,000, Problem 11., Mr. X has let out one house property @ `70000 per month and there is unrealised Rent of 2 months and, there is vacancy of 3 month. Fair rent `60,000 per month, municipal valuation `55,000 per month and, standard rent `80,000 per month. Municipal tax paid `62,000. Interest on loan for construction of the house, property is `75,000.The assessee has unrealised Rent of `2,00,000 in P.Y. 2018-19 and he has recovered, `1,50,000 in P.Y. 2021-22 and interest of `18,000 and he has incurred `11,000 as legal expense., Compute his tax liability for assessment year 2022-23., Answer: Tax Liability: `14,790, Problem 12., Mr. X (non-resident) has one house with fair rent `20,000 p.m., municipal valuation `10,000 p.m., standard, rent `18,000 p.m. It was let out for `12,000 p.m. but it remains vacant for 1½ months and there was, unrealised rent for 2 months. Municipal taxes paid are `11,000 and interest on capital borrowed for, construction of the house is `3,00,000., Mr. X has income under the head other sources `7,00,000., Compute his total income and tax liability for the assessment year 2022-23., Answer = Total Income: `5,43,500; Tax Liability: `22,050
Page 206 :
Income Under The Head House Property, , 206, , Problem 13., Mrs. X has taken a loan of ` 11,00,000 on 01.07.2015 at a rate of 10% per annum from SBI for construction, of one house which was completed on 31.03.2017 and the house was let out at a rate of `80,000 per month, w.e.f. 01.11.2020 and fair rent is `1,00,000 per month. Municipal taxes paid in previous year 2021-2022, `30,000. She has taken a fresh loan of `11,00,000 on 01.07.2020 @ 11% per annum and it was utilized to, repay the original amount., Compute her income tax liability for assessment year 2022-23., Answer: Income Tax Liability: `54,180, Problem 14., Mr. X took a loan of ` 6,10,500 @ 7% p.a. on 01.09.2018 from his friend for construction of one house, which was completed on 01.06.2021 and it was let out @ `9,000 p.m. It remained vacant for 1½ month and, there is unrealised rent of `1,000. The fair rent of house is `10,000 p.m. Assessee has repaid half of the loan, amount on 01.07.2020 and remaining amount on 01.02.2022. He has also paid municipal tax of `3,000. His, income under the head salary `2,65,000., Compute his total income and tax liability for the assessment year 2022-23., Answer = Total Income: `2,96,220; Tax Liability: Nil, Problem 15., Mr. X has taken a loan on 01.07.2018 from SBI @ 11% p.a. of `15,00,000 for construction of one house, which was completed on 01.11.2020 and was self occupied and municipal taxes paid in previous year, 2021-22 `32,000. He has given repayment of loan of `70,000 on 01.01.2022. He has submitted a certificate, confirming the amount of interest., He has income under the head Salary `6,00,000, Compute income tax liability for assessment year 2022-23., Answer: Tax Liability: Nil, Problem 16., Mrs. X has taken a loan on 01.11.2017 from PNB @ 10% p.a. of `10,00,000 for purchase of one house, which was purchased on 01.01.2018 and was self occupied and municipal taxes paid in previous year 20212022 `30,000. She has repaid the loan amount in annual installments of `50,000 starting from 01.01.2019., The house was vacant for 1 month in previous year 2021-22. She has submitted a certificate confirming the, amount of interest., She has short term capital gains under section 111A `10,00,000., Compute Income Tax Liability for assessment year 2022-23., Answer: Tax Liability: `1,03,940, Problem 17., Mr. X has taken a loan of `15,00,000 from State Bank on 01.07.2019 @ 10% p.a. and the residential house, was completed on 01.05.2021 and was let out w.e.f. 01.06.2021 @ 80,000 p.m. and fair rent of the house is, `90,000 p.m., He repaid half of the loan amount on 01.01.2022., Compute his Income Tax Liability for assessment year 2022-23., Answer = Total Income: `3,59,250; Tax Liability: Nil, Problem 18., Mr. X has taken a loan of `11,00,000 on 01.07.2018 @ 10% p.a. from his friend for construction of one, house which was completed on 01.09.2020 and the house is self occupied during the previous year 2021-22, and Mr. X has paid municipal tax of `12,000., The assessee has submitted a certificate confirming the amount of interest. Mr. X has short term capital, gains under section 111A `120 lakhs., Compute his income and Tax Liability for the assessment year 2022-23., Answer: Total Income: ` 118,51,500; Tax Liability: `20,81,310
Page 207 :
Income Under The Head House Property, , 207, , Problem 19., Mr. X has 2 houses. First is self occupied with fair rent `20,000 p.a., municipal valuation is `55,000 p.a.., Fair rent as per Rent Control Act is `50,000 p.a.. However the house remains vacant for 2 months Architect, has issued completion certificate on 01.07.2019. Mr. X has taken loan for addition to house `3,50,000 on, 01.04.2021 @ 13% p.a. The loan was repaid on 01.03.2022 and assessee has submitted a certificate from the, person from whom he has taken the loan certifying that the amount of interest claimed by Mr. X is correct., In the earlier years, the house was let out and the assessee has recovered unrealised rent of `2,000 in the, previous year 2021-22. The assessee has also incurred legal expenses of `350., The second house is also self-occupied. However its similar building rent is `64,000 p.a. and rent, determined by municipality for charging house tax is `66,000 p.a. Its standard rent is `6,000 p.m. municipal, tax payable are `5,000., He has long term capital gains `20,00,000., Compute his income tax liability for Assessment Year 2022-23., Answer = Income Tax Liability: `3,58,050, Problem 20., Mr. X has let out one house @ `45,000 p.m., but this house was vacated on 01.11.2021. The house was self, occupied w.e.f. 01.01.2022. Fair rent of this house is `50,000 p.m., municipal valuation is `47,000 p.m. and, standard rent is `48,000 p.m. The assessee has paid municipal taxes @ 10% of municipal valuation. Interest, on capital borrowed is `42,000. Land revenue paid by the assessee is `11,000 and ground rent paid by him is, `3,000. The assessee has taken a loan for payment of municipal tax and interest paid on loan is `500., Compute his income under the head house property and tax liability for assessment year 2022-23., Answer = Income under the head House Property: `3,21,720; Tax Liability: Nil, Problem 21., Mr. X has two houses one of which is self occupied throughout the year. Its fair rent is `10,000 p.m.,, municipal valuation `11,000 p.m. and standard rent is `10,500 p.m. Municipal taxes paid are `6,000 and, interest on capital borrowed is `41,000. The assessee has taken the loan for construction of the house on, 01.04.1998., Second house is self occupied for 4 months and let out for 8 months @ of `45,000 p.m. Its fair rent is, `20,000 p.m., municipal valuation is `18,000 p.m. and standard rent `15,000 p.m. Municipal taxes paid are, `20,000 and interest on capital borrowed is `45,000. The assessee has taken the loan for construction of the, house on 01.04.1998., Compute his income under the head house property for the assessment year 2022-23., Answer = Income under the head House Property: ` 1,63,000, Problem 22., Mr. X has let out one showroom building in Pitam Pura @ 1,00,000 p.m. and has paid municipal tax, `85,000 and fair rent of the house is `98,000 p.m., He has received arrears of rent `3,00,000 relating to the previous year 2020-21., He has also received unrealized rent of `4,00,000 of previous year 2019-20 and also interest of `20,000 on, such unrealised rent and he has paid `27,000 to the advocate in connection with recovery of unrealized rent., Compute his income tax liability for assessment year 2022-23., Answer: Tax Liability: `2,07,640, Problem 23., Mr. X occupied two flats for his residential purposes, particulars of which are as follows:, Particulars, Flat I, Flat II, (in `), (in `), Municipal Valuation, 95,000 p.a., 50,000 p.a., Fair Rent, 1,25,000 p.a., 45,000 p.a., Fair Rent under Rent Control Act, 85,000 p.a., Not available
Page 208 :
Income Under The Head House Property, , 208, , Municipal taxes paid, 10%, 10%, Fire Insurance paid, 1,500, 650, Ground rent due, 700, 900, Land revenue paid, 600, 800, Interest payable on capital borrowed for purchase of flat, 45,000, Nil, Income of Mr. X from his proprietary business–warehousing corporation is `7,00,000. Determine the total, income and tax liability for the assessment year 2022-23, you are informed that Mr. X could not occupy flat, for 2 months commencing from December 1st, 2021 and that he has attained the age of 82 on 23.08.2021., Answer = Total Income: `7,01,500; Tax Liability: `35,360, Problem 24., Mr. X and Mr. Y constructed their houses on a piece of land purchased by them at New Delhi. The built up, area of each house was 1,000 sq. ft. ground floor and an equal area at the first floor., Mr. X started construction of the house on 01.04.2020 and completed it on 31.03.2021. Mr. X occupied the, entire house on 01.04.2021. Mr. X has availed a housing loan of `25 lakhs @ 12% p.a. on 01.04.2020 and, has also submitted a certificate from the lender certifying the amount of interest., Mr. Y started construction on 01.04.2020 and completed it on 30.06.2021. Mr. Y occupied the ground floor, on 01.07.2021 and let out the first floor for a rent of `20,000 per month. However, the tenant vacated the, house on 31.12.2021 and Mr. Y occupied the entire house during the period 01.01.2022 to 31.03.2022. Mr., Y has availed a housing loan of `15 lakhs @ 10% p.a. on 01.07.2020 and has also submitted a certificate, from the lender certifying the amount of interest., Following are the other information:, `, (i) Fair rental value of each unit, 1,20,000 Per annum, (Ground floor / first floor), (ii) Municipal value of each unit, 92,000 Per annum, (Ground floor / first floor), (iii) Municipal taxes paid by, X, - 10,000, Y, - 10,000, (iv) Repair and maintenance charges paid by, X, - 30,000, Y, - 32,000, No repayment was made by either of them till 31.03.2022. Compute income from house property for Mr. X, and Mr. Y for the previous year 2021-22 (assessment year 2022-23)., Answer = Mr. X: ` (2,00,000); Mr. Y: ` (92,000), Problem 25., Mrs. X is the owner of a house property. She borrowed `60,000 from life insurance corporation of India on, 1st September 2014 @ 15% p.a. for the construction of this house. The construction was completed on, 31.03.2017. Since then the house is under her self-occupation. On 1st June 2021 the house was let out @, `3,000 p.m. The tenant vacated the house on 1st August 2021. She occupied the house for self-occupancy., The house is again let out @ `3,500 p.m. from 1st October 2021., Other particulars of the house for the previous year 2021-22., `, Municipal Valuation, 22,000 p.a., Municipal taxes disputed, hence not paid, 2,200 p.a., Ground rent for the previous year 2021-22 outstanding, 3,200, Insurance premium paid, 1,200, Refund of first loan instalment to LIC on 01.10.2021, 15,000, Compute the income from house property for assessment year 2022-23., Answer = Income under the head House Property: `11,025, Problem 26., Mr. X owns a residential house property. It has two identical units—unit I and unit II. Unit I is self–occupied, by Mr. X and his family members, unit II is let out (rent being `7,500 per month, this unit remained vacant, for one month during which it was self-occupied). Municipal value of the property is `1,30,000. Standard
Page 209 :
Income Under The Head House Property, , 209, , rent is `1,40,000 and fair rent is `1,53,000. Municipal taxes is imposed @ 12% (on municipal value) which, is paid by Mr. X. Other expenses for the previous year 2021-22 being repairs `5,100 and insurance `6,300., Mr. X borrowed `9,00,000 on 01.07.2018 from LIC @ 12% p.a. to construct the property. Construction of, the house was completed on 30.06.2020. The entire loan is still unpaid., Compute the total income and tax liability of Mr. X for the assessment year 2022-23 on the assumption that, income of Mr. X from other sources is `3,90,000., Answer = Total Income: `3,39,390; Tax Liability: Nil, Problem 27., Mr. X has a house property situated in Mumbai which has two units. Unit I has a floor area of 70% whereas, the unit II has a floor area of 30%. Both the units were self-occupied by the assessee. As the assessee was, allowed a rent free accommodation by his employer w.e.f. 01.04.2021, he vacated both of the units and let, out unit I at a rent of `13,000 p.m. and unit II for `5,000 p.m. unit I remained vacant for 1½ months whereas, unit II was vacant for one month. Other particulars of the house property are asunder:, `, Municipal Valuation, 1,55,000, Fair Rent, 1,75,000, Standard Rent, 1,65,000, Municipal taxes paid, 35,000, Ground rent due, 15,000, Compute income from house property for the assessment year 2022-23., Answer = Income under the head House Property: `1,09,550, Problem 28., Mr. X is the owner of a residential house whose construction was completed on 31.08.2017. It has been let, out from 01.01.2018 for residential purposes. Its particulars for the financial year 2021-22 are given below:, `, (i) Municipal Valuation (p.a.), 68,000, (ii) Expected Fair Rent (p.a.), 75,000, (iii) Standard Rent under the Rent Control Act (p.m.), 7,200, (iv) Actual Rent (p.m.), 7,200, (v) Municipal taxes paid (including `7,000 paid by tenant), 21,000, (vi) Water/sewerage benefit tax, levied by State Government paid under protest, 5,100, (vii) Interest on loan taken for the construction of the house. The interest has been paid, outside India to a non-resident without deduction of tax at source, 20,000, (viii) Stamp duty and registration charges incurred in respect of the lease agreement of the house, 2,500, (ix) The unrealised rent for previous year 2020-21 amounts to `42,000., There is recovery of `22,000 from the defaulting tenant., Legal charges for the recovery of rent, 4,500, Compute income from house property for the assessment year 2022-23., Answer = Income under the head House Property: `66,080, Problem 29., Mr. X has three houses with details given below:, House I, It is self occupied with fair rent of `20,000, municipal valuation `55,000, rent as per Rent Control Act is, `50,000. However the house remains vacant for 2 months. Architect has issued completion certificate on, 01.07.2019. Loan taken for addition to the house `5,00,000 on 01.04.2021 @ 13% p.a. and loan amount was, repaid on 01.03.2022. The assessee has submitted a certificate from the person from whom he has taken the, loan certifying the amount of the interest claimed., In the earlier years the house was let out and the assessee has recovered unrealised rent of `2,000 in the, previous year 2021-22 and interest on such unrealised rent also amounting to `250. However the assessee, has incurred legal expenses of `350.
Page 210 :
Income Under The Head House Property, , 210, , House II, It is self occupied. Its similar building rent is `64,000 and rent determined by municipality for charging, house tax is `66,000 and its fair rent under Rent Control Act (p.m.) is `6,000. Municipal taxes payable, `5,000., The assessee has also recovered unrealised rent of `2,000 in the previous year 2019-20 but the expenses, thereon are paid in the year 2021-22 amounting to `200., House III, It is let out @ `50,000 p.m. and fair rent is `60,000 p.m. Water tax and house tax paid to municipality is, `11,000. Insurance premium paid `6,500 and expenses on repairs `3,000., Interest on capital borrowed for purchase of house is `55,000., He has long term capital gains of `3,50,000., Compute his total income and tax liability for assessment year 2022-23., Answer = Total Income: `7,62,950; Tax Liability: `81,270, Problem 30., Determine the income head under which the following incomes shall be taxable., (i), Mr. X has income from letting out house property., (ii), Mr. X has sold one house property., (iii) ABC Ltd. has 500 flats for the purpose of sale/purchase., (iv), Mr. X has let out an open land., (v), ABC Ltd. has 500 flats for the purpose of letting out and ABC Ltd. is engaged in the business, of letting out., (vi), ABC Ltd. has constructed flats within its premises for letting out to the employees., (vii) Mr. X is engaged in the business of providing paying guest accommodation in his own, building., (viii) Mr. X is engaged in the business of warehousing., (ix), Mr. X has sublet one house property., (x), Mr. X has let out his hotel building., Answer = (i) House Property; (ii) Capital Gains; (iii) Business/Profession; (iv) Other Sources; (v), Business/Profession; (vi) Business/Profession; (vii) Business/Profession; (viii) Business/Profession; (ix), Other Sources; (x) House Property, , Check Detailed Solution given on our website www.mkgeducation.com
Page 211 :
Income Under The Head House Property, , 211, , EXAMINATION QUESTIONS, , Solutions of All Examination Questions are given on, our website www.mkgeducation.com, NOV – 2018 (NEW COURSE), Question 3(a), Marks 7, Mrs. Disha Khanna, a resident of India, owns a house property at Bhiwani in Haryana. The Municipal, value of the property is `7,50,000, Fair Rent of the property is `6,30,000 and Standard Rent is `7,20,000, per annum., The property was let out for `75,000 per month for the period April 2021 to December 2021., Thereafter, the tenant vacated the property and Mrs. Disha Khanna used the house for self-occupation. Rent, for the months of November and December 2021 could not be realize from the tenant. The tenancy was, bonafide but the defaulting tenant was in occupation of another property of the assessee, paying rent, regularly., She paid municipal taxes @ 12% during the year and paid interest of `35,000 during the year for amount, borrowed towards repairs of the house property., You are required to compute her income from "House Property” for the A.Y. 2022-23., , NOV – 2018 (OLD COURSE), Question 2(b), Marks 5, Mr. Chakrobarty, a resident, aged 35 years, works as a deputy manager in Dews Limited, located in Noida, since April 2014. He own two houses and uses it for self-purpose. The following information relates to the, houses for the previous year 2021-2022:, Location, House-1, House-II, Noida, Gurgaon, (He and his family reside), (His parents reside), 8,00,000, 9,00,000, Municipal value per annum (`), 9,20,000, 8,80,000, Fair rent (`), 8,40,000, 9,20,000, Standard rent per annum (`), Actual rent (per month) (`), Municipal taxes paid during the year, 8%, 10%, Date of completion of construction of properties, 31st March, 2014, 25th May, 2020, He had taken a loan of `18,00,000 for the construction of the House-II on 1st April, 2018. Interest was payable, @ 10% per annum. Certificate confirming interest has been submitted. Till date, no payment was made, towards the principal amount., Compute his income under the head House Property, , MAY – 2018 (OLD COURSE), Question 3 (b), Marks 5, Mr. Rohan a residential individual, owns 3 houses in Chennai. One house is self-occupied by him, second, house is self-occupied by his major son and the third house is vacant during the year., You are required to highlight the steps involved to compute Income from House Property for Mr. Rohan, under deemed to be let out concept.
Page 212 :
Income Under The Head House Property, , 212, , NOV – 2017, Question 4(a), (Marks 5), Mr. Aditya, a resident but not ordinarily resident in India during the Assessment Year 2022-23. He owns, two houses, one in Dubai and the other in Mumbai. The house in Dubai is let out there at a rent of DHS, 20,000 p.m. (1DIRHAM=INR 18). The entire rent is received in India. He paid Property tax of DHS 2,500, and Sewerage Tax DHS 1,500 there, for the Financial Year 2021-22. The house in Mumbai is selfoccupied., He has taken a loan of `25,00,000 to construct the house on 1st June, 2018 @ 12%. The construction was, completed on 31st May, 2020 and he occupied the house on 1st June, 2020. The entire loan is outstanding as, on 31st March, 2022. Property tax paid in respect of the second house is `2,400 for the Financial Year, 2021-22. Compute the income chargeable under the head “Income from House property” in the hands of, Mr. Aditya for the Assessment Year 2022-23., , MAY – 2017, Question 4(a) (ii), (4 Marks), Mr. Ganesh owns a commercial building whose construction got completed in June 2020. He took a loan of, `15 lakhs from his friend on 1-8-2019 and had been paying interest calculated at 15% per annum. He is, eligible for pre-construction interest as deduction as per the provisions of the Income Tax Act., Mr. Ganesh has let out the commercial building at a monthly rent of `40,000 during the financial year 202122. He paid municipal tax of `18,000 each for the financial year 2020-21 and 2021-22 on 1-5-2021 and 5-42022 respectively., Compute income under the head. 'House Property' of Mr. Ganesh for the Assessment Year 2022-23., , NOV – 2015, Question 6(a)., (8 Marks), Mr. X constructed a shopping complex. He had taken a loan of `25 lakhs for construction of the said, property on 01.08.2019 from SBI @ 10% for 5 years. The construction was completed on 30.06.2020., Rental income received from shopping complex `30,000 per month let out for the whole year. Municipal, Taxes paid for shopping complex `8,000., Arrears of rent received from shopping complex `1,20,000., Interest paid on loan taken from SBI for purchase of house for use as own residence for the period 20212022 `3 lakhs. The loan was taken after 01.04.1999 and house was purchased within 5 years from the end of, the year in which loan was taken and assessee has submitted certificate certifying the amount of interest., You are required to compute Income from House property of Mr. X for AY 2022-2023 as per Income Tax, Act, 1961.
Page 213 :
Deduction From Gross Total Income, , 213, , DEDUCTION FROM GROSS, TOTAL INCOME, CHAPTER VI-A, SECTION 80C TO 80U, PARTICULARS, , SECTIONS, , 1. Deduction in respect of investment in NSC/PPF/Bank FD etc., 2. Deduction in respect of contribution to certain pension funds, 3. Deduction in respect of medical insurance premia, 4. Deduction in respect of maintenance including medical treatment of a, dependent who is a person with disability, 5. Deduction in respect of medical treatment, etc., 6. Deduction in respect of interest on loan taken for higher education, 7. Deduction in respect of interest on loan taken for certain house property, , 80C, 80CCC, 80D, 80DD, , 8. Deduction in respect of purchase of electric vehicle., , 80DDB, 80E, 80EEA, 80EEB, 80G, 80GG, 80GGA, , 9. Deduction in respect of donations to certain funds, charitable institutions, etc., 10. Deduction in respect of rents paid, 11. Deduction in respect of certain donations for scientific research or rural, development, 12. Deduction in case of donation to political parties, 80GGB/80GGC, 13. Deduction in respect of employment of new workmen, 80JJAA, 14. Deduction in respect of royalty income, etc., of authors of certain books, 80QQB, 15. Deduction in respect of royalty on patents, 80RRB, 16. Deduction from interest on savings bank account, 80TTA, 17. Deduction from interest on savings bank account to Senior Citizens, 80TTB, 18. Deduction in case of disabled person, 80U
Page 214 :
Deduction From Gross Total Income, , 214, , DEDUCTION FROM GROSS TOTAL INCOME, CHAPTER VI-A, SECTION 80C to 80U, Deductions under section 80C to 80U are allowed from gross total income to compute total income however, such deduction is allowed only from normal income., As per section 112, such deductions are not allowed from long term capital gains., As per section 58(4), such deductions are not allowed from casual income., As per section 111A, such deduction are not allowed from short term capital gains on the sale of, short term equity shares or short term units of equity oriented mutual funds provided securities, transaction tax has been paid., As per section 112A, such deduction are not allowed from long term capital gains on the sale of long, term equity shares or long term units of equity oriented mutual funds provided securities transaction, tax has been paid., Example, Mr. X has income under the head salary `75,000, income from long term capital gains `2,10,000 and casual, income `35,000, in this case maximum amount of deductions allowed shall be `75,000., Question 1 [V. Imp.]. Write a note on deduction under Section 80C., Answer: Deduction under section 80C shall be allowed only to, (i), an individual, (ii), Hindu Undivided Family, (Deduction under section 80C is not allowed to any partnership firm or a company etc.), Deduction shall be allowed to the extent of the following investments but as per section 80CCE, maximum, deduction allowed shall be `1,50,000 (Including deduction under section 80CCC and section 80CCD(1))., 1. Deduction shall be allowed if amount has been invested in National Saving Certificate (NSC) and NSC, are just like a fixed deposit with a bank. Amount can be invested in the name of self, spouse or minor, children and HUF can invest the amount in the name of any of its members. Deduction shall be allowed, equal to the amount invested and amount received on maturity shall be exempt from income tax but, interest shall be taxable every year on accrual basis but payment of interest shall be received on maturity., Deduction under section 80C shall also be allowed for such accrued interest but no deduction shall be, allowed for accrued interest of the year in which assessee has received payment. NSC are issued for 5, years., Example, Mr. X has income under the head House Property `10 lakh and he invested `50,000 in NSC on, 01.10.2021. He has invested `40,000 in previous year 2020-21 also and there is accrued interest of, `4,000 in previous year 2021-22. He has also received `1,00,000 on maturity of NSC which were, invested in the earlier year and original amount is `60,000 and interest for current year is `8,000, in this, case his tax liability shall be, Income under the head House Property, 10,00,000, Income under the head Other Sources (4,000+ 8,000), 12,000, Gross Total Income, 10,12,000, Less: Deduction u/s 80C, Investment in current year, 50,000, Accrued interest, 4,000, (54,000), (no deduction shall be allowed for interest received on maturity), Total Income, 9,58,000, Tax on `9,58,000 at slab rate, 1,04,100, Add: HEC @ 4%, 4,164, Tax Liability, 1,08,264
Page 215 :
Deduction From Gross Total Income, , 215, , Rounded off u/s 288B, 1,08,260, 2. Public provident fund is a deposit scheme run by Central Government and account can be opened in the, bank or post office and maturity shall be after 15 years and the account can be opened in the name of, self, spouse or children. HUF can open the account in the name of any of its members. Amount, received on maturity shall be exempt from income tax and also interest is exempt from income tax. No, deduction is allowed under section 80C for interest., 3. Investment in fixed deposit for a period of 5 years or more with scheduled banks, provided the term, deposit are issued in accordance with a scheme notified by the Central Government. (Bank Term Deposit, Scheme, 2006 – depositor can be individual or Hindu Undivided Family. The deposit should be for a, minimum period of 5 years. Interest income shall be taxable on accrual basis and it will not qualify for, deduction under section 80C.) Principal amount received on maturity shall be exempt. Individual can, deposit the amount only in his own name and HUF can deposit the amount in the name of any of its, member., 4. Five Year Post Office Time Deposit Account., An assessee is allowed to invest the amount in five year post office time deposit account and deduction, shall be allowed equal to the amount invested. Interest shall be paid on annual basis and it will be, taxable and deduction under section 80C is not allowed. Amount received on maturity shall be, exempt. Individual can invest the amount in his name and HUF can invest the amount in the name of, any of its member., Pre-mature payment is allowed but amount received on pre-mature payment shall be taxable., 5. If an assessee has taken a loan from a notified organization like banks or financial institution etc. for, purchase or construction of a residential house, in such cases deduction shall be allowed equal to the, amount re-paid by the assessee towards principal (not towards interest)., If loan has been taken for Addition, Alteration, or Repairs etc of the house property, no deduction is, allowed., If the assessee has transferred the house property before the expiry of 5 years from the end of the, financial year in which possession of such properties was taken by him, no deduction shall be allowable, in the previous year in which the house property has been transferred. The deduction allowed in the past, years shall be considered to be income of the assessee of the previous year in which the house property, is transferred., Deduction shall also be allowed for stamp duty , registration fee and other expenses for the purpose of, transfer of such house property to the assessee., 6. If any individual or HUF has taken life policy, deduction shall be allowed for the premium paid for such, life policy and individual can take the policy in the name of self, spouse and children and Hindu, Undivided Family can take the policy in the name of any of its members. (Children may be, dependant or independent or may be married or unmarried or step or adopted.), Deduction is allowed equal to the premium paid but maximum upto 10% of capital sum assured, i.e. if, premium paid is more than 10% of capital sum assured, deduction shall be allowed only for 10% of sum, assured. (In respect of policy issued before 01.04.2012, 10% shall be taken as 20%), If LIC policy has been taken in the name of a person who is suffering from disability given under section, 80U or from a specified disease given under section 80DDB, 10% shall be taken as 15% but it is, applicable for the policies taken w.e.f 01.4.2013 onwards., If an assessee has discontinued a life insurance policy before paying premium for a period of atleast 2, years, deduction allowed in the earlier years shall be considered to be income of the year in which, policy has been discontinued., As per section 10(10D), any payment received on maturity of insurance policy shall be exempt from, income tax i.e. even the amount of bonus received shall be exempt from income tax. If the policy, holder has paid premium of more than the specified limit (10% / 15% / 20%) in any of the years,, amount received on maturity shall be chargeable to tax but if the amount has been received on the, death of the policy holder, it will be exempt from income tax.
Page 216 :
Deduction From Gross Total Income, , 216, , e.g. Mr. X has paid premium of one life policy `25,000 and sum assured is ` 1,00,000 and policy was, taken on 01.04.2012 onwards, in this case deduction allowed shall be `10,000 but if policy was taken, before 01.04.2012, deduction allowed shall be `20,000. If Mr. X is a handicapped person and policy was, taken w.e.f 01.04.2013 onwards, deduction allowed shall be `15,000, 7. Payment of tuition fees to School, College, University or any other Educational Institution in India, provided the fees has been paid in connection with the children of the assessee and further for, maximum two children and it should be whole time education. Children shall include even adopted, and step children also. Deduction is not allowed to HUF. If payment is made outside India, deduction is, not allowed. Similarly if payment is given for part time education or correspondence course, deduction is, not allowed., 8. Employees contribution to statutory provident fund or recognised provident fund or approved, superannuation fund, 9. Investment in Units of Unit trust of India or mutual fund including Unit Linked Insurance Plan of, UTI or mutual fund., 10. Subscription to Notified Deposit Schemes of NHB e.g. subscription to Home Loan Account Scheme, of NHB and National Housing Bank (tax saving) term deposit scheme 2008, 11. Investment in equity shares or debentures etc forming part of an eligible issue., Eligible issue means an issue made by an Indian Public Ltd Company or a Public Financial, Institution, a Mutual Fund etc. and the funds so collected are utilised for Developing, Maintaining, and Operating Infrastructure Facility., If any such equity shares etc. have been sold within a period of 3 years from the date of purchase, in, such cases deduction earlier allowed shall be considered to be income of such year, 12. Investment in notified bonds issued by the National Bank for Agriculture and Rural Development., 13. Senior Citizens Savings Scheme. Amount should be invested in the name of self and amount received, on maturity shall be exempt and interest shall be payable on quarterly basis and interest received is, taxable. Deduction under section 80C for interest is not allowed., 14. Investment in Sukanya Samridhi Account and amount can be invested by an individual as guardian in, the name of girl child who is of the age of 10 years or less. Interest received is exempt. Amount received, on maturity is exempt. Account can be closed after the completion of 21 years of age. In case of, marriage, payment is allowed after completion of 18 years of age., Payment from Sukanya Samriddhi Account [Section 10(11A)], Section 10(11A) provides that any payment from an account opened in accordance with the Sukanya, Samriddhi Account Rules, 2014, made under the Government Savings Bank Act, 1873, shall not be, included in the total income of the assessee. Accordingly, the interest accruing on deposits in, and, withdrawals from any account under the said scheme would be exempt., 15. Contribution to additional account under NPS, Contribution by a Central Government employee to additional account under NPS (specified account), referred to in section 80CCD for a fixed period of not less than 3 years and which is in accordance with the, scheme notified by the Central Government for this purpose qualifies for deduction under section 80C. It, may be noted that only the contribution to the additional account under NPS will qualify for deduction under, section 80C., There are two types of NPS account i.e., Tier I and Tier II, to which an individual can contribute. Section, 80CCD provides deduction in respect of contribution to individual pension account [Tier I account] under, the NPS [referred to in section 20(2)(a) of the Pension Fund Regulatory and Development Authority Act,, 2013 (PFRDA)] whereas deduction under section 80C is allowable in respect of contribution by Central, Government employee to additional account [Tier II account] of NPS [referred to in section 20(3) of the, PFRDA], which does not qualify for deduction under section 80CCD. Thus, Tier II account is the additional, account under NPS, contribution to which would qualify for deduction under section 80C only in the hands, of a Central Government employee., Deduction shall be allowed only if the amount has been actually paid by the assessee i.e. if the amount is, due but not paid deduction is not allowed. E.g. Premium of `25,000 was due on 27.03.2022 but it was
Page 217 :
Deduction From Gross Total Income, , 217, , paid on 10.04.2022, in this case no deduction is allowed in the previous year 2021-22, rather deduction shall, be allowed in the previous year 2022-23., Illustration 1: Mr. X, aged about 61 years, has earned a lottery income of ` 1,20,000 (gross) during the P.Y., 2021-22. He also has a business income of `6,00,000. He invested an amount of `10,000 in Public Provident, Fund account and ` 24,000 in National Saving Certificates and `60,000 in eligible mutual funds. He has, paid premium of `30,000 for a life policy having sum assured `2,00,000 and policy was taken after, 01.04.2012. Compute his tax liability for assessment year 2022-23., Solution:, Computation of total taxable income of Mr. X for A.Y.2022-23, Income under the head business/profession, 6,00,000, Income under the head Other Sources (casual income), 1,20,000, Gross Total Income, 7,20,000, Less: Deduction u/s 80C, 1. Public Provident Fund, (10,000), 2. Investment in NSC, (24,000), 3. Mutual Fund, (60,000), 4. Payment of premium for LIC policy, (20,000), Total Income, 6,06,000, Computation of Tax Liability, Tax on casual income `1,20,000 @ 30%, 36,000, Tax on `4,86,000 at slab rate, 9,300, Tax before health & education cess, 45,300, Add: HEC @ 4%, 1,812, Tax Liability, 47,112, Rounded off u/s 288B, 47,110, Illustration 2: Mr. X has income under the head Business/Profession `19,90,000., His investments are as given below:, 1. Investment in NSC `50,000, 2. Investment in PPF in name of Mrs. X `5,000, 3. Payment of premium for LIC policy taken in the name of dependent father on 16.06.2021 and its premium, paid is `11,000, 4. Payment of premium for LIC policy taken in the name of independent son on 15.04.2021 and its premium, paid is `6,000 (sum assured `1,00,000), 5. Payment of premium for LIC policy taken in the name of independent married daughter on 11.01.2022, and its premium paid is `21,000 (sum assured `1,00,000), Compute Income Tax liability for the A.Y. 2022-23., Solution:, `, Income under the head business/profession, 19,90,000, Gross Total Income, 19,90,000, Less: Deduction u/s 80C, 1. Investment in NSC, (50,000), 2. Investment in PPF in the name of Mrs. X, (5,000), 3. Payment of premium for LIC policy taken in the name of dependent father, NIL, 4. Payment of premium for LIC policy taken in the name of independent son, (6,000), 5. Payment of premium for LIC policy taken in the name of independent married, daughter (allowed 10% of sum assured), (10,000), Total Income, 19,19,000, Computation of Tax Liability, Tax on `19,19,000 at slab rate, 3,88,200, Add: HEC @ 4%, 15,528, Tax Liability, 4,03,728
Page 218 :
Deduction From Gross Total Income, Rounded off u/s 288B, , 218, 4,03,730, , Illustration 3: Mr. X furnishes you the following information:, Raw material purchased `5,00,000. Manufacturing expenses (revenue nature) `2,00,000., Sale price `18,00,000. Plant & machinery acquired `2,60,000. Depreciation is allowed @ 15%., He has made the investments as given below:, (i), Fixed deposit with State Bank for two years `5,000., (ii), Investment in National Saving Certificates `5,000., (iii) Deposit in Public Provident Fund Account in the name of major married independent son `5,000., (iv), Deposit in Public Provident Fund Account in the name of minor son `5,000., (v), Payment of premium for LIC policy in name of major married independent daughter on, 15.09.2021 `5,000. (sum assured `1,00,000)., (vi), Payment of premium for LIC policy in name of major married independent son on 11.11.2021, `5,000. (sum assured `20,000), (vii) Investment in Home Loan Account Scheme of National Housing Bank `5,000 (Investment was, made out of past savings)., (viii) Investment in units of Mutual Funds notified under section 10(23D) `5,000. (Investment was, made out of current income exempt from income tax)., (ix), Investment in Equity Shares of Infrastructure Companies `5,000., (x), Payment of Tuition fees of his son to a private coaching centre for coaching in taxation `5,000., Compute his income and tax liability for assessment year 2022-23., Solution:, Computation of income under the head Business/profession, Sale price, 18,00,000.00, Less: Purchase Price, (5,00,000.00), Less: Manufacturing expenses, (2,00,000.00), Less: Depreciation on plant and machinery (2,60,000 x 15%), (39,000.00), Income under the head Business/profession, 10,61,000.00, Gross Total Income, 10,61,000.00, Less: Deduction u/s 80C, National Saving Certificate, (5,000.00), Public Provident Fund, (10,000.00), LIC policy in name of major married independent daughter, (5,000.00), LIC policy in name of major married independent son, (2,000.00), Home Loan Account Scheme, (5,000.00), Units of Mutual Funds, (5,000.00), Equity Shares of Infrastructure Companies, (5,000.00), Total Income, 10,24,000.00, Computation of Tax Liability, Tax on `10,24,000 at slab rate, 1,19,700.00, Add: HEC @ 4%, 4,788.00, Tax Liability, 1,24,488.00, Rounded off u/s 288B, 1,24,490.00, Illustration 4: Compute the eligible deduction under section 80C for A.Y.2022-23 in respect of life, insurance premium paid by Mr. X during the P.Y.2021-22, the details of which are given hereunder –, Date of issue, Person insured, Actual capital sum Insurance premium, of policy, paid during 2021-22, assured (`), (`), (i), 01.04.2011, Self, 2,00,000, 50,000, (ii), 01.05.2017, Spouse, 1,50,000, 20,000, (iii), 01.06.2018, Handicapped Son, 3,00,000, 60,000
Page 219 :
Deduction From Gross Total Income, , 219, , (section 80U disability), Solution:, Date of issue Person insured, of policy, (i), (ii), (iii), , 01.04.2011, 01.05.2017, 01.06.2018, , Self, Spouse, Handicapped, Son (section, 80U disability), , Actual, capital sum, assured, 2,00,000, 1,50,000, 3,00,000, , Insurance, premium, paid during, 2021-22, 50,000, 20,000, 60,000, , Deduction, u/s 80C for, A.Y.2022-23, 40,000, 15,000, 45,000, , Total, , Remark, (restricted to, % of sum, assured), 20%, 10%, 15%, , 1,00,000, , Illustration 5 (From RTP): Compute the eligible deduction under section 80C for A.Y.2022-23 in respect, of life insurance premium paid by Mr. X during the P.Y.2021-22, the details of which are given hereunder –, Date of issue, of policy, (i), (ii), (iii), (iv), , 01.06.2011, 01.05.2017, 01.07.2018, 01.07.2018, , Person insured, Mr. X, Mrs. X, his wife, Ms. Y, his handicapped, daughter, (section, disability), Mr. Z, his son, , Actual capital sum, Assured (`), 4,00,000, 1,00,000, 5,00,000, , Insurance premium, paid during 2021-22, (`), 75,000, 25,000, 60,000, , 1,00,000, Total Premium paid, , 25,000, 1,85,000, , 80U, , Solution:, Computation of eligible deduction under section 80C for A.Y. 2022-23, (in respect of life insurance premium paid by Mr. X), Insurance, Date of issue Person insured, Actual, premium, of policy, capital sum, paid during, assured, 2021-22, (i), 01.06.2011, Mr. X, 4,00,000, 75,000, (ii), 01.05.2017, Mrs. X, 1,00,000, 25,000, (iii), 01.07.2018, Handicapped, 5,00,000, 60,000, daughter, (iv), 01.07.2018, Son, 1,00,000, 25,000, , Restricted, to % of, sum assured, , Deduction, u/s 80C for, A.Y.2022-23, , 20%, 10%, 15%, , 75,000, 10,000, 60,000, , 10%, , 10,000, , Total, Maximum deduction u/s 80C restricted to, , 1,55,000, 1,50,000, , NOV – 2014 (2 Marks), Determine the eligibility and quantum of deduction under Chapter VI-A in the following case:, Mr. X has paid life insurance premium of `70,000 during the year. (Sum Assured 3,50,000 policy issued on, 31.03.2011), Solution:, Deduction for Life Insurance Premium shall be available under section 80C of `70,000 (being maximum, 20% of `3,50,000 as the policy was issued before 01.04.2012)
Page 220 :
Deduction From Gross Total Income, , 220, , MAY – 2001 (5 Marks), The particulars of income of Mrs. X. aged 55 years for the financial year 2021-22 are given below: `, (1) Income under the head salary received from M/s ABC Ltd. for the year, 4,00,000, (2) Rental income received from a commercial complex, 12,000 p.m., (3) Arrears of rent received from the complex, which were not charged to, tax in any earlier years, 30,000, (4) Interest paid on loan taken for the purchase of a house from a scheduled, bank for use as own residence, 1,20,000, (5) Repayment of instalments of loan taken from the bank for the purchase, of the above property, 60,000, (6) Deposits in public provident fund account, (i) Towards loan taken from public provident account, 20,000, (ii) Out of current year’s income, 40,000, (7) Investment made in units of a mutual fund approved by the board under, section 80C of the Income-Tax Act., 40,000, Compute the total income of Mrs. X and the tax payable thereon in respect of assessment year 2022-23., Answer:, `, `, Computation of total income and tax liability Mrs. X, Income from under the head salary, Income under the head Salary, 4,00,000.00, Income from house property, Let out commercial complex, Gross Annual Value (12,000 x 12), Less: Municipal taxes, Net Annual Value, Less: 30% of NAV u/s 24(a), Less: Interest on capital borrowed u/s 24(b), Income from let out property, Property self- occupied for residence, Net Annual Value, Less: Interest on capital borrowed u/s 24(b), Loss from self–occupied property, Arrears of rent Section 25A, Less: (30% of `30,000), Income under the head House Property, Gross Total Income, Less: Deduction u/s 80C, Repayment of loan taken to purchase residential house property, Deposit in public provident fund out of current income, Investment made in units of mutual fund for infrastructure facility, Total Income, Computation of Tax Liability, Tax on `3,51,800 at slab rate, Less: Rebate u/s 87A, Add: HEC @ 4%, Tax Liability, , 1,44,000.00, Nil, 1,44,000.00, (43,200.00), Nil, 1,00,800.00, Nil, (30,000.00), (30,000.00), 30,000, (9,000), , 21,000.00, 91,800.00, 4,91,800.00, (1,40,000.00), , 60,000, 40,000, 40,000, 3,51,800.00, 5,090.00, (5,090.00), Nil, Nil, , Question 2: Write a note on deduction in case of contribution to a Pension Fund., Answer: Deduction in respect of contribution to certain pension Funds Section 80CCC, In general in case of life insurance, lump sum amount is paid to the policyholder but in some of the life, policies pension is given instead of lump sum amount e.g. Jeevan Suraksha policy. If any person has paid, premium for such policy, deduction is allowed under section 80CCC instead of section 80C. Deduction is
Page 221 :
Deduction From Gross Total Income, , 221, , allowed only to an individual and individual can take the policy only in his name (and not in name of spouse, or children). Any pension received shall be taxable under the head Other Sources. If the assessee has, surrendered the policy, amount received shall be taxable under the head Other Sources., e.g. (i) Mr. X has taken Jeevan Suraksha Policy in his name and has paid premium of `10,000, in this case, deduction allowed shall be `10,000, (ii) Mr. X has taken Jeevan Suraksha Policy in the name of Mrs. X and has paid premium of `10,000, in this, case deduction is not allowed., (iii) Mr. X has taken Jeevan Suraksha Policy in the name of his dependant son and has paid premium of, `10,000, in this case deduction is not allowed., As per section 80CCE, Maximum Deduction allowed u/s 80C + 80CCC + 80CCD(1) shall be ` 1,50,000., Illustration 6: Mr. X has income under the head business/profession `3,35,000., He has made the following investments:• NSC `10,000, • Investment in post office 5 year time deposit account `15,000, • Payment of premium for life policy in the name of major married independent son on 10.10.2021, `30,000 (sum assured `90,000), • Paid premium of ` 11,000 for Jeeven Suraksha policy taken in name of Mr. X on 11.11.2021., Compute income tax liability for A.Y 2022-23., Solution:, `, Income under the head Business/Profession, 3,35,000.00, Gross Total Income, 3,35,000.00, Less: Deduction u/s 80C, NSC, (10,000.00), Investment in 5 years post office, (15,000.00), Payment of premium of LIC, (9,000.00), Less: Deduction u/s 80CCC, (11,000.00), Total Income, 2,90,000.00, Computation of Tax Liability, Tax on `2,90,000 at slab rate, Less: Rebate u/s 87A, Tax Liability, , 2,000.00, (2,000.00), Nil, , Illustration 7: The gross total income of Mr. X for the A.Y.2022-23 is ` 5,00,000. He has made the, following investments/payments during the F.Y.2021-22, `, (1) Contribution to PPF, 90,000, (2) Payment of tuition fees to Apeejay School, New Delhi, for education of his son, studying in Class XI, 45,000, (3) Repayment of housing loan taken from Standard Chartered Bank, 25,000, (4) Contribution to approved pension fund of LIC, 10,000, Compute the eligible deduction under Chapter VI-A for the A.Y.2022-23., Solution:, Computation of deduction under Chapter VI-A for the A.Y.2022-23, Particular, Deduction under section 80C, (1) Contribution to PPF, (2) Payment of tuition fees to Apeejay School, New Delhi, for education of his son studying in, Class XI, (3) Repayment of housing loan, , `, 90,000, 45,000, 25,000, 1,60,000
Page 222 :
Deduction From Gross Total Income, Deduction under section 80CCC, (1) Contribution to approved pension fund of LIC, As per section 80CCE, the aggregate deduction under section 80C, 80CCC and 80CCD has to be, restricted to ` 1,50,000, Deduction allowable under Chapter VI-A for the A.Y.2022-23, , 222, 10,000, 1,70,000, 1,50,000, , Deduction under section 80CCD shall be discussed under the head Salary., Question 3 [V. Imp.]: Write a note on deduction in case of payment of premium for Medical, Insurance/Mediclaim Policy., Answer: Deduction in respect of Medical Insurance Premia Section 80D, 1. Deduction shall be allowed only to an individual or Hindu Undivided Family., 2. Deduction shall be allowed if the assessee has made payment towards, (i), Medical Insurance or, (ii), Central Government Health Scheme or such other scheme as may be notified by the, Central Government in this behalf, (iii) Preventive Health Check-up, 3. Individual can make payment for wife or husband or dependent children and deduction shall be, allowed equal to the amount paid but subject to a maximum of `25,000 but in case of senior, citizen deduction shall be allowed upto `50,000., If the individual has taken policy in the name of parents (dependent or independent), additional, deduction shall be allowed to the extent of the premium paid but maximum `25,000, however, if, the policy has been taken in the name of senior citizen, deduction shall be allowed to the extent of, `50,000., Deduction for Preventive Health Check up shall be maximum `5,000 in aggregate for self, spouse,, dependant children and parents. Parents may be dependant or independant, Hindu Undivided Family can take the policy in the name of any of its members and deduction shall, be allowed in the similar manner., Payment should be made otherwise than in cash but payment for preventive health check-up can be, made in any manner., In case of a senior citizen, in general medi-claim policy is not issued hence expenditure can be incurred, on their medical treatment and deduction for such expenditure shall also be allowed but limit shall be the, same as given above., e.g. (i) Mr. X has taken medi claim policy in his name and paid premium `27,000 by cheque, in this case, deduction allowed shall be `25,000 but if Mr. X is a senior citizen, deduction allowed shall be `27,000, (ii) Mr. X has paid `7,000 for self for preventive health checkup and `7,000 for preventive health checkup, of his father, in this case deduction allowed shall be `5,000, (iii) Mr. X paid premium of medi claim policy by cheque for self, spouse and children `22,000 and for, parents `28,000, in this case deduction allowed shall be `47,000, (iv) Mr. X paid premium of medi claim policy `15,000 in cash, deduction allowed shall be Nil, (v) Mr. X paid premium of medi claim policy by cheque `18,000 in the name of his father who is not, dependant on Mr. X, deduction allowed shall be `18,000, In case premium is paid for more than one year, proportionate deduction shall be allowed e.g. Mr. X paid, premium of ₹ 30,000 for 2 years, in this case deduction allowed shall be 15,000 for each of the year., Illustration 8: Mr. X, aged 40 years, paid medical insurance premium of `18,000 by cheque during the, P.Y.2021-22 to insure his health as well as the health of his spouse. He also paid medical insurance premium, of `26,000 by cheque during the year to insure the health of his father, aged 63 years, who is not dependent, on him. He contributed ` 5,000 by cheque to Central Government Health Scheme during the year. He has, incurred `3,000 in cash on preventive health check-up of himself and his spouse and `4,000 by cheque on, preventive health check-up of his father., Compute the deduction allowable under section 80D for the A.Y.2022-23.
Page 223 :
Deduction From Gross Total Income, Solution:, Deduction allowable under section 80D for the A.Y.2022-23, Particulars, , A., (i), (ii), (iii), , Premium paid and medical expenditure incurred for self and spouse, Medical insurance premium paid for self and spouse, Contribution to CGHS, Exp. on preventive health check-up of self & spouse, , B., , Premium paid and medical expenditure incurred for father, who is a, senior citizen, Mediclaim premium paid for father, who is 63 years of age, Expenditure on preventive health check-up of father, , (i), (ii), , 223, , `, Actual, Payment, , `, Maximum, deduction, allowable, , 18,000, 5,000, 3,000, 26,000, , 18,000, 5,000, 2,000, 25,000, , 26,000, 4,000, 30,000, , 26,000, 3,000, 29,000, 54,000, , Total deduction under section 80D (25,000 + 29,000), , Illustration 9: Mr. X, aged 40 years, paid medical insurance premium of ` 20,000 by cheque during the, P.Y.2021-22 to insure his health as well as the health of his spouse and dependent children. He also paid, medical insurance premium of `51,000 by cheque during the year to insure the health of his father, aged 67, years, who is not dependent on him. He contributed `6,000 by cheque to Central Government Health, Scheme during the year. Compute the deduction allowable under section 80D for the A.Y.2022-23., Solution:, Deduction allowable under section 80D for the A.Y.2022-23, Particulars, `, 20,000, (i) Medical insurance premium paid for self, spouse and dependent children, 6,000, (ii) Contribution to CGHS, 26,000, Deduction allowed, 25,000, (iii) Mediclaim premium paid for father, who is 67 years of age (restricted to 50,000), 50,000, 75,000, Note – The total deduction under (i) and (ii) above should not exceed ` 25,000. In this case, since the total, of (i) and (ii) is exceeding `25,000 (i.e., ` 26,000) hence it is restricted to ` 25,000., Illustration 10: Mr. Arjun (52 years old) furnishes the following particulars in respect of the following, payments:, S. No. Particulars, Amount (`), 1., Premium paid for insuring the health of –, 10,000, • Self, 8,000, • Spouse, 4,000, • Dependant son, 18,000, • Mother, 2., Paid for Preventive Health Check-up of, 2,000, • himself, 1,500, • spouse, 4,000, • mother, 3., Incurred medical expenditure of ` 25,000 and `15,000 for his mother, aged 80 years, and father, aged 85 years. Both mother and father are resident in India., Compute the deduction available to Mr. Arjun under section 80D for the A.Y. 2022-23., Solution:
Page 224 :
Deduction From Gross Total Income, Computation of deduction under section 80D for the A.Y. 2022-23, S. No., Particulars, 1., I. In respect of premium paid for insuring the health of • Self, • Spouse, • Dependant son, II. In respect of expenditure on preventive health check-up of • Self, • Spouse, , 2., , Restricted to [`25,000 – ` 22,000, since maximum deduction is `25,000], Aggregate of deduction (I+II) under (1) restricted to, I. In respect of payment towards health insurance premium for his mother, II. In respect of preventive health check-up of his mother [`4,000,, restricted to `2,000, (`5,000 – `3,000), since maximum deduction for, preventive health check-up under section 80D is `5,000], III. Medical expenditure for father would only be eligible for deduction, [See Note below], , 224, Amount (`), 10,000, 8,000, 4,000, 22,000, 2,000, 1,500, 3,500, 3,000, 25,000, 18,000, 2,000, 15,000, 35,000, , 35,000, Amount of deduction under (2), Total deduction under section 80D [(1) + (2)], 60,000, Note: Irrespective of the fact that the mother of Arjun is a very senior citizen the deduction under section, 80D would not available to him in respect of the medical expenditure incurred for his mother, since Mr., Arjun has taken a health insurance policy for his mother., MAY – 2015 (4 Marks), Compute the eligible deduction under Chapter VI-A for the Assessment year 2022-23 of Ms. Roma, who has, a gross total income of `15,00,000 for the assessment year 2022-23 and provide the following informations, about his investments/payments during the financial year 2021-22:, SI., Particulars, Amount, No., (` ), 35,000, 1., Life Insurance premium paid (Policy taken on 01.01.2012 and sum assured is, `1,50,000), 2., Public Provident Fund contribution., 90,000, 3., Repayment of Housing loan to Bhartiya Mahila Bank, Bangalore., 20,000, 4., Payment to L.I.C. Pension Fund, 25,000, 5., Mediclaim Policy taken for self, spouse and dependent children, premium paid by, 20,000, cheque, 6., Medical Insurance premium paid for parents (Senior Citizen), premium paid by cheque, 25,000, Solution:, `, `, Gross Total Income, 15,00,000, Less:, Deduction u/s 80C, L.I.C. Premium paid, 30,000, (Paid `35,000, but maximum allowed 20% of `1,50,000), P.P.F. amount paid, 90,000, Repayment of housing loan to Bhartiya Mahila Bank, 20,000, Deduction u/s 80 CCC, L.I.C. pension fund, 25,000, Total, 1,65,000, Maximum deduction allowed 80CCE, (1,50,000)
Page 225 :
Deduction From Gross Total Income, , 225, , Deduction u/s 80D, Mediclaim for self, spouse and dependent children, (assumed amount paid through cheque), Mediclaim premium for parents, (assumed amount paid through cheque), Total Income, , (20,000), (25,000), 13,05,000, , NOV – 2011 (3 Marks), The gross total income of Mr. X for the Assessment Year 2022-23, was `12,00,000. He has made the, following investments/payments during the financial year 2021-22., 1. L.I.C. Premium paid (Policy value `1,00,000) (Policy taken after 01.04.2012), 25,000, 2. P.P.F. amount paid, 70,000, 3. Repayment of housing Loan to Indian Bank, 50,000, 4. Payment made to L.I.C. pension fund, 25,000, 5. Medical insurance premium for self, wife and dependant Children., 18,000, 6. Mediclaim premium for parents (aged over 80 years), 30,000, Compute eligible deduction under Chapter VI-A for the Assessment Year 2022-23 and also compute his tax, liability., Answer:, `, `, Gross Total Income, 12,00,000, Less:, Deduction u/s 80C, L.I.C. Premium paid, (Paid `25,000, but maximum allowed 10% of `1,00,000), P.P.F. amount paid, Repayment of housing loan to Indian Bank, Deduction u/s 80 CCC, L.I.C. pension fund, Total, Maximum deduction allowed 80CCE, Deduction u/s 80D, Mediclaim for self, wife and dependant children, (assumed amount paid through cheque), Mediclaim premium for parents, (assumed amount paid through cheque), Total Income, Computation of Tax Liability, Tax on `10,02,000 at slab rate, Add: HEC @ 4%, Tax Liability, Rounded off u/s 288B, , 10,000, 70,000, 50,000, 25,000, 1,55,000, (1,50,000), (18,000), (30,000), 10,02,000, 1,13,100, 4,524, 1,17,624, 1,17,620, , JUNE – 2009 (2 Marks), Mr. X, an individual, made payment of health insurance premium to General Insurance Corporation in an, approved scheme. Premium paid on his health is `10,000 and his spouse’s health is `15,000 during the year, 2021-22. He also paid health insurance premium of `25,000 on his father’s health who is a senior citizen and, not dependent on him. The payments have not been made by cash. Compute the amount of deduction under, Chapter VI-A of the Act, available to Mr. X from his gross total income for the assessment year 2022-23.
Page 226 :
Deduction From Gross Total Income, , 226, , Answer., Mr. X will be eligible to claim deduction under section 80D on payment of health insurance premium. The, premium is paid otherwise than by way of cash and hence qualifies for deduction under section 80D., Therefore, the amount of deduction under section 80D would be –, Particulars, Amount (`), On health insurance premium paid on the health of himself and his spouse, (`10,000 + `15,000 = `25,000), 25,000, On health insurance premium paid on the health of his father, `25,000, (whether dependent or not), 25,000, Total deduction under section 80D, 50,000, Question 4 [Imp.]: Deduction in case of expenditure in connection with handicapped dependant, relative., Answer: Deduction in respect of maintenance including medical treatment of a dependant who is a, person with disability Section 80DD, 1. Deduction is allowed only to a resident individual and a resident Hindu Undivided Family., 2. Deduction is allowed if the assessee has incurred any expenditure for the medical treatment, training, and rehabilitation etc. of a dependant disabled person, or has deposited any amount with LIC or any, other insurer for the benefit of such dependant., 3. “Dependant” in the case of an individual, means the spouse, children, parents, brothers and sisters, who are dependant on the individual and in the case of Hindu Undivided Family means any member of, the Hindu Undivided Family who is dependant on such Hindu Undivided Family., 4. Deduction allowed shall be `75,000 irrespective of the expenditure incurred by the assessee and in case, of severe disability, deduction allowed shall be `1,25,000., 5. The assessee should enclose a certificate with the return from prescribed medical authority., Illustration 11: Mr. X is a resident individual. He deposits a sum of `25,000 with Life Insurance, Corporation every year for the maintenance of his handicapped grandfather. A copy of the certificate from, the medical authority is submitted. Compute the amount of deduction available under section 80DD for the, A.Y. 2022-23., Solution: Since the amount deposited by Mr. X was for his grandfather, he will not be allowed any, deduction under section 80DD. The deduction is available if the individual assessee incurs any expense for a, dependant disabled relative. Grandfather does not come within the definition of dependant relative., Illustration 12: What will be the deduction if Mr. X had made this deposit for his dependant father?, Solution: Since the expense was incurred for a dependant disabled relative, Mr. X will be entitled to claim a, deduction of `75,000 under section 80DD, irrespective of the amount deposited. In case his father has severe, disability, the deduction would be `1,25,000., MAY – 1997 (4 Marks), In respect of assessment year 2022-23, an author of text-books for schools furnishes the following, particulars and request you to work out his tax liability:, `, `, 1. Royalty from Printers Ltd. on publication of books, 2,20,000, 2. Capital gains long term, 1,90,000, 3. Other Sources:, (a) Interest on Bank fixed Deposits, 12,000, (b) Dividend income from Indian company, 3,000, (c) Income from units of U.T.I., 5,000, 20,000, Deductions: (i) Contributions towards:, (a) LIC Pension Scheme, 15,000, (b) LIC Premium, 10,000, (ii) Contribution to public provident fund, 10,000, (iii) Investment in National Savings Certificates, 50,000, (iv) Medical treatment of handicapped dependent relative, 20,000
Page 227 :
Deduction From Gross Total Income, Answer:, Income under the head Capital Gains (LTCG), Computation of income under the head Other Sources, Royalty income, Interest from bank fixed deposits, Dividend income, Interest from units of UTI, Income under the head Other Sources, Gross Total Income, Less: Deduction u/s 80C, LIC Premium, Contribution in Public provident fund, National Saving Certificate, Less: Deduction u/s 80CCC, Less: Deduction u/s 80DD, Total Income, Computation of Tax Liability, Tax on LTCG `20,000 (`1,90,000 – `1,70,000) @ 20% u/s 112, Tax on `80,000 at slab rate, Less: Rebate u/s 87A, Tax Liability, , 227, `, 1,90,000, 2,20,000, 12,000, 3,000, 5,000, 2,40,000, 4,30,000, (10,000), (10,000), (50,000), (15,000), (75,000), 2,70,000, 4,000, Nil, (4,000), Nil, , Question 5 [V. Imp.]: Briefly explain the provisions under the Income Tax Act relating to deduction, from gross total income in the case of blind or physically handicapped persons., Answer:, Deduction in case of handicapped person Section 80U, (1) In computing the total income of an individual, being a resident, who, at any time during the previous, year, is certified by the medical authority to be a person with disability, there shall be allowed a, deduction of a sum of `75,000., Provided that where such individual is a person with severe disability, the provisions shall have effect, as if for the words “`75,000”, the words “`1,25,000” had been substituted., (2) The assessee should enclose a certificate with the return from prescribed medical authority., e.g. (i) Mr. X is suffering from a disability and has income under the head salary `10,00,000 and he has, invested `1,00,000 in NSC, in this case deduction allowed under section 80C shall be `1,00,000 and under, section 80U shall be `75,000, (ii) Mr. X is suffering from a severe disability and has income under the head salary `10,00,000 and he has, invested `2,00,000 in NSC, in this case deduction allowed under section 80C shall be `1,50,000 and under, section 80U shall be `1,25,000., NOV – 2008 (4 Marks), Briefly explain provisions of section 80U of the Income-tax Act, 1961, in respect of deduction available on, permanent physical disability., Solution: Refer Answer given in the Chapter, Question 6 [Imp.]: Write a note on deduction in case of expenditure on the treatment of Specified, Disease., Answer: Deduction in respect of medical treatment, etc. of specified disease, Section 80DDB, 1. Deduction is allowed only to a resident individual or resident Hindu Undivided Family., 2. Deduction is allowed if the assessee has incurred any amount for treatment of such disease as are, specified in the rule 11DD., 3. The expenditure can be incurred for himself or a dependant person, and in case of an individual,, such person may be spouse, children, parents, brothers or sisters who are dependant on such, individual and in case of Hindu Undivided Family such person may be any member of the Hindu
Page 228 :
Deduction From Gross Total Income, , 228, , Undivided Family who is dependant on the Hindu Undivided Family., 4. Deduction allowed shall be the amount incurred or `40,000 whichever is less and if the amount has, been paid with regard to a Senior Citizen, deduction allowed shall be upto `1,00,000., 5. Deduction allowed shall be reduced by the amount received under medi claim insurance and also, by the amount which has been paid by the employer., 6. The assessee should enclose a certificate with the return from prescribed medical authority., Example, Mr. X has incurred `1,25,000 on the treatment of a specified disease for himself, in this case deduction, allowed shall be `40,000 but if a claim of `10,000 has been received under medi-claim policy, deduction, allowed shall be `30,000. If Mr. X is a senior citizen, deduction allowed shall be 1,00,000 – 10,000 = 90,000, Question 7 [Imp.]: Write a note on deduction in case of payment of interest on loan taken for Higher, Education., Answer: Deduction in case of payment of interest on loan taken for pursuing Higher Education, Section 80E, 1. Deduction is allowed only to an individual., 2. Deduction is allowed if the assessee has paid interest on loan taken by him from any financial, institutions or any approved charitable institution., 3. The loan should have taken for pursuing higher education which means any course of study pursued, after passing the Senior Secondary Examination or its equivalent., 4. Education can be either of self or spouse or children or any person for whom the assessee is legal, guardian., 5. The entire amount of interest paid by an individual is allowed as deduction., 6. No deduction shall be allowed for repayment of the principal loan amount., 7. Deduction is allowed for a maximum period of 8 years starting from the year in which first payment of, interest was given., 8. Approved charitable institution means the institution notified by the Central Government., Financial institution means banking company or other financial institution notified by the Government., 9. No deduction is allowed after the period of 8 years., Example, Mr. X has taken a loan of `2,00,000 from State Bank on 01.10.2010 for pursuing MBBS course & after, becoming a doctor he has given payment of interest of `45,000 on 01.10.2021, in this case deduction, allowed shall be `45,000., Illustration 13: Mr. X has taken three education loans on April 1st, 2021, the details of which are given, below:, Loan 1, Loan 2, Loan 3, For whose education loan was taken, Mr. X, Son of Mr. X, Daughter of Mr. X, Purpose of loan, MBA, B. Sc., B.A., Amount of loan (`), 5,00,000, 2,00,000, 4,00,000, Annual repayment of loan (`), 1,00,000, 40,000, 80,000, 20,000, 10,000, 18,000, Annual repayment of interest (`), Compute the amount deductible under section 80E for the A.Y.2022-23., Solution:, Deduction under section 80E is available to an individual assessee in respect of any interest paid by him in, the previous year in respect of loan taken for pursuing his higher education or higher education of his spouse, or children. Higher education means any course of study pursued after senior secondary examination., Therefore, interest repayment in respect of all the above loans would be eligible for deduction., Deduction under section 80E = `20,000 + `10,000 + `18,000 = `48,000
Page 229 :
Deduction From Gross Total Income, , 229, , Question 8 : Deduction in respect of interest on loan taken for certain house property., Answer: Deduction in respect of interest on loan taken for certain house property. Section 80EEA., Deduction is allowed only to an individual in connection with interest for the loan taken for purchase or, construction of a residential house and such loan should have been sanctioned by the financial institution, from 1st April 2019 to 31st March 2022. Stamp duty value of such house should not exceed 45 lakhs. The, Assessee should not have any residential house property in his name on the date of sanction of loan., Maximum deduction allowed shall be equal to the interest payable on such loan but maximum `1,50,000 per, year (this is in addition to deduction u/s 24(b)). Loan should have been taken from any financial institution,, which includes banks and housing finance company. Housing finance companies means a public company, formed or registered in India with the main object of carrying on the business of providing long-term finance, for construction or purchase of houses in India for residential purposes., Original Text, (1) In computing the total income of an assessee, being an individual, there shall be deducted, in accordance, with and subject to the provisions of this section, interest payable on loan taken by him from any financial, institution for the purpose of acquisition of a residential house property., (2) The deduction under sub-section (1) shall not exceed one lakh and fifty thousand rupees and shall be, allowed in computing the total income of the individual for the assessment year beginning on the 1st day of, April, 2020 and subsequent assessment years., (3) The deduction under sub-section (1) shall be subject to the following conditions, namely:—, (i) the loan has been sanctioned by the financial institution during the period beginning on the 1st day of, April, 2019 and ending on the 31st day of March, 2022;, (ii) the stamp duty value of residential house property does not exceed forty-five lakh rupees;, (iii) the assessee does not own any residential house property on the date of sanction of loan., (4) Where a deduction under this section is allowed for any interest referred to in sub-section (1), deduction, shall not be allowed in respect of such interest under any other provision of this Act for the same or any, other assessment year., (5) For the purposes of this section,––, The expression “stamp duty value” means value adopted or assessed or assessable by any authority of the, Central Government or a State Government for the purpose of payment of stamp duty in respect of an, immovable property., "financial institution" means a banking company to which the Banking Regulation Act, 1949 applies, or any bank, or banking institution referred to in section 51 of that Act or a housing finance company;, "housing finance company" means a public company formed or registered in India with the main object of, carrying on the business of providing long-term finance for construction or purchase of houses in India for, residential purposes.], , Illustration 14: Mr X has taken a loan on 01.05.2021 from State Bank of India `40,00,000 @ 10% p.a. and, purchased one house which is self - occupied. He has submitted a certificate confirming the amount of, interest. He repaid `2,00,000 on 01.03.2022 he don’t have any other house. He paid municipal tax `30,000., He has income under the head salary `10,00,000. Compute his total income and tax liability., (b)Presume it is let out @ `10,000 per month from 01.05.2021., Answer:, (a), Computation of Total Income of Mr. X, Income under the head house property, GAV, Less: Municipal Tax, NAV, , Nil, Nil, Nil
Page 230 :
Deduction From Gross Total Income, , 230, , Less: Standard Deduction u/s 24(a) @ 30%, Less: Interest on Loan u/s 24(b), (`40,00,000 x 10% x 10/12 = 3,33,333.33, `38,00,000 x 10% x 1/12 =, 31,666.67 = 3,65,000 but limited to 2,00,000), Loss under the head house property, , (2,00,000), , Income under the head salary, Less: Loss of House property can be set off from salary income, Income under the head salary, , 10,00,000, (2,00,000), 8,00,000, , Gross total Income, Less: Repayment of housing loan u/s 80C, Less: Deduction u/s 80EEA for interest (3,65,000-2,00,000 but limited to 1,50,000), Total Income, , 8,00,000, (1,50,000), (1,50,000), 5,00,000, , Computation of Tax Liability, Tax on `5,00,000 at slab rate, Less: Rebate u/s 87A, Tax Liability, (b), Computation of Total Income of Mr. X, Option 1: Interest is claimed under section 24(b), Income under the head house property, GAV (10,000 x 11), Less: Municipal Tax, NAV, Less: Standard Deduction u/s 24(a) @ 30%, Less: Interest on Loan u/s 24(b), (`40,00,000 x 10% x 10/12 = 3,33,333.33, `38,00,000 x 10% x 1/12 =, 31,666.67 = 3,65,000), Loss under the head house property, , Nil, (2,00,000), , 12,500, (12,500), Nil, , 1,10,000, (30,000), 80,000, (24,000), (3,65,000), (3,09,000), , Income under the head salary, Less: Loss of House Property, House property loss can be set off from salary head income but maximum allowed 2,00,000, and balance can be carried forward, Income under the head salary, , 10,00,000, (2,00,000), , Gross Total Income, Less: Repayment of housing loan u/s 80C, Less: Deduction u/s 80EEA for interest, Total Income, Computation of Tax Liability, Tax on `6,50,000 at slab rate, Add: HEC @ 4%, Tax Liability, , 8,00,000, (1,50,000), Nil, 6,50,000, , 8,00,000, , 42,500, 1,700, 44,200, , (b), Computation of Total Income of Mr. X, Option 2: Interest of 2,15,000 is claimed u/s 24 (b) and balance is claimed u/s 80EEA, Income under the head house property, GAV (10,000 x 11), , 1,10,000
Page 231 :
Deduction From Gross Total Income, Less: Municipal Tax, NAV, Less: Standard Deduction u/s 24(a) @ 30%, Less: Interest on Loan u/s 24(b) (claimed maximum 2,15,000), (`40,00,000 x 10% x 10/12 = 3,33,333.33, `38,00,000 x 10% x 1/12 =, 31,666.67 = 3,65,000), Loss under the head house property, , 231, (30,000), 80,000, (24,000), (2,15,000), (1,59,000), , Income under the head salary, Less: Loss of House Property, (House property loss can be setoff from salary head income), Income under the head salary, , 10,00,000, (1,59,000), , Gross total Income, Less: Repayment of housing loan u/s 80C, Less: Deduction u/s 80EEA for interest (3,65,000-2,15,000), Total Income, , 8,41,000, (1,50,000), (1,50,000), 5,41,000, , 8,41,000, , Computation of Tax Liability, Tax on `5,41,000 at slab rate, Add: HEC @ 4%, Tax Liability, , 20,700, 828, 21,528, , Rounded off u/s 288B, , 21,530, , Illustration 15:, The following are the particulars relating to Mr. A, Mr. B, Mr. C and Mr. D, salaried individuals for A.Y., 2022-23 –, Particulars, Mr. A, Mr. B, Amount of loan taken, ` 43 lakhs, ` 45 lakhs, Loan taken from, HFC, Deposit taking NBFC, Date of sanction of loan, 01.04.2021, 01.04.2021, Date of disbursement of loan, 01.05.2021, 01.05.2021, Purpose of loan, Acquisition of residential, Acquisition of residential, house property for selfhouse property for selfoccupation, occupation, Stamp duty value of house property, ` 45 lakhs, ` 48 lakhs, Cost of electric vehicle, –, –, Rate of interest, 9% p.a., 9% p.a., Compute the amount of deduction, if any, allowable under the provisions of the Income-tax Act, 1961 for, A.Y.2022-23 in the hands of Mr. A, Mr. B. Assume that there has been no principal repayment during the, P.Y.2021-22., Solution:, Particulars, Mr. A, Interest deduction for A.Y.2022-23, (i) Deduction allowable while computing income under the head “Income from house, property”, Deduction u/s 24(b) ` 3,54,750, , `
Page 232 :
Deduction From Gross Total Income, [` 43,00,000 X 9% X 11/12], Restricted to, (ii) Deduction under Chapter VI-A from Gross Total Income, Deduction u/s 80EEA ` 1,54,750, (` 3,54,750 – ` 2,00,000), Restricted to, , 232, , 2,00,000, , 1,50,000, , Mr. B, Interest deduction for A.Y.2022-23, (i) Deduction allowable while computing income under the head “Income from house, property”, Deduction u/s 24(b) ` 3,71,250, [` 45,00,000 X 9% X 11/12], 2,00,000, Restricted to, (ii) Deduction under Chapter VI-A, Nil, Deduction u/s 80EEA is not permissible since Stamp duty value exceeds ` 45 lakh., Question 9: Deduction in respect of purchase of electric vehicle., Answer: Deduction in respect of purchase of electric vehicle. Section 80EEB., (1) In computing the total income of an assessee, being an individual, there shall be deducted, in accordance, with and subject to the provisions of this section, interest payable on loan taken by him from any financial, institution for the purpose of purchase of an electric vehicle., (2) The deduction under sub-section (1) shall not exceed one lakh and fifty thousand rupees and shall be, allowed in computing the total income of the individual for the assessment year beginning on the 1st day of, April, 2020 and subsequent assessment years., (3) The deduction under sub-section (1) shall be subject to the condition that the loan has been sanctioned by, the financial institution during the period beginning on the 1st day of April, 2019 and ending on the 31st day, of March, 2023., (4) Where a deduction under this section is allowed for any interest referred to in sub-section (1), deduction, shall not be allowed in respect of such interest under any other provision of this Act for the same or any, other assessment year., (5) For the purposes of this section,––, (a) “electric vehicle” means a vehicle which is powered exclusively by an electric motor whose traction, energy is supplied exclusively by traction battery installed in the vehicle and has such electric regenerative, braking system, which during braking provides for the conversion of vehicle kinetic energy into electrical, energy;, (b) “financial institution” means a banking company to which the Banking Regulation Act, 1949 applies, or, any bank or banking institution referred to in section 51 of that Act and includes any deposit taking nonbanking financial company or a systemically important non-deposit taking non-banking financial company, as defined in clauses (e) and (g) of Explanation 4 to section 43B., Illustration 16:, The following are the particulars relating to Mr. A, Mr. B, Mr. C and Mr. D, salaried individuals for A.Y., 2022-23 –, Particulars, Mr. X, Mr. Y, Amount of loan taken, ` 20 lakhs, ` 15 lakhs, Loan taken from, Deposit taking NBFC, Public sector bank, Date of sanction of loan, 01.04.2021, 31.03.2019, Date of disbursement of loan, 01.05.2021, 01.05.2019, Purpose of loan, Purchase of electric vehicle Purchase of electric vehicle, for personal use, for personal use
Page 233 :
Deduction From Gross Total Income, Stamp duty value of house property, Cost of electric vehicle, Rate of interest, , 233, , –, ` 22 lakhs, 10% p.a., , –, ` 18 lakhs, 10% p.a., , Compute the amount of deduction, if any, allowable under the provisions of the Income-tax Act, 1961 for, A.Y.2022-23 in the hands of Mr. X, Mr. Y. Assume that there has been no principal repayment during the, P.Y.2021-22., Solution:, Particulars, `, Mr. X, Deduction under Chapter VI-A, Deduction u/s 80EEB for interest payable on loan taken for purchase of electric vehicle [` 20 1,50,000, lakhs X 10% X 11/12 = ` 1,83,333, restricted to ` 1,50,000, being the maximum permissible, deduction], Mr. Y, Deduction under Chapter VI-A, Deduction u/s 80EEB is not permissible since loan was sanctioned before 01.04.2019, , Nil, , Question 10 [V. Imp.]: Write a note on deduction in case of donations to the Notified Institutions., Answer: Deduction in respect of donations to certain Funds, Charitable Institutions, etc. Section 80G, Deduction is allowed to all the assessees if they have given any donation or contribution to any of the below, mentioned institutions or funds and deduction allowed shall be either 100% or 50% and for this purpose,, the organisations shall be divided into four categories, Category A Deduction is allowed equal to 100% of donation, 1. The Prime Minister’s National Relief Fund, 2. The Prime Minister’s Armenia Earthquake Relief Fund, 3. The National Foundation for Communal Harmony, 4. The National Defence Fund, 5. The National Children’s Fund, 6. The Africa Fund, 7. A University or any educational institution of national eminence as may be approved by the prescribed, authority in this behalf, 8. The Chief Minister’s Earthquake Relief Fund, Maharashtra, 9. The Andhra Pradesh Chief Minister’s Cyclone Relief Fund, 1996, 10. Any fund set up by the State Government of Gujarat for providing relief to the victims of earthquake, 11. The Chief Minister’s Relief Fund, 12. The Lieutenant Governor’s Relief Fund in respect of any Union territory, 13. Zila Saksharta Samiti., 14. The National Blood Transfusion Council, 15. The State Blood Transfusion Council., 16. The National Illness Assistance Fund, 17. The Army Central Welfare Fund, 18. The Air Force Central Welfare Fund, 19. The Indian Naval Benevolent Fund, 20. The National Sports Fund, 21. The National Cultural Fund, 22. The Fund for Technology Development and Application, 23. Any fund set up by a State Government to provide medical relief to the poor, 24. The National Trust for Welfare of Persons suffering with Autism, Cerebral Palsy, Mental Retardation, and Multiple Disabilities.
Page 234 :
Deduction From Gross Total Income, , 234, , 25. Swachh Bharat Kosh, 26. Clean Ganga Fund, 27. National Fund for Control of Drugs, 28. The Prime Minister’s Citizen Assistance and Relief in Emergency Situations Fund (PM CARES, FUND), Category B Deduction shall be allowed equal to 50% of Donation, 1. The Jawaharlal Nehru Memorial Fund, 2. The Indira Gandhi Memorial Trust,, 3. The Rajiv Gandhi Foundation, 4. The Prime Minister’s Drought Relief Fund, Category C Deduction shall be allowed equal to 100% of Qualifying amount (Not Donation), 1. Donation to government or to any such local authority (e.g. MCD), Institution or association as may be, approved by the central government and donation should be utilized for the purpose of promoting, family planning., 2. any sums paid by the assessee, being a company, in the previous year as donations to the Indian Olympic, Association or to any other association or institution established in India, as the Central Government may,, having regard to the prescribed guidelines, by notification in the Official Gazette, specify in this behalf, for—(i) the development of infrastructure for sports and games; or (ii) the sponsorship of sports and games,, in India., Category D Deduction shall be allowed equal to 50% of Qualifying amount (Not Donation), 1. the Government or any local authority, to be utilised for any charitable purpose other than the purpose of, promoting family planning; or, 2. Donation to an authority constituted in India for the purpose of dealing with and satisfying the need for, housing accommodation or for the purpose of planning, development or improvement of cities, towns and, villages, or for both. (like DDA), 3. donations for the renovation or repair of any such temple, mosque, gurdwara, church or other place as is, notified by the Central Government in the Official Gazette to be of historic, archaeological or artistic, importance or to be a place of public worship of renown throughout any State or States., 4. Donation to any other fund or any institution notified under section 80G like social, religious, charitable or, other similar organisations., Qualifying amount = 10% of the adjusted gross total income or the donation (except donation to the above, mentioned 28+4 funds) given, whichever is less., Adjusted gross total income = Gross Total Income – Long term capital Gains (including LTCG u/s 112A) –, Short term capital gains u/s 111A – All Deduction under section 80C to 80U except section 80G, No deduction shall be allowed under this section in respect of any donation unless such donation is of a sum, of money i.e. if donation is given in kind, deduction is not allowed., No deduction shall be allowed under this section in respect of donation of any sum exceeding two, thousand rupees unless such sum is paid by any mode other than cash., Illustration 17: Mr. X has income from business/profession `6,00,000 and long term capital gain `4,00,000, and short term capital gain u/s 111A `2,00,000 and casual income `1,00,000., He has paid premium of a mediclaim policy amounting to `20,000 taken in the name of his dependant grand, father who is senior citizen and payment was made by a cheque on 09.01.2022., He has given premium of Jeevan Suraksha policy `7,000, has donated `12,000 to the National Defence, Fund, `4,000 to Rajiv Gandhi Foundation and `3,00,000 to a charitable institution and `1,00,000 to a social, organization and `4,00,000 to religious organization and all such organization are notified under section, 80G. (all the donations was made by cheque), Compute his total income and tax liability for A.Y. 2022-23., (b) Presume in the above question the assessee has given donation of `10,000 by cheque for family planning, also to the Government., Compute his total income and tax liability for the assessment year 2022-23., (c) Presume in part (b), donation to government for family planning is `2,00,000 by cheque., Solution 17(a):, `, Income under the head Business/Profession, 6,00,000.00
Page 235 :
Deduction From Gross Total Income, Income under the head Capital Gain (LTCG), Income under the head Capital Gain (STCG u/s 111A), Income under the head Other Sources (casual income), Gross Total Income, Less: Deduction u/s 80CCC, Less: Deduction u/s 80G, (i) National Defence Fund, (ii) Rajiv Gandhi Foundation, (iii) Charitable Institution/ Social organization/ Religious organization, Working Note:, Charitable Institution, Social organization, Religious organization, , 235, 4,00,000.00, 2,00,000.00, 1,00,000.00, 13,00,000.00, (7,000.00), (12,000.00), (2,000.00), (34,650.00), 3,00,000, 1,00,000, 4,00,000, 8,00,000, , AGTI = GTI – LTCG – STCG u/s 111A – Deduction u/s 80C to 80U (except 80G), = 13,00,000 – 4,00,000 – 2,00,000 – 7,000, = 6,93,000, Qualifying amount = 10% of AGTI or donation whichever is less, = 69,300 or 8,00,000 whichever is less, = 69,300, 50% of the qualifying amount = 34,650, Total Income, Computation of Tax Liability, Tax on casual income `1,00,000 @ 30% u/s 115BB, Tax on STCG `2,00,000 @ 15% u/s 111A, Tax on LTCG `4,00,000 @ 20% u/s 112, Tax on normal income `5,44,350 at slab rate, Tax before health & education cess, Add: HEC @ 4%, Tax Liability, Rounded off u/s 288B, Solution 17(b):, Computation of Total Income, Gross Total Income, Less: Deduction u/s 80CCC, Less: Deduction u/s 80G, (i) National Defence Fund, (ii) Rajiv Gandhi Foundation, (iii) Other Donations u/s 80G, Working Note:, Charitable Institution, Social organization, religious organization, Family planning, , `, 3,00,000, 1,00,000, 4,00,000, 10,000, 8,10,000, AGTI = GTI – LTCG – STCG u/s 111A – Deduction u/s 80C to 80U (except 80G), = 13,00,000 – 4,00,000 – 2,00,000 – 7,000, = 6,93,000, Qualifying amount = 10% of AGTI or donation whichever is less, = 69,300 or 8,10,000 whichever is less, , 12,44,350.00, 30,000.00, 30,000.00, 80,000.00, 21,370.00, 1,61,370.00, 6,454.80, 1,67,824.80, 1,67,820.00, , 13,00,000.00, (7,000.00), (12,000.00), (2,000.00), (39,650.00)
Page 236 :
Deduction From Gross Total Income, = 69,300, Deduction of donation for family planning `10,000, 50% of balance amount (i.e. `59,300) `29,650, Total deduction allowed 10,000 + 29,650 = 39,650, Total Income, Computation of Tax Liability, Tax on casual income `1,00,000 @ 30% u/s 115BB, Tax on STCG `2,00,000 @ 15% u/s 111A, Tax on LTCG `4,00,000 @ 20% u/s 112, Tax on normal income `5,39,350 at slab rate, Tax before health & education cess, Add: HEC @ 4%, Tax Liability, Rounded off u/s 288B, Solution 17(c):, Computation of Total Income, Gross Total Income, Less: Deduction u/s 80CCC, Less: Deduction u/s 80G, (i) National Defence Fund, (ii) Rajiv Gandhi Foundation, (iii) Other donations u/s 80G, Working Note:, Charitable Institution, Social organization, religious organization, Family planning, , `, 3,00,000, 1,00,000, 4,00,000, 2,00,000, 10,00,000, AGTI = GTI – LTCG – STCG u/s 111A – Deduction u/s 80C to 80U (except 80G), = 13,00,000 – 4,00,000 – 2,00,000 – 7,000, = 6,93,000, Qualifying amount = 10% of AGTI or donation whichever is less, = 69,300 or 10,00,000 whichever is less, = 69,300, Donation for family planning is `2,00,000 but maximum deduction allowed shall be, `69,300, Total Income, Computation of tax liability, Tax on casual income `1,00,000 @ 30% u/s 115BB, Tax on STCG `2,00,000 @ 15% u/s 111A, Tax on LTCG `4,00,000 @ 20% u/s 112, Tax on normal income `5,09,700 at slab rate, Tax before health & education cess, Add: HEC @ 4%, Tax Liability, Rounded off u/s 288B, , 236, , 12,39,350.00, 30,000.00, 30,000.00, 80,000.00, 20,370.00, 1,60,370.00, 6,414.80, 1,66,784.80, 1,66,780.00, `, 13,00,000.00, (7,000.00), (12,000.00), (2,000.00), (69,300.00), , 12,09,700.00, 30,000.00, 30,000.00, 80,000.00, 14,440.00, 1,54,440.00, 6,177.60, 1,60,617.60, 1,60,620.00, , MAY – 2017, Question 6(a) (ii), (4 Marks), Mr. Rohan, a resident individual has Gross Total Income of `7,50,000 comprising of Income from Salary, and income from house property for the assessment year 2022-23. He provides the following information:
Page 237 :
Deduction From Gross Total Income, , 237, , Paid `70,000 towards premium on life insurance policy of his Handicapped Son (Section 80U disability)., Sum assured `4,00,000 ; and date of issue of policy 1-8-2018., Deposited `90,000 in tax saver deposit in the name of his major son in State Bank of India., Contributed by cheque `25,000 to The Clean Ganga Fund, set up by the Central Government., Compute the Total Income and deduction under Chapter VI-A for the Assessment year 2022-23., Solution:, Computation of Total Income and deduction under chapter VI-A, Gross Total Income, Less: Deduction under chapter VI-A, Deduction u/s 80C for LIC Premium (15% of 4,00,000), Deposited in Tax Saver Deposit in the name of major son, Deduction u/s 80G – Contribution in Clean Ganga Fund (100%), Total Income, , `, 7,50,000, (60,000), (Nil), (25,000), 6,65,000, , Note: Tax Saver deposits in the name of major son does not qualify for deduction u/s 80C since such, deposits has to be made in the name of assessee himself., NOV – 2008 (4 Marks), Mr. X declares gross total income `4,00,000 for the assessment year 2022-23. The gross total income, includes taxable long term capital gain ` 65,000 and short term capital gain `35,000 which is taxable @, 15% under section 111A of the Income-tax Act, 1961. The details of fund investment made during the year, 2021-22 are:, `, (i) Medical insurance premium paid by cheque –, (a) in the name of Mr. X, 4,000, (b) in name of Mrs. X, 5,000, (ii) Contribution made to –, (a) Indira Gandhi Memorial Trust by cheque, 7,000, (b) Delhi University (declared as an institution of national eminence) by cheque, 3,000, (c) Zila Saksharta Samiti by cheque, 5,000, (d) An approved charitable institute by cheque, 30,000, (e) Government by cheque for the purpose of promoting family planning, 10,000, (f) Hanuman Temple in Mohalla by cheque, 20,000, Compute the total income of Mr. X chargeable to tax for the Assessment year 2022-23 and also compute his, tax liability., Answer., (`), Computation of Total Income of Mr. X for the A.Y. 2022-23, Gross Total Income, 4,00,000.00, Less : Deduction, Section 80D, Medical insurance premium paid by cheque, (i) in the name of Mr. Prasad, (4,000.00), (ii) in name of Mrs. Prasad, (5,000.00), Section 80G, Donation to Indira Gandhi Memorial trust @ 50% of ` 7,000, (3,500.00), Donation to Delhi University @ 100%, (3,000.00), Donation to Zila Saksharta Samiti @ 100%, (5,000.00), Other donations u/s 80G, (19,550.00), Working Note:, Donation to approved Charitable Institute, 30,000, Donation to Government for promoting family planning 10,000, 40,000
Page 238 :
Deduction From Gross Total Income, AGTI = GTI – LTCG – STCG u/s 111A – Deduction u/s, 80C to 80U (Except 80G), = 4,00,000 – 65,000 – 35,000 – 9,000, = 2,91,000, Qualifying amount = 10% of AGTI or donation whichever is less, = 29,100 or 40,000, = 29,100, Deduction for family planning `10,000, 50% of balance amount (i.e. 19,100), = 9,550, Total deduction = 10,000 + 9,550 = 19,550, Total Income, Computation of Tax Liability, Tax on LTCG `65,000 @ 20% u/s 112, Tax on STCG `35,000 @ 15% u/s 111A, Tax on `2,59,950 at slab rate, Less: Rebate u/s 87A, Tax before health & education cess, Add: HEC @ 4%, Tax Liability, Rounded off u/s 288B, , 238, , 3,59,950.00, 13,000.00, 5,250.00, 497.50, (12,500.00), 6,247.50, 249.90, 6,497.40, 6,500.00, , NOV – 2002 (4 Marks), For the assessment year 2022-23, the gross total income of Mr. X was `4,50,240 which includes long term, capital gain of `45,000 and short term capital gain of `80,000.The gross total income also includes interest, income (fixed deposits) from banks of `12,000., Mr. X has invested in public provident fund `60,000 and also paid medical insurance premium `11,000 by, cheque. Mr. X also contributed `15,000 by cheque to public charitable trust eligible for deduction u/s 80G., Compute the total income and tax thereon of Mr. X, who is 50 years old as on 31.03.2022., Answer:, Computation of Total Income, `, Long Term Capital Gain, 45,000.00, Short Term Capital Gain, 80,000.00, Bank Interest, 12,000.00, Other Income, 3,13,240.00, Gross Total Income, 4,50,240.00, Less: Deduction u/s 80C {Public Provident Fund}, (60,000.00), Less: Deduction u/s 80D, (11,000.00), Less: Deduction u/s 80G, (7,500.00), Working Note:, AGTI = GTI – LTCG – Deductions u/s 80C to 80U (Except 80G), = 4,50,240 – 60,000 – 45,000 – 11,000, = `3,34,240, Qualifying Amount = 10% of AGTI or Donation given whichever is less, = `33,424 or 15,000, = 15,000, 50% of qualifying amount = `7,500, Total Income, 3,71,740.00, Computation of Tax Liability, Tax on long term capital gain `45,000 @ 20% u/s 112, 9,000.00, Tax on normal income `3,26,740 at slab rate, 3,837.00, Less: Rebate u/s 87A, (12,500.00)
Page 239 :
Deduction From Gross Total Income, Tax before HEC, Add: HEC @ 4%, Tax liability, Rounded off u/s 288B, , 239, 337.00, 13.48, 350.48, 350.00, , Question 11 [V. Imp.]: Write a note on deduction in case of payment of rent., Answer: Deduction in case of payment of rent Section 80GG, 1. Deduction is allowed only to an individual., 2. He should not be getting any house rent allowance and also he is not being provided with Rent Free, Accommodation by his employer., 3. He should not have any house in his name or in the name of the spouse or in the name of minor child, or in the name of Hindu Undivided Family of which he is a member, at a place where he ordinarily, resides or performs duties of his office or carries on his business or profession., 4. The assessee may have house at any other place but it should not be self occupied i.e. it may be let out or, vacant., 5. He has paid rent for the accommodation taken by him for his residence., 6. Deduction shall be allowed to such individual in case of payment of rent and deduction shall be allowed, to the extent of the least of the following:, (i) Rent paid over 10% of the adjusted gross total income, (ii) `5,000 p.m., (iii) 25% of the adjusted gross total income, Adjusted Gross Total Income = Gross Total Income – Long term capital gains (including LTCG u/s, 112A) – Short term capital gains u/s 111A – All Deduction of section 80C to 80U except section 80GG., • Deduction can be allowed even where the assessee is not an employee i.e. the persons having, business/profession can also avail deduction under section 80GG., Illustration 18: Mr. X has income under the head Business/Profession `5,00,000 and LTCG of `2,00,000,, STCG u/s 111A `3,00,000 and casual income of `1,00,000., He is paying rent for a house of `40,000 p.m. He has deposited `30,000 in home loan account scheme of, National Housing Bank., He has complied with all the condition of section 80GG., Compute income tax liability for A.Y. 2022-23., Solution:, `, Income under the head Business/Profession, 5,00,000, Computation of income under the head Capital Gain, Long Term Capital Gain, 2,00,000, Short Term Capital Gain u/s 111A, 3,00,000, Income under the head capital gain, 5,00,000, Computation of income under the head Other Sources, Casual income, 1,00,000, Income under the head Other Sources, 1,00,000, Gross Total Income, 11,00,000, Less:, Deduction u/s 80C, (30,000), Deduction u/s 80GG, (60,000), Working Note:, Least of the following:, 1. `4,80,000 – 57,000 = 4,23,000, 2. 60,000, 3. 25% x 5,70,000 = 1,42,500, AGTI = GTI – LTCG – STCG 111A – Deduction u/s 80C to 80U, (except 80GG), = 11,00,000 – 2,00,000 – 3,00,000 – 30,000
Page 240 :
Deduction From Gross Total Income, = 5,70,000, Total Income, Computation of Tax Liability, Tax on casual income ` 1,00,000 @ 30%, Tax on LTCG `2,00,000 @ 20%, Tax on STCG 111A ` 3,00,000 @ 15%, Tax on `4,10,000 at slab rate, Tax before health & education cess, Add: HEC @ 4%, Tax Liability, , 240, , 10,10,000, 30,000, 40,000, 45,000, 8,000, 1,23,000, 4,920, 1,27,920, , MAY – 2000 (4 Marks), Mr. X is a retired Government officer aged 40 years, who derived the following income in respect of, financial year 2021-22. He resides in Cochin:, `, Salary, 6,00,000, Interest from bank deposits (fixed deposits), 2,00,000, He has paid `28,000 as premium to effect an insurance on his health and it was paid by a cheque. He pays a, rent of `10,000 per month in respect of furnished accommodation. He is eligible for deduction under Section, 80GG. Compute his total income and tax liability for assessment year 2022-23., What are the conditions to be satisfied by him to qualify for the deduction?, Answer:, `, Salary, 6,00,000.00, Less: Deduction u/s 16(ia), (50,000.00), Income under the head Salary, 5,50,000.00, Income under the head Other Sources {Bank Interest}, 2,00,000.00, Gross Total Income, 7,50,000.00, Less: Deduction under section 80D, (25,000.00), Less: Deduction under section 80GG, (1,81,250.00), Working Note:, Least of the following:, 1. `1,20,000 – `72,500 = `47,500, 2. `60,000, 3. 25% x 7,25,000 = `1,81,250, (AGTI = `7,50,000 – `25,000 = `7,25,000), Total Income, 5,43,750.00, Computation of Tax Liability, Tax on `5,43,750 at slab rate, 21,250, Add: HEC @ 4%, 850, Tax Liability, 22,100, Conditions to be fulfilled for grant of deduction under section 80GG :, 1. The assessee should not be getting any house rent allowance and also he is not being provided with Rent, Free Accommodation by his employer., 2. The assessee should not have any house in his name or in the name of the spouse or in the name of minor, child or in the name of Hindu Undivided Family of which he is a member, at a place where he ordinarily, resides or performs duties of his office or employment or carries on his business or profession., 3. Also he should not have house even at any other place which he has declared to be self occupied., 4. The assessee has paid rent for the accommodation taken by him for his residence.
Page 241 :
Deduction From Gross Total Income, , 241, , Question 12: Write a note on deductions under section 80GGA in respect of donations etc to certain, notified institutions., Answer: Deduction in case of certain donation Section 80GGA, Deduction is allowed to all the assessees except the assessees whose gross total income includes income, which is chargeable under the head “Profits and gains of business or profession”. (because such assesses is, allowed to debit the amount to profit and loss account of business/profession), Deduction is allowed in case of donation or contributions to any of the below mentioned institutions., Deduction allowed is equal to the amount of donations., (i) Donation given to an institution notified under section 35 for scientific research / research in social, science or statistical research., (ii) Donation given to an institution notified under section 35AC for eligible project i.e. the projects of, social or economic importance like construction of houses for the poor person or taking up drinking, water project or other similar projects., (iii) Donation given to an institution notified under section 35CCA for rural development including, donation to Rural Development Fund setup by central Government or donation to National, Urban Poverty Eradication Fund., No deduction shall be allowed under this section in respect of any sum exceeding two thousand rupees, unless such sum is paid by any mode other than cash and deduction shall be allowed to the donor only, on the basis of the statement filed by the payee., Example:, (i) Mr. X has donated `2,00,000 by cheque to an institution notified under section 35AC for eligible projects, and Mr. X do not have any business/profession, in this case he will be allowed deduction under section, 80GGA for `2,00,000 provided the payee has filed the statement but if he has business/profession, he will, not be allowed deduction under section 80GGA rather he will be allowed to debit the amount to profit and, loss account., (ii) ABC Ltd. has donated `2,00,000 by cheque to an institution notified under section 35CCA for rural, development and company has business/profession, in this case deduction under section 80GGA is not, allowed but company can debit the amount to profit and loss account., Illustration 19: Mr. X has income under the head salary `6,00,000 and income under the head house, property `7,00,000 and he has submitted information as given below:, (i) Paid premium of life policy `40,000 (sum assured `1,50,000) and policy has taken before 01.04.2012 in, the name of Mr. X, (ii) Paid premium of life policy `40,000 (sum assured `1,50,000) and policy has taken after 01.04.2012 in, the name of Mrs. X, (iii) Paid premium of life policy `40,000 (sum assured `1,50,000) and policy has taken before 01.04.2012 in, the name of father of Mr. X who is dependant on Mr. X., (iv) Paid premium of life policy `40,000 (sum assured `1,50,000) and policy has taken before 01.04.2012 in, the name of son of Mr. X who is not dependant on Mr. X., (v) He has donated `1,00,000 by cheque in rural development fund setup by government., (vi) He has paid premium of Jeevan Suraksha Policy `10,000 by cheque in the name of Mrs. X., (vii) He has paid `15,000 in cash in connection with preventive health checkup for his father., (viii) He has donated `60,000 by cheque to a charitable institution notified under section 80G, Compute his tax liability assessment year 2022-23., Solution:, `, Income under the head Salary, 6,00,000, Income under the head House Property, 7,00,000, Gross Total Income, 13,00,000, Less: Deduction u/s 80C, Premium of life policy in name of Mr. X, (30,000), Premium of life policy in name of Mrs. X, (15,000), Premium of life policy in name of Son, (30,000), Less: Deduction u/s 80D (Preventive Health Checkup), (5,000)
Page 242 :
Deduction From Gross Total Income, Less: Deduction u/s 80GGA, Less: Deduction u/s 80G, Charitable Institution, Working Note:, AGTI = GTI – Deductions u/s 80C to 80U (Except 80G), = 13,00,000 – 30,000 – 15,000 – 30,000 – 1,00,000 – 5,000, = `11,20,000, Qualifying Amount = 10% of AGTI or Donation given whichever is less, = `1,12,000 or 60,000, = 60,000, 50% of qualifying amount = `30,000, Total Income, Computation of Tax Liability, Tax on `10,90,000 at slab rate, Add: HEC @ 4%, Tax Liability, , 242, (1,00,000), (30,000), , 10,90,000, 1,39,500, 5,580, 1,45,080, , MAY – 2011 (3 Marks), Mr. X having gross total income of `6,35,000 for the financial year 2021-22 furnishes you the following, information:, (i), Paid `25,000 towards premium on life insurance policy of his married daughter., (ii), Contributed `10,000 by cheque to Prime Minister’s National Relief Fund., (iii) Donated `20,000 to a Government recognized institution (by cheque) for scientific research., Note: Assume that the gross total income of Mr. X does not include any income under the head ‘profits and, gains of business or profession’., Compute the total income of Mr. X for the assessment year 2022-23., Answer., Computation of total income of Mr. X for the A.Y. 2022-23, Particulars, `, `, 6,35,000, Gross Total Income, Less: Deductions under Chapter VI-A, 25,000, (i) Premium on life insurance policy of his married daughter - Eligible for, deduction under section 80C, 10,000, (ii) Contribution of `10,000 to PM’s National Relief Fund - Eligible for 100%, deduction under section 80G, 20,000, (iii) Payment of `20,000 to a Government recognized institution for scientific, (55,000), research - Eligible for deduction under section 80GGA, 5,80,000, Total Income, Question 13: Write a note on deduction in case of donation to the political parties or an Electoral, trust., Answer: Deduction in respect of contributions given by companies to political parties Section 80GGB, (i) Deduction & Conditions: This section provides for deduction of any sum contributed in the previous, year by an Indian company to any political party or an electoral trust. However, no deduction shall be, allowed in respect of any sum contributed by way of cash., (ii) Meaning of “Contribute”: For the purposes of this section, the word “contribute” has the same, meaning assigned to it under section 182 of the Companies Act, 2013, which provides that (a) a donation or subscription or payment given by a company to a person for carrying on any activity which, is likely to effect public support for a political party shall also be deemed to be contribution for a political, purpose;, (b) the expenditure incurred, directly or indirectly, by a company on advertisement in any publication (being, a publication in the nature of a souvenir, brochure, tract, pamphlet or the like) by or on behalf of a political
Page 243 :
Deduction From Gross Total Income, , 243, , party or for its advantage shall also be deemed to be a contribution to such political party or a contribution, for a political purpose to the person publishing it., (iii) Meaning of “Political party”: It means a political party registered under section 29A of the, Representation of the People Act, 1951., Illustration 20: During the P.Y.2021-22, ABC Ltd., an Indian company,, (1) contributed a sum of `2 lakh to an electoral trust; and, (2) incurred expenditure of `25,000 on advertisement in a brochure of a political party., Is the company eligible for deduction in respect of such contribution/expenditure, assuming that the, contribution was made by cheque? If so, what is the quantum of deduction?, Solution:, An Indian company is eligible for deduction under section 80GGB in respect of any sum contributed by it in, the previous year to any political party or an electoral trust. Further, the word “contribute” in section 80GGB, has the meaning assigned to it in section 182 of the Companies Act, 2013, and accordingly, it includes the, amount of expenditure incurred on advertisement in a brochure of a political party., Therefore, ABC Ltd. is eligible for a deduction of `2,25,000 under section 80GGB in respect of sum of `2, lakh contributed to an electoral trust and `25,000 incurred by it on advertisement in a brochure of a political, party., It may be noted that there is a specific disallowance under section 37(2B) in respect of expenditure incurred, on advertisement in a brochure of a political party. Therefore, the expenditure of `25,000 would be, disallowed while computing business income/gross total income. However, the said expenditure incurred by, an Indian company is allowable as a deduction from gross total income under section 80GGB., Deduction in respect of contributions given by any person to political parties [Section 80GGC], (i) Deduction & Conditions: This section provides for deduction of any sum contributed in the previous, year by any person to a political party or an electoral trust. However, no deduction shall be allowed in, respect of any sum contributed by way of cash., (ii) Persons not eligible for deduction: This deduction will, however, not be available to a local authority, and an artificial juridical person, wholly or partly funded by the Government., (iii) Meaning of “Political party”: It means a political party registered under section 29A of the, Representation of the People Act, 1951, MAY – 2012 (4 Marks), Explain how contributions to political parties are deductible in the hands of corporate and non-corporate, assesses under the Income tax law., Answer: refer answer given above, Question 14: Write a note on deduction in case of employment by any Person., Answer: Deduction in case of new employment Section 80JJAA, 1. Deduction is allowed to all assessee whose accounts are required to be audited., 2. Deduction shall be allowed equal to 30% of additional employee cost incurred., 3. Deduction is allowed for 3 assessment years including the assessment year in which such employment, is provided., 4. It should be a new business., 5. Emoluments should be paid through account payee cheque, an account payee bank draft or by use of, electronic clearing system through a bank account or through such other electronic modes as may be, prescribed. (Other electronic mode means Credit Card, Debit Card, Net Banking, IMPS (Immediate, Payment Service), UPI (Unified Payment Interface), RTGS (Real Time Gross Settlement), NEFT, (National Electronic Funds Transfer), and BHIM (Bharat Interface for Money) Aadhaar Pay), 6. “Additional employee cost” means total emoluments paid or payable to additional employees employed, during the previous year.
Page 244 :
Deduction From Gross Total Income, , 244, , Provided that in the case of an existing business, the additional employee cost shall be nil, if there is no, increase in the number of employees from the total number of employees employed as on the last day of the, preceding year., 7. “Additional employee” means an employee who has been employed during the previous year and whose, employment has the effect of increasing the total number of employees employed by the employer as on the, last day of the preceding year, but does not include,—, (a) an employee whose total emoluments are more than twenty-five thousand rupees per month; or, (b) an employee employed for a period of less than two hundred and forty days during the previous year., (c) an employee who does not participate in the recognised provident fund., In the case of an assessee who is engaged in the business of manufacturing of apparel or footwear or leather, products, 240 days shall be taken as 150 days., Where an employee is employed during the previous year for a period of less than two hundred and forty, days or one hundred and fifty days, as the case may be, but is employed for a period of two hundred and, forty days or one hundred and fifty days, as the case may be, in the immediately succeeding year, he shall be, deemed to have been employed in the succeeding year and the provisions of this section shall apply, accordingly., The Assessee shall be required to submit a certificate from a Chartered Accountant in form No. 10DA, certifying the amount of deduction claimed., NOV-2016 (4 Marks), Mr. Satya is a manufacturer of household goods in a factory located in Navi Mumbai and commenced his, business on 1st April 2021 and he employed 120 new work men during the previous year 2021-22 which, included:, (a) 20 employee whose total emoluments paid @ `30,000 p.m. per employee;, (b) 40 worker employed on 01st April,2021, (c) 35 worker employed on 1st May,2021, (d) 25 worker employed on 5th October,2021, Compute the Deduction under Section 80JJAA, if available to Mr. Satya for Assessment year 2022-23, if, wages are paid to each worker @ `3,000 per month. His profit from the manufacture of goods for, Assessment year 2022-23 is `9.50 lakhs. (after debiting all the wages paid), The Assessee is liable to Audit his accounts., If in the previous year 2022-23, he has given employment to 35 workmen on 01-05-2022 and each getting, salary 15,000 per month. Compute deduction allowed in previous year 2022-23., If in the previous year 2023-24, he has given employment to 40 workmen on 01-05-2023 and each getting, salary 12,000 per month. Compute deduction allowed in previous year 2023-24., Solution:, Mr. Satya is eligible for deduction under section 80JJAA since he is liable to Audit of his accounts in the, previous year 2021-22. Deduction allowed shall be as given below:, (a) 20 employee whose total emoluments paid @ `30,000 p.m. per employee - Not eligible, (b) 40 worker employed on 01st April,2021 - Eligible – 3,000 x 12 x 40 = 14,40,000, (c) 35 worker employed on 1st May,2021 Eligible – 3,000 x 11 x 35 = 11,55,000, th, (d) 25 worker employed on 5 October,2021 - will be considered next year, Total, 25,95,000, Deduction allowed @ 30%, 7,78,500, Previous year 2022-23, (a) 25 worker employed on 5th October,2021 - eligible – 3,000 x 12 x 25 =, 9,00,000, (b) New Employed in P.Y. 2022-23(01.05.2022)eligible – 15,000 x 11 x 35=, 57,75,000, Total, 66,75,000
Page 245 :
Deduction From Gross Total Income, Deduction allowed @ 30%, Previous year 2023-24, (a) New Employed in P.Y. 2023-24 (01.05.2023) eligible – 12,000 x 11 x 40=, Total, Deduction allowed @ 30%, , 245, 20,02,500, , 52,80,000, 52,80,000, 15,84,000, , Question 15. Write a note on deduction in case of royalty income from certain books., Answer: Deduction in respect of royalty income, etc., of authors of certain books other than text books, Section 80QQB, 1. Deduction is allowed only to a resident individual who is an author., 2. He should have income through his copyright in a book which is a work of literary, artistic or, scientific nature but such should not be text-books for schools/colleges etc. and also it should not be, any help book or guide etc. or any newspaper or magazine etc., 3. Deduction allowed shall be equal to the amount of royalty income or `3,00,000 whichever is less., 4. Royalty received by the author in excess of 15% of the value of such books sold during the previous, year shall be ignored., e.g. Mr. X is an author of a book of literary nature and print price is `200 and total copies sold are 3000, and he claims that he is getting royalty @ 50% of print price, in this case deduction allowed shall be, 200 x 15% x 3000 = 90,000, 5. In respect of any income earned from any source outside India, so much of the income shall be taken into, account for the purpose of this section as is brought into India by the assessee in convertible foreign, exchange within a period of six months from the end of the previous year in which such income is earned, or within such further period as the competent authority may allow in this behalf. E.g. Mr. X received a, royalty of `4,00,000 from outside India in connection with a book of literary nature but amount brought, in India within 6 months from the end of relevant previous year is `2,30,000, in this case amount to be, added to income shall be `4,00,000 but deduction allowed shall be `2,30,000., 6. The Assessee should retain information with him in form no. 10CCD and it should be produced when, demanded by the department., Illustration 21: Mrs. X is author of one book of scientific nature and its print price is `500 and total copies, sold are 2000 and she has received royalty @ 50%., She has taken a loan from State Bank in 2012 for pursuing bachelor’s degree in Engineering and she has, given repayment of principal amount `80,000 and interest `20,000 to State Bank. (payment of interest was, given for the first time in financial year 2018-19), She has paid tuition fee of her son for whole time education ` 3,000 in India, Compute Income Tax liability A.Y. 2022-23., Solution:, `, Income under the head Other Sources, 500 x 50% x 2000, 5,00,000.00, Gross Total Income, 5,00,000.00, Less: Deduction u/s 80C, Tuition Fee, (3,000.00), Less: Deduction u/s 80E, Payment of Interest, (20,000.00), Less: Deduction u/s 80QQB, 500 x 15% x 2000, (1,50,000.00), Total Income, 3,27,000.00, Computation of tax liability, Tax on `3,27,000 at slab rate, 3,850.00, Less: Rebate u/s 87A, (3,850.00), Tax Liability, Nil
Page 246 :
Deduction From Gross Total Income, , 246, , Example 1: Mr. X received royalty of `2,00,000 from abroad for a book authored by him which is a work, of artistic nature. The rate of royalty is 20% of value of books and expenditure made for earning this royalty, was `50,000. The amount remitted to India till 30th September, 2021 is `1,20,000. Compute deduction u/s, 80QQB and also compute income to be added in Gross Total Income., Solution:, Amount to be added in Income (2,00,000-50,000), 1,50,000, Deduction allowed u/s 80QQB, Deductions u/s 80QQB, 15% of value of books, 1,50,000, (2,00,000/20% x 15%), but cannot exceed amount brought in India within 6, months from the end of the previous year i.e., 1,20,000, Allowed, 1,20,000, Less: Expenses, (50,000), Deduction allowed, 70,000, , 70,000, , Example 2: Mr. X received royalty of `6,00,000 from abroad for a book authored by him which is a work, of artistic nature. The rate of royalty is 25% of value of books and expenditure made for earning this royalty, was `2,50,000. The amount remitted to India till 30th September, 2022 is `3,80,000. Compute deduction u/s, 80QQB and also compute income to be added in Gross Total Income., Solution:, Amount to be added in Income (6,00,000-2,50,000), 3,50,000, Deduction allowed u/s 80QQB, Deductions u/s 80QQB, 15% of value of books, 3,60,000, (6,00,000/25% x 15%), but cannot exceed amount brought in India within 6, months from the end of the previous year i.e., 3,80,000, Allowed, 3,60,000, Less: Expenses, (2,50,000), Deduction allowed, 1,10,000, , 1,10,000, , Example 3: Mr. X received royalty of `10,00,000 from ABC Ltd. situated in India for a book authored by, him which is a work of artistic nature. The rate of royalty is 10% of value of books and expenditure made, for earning this royalty was `5,00,000. Compute deduction u/s 80QQB and also compute income to be, added in Gross Total Income., Solution:, Amount to be added in Income (10,00,000-5,00,000), 5,00,000, Deduction allowed u/s 80QQB, Deductions u/s 80QQB, 5,00,000, Royalty received, 10,00,000, Less: Expenses, (5,00,000), Deduction allowed, 5,00,000, Example 4: Mr. X received royalty (lumpsum) of `6,00,000 from abroad for a book authored by him which, is a work of artistic nature. Expenditure made for earning this royalty was `2,50,000. The amount remitted, to India till 30th September, 2022 is `4,00,000. Compute deduction u/s 80QQB and also compute income to, be added in Gross Total Income.
Page 247 :
Deduction From Gross Total Income, Solution:, Amount to be added in Income (6,00,000-2,50,000), Deduction allowed u/s 80QQB, Deductions u/s 80QQB, Royalty received, 6,00,000, but cannot exceed amount brought in India within 6, months from the end of the previous year i.e., 4,00,000, Allowed, 4,00,000, Less: Expenses, (2,50,000), Deduction allowed, 1,50,000, , 247, 3,50,000, 1,50,000, , NOV – 2020, Mr. X received royalty of `2,88,000 from abroad for a book authored by him which is a work of artistic, nature. The rate of royalty is 18% of value of books and expenditure made for earning this royalty was, `40,000. The amount remitted to India till 30th September, 2022 is `2,30,000. Compute deduction u/s, 80QQB. Also compute tax liability if he has income under the head House Property `7,00,000., Solution: Deduction allowed u/s 80QQB, Deductions u/s 80QQB, 1,90,000, 15% of value of books, 2,40,000, (2,88,000/18% x 15%), but cannot exceed amount received within 6 months, from the end of the previous year i.e. 2,30,000, Allowed, 2,30,000, Less: Expenses, (40,000), Deduction allowed, 1,90,000, Computation of Total Income & Tax Liability, Income under the head house property, Income under the head other sources, Royalty Received, Less: Expenses, Income under the head other sources, Gross Total Income, Less: Deduction u/s 80QQB, Total Income, Computation of Tax Liability, Tax on 7,58,000 at slab rate, Add: HEC @ 4%, Tax Liability, Rounded off u/s 288B, , 7,00,000, 2,88,000, (40,000), 2,48,000, 9,48,000, (1,90,000), 7,58,000, 64,100, 2,564, 66,664, 66,660, , NOV – 2001 (3 Marks), Mr. X, a writer and a professional furnishes the following particulars for the previous year ended, 31.03.2022:, `, (a) Royalty on books (eligible for deduction u/s 80QQB), 42,000, (b) Expenditure on books, 8,000, (c) Income from profession, 3,80,000, (d) Deposited in public provident fund (15.03.2022), 70,000, You are required to compute
Page 248 :
Deduction From Gross Total Income, (i) Taxable income,, (ii) Tax payable for assessment year 2022-23., Answer:, Computation of total income and tax payable by Mr. X, Income from business/profession, Income from profession, Income under the head Business/Profession, Income under the head Other Sources, Royalty on books, Less: Expenses, Income under the head Other Sources, Gross Total Income, Less: Deduction u/s 80C, Less: Deduction u/s 80QQB, Total Income, Computation of Tax Payable, Tax on `3,10,000 at slab rate, Less: Rebate u/s 87A, Tax Payable, , 248, , `, 3,80,000, 3,80,000, 42,000, (8,000), 34,000, 4,14,000, (70,000), (34,000), 3,10,000, 3,000, (3,000), Nil, , Question 16: Write a note on deduction in case of Royalty on Patents., Answer: Deduction in respect of royalty on patents Section 80RRB, 1. Deduction is allowed only to resident individual., 2. His gross total income should include royalty in respect of a patent., 3. Deduction allowed shall be equal to the amount of royalty or `3,00,000 whichever is less., 4. In respect of any income earned from any source outside India, so much of the income, shall be taken, into account for the purpose of this section as is brought into India by the assessee in convertible foreign, exchange within a period of six months from the end of the previous year in which such income is earned, or within such further period as the competent authority may allow in this behalf., 5. The Assessee should retain information with him in form no. 10CCE and it should be produced when, demanded by the department., Question 17: Write a note on deduction in respect of interest on deposits in savings account., Answer: Deduction in respect of interest on deposits in savings account Section 80TTA, 1. Deduction is allowed only to an individual or HUF. (Other than those covered in 80TTB), 2. Deduction is allowed is the assessee has interest income on saving bank accounts with any bank/ Post, Office., 3. No deduction is allowed from interest on time deposit/ fixed deposit., 4. Deduction is allowed to the extent of `10,000., E.g. Mr. X has interest income `8,000 from savings bank account with State Bank and interest income of, `13,000 from fixed deposit with State Bank, deduction allowed under section 80TTA shall be `8,000., As per section 10(15), Interest on Post Office Savings Bank Account to the extent of `3,500 per year shall, be exempt from income tax and in the case of joint account, exemption shall be allowed upto `7,000 per, year., Example: Mr. X has Income under the head salary `7,00,000 and interest on post office savings bank, account `7,000 and interest on savings bank account with State Bank `9,000, in this case tax liability of Mr., X shall be, `, Income under the head Salary, 7,00,000, Income under the head Other sources
Page 249 :
Deduction From Gross Total Income, Interest on Post office Saving Bank Account, Less: Exemption u/s 10(15), Interest on Saving Bank Account with SBI, Income under the head other sources, , 249, 7,000, (3,500), , Gross Total Income, Less: Deduction u/s 80TTA, Total Income, Computation of Tax Liability, Tax on ` 7,02,500 at slab rate, Add: HEC @ 4%, Tax Liability, , 3,500, 9,000, 12,500, 7,12,500, (10,000), 7,02,500, 53,000, 2,120, 55,120, , Question 18: Write a note on deduction in respect of interest on deposits in savings account., Answer: Deduction in respect of interest on deposits in savings account Section 80TTB, Deduction shall be allowed only to a senior citizen with regard to interest income from banks/cooperative, bank/ cooperative society/post office and further it may be in connection with time deposits/saving bank, account or any other deposits., Deduction shall be allowed upto such income but maximum ` 50,000., (Deduction 80TTA not allowed), Illustration 22: Mr. X, aged 62 years, earned professional income (computed) of `5,50,000 during the year, ended 31.03.2022. He has earned interest of `14,500 on the saving bank account with State Bank of India, during the year. Compute the total income of Mr. X for the assessment year 2022-23 from the following, particulars:, (i) Life insurance premium paid to Birla Sun life Insurance in cash amounting to `25,000 for insurance of, life of his dependent parents. The insurance policy was taken on 15.07.2021 and the sum assured on life of, his dependent parents is ` 1,25,000., (ii) Life insurance premium of ` 25,000 paid for the insurance of life of his major son who is not dependent, on him. The sum assured on life of his son is `1,75,000 and the life insurance policy was taken on, 18.04.2011., (iii) Life insurance premium paid by cheque of ` 22,500 for insurance of his life. The insurance policy was, taken on 08.09.2021 and the sum assured is ` 2,00,000., (iv) Premium of ` 16,000 paid by cheque for health insurance of self and his wife (`8,000 for self and, `8,000 for spouse)., (v) `1,500 paid in cash for his health check-up and ` 4,500 paid in cheque for health checkup for his parents., (vi) Paid interest of ` 6,500 on loan taken from bank for MBA course pursued by his daughter., (vii) A sum of ` 15,000 donated in cash to an institution approved for purpose of section 80G for promoting, family planning., (viii) Contribution ` 10,500 made in cheque to an electoral trust., Solution: Computation of total income of Mr. X for the Assessment Year 2022-23, Particulars, Professional Income (computed), Interest on saving bank deposit, Gross Total Income, Less: Deduction under Chapter VIA, Under section 80C, Life insurance premium paid for life insurance of:, - major son, - self ` 22,500 restricted to 10% of ` 2,00,000, Under section 80D, Premium paid for health insurance of self and wife by cheque, , `, 5,50,000, 14,500, 5,64,500, , (25,000), (20,000), (16,000)
Page 250 :
Deduction From Gross Total Income, Payment made for health check-up: - Self, Parents, ` 4,500 (but maximum amount PHC is `5,000), Under section 80E, For payment of interest on loan taken from bank for MBA course of his daughter, Under section 80GGC, Contribution to electoral trust, Under section 80TTB, Interest on savings bank account, Total Income, , 250, (1,500), (3,500), (6,500), (10,500), (14,500), 4,67,000, , Illustration 23: For the Assessment year 2022-23, the Gross Total Income of Mr. Chaturvedi, a resident in, India, was `8,18,240 which includes long-term capital gain of `2,45,000 taxable under section 112 and, Short-term capital gain of `58,000. The Gross Total Income also includes interest income of `12,000 from, savings bank deposits with banks and `40,000 interest on fixed deposits with banks. Mr. Chaturvedi has, invested in PPF `1,20,000 and also paid a medical insurance premium `51,000. Mr. Chaturvedi also, contributed `50,000 to Public Charitable Trust eligible for deduction under section 80G by way of an, account payee cheque. Compute the total income and tax thereon of Mr. Chaturvedi, who is 70 years old as, on 31.3.2022., Solution:, Computation of total income and tax payable by Mr. Chaturvedi for, the A.Y. 2022-23, Particulars, `, `, Gross total income including capital gain, 8,18,240, Less: Long term capital gain u/s 112, (2,45,000), 5,73,240, Less: Deductions under Chapter VI-A:, 1,20,000, Under section 80C in respect of PPF deposit 1,20,000, 50,000, Under section 80D (it is assumed that premium of `51,000 is paid by, otherwise than by cash. The deduction would be restricted to `50,000, since, Mr. Chaturvedi is a senior citizen), 17,662, Under section 80G (See Notes 1 & 2 below), 50,000, (2,37,662), Under section 80TTB (See Note 3 below), 3,35,578, Total income (excluding long term capital gains), 5,80,578, Total income (including long term capital gains), 5,80,580, Total income rounded off u/s 288A, Computation of Tax Liability, `, LTCG `2,45,000 @ 20% u/s 112, 49,000.00, Balance total income `3,35,580 at slab rate, 1,779.00, 50,779.00, Add: Health and Education cess @4%, 2,031.16, Total tax liability (Rounded off u/s 288B), 52,810.00, Notes:, 1. Computation of deduction under section 80G:, Particulars, Gross total income (excluding long term capital gains), Less : Deduction under section 80C, 80D & 80TTB, Adjusted Gross Total Income, 10% of the above, Contribution made, Lower of the two eligible for deduction under section 80G, Deduction under section 80G – 50% of `35,324, , `, 5,73,240, 2,20,000, 3,53,240, 35,324, 50,000, 35,324, 17,662
Page 251 :
Deduction From Gross Total Income, , 251, , 2. Deduction under section 80G is allowed only if amount is paid by any mode other than cash, in case of, amount exceeding `2,000. Therefore, the contribution made to public charitable trust is eligible for, deduction since it is made by way of an account payee cheque., 3. Deduction of upto `50,000 under section 80TTB is allowed to a senior citizen if gross total income, includes interest income on bank deposits, both fixed deposits and savings account., Illustration 24: Mr. Rajmohan whose gross total income was `6,40,000 for the financial year 2021–22,, furnishes you the following information:, (i) Stamp duty paid on acquisition of residential house (self-occupied) - `50,000., (ii) Five year post office time deposit - `20,000., (iii) Donation to a recognized charitable trust `25,000 which is eligible for deduction under section 80G at, the applicable rate., (iv) Interest on loan taken for higher education of spouse paid during the year - `10,000., Compute the total income of Mr. Rajmohan for the Assessment year 2022-23., Answer, Computation of total income of Mr. Rajmohan for the A.Y.2022-23, `, Gross Total Income, 6,40,000, Less: Deduction under Chapter VI-A, Under section 80C, Stamp duty paid on acquisition of residential house, (50,000), Five year time deposit with Post Office, (20,000), Under section 80E, Interest on loan taken for higher education of spouse, being a relative., (10,000), Under section 80G (See Note below), Donation to recognized charitable trust, (50% of `25,000), (12,500), Total Income, 5,47,500, Note: In case of deduction under section 80G in respect of donation to a charitable trust, the net qualifying, amount has to be restricted to 10% of adjusted total income, i.e., gross total income less deductions under, Chapter VI-A except 80G. The adjusted total income is, therefore, `5,60,000 (i.e. 6,40,000 – `80,000), 10%, of which is `56,000, which is higher than the actual donation of `25,000. Therefore, the deduction under, section 80G would be `12,500, being 50% of the actual donation of `25,000., MAY – 2019 (OLD COURSE) 4 Marks, (i) Prakash is retired Government Officer aged 65 years, resides in Cochin, derived following income:, `, Pension, Interest from bank on fixed deposits (Gross), , 6,60,000, 55,000, , Compute the total income of Mr. Prakash for the assessment year 2022-23 from the following, particulars:, (i) Life insurance premium paid by cheque `22,500 for insurance of his life. The insurance policy was, taken on 08-09-2017 and the sum assured is `2,00,000., (ii) Premium of ` 26,000 paid by cheque for health insurance of self., (iii) ` 1,500 paid in cash and ` 4,500 paid through cheque for preventive health check-up of his parents,, who are senior citizens., (iv) Paid interest ` 6,500 on loan taken from bank for MBA course pursued by his daughter., (v) A sum of ` 15,000 donated in cash to an institution approved for purpose of section 80G for, promoting family planning.
Page 252 :
Deduction From Gross Total Income, , 252, , Solution:, Computation of Total Income for the A.Y. 2022-23, Income under the head salary, `, Pension, 6,60,000, Gross salary, 6,60,000, Less: Standard deduction u/s 16(ia), (50,000), Income under the head salary, 6,10,000, Income under the head other sources, Interest income on Fixed Deposits, 55,000, Income under the head other sources, 55,000, Gross Total Income, 6,65,000, Less: Deduction u/s 80C- LIC (22,500 limited to 10% of 2,00,000), (20,000), Less: Deduction u/s 80D- Health Insurance, (26,000), Less: Deduction u/s 80D- PHC of parents (6,000 limited to 5,000), (5,000), Less: Deduction u/s 80E Interest paid on higher studies, (6,500), Less: Deduction u/s 80TTB - Interest on FD, (50,000), Total Income, 5,57,500, Note:, 1. As per section 80D, Maximum deduction of PHC can be allowed is 5,000 whether paid in cash or, by cheque., 2. As per section 80G, Deduction of Donation is not allowed if the payment is made in cash in, excess of 2,000., 3. As per section 80TTB, Deduction shall be allowed in case of senior citizen receiving interest, income from saving account or from FD. Maximum deduction can be 50,000.
Page 253 :
Deduction From Gross Total Income, , 253, , MULTIPLE CHOICE QUESTIONS, 1. As per section 80CCE, `1.5 lakh is the maximum qualifying limit for deduction under (a) Section 80C and 80CCD., (b) Sections 80CCC and 80CCD, (c) Sections 80C, 80CCC and 80CCD, (d) Section 80C, 80CCC and 80D, 2. Deduction u/s 80C in respect of Life Insurance Premium, Contribution to provident fund, etc. is, allowed to:, (a) Any assessee, (b) an individual, (c) An individual or HUF, (d) An individual or HUF who is resident in India, 3. An assessee has paid life insurance premium of ` 25,000 during the previous year 2021-22 for a, policy of `1,00,000 taken on 1.4.2015. He shall:, (a) not be allowed deduction u/s 80C, (b) be allowed deduction of ` 20,000 u/s 80C, (c) be allowed deduction of ` 25,000 under section 80C, (d) be allowed deduction of ` 10,000 u/s 80C, 4. Mr. Srivastav, aged 72 years, paid medical insurance premium of `40,000 by cheque and `4,000 by, cash during May, 2021 under a Medical Insurance Scheme of the General Insurance Corporation., The above sum was paid for insurance of his own health. He would be entitled to a deduction under, section 80D of a sum of(a) `30,000 (b) `40,000 (c) `52,000 (d) `56,000, 5. The payment for Insurance premium under section 80D should be paid:, (a) in cash, (b) by any mode other than cash, (c) by cheque, (d) through account payee cheque/ account payee bank draft, 6. Rajan, a resident Indian, has incurred `15,000 for medical treatment his dependent brother, who is, a person with severe disability and has deposited `20,000 with LIC for his maintenance. For, A.Y.2022-23, Rajan would be eligible for deduction under section 80DD of an amount equal to –, (a) `15,000 (b) `35,000 (c) `75,000 (d) `1,25,000, 7. The maximum amount which can be donated in cash for claiming deduction under section 80G is–, (a) ` 5,000, (b) ` 10,000 (c) ` 1,000, (d) ` 2,000, 8. Mr. Shiva made a donation of `50,000 to National Children's Fund and `20,000 to Rajiv Gandhi, Foundation by cheque. He made a cash donation of `10,000 to a public charitable trust. The deduction, allowable to him under section 80G for A.Y.2022-23 is, (a) `80,000 (b) `70,000 (c) `60,000 (d) `35,000, 9. Mr. Ramesh pays a rent of `5,000 per month. His adjusted gross total income is `2,80,000. He is, also in receipt of HRA. He would be eligible for a deduction under section 80GG of an amount of (a) `60,000 (b) `32,000 (c) `70,000 (d) Nil, 10. Mr. Shaleen, a businessman, whose adjusted gross total income for A.Y.2022-23 is `4,60,000, paid, house rent at `12,000 p.m. in respect of residential accommodation occupied by him at Chennai. The, deduction allowable to him under section 80GG for A.Y.2022-23 is, (a) `98,000 (b) `1,15,000 (c) `60,000 (d) `24,000, 11. Mr. Anuj, a businessman, whose adjusted gross total income for AY 2022-23 is `5,95,000. He does, not own any house property and is staying in a rented accommodation in Patna for a monthly rent of, `9,000. Deduction allowance under section 80GG for A.Y. 2022-23 is:, (a) `48,500, (b) `1,48,750, (c) `60,000, (d) `1,08,000
Page 254 :
Deduction From Gross Total Income, , 254, , 12. Under section 80GGB, deduction is allowable in respect of contribution to political parties by (a) any person other than local authority and every artificial juridical person wholly or partly funded by the, Government, (b) Local authority and every artificial juridical person wholly or partly funded by the Government, (c) An Indian company, (d) Any assessee, 13. The deduction under section 80QQB in respect of royalty income of authors of certain books is, subject to a maximum limit of(a) `1,00,000, (b) `3,00,000, (c) `5,00,000, (d) `2,00,000, 14. Deduction allowed under section 80JJAA is, (a) 20%, (b) 30%, (c) 33%, (d) none of these, 15. Deduction allowed under section 80JJAA is for, (a) 2 year, (b) 1 year, (c) 4 year, (d) 3 year, (e) none of these, 16. Deduction under section 8QQB is allowed for, (a) book of literary nature, (b) book of artistic nature, (c) book of scientific nature, (d) any of the above, (e) none of the above, 17. Deduction under section 80TTA is allowed, (a) to every person for interest on saving accounts, (b) to an individual for interest on fixed deposit, (c) to an individual and HUF for interest on savings account, (d) to an individual and HUF for interest on any account, 18. Deduction under section 80TTB is allowed, (a) to all individual upto ` 40,000, (b) to all individual upto ` 10,000, (c) to all individual upto ` 50,000, (d) to a senior citizen upto ` 40,000, (e) none of these, 19. Deduction for investment in Kisan Vikas Patra is allowed under section, (a) 80C, (b) 80CCC, (c) 80CCD, (d) 80D, (e) none of these, 20. Deduction for repayment of principal amount under section 80C is allowed, (a) for any house property, (b) repairs of residential house property, (c) purchase of residential house property, (d) construction of residential house property, (e) (c) or (d), (f) none of these
Page 255 :
Deduction From Gross Total Income, , 255, , 21. Deduction under section 80C to 80U is allowed from, (a) normal income, (b) normal income including capital gains, (c) only capital gains, (d) only casual income, (e) none of these, 22. Mr. X has income under the head salary `1,25,000 and income from long term capital gains, `3,50,000, in this case maximum amount of deduction allowed shall be, (a) `3,50,000, (b) `4,75,000, (c) `1,25,000, (d) Nil, 23. Mr. X has taken a Life Insurance policy in the name of dependent father on 10.01.2022 and paid, premium by cheque of `25,000 and sum assured shall be `3,00,000 , in this case maximum amount of, deduction allowed u/s 80C shall be, (a) `25,000, (b) `2,500, (c) `30,000, (d) Nil, 24. Mr. X has taken a Life Insurance policy in the name of dependent son on 10.01.2022 and paid, premium by cheque of `25,000 and sum assured shall be `3,00,000 , in this case maximum amount of, deduction allowed u/s 80C shall be, (a) `25,000, (b) `2,500, (c) `30,000, (d) Nil, 25. Mr. X has purchased NSC of `50,000 in the name of dependent father on 10.01.2022, in this case, maximum amount of deduction allowed u/s 80C shall be, (a) `50,000, (b) `5,000, (c) `75,000, (d) Nil, 26. Mr. X has purchased NSC of `50,000 in the name of dependent minor son on 10.01.2022, in this, case maximum amount of deduction allowed u/s 80C shall be, (a) `50,000, (b) `5,000, (c) `75,000, (d) Nil, 27. Mr. X has taken a Life Insurance policy in the name of handicapped daughter on 10.01.2022 and, paid premium by cheque of `60,000 and sum assured shall be `5,00,000, in this case maximum, amount of deduction allowed u/s 80C shall be, (a) `50,000, (b) `60,000, (c) `5,00,000, (d) Nil, 28. Deduction u/s 80D in respect of payment of premium of Medical Insurance or Medical Policy, etc., is allowed to:, (a) Any assessee, (b) an individual, (c) An individual or HUF, (d) An individual or HUF who is resident in India
Page 256 :
Deduction From Gross Total Income, , 256, , 29. Mr. X taken Mediclaim policy for self and his spouse and paid premium by cheque of `35,000., Deduction allowed u/s 80D shall be:, (a) `25,000, (b) `35,000, (c) `60,000, (d) Nil, 30. Mr. X taken Mediclaim policy for his parents who is senior citizen and paid premium by cheque of, `35,000. Deduction allowed u/s 80D shall be:, (a) `25,000, (b) `35,000, (c) `50,000, (d) Nil, 31. Mr. X taken Preventive health check up of `3,000 (paid in cash) for himself and also taken in the, name of parents who is senior citizen and paid `5,000 (in cash). Deduction allowed u/s 80D shall be:, (a) `5,000, (b) `8,000, (c) `3,000, (d) Nil, 32. Deduction u/s 80DD in case of expenditure in connection with handicapped dependent relative is, allowed to:, (a) Any assessee, (b) an individual, (c) An individual or HUF, (d) An individual or HUF who is resident in India, 33. Deduction u/s 80U in case of blind or physically handicapped persons is allowed to:, (a) Any assessee, (b) an individual, (c) An individual or HUF, (d) An individual who is resident in India, 34. Deduction u/s 80E in case of payment of interest on loan taken for pursuing higher education is, allowed to:, (a) Any assessee, (b) an individual, (c) An individual or HUF, (d) An individual who is resident in India, 35. Deduction u/s 80E in case of payment of interest on loan taken for pursuing higher education is, allowed for maximum period of:, (a) 5 years, (b) 1 year, (c) 10 years, (d) 8 years, , Solutions are given on our website, www.mkgeducation.com
Page 257 :
Deduction From Gross Total Income, , 257, , PRACTICE PROBLEMS, TOTAL PROBLEMS 17, Problem 1., Mr. X has taken a loan of `10,00,000 from S.B.I @ 10 % p.a. on 01.07.2017 for construction of one, residential house which was completed on 01.07.2019. It was let out @ ` 80,000 p.m. w.e.f 01.04.2021 and, Mr. X has paid Municipal tax of `20,000 though the amount due is `30,000., He has repaid Principal amount of `70,000 on 01.07.2021., He has Agricultural income of ` 3,00,000 and unadjusted loss of house property of P.Y. 2012-13 `10,000, and P.Y. 2013-14 `21,000., He has invested `10,000 in NSC and `5,000 in Public Provident Fund and `5,000 in Post Office 5 Year, Time Deposit., Compute his Income Tax Liability for the A.Y. 2022-23., Answer: Tax Liability: `21,790, Problem 2., Mr. X has taken a loan of `12,00,000 @ 10 % p.a. on 01.07.2017 and the house completed on 01.05.2021. It, was let out @ ` 30,000 p.m. w.e.f 01.08.2021 and the loan was repaid in annual installment of `40,000, starting from 01.01.2019., Mr. X has STCG 111A `10,00,000., He has paid premium of life policy `40,000 on 12.12.2021 and sum assured is `1,00,000., He has paid premium of Jeevan Suraksha Policy `20,000., Compute his Total Income and Tax Liability for the A.Y. 2022-23., Answer: Total Income: `9,74,000; Tax Liability: `1,12,940, Problem 3., Mr. X has income from business `410000 and he has received income of `3,80,000 from subletting also., He has paid premium of medi-claim policy of `31,000. It was paid by cheque and the policy was taken on, 02.01.2022 in the name of his father., Compute his total income and tax liability for assessment year 2022-23., Answer: Total Income: `7,65,000; Tax Liability: `68,120, Problem 4., Mr. X has income from business `7,00,000., Mr. X has incurred `65,000 on the treatment of his dependent brother who is suffering from a disease, notified under Rule 11DD and he has received claim under medi-claim policy `35,000., Compute his income and tax liability for assessment year 2022-23., Answer: Total Income: `6,95,000; Tax Liability: `53,560, (b) Presume assessee incurred `65,000 on the treatment of his independent brother., Answer: Total Income: `7,00,000; Tax Liability: `54,600, Problem 5., Mrs. X has let out one residential house property @ `1,00,000 p.m. and she has paid municipal tax of, `1,00,000., She has taken a Medi-claim policy on 17.07.2021 in the name of Mr. X and paid premium of `18,000 by, cheque, She has also taken a Medi-claim policy on 15.10.2021 in the name of her Father in law who is aged 66 years, and paid premium of `16,000 by cheque., She has incurred `21,000 on the treatment of her brother who is dependent on her and suffering from severe, disability., She has purchased N.S.C. in P.Y. 2019-20 and there is accrued interest of `30,000 and also there is accrued, interest of PPF `10,000
Page 258 :
Deduction From Gross Total Income, , 258, , She has taken Jeevan Suraksha Policy in the name of Mr. X and paid premium of `19,000., She has taken a loan in 2015-16 from SBI for the education of his son who is studying in B.com (Hons) in, SRCC and she had paid principal amount of `60,000 and interest `10,000 in P.Y.2021-22., She has Agricultural Income `1,00,000, Compute her Income Tax Liability for the A.Y.2022-23., Answer: Tax Liability: `52,940, Problem 6., Mr. X has incomes asunder:, `, 1. Rent from letting out one house property, 3,00,000, 2. Long term capital gains, 2,00,000, He has donated `5,000 by cheque to MCD which is notified under section 80G and has donated `2,000 by, cheque to National Children’s Fund and `2,000 by cheque to the Government for promotion of family, planning norms., He has invested `6,000 in NSC. He is aged about 67 years., Compute his total income and tax liability for the assessment year 2022-23., Answer: Total Income: `3,97,500; Tax Liability: `7,280, Problem 7., Mr. X is a Practicing Chartered Accountant and he started his practice from 01.04.2021 and he has income, from profession `8,00,000., He has LTCG of `3,00,000, STCG 111A of `1,00,000, casual income `2,00,000., Investment and donations are as given below:• NSC `10,000., • Medi-claim premium (by cheque) of `15,000., • Prime Minister’s National Relief Fund `10,000. (Paid by cheque), • Rajiv Gandhi Foundation `8,000. (Paid by cheque), • Donation to Birla Temple (Notified u/s 80G) `1,60,000. (Paid by cheque), • Charitable institution (Notified u/s 80G) `40,000. (Paid by cheque), • Social organization (Notified u/s 80G) `20,000. (Paid by cheque), • Municipal Corporation of Delhi (notified under section 80G) `10,000. (Paid by cheque), Compute income tax liability for A.Y. 2022-23., Answer: Tax Liability: `1,97,550, (b) Presume in the above question the assessee has given donation to the Government also for family, planning is ` 20,000., Answer: Tax Liability: `1,95,470, (c) Presume in the above question the assessee has given donation to the Government also for family, planning is ` 3,00,000., Answer: Tax Liability: `1,87,410, Problem 8., Mr. X has short term capital gain of `6 lakhs and he has donated `20,000 by cheque to a charitable, institution which is notified under section 80G and he has spent `25,000 on the treatment of his handicapped, dependant brother., Compute his total income and tax liability for the assessment year 2022-23., Answer: Total Income: `5,15,000; Tax Liability `16,120, Problem 9., Mr. Ram Kumar has incomes asunder:, 1. He has income from Business/Profession `1,22,000., 2. He has long term capital gains of `2,00,000.
Page 259 :
Deduction From Gross Total Income, , 259, , 3. He has donated `10,000 by cheque to MCD for family planning and has donated `3,000 by cheque to a, charitable institution notified under section 80G., Compute his total income and tax liability for the assessment year 2022-23., Answer: Total Income: `3,10,900; Tax Liability: Nil, Problem 10., Mr. X has incomes asunder:, 1. Short term capital gains on sale of a capital asset `5,00,000., 2. Mr. X has donated `7,000 by cheque to the Prime Minister’s National Relief Fund and `20,000 by, cheque to Birla temple which is notified under section 80G., Compute his total income and tax liability for the assessment year 2022-23., Answer: Total Income: `4,83,000; Tax Liability: Nil, Problem 11., Mr. X has incomes asunder:, 1. Income from Business/Profession, 2. He has long term capital gains, 3. He has income from other sources, 4. He has donated `10,000 by cheque to National Defence Fund, 5. He has donated ` 5,000 by cheque to charitable institution notified under section 80G., Compute his total income and tax liability for assessment year 2022-23., Answer: Total Income: `4,47,500; Tax Liability: `28,080, , `, 1,00,000, 2,50,000, 1,10,000, , Problem 12., Mr. X has incomes asunder:, Long Term Capital Gains: `1,00,000, Short Term Capital Gains: `2,55,000, Casual income: `10,000, Donations given to charitable institutions notified under section 80G `45,000 paid by cheque and donation, to MCD for family planning `3,000 paid by cheque., Compute his total income and tax liability for the assessment year 2022-23., Answer: Total Income: `3,50,250; Tax Liability: `8,890, Problem 13., Mr. X is engaged in the business of manufacturing chemicals and has income under the head, business/profession of `5,00,000 and has paid rent of `10,000 p.m. for taking a house on rent because he did, not have any house in his name or in the name of his spouse or minor child or the Hindu Undivided Family, of which he is a member. He has invested `75,000 in NSC which were taken in the name of his spouse., Compute his total income and tax liability for assessment year 2022-23., Answer: Total Income: `3,65,000; Tax Liability: Nil, Problem 14., During the previous year 2021-22, Mr. X has income under the head house property `4,00,000. He has, donated `12,000 by cheque to a notified institution for the purpose of scientific research., Compute his total income and tax liability for the assessment year 2022-23., Answer: Total Income: `3,88,000; Tax Liability: Nil, Problem 15., For the assessment year 2022-23, Mr. X submits the following information:, Income from business, Property income, House I, `, Fair Rent, 75,000, , 9,800, House II, `, 82,000
Page 260 :
Deduction From Gross Total Income, , 260, , Rent Received/Receivable, 78,000, 85,000, Municipal Valuation, 76,000, 75,000, Municipal Taxes (due but outstanding), 13,000, 14,000, Repairs, 3,500, 47,000, Insurance, 2,000, 3,000, Land Revenue (Paid), 2,500, 4,000, Ground Rent (due but outstanding), 1,600, 6,000, Interest on capital borrowed by mortgaging house I, (Funds are used for construction of house II), 14,000, ———, Nature of Occupation, Let out for Residence, Let out for Business, Date of completion of construction, 30.04.2017, 07.04.2019, Assume that standard rent is less than rent actually received., Mr. X has brought forward losses of house property asunder, `, Assessment year 2013-14, 1,00,000, Assessment year 2017-18, 10,100, Other incomes of Mr. X, 1. Vacant site lease rent, 4,12,000, 2. Rent from house property at Chennai, 3,000 p.m., This house was constructed by taking a loan of ` 2,00,000 @ 10% from State Bank of India but it was, repaid on 01.10.2021 by taking a loan of `2,00,000 from Punjab National Bank on 01.10.2021 @ 9.5%, p.a., 3. He has also received `3,000 during the year from Calcutta University for acting as an examiner and, `1,500 from Delhi University also, for acting as an examiner., 4. He has received from Life Insurance Corporation of India `1,20,000 being the maturity amount of life, insurance policy., 5. He has received a reward of `5,000 from Central Government and the reward is notified under section, 10(17A)., He has invested `1,000 in the notified bonds of NABARD eligible for deduction under section 80C, but, the investment is out of past savings and has invested `1,000 in master equity plan of Unit Trust of India., He is eligible for deductions under section 80D to 80U amounting to `1,005., Compute his income and also tax liability for assessment year 2022-23., Answer = Total Income: `3,71,000; Tax Liability: Nil, Problem 16., Mrs. X has income under the head Business/Profession `6,00,000., She has spent `30,000 on the treatment and education of her sister who is a disabled person as per section, 80U and she is dependent on Mrs. X., She has paid premium of medi-claim policy `18,000 by cheque and policy is taken on 15.11.2021 in the, name of her father who is not dependent on Mrs. X., She is holder of one patent right and has received royalty of `3,20,000, She has paid rent of `30,000 p.m. and she has complied with all the conditions of section 80GG, You are required to compute her Income tax liability for the A.Y. 2022-23., Answer: Tax Liability: Nil, Problem 17., Mr. X has received royalty of `8,00,000 in connection with a patent right registered in his name., Other informations are as given below:, 1. He has donated ` 30,000 to a political party by cheque., 2. He donated `10,000 by cheque to Delhi University notified under section 80G, 3. He donated `10,000 by cheque to Government for the purpose of promoting family planning., 4. He paid premium of medi-claim policy `6,000 by cheque in the name of his major married independent, son., 5. LIC premium paid `25,000 on 15.01.2022 (Policy value `1,00,000)
Page 261 :
Deduction From Gross Total Income, , 261, , 6. Repayment of housing loan to Indian Bank `50,000, 7. Payment made to LIC pension fund notified under section 80CCC `20,000, Compute income tax liability for A.Y 2022-23., Answer: Tax Liability: Nil, , Detailed Solutions of All Questions are given on our, website www.mkgeducation.com
Page 262 :
Deduction From Gross Total Income, , 262, , Question 1 [V. Imp.]: Explain Deduction in case of Units established in Special Economic Zone., Section 10AA, Answer:, Units established in Special Economic Zone, 1. Deduction shall be allowed to all the assessees, may be individual, firm, company etc. provided the, assessee has its unit in Special Economic Zone and it is engaged in manufacturing or in providing services, including computer software, 2. Quantum of deduction:, Deduction shall be allowed to the units in the Special Economic Zone for a continuous period of 15 years in, the manner given below:, For first 5 Assessment Years, 100% of export profits, For next 5 Assessment Years, 50% of export profits, For next 5 Assessment Years, 50% of export profit provided such profits have been credited, to the Special Economic Zone Re-investment Reserve, Account., Export profits means, , Profits of Business x Export Turnover, Total Turnover, e.g. ABC Ltd. has one unit in SEZ and total turnover is `1000 lakhs and profits `400 lakhs and export, turnover `800 lakhs, in this case export profits shall be 400 / 1000 x 800 = 320 lakhs, The amount credited to the Special Economic Zone Reinvestment Reserve Account should be utilised for, acquiring a new plant and machinery within a period of 3 years. The period of 3 years shall be determined, from the end of the previous year in which the reserve was created e.g. If amount has been transferred in, reserve account in the previous year 2021-22, amount should be utilized for purchasing plant and machinery, upto 31.03.2025., 3. If the amount credited to the Special Economic Zone Reinvestment Reserve Account is not utilised within, 3 years, it will be taxable in the 4th year. Till the acquisition of plant and machinery amount will be, utilized for the purpose of business/profession but it should not be used for distribution as dividends or for, remittance out of India or for creation of asset out of India. After the assessee has purchased plant and, machinery, information should be retained in form no.56FF, If the amount is misutilised within the period of 3 years, it will be taxable in the year in which it was, misutilised., 4. Export turnover means the consideration in respect of export by the undertaking, being the Unit of articles, or things or services received in, or brought into, India by the assessee but does not include freight,, telecommunication charges or insurance attributable to the delivery of the articles or things outside India or, expenses, if any, incurred in foreign exchange in rendering of services (including computer software), outside India., 5. The assessee should furnish in the prescribed form (56F), before the date specified in section 44AB i.e.,, one month prior to the due date for furnishing return of income u/s 139(1), the report of a chartered, accountant certifying that the deduction has been correctly claimed., Example: An individual, subject to tax audit u/s 44AB, claiming deduction u/s 10AA is required to furnish, return of income on or before 31.10.2022 and the report of a chartered accountant before 30.09.2022,, certifying the deduction claimed u/s 10AA., 6. The profits and gains derived from the on site development of computer software (including services for, development of software) outside India shall be deemed to be the profits and gains derived from the export, of computer software outside India., 7. Deduction allowable in case of amalgamation and demerger, In the event of any undertaking, being the Unit which is entitled to deduction under this section, being, transferred, before the expiry of the period specified in this section, to another undertaking, being the Unit in, a scheme of amalgamation or demerger, (a) no deduction shall be admissible under this section to the amalgamating or the demerged Unit for the, previous year in which the amalgamation or the demerger takes place; and
Page 263 :
Deduction From Gross Total Income, , 263, , (b) the provisions of this section would apply to the amalgamated or resulting Unit, as they would have, applied to the amalgamating or the demerged Unit had the amalgamation or demerger had not taken place., MAY – 2019 (OLD COURSE) Marks 3, Krishna furnishes the following particulars for the previous year 2020-21 and 2021-22 in respect of an, industrial undertaking established in “Special Economic Zone” during the financial year 2016-17., 2020-21, 2021-22, Particulars, (`), (`), Total sales, 60,00,000, 85,00,000, Export sales, 48,00,000, 55,00,000, Domestic sales, 12,00,000, 30,00,000, Money received in or brought to India in convertible foreign exchange up to, 43,20,000, 40,00,000, 30-09-2021/30-09-2022., Profit from the above undertaking, 6,00,000, 10,00,000, Total Sales of F.Y. of 2021-22 includes freight of ` 5 lacs for delivery of goods outside India. Compute the, amount of deduction available to Mr. Krishna under section 10AA., Solution:, Computation of the amount of deduction available to Mr. Krishna u/s 10AA, Since A.Y. 2022-23 is the 6th assessment year from A.Y. 2017-18, relevant to the previous year 2016-17, in, which the SEZ unit began manufacturing of articles or things, it shall be eligible for deduction of 50% of the, profits derived from export of such articles or things, assuming all the other conditions specified in section, 10AA are fulfilled., = (Profit of Unit in SEZ /Total Turnover of Unit in SEZ X Export turnover of Unit in SEZ) X 50%, = (10,00,000 /80,00,000 X 35,00,000) X 50%, = 2,18,750, Working Note:, Export Turnover, Sale proceeds received in India, Less: Freight not includible in export turnover, Total turnover, Less: Freight not includible [Since freight has been excluded from export, turnover, the same has to be excluded from total turnover also]., , 40,00,000, (5,00,000), 35,00,000, 85,00,000, (5,00,000), 80,00,000, , Notes:, (1) The question mentions the quantum of money received in India up to 30.9.2021/30.9.2022. Section10AA, does not specify any time limit for repatriation of export proceeds. As per RBI Master Direction No., 16/2015-16 [RBI/FED/2015-16/11 FED] updated upto 12.1.2018, the period of realization and repatriation, of export proceeds shall be nine months from the date of export for all exporters including Units in Special, Economic Zones (SEZs). Erstwhile section 10A(3) specified a time limit of six months from the end of the, previous year. Section 10AA, however, does not specify any time limit. The only requirement is that the, consideration has to be brought into India. Accordingly, this question has been solved assuming that `40, lakhs is the money received or brought into India within the time limit permitted by RBI. Further, it is, presumed that `5 lakhs for delivery of goods outside India is included in this sum. Alternatively, it is, possible to work out the deduction under section 10AA assuming that the entire sum of `55 lakhs has been, brought into India within the time limit prescribed by RBI., (2) Since the question does not specify for which assessment year the deduction under section 10AA is, required to be computed, the deduction available under section 10AA is computed for the assessment year, 2022-23 only. Also, in the instructions relating to Part II Section A given in page 1 of the question paper,, point no.4 clearly states that answers to all questions should relate to A.Y.2022-23, unless otherwise stated.
Page 264 :
Deduction From Gross Total Income, , 264, , (3) Since question mentions that the SEZ unit was established in the P.Y. 2016-17, it is logical to assume, that it began manufacturing in the same previous year., NOV – 2018 (NEW COURSE) 6 Marks, Mrs. Vibha Gupta, a resident individual is running a SEZ unit, as well as a unit in Domestic Tariff Area, (DTA). She furnishes the following details relating to the year ended 31-3-2022, pertaining to these two, units (` in lakhs), DTA Unit, SEZ Unit, Export turnover, , 100, , 1000, , Total turnover, , 400, , 1100, , Net profit, , 50, , 220, , Compute the deduction available u/s 10AA:, (i) When the SEZ unit had been set up on 12-3-2014, and, (ii) When the SEZ unit had been set up on 12-8-2019, Solution:, (i) 50% of the profit derived from export of articles or things or services is eligible for deduction under, section 10AA, F.Y.2021-22 falls in the next five year period commencing from the year of manufacture or, production of articles or things or provision of services by the Unit in SEZ. As per section 10AA, the profit, derived from export of articles or things or services shall be, = Profit of the business of Unit in SEZ x, Export Turnover of Unit in SEZ, Total Turnover of Unit in SEZ, = 50% of ` 220 lakhs x, 1000 Lakhs, 1100 Lakhs, = 50% x `200 lakhs, = `100 lakhs, (ii) 100% of the profit derived from export of articles or things or services is eligible for deduction under, section 10AA, F.Y.2021-22 falls in the first five year period commencing from the year of manufacture or, production of articles or things or provision of services by the Unit in SEZ. As per section 10AA, the profit, derived from export of articles or things or services shall be, = Profit of the business of Unit in SEZ x Export Turnover of Unit in SEZ, Total Turnover of Unit in SEZ, = 100% of ` 220 lakhs x, 1000 Lakhs, 1100 Lakhs, = 100% x `200 lakhs, = `200 lakhs, MAY – 2016 (4 Marks), Mr. Suresh has set up an undertaking in SEZ (Unit A) and another undertaking in DTA (Unit B) in the, financial year 2016-17. In the Previous year 2021-22, total turnover of the unit A is `180 lacs and total, turnover of Unit B is `120 lacs. Export Turnover of Unit A for the year is `150 lacs and Profit for the unit A, is `60 lacs., Calculate the deduction available, if any, to Mr. Suresh under section 10AA of the Income-tax Act, 1961, for, the Assessment Year 2022-23, If the manufacturing started in Unit A in the Financial year 2016-17., Solution:, As per Section 10AA,in case of a unit in SEZ, Deduction shall be allowed to the extent of 100% of Export, Profits in the first five years and 50% of export profits in the next five years. In the given case Unit was, setup in Financial year 2016-17 hence it is sixth year in Previous year 2021-22 and Deduction shall be, allowed to the extent of 50% of export profit and shall be as given below:
Page 265 :
Deduction From Gross Total Income, = Profit of the business of Unit in SEZ x, = `60 lacs x, , 265, , Export Turnover of Unit in SEZ, Total Turnover of Unit in SEZ, , 150 Lacs, 180 Lacs, , = `50lacs, Deduction allowed = 50% of `50 lacs, = `25lacs, Taxable amount of Unit A (`60 lacs – `25 lacs) = `35 lacs, MAY – 2015 (4 Marks), Rudra Ltd. has one unit at Special Economic Zone (SEZ) and other unit at Domestic Tariff Area (DTA), the, company provides the following details for the previous year 2021-22., Particulars, Rudra Ltd. Unit in DTA, (`), (`), Total Sales, 6,00,00,000, 2,00,00,000, Export Sales, 4,60,00,000, 1,60,00,000, Net Profit, 80,00,000, 20,00,000, Calculate the eligible deduction under section 10AA of the Income-tax Act, 1961, for the Assessment Year, 2022-23, in the following situations:, (i) If both the units were set up and start manufacturing from 22.05.2015., (ii) If both the units were set up and start manufacturing from 14.05.2019., Solution:, (i) 50% of the profit derived from export of articles or things or services is eligible for deduction under, section 10AA, F.Y.2021-22 falls in the next five year period commencing from the year of manufacture or, production of articles or things or provision of services by the Unit in SEZ. As per section 10AA, the profit, derived from export of articles or things or services shall be, = Profit of the business of Unit in SEZ x, Export Turnover of Unit in SEZ, Total Turnover of Unit in SEZ, = 50% of ` 60 lakhs x 300 Lakhs, 400 Lakhs, = 50% x `45 lakhs, = `22.5 lakhs, (ii) 100% of the profit derived from export of articles or things or services is eligible for deduction under, section 10AA, F.Y.2021-22 falls in the first five year period commencing from the year of manufacture or, production of articles or things or provision of services by the Unit in SEZ. As per section 10AA, the profit, derived from export of articles or things or services shall be, = Profit of the business of Unit in SEZ x Export Turnover of Unit in SEZ, Total Turnover of Unit in SEZ, = 100% of `60 lakhs x 300 Lakhs, 400 Lakhs, = 100% x `45 lakhs, = `45 lakhs, NOV – 2013 (4 Marks), Mr. X is running two industrial undertakings, one in a SEZ (Unit A) and another in a Domestic Tariff Area, (Unit B). The brief details for the year ended 31.03.2022 are as under:, Particulars, Domestic turnover, Export turnover, Gross Profit, Less: Expenses and depreciation, , Amount (` in lacs), Unit A, Unit B, 10, 100, 120, Nil, 20, 10, (07), (05)
Page 266 :
Deduction From Gross Total Income, , 266, , Profits derived from the units, 13, 05, The brought forward business loss pertaining to assessment year 2019-20 for Unit B is `3.2 lacs. Briefly, compute the business income of the assessee., Answer., Computation of business income of Mr. X, Particulars, ` (in lacs), Profit derived from Units A (20 lacs -7 lacs), 13.0, (12.0), 13 × 120, ), Less: Deduction under section 10AA (, 130, Income from unit A, 1.0, 05.0, Profit derived from Units B (` 10 lacs -` 5 lacs), Less: Set-off of brought forward business loss as per section 72, (3.2), Income from unit B, 1.8, Total Income from unit A & B (1+1.8), 2.8, Note - 100% of the profit derived from export of articles or things or services is eligible for deduction under, section 10AA, assuming that F.Y.2021-22 falls within the first five year period commencing from the year, of manufacture or production of articles or things or provision of services by Unit A in SEZ., MAY – 2011 (5 Marks), ABC Ltd. is running two industrial undertakings, one in a SEZ (Unit S) and another in a normal area (Unit, N). The brief summarized details for the year ended 31.03.2022 are as under:, (` in lacs), S, N, Domestic turnover, 10, 100, Export turnover, 120, Nil, Gross profit, 20, 10, Less: Expenses and depreciation, (7), (6), Profits derived from the unit, 13, 4, The brought forward business loss pertaining to Unit N is `2 lacs. Briefly compute the business income of, the assessee., Answer. Computation of business income of ABC Ltd., Particulars, Total profit derived from Units S & N (`13 lacs + `4 lacs), 13 × 120, ), Less: Deduction under section 10AA (, 130, Less: Brought forward business loss, , ` in lacs, 17, (12), 5, (2), 3, Note – 100% of the profit derived from export of articles or things or from services is eligible for deduction, under section 10AA, assuming that F.Y. 2021-22 falls within the first five year period commencing from the, year of manufacture or production of articles or things or provision of services by the Unit in SEZ., MAY – 2011 (4 Marks), Y Co. Ltd. Furnishes you the following information for the year ended 31.03.2022:, Total turnover of Unit A located in Special Economic Zone, Profit of the business of Unit A, Export turnover of Unit A, Total turnover of Unit B located in Domestic Tariff Area (DTA), Profit of the business of Unit B, Compute deduction under section 10AA for the assessment year 2022-23., , `, 100 lakhs, 30 lakhs, 50 lakhs, 200 lakhs, 20 lakhs
Page 267 :
Deduction From Gross Total Income, , 267, , Answer., 100% of the profit derived from export of articles or things or services is eligible for deduction under section, 10AA, assuming that F.Y.2021-22 falls within the first five year period commencing from the year of, manufacture or production of articles or things or provision of services by the Unit in SEZ. As per section, 10AA, the profit derived from export of articles or things or services shall be, = Profit of the business of Unit A x Export Turnover of Unit A, Total Turnover of Unit A, = ` 30 lakhs x 50, 100, = `15 lakhs.
Page 268 :
Agricultural Income, , 268, , AGRICULTURAL INCOME, PARTICULARS, Definition of agricultural income, • Rent or revenue derived from agricultural land, • Income derived from agricultural land by agricultural operations, • Income of a farm building, Exemption of agricultural income, Income which is partially agricultural and partially from business, Income from growing and manufacturing of rubber, Income from growing and manufacturing of coffee, Income from growing and manufacturing of tea, , SECTIONS, 2(1A), 2(1A)(a), 2(1A)(b), 2(1A)(c), 10(1), Rule 7, Rule 7A, Rule 7B, Rule 8, , Question 1. [V. Imp.] Explain meaning of agricultural income., Answer: Meaning of Agricultural Income Section 2(1A), The term Agricultural Income is defined in three parts under Income Tax Act under section 2(1A) (a), 2(1A), (b), 2(1A) (c) as given below:, Income from leasing out of agricultural land Section 2(1A) (a), If any person has given any agricultural land on rent, rent so received shall be considered to be agricultural, income and shall be exempt from income tax e.g. Mr. X has ten acres of agricultural land in India which is, given on lease at a rent of ` 2,00,000. It will be considered to be agricultural income., If rent is received in kind, still it will be considered to be agricultural income e.g. Mr. X has leased out ten, acres of agricultural land and has received wheat crop worth `2,00,000. In this case, `2,00,000 shall be, considered to be his agricultural income., The rent can either be received by the owner of the land or by the original tenant from the sub-tenant. It, implies that ownership of land is not necessary. Thus, the rent received by the original tenant from subtenant would also be agricultural income subject to the other conditions mentioned above., The scope of the term “Revenue” is much broader than rent. It includes income other than rent. For example,, fees received for renewal for grant of land on lease would be revenue derived from land., , If rent to be received has not been received in time and accordingly interest has been received, such interest, shall not be considered to be agricultural income, rather it is his income under the head other sources., If the agricultural land is situated outside India, income from agricultural land is taxable as income from, other sources., Rent received for letting out agricultural land for a movie shooting shall not be considered to be agricultural, income., (One acre is equal to an area of 4,840 square yards (0.405 hectare)), Income from Agricultural Operations Section 2(1A)(b), If any person is engaged in agricultural activities, income derived from such agricultural operations shall be, considered to be agricultural income. If any company is doing agriculture, its income shall also be exempt., Dividends received by a shareholder from the company having agricultural income, If any shareholder has received dividend from a company having income from agricultural activities, such, income shall not be considered to be agricultural income rather it will be considered to be dividend income., E.g. ABC Ltd. an Indian company has agricultural income of `500 lakhs and company has distributed, dividend of `50 lakh and one of the shareholder Mr. X has received dividend of `8 lakh, in this case tax, treatment shall be: Tax liability of ABC Ltd. Shall be nil as per section 10(1) and dividend received by Mr., X shall be taxable., If any foreign company is doing agriculture, its agricultural income in India shall also be exempt and if the, company has paid dividend, it will be taxable in the hands of the shareholder e.g. If in the above case it is a, foreign company, its tax liability shall be nil and tax liability of shareholder shall be as given below:
Page 269 :
Agricultural Income, , 269, , Tax on `8,00,000 at slab rate, 72,500, Add: HEC @ 4%, 2900, Tax Liability, 75,400, Illustration 1: ABC Ltd. an Indian company has agricultural income `350 lakhs and company has, distributed dividend of `60 lakhs to its shareholders and one of the shareholder Mr. X has received dividend, of `7,00,000. Compute tax liability of the company and tax liability of shareholder., Solution:, Tax liability of ABC Ltd. Shall be nil as per section 10(1),, Tax liability of shareholder shall be as given below:, Tax on `7,00,000 at slab rate, 52,500, Add: HEC @ 4%, 2,100, Tax Liability, 54,600, (b) Presume it is foreign company., Solution:, It is a foreign company, its tax liability shall be nil and tax liability of shareholder shall be as given below:, Tax on `7,00,000 at slab rate, 52,500, Add: HEC @ 4%, 2,100, Tax Liability, 54,600, (c) Presume it is Indian company and income is from business and not from agriculture and turnover during, F.Y. 2018-19 was 500 Crores., Solution:, Tax liability of the company shall be as given below:, Profit before tax, 350,00,000.00, Income tax on `350,00,000 @ 30%, 105,00,000.00, Add: Surcharge @ 7%, 7,35,000.00, Add: HEC @ 4%, 4,49,400.00, Income tax liability, 116,84,400.00, Tax liability of the shareholder shall be as given below:, Tax on `7,00,000 at slab rate, 52,500, Add: HEC @ 4%, 2,100, Tax Liability, 54,600, (d) Presume it is foreign company and income is from business and not from agriculture, Solution:, Tax liability of the company shall be as given below:, Profit before tax, 350,00,000.00, Income tax on `350,00,000 @ 40%, 140,00,000.00, Add: Surcharge @ 2%, 2,80,000.00, Add: HEC @ 4%, 5,71,200.00, Income tax liability, 148,51,200.00, Tax liability of the shareholder shall be as given below:, Dividend from foreign company, 7,00,000, Tax on `7,00,000 at slab rate, 52,500, Add: HEC @ 4%, 2,100, Tax Liability, 54,600, Payments received by a partner from the partnership firm, If any partnership firm has agricultural income, it will be exempt from income tax and if partnership firm, has paid any salary or interest to the partners, it will be considered to be agricultural income to the partners, as decided in R.M. Chidambaram Pillai v CIT (SC), If any partner has received any share out of profits of partnership firm, it will be exempt under section, 10(2A) and it do not matter whether partnership firm has agricultural income or non-agricultural income., If partnership firm has non-agricultural income, salary or interest received by a partner from the partnership, firm shall be considered to be their income under the head business/profession as per section 28 and shall be, taxable in the hands of partner e.g. XY partnership firm has two partners Mr. X and Mr. Y and profit sharing
Page 270 :
Agricultural Income, , 270, , ratio is 1:1 and the firm has agricultural income `300 lakhs without debiting salary or interest to the, partners. The firm has paid salary of `8 lakh to each of the partner and interest of `4 lakh to each of the, partner. Mr. X has income under the head house property `6 lakh and Mr. Y has income under the head, house property `7 lakh. Compute tax liability of the firm and also that of partners., Solution:, Since partnership firm has agricultural income, it is exempt from income tax under section 10(1)., Tax liability of Mr. X shall be, Income under the head House Property, 6,00,000, Agricultural income (8,00,000 + 4,00,000), 12,00,000, Partial integration, 6,00,000 + 12,00,000 = 18,00,000 at slab rate, 3,52,500, 2,50,000 + 12,00,000 = 14,50,000 at slab rate, 2,47,500, (3,52,500 – 2,47,500), 1,05,000, Tax before health & education cess, 1,05,000, Add: HEC @ 4%, 4,200, Tax Liability, 1,09,200, Tax liability of Mr. Y shall be, Income under the head House Property, 7,00,000, Agricultural income (8,00,000 + 4,00,000), 12,00,000, Partial integration, 7,00,000 + 12,00,000 = 19,00,000 at slab rate, 3,82,500, 2,50,000 + 12,00,000 = 14,50,000 at slab rate, 2,47,500, (3,82,500 – 2,47,500), 1,35,000, Add: HEC @ 4%, 5,400, Tax Liability, 1,40,400, Share received out of profits is exempt under section 10(2A)., Presume in the above case partnership firm has income from business and not agricultural income., Solution:, Tax Liability of partnership firm shall be as given below:, Profits before debiting salary and interest, 300,00,000, Less: Salary and Interest, 24,00,000, Income under the head Business/Profession, 276,00,000, Gross Total Income/Total Income, 276,00,000, Tax Liability 276,00,000 x 30%, 82,80,000, Add: Surcharge @ 12%, 9,93,600, Tax before health & education cess, 92,73,600, Add: HEC @ 4%, 3,70,944, Tax Liability, 96,44,544, Rounded off u/s 288B, 96,44,540, Tax Liability of Mr. X, Income under the head Business/Profession (salary + interest), 12,00,000, Income under the head House Property, 6,00,000, Gross Total Income/Total Income, 18,00,000, Tax on `18,00,000 at slab rate, 3,52,500, Add: HEC @ 4%, 14,100, Tax Liability, 3,66,600, Tax Liability of Mr. Y, Income under the head Business/Profession (salary + interest), 12,00,000, Income under the head House Property, 7,00,000, Gross Total Income/Total Income, 19,00,000, Tax on `19,00,000 at slab rate, 3,82,500, Add: HEC @ 4%, 15,300, Tax Liability, 3,97,800
Page 271 :
Agricultural Income, , 271, , Meaning of Agriculture: The term agriculture and agricultural purposes has not been defined under Income, Tax Act, accordingly its meaning has been explained in Raja Benoy Kumar Sahas Roy v CIT (SC). If any, person has performed the following two operations, it will be called agriculture., 1. Basic Operations:, In order to constitute agriculture, there must be basic operations like ploughing of land, sowing of seeds,, planting and similar kind of operations on the land., 2. Subsequent Operations:, After carrying out basic operations, there must be subsequent operations like weeding, digging the soil, around the growth, watering of the plant at regular intervals, using pesticides and insecticides to, protect the crop and it will also include pruning, cutting, harvesting etc., (Pruning means to trim (a tree, shrub, or bush) by cutting away dead or overgrown branches or stems,, especially to encourage growth.), If there are basic and subsequent operations, it will be considered to be agricultural income even if what is, produced is not food grains, example:, (i) If a person is growing betel, coffee, tea, spices etc through basic and subsequent operations, it will be, agricultural income., (ii) If a person is growing commercial crops like cotton, flax, jute, indigo etc. through basic and, subsequent operations, it will be considered to be agricultural income., (iii)If a person is growing trees like Sal, Seesam, Sangwan etc for obtaining timber, it will be considered to, be agricultural income, provided there are basic and subsequent operations., Income which is partially agricultural and partially from business Rule 7, If any person is engaged in growing as well as manufacturing activity, in such cases it will be presumed that, he has transferred his agricultural produce to his industrial undertaking at the market price and expenses on, agriculture shall be deducted from such amount and balance shall be agricultural income. While computing, income of business, such market price is allowed to be deducted as cost of raw material. E.g. Mr. X is, engaged in growing of sugarcane and also has a sugar factory. He has incurred expenses of `3,00,000 in, connection with growing of sugarcane crop. Entire sugarcane crop was transferred to the industrial unit, when market price of sugarcane was ` 10,00,000. In this case, agricultural income of Mr. X shall be `, 7,00,000. While computing income of sugar factory, ` 10,00,000 shall be debited to profit and loss account, as the cost of raw material., Example, Mr. X grows sugarcane and uses the same for the purpose of manufacturing sugar in his factory., 50% of sugarcane produce is sold for `10 lacs, and the cost of cultivation of such sugarcane is `3 lacs., The cost of cultivation of the balance sugarcane (50%) is 3 lacs and the market value of the same is `10 lacs., After incurring `1.5 lacs in the manufacturing process on the balance sugarcane, the sugar was sold for `25, lacs., Compute Mr. X’s business income and agricultural income. Compute his Tax Liability., Solution:, Agricultural income = Actual sale of sugarcane + Market value of sugarcane transferred to the, manufacturing unit – Cost of cultivation, = [`10 lacs + `10 lacs] – [`3 lacs + `3 lacs], = `20 lacs – `6 lacs, = `14 lacs, Business income = Sales – Market value of 50% of sugarcane produce – Manufacturing expenses, = `25 lacs – 10 lacs – 1.5 lacs, = `13.5 lacs, Computation of tax liability, 6,37,500, Step 1: Tax on (`14,00,000 + ` 13,50,000 = ` 27,50,000), Step 2: Tax on (` 2,50,000 + ` 14,00,000) = ` 16,50,000), (3,07,500), Step 3: ` 6,37,500 – ` 3,07,500, 3,30,000, Tax before health & education cess, 3,30,000, Add : HEC @ 4%, 13,200, Tax Liability, 3,43,200
Page 272 :
Agricultural Income, , 272, , Computation of income in case of growing and manufacturing of Rubber Rule 7A, If any person is engaged in growing and manufacturing of rubber, income shall be computed combined for, agriculture as well as business and 35% of such income shall be business income and balance shall be, agricultural income e.g. If income from growing + manufacturing is `100 lakhs, income from business shall, be `35 lakhs and income from agriculture shall be `65 lakhs., Computation of income from the growing and manufacturing of Coffee Rule 7B, If any person is engaged in growing and manufacturing of coffee, income shall be computed combined for, agriculture as well as business and 40% of such income shall be business income and balance shall be, agricultural income e.g. If income from growing + manufacturing is `100 lakhs, income from business shall, be `40 lakhs and income from agriculture shall be `60 lakhs., If any person is engaged in growing and curing of coffee, 25% of such income shall be business income and, balance shall be agricultural income., Computation of income in case of persons Growing and Manufacturing Tea Rule 8, If any person is engaged in growing and manufacturing of tea, income shall be computed combined for, agriculture as well as business and 40% of such income shall be business income and balance shall be, agricultural income e.g. If income from growing + manufacturing is `100 lakhs, income from business shall, be `40 lakhs and income from agriculture shall be `60 lakhs., MTP NOV -2020, 1. (a) Miss Deepika, a citizen of India, got married to Mr. John of Australia and left India for the first time, on 20.8.2021. She has not visited India again during the P.Y. 2021-22. She has derived the, following income for the year ended 31-3-2022:, , (i), (ii), , Particulars, Income from sale of centrifuged latex processed from rubber plants grown in, Kanyakumari., Income from sale of coffee grown and cured in Kodagu, Karnataka, , `, , 1,50,000, 2,00,000, , (iii) Income from sale of coffee grown, cured, roasted and grounded in Colombo., 5,00,000, Sale consideration was received in Chennai., (iv) Income from sale of tea grown and manufactured in West Bengal., 12,00,000, (v) Income from sapling and seedling grown in a nursery at Cochin. Basic, 2,00,000, operations were not carried out by her on land, You are required to determine the residential status of Miss Deepika and compute the business, income and agricultural income of Miss. Deepika for the Assessment Year 2022-23., (7 Marks), Answer:, Miss Deepika is said to be resident if she satisfies any one of the following basic conditions:, , (i) Has been in India during the previous year for a total period of 182 days or more, (or), (ii) Has been in India during the 4 years immediately preceding the previous year for a total, period of 365 days or more and has been in India for at least 60 days during the previous year., Miss Deepika’s stay in India during the P.Y.2021-22 is 142 days [30+31+30+31+20] which is less, than 182 days. However, her stay in India during the P.Y.2021-22 exceeds 60 days. Since, she left, India for the first time, her stay in India during the four previous years prior to P.Y.2021-22, would be more than 365 days. Hence, she is a resident for P.Y.2021-22., Further, Miss Deepika would be “Resident and ordinarily resident” in India in during the previous year, 2021-22, since her stay in India in the last seven previous years prior to P.Y.2021-22 is more than 730, days and she must be resident in the preceding ten years.
Page 273 :
Agricultural Income, , 273, , Computation of business income and agricultural income of Miss Deepika for A.Y. 2022-23, Particulars, Income, Business Agricultural, Income, Income, `, `, (i) Income from sale of centrifuged latex processed, from rubber plants grown in Kanyakumari (Apportioned, between business and agricultural income in the ratio of, 97,500, 52,500, 1,50,000, 35:65 as per Rule 7A of Income-tax Rules, 1962), (ii) Income from sale of coffee grown and cured in Kodagu,, Karnataka (Apportioned between business and, agricultural income in the ratio of 25:75, as per Rule, 7B(1) of the Income-tax Rules, 1962), 2,00,000, 50,000, 1,50,000, (iii) Income from sale of coffee grown, cured, roasted and, grounded in Colombo and received in Chennai [See, Note 1 below], 5,00,000 5,00,000, –, (iv) Income from sale of tea grown and manufactured in, West Bengal (Apportioned between business and, agricultural income in the ratio of 40:60 as per Rule 8 of, 7,20,000, 12,00,000 4,80,000, the Income-tax Rules, 1962), (v) Income from sapling and seedling grown in a nursery at, Cochin. Basic operations were not carried out on land, [See Note 2 below], 2,00,000, –, 2,00,000, 22,50,000 10,82,500, 11,67,500, Notes:, (1) Since Miss Deepika is resident and ordinarily resident in India for A.Y. 2022-23, her global income is, taxable in India. Entire income from sale of coffee grown, cured, roasted and grounded in Colombo is, taxable as business income since such income is earned from sale of coffee grown, cured, roasted and, grounded outside India i.e., in Colombo., , (2) As per Explanation 3 to section 2(1A), income derived from sapling or seedlings grown in a nursery, would be deemed to be agricultural income, whether or not the basic operations were carried out on, land. Hence, income of `2,00,000 from sapling and seedling grown in a nursery at Cochin is, agricultural income., Marketing operations /Marketing process, Process ordinarily employed to render the produce fit to be taken to the market: Sometimes, to make, the agricultural produce a saleable commodity, it becomes necessary to perform some kind of process on the, produce. The income from the process employed to render the produce fit to be taken to the market would be, agricultural income., However, it must be a process ordinarily employed by the cultivator or receiver of rent in kind and the, process must be applied to make the produce fit to be taken to the market., The ordinary process employed to render the produce fit to be taken to market includes thrashing,, winnowing, cleaning, drying, crushing etc., For example, the process ordinarily employed by the cultivator to obtain the rice from paddy is to first, remove the hay from the basic grain, and thereafter to remove the chaff from the grain. The grain has to be, properly filtered to remove stones etc. and finally the rice has to be packed in gunny bags for sale in the, market., After such process, the rice can be taken to the market for sale. This process of making the rice ready for the, market may involve manual operations or mechanical operations. All these operations constitute the process, ordinarily employed to make the product fit for the market., The produce must retain its original character in spite of the processing unless there is no market for selling
Page 274 :
Agricultural Income, , 274, , it in that condition., However, if marketing process is performed on a produce which can be sold in its raw form, income derived, therefrom is partly agricultural income and partly business income., Example, Threshing is done in case of wheat crop to render it fit for sale, similarly, tobacco leaves are dried to make, them fit for sale. In all such cases, it will continue to be agricultural income., Income derived from animal husbandry, fisheries, poultry farming, dairy farming etc. shall not, be considered to be agricultural income., Income derived from saplings or seedlings growing in a nursery shall be considered to be, agricultural income. (whether basic and subsequent operations have been carried out or not), Income from sale of agricultural land shall not be considered to be agricultural income rather, it will be considered to be capital gain., RTP NOV -2020, 1. Explain with brief reasons, whether the following income can be regarded as agricultural income, as per, the provisions of the Income-tax Act, 1961:, (i) Rent received for letting out agricultural land for a movie shooting., (ii) Income from sale of seedlings in a nursery adjacent to the agricultural lands owned by an assessee., Answer:, (i) Rent received for letting out agricultural land for a movie shooting:, As per section 2(1A), “agricultural income” means, inter alia,, • any rent or revenue derived from land, • which is situated in India and is used for agricultural purposes., In the present case, rent is being derived from letting out of agricultural land for a movie shoot, which is not, an agricultural purpose and hence, it does not constitute agricultural income., , (ii) Income from sale of seedlings in a nursery:, As per Explanation 3 to section 2(1A), income derived from saplings or seedlings grown in a nursery is, deemed to be agricultural income, whether or not the basic operations were carried out on land., Therefore, the amount received from sale of seedlings in a nursery adjacent to the agricultural lands owned, by the assessee constitutes agricultural income., MTP NOV -2020, 1. Sham Singh spends `1,00,000 on cultivation and harvesting of his agricultural produce. 50% of, the production is sold for `1,10,000 and rest is stored for self consumption. What is the amount of, the agricultural income?, (a) ` 60,000, (b) ` 1,10,000, (c) ` 1,20,000, (d) ` 1,00,000, Answer: (a), Amount of agricultural income, Sales, Less: cost of cultivation and harvesting (50%), Agricultural income, , (2 Marks), , 1,10,000, (50,000), 60,000
Page 275 :
Agricultural Income, , 275, , Illustration 2: Mr. X, a resident, has provided the following particulars of his income for the P.Y.2021-22., i. Income under the head salary, `3,40,000, ii. Income under the head house property, `3,00,000, iii. Agricultural income from a land in Jaipur, `1,80,000, iv. Expenses incurred for earning agricultural income, `1,20,000, Compute his tax liability assuming his age is (a) 45 years, (b) 70 years, Solution:, Computation of total income of Mr. X for the A.Y.2022-23, (a) Computation of tax liability (age 45 years), Particulars, `, Income under the head salary, 3,40,000, Income under the head house property, 3,00,000, Gross Total Income, 6,40,000, Less: Deductions under Chapter VI-A, Nil, Total Income, 6,40,000, Agricultural income (1,80,000 – 1,20,000) =, 60,000, Computation of tax liability, 52,500, Step 1: Tax on (`6,40,000 + ` 60,000 = ` 7,00,000), Step 2: Tax on (` 2,50,000 + ` 60,000) = ` 3,10,000), (3,000), Step 3: ` 52,500 – ` 3,000, 49,500, Tax before health & education cess, 49,500, Add : HEC @ 4%, 1980, Tax Liability, 51,480, (b) Computation of tax liability (age 70 years), Step 1: Tax on (` 6,40,000 + ` 60,000 = ` 7,00,000), 50,000, Step 2: Tax on (` 3,00,000 + ` 60,000 = ` 3,60,000), (3,000), Step 3: ` 50,000 – ` 3,000, 47,000, Tax before health & education cess, 47,000, Add : HEC @ 4%, 1880, Tax Liability, 48,880, Illustration 3: Mr. X grows sugarcane and uses the same for the purpose of manufacturing sugar in his, factory. 30% of sugarcane produce is sold for ` 10 lacs, and the cost of cultivation of such sugarcane is ` 5, lacs. The cost of cultivation of the balance sugarcane (70%) is `14 lacs and the market value of the same is, `22 lacs. After incurring ` 1.5 lacs in the manufacturing process on the balance sugarcane, the sugar was, sold for ` 25 lacs. Compute Mr. X’s business income and agricultural income., Solution:, Income from sale of sugarcane gives rise to agricultural income and from sale of sugar gives rise to business, income., Business income, = Sales – Market value of 70% of sugarcane produce – Manufacturing expenses, = `25 lacs – `22 lacs - `1.5 lacs = `1.5 lacs., Agricultural income = Market value of sugarcane produce – Cost of cultivation, = [`10 lacs + `22 lacs] – [`5 lacs + `14 lacs], = `32 lacs – `19 lacs, = `13 lacs., Illustration 4: Mr. X is engaged in growing and manufacturing of rubber. These are then sold in the market, for `30 lacs. The cost of growing rubber plants is `10 lacs and that of manufacturing rubber is `8 lacs., Compute his total income., Solution:, The total income of Mr. X comprises of agricultural income and business income., Total profits from the sale of rubber, = `30 lacs – `10 lacs – `8 lacs = `12 lacs., Agricultural income, = 65% of `12 lacs. = `7.8 lacs, Business income, = 35% of `12 lacs. = `4.2 lacs
Page 276 :
Agricultural Income, , 276, , Illustration 5: Mr. X has estates in rubber, tea and coffee. He derives income from them. He has a nursery, wherein he grows and sells the plants. For the previous year ending 31.03.2022, he furnishes the following, particulars of his income from estates and sale of plants. You are requested to compute the taxable income, and tax liability for the assessment year 2022-23:, `, (i) Growing and manufacturing of rubber, 5,00,000, (ii) Sale of coffee grown and cured, 3,50,000, (iii) Growing and manufacturing of tea, 7,00,000, (iv) Sale of plants from nursery, 1,00,000, He has long term capital gain on the sale of agricultural land in Delhi `3,13,500. He has received rent of, `7,000 p.m. by letting out one farm house near Delhi and he has incurred `20,000 on the repairs of the farm, house. He has not paid municipal taxes for the last ten years in connection with farm house and MCD has, issued him a notice for selling of farm house, hence he has paid municipal tax of `90,000., Solution:, Agricultural Income Business Income, (a) Income from growing and manufacturing of Rubber {Rule 7A}, [Agricultural income 65% and business income 35%], 3,25,000, 1,75,000, (b) Income from Coffee grown and cured {Rule 7B}, [Agricultural income 75% and business income 25%], 2,62,500, 87,500, (c) Income from growing and manufacturing of Tea {Rule 8}, [Agricultural income 60% and business income 40%], 4,20,000, 2,80,000, (d) Income from growing and selling of plants, 1,00,000, ---------Total, 11,07,500, 5,42,500, Computation of Income under the head House Property, `, Gross Annual Value (7,000 x 12), 84,000.00, Less: Municipal Taxes, (90,000.00), Net Annual Value, (6,000.00), Less: 30% of NAV u/s 24(a), Nil, Less: Interest on capital borrowed u/s 24(b), Nil, Loss under the head House Property, (6,000.00), Option I, Loss under the head house property set off from long term capital gains, Long term capital gains, 3,13,500.00, Loss under the head House Property, (6,000.00), Long term capital gains after adjusting loss from house property, 3,07,500.00, Income under the head Business/Profession, 5,42,500.00, Gross Total Income, 8,50,000.00, Less: Deduction under section 80C to 80U, Nil, Total Income, 8,50,000.00, Computation of Tax Liability, Tax on long term capital gain `3,07,500 @ 20% u/s 112, 61,500.00, Normal income `5,42,500, Tax on (5,42,500 + 11,07,500), 3,07,500.00, Tax on(2,50,000 + 11,07,500), (2,19,750.00), Tax on Normal Income (3,07,500 – 2,19,750), 87,750.00, Tax before health & education cess, 1,49,250.00, Add: HEC @ 4%, 5,970.00, Tax Liability, 1,55,220.00, Option II, Loss under the head house property set off from business income, Income under the head Business/Profession, 5,42,500.00, Loss under the head House Property, (6,000.00), Income under the head Business/Profession, 5,36,500.00, Income under the head Capital gain (LTCG), 3,13,500.00
Page 277 :
Agricultural Income, Gross Total Income, Less: Deduction under section 80C to 80U, Total Income, Computation of Tax Liability, Tax on long term capital gain `3,13,500 @ 20% u/s 112, Normal income `5,36,500, Tax on (5,36,500 + 11,07,500), Tax on (2,50,000 + 11,07,500), Tax on Normal Income (3,05,700 – 2,19,750), Tax before health & education cess, Add: HEC @ 4%, Tax Liability, Rounded off u/s 288B, Hence Option II is better., , 277, 8,50,000.00, Nil, 8,50,000.00, 62,700.00, 3,05,700.00, (2,19,750.00), 85,950.00, 1,48,650.00, 5,946.00, 1,54,596.00, 1,54,600.00, , Income from a Farm Building Section 2(1A)(c), If any building is in the agricultural field or is very near to the agricultural field and it is being used for, storing agricultural produce or for storing agricultural implements or it is being used as dwelling unit by the, farmer himself, such building is called farm building and its income shall be computed as per provisions, given under the head house property and income shall be considered to be agricultural income., , In addition to the above conditions any one of the following two conditions should also be satisfied:, (i) The land should either be assessed to land revenue in India or be subject to a local rate assessed and, collected by the officers of the Government as such or;, (ii) Where the land is not so assessed to land revenue in India or is not subject to local rate:a. It should not be situated in any area as comprised within the jurisdiction of a municipality or a cantonment, board and which has a population not less than 10,000 or, b. It should not be situated in any area within such distance, measured aerially, in relation to the range of, population as shown hereunder –, Shortest aerial distance from the local limits Population according to the last preceding census of, of a municipality or cantonment board which the relevant figures have been published before, referred to in item a., the first day of the previous year, (i) ≤ 2 kilometers, > 10,000 ≤ 1,00,000, (ii) ≤ 6 kilometers, > 1,00,000 ≤ 10,00,000, (iii) ≤ 8 kilometers, > 10,00,000, Examples:, Area, , (i), (ii), (iii), (iv), (v), (vi), (vii), (viii), (ix), (x), , A, B, C, D, E, F, G, H, I, J, , Shortest aerial distance from, the local limits of a, municipality or cantonment, board referred to in item a., , 1 km, 1.5 kms, 2 kms, 3 kms, 4 kms, 5 kms, 6 kms, 7 kms, 8 kms, 9 kms, , Population according to the, last preceding census of, which the relevant figures, have been published before, the first day of the previous, year, 9,000, 12,000, 11,00,000, 80,000, 3,00,000, 12,00,000, 8,000, 4,00,000, 10,50,000, 15,00,000, , Would, income, derived from farm, building situated in, this area be treated as, agricultural income?, , Yes, No, No, Yes, No, No, Yes, Yes, No, Yes
Page 278 :
Agricultural Income, , 278, , Self Reading, Judicial Decisions, B. Gupta Private Ltd. v CIT, (HC), Compensation received from an insurance company on account of damage caused to the crops is, agricultural income., Venkataswamy Naidu v CIT, (SC), Income from butter and cheese making is not agricultural income., Sri Ranga Vilas Ginning & Oil Mills v. CIT, (HC), Income from supplying surplus water to other agriculturists is not agricultural income., New Ambadi Estates Ltd. v CIT, (SC), Harvest crops on purchased land is not agricultural income., K. Lakshmansa & Co. v CIT, (SC), If the assessee was growing mulberry leaves, feeding them to silkworms and obtaining silk cocoons,, income from sale of silk cocoons would not be agricultural income., Illustration 6: Mr. X is employed in MP Agricultural University and getting basic pay `20,000 p.m. He, claims that it is his agricultural income. Discuss., Solution: Income from an agricultural university cannot be considered to be agricultural income rather it is, his income under the head salary., Illustration 7: Mr. X has sold his agricultural land in Delhi and there are long term capital gains of, `10,00,000. Mr. X claims it to be his agricultural income. Discuss., Solution: Income from sale of agricultural land cannot be considered to be agricultural income and, accordingly it is chargeable to tax under the head capital gains., Illustration 8: Mr. X holds shares in ABC Ltd., an Indian Company, which is engaged in agricultural, operations. He has received dividends of `1,20,000 from ABC Ltd. and claims that it is his agricultural, income. Discuss., Solution: Dividend from a company which is engaged in agricultural operations cannot be considered to be, agricultural income rather it is dividend income of the recipient. Such dividend income shall be taxable in, the hands of shareholder., Examples from ICAI Module, Example 1: A rural society has as its principal business the selling on behalf of its member societies, butter, made by these societies from cream sold to them by farmers. The making of butter was a factory process, separated from the farm., The butter resulting from the factory operations separated from the farm was not an agricultural product and, the society was, therefore, not entitled to exemption under section 10(1) in respect of such income., Example 2: X was the managing agent of a company. He was entitled for a commission at the rate of 10%, p.a. on the annual net profits of the company. A part of the company’s income was agricultural income. X, claimed that since his remuneration was calculated with reference to income of the company, part of which, was agricultural income, such part of the commission as was proportionate to the agricultural income was, exempt from income tax., Since, X received remuneration under a contract for personal service calculated on the amount of profits, earned by the company; such remuneration does not constitute agricultural income., Example 3: Y owned 100 acres of agricultural land, a part of which was used as pasture for cows. The lands, were purely maintained for manuring and other purposes connected with agriculture and only the surplus, milk after satisfying the assessee’s needs was sold. The question arose whether income from such sale of, milk was agricultural income., The regularity with which the sales of milk were effected and quantity of milk sold showed that the assessee, carried on regular business of producing milk and selling it as a commercial proposition. Hence, it was not, agricultural income., Example 4: In regard to forest trees of spontaneous growth which grow on the soil unaided by any human, skill and labour there is no cultivation of the soil at all., Even though operations in the nature of forestry operations performed by the assessee may have the effect of, nursing and fostering the growth of such forest trees, it cannot constitute agricultural operations. Income, from the sale of such forest trees of spontaneous growth does not, therefore, constitute agricultural income.
Page 279 :
Agricultural Income, , 279, , MULTIPLE CHOICE QUESTIONS, AGRICULTURAL INCOME, 1. Which of the following would be agricultural income (a) Income from breeding of livestock, (b) Income from poultry farming, (c) Rent received from land used for movie shooting, (d) Rent received from land used for grazing of cattle required for agricultural activities, 2. The proportion of agricultural and business income in case of income derived from the sale of coffee, grown and cured by the assessee in India is (a) 65% and 35%, respectively, (b) 75% and 25%, respectively, (c) 60% and 40%, respectively, (d) 70% and 30%, respectively, 3. The proportion of agricultural and business income in case of income derived by the assessee from, growing of tea leaves in India and manufacturing of tea is (a) 65% and 35%, respectively, (b) 75% and 25%, respectively, (c) 60% and 40%, respectively, (d) 70% and 30%, respectively, 4. In case of an individual aged 61 years, partial integration of agricultural income is not required if, his–, (a) Net agricultural income does not exceed `5,000., (b) Non-agricultural income does not exceed `2,50,000., (c) Non-agricultural income does not exceed `3,00,000., (d) Either (a) or (c) above., 5. Income derived from farm building situated in the immediate vicinity of an agricultural land (not, assessed to land revenue) would be treated as agricultural income if such land is situated in –, (a) an area at a distance of 3 kms from the local limits of a municipality which has a population of 80,000 as, per last census, (b) an area within 1.5 kms from the local limits of a municipality and has a population of 12,000 as per last, census, (c) an area within 2 kms from the local limits of a municipality and has a population of 11,00,000 as per last, census, (d) an area within 8 kms from the local limits of a municipality and has a population of 10,50,000 as per last, census, 6. Mr. Harini earned income of `4,00,000 from sale of tea grown and manufactured in Shimla. Income, from sapling and seedling grown in nursery at Cochin is `80,000. Her agricultural income is, (a) `4,80,000, (b) `4,00,000, (c) `2,40,000, (d) `3,20,000, 7. Mr. Prem earned income of `22 lakhs from manufacture and sale of coffee grown, cured, roasted, and grounded by him in India. The business income chargeable to tax in his hands would be –, (a) `8,80,000, (b) `5,50,000, (c) `13,20,000, (d) `16,50,000, 8. Mr. X has agricultural land which he has given on rent, in this case income shall be, (a) taxable under the head other sources, (b) taxable under the head house property, (c) taxable under the head Business/Profession, (d) Agricultural income, (e) none of these, 9. Which of the following statements is correct?, (a) Dividend received from a domestic company doing agriculture is agricultural income, (b) Share received by a partner out of profits of partnership firm doing agriculture is taxable under the head, Business/Profession, (c) Salary and interest received by a partner from partnership firm doing agriculture is taxable under the, head Business/Profession, (d) none of these
Page 280 :
Agricultural Income, , 280, , 10. Which of the following statements is correct?, (a) Dividend received from a foreign company doing agriculture is agricultural income, (b) Share received by a partner out of profits of partnership firm doing agriculture is exempt u/s 10(2A), (c) Salary received by a partner from partnership firm doing agriculture is taxable under the head Salary, (d) none of these, 11. Which of the following statements is correct?, (a) Dividend received from a foreign company doing agriculture is exempt., (b) Share received by a partner out of profits of partnership firm doing agriculture is taxable under the head, other sources, (c) Salary received by a partner from partnership firm doing agriculture is taxable under the head Salary, (d) Interest received by a partner from partnership firm doing agriculture is taxable under the head other, sources, (e) none of these, 12. Which of the following statements is correct?, (a) in order to constitute agriculture there must be basic operations, (b) in order to constitute agriculture there must be subsequent operations, (c) in order to constitute agriculture there must be both basic and subsequent operations, (d) in order to constitute agriculture there must be ploughing of field, (e) in order to constitute agriculture there must be either basic or subsequent operations, (f) none of these, 13. Which of the following statements is not correct?, (a) If any person is engaged in growing and manufacturing of rubber, income from business shall be 35%, (b) If any person is engaged in growing and manufacturing of coffee, income from business shall be 35%, (c) If any person is engaged in growing and manufacturing of tea, income from business shall be 40%, (d) If any person is engaged in growing and curing of coffee, income from business shall be 25%, (e) none of these, 14. Which of the following statements is not correct?, (a) Income from animal husbandry or fisheries etc. shall be considered to be agriculture income, (b) Income from sale of sapling or seeding grown in nursery shall be considered to be agriculture income, (c) Income from sale of agricultural land is taxable under the head capital gains, (d) Rent received for letting out agriculture land for a movie shooting shall not be considered to be, agriculture income, (e) none of these, 15. Which of the following statements is not correct?, (a) If an individual has income from salary ` 3,50,000 and agricultural income ` 2,00,000, partial integration, is applicable, (b) If an individual has income from salary ` 5,00,000 and agricultural income ` 50,000, partial integration, is applicable, (c) If an individual has income from salary `2,00,000 and agricultural income `10,00,000, partial integration, is applicable, (d) If an individual has income from salary ` 3,00,000 and agricultural income ` 5,000, partial integration is, applicable, (e) (b) & (c), (f) (c) & (d), (g) none of these, , Solutions are given on our website, www.mkgeducation.com
Page 281 :
Agricultural Income, , 281, , PRACTICE PROBLEMS, TOTAL PROBLEMS 6, Problem 1., Mr. X (non-resident, aged 68 years) has incomes as given below:, (i) Income under the head Salary `3,00,000, (ii) Income under the head House Property `1,20,000, (iii) Income from long term capital gains `50,000, (iv) Casual income `30,000, (v) Agricultural income `60,000, (vi) Deductions under section 80D to 80U `1,40,000, (vii) He has invested `40,000 in Kisan Vikas Patra, `20,000 in equity shares of infrastructure, development companies., Compute his total income and tax liability for the assessment year 2022-23., Answer: Total Income: `3,40,000; Tax Liability: `20,280, Problem 2., Mrs. X (aged 58 years) has income and losses as given below:, (i) Income from growing and manufacturing of Rubber `3,00,000, (ii) Income from growing and curing coffee `2,00,000, (iii) Income under the head Salary `2,40,000, (iv) Loss under the head House Property `1,00,000, (v) Income from short term capital gains `40,000, (vi) Income from long term capital gains `50,000, (vii) Casual income `60,000, Compute her total income and tax liability for the assessment year 2022-23., Answer: Total Income: `4,45,000; Tax Liability: `33,800, Problem 3., Mrs. X (resident but not ordinarily resident) have incomes as given below:, (i) Income from growing and manufacturing of Tea in India `10,00,000, (ii) Income from house property situated outside India `3,50,000, received outside India., (iii) Income from agriculture in Nepal `1,50,000, received in India, (iv) Income from business in Paris and received in Paris ` 1,00,000, Compute her total income and tax liability for the assessment year 2022-23., Answer: Total Income: `5,50,000; Tax Liability: `78,000, Problem 4., Mr. X (resident but not ordinarily resident) have incomes and losses as given below:, (i) Loss from house I in India `80,000, (ii) Income from house II in India `1,00,000, (iii) Carried forward loss assessment year 2011-12 from house III in India `50,000, (iv) Income under the head Business/Profession in India `2,20,000, (v) Royalty received in the UK for use of formula in U.K. `30,000, (vi) Long term capital gains in India `1,00,000, (vii) Income from agriculture in Indonesia but received in India and subsequently invested it in, Indonesia `50,000, (viii) Income from agriculture in India `2,00,000, Compute his total income and tax liability for the assessment year 2022-23., Answer: Total Income: `3,90,000; Tax Liability: `9,880, Problem 5., A partnership firm XY has agricultural income `2,00,000, income under the head business/profession, `1,00,000 and long term capital gains `10,000., Compute its tax liability for the assessment year 2022-23.
Page 282 :
Agricultural Income, , 282, , Answer: Tax Liability: `33,280, Problem 6., A partnership firm Z & Co. has agricultural income `20,00,000 and its partner Mr. Z has received `5,00,000, being his share in the profits of partnership. Mr. Z has income under the head house property `2,75,000., Compute tax liability of the partnership firm and also that of Mr. Z., Answer: Tax Liability: Partnership Firm: Nil; Mr. Z: Nil, , Detailed Solutions of All Questions are given on our, website www.mkgeducation.com
Page 283 :
Agricultural Income, , 283, , EXAMINATION QUESTIONS, MAY – 2018 (Old Course), Question 2(a), (5 Marks), Miss. Kavita, a resident and ordinarily resident in India, has derived the following income for the year ended, 31-3-2022., `, (i) Income from sale of centrifuged latex processed from rubber plants grown in Darjeeling., 1,00,000, (ii) Income from sale of coffee grown and cured in Yercaud, Tamil Nadu, 2,00,000, (iii) Income from sale of coffee grown, cured in Colombo., 5,00,000, Sale Consideration was received in Chennai., (iv) Income from sale of tea grown and manufactured in Shimla., 10,00,000, (v) Income from sapling and seedling grown in a nursery at Cochin., 2,00,000, Basic operations were not carried out by her on land., You are required to compute the Income of Miss. Kavita for the Assessment Year 2022-23., Solution:, Computation of Income, Sources, Agricultural, Business Income, Other, Income, Sources, 65,000, 35,000, (i) Income from growing and manufacturing of, Rubber {Rule 7A}, Agricultural income 65% and business income 35%, (ii) Income from Coffee grown and cured {Rule 7B}, 1,50,000, 50,000, Agricultural income 75% and business income 25%, (iii) Income from Coffee grown and cured outside, 1,25,000, 3,75,000, India, (iv) Income from growing and manufacturing of Tea, 6,00,000, 4,00,000, {Rule 8} Agricultural income 60% and business, income 40%, (v) Income from sapling and seedling grown in a, 2,00,000, nursery at Cochin, Total, 10,15,000, 6,10,000, 3,75,000, , MAY – 2018 (New Course), Question 5(b), (7 Marks), Mr. Avani, a resident aged 25 years, manufactures tea leaves from the tea plants grown by him in India., These are then sold in the Indian market for `40 lakhs. The cost of growing tea plants was `15 lakhs and the, cost of manufacturing tea leaves was `10 lakhs., Compute her tax liability for the Assessment year 2022-23., Solution:, As per Rule 8, If any person is engaged in growing and manufacturing of tea, income shall be computed, combined for agriculture as well as business and 40% of such income shall be business income and balance, shall be agricultural., Income, = Sales – Cost of growing tea plants – cost of manufacturing tea leaves, = `40 lacs – `15 lacs - `10 lacs = `15 lacs., , Business Income = 15,00,000 x 40% = 6,00,000, Agriculture Income = 15,00,000 x 60% = 9,00,000, Computation of Tax Liability, Tax on (6,00,000 + 9,00,000) at slab rate, Tax on (2,50,000 + 9,00,000) at slab rate, , `, 2,62,500, (1,57,500)
Page 284 :
Agricultural Income, Tax on normal income (2,62,500 – 1,57,500), Add: HEC @ 4%, Tax Liability, , 284, 1,05,000, 4,200, 1,09,200, , MAY – 2017, Question 2(a) (ii), (4 Marks), Discuss with brief reasons, whether rent received for letting out agricultural land for a movie shooting and, amounts received from sale of seedlings in a nursery adjacent to the agricultural lands owned by an assessee, can be regarded as agricultural income, as per the provisions of the provisions of the Income tax Act,1961., Answer:, Rent received from letting out agricultural land for a movie shooting: As per section 2(1A) Agricultural, income means, any rent or revenue derived from land which is situated in India and is used for agricultural, purposes., In the present case, rent is being derived from letting out of agricultural land for a movie shoot, which is not, an agricultural purpose. Hence, Rent received from letting out agricultural land for a movie shooting is not, Agricultural income, Amount received from sale of seedlings in a nursery: As per Section 2(1A), Income derived from sapling, or seedling grown in nursery is deemed to be agricultural Income., Therefore, Amount received from sale of seedlings in a nursery adjacent to the agricultural lands is, Agricultural income., , NOV – 2016, Question 5(a), (4 Marks), Mr. Kamal grows paddy and uses the same for the purpose of manufacturing of rice in his own Rice Mill., The cost of cultivation of 40% of paddy produce is `7,00,000 which is sold for `15,00,000; and the cost of, cultivation of balance 60% of paddy is `12,00,000 and the market value of such paddy is `24,00,000. To, manufacture the rice, he incurred `2,00,000 in the manufacturing process on the balance (60%) paddy., The rice was sold for `30,00,000., Compute the Business income and Agriculture Income of Mr. Kamal., Solution:, As per Rule 7 of Income Tax Rules 1962, if any person is growing agricultural produce and is using it in his, own factory to manufacture a product, in such cases it will be presumed that the assessee has sold his, agricultural produce to his manufacturing unit at the market price and income shall be computed, accordingly. In the given case, computation of income shall be as given below:, , Agricultural income = Actual sale of paddy + Market value of paddy transferred to the, manufacturing unit – Cost of cultivation, = [`15 lacs + `24 lacs] – [`7 lacs + `12 lacs], = `39 lacs – `19 lacs, = `20 lacs, Business income = Sales – Market value of 60% of paddy produce – Manufacturing expenses, = `30 lacs – 24 lacs – 2 lacs, = 4 lacs, , NOV – 2011, Question 4, (2 Marks), Mr. X, a 60 years old individual, is engaged in the business of roasting and grinding of coffee, derives, income `10 lacs during the financial year 2021-22. Compute the tax payable by him assuming he has not, earned any other income during the financial year 2021-22., Answer:, Computation of income from the growing and manufacturing of Coffee Rule 7B, Income derived from the sale of coffee grown and cured by the seller in India shall be computed as if it were
Page 285 :
Agricultural Income, , 285, , income derived from business, and 25% of such income shall be deemed to be income liable to tax., Income derived from the sale of coffee grown and manufactured by the seller in India, with or without, mixing of chicory or other flavouring ingredients, shall be computed as if it were income derived from, business, and 40% of such income shall be deemed to be income liable to tax., In the given case, assessee is not engaged in growing of coffee hence entire income is business income and, tax liability shall be as given below:, `, Total income, 10,00,000, Tax + HEC, 1,14,400, , MAY – 2011, Question 1, (2 Marks), Mr. X earned `5,00,000 from sale of Coffee grown and cured by him. He claims the entire income as, agricultural income, hence exempt from tax. Is he correct?, Answer., Mr. X is not correct in claiming the entire income as agricultural income. As per rule 7B, in the case of, income derived from the sale of coffee grown and cured by the seller in India, 25% of such income is, taxable as business income under the head ‘Profits and gains from business or profession’ and the balance, (i.e. 75%) is agricultural income. Hence, only `3,75,000 (75% of `5,00,000) being agricultural income is, exempt from tax., , JUNE – 2009, Question 1, (2 Marks), Whether the income derived from saplings or seedlings grown in a nursery is taxable under the Income-tax, Act, 1961?, Answer ., As per Explanation 3 to section 2(1A) of the Act, income derived from saplings or seedlings grown in a, nursery shall be deemed to be agricultural income and exempt from tax, whether or not the basic operations, were carried out on land., , NOV – 2004, Question 1, (3 Marks), Mr. X has estates in Rubber, Tea and Coffee. He derives income from them. He has also a nursery wherein, he grows plants and sells. For the previous year ending 31.03.2022, he furnishes the following particulars of, his sources of income from estates and sale of Plants., You are requested to compute the taxable income and tax liability for the assessment year 2022-23., `, (i) Manufacture of rubber, 5,00,000, , (ii) Manufacture of coffee grown and cured, , 3,50,000, , (iii) Manufacture of tea, , 7,00,000, , (iv) Sale of plants from nursery, , 1,00,000, , Answer:, , Income from growing and manufacturing of Rubber {Rule 7A}, Agricultural income 65% and business income 35%, Income from Coffee grown and cured {Rule 7B}, Agricultural income 75% and business income 25%, Income from growing and manufacturing of Tea {Rule 8}, Agricultural income 60% and business income 40%, Income from growing and selling of plants, Total, Computation of Tax Liability, Normal income `5,42,500, Tax on (5,42,500 + 11,07,500), , Agricultural, Income, , Business, Income, , 3,25,000, , 1,75,000, , 2,62,500, , 87,500, , 4,20,000, 1,00,000, 11,07,500, , 2,80,000, xxxxx, 5,42,500, 3,07,500.00
Page 286 :
Agricultural Income, Tax on (2,50,000 + 11,07,500), Tax before education cess (3,07,500 – 2,19,750), Add: HEC @ 4%, Tax Liability, , 286, (2,19,750.00), 87,750.00, 3,510.00, 91,260.00, , MAY – 1998, Question 2, (3 Marks), From the following information, compute taxable income and tax liability of Mrs. X for the assessment year, 2022-23., `, Income from business – letting cycles on hire, 2,40,000, Fixed deposit interest received from companies on deposits made of sale proceeds of land, 18,000, Dividends from an Indian company having rubber plantations, 6,000, Salary received as a partner from a firm growing and manufacturing tea, 40,000, Sale of agricultural produce, 1,75,000, Payment of government tax on agricultural lands, 6,000, Expenses on power, irrigation cess and farm labour, 10,000, Purchase of seeds, 1,000, Tractor hire charges (for agricultural operations), 2,500, Answer:, Computation of income from agriculture, Salary from firm growing and manufacturing tea, 24,000, 40,000 x 60% (as per decision in R.M. Chidambaram Pillai v CIT), Sale of agricultural produce, 1,75,000, Less : Government tax, (6,000), Power, Irrigation cess etc, (10,000), Purchase of seeds, (1,000), Tractor hire charges, (2,500), Agricultural income, 1,79,500, Computation of Non agricultural income :, Income from Business:, Cycle hire charges, 2,40,000, Salary from firm (non –agricultural part – 40,000 x 40%), 16,000, Other sources:, Dividends from Plantation company –, 6,000, Interest on fixed deposit with companies:, 18,000, Non-Agricultural Income, Computation of Tax Liability, Tax on (2,80,000 + 1,79,500) at slab rate, Tax on (2,50,000 + 1,79,500) at slab rate, Tax liability (10,475 – 8,975), Less: Rebate u/s 87A, Tax Liability, , 2,80,000, , 10,475, (8,975), 1,500, (1,500), Nil
Page 287 :
Clubbing of Income, , 287, , CLUBBING OF INCOME, (INCOME OF OTHER PERSONS INCLUDED, IN ASSESSEE’S TOTAL INCOME), , SECTION 60 TO 65, PARTICULARS, SECTIONS, Transfer of income without transfer of assets, 60, Revocable transfer of assets, 61, Transfer irrevocable for a specified period, 62, Transfer and revocable transfer, 63, Income from assets transferred to the spouse, 64(1), Income from assets transferred to son’s wife, 64(1), Income from assets transferred to any person for the benefit of the spouse of the, 64(1), transferor/ son’s wife of the transferor, Remuneration of a spouse from a concern in which the other spouse has substantial, 64(1), interest, Clubbing of income of a minor child, 64(1A), Income from self acquired property converted to joint family property, 64(2), Liability of person in respect of income included in the income of another person, 65, (Deleted from syllabus), , In general a person has to pay tax only on his own income but sometimes incomes of other persons is added, to his income to charge tax from him, it is called ‘clubbing of income’. Clubbing provision are applicable to, check tax evasion., Clubbing provision are applicable in the following cases: 1. Transfer of income without transferring the asset Section 60, If any person has transferred any income without transferring the asset, in such cases clubbing provision, shall be applicable., Example, Mr. X has two deposits of `10 lakhs each and interest income of each deposit is `1.5 lakhs. He has, transferred income of one of the deposit to his brother Mr. Y. In this case, clubbing provision shall be, applicable and income shall be taxable in the hands of Mr. X., , MAY – 2008, Question 1, (2 Marks), Mr. X has transferred through a duly registered document the income arising from a godown, to his son,, without transferring the godown. In whose hands will the rental income from godown be charged?, Answer., Section 60 expressly states that where there is transfer of income from an asset without transfer of the asset, itself, such income shall be included in the total income of the transferor. Hence, the rental income derived, from the godown shall be charged in the hands of Mr. X., 2. Transfer of asset through revocable transfer, Section 61, If any person has transferred any asset through revocable transfer, income from that asset shall be clubbed in, the income of transferor.
Page 288 :
Clubbing of Income, , 288, , Example, Mr. X has transferred a deposit of `10 lakhs to his friend Mr. Y with the condition that the deposit can be, taken back by him at any time. In this case, clubbing provision shall be applicable., 3. Transfer of an asset through irrevocable transfer, Section 62, If any person has transferred any asset through irrevocable transfer, in this case clubbing provision shall not, apply. Similarly If any person has transferred any asset through a transfer which is not revocable during the, life time of the beneficiary, clubbing provision shall not apply., Example: Mr. X has transferred one asset to Mr. Y with the condition that the asset shall be retained by Mr., Y as long as he is alive and after that the asset shall be taken back by Mr. X. In this case, clubbing provision, shall not apply., , Provided that the transferor derives no direct or indirect benefit from such income in either case., All income arising to any person by virtue of any such transfer shall be chargeable to income-tax as the, income of the transferor as and when the power to revoke the transfer arises, and shall then be included in, his total income, Meaning of revocable transfer Section 63, For the purposes of sections 60, 61 and 62 and of this section,—, A transfer shall be deemed to be revocable if—, (i) it contains any provision for the re-transfer directly or indirectly of the whole or any part of the income or, assets to the transferor, or, (ii) it, in any way, gives the transferor a right to re-assume power directly or indirectly over the whole or any, part of the income or assets, 4. TRANSFER OF ASSETS TO SPOUSE SECTION 64(1)[V. IMP.], (i) If any person has transferred any asset, other than a house property to his or her spouse directly or, indirectly without adequate consideration, in such cases, income of the asset shall be clubbed in the, income of transferor., (ii) If the asset is transferred for adequate consideration, clubbing provisions are not applicable. Similarly if, the asset is transferred under an agreement to live apart, clubbing provision shall not apply., Example, Mr. X has transferred one deposit to his wife Mrs. X by charging full consideration of `10,00,000. In this, case, interest income shall not be clubbed in the income of Mr. X., (iii) If there is inadequate consideration, clubbing provisions shall be applicable only with regard to the, income relating to that part of the consideration which is considered to be inadequate., Example, Mr. X has transferred one deposit of `10,00,000 for a consideration of `7,00,000 and there is interest, income of `1,00,000 from the said deposit, in this case income of `30,000 shall be clubbed., (iv) In order to apply clubbing provision relationship of husband and wife must exist on the date of transfer, of the asset and also on the date of accrual of income otherwise clubbing provision shall not be apply, as, decided in Philip Johan Plasket Thomas v. CIT (SC)., Example, Mr. X has transferred certain assets on 01.01.2022 to his would be wife. He got married on 10.01.2022, in, this case clubbing provision shall not apply., (v) If any person has transferred the asset to the spouse and there is accretion to the asset, income from such, accretion shall not be clubbed, as decided in case of M. P. Birla (HC).e.g. Mr. X gifted certain shares to, Mrs. X and Mrs. X has received bonus shares. In this case dividend or capital gains from original shares, shall be clubbed in the hands of Mr. X but dividend or capital gains from bonus shares shall not be, clubbed rather Mrs. X herself has to pay tax., , Similarly if any asset has been transferred to spouse, income from the asset shall be clubbed but if same, income is invested further, income from such income shall not be clubbed e.g. Mr. X has gifted one fixed
Page 289 :
Clubbing of Income, , 289, , deposit to Mrs. X, interest income from such fixed deposit shall be clubbed but if interest income is, invested further, any fresh income from such income shall not be clubbed., , MAY – 2011, Question 1, Mr. X bought 200 listed shares on 19.04.2020 @ `2,000 per share., He gifted these shares to Mrs. X on 21.03.2021., On 01.04.2021, bonus shares were allotted in the ratio of 1:1., All these shares were sold by Mrs. X as under:, Date of sale, Manner of sale, No. of shares, 10.04.2021, Private sale, to her friend Mrs. Y (Market value 100 original shares, on this date was `2,50,000), 21.05.2021, Sold in recognized stock exchange, STT paid, 100 original shares, , 21.07.2021, , Private sale to an outsider, , All bonus shares, , (5 Marks), , Net sales value (`), 1,70,000, , 4,20,000, 2,50,000, , Briefly state the income-tax consequences in respect of the sale of the shares by Mrs. X, showing clearly the, person in whose hands the same is chargeable, the quantum and the head of income in respect of the above, transactions. Detailed computation of total income is NOT required. Also Discuss tax consequences in the, hands of Mrs.Y., Answer. Where an asset has been transferred by an individual to his spouse otherwise than for adequate, consideration, the income arising from the sale of the said asset by the spouse will be clubbed in the hands, of the individual., Where there is any accretion to the asset transferred, income arising to the transferee from such accretion, will not be clubbed. Hence, the profit from sale of bonus shares allotted to Mrs. X will be chargeable to tax, in the hands of Mrs. X., Therefore, the capital gains arising from the sale of the original shares has to be included in the hands of Mr., X, and the capital gains arising from the sale of bonus shares would be taxable in the hands of Mrs. X., Income/loss to be clubbed in the hands of Mr. X, `, (i) 100 Original shares sold on 10.04.2021, Sale consideration, 1,70,000.00, (2,00,000.00), Less: Cost of acquisition of 100 shares (` 2,000 x 100), (30,000.00), Short term capital loss to be included in the hands of Mr. X, (ii) 100 Original shares sold on 21.05.2021, Sale consideration, Less: Cost of acquisition of 100 shares (` 2,000 x 100), Long term capital gains u/s 112A to be clubbed in the income of Mr. X, , 4,20,000, (2,00,000), 2,20,000, , Income taxable in the hands of Mrs. X Short-term capital gains (on sale of 200 bonus shares), Bonus shares, `, Sale consideration, 2,50,000, Less: Cost of acquisition of bonus shares, Nil, Short-term capital gains, 2,50,000, Taxability in the hands of Mrs. Y under the head “Income from other sources”, Mrs. Y has received shares from her friend for `1,70,000 but market value is `2,50,000 hence `80,000 is, taxable as gift under the head other sources as per section 56., Illustration 1: Mr. X transferred 2,000 debentures of `100 each of Wild Fox Ltd. to Mrs. X on 03.04.2021, without consideration. The company paid interest of `30,000 in September, 2021 which was deposited by
Page 290 :
Clubbing of Income, , 290, , Mrs. X with Kartar Finance Co. in October, 2021. Kartar Finance Co. paid interest of `3,000 upto March,, 2022. How would both the interest income be charged to tax in assessment year 2022-23?, Solution:, As per section 64(1), income arising from assets transferred without adequate consideration by an individual, to his spouse is liable to be clubbed in the hands of the individual, but if there is any further income from, such income, it will not be clubbed., Therefore, `30,000, being the interest on debentures received by Mrs. X in September, 2021 will be clubbed, with the income of Mr. X, since he had transferred the debentures of the company without consideration to, her., However, the interest of `3,000 upto March 2022 earned by Mrs. X on the interest of the debentures, deposited by her with Kartar Finance Company shall be taxable in her individual capacity and will not be, clubbed with the income of Mr. X., (vi) Where the asset transferred directly or indirectly by an individual to the spouse has been invested by the, transferee in any business, the income arising out of the business to the transferee in any previous year, shall be clubbed in the income of transferor but for this purpose capital as on first day of relevant, previous year shall be taken into consideration., Example, (a) Mr. X has gifted `5,00,000 to Mrs. X on 01.04.2021 and She invested it in the proprietary business on, the same date and there were profits of `2,00,000. In this case, entire income of `2,00,000 shall be, clubbed in the income of Mr. X., (b) Mrs. X has one business on 01.04.2021 with capital of `5 lakh and Mr. X has gifted `5,00,000 to Mrs. X, on 01.04.2021 and She invested it in the proprietary business on the same date and there were profits of, `2,00,000. In this case, income of `1,00,000 shall be clubbed in the income of Mr. X., (c) Mrs. X has one business on 01.04.2021 with capital of `5 lakh and Mr. X has gifted `5,00,000 to Mrs. X, on 20.04.2021 and She invested it in the proprietary business on the same date and there were profits of, `2,00,000. In this case, income from business shall not be clubbed in the income of Mr. X because, amount was transferred in business after first day of previous year ., Illustration 2: A proprietary business was started by Mrs. X in the year 2019. As on 01.04.2021 her capital, in business was `4,00,000. Her husband gifted `3,00,000, on 01.04.2021, which Mrs. X invested in her, business on the same date. Mrs. X earned profits from her proprietary business for the, Financial year 2021-22 `2,00,000, Financial year 2022-23 `2,40,000, Financial year 2023-24 `2,80,000, Financial year 2024-25 `3,00,000, Amount of profit was further invested in the business., Compute amount to be clubbed in the income of Mr. X in each of the year., Solution:, Amount to be clubbed in various years shall be as given below:, (i) Previous Year 2021-22: amount to be clubbed shall be as given below:, 2,00,000 / 7,00,000 x 3,00,000 = 85,714.29, (ii) Previous Year 2022-23: amount to be clubbed shall be, 2,40,000 / 9,00,000 x 3,00,000 = 80,000, (iii) Previous Year 2023-24: amount to be clubbed shall be, 2,80,000 / 11,40,000 x 3,00,000 = 73,684.21, (iv) Previous Year 2024-25: amount to be clubbed shall be, 3,00,000 / 14,20,000 x 3,00,000 = 63,380.28, (b) Presume amount was gifted on 10.04.2021., Solution:, Amount to be clubbed in various years shall be as given below:, (i) Previous Year 2021-22: amount to be clubbed shall be Nil because amount was not invested in business, on the first day of the previous year
Page 291 :
Clubbing of Income, , 291, , (ii) Previous Year 2022-23: amount to be clubbed shall be, 2,40,000 / 9,00,000 x 3,00,000 = 80,000, (iii) Previous Year 2023-24: amount to be clubbed shall be, 2,80,000 / 11,40,000 x 3,00,000 = 73,684.21, (iv) Previous Year 2024-25: amount to be clubbed shall be, 3,00,000 / 14,20,000 x 3,00,000 = 63,380.28, , NOV – 2011, Question 1, (3 Marks), Mr. X started a proprietary business on 01.04.2020 with a capital of `5,00,000. He incurred a loss of, `2,00,000 during the year 2020-21. To overcome the financial position, Mrs. X, a software Engineer gave a, gift of `5,00,000 on 01.04.2021, which was immediately invested in the business by Mr. X. He earned a, profit of `4,00,000 during the year 2021-22. Compute the amount to be clubbed in the hands of Mrs. X for, the Assessment Year 2022-2023. If Mrs. X gave the said amount as loan, what would be the amount to be, clubbed?, Answer:, Computation of amount to be clubbed, `, Capital of Mr. X as on 01.04.2020, Opening Capital, 5,00,000, Less: Loss from the business during 2020-21, (2,00,000), Capital as on 01.04.2021, 3,00,000, , Amount to be clubbed in the income of Mrs. X, = 4,00,000 /8,00,000 x 5,00,000, In case it has been given as loan, then no clubbing shall be done, , 2,50,000, , MAY – 2010, Question 3, (3 Marks), Mr. X commenced a proprietary business in the year 2016. His capital as on 01.04.2020 was `6,00,000., On 10.04.2020 his wife gifted `2,00,000 which he invested in the business on the same date., Mr. X earned profit from his proprietary business as given below:, Previous year 2020-21, =, Profit `3,00,000, Previous year 2021-22, =, Profit `4,40,000, During the Previous Year 2021-22, he sold a vacant site which resulted in chargeable long-term capital gain, of `5,00,000 (computed). The vacant site was sold on 20.12.2021., Compute the total income and tax liability of Mr. X., Answer: Computation of business income of Mr. X, `, Capital as on 01.04.2020, 6,00,000, Add: Gift from wife (10.04.2020), 2,00,000, 8,00,000, Add: Profit for the year ended 31.03.2021, 3,00,000, Capital as on 01.04.2021, 11,00,000, Profit for the year ended 31.03.2022, 4,40,000, Amount of income to be clubbed in the hands of Mrs. X under section 64(1), `4,40,000 /11,00,000 x 2,00,000, 80,000, Income assessable in the hands of Mr. X (4,40,000 – 80,000), 3,60,000, Total Income of Mr. X, Income from Business, 3,60,000, Long term capital gain, 5,00,000, Gross Total Income, 8,60,000, Less: Deduction u/s 80C to 80U, Nil, Total Income, 8,60,000
Page 292 :
Clubbing of Income, Tax liability on normal income of ` 3,60,000, On LTCG @ 20% on ` 5,00,000, , 292, 5,500, 1,00,000, 1,05,500, 4,220, 1,09,720, , Add: HEC @ 4%, Tax Liability, , NOV – 2004, Question 1, (3 Marks), A proprietary business was started by Smt. X in the year 2019. As on 01.04.2020 her capital in business was, `3,00,000. Her husband gifted `2,00,000, on 10.04.2020, which amount Smt. X invested in her business on, the same date. Smt. X earned profits from her proprietary business for the financial years 2020-21,, `1,50,000 and financial year 2021-22 `3,90,000., Compute the income, to be clubbed in the hands of X’s husband for the assessment year 2022-23 with, reasons., Answer:, `, Capital as on 01.04.2021 (3,00,000 + 2,00,000 + 1,50,000), 6,50,000, Total profits, 3,90,000, Amount to be clubbed (3,90,000/6,50,000 x 2,00,000), 1,20,000, Income relating to the amount gifted shall be clubbed and not any subsequent income from the income so, clubbed, (vii) If any person has transferred the asset to the spouse and the spouse has invested it in some partnership, firm as capital contribution or otherwise, in this case interest received from the partnership firm shall be, clubbed in the income of the transferor and capital as on first day of relevant previous year shall be taken, into consideration., If any salary has been received from partnership firm, it will not be clubbed., If any share has been received from the profits of partnership firm, such shares shall be exempt under, section 10(2A)., (viii). If any person has transferred any asset to the spouse and spouse has further transferred this asset, in, this case, capital gain shall be considered to be the income of the transferor., (ix) Cross-transfers are also covered, The Supreme Court, in case of Keshavji Morarji, observed that clubbing provisions shall be applicable in, case of cross transfers also e.g. A making gift of ` 50,000 to the wife of his brother B for the purchase of a, house by her and a simultaneous gift by B to A’s minor son of shares in a foreign company worth ` 50,000, owned by him, in the case, the income arising to Mrs. B from the house property should be included in the, total income of B and the dividend from shares transferred to A’s minor son would be taxable in the hands, of A., Example: Mr. Vasudevan gifted a sum of `6 lakhs to his brother's wife on 14-6-2021. On 12-7-2021, his, brother gifted a sum of `5 lakhs to Mr. Vasudevan's wife. The gifted amounts were invested as fixed, deposits in banks by Mrs. Vasudevan and wife of Mr. Vasudevan's brother on 01-8-2021 at 9% interest., Examine the consequences of the above under the provisions of the Income-tax Act, 1961 in the hands of, Mr. Vasudevan and his brother., Answer: In the given case, Mr. Vasudevan gifted a sum of `6 lakhs to his brother’s wife on 14.06.2021 and, simultaneously, his brother gifted a sum of `5 lakhs to Mr. Vasudevan’s wife on 12.07.2021. The gifted, amounts were invested as fixed deposits in banks by Mrs. Vasudevan and his brother’s wife. These transfers, are in the nature of cross transfers., Accordingly, the income from the assets transferred would be assessed in the hands of the deemed transferor, because the transfers are so intimately connected to form part of a single transaction and each transfer, constitutes consideration for the other by being mutual or otherwise.
Page 293 :
Clubbing of Income, , 293, , If two transactions are inter-connected and are part of the same transaction in such a way that it can be said, that the circuitous method was adopted as a device to evade tax, the implication of clubbing provisions, would be attracted. It was so held by the Apex Court in CIT vs. Keshavji Morarji (1967) 66 ITR 142., Accordingly, the interest income arising to Mrs. Vasudevan in the form of interest on fixed deposits would, be included in the total income of Mr. Vasudevan and interest income arising in the hands of his brother’s, wife would be taxable in the hands of Mr. Vasudevan’s brother as per section 64(1), to the extent of amount, of cross transfers i.e., `5 lakhs., This is because both Mr. Vasudevan and his brother are the indirect transferors of the income to their, respective spouses with an intention to reduce their burden of taxation However, the interest income earned, by his spouse on fixed deposit of `5 lakhs alone would be included in the hands of Mr. Vasudevan’s brother, and not the interest income on the entire fixed deposit of `6 lakhs, since the cross transfer is only to the, extent of `5 lakhs., RTP NOV -2020, 2. Mr. Karan gifted a sum of ` 9 lakhs to his brother’s minor son on 1-5-2021. On the same date, his, brother gifted debentures worth ` 10 lakhs to Mrs. Karan. Son of Mr. Karan’s brother invested the, amount in fixed deposit with Canara Bank @ 9% p.a. interest and Mrs. Karan received interest of, `81,000 on these debentures during the previous year 2021-22. Discuss the tax implications under the, provisions of the Income- tax Act, 1961., Answer:, In the given case, Mr. Karan gifted a sum of ` 9 lakhs to his brother’s minor son on 1.5.2021 and, simultaneously, his brother gifted debentures worth ` 10 lakhs to Mr. Karan’s wife on the same date. Mr., Karan’s brother’s minor son invested the gifted amount of ` 9 lakhs in fixed deposit with Canara Bank., , These transfers are in the nature of cross transfers. Accordingly, the income from the assets transferred, would be assessed in the hands of the deemed transferor because the transfers are so intimately connected to, form part of a single transaction and each transfer constitutes consideration for the other by being mutual or, otherwise., If two transactions are inter-connected and are part of the same transaction in such a way that it can be said, that the circuitous method was adopted as a device to evade tax, the implication of clubbing provisions, would be attracted., As per section 64(1A), all income of a minor child is includible in the hands of the parent, whose total, income, before including minor’s income is higher. Accordingly, the interest income arising to Mr. Karan’s, brother’s son from fixed deposits would be included in the total income of Mr. Karan’s brother, assuming, that Mr. Karan’s brother’s total income is higher than his wife’s total income, before including minor’s, income. Mr. Karan’s brother can claim exemption of ` 1,500 under section 10(32)., Interest on debentures arising in the hands of Mrs. Karan would be taxable in the hands of Mr. Karan as per, section 64(1)(iv)., This is because both Mr. Karan and his brother are the indirect transferors of the income to their spouse and, minor son, respectively, with an intention to reduce their burden of taxation., In the hands of Mr. Karan, interest received by his spouse on debentures of ` 9 lakhs alone would be, included and not the entire interest income on the debentures of `10 lakhs, since the cross transfer is only to, the extent of ` 9 lakhs., Hence, only proportional interest (i.e., 9/10th, includible in the hands of Mr. Karan., , of interest on debentures received) ` 72,900 would be
Page 294 :
Clubbing of Income, , 294, , The provisions of section 56(2)(x) are not attracted in respect of sum of money transferred or value of, debentures transferred, since in both the cases, the trans fer is from a relative, MTP NOV -2020, 3. Nishant gifted ` 10 lakhs to his wife, Nisha on her birthday on, 1st January, 2021. Nisha lent `5,00,000, out of the gifted amount to Krish on 1st April, 2021 for six months on which she received interest of `, 50,000. The said sum of ` 50,000 was invested in shares of a listed company on 15th October, 2021,, which were sold for ` 75,000 on 30th December, 2021. Securities transaction tax was paid on such sale., The balance amount of gift was invested as capital by Nisha in a newly business started on 1.4.2021. She, suffered loss of ` 15,000 in the business in Financial Year 2021-22., In whose hands the above income and loss shall be included in Assessment Year 2022-23? Support your, (4 Marks), answer with brief reasons., Answer:, Interest on loan, As per section 64(1)(iv), in computing the total income of any individual, there shall be included all such, income as arises directly or indirectly, to the spouse of such individual from assets transferred directly or, indirectly, to the spouse by such individual otherwise than for adequate consideration or in connection with, an agreement to live apart., Accordingly, ` 50,000, being the amount of interest on loan received by Ms. Nisha, wife of Mr. Nishant,, would be includible in the total income of Mr. Nishant, since such loan was given by her out of the sum of, money received by her as gift from her husband., Loss from business, Since the capital was invested in business by Ms. Nisha on 1st April, 2021, and capital invested was entirely, out of the funds gifted by her husband, the entire loss of `15,000 from the business carried on by Ms. Nisha, would also be includible in the total income of Mr. Nishant., Since income includes loss as per Explanation 2 to section 64, clubbing provisions would be attracted even, if there is loss and not income., Capital Gain on sale of shares of listed company, The short-term capital gain of ` 25,000 (` 75,000, being the sale consideration less ` 50,000, being the cost, of acquisition) arising in the hands of Ms. Nisha from sale of shares acquired by investing the interest, income of ` 50,000 earned by her (from the loan given out of the sum gifted to her by her husband), would, not be included in the hands of Mr. Nishant., Income from the accretion of the transferred asset is not liable to be included in the hands of the transferor, and therefore such income is taxable in the hands of Ms. Nisha. Since securities transaction tax has been, paid, such short-term capital gain on sale of listed shares is taxable@15% in the hands of Ms. Nisha, , MAY – 2015, Question 6(a)(ii)., (4 Marks), Mr. Ramesh gifted a sum of `5 lacs to his brother's minor son on 16.04.2021. On 18.04.2021, his brother, gifted debentures worth `6 lacs to Mrs. Ramesh. Son of Mr. Ramesh' brother invested the amount in fixed, deposit with Bank of India @ 9% p.a. interest and Mrs. Ramesh received interest of `45,000 on debentures, received by her., Discuss the implications under the provisions of the Income tax Act, 1961., Answer: In the given case, Mr. Ramesh gifted a sum of `5 lacs to his brother’s minor son on 16.4.2021 and, simultaneously, his brother gifted debentures worth `6 lacs to Mr. Ramesh’s wife on 18.4.2021. Mr., Ramesh’s brother’s minor son invested the gifted amount of `5 lacs in fixed deposit with Bank of India., These transfers are in the nature of cross transfers. Accordingly, the income from the assets transferred, would be assessed in the hands of the deemed transferor because the transfers are so intimately connected to, form part of a single transaction and each transfer constitutes consideration for the other by being mutual or, otherwise. If two transactions are inter-connected and are part of the same transaction in such a way that it, can be said that the circuitous method was adopted as a device to evade tax, the implication of clubbing, provisions would be attracted .
Page 295 :
Clubbing of Income, , 295, , As per section 64(1A), all income of a minor child is includible in the hands of the parent, whose total, income, before including minor’s income is higher. Accordingly, the interest income arising to Mr., Ramesh’s brother’s son from fixed deposits would be included in the total income of Mr. Ramesh’s brother,, assuming that Mr. Ramesh’s brother’s total income is higher than his wife’s total income, before including, minor’s income. Mr. Ramesh’s brother can claim exemption of `1,500 under section 10(32). Interest on, debentures arising in the hands of Mrs. Ramesh would be taxable in the hands of Mr. Ramesh as per section, 64(1)(iv)., This is because both Mr. Ramesh and his brother are the indirect transferors of the income to their spouse, and minor son, respectively, with an intention to reduce their burden of taxation. In the hands of Mr., Ramesh, interest received by his spouse on debentures of `5 lacs alone would be included and not the entire, interest income on the debentures of `6 lacs, since the cross transfer is only to the extent of `5 lacs., Hence, only proportional interest (i.e., 5/6th of interest on debentures received) `37,500 would be includible, in the hands of Mr. Ramesh. The provisions of section 56(2)(vii) are not attracted in respect of sum of, money transferred or value of debentures transferred, since in both the cases, the transfer is from a relative., (x) If there is indirect transfer, clubbing provisions shall be applicable in that case also e.g. Mr. X gifted, certain cash/ asset to his major son and son gifted the same asset to mother, in this case it will be considered, transfer and income shall be clubbed in the income of Mr. X., (xi) If any person has given loan to the spouse, income from such loan shall not be clubbed., Transfer of house property: In the case of transfer of house property, the provisions are contained in, section 27. If an individual transfers a house property to his spouse, without adequate consideration or, otherwise than in connection with an agreement to live apart, the transferor shall be deemed to be the owner, of the house property and its annual value will be taxed in his hands., 5. Transfer of the asset to the son’s wife Section 64(1), If any person has transferred the asset to the son’s wife, in this case, clubbing provision shall apply in the, similar manner as in the case of transfer of the assets to the spouse., Such clubbing provisions are applicable from 01.06.1973., 6. Transfer of assets to any other person Section 64(1), If any person has transferred the asset to any other person, clubbing provision shall not be applicable, but if, the transferor has any right to receive any benefit from the asset or the benefit shall be received by the, spouse of the transferor or by the son’s wife of the transferor, in that case, clubbing provision shall be, applicable., , NOV – 2010, Question 1, (3 Marks), Mrs. X transferred her immovable property to ABC Co. Ltd. subject to a condition that out of the rental, income, a sum of `36,000 per annum shall be utilized for the benefit of her son’s wife., Mrs. X claims that the amount of `36,000 (utilized by her son’s wife) should not be included in her total, income as she no longer owned the property., State with reasons whether the contention of Mrs. X is valid in law?, Answer: The clubbing provisions are attracted in case of transfer of any asset, directly or indirectly,, otherwise than for adequate consideration, to any person to the extent to which the income from such asset is, for the immediate or deferred benefit of son’s wife. Such income shall be included in computing the total, income of the transferor-individual. Therefore, income of `36,000 meant for the benefit of daughter-in-law, is chargeable to tax in the hands of transferor i.e., Smt. X in this case. The contention of Smt. X is, hence,, not valid in law., 7. Salary/commission/fee etc. from a concern in which the spouse has substantial interest Section, 64(1), (i) If any person is getting salary, commission, fee or any other remuneration whether in cash or in kind
Page 296 :
Clubbing of Income, , 296, , from a concern in which his or her spouse has substantial interest and further salary etc. is being received, without any technical or professional qualification, in such case, salary etc. so received shall be, clubbed in the income of the spouse having substantial interest. However clubbing shall not be applicable, in relation to any income arising to the spouse where the spouse possesses technical or professional, qualifications and the income is solely attributable to the application of his or her technical or professional, knowledge and experience., If the spouse has substantial interest along with his relative, even in that case clubbing provisions are, applicable., Example, Mr. X is holding 11% shares of ABC Ltd. and his father is holding 10% shares in ABC Ltd. and his wife, Mrs. X is employed in ABC Ltd. without any technical or professional qualification, in this case, salary, income of Mrs. X shall be clubbed in the income of Mr. X., (ii) Technical and professional qualification shall include not only degree or membership but also any, experience or expertise or any natural talent also, as decided in Batta Kalyani v. CIT, (HC)., (iii) As per section 2(41), Relative, means the husband, wife, brother or sister or any lineal ascendant or, descendant., (iv) As per section 2(32), Substantial Interest means having 20% or more of the equity shares in a, company or having 20% of more of the shares in profits in any other concern., (v) Both husband and wife have substantial interest in a concern: Where both husband and wife have, substantial interest in a concern and both are in receipt of income by way of salary etc. from the said, concern, such income will be includible in the hands of that spouse, whose total income, excluding such, income is higher. E.g. Mr. X has 12% shares in ABC limited and Mrs. X has 13% shares in ABC limited, and both are getting salary of 13,00,000 and 10,00,000 from ABC limited without any technical or, professional qualification. Mr. X has income under the head house property 6,00,000 and Mrs. X has, income under the head house property 7,00,000, in this case salary income of both of them shall be, clubbed in the income of Mrs. X. Tax liability of each one of them shall be:, Mr. X, Income under the head house property, 6,00,000, Gross Total Income, 6,00,000, Less: Deductions u/s 80C to 80U, Nil, Total Income, 6,00,000, Computation of Tax Liability, Tax on 6,00,000 at slab rate, 32,500, Add: HEC @ 4%, 1,300, Tax Liability, 33,800, Mrs. X, Income under the head house property, Income under the head salary, Salary of Mr. X, Salary of Mrs. X, Gross salary, Less: Standard Deduction u/s 16(ia), Income under the head salary, , Gross Total Income, Less: Deductions u/s 80C to 80U, Total Income, Computation of Tax Liability, Tax on 29,50,000 at slab rate, Add: HEC @ 4%, Tax Liability, , 7,00,000, 13,00,000, 10,00,000, 23,00,000, (50,000), 22,50,000, 29,50,000, Nil, 29,50,000, 9,67,500, 38,700, 10,06,200
Page 297 :
Clubbing of Income, , 297, , Illustration 3: Mr. X is an employee of X Ltd. and he has 25% shares of that company. His salary is, `50,000 p.m. Mrs. X is working as a computer software programmer in X Ltd. at a salary of `30,000 p.m., She is, however, not qualified for the job. Compute the gross total income of Mr. X and Mrs. X for the, A.Y.2022-23, assuming that they do not have any other income., Solution:, Mr. X is an employee of X Ltd and has 25% shares of X Ltd i.e. a substantial interest in the company. His, wife is working in the same company without any professional qualifications for the same. Thus, by virtue, of the clubbing provisions of the Act, the salary received by Mrs. X from X Ltd. will be clubbed in the hands, of Mr. X., Computation of Gross Total Income of Mr. X, Particulars, `, 6,00,000, Salary received by Mr. X (` 50,000 x 12), 3,60,000, Salary received by Mrs. X (` 30,000 x 12), 9,60,000, Gross Salary, (50,000), Less: Deduction u/s 16(ia), Income under the head salary, 9,10,000, Gross Total Income, 9,10,000, The gross total income of Mrs. X is nil., Illustration 4: Will your answer be different if Mrs. A was qualified for the job?, Solution:, If Mrs. A possesses professional qualifications for the job, then the clubbing provisions shall not be, applicable., Gross total income of Mr. X = Salary received by Mr. A [` 50,000 × 12 ] = ` 6,00,000 - 50,000 = 5,50,000, Gross total income of Mrs. X = Salary received by Mrs. A [` 30,000×12] = ` 3,60,000 - 50,000= 3,10,000, Illustration 5: Mr. X is an employee of Y Ltd. and has substantial interest in the company. His salary is, `20,000 p.m. Mrs. X is also working in Y Ltd. at a salary of `12,000 p.m. without any qualifications. Mr. X, also receives `30,000 as interest on securities. Mrs. X owns a house property which she has let out. Rent, received from tenants is `6000 p.m. Compute the gross total income of Mr. X and Mrs. X for the A.Y.202223., Solution:, Since Mrs. X is not professionally qualified for the job, the clubbing provisions shall be applicable., Computation of Gross Total Income of Mr. X, Particulars, `, Income from Salary, 2,40,000, Salary received by Mr. X (` 20,000 × 12), 1,44,000, Salary received by Mrs. X (` 12,000 × 12), 3,84,000, Gross salary, (50,000), Less: Deduction u/s 16(ia), 3,34,000, Income from salary, Income from other sources, 30,000, Interest on securities, 3,64,000, Computation of Gross Total Income of Mrs. X, Particulars, `, `, Nil, Income from Salary, [clubbed in the hands of Mr. X], Income from house property, 72,000, Gross Annual Value [` 6,000 × 12], -
Page 298 :
Clubbing of Income, Less: Municipal taxes paid, Net Annual Value (NAV), Less: Deductions under section 24, - 30% of NAV i.e., 30% of ` 72,000, - Interest on loan, , 298, 72,000, , (21,600), - 50,400, 50,400, , Gross Total Income, , MAY – 2013, Question 5(a), (4 Marks), Mr. X is an employee of Larsen Limited and has substantial interest in the company. His salary is `25,000, p.m. Mrs. X also is working in that company at a salary of `10,000 p.m. without any professional, qualification., Mr. X also receives ` 30,000 as income from securities, Mrs. X owns a house property which she has let out., Rent received from such house property is ` 12,000 p.m., Mr. & Mrs. X have three minor children-two twin daughters and one son. Income of the twin daughters is, `2,000 p.a. each and that of his son is ` 1,200 p.a. Compute the income of Mr. & Mrs. X., Solution: Computation of Total Income of Mr. X and Mrs. X for the A.Y. 2022-23, Particulars, Mr. X, Mrs. X, (`), (`), Income from Salaries, 3,00,000, Salary income of Mr. X (` 25,000 × 12), 1,20,000, Salary income of Mrs. X (` 10,000 × 12) (Note 1), (50,000), Less: Deduction u/s 16(ia), , Income from House Property, 1,44,000, Rent received (` 12,000×12), Less: Deduction under section 24 @ 30%, (43,200), Income from other sources, Income from securities, Income before including income of minor children under section 64(1A), (Note 2), Income of twin daughters, ` 4,000, , 1,00,800, 30,000, 4,00,000, , 1,00,800, , -, , (` 2,000 per child x 2), Less: Exempt u/s 10(32) (`1,500 x 2), ( ` 3,000), 1,000, Income of the minor son, ` 1,200, Less: Exempt u/s 10(32), (`1,200), 4,01,000 1,00,800, Total Income, Notes:, (1) As per section 64(1), the salary of ` 10,000 p.m. received by Mrs. X from the company has to be, included in the total income of Mr. X, as Mrs. X does not possess any technical or professional qualification, for earning such income and Mr. X has substantial interest in the company., (2) As per section 64(1A), the income of a minor child is to be included in the total income of the parent, whose total income (excluding the income of minor child to be so clubbed) is greater. Further, as per section, 10(32), income of a minor child which is includible in the income of the parent shall be exempt to the extent, of ` 1,500 per child., 8. Asset held by Minor Child Section 64(1A) [V. IMP.], (i) If any income accrues or arises to a minor child, such income shall be clubbed in the income of mother or, father whosoever has higher income before taking into consideration the income to be clubbed.
Page 299 :
Clubbing of Income, , 299, , (ii) If the marriage of mother, father doesn’t subsist, in that case, income shall be clubbed in the income, of mother or father whosoever maintains the minor child., (iii) Minor child for this purpose shall include even an adopted child and also step child, however, it will not, include the minor child suffering from a disability mentioned under section 80U. e.g. Minor son of, Mr. X has interest income of `2,00,000 and the minor child is suffering from a disability, in this case,, clubbing provisions shall not be applicable., (iv) If any minor child has income through, (i), Manual labour or, (ii), has income through activity involving application of his skill, talent or specialized, knowledge and experience,, in this case, clubbing provision shall not apply, rather it will be considered to be the income of minor, child and his tax liability shall be computed separately but the return shall be filed by his father as his, guardian., (v) If the income of minor child is to be clubbed, exemption shall be allowed under section 10(32) upto, `1,500 per annum per child. E.g. Minor son of Mr. X has interest income from bank fixed deposit, `35,000, in this case amount to be clubbed shall be 35,000 – 1,500 = 33,500, If any minor child has casual income and such income is to be clubbed, in that case exemption of `1,500, under section 10(32) shall be allowed or not is controversial., (vi) If any person has transferred any asset to minor married daughter, clubbing provision shall applicable in, that case also e.g. Minor married daughter of Mr. X has interest income of `1,00,000 from bank deposit,, in this case income shall be clubbed in the income of mother or father whosoever has higher income and, exemption under section 10(32) shall be allowed., (vii) If any minor child has income from manual labour or through activity involving application of his skill,, talent or specialized knowledge and experience, such income shall not be clubbed but if such income has, been invested further, any new income shall be clubbed in the income of mother or father., Example, Minor son of Mr. X is a child actor. He has income of `5,00,000 from stage acting, this income will not be, clubbed but if this amount was invested by him in a bank as fixed deposit, interest received by him shall be, clubbed., Illustration 6: Mr. X has three minor children – two twin daughters and one son. Income of the twin, daughters is `2,000 p.a. each and that of the son is `1,200 p.a. Compute the income, in respect of minor, children, to be clubbed in the hands of Mr. X., Solution:, Taxable income, in respect of minor children, in the hands of Mr. X is, Particulars, `, `, 4,000, Twin minor daughter [` 2,000 × 2], (3,000), 1,000, Less: Exempt under section 10 (32) [` 1,500 × 2], , Minor son, Less: Exempt under section 10 (32), Income to be clubbed in the hands of Mr. X, , 1,200, (1,200), , Nil, 1,000, , Illustration 7: Mr. X, a mentally retarded minor, has a total income of `1,20,000 for the assessment year, 2022-23. The total income of his father Mr. Y and of his mother Mrs. Y for the relevant assessment year is, `2,40,000 and `1,80,000 respectively. Discuss the treatment to be accorded to the total income of Mr. X for, the relevant assessment year., Solution: Section 64(1A) provides that all income accruing or arising to a minor child has to be included in, the income of that parent, whose total income is greater. However, the income of a minor child suffering, from any disability of the nature specified in section 80U shall not be included in the income of the parents, but shall be assessed in the hands of the child. Thus, the total income of Mr. X has to be assessed in his, hands and cannot be included in the total income of either his father or his mother.
Page 300 :
Clubbing of Income, , 300, , NOV – 2020 (New Course), Question 3 (b), 6 Marks, Determine the Gross total income of Shri Ram Kumar and Smt. Ram Kumar for the assessment year 202223 from the following:, (i) Salary received by Shri Ram Kumar from a company `1,80,000 per annum and Smt. Ram Kumar also, doing job in a company and getting salary of `2,40,000 per annum, (ii) Shri Ram Kumar transferred a flat to his wife Smt. Ram Kumar on 1st September, 2021 for adequate, consideration. The rent received from this let-out flat is `9,000 per month., (iii) Shri Ram Kumar and his wife Smt. Ram Kumar both are partners in a firm. Shri Ram Kumar received, `36,000 and Smt. Ram Kumar received `64,000 as interest from the firm and also had a share of profit of, `12,000 and `26,000 respectively., (iv) Smt. Ram Kumar transferred 10% debentures worth `3,00,000 to Shri Ram Kumar. The whole amount, of `3,30,000 invested by Shri Ram Kumar in the similar investments and earned income of `39,000., (v) Mother of Shri Ram Kumar transferred a property to Master Rohit (son of Shri Ram Kumar) in the, year 2020. Master Rohit (Aged 13 years) received of `15,000 as income from this property on 20th, February, 2022., Solution: Computation of Gross Total Income of Shri Ram Kumar, Income under the head salary, Salary received, 1,80,000.00, Less: Standard deduction u/s 16(ia), (50,000.00), Income under the head salary, 1,30,000.00, Income under the head house property, Gross Annual value (9,000 x 5), Less: Municipal taxes, Net Annual value, Less: Standard deduction @ 30% u/s 24(a), Less: Interest on capital borrowed u/s 24(b), Income from house property, Income under the business/Profession, Interest income from Firm, Income under the head other sources, Interest income of debentures, (39,000 /3,30,000 x 30,000), Gross Total Income, Computation of Gross Total Income of Smt. Ram Kumar, Income under the head salary, Salary received, Less: Standard deduction u/s 16(ia), Income under the head salary, Income under the head house property (Transferred with adequate consideration), Gross Annual value (9,000 x 7), Less: Municipal taxes, Net Annual value, Less: Standard deduction @ 30% u/s 24(a), Less: Interest on capital borrowed u/s 24(b), Income from house property, Income under the business/Profession, Interest income from Firm, , 45,000.00, Nil, 45,000.00, (13,500.00), Nil, 31,500.00, 36,000.00, 3,545.45, 2,01,045.45, , 2,40,000, (50,000), 1,90,000, 63,000, Nil, 63,000, (18,900), Nil, 44,100, 64,000
Page 301 :
Clubbing of Income, Income of Minor son – Rohit, Income from house property, Less: Exemption u/s 10(32), Income under the head house property, , 301, 15,000, (1,500), 13,500, , Income under the head other sources, Interest on debentures (3,00,000 x 10%), 30,000.00, Interest income of debentures shall be clubbed u/s 64(1), 35,454.55, (39,000 /3,30,000 x 3,00,000), Income under the head other sources, 65,454.55, Gross Total Income, 3,77,054.55, Note:, (i) Mother of Shri Ram Kumar transferred a Property to master Rohit, it is not mentioned it is House, Property, hence it is presumed that it is other than House Property. Accordingly income has been clubbed, after exemption under section 10(32)., (ii) In case of transfer of debentures date of transfer is not given and whether it is transferred for adequate, consideration or not is not mentioned. Above solution is given on the assumption that it is transferred for, inadequate consideration and clubbing provisions shall be applicable., , NOV – 2018 (New Course), Question 6(b)., (5 Marks), Mrs. and Mr. Vinod Amin have two minor children M and N. The following are the receipts in the hands of, M and N during the year ended 31-3-2022:, (i) M received a gift of `70,000 from her friend’s father on the occasion of her birthday., (ii) M won a prize money of `3,00,000 in National Quiz competition., This was invested in debentures of a company, from which interest of `19,000 (gross) accrued during the, year., (iii) N won prize in a lottery. The net amount received after deduction of tax at source was `1,05,000., Mr. Vinod Amin's income before considering clubbing provisions is higher than that of his wife. Discuss, how these items will be considered for taxation under the provisions of the Income Tax Act, 1961. Detailed, computation of income is not required., Solution:, As per section 64(1A), If any income accrues or arises to a minor child, such income shall be clubbed in the, income of mother or father whosoever has higher income before taking into consideration the income to, be clubbed but if any minor child has income through Manual labour or has income through activity, involving application of his skill, talent or specialized knowledge and experience, in this case, clubbing, provision shall not apply, rather it will be considered to be the income of minor child and his tax liability, shall be computed separately but the return shall be filed by his father as his guardian. If the income of, minor child is to be clubbed, exemption shall be allowed under section 10(32) upto `1,500 per annum, per child., In the given case income of father is higher then the mother hence all income to be clubbed shall be clubbed, with the income of the Father., (i) Gift of `70,000 received by M from her friend father is taxable as it is not received from any relative of, M., (ii) Prize money earned by M in National Quiz Competition shall not be clubbed as it is earned through, application of her skill, talent or specialized knowledge and experience but income from such income, i.e. interest on debentures (`19,000) shall be clubbed with the father., (iii) Prize Money earned by N shall be taxable and shall be clubbed with the income of the father., (`1,05,000/70% = `1,50,000 shall be clubbed), , Exemption shall be allowed u/s 10(32) upto `1500 per annum per child but in case of casual income such, exemption is allowed or not is controversial.
Page 302 :
Clubbing of Income, , 302, , NOV – 2014, Question 6(a)(ii), (4 Marks), Mr. X has four minor children consisting of three daughters and one son. The annual income of all the, children for the Assessment Year 2022-23 were as follows:, 10,000, First daughter (including Scholarship received `5,000), Second Daughter, 8,500, Third Daughter (Suffering from disability specified U/s 80U), 4,500, Son, 40,000, Mr. X gifted `2,00,000 to his minor Son who invested the same in the business and derived income of, `20,000 which is included above., Compute the amount of Income earned by Minor Children to be clubbed in the hands of Mr. X., Solution:, Computation of Amount of Income of minor children to be clubbed in the income of Mr. X, `, (i) Income of First Daughter, 10,000, Less: Scholarship received exempt u/s 10(16) (assumed received for education), (5,000), Less: Exempt u/s 10(32), (1,500), 3,500, (ii) Income of Second Daughter, 8,500, Less: Exempt u/s 10(32), (1,500), 7,000, (iii) Income of Third Daughter who is suffering from disability shall not be clubbed, (iv) Income of Son, 40,000, Less: Exempt u/s 10(32), (1,500), 38,500, Total Income to be clubbed (3,500 + 7,000 + 38,500), 49,000, , NOV – 2012, Question No. 2(a), Mr. X is the Karta of an HUF, whose members derive income as given below:, (i) Income from Mr. X’s Business, (ii) Mrs. X’s Salary Income (Computed) as fashion designer, (Mrs. X is working as fashion designer in the business of Mr. X), (iii) Minor son D (interest on fixed deposits with a bank which were gifted to him by his uncle), (iv) Minor daughter P’s earning from sports, (v) D’s winning from lottery (gross), , (3 Marks), `, 4,50,000, , 7,60,000, 10,000, 95,000, 1,95,000, , Discuss the tax implications in the hands of Mr. and Mrs. X., Answer:, Computation of income of Mr. X and Mrs. X, Income from X’s Business, Salary as fashion designer, Bank Interest to Minor Son D (10,000-1,500) `1,500 exempt u/s 10(32), Income of Minor Daughter from Sports (since she is earning income from her, (own talent, sports, income is not to be clubbed), Lottery income to minor son D, TOTAL, , Mr. X, 4,50,000, -----------------, , Mrs. X, ------7,60,000, 8,500, ------, , ------4,50,000, , 1,95,000, 9,63,500, , Note: Whether exemption under section 10(32) shall be allowed from casual income or not is controversial.
Page 303 :
Clubbing of Income, , 303, , MAY – 2012, Question 2, (3 Marks), Mr. X has four children consisting 2 daughters and 2 sons. The annual income of 2 daughters were `9,000, and `4,500 and of sons were `6,200 and `4,300 respectively. The daughter who has income of `4,500 was, suffering from a disability specified under section 80U. Compute the amount of income earned by minor, children to be clubbed in hands of Mr. X., Answer:, Computation of Amount of Income of minor children to be clubbed in the income of Mr. X, `, (i) Income of First Daughter, 9,000, Less: Exempt u/s 10(32), (1,500), 7,500, (ii) Income of Second Daughter who is suffering from disability shall not be clubbed, (iii) Income of First Son, 6,200, Less: Exempt u/s 10(32), (1,500), 4,700, (iv) Income of Second Son, 4,300, Less: Exempt u/s 10(32), (1,500), 2,800, Total Income to be clubbed (7,500 + 4,700 + 2,800), 15,000, , MAY – 2008, Question 4, Mr. X and Mrs. X furnish the following information:, , (4 Marks), , `, (i) Income under the head salary of Mrs. X (Computed), 4,60,000, (ii) Income of minor son ‘B’ who suffers from disability specified in Section 80U, 1,08,000, (iii) Income of minor daughter ‘C' from singing, 86,000, (iv) Income from profession of Mr. X, 7,50,000, (v) Cash gift received by 'C' on 02.10.2021 from friend of Mrs. X, 48,000, (vi) Income of minor married daughter ‘A’ from company deposit, 30,000, Compute the total income of Mr. X and Mrs. X for the assessment year 2022-23., Answer: Computation of Total Income of Mr. X and Mrs. X for the A.Y. 2022-23, Particulars, Mr. X, Mrs. X, `, `, Income under the head Salary, 4,60,000, Profits and gains of business or profession, 7,50,000, Income from other sources, Income by way of interest from company, deposit earned by minor daughter A, [Note (iv)], 30,000, Less: Exemption under section 10(32), (1,500) 28,500, Gross Total Income, 7,78,500, 4,60,000, Less: Deduction u/s 80C to 80U, Nil, Nil, Total Income, 7,78,500, 4,60,000, Notes:, (i) The income of a minor child suffering from any disability of the nature specified in section 80U shall not, be included in the hands of the parents., (ii) The income derived by the minor from manual work or from any activity involving exercise of his skill,, talent or specialized knowledge or experience will not be included in the income of his parent., (iii) Under section 56, cash gifts received from any person/persons exceeding ` 50,000 during the year in, aggregate are taxable. Since the cash gift in this case does not exceed ` 50,000 the same is not taxable.
Page 304 :
Clubbing of Income, , 304, , (iv) The clubbing provisions are attracted even in respect of income of minor married daughter. The income, of the minor will be included in the income of that parent whose total income is greater. Hence, income of, minor married daughter ‘A’ from company deposit shall be clubbed in the hands of the Mr. X and, exemption under section 10(32) of ` 1,500 per child shall be allowed in respect of such income., , NOV – 2007, Question 4, (3 Marks), Mr. X has four minor children consisting 2 daughters and 2 sons. The annual income of 2 daughters was, `7,500 and ` 5,000 and of sons was ` 5,500 and ` 1,250 respectively. The daughter who was having income, of `5,000 was suffering from a disability specified under section 80U. Work out the amount of income, earned by minor children to be clubbed in the hands of Mr. X., Answer., Income earned by minor children to be clubbed with the income of Mr. X, Income of first daughter to be clubbed (7,500 – 1,500 u/s 10(32)), 6,000, Income of second daughter suffering from disability shall not be clubbed, Nil, Income of first son to be clubbed (5,500 – 1,500 u/s 10(32)), 4,000, Income of second son to be clubbed (1,250 – 1,250 u/s 10(32)), Nil, Total Income to be clubbed as per section 64(1A), 10,000, The income of daughter suffering from disability specified under section 80U is not to be clubbed with the, income of Mr. X., , NOV – 2005, Question 1, Compute the total income of Mr. & Mrs. X from the following information:, (a) Salary Income (computed) of Mrs. X, (b) Income from profession of Mr. X, (c) Income of minor son A from company deposit, (d) Income of minor daughter B from special talent, (e) Interest from bank received by B on fixed deposit made out of her special talent, (f) Gift received by B on 30.09.2021 from friend of Mrs. X, Answer:, Since income of Mr. X is higher, income shall be clubbed in the income of Mr. X, Interest income (minor son A) (15,000 – 1,500), Income of minor daughter B from special talent shall not be clubbed, Interest income of minor daughter B (3,000 – 1,500), Total income of Mr. X (3,90,000 + 13,500 + 1,500), Total income of Mrs. X shall be, , (3 Marks), `, 2,30,000, 3,90,000, 15,000, 32,000, 3,000, 2,500, `, , 13,500, Nil, 1,500, 4,05,000, 2,30,000, , NOV – 1999, Question 1, (4 Marks), Mr. X is the Karta of a HUF, whose members derive income as given below:, `, (i) Income of Mr. X from own business, 5,00,000, (ii) Income of Mrs. X under the head Salary, 8,00,000, (iii) Minor son Deepak (earning interest on fixed deposits, with bank, which were gifted to him by his grandfather), 15,000, (iv) Deepak got winnings from lottery (gross), 2,00,000, (v) Minor daughter Priya gave a dance performance and received remuneration, 1,00,000, Explain how the above will be taxed., Answer:, `, Since Mrs. X has higher income, income of minor children shall be clubbed in the income of Mrs. X and, incomes to be clubbed shall be as given below:
Page 305 :
Clubbing of Income, , 305, , Interest income + Income of lottery of minor son – Deepak (15,000 + 2,00,000) – 1,500, 2,13,500, Income of minor daughter Priya shall not be clubbed, Nil, Note 1: Income of Priya arises out of an activity involving application of her talent and is therefore not to be, included in the taxable income of her parents [Section 64(1A)]., Note 2: Whether exemption under section 10(32) shall be allowed from casual income or not is, controversial., , NOV – 1998, Question 3, (3 Marks), Mr. X is a trader. Particulars of his income and those of the members of his family are given below. These, incomes relate to the previous year ended 31st March, 2022:, `, (i) Income from business—Mr. X’s, 9,00,000, (ii) Salary derived from an educational institution by Mrs. X., She is the principal of the institution, 5,00,000, (iii) Interest on company deposits derived by master Deep Singh, (minor son). These deposits were made in the name of master Deep Singh, by his father’s father about 6 years ago (Gross), 12,000, (iv) Receipts from sale of paintings and drawings made by minor, Dipali Singh (minor daughter and noted child artist), 60,000, (v) Income by way of lottery earnings by Master Dipindar Singh (minor son), 6,000, Discuss whether the above will form part of the assessable income of any individual and also compute the, assessable income of Mr. X., Answer: Since income of Mr. X is higher, income shall be clubbed in the income of Mr. X and such, incomes shall be, Interest income of master Deep Singh (12,000 – 1,500), 10,500, Income of minor daughter Dipali Singh shall not be clubbed, Nil, Lottery income of master Dipindar Singh (6,000 – 1,500), 4,500, Total income of Mr. X shall be (9,00,000 + 10,500 + 4,500), 9,15,000, Note: Whether exemption under section 10(32) shall be allowed from casual income or not is controversial, 9. Transfer of the asset by the member of Hindu Undivided Family to the Hindu undivided family, Section 64(2)(Conversion of self-acquired property into common property of HUF), If any member of HUF has gifted any asset to the HUF, income from such asset shall be clubbed in the, income of such member but if partition has been taken place, in that case clubbing provision shall not be, applicable however income from that part of asset which has been received by the spouse of such person,, shall be clubbed in the income of such member., Rules for Clubbing of Income, Clubbing of income will also include clubbing of loss and income shall be clubbed in the following manner:, Step 1: Calculate the income/loss on the hands of recipient as if it is the income of recipient., Step 2: The income or loss calculated as above will then be clubbed with the income of the transferor under, the same head (i.e. the head to which such income belongs). Exemption upto 1500 per child per annum is, allowed under section 10(32) in respect of clubbing of income of a minor child., Step 3: Such clubbed income is the income of the transferor and provisions relating to set off and carry, forward of losses shall apply in the normal manner. Deductions under section 80C to 80U shall also be, allowed in the normal manner., Illustration 8: Mr. X gifts `1 lakh to his wife Mrs. X on April 1, 2021 which she invests in a firm on, interest rate of 14% per annum. On January 1, 2022, Mrs. X withdraws the money and gift it to her son’s, wife. She claims that interest which has accrued to the daughter-in-law, from January 1, 2022 to March 31,
Page 306 :
Clubbing of Income, , 306, , 2022 on investment made by her is not assessable in her hands but in the hands of Mr. X. Is this correct?, What would be the position, if Mrs. X has gifted the money to minor grandson, instead of the daughter-inlaw?, Solution: Section 64(1) provides that in computing the total income of any individual, there shall be clubbed, all such income as arises directly or indirectly to the son’s wife, of such individual, from assets transferred, directly or indirectly to the son’s wife by such individual otherwise than for adequate consideration., There is an indirect transfer by Mr. X to the daughter-in-law and therefore, the interest income shall be, clubbed with income of Mr. X., If Mrs. X had gifted the money to her minor grandson, then the interest income arising to the minor shall be, clubbed under section 64(1A) in the total income of that parent (son/daughter-in-law) whose total income, (before including such income) is higher., Illustration 9: Mr. X, entered into the following transactions during the previous year 2021-22:, (a) Mr. X had a fixed deposit of ` 8,00,000 with State Bank of India. He instructed the bank to credit the, interest on the deposit @ 9% from 01.04.2021 to 31.03.2022 to the savings bank account of Ms. Y, his, niece, to help her in her higher education., (b) Mr. X holds 51% share in a partnership firm. Mrs. X (wife of Mr. X) received a remuneration of `45,000, from the firm for writing its books of accounts. Mrs. X, being a fashion designer, does not possess any, qualification or training in the accountancy field., (c) Mr. X gifted a flat to Mrs. X on April 1, 2021. During the previous year 2021-22, she received rent of, `8,500 p.m. from letting out of the flat., (d) Mr. X gifted ` 4,00,000 to his minor son who invested the same in a business and he derived income of, `40,000 from the investment., (e) Mr. X’s minor daughter derived an income of ` 25,000 from participation in music shows., During the year, Mr. X got a monthly pension of ` 18,000. He had no other income. Mrs. X received salary, of ` 25,000 per month from a part time job as a fashion designer., Discuss the tax implications of each transaction and compute the total income of Mr. X and Mrs. X., Solution:, `, Computation of Total Income of Mr. X, (a) Interest income received by Miss. Y shall be clubbed in the income of Mr. X, as per section 60 (8,00,000 x 9%), 72,000, (b) Remuneration of `45,000 received by Mrs. X shall be clubbed in the income of Mr. X, as per section 64(1), 45,000, (c) Income from House Property gifted to Mrs. X shall be taxable in the hands of Mr. X, because as per section 27 Mr. X is the deemed owner, 71,400, (Rent received (i.e. ` 1,02,000) is taken as Gross Annual Value. Deduction @ 30% of, Net Annual Value is allowed u/s 24. The net income from house property would be, `71,400 (i.e. ` 1,02,000- `30,600 being 30% of NAV), (d) Income of minor child shall be clubbed in the income of Mr. X as per section 64(1A), because Mr. X has higher income (40,000 – 1,500), 38,500, (e) Income of minor daughter from music show shall not be clubbed, Nil, Pension income of Mr. X (` 18,000×12), 2,16,000, Less: Standard deduction u/s 16(ia), (50,000), Income under the head salary, 1,66,000, Total Income, 3,92,900, Salary income of Mrs. X, 3,00,000, Less: Standard deduction u/s 16(ia), (50,000), Income under the head salary, 2,50,000, Illustration 10: Mr. A has gifted a house property valued at `50 lakhs to his wife, Mrs. B, who in turn has, gifted the ·same to Mrs. C, their daughter-in-law. The house was let out at `25,000 per month throughout the, year. Compute the total income of Mr. A and Mrs. C. Will your answer be different if the said property was, gifted to his son, husband of Mrs. C?
Page 307 :
Clubbing of Income, , 307, , Answer: As per section 27(i), an individual who transfers otherwise than for adequate consideration any, house property to his spouse, not being a transfer in connection with an agreement to live apart, shall be, deemed to be the owner of the house property so transferred. Therefore, in this case, Mr. A would be the, deemed owner of the house property transferred to his wife Mrs. B without consideration. As per section, 64(1)(vi), income arising to the son’s wife from assets transferred, directly or indirectly, to her by an, individual otherwise than for adequate consideration would be included in the total income of such, individual. Income from let-out property is `2,10,000 [i.e., `3,00,000, being the actual rent calculated at `, 25,000 per month less `90,000, being deduction under section 24 @ 30% of `3,00,000], In this case, income of `2,10,000 from let-out property arising to Mrs. C, being Mr. A’s son’s wife, would, be included in the income of Mr. A, applying the provisions of section 27(i) and section 64(1)(vi). Such, income would, therefore, not be taxable in the hands of Mrs. C. In case the property was gifted to Mr. A’s, son, the clubbing provisions under section 64 would not apply, since the son is not a minor child. Therefore,, the income of `2,10,000 from letting out of property gifted to the son would be taxable in the hands of the, son. It may be noted that the provisions of section 56(2)(x) would not be attracted in the hands of the, recipient of house property, since the receipt of property in each case was from a “relative” of such, individual. Therefore, the stamp duty value of house property would not be chargeable to tax in the hands of, the recipient of immovable property, even though the house property was received by her or him without, consideration., , Note - The first part of the question can also be answered by applying the provisions of section 64(1)(vi), directly to include the income of `2,10,000 arising to Mrs. C in the hands of Mr. A. [without first applying, the provisions of section 27(i) to deem Mr. A as the owner of the house property transferred to his wife Mrs., B without consideration], since section 64(1)(vi) speaks of clubbing of income arising to son’s wife from, indirect transfer of assets to her by her husband’s parent, without consideration. Gift of house property by, Mr. A to Mrs. C, via Mrs. B, can be viewed as an indirect transfer by Mr. A to Mrs. C.
Page 308 :
Clubbing of Income, , 308, , MULTIPLE CHOICE QUESTIONS, 1. Income of a minor child suffering from any disability of the nature specified in section 80U is (a) to be assessed in the hands of the minor child, (b) to be clubbed with the income of that parent whose total income, before including minor’s income, is, higher, (c) completely exempt from tax, (d) to be clubbed with the income of father, 2. Income arising to a minor married daughter is (a) to be assessed in the hands of the minor married daughter, (b) to be clubbed with the income of that parent whose total income, before including minor’s income, is, higher, (c) completely exempt from tax, (d) to be clubbed with the income of her husband, 3. Where a member of a HUF has converted or transferred his self-acquired property for inadequate, consideration into joint family property, income arising therefrom is (a) taxable as the income of the transferor-member., (b) taxable in the hands of the HUF., (c) taxable in the hands of the karta of the HUF., (d) exempt from tax., 4. If the converted property is subsequently partitioned among the members of the family, the income, derived from such converted property as is received by the spouse of the transferor will be taxable (a) as the income of the karta of the HUF, (b) as the income of the spouse of the transferor, (c) as the income of the HUF., (d) as the income of the transferor-member, 5. Exemption of a certain amount (not exceeding the income clubbed) is available under section, 10(32), where a minor’s income is clubbed with the income of the parent. The maximum exemption, available is (a) upto `1,500 in respect of each minor child, (b) upto `1,500 in respect of each minor child maximum of two children, (c) upto `2,000 in respect of each minor child, (d) upto `2000 in respect of each minor child maximum of two children, 6. Mr. A gifts a sum of ` 1,00,000 to his brother’s wife Mrs. B. Mr. B gifts a sum of ` 1,00,000 to Mrs., A. From the sum gifted to her, Mrs. B invests in a fixed deposit, income therefrom is `10,000., Aforesaid `10,000 will be included in the total income of ............, (a) Mr. A, (b) Mrs. A, (c) Mrs. B, (d) Mr. B, 7. Scholarship received by a minor child is –, (a) to be assessed in the hands of the minor child, (b) to be clubbed with the income of that parent whose total income, before including minor’s income, is, higher, (c) completely exempt from tax u/s 10(16), (d) to be clubbed with the income of father, 8. Income of a minor child from a fixed deposit with a bank, made out of income earned from, scholarship is–, (a) to be assessed in the hands of the minor child, (b) to be clubbed with the income of that parent whose total income, before including minor’s income, is, higher, (c) completely exempt from tax
Page 309 :
Clubbing of Income, , 309, , (d) to be clubbed with the income of father, 9. Mr. X transfers income of `51,000 from rent to his major son without transfer of house property., Rent of `51,000 is –, (a) taxable in the hands of the transferor-father, (b) taxable in the hands of the his son, (c) taxable in the hands of the that parent whose total income is higher, (d) exempt from tax, 10. Interest from a fixed deposit received by a minor married daughter is –, (a) to be assessed in the hands of the minor child, (b) to be clubbed with the income of that parent whose total income, before including minor’s income, is, higher, (c) completely exempt from tax, (d) to be clubbed with the income of her husband, 11. Mr. Aarav gifted a house property valued at `50 lakhs to his wife, Geetha, who in turn has gifted, the same to her daughter-in-law Deepa. The house was let out at `25,000 per month throughout the, P.Y.2021-22. Compute income from house property for A.Y.2022-23., In whose hands is the income from house property chargeable to tax?, (a) `3,00,000 in the hands of Mr. Aarav, (b) `2,10,000 in the hands of Mr. Aarav, (c) `2,10,000 in the hands of Geetha, (d) `2,10,000 in the hands of Deepa, 12. As per section 60, which of the following statement is correct?, (a) If any person has not transferred the asset but has transferred income from such asset to any person, such, income shall be included in the income of transferor, (b) If any person has not transferred the asset but has transferred income from such asset to any person, such, income shall be included in the income of transferee, (c) If any person has not transferred the asset but has transferred income from such asset to any person, such, income is not taxable in the hands of transferor or transferee, (d) none of these, 13. Which of the following is correct as per section 61?, (a) All income arising to any person by virtue of a revocable transfer of assets shall be chargeable to, income-tax as the income of the transferee and shall be included in his total income., (b) All income arising to any person by virtue of a revocable transfer of assets shall be chargeable to, income-tax as the income of the transferor and shall be included in his total income., (c) All income arising to any person by virtue of a irrevocable transfer of assets shall be chargeable to, income-tax as the income of the transferor and shall be included in his total income., (d) none of these, 14. Which of the following statement is correct as per section 64(1)?, (a) If any individual has transferred any asset other than house property to the spouse, directly or indirectly,, income from the asset transferred, directly or indirectly shall be included in the income of transferor, provided the asset was transferred for adequate consideration or in connection with an agreement to live, apart., (b) If any individual has transferred any asset other than house property to the spouse, directly or indirectly,, income from the asset transferred, directly or indirectly shall be included in the income of transferee, provided the asset was transferred otherwise than for adequate consideration or in connection with an, agreement to live together., (c) If any individual has transferred any asset other than house property to the spouse, directly or indirectly,, income from the asset transferred, directly or indirectly shall be included in the income of transferor, provided the asset was transferred otherwise than for adequate consideration or in connection with an, agreement to live apart., (d) none of these
Page 310 :
Clubbing of Income, , 310, , 15. Which of the following statements is correct?, (a) As per section 64(1A), in computing the total income of any individual, there shall be included all such, income as arises or accrues to his minor child, including a minor child suffering from any disability of the, nature specified in section 80U:, Provided that nothing contained in this sub-section shall apply in respect of such income as arises or, accrues to the minor child on account of any—, (a) manual work done by him; or, (b) activity involving application of his skill, talent or specialised knowledge and experience., (b) As per section 64(1A), in computing the total income of any individual, there shall be included all such, income as arises or accrues to his minor child, not being a minor child suffering from any disability of the, nature specified in section 80U:, This section shall also apply in respect of such income as arises or accrues to the minor child on account of, any—, (a) manual work done by him; or, (b) activity involving application of his skill, talent or specialised knowledge and experience., (c) As per section 64(1A), in computing the total income of any individual, there shall be included all such, income as arises or accrues to his minor child, not being a minor child suffering from any disability of the, nature specified in section 80U:, Provided that nothing contained in this sub-section shall apply in respect of such income as arises or, accrues to the minor child on account of any—, (a) manual work done by him; or, (b) activity involving application of his skill, talent or specialised knowledge and experience., (d) As per section 64(1A), in computing the total income of any individual, there shall be included all such, income as arises or accrues to his major child, not being a major child suffering from any disability of the, nature specified in section 80U:, Provided that nothing contained in this sub-section shall apply in respect of such income as arises or, accrues to the minor child on account of any—, (a) manual work done by him; or, (b) activity involving application of his skill, talent or specialised knowledge and experience., (e) none of these, , Solutions are given on our website, www.mkgeducation.com
Page 311 :
Clubbing of Income, , 311, , EXAMINATION QUESTIONS, , Solutions of All Examination Questions are given on, our website www.mkgeducation.com, MAY – 2019 (Old Course), Question 4(b)., (3 Marks), Briefly explain with example, the meaning of Cross Transfer, the objective to make such transactions, and implications thereof under the Income Tax Laws., Answer: Refer answer given in the book., , NOV – 2018 (Old Course), Question 3(b)., (5 Marks), Mr. Sharma, a resident individual, aged 40 years, suffers from severe disability as certified by medical, authority. He gives the following information for the previous year 2021-2022:, (i) He had written a book for Himalaya Publication on "Yoga and its benefits". A lump sum amount of royalty, income earned in the previous year 2021-2022 amounted to `6,00,000. Expenses incurred for writing the, book amounted to `20,000., (ii) His friends gifted a statue of Lord Ganesh to his daughter Ms. Diya (aged 14 years) on the successful, completion of her secondary school. Fair market value of the statue is `55,000., (iii) The following gift was received on the occasion of his son's (aged 10 years) thread ceremony;, • in-laws-gold chain worth `35,000, (iv) He had deposited `50,000 in fixed deposit with Bank of Baroda in the name of his son in March 2020., Interest earned on such deposit `5000., (v) He donated `5,000 in cash to Swabhiman, a NGO set up for the destitute (the association was registered, under section 80G of the Income Tax Act, 1961)., (vi) He paid life insurance premium on his life `10,000 (sum assured `1,00,000), Compute the gross total income of Mr. Sharma for the assessment year 2022-2023., , NOV – 2017, Question 6(b)., (5 Marks), st, Kamal gifted `10 lakhs to his wife, Sulochana on her birthday on, 1 January, 2021., Sulochana lent `5,00,000 out of the gifted amount to Krishna on 1st April, 2021 for six months on which, she received interest of `50,000. The said sum of `50,000 was invested in shares of a listed company on 15th, October, 2021, which were sold for `75000 on 30th December, 2021. Securities transactions tax was paid on, such sale. The balance amount of gift was invested as capital by Sulochana in a business. She suffered loss, of `15,000 in the business in financial Year 2021-22., In whose hands the above income and loss shall be included in Assessment Year 2022-23? Support your, answer with brief reasons., , MAY – 2012, Question 5, (5 Marks), During the previous year 2021-22 the following transactions occurred in respect of Mr. X., (a) Mr. X had a fixed deposit of ` 5,00,000 in Bank of India. He instructed the bank to credit the interest, on the deposit @ 9% from 01.04.2021 to 31.03.2022 to the savings bank account of Mr. B, son of his, brother, to help him in his education.
Page 312 :
Clubbing of Income, , 312, , (b) Mr. X holds 75% share in a partnership firm. Mrs. X received a commission of `25,000 from the, firm for promoting the sales of the firm. Mrs. X possesses no technical or professional qualification., (c) Mr. X gifted a flat to Mrs. X on April 1, 2021. During the previous year the flat had income under, the head House Property `52,000 to Mrs. X., (d) Mr. X gifted `2,00,000 to his minor son who invested the same in a business and he got a share, income of ` 20,000 from the investment., (e) Mr. X’s minor son derived an income of `20,000 through a business activity involving application of, his skill and talent., During the year Mr. X got a monthly pension of `10,000. He had no other income. Mrs. X received, salary of ` 20,000 per month from a part time job., Discuss the tax implications of each transaction and compute the total income of Mr. X, Mrs. X and, their minor child., , NOV – 2011, Question 1, (4 Marks), Mr. X (age 67 years) gifted a building owned by him to his son’s wife Mrs. X on 01.10.2020. The building, fetched a rental income of `10,000 per month throughout the year. Municipal tax for the first half-year of, `5,000 was paid in June 2021 and the municipal tax for the second half-year was not paid till 30.09.2022., Incomes of Mr. X and Mrs. X other than income from house property are given below:, Name, Business income, Capital gain, Other sources, Mr. X, `1,00,000, `50,000 (long-term), `1,50,000, , Mrs. X, `(75,000), `2,00,000 (short-term), `50,000, Note: Capital gain does not relate to gain from shares and securities., Compute the total income of Mr. X and Mrs. X taking into account income from property given above and, also compute their income-tax liability for the assessment year 2022-23., , MAY – 2010, Question 5, (4 Marks), In whose hands the income from an asset is chargeable to tax in the case of transfer which is not revocable, during the life time of the beneficiary/transferee?
Page 313 :
Income Under The Head Other Sources, , 313, , INCOME UNDER THE HEAD, OTHER SOURCES, SECTION 56 TO 59, PARTICULARS, Income from other sources, Deductions while computing income under the head other sources, Amounts not deductible, Deemed income chargeable to tax, Deemed dividend, Method of accounting, , SECTIONS, 56, 57, 58, 59, 2(22), 145, , Question 1: What are the incomes taxable under the head Other Sources., Answer: Incomes taxable under the head Other Sources Section 56, If any income cannot be taxed under first 4 heads, such income shall be taxable under the head other sources, and such income may be, 1. Interest income, 2. Dividend income, 3. Casual income, 4. Gift, 5. Family pension, 6. Payment received under keyman insurance policy to a person who is not an employee, 7. Income from owning and maintaining of race horses, 8. Forfeiture of advance money, 9. Any other income which is not taxable under first four heads., Question 2 [Imp.]: Discuss the deductions allowable under section 57 of the Income Tax Act, 1961, in, respect of Income from Other Sources., Answer:, Deductions allowable under Section 57, While computing income under the head other sources, expenses incurred in connection with earning of, such income shall be allowed to be deducted. However, in case of dividend income or income in respect of, units of mutual fund specified u/s 10(23D) or units of UTI, deduction shall be allowed only for the, interest expenses and that too shall also be restricted to 20% of the such income. Mr. X has taken a loan of, ₹10 Lakh and paid interest ₹1 lakh and amount was invested in shares of a company and dividend received, is ₹ 2 lakh, in this case ₹1 lakh shall not be allowed to be deducted rather amount allowed to be deducted, shall be ₹40,000 and income shall be considered to be ₹1,60,000., Amounts not deductible Section 58, While computing income, any personal expense shall not be allowed to be deducted and also in case of, capital expenditure only depreciation shall be allowed., As per section 58(4), no deduction in respect of any expenditure or allowance shall be allowed in, computing the income by way of any winnings from lotteries, crossword puzzles, races including horse, races, card games and other games of any sort or from gambling or betting of any form or nature,, whatsoever., e.g. Mr. X purchased lottery tickets of `10,000 and he had a winning of `1,00,000, in this case, his income
Page 314 :
Income Under The Head Other Sources, , 314, , shall be considered to be `1,00,000 and expenditure of `10,000 shall not be allowed., Profits chargeable to tax Section 59, If the assessee has claimed any expenditure while computing income and subsequently he has recovered the, same amount, the amount so recovered shall be considered to be income of the year in which amount has, been recovered, e.g: Mr. X has received a cheque of ` 1,00,000 being interest from ABC Ltd. and cheque, was deposited in his bank account and bank has deducted ` 1,000 being collection charge. His income was, considered to be ` 99,000. In the next year bank has refunded ` 500 being excess charges collected, in this, case ` 500 shall be considered to be income of the year in which the amount has been received, Question 3: Write a note on Family Pension., Answer: Family Pension, Regular payments given by the employer to the employee after retirement is called pension and it is taxable, under the head Salary and standard deduction is allowed under section 16(ia), e.g. Mr. X is retired from, ABC Ltd. and is getting pension ` 40,000 per month, in this case taxable amount under the head salary shall, be 40,000 X 12 – 50,000 = 4,30,000., After the death of the employee, employer may pay some pension to the family member of the employee, and it is called family pension. It is taxable under the head Other Sources but as per section 57 deduction is, allowed equal to 1/3rd of such pension but maximum `15,000 E.g. Mrs. X is getting family pension of, `5,000 p.m., in this case taxable amount shall be (5,000 x 12)minus 1/3 of `60,000 or `15,000 whichever is, less i.e. taxable amount shall be `45,000. If family pension is ` 3,000 per month, taxable amount shall be, 36,000 – 12,000 = 24,000, Illustration 1: Mrs. X is getting family pension of `7,000 p.m. She also has dividend income from domestic, company of `31,000. She has long term capital gain under section 112A `3,00,000., Compute her tax liability for assessment year 2022-23., Solution:, `, `, Income under the head Other Sources, Family Pension, 84,000, (7,000 x 12), Less: Deduction u/s 57, (15,000), 69,000, 1/3 of `84,000 or `15,000 whichever is less, Dividend income, 31,000, Income under the head Other Sources, 1,00,000, Income under the head Capital Gains, Long term capital gain 112A, 3,00,000, Gross Total Income, 4,00,000, Less: Deduction u/s 80C to 80U, Nil, Total Income, 4,00,000, Computation of Tax Liability, Tax on LTCG 112A `3,00,000 – 1,50,000 – 1,00,000 @ 10%, 5,000, Tax on `1,00,000 at slab rate, Nil, Tax before health & education cess, 5,000, Add: HEC @ 4%, 200, Tax Liability, 5,200, Note: Rebate not allowed from LTCG 112A, Question 4: Write a note on taxability of interest received on payment of compensation from the, government., Answer:, As per section 145B, interest received for payment of compensation from the Government or other, similar agency in connection with compulsory acquisition of land or building shall be taxable in the year in, which it has been received and it will be taxable under the head other sources however, as per section 57, deduction shall be allowed @ 50% of such interest. e.g. Government has acquired one land of Mr. X in, Noida in 2013 and payment was given by the Government in the year 2021-22 and has also paid interest of, `1,00,000, in this case, taxable amount shall be `1,00,000 – `50,000 = `50,000.
Page 315 :
Income Under The Head Other Sources, , 315, , Illustration 2: Interest on enhanced compensation received by Mr. X during the previous year 2021-22 is, `6,50,000. Out of this interest, ` 2,00,000 relates to the previous year 2018-19, `2,15,000 relates to previous, year 2019-20 and `2,35,000 relates to previous year 2020-21. Discuss the tax implication, if any, of such, interest income for A.Y.2022-23., Solution:, The entire interest of ` 6,50,000 would be taxable in the year of receipt, namely, P.Y.2021-22., Particulars, `, Interest on enhanced compensation taxable u/s 56, 6,50,000, Less: Deduction under section 57 @ 50%, (3,25,000), Interest chargeable under the head “Income from other sources”, 3,25,000, NOV – 2013 (4 Marks), State with brief reasoning whether the following receipts are chargeable to income-tax or are exempt (if, chargeable, the amount taxable is to be mentioned) for the assessment year 2022-23:, (i) Interest on enhanced compensation received on 12.03.2022 for acquisition of urban land, of which 40%, relates to the earlier year., 96,000, (ii) Rent received for letting out agricultural land for a movie shooting., 72,000, Computation is NOT required., Solution:, (i) Yes, it is chargeable to tax., As per section 145B, interest received by the assessee on enhanced compensation shall be deemed to be the, income of the year in which it is received, irrespective of the method of accounting followed by the assessee., Interest of ` 96,000 on enhanced compensation is chargeable to tax in the year of receipt i.e. P.Y.2021-22, under section 56 after providing deduction of 50% under section 57. Therefore, ` 48,000 is chargeable to tax, under the head “Income from other sources”., (ii) Yes, it is chargeable to tax., Agricultural income is exempt from tax as per section 10(1). Agricultural income means, inter alia, any rent, or revenue derived from land which is situated in India and is used for agricultural purposes. In the present, case, rent is being derived from letting out of agricultural land for a movie shoot, which is not an agricultural, purpose. In effect, the land is not being put to use for agricultural purposes. Therefore, ` 72,000, being rent, received from letting out of agricultural land for movie shooting, is not exempt under section 10(1). The, same is chargeable to tax under the head “Income from other sources”., NOV – 2011 (2 Marks), On 10.10.2021, Mr. X (a bank employee) received `5,00,000 towards interest on enhanced compensation, from State Government in respect of compulsory acquisition of his land effected during the financial year, 2017-18., Out of this interest, `1,50,000 relates to the financial year 2018-19; `1,65,000 to the financial year 2019-20;, and `1,85,000 to the financial year 2020-21. He incurred `50,000 by way of legal expenses to receive the, interest on such enhanced compensation., How much of interest on enhanced compensation would be chargeable to tax for the assessment year 202223?, Answer:, Computation of amount chargeable to tax, `, Interest on enhanced compensation received, 5,00,000.00, Less: Deduction u/s 57, (2,50,000.00), (5,00,000 x 50/100), Amount chargeable to tax, 2,50,000.00, Question 5 [V. Imp.]: Write a note on taxability of Dividend Income., Answer: Dividend Income Section 56, Dividend income from the domestic company shall be Taxable in the hands of the shareholder. Dividends, from a foreign company shall also be taxable in the hands of the shareholder.
Page 316 :
Income Under The Head Other Sources, , 316, , If any such person is engaged in the business of sale purchase of shares , even in that case dividend income, shall be taxable under the head other sources., As per section 57, in case of dividend income, deduction shall be allowed only for the interest expenses, and that too shall also be restricted to 20% of the dividend income., MEANING OF DIVIDEND SECTION 2(22), The term dividend has a very limited meaning under Companies Act but it has a very wide meaning under, Income Tax Act and is called deemed dividend and it is divided into 5 parts:, (i) Distribution in cash or as assets Section 2(22)(a), (ii) Issue of bonus shares etc. Section 2(22)(b), (iii) Distribution on liquidation Section 2(22)(c), (iv) Distribution on reduction of share capital Section 2(22)(d), (v) Loan and advance by a closely held company Section 2(22)(e), (i) Distribution in cash or as assets Section 2(22)(a), If any company has distributed any amount to its shareholders either in cash or in kind, it will be considered, to be dividend but only to the extent of accumulated profits including capitalized profit., Example, Balance sheet of ABC limited., Liability, Amount, Assets, Amount, Share Capital (includes Bonus shares, 17,00,000, Assets, 25,00,000, of ` 2 Lacs), Reserve and Surplus, 3,00,000, Liability, 5,00,000, Total, 25,00,000, Total, 25,00,000, Company distributed assets having Book value of ₹3,00,000 to is shareholders. Calculate the Deemed, Dividend u/s 2(22)(a) having market Value1. 5,00,000, 2. 7,00,000, Solution:, Accumulated Profit of the Company (whether Capitalised or Not), - Reserves and Surplus + Bonus Shares, - 3,00,000 + 2,00,000 = 5,00,000, , Deemed Dividend u/s 2(22)(a) shall be as followsMarket Value, 5,00,000, 7,00,000, , Deemed Dividend, 5,00,000, 5,00,000, , (ii) Issue of bonus shares etc. Section 2(22)(b), If any company has issued bonus shares to the equity shareholders, it will not be considered to be dividend, but if the bonus shares have been issued to the preference shareholders, it will be considered to be dividend, but to the extent of accumulated profits whether capitalised or not. Further, market value of the bonus shares, shall be taken into consideration., Example, Mr. X is holding 100 preference share in ABC Ltd. The company has issued him 100 bonus shares and their, market value is `1,200. In this case, it will be considered to be dividend but only to the extent of, accumulated profits whether capitalized or not.
Page 317 :
Income Under The Head Other Sources, , 317, , (iii) Distribution on liquidation Section 2(22)(c), If any company has distributed any amount to its shareholders in connection with its liquidation, it will be, considered to be dividend but only to the extent of accumulated profits and any excess over it shall be, considered to be full value of consideration as per section 46 and capital gains shall be computed, accordingly., Example, ABC Ltd. has 1,00,000 equity shares of `10 each and the company goes into liquidation on 31.07.2021 and, company has net distributable amount of `60 lakhs after discharging all the liabilities including income tax, and it includes accumulated profits of `20 lakhs and the entire amount was distributed among the, shareholders and Mr. X is holding 10,000 equity shares which were purchased by him on 01.03.2021 for `, 1,10,000, in this case, tax treatment shall be as given below:, `, Net Distributable Amount, 60,00,000, Share of Mr. X (10%), 6,00,000, Share of Mr. X out of accumulated profits which is considered dividend u/s 2(22)(c), (2,00,000), Balance to be considered full value of consideration, 4,00,000, Less: Cost of acquisition of shares, (1,10,000), Short term Capital Gain, 2,90,000, Dividend u/s 2(22)(c), 2,00,000, Tax liability on ` 4,90,000 at slab rate, 12,000, Less: Rebate u/s 87A, (12,000), Tax Liability, Nil, MAY – 2008 (5 Marks), Mr. X purchased 10,000 equity shares of ABC Co. Pvt. Ltd. on 28.02.2021 for ` 1,20,000. The company, was wound up on 31.07.2021. The following is the summarized financial position of the company as on, 31.07.2021:, Liability, Assets, `, `, 1,00,000 Equity shares, 10,00,000 Land, 42,00,000, General reserve, 40,00,000 Cash at bank, 10,50,000, Provision for taxation, 2,50,000, 52,50,000, 52,50,000, The tax liability was ascertained at `3,00,000. The remaining assets were distributed to the shareholders in, the proportion of their shareholding. The market value of land as on 31.07.2021 is `100,00,000., The land received above was sold by Mr. X on 28.02.2022 for ` 15,00,000., Discuss the tax consequences in the hands of the company and Mr. X, Answer., In the hands of the company:, As per section 46, if any company is in liquidation and it has distributed its assets to its shareholders, no, capital gains shall be computed in the hands of the company hence in the given case there is no capital gains, for sale of land to shareholder., In the hands of Mr. X (shareholder), Particulars, Amount (`), Mr. X holds 1/10th of the shareholding of the company, Market value of land received (1/10 x 100,00,000), 10,00,000, Cash at bank [1/10th of ` (10,50,000 – ` 3,00,000)], 75,000, 10,75,000, Less: Deemed Dividend u/s 2(22)(c) 1/10th of (`40,00,000-` 50,000), (3,95,000), Consideration for computing Capital Gain, 6,80,000, Cost of acquisition of Shares, 1,20,000, Short term capital gains, 5,60,000, Mr. X has received the land from the company for `10 lakh but it has been sold by him for `15 lakh, in this, case capital gains shall be computed in the manner given below:, Particulars, `
Page 318 :
Income Under The Head Other Sources, , 318, , Sale consideration, 15,00,000, Less : Fair market value of the land on the date of distribution, (10,00,000), Short term capital gain, 5,00,000, Dividend u/s 2(22)(c) ` 3,95,000 will be Taxable in the hands of Mr. X (Shareholder) during the F.Y.202021., Original text Section 46., Capital gains on distribution of assets by companies in liquidation., (1) Notwithstanding anything contained in section 45, where the assets of a company are distributed to its, shareholders on its liquidation, such distribution shall not be regarded as a transfer by the company for the, purposes of section 45., (2) Where a shareholder on the liquidation of a company receives any money or other assets from the, company, he shall be chargeable to income-tax under the head "Capital gains", in respect of the money so, received or the market value of the other assets on the date of distribution, as reduced by the amount, assessed as dividend within the meaning of sub-clause (c) of clause (22) of section 2 and the sum so arrived, at shall be deemed to be the full value of the consideration for the purposes of section 48., (iv) Distribution on reduction of share capital Section 2(22)(d), Any distribution to its shareholders by a company on the reduction of its capital, to the extent to which the, company possesses accumulated profits e.g. Mr. X is holding 100 shares in ABC Ltd. of `10 each and, company has paid `5 per share in connection with reduction of share capital, in this case amount so received, shall be considered to be dividend but only to the extent of accumulated profits including capitalized profits., Example, Mr. X is holding 1000 shares of ABC Ltd. of `10 each and company has reduced its share capital and has, refunded `5 per share to the shareholders, the amount so received by the shareholders shall be considered to, be dividend to the extent of accumulated profit., ABC Ltd. has share capital 50,00,000 and Reserve and Surplus ` 10,00,000 and company has distributed `, 15,00,000 in correction with reduction of share capital, in this case dividend under section 2(22)(d) shall be, ` 10,00,000., (v) Loan and advance by a closely held company Section 2(22)(e), If any closely held company (also called company in which public are not substantially interested) has given, any loan or advance to an equity shareholder who is holding not less than 10% of the voting power of the, company, in such cases such loan or advance shall be considered to be dividend in the hands of such, shareholder but only to the extent of accumulated profits excluding capitalized profits e.g. ABC Pvt. Ltd. a, closely held company has general reserves of `7,00,000 and current profits of `2,00,000. The company has, given a loan of `3,00,000 to one such shareholder Mr. X. in this case, it will be considered to be dividend in, the hands of Mr. X. If loan given by the company is `10,00,000, the amount of dividend shall be `9,00,000., , If the loan or advance has been given to any concern (Partnership firm, company, AOP, BOI etc.) in which, such a shareholder has substantial interest, such loan or advance shall also be considered to be dividend in, the hands of such concern but only to the extent of accumulated profits excluding capitalized profits ., Example, (i) Mr. X is the beneficial owner of 10% equity shares in ABC Pvt. Ltd. (A closely held company) and the, company has general reserve of `10,00,000 and has given a loan of `6,00,000 to a partnership firm XY in, which Mr. X is holding 20% shares. In this case, the loan so given shall be considered to be dividend in the, hands of partnership firm ., (ii) Mr. X is a shareholder in a Company A (A Closely held company) as well as Company B. He has 10%, shareholding in Company A and 20% shareholding in Company B. The accumulated profits of Company A, = `10 lakh. A loan of `12 lakh is given by Company A to Company B., This loan up to the extent of accumulated profits of ` 10 lakh is treated as dividend and is taxable in the, hands of Company B., If the loan or advance has been given to any person on behalf of such a shareholder, it will also be, considered to be dividend.
Page 319 :
Income Under The Head Other Sources, , 319, , Where a loan had been treated as dividend and subsequently, the company declares and distributes dividend, to all its shareholders including the borrowing shareholder, and the dividend so paid is set off by the, company against the previous borrowing, the adjusted amount will not be again treated as a dividend., E.g. Mr. X is holding 10% shares in ABC private limited a closely held company and has taken a loan of, 10,00,000 and it was considered to be dividend under section 2(22)(e) and in subsequent year the company, has declared dividend of 10,00,000 which was deposited in the loan account of Mr. X, in this case it will not, be considered to be dividend., If any such company has the business of lending as substantial part of its business, in such cases the above, provisions shall not apply e.g. ABC Pvt. Ltd. is a closely held company and is engaged in banking business, (lending of money), in this case section 2(22)(e) is not applicable for ABC Pvt. Ltd., As per section 2(22)(e), if any trade advance is given to the shareholder covered under section 2(22)(e), it, will not be considered to be dividend, eg. Mr. X is holding 10% share in XYZ private limited a closely held, company and Mr. X is supplying certain goods to the company and has received some advance, it will not be, considered to be dividend., NOV – 2020 (4 Marks), Question 4 (c), Ms. Julie received following amounts during the previous year 2021-22., (1) Received loan of `5,00,000 year from the ABC Private Limited, a closely held company engaged in, textile business. She is holding 10% of the equity share capital in the said company. The accumulated profit, of the company was `2,00,000 on the date of the loan., (2) Received Interest on enhanced compensation of `5,00,000. Out of this interest, `1,50,000 relates to the, previous year 2018-19, `1,90,000 relates to previous year 2019-20 and `1,60,000 relates to previous, year 2020-21. She paid 1 lakh to her advocate for his efforts in the matter., Discuss the tax implications, if any, arising from these transactions in her hand with reference to, Assessment Year 2022-23., Answer:, (1) As per section 2(22)(e), If any closely held company (also called company in which public are not, substantially interested) has given any loan or advance to an equity shareholder who is holding not less than, 10% of the voting power of the company, in such cases such loan or advance shall be considered to be, dividend in the hands of such shareholder but only to the extent of accumulated profits excluding capitalized, profits., In the given case, She is holding 10% and company is a closely held company, hence amount received to the, extent of accumulated profits i.e. 2,00,000 shall be considered to be deemed dividend u/s 2(22)(e)., (2) As per section 145B, interest received for late payment of compensation from the Government or, other similar agency in connection with compulsory acquisition of land or building shall be taxable in the, year in which it has been received and it will be taxable under the head other sources however, as per, section 57 deduction shall be allowed @ 50% of such interest., Interest on enhanced compensation, 5,00,000, Less: Deduction @ 50% u/s 57, 2,50,000, Income under the head other sources, 2,50,000, , NOV – 2016, Question 3(a), (4 Marks), Mr. Rakesh has 15% share holding in RSL (P) Ltd and has also 50% share in Rakesh & Sons, a partnership, firm., The accumulated profit of RSL (P) Ltd. is 20 Lakh. Rakesh & Sons had taken a loan of `25 Lakh, from RSL, (P) Ltd. Explain, whether the above loan is treated as dividend, as per the provision of Income Tax Act,, 1961.
Page 320 :
Income Under The Head Other Sources, , 320, , Solution:, As per Section 2(22)(e), If the loan or advance has been given to any concern in which shareholder has, substantial interest, such loan or advance shall be considered to be dividend in the hands of such concern but, only to the extent of accumulated profits excluding capitalized profits., In this case dividend in the hands of the shareholder is nil and in hands of the firm are `20 lakhs., MAY – 2011 (2 Marks), Mr. X holding 28% of equity shares in a company took a loan of `5,00,000 from the same company. On the, date of granting the loan, the company had accumulated profit of `4,00,000. The company is engaged in, some manufacturing activity., (i), Is the amount of loan taxable as deemed dividend in the hands of Mr. X, if the company is a, company in which the public are substantially interested?, What would be your answer, if the lending company is a private limited company (i.e.) a, (ii), company in which the public are not substantially interested?, Answer:, Any payment by a company, other than a company in which the public are substantially interested, of any, sum by way of advance or loan to an equity shareholder, being a person who is the beneficial owner of, shares holding not less than 10% of the voting power, is deemed as dividend under section 2(22)(e), to the, extent the company possesses accumulated profits., (i) The provisions of section 2(22)(e), however, will not apply where the loan is given by a company in, which public are substantially interested. In such a case, the loan would not be taxable as deemed dividend., (ii) However, if the loan is taken from a private company (i.e. a company in which the public are not, substantially interested), which is a manufacturing company and not a company where lending of money is a, substantial part of the business of the company, then, the provisions of section 2(22)(e) would be attracted,, since Mr. X holds more than 10% of the equity shares in the company., The amount chargeable as deemed dividend cannot, however, exceed the accumulated profits held by the, company on the date of giving the loan. Therefore, the amount taxable as deemed dividend would be limited, to the accumulated profit i.e., `4,00,000 and not the amount of loan which is `5,00,000., Question 6 [V. Imp.]: Write a note on taxability of Casual Income., Answer: Casual Income Section 56, Under section 2(24)(ix), casual income shall include card games, cross word puzzles, betting, races, including horse races, any game show on electronic media or any other gambling., While computing income from casual income, as per section 58(4) no expenditure or allowance or, deductions shall be allowed and accordingly the gross receipt itself shall be considered to be income., Example, Mr. X purchased one lottery ticket of `10,000 and there was a winning of `1,20,000. Deductions allowed, under section 80C to 80U ` 1,00,000. He has loss under the head house property ` 50,000, In this case, his, taxable income shall be `1,20,000 and tax liability shall be, Tax on `1,20,000 @ 30%, 36,000, Less: Rebate u/s 87A, (12,500), Tax before HEC, 23,500, Add: HEC @ 4%, 940, Tax Liability, 24,440, Note: No expenditure or deduction or loss is allowed to be adjusted from casual income however rebate is, allowed from tax of casual income, Question 7. Write a note on taxability of income from Owning and Maintaining of Race Horses., Answer: Income from Owning and Maintaining of Race Horses Section 56, If any person has income from owning and maintaining of race horses, such income shall be taxable under, the head other sources and income shall be computed in the normal manner and will be taxed at the, normal rates., As per Section 74A, If any person has any loss from the activities of owning and maintaining race horse,, such loss is not allowed to be set off from any income under any head. However, if the assessee has any
Page 321 :
Income Under The Head Other Sources, , 321, , other business of owning and maintaining race horses, loss of one such business can be set off from the, income of other such business., If the loss can not be set off, it will be allowed to be carried forward, but such carry forward is allowed for a, maximum period of four years and brought forward loss can be set off only from the income of owning and, maintaining race horses., Income from owning and maintaining of any other animal, If the assessee is engaged in the business of owning and maintaining any other animal, his income shall be, computed under the head business/profession because section 56 includes only income from owning and, maintaining race horses. E.g. Mr. X has income from owning and maintaining of race camels, in this case, income shall be taxable under the head business/profession., E.g. (i) Mr. X has loss of `5,00,000 from owning and maintaining of race horses and income under the head, house property `5,00,000, in this case loss is not allowed to be setoff, however its carry forward is allowed, for a period of 4 years., (ii) Mr. X has loss of `2,00,000 from house property and income from owning and maintaining of race, horses `2,00,000, in this case loss is allowed to be setoff., (iii) Mr. X has loss of `5,00,000 from business/profession and income from owning and maintaining of race, horses `5,00,000, in this case loss is allowed to be setoff., (iv) Mr. X has loss of `5,00,000 from owning and maintaining of race horses and income under the head, capital gains `5,00,000, in this case loss is not allowed to be setoff, however its carry forward is allowed for, a period of 4 years., Question 8 [Imp.]: Write a note on taxability of interest income., Answer: Taxability of interest income Section 56, Any Interest income shall be taxable under the head Other Sources however some of the interest incomes, shall be exempt from income tax under section 10(15) and are as given below:, 1. Interest on Capital Investment Bonds issued by the Government., 2. Interest on Relief Bonds issued by RBI., 3. Interest on Post Office Savings Bank Account to the extent of `3,500 per year and in the case of joint, account, exemption shall be allowed upto `7,000 per year., 4. Interest on Public Provident Fund Account, 5. Any other interest income notified under section 10(15)., Question 9: Explain taxability of income from letting out of building alongwith furniture, fixtures etc., Answer: If any person has let out any building alongwith plant and machinery and furniture, fixtures etc., and it is not a case of composite rent and also income is not taxable under the head business/profession, in, such cases income shall be taxable under the head Other sources and while computing income all expenses, incurred shall be allowed to be deducted e.g. Mr. X has one factory building along with machines and, furniture in Bombay which has been let out @ `50,000 p.m. Repair charges of the building is `7,000 and, that of furniture fixtures are `4,000, insurance premium paid `3,000 and depreciation is `27,000, in this case, income shall be computed in the manner given below:, Solution:, `, Gross Rent (50,000 x 12), 6,00,000, Less: Repair of building, (7,000), Less: Repair of Furniture and fixtures, (4,000), Less: Insurance premium, (3,000), Less: Depreciation, (27,000), Income under the head Other Sources, 5,59,000, Question 10: Write a note on Books of Accounts., Answer: Books of Accounts Section 145, A person is not required to maintain any books of accounts under the head salary or house property or, capital gains and income has to be computed as per the procedure given in the relevant head., Books of accounts are required under the head Business/Profession and under the head Other Sources. An, assessee has the option to maintain books of accounts either on the basis of mercantile system of accounting, or on cash basis. Any system of accounting once adopted has to be followed consistently, however it can be
Page 322 :
Income Under The Head Other Sources, , 322, , changed with the permission of Assessing Officer., Example, Mr. X has deposited `10,00,000 in ABC Ltd. @ 10% p.a. and interest income is due on yearly basis on 31st, March every year. Interest income which was due on 31.03.2022 was received on 01.04.2022. In this case, if, the assessee is maintaining books of account on the basis of mercantile system of accounting, income is, taxable in previous year 2021-22, and if the books are maintained on cash basis, income is taxable in the, previous year 2022-23., Question 11: Write a note on payment under keyman insurance policy., Answer: Payment under Keyman Insurance Policy, Sometimes employer may take a life policy in the name of any of his employees who are considered to be, very important for business or profession and such policy is called keyman insurance policy and premium is, paid by employer and employer is allowed to debit it to profit and loss account and amount received on, maturity shall be considered to be income of employer as per section 28., If any payment has been received by the employee, it will be considered to be income under the head salary., Similarly a policy may be taken in the name of any other person who is considered to be very important for, the business of the employer, such policy is also called keyman insurance policy. If payment has been, received by such other person, it will be considered to be his income under the head other sources as per, section 56., Question 12: Write a note on forfeiture of advance money., Answer: Forfeiture of advance money, If any person has entered into an agreement to sell any capital asset and some advance money was received, but the buyer refused to purchase the capital asset and advance money was forfeited, in such cases the, amount so forfeited shall be considered to be income under the head Other Sources. e.g. Mr. X has entered, into agreement to sell a house property for `50 lakh to Mr. Y and advance money of `5,00,000 was received, but Mr. Y refused to purchase the property and advance money was forfeited, in this case `5,00,000 shall be, considered to be income of Mr. X under the head Other Sources., Question 13 [Imp.]: Write short note on Set Off and Carry Forward of loss arising under the head, “Income from Other Sources”., Answer: Set Off and Carry Forward of Losses under the head “Income from Other Sources”, As per section 70, if there is loss in one source under the head other sources, such loss can be set off from, income of any other source under the same head., However, as per section 58(4), no deduction or set off shall be allowed from the income by way of any, winnings from lotteries, crossword puzzles, races including horse races, card games, and other games of any, sort or from gambling or betting of any form whatsoever., As per section 71, if the loss can not be set off under the same head, it can be set off from the incomes of, other heads., If the loss can not be set off even from the incomes of other heads, its carry forward is not allowed., e.g. (i) Mr. X has loss under the head other sources `2,00,000 and income under the head other sources, `5,00,000, in this case loss is allowed to be setoff., (ii) Mr. X has loss under the head other sources `2,00,000 and income under the head house property, `5,00,000, in this case loss is allowed to be setoff., (iii) Mr. X has loss under the head other sources `2,00,000 and income from owning and maintaining of race, horses `5,00,000, in this case loss is allowed to be setoff., (iv) Mr. X has loss under the head other sources `2,00,000 but do not have income under any other head, in, this case carry forward of loss is not allowed., Question 14: Write a note on income of closely held company by issue of Shares., Answer:, As per section 56 (2) (viib), where a company, not being a company in which the public are substantially, interested, receives, in any previous year, from any person being a resident, any consideration for issue of, shares that exceeds the face value of such shares, the aggregate consideration received for such shares as, exceeds the fair market value of the shares. In other words if shares are issued at a price which is higher than
Page 323 :
Income Under The Head Other Sources, , 323, , the market value and also higher than the face value, in that case taxable amount shall be the issue price less, market price. E.g. ABC Pvt. Ltd. a closely held company has submitted information as given below:, 1. Face value ` 100 per share, Market value ` 120 per share and issue price ` 150 per share, in this case, taxable amount shall be ` 30 per share., 2. Face value ` 100 per share, Market value ` 80 per share and issue price ` 95 per share, in this case, taxable amount shall be Nil because issue price is not exceeding the face value., 3. Face value ` 100 per share, Market value ` 80 per share and issue price ` 110 per share, in this case, taxable amount shall be 110 – 80 = 30 because issue price is exceeding the face value and also market, value., , NOV – 2019 (NEW COURSE), Question.3. (c), (4 Marks), MLX Investments (P) Ltd. was incorporated during P.Y. 2019-20 having a paid up capital of ` 10 Lakhs. In, order to increase its capital, the company further issues, 1,00,000 shares (having face value of `100 each), during the year at par as on 01-08-2021. The FMV of such share as on 01-08-2021 was ` 85., (i) Determine the tax implications of the above transaction in the hands of company, assuming it is the only, transaction made during the year., (ii) Will your answer change, if shares were issued at `105 each?, , (iii) What will be your answer, if shares were issued at `105 and FMV of the share was `120 as on 01-082021?, Answer:, As per section 56(2)(viib), where a company, not being a company in which public are substantially, interested, receives in any previous year, from any resident person, any consideration for issue of shares that, exceeds the face value of the shares, aggregate consideration received for such shares as exceeds the Fair, market value of the shares shall be taxable under the head other sources., (i) In the given case, shares are issued at face value and does not issue in price exceeding face value of the, shares. Hence no amount is taxable., (ii) In the given case, shares are issued exceeding face value hence taxable amount shall be, (Issue price – FMV of the share) x No. of shares, = (105-85) x 1,00,000 = 20,00,000, (iii) In the given case, shares are issued exceeding face value but shares are issued lower than FMV, hence, no amount shall be taxable., Question 15: Write a note on Bond Washing Transactions., Answer: Bond Washing Transactions Section 94, If any person has transferred any security in the name of any other person sometimes before the due date and, has reacquired it sometimes after the due date in order to evade tax, it will be considered to be a bond, washing transaction and income shall be considered to be of the person who has manipulated in this manner., Example, Mr. Yuvraj Arora has purchased security of `10,00,000 in ABC Ltd. on 01.04.2021 @ 10% and interest is, due on half yearly basis i.e. on 30th Sept and 31st March of every year. If Mr. Yuvraj Arora has transferred, this security just before the due date in the name of any other person through a fictitious sale transaction and, has re-transferred it in his name after the due date through a fictitious purchase transaction so that he can, evade tax, it will be called bond washing transaction and in such cases interest income is taxable in the, hands of Mr. Yuvraj Arora., [This practice is generally adopted by high-income class assessees to evade the tax by transferring securities, to low income class assessee on the eve of due date of payment of interest.]
Page 324 :
Income Under The Head Other Sources, , 324, , Question 16: Explain taxability of compensation or other payment in connection with employment., Answer: As per section 56(2)(xi), any compensation or other payment, due to or received by any person, by, whatever name called, in connection with the termination of his employment or the modification of the, terms and conditions relating thereto, shall be taxable under the head other sources., Illustration 3: Mr. X has taken a loan of `1,00,000 @ 10%. The amount was invested by him in the, securities of one company. During the year he has received gross interest of `18,000 and has paid collection, charges to the bank `500. He has paid interest `10,000 on the loan taken by him for investment and has long, term capital gain under section 112A of `4,00,000 and casual income ` 10,000. Deductions allowed under, section 80C to 80U ` 10,000, Compute his tax liability for assessment year 2022-23., Solution:, `, Gross Interest, 18,000, Less:, (i) Bank Charges u/s 57, (500), (ii) Interest paid for borrowing the amount u/s 57, (10,000), 7,500, Casual Income, 10,000, Income under the head Other Sources, 17,500, Income under the head Capital Gains (LTCG 112A), 4,00,000, Gross Total Income, 4,17,500, Less: Deductions under section 80C to 80U, (7,500), Total Income, 4,10,000, Computation of Tax Liability, Tax on Casual Income 10,000 X 30%, 3,000, Tax on LTCG 112A (4,00,000 – 2,50,000 – 1,00,000) X 10%, 5,000, Less: Rebate u/s 87A, (3,000), Tax before HEC, 5,000, Add: HEC @ 4%, 200, Tax Liability, 5,200, Note: Rebate is not allowed from tax on LTCG 112A, Illustration 4 (From RTP): Mr. X reports the following transactions to you:, (i) Received cash gifts on the occasion of his marriage on 15.06.2021 of ` 1,08,000. It includes gift of, `28,000 received from non-relatives., (ii) On 15.08.2021, being his birthday, he received a gift by means of cheque from his mother's maternal, aunt for an amount of ` 49,000., (iii) On 25.12.2021 he acquired a vacant site from his friend for ` 1,50,000. The State stamp valuation, authority fixed the value of site at ` 2,25,000 for stamp duty purpose., (iv) He bought 200 equity shares of a listed company from another friend for ` 75,000., The value of shares in the stock exchange on the date of purchase was ` 1,75,000., (v) A cell phone worth ` 21,000 is gifted by his friend on 16.8.2021, Determine the amount chargeable to tax in the hands of Mr. X for the Assessment Year 2022-23., Your answer should be supported with reasons., Solution:, Computation of amount chargeable to tax in hands of Mr. X for A.Y. 2022-23, , (i), (ii), , Particulars, Cash gift of ` 1,08,000 received on the occasion of his marriage is not taxable since gifts, received by an individual on the occasion of marriage are excluded under section 56, even, if the same are from non-relatives., Even though mother’s maternal aunt does not fall within the definition of “relative” under, , `, Nil, , Nil
Page 325 :
Income Under The Head Other Sources, , 325, , section 56, gift of ` 49,000 received from her by cheque is not chargeable to tax since the, aggregate sum of money received by Mr. X without consideration from non-relatives, (other than on the occasion of marriage) during the previous year 2021-22 does not exceed, ` 50,000., (iii) Purchase of land for inadequate consideration on 25.12.2021 would attract the provisions 75,000, of section 56. Where any immovable property is received for a consideration which is less, than the stamp duty value of the property by an amount exceeding `50,000, the difference, between the stamp duty value and consideration is chargeable to tax in the hands of the, individual. Therefore, in the given case `75,000 is taxable in the hands of Mr. X., (Difference amount is more than `50,000 and more than 10% of the consideration), (iv) Since shares are included in the definition of “property” and difference between the 1,00,000, purchase price and fair market value of shares is ` 1,00,000 (` 1,75,000 – ` 75,000) i.e. it, exceeds ` 50,000, the difference would be taxable under section 56., (v) Cell phone is not included in the definition of “property” as per section 56. Hence, it is not, Nil, taxable., Amount chargeable to tax, 1,75,000
Page 326 :
Income Under The Head Other Sources, , 326, , MULTIPLE CHOICE QUESTIONS, 1. Income from letting of machinery, plant and furniture is (a) always chargeable to tax under the head “Profits and gains of business and profession”, (b) always chargeable to tax under the head “Income from other sources”, (c) chargeable under the head “Income from other sources” only if not chargeable under the head “Profits, and gains of business and profession”, (d) chargeable to tax under the head “Income from house property”, 2. In respect of winnings from lottery, crossword puzzle or race including horse race or card game etc., (a) no deduction under Chapter VI-A is allowed and basic exemption limit cannot be exhausted, (b) no deduction under Chapter VI-A is allowed but unexhausted basic exemption can be exhausted, (c) both deduction under Chapter VI-A and basic exemption are allowed, (d) deduction under Chapter VI-A is allowed but basic exemption limit cannot be exhausted, 3. The deduction allowable in respect of family pension taxable under “Income from other sources” is, (a) 33-1/3% of the pension, (b) 30% of the pension or `15,000, whichever is less, (c) 33-1/3% of the pension or `15,000, whichever is less, (d) 30% of the pension, 4. The deduction in respect of interest on enhanced compensation of `1,50,000 received during the, previous year 2021-22, would be –, (a) ` 1,50,000, being 100% of ` 1,50,000, (b) ` 75,000, being 50% of ` 1,50,000, (c) ` 45,000, being 30% of ` 1,50,000, (d) Nil, 5. Mr. Arjun, aged 53 years, has a total income of `51 lakhs for A.Y.2022-23. His total income, comprises of salary, income from house property and interest on savings bank account. His tax, liability for A.Y.2022-23 would be –, (a) `13,96,200, (b) `14,82,000, (c) `15,35,820, (d) `14,69,000, 6. Mr. Harish, aged 40 years, earned interest of `15,000 during P.Y. 2021-22 from post office savings, bank account. What portion of such income would be ultimately included in his total income?, (a) `15,000, (b) `5,000, (c) `1,500, (d) Nil, 7. Which of the following statement is not correct?, (a) Loss from owning and maintaining of race horses cannot be set off from the income of house property, (b) Loss from owning and maintaining of race horses cannot be set off from any income except income of, owning and maintaining race horses, (c) Loss from owning and maintaining of race horses is allowed to be carry forward for a period of 4 years, (d) Loss from owning and maintaining of race horses is allowed to be carry forward for a period of 4 years, and in subsequent years it can be set off from any income, 8. Jenny has invested in debt securities of Haryali Pvt. Ltd., a company deriving its main source of, income from business of growing and processing organic vegetables and fruits. Thus, the company has, 80% of income exempt as agricultural income and 20% is taxable as business income. During the P.Y., 2021-22, Jenny derived ` 5,000 as interest income from the above investments. Which of the following, statements are correct on taxability:, (a) Interest will be exempt from tax to the extent of 80%, since Hariyali Pvt. Ltd has 80% exempted income., (b) Interest will be exempt from tax to the extent of 20%, since Hariyali Pvt. Ltd has claimed 80% of income, as exempt., (c) Interest will be fully taxable, (d) Interest will be fully exempt, 9. Which of the following statements is not correct?, (a) Issue of bonus shares to the preference shareholders shall be considered to be dividend, (b) Issue of bonus shares to the equity shareholders shall be considered to be dividend, (c) Distribution on the liquidation to the extent of accumulated profits shall be considered to be dividend, (d) None of these
Page 327 :
Income Under The Head Other Sources, , 327, , 10. Loan or advance by a closely held company shall be dividend, (a) If such loan and advance has been given to any equity shareholder who is holding not less than 20% of, the voting power of the company, (b) If such loan and advance has been given to any equity shareholder who is holding not less than 10% of, the voting power of the company, (c) If such loan and advance has been given to any equity shareholder who is holding not less than 15% of, the voting power of the company, (d) If such loan and advance has been given to any equity shareholder who is holding not less than 12% of, the voting power of the company, (e) None of these, 11. Which is the correct statement., (a) Section (2)(22)(a), 2(22)(b) and 2(22)(e) is applicable only in case of a closely held company, (b) Section (2)(22)(a), 2(22)(b) and 2(22)(e) is applicable only in case of a widely held company, (c) Section (2)(22)(a), 2(22)(b) and 2(22)(c) is applicable only in case of a closely held company, (d) Section (2)(22)(a), 2(22)(b) and 2(22)(c) is applicable in case of all the companies, (e) None of these, 12. Salary received by a member of parliament is taxable under the head, (a) Salary, (b) Business/Profession, (c) Capital Gains, (d) Other Sources, (e) None of these, 13. Salary and interest received by a partner from a partnership firm shall be, (a) Exempt from Income tax, (b) Salary taxable under the head salary and interest taxable under the head other sources, (c) Salary taxable under the head Business/Profession and interest taxable under the head other sources, (d) Salary taxable under the head salary and interest taxable under the head Business/Profession, (e) Salary taxable under the head Business/Profession and interest taxable under the head Business/, Profession, (f) None of these, 14. Mr. X has taken a loan and the amount was given as deposit to a company and interest received is, less than interest paid, in this case loss can be, (a) Set off within the same head including casual income, (b) Set off within the same head excluding casual income and also its carry forward is allowed, (c) Set off within the same head excluding casual income or it can be set off from the income of other heads, but its carry forward is not allowed, (d) None of these, 15. Mr. X received a cell phone as a gift from his friend valued ` 1,00,000, in this case, (a) It is taxable under the head Other Sources, (b) It is taxable under the head Salary, (c) It is taxable under the head Business/Profession, (d) It is not taxable, (e) None of these, , Solutions are given on our website, www.mkgeducation.com
Page 328 :
Income Under The Head Other Sources, , 328, , PRACTICE PROBLEMS, TOTAL PROBLEMS 6, Problem 1., Mr. X has income from business of owning and maintaining race camels `60,000, loss from owning and, maintaining race horses `7,000 and income from horse races `7,000. He has brought forward business loss, of `7,000 of the assessment year 2009-10 and brought forward business loss of `7,000 of the assessment, year 2017-18., Compute his tax liability for the assessment year 2022-23., Answer = Total Income: `60,000; Tax Liability: Nil; Carry forward loss from owning and maintaining race, horses: `7,000, Problem 2., Mr. X has income from owning and maintaining of race horses ` 4 lakhs and loss from horse races ` 10, lakhs., Determine his tax liability for the assessment year 2022-23., Answer = Tax Liability: Nil, Problem 3., Mr. X has loss from owning and maintaining of race horses `4 lakhs and income from owning and, maintaining of race camels `4 lakhs., Determine his tax liability for the assessment year 2022-23., Answer = Tax Nil; Carry forward loss from owning and maintaining of race horses: `4,00,000, Problem 4., Find the tax liability of Mrs. X (age 40 years), a resident individual, for the assessment year 2022-23. From, the following particulars of her incomes and spending for the previous year ending March 31st, 2022., `, Income from house property (Computed), 90,000, Dividend from UTI, 35,000, Family pension (gross), 90,000, Interest on bank FD (gross), 14,000, Dividend from foreign company, 36,000, Gift received from her sister, 26,000, Winnings from lotteries (gross), 70,000, Long-term capital gain, 1,20,000, Payment for purchase of National Savings Certificates, 35,000, Answer = Tax Liability: `26,520, Problem 5., Mr. X has submitted information given below., i), Income from owning and maintaining of race horse `2,00,000., ii), Income from owning and maintaining of race camels `1,00,000., iii), He had winning of `1,60,000 from horse race on 01.12.2021 and winning from camel race, `1,80,000 on 07.12.2021., iv), He purchased lottery tickets of `10,000 on 01.02.2022 and had winning of `2,00,000 on, 12.02.2022., v), He has received Royalty of book of literary nature @ 50% of print price of ` 600 and total copies, sold are 2,000, He has paid advance tax as given below:, vi), Upto 15.06.2021, ` 20,000, Upto 15.09.2021, ` 35,000
Page 329 :
Income Under The Head Other Sources, , 329, , Upto 15.12.2021, ` 80,000, Upto 15.03.2022, ` 1,30,000, Balance was paid on 10.06.2022, Compute tax liability for the A.Y 2022-23 and interest under section 234A, 234B and 234C., Answer = Tax Liability: `2,27,240; Interest under section 234A: Nil; 234B: `2,916; 234C: `2,280, Problem 6., ABC Ltd. has 1,00,000 equity shares of `10 each and Mr. X purchased 10,000 equity shares on 01.01.2021, of `10 each and the company goes into liquidation on 31.07.2021 and company has net distributable amount, of `60 lakhs after discharging all the liabilities and it includes accumulated profits of ` 20 lakhs and the, entire amount was distributed among the shareholders., Minor son of Mr. X has interest income of `2 lakhs from one bank deposit which was gifted to him by his, grand father., Mrs. X has one business and income from business is `1 lakh entire capital was gifted by Mr. X., Mr. X is growing flowers and has income of ` 7 lakh from sale of flowers., Compute his tax liability for Assessment Year 2022-23., Answer = Tax Liability: `1,65,930, , Detailed Solutions of All Questions are given on our, website www.mkgeducation.com
Page 330 :
Income Under The Head Other Sources, , 330, , EXAMINATION QUESTIONS, , Solutions of All Examination Questions are given on, our website www.mkgeducation.com, NOV – 2019 (NEW COURSE), Question.2. (a), (7 Marks), Mr. Jagdish, aged 61 years, has set-up his business in Thailand and is residing in Thailand since last 20, , years. He owns a house property in Bangkok, half of which is used as his residence and half is given on rent, (such rent received, converted in INR is `6,00,000). The annual value of the house in Thailand is `50,00,000, i.e. converted value in INR., He purchased a flat in Pune during F.Y. 2017-18, which has been given on monthly rent of `27,500 since, 01.07.2020. The annual property tax of Pune flat is ` 40,000 which is paid by Mr. Jagdish whenever he, comes to India. Mr. Jagdish last visited India in July 2020. He has taken a loan Union Bank of India for, purchase of the Pune flat amounting to `15,00,000. The interest on such loan for the F.Y. 2021-22 was, `84,000. However, interest for March 2022 quarter has not yet been paid by Mr. Jagdish., He had a house in Jaipur which was sold in May 2017. In respect of this house he received arear of rent of, `96,000 in Feb. 2022 (not taxed earlier)., He also derived some other incomes during F.Y. 2021-22 which are as follows., Profit from business in Thailand `2,75,000, Interest on bonds of a Japanese Co. `45,000 out of which 50% was received in India., Income from Apple Orchid in Nepal given on contract and the yearly contract fee of `5,00,000, for F.Y., 2021-22 was deposited directly by the contractor in Kathmandu branch of Union Bank of India in Mr., Jagdish’s bank account maintained with Union Bank of India’s Pune Branch., Compute the total income of Mr. Jagdish for Assessment Year 2022-23 chargeable to income tax in India., Question.4. (a), (5 Marks), Mr. Mahadev, a noted bhajan singer of Rajasthan and his wife Mrs. Dariya furnish the following, information relating to the Assessment Year 2022-23., `, , 1, , Income of Mr. Mahadev- professional bhajan singer (computed), , 5,65,000, , 2, , Income under the head salary of Mrs. Dariya (computed), , 3,80,000, , 3, , Loan received by Mrs. Dariya from Ramu & Jay (Pvt) Ltd. (Mrs. Dariya holds 35% shares of 2,50,000, the Co. has incurred losses since its inception 2 years back), , 4, , Income of their minor son Golu winning signing reality show on T.V., , 5, , Cash gift received by Golu from friend of Mr. Mahadev on winning the show, , 21,000, , 6, , Interest income received by minor married daughter Gudia from deposit with Ramu & Jay, Pvt Ltd., , 40,000, , 2,50,000
Page 331 :
Income Under The Head Other Sources, , 331, , Compute total taxable income of Mr. Mahadev & Mrs. Dariya for the Assessment Year 2022-23., , MAY – 2018, Question 3(b), (3 Marks), XYZ Ltd. A domestic company, declared dividend of `170 lakh for the Financial Year 2020-21 and, distributed the same on 31-07-2021. Mr. A holding 10% share in XYZ Ltd. received dividend of `17 lakh in, July, 2021. Mr. B holding 5 % share in XYZ Ltd. received dividend of `8.5 lakh in July 2021., Discuss the tax liabilities in the hands of Mr. A and Mr. B assuming that Mr. A and Mr. B have not received, dividend from any other domestic company during the year., , NOV – 2016, Question 4(a), (2 Marks), State with reasons whether the following receipts are taxable or not under the provisions of Income-tax Act,, 1961?, Mr. Suman received an advance of `3 lakhs on 06-06-2021 to transfer his residential house property. Since, the transfer was not effected during the previous year due to failure in negotiations, he deducted the advance, money forfeited from the cost of acquisition of the property., , MAY – 2016, Question 4(a), (2 x 2 = 4 Marks), Discuss the taxability or otherwise in the hands of the recipients, as per the provisions of the Income-tax, Act, 1961:, (i) ABC Private Limited, a closely held company, issued 10,000 share at `130 per share. (The face, value of the share is `100 per share and the fair market value of the share is `120 per share)., (ii) Mr. A received an advance of `50,000 on 01.09.2021 against the sale of his house. However, due to, non-payment of instalment in time, the contract has cancelled and the amount of `50,000 was, forfeited., Question 7(a)(iii), (2 Marks), Discuss with reason, whether the following transactions are true or false, as per the provisions of Income, Tax Act, 1961:, Dividend received by a dealer in shares or one engaged in buying/selling of shares, is chargeable under the, head “Income from other sources”. (Discussion must be on the head of income)., , NOV – 2010, Question 7, State under which heads the following incomes are taxable:, (i) Rental income in case of a dealer engaged in business of letting out of house property, (ii) Dividend on shares in case of a dealer in shares, (iii) Salary by a partner from his partnership firm, (iv) Rental income of machinery, (v) Winnings from lotteries by a person having the same as business activity, (vi) Salaries payable to a Member of Parliament, (vii) Receipts without consideration, , (4 Marks)
Page 332 :
Deduction Of Tax At Source, , 332, , DEDUCTION OF TAX AT SOURCE, SECTION 190 TO 206AA, •, •, •, •, •, •, •, •, •, •, •, •, •, •, •, •, •, •, •, •, •, •, •, •, •, •, •, •, •, •, •, •, •, •, •, •, •, •, , Section 190 – Deduction at source and advance payment, Section 191 – Direct payment, Section 192 – Deduction of tax from salary income, Section 192A – Deduction of tax at source in case of payment from recognized provident fund, Section 193 – Deduction of tax from interest on securities, Section 194- Deduction of Tax in case of Dividends, Section 194A – Deduction of tax from interest other than “interest on securities”, Section 194B – Deduction of tax from winnings from lottery, crossword puzzle, card game, etc., Section 194BB – Deduction of tax from winnings from horse races, Section 194C – Deduction of tax from payment to contractors (V. Imp.), Section 194D – Deduction of tax from insurance commission, Section 194DA – Payment in respect of life insurance policy, Section 194E – Payments to non-resident sportsmen or sports associations, Section 194EE – Payments in respect of deposits under National Savings Scheme, etc, Section 194G – Deduction of tax from commission, etc. on sale of lottery tickets, Section 194H – Commission or brokerage, Section 194-I – Deduction of tax from rent (V. Imp.), Section 194-IA – Payment for purchase of immovable property, Section 194-IB – Payment of rent by certain individuals or HUF., Section 194-IC – Payment under specified agreement., Section 194J – Deduction of tax from fees for professional or technical services, Section 194K- Deduction of tax in case of Income on units of Mutual Fund, Section 194LA – Payment of compensation on acquisition of certain immovable property, Section 194M – Payment of certain sums by certain individuals or Hindu undivided family, Section 194N – Payment of certain amounts in cash, Section 194-O- TDS in case of Payment of certain sums by e-commerce operator to ecommerce participant, Section 194P – Deduction of tax in case of specified senior citizen, Section 194Q – Deduction of tax at source on payment of certain sum for purchase of goods, Section 195 – Payments to non-residents, Section 197 – Certificate for deduction at lower rate (V. Imp.), Section 197A – Declaration for not Deducting Tax at Source, Section 200 – Duty of person deducting tax (V. Imp.), Section 201 – Consequences of failure to deduct or pay (V. Imp.), Section 203 – Certificate for tax deducted, Section 203A – Tax deduction account number, Section 206AA – Requirements to furnish Permanent Account Number, Section 206AB – Special provision for deduction of tax at source for non-filers of income-tax, return., Section 206C – Collection of Tax Source
Page 333 :
Deduction Of Tax At Source, , 333, , Deduction at Source and Advance Payment Section 190, Every person shall be required to pay tax through TDS/TCS and advance tax and exact tax shall be, computed in the assessment year and balance if any shall be paid in the assessment year and it is called self, assessment tax., Direct Payment Section 191, If tax is not to be deducted at source with regard to any income, assessee shall pay advance tax. Similarly if, tax was to be deducted at source but it has not been deducted at source, in such cases also the assessee is, required to pay advance tax., Question 1: Write a note on Deduction of Tax at Source with regard to Salary Income., Answer: Deduction of Tax at Source with regard to Salary Income Section 192, 1. Every person (including individual and HUF even if limit prescribed under section 44AB has not, exceeded in the preceding year) making payment of salary income to resident or non-resident shall deduct, tax at source and for this purpose the employer shall estimate tax liability of the employee and tax so, estimated shall be deducted in 12 monthly equal installments. While estimating tax liability, deduction under, section 80C to 80U shall be allowed. It can be shown in the manner given below:, Mr. X is employed in ABC Ltd. and salary is `60,000 p.m. and he has invested `50,000 in NSC. In this case,, tax to be deducted at source at the time of payment of salary shall be:, `, Gross Salary (60,000 x 12), 7,20,000.00, Less: Standard Deduction u/s 16(ia), (50,000.00), Income under the head Salary, 6,70,000.00, Gross Total Income, 6,70,000.00, Less: Deduction u/s 80C {NSC}, (50,000.00), Total Income, 6,20,000.00, Tax on `6,20,000 at slab rate, 36,500.00, Add: HEC @ 4%, 1,460.00, Tax Liability, 37,960.00, Monthly installment shall be 37,960 / 12, 3,163.33, If employer has deducted tax at source for the month of April and May and salary was increased to `70,000, p.m. w.e.f. 01.06.2021, tax to be deducted in subsequent installments shall be, Gross Salary (60,000 x 2) + (70,000 x 10), 8,20,000.00, Less: Standard Deduction u/s 16(ia), (50,000.00), Income under the head Salary, 7,70,000.00, Gross Total Income, 7,70,000.00, Less: Deduction u/s 80C {NSC}, (50,000.00), Total Income, 7,20,000.00, Tax at slab rate including HEC, 58,760.00, Tax deducted at source in April and May (3,163.33 x 2), (6,326.66), Balance amount of tax, 52,433.34, Tax to be deducted in subsequent installments (52,433.34 / 10), 5,243.33, , Rule 26C requires furnishing of evidence of the following claims by an employee to the person responsible, for making payment in Form No.12BB for the purpose of estimating his income or computing the tax, deduction of tax at source:, S. No., Nature of Claim, Evidence or particulars, 1., , House Rent Allowance, , Name, address and PAN of the landlord(s) where the, aggregate rent paid during the previous year exceeds, ₹1 lakh., , 2., , Leave Travel Concession or Assistance, , Evidence of expenditure, , 3., , Deduction of interest under the head, , Name, address and PAN of the lender
Page 334 :
Deduction Of Tax At Source, , 334, , “Income from house property”, 4., , Deduction under Chapter VI-A, , Evidence of investment or expenditure., , 2. If any person is working with two or more employers, in that case he should submit the particulars of his, salary income from all the employers to one of the employer who will deduct tax at source taking into, consideration income from all employers. (Information has to be given in Form 12B), Example, Mr. X is working with two employer A Ltd. and B Ltd. and is getting basic pay of `30,000 p.m. from each, of the employer. In this case, he must inform one of the employer regarding his salary income from other, employer and such employer shall deduct tax at source taking into consideration income from other, employer., 3. If any employee has income under any other head, the employee shall be allowed even to report such, incomes to the employer and the employer shall take it into consideration. If employee has loss under the, head house property, he shall be allowed to report such loss to the employer. The employee shall be required, to give proof., 4. For taxability of Salary under new concessional rates (Section 115BAC), employee to intimate, employer of intention to opt for new concessional rates. Intimation so given will be applicable for the year, & can't be modified., Question 2: Write a note on deduction of tax at source in case of payment from recognized provident, fund., Answer: Deduction of tax at source in case of payment from recognized provident fund Section 192A, The person responsible for making payment of recognized provident fund to any person shall deduct tax, at source if the amount to be paid is taxable and tax shall be deducted at source @ 10% provided the amount, paid or payable during a particular year is `50,000 or more. If any person has failed to give permanent, account number, tax shall be deducted at source at maximum marginal rate that is 30% + Surcharge 37% +, HEC 4%., (29C) "maximum marginal rate" means the rate of income-tax (including surcharge on income-tax, if any), applicable in relation to the highest slab of income in the case of an individual [,association of persons or, as the, case may be, body of individuals] as specified in the Finance Act of the relevant year., , Payments received from recognised provident fund shall be exempt from income tax if the employee has, complied with any of the conditions given below:, (i) If the employee has rendered continuous service for a period of 5 years or more, or, (ii) If he has not rendered such continuous service, the service has been terminated by reason of the, employee’s ill-health, or by the contraction or discontinuance of the employer’s business or other cause, beyond the control of the employee, or, (iii) If the employee obtains employment with any other employer and the provident fund has been, transferred to such employer and the total service with the former employer and the current employer is, of 5 years or more., If the employee has not complied with even a single condition, in that case amount received by him shall be, taxable but only the amount which was exempt earlier., (To be discussed under the head Salary), Question 3: Write a note on TDS in case of interest on securities., Answer: TDS in case of interest on securities Section 193, Every person responsible for making payment of interest on securities to any resident shall deduct tax at, source @ 10%., No tax shall be deducted at source in the following cases:, 1. In case of, any interest payable to an individual or a Hindu undivided family, who is resident in India, on any, debenture issued by a company in which the public are substantially interested, if the aggregate amount of such
Page 335 :
Deduction Of Tax At Source, , 335, , interest paid or likely to be paid on such debenture during the financial year by the company to such individual or, Hindu undivided family does not exceed five thousand rupees; and such interest is paid by the company by an, account payee cheque;], 2. Any interest payable on any security of the Central Government or a State Government., 3. Any interest being paid to Bank/LIC or other notified financial organizations., 4. Interest payable by a company in connection with security held in dematerialised form., “Interest on securities” as per section 2(28B), interest on securities means interest on bond / debenture etc., issued by Government / local authority / company or statutory corporation etc., Example, (i) ABC Ltd. has to pay interest of `2,00,000 to Mr. X in connection with listed debentures, amount of TDS, shall be ` 20,000., (ii) ABC Ltd. has to pay interest of `12,00,000 to Mr. X in connection with listed debentures, amount of, TDS shall be `1,20,000, (iii) ABC Ltd. has to pay interest of `5,000 to Mr. X, no tax shall be deducted at source in this case., , Illustration 1: Mr. X has invested some amount in ABC Ltd. and the company has paid him interest of, `5,40,000 after deducting tax at source @ 10%. The cheque was collected by the bank and the bank charges, were 1%. He has income under the head house property ₹ 10,00,000, Compute his tax liability and tax refund for assessment year 2022-23., Solution:, `, Income under the head House Property, 10,00,000.00, Gross interest (5,40,000 x 100 /90), 6,00,000.00, Less: bank charges u/s 57 (1% of 5,40,000), (5,400.00), Income under the head Other Sources, 5,94,600.00, Gross Total Income, 15,94,600.00, Less: Deductions u/s 80C to 80U, Nil, Total Income, 15,94,600.00, Computation of Tax Liability, Tax on `15,94,600 at slab rate, 2,90,880.00, Add: HEC 4%, 11,635.20, Tax Liability, 3,02,515.20, Less: TDS, (60,000.00), Tax Payable, 2,42,515.20, Rounded off u/s 288B, 2,42,520.00, Illustration 2: Mr. X has invested some amount in ABC Ltd. and the company has paid him interest of, `2,70,000 after deducting tax at source @ 10%. The cheque was collected by the bank and the bank charges, were 1%., Compute his tax liability and tax refund for assessment year 2022-23., Solution:, `, Gross interest (2,70,000 x 100 /90), 3,00,000.00, Less: bank charges u/s 57 (1% of 2,70,000), (2,700.00), Income under the head Other Sources, 2,97,300.00, Total Income, 2,97,300.00, Computation of Tax Liability, Tax on `2,97,300 at slab rate, 2,365.00, Less: Rebate u/s 87A, (2,365.00), Tax Liability, Nil, Less: TDS, (30,000.00), Refund, 30,000.00, Assesse can take benefit of section 197 (not 197A)
Page 336 :
Deduction Of Tax At Source, , 336, , Illustration 3: Mr. X has invested some amount in ABC Ltd. and the company has paid him interest of, `1,80,000 after deducting tax at source @ 10%. The cheque was collected by the bank and the bank charges, were 1%., Compute his tax liability and tax refund for assessment year 2022-23., Solution:, `, Gross interest (1,80,000 x 100 /90), 2,00,000.00, Less: bank charges u/s 57 (1% of 1,80,000), (1,800.00), Income under the head Other Sources, 1,98,200.00, Total Income, 1,98,200.00, Computation of Tax Liability, Tax Liability, Nil, Less: TDS, (20,000.00), Refund, 20,000.00, In this case assesse can avail benefit of section 197 or 197A, Illustration 4: Mr. X has borrowed `1,00,000 from the market. The amount was invested in security of, some company and the assessee has received a cheque for ` 45,000 (after TDS @ 10%) being the amount of, interest and assessee has paid interest of ` 11,000. He has casual income ₹ 2,00,000, The cheque was given for collection to a bank and the bank has deducted collection charges of 2%., Mr. X has income under the head house property ` 2,50,000., Compute his tax liability / tax payable for assessment year 2022-23., Solution:, `, Income under the head House Property, 2,50,000.00, Income under the head other sources, Interest income, 38,100.00, (45,000 x 100 / 90) -11,000-900}, (50,000 – 11,000 – 900), Casual income, 2,00,000.00, Income under the head other sources, 2,38,100.00, Gross Total Income, 4,88,100.00, Less: Deduction u/s 80C to 80U, Nil, Total Income, 4,88,100.00, Computation of Tax Liability, Tax on Casual Income 2,00,000 X 30%, 60,000.00, Tax on `2,88,100 at slab rate, 1,905.00, Total Income, 61,905.00, Less: Rebate u/s 87A, (12,500.00), Tax Liability, 49,405.00, Less: TDS, (5,000.00), Tax Payable, 44,405.00, Rounded off u/s 288B, 44,410.00, Question 4: Write a note on TDS in case of “Dividends”., Answer: TDS in case of “Dividends” Section 194, Every company making payment of dividends to any resident shareholder shall deduct tax at source @ 10%,, however, no tax shall be deducted at source if the following conditions have been satisfied:, (i) dividend has been paid to an individual, and, (ii) payment is made by any mode other than cash, and, (iii) the amount being paid or payable during a particular year to such individual does not exceed, `5,000., , No tax shall be deducted at source in case of payment of dividend to Life Insurance Corporation of India,, General Insurance Corporation of India, or any other insurer
Page 337 :
Deduction Of Tax At Source, , 337, , Question 5: Write a note on TDS in case of Interest other than “Interest on Securities”., Answer: TDS in case of Interest other than “Interest on Securities” Section 194A, Every person making payment of interest other than interest on securities to any resident shall deduct tax at, source @ 10% provided the amount being paid or payable during a particular year to a particular person is, exceeding `5,000 but if payment is being made by bank or post office or Co-Operative Society, tax shall be, deducted only if interest being paid or payable is exceeding `40,000, however if the payee is senior citizen,, `40,000 shall be taken as `50,000 ., Further TDS shall be only on time deposit including recurring deposit. Limit of `40,000 (`50,000 for senior, citizen) shall be per branch of the bank but if the bank has core banking solution, limit shall be per bank and, not per branch., An Individual or Hindu Undivided Family shall be required to deduct tax at source only if the turnover in, case of business has exceeded ` 1 crore and gross receipts in case of profession has exceeded ` 50 lakhs,, during the financial year immediately preceding the relevant year., A co-operative society shall be required to deduct the TDS, if the total sales, gross receipt or turnover of, the Cooperative Society exceeds ` 50 crore during the financial year immediately preceding the relevant, year., Example, (i) Punjab National Bank has to pay interest of `1,00,000 to Mr. X. In this case, amount of TDS shall be, `10,000., (ii) Punjab National Bank has to pay interest of `10,00,000 to Mr. X. In this case, amount of TDS shall be, `1,00,000., (iii) Punjab National Bank has to pay interest of `1,00,000 to X Ltd. In this case, amount of TDS shall be, `10,000., (iv) Punjab National Bank has to pay interest of `1,00,000 to an Hindu Undivided Family. In this case,, amount of TDS shall be `10,000., (v) Punjab National Bank has to pay interest of `1,000 to a Hindu Undivided Family. In this case, amount of, TDS shall be Nil., (vi) Punjab National Bank has to pay interest of `39,900 to Mr. X. In this case, amount of TDS shall be Nil., No tax shall be deducted at source in the following cases:, (1) Interest paid by a firm to a partner of the firm;, (2) Any interest being paid to Bank/LIC or other notified financial organizations, (3) Interest on income tax refund or wealth tax refund etc., (4) Income paid in relation to a Zero Coupon Bond., (5) Interest paid in respect of deposits under any scheme notified by the government., "Zero Coupon Bond" Section 2(48), means a bond which are issued by the specified companies and which are issued for minimum 10 years and, maximum 20 years and in respect of which no payment and benefit is received before maturity or, redemption from such specified company and further such bonds shall be notified by the Central, Government., Additional amount received on redemption shall be considered to be capital gain., “Interest” Section 2(28A) means interest payable in any manner in respect of any moneys borrowed or debt, incurred (including a deposit, claim or other similar right or obligation) and includes any service fee or other, charge in respect of the moneys borrowed or debt incurred or in respect of any credit facility which has not been, utilized, , Illustration 5: Mrs. Nupur Sharma is getting a family pension of `7,000 p.m. She has also received interest, on fixed deposit of `90,000 after deducting tax at source of `10,000. The bank has deducted collection, charges @ 1.5%. She has short term capital gains under section 111A ` 4,00,000. Deductions under section, 80C to 80U ` 67,650, Compute her tax liability for assessment year 2022-23., Solution:, `, `, Family Pension, 84,000
Page 338 :
Deduction Of Tax At Source, (7,000 x 12), Less: Deduction u/s 57, 1/3 of `84,000 or `15,000 whichever is less, Interest, Less: Bank Charges [90,000 @ 1.5 %], Income under the head Other Sources, Income under the capital gains (STCG 111A), Gross Total Income, Less: Deductions 80C to 80U, Total Income, Computation of tax liability, Tax on Normal Income ` 1,00,000 at slab rate, Tax on STCG 111A (4,00,000 – 1,50,000) X 15%, Less: Rebate 87A, Tax before HEC, Add: HEC @ 4%, Tax Liability, , 338, (15,000), 1,00,000, (1,350), , 69,000, 98,650, 1,67,650, 4,00,000, 5,67,650, (67,650), 5,00,000, Nil, 37,500, (12,500), 25,000, 1,000, 26,000, , Illustration 6 (From RTP): Examine the implications of tax deduction at source under section 194A in the, cases mentioned hereunder, based on the provisions of the Income-tax Act, 1961., (i) On 01.10.2021, Mr. Mohit made a six-month fixed deposit of ` 12 lakh @ 8% p.a. with Theta Cooperative Bank. The fixed deposit matures on 31.3.2022., (ii) Mr. Harish made fixed deposits carrying interest @10% p.a. with the following branches of Omega, Bank, a bank which has adopted CBS., Branch, Amount (`), Date of deposit, Date of Maturity, Adyar, 60,000, 01.06.2021, 31.03.2022, Anna Nagar, 80,000, 01.07.2021, 31.03.2022, Nungambakkam, 75,000, 01.08.2021, 31.03.2022, (iii) On 01.04.2021, Ms. Meena started a 1 year recurring deposit of ` 50,000 per month @ 10% p.a. with, Gamma Bank. The recurring deposit matures on 31.3.2022. Gamma bank pays interest of `43,000., Solution:, (i) Theta Co-operative Bank has to deduct tax at source @ 10% on the interest of ` 48,000 (8% × ` 12 lakh, × ½) under section 194A., (ii) Since Omega Bank has adopted CBS, the aggregate interest credited/paid by all branches has to be, considered, and if the same exceeds ` 40,000, tax is deductible under section 194A. Omega Bank is not, required to deduct tax at source @10% under section 194A, since the aggregate interest on fixed deposit, with the three branches of the bank is ` 16,000, which does not exceed the threshold limit of ` 40,000., Branch, Amount of, Rate of, Period in, Amount of, Interest, months, Interest (`), deposit (`), Adyar, 60,000, 10%, 10, 5,000, Anna Nagar, 80,000, 10%, 9, 6,000, Nungambakkam, 75,000, 10%, 8, 5,000, Total, 16,000, (iii) Tax has to be deducted @ 10% under section 194A by Gamma Bank on the interest of `43,000 on, recurring deposit on 31.3.2022 to Ms. Meena, since –, (1) ―recurring deposit has been included in the definition of “time deposit”; and, (2) such interest exceeds the threshold limit of ` 40,000., Illustration 7 (From RTP): Examine the TDS implications under section 194A in the cases mentioned, hereunder –, (i) On 1.10.2021, Mr. Harish made a six-month fixed deposit of ` 10 lakh @ 9% p.a. with ABC Cooperative Bank. The fixed deposit matures on 31.3.2022.
Page 339 :
Deduction Of Tax At Source, , 339, , (ii) On 01.06.2021, Mr. Ganesh made three nine month fixed deposits of ` 2 lakh each carrying interest @, 9% per annum with Dwarka Branch, Janakpuri Branch and Rohini Branches of XYZ Bank, a bank which, has adopted CBS. The fixed deposits mature on 28.2.2022., (iii) On 01.04.2021, Mr. Rajesh started a 1 year recurring deposit of ` 60,000 per month @ 8% p.a. with, PQR Bank. The recurring deposit matures on 31.03.2022. PQR bank pays interest of `50,400., Answer:, (i) ABC Co-operative Bank has to deduct tax at source @10% on the interest of ` 45,000 (9% × ` 10 lakh ×, ½) under section 194A. The tax deductible at source under section 194A from such interest is, therefore,, `4,500., (ii) XYZ Bank has to deduct tax at source@10% under section 194A, since the aggregate interest on fixed, deposit with the three branches of the bank is ` 40,500 [2,00,000 × 3 × 9% × 9/12], which exceeds the, threshold limit of ` 40,000. Since XYZ Bank has adopted CBS, the aggregate interest credited/paid by all, branches has to be considered. Since the aggregate interest of ` 40,500 exceeds the threshold limit of `, 40,000, tax has to be deducted @ 10% under section 194A., (iii) Tax has to be deducted under section 194A @ 10% by PQR Bank on the interest of `50,400 on, recurring deposit on 31.03.2022 to Mr. Rajesh, since –, (1) “recurring deposit” has been included in the definition of “time deposit”; and, (2) such interest exceeds the threshold limit of ` 40,000., Question 6: Write a note on TDS in case of Winnings from Lottery or Crossword Puzzle etc., Answer: TDS in case of Winnings from Lottery or Crossword Puzzle etc. Section 194B, Every person (including individual and HUF) responsible for paying to any resident or non-resident, any, income by way of winnings from any lottery or crossword puzzle or card game and other game of any, sort shall deduct tax at source @ 30% provided the amount being paid or payable is exceeding `10,000 e.g., If ABC Ltd. has to pay `7,000 being winning of a lottery, no tax shall be deducted at source but if amount, being paid is `10 lakh, tax to be deducted at source shall be `10,00,000 x 30% = `3,00,000, If any such winning is in kind, winning shall be released only after collecting the amount of tax e.g. Mr. X, has won a motor car valued `5,00,000, in this case the organizer should collect tax of `1,50,000 and only, after that motor car shall be released., Question 7: Write a note on TDS in case of Winnings from Horse Race., Answer: TDS in case of Winnings from Horse Race Section 194BB, Every person (including individual and HUF), responsible for paying to any resident or non-resident, shall, be required to deduct tax at source @ 30% in case of payment of winning from horse races but tax shall be, deducted at source only if amount paid or payable during a particular year to a particular person is exceeding, `10,000., Example, ABC Ltd. has to pay winnings of horse race `3,00,000 to Mr. X, amount of TDS shall be `90,000 but if the, amount to be paid is `2,000, amount of TDS shall be Nil., Illustration 8: Mr. X purchased 20 lottery tickets of ` 250 each with a winning of `2.80 lakhs (after TDS, @ 30%). He has also received interest of `72,000 after deducting tax at source @ 10% in connection with, fixed deposit. and the cheque was collected by bank and service charges @ 2% was taken by the bank. He, has income from subletting of house property `9,000 p.m. He has received family pension of ` 4,000 p.m., Compute his tax liability and also tax payable/refund for assessment year 2022-23., Solution:, `, `, Income from lottery (2,80,000 /70% x 100%), 4,00,000.00, Interest (72,000 / 90% X 100%), 80,000.00, (1,440), 78,560.00, Less: Bank charges u/s 57 (2% of `72,000), Sub-letting of house property (9,000 x 12), 1,08,000.00, Family pension, 48,000, Less: 1/3 of income or `15,000 whichever is less, (15,000), 33,000.00, Income under the head Other Sources, 6,19,560.00
Page 340 :
Deduction Of Tax At Source, Computation of Tax Liability, Tax on casual income `4,00,000 @ 30% u/s 115BB, Tax on `2,19,560 at slab rate, Tax before health & education cess, Add: HEC @ 4%, Tax Liability, Less: TDS (1,20,000 + 8,000), Tax Refund, , 340, , 1,20,000.00, Nil, 1,20,000.00, 4,800.00, 1,24,800.00, (1,28,000.00), 3,200.00, , Question 8: Write a note on TDS in case of Payments to Contractors., Answer: TDS in case of Payments to Contractors Section 194C, 1. Every person responsible for making payment to a resident contractor in connection with any work shall, deduct tax at source @ 2% and in case of payment to individual or Hindu Undivided Family, the rate of, TDS shall be 1%. Tax shall be deducted at source only if the amount being paid is exceeding `30,000 or the, amount paid or payable during a particular financial year to a particular person exceeds `1,00,000., Example, ABC Ltd. makes the following payments to Mr. X, a contractor, for contract work during the P.Y.2021-22 –, ` 15,000 on 01.05.2021, ` 25,000 on 01.08.2021, ` 30,000 on 01.12.2021, On 01.03.2022, a payment of ` 48,000 is due to Mr. X on account of a contract work., Discuss whether ABC Ltd. is liable to deduct tax at source under section 194C from payments made to Mr., X., Solution:, In this case, the individual contract payments made to Mr. X does not exceed ` 30,000. However, since the, aggregate amount paid to Mr. X during the P.Y.2021-22 exceeds ` 1,00,000 (on account of the last payment, of `48,000, due on 01.03.2022, taking the total from ` 70,000 to ` 1,18,000), the TDS provisions under, section 194C would get attracted. Tax has to be deducted @ 1% on the entire amount of 1,18,000 from the, last payment of ` 48,000 and the balance of ` 46,820 (i.e. ` 48,000 – ` 1,180) has to be paid to Mr. X., Example, (i) If DDA has to pay a sum of `5,00,000 to Mr. X in connection with a particular contract, amount of TDS, shall be `5,000., (ii) If in the above case amount is to be paid to X Ltd. An Indian company, amount of TDS shall be `10,000., 2. An Individual or Hindu Undivided Family shall be required to deduct tax at source only if the turnover, in case of business has exceeded ` 1 crore and gross receipts in case of profession has exceeded ` 50, lakhs, during the financial year immediately preceding the relevant year., Example, If Mr. X is engaged in a business and turnover of business is `41,00,000 in the previous year 2020-21 and he, has to pay `1,10,000 to Mr. Y in the previous year 2021-22 in connection with a contract, amount of TDS, shall be Nil but if his turnover in previous year 2020-21 was `110,00,000, amount of TDS shall be `1,100, but if payment is to given to Y Ltd., amount of TDS shall be `2,200., 3. No individual or HUF shall deduct tax at source under this section, if the amount is paid for personal, purpose of such individual or HUF., Example, If in the above case, Mr. X has to pay `1,10,000 to Mr. Y in connection with a contract which is for personal, purpose of Mr. X, TDS under section 194C, shall be Nil., 4. Contract for this purpose shall include every type of contract e.g. Advertising contract/Broadcasting and, telecasting contract / Carriage of passenger by any mode of transport / Catering contract / Contract for, construction / Contract for courier services / Contract of maintenance of plant and machinery etc., 5. If any person making payment for purchase of goods, no tax shall be deducted at source but TDS shall be, applicable in case of manufacturing or supplying a product according to the requirement or specification of a
Page 341 :
Deduction Of Tax At Source, , 341, , customer by using material purchased from such customer or its associate, i.e. a person covered u/s, 40A(2)(b), but does not include manufacturing or supplying a product according to the requirement or, specification of a customer by using material purchased from a person, other than such customer or associate, of such customer., Further tax shall be deducted at source on the invoice value excluding the value of material, if such value is, mention separately in the invoice. If value is not mention separately, tax shall be deducted at source on, whole of the invoice value., Example, ABC Ltd. has given orders to Mr. X to stitch uniform for their employees and Mr. X purchased material, from the market and has stitched uniform for ABC Ltd. and has charged `7,00,000, in this case amount of, TDS shall be nil but if material is supplied by ABC Ltd. or its associates and Mr. X has charged `1,10,000, as labour charge, tax shall be deducted at source @ 1% i.e. `1,100. If value of material and amount for, labour is not shown separately, tax shall be deducted at source on the entire amount., 6. No tax shall be deducted at source in case of payment to a contractor in connection with transportation, of goods where such contractor do not own more than 10 goods carriages at any time during the year and, also submitted a declaration in this regard and has also furnished permanent account number., Example, ABC Ltd. has paid `5,00,000 to Mr. X for transportation of goods and Mr. X do not have more than 10, goods carriages and he has furnished a declaration in this regard and has submitted permanent account, number, in this case no tax shall be deducted at source but if PAN has not been provided, tax shall be, deducted at source @ 20%., TDS provisions on payments by television channels and publishing houses to advertisement companies, for procuring or canvassing for advertisements [Circular No. 05/2016, dated 29-2-2016], , There are two types of payments involved in the advertising business:, (i) Payment by client to the advertising agency, and, (ii) Payment by advertising agency to the television channel/newspaper company, The applicability of TDS on these payments has been dealt with in Circular No. 715 dated 8-8-1995, where, it has been clarified that while TDS under section 194C (as work contract) will be applicable on the first, type of payment, there will be no TDS under section 194C on the second type of payment e.g. payment by, advertising agency to the media company., However, another issue has been raised in various cases as to whether the fees/charges taken or retained by, advertising companies from media companies for canvasing/booking advertisements (typically 15% of the, billing) is 'commission' or 'discount' for attracting the provisions of section 194H., The CBDT has clarified that no TDS is attracted on payments made by television channels/newspaper, companies to the advertising agency for booking or procuring of or canvassing for advertisements., TDS provisions on payments by broadcasters or Television Channels to production houses for, production of content or programme for telecasting [Circular No. 04/2016, dated 29-2-2016], The issue under consideration is whether payments made by the broadcaster/telecaster to production houses, for production of content/programme are payments under a ‘work contract’ liable for tax deduction at source, under section 194C or a contract for ‘professional or technical services’ liable for tax deduction at source, under section 194J., , In this regard, the CBDT has clarified that while applying the relevant provisions of TDS on a contract for, content production, a distinction is required to be made between:, (i) a payment for production of content/programme as per the specifications of the broadcaster/telecaster;, and, (ii) a payment for acquisition of broadcasting/ telecasting rights of the content already produced by the, production house., In the first situation where the content is produced as per the specifications provided by the broadcaster/
Page 342 :
Deduction Of Tax At Source, , 342, , telecaster and the copyright of the content/programme also gets transferred to the telecaster/ broadcaster,, such contract is covered by the definition of the term work’ in section 194C and, therefore, subject to TDS, under that section., However, in a case where the telecaster/broadcaster acquires only the telecasting/ broadcasting rights of the, content already produced by the production house, there is no contract for “carrying out any work”, as, required in section 194C. Therefore, such payments are not liable for TDS under section 194C. However,, payments of this nature may be liable for TDS under other sections of Chapter XVII-B of the Act., ILLUSTRATION, Moon TV, a television channel, made payment of `50 lakhs to a production house for production of, programme for telecasting as per the specifications given by the channel. The copyright of the programme is, also transferred to Moon TV. Would such payment be liable for tax deduction at source under section 194C?, Discuss., , Also, examine whether the provisions of tax deduction at source under section 194C would be attracted if, the payment was made by Moon TV for acquisition of telecasting rights of the content already produced by, the production house., SOLUTION:, In this case, since the programme is produced by the production house as per the specifications given by, Moon TV, a television channel, and the copyright is also transferred to the television channel, the same falls, within the scope of definition of the term ‘work’ under section 194C. Therefore, the payment of `50 lakhs, made by Moon TV to the production house would be subject to tax deduction at source under section 194C., , If, however, the payment was made by Moon TV for acquisition of telecasting rights of the content already, produced by the production house, there is no contract for “carrying out any work”, as required in section, 194C . Therefore, such payment would not be liable for tax deduction at source under section 194C., Question 9: Write a note on TDS in case of payment of Insurance Commission., Answer: TDS in case of payment of Insurance Commission Section 194D, Every person responsible for making payment for insurance commission to a resident insurance agent shall, deduct tax at source @ 5% provided the amount paid or payable during a particular year to a particular agent, is exceeding `15,000 e.g. If LIC has to pay commission of `5,00,000 to one of the agent Mr. X, amount of, TDS shall be `5,00,000 x 5% = 25,000, Question 10: write a note on Payment on maturity of life insurance policy., Answer: Payment on maturity of life insurance policy Section 194DA, In general payment on maturity of Life policy is exempt from income tax under section 10(10D) however, sometimes the amount is taxable (if premium paid has exceeded the prescribed percentage (i.e. 10% / 15% /, 20%)) and in that case tax has to deducted at source @ 5% on the amount of income provided the amount, paid or payable to any resident during a particular financial year is `1,00,000 or more., Example, Examine the applicability of the provisions for tax deduction at source under section 194DA in the following, cases (i) Mr. X, a resident, is due to receive ` 4.50 lakhs on 31.03.2022, towards maturity proceeds of LIC policy, taken on 01.4.2019, for which the sum assured is ` 4 lakhs and the annual premium is ` 1,25,000., , (ii) Mr. Y, a resident, is due to receive ` 3.25 lakhs on 31.03.2022 on LIC policy taken on 31.03.2012, for, which the sum assured is ` 3 lakhs and the annual premium is `35,000., (iii) Mr. Z, a resident, is due to receive `95,000 on 01.8.2021 towards maturity proceeds of LIC policy taken, on 01.08.2013 for which the sum assured is ` 90,000 and the annual premium was ` 12,000., Answer, (i) Since the annual premium exceeds 10% of sum assured in respect of a policy taken on 01.04.2019, the, maturity proceeds of ` 4.50 lakhs are not exempt under section 10(10D) in the hands of Mr. X. Therefore,
Page 343 :
Deduction Of Tax At Source, , 343, , tax is required to be deducted @ 5% under section 194DA on the amount of income of `75,000 (`4,50,0003,75,000)., (ii) Since the annual premium is less than 20% of sum assured in respect of a policy taken before, 01.04.2012, the sum of `3.25 lakhs due to Mr. Y would be exempt under section 10(10D) in his hands., Hence, no tax is required to be deducted at source under section 194DA on such sum payable to Mr. Y., (iii) Even though the annual premium exceeds 10% of sum assured in respect of a policy taken after, 31.03.2012, and consequently, the maturity proceeds of `95,000 due on 01.08.2021 would not be exempt, under section 10(10D) in the hands of Mr. Z, the tax deduction provisions under section 194DA are not, attracted since the maturity proceeds are less than ` 1 lakh., Question 11: Write a note on Payments to non-resident sportsmen or sports association., Answer: Payments to non-resident sportsmen or sports association, Section 194E, Every person (including individual and HUF) responsible for making payment to a non-resident sportman /, athlete / entertainer shall deduct tax at source @ 20% plus HEC. Further such non-resident should not be a, citizen of India. The incomes should be such as are covered under section 115BBA i.e. the incomes listed, below:, (a) participation in any game or sport in India or, (b) advertisement; or, (c) Contribution of articles relating to any game or sport in India in newspapers, magazines or journals., Example: Madona, a non-resident, received ` 40 lakh for her stage shows in India from Optimistic Ltd., an, event management company in India, on 26.12.2021, in this case, payments made to a non-resident, entertainer, shall be subject to tax deduction @ 20% under the provisions of section 194E plus HEC @ 4%., Tax deductible under section 194E = ` 40 lakh × 20.8% = ` 8,32,000, Question 12: Write a note on Payments in respect of deposits under National Savings Scheme, etc, Answer: Payments in respect of deposits under National Savings Scheme, etc Section 194EE, The person responsible for paying to any person any amount u/s 80CCA(2)(a) in connection with National, Saving Scheme shall deduct tax at a rate of 10% provide the amount being paid during a financial year to a, particular person is ₹ 2,500 or more., , No TDS in case of payment to the legal heirs of the assessee., Investment u/s Section 80CCA has been discontinued w.e.f. 01.04.1992., Question 13: Write a note on TDS in case of Commission, etc., on the Sale of Lottery Tickets., Answer: TDS in case of Commission, etc., on the Sale of Lottery Tickets Section 194G, Every person (including individual and HUF) making payment of commission for sale of lottery tickets to, any person resident or non-resident, shall deduct tax at source @ 5% provided the amount paid or payable to, a particular person during a particular year is exceeding `15,000., Question 14: Write a note on TDS on payment of Commission or Brokerage., Answer: TDS on payment of Commission or Brokerage Section 194H, Every person making payment of any commission or brokerage to a resident shall, deduct income-tax at, the rate of 5%, provided amount paid or payable during a particular year to a particular person is exceeding, `15,000., An Individual or Hindu Undivided Family shall be required to deduct tax at source only if the turnover in, case of business has exceeded ` 1 crore and gross receipts in case of profession has exceeded ` 50 lakhs,, during the financial year immediately preceding the relevant year., Example, Manoj trading limited rendered services in relation to sale of mustard oil to Ashish oils limited and, commission charged is `7,00,000, in this case, tax to be deducted at source by Ashish oils limited shall be, 7,00,000 x 5% = `35,000 and amount payable to Manoj trading limited shall be, Commission, 7,00,000, Less: TDS(7,00,000 x 5%), (35,000), Amount Payable, 6,65,000
Page 344 :
Deduction Of Tax At Source, , 344, , Question 15: Write a note on TDS in case of payment of rent., Answer: TDS in case of payment of rent Section 194-I, Every person making payment of rent to a resident shall deduct tax at source provided the amount paid or, payable during a particular year is exceeding `2,40,000. Tax shall be deducted at source @ 2% if rent is for, plant and machinery but @ 10% if rent is for land/building / furniture / fixture etc., An Individual or Hindu Undivided Family shall be required to deduct tax at source only if the turnover in, case of business has exceeded ` 1 crore and gross receipts in case of profession has exceeded ` 50 lakhs,, during the financial year immediately preceding the relevant year., Illustration 9:, (i) Amount of TDS in the following cases shall be:, Person receiving the payment, Mr. A, Person making the payment, Individual, (not, exceeded the limit, in P.Y. 2020-21), , Mr. B, Mr. C, Individual, Partnership firm, (exceeded, the, limit in P.Y., 2020-21), `3,00,000, `3,00,000, `30,000, `30,000, `3,00,000, `3,00,000, `6,000, `6,000, `1,00,000, `1,00,000, Nil, Nil, `1,00,000, `1,00,000, Nil, Nil, , Mr. D, Company, (Indian, company), , Rent for house property, `3,00,000, `3,00,000, Amount of TDS, Nil, `30,000, Rent for plant and machinery, `3,00,000, `3,00,000, Amount of TDS, Nil, `6,000, Rent for house property, `1,00,000, `1,00,000, Amount of TDS, Nil, Nil, Rent for plant and machinery, `1,00,000, `1,00,000, Amount of TDS, Nil, Nil, Example 1., XYZ Ltd. raised an invoice of `3,00,000/- to ABC Limited for renting of commercial building. The above, figure includes `50,000/- of parking charges. The bill is raised on 30th June, 2021 and ABC Limited made, the payment on the same date., Compute the Amount of TDS required to be deducted by ABC Limited and the due date of deposit of TDS, amount and last date of filing of quarterly Statement?, Solution:, Rent, 3,00,000, Less: TDS(3,00,000 x 10%), (30,000), Amount payable, 2,70,000, Last date of deposit = 7th July, 2021., Last date of filing of quarterly Statement = 31st July, 2021., Example 2, ABC limited has let out one commercial building to Idea cellular limited at Gurgaon and rent charged is, `2,50,000 per month, in this case, tax to be deducted at source by Idea cellular limited shall be as given, below:, Solution:, Rent (2,50,000 x 12), 30,00,000, Less: TDS(30,00,000 x 10%), (3,00,000), Amount payable, 27,00,000, Question 16: Write a note on TDS in case of Payment for purchase of immovable property., Answer: TDS in case of Payment for purchase of immovable property Section 194-IA, 1. Every person (including individual and HUF) making payment to a resident for purchase of immovable, property of `50 lakhs or more shall deduct tax at source @ 1% of such sum., 2. Consideration for immovable property” shall include all charges of the nature of club membership fee,, car parking fee, electricity or water facility fee, maintenance fee, advance fee or any other charges of, similar nature, which are incidental to transfer of the immovable property.
Page 345 :
Deduction Of Tax At Source, , 345, , 3. No tax shall be deducted at source in case of payment for purchase of agricultural land which is, situated in the rural area., E.g. Mr. X has purchased one building for `65 lakhs, in this case amount of TDS shall be 65,00,000 x 1% =, `65,000 but if building was purchased for `47 lakhs, amount of TDS shall be nil., 4. The person deducting tax at source shall not be required to obtain Tax Deduction Account Number as per, section 203A., Example: Mr. X sold his house property in Chennai for a consideration of `75 lakh to Mr. Y on 31.01.2022,, in this case, Mr. Y is required to deduct tax at source under section 194-IA @ 1% of `75 lakh and tax, deductible under section 194-IA shall be ` 75 lakh × 1% = ` 75,000, Question 17: Write a note on TDS in case of Payment of Rent by Certain Individual and HUF., Answer: TDS in case of Payment of Rent by Certain Individual and HUF Section 194-IB, (1) Any person, being an individual or a Hindu undivided family not covered under section 194-I,, responsible for paying to a resident any income by way of rent exceeding `50,000 for a month or part of a, month during the previous year, shall deduct tax @ 5%., (2) Tax shall be deducted at the time of making payment of rent for the last month of the previous year or, the last month of tenancy whichever is earlier., (3) No requirement to take tax deduction account number., (4) If the person receiving payment of rent has not submitted PAN, tax shall be deducted @ 20% but, maximum rent payable for the last month., (5) If the person receiving payment of rent has not filed his/her Income Tax Returns for the both of the, two assessment years relevant to the two previous years which are immediately before the previous year in, which tax is required to be deducted or collected, then tax shall be deducted at the higher of the following, rates:, • twice the rate specified in the relevant provision of the Act; or, • twice the rate or rates in force; or, • the rate of five per cent, However, maximum TDS can not be more than the rent payable for the last month., Example: Mr. X has taken a house on rent `60,000 p.m. not required to deduct tax at source under section, 194-I, in this case he will be required to deduct tax at source @ 5% but tax is to be deducted in the last, month instead of every month. While paying rent of ` 60,000 for March 2022 he should deduct tax at source, ` 7,20,000 x 5% = 36,000 but if person receiving payment has not submitted PAN, amount of TDS shall be, 7,20,000 x 20% = 1,44,000 but maximum ` 60,000., Question 18: Write a note on TDS in case of Payment under specified agreement., Answer: TDS in case of Payment under specified agreement Section 194-IC, Notwithstanding anything contained in section 194-IA, any person responsible for paying to a resident any, sum by way of consideration, not being consideration in kind, under the agreement referred to in section 45, (5A), shall deduct tax @ 10%., Question 19: Write a note on TDS in case of Fees for Professional or Technical Services., Answer: TDS in case of Fees for Professional or Technical Services Section 194J, 1. Every person, who is responsible for paying to a resident any sum by way of –, (i) fees for Professional services, (ii) fees for Technical services, (iii) any Remuneration or fees or commission to a director of a company (in case salary is being paid to a, director, tax shall be deducted at source under section 192)., (iv) Royalty, (v) Non-compete fee referred to in section 28, shall deduct tax at source at the rate of 10%, however rate of TDS shall be 2% in the following cases, (i), in case of Fees for Technical Services., (ii), royalty where such royalty is in the nature of consideration for sale, distribution or exhibition of, cinematographic films
Page 346 :
Deduction Of Tax At Source, (iii), , 346, , in case of a payee engaged only in the business of operation of a call centre., , An Individual or Hindu Undivided Family shall be required to deduct tax at source only if the turnover in, case of business has exceeded ` 1 crore and gross receipts in case of profession has exceeded ` 50 lakhs,, during the financial year immediately preceding the relevant year., 2. No tax shall be deducted at source where the amount paid or payable during the year do not exceed, `30,000. (limit of `30,000 is applicable separately for each of the above payments). There is no such limit in, case of payment to a director i.e. tax has to be deducted at source in case of payment to a director, irrespective of the amount to be paid., Illustration 10: XYZ Ltd. makes a payment of `28,000 to Mr. X on 02.08.2021 towards fees for, professional services and another payment of `25,000 to him on the same date towards fees for technical, services. Discuss whether TDS provisions under section 194J are attracted., Solution:, TDS provisions under section 194J would not get attracted, since the limit of `30,000 is applicable for fees, for professional services and fees for technical services, separately. It is assumed that there is no other, payment to Mr. X towards fees for professional services and fees for technical services during the P.Y.202122., Example: X Ltd. paid retainership fees of `25,000 to its Director, Mr. Ram Sharma, on 30.01.2022, as per, section 194J, the company shall be liable to deduct tax at source @ 10% on any remuneration or fees or, commission paid to a director, on which the tax is not deductible under section 192. The limit of `30,000, under section 194J is not applicable on any remuneration or fees or commission payable to director of, a company., Tax deductible under section 194J = ` 25,000 x 10% = ` 2,500, Example, (i) If ABC Ltd. has to pay a sum of `2,00,000 to an architect, amount of TDS shall be `20,000., (ii) If ABC Ltd. has to pay `10,00,000 to a Chartered Accountant, amount of TDS shall be `1,00,000., (iii) If Mr. X has to pay `50,000 to an advocate, amount of TDS shall be Nil and if Turnover of Mr. X was, exceeded the prescribed limit during 2020-21, amount of TDS shall be `5,000., (iv) If Z Ltd. has to pay `15,000 in connection with technical services, amount of TDS shall be Nil., 3. If individual or HUF is making payment for professional services and it is for personal purpose, no tax, shall be deducted at source., 4. Meaning of “Professional services”, As per section 44AA, rule 6F, “Professional services” means:, , (a) Legal profession, (b) Medical Profession, (c) engineering Profession, (d) architectural profession, (e) profession of accountancy, (f) technical consultancy, (g) interior decoration, (h) advertising, (i) Profession of “authorised representatives”;, (j) Profession of “film artist”;, (k) Profession of “company secretary”;
Page 347 :
Deduction Of Tax At Source, , 347, , (l) Profession of “information technology”., The CBDT has notified the services also as professional services, , (a) Sports Persons,, (b) Umpires and Referees,, (c) Coaches and Trainers,, (d) Team Physicians and Physiotherapists,, (e) Event Managers,, (f) Commentators,, (g) Anchors and, (h) Sports Columnists., Accordingly, the requirement of TDS as per section 194J would apply to all the aforesaid professions. The, term “profession”, as such, is of a very wide import. However, the term has been defined in this section, exhaustively. For the purposes of TDS, therefore, all other professions would be outside the scope of section, 194J. For example, this section will not apply to professions of teaching, sculpture, painting etc. unless they, are notified., Meaning of “Fees for technical services”, , The term ‘fees for technical services’ means any consideration (including any lump sum consideration) for, rendering of any of the following services:, (i) Managerial services;, (ii) Technical services;, (iii) Consultancy services;, (iv) Provision of services of technical or other personnel., It is expressly provided that the term ‘fees for technical services’ will not include following types of, consideration:, (i) Consideration for any construction, assembly, mining or like project, or, (ii) Consideration which is chargeable under the head ‘Salaries’., Question 20: Write a note on TDS in case of “Income on units of Mutual Fund”., Answer: TDS in case of “Income on units of Mutual Fund” Section 194K, Any person making payment of Income on units of Mutual Fund to a resident shall deduct tax at source, @ 10%, provided the amount being paid or payable during a particular year to a particular person is, exceeding `5,000. Also TDS shall not be deducted, if the income is of the nature of capital gains., Question 21: Write a note on TDS in case of Payment of Compensation on Acquisition of certain, Immovable Property., Answer: TDS in case of Payment of Compensation on Acquisition of certain Immovable Property, Section 194LA, If any land or building has been acquired by the government or other similar agency, tax shall be deducted at, source @ 10% provided the amount paid or payable to any resident is exceeding `2,50,000. No tax shall be, deducted at source if the payment relates to acquisition of agricultural land., , No deduction shall be made under this section where such payment is made in respect of any award or, agreement which has been exempted from levy of income-tax under section 96 of the Right to Fair, Compensation and Transparency in Land Acquisition, Rehabilitation and Resettlement Act, 2013.
Page 348 :
Deduction Of Tax At Source, , 348, , Example: If `3,00,000 is to be paid to Mr. X on 05.05.2021 by State Government on compulsory, acquisition of his urban land, amount of TDS shall be 3,00,000 x 10% = 30,000., Question 22: Write a note on TDS in case of Payment of certain sums by certain individuals or Hindu, undivided family., Answer: TDS in case of Payment of certain sums by certain individuals or Hindu undivided family, Section 194M, Any person, being an individual or a Hindu undivided family (other than those who are required to deduct, income-tax as per the provisions of section 194C or section 194H or section 194J) responsible for paying, any sum to any resident for carrying out any work (including supply of labour for carrying out any work) in, pursuance of a contract or by way of fees for professional services during the financial year, shall, at the, time of credit of such sum or at the time of payment of such sum in cash or by issue of a cheque or draft or, by any other mode, whichever is earlier, deduct an amount equal to five per cent. of such sum as income -tax, thereon:, Provided that no such deduction under this section shall be made if such sum or, as the case may be,, aggregate of such sums, credited or paid to a resident during a financial year does not exceed fifty lakh, rupees., The provisions of section 203A shall not apply to a person required to deduct tax in accordance with the, provisions of this section., Illustration 11: Examine whether TDS provisions would be attracted in the following cases, and if so, under, which section. Also specify rate of TDS applicable in each case. Assume that all payments are made to, residents., Particulars of the payer, Nature of payment, Aggregate of payments, made in the F.Y.202122, 1. Mr. Ganesh, an individual carrying on Contract Payment for repair of `5 lakhs, retail business with turnover of ` 2.5 residential house, crores in the P.Y.2020-21, Payment of commission to Mr. `80,000, Vallish for business purposes, 2. Mr. Rajesh, a wholesale trader and Contract payment for reconstruction `20 lakhs in January,, turnover for P.Y.2020-21 is 95 lakhs of residential house (made during 2022, `15 lakhs in Feb, the period January-March, 2022), and for P.Y.2021-22 105 lakhs, 2022 and `20 lakhs in, March 2022., 3. Mr. Satish, a salaried individual, Payment of brokerage for buying a `51 lakhs, residential house in March, 2022, 4. Mr. Dheeraj, a pensioner, Contract payment made during, `48 lakhs, October–November 2021 for, reconstruction of residential house, Solution:, Particulars, payer, , of, , the Nature of payment, , 1. Mr. Ganesh, an, individual carrying on, retail business with, turnover of ` 2.5 crores, in the P.Y.2020-21, , Aggregate of Whether TDS provisions are, payment, in attracted?, the F.Y. 202122, Contract Payment for ` 5 lakhs, No, TDS under section 194C is, repair of residential, not attracted since the payment, house, is for personal purpose and TDS, under section 194M is not, attracted as aggregate of contract, payment to the payee in the
Page 349 :
Deduction Of Tax At Source, , 2. Mr. Rajesh, , Payment, of ` 80,000, commission to Mr., Vallish for business, purposes, Contract Payment for, ` 55 lakhs, reconstruction of, residential house, , 3. Mr. Satish, a salaried Payment of brokerage ` 51 lakhs, individual, for buying a residential, house, , 4. Mr., Dheeraj,, pensioner, , a Contract payment for, reconstruction of, residential house, , ` 48 lakhs, , 349, P.Y.2021-22 does not exceed `, 50 lakh., Yes, u/s 194H, since the, payment exceeds `15,000, and, Mr. Ganesh’s turnover exceeds, `1 crore in the P.Y.2020-21., Yes, under section 194M, since, the aggregate of payments (i.e.,, `55 lakhs) exceed `50 lakhs,, Since his turnover is below `100, lakhs in the P.Y.2020-21. Hence,, TDS provisions under section, 194C are not attracted in respect, of payments made in the, P.Y.2021-22., Yes, under section 194M, since, the payment of ` 51 lakhs made, in March 2022 exceeds the, threshold limit of `50 lakhs., Since Mr. Satish is a salaried, individual, the provisions of, section 194H are not applicable, in this case., TDS provisions under section, 194C are not attracted since Mr., Dheeraj is a pensioner and TDS, provisions under section 194M, are also not applicable in this, case, since the payment of ` 48, lakhs does not exceed the, threshold limit of ` 50 lakhs., , NOV – 2020 EXAM (NEW COURSE), Question 2 (b), 5 Marks, State in brief the applicability of tax deduction at source provisions, the rate and amount of tax deduction in, the following cases for the financial year 2021-2022 under the Income -tax Act,1961. Assume that all, payments are made to residents:, (i) Sanjay, a resident individual, not deriving any income from business or profession makes payment of `12, lakhs in January, 2022, `20 lakh in February, 2022 and `20 lakh in March, 2022 to Mohan, a contractor, for reconstruction of his residential house., (ii) ABC Ltd. makes the payment of `1,50,000 to Ramlal, an individual transporter who owned, 6 goods carriages throughout the previous year, He does not furnish his PAN., (iii) Smt. Sarita paid `5,000 on 17th April,2021 to Smt. Deepa from the deposits in National savings, Scheme account., Answer:, (i) Yes, under section 194M since the aggregate of payments (i.e., `52 lakhs) exceeds `50 lakhs and his, turnover is below `100 lakhs in the P.Y.2020-21. Hence, TDS provisions under section 194C are not
Page 350 :
Deduction Of Tax At Source, , 350, , attracted in respect of payments made in the P.Y.2021-22 and section 194M gets attracted as the aggregate, payments exceeds 50 lakhs, hence he is liable to deduct TDS @ 5% on 52,00,000 = 2,60,000., (ii) As per section 194C, No tax shall be deducted at source in case of payment to a contractor in connection, with transportation of goods where such contractor do not own more than 10 goods carriages at any time, during the year and also submitted a declaration in this regard and has also furnished permanent account, number. But in the given case transporter has not furnished his PAN hence ABC limited can deduct TDS u/s, 194C., As transporter has not furnished his PAN then section 206AA shall also be applicable and TDS shall be, deducted @ 20% on 1,50,000 = 30,000., (iii) As per section 194EE, the person responsible for paying to any person any amount from deposits under, National saving scheme shall, at the time of payment thereof, deduct income-tax thereon at the rate of 10%, provided amount is exceeding 2,500 in a financial year. In the given case amount exceeds 2,500 hence TDS, shall be deducted @ 10% on 5,000 = 500., MTP NOV -2020, 3. Mr. Ajay Sahu, proprietor of M/s Blue Bird Enterprises having turnover of ` 65 lakhs and, Turnover is less than 100 lakhs during P.Y. 2020-21, has received two bills for payment. The first, bill is for ` 42,00,000 from Vijay Associates, an advocate and property dealer firm, for his, daughter’s hearing and ` 21,00,000 from same Vijay Associates for brokerage service provided in, relation to purchase of one property. Both bills were raised on 21-12-2021 but payment were, made in instalments. 1st Instalment of `5,00,000 as advance was payment on 15-11-2021, 2nd, Instalment of `45,00,000 on 25-03-2022 and balance amount `13,00,000 on 31-03-2022., Determine the TDS liability for Mr. Ajay Sahu, if any, for A.Y. 2022-23?, (a) ` 2,50,000, (b) ` 3,15,000, (c) ` 65,000, (d) Nil, , (2 Marks), , Answer: (b), As per section 194M, any individual and HUF (not liable to deduct tax at source u/s 194C/194J/194H) is, liable to deduct tax at source @ 5% if aggregate of such payment is exceeding 50,00,000. Payment is for, commission or brokerage or by way of fees for professional services., In the given case, it is covered u/s 194M, hence TDS shall be deducted @ 5% on 63,00,000 = 3,15,000, Question 23: Write a note on TDS in case of Payment of certain amounts in cash., Answer: TDS in case of Payment of certain amounts in cash Section 194N, Every person, being,––, (i) a banking company to which the Banking Regulation Act, 1949 applies, (ii) a co-operative society engaged in carrying on the business of banking; or, (iii) a post office,, who is responsible for paying any sum, being the amount or the aggregate of amounts, as the case may be, in, cash exceeding one crore rupees during the previous year, to any person (herein referred to as the recipient), from one or more accounts maintained by the recipient with it shall, at the time of payment of such sum,, deduct an amount equal to two per cent of such sum, as income-tax:, Provided that in case of a recipient who has not filed the returns of income for all of the three assessment, years relevant to the three previous years, for which the time limit to file return of income under sub-section, (1) of section 139 has expired, immediately preceding the previous year in which the payment of the sum is, made to him, the provision of this section shall apply with the modification that(i) the sum shall be the amount or the aggregate of amounts, as the case may be, in cash exceeding, twenty lakh rupees during the previous year; and
Page 351 :
Deduction Of Tax At Source, , 351, , (ii) the deduction shall be—, (a) an amount equal to two per cent. of the sum where the amount or aggregate of amounts, as the, case may be, being paid in cash exceeds twenty lakh rupees during the previous year but does not, exceed one crore rupees; or, (b) an amount equal to five per cent. of the sum where the amount or aggregate of amounts, as the, case may be, being paid in cash exceeds one crore rupees during the previous year:, Provided also that nothing contained in this section shall apply to any payment made to,––, (i) the Government;, (ii) any banking company or co-operative society engaged in carrying on the business of banking or a post, office;, (iii) any business correspondent of a banking company or co-operative society engaged in carrying on the, business of banking., (iv) any white label automated teller machine operator of a banking company, including Cash, Replenishment Agencies (CRA’s) and franchise agents of White Label Automated Teller Machine, Operators (WLATMO’s) maintaining a separate bank account from which withdrawal is made only for the, purposes of replenishing cash in the Automated Teller Machines (ATM’s) operated by such WLATMO., (v) Commission agent or trader, operating under Agriculture Produce Market Committee (APMC), and, registered under any Law relating to Agriculture Produce Market of the concerned State., (vi) The authorised dealer and its franchise agent and sub-agent; and Full-Fledged Money Changer (FFMC), licensed by the RBI and its franchise agent;, Example, Mr. X has withdrawn ₹120 Lakh in F.Y. 2021-22 in cash, amount of TDS shall be 120 lakh x 2% =, ₹2,40,000 but if he has not filing return for last three years and time limit has expired in the preceding year,, amount of TDS shall be 100 lakh x 2% i.e. ₹2,00,000 and 20 lakh x 5% = 1,00,000. If amount withdrawn is, ₹15 lakh, there is no TDS. If amount withdrawn is ₹70 lakh, there is no TDS but if return has not been filed, for last three years, amount of TDS shall be 70 lakh x 2% = ₹1,40,000., NOV -2020 (4 Marks), Question 4 (c), Briefly explain the provisions relating to tax deduction at source on cash withdrawal under section, 194N of the Income Tax Act, 1961., Answer: Refer answer given in the chapter, RTP NOV -2020 (Modified), 4. Mr. Nihar maintains a Savings A/c and a Current A/c in Mera Bank Ltd. The details of withdrawals on, various dates during the previous year 2021-22 are as follows:, Date of Cash withdrawal, Saving Account, Current account, 05.04.2021, 15,00,000, 10.05.2021, 22,00,000, 25.06.2021, 20,00,000, 17.07.2021, 5,00,000, 28.10.2021, 35,00,000, 10.11.2021, 38,00,000, 12.12.2021, 25,00,000, Is Mera Bank Limited required to deduct tax at source on the withdrawals made by Mr. Nihar during the, previous year 2021-22? If yes, what would the amount of tax deducted at source? If Mr. Nihar is in normal, category and special category.
Page 352 :
Deduction Of Tax At Source, , 352, , Solution: Total withdrawal upto 31/03/2022 are ₹ 160 lakhs hence TDS shall be at a rate of 2% on the entire, amount of ₹ 160 lakhs. Amount of TDS shall be ₹ 160 lakhs X 2% = ₹ 3,20,000., If he is covered in the special category i.e. return has not been filed upto 3 years, TDS shall be on ₹ 100, lakhs at a rate of 2% = ₹ 2,00,000 and on ₹ 60 lakhs at a rate of 5% = ₹ 3 lakhs., Question 24: Write a note on TDS in case of Payment of certain sums by e-commerce operator to ecommerce participant., Answer: TDS in case of Payment of certain sums by e-commerce operator to e-commerce participant, Section 194O, Every E-Commerce Operator shall deduct tax at source @ 1% while making payment to any E-Commerce, participant for supply of goods or services however no tax shall be deducted at source if such E-Commerce, participant is an individual or HUF and gross amount of sale or services is upto 5 lakh. If E-Commerce, participant has received any payment directly from buyer, it will also be considered to be a payment made, by E-Commerce Operator and tax shall be deducted at source. If tax has been deducted by E-Commerce, Operator or TDS is not applicable because sale is upto 5 lakh, in such case no TDS shall be applicable under, any other TDS provision but TDS shall be applicable in case of amount received or receivable in connection, with Hosting advertisement or providing any other service which is not connected with the sale or service, given by E-Commerce participant., (a) “electronic commerce” means the supply of goods or services or both, including digital products,, over digital or electronic network;, (b) “e-commerce operator” means a person who owns, operates or manages digital or electronic, facility or platform for electronic commerce;, (c) “e-commerce participant” means a person resident in India selling goods or providing services or, both, including digital products, through digital or electronic facility or platform for electronic, commerce;, (d) “services” includes ‘‘fees for technical services’’ and fees for ‘‘professional services’’, as defined, in the Explanation to section 194J.’., Example 1, ABC Ltd. has sold 1,000 television sets at ₹50,000 each through Amazon on 01.12.2021. Amazon receives, payment of ₹5,00,00,000 from various buyers and deducts commission of 10% and wants to remit, ₹4,50,00,000 to ABC Ltd., in this case ABC Ltd. is the E-commerce Participant and Amazon is the Ecommerce Operator. As per section 194-O, Amazon shall deduct TDS of 1% on ₹5,00,00,000 i.e. ₹5,00,000., Therefore Amazon will remit ₹4,45,00,000 to ABC Ltd. after deducting TDS of ₹5,00,000., There is no liability of ABC Ltd. to deduct TDS under section 194H on the commission it pays to Amazon, since:, (i) Section 194-O overrules the entire chapter of TDS and, (ii) As per section 194-O, a transaction on which tax has been deducted by e-commerce operator, under section 194-O, shall not be liable to tax deduction under any other section. TDS has been, deducted on the entire transaction of ₹5,00,00,000., Example 2, Mr. A, a resident manufactures perfumes and sell through Flipkart. The total sales made by Mr. A through, Flipkart are ₹5,00,000 in previous year 31.03.2022. Flipkart deducts commission @ 10% and remits, ₹4,50,000 to Mr. A, in this case Section 194-O shall not be applicable since the gross sales do not exceeds, ₹5,00,000. Hence Flipkart shall not deduct TDS provided that Mr. A gives his PAN/Aadhaar to Flipkart. If, he does not give PAN/Aadhaar to Flipkart, then TDS shall be deducted @ 5% on ₹5,00,000., TDS on commission paid to Flipkart is also not required to be deducted., Example 3, Will your answer be different if in example 2, M/s ABC Partnership was there instead of Mr. A.
Page 353 :
Deduction Of Tax At Source, , 353, , Solution:, Section 194-O shall be applicable since exemption is only in case of individuals and HUF whose gross sales, through E-Commerce Operator does not exceed ₹5,00,000., , Flipkart will deduct TDS @ 1% under section 194-O on ₹5,00,000. M/s ABC shall not deduct TDS on, commission paid to Flipkart., Example 4, M/s ABC sells Women Apparel through Myntra and total sales of M/s ABC through Myntra are, ₹2,00,00,000 during the previous year 31.03.2021. Myntra charges 10% commission on sales made by it., The payment gateway of Myntra is so designed that ₹20,00,000 comes to Myntra from customers and, ₹1,80,00,000 goes to M/s ABC directly., Solution:, As per section 194-O, any payment by purchaser directly to E-Commerce participant for sale of goods, facilitated by an E-Commerce Operators shall be deemed to be the amount credited or paid by E- Commerce, Operator to E-Commerce participant and shall be included in gross amount of sale for purpose of tax, deduction under section 194-O, Therefore Myntra shall pay TDS of 1% of ₹2,00,00,000 i.e. ₹2,00,000 to the credit of Government and shall, recover ₹2,00,000 from M/s ABC., Example 5, A Chartered Accountant renders professional advices to various clients through “Consult-CA”, a website run, by E-Commerce Operator. Various clients pays ₹1,00,00,000 to Consult-CA and Consult-CA deducts 20%, commission and remits ₹80,00,000 to the Chartered Accountant for the financial year ending 31.03.2021, Solution:, As per section 194-O, “Services” include professional services. Therefore Consult-CA will deduct 1% TDS, on ₹1,00,00,000 and remit ₹79,00,000 to the Chartered Accountant. Section 194J shall not apply and TDS, shall not be deducted., Question 25: Write a note on Deduction of tax in case of specified senior citizen., Answer: TDS in case of specified senior citizen Section 194P, (1) Notwithstanding anything contained in the provisions of Chapter XVII-B, in case of a specified senior, citizen, the specified bank shall, after giving effect to the deduction allowable under Chapter VI-A and, rebate allowable under section 87A, compute the total income of such specified senior citizen for the, relevant assessment year and deduct income-tax on such total income on the basis of the rates in force., (2) The provisions of section 139 shall not apply to a specified senior citizen for the assessment year, relevant to the previous year in which the tax has been deducted under sub-section (1)., Explanation.–– For the purposes of this section,––, (a) “specified bank” means a banking company as the Central Government may, by notification in Official, Gazette, specify;, (b) “specified senior citizen” means an individual, being a resident in India –, (i) who is of the age of seventy-five years or more at any time during the previous year;, (ii) who is having income of the nature of pension and no other income except the income of the nature, of interest received or receivable from any account maintained by such individual in the same, specified bank in which he is receiving his pension income; and, (iii) has furnished a declaration to the specified bank containing such particulars, in such form and, verified in such manner, as may be prescribed.’., Question 26: Write a note on Deduction of tax at source on payment of certain sum for purchase of, goods., Answer: TDS in case of payment of certain sum for purchase of goods Section 194Q, (1) Any person, being a buyer who is responsible for paying any sum to any resident (hereafter in this, section referred to as the seller) for purchase of any goods of the value or aggregate of such value
Page 354 :
Deduction Of Tax At Source, , 354, , exceeding fifty lakh rupees in any previous year, shall, at the time of credit of such sum to the account of the, seller or at the time of payment thereof by any mode, whichever is earlier, deduct an amount equal to 0.1, per cent. of such sum exceeding fifty lakh rupees as income-tax., Explanation.––For the purposes of this sub-section, “buyer” means a person whose total sales, gross, receipts or turnover from the business carried on by him exceed ten crore rupees during the financial year, immediately preceding the financial year in which the purchase of goods is carried out, not being a person,, as the Central Government may, by notification in the Official Gazette, specify for this purpose, subject to, such conditions as may be specified therein., (2) Where any sum referred to in sub-section (1) is credited to any account, whether called “suspense, account” or by any other name, in the books of account of the person liable to pay such income, such credit, of income shall be deemed to be the credit of such income to the account of the payee and the provisions of, this section shall apply accordingly., (3) If any difficulty arises in giving effect to the provisions of this section, the Board may, with the previous, approval of the Central Government, issue guidelines for the purpose of removing the difficulty., (4) Every guideline issued by the Board under sub-section (3) shall, as soon as may be after it is issued, be, laid before each House of Parliament, and shall be binding on the income-tax authorities and the person, liable to deduct tax., (5) The provisions of this section shall not apply to a transaction on which –, (a) tax is deductible under any of the provisions of this Act; and, (b) tax is collectible under the provisions of section 206C other than a transaction to which sub-section, (1H) of section 206C applies.’., Question 27: Write a note on TDS in case of Payment to Non-Resident or Foreign Company., Answer: TDS in case of Payment to Non-Resident or Foreign Company Section 195, Every person making any payment to a non-resident or to a foreign company shall deduct tax at source at the, prescribed rate., Question 28: Explain TDS provision in case of payment to Government., Answer: Interest or dividend or other sums payable to Government, Reserve Bank or certain, corporations. Section 196, Notwithstanding anything contained in the foregoing provisions of this Chapter, no deduction of tax shall be, made by any person from any sums payable to—, (i) the Government, or, (ii) the Reserve Bank of India, or, (iii) a corporation established by or under a Central Act which is, under any law for the time being in force,, exempt from income-tax on its income, or, (iv) a Mutual Fund specified under clause (23D) of section 10, where such sum is payable to it by way of, interest or dividend in respect of any securities or shares owned by it., Question 29: Write a note on TDS in case of payment by individual or Hindu Undivided Family., Answer: TDS in case of payment by individual or Hindu Undivided Family, An Individual or Hindu Undivided Family shall be required to deduct tax at source only if the turnover in, case of business has exceeded ` 1 crore and gross receipts in case of profession has exceeded ` 50 lakhs,, during the financial year immediately preceding the relevant year., The above provisions are applicable for TDS under section 194A, 194C, 194H, 194I, 194J., The above provisions are not applicable for TDS under other sections like 192, 192A, 193, 194, 194B,, 194BB, 194D, 194DA, 194E, 194G, 194-IA,194-IB, 194LA, 194K, 194M, 194N, 194O, 195., Question 30: Write a note on Deduction of tax at Lower Rate., Answer: Deduction of tax at Lower Rate Section 197, If on income of any person, income-tax is required to be deducted at the time of payment under section 192,, 193, 194, 194A, 194C, 194D, 194G, 194H, 194I, 194J, 194K, 194LA,194M, 194O and 195 and the
Page 355 :
Deduction Of Tax At Source, , 355, , Assessing Officer is satisfied that the total income of the recipient justifies the deduction of income-tax at, any lower rates or no deduction of income-tax, as the case may be, the Assessing Officer shall, on an, application made by the assessee in this behalf, give to him such certificate as may be appropriate., Application should be given in Form No. 13., (Section 197 is not applicable in case of TDS under section 192A, 194B, 194BB, 194DA, 194E, 194-IA,, 194-IC, 194N), Question 31: Write a note on self declaration for not deducting tax at source., Answer: Self declaration for not deducting tax at source Section 197A, As per section 197A, if any individual or Hindu Undivided Family has total income not exceeding the, exemption limit and also his tax liability is nil, such individual or HUF can furnish a declaration in Form, No. 15G to the person making payment of interest and in that case no tax shall be deducted at source. A, senior citizen can give a declaration in Form No 15H if his tax liability is nil., Similar provision shall be applicable in case of section 192A, 193, 194, 194A, 194D, 194DA & 194I., Question 32: Write a note on time period for depositing tax deducted at source., Answer: Time period for depositing tax deducted at source Section 200 / Rule 30, As per Rule 30, the payment is to be made in general within 7 days from the last day of the month in, which the deduction is made., If the tax has been deducted in the month of March, tax should be deposited on or before 30th April., In certain cases, Assessing Officer may permit the payments on quarterly basis., Question 33: Write a note on filing of quarterly statement of TDS., Answer: Filing of quarterly statement of TDS, Section 200 / Rule 31A, Every person deducting tax at source has to submit quarterly statement containing details of the tax deducted, at source. The statement should be submitted latest by 31st of the month succeeding the relevant quarter but, statement for the quarter ending March can be submitted upto 31st May. e.g. Statement for quarter ending, March, 2022 can be submitted upto 31st May, 2022., Question 34: Write a note on Consequences of Failure to Deduct or Pay., Answer: Consequences of Failure to Deduct or Pay Section 201, 1. If any person has failed to deduct tax at source, interest shall be charged @ 1% p.m. or part of a, month for the period of delay. E.g. ABC Ltd. has made one payment on 03.01.2022 but tax was deducted at, source on 20.01.2022, in this case interest shall be charged @ 1% for one month., 2. If person has deducted tax at source but tax was not deposited within the time allowed under section, 200, interest shall be charged @ 1.5% p.m. or part of a month from date of deducting tax at source upto, the date of depositing the amount, Example, Assessee deduct TDS on 10.10.2021 but pays TDS on 31.12.2021, Interest under section 201 shall be, charged from 10.10.2021 to 31.12.2021 @ 1.5% per month i.e., for 3 months., If in the above case assessee has not deducted tax at source on 10.10.2021 rather assessee deducted TDS on, 31.12.2021 and assessee pays TDS on 17.01.2022, interest shall be charged in the manner given below:, (i) Interest under section 201 shall be charged for 3 months @ 1% for the period 10.10.2021 to 31.12.2021., (ii) Interest under section 201 shall be charged @ 1.5% per month for one month from 31.12.2021 to, 17.01.2022., 3. Assessee shall also be considered to be assessee in default and penalty may be imposed equal to the, amount which he has failed to deduct or pay but in following two situations he will not be considered to be, assessee in default, 1. If there were sufficient reasons for not deducting tax at source, 2. Payment was made to a payee and such payee has shown the amount in his income and has paid tax and, also return has been filed and it has been confirmed by a Chartered Accountant., In this case assessee shall pay interest from the date when tax was to be deducted upto the date of filing the, return.
Page 356 :
Deduction Of Tax At Source, , 356, , Example, , ABC Ltd. paid certain amount on 05.01.2022 to Mr. X and tax was not deducted at source but Mr. X himself, has paid his tax and return was filed on 31.07.2022, in this case interest shall be charged @ 1% p.m. for a, period of 7 months i.e. from 05.01.2022 to 31.07.2022., Illustration 12: An amount of `40,000 was paid to Mr. X on 01.07.2021 towards fees for professional, services without deduction of tax at source. Subsequently, another payment of `50,000 was due to Mr. X on, 28.02.2022, from which tax @ 10% (amounting to `9,000) on the entire amount of ` 90,000 was deducted., However, this tax of ` 9,000 was deposited only on 22.06.2022., Compute the interest chargeable under section 201., Solution:, Interest under section 201 would be computed as follows –, Particulars, `, 320, 1% on tax deductible but not deducted i.e., 1% on ` 4,000 for 8 months, (01.07.2021 to 28.02.2022), 540, 1½% on tax deducted but not deposited i.e. 1½% on ` 9,000 for 4 months, (28.02.2022 to 22.06.2022), 860, Question 35: Write a note on Certificate for Tax Deducted., Answer: Certificate for Tax Deducted Section 203/ Rule 31, TDS Certificate, Every person deducting tax at source shall issue a certificate to the person with regard to whom tax has been, deducted at source. In case of payment of salary, certificate shall be issued in Form No. 16 and in other, cases it will be in Form No. 16A., The certificate in Form No. 16 should be given upto 31st May of the succeeding year in case of an employee, and it will be an annual certificate., In other cases certificate in Form No. 16A should be issued on quarterly basis and it should be issued within, 15 days from the last date of submitting the quarterly statement under section 200., Question 36: Write a note on Tax Deduction Account Number (TAN)., Answer: Tax Deduction Account Number (TAN) Section 203A, Every person, deducting tax at source shall apply for allotment of tax deduction account number and, application has to be given in Form No.49B within one month from the end of the month in which tax, was deducted for the first time., Application for allotment of a tax deduction and collection account number., 114A. An application under section 203A for the allotment of a tax deduction and collection account, number shall be made in duplicate in Form No. 49B and application should be given within one month from, the end of the month in which tax was deducted or collected for the first time, Provided that an applicant may apply for allotment of a tax deduction and collection account number, through a common application form notified by the Central Government in the Official Gazette, and the, Principal Director General of Income-tax (Systems) or Director General of Income-tax (Systems) shall, specify the classes of persons, applicable forms and formats along with procedure for safe and secure, transmission of such forms and formats in relation to furnishing of tax deduction and collection account, number.], Question 37: Write a note on Requirement to Furnish Permanent Account Number., Answer: Requirement to Furnish Permanent Account Number Section 206AA, Every person on whose behalf, tax is being deducted at source shall submit his PAN to the person deducting, tax at source otherwise rate of TDS shall be the actual rate or 20% whichever is higher. The person, deducting tax at source has to mention such PAN in the quarterly statement. However, if the assessee,, whose tax is required to be deducted under section 194-O or 194Q, does not submit his PAN or Aadhar,, rate of TDS shall be 5% instead of 20%.
Page 357 :
Deduction Of Tax At Source, , 357, , TDS shall be on the amount excluding GST, As per Circular No. 23/2017, Dated 19-7-2017, wherever in terms of the agreement/contract between the, payer and the payee, the GST component comprised in the amount payable to a resident is indicated, separately, tax shall be deducted at source on the amount paid/payable without including such GST, component., Question 38: Write a note on Special provision for deduction of tax at source for non-filers of incometax return., Answer: Special provision for deduction of tax at source for non-filers of income-tax return Section, 206AB, (1) Notwithstanding anything contained in any other provisions of this Act, where tax is required to be, deducted at source under the provisions of Chapter XVIIB, other than sections 192, 192A, 194B, 194BB,, 194LBC or 194N on any sum or income or amount paid, or payable or credited, by a person (hereafter, referred to as deductee) to a specified person, the tax shall be deducted at the higher of the following rates,, namely:–, a. at twice the rate specified in the relevant provision of the Act; or, b. at twice the rate or rates in force; or, c. at the rate of five per cent., (2) If the provisions of section 206AA is applicable to a specified person, in addition to the provision of this, section, the tax shall be deducted at higher of the two rates provided in this section and in section 206AA., (3) For the purposes of this section “specified person” means a person who has not filed the returns of, income for both of the two assessment years relevant to the two previous years immediately prior to the, previous year in which tax is required to be deducted, for which the time limit of filing return of income, under sub-section (1) of section 139 has expired; and the aggregate of tax deducted at source and tax, collected at source in his case is rupees fifty thousand or more in each of these two previous years:, Provided that the specified person shall not include a non-resident who does not have a permanent, establishment in India., Explanation.––For the purposes of this sub-section, the expression “permanent establishment” includes a, fixed place of business through which the business of the enterprise is wholly or partly carried on.’., Illustration 13: Mrs. X has received incomes as given below during the previous year 2021-22:, 1. Interest on savings bank account with State Bank `50,000 (gross)., 2. Interest from Government securities `1,00,000 on 01.01.2022 (collection charge paid to the bank @, 1.5%)., 3. Interest from ABC Ltd on non listed debentures `3,60,000 (after TDS) on 01.03.2022 (collection, charge paid to the bank `30)., 4. Interest credited to post office savings bank account during the year ` 10,000., 5. Interest credited to public provident fund during the year ` 15,000., 6. Interest received from XYZ Ltd on listed debentures ` 1,35,000 (Net)., (Collection charge `30) The amount was invested by taking a loan of `15,00,000 @ 12% p.a., 7. Mrs. X received rent of house property ` 72,000 per month after TDS., 8. Winnings from a lottery `70,000 (after TDS), Compute her tax liability and also tax payable for the assessment year 2022-23., Solution:, `, Income under the head other sources, Gross interest from State Bank of India, 50,000.00, Interest from Government securities, {`1,00,000 – `1,500}, 98,500.00, Interest from ABC Ltd, 3,99,970.00, {(`3,60,000 / 90 x 100) – `30}, Interest on P.O.S.B (10,000 – 3,500), 6,500.00, Interest on PPF (exempt u/s 10(15)), Nil
Page 358 :
Deduction Of Tax At Source, Interest from XYZ Ltd., {Gross interest = `1,35,000 / 90 x 100 = 1,50,000, Less: Collection charges = (`30), Less: Interest paid on loan = (`1,80,000)}, Winning from lottery, (70,000 / 70 x 100), Income under the head Other Sources, Income under the head House Property, Gross Annual Value (72,000/90% x 12), Less: Municipal Tax, Net Annual Value, Less: 30% of NAV u/s 24(a), Less: Interest on capital borrowed u/s 24(b), Income from house property, Gross Total Income, Less: Deduction u/s 80TTA, Total Income, Computation of tax liability, Tax on winning from lottery `1,00,000 @ 30% u/s 115BB, Tax on `11,86,940 at slab rate, Tax before health & education cess, Add: HEC @ 4%, Tax Liability, Less: TDS u/s 193 on Non Listed Debentures, Less: TDS u/s 193 on Listed Debentures, Less: TDS u/s 194B on winning from lottery, Less: TDS u/s 194I, Tax Payable, Rounded off u/s 288B, , 358, (30,030.00), , 1,00,000.00, 6,24,940.00, 9,60,000.00, (Nil), 9,60,000.00, (2,88,000.00), (Nil), 6,72,000.00, 12,96,940.00, (10,000.00), 12,86,940.00, 30,000.00, 1,68,582.00, 1,98,582.00, 7,943.28, 2,06,525.28, (40,000.00), (15,000.00), (30,000.00), (96,000.00), 25,525.28, 25,530.00, , Illustration 14: Mr. X has let out one House property and rent received is `90,000 p.m. after TDS. He paid, Municipal Tax `1,00,000 and Interest u/s 24 (b) is `2,00,000. He has received ` 9,00,000 in connection, with professional services after TDS. The Assessee made the payment of tax on 10.05.2022. Compute Total, Income and Tax Payable and also Compute Interest u/s 234A, 234B & 234C., Solution:, Computation of income under the head House Property, `, Gross Annual Value (90,000/90% x 12), 12,00,000.00, Less: Municipal Tax, (1,00,000.00), Net Annual Value, 11,00,000.00, Less: 30% of NAV u/s 24(a), (3,30,000.00), Less: Interest on capital borrowed u/s 24(b), (2,00,000.00), Income from house property, 5,70,000.00, Income under the head Business/Profession (9,00,000/90%), 10,00,000.00, Gross Total Income, 15,70,000.00, Less: Deduction u/s 80C to 80U, Nil, Total Income, 15,70,000.00, Computation of Tax Liability, Tax on ` 15,70,000 at slab rate, Add: HEC @ 4%, Tax Liability, Less: TDS u/s 194I, , 2,83,500.00, 11,340.00, 2,94,840.00, (1,20,000.00)
Page 359 :
Deduction Of Tax At Source, Less: TDS u/s 194J, Tax Payable, Interest under section 234C shall be computed in the manner given below:, Tax Payable, Tax Paid, 15.06.2021, 11,226 (74,840 x 15%), Nil, Rounded off Rule 119A = 11,200, Interest u/s 234C = 11,200 x 1% x 3 = 336, 15.09.2021, 33,678 (74,840 x 45%), Nil, Rounded off Rule 119A = 33,600, Interest u/s 234C = 33,600 x 1% x 3 = 1,008, 15.12.2021, 56,130 (74,840 x 75%), Nil, Rounded off Rule 119A = 56,100, Interest u/s 234C = 56,100 x 1% x 3 = 1,683, 15.03.2022, 74,840 (74,840 x 100%), Nil, Rounded off Rule 119A = 74,800, Interest u/s 234C = 74,800 x 1% x 1 = 748, Total interest payable u/s 234C, , 359, (1,00,000.00), 74,840.00, Shortfall, 11,226, , 33,678, 56,130, 74,840, , 3,775, , Interest under section 234B shall be computed from 01.04.2022 to 10.05.2022 and is as given below:, Tax Liability – TDS shall be considered to be tax payable i.e. 74,840, 74,840 = 74,800 x 1% x 2 =, 1,496, (Rounded off Rule 119A = 74,800), Total interest payable (3,775 + 1,496), 5,271, Rounded off u/s 288B, 5,270
Page 360 :
Deduction Of Tax At Source, , 360, , TAX COLLECTION AT SOURCE, 206C, Question : Write a note on TCS under the Income- tax Act, 1961., Answer:, If the person making the payment has deducted tax, it is called TDS but if the person collecting payment for, goods or other purpose has also collected tax, it is called TCS and it is applicable in the following cases:, 1. As per section 206C(1), every seller shall collected tax in the following cases:, (i) Alcoholic Liquor for human consumption, 1%, (ii) Tendu leaves, 5%, (iii)Timber obtained under a forest lease, 2.5%, (iv) Timber obtained by any mode other than under a forest lease, 2.5%, (v) Any other forest produce not being timber or tendu leaves, 2.5%, 1%, (vi) Scrap, (vii) Minerals, being coal or lignite or iron ore, 1%, Buyer" means a person who obtains in any sale, by way of auction, tender or any other mode, or the right to, receive any such goods but does not include,— a public sector company, the Central Government, a State, Government, and an embassy, a High Commission, and a club. It will also not include a buyer in the retail sale of, such goods purchased by him for personal consumption;, "seller" means the Central Government, a State Government or any local authority or corporation or authority, established by or under a Central, State or Provincial Act, or any company or firm or co-operative society and, also includes an individual or a Hindu undivided family whose total sales, gross receipts or turnover from the, business or profession carried on by him exceed one crore rupees in case of business or fifty lakh rupees in case, of profession during the financial year immediately preceding the financial year in which the goods as specified, above are sold, As per section 206CC, the person from whom tax is being collected shall furnish his permanent account, number otherwise tax shall be collected at the higher of the following rates, (i) Twice the rate applicable, (ii) At a rate of 5%, 2. As per section 206C(1C), every person giving license shall collect tax in the following cases:, (i) Parking lot, (ii) Toll plaza, (iii) Mining and quarrying, , 2%, 2%, 2%, , As per section 206CC, the person from whom tax is being collected shall furnish his permanent account, number otherwise tax shall be collected at the higher of the following rates, (i) Twice the rate applicable, (ii) At a rate of 5%, 3. As per section 206C(1F), every seller of motor vehicle shall collect tax at source in the following cases:, Motor vehicle exceeding value ₹ 10 lakh, 1%, “Buyer” means a person who obtains in any sale motor vehicle, but does not include,— the Central Government,, a State Government and an embassy, a High Commission, a local authority, a public sector company which is, engaged in the business of carrying passengers.], It will also not include sale by manufacturer to dealers and distributors, "Seller" means the Central Government, a State Government or any local authority or corporation or authority, established by or under a Central, State or Provincial Act, or any company or firm or co-operative society and, also includes an individual or a Hindu undivided family whose total sales, gross receipts or turnover from the, business or profession carried on by him exceed one crore rupees in case of business or fifty lakh rupees in case, of profession during the financial year immediately preceding the financial year in which the goods as specified, above are sold.
Page 361 :
Deduction Of Tax At Source, , 361, , As per section 206CC, the person from whom tax is being collected shall furnish his permanent account, number otherwise tax shall be collected at the higher of the following rates, (i) Twice the rate applicable, (ii) At a rate of 5%, Q.1 Whether TCS @ 1% is on sale of motor vehicle at retail level or also on sale of motor vehicles by, manufacturers to dealers/ distributors?, A. To bring high value transactions within the tax net, section 206C has been amended to provide that the, seller shall collect the tax @ 1% from the purchaser on sale of motor vehicle of the value exceeding ` 10, lakhs. This is brought to cover all transactions of retail sales and accordingly, it will not apply on sale of, motor vehicles by manufacturers to dealers/distributors., Q.2 Whether TCS @ 1% on sale of motor vehicle is applicable only to luxury cars?, A. No, as per section 206C(1F), the seller shall collect tax@1% from the purchaser on sale of any motor, vehicle of the value exceeding ` 10 lakhs., Q.3 Whether TCS @ 1% is applicable in the case of sale to Government Departments, Embassies,, Consulates and United Nation Institutions, of motor vehicle or any other goods or provision of, services?, A. Government, institutions notified under United Nations (Privileges and Immunities) Act 1947, and, Embassies, Consulates, High Commission, Legation, Commission and trade representation of a foreign State, shall not be liable to levy of TCS., Q.4 Whether TCS is applicable on each sale of motor vehicle or on aggregate value of sale during the, year?, A. Tax is to be collected at source@1% on sale consideration of a motor vehicle exceeding ` 10 lakhs. It is, applicable to each sale and not to aggregate value of sale made during the year., Q.5 Whether TCS @ 1% on sale of motor vehicle is applicable in case of an individual?, A. An individual who is liable to audit as per the provisions of section 44AB during the financial year, immediately preceding the financial year in which the motor vehicle is sold shall be liable for collection of, tax at source on sale of motor vehicle by him., Q.6 How would the provisions of TCS on sale of motor vehicle be applicable in a case where part of, the payment is made in cash and part is made by cheque?, A. The provisions of TCS on sale of motor vehicle exceeding ` 10 lakhs is not dependent on mode of, payment. Any sale of motor vehicle exceeding ` 10 lakhs would attract TCS @ 1%., 4. As per section 206C(1G), tax shall be collected at source in the following cases:, 1. Every authorized dealer shall collect tax at source from the person who is making remittance out of India, of ₹ 7 lakh or more in a financial year and tax shall be collected at a rate of 5% on the amount exceeding, ₹ 7 lakh but if it is a remittance for payment of loan for the purpose of education, TCS shall be at a rate, of 0.5%., , 2. Every seller who is selling overseas tour program package shall collect at source at a rate of 5%,, irrespective of the amount, Overseas tour programme package" means any tour package which offers visit to a country or countries, or territory or territories outside India and includes expenses for travel or hotel stay or boarding or lodging, or any other expenditure of similar nature or in relation thereto., , The provisions of section 206C(1G) shall not apply if the buyer is the Central Government, a State, Government, an embassy, a High Commission, a local authority or any other person as the Central Government, may notify.
Page 362 :
Deduction Of Tax At Source, , 362, , As per section 206CC, the person from whom tax is being collected shall furnish his permanent account, number otherwise tax shall be collected at the higher of the following rates, (i) Twice the rate applicable, (ii) At a rate of 5%, 5. As per 206C(1H), every seller whose turnover in the preceding year was exceeding ₹ 10 crores shall, collect tax at source from a buyer at a rate of 0.1% provided sale consideration is exceeding ₹ 50 lakh and, TDS shall be only on the amount exceeding ₹ 50 lakhs. The provisions shall not apply in case of goods, being exported out of India or in case of goods covered u/s 206C(1), 206C(1F) and 206C(1G), As per section 206CC, every person from whom tax is being collected shall furnish his permanent account, number otherwise tax shall be collected at twice the rate applicable or 5% whichever is higher however in, case of section 206C(1H), rate shall be double of the rate or 1% whichever is higher., "Buyer" means a person who purchases any goods, but does not include,—the Central Government, a State, Government, an embassy, a High Commission, a local authority, a person importing goods into India or any, other person as the Central Government may notify., , As per section 206CC, the person from whom tax is being collected shall furnish his permanent account, number otherwise tax shall be collected at the higher of the following rates, (i) Twice the rate applicable, (ii) At a rate of 1%, Time of Collection of tax, The tax should be collected at the time of debiting of the amount payable by the buyer or licensee or lessee,, as the case may be, to his account or at the time of receipt of such amount from the buyer or licensee or, lessee, as the case may be, in cash or by the issue of a cheque or draft or any other made, whichever is, earlier., In case of sale of a motor vehicle of the value exceeding ` 10 lakhs, tax shall be collected at the time of, receipt of such amount., Tax collection Account Number Section 206CA, Every person collecting tax as per section 206C shall apply for allotment of tax collection account number in, the similar manner as in case of tax deduction account number., MTP NOV -2020, 4. Mention the significant differences between TDS and TCS., Answer:, Significant Differences between TDS and TCS, TDS, , (3 Marks), , TCS, , (1), , TDS is tax deduction at source, , TCS is tax collection at source., , (2), , Person responsible for paying is required to deduct, tax at source at the prescribed rate, , Seller of certain goods or provider of services, is responsible for collecting tax at source at, the prescribed rate from the buyer., Person who grants licence or lease (in respect, of any parking lot, toll plaza, mine or quarry), is responsible for collecting tax at source at, the prescribed rate from the licensee or, lessee, as the case may be., , (3), , Generally, tax is required to be deducted at the time, of credit to the account of the payee or at the time of, payment, whichever is earlier., , Generally, tax is required to be collected at, source at the time of debiting of the amount, payable by the buyer of certain goods to the
Page 363 :
Deduction Of Tax At Source, However, in case of payment of salary, payment in, respect of life insurance policy etc., tax is required to, be deducted at the time of payment., , 363, , account of the buyer or at the time of receipt, of such amount from the said buyer,, whichever is earlier., However, in case of sale of motor vehicle of, the value exceeding `10 lakhs, tax collection, at source is required at the time of receipt of, sale consideration.
Page 364 :
Deduction Of Tax At Source, , 364, , MULTIPLE CHOICE QUESTIONS, 1. The rate of TDS on rental payments of plant, machinery or equipment is (a) 2%, (b) 5%, (c) 10%, (d) 1%, 2. Advance tax will not be paid if tax payable after TDS is less than–, (a) ` 10,000, (b) `5,000, (c) `20,000, (d) `25,000, 3. For deferment of advance tax (a) interest is payable under section 234A, (b) interest is payable under section 234B, (c) interest is payable under section 234C, (d) interest is payable under all the three sections 234A, 234B and 234C, 4. Mr. X, a resident Indian, wins `10,000 in a lottery. Which of the statement is true?, (a) Tax is deductible u/s 194B @ 30%, (b) Tax is deductible u/s 194B @ 30.9%, (c) No tax is deductible at source, (d) None of the above, 5. Mr. X paid fees for professional services of `40,000 to Mr. Y, who is engaged only in the business of, operation of call centre, on 15.7.2021. Tax is to be deducted by Mr. X at the rate of –, (a) 1%, (b) 2%, (c) 10%, (d) 20%, 6. Mr. A, a salaried individual, pays rent of `51,000 per month to Mr. B from June,2021. Which of the, statement is true?, (a) No tax is deductible at source since Mr. A is not liable to tax audit u/s 44AB., (b) Tax is deductible at source every month @ 10% on rent paid to Mr. B., (c) Tax is deductible at source every month @ 5% on rent paid to Mr. B., (d) Tax is deductible at source @ 5% on annual rent from the rent paid for March 2022., 7. `2 lakh is paid to Mr. Vallish, a resident individual on 15.3.2022 by the State of Haryana on, compulsory acquisition of his urban agricultural land., (a) No tax is deductible at source, (b) Tax is deductible@ 1%, (c) Tax is deductible @ 5%, (d) Tax is deductible@ 10%, 8. Two motor cars of the value of `12 lakhs and `8 lakhs was sold by a dealer to two different, customers., (a) Tax @ 1% has to be collected on `20 lakhs, (b) Tax @ 1% has to be collected on `12 lakhs, (c) Tax @ 1% has to be collected on `8 lakhs, (d) No tax collection at source is required in this case, 9. A registered firm pays salary and interest on capital to its resident partners. Which of the following, statements is true?, (a) Tax has to be deducted u/s 192 on salary and u/s 194A on interest, (b) Tax has to be deducted u/s 192 on salary but no tax needs to be deducted on interest, (c) No tax has to be deducted on salary but tax has to be deducted u/s 194A on interest, (d) No tax has to be deducted at source on either salary or interest, 10. M/S Mohan & Sons (liable to audit) paid `35,000 to Mr. Goel on 01.05.2021 towards fee for legal, advisory services without deduction of tax at source. Another payment of `47,000 was due to Mr. Goel, on 31.07.2021 and TDS on entire amount (i.e. `35,000 plus `47,000) was deducted and then the net, amount was paid. However, the total tax deducted was deposited on 15.11.2021. The interest, chargeable under section 201 will be:, (a) `650, (b) `433, (c) `486, (d) `597
Page 365 :
Deduction Of Tax At Source, , 365, , 11. Which of the following statement is correct., (a) As per section 192A tax shall be deducted at source @ 12% provided the amount paid or payable during, a particular year is ` 50,000 or more, (b) As per section 192A tax shall be deducted at source @ 10% provided the amount paid or payable during, a particular year is ` 1,50,000 or more, (c) As per section 192A tax shall be deducted at source @ 10% provided the amount paid or payable during, a particular year is ` 50,000 or more, (d) None of these, 12. Which of the following statement is correct., (a) As per section 193 tax shall be deducted at source @ 10% provided the amount paid or payable during a, particular year is exceeding ` 10,000, (b) As per section 193 tax shall be deducted at source @ 10% provided the amount paid or payable during a, particular year is exceeding ` 50,000, (c) As per section 193 tax shall be deducted at source @ 10% provided the amount paid or payable during a, particular year is exceeding ` 5,000, (d) None of these, 13. Which of the following statement is correct., (a) As per section 194A, in case of payment of interest on fixed deposit by a bank to a senior citizen, tax, deducted at source @ 10% provided the amount paid or payable during a particular year is exceeding, `50,000, (b) As per section 194A, in case of payment of interest on fixed deposit by a bank to a senior citizen, tax, deducted at source @ 10% provided the amount paid or payable during a particular year is exceeding, `40,000, (c) As per section 194A, in case of payment of interest on fixed deposit by a bank to a senior citizen, tax, deducted at source @ 10% provided the amount paid or payable during a particular year is exceeding, `1,50,000, (d) None of these, 14. Which of the following statement is correct., (a) As per section 194C, tax shall be deducted at source @ 2% in case of payment to individual or HUF and, @ 1% in case of payment to any other person provided the amount being paid is exceeding ` 1,00,000, (b) As per section 194C, tax shall be deducted at source @ 10% in case of payment to individual or HUF, and @ 1% in case of payment to any other person provided the amount being paid is exceeding ` 30,000, (c) As per section 194C, tax shall be deducted at source @ 1% in case of payment to individual or HUF and, @ 2% in case of payment to any other person provided the amount being paid is exceeding ` 30,000, (d) None of these, 15. Which of the following statement is correct., (a) As per section 194H tax shall be deducted at source @ 5% provided the amount being paid or payable to, a particular person during a particular year is ` 15,000 or more, (b) As per section 194H tax shall be deducted at source @ 5% provided the amount being paid or payable to, a particular person during a particular year is exceeding ` 15,000, (c) As per section 194H tax shall be deducted at source @ 10% provided the amount being paid or payable, to a particular person during a particular year is exceeding ` 15,000, (d) None of these, , Solutions are given on our website, www.mkgeducation.com
Page 366 :
Deduction Of Tax At Source, , 366, , EXAMINATION QUESTIONS, , Solutions of All Examination Questions are given on, our website www.mkgeducation.com, NOV – 2019 (NEW COURSE), Question. 2. (b), (7 Marks), Examine & explain the TDS implications in the following cases along with reasons thereof, assuming that, the deductees are residents and having a PAN which they have duly furnished to the respective deductors., (i) Mr. Tandon received a sum of `1,75,000 as pre-mature withdrawal from Employees Provident Fund, Scheme before continuous service of 5 years on account of termination of employment due to ill-health., (ii) A sum of `42,000 has been credited as interest on recurring deposit by a banking company to the, account of Mr. Hasan (aged 63 years)., (iii) Ms. Kaul won a lucky draw prize of `21,000. The lucky draw was organized by M/s. Maximus Retail, Ltd. for its customer., (iv) Finance Bank Ltd. sanctioned and disbursed a loan of `10 crores to Borrower Ltd. on 31-3-2022., Borrower Ltd. paid a sum of `1,00,000 as service fee to Finance Bank Ltd. for processing the loan, application., (v) Mr. Ashok, working in a private company, is on deputation for 3 months (from December, 2021 to, February, 2022) at Hyderabad where he pays a monthly house rent of `52,000 for those three months,, totaling to `1,56,000. Rent is paid by him on the first day of the relevant month., Question.4. (c), (4 Marks), What are the clarifications made by CBDT with respect to Section 206 C (1F) relating to following issues:, (i) Whether TCS on sale of motor vehicle is applicable only to luxury car?, (ii) Whether TCS is applicable on each sale or aggregate value of sale motor vehicle, exceeding ` 10 lakhs?, , (iii) Whether TCS is applicable in case of an individual?, (iv) Whether TCS on sale of motor vehicle is at retail level or only by manufacturer to distributor or dealer?, , MAY – 2019 (NEW COURSE), Question 2 (b), (7 Marks), Examine the TDS implications in the following cases along-with reasons thereof;, (i) Ms. Varsha received a sum of ` 95,000 on 31st December 2021 towards maturity proceeds of LIC taken, on 1st October 2013 for which sum assured was ` 80,000 and annual premium was ` 10,000., (ii) Mr. Deepak transferred a residential house property to Mr. Karan for ` 45 lacs. The stamp duty value of, such property is ` 55 lacs., (iii) XYZ Private Limited pays the following amounts to Mr. Narayan during previous year 2021-22:, (1) ` 22,000 towards fee for professional services, (2) ` 18,000 towards royalty, (iv) Payment of ` 1,75,000 made to Mr. Vaibhav for purchase of calendar according to specification of M/s., ABC Limited. However, no material was supplied for such calendar by ABC Limited to Mr. Vaibhav., (v) Talent Private Limited pays ` 12,000 to Ms. Sudha, its director, towards sitting fee which is not taxable, u/s 192., (vi) Radha Limited is engaged for Shyam Limited only in the business of operation of call centre. On 18-032022, the total amount credited by Shyam Limited in the ledger account of Radha Limited is ` 70,000
Page 367 :
Deduction Of Tax At Source, , 367, , regarding service charges of call centre. The amount is paid through cheque on 28/03/2022 by Shyam, Limited., , MAY – 2019 (OLD COURSE), Question 2 (b), (3 Marks), The following issues arise in connection with the deduction of tax at sources under chapter XVII-B. Discuss, the liability for tax deduction in these cases:, (i) An employee of the Central Government receives arrears of salary for the earlier 3 years. He inquires, whether amount will be received after deduction of tax at source during the current year., (ii) A T.V. channel pays ` 10 lacs as prize money to the winner of a quiz programme., (iii) A Nationalized bank pays ` 50,000 per month as rent to ABC limited for a building in which one of its, branch is situated., (iv) A television company pays ` 50,000 to a cameraman for shooting of a documentary film., , NOV – 2018 (NEW COURSE), Question 2 (b), (2 Marks), Mr. Dhanapal wishes to purchase a residential house costing `60 lakhs from Ms. Saipriya. The house is, situated at Chennai. He also wants to purchase agricultural lands in a rural area for `65 lakhs. He wants to, know whether there will be any obligation to deduct tax at source in these two situations. Both the buyer as, well as the sellers are residents in India. Advise Mr. Dhanapal suitably., Question 2(c), (2 Marks), Rahil & Co., a partnership firm is having a car dealership show-room. They have purchased cars for `2, crores from XYZ Ltd., car manufacturers, the cost of each car being more than `12 lakhs., They sell the cars to individual buyers at a price yielding 10% margin on cost. State whether there will be, any ob1igation to collect tax in the above two situations., , NOV – 2018 (OLD COURSE), Question 6 (a), (5 Marks), Examine the applicability of the provisions for tax deduction at source under Income Tax Act, 1961., (i) Mr. Z, a resident, is due to receive, `95,000 on 01.10.2021 towards maturity proceeds of LIC, policy taken on 01.10.2012 for which sum assured was `90,000 and the annual premium was `15,000., (ii) Mudra Ltd., an advertising Company, has retained a sum of `15 Lakhs, towards charges for procuring, and canvassing advertisement, from payment of 1 crore due to Cloud TV, A television channel and, remitting the balance amount of `85 Lakhs to the television channel. Would the provisions of Tax, deduction at source under section 194H be attracted on the sum of `15 Lakhs retained by the, advertising company?, (iii) Mr. X is salaried Individual pays rent of 55,000 per month to Mr. Y (does not have PAN) from June, 2021. ls he required to deduct TDS ? If so, when is he required to deduct tax ? Mr. X vacated the, premises on 31st December, 2021., 6 (c). The following details are provided by Mr. Pinto, an individual, for the assessment year 2022-2023., Amount (`), Total estimated tax payable, 2,00,000, TDS (estimated but not deducted), 55,000, Determine the advance tax payable with their due dates for the assessment year 2022-2023., , MAY – 2018, Question 6 (a), (5 Marks), Discuss the applicability of provisions of Tax Deduction at source, the rate and amount of tax deduction to, be made in the following cases for the financial year 2021-22.
Page 368 :
Deduction Of Tax At Source, , 368, , (A) Mr. Bobby, a resident whose turnover during the previous financial year is `205 Lakhs and for the, current year 2021-22 it is `80 lakhs., (i) Shop rent paid to Mr. Rajasekharan, a resident `25,000 per month., (ii) On 1-11-2021 paid towards fee for technical services `25,000 and royalty of `20,000 to Mr., Swamy, a resident who is having PAN. No other payment made to Mr. Swamy., (iii) On 01-10-2021 payment of `2,00,000 made to Mr. A for purchase of diaries according to, specifications. However, no material was supplied for such diaries., (iv) Contract payments made to Mr. Satheesan on 01-05-2021 for painting `25,000 and another, contract for interior furnishing on 22-03-2022 for `20,000., (B) Mr. Thrilok an individual not assessed to tax pays towards rent `60,000 per month., Question 6 (b), (Marks 5), Write any four cases where seller of certain goods is required to collect tax from buyers and also state the, circumstances where TCS is not applicable., Answer: See Answer in the book., , MAY – 2017, Question 7 (a), (4 Marks), Pallavi Bank Ltd., has paid interest of `29,000 to Mr. A, a resident Indian, from its Chennai branch and, `28,000 from Bangalore branch. If there is no core banking services in the bank, is tax required to be, deducted at source from such interest payments made on 31-3-2022? Will your answer be different if there, is core banking service present in the bank? Also, explain the provisions of the Income-tax Act, 1961 in this, regard., Question 7 (c), (4 Marks), Mr. Sachal , a resident individual aged 54, furnishes income details as under:, (i) Wholesale Cloth business, whose turnover is `150 lakhs, for which accounts are audited u/s 44AB., Income from such business `8,10,000., (ii) Income from other sources `2,70,000., (iii) Tax deducted at source `25,000., (iv) Advance tax paid `1,03,000 on 14-3-2022., Return of income will be filed on 11-12-2022. The assesse is willing to pay the requisite self-assessment tax., Calculate the interest payable under section 234C and 234B of the income-tax Act, 1961. Assume that the, return of income would be processed on the same day of filing of return., , NOV – 2016, Question 4(a), (2 Marks), State with reasons whether TDS is applicable or not on the following payment under the provisions of, Income-tax Act, 1961?, TDS is not applicable in respect of payment of `1,00,000 to Mr. Pandey a resident, being interest on, recurring deposit with SBI., Question 7(a), (4 Marks), (i) Discuss the provisions, relating to the premature withdrawal from Employees Provident Fund, under, section 192A, for A.Y. 2022-23., Answer: Refer Answer given in the book, (4 Marks), (ii) Mr. Barun provides you the following information and requests you to determine the Advance Tax, liability with due dates for the financial year 2021-22., Estimated tax liability for the financial year 2021-22, `65,000, Tax deducted at source for this year, ` 5,000
Page 369 :
Deduction Of Tax At Source, , 369, , MAY – 2016, Question 7(a)(i), (4 Marks), Ashwin a resident Individual carrying on business, furnishes you the following information:, Total turnover for the financial year, `, 2020-21, 120,00,000, 2021-22, 98,00,000, State whether tax deduction at source provisions are attracted for the under - mentioned expenses incurred, during the financial year 2021-22:, Particulars, `, Commission paid to Babloo, 18,500, Payment to Vijay for repair of office building, 23,000, Payment of fees for Technical Services, to Vivek, 35,000, All payments are made to residents., If Tax has to be deducted at source, state the amount of tax to be deducted at source., , NOV – 2015, Question 7(a)., (4 Marks), What are the consequences of failure to deduct or pay the tax under section 201 of the Income Tax Act,, 1961?, Refer Answer given in the Chapter, , MAY – 2015, Question 7(a)(ii)., (4 Marks), Mr. Madan sold his house property in Surat as well as his rural agricultural land for a consideration of `65, lakhs and `20 lakhs respectively, to Mr. Raman on 01.10.2021. He has purchased the house property for `40, lakhs and the land for `15 lakhs, in the year 2015. There was no difference in the stamp valuation. You are, required to determine TDS implications, if any, assuming both persons are resident Indians., Question 7(a)(iii)., What is the difference between TDS and TCS under the Income- tax Act, 1961?, Answer: Refer Answer given in the Chapter, , (4 Marks), , NOV – 2014, Question 7(a), (4 Marks), State in brief the applicability of tax deduction at source provisions, the rate and amount of tax deduction in, the following cases for the financial year 2021-22, (1) Payment of `27,000 made to a South African cricketer, by an Indian newspaper agency on 02.07.2021, for contribution of articles in relation to the sport of cricket., (2) Rent of `1,70,000 paid by a partnership firm for use of plant and machinery., (3) Winning from horse race `1,50,000., (4) `2,00,000 paid to Mr. X, a resident individual on 22.02.2022 by the State of Uttar Pradesh on, compulsory acquisition of his urban land., , MAY – 2014, Question 7(A)., (2 x 2 = 4 Marks), What are the provisions relating to tax deduction at source in respect of:, (A) ABC and Co. Ltd. Paid `19,000 to one of its Directors as sitting fees on 01.01.2022., (B) Mr. X sold his House to Mr. Y on 01.02.2022 for `60 lacs?, , NOV – 2012, Question No. 7(a), (i) Mr. X doing textiles business furnishes you the following information:, , (4 Marks)
Page 370 :
Deduction Of Tax At Source, , 370, , Total turnover for the financial year:, `, 2020-21, 201,00,000, 2021-22, 95,00,000, State whether the provisions of tax deduction at source are attracted for the following expenses incurred, during the financial year 2021-22;, Interest paid to Indian Bank on Term Loan, 92,800, 58,000, Advertisement expenses to Mr. X (two individual expense of `24,000 and `34,000), Factory rent paid to C, 2,85,000, Brokerage paid to B, a sub-broker, 16,000, , NOV – 2011, Question 7, State the applicability of TDS provisions and TDS amount in the following cases:, (a) Rent paid for hire of machinery by ABC Ltd. to Mr. X `3,10,000., (b) Fee paid to Dr. Y by X (HUF) `35,000 for surgery performed to a member of the family., , (4 Marks), , NOV – 2011, Question 6, (6 Marks), Compute amount of tax to be deducted at source on the following payments made by M/s ABC Ltd. during, the financial year 2021-22 as per the provisions of the Income-Tax Act, 1961., Sr. No. Date, Nature of Payment, (i), 01.10.2021, Payment of `2,00,000 to Mr. “X” a transporter who is having PAN, and who do not have more than 10 goods carriages., (ii), 01.11.2021, Payment of fee for technical services of `45,000 to Mr. X who is, having PAN., (iii), 30.12.2021, Payment of `25,000 to M/s X Ltd. for repair of building., (iv), 01.01.2022, Payment of `2,00,000 made to Mr. Y for purchase of diaries made, according to specifications of M/s ABC Ltd. However, no material, was supplied for such diaries to Mr. Y by M/s ABC Ltd., (v), 01.01.2022, Payment made `2,20,000 to Mr. Z for compulsory acquisition of his, house as per Law of the State Government., (vi), 01.02.2022, Payment of commission of 25,000 to Mr. A., , MAY – 2011, Question 5, List any 5 instances where the tax deductible at source in terms of section 194A will not apply., , (2 Marks), , Question 7, (4 Marks), Mr. X doing manufacture and wholesale trade furnishes you the following information:, Total turnover for the financial year, `, 2020-21, 215,00,000, 2021-22, 105,00,000, State whether tax deduction at source provisions are attracted for the below said expenses incurred during, the financial year 2021-22:, `, Interest paid to UCO Bank, 41,000, Contract payment to Mr. Y (2 contracts of `12,000 each), 24,000, Shop rent paid (one payee), 2,90,000, Commission paid to Mr. Z, 18,000
Page 371 :
Deduction Of Tax At Source, , 371, , NOV – 2010, Question 4, (4 Marks), Explain the consequences of not deducting tax and paying to Govt. account under section 201 of the Income, Tax Act, 1961., Refer Answer given in the Chapter, , MAY – 2010, Question 4, (2 Marks), State the concessions granted to transport operators in the context of deduction of tax at sources under, section 194-C., , MAY – 2010, Question 4, (4 Marks), State with reasons, whether tax deduction at source provisions are applicable to the following transactions, and if so, the rate of tax deduction:, (i) X & Co. (Firm) engaged in wholesale business assigned a contract for construction of its godown, building to Mr. X, a contractor. It paid `25,00,000 to Mr. X as contract payment., (ii) Y & Co. engaged in real estate business conducted a lucky dip and gave Maruti car to a prize winner., (iii) An Insurance Company paid `45,000 as Insurance Commission to its agent Mr. Y., (iv) AB Ltd. allowed a discount of `50,000 to XY & Co. (a firm) on prompt (immediate) payment towards, supply of automobile parts., , NOV – 2009, Question 2, (6 Marks), From the following particulars of Mr. X for the previous year ended 31st March, 2022 compute total income, and tax liability for assessment year 2022-23., `, (i), Directors Fee from a company, 3,00,000, (ii), Interest on saving bank deposits, 23,000, (iii) Winnings from Lotteries (Net), 35,000, 2,09,000, (iv) Royalty on a book written by him, (v), Lectures in Seminars, 5,000, (vi) Interest on loan given to a relative, 7,000, (vii) Interest on debentures of a company (listed in a Recognised Stock Exchange) net of, 36,000, TDS, (viii) Interest on Post Office Savings Bank Account, 500, (ix) Interest on Government Securities, 2,200, (x), Interest on Monthly Income Scheme of Post office, 33,000, He paid `10,000 for typing the manuscript of book written by him., Question 4, (4 Marks), Explain the consequences of not deducting tax and paying to Govt. account under section 201 of the Income, Tax Act, 1961., Refer Answer given in the Chapter, , NOV – 2009, Question 1, (2 Marks), State with reasons, whether the following statements are true or false having regard to the provisions of the, Income-tax Act, 1961, for the assessment year 2022-23:, An individual having gross receipts of `110 lacs during the financial year 2020-21 is not required to deduct, tax at source under section 194C of the Income-tax Act, 1961, on payment made to contractors during the
Page 372 :
Deduction Of Tax At Source, , 372, , financial year 2021-22., Question 3, (4 Marks), Mrs. X, a landlord, derived income from rent from letting a house property to M/s ABC Corporation Ltd. of, `1,00,000 per month. Calculate the deduction of tax at source (TDS) to be made by M/s ABC Corporation, Ltd. on payment made to Mrs. X and narrate related formalities in relation to TDS., , JUNE – 2009, Question 1, (2 Marks), When will tax not required to be deducted at source on interest payable to a resident on any bond or security, issued by a company though the aggregate amount of interest exceeds `5,000, the basic exemption limit, under section 193 of the Act?, , MAY – 2008, Question 1, (2 Marks), Mrs. X has made payments of ` 5 lacs to a contractor (for business purposes) during the last two quarters of, the year ended 31.03.2022. Her turnover for the year ended 31.03.2021 was `215 lacs. Is there any, obligation to deduct tax at source?, , NOV – 2007, Question 5, (4 Marks), Briefly explain the provisions of section 197 in respect of obtaining certificate for deduction of tax at a, lower rate., Refer Answer given in the Chapter, , MAY – 2007, Question 5, (4 Marks), What are the consequences of failure to deduct tax at source or pay the tax deducted at source to the credit of, Central Government?, Answer: Refer Answer given in the Chapter
Page 373 :
Deduction Of Tax At Source, , 373, , T.D.S./TCS TAX CHALLAN, CHALLAN NO., ITNS, 281, , Tax Applicable(Tick One)*, , Assessment Year, , TAX DEDUCTED/COLLECTED AT SOURCE FROM, (0020) Company, , (0021) Non-Company, , -, , Deductees, , Deductees, Tax Deduction Account No.(T.A.N.), Full Name, Complete Address with City & State, , Tel. No., , Pin, , Type of Payment, (Tick One), TDS/TCS Payable by Taxpayer, , (200), , TDS/TCS Regular Assessment (Raised by I.T. Deptt.), , (400), , DETAILS OF PAYMENTS, Income Tax, , FOR USE IN, RECEIVING BANK, Debit to A/c / Cheque credited on, , Amount (in Rs. Only), , _, Surcharge, , D D, , Health & education cess, Interest, , Y, , Y, , SPACE FOR BANK SEAL, , Penalty, Total, Total (in words), CRORES LACS, , M M, , THOUSANDS, , Paid in Cash/Debit to A/c /Cheque No., Drawn on, , HUNDREDS, , TENS, , UNITS, , Dated, , (Name of the Bank and Branch), Date:, Signature of person making payment, , Rs., SPACE FOR BANK SEAL, , Taxpayers Counterfoil (To be filled by the taxpayer), TAN, Received From, (Name), Cash/Debit to A/c / Cheque No., , For Rs., , Rs. (in words), drawn on, (Name of the Bank and Branch), Company/Non-Company Deductees, on account of Tax Deducted at Source(TDS)/Tax Collected at, Source(TCS), from___________(Fill up Code), , (Strike out whichever is not applicable), For the Assessment Year, -, , Rs.
Page 374 :
Miscellaneous Topics, , 374, , MISCELLANEOUS TOPICS, Assessee Section 2(7), “Assessee” means, • Any person who is liable to pay income tax or interest or penalty or any other sum under Income Tax, Act, • Any person with regard to whom any proceedings are pending under Income Tax Act for assessment, of income or loss or refund or any other proceeding, • Any person who is assessable on behalf of any other assessee i.e. deemed assessee and any proceeding, is pending with regard to such other person like assessment of income or loss or refund or any other, proceeding e.g. Minor son of Mr. X has income from talent `5,00,000, in this case Mr. X shall be, deemed to be an assessee., MAY – 2018, Question 7(b), (4 Marks), Briefly explain the purpose for which the words “PROVISO” and “EXPLANATION” are incorporated, under various sections of the Income Tax Act, 1961., Answer:, The Proviso to a section/sub-section/clause refers to sometimes additional provision, sometimes a special, condition and sometimes exceptions to the main provision, eg. Proviso to section 23(1) refers to deduction, of municipal tax but section 23(1) refers to computation of gross annual value i.e. it is an additional, provision. Similarly proviso to section 17(2) refers to medical facility but section 17(2) refers to different, facilities., The Explanation to a section/sub-section/clause gives a clarification relating to the provision contained in, the respective section/sub-section/clause, eg. Explanation to section 23(1) covers unrealised rent i.e. an, explanation to section 23(1) which refers to gross annual value., MAY – 2016, Question 2(a)(ii), How is the term “Assessee” defined under the Provisions of the Income - Tax Act, 1961?, Answer: Refer answer given above, , (4 Marks), , Assessment year, As per section 2(9), assessment year means a period of 12 months starting from 01st April of every year, ending with 31st March i.e. every financial year is an assessment year e.g. Financial year 2022-23 is one, assessment year., Previous year, As per section 3, previous year means financial year preceding the assessment year e.g. If financial year, 2022-23 is one assessment year, financial year 2021-22 is the previous year for such assessment year., Income of previous year is taxable in its assessment year i.e. exact tax liability for previous year 2021-22, shall be determined in assessment year 2022-23 (however the person has to pay advance tax on estimated, basis in previous year 2021-22.), In general previous year shall be of full year but in case of a newly setup business or profession, first, previous year will start from the date of commencement of business / profession e.g. If Mr. X started, business on 01.07.2021, first previous year shall be from 01.07.2021 to 31.03.2022., Income Section 2(24), Every person shall be required to pay tax on his income as per section 4 and the term income is divided into, 5 different categories which are called heads of income and such incomes shall include, 1. Payment by employer to employee., 2. Rent received or receivable in connection with house property, 3. Payments in connection with business/profession as per section 28, 4. Profit and gains on the transfer of capital asset as per section 45(1), 5. Incomes under section 56 like, dividend, interest, casual income, gift etc.
Page 375 :
Miscellaneous Topics, , 375, , 6. Any other income given under section 2(24)., Charging section of Income-Tax Section 4, Every person shall be liable to pay income tax on his income. Normal income of every person shall be, taxable at the rates given in the relevant Finance Act. Special incomes like Long term capital gains or Short, term capital gains under section 111A or casual income shall be taxable at the rates given in the Income Tax, Act. Tax shall be deducted at source as per the relevant provision also advance tax is to be paid as per the, relevant provision., Expenditure incurred in relation to income not includible in Total Income Section 14A, If any income is exempt from income tax, expenditure incurred in connection with such income shall not be, allowed to be deducted either from same income or from some other income. If expenditure is incurred for, taxable income as well as exempt income, only expenditure relating to taxable income shall be allowed to be, deducted., Question 1 [V. Imp.]: Discuss exceptions to the General Rule that the income of the Previous Year is, taxed in the Assessment Year., Answer: Exceptions to the General Rule that the income of the Previous Year is taxed in the, Assessment Year, Generally the income of the Previous Year is taxable in the immediately succeeding year called the, assessment year. But, in the following cases, the income of the current year may be brought to tax in the, same year–being exception to the general rule that incomes earned in the previous year are taxed in its, assessment year following: 1. Profits of non-resident from Shipping Business Section 172: If any ship owned by a non-resident, has entered in India, the ship shall not be allowed to leave India unless tax has been paid and return has been, filed, 2. Assessment of persons Leaving India Section 174: If any person is leaving India with no present, intention of returning to India, the total income of such individual up to the probable date of his departure, from India shall be chargeable to tax in the previous year itself., 3. Assessment of association of persons or Body of Individuals or Artificial Juridical Person formed, for a particular event or purpose Section 174A: If any association of persons or a body of individuals, etc. has been incorporated for a particular event or purpose and it is likely to be dissolved in the same year in, which it was formed, the total income of such association or body or juridical person for the period up to the, date of its dissolution shall be chargeable to tax in that year itself., 4. Assessment of persons likely to transfer property to avoid tax, Section 175: If it appears to the, Assessing Officer that any person is likely to sell, transfer or otherwise part with any of his assets with a, view to avoid payment of any liability under the provisions of this Act, the total income of such person shall, be taxable in the same previous year., 5. Discontinued Business Section 176: If any person has closed down his business/profession, such person, should inform Income Tax Department within 15 days of closing down such business/profession and the, Department may direct such a person to pay tax and file return in the previous year itself., 10(2) Share received by a member of Hindu undivided family from income of Hindu Undivided, Family, If any Hindu Undivided Family has paid income tax on the income of the family, such income shall not be, taxable in the hands of its members. E.g. Mr. X is a member of one HUF and has received `3,00,000 from, HUF as his share, it will be exempt from income tax under section 10(2)., 10(10BC) Compensation received or receivable on account of any disaster, As per section 10(10BC), if any person has received any payment from government or other similar, authority as compensation for loss or damage caused by any disaster whether natural calamity or any, accident etc., such compensation shall be exempt from income tax.
Page 376 :
Miscellaneous Topics, , 376, , NOV – 2016, Question 4(a), (2 Marks), State with reasons whether the following receipts are taxable or not under the provisions of Income-tax Act,, 1961?, Mr. Suri received a sum of `5,00,000 as compensation, from ‘Yatra Foundation’, towards the loss of, property on account of Flood Disaster at Chennai during December 2021., Answer:, As per section 10(10BC), if any person has received any payment from government or other similar, authority as compensation for loss or damage caused by any disaster whether natural calamity or any, accident etc., such compensation shall be exempt from income tax. In the given case compensation is not, received from Government hence the compensation received is taxable., MAY – 2016, Question 7(a)(iii), (2 Marks), Discuss with reason, whether the following transactions are true or false, as per the provisions of Income, Tax Act, 1961:, Any amount received by an individual or his legal heir as compensation for natural disaster from the, Government, is taxable., Answer:, False: As per section 10(10BC), if any person has received any payment from government or other similar, authority as compensation for loss or damage caused by any disaster whether natural calamity or any, accident etc., such compensation shall be exempt from income tax., 10(26AAA) Exemption in case of Income of an Individual being Sikkimese, As per section 10(26AAA), income of an individual of Sikkim shall be exempt from income tax if such, income is from Sikkim. If he has received dividend or interest on securities from any where, it will also be, exempt from income tax. If any woman from Sikkim has married an individual who is not a Sikkimese, in, that case such woman shall not be eligible for exemption., Question 2: Explain the Treatment of Unexplained money, investments etc., Answer: Unexplained money, investments etc. to attract tax @60% [Section 115BBE], (i) In order to control laundering of unaccounted money by availing the benefit of basic exemption limit,, the unexplained money, investment, expenditure, etc. deemed as income under section 68 or section 69 or, section 69A or section 69B or section 69C would be taxed at the rate of 60% plus surcharge @25% of tax., Thus, the effective rate of tax (including surcharge @25% of tax and HEC @4% of tax and surcharge) is, 78%., (ii) No basic exemption or allowance or expenditure shall be allowed to the assessee under any provision of, the Income-tax Act, 1961 in computing such deemed income., (iii) Further, no set off of any loss shall be allowable against income brought to tax under sections 68 or, section 69 or section 69A or section 69B or section 69C., (i) Cash Credits [Section 68]: Where any sum is found credited in the books of the assessee and the, assessee offers no explanation about the nature and source or the explanation offered is not satisfactory in, the opinion of the Assessing Officer, the sum so credited may be charged as income of the assessee of that, previous year., (ii) Unexplained Investments [Section 69]: Where in the financial year immediately preceding the, assessment year, the assessee has made investments which are not recorded in the books of account and the, assessee offers no explanation about the nature and the source of investments or the explanation offered is, not satisfactory in the opinion of the Assessing Officer, the value of the investments are taxed as deemed, income of the assessee of such financial year., (iii) Unexplained money etc. [Section 69A]: Where in any financial year the assessee is found to be the, owner of any money, bullion, jewellery or other valuable article and the same is not recorded in the books of, account and the assessee offers no explanation about the nature and source of acquisition of such money,
Page 377 :
Miscellaneous Topics, , 377, , bullion etc. or the explanation offered is not satisfactory in the opinion of the Assessing Officer, the money, and the value of bullion etc. may be deemed to be the income of the assessee for such financial year., (iv) Amount of investments etc., not fully disclosed in the books of account [Section 69B]: Where in any, financial year the assessee has made investments or is found to be the owner of any bullion, jewellery or, other valuable article and the Assessing Officer finds that the amount spent on making such investments or, in acquiring such articles exceeds the amount recorded in the books of account maintained by the assessee, and he offers no explanation for the difference or the explanation offered is unsatisfactory in the opinion of, the Assessing Officer, such excess may be deemed to be the income of the assessee for such financial year., Example: If the assessee is found to be the owner of say 300 gms of gold (market value of which is, `15,00,000) during the financial year ending 31.3.2022 but he has recorded to have spent `10,00,000 in, acquiring it, the Assessing Officer can add `5,00,000 as the income of the assessee, if the assessee offers no, satisfactory explanation thereof, (v) Unexplained expenditure [Section 69C]: Where in any financial year an assessee has incurred any, expenditure and he offers no explanation about the source of such expenditure or the explanation is, unsatisfactory in the opinion of the Assessing Officer, Assessing Officer can treat such unexplained, expenditure as the income of the assessee for such financial year. Such unexplained expenditure which is, deemed to be the income of the assessee shall not be allowed as deduction under any head of income., Question 3: Describe average rate of tax and maximum marginal rate under section 2(10) and 2(29C), of the Income- tax Act, 1961., Answer: As per section 2(10), “Average Rate of tax” means the rate arrived at by dividing the amount of, income-tax calculated on the total income, by such total income., Section 2(29C) defines “Maximum marginal rate” to mean the rate of income-tax (including surcharge on, the income-tax, if any) applicable in relation to the highest slab of income in the case of an individual, AOP, or BOI, as the case may be, as specified in Finance Act of the relevant year., Question 4: Explain Interest on notified securities and bonds issued to non-residents., Answer: Interest on notified securities and bonds issued to non-residents [Section 10(4)], As per section 10(4), in the case of an individual, any income by way of interest on moneys standing to his, credit in a Non-resident (External) Account (NRE A/c) in any bank in India, would be exempt., THE FINANCE ACT, Every year, the Finance Minister of the Government of India introduces the Finance Bill in the Parliament’s, Budget Session. When the Finance Bill is passed by both the houses of the Parliament and gets the assent of, the President, it becomes the Finance Act. Amendments are made every year to the Income-tax Act, 1961, and other tax laws by the Finance Act., The First Schedule to the Finance Act contains four parts which specify the rates of tax • Part I of the First Schedule to the Finance Act specifies the rates of tax applicable for the current, Assessment Year. (i.e. for earlier previous year), • Part II specifies the rates at which tax is deductible at source for the current Financial Year., • Part III gives the rates for current previous year, • Part IV gives the rules for computing net agricultural income., INCOME-TAX RULES, 1962, The administration of direct taxes is looked after by the Central Board of Direct Taxes (CBDT)., • The CBDT is empowered to make rules for carrying out the purposes of the Act., • For the proper administration of the Income-tax Act, 1961, the CBDT frames rules from time to time., These rules are collectively called Income-tax Rules, 1962., • Rules also have sub-rules, provisos and Explanations. The proviso to a Rule/Sub-rule spells out the, exception/conditions, to the Rule/Sub-rule. The Explanation gives clarification for the purposes of the Rule.
Page 378 :
Miscellaneous Topics, , 378, , CIRCULARS AND NOTIFICATIONS, Circulars, • Circulars are issued by the CBDT from time to time to deal with certain specific problems and to clarify, doubts regarding the scope and meaning of certain provisions of the Act., • Circulars are issued for the guidance of the officers and/or assessees., Notifications, Notifications are issued by the Central Government to give effect to the provisions of the Act. The CBDT is, also empowered to make and amend rules for the purposes of the Act by issue of notifications., Case Laws, Case Laws refer to decision given by courts. The study of case laws is an important and unavoidable part of, the study of Income-tax law. It is not possible for Parliament to conceive and provide for all possible issues, that may arise in the implementation of any Act. Hence the judiciary will hear the disputes between the, assessees and the department and give decisions on various issues., Assessment [Section 2(8)], This is the procedure by which the income of an assessee is determined. It may be by way of a normal, assessment or by way of reassessment of an income previously assessed. Assessment Procedure will be dealt, with in detail at the Final level., Payments to MPs & MLAs [Section 10(17)], The following incomes of Members of Parliament or State Legislatures will be exempt:, (i) Daily Allowance - Daily allowance received by any Member of Parliament or of any State Legislatures or, any Committee thereof., (ii) Constituency Allowance of MPs - In the case of a Member of Parliament, any allowance received under, Members of Parliament (Constituency Allowance) Rules, 1986; and, (iii) Constituency allowance of MLAs - Any constituency allowance received by any person by reason of his, membership of any State Legislature under any Act or rules made by that State Legislature, Income of member of a Scheduled Tribe [Section 10(26)], A member of a Scheduled Tribe residing in (i) any area specified in the Constitution i.e., The North Cachar Hills District, The Karbi Anglong District,, The Bodoland Territorial Areas District, Khasi Hills District, Jaintia Hills District or The Garo Hills District, or, (ii) in the States of Manipur, Tripura, Arunachal Pradesh, Mizoram and Nagaland, or, (iii) in Ladakh, is exempt from tax on his income arising or accruing –, (a) from any source in the areas or States aforesaid., (b) by way of dividend or interest on securities., Tea board subsidy [Section 10(30)], The amount of any subsidy received by any assessee engaged in the business of growing and manufacturing, tea in India through or from the Tea Board will be wholly exempt from tax., Conditions:, (a) The subsidy should have been received under any scheme for replantation or replacement of the bushes, or for rejuvenation or consolidation of areas used for cultivation of tea, as notified by the Central, Government., (b) The assessee should furnish a certificate from the Tea Board, as to the amount of subsidy received by, him during the previous year, to the Assessing Officer along with his return of the relevant assessment year, or within the time extended by the Assessing Officer for this purpose., Other subsidies [Section 10(31)]
Page 379 :
Miscellaneous Topics, , 379, , Amount of any subsidy received by an assessee engaged in the business of growing and manufacturing, rubber, coffee, cardamom or other specified commodity in India, as notified by the Central Government,, will be wholly exempt from tax., Conditions:, (a) The subsidies should have been received from or through the Rubber Board, Coffee Board, Spices Board, or any other Board in respect of any other commodity under any scheme for replantation or replacement of, rubber, coffee, cardamom or other plants or for rejuvenation or consolidation of areas used for cultivation of, all such commodities., (b) The assessee should furnish a certificate from the Board, as to the amount of subsidy received by him, during the previous year, to the Assessing Officer along with his return of the relevant assessment year or, within the time extended by the Assessing Officer for this purpose.
Page 380 :
Miscellaneous Topics, , 380, , MULTIPLE CHOICE QUESTIONS, 1. Which of the following income would be exempt in the hands of a Sikkimese Individual?, (a) only income from any source in the State of Sikkim, (b) only income by way of dividend, (c) only income from interest on securities, (d) All the above, 2. In case of a Member of Parliament –, (a) Daily allowance is exempt but constituency allowance received as per applicable Rules is taxable., (b) Constituency allowance received as per applicable Rules is exempt but daily allowance is taxable., (c) Both daily allowance and constituency allowance received as per applicable Rules are taxable., (d) Both daily allowance and constituency allowance received as per applicable Rules are exempt., 3. The quantum of deduction available under section 10AA in respect of profits and gains derived by a, SEZ unit from export of articles is –, (a) 100% of export profits for first 10 consecutive AYs and 50% for next 5 consecutive AYs, (b) 100% of export profits for first 5 consecutive AYs and 50% for next 10 consecutive AYs, (c) 100% of export profits for first 15 consecutive AYs, (d) 100% of export profits for first 5 consecutive AYs, 50% for export profits for next 5 consecutive AYs, and upto 50% of export profits for next 5 consecutive AYs, as is credited to Special Reserve Account, 4. Which of the following income is not exempt under section 10?, (a) Share income of a member from a HUF, (b) Share income of a partner from a firm, (c) Salary received by a partner from a firm, (d) Both (b) and (c), 5. Income under the Income-tax Act, 1961, is to be computed under (a) five heads, (b) six heads, (c) four heads, (d) seven heads, 6. Share of profit of Mr. P, who is a partner in M/s PQR, a firm resident in India, is –, (a) exempt from tax, (b) taxable as his business income, (c) taxable as his salary, (d) taxable as other sources, 7. Mr. A, whose total sales is `201 lakhs, declare profit of `10 lakhs for the F.Y. 2021-22 . He is liable, to pay advance tax(a) in one instalment, (b) in two instalments, (c) in three instalments, (d) in four instalments, 8. Mr. Ramanan, a resident aged 40 years, has a total income of `3,25,000 for A.Y.2022-23,, comprising of his salary income and income from house property. His tax liability for A.Y.2022-23, would be –, (a) `3,900 , (b) `1,300, (c) `2,600, (d) Nil, 9. Which of the following benefits are not allowable to Ms. Geetha, a non-resident, while computing, her total income and tax liability for A.Y.2022-23 under the Income-tax Act, 1961?, (a) Deduction of 30% of gross annual value while computing her income from house property in Bangalore,, India, (b) Tax rebate of `12,500 from tax payable on her total income of `3,40,000, (c) Deduction for donation made by her to Prime Minister’s National Relief Fund, (d) Deduction for interest earned by her on NRO savings account.
Page 381 :
Instruction to Examinees, , 381, , THE INSTITUTE OF CHARTERED ACCOUNTANTS OF INDIA, (EXAMINATION DEPARTMENT), INSTRUCTIONS TO EXAMINEES - July, 2021, TIMING OF EXAMINATIONS: 02.00 PM TO 05.00 PM (IST) on all days., Reading time (for question paper) starts at 1.45 P.M (IST), Do's, 1. Reach the exam Centre well in time., 2. Occupy the seat earmarked against the Roll Number., 3. Carry your own pen, stapler, ink, blotting paper, scale, and battery operated noiseless/cordless portable, calculator with upto 6 functions, 12 digits and upto two memories., 4. Use black ink ball point pen only., 5. Write Roll Number at specified place in the question paper., 6. Check that the question paper bears the Code as displayed on the notice/black board / announced in the, examination room/hall., 7. Check that the question paper is complete with reference to the number of pages mentioned thereon., 8. Write Roll Number at the specified place in the (main) answer book only., 9. Write answers in the medium (English/Hindi) opted and mentioned on the Admit Card., 10. write the answers to MCQs (in composite papers) on the OMR answer sheet only. Answer of MCQs, written on the descriptive answer book or in the MCQ booklet will not be taken cognizance of and will not, be evaluated., 11. Mark √ tick against the question numbers attempted in the cages provided on the front/cover page of the, answer book., 12. For paper 1,2 &5 submit descriptive answer book, for paper 3,4,6,7 & 8 submit (a) OMR answer sheet, (b) MCQ question paper booklet & (c) Descriptive answer without fail and take acknowledgement from the, invigilator before leaving the exam hall., 13. Fasten all answer books including additional answer books used with a tag and also staple them., 14. Sign the attendance register without fail., Don'ts, 1. Use ink, other than black sketch pen, highlighter for underlining or highlighting., 2. Make/Write any notings/rough work on the question paper., 3. Carry any paper, book, notes or any other written material in the examination room/hall., 4. Write Roll Number or Registration Number or name anywhere (except on the front page of the answer, book) in the answer book, including additional answer book(s), graph/blotting paper., 5. Make/write religious symbols/prayers like God's name, Guru's Name, OM, Swastika, 786 and the like, anywhere in the answer book, including additional answer book(s), graph/blotting paper., 6. Write / make extraneous (irrelevant/ unrelated) notes/remarks. Write / make appeal for marks etc., 7. Carry printing models or scientific calculator., 8. Exchange pen, scale, calculator etc., 9. Talk/communicate with any other examinees., 10. Tear and carry any page/leave from the answer book, including additional answer book., 11. Leave seat without permission., 12. Carry mobile phones or any other electronic gadget/device in any form [except calculator]., 13. Smoke, chew tobacco/betel nut, gum etc., 14. Write the answers of MCQs on the descriptive type answer., 15. Staple or tag the OMR answer sheet with the descriptive type answer book /MCQ booklet., 16. Keep the OMR answer sheet inside the MCQ booklet while submitting them to the invigilator. Submit, them separately, A. Before the Exam, 1. Immediately on receipt of the Admit Card, every candidate is expected to verify all the facts, i.e. Roll No.,, Registration Number, Name, Group or Groups or Unit, Examination Timing/Session, Dates of examination,, Sequence of Papers, Examination Centre etc. In case of doubt he/she is expected to approach the Joint, Secretary (Exams.), sufficiently in advance.
Page 382 :
Instruction to Examinees, , 382, , 2. Each candidate is advised to familiarise himself/herself with the location of the examination centre by, visiting the centre a day prior to the commencement of the examinations and to also satisfy that he/she has, visited/seen the allotted centre., 3. The doors of the examination halls will be opened half an hour before the time specified for the, commencement of the examination in each session. All candidates should be inside the examination hall, before the scheduled time of the commencement of reading time / the examination. It may be noted that the, Superintendents have been advised to verify the identity of each candidate at the time of his/her entry into, the examination hall. Candidates are, however, advised, in their own interest, to carry with them (along with, the admit card down-loaded from website) the photo identity cards (such as PAN card, Aadhar card etc.), issued by the government or the identity card issued to them by the Decentralized Office at the time of their, registration as articled assistants., 4. Exam centres have been instructed to undertake frisking of all candidates, before they enter the exam, hall, which may involve some time. Hence candidates are advised to reach the exam centre well in time., B. Rules relating to Entry & Exit from exam hall:, Late entry is permitted only upto 2:15 PM (IST), No candidate shall be permitted to leave the hall until the conclusion of the exam, even if he has completed, the paper., C. During the Exam:, 1. Candidates will find their roll numbers written against the seats allotted to them. They should find and, occupy their allotted seats., 2. Candidates are allowed 15 minutes reading time before the scheduled commencement of the examination., Accordingly question paper will be distributed at 1.45 p.m (I.S.T) and the answer book at 2.00 p.m (I.S.T), each day., 3. Immediately on receipt of the question paper, every candidate must write his/her Roll Number on his/her, copy of the question paper at the specified space provided on the cover page. By reference to the code of the, question paper displayed on the notice board, every candidate is expected to satisfy himself/herself that, he/she has received the correct question paper. By reference to the number of printed pages and the number, of questions in the question paper which will be found printed on the front page, every candidate is expected, to satisfy himself/herself that the question paper issued to him/her is complete., Similarly, if the answer book supplied is defective or the candidate has inadvertently written wrong Roll, Number etc. or at wrong place, the answer book should be exchanged before the commencement of, examination., 4. Candidates should use all the pages in the main answer book/s supplied, before asking for the additional, answer books. In order to avoid wastage, candidates will be issued additional answer books only after they, have used the main answer book. No candidate shall tear a leaf of an answer book. Tearing of any page of an, answer book is deemed to be an act of unfair means & will be dealt with accordingly. The candidate should, write answers on both sides of the paper. Rough work when necessary, should be done on the left hand side, on the pages of the answer book only. Rough work should not be done on the question paper. The candidates, are advised, in their own interest to ensure that working notes should form part of the answer/s., 5. Candidates should write the roll no in words and numerals inside the boxes and darken the corresponding, OMR circles provided on the cover page of the main answer book only i.e. Descriptive & OMR answer, sheet, in black ball point pen and in no other part of the answer book, additional sheet, graph paper etc. Any, violation of this instruction will tanta mount to adoption of unfair means and will attract punishment which, may include debarring from appearing in the examination. The attendance register contains the roll number, sticker of the candidate, in the Paper Number column, below the space where the candidate is required to, sign. Candidates should remove the correct roll number sticker of relevant paper/subject against his/her, name from the attendance register and affix the same within the box provided in the top right hand corner of, the cover page of the answer book. Since a machine will read the roll no. candidates should check and, ensure that the roll number written in words, numbers and circles darkened are correct. In case this, information is filled wrongly, Institute will not take any responsibility for rectifying the mistake. Candidates, should also affix their signature within the box provided for the purpose, on the cover page of answer book.
Page 383 :
Instruction to Examinees, , 383, , Candidate has also to additionally fill MCQ booklet serial number & MCQ booklet set code in the, attendance register wherever applicable., 6. Statistical/Mathematical tables to be provided and graph, paper, wherever needed, for solving the, questions are indicated on the question paper itself and would be made available to the candidates on, demand, by the Superintendent., 7., (a) The answers should be written neatly and legibly and should, as far as possible, be brief. However, avoid, use of SMS type of language, while writing your answers., (b) The answer to each question must be commenced on a fresh page and the question number clearly and, prominently written at the top of each answer. Candidates are advised in their own interest not to leave full, pages blank in between the answers. Candidates an advised to check the answer book carefully after, completing the paper & score off any blank pages, i.e. draw a line across the blank page, if any., (c) The answer to each question in all parts should be completed fully in one page, or in a consecutive set of, pages before the next question is taken up i.e. all parts of a question be done together., (d) Candidates are required to answer the requisite number of questions as per instruct ions printed on each, question paper. In case any candidate answers extra questions/sub questions over and above the required, number, then the requisite number of questions first answered in the answer book shall be valued and, subsequent questions answered extra will be ignored., (e) Candidates should write the answers only in black ink and in no other colour. Though there is no specific, prohibition against the use of Gel pen / Fountain pen, it will be advisable to use ball point pen only since its, ink is indelible. Candidates are also advised not to use red, green ink, highlighter, sketch pen etc. for, underlining or highlighting any sentence/Para/phrase as it amounts to making distinguishing mark which is, prohibited as stated in Para 17 below. Candidates are permitted to use pencils for drawing graphs, diagrams, etc. However, Candidates should take care not to use pencil for writing answers., 8. The candidates should write the question number and the Sub-question number, if any, very clearly., Candidate should thoroughly check & ensure that the question number written by him is correct, in his own, interest. The candidates should also leave a margin on each page wherever margin is not provided in the, answer book., 9. Candidates should not tie up the unused answer books along with the used books. Unused answer books, should be handed over to the invigilator., 10. The candidates are required to provide themselves with their own pen, HB pencil, eraser, stapler, ink and, blotting paper., 11. The answer book and additional sheets used if any, should be fastened together with a tag supplied for, the purpose. They should also be stapled. Candidates are to bring their stapler and staple them after tagging., The number of answer books used must be clearly stated in the space provided on the cover page of the, answer book., 12. Similarly, the number of questions attempted should also be indicated, by way of a tick (√) mark against, the question/s attempted, in the cages provided for the purpose on the cover page of the answer book., 13. The candidates should not write anything in the portion provided on the cover page for noting the marks., It is intended for the use of the examiner., 14. No candidate shall, without the special permission of the Superintendent/invigilator, leave his/her seat in, the examination hall during the hours of the examination. At the expiry of the time allowed for each paper,, the answer books (including the MCQ booklet and OMR answer sheet, wherever applicable) must be, surrendered immediately to the Invigilator concerned., 15. The Institute has extended the facility of obtaining confirmation, from the invigilator concerned for the, answer books surrendered. Candidates may, therefore, obtain the signature of the invigilator concerned,, immediately on submission of their answer books, in the relevant column on the area provided for the same, in the Admit Card., The Superintendents have been advised to issue receipt in the aforesaid manner through the invigilator, for, the answer books surrendered. Candidates should not leave the hall without surrendering their answer books, to the Invigilator or the Superintendent., 16. All the answer books including additional sheets written by the candidates should be fastened together, and also stapled before surrendering to the Invigilator or the Superintendent of the examination. Any
Page 384 :
Instruction to Examinees, , 384, , representation regarding omission to surrender the written answer book or any part thereof which, tantamounts to adoption of unfair means will not be entertained after the examination is over., 17. The candidates should write their Roll numbers only in the space provided on the cover page of the main, answer book / OMR answer sheet and in no other part of the answer books. They should not write Roll, Number in the additional answer books and graph paper. Writing of Roll number in place/s other than the, space provided for the purpose in the cover page shall tantamount to adoption of "unfair means". Similarly,, they should not make any distinguishing mark including religious symbols/prayers like God's name, any, telephone numbers, Guru's name, OM, Swastika, 786 etc. in any part of their answer books. Infringement of, these instructions is punishable, which may include debarring from appearing in the examination., 18. Similarly, the candidates are prohibited from writing their Articles registration Number, Name, any, extraneous notes, remarks, or appeals in their answer books and any violation shall tantamount to adoption, of unfair means., 19. Every candidate must sign the attendance sheets in the appropriate column against his/her Roll No. and, name and in no case shall leave the examination hall without signing these sheet., 20. The candidates should not write any matter on the question paper except their Roll Number. They should, not remove any paper or papers from the examination hall (except the descriptive type question paper given, to them.), 21. Smoking, chewing of tobacco/betel nut, intoxicant, etc is strictly prohibited in the examination hall., 22. Candidates have been allowed to answer in Hindi in all papers of a group or both Groups in entirety as, may be applicable and no paper-wise option is allowed. The option once exercised is final and cannot be, changed subsequently. In the absence of any clear option, English Medium is reckoned as the medium of, answering in the examination., In respect of final examination, the questions papers will be in English only for all the candidates though, Hindi medium candidates are allowed to write their answers in Hindi., Candidates opting for Hindi medium:, i) All questions including, parts, if any, in all the papers have to answered in Hindi medium only. However, candidates can write number, technical terms, phrases and figures in English and can also solve numerical, questions in English., ii) Candidates who opt for Hindi medium and answer any or all questions in English medium (except, numerical questions) will not get any credit for such question/s and zero marks will be awarded., Candidates opting for English medium:, i) All questions including parts, if any, in all the parts have to be answered in English medium only., ii) Candidates who opt for English medium and answer any or all questions in Hindi medium will not get, any credit for such question/s and zero marks will be awarded., 23. Students are allowed to use battery operated portable calculators in all the subjects. The calculators can, be of any type with up to 6 functions, 12 digits and up to two memories. (Attempt to use any other type of, calculators not complying with the specifications indicated above or having more features than mentioned, above shall tantamount to use of "unfair means" and would fall within the purview of paragraph F stated, below.), Note:, i) Printing models of calculators are not allowed., ii) Exchange of calculators between the students is not permitted., iii) The calculators should be noiseless and cordless., iv) The Superintendent of the examination has complete authority to disallow the use of a particular, calculator not complying with the conditions stated above. (It may be noted that each step/working of any, problem should invariably be indicated by the candidate in the answer book, irrespective of use of, calculator. Candidates are advised to follow this instruction in their own interest.), v) Scientific calculators are not allowed., vi) Candidates are advised not to bring the pager, cellular phone, digital diary, smart watch or other, electronic gadgets /device in any form inside the examination hall except the calculator as defined above., Violation of these instructions shall tantamount to adoption of unfair means and the candidates will be liable, for punishment which may include debarring from appearing in the examination.
Page 385 :
Instruction to Examinees, , 385, , 24. No candidate shall bring with him/her into the examination hall or carry on his/her person any paper,, book, notes or any other material, nor shall he/she communicate with any other candidate in the examination, hall or in the premises, when the examination is in progress. Any infringement of this instruction is likely to, result in the candidate concerned being expelled from the examination hall or otherwise dealt with., MOBILE PHONES OR ANY OTHER ELECTRONIC GADGETS/DEVICE IN ANY FORM [EXCEPT, THE CALCULATOR AS DEFINED IN PARA 23 OF THESE INSTRUCTIONS] ARE BANNED IN THE, EXAMINATION CENTRES, D. Rules Relating To Exemptions:, 1. Candidates are advised to go through the rules relating to "Exemption in a paper(s)" contained in the, Guidance Notes hosted along with the Examination form, CA Regulations 1988 and the "Frequently Asked, Questions (FAQs)" hosted on the institute's website www.icai.org and be clear about their exemption, entitlements and not miss out on appearing in those paper/s where they are not eligible for exemption., Candidates can also check the status of the paper(s) in which they are eligible for exemption in the, forthcoming examination, at http://icaiexam.icai.org., E. Act of Unfair Means:, Violation of Instructions to Examinees as contained above tantamounts to adoption of unfair means. The, nature of such violation, inter alia, includes the following., 1. Writing/jotting on the question paper (other than Roll Number at the specified place), 2. Writing in the answer book or additional book of, e.g. Roll Number [other than at the specified space]/, Registration Number, Name, Mobile number, unwarranted Remarks, irrelevant notes etc., 3. Possession of material inside the examination hall/room/washroom, e.g. writing/copying material / books, / notes / writing on desk/writing on writing pad/geometric box/admit card (relevant for the day of the, examination or otherwise), mobile phone [in switched off mode or otherwise], I Pod etc., 4. Seeking sympathy/making appeal, e.g. parent or relative passed away, met with accident /was, hospitalized/ award marks/minimum required marks, inducement to examiner/writing irrelevant / unrelated, remarks etc., 5. Writing/making in the answer book or additional answer book distinguishing marks - e.g. religious, symbols, prayers, Om, Swastika, 786, etc., 6. Leaving the examination hall without submitting the (1) OMR answer sheet (2) MCQ booklet and (3), Descriptive type answer book to the invigilator;, 7. Misbehaving with the examination functionaries, 8. Using of different inks/highlighter, 9. Answering the questions in different hand writing on different pages, 10. Tearing off any sheet from the answer book, 11. In the case of persons with disabilities, using the services of a writer who does not fulfill the eligibility, requirements by making a false declaration regarding his educational qualifications of a writer., 12. Using ink, other than black, sketch pen, highlighter for underlining or high lighting., 13. Carrying any paper, book, notes or any other written material in the examination room/hall., 14. Writing / making extraneous (irrelevant/ unrelated) notes/remarks. Write / make appeal for marks etc., 15. Carrying printing models or scientific calculator, 16. Exchange pen, scale, calculator etc., 17. Talking/communicate with any other examinees., 18. Tearing and carrying any page/leave from the answer book, including additional answer book., 19. Leaving seat without permission., 20. Carrying mobile phones or any other electronic gadget/device in any form [except calculator]., 21. Smoking, chewing tobacco/betel nut, gum etc., 22. If a candidate is found to have resorted to or has made attempts to resort to unfair means pertaining to an, examination, the Council may, on receipt of a report to that effect and after such investigation as it may, deem necessary, take such disciplinary action against the candidate concerned as it may think fit. The, Superintendent of the examination has absolute power to expel a candidate from the examination hall, if in, his opinion the candidate has adopted or attempted to adopt unfair means in connection with the, examination. Any candidate expelled from the examination hall must, before leaving the hall, submit to the, Council his/her explanation in writing through the Superintendent of the examination.
Page 386 :
Instruction to Examinees, , 386, , The above cases will be considered by the Examination Committee in accordance with the provisions of, Regulation 41, read with Regulation 176, of the Chartered Accountants Regulations, 1988. The decision, taken by the Committee includes cancellation of result and debarment from appearing in the examination in, future., In view of the above, candidates are advised to read the instructions carefully and familiarize themselves, with the same to avoid falling within the ambit of unfair means leading to avoidable difficulties., G. General:, In case of any inadvertent mistake in printing or framing of a question in a paper, candidates can bring it to, the notice of the Joint Secretary (Exams), within a week from the last date of examination.
Page 387 :
387, , Special provision for collection of tax at source for non-filers, of income-tax return, Section 206CCA, (1) Notwithstanding anything contained in any other provisions of this, Act, where tax is required to be collected at source under the provisions, of Chapter XVII-BB, on any sum or amount received by a person, (hereafter referred to as collectee) from a specified person, the tax shall, be collected at the higher of the following two rates, namely:—, (i) at twice the rate specified in the relevant provision of the Act; or, (ii) at the rate of five per cent., (2) If the provisions of section 206CC is applicable to a specified person,, in addition to the provisions of this section, the tax shall be collected at, higher of the two rates provided in this section and in section 206CC., (3) For the purposes of this section "specified person" means a person, who has not filed the returns of income for both of the two assessment, years relevant to the two previous years immediately prior to the previous, year in which tax is required to be collected, for which the time limit of, filing return of income under sub-section (1) of section 139 has expired;, and the aggregate of tax deducted at source and tax collected at source in, his case is rupees fifty thousand or more in each of these two previous, years:, Provided that the specified person shall not include a non-resident who, does not have a permanent establishment in India., Explanation.—For the purposes of this sub-section, the expression, "permanent establishment" includes a fixed place of business through, which the business of the enterprise is wholly or partly carried on.
Page 388 :
388, ILLUSTRATION NO. 12 OF ICAI STUDY MATERIAL PAGE NO. 9.63, , Mr. Gupta, a resident Indian, is in retail business and his turnover for F.Y.2020-21 was ₹12 crores., He regularly purchases goods from another resident, Mr. Agarwal, a wholesaler, and the aggregate, payments during the F.Y.2021-22 was ₹ 95 lakh (₹ 20 lakh on 1.6.2021, ₹ 25 lakh on 12.8.2021, ₹ 22, lakh on 23.11.2021 and ₹ 28 lakh on 25.3.2022). Assume that the said amounts were credited to Mr., Agarwal’s account in the books of Mr. Gupta on the same date. Mr. Agarwal’s turnover for F.Y.202021 was ₹ 15 crores., (1) Based on the above facts, examine the TDS/TCS implications, if any, under the Income-tax, Act, 1961., (2) Would your answer be different if Mr. Gupta’s turnover for F.Y.2020-21 was ₹ 8 crores, all, other facts remaining the same?, (3) Would your answer to (1) and (2) change, if PAN has not been furnished by the buyer or seller,, as required?, Solution:, (1) Since Mr. Gupta’s turnover for F.Y.2020-21 exceeds 10 crores, and payments made by him to, Mr. Agarwal, a resident seller exceed ₹ 50 lakhs in the P.Y.2021-22, he is liable to deduct tax @, 0.1% of ₹ 45 lakhs (being the sum exceeding ₹ 50 lakhs) in the following manner –, No tax is to be deducted u/s 194Q on the payments made on 1.6.2021 and 12.8.2021, since the, aggregate payments till that date i.e. 45 lakhs, has not exceeded the threshold of ₹ 50 lakhs., Tax of ₹ 1,700 (i.e., 0.1% of ₹ 17 lakhs) has to be deducted u/s 194Q from the payment/ credit of, ₹ 22 lakh on 23.11.2021 [₹ 22 lakh – ₹ 5 lakhs, being the balance unexhausted threshold limit]., Tax of ₹ 2,800 (i.e., 0.1% of ₹ 28 lakhs) has to be deducted u/s 194Q from the payment/ credit of, ₹ 28 lakhs on 25.3.2022., Note – In this case, since both section 194Q and 206C(1H) applies, tax has to be deducted u/s, 194Q., (2) If Mr. Gupta’s turnover for the F.Y.2020-21 was only ₹ 8 crores, TDS provisions under section, 194Q would not be attracted. However, TCS provisions under section 206C(1H) would be, attracted in the hands of Mr. Agarwal, since his turnover exceeds ₹ 10 crores in the F.Y.2020-21, and his receipts from Mr. Gupta exceed ₹ 50 lakhs., No tax is to be collected u/s 206C(1H) on 1.6.2021 and 12.8.2021, since the aggregate receipts, till that date i.e. 45 lakhs, has not exceeded the threshold of ₹ 50 lakhs., Tax of ₹ 1,700 (i.e., 0.1% of ₹ 17 lakhs) has to be collected u/s 206C(1H) on 23.11.2021 (₹ 22, lakh – ₹5 lakhs, being the balance unexhausted threshold limit)., Tax of ₹2,800 (i.e., 0.1% of ₹28 lakhs) has to be collected u/s 206C(1H) on 25.3.2022., (3) In case (1), if PAN is not furnished by Mr. Agarwal to Mr. Gupta, then, Mr. Gupta has to deduct, tax @ 5%, instead of 0.1%. Accordingly, tax of ₹ 85,000 (i.e., 5% of ₹17 lakhs) and ₹1,40,000, (5% of ₹28 lakhs) has to be deducted by Mr. Gupta u/s 194Q on 23.11.2021 and 25.3.2022,, respectively., In case (2), if PAN is not furnished by Mr. Gupta to Mr. Agarwal, then, Mr. Agarwal has to, collect tax@1% instead of 0.1%. Accordingly, tax of ₹17,000 (i.e., 1% of ₹17 lakhs) and ₹ 28,000, (1% of ₹ 28 lakhs) has to be collected by Mr. Agarwal u/s 206C(1H) on 23.11.2021 and, 25.3.2022, respectively.
Page 389 :
389, , TAXATION PAPER, DEC – 2021, ETM2, Roll No………………………., Total No. of printed Pages – 6, Total No. of Questions – 8, Maximum Marks – 70, GENERAL INSTRUCTIONS TO CANDIDATES, 1. The question paper comprises two parts, Part I and Part II., 2. Part I comprises Multiple Choice Question (MCQs)., 3. Part II comprises questions which require descriptive type answers., 4. Ensure that you receive the question paper relating to both the parts. If you have not received both, bring it, to the notice of the invigilator., 5. Answers to Questions in Part I are to be marked on the OMR answer sheet given on the Cover Page of, Section A of descriptive type answer book. Answers to questions in Part II are to be written on the same, descriptive type answer book. Answers to MCQs, if written inside the descriptive answer book, will not be, evaluated., 6. OMR answer sheet given on the Cover Page of Section A of descriptive answer book will be in English, only for all candidates, including for Hindi medium candidates., 7. The bar coded sticker provided in the attendance register, is to be affixed only on the descriptive type answer, book., 8. You will be allowed to leave the examination hall only after the conclusion of the exam. If you have, completed the paper before time, remain in your seat till the conclusion of the exam., 9. Duration of the examination is 3 hours. You will be required to submit (a) Part I of the question paper, containing MCQs, and (b) the answer book in respect of descriptive type answer book with OMR Cover Page, to the invigilator before leaving the exam hall, after the conclusion of the exam., 10. The invigilator will give you acknowledgement on Page 2 of the admit card, upon receipt of the abovementioned items., 11. Candidate found copying or receiving or giving any help or defying instructions of the invigilators will be, expelled from the examination and will also be liable for further punitive action., PART II, 1. Section – A comprises questions 1-4. In Section – A, answer question No.1 which is compulsory and any, 2 questions from question Nos. 2-4. All questions in Section – A relate to assessment year 2022-23, unless, otherwise stated., Section – B comprises questions 5-8. In Section – B, answer question No.5 which is compulsory and any 2, questions from question Nos. 6-8, 2. Working notes should form part of the answer., 3. Answers to the questions are to be given only in English except in the case of candidates who have opted, for Hindi Medium. If a candidate has not opted for Hindi Medium, his/her answers in Hindi will not be, evaluated., 4. All questions in Section – B should be answered on the basis of position of GST law as amended by, significant notification/circulars issued upto 31st Oct, 2021., PART – II, SECTION – A, 1. Mr. Shivansh, a resident and ordinarily resident aged 61 years, is engaged in the business of manufacturing, of motor parts. He is subject to tax audit under section 44AB of Income Tax Act, 1961. He has provided, following information:, Profit & Loss account for the year ended 31st March, 2022, Particulars, (₹), Particulars, (₹), To Administrative expenses, 4,30,000 By Gross Profit, 58,30,000
Page 390 :
390, To Salaries & wages, To Interest on loans, , 20,00,000 By Profit on sale of asset of scientific, research, 7,50,000 By Winning from lottery, (Net of TDS @ 30%), 6,17,000, 2,70,000, 2,80,000, 1,40,000, 15,74,500, 60,61,500 Total, , 2,00,000, 31,500, , To Depreciation, To Professional fees, To Rent, rates & taxes, To Travelling & conveyance, To Net Profit, Total, 60,61,500, Explanatory information:, (i), Opening and closing stock of finished goods were undervalued by 10%. Opening stock of, ₹4,50,000 and closing stock of ₹5,58,000 was shown., (ii), Salaries & wages include following items:, (a) Contributed 20% of basic salary in National Pension Scheme referred to section 80CCD, regarding salary paid to an employee Mr. Ganesh who has withdrawn basic salary of, ₹3,00,000, and Dearness allowance is 40% of basic salary. 50% of Dearness allowance forms, part of the salary., (b) Some of the employees opted for retirement under the voluntary retirement scheme; a sum of, ₹2,40,000 was paid to them on 1st January, 2022., (iii) Interest on loan includes interest paid @ 15% per annum on loan of ₹12,00,000 which was taken, from State Bank of India on 01.05.2021 for purchased of new electric car of ₹15,00,000. The car, is used for personal purpose., (iv), Depreciation allowable as per Income Tax Rules, 1962 is ₹4,50,000 but during the calculation of, such depreciation following addition was not considered:, Motor car purchased for ₹3,00,000 for supply of finished goods to dealers on 25.08.2020., (v), An asset was purchased for ₹6,00,000 on 17.11.20 for conducting scientific research and the, deduction was claimed under section 35 of the Income-tax Act, 1961. This asset was sold on, 05.09.2021 for a consideration of ₹8,00,000., Other information:, A plot of Industrial land which was used by Mr. Shivansh for business purpose for last 10 years was, compulsorily acquired by Central Government on 07.05.2021. The compensation of ₹ 12,00,000 was received, on 27.02.2022. Such property was purchased by him on 08.08.2005 for ₹2,00,000. He has purchased another, plot of industrial land on 21.04.2022 for ₹6,00,000. Government has also paid ₹54,000 as interest on such, compensation on 28.03.2022., Cost Inflation Indices: FY 2021-22: 317, FY 2005-06: 117, Compute the total income and tax liability of Mr. Shivansh for the assessment year 2022-23 assuming that he, has not opted provisions of section 115BAC. Ignore Provisions relating to AMT., (14 Marks), 2. (a) Examine the tax implications of the following transactions for the assessment year 2022-23: (Give brief, reason), (i) Government of India has appointed Mr. Rahul as an ambassador in Japan. He received salary of ₹7,50,000, and allowances of ₹2,40,000 during the previous year 2021-22 for rendering his services in Japan. He is, an Indian citizen having status of non-resident in India for the previous year 2021-22., (ii) Ms. Juhi, a non-resident in India is engaged in operations which are confined to purchase of goods in, India for the purpose of export. She has earned ₹2,50,000 during the previous year 2021-22., (iii) Mr. Naveen, a non-resident in India, has earned ₹3,00,000 as royalty for a patent right made available to, Mr. Rakesh who is also a non-resident. Mr. Rakesh has utilized patent rights for development of a product, in India and 50% royalty is received in India and 50% outside India.
Page 391 :
391, (iv) Mr. James, a NRI, borrowed ₹10,00,000 on 01.04.2021 from Mr. Akash who is also non-resident and, invested such money in the shares of an Indian Company. Mr. Akash has received interest @ 12% per, annum., (7 Marks), (b) Ms. Mishika has entered into an agreement with M/s CVM Build Limited on 25.04.2017 in which she, agrees to allow such Company to develop a shopping mall on land owned by her in New Delhi. She purchased, such land on 05.05.2009 in ₹15,00,000. In consideration, M/s CVM Build Limited will provide 20% share in, shopping mall to Mishika. The certificate of completion of shopping mall was issued by authority as on, 26.12.2021. On such date, Stamp duty value of shopping mall was ₹4,14,00,000. Subsequently on 18.03.2022,, she sold her 15% share in shopping mall to Mr. Ketav in consideration of ₹65,00,000., She has also purchased a house on 09.05.2021 in consideration of ₹46,00,000 and occupied for own residence., Punjab National Bank has sanctioned a loan of ₹35,50,000 (80% of stamp value) at the interest rate of 12%, per annum on 01.05.2021 and disbursement was made on 01.06.2020. She does not own any other residential, house on the date of sanction of loan. Principal amount of ₹1,30,000 was paid during the financial year 202021., Cost Inflation Indices: 2021-22: 317,2009-10: 148, Compute total income of Ms. Mishika for the assessment year 2022-23 assuming that she has not opted, provisions under section 115BAC., (7 Marks), 3.(a) State in brief the applicability of provisions of tax deduction at source, the rate and amount of tax, deduction in the following cases for the financial year 2021-22 under Income Tax Act, 1961. Assume that all, payments are made to residents:, (i) Mr. Mahesh has paid ₹6,00,000 on 15.10.2021 to M/s Fresh Cold Storage Pvt. Ltd. for preservation of, fruits and vegetables. He is engaged in the wholesale business of fruits & vegetable in India having, turnover of ₹3 Crores during the previous year 2020-21., (ii) Mr. Ramu, a salaried individual, has paid rent of ₹60,000 per month to Mr. Shiv Kumar from 1st July, 2021, to 31st March, 2022. Mr. Shiv Kumar has not furnished his Permanent Account Number., (4 Marks), (b) Examine the following transactions with reference to applicability of the provision of tax collected at, source and the rate and amount of the TCS for the Assessment year 2022-23., (i) Mr. Kalpit bought an overseas tour programme package for Singapore for himself and his family of ₹5, lakhs on 01.11.2021 from an agent who is engaged in organising foreign tours in course of his business., He made the payment by an account payee cheque and provided the permanent account number to the, seller. Assuming Kalpit is not liable to deduct tax at source under any other provisions of the Act., (ii) Mr. Anu doing business of textile as a proprietor. His turnover in the business is ₹11 crores in the previous, year 2020-21. He received payment against sale of textile goods from Mr. Ram ₹75 lakhs against the sales, made to him in the previous year and proceeding previous years. (Assuming all the sales are domestic, sales and Mr. Ram is neither liable to deduct tax on the purchase from Mr. Anu nor he deducted any tax, at source)., (4 Marks), (c) Mr. Ravi, a resident and ordinarily resident in India, owns a let out house property having different flats in, Kanpur which has municipal value of ₹27,00,000 and standard rent of ₹29,80,000. Market rent of similar, property is ₹30,00,000. Annual rent was ₹40,00,000 which includes ₹10,00,000 pertaining to different, amenities provided in the building. One flat in the property (annual rent is 2,40,000) remains vacant for 4, months during the previous year. He has incurred following expenses in respect of aforesaid property:, Municipal taxes of ₹4,00,000 for the financial year 2021-22 (10% rebate is obtained for payment before due, date.) Arrears of municipal tax of financial year 2020-21 paid during the year of ₹1,40,000 which includes, interest on arrears of ₹25,000., Lift maintenance expenses of ₹2,40,000 which includes a payment of ₹30,000 which made in cash., Salary of ₹88,000 paid to staff for collecting house rent and other charges. ., Compute the total income of Mr. Ravi for the assessment year 2022-23 assuming that Mr. Ravi has not opted, provisions under section 115BAC., (6 Marks)
Page 392 :
392, 4. (a) Details of Income of Mr. R and his wife Mrs. R for the previous year 2021-22 are as under :, (i) Mr. R transferred his self-occupied property without any consideration to the HUF of which he is a, member. During the previous year 2021-22 the HUF earned an income of ₹50,000 from such property., (ii) Mr. R transferred ₹4,00,000 to his wife Mrs. R on 01.04.2006 without any consideration which was given, as a loan by her to Mr. Girish. She earned ₹3,50,000 as interest during the earlier previous years which, was also given as a loan to Mr. Girish. During the previous year 2021-22, she earned interest @ 11% per, annum., (iii) Mr. R and Mrs. R both hold equity shares of 27% and 25% respectively in AMG Limited. They are also, working as employees in such Company. During the financial year 2021-22 they have withdrawn a salary, of ₹3,20,000 and ₹ 2,70,000 respectively., (iv) Mrs. R transferred 5,000 equity shares of RSB Ltd. on 17.09.2013 to Mr. R without any consideration., The Company issued 3,000 bonus shares to Mr. R in 2016. On 04.03.2022, Mr. R sold entire share, holdings and earned ₹5,20,000 as capital gains., Apart from above income, Mr. R has income from commission ₹4,00,000 and Mrs. R has interest income of, ₹3,30,000., Compute Gross Total income of Mr. R and Mrs. R for the assessment year 2022-23., (4 Marks), (b) Mr. X, an employee of the Central Government is posted at New Delhi. He joined the service on 1st, February, 2017. Details of his income for the previous year 2021-22, are as follows :, (i) Basic salary: ₹3,80,000, (ii) Dearness allowance : ₹1,20,000 (40% forms part of pay for retirement benefits), (iii) Both Mr. X and Government contribute 20% of basic salary to the pension scheme referred to in Section, 80CCD., (iv) Gift received by X’s minor son on his birthday from friend : ₹70,000. (No other gift is received by him, during the previous year 2021-22), (v) During the year 2013-14, Mr. X gifted a sum of ₹6,00,000 to Mrs. X. She started a business by, introducing such amount as her capital. On 1st April, 2021, her total investments in business was, ₹10,00,000. During the previous year 2021-22, she has loss from such business ₹1,30,000., (vi) Mr. X deposited ₹70,000 in Sukanya Samridhi account on 23.01.2022. He also contributed ₹40,000 in, an approved annuity plan of LIC to claim deduction u/s 80CCC., (vii) He has taken an educational loan for his major son who is pursuing MBA course from Gujarat, University. He has paid ₹15,000 as interest on such loan which includes ₹5,000 for the financial year, 2020-21., Determine the total income of Mr. X for the assessment year 2022-23. Ignore provisions under section, 115BAC., (6 Marks), (c) Mr. Kailash, a resident and ordinarily resident in India, could not file his return of Income for the, assessment year 2022-23 before due date prescribed under section 139(1). Advise Mr. Kailash as a tax, consultant:, What are the consequences for non-filing of return of Income within the due date under section 139(1)?, (4 Marks), OR, Mr. Sitaram is engaged in the business of trading of cement having turnover of ₹10 crores during the financial, year 2021-22. As a tax consultant advise him what are the particulars to be furnished under section 139(6A), along with Return of Income?, (4 Marks), SECTION-B, 5. M/s ABC Ltd., a registered supplier in Surat, Gujarat and it has calculated output net GST liability after, adjusting ITC in the books for the month of February 2022:, CGST : ₹3,00,000, SGST : ₹2,50,000, IGST : ₹3,00,000, During the above month, the following additional information provided by M/s ABC Ltd:
Page 393 :
393, S. No., 1, , Particulars, , Amount, (excluding, GST) ₹, The company had given on hire 5 trucks to one of the transporters of Vadodara, 3,75,000, (a goods transport agency) for transporting goods for 10 days. The hiring, charges for the trucks were ₹7,500 per truck per day, , 2, , The company sold goods to X & Co. of Delhi on 6th January 2022 with a, condition that interest @ 2% per month will be charged on invoice value if X, & Co. failed to make payment within 30 days of the delivery of the goods., Goods were delivered and also the invoice was issued on 6th January 2022. X, & Co. paid the consideration for the goods on 20th February along with, applicable interest., , 5,00,000, , 3, , The company sought legal consultancy services for it’s business from A &, Advocates, a partnership firm of advocates situated at Bhuj, Gujarat., , 1,50,000, , 4, , The company ordered 3,000 packets of tools which are to be delivered by the, supplier of Delhi via 3 lots of 1,000 packets monthly. The supplier raised the, invoice for full quantity in February 2022 and the last lot would be delivered, in April 2022., , 5,00,000, , 5, , The company supplied 10,000 packets of tools to one of it’s customer at ₹10/- per packet in, Gujarat in January 2022. Afterwards, the company re-values it at ₹9 per packet in February 2022, and the company issued credit note to the customer for ₹1 per packet., , The rate of GST is 9% CGST, 9% SGST and 18% IGST., You are required to compute the actual net liability of GST to be paid in cash along with working notes for, the month of February 2022., (8 Marks), 6. (a) M/s AB Ltd., a registered company of Chennai, Tamil Nadu has provided following services for the, month of October 2022, Particulars, Amount, (₹), Services of transportation of students, faculty and staff from home to college and back to, Commerce College, (a private college) providing degree courses in BBA, MBA, B.Com.,, M.Com, , 2,50,000, , Online monthly magazine containing question bank and latest updates in law to students, of PQR Law College offering degree courses in LLB and LLM, Housekeeping services to T Coaching Institute, Security services to N Higher Secondary School, Services of providing breakfast, lunch and dinner to students of ABC Medical College, offering degree courses recognized by law in medical field, , 1,00,000, 50,000, 3,25,000, 5,80,000, , All the above amounts are exclusive of GST., Compute the taxable supplies of M/s AB Ltd. for the month of October 2022 with necessary explanations., (6 Marks), (b) Q Ltd. is engaged exclusively in supply of taxable goods from the following states. The particulars of, intra-state supplies for the month of May 2022 are as follows :, STATE, TURNOVER (₹), Madhya Pradesh, , 5,00,000
Page 394 :
394, Gujarat, , 14,00,000, , Tripura, 12,00,000, (i) Q Ltd. seeks to know whether it is liable for registration under GST. Give your explanation., (ii) Will your answer be different if Q Ltd. supplies only petrol & diesel from Tripura instead of any other, taxable goods?, (4 Marks), 7.(a)(i) An order is placed to T & Co., Sholapur on 18th August, 2021 for supply of fabrics to make garments., Company delivered the fabrics on 4th September, 2021 and after completion of the order issued the invoice on, 15th September, 2021. The payment against the same was received on 30th September, 2021. Determine the, time of supply for the purpose of payment under CGST Act, 2017 with your explanations., (ii) HM Industries Ltd. engaged the services of a transporter for road transport of a consignment on 20th May, 2021. However, the consignment could not be sent immediately on account of a strike in the factory, and, instead was sent on 20th July 2021. Invoice was received from the transporter on 20th June 2021 and payment, was made on 25th August 2021., What is the time of supply of the transporter’s service?, (5 Marks), (b) M/s PQR Ltd., have filed their GSTR3B return for the month of August, 2021 within the due date i.e., 20.09.2021. It was noticed in October 2021 that tax dues for the month of August, 2021 have been short paid, for by ₹10,000. The shortfall of ₹10,000 was paid through cash ledger and credit ledger amounting to ₹7,500, and ₹2,500 respectively while filing GSTR3B of October 2021 which was filed on 20.11.2021., (i) Examine and compute the Interest payable if any under the CGST Act, 2017., (ii) What would be your answer if, GSTR-3B for the month of August 2021 had been filed belatedly on, 20.11.2021 and the tax of ₹10,000/- has been paid on 20.11.2021 as above., Note: Ignore the effect of the leap year. Electronic cash ledger and credit ledger carried sufficient balance for, the above shortfall., (5 Marks), 8. (a) Mr. Q, a Casual Taxable Person of Gujarat state is a trader of taxable notified handicraft goods. It makes, supplies to the states of Maharashtra, Rajasthan and Andhra Pradesh. Turnover for October 2021 is 18 Lakhs., (i) Explain the provisions of registration for Casual Taxable person under GST. Examine whether Mr. Q is, liable for registration or not?, (ii) What will be the answer if Mr. Q makes trading in taxable notified products instead of taxable notified, handicraft goods which involves 75% making on machine and 25% by hand?, (5 Marks), (b) Is Dynamic Quick Response (QR) Code applicable to suppliers who issue invoice to unregistered persons?, If no, list the suppliers to whom Dynamic QR Code is not applicable., (5 Marks), OR, (i) What is ‘e-invoicing’?, (ii) What is the threshold limit for mandatory issuing of E-invoice for all registered businesses?, (iii) A consignor hands over his goods for transportation on a Friday to the transporter. However, assigned, transporter starts the movement of goods from consigner’s warehouse to its depot located at distance of 600, Km. on Monday., When will the e-way bill be generated and for how many days it will be valid?, (5 Marks)
Page 395 :
395, CORRECTION IN INCOME TAX VOLUME 1 (47th Edition) PAGE NO. 16, (CORRECTED PART IS GIVEN IN BOLD ITALICS AND COLOURED), , (ii) Total income, On first ₹2,50,000, On next ₹2,50,000 @ 5%, On balance ₹3,00,000 @ 20%, Tax before health and education cess, Add: health & education cess @ 4%, Tax Liability, , 8,00,000, Nil, 12,500, 60,000, 72,500, 2,900, 75,400, , (iii) Total income, On first ₹2,50,000, On next ₹2,50,000 @ 5%, On balance ₹5,00,000 @ 20%, Tax before health and education cess, Add: health & education cess @ 4%, Tax Liability, , 10,00,000, Nil, 12,500, 1,00,000, 1,12,500, 4,500, 1,17,000, , (iv) Total income, On first ₹2,50,000, On next ₹2,50,000 @ 5%, On next ₹5,00,000 @ 20%, On balance ₹2,00,000 @ 30%, Tax before health and education cess, Add: health & education cess @ 4%, Tax Liability, , 12,00,000, Nil, 12,500, 1,00,000, 60,000, 1,72,500, 6,900, 1,79,400, , (v) Total income, On first ₹2,50,000, On next ₹2,50,000 @ 5%, On next ₹5,00,000 @ 20%, On balance ₹10,00,000 @ 30%, Tax before health and education cess, Add: health & education cess @ 4%, Tax Liability, , 20,00,000, Nil, 12,500, 1,00,000, 3,00,000, 4,12,500, 16,500, 4,29,000, , Question 3: Explain Health and Education Cess, Answer: Health and Education Cess, If any tax is charged for any specific purpose, it is called Cess. Health and Education Cess shall be charged @, 4% on the amount of income tax., Rounding off of Tax Section 288B, Any amount payable, and the amount of refund due, shall be rounded off in the multiples of ₹10 in the similar, manner as in case of total income under section 288A., Question 4: Explain Previous Year and Assessment Year, Answer: Every person has to pay tax on Income of a particular financial year and such year is called previous, year. Further computation of income and tax liability is computed in the subsequent year and it is called, assessment year, eg. if income is to be computed for financial year 2021-22, it will be called previous year, and subsequent year i.e. 2022-23 shall be called assessment year. The term previous year is defined u/s 3 and, assessment year is defined u/s 2(9).
Page 396 :
396, CORRECTION IN INCOME TAX VOLUME 1 (47th Edition) PAGE NO. 53, (CORRECTED PART IS GIVEN IN BOLD ITALICS AND COLOURED), , Tax Before Surcharge, Add: Surcharge @ 15% on 12,50,000, Add: Surcharge on Dividend Income, (208,12,500/7,00,00,000 x 1,00,00,000=29,73,214.29) X 15%, Add: Surcharge on PGBP Income, (208,12,500/7,00,00,000 x 6,00,00,000 = 178,39,285.71) X 37%, Tax Before cess, Add: Health and education cess @ 4%, Tax Liability, Rounded off u/s 288B, (c), Solution:, Computation of Total Income and Tax Liability of Mr. X, Income under the head PGBP, LTCG 112A, STCG 111A, Dividend from domestic company, Gross Total Income, Less: Deductions u/s 80C to 80U, Total Income, Computation of Tax Liability, Tax on LTCG 50,00,000 (51,00,000-1,00,000) @ 10% u/s 112A, Tax on STCG 50,00,000 @15% u/s 111A, Tax on 1,90,00,000 at slab rate, Tax Before Surcharge, Add: Surcharge @ 15%, Tax Before cess, Add: Health and education cess @ 4%, Tax Liability, Example 2: Mr. X has income as given below:, Income under the head Salary, LTCG 112A, STCG 111A, Dividend from domestic company, Compute his tax liability for A.Y. 2022-23., (b) Suppose income under the head Salary ₹300,00,000, (c) Suppose income under the head Salary ₹500,00,000, Solution:, Computation of Total Income and Tax Liability of Mr. X, Income under the head Salary, LTCG 112A, STCG 111A, Dividend from domestic company, Gross Total Income, Less: Deductions u/s 80C to 80U, Total Income, Computation of Tax Liability, Tax on LTCG 20,00,000 (21,00,000-1,00,000) @ 10% u/s 112A, Tax on STCG 10,00,000 @15% u/s 111A, Tax on 1,80,00,000 at slab rate, Tax Before Surcharge, , 220,62,500.00, 1,87,500.00, 4,45,982.14, 66,00,535.71, 292,96,517.85, 11,71,860.71, 304,68,378.56, 304,68,380.00, ₹, 90,00,000, 51,00,000, 50,00,000, 100,00,000, 291,00,000, Nil, 291,00,000, 5,00,000.00, 7,50,000.00, 55,12,500.00, 67,62,500.00, 10,14,375.00, 77,76,875.00, 3,11,075.00, 80,87,950.00, ₹150,00,000, ₹21,00,000, ₹10,00,000, ₹30,00,000, , ₹, 150,00,000, 21,00,000, 10,00,000, 30,00,000, 211,00,000, Nil, 211,00,000, 2,00,000.00, 1,50,000.00, 52,12,500.00, 55,62,500.00
Page 397 :
397, CORRECTION IN INCOME TAX VOLUME 1 (47th Edition) PAGE NO. 57, (CORRECTED PART IS GIVEN IN BOLD ITALICS AND COLOURED), , Example 6: Mr. X has income from business ₹ 550 lakhs and short term capital gain under section 111A, ₹ 30 lakhs and long term capital gains under section 112A ₹ 41 lakhs, in this case tax liability shall be, computed in the manner given below, Solution:, Income under the head business/profession from business, 550,00,000, Income under the head capital gains, Short term capital gains under section 111A, 30,00,000, Long term capital gains under section 112A, 41,00,000, Gross total income/total income, 621,00,000, Computation of Tax Liability, Total Income, 550,00,000, Tax on ₹550,00,000 at slab rate, 163,12,500, Add: Surcharge @ 37%, 60,35,625, Tax on normal income, 223,48,125, Tax on short term capital gain under section 111A 30,00,000 X 15%, Add: Surcharge @ 15%, Tax on long term capital gain under section 112A (41,00,000 – 1,00,000) X 10%, Add: Surcharge @ 15%, Tax liability before HEC, Add: HEC @ 4%, Tax Liability, , 4,50,000, 67,500, 4,00,000, 60,000, 233,25,625, 9,33,025, 242,58,650, , Question 10: Write a note on taxability of income of Partnership Firm/Limited Liability Partnership, Firm., Answer: Partnership firm/LLP, Long term capital gains are taxable @ 20%, STCG u/s 111A shall be taxable @ 15% , LTCG u/s 112A shall, be taxable in excess of 1,00,000 @ 10% and casual income @ 30% and other incomes are also taxable @, 30%., Surcharge shall be applicable @ 12% provided total income is exceeding ₹ 1 crore., Marginal Relief, Marginal relief shall be allowed if income has exceeded ₹100 lakhs., Health & education cess is applicable @ 4%, Deductions under section 80C to 80U shall be allowed in the normal manner., Partnership firm is regulated through Partnership Act,1932 and Limited Liability Partnership firm is regulated, through Limited Liability Partnership Act, 2008., Question 11: Write a note on taxability of income of domestic company., Answer: Domestic Company, Long term capital gains are taxable @ 20%, STCG u/s 111A shall be taxable @ 15% , LTCG u/s 112A shall, be taxable in excess of 1,00,000 @ 10% and casual income @ 30% and other incomes are also taxable @, 30%., Surcharge shall be applicable, - @ 7% provided total income is exceeding ₹100 lakhs but it is upto ₹1000 lakhs, - @ 12% provided total income is exceeding ₹1000 lakhs., Marginal relief shall be allowed if income has exceeded ₹100 lakhs / 1000 lakhs, Health & education cess is applicable @ 4%, Deductions under section 80C to 80U shall be allowed in the normal manner., (If total turnover or gross receipts in P.Y. 2019-20 does not exceed 400 crores, tax rate shall be 25% instead
Page 398 :
398, CORRECTION IN INCOME TAX VOLUME 1 (47th Edition) PAGE NO. 64, (CORRECTED PART IS GIVEN IN BOLD ITALICS AND COLOURED), , Question 14: Write a note on Computation of Tax Liability of HUF., Answer: Tax liability of Hindu undivided family, Hindu undivided family means any family which is Hindu by religion and its senior most male member is, called karta and karta is responsible for control and management of HUF. Parental property / business etc, received by karta shall be considered to be common property and taxability shall be as given below: Normal, income of Hindu undivided family shall be computed at the normal slab rate as given below:, Income shall be taxable at the slab rates given below:, If total Income upto ₹2,50,000, NIL, On next ₹2,50,000, 5%, On next ₹5,00,000, 20%, On Balance amount, 30%, Slab rate of senior citizen is not applicable to HUF even if age of Karta or its member is exceeding 60 years, or 80 years., Surcharge shall be applicable, @ 10% if total income has exceeded ₹50 lakhs but upto ₹100 lakhs., @ 15% if total income has exceeded ₹100 lakhs but upto ₹200 lakhs., @ 25% if total income has exceeded ₹200 lakhs but upto ₹500 lakhs., @ 37% if total income has exceeded ₹500 lakhs., Surcharge of 25% or 37% shall be applicable only if total income excluding short term capital gain under, section 111A and long term capital gain under section 112A and dividend income, is exceeding ₹ 200 Lakhs, or ₹ 500 Lakhs, All other provisions shall be similar to individual but rebate under section 87A is not allowed. Tax rates for, LTCG /LTCG 112A/ STCG u/s 111A and casual income are the same for all the persons., If normal income of resident HUF is less than the exemption limit, the difference of the amount shall be, allowed to be deducted from long term capital gain and if long term capital gains are not sufficient, it will be, allowed to be adjusted from short term capital gains under section 111A or long term capital gains u/s 112A, but it will not be allowed to be adjusted from casual income., (What is HUF is given in the Hindu Law and it is not covered in the syllabus), Example, XY HUF has income under the head business/profession ₹20 lakhs and its Karta Mr. X has individual income, ₹12 lakhs, in this case tax liability of HUF and that of Karta shall be, Tax liability of HUF ₹20 lakhs at slab rate, 4,12,500, Add: HEC @ 4%, 16,500, Tax Liability, 4,29,000, Tax Liability of Karta ₹12 lakhs at slab rate, 1,72,500, Add: HEC @ 4%, 6,900, Tax Liability, 1,79,400, Question 15: Write a note on Computation of Tax Liability of Body of Individuals/Association of, Persons., Answer: Tax liability of BOI/AOP, Body of individual means a group of individuals which is neither a company nor a partnership firm. If it is, registered in some other Act, it will be called incorporated BOI. E.g. cooperative society or Trust etc. If such, a group includes persons other than individual also, it will be called AOP., In general normal income shall be taxable at normal slab rate but rate may change as per provisions of section, 167B. (NOT COVERED IN SYLLABUS), Surcharge shall be applicable, @ 10% if total income has exceeded ₹50 lakhs but upto ₹100 lakhs., @ 15% if total income has exceeded ₹100 lakhs but upto ₹200 lakhs., @ 25% if total income has exceeded ₹200 lakhs but upto ₹500 lakhs., @ 37% if total income has exceeded ₹500 lakhs.
Page 399 :
399, CORRECTION IN INCOME TAX VOLUME 1 (47th Edition) PAGE NO. 65, (CORRECTED PART IS GIVEN IN BOLD ITALICS AND COLOURED), , Surcharge of 25% or 37% shall be applicable only if total income excluding short term capital gain under, section 111A and long term capital gain under section 112A and dividend income, is exceeding ₹ 200 Lakhs, or ₹ 500 Lakhs, Deductions under section 80C to 80U shall be allowed in the normal manner., Question 16: Write a note on Computation of Tax Liability of Local Authority., Answer: Tax liability of local authority, In order to maintain any town or city, there is always some authority responsible and such authority is called, local authority e.g. MCD in Delhi. Such authority is allowed to collect house tax with regard to every type of, house property and also some other tax are collected by such authority. In general income of such authority is, exempt from income tax under section 10(20) but if such authority is doing any business, its income is taxable, just like a partnership firm. Deductions under section 80C to 80U shall be allowed in the normal manner., Question 17: Explain meaning of Person Section 2(31)., Answer: Meaning of Person Section 2(31), “Person” includes—, (i) an individual,, (ii) a Hindu undivided family,, (iii) a company,, (iv) a firm,, (v) an association of persons or a body of individuals, whether incorporated or not,, (vi) a local authority,, (vii) every artificial juridical person, not covered above and income is taxable as slab rate (juridical means, legal) e.g. ICAI or Delhi University etc., Question 18 [V. Imp.]: Discuss Partial Integration of Agricultural Income?, Or, Discuss Indirect Taxing of Agricultural Income?, Or, Under the Constitution, the power to levy a tax on agricultural income vests in the States. However,, Parliament has also levied a tax on such income. Explain how this has been achieved?, Answer:, Agricultural Income Section 10(1), Under section 10(1), any agricultural income in India is fully exempt from income tax but if the agricultural, income is from outside India, it is chargeable to tax. (As per entry no. 82 of Union List, Central Government, has the power to levy income tax on income except agricultural income and power to levy tax on agricultural, income has been given to the State Government vide entry no. 46 of State List), Indirect taxing of agricultural income or partial integration of agricultural income (Under the, constitution, the power to levy a tax on agricultural income vests in the states. However, parliament has, also levied a tax on such income. Explain how this has been achieved?), If any person has agricultural income as well as non-agricultural income, his tax liability shall be computed, in the manner given below:, 1. Compute tax on the total of agricultural income and non- agricultural income considering it to be total, income of the assessee., 2. Compute tax on exemption limit (₹2,50,000 / 3,00,000 / 5,00,000) and agricultural income considering, it to be total income., 3., Deduct tax computed under Step 2 from Step 1 and apply surcharge if any and allow rebate if any and, health & education cess., 4., Long term capital gain, casual income and short term capital gain u/s 111A shall not be taken into, consideration for the purpose of partial integration
Page 400 :
400, CORRECTION IN INCOME TAX VOLUME 1 (47th Edition) PAGE NO. 76, (CORRECTED PART IS GIVEN IN BOLD ITALICS AND COLOURED), , MULTIPLE CHOICE QUESTIONS, 1. The basic source of income-tax law is (a) Income-tax Act, 1961, (b) Income-tax Rules, 1962, (c) Circulars/Notifications issued by CBDT, (d) Judgments of Courts, 2. A domestic company means (a) Only an Indian company, (b) Both Indian company and a foreign company having a branch in India, (c) Both Indian company and a foreign company having business connection in India, (d) Both Indian company and a foreign company which has made the prescribed arrangement for declaration, and payment of dividends in India out of the income chargeable to tax in India, 3. The rates of income-tax are mentioned in (a) The Income-tax Act, 1961 only, (b) Both Income-tax Act, 1961 and Income-tax Rules, 1962, (c) The First Schedule to the Annual Finance Act, (d) Both Income-tax Act, 1961 and the First Schedule to the Annual Finance Act, 4. The surcharge applicable in the case of an individual is (a) 10% of tax payable if total income exceeds ₹50 lakhs but does not exceed ₹1 crore, (b) 10% of tax payable if total income exceeds ₹1 crore but does not exceed ₹2 crore, (c) 15% of tax payable if total income exceeds ₹1 crore but does not exceed ₹2 crore, (d) Both (a) and (c), as the case may be., 5. In respect of a non-resident assessee, who is of the age of 60 years or more but less than 80 years at, any time during the previous year 2021-22, (a) Basic exemption of ₹2,50,000 is available, (b) Basic exemption of ₹3,00,000 is available, (c) Basic exemption of ₹5,00,000 is available, (d) No basic exemption limit would be available, 6. In case of a domestic company whose gross receipts for the P.Y. 2019-20 is upto ₹400 crores, the, rate of tax applicable is (a) 29% , (b) 25% , (c) 30% , (d) None of the above, 7. The surcharge applicable to a domestic company for A.Y. 2022-23 is (a) 5%, if total income exceeds ₹1 crore., (b) 10%, if the total income exceeds ₹1 crore, (c) 7%, if the total income exceeds ₹1 crore but does not exceed ₹10 crore, and 15%, if the total income, exceeds ₹10 crore., (d) 7%, if the total income exceeds ₹1 crore but does not exceed ₹10 crore, and 12%, if the total income, exceeds ₹10 crore., 8. The surcharge applicable to a foreign company for A.Y. 2022-23 is (a) 5%, if the total income exceeds ₹1 crore., (b) 10%, if the total income exceeds ₹1 crore., (c) 2%, if the total income exceeds ₹1 crore but does not exceed ₹10 crore and 5% if the total income exceeds, ₹10 crore., (d) 2%, if the total income exceeds ₹10 crore., 9. The rate of tax applicable to a partnership firm for A.Y. 2022-23 is (a) 25%, (b) 30%, (c) 35%, (d) 40%, 10. Where the total income of an artificial juridical person is ₹3,10,000, the income-tax before cess, payable is ₹............... and surcharge payable is ₹.............., (a) ₹3,000; surcharge – nil.
Page 401 :
401, CORRECTION IN INCOME TAX VOLUME 1 (47th Edition) PAGE NO. 90, (CORRECTED PART IS GIVEN IN BOLD ITALICS AND COLOURED), , than 10% but if stamp duty value is ₹ 111,00,000, taxable amount shall be ₹ 11,00,000 because stamp duty is, exceeding by more than 10% of actual consideration., If the date of the agreement fixing the amount of consideration for the transfer of immovable property and the, date of registration are not the same and in such cases, the stamp duty value on the date of the agreement shall, be taken into consideration but part of consideration should have been paid by account payee cheque, an, account payee bank draft or by use of electronic clearing system through a bank account or through such, other electronic modes as may be prescribed. (Other electronic mode means Credit Card, Debit Card, Net, Banking, IMPS (Immediate Payment Service), UPI (Unified Payment Interface), RTGS (Real Time Gross, Settlement), NEFT (National Electronic Funds Transfer), and BHIM (Bharat Interface for Money), Aadhaar Pay) on or before the date of agreement. E.g. Mr. X has entered into agreement with a builder ABC, Limited on 01.07.2016 for purchase of one building for ₹20,00,000 but stamp duty value was ₹27,00,000 and, advance of ₹3,00,000 was given by account payee cheque but property was transferred in his name on, 01.07.2021 and on that date stamp duty value was ₹35,00,000, in this case amount of gift shall be ₹7,00,000, (27,00,000 – 20,00,000). (Difference amount is more than ₹50,000 and more than 10% of the consideration)., Similarly, it will also be considered to be normal income., The gift is exempt in the following cases, (a) If any individual has received any gift from any of his relative, it will be exempt from income tax. The, term relative shall include, (a) spouse of the individual;, (b) brother or sister of the individual;, (c) brother or sister of the spouse of the individual;, (d) brother or sister of either of the parents of the individual;, (e) any lineal ascendant or descendant of the individual; (ascendant means mother/ father/ grand mother /, grand father and so on: Descendant means son / daughter / grand son / grand daughter etc., (f) any lineal ascendant or descendant of the spouse of the individual;, (g) spouse of the person referred to in items (b) to (f), Whether mother’s parents shall be included in lineal ascended is a question of law., (b) If any individual has received any gift from any person of any amount on the occasion of his/her marriage., If gift is received by the parents of such individual, in that case it will be taxable. If any individual has received, gift on the occasion of anniversary, it will be taxable., (c) If any person has received any gift under a will/ inheritance, it will be exempt from income tax., (d) in contemplation of death of the payer or donor (Contemplation of Death means the apprehension of an, individual that his life will end in the immediate future by a particular illness etc.), (e) from any local authority or charitable hospital or charitable educational institution or charitable trust or, other similar organisation., Question 2: Explain meaning of property., Answer:, "PROPERTY" means the following capital asset of the assessee, namely:—, (i) immovable property being land or building or both;, (ii) shares and securities;, (iii) jewellery;, (iv) archaeological collections (relating to past/ ancient), (v) drawings (a picture or diagram made with a pencil, pen, or crayon without paint.), (vi) paintings;, (vii) sculptures;, (viii) any work of art; or, (ix) bullion (Gold and silver in the form of biscuits / bricks / bars), If any person has received gift of any other property, it will not be taxable e.g. motor car or plant and, machinery or a watch or a mobile phone etc., E.g. Mr. X received a mobile phone valued ₹70,000 from his friend, in this case, it will be exempt from
Page 402 :
402, CORRECTION IN INCOME TAX VOLUME 1 (47th Edition) PAGE NO. 208, (CORRECTED PART IS GIVEN IN BOLD ITALICS AND COLOURED), , Municipal taxes paid, 10%, 10%, Fire Insurance paid, 1,500, 650, Ground rent due, 700, 900, Land revenue paid, 600, 800, Interest payable on capital borrowed for purchase of flat, 45,000, Nil, Income of Mr. X from his proprietary business–warehousing corporation is ₹7,00,000. Determine the total, income and tax liability for the assessment year 2022-23, you are informed that Mr. X could not occupy flat, for 2 months commencing from December 1st, 2021 and that he has attained the age of 82 on 23.08.2021., Answer = Total Income: ₹6,70,000; Tax Liability: ₹35,360, Problem 24., Mr. X and Mr. Y constructed their houses on a piece of land purchased by them at New Delhi. The built up, area of each house was 1,000 sq. ft. ground floor and an equal area at the first floor., Mr. X started construction of the house on 01.04.2020 and completed it on 31.03.2021. Mr. X occupied the, entire house on 01.04.2021. Mr. X has availed a housing loan of ₹25 lakhs @ 12% p.a. on 01.04.2020 and has, also submitted a certificate from the lender certifying the amount of interest., Mr. Y started construction on 01.04.2020 and completed it on 30.06.2021. Mr. Y occupied the ground floor, on 01.07.2021 and let out the first floor for a rent of ₹20,000 per month. However, the tenant vacated the, house on 31.12.2021 and Mr. Y occupied the entire house during the period 01.01.2022 to 31.03.2022. Mr. Y, has availed a housing loan of ₹15 lakhs @ 10% p.a. on 01.07.2020 and has also submitted a certificate from, the lender certifying the amount of interest., Following are the other information:, ₹, (i) Fair rental value of each unit, 1,20,000 Per annum, (Ground floor / first floor), (ii) Municipal value of each unit, 92,000 Per annum, (Ground floor / first floor), (iii) Municipal taxes paid by, X, - 10,000, Y, - 10,000, (iv) Repair and maintenance charges paid by, X, - 30,000, Y, - 32,000, No repayment was made by either of them till 31.03.2022. Compute income from house property for Mr. X, and Mr. Y for the previous year 2021-22 (assessment year 2022-23)., Answer = Mr. X: ₹ (2,00,000); Mr. Y: ₹ (92,000), Problem 25., Mrs. X is the owner of a house property. She borrowed ₹60,000 from life insurance corporation of India on, 1st September 2014 @ 15% p.a. for the construction of this house. The construction was completed on, 31.03.2017. Since then the house is under her self-occupation. On 1st June 2021 the house was let out @, ₹3,000 p.m. The tenant vacated the house on 1st August 2021. She occupied the house for self-occupancy. The, house is again let out @ ₹3,500 p.m. from 1st October 2021., Other particulars of the house for the previous year 2021-22., ₹, Municipal Valuation, 22,000 p.a., Municipal taxes disputed, hence not paid, 2,200 p.a., Ground rent for the previous year 2021-22 outstanding, 3,200, Insurance premium paid, 1,200, Refund of first loan instalment to LIC on 01.10.2021, 15,000, Compute the income from house property for assessment year 2022-23., Answer = Income under the head House Property: ₹11,025, Problem 26., Mr. X owns a residential house property. It has two identical units—unit I and unit II. Unit I is self–occupied, by Mr. X and his family members, unit II is let out (rent being ₹7,500 per month, this unit remained vacant, for one month during which it was self-occupied). Municipal value of the property is ₹1,30,000. Standard
Page 403 :
403, CORRECTION IN INCOME TAX VOLUME 1 (47th Edition) PAGE NO. 246, (CORRECTED PART IS GIVEN IN BOLD ITALICS AND COLOURED), , Example 1: Mr. X received royalty of ₹2,00,000 from abroad for a book authored by him which is a work of, artistic nature. The rate of royalty is 20% of value of books and expenditure made for earning this royalty was, ₹50,000. The amount remitted to India till 30th September, 2021 is ₹1,20,000. Compute deduction u/s 80QQB, and also compute income to be added in Gross Total Income., Solution:, Amount to be added in Income (2,00,000-50,000), 1,50,000, Deduction allowed u/s 80QQB, Deductions u/s 80QQB, 15% of value of books, 1,50,000, (2,00,000/20% x 15%), but cannot exceed amount brought in India within 6, months from the end of the previous year i.e. 1,20,000, Allowed, 1,20,000, Less: Expenses, (50,000), Deduction allowed, 70,000, , 70,000, , Example 2: Mr. X received royalty of ₹6,00,000 from abroad for a book authored by him which is a work of, artistic nature. The rate of royalty is 25% of value of books and expenditure made for earning this royalty was, ₹2,50,000. The amount remitted to India till 30th September, 2022 is ₹3,80,000. Compute deduction u/s, 80QQB and also compute income to be added in Gross Total Income., Solution:, Amount to be added in Income (6,00,000-2,50,000), 3,50,000, Deduction allowed u/s 80QQB, Deductions u/s 80QQB, 15% of value of books, 3,60,000, (6,00,000/25% x 15%), but cannot exceed amount brought in India within 6, months from the end of the previous year i.e. 3,80,000, Allowed, 3,60,000, Less: Expenses, (2,50,000), Deduction allowed, 1,10,000, , 1,10,000, , Example 3: Mr. X received royalty of ₹10,00,000 from ABC Ltd. situated in India for a book authored by, him which is a work of artistic nature. The rate of royalty is 10% of value of books and expenditure made for, earning this royalty was ₹5,00,000. Compute deduction u/s 80QQB and also compute income to be added in, Gross Total Income., Solution:, Amount to be added in Income (10,00,000-5,00,000), 5,00,000, Deduction allowed u/s 80QQB, Deductions u/s 80QQB, 3,00,000, Royalty received, 10,00,000, Less: Expenses, (5,00,000), Deduction allowed, 5,00,000, But maximum ₹3,00,000, Example 4: Mr. X received royalty (lumpsum) of ₹6,00,000 from abroad for a book authored by him which, is a work of artistic nature. Expenditure made for earning this royalty was ₹2,50,000. The amount remitted to, India till 30th September, 2022 is ₹4,00,000. Compute deduction u/s 80QQB and also compute income to be, added in Gross Total Income.
Page 404 :
Revised on 22.10.2021, , CA EDUCATION, 9811429230 / 9212011367, WWW.MKGEDUCATION.COM, , PRACTICE PROBLEMS, SOLUTIONS INCOME TAX, (Volume – 1), , , , , , , , , , , , , COMPUTATION OF TOTAL INCOME AND TAX LIABILITY, 02-25, TAXABILITY OF GIFT, 26-29, ADVANCE PAYMENT OF TAX, 30-33, RESIDENTIAL STATUS & SCOPE OF TOTAL INCOME, 34-50, INCOME UNDER THE HEAD HOUSE PROPERTY, 51-76, DEDUCTION FROM GROSS TOTAL INCOME, 77-88, AGRICULTURAL INCOME, 89-93, CLUBBING OF INCOME, 94-97, INCOME UNDER THE HEAD OTHER SOURCES, 98-104, DEDUCTION OF TAX AT SOURCE, 105-115, MISCELLANEOUS TOPICS, 116-116, After the book has been published, some error/mistake etc. may be detected/or there, may be some amendments etc., all such corrections/amendments shall be uploaded on, our website. Students are requested to visit our website in order to update the book, , 47th Edition, CA (INTER), , MAY-2022/NOV-2022, P.Y. 2021-22, A.Y. 2022-23, F.A. – 2021, , Author, This Book is the result of combined efforts of, Chartered Accountants/ company executives /, other professionals / feedback of our thousands of students, ₹750
Page 405 :
Computation of Total Income And Tax Liability, , 2, , COMPUTATION OF TOTAL, INCOME AND TAX LIABILITY, SOLUTIONS OF MCQS, 1. (a); 2. (d); 3. (d); 4. (d); 5. (a); 6. (b); 7. (d); 8. (c); 9. (b); 10. (a); 11.(b); 12.(a); 13.(a); 14.(a);, 15.(c);16.(c); 17.(e); 18.(d); 19(c); 20.(d); 21 (c); 22 (c); 23 (a); 24 (a); 25 (a); 26 (a); 27 (a); 28 (c); 29 (b);, 30 (c);, Hint for answer 2. As per rule 27, meaning of making prescribed arrangement for declaration and payment, of dividends in India means that the company has compiled with the following conditions:, 1. The share-register of the company for all shareholders shall be regularly maintained at its principal place, of business within India, in respect of any assessment year from a date not later than the 1st day of April of, such year., 2. The general meeting for passing the accounts of the previous year relevant to the assessment year and for, declaring any dividends in respect thereof shall be held only at a place within India., 3. The dividends declared, if any, shall be payable only within India to all shareholders., Hint for answer 10., Total income, 3,10,000, On first ₹2,50,000, Nil, On next ₹60,000 @ 5%, 3,000, Tax before health and education cess, 3,000, Hint for answer 14., Computation of Tax Liability, Tax on LTCG (3,50,000 – 2,50,000) @ 20% u/s 112, 20,000, Tax before Rebate u/s 87A, 20,000, Less: Rebate u/s 87A, (12,500), Tax before health & education cess, 7,500, Add: HEC @ 4%, 300, Tax Liability, 7,800, Hint for answer 26., Computation of Tax Liability, Tax on STCG (3,50,000 – 2,50,000) @ 15% u/s 111A, 15,000, Tax before Rebate u/s 87A, 15,000, Less: Rebate u/s 87A, (12,500), Tax before health & education cess, 2,500, Add: HEC @ 4%, 100, Tax Liability, 2,600, Hint for answer 27., Computation of Tax Liability, Tax on Casual Income 3,50,000 30% u/s 115BB, 1,05,000, Tax before Rebate u/s 87A, 1,05,000, Less: Rebate u/s 87A, (12,500), Tax before health & education cess, 92,500, Add: HEC @ 4%, 3,700, Tax Liability, 96,200
Page 406 :
Computation of Total Income And Tax Liability, Hint for answer 28., Computation of Tax Liability, Tax on STCG 3,50,000 @ 15% u/s 111A, Tax before Rebate u/s 87A, Less: Rebate u/s 87A, Tax before health & education cess, Add: HEC @ 4%, Tax Liability, , 3, , 52,500, 52,500, Nil, 52,500, 2,100, 54,600
Page 407 :
Computation of Total Income And Tax Liability, , 4, , SOLUTIONS, TO, , PRACTICE PROBLEMS, Solution 1:, (i) Computation of Tax Liability, Total Income, Tax on ₹22,00,000 at slab rate, Add: HEC @ 4%, Tax Liability, (ii) Computation of Tax Liability, Total Income, Tax on ₹22,00,000 at slab rate, Add: HEC @ 4%, Tax Liability, (iii) Computation of Tax Liability, Total Income, Tax on ₹22,00,000 at slab rate, Add: HEC @ 4%, Tax Liability, (iv) Computation of Tax Liability, Total Income, Tax on ₹22,00,000 at slab rate, Add: HEC @ 4%, Tax Liability, (v) Computation of Tax Liability, Total Income, Tax on ₹22,00,000 at slab rate, Add: HEC @ 4%, Tax Liability, (vi) Computation of Tax Liability, Total Income, Tax on ₹22,00,000 at slab rate, Add: HEC @ 4%, Tax Liability, (vii) Computation of Tax Liability, Total Income, Tax on ₹22,00,000 at slab rate, Add: HEC @ 4%, Tax Liability, (viii) Computation of Tax Liability, Total Income, Tax on ₹22,00,000 at slab rate, Add: HEC @ 4%, Tax Liability, (ix) Computation of Tax Liability, Total Income, Tax on ₹22,00,000 at slab rate, , ₹, 22,00,000, 4,72,500, 18,900, 4,91,400, 22,00,000, 4,72,500, 18,900, 4,91,400, 22,00,000, 4,72,500, 18,900, 4,91,400, 22,00,000, 4,72,500, 18,900, 4,91,400, 22,00,000, 4,70,000, 18,800, 4,88,800, 22,00,000, 4,70,000, 18,800, 4,88,800, 22,00,000, 4,72,500, 18,900, 4,91,400, 22,00,000, 4,72,500, 18,900, 4,91,400, 22,00,000, 4,60,000
Page 408 :
Computation of Total Income And Tax Liability, Add: HEC @ 4%, Tax Liability, (x) Computation of Tax Liability, Total Income, Tax on ₹22,00,000 at slab rate, Add: HEC @ 4%, Tax Liability, (xi) Computation of Tax Liability, Total Income, Tax on ₹22,00,000 at slab rate, Add: HEC @ 4%, Tax Liability, (xii) Computation of Tax Liability, Total Income, Tax on ₹22,00,000 at slab rate, Add: HEC @ 4%, Tax Liability, Solution 2:, (i) Computation of Tax Liability, Total Income, Tax on ₹100,05,000 at slab rate, Add: Surcharge @ 15%, Tax before marginal relief, Less: Marginal Relief, Working Note:, Tax + surcharge on income of ₹100,05,000, Tax + surcharge on income of ₹100,00,000, Increase in tax, Increase in income, Marginal Relief (1,42,350– 5,000), Tax after marginal relief, Add: HEC @ 4%, Tax Liability, (ii) Computation of Tax Liability, Total Income, Tax on ₹102,00,000 at slab rate, Add: Surcharge @ 15%, Tax before marginal relief, Less: Marginal Relief, Working Note:, Tax + surcharge on income of ₹102,00,000, Tax on income of ₹100,00,000, Increase in tax, Increase in income, Marginal Relief (2,09,625 – 2,00,000), Tax after marginal relief, Add: HEC @ 4%, Tax Liability, (iii) Computation of Tax Liability, Total Income, Tax on ₹90,00,000 at slab rate, Add: Surcharge @ 10%, , 5, 18,400, 4,78,400, 22,00,000, 4,60,000, 18,400, 4,78,400, 22,00,000, 4,72,500, 18,900, 4,91,400, 22,00,000, 4,72,500, 18,900, 4,91,400, ₹, 100,05,000.00, 28,14,000.00, 4,22,100.00, 32,36,100.00, (1,37,350.00), , 32,36,100, (30,93,750), 1,42,350, 5,000, 1,37,350, 30,98,750.00, 1,23,950.00, 32,22,700.00, 102,00,000.00, 28,72,500.00, 4,30,875.00, 33,03,375.00, (9,625.00), 33,03,375, (30,93,750), 2,09,625, 2,00,000, 9,625, 32,93,750.00, 1,31,750.00, 34,25,500.00, 90,00,000.00, 25,12,500.00, 2,51,250.00
Page 409 :
Computation of Total Income And Tax Liability, Tax before health & education cess, Add: HEC @ 4%, Tax Liability, (iv) Computation of Tax Liability, Total Income, Tax on ₹4,98,000 at slab rate, Add: HEC @ 4%, Tax Liability, Rounded off u/s 288B, (v) Computation of Tax Liability, Total Income, Tax on ₹4,05,000 at slab rate, Less: Rebate u/s 87A, Tax before health & education cess, Add: HEC @ 4%, Tax Liability, (vi) Computation of Tax Liability, Total Income, Tax on ₹102,05,000 at slab rate, Add: Surcharge @ 15%, Tax before marginal relief, Less: Marginal Relief, Working Note:, Tax + surcharge on income of ₹102,05,000, Tax on income of ₹100,00,000, Increase in tax, Increase in income, Marginal Relief (2,11,225 – 2,05,000), Tax after marginal relief, Add: HEC @ 4%, Tax Liability, (vii) Computation of Tax Liability, Total Income, Tax on ₹25,00,000 at slab rate, Add: HEC @ 4%, Tax Liability, (viii) Computation of Tax Liability, Total Income, Tax on ₹4,50,000 at slab rate, Add: HEC @ 4%, Tax Liability, (ix) Computation of Tax Liability, Total Income, Tax on ₹3,80,000 at slab rate, Tax Liability, (x) Computation of Tax Liability, Total Income, Tax on ₹110,00,000 at slab rate, Add: Surcharge @ 15%, Tax before health & education cess, Add: HEC @ 4%, Tax Liability, , 6, 27,63,750.00, 1,10,550.00, 28,74,300.00, 4,98,000.00, 12,400.00, 496.00, 12,896.00, 12,900.00, 4,05,000.00, 5,250.00, (5,250.00), Nil, Nil, Nil, 102,05,000.00, 28,71,500.00, 4,30,725.00, 33,02,225.00, (6,225.00), , 33,02,225, (30,91,000), 2,11,225, 2,05,000, 6,225, 32,96,000.00, 1,31,840.00, 34,27,840.00, 25,00,000.00, 5,62,500.00, 22,500.00, 5,85,000.00, 4,50,000.00, 10,000.00, 400.00, 10,400.00, 3,80,000.00, Nil, Nil, 110,00,000.00, 31,00,000.00, 4,65,000.00, 35,65,000.00, 1,42,600.00, 37,07,600.00
Page 410 :
Computation of Total Income And Tax Liability, (xi) Computation of Tax Liability, Total Income, Tax on ₹99,99,000 at slab rate, Add: Surcharge @ 10%, Tax before health & education cess, Add: HEC @ 4%, Tax Liability, Rounded off u/s 288B, (xii) Computation of Tax Liability, Total Income, Tax on ₹12,00,000 at slab rate, Add: HEC @ 4%, Tax Liability, (xiii) Computation of Tax Liability, Total Income, Tax on ₹202,00,000 at slab rate, Add: Surcharge @ 25%, Tax before marginal relief, Less: Marginal Relief, Working Note:, Tax + surcharge @25% on income of ₹202,00,000, Tax + surcharge @15% on income of ₹200,00,000, Increase in tax, Increase in income, Marginal Relief (6,56,250 – 2,00,000), Tax after marginal relief, Add: HEC @ 4%, Tax Liability, , 99,99,000.00, 28,12,200.00, 2,81,220.00, 30,93,420.00, 1,23,736.80, 32,17,156.80, 32,17,160.00, 12,00,000.00, 1,72,500.00, 6,900.00, 1,79,400.00, 202,00,000, 58,72,500, 14,68,125, 73,40,625, (4,56,250), 73,40,625, (66,84,375), 6,56,250, 2,00,000, 4,56,250, , (xiv) Computation of Tax Liability, Total Income, Tax on ₹502,00,000 at slab rate, Add: Surcharge @ 37%, Tax before marginal relief, Less: Marginal Relief, Working Note:, Tax + surcharge @37% on income of ₹502,00,000 203,75,325, Tax + surcharge @25% on income of ₹500,00,000 (1,85,15,625), Increase in tax, 18,59,700, Increase in income, 2,00,000, Marginal Relief (18,59,700 – 2,00,000), 16,59,700, Tax after marginal relief, Add: HEC @ 4%, Tax Liability, Solution 3:, (i) Computation of Tax Liability, Total Income, Tax on ₹50,20,000 at slab rate, Add: Surcharge @ 10%, Tax before marginal relief, Less: Marginal Relief, , 7, , 68,84,375, 2,75,375, 71,59,750, 502,00,000, 1,48,72,500, 55,02,825, 2,03,75,325, (16,59,700), , 1,87,15,625, 7,48,625, 1,94,64,250, ₹, 50,20,000, 13,18,500, 1,31,850, 14,50,350, (1,17,850)
Page 411 :
Computation of Total Income And Tax Liability, Working Note:, Tax + surcharge on income of ₹50,20,000, Tax on income of ₹50,00,000, Increase in tax, Increase in income, Marginal Relief (1,37,850 –20,000), Tax after marginal relief, Add: HEC @ 4%, Tax Liability, (ii) Computation of Tax Liability, Total Income, Tax on ₹53,00,000 at slab rate, Add: Surcharge @ 10%, Tax before health & education cess, Add: HEC @ 4%, Tax Liability, (iii) Computation of Tax Liability, Total Income, Tax on ₹51,00,000 at slab rate, Add: Surcharge @ 10%, Tax before marginal relief, Less: Marginal Relief, Working Note:, Tax + surcharge on income of ₹51,00,000, Tax on income of ₹50,00,000, Increase in tax, Increase in income, Marginal Relief (1,64,000 –1,00,000), Tax after marginal relief, Add: HEC @ 4%, Tax Liability, (iv) Computation of Tax Liability, Total Income, Tax on ₹54,00,000 at slab rate, Add: Surcharge @ 10%, Tax before health & education cess, Add: HEC @ 4%, Tax Liability, (v) Computation of Tax Liability, Total Income, Tax on ₹51,22,000 at slab rate, Add: Surcharge @ 10%, Tax before marginal relief, Less: Marginal Relief, Working Note:, Tax + surcharge on income of ₹51,22,000, Tax on income of ₹50,00,000, Increase in tax, Increase in income, Marginal Relief (1,71,510 –1,22,000), Tax after marginal relief, Add: HEC @ 4%, Tax Liability, , 8, , 14,50,350, (13,12,500), 1,37,850, 20,000, 1,17,850, 13,32,500, 53,300, 13,85,800, 53,00,000.00, 14,02,500.00, 1,40,250.00, 15,42,750.00, 61,710.00, 16,04,460.00, 51,00,000, 13,40,000, 1,34,000, 14,74,000, (64,000), 14,74,000, (13,10,000), 1,64,000, 1,00,000, 64,000, 14,10,000, 56,400, 14,66,400, 54,00,000.00, 14,20,000.00, 1,42,000.00, 15,62,000.00, 62,480.00, 16,24,480.00, 51,22,000.00, 13,49,100.00, 1,34,910.00, 14,84,010.00, (49,510), 14,84,010, (13,12,500), 1,71,510, 1,22,000, 49,510, 14,34,500 .00, 57,380.00, 14,91,880.00
Page 412 :
Computation of Total Income And Tax Liability, (vi) Computation of Tax Liability, Total Income, Tax on ₹50,80,000 at slab rate, Add: Surcharge @ 10%, Tax before marginal relief, Less: Marginal Relief, Working Note:, Tax + surcharge on income of ₹50,80,000, Tax on income of ₹50,00,000, Increase in tax, Increase in income, Marginal Relief (1,57,650 –80,000), Tax after marginal relief, Add: HEC @ 4%, Tax Liability, , 9, 50,80,000.00, 13,36,500.00, 1,33,650.00, 14,70,150.00, (77,650), , 14,70,150, (13,12,500), 1,57,650, 80,000, 77,650, , Solution 4:, (i) Computation of Tax Liability, Total Income, Tax on ₹4,90,000 at slab rate, Less: Rebate u/s 87A, Tax before health & education cess, Add: HEC @ 4%, Tax Liability, (ii) Computation of Tax Liability, Total Income, Tax on ₹4,90,000 at slab rate, Add: HEC @ 4%, Tax Liability, Note: Rebate under section 87A is not allowed to non-resident., (iii) Computation of Tax Liability, Total Income, Tax on ₹4,90,000 at slab rate, Less: Rebate u/s 87A, Tax before health & education cess, Add: HEC @ 4%, Tax Liability, (iv) Computation of Tax Liability, Total Income, Tax on ₹4,90,000 at slab rate, Add: HEC @ 4%, Tax Liability, Note: Rebate under section 87A is not allowed to non-resident., (v) Computation of Tax Liability, Total Income, Tax on ₹4,90,000 at slab rate, Less: Rebate u/s 87A, Tax Liability, (vi) Computation of Tax Liability, Total Income, Tax on ₹4,90,000 at slab rate, Less: Rebate u/s 87A, Tax Liability, , 13,92,500 .00, 55,700.00, 14,48,200.00, ₹, 4,90,000, 12,000, (12,000), Nil, Nil, Nil, 4,90,000, 12,000, 480, 12,480, 4,90,000, 12,000, (12,000), Nil, Nil, Nil, 4,90,000, 12,000, 480, 12,480, 4,90,000, 9,500, (9,500), Nil, 4,90,000, 9,500, (9,500), Nil
Page 413 :
Computation of Total Income And Tax Liability, , 10, , (vii) Computation of Tax Liability, Total Income, Tax on ₹4,90,000 at slab rate, Add: HEC @ 4%, Tax Liability, Note: Rebate under section 87A is not allowed to non-resident., (viii) Computation of Tax Liability, Total Income, Tax on ₹4,90,000 at slab rate, Add: HEC @ 4%, Tax Liability, Note: Rebate under section 87A is not allowed to non-resident., (ix) Computation of Tax Liability, Total Income, Tax Liability, (x) Computation of Tax Liability, Total Income, Tax Liability, (xi) Computation of Tax Liability, Total Income, Tax on ₹4,90,000 at slab rate, Add: HEC @ 4%, Tax Liability, Note: Rebate under section 87A is not allowed to non-resident., (xii) Computation of Tax Liability, Total Income, Tax on ₹4,90,000 at slab rate, Add: HEC @ 4%, Tax Liability, Note: Rebate under section 87A is not allowed to non-resident., Solution 5:, Computation of Total Income, Income under the head Salary, Income under the head House Property, Income under the head Business/Profession, Income under the head Capital gains, Long term capital gains, Short term capital gains, Income under the head Other Sources (Casual Income), Gross Total Income, Less: Deductions u/s 80C to 80U, Total Income, Computation of Tax Liability, Tax on Long term capital gains ₹1,20,000 @ 20% u/s 112, Tax on Casual Income ₹65,000 @ 30% u/s 115BB, Tax on Normal income ₹7,15,000 at slab rate, Tax before health & education cess, Add: HEC @ 4%, Tax Liability, , 4,90,000, 12,000, 480, 12,480, 4,90,000, 12,000, 480, 12,480, 4,90,000, Nil, 4,90,000, Nil, 4,90,000, 12,000, 480, 12,480, 4,90,000, 12,000, 480, 12,480, ₹, , ₹, 2,40,000, 1,55,000, 3,30,000, , 1,20,000, 35,000, , 1,55,000, 65,000, 9,45,000, (45,000), 9,00,000, 24,000, 19,500, 55,500, 99,000, 3,960, 1,02,960
Page 414 :
Computation of Total Income And Tax Liability, , 11, , Solution 5(b):, Total Income, , ₹, 9,00,000, , Computation of Tax Liability, Tax on Long term capital gains ₹1,20,000 @ 20% u/s 112, Tax on Casual Income ₹65,000 @ 30% u/s 115BB, Tax on Normal income ₹7,15,000 at slab rate, Tax before health & education cess, Add: HEC @ 4%, Tax Liability, , 24,000, 19,500, 55,500, 99,000, 3,960, 1,02,960, , Solution 5(c):, Total Income, Computation of Tax Liability, Tax on Long term capital gains ₹1,20,000 @ 20% u/s 112, Tax on Casual Income ₹65,000 @ 30% u/s 115BB, Tax on Normal income ₹7,15,000 at slab rate, Tax before health & education cess, Add: HEC @ 4%, Tax Liability, , ₹, 9,00,000, , Solution 5(d):, Total Income, Computation of Tax Liability, Tax on Long term capital gains ₹1,20,000 @ 20% u/s 112, Tax on Casual Income ₹65,000 @ 30% u/s 115BB, Tax on Normal income ₹7,15,000 at slab rate, Tax before health & education cess, Add: HEC @ 4%, Tax Liability, , ₹, 9,00,000, , Solution 5(e):, Total Income, Computation of Tax Liability, Tax on Long term capital gains ₹1,20,000 @ 20% u/s 112, Tax on Casual Income ₹65,000 @ 30% u/s 115BB, Tax on Normal income ₹7,15,000 at slab rate, Tax before health & education cess, Add: HEC @ 4%, Tax Liability, , ₹, 9,00,000, , Solution 5(f):, Total Income, Computation of Tax Liability, Tax on Long term capital gains ₹1,20,000 @ 20% u/s 112, Tax on Casual Income ₹65,000 @ 30% u/s 115BB, Tax on Normal income ₹7,15,000 at slab rate, Tax before health & education cess, Add: HEC @ 4%, Tax Liability, , ₹, 9,00,000, , 24,000, 19,500, 53,000, 96,500, 3,860, 1,00,360, , 24,000, 19,500, 55,500, 99,000, 3,960, 1,02,960, , 24,000, 19,500, 55,500, 99,000, 3,960, 1,02,960, , 24,000, 19,500, 43,000, 86,500, 3,460, 89,960
Page 415 :
Computation of Total Income And Tax Liability, Solution 5(g):, Total Income, Computation of Tax Liability, Tax on Long term capital gains ₹1,20,000 @ 20% u/s 112, Tax on Casual Income ₹65,000 @ 30% u/s 115BB, Tax on Normal income ₹7,15,000 at slab rate, Tax before health & education cess, Add: HEC @ 4%, Tax Liability, Solution 6:, (i), Computation of Total Income, Income under the head Salary, Income under the head House Property, Income under the head Capital Gains (LTCG), Gross Total Income, Less: Deduction u/s 80C to 80U, Total Income, Computation of Tax Liability, Tax on LTCG ₹6,30,000 (8,00,000 – 1,70,000) @ 20% u/s 112, Tax on ₹80,000 at slab rate, Tax before health & education cess, Add: HEC @ 4%, Tax Liability, (ii), Total Income, Computation of Tax Liability, Tax on LTCG ₹6,30,000 (8,00,000 – 1,70,000) @ 20% u/s 112, Tax on ₹80,000 at slab rate, Tax before health & education cess, Add: HEC @ 4%, Tax Liability, (iii), Total Income, Computation of Tax Liability, Tax on LTCG ₹5,80,000 (8,00,000 – 2,20,000) @ 20% u/s 112, Tax on ₹80,000 at slab rate, Tax before health & education cess, Add: HEC @ 4%, Tax Liability, (iv), Total Income, Computation of Tax Liability, Tax on LTCG ₹5,80,000 (8,00,000 – 2,20,000) @ 20% u/s 112, Tax on ₹80,000 at slab rate, Tax before health & education cess, Add: HEC @ 4%, Tax Liability, (v), Total Income, Computation of Tax Liability, , 12, ₹, 9,00,000, 24,000, 19,500, 55,500, 99,000, 3,960, 1,02,960, ₹, 30,000, 50,000, 8,00,000, 8,80,000, Nil, 8,80,000, 1,26,000, Nil, 1,26,000, 5,040, 1,31,040, 8,80,000, 1,26,000, Nil, 1,26,000, 5,040, 1,31,040, 8,80,000, 1,16,000, Nil, 1,16,000, 4,640, 1,20,640, 8,80,000, 1,16,000, Nil, 1,16,000, 4,640, 1,20,640, 8,80,000
Page 416 :
Computation of Total Income And Tax Liability, Tax on LTCG ₹3,80,000 (8,00,000 – 4,20,000) @ 20% u/s 112, Tax on ₹80,000 at slab rate, Tax before health & education cess, Add: HEC @ 4%, Tax Liability, (vi), Total Income, Computation of Tax Liability, Tax on LTCG ₹3,80,000 (8,00,000 – 4,20,000) @ 20% u/s 112, Tax on ₹80,000 at slab rate, Tax before health & education cess, Add: HEC @ 4%, Tax Liability, (vii)In situation (i), Total Income, Computation of Tax Liability, Tax on LTCG ₹8,00,000 @ 20% u/s 112, Tax on ₹80,000 at slab rate, Tax before health & education cess, Add: HEC @ 4%, Tax Liability, In situation (ii), Total Income, Computation of Tax Liability, Tax on LTCG ₹8,00,000 @ 20% u/s 112, Tax on ₹80,000 at slab rate, Tax before health & education cess, Add: HEC @ 4%, Tax Liability, In situation (iii), Total Income, Computation of Tax Liability, Tax on LTCG ₹8,00,000 @ 20% u/s 112, Tax on ₹80,000 at slab rate, Tax before health & education cess, Add: HEC @ 4%, Tax Liability, In situation (iv), Total Income, Computation of Tax Liability, Tax on LTCG ₹8,00,000 @ 20% u/s 112, Tax on ₹80,000 at slab rate, Tax before health & education cess, Add: HEC @ 4%, Tax Liability, In situation (v), Total Income, Computation of Tax Liability, Tax on LTCG ₹8,00,000 @ 20% u/s 112, Tax on ₹80,000 at slab rate, Tax before health & education cess, Add: HEC @ 4%, , 13, 76,000, Nil, 76,000, 3,040, 79,040, 8,80,000, 76,000, Nil, 76,000, 3,040, 79,040, 8,80,000, 1,60,000, Nil, 1,60,000, 6,400, 1,66,400, 8,80,000, 1,60,000, Nil, 1,60,000, 6,400, 1,66,400, 8,80,000, 1,60,000, Nil, 1,60,000, 6,400, 1,66,400, 8,80,000, 1,60,000, Nil, 1,60,000, 6,400, 1,66,400, 8,80,000, 1,60,000, Nil, 1,60,000, 6,400
Page 417 :
Computation of Total Income And Tax Liability, Tax Liability, In situation (vi), Total Income, Computation of Tax Liability, Tax on LTCG ₹8,00,000 @ 20% u/s 112, Tax on ₹80,000 at slab rate, Tax before health & education cess, Add: HEC @ 4%, Tax Liability, Solution 7:, (i), Computation of Total Income, Income under the head Salary, Income under the head House Property, Income under the head Capital Gains, Long term capital gains, Short term capital gains u/s 111A, Income under the head Other Sources (Casual Income), Gross Total Income, Less: Deduction u/s 80C to 80U, Total Income, Computation of Tax Liability, Tax on LTCG (2,30,000 – 2,30,000) @ 20% u/s 112, Tax on STCG ₹2,20,000 (₹2,40,000 – 20,000) @ 15% u/s 111A, Tax on Casual Income ₹70,000 @ 30% u/s 115BB, Tax on normal income at slab rate, Tax before health & education cess, Add: HEC @ 4%, Tax Liability, (ii), Total Income, Computation of Tax Liability, Tax on LTCG (2,30,000 – 2,30,000) @ 20% u/s 112, Tax on STCG ₹2,20,000 (₹2,40,000 – 20,000) @ 15% u/s 111A, Tax on Casual Income ₹70,000 @ 30% u/s 115BB, Tax on normal income at slab rate, Tax before health & education cess, Add: HEC @ 4%, Tax Liability, (iii), Total Income, Computation of Tax Liability, Tax on LTCG (2,30,000 – 2,30,000) @ 20% u/s 112, Tax on STCG ₹1,70,000 (2,40,000 – 70,000) @ 15% u/s 111A, Tax on Casual Income ₹70,000 @ 30% u/s 115BB, Tax on normal income at slab rate, Tax before health & education cess, Add: HEC @ 4%, Tax Liability, (iv), Total Income, , 14, 1,66,400, 8,80,000, 1,60,000, Nil, 1,60,000, 6,400, 1,66,400, , 90,000, 60,000, 2,30,000, 2,40,000, , 4,70,000, 70,000, 6,90,000, (1,50,000), 5,40,000, Nil, 33,000, 21,000, Nil, 54,000, 2,160, 56,160, 5,40,000, Nil, 33,000, 21,000, Nil, 54,000, 2,160, 56,160, 5,40,000, Nil, 25,500, 21,000, Nil, 46,500, 1,860, 48,360, 5,40,000
Page 418 :
Computation of Total Income And Tax Liability, Computation of Tax Liability, Tax on LTCG (2,30,000 – 2,30,000) @ 20% u/s 112, Tax on STCG ₹1,70,000 (2,40,000 – 70,000) @ 15% u/s 111A, Tax on Casual Income ₹70,000 @ 30% u/s 115BB, Tax on normal income at slab rate, Tax before health & education cess, Add: HEC @ 4%, Tax Liability, (v), Total Income, Computation of Tax Liability, Tax on LTCG ₹2,30,000 @ 20% u/s 112, Tax on STCG ₹2,40,000 @ 15% u/s 111A, Tax on Casual Income ₹70,000 @ 30% u/s 115BB, Tax on normal income at slab rate, Tax before health & education cess, Add: HEC @ 4%, Tax Liability, (vi), Total Income, Computation of Tax Liability, Tax on LTCG (2,30,000 – 2,30,000) @ 20% u/s 112, Tax on STCG (2,40,000 – 2,40,000) @ 15% u/s 111A, Tax on Casual Income ₹70,000 @ 30% u/s 115BB, Tax on normal income at slab rate, Tax before health & education cess, Add: HEC @ 4%, Tax Liability, (vii), Total Income, Computation of Tax Liability, Tax on LTCG (2,30,000 – 2,30,000) @ 20% u/s 112, Tax on STCG (2,40,000 – 2,40,000) @ 15% u/s 111A, Tax on Casual Income ₹70,000 @ 30% u/s 115BB, Tax on normal income at slab rate, Tax before health & education cess, Add: HEC @ 4%, Tax Liability, (viii), Total Income, Computation of Tax Liability, Tax on LTCG ₹2,30,000 @ 20% u/s 112, Tax on STCG ₹2,40,000 @ 15% u/s 111A, Tax on Casual Income ₹70,000 @ 30% u/s 115BB, Tax on normal income at slab rate, Tax before health & education cess, Add: HEC @ 4%, Tax Liability, Solution 8:, (i), Income under the head House Property, , 15, Nil, 25,500, 21,000, Nil, 46,500, 1,860, 48,360, 5,40,000, 46,000, 36,000, 21,000, Nil, 1,03,000, 4,120, 1,07,120, 5,40,000, Nil, Nil, 21,000, Nil, 21,000, 840, 21,840, 5,40,000, Nil, Nil, 21,000, Nil, 21,000, 840, 21,840, 5,40,000, 46,000, 36,000, 21,000, Nil, 1,03,000, 4,120, 1,07,120, ₹, 8,42,324.00
Page 419 :
Computation of Total Income And Tax Liability, Gross Total Income, Less: Deduction u/s 80C to 80U, Total Income, Rounded off u/s 288A, Computation of Tax Liability, Tax on ₹8,42,320 slab rate, Add: HEC @ 4%, Tax Liability, Rounded off u/s 288B, (ii), Income under the head Business/Profession, Gross Total Income, Less: Deduction u/s 80C to 80U, Total Income, Rounded off u/s 288A, Computation of Tax Liability, Tax on ₹14,42,340 slab rate, Add: HEC @ 4%, Tax Liability, Rounded off u/s 288B, (iii), Income under the head Capital Gains (LTCG), Gross Total Income, Less: Deduction u/s 80C to 80U, Total Income, Rounded off u/s 288A, Computation of Tax Liability, Tax on ₹8,35,340 (11,35,340 – 3,00,000) @ 20% u/s 112, Add: HEC @ 4%, Tax Liability, Rounded off u/s 288B, (iv), Income under the head Capital Gains (LTCG), Gross Total Income, Less: Deduction u/s 80C to 80U, Total Income, Rounded off u/s 288A, Computation of Tax Liability, Tax on ₹10,85,330 (13,35,330 – 2,50,000) @ 20% u/s 112, Add: HEC @ 4%, Tax Liability, Rounded off u/s 288B, (v), Income under the head Capital Gains (STCG), Gross Total Income, Less: Deduction u/s 80C to 80U, Total Income, Rounded off u/s 288A, Computation of Tax Liability, Tax on ₹7,70,340 (10,20,340 – 2,50,000) @ 15% u/s 111A, Add: HEC @ 4%, Tax Liability, Rounded off u/s 288B, , 16, 8,42,324.00, Nil, 8,42,324.00, 8,42,320.00, 80,964.00, 3,238.56, 84,202.56, 84,200.00, 14,42,336.00, 14,42,336.00, Nil, 14,42,336.00, 14,42,340.00, 2,45,202.00, 9,808.08, 2,55,010.08, 2,55,010.00, 11,35,335.00, 11,35,335.00, Nil, 11,35,335.00, 11,35,340.00, 1,67,068.00, 6,682.72, 1,73,750.72, 1,73,750.00, 13,35,334.90, 13,35,334.90, Nil, 13,35,334.90, 13,35,330.00, 2,17,066.00, 8,682.64, 2,25,748.64, 2,25,750.00, 10,20,335.00, 10,20,335.00, Nil, 10,20,335.00, 10,20,340.00, 1,15,551.00, 4,622.04, 1,20,173.04, 1,20,170.00
Page 420 :
Computation of Total Income And Tax Liability, (vi), Income under the head Capital Gains (LTCG), Gross Total Income, Less: Deduction u/s 80C to 80U, Total Income, Rounded off u/s 288A, Computation of Tax Liability, Tax on ₹5,40,340 @ 20% u/s 112, Add: HEC @ 4%, Tax Liability, Rounded off u/s 288B, (vii), Income under the head Other Sources (winning from lottery), Gross Total Income, Less: Deduction u/s 80C to 80U, Total Income, Computation of Tax Liability, Tax on ₹7,20,000 @ 30% u/s 115BB, Add: HEC @ 4%, Tax Liability, (viii), Income under the head Capital Gains (LTCG), Gross Total Income, Less: Deduction u/s 80C to 80U, Total Income, Rounded off u/s 288A, Computation of Tax Liability, Tax on ₹10,65,390 (15,65,390 – 5,00,000) @ 20% u/s 112, Add: HEC @ 4%, Tax Liability, Rounded off u/s 288B, (ix), Income under the head Other Sources (winning from lottery), Gross Total Income, Less: Deduction u/s 80C to 80U, Total Income, Computation of Tax Liability, Tax on ₹10,20,000 @ 30% u/s 115BB, Add: HEC @ 4%, Tax Liability, , 17, 5,40,337.00, 5,40,337.00, Nil, 5,40,337.00, 5,40,340.00, 1,08,068.00, 4,322.72, 1,12,390.72, 1,12,390.00, 7,20,000.00, 7,20,000.00, Nil, 7,20,000.00, 2,16,000.00, 8,640.00, 2,24,640.00, 15,65,385.00, 15,65,385.00, Nil, 15,65,385.00, 15,65,390.00, 2,13,078.00, 8,523.12, 2,21,601.12, 2,21,600.00, 10,20,000.00, 10,20,000.00, Nil, 10,20,000.00, 3,06,000.00, 12,240.00, 3,18,240.00, , Solution 9:, (i) Computation of Tax Liability, Income under the head Business/Profession, Total Income, Tax on ₹70,000 @ 30%, Add: HEC @ 4%, Tax Liability, , ₹, 70,000, 70,000, 21,000, 840, 21,840, , (ii) Computation of Tax Liability, Income under the head Business/Profession, Total Income, , 150,00,000, 150,00,000
Page 421 :
Computation of Total Income And Tax Liability, Tax on ₹150,00,000 @ 30%, Add: Surcharge @ 7%, Add: HEC @ 4%, Tax Liability, (iii) Computation of Tax Liability, Income under the head Business/Profession, Total Income, Tax on ₹6,00,000 @ 30%, Add: HEC @ 4%, Tax Liability, (iv) Computation of Tax Liability, Income under the head Business/Profession, Total Income, Tax on ₹10,30,000 @ 30%, Add: HEC @ 4%, Tax Liability, (v) Computation of Tax Liability, Income under the head Capital Gains (long term capital gains), Total Income, Tax on ₹700,00,000 @ 20%, Add: Surcharge @ 7%, Add: HEC @ 4%, Tax Liability, (vi) Computation of Tax Liability, Income under the head Capital Gains (long term capital gains), Total Income, Tax on ₹1,50,000 @ 20%, Add: HEC @ 4%, Tax Liability, (vii) Computation of Tax Liability, Income under the head Capital Gains (long term capital gains), Total Income, Tax on ₹6,00,000 @ 20%, Add: HEC @ 4%, Tax Liability, (viii) Computation of Tax Liability, Income under the head Capital Gains (long term capital gains), Total Income, Tax on ₹10,30,000 @ 20%, Add: HEC @ 4%, Tax Liability, (ix) Computation of Tax Liability, Income under the head Other Sources (Casual Income), Total Income, Tax on ₹400,00,000 @ 30%, Add: Surcharge @ 7%, Add: HEC @ 4%, Tax Liability, Solution 9(b):, (i) Computation of Tax Liability, Income under the head Business/Profession, Total Income, , 18, 45,00,000, 3,15,000, 1,92,600, 50,07,600, 6,00,000, 6,00,000, 1,80,000, 7,200, 1,87,200, 10,30,000, 10,30,000, 3,09,000, 12,360, 3,21,360, 700,00,000, 700,00,000, 140,00,000, 9,80,000, 5,99,200, 155,79,200, 1,50,000, 1,50,000, 30,000, 1,200, 31,200, 6,00,000, 6,00,000, 1,20,000, 4,800, 1,24,800, 10,30,000, 10,30,000, 2,06,000, 8,240, 2,14,240, 400,00,000, 400,00,000, 120,00,000, 8,40,000, 5,13,600, 133,53,600, ₹, 70,000, 70,000
Page 422 :
Computation of Total Income And Tax Liability, Tax on ₹70,000 @ 40%, Add: HEC @ 4%, Tax Liability, (ii) Computation of Tax Liability, Income under the head Business/Profession, Total Income, Tax on ₹150,00,000 @ 40%, Add: Surcharge @ 2%, Add: HEC @ 4%, Tax Liability, (iii) Computation of Tax Liability, Income under the head Business/Profession, Total Income, Tax on ₹6,00,000 @ 40%, Add: HEC @ 4%, Tax Liability, (iv) Computation of Tax Liability, Income under the head Business/Profession, Total Income, Tax on ₹10,30,000 @ 40%, Add: HEC @ 4%, Tax Liability, (v) Computation of Tax Liability, Income under the head Capital Gains (long term capital gains), Total Income, Tax on ₹700,00,000 @ 20%, Add: Surcharge @ 2%, Add: HEC @ 4%, Tax Liability, (vi) Computation of Tax Liability, Income under the head Capital Gains (long term capital gains), Total Income, Tax on ₹1,50,000 @ 20%, Add: HEC @ 4%, Tax Liability, (vii) Computation of Tax Liability, Income under the head Capital Gains (long term capital gains), Total Income, Tax on ₹6,00,000 @ 20%, Add: HEC @ 4%, Tax Liability, (viii) Computation of Tax Liability, Income under the head Capital Gains (long term capital gains), Total Income, Tax on ₹10,30,000 @ 20%, Add: HEC @ 4%, Tax Liability, (ix) Computation of Tax Liability, Income under the head Other Sources (Casual Income), Total Income, Tax on ₹400,00,000 @ 30%, Add: Surcharge @ 2%, Add: HEC @ 4%, , 19, 28,000, 1,120, 29,120, 150,00,000, 150,00,000, 60,00,000, 1,20,000, 2,44,800, 63,64,800, 6,00,000, 6,00,000, 2,40,000, 9,600, 2,49,600, 10,30,000, 10,30,000, 4,12,000, 16,480, 4,28,480, 700,00,000, 700,00,000, 140,00,000, 2,80,000, 5,71,200, 148,51,200, 1,50,000, 1,50,000, 30,000, 1,200, 31,200, 6,00,000, 6,00,000, 1,20,000, 4,800, 1,24,800, 10,30,000, 10,30,000, 2,06,000, 8,240, 2,14,240, 400,00,000, 400,00,000, 120,00,000, 2,40,000, 4,89,600
Page 423 :
Computation of Total Income And Tax Liability, Tax Liability, Solution 10:, Computation of Total Income, Income under the head Business/Profession, Income under the head House Property, Income under the head Capital Gains, Long term capital gains, Short term capital gains u/s 111A, Income under the head Other Sources (Casual Income), Gross Total Income, Less: Deduction u/s 80C to 80U, Total Income, Computation of Tax Liability, Tax on LTCG ₹4,50,000 @ 20% u/s 112, Tax on STCG ₹3,50,000 @ 15% u/s 111A, Tax on Casual income ₹3,50,000 @ 30% u/s 115BB, Tax on ₹8,75,000 at slab rate, Tax before health & education cess, Add: HEC @ 4%, Tax Liability, Solution 11:, Computation of Total Income, Income under the head Salary, Income under the head House Property, Income under the head Capital Gains (Long term capital gains), Gross Total Income, Less: Deduction u/s 80C to 80U, Total Income, Agricultural Income, Computation of Tax Liability, Tax on long term capital gain ₹1,00,000 @ 20% u/s 112, Normal income ₹18,40,000, Step 1. Tax on (18,40,000 + 4,00,000), Step 2. Tax on (₹2,50,000 + 4,00,000) at slab rates, Step 3. Deduct Tax at Step 2 from Tax at Step 1, Tax before health & education cess, Add: HEC @ 4%, Tax Liability, Solution 11(b):, Computation of Total Income, Income under the head Salary, Income under the head House Property, Income under the head Capital Gains (Long term capital gains), Gross Total Income, Less: Deduction u/s 80C to 80U, Total Income, Agricultural Income, Computation of Tax Liability, Tax on long term capital gain ₹1,00,000 @ 20% u/s 112, , 20, 127,29,600, ₹, 6,00,000.00, 4,00,000.00, , 4,50,000, 3,50,000, , 8,00,000.00, 3,50,000.00, 21,50,000.00, (1,25,000.00), 20,25,000.00, 90,000.00, 52,500.00, 1,05,000.00, 87,500.00, 3,35,000.00, 13,400.00, 3,48,400.00, ₹, 3,00,000, 16,00,000, 1,00,000, 20,00,000, (60,000), 19,40,000, 4,00,000, 20,000, 4,84,500, (42,500), 4,42,000, 4,62,000, 18,480, 4,80,480, ₹, 3,00,000, 16,00,000, 1,00,000, 20,00,000, (60,000), 19,40,000, 4,00,000, 20,000
Page 424 :
Computation of Total Income And Tax Liability, Step 1. Tax on (18,40,000 + 4,00,000), Step 2. Tax on (₹3,00,000 + 4,00,000) at slab rates, Step 3. Deduct Tax at Step 2 from Tax at Step 1, Tax before health & education cess, Add: HEC @ 4%, Tax Liability, Solution 12:, Computation of Total Income, Income under the head House Property, Income under the head Capital gains (long term capital gains), Gross Total Income, Less: Deduction u/s 80C to 80U, Total Income, Agricultural income, Computation of Tax Liability, Tax on normal income ₹2,00,000 at slab rate, Tax on long term capital gain ₹4,50,000 (₹5,00,000 – ₹50,000) @ 20% u/s 112, Tax before health & education cess, Add: HEC @ 4%, Tax Liability, , 21, 4,82,000, (50,000), 4,32,000, 4,52,000, 18,080, 4,70,080, ₹, 3,00,000, 5,00,000, 8,00,000, (1,00,000), 7,00,000, 3,00,000, Nil, 90,000, 90,000, 3,600, 93,600, , Solution 12(b):, Computation of Total Income, ₹, Income under the head House Property, 3,00,000, Income under the head Capital gains (long term capital gains), 5,00,000, Gross Total Income, 8,00,000, Less: Deduction u/s 80C to 80U, (1,00,000), Total Income, 7,00,000, Agricultural income, 3,00,000, Computation of Tax Liability, Tax on normal income ₹2,00,000 at slab rate, Nil, Tax on long term capital gain ₹2,00,000 (₹5,00,000 – ₹3,00,000) @ 20% u/s 112, 40,000, Tax before health & education cess, 40,000, Add: HEC @ 4%, 1,600, Tax Liability, 41,600, Note: If non-agricultural income is upto the limit not chargeable to tax (₹2,50,000/3,00,000/5,00,000),, partial integration is not applicable., Solution 13:, Computation of Total Income, Income under the head Other Sources (Casual income), Income under the head Capital gains (STCG u/s 111A), Gross Total Income, Less: Deduction u/s 80C to 80U, Total Income, Agricultural income, Computation of Tax Liability, Tax on STCG ₹4,50,000 (₹7,00,000 – 2,50,000) @ 15% u/s 111A, Tax on casual income ₹5,00,000 @ 30%, Tax before health & education cess, Add: HEC @ 4%, Tax Liability, , ₹, 5,00,000, 7,00,000, 12,00,000, Nil, 12,00,000, 3,00,000, 67,500, 1,50,000, 2,17,500, 8,700, 2,26,200
Page 425 :
Computation of Total Income And Tax Liability, , 22, , Note: On Long term capital gain, casual income and short term capital gain u/s 111A partial integration, shall not be applicable., Solution 13(b):, Computation of Total Income, ₹, Income under the head Other Sources (Casual income), 5,00,000, Income under the head Capital gains (STCG u/s 111A), 7,00,000, Gross Total Income, 12,00,000, Less: Deduction u/s 80C to 80U, Nil, Total Income, 12,00,000, Agricultural income, 3,00,000, Computation of Tax Liability, Tax on STCG ₹7,00,000 @ 15% u/s 111A, 1,05,000, Tax on casual income ₹5,00,000 @ 30%, 1,50,000, Tax before health & education cess, 2,55,000, Add: HEC @ 4%, 10,200, Tax Liability, 2,65,200, Note: On Long term capital gain, casual income and short term capital gain u/s 111A partial integration, shall not be applicable., Solution 14:, Computation of Total Income, Income under the head Business/Profession, Income under the head Other Sources (Casual income), Gross Total Income, Less: Deduction u/s 80C to 80U, Total Income, Agricultural income, Computation of Tax Liability, Tax on casual income ₹5,00,000 @ 30% u/s 115BB, Normal income ₹12,00,000, Step 1. Tax on (12,00,000 + 10,00,000), Step 2. Tax on (₹5,00,000 + 10,00,000) at slab rates, Step 3. Deduct Tax at Step 2 from Tax at Step 1, Tax before health & education cess, Add: HEC @ 4%, Tax Liability, Solution 15:, Computation of Total Income, Income under the head Salary, Income under the head House Property, Income under the head Capital Gains, Short Term Capital Gain, Short Term Capital Gain111A, Long Term Capital Gain, Income under the head Capital Gains, Casual Income, Gross Total Income, Less: Deduction u/s 80C to 80U, Total Income, Agricultural Income, , ₹, 12,00,000, 5,00,000, 17,00,000, Nil, 17,00,000, 10,00,000, 1,50,000, 4,60,000, (2,50,000), 2,10,000, 3,60,000, 14,400, 3,74,400, ₹, 3,00,000, 1,00,000, 50,000, 2,00,000, 1,50,000, 4,00,000, 70,000, 8,70,000, (1,10,000), 7,60,000, 5,00,000
Page 426 :
Computation of Total Income And Tax Liability, Computation of Tax Liability, Tax on casual income ₹70,000 @ 30% u/s 115BB, Tax on long term capital gain ₹1,50,000 @ 20% u/s 112, Tax on short term capital gain ₹2,00,000 @ 15% u/s 111A, Normal income ₹3,40,000, Step 1. Tax on (3,40,000 + 5,00,000), Step 2. Tax on (₹2,50,000 + 5,00,000) at slab rates, Step 3. Deduct Tax at Step 2 from Tax at Step 1, Tax before health & education cess, Add: HEC @ 4%, Tax Liability, , 23, 21,000, 30,000, 30,000, 80,500, (62,500), 18,000, 99,000, 3,960, 1,02,960, , Solution 15(b):, Computation of Total Income, ₹, Income under the head Salary, 3,00,000, Income under the head House Property, 1,00,000, Income under the head Capital Gains, Short Term Capital Gain, 50,000, Short Term Capital Gain111A, 2,00,000, Long Term Capital Gain, 1,50,000, Income under the head Capital Gains, 4,00,000, Casual Income, 70,000, Gross Total Income, 8,70,000, Less: Deduction u/s 80C to 80U, (1,10,000), Total Income, 7,60,000, Agricultural Income, 5,00,000, Computation of Tax Liability, Tax on casual income ₹70,000 @ 30% u/s 115BB, 21,000, Tax on long term capital gain Nil (₹1,50,000-1,50,000) @ 20% u/s 112, Nil, Tax on short term capital gain ₹ 1,90,000(₹2,00,000-₹10,000) @ 15% u/s 111A, 28,500, Tax on normal income ₹3,40,000 at slab rate, Nil, Tax before health & education cess, 49,500, Add: HEC @ 4%, 1,980, Tax Liability, 51,480, Note: If non-agricultural income is upto the limit not chargeable to tax (₹2,50,000/ 3,00,000/5,00,000),, partial integration is not applicable., Solution 15(c):, Computation of Total Income, Income under the head Salary, Income under the head House Property, Income under the head Capital Gains, Short Term Capital Gain, Short Term Capital Gain111A, Long Term Capital Gain, Income under the head Capital Gains, Casual Income, Gross Total Income, Less: Deduction u/s 80C to 80U, Total Income, Agricultural Income, Computation of Tax Liability, Tax on casual income ₹70,000 @ 30% u/s 115BB, , ₹, 3,00,000, 1,00,000, 50,000, 2,00,000, 1,50,000, 4,00,000, 70,000, 8,70,000, (3,00,000), 5,70,000, 5,00,000, 21,000
Page 427 :
Computation of Total Income And Tax Liability, , 24, , Tax on long term capital gain ₹1,50,000 @ 20% u/s 112, 30,000, Tax on short term capital gain ₹2,00,000 @ 15% u/s 111A, 30,000, Tax on normal income ₹1,50,000 at slab rate, Nil, Tax before health & education cess, 81,000, Add: HEC @ 4%, 3,240, Tax Liability, 84,240, Note: If non-agricultural income is upto the limit not chargeable to tax (₹2,50,000/ 3,00,000/5,00,000),, partial integration is not applicable., Solution 16:, Computation of Total Income, ₹, Income under the head House Property, 2,00,000, Income under the head capital gains (long term capital gains), 10,00,000, Gross Total Income, 12,00,000, Less: Deduction u/s 80C to 80U, (60,000), Total Income, 11,40,000, Agricultural income, 7,00,000, Computation of Tax Liability, Tax on normal income ₹1,40,000 at slab rate, Nil, Tax on long term capital gain ₹8,90,000 (₹10,00,000 – ₹1,10,000) @ 20% u/s 112, 1,78,000, Tax before health & education cess, 1,78,000, Add: HEC @ 4%, 7,120, Tax Liability, 1,85,120, Note: If non-agricultural income is upto the limit not chargeable to tax (₹2,50,000/ 3,00,000/5,00,000),, partial integration is not applicable., Solution 16(b):, Computation of Total Income, Income under the head House Property, Income under the head capital gains (long term capital gains), Gross Total Income, Less: Deduction u/s 80C to 80U, Total Income, Agricultural income, Computation of Tax Liability, Tax on long term capital gain ₹10,00,000 @ 20% u/s 112, Normal income ₹9,40,000, Step 1. Tax on (9,40,000 + 7,00,000) at slab rates, Step 2. Tax on (₹3,00,000 + 7,00,000) at slab rates, Step 3. Deduct Tax at Step 2 from Tax at Step 1, Tax before health & education cess, Add: HEC @ 4%, Tax Liability, Solution 17:, Computation of Total Income, Income under the head Business/Profession, Income under the head House Property, Income under the head Capital Gains, Long term capital gains, Short term capital gains u/s 111A, Income under the head Other Sources (Casual Income), Gross Total Income, , ₹, 10,00,000, 10,00,000, 20,00,000, (60,000), 19,40,000, 7,00,000, 2,00,000, 3,02,000, (1,10,000), 1,92,000, 3,92,000, 15,680, 4,07,680, ₹, 6,00,000.00, 4,00,000.00, 4,50,000, 3,50,000, , 8,00,000.00, 3,50,000.00, 21,50,000.00
Page 428 :
Computation of Total Income And Tax Liability, , 25, , Less: Deduction u/s 80C to 80U (not allowed), Total Income, Computation of Tax Liability as per section 115BAC, Tax on LTCG ₹4,50,000 @ 20% u/s 112, Tax on STCG ₹3,50,000 @ 15% u/s 111A, Tax on Casual income ₹3,50,000 @ 30% u/s 115BB, Tax on ₹10,00,000 at slab rate (115BAC), Tax before health & education cess, Add: HEC @ 4%, Tax Liability, , Nil, 21,50,000.00, , Solution 18:, Computation of Total Income, Income under the head Business/Profession, Income under the head House Property, Income under the head salary, Gross Total Income, Less: Deduction u/s 80C to 80U (not allowed), Total Income, Computation of Tax Liability as per section 115BAC, Tax on ₹13,00,000 at slab rate (115BAC), Tax before health & education cess, Add: HEC @ 4%, Tax Liability, , ₹, , 90,000.00, 52,500.00, 1,05,000.00, 75,000.00, 3,22,500.00, 12,900.00, 3,35,400.00, , 9,00,000.00, 1,00,000.00, 3,00,000.00, 13,00,000.00, Nil, 13,00,000.00, 1,37,500.00, 1,37,500.00, 5,500.00, 1,43,000.00
Page 429 :
Taxability of Gift, , 26, , TAXABILITY OF GIFT, SECTION 56, SOLUTIONS OF MCQS, 1. (b); 2. (c); 3. (a); 4. (e); 5. (c); 6. (c); 7. (d); 8. (c); 9. (a); 10. (d); 11.(c); 12.(a);13.(c); 14. (a) 15. (a) ; 16., (d) ; 17. (c); 18. (a); 19. (d); 20. (c)
Page 430 :
Taxability of Gift, , 27, , SOLUTIONS, TO, , PRACTICE PROBLEMS, Solution 1:, (i) Mr. X has received gift of ₹50,000 in cash from his friend, in this case it will not be considered to be his, income., (ii) Mr. X has received gift of ₹2,50,000 in cash from his brother, in this case it will not be considered to be, his income., (iii) Mr. X has received gift of ₹ 2,50,000 in cash from his mother’s sister, in this case it will not be, considered to be his income., (iv) Mr. X has received gift of ₹2,50,000 in cash from his father’s brother, in this case it will not be, considered to be his income., (v) Mr. X has received gift of ₹2,50,000 in cash from his cousin, in this case it will be chargeable to tax., (vi) Mr. X has received gift of ₹2,50,000 in cash from brother of his spouse, in this case it will not be, considered to be his income., (vii) Mr. X has received gift of ₹2,50,000 in cash from his grand father, in this case it will not be considered, to be his income., (viii) Mr. X has received gift of ₹2,50,000 in cash from spouse of his brother, in this case it will not be, considered to be his income., (ix) Mr. X has received gift of ₹2,50,000 in cash from husband of his sister, in this case it will not be, considered to be his income., (x) Mr. X has received gift of ₹2,50,000 in cash from sister of his brother’s wife, in this case it will be, considered to be his income., (xi) Mr. X has received gift of ₹2,50,000 in cash from the sister of his spouse, in this case it will not be, considered to be his income., (xii) Mr. X has received gift of ₹6,000 in cash on his birthday from each of his eleven friends, in this case it, will be considered to be his income because the total amount is exceeding ₹50,000., (xiii) Mr. X has received gift of ₹2,50,000 in kind from his friend, in this case it will be considered to be his, income., (xiv) Mr. X has received gift of ₹2,50,000 in cash from his friend on the occasion of his marriage, in this, case it will not be considered to be his income., (xv) Mr. X has received gift of ₹1,00,000 in cash and ₹1,00,000 as property from his fiancee, in this case, gift in cash will be considered to be his income and the gift in kind shall also be considered to be his income., Solution 2:, Computation of income under the head Salary, Gift in kind from his employer (70,000 – 5,000), Less: Standard deduction u/s 16(ia), Income under the head Salary, Computation of income under the head Other Sources, Gift received from friend, Gifts received from neighbours, Gift received from friend in kind, Income under the head Other Sources, Gross Total Income, Less: Deduction u/s 80C to 80U, Total Income, , ₹, 65,000.00, (50,000.00), 15,000.00, 27,000.00, 60,000.00, 2,00,000.00, 2,87,000.00, 3,02,000.00, Nil, 3,02,000.00
Page 431 :
Taxability of Gift, Computation of Tax Liability, Tax on ₹3,02,000 at slab rate, Less: Rebate u/s 87A, Tax before health & education cess, Add: HEC @ 4%, Tax Liability, Rounded off u/s 288B, Solution 3:, Computation of income under the head Other Sources, Gift received from son of his father’s brother, Gift received from bother of father’s of Mrs. X, Income under the head Other Sources, Gross Total Income, Less: Deduction u/s 80C to 80U, Total Income, Agricultural Income, Computation of Tax Liability, Step 1. Tax on (6,00,000 + 5,00,000) at slab rates, Step 2. Tax on (₹2,50,000 + 5,00,000) at slab rates, Step 3. Deduct Tax at Step 2 from Tax at Step 1, Tax before health & education cess, Add: HEC @ 4%, Tax Liability, Solution 3(b):, Total Income, Agricultural Income, Computation of Tax Liability, Step 1. Tax on (6,00,000 + 5,00,000) at slab rates, Step 2. Tax on (₹5,00,000 + 5,00,000) at slab rates, Step 3. Deduct Tax at Step 2 from Tax at Step 1, Tax before health & education cess, Add: HEC @ 4%, Tax Liability, Solution 3(c):, Total Income, Agricultural Income, Computation of Tax Liability, Step 1. Tax on (6,00,000 + 5,00,000) at slab rates, Step 2. Tax on (₹2,50,000 + 5,00,000) at slab rates, Step 3. Deduct Tax at Step 2 from Tax at Step 1, Tax before health & education cess, Add: HEC @ 4%, Tax Liability, Solution 4:, Computation of income under the head Other Sources, Gift in kind from brother of his grand father, Income under the head Other Sources, Gross Total Income, Less: Deduction u/s 80C to 80U, Total Income, , 28, 2,600.00, (2,600.00), Nil, Nil, Nil, Nil, ₹, 5,00,000, 1,00,000, 6,00,000, 6,00,000, Nil, 6,00,000, 5,00,000, 1,42,500, (62,500), 80,000, 80,000, 3,200, 83,200, 6,00,000, 5,00,000, 1,30,000, (1,00,000), 30,000, 30,000, 1,200, 31,200, 6,00,000, 5,00,000, 1,42,500, (62,500), 80,000, 80,000, 3,200, 83,200, ₹, 8,00,000, 8,00,000, 8,00,000, Nil, 8,00,000
Page 432 :
Taxability of Gift, Agricultural Income, Computation of Tax Liability, Step 1. Tax on (8,00,000 + 1,00,000) at slab rates, Step 2. Tax on (₹2,50,000 + 1,00,000) at slab rates, Step 3. Deduct Tax at Step 2 from Tax at Step 1, Tax before health & education cess, Add: HEC @ 4%, Tax Liability, Solution 5:, Computation of Total Income of Mrs. X for the A.Y. 2022-23, Gift received on the occasion of marriage are exempt, Cash gift received from cousin of Mrs. X’s mother is taxable under section 56(2)(x), (Cousin of Mrs. X’s mother is not a relative), Mobile phone gifted by her friend is not taxable since it is not included in the definition, of “property” under section 56(2)(x), Cash gift received from elder brother of husband’s grandfather is taxable, (Brother of husband’s grandfather is not a relative), Cash gift from friend is taxable, Since bullion is included in the definition of property, therefore, when bullion is, received without consideration, the same is taxable, since the aggregate fair, market value exceeds ₹50,000, Difference of ₹2.5 lakh in the value of jewellery purchased from her friend, is not, taxable as it represents the stock-in-trade of Mrs. X. Since Mrs. X is carrying, jewellery business and it has been mentioned that the jewellery were subsequently, sold in the course of her business, such jewellery represent the stock-in-trade, of Mrs. X., Income under the head Other Sources, Gross Total Income, Less: Deduction u/s 80C to 80U, Total Income, Computation of Tax Liability, Tax on ₹ 15,65,000 at slab rate, Add: HEC @ 4%, Tax Liability, , 29, 1,00,000, 92,500, (5,000), 87,500, 87,500, 3,500, 91,000, ₹, -2,50,000, -2,40,000, 6,00,000, 4,75,000, , Nil, 15,65,000, 15,65,000, Nil, 15,65,000, 2,82,000, 11,280, 2,93,280
Page 433 :
Advance Payment of Income Tax, , ADVANCE PAYMENT OF TAX, OR, , PAY AS YOU EARN SCHEME, SECTION 207 TO 219, , SOLUTIONS OF MCQS, Answer, 1. (b); 2. (b); 3. (c); 4. (d); 5. (b); 6. (c); 7. (c); 8. (c); 9. (c); 10. (b), , 30
Page 434 :
Advance Payment of Income Tax, , 31, , SOLUTIONS, TO, , PRACTICE PROBLEMS, Solution 1:, Computation of Total Income, Income under the head House Property, Income under the head Other Sources, Gift in kind received from a friend, Gross Total Income, Less: Deduction u/s 80C to 80U, Total Income, Agricultural Income, Computation of Tax Liability, Step 1. Tax on (17,00,000 + 3,00,000) at slab rates, Step 2. Tax on (`2,50,000 + 3,00,000) at slab rates, Step 3. Deduct Tax at Step 2 from Tax at Step 1, Add: HEC @ 4%, Tax Liability, , `, 15,00,000, 2,00,000, 17,00,000, Nil, 17,00,000, 3,00,000, 4,12,500, (22,500), 3,90,000, 15,600, 4,05,600, , Interest u/s 234C, Amount payable, `, Upto 15.06 2021 (4,05,600 x 15%), 60,840, Rounded off under rule 119A = 45,800, Interest u/s 234C = 45,800 x 1% x 3 month = 1,374, , Amount actually paid, `, 15,000, , Shortfall, `, 45,840, , Upto 15.09 2021 (4,05,600 x 45%), 1,82,520, Rounded off under rule 119A = 1,52,500, Interest u/s 234C = 1,52,500 x 1% x 3 month = 4,575, , 30,000, , 1,52,520, , Upto 15.12.2021 (4,05,600 x 75%) 3,04,200, Interest u/s 234C = 2,54,200 x 1% x 3 month = 7,626, , 50,000, , 2,54,200, , Upto 15.03.2022 (4,05,600 x 100%) 4,05,600, Interest u/s 234C = 3,45,600 x 1% x 1 month = 3,456, , 60,000, , 3,45,600, , Interest liability under section 234C, Interest under section 234B (01-04-2022 to 10-09-2022), 3,45,600 x 1% x 6, Interest under section 234A (01-08-2022 to 10-09-2022), 3,45,600 x 1% x 2, Solution 2:, Total Income, Computation of Tax Liability, Tax on `27,00,000 @ 30%, , `17,031, `20,736, ` 6,912, `, 27,00,000, 8,10,000
Page 435 :
Advance Payment of Income Tax, Add: HEC @ 4%, Tax Liability, , 32, 32,400, 8,42,400, , Interest u/s 234C, Amount payable, `, Upto 15.06 2021 (8,42,400 x 15%), 1,26,360, Rounded off under rule 119A = 1,06,300, Interest u/s 234C = 1,06,300 x 1% x 3 month = 3,189, , Amount actually paid, `, 20,000, , Shortfall, `, 1,06,360, , Upto 15.09 2021 (8,42,400 x 45%), 3,79,080, Rounded off under rule 119A = 3,49,000, Interest u/s 234C = 3,49,000 x 1% x 3 month = 10,470, , 30,000, , 3,49,080, , Upto 15.12.2021 (8,42,400 x 75%), 6,31,800, Interest u/s 234C = 5,51,800 x 1% x 3 month = 16,554, , 80,000, , 5,51,800, , 1,00,000, , 7,42,400, , Upto 15.03.2022 (8,42,400 x 100%), 8,42,400, Interest u/s 234C = 7,42,400 x 1% x 1 month = 7,424, Interest liability under section 234C, , `37,637, , Interest under section 234B (01-04-2022 to 10-12-2022), 7,42,400 x 1% x 9, , `66,816, , Interest under section 234A (01-11-2022 to 10-12-2022), 7,42,400 x 1% x 2, , `14,848, , Solution 3:, Computation of Total Income, Income under the head House Property, Income under the head Other Sources, Gift received from sister of her mother in law, Gross Total Income, Less: Deduction u/s 80C to 80U, Total Income, Agricultural Income, Computation of Tax Liability, Step 1. Tax on (19,20,000 + 4,00,000) at slab rates, Step 2. Tax on (`2,50,000 + 4,00,000) at slab rates, Step 3. Deduct Tax at Step 2 from Tax at Step 1, Add: HEC @ 4%, Tax Liability, , `, 18,00,000, 1,20,000, 19,20,000, Nil, 19,20,000, 4,00,000, 5,08,500, (42,500), 4,66,000, 18,640, 4,84,640, , Interest u/s 234C, Amount payable, `, Upto 15.06 2021 (4,84,640 x 15%), 72,696, Rounded off under rule 119A = 57,600, Interest u/s 234C = 57,600 x 1% x 3 month = 1,728, Upto 15.09 2021 (4,84,640 x 45%), 2,18,088, Rounded off under rule 119A = 1,73,000, , Amount actually paid, `, 15,000, , 45,000, , Shortfall, `, 57,696, , 1,73,088
Page 436 :
Advance Payment of Income Tax, , 33, , Interest u/s 234C = 1,73,000 x 1% x 3 month = 5,190, Upto 15.12.2021 (4,84,640 x 75%), 3,63,480, Rounded off under rule 119A = 2,88,400, Interest u/s 234C = 2,88,400 x 1% x 3 month = 8,652, , 75,000, , 2,88,480, , Upto 15.03.2022 (4,84,640 x 100%), 4,84,640, Interest u/s 234C = 3,84,600 x 1% x 1 month = 3,846, , 1,00,000, , 3,84,640, , Interest liability under section 234C, Interest under section 234B (01-04-2022 to 10-12-2022), 3,84,600 x 1% x 9, Interest under section 234A (01-08-2022 to 10-12-2022), 3,84,600 x 1% x 5, , `19,416, `34,614, `19,230
Page 437 :
Residential Status and Scope of Total Income, , 34, , RESIDENTIAL STATUS, &, , SCOPE OF TOTAL INCOME, SECTION 5 TO 9, SOLUTIONS OF MCQS, 1. (b); 2. (b); 3. (d); 4. (a); 5. (d); 6. (a); 7. (d); 8. (d); 9. (a); 10. (d); 11. (b); 12. (b); 13. (c); 14. (b); 15. (a);, 16. (a); 17. (b); 18. (b); 19. (b); 20. (c); 21. (c); 22. (c); 23. (d); 24. (a); 25. (a)
Page 438 :
Residential Status and Scope of Total Income, , 35, , SOLUTIONS, TO, , PRACTICE PROBLEMS, Solution 1:, 2021-22, Resident, 2020-21, Resident, 2019-20, Resident, 2018-19, Non-Resident, 2017-18, Resident, 2016-17, Resident, 2015-16, Resident, 2014-15, Resident, 2013-14, Resident, 2012-13, Resident, 2011-12, Resident, Total stay in 7 years preceding the relevant previous year is 732 days., Since the assessee is not able to comply with any of the conditions of section 6(6)(a), as listed below, he will, be considered to be ROR., 1. He is non resident in India in at least nine out of ten previous years preceding that year., or, 2. He has during the seven previous years preceding that year been in India for a period of 729 days or, less., Solution 2:, 2021-22, Resident, 2020-21, Resident, 2019-20, Resident, 2018-19, Resident, 2017-18, Resident, 2016-17, Resident, 2015-16, Resident, 2014-15, Resident, 2013-14, Resident, 2012-13, Resident, 2011-12, Resident, Total stay in 7 years preceding the relevant previous year is 754 days., Since the assessee is not able to comply with any of the conditions of section 6(6)(a), as listed below, he will, be considered to be ROR., 1. He is non resident in India in at least nine out of ten previous years preceding that year., or, 2. He has during the seven previous years preceding that year been in India for a period of 729 days or, less., Solution 3:, 2021-22, Resident, 2020-21, Resident
Page 439 :
Residential Status and Scope of Total Income, , 36, , 2019-20, Resident, 2018-19, Resident, 2017-18, Resident, 2016-17, Resident, 2015-16, Resident, 2014-15, Resident, 2013-14, Non-Resident, 2012-13, Non-Resident, 2011-12, Non-Resident, Total stay in 7 years preceding the relevant previous year is 742 days., Since the assessee is not able to comply with any of the conditions of section 6(6)(a), as listed below, he will, be considered to be ROR., 1. He is non resident in India in at least nine out of ten previous years preceding that year., or, 2. He has during the seven previous years preceding that year been in India for a period of 729 days or, less., Solution 4:, 2021-22, Resident, 2020-21, Resident, 2019-20, Resident, 2018-19, Resident, 2017-18, Resident, 2016-17, Resident, 2015-16, Resident, 2014-15, Non-Resident, 2013-14, Non-Resident, 2012-13, Non-Resident, 2011-12, Non-Resident, Total stay in 7 years preceding the relevant previous year is 772 days., Since the assessee is not able to comply with any of the conditions of section 6(6)(a), as listed below, he will, be considered to be ROR., 1. He is non resident in India in at least nine out of ten previous years preceding that year., or, 2. He has during the seven previous years preceding that year been in India for a period of 729 days or, less., Solution 5:, 2021-22, Resident, 2020-21, Resident, 2019-20, Resident, 2018-19, Resident, 2017-18, Resident, 2016-17, Resident, 2015-16, Resident, 2014-15, Non-Resident, 2013-14, Resident, 2012-13, Non-Resident, 2011-12, Resident, Total stay in 7 years preceding the relevant previous year is 868 days., Since the assessee is not able to comply with any of the conditions of section 6(6)(a), as listed below, he will, be considered to be ROR., 1. He is non resident in India in at least nine out of ten previous years preceding that year.
Page 440 :
Residential Status and Scope of Total Income, , 37, , or, 2. He has during the seven previous years preceding that year been in India for a period of 729 days or, less., Solution 6:, 2021-22, Resident, 2020-21, Resident, 2019-20, Resident, 2018-19, Resident, 2017-18, Resident, 2016-17, Resident, 2015-16, Resident, 2014-15, Non-Resident, 2013-14, Resident, 2012-13, Non-Resident, 2011-12, Non-Resident, Total stay in 7 years preceding the relevant previous year is 706 days., Since the assessee is able to comply with any of the conditions of section 6(6)(a), as listed below, he will be, considered to be NOR., 1. He is non resident in India in at least nine out of ten previous years preceding that year., or, 2. He has during the seven previous years preceding that year been in India for a period of 729 days or, less., Solution 7:, 2021-22, Resident, 2020-21, Resident, 2019-20, Resident, 2018-19, Resident, 2017-18, Resident, 2016-17, Resident, 2015-16, Resident, 2014-15, Resident, 2013-14, Resident, 2012-13, Non-Resident, 2011-12, Non-Resident, Total stay in 7 years preceding the relevant previous year is 787 days., Since the assessee is not able to comply with any of the conditions of section 6(6)(a), as listed below, he will, be considered to be ROR., 1. He is non resident in India in at least nine out of ten previous years preceding that year., or, 2. He has during the seven previous years preceding that year been in India for a period of 729 days or, less., Solution 8:, 2021-22, 2020-21, 2019-20, 2018-19, 2017-18, 2016-17, 2015-16, 2014-15, , Resident, Resident, Resident, Resident, Resident, Resident, Resident, Resident
Page 441 :
Residential Status and Scope of Total Income, , 38, , 2013-14, Non-Resident, 2012-13, Non-Resident, 2011-12, Non-Resident, Total stay in 7 years preceding the relevant previous year is 781 days., Since the assessee is not able to comply with any of the conditions of section 6(6)(a), as listed below, he will, be considered to be ROR., 1. He is non resident in India in at least nine out of ten previous years preceding that year., or, 2. He has during the seven previous years preceding that year been in India for a period of 729 days or, less., Solution 9:, 2021-22, Resident, 2020-21, Resident, 2019-20, Resident, 2018-19, Resident, 2017-18, Resident, 2016-17, Non-Resident, 2015-16, Resident, 2014-15, Non-Resident, 2013-14, Non-Resident, 2012-13, Non-Resident, 2011-12, Non-Resident, Total stay in 7 years preceding the relevant previous year is 774 days., Since the assessee is not able to comply with any of the conditions of section 6(6)(a), as listed below, he will, be considered to be ROR., 1. He is non resident in India in at least nine out of ten previous years preceding that year., or, 2. He has during the seven previous years preceding that year been in India for a period of 729 days or, less., Solution 10:, Mr. X is in India for 60 days or more in 2021-22 but for less than 365 days in 4 years immediately preceding, 2021-22, so he is non-resident in 2021-22., Solution 11:, Stay of Mr. X in various years is as given below., In P.Y. 2017-18, {July – 31, August – 31, September – 30, October – 31, November – 30, December – 11, March – 5}, Days of stay in India are 169, so Mr. X is non-resident., In P.Y. 2018-19, {April – 30, May – 31, June – 30, July – 21, September – 21, October – 31, November – 30, December – 31,, January – 31, February – 28, March – 1}, Days of stay in India are 285. So, he is resident and also he is non-resident in at least 9 years out of 10 years, preceding the relevant previous year, hence he is NOR., In P.Y. 2019-20, {January – 31, February – 29, March – 31}, Days of stay in India are 91. So, he is resident and also he is non-resident in at least 9 years out of 10 years, preceding the relevant previous year, hence he is NOR., In P.Y. 2020-21, {April – 30, May – 31, June – 30, July – 31, August – 31, September – 23, February – 28, March – 31}, Days of stay in India are 235. So, he is resident and also his stay during seven years preceding the relevant, previous year is 729 days or less, hence he is NOR.
Page 442 :
Residential Status and Scope of Total Income, , 39, , In P.Y. 2021-22, {April – 30, May – 31, June – 30, July – 1, February – 18, March – 31}, Days of stay in India are 141 and during the previous 4 years his stay is for 365 days or more so he is, resident and also he is ROR because he is not able to fulfil any of the conditions of section 6(6)(a). i.e., 1. He is non resident in India in at least nine out of ten previous years preceding that year., or, 2. He has during the seven previous years preceding that year been in India for a period of 729 days or, less., Hence he is ROR ., Solution 12:, In P.Y. 2017-18, {July – 31, August – 31, September – 7, January – 31, February – 28, March – 8}, Days of stay in India are 136, so Mr. Daniel is non-resident., In P.Y. 2018-19, {July – 21, August – 31, September – 20, February – 19, March – 31}, Days of stay in India are 122, so, he is non-resident., In P.Y. 2019-20, {April – 30, May – 9, January – 31, February – 29, March – 31}, Days of stay in India are 130, so, he is non-resident., In P.Y. 2020-21, {April – 30, May – 20, March – 21}, Days of stay in India are 71 and also he stays for 365 days or more during 4 years preceding the relevant, previous year and also he is able to comply with at least one of the conditions of section 6(6)(a) as given, below., 1. He is non resident in India in at least nine out of ten previous years preceding that year., or, 2. He has during the seven previous years preceding that year been in India for a period of 729 days or, less., Hence he is NOR., In P.Y. 2021-22, {April – 30, May – 31, June – 21, March – 5}, Days of stay in India are 87 and during the previous 4 years his stay is more than 365 days. So he is resident, but not ordinarily resident because he is able to fulfill at least one of the two condition given u/s 6(6)(a)., Solution 13:, Since he is out of India every year for 274 days so his days of stay in India are –, In 2021-22, 91 Days, In 2020-21, 92 Days, In 2019-20, 91 Days, In 2018-19, 91 Days, In 2017-18, 91 Days, So his stay in India during the seven years immediately preceding the relevant previous year is less than 729, days, so he is resident but not ordinarily resident., Solution 14:, Days of stay in India in P.Y. 2021-22 are 182., {October – 31, November – 30, December – 31, January – 31, February – 28, March – 31}, So he is resident and also he will be able to comply with at least one of the conditions of section 6(6)(a) as, given below., 1. He is non resident in India in at least nine out of ten previous years preceding that year., or, 2. He has during the seven previous years preceding that year been in India for a period of 729 days or
Page 443 :
Residential Status and Scope of Total Income, , 40, , less., Hence he is NOR., Solution 15:, His days of stay in India are as under –, In P.Y. 2011-12, 275 days, {July – 31, August – 31, September – 30, October – 31, November – 30, December – 31, January – 31,, February – 29, March – 31}, In P.Y. 2012-13, 366, In P.Y. 2013-14, 365, In P.Y. 2014-15, 365, In P.Y. 2015-16, 366, In P.Y. 2016-17, 365, In P.Y. 2017-18, 365, In P.Y. 2018-19, 365, In P.Y. 2019-20, 366, In P.Y. 2020-21, 215, {April – 30, May – 31, June – 30, July – 31, August – 31, September – 30, October – 31, November – 1}, In P.Y. 2021-22, 61, {January – 31, February – 29, March – 1}, He is resident in 2021-22 but he is not able to comply with any of the conditions of section 6(6)(a) hence he, is resident and ordinarily resident., Solution 16:, His days of stay in India in year 2020-21 are 313., {May – 9, June – 30, July – 31, August – 31, September – 30, October – 31, November – 30, December –, 31, January – 31, February – 28, March – 31}, So he is resident and he is also able to comply with one of the condition of section 6(6)(a) hence he will be, considered to be resident but not ordinarily resident., His days of stay in India in 2021-22 are 60., {April – 30 and May – 30}, So he is non–resident in the year 2021-22., Solution 17:, His days of stay in India during 2021-22 are 182., {April – 30, May – 31, June – 30, July – 31, August – 31, September – 29}, So Mr. X is resident in previous year 2021-22 and also he is not able to comply with any of the conditions of, section 6(6)(a) hence he will be considered to be ROR., Solution 18:, It will be accruing arising abroad because if any loan has been taken by a person resident in India from, outside India then interest income shall be accruing arising in India only if such resident has utilized the loan, amount in India., Solution 19:, Particulars, (i) Income accruing/arising outside India but received in India, (ii) Income accruing/arising outside India and also received abroad., (iii) Income accruing/arising in India, (iv) Income accruing/arising outside India and also received abroad., (v) Income accruing/arising outside India and also received outside India but, from a business controlled from India, (vi) Income accruing/arising in India, , ROR, 10,000, 5,000, 10,000, 2,000, 50,000, , NOR, 10,000, -------10,000, -------50,000, , NR, 10,000, -------10,000, ---------------, , 12,000, , 12,000, , 12,000
Page 444 :
Residential Status and Scope of Total Income, (vii) Past profits, Taxable Income, , 41, -------89,000, , ------82,000, , -------32,000, , Solution 20:, Particulars, (1) Income accruing/arising outside India and received outside India, (2) Income accruing/arising in India, (3) Past profits, (4) Income accruing/arising and received outside India, (5) Income accruing/arising and received outside India, but business, controlled from India, (6) Income deemed to be accruing/ arising in India, (7) Income deemed to be accruing/ arising in India, Gross Total Income, , ROR, 40,000, 15,000, ------1,00,000, 1,75,000, , NOR, ------15,000, ------------1,75,000, , NR, ------15,000, -------------------, , 20,000, 2,00,000, 5,50,000, , 20,000, 2,00,000, 4,10,000, , 20,000, 2,00,000, 2,35,000, , ROR, 1,20,000, 2,25,000, 1,45,000, 1,50,000, 1,50,000, 75,000, 65,000, 80,000, 40,000, 10,50,000, , NOR, 1,20,000, 2,25,000, ------1,50,000, ------------65,000, 80,000, 40,000, 6,80,000, , NR, 1,20,000, 2,25,000, ------1,50,000, ------------65,000, 80,000, 40,000, 6,80,000, , ROR, 60,000, , NOR, 60,000, , NR, 30,000, , 10,000, 1,70,000, 2,20,000, 1,00,000, 60,000, 90,000, , -----1,70,000, ----------60,000, 90,000, , -----1,70,000, ----------60,000, ------, , 19,000, 39,000, 4,80,000, , 19,000, 39,000, 4,80,000, , 19,000, 39,000, ------, , 12,000, , 6,000, , 6,000, , 50,000, , 10,000, , 10,000, , 13,10,000 9,34,000, , 3,34,000, , Solution 21:, Particulars, (1) Income accruing /arising in India, (2) Income accruing/arising in India, (3) Income accruing/arising and received outside India, (4) Income accruing/arising in India, (5) Income accruing/arising outside India and received outside India, (6) Income accruing/arising outside India and received outside India, (7) Income accruing/arising in India, (8) Income accruing/arising in India, (9) Income received in India, Gross Total Income, Solution 22:, (1) Income accruing/arising outside India from a business controlled in, India, half of the income received in India, (2) Income accruing/arising outside India and received outside India, (3) Income accruing/arising in India, (4) Income accruing/arising outside India and received outside India, (5) Income accruing/arising outside India and received outside India, (6) Income received in India, (7) Income accruing/arising and received outside India, but profession, set up in India, (8) Income accruing/arising in India, (9) Income accruing/arising in India, (10) Income accruing/arising outside India and received outside India,, but business controlled from India, (11) Income accruing/arising outside India, half received outside India, and half in India, (12) Income accruing/arising outside India, 4/5th received outside India, and 1/5th in India, Gross Total Income, , Solution 23:, Since Mr. X is covered in special category and will be resident, if his stay in India in relevant previous year
Page 445 :
Residential Status and Scope of Total Income, , 42, , is 182 days or more, hence Mr. X is a non–resident as his stay in India is less than 182 days and his income, taxable in India shall be, `, Income under the head Salary, Income accruing/arising in India, 3,00,000.00, (60,000 x 5), Income received in India, 60,000.00, (60,000 x 1), Gross Salary, 3,60,000.00, Less: Standard Deduction u/s 16(ia), (50,000.00), Gross Salary, 3,10,000.00, Income under the head Other Sources, Gift of gold (`11,00,000 – `8,00,000), 3,00,000.00, Gift of building (`13,00,000 – `10,00,000), 3,00,000.00, Income under the head Other Sources, 6,00,000.00, Gross Total Income, 9,10,000.00, Less: Deduction u/s 80C to 80U, Nil, Total Income, 9,10,000.00, Computation of Tax Liability, Tax on `9,10,000 at slab rate, 94,500.00, Add: HEC @ 4%, 3,780.00, Tax Liability, 98,280.00, Note: STCG is received in USA is not taxable in India as the assessee is a non-resident., Solution 24:, In this case, Mrs. X stays in India for more than 182 days during the previous year 2021-22 and also she is, not able to comply with any of the conditions of section 6(6)(a), she will be considered to be ROR., Her incomes taxable in India shall be, `, Income under the head Salary, Income accruing/arising in India, 24,00,000.00, (2,00,000 x 12), Gross Salary, 24,00,000.00, Less: Standard deduction u/s 16(ia), (50,000.00), Income under the head Salary, 23,50,000.00, Income under the head Other Sources, Gift from friend (immovable property), 51,000.00, Interest from UK Development bond, 2,00,000.00, (Received in USA), Income under the head Other Sources, 2,51,000.00, Gross Total Income, 26,01,000.00, Less: Deduction u/s 80C to 80U, Nil, Total Income, 26,01,000.00, Computation of Tax Liability, Tax on `26,01,000 at slab rate, 5,92,800.00, Add: HEC @ 4%, 23,712.00, Tax Liability, 6,16,512.00, Rounded off u/s 288B, 6,16,510.00
Page 446 :
Residential Status and Scope of Total Income, , 43, , SOLUTIONS OF EXAMINATIONS QUESTIONS, MAY – 2019 (NEW COURSE), Solution 2(a) :, As per section 5, All Global Income of ROR shall be taxable in India but in case of NOR income accruing, arising in India or received in India shall be taxable in India. In case of NOR, income accruing / arising, aboard and received aboard but from a business controlled from India or from a profession which was set up, in India shall be taxable in India., S.No., (i), (ii), (iii), (iv), (v), (vi), , Particulars, Income from business in India, controlled from London, Profit from business in Japan, controlled from India, Past years untaxed foreign income brought to India, Royalty Income from a resident for technical service to run business, outside India (assumed amount received in India), Agriculture Income from Bhutan (i.e. outside India) assumed received, in Bhutan, Income from house property in Dubai received in Dubai, Gross Total Income, , ROR, 2,00,000, 70,000, 4,00,000, , NOR, 2,00,000, 70,000, 4,00,000, , 90,000, , -, , 73,000, 8,33,000, , 6,70,000, , Note: Student can take assumption that royalty received outside India, in such case royalty shall be, taxable in case of ROR only., Note: In the above solution income of 73,000 is presumed to be computed income under the head, house property. Student can also presume such amount as rent received (as the amount is deposited in, bank account) and standard deduction u/s 24(a) @ 30% shall be allowed from 73,000 and taxable, amount shall be 51,100., , MAY – 2019 (OLD COURSE), Solution 2(a):, As per section 5, All Global Income of ROR shall be taxable in India but in case of NOR/NR income, accruing arising in India or received in India shall be taxable in India. In case of NOR, income accruing /, arising abroad and received abroad but from a business controlled from India or from a profession which, was set up in India shall be taxable in India., S.No., Particulars, (i), Short term capital gains on sale of shares in Indian, company received in Japan (income accruing/arising, from India), (ii), Rent from property in Bangladesh deposited in a bank, at Dhaka, later on remitted to India through approved, banking channels (income accruing arising abroad, received abroad) (96,000 – 30% of 96,000), Gross Total Income, Less: Deduction u/s 80C to 80U, Total Income, , ROR, 85,000, , NOR, 85,000, , NR, 85,000, , 67,200, , -, , -, , 1,52,200, 1,52,200, , 85,000, 85,000, , 85,000, 85,000
Page 447 :
Residential Status and Scope of Total Income, , 44, , NOV – 2018 (NEW COURSE), Solution 2(a):, As per section 5, Income of ROR is taxable if income is accruing/arising in India or abroad but in case of, NOR & Non- Resident Income is taxable if it is accruing arising or deemed to accrue/arise in India or, received or deemed to be received in India even if accrued outside India. All Global Income of ROR is, taxable in India., Particulars, ROR, NOR, Non –, Resident, Pension received from the US Government (Income, 3,20,000, accruing/arising abroad and received abroad), Agricultural income from lands in Malaysia (Income, 2,70,000, accruing/arising abroad and received abroad), Rent received from let out property in Colombo, Sri, 2,94,000, Lanka (4,20,000 – 30% of 4,20,000) (Income, accruing/arising abroad and received abroad), Total, 8,84,000, -, , NOV – 2018 (OLD COURSE), Solution 2(a):, (i) As per section 6(1), an individual is said to be resident in India in any previous year, if he satisfies any, one of the following conditions:, (i) He stays in India for 182 days or more during the relevant previous year, (ii) He stays in India for 60 days or more and also for 365 days or more during 4 years preceding the, relevant previous year., If the individual satisfies any one of the conditions mentioned above, he is a resident, otherwise the, individual is a non-resident., In the given case his stay in India in the previous year is less than 60 days hence he will be Non-Resident., As per section 5, in case of Non-resident –The following incomes shall be taxable., (i), income accruing /arising in India., (ii), income received or deemed to be received in India even if accruing /arising abroad., (ii) Computation of Total Income for the A.Y. 2022-23, (1) Short term capital gains on the sale of shares of an Indian company is income accruing/arising in, India but the amount is received outside India. As Income is accruing/arising in India hence it is, taxable in case of Non-Resident., (2) Interest on Fixed Deposits with State Bank of India (Mumbai) is income accruing/arising in India, hence taxable in case of Non-resident., Short term capital gains, Interest on fixed deposits, Gross Total Income, Less: Deduction u/s 80C to 80U, Total Income, , `, 35,000, 8,000, 43,000, Nil, 43,000
Page 448 :
Residential Status and Scope of Total Income, , 45, , MAY – 2018 (NEW COURSE), Solution 3:, S.No., Particulars, (i), Interest from German Derivatives Bonds (1/3 received in India), (ii) Income from agriculture land situated in Malaysia ,remitted to India, (iii) Income earned from business in Dubai, Controlled from India (`20,000, received in India)., (iv) Profit from business in Mumbai, controlled from Australia, (v), Interest received from Mr. Ashok (NRI) on loan provided to him for, business in India, (vi) Profit from business in Canada controlled from Mumbai (60% of profits, deposited in a bank in Canada and 40% remitted to India), Note: The language is not clear. It should be 100% deposited in, Canada but 40% remitted to India. OR It should be 60% deposited, in Canada and 40% deposited in India and in that case 24,000 (40%, of 60,000) shall be taxable in case of NR., (vii) Amount received from an NRI for the use of know-how for his business, in Singapore, (viii) Past years untaxed foreign income brought to India, Total, , ROR, 21,000, 51,000, 75,000, , NR, 7,000, 20,000, , 1,75,000, 35,000, , 1,75,000, 35,000, , 60,000, , -, , 8,00,000, , -, , 12,17,000, , 2,37,000, , MAY – 2018 (OLD COURSE), Solution 3(a):, Under section 6(1), an individual is said to be resident in India in any previous year, if he satisfies any one, of the following conditions:, (i) He stays in India for 182 days or more during the relevant previous year, (ii) He stays in India for 60 days or more and also for 365 days or more during 4 years preceding the, relevant previous year., If the individual satisfies any one of the conditions mentioned above, he is a resident, otherwise the, individual is a non-resident., Meaning of Not-ordinarily resident Section 6(6)(a), An individual who is resident of India shall be considered to be NOR if he has complied with at least one of, the conditions given below:, (i), If such individual has during the 7 previous years preceding the relevant previous year been in India, for a period of 729 days or less or, (ii), If such individual has been non-resident in India in 9 years out of 10 previous years preceding the, relevant previous year, If he has not complied with even a single condition, he will be considered to be ROR., Stay and Status of Mrs. Karuna Kapoor in various years is as given below., In P.Y. 2013-14, {April – 28, May – 31, June – 30, July-11}, Days of stay in India are 100, so she is non-resident., In P.Y. 2014-15, {April – 28, May – 31, June – 30, July-11}, Days of stay in India are 100, so she is non-resident., In P.Y. 2015-16, {April – 28, May – 31, June – 30, July-11}, Days of stay in India are 100, so she is non-resident.
Page 449 :
Residential Status and Scope of Total Income, , 46, , In P.Y. 2016-17, {April – 28, May – 31, June – 30, July-11}, Days of stay in India are 100, so she is non-resident., In P.Y. 2017-18, { May – 15, June – 30, July-31, August- 31, September - 30 }, Days of stay in India are 137, so she is resident and also she is non-resident in at least 9 years out of 10 years, preceding the relevant previous year, hence she is NOR., In P.Y. 2018-19, { September – 24, October – 31, November – 30, December – 31, January – 31, February – 28, March – 26}, Days of stay in India are 201, so she is resident and also she is non-resident in at least 9 years out of 10 years, preceding the relevant previous year, hence she is NOR., In P.Y. 2019-20, {February – 20, March – 26}, Days of stay in India are 46. So, she is non - resident., In P.Y. 2020-21, {June – 9, July – 11}, Days of stay in India are 20. So, she is non – resident, In P.Y. 2021-22, {April – 28, May – 31, June – 30, July – 11}, Days of stay in India are 100. So, she is resident and also her stay during seven years preceding the relevant, previous year is 729 days or less, hence she is NOR., , NOV – 2017, Question 2(a), (5 Marks), Answer:, (i) As per section 6(3), A company would be resident in India if, (a) it is an Indian Company; or, (b) its place of effective management in that year is in India., In the given case, DAISY Ltd. a foreign company therefore it would be resident in India if its place of, effective management in that year is in India., Section 6(3) defines “place of effective management” to mean a place where key management and, commercial decisions that are necessary for the conduct of the business of an entity as a whole are, in, substance made. In the case of DAISY Ltd., its place of effective management for P.Y.2021-22 is not in, India, since the significant management and commercial decisions are, in substance, made by the Board of, Directors outside India in USA., Hence, DAISY Ltd, being a foreign company is a non-resident for A.Y.2022-23, since its place of effective, management is outside India in the P.Y.2021-22., (ii) If any non - resident is purchasing goods from India for the purpose of export, such income shall not be, accruing/arising in India, hence it is not taxable in India., , MAY – 2017, Question 2(a) (i), (4 Marks), Answer:, As per section 6(1), Mr. Damodhar is covered in special category., Period of stay in India shall exclude time period from 12th August 2021 to 21st January, 2022 and also 29, days for which he stays out of India., His stay in India shall be 173 days [April – 30 + May – 2 + June – 30 + July – 31 + August – 11 + January –, 10 + February – 28 + March – 31], Since his period of stay in India during the P.Y.2021-22 is less than 182 days, he is a non-resident for, A.Y.2022-23.
Page 450 :
Residential Status and Scope of Total Income, , 47, , Note - Since the residential status of Mr. Damodhar is “non-resident” for A.Y.2022 – 23 consequent to his, number of days of stay in P.Y.2021–22 being less than 182 days, his period of stay in the earlier previous, years become irrelevant., Question 4(a), , (4 Marks), , Solution:, (a) Taxable in India: As per section 9, Amount received for use of Patent for a Business in India shall, be deemed as Income accruing arising in India and chargeable to tax in India., (b) Not Taxable in India: As per section 9, Amount received in Japan for use of know - how for a, business in Sri Lanka shall not be deemed as Income accruing arising from India and not chargeable, to tax in India., (c) Taxable in India: As per section 9, Amount received for use of Technical know - how in India shall, be deemed as Income accruing arising in India and shall be taxable in India., (d) Not Taxable in India: As per section 9,Amount received in Nepal for Feasibility study conducted, for the new project in Nepal shall not be deemed as Income accruing arising from India and not, chargeable to tax in India., , NOV – 2016, Question 4(a), (2 Marks), Answer:, As per section 9, if source of income is in India, income shall be accruing/arising in India and shall be, taxable in all the three status even if income has been received outside India, hence in the given case Income, is chargeable to tax in India as income is accruing and arising from India as property is situated in India., , NOV – 2015, Question 2(a)., (4 Marks), Solution:, Residential status of Mr. X for the Assessment Year 2022-23, As per Section 6(1), any individual who is a citizen of India and has left India for taking up any business or, profession or employment outside India are covered in the special category and they will be considered to be, resident only if they stay in India for 182 days or more i.e. second condition of 60 plus 365 days shall not be, applicable., In the given case Mr. X is covered under special category and stay in the current year is 99 days only which, is less than 182 days. So he is non-resident., As per section 9, if any non-resident has provided any patent right or any managerial, technical services and, such patent right etc. was used in India, in such cases any royalty or fee received by non-resident shall be, considered to be income accruing/arising in India and shall be taxable and it do not matter that the nonresident do not have residence or place of business or business connection in India i.e. there is no territorial, nexus or non-resident has not rendered services in India., In the given case Fees charged from XYZ Limited is taxable in India because the assesse has transferred, technical documents and designs to setup an automobile factory in Faridabad (i.e. in India)., , MAY – 2015, Question 2(a)(i)., (4 Marks), Solution:, (a) Taxable in India: As Mr. David is an Central Government employee and salary paid by Central, Government for the services rendered outside India is taxable in India., (b) Taxable in India: As service is rendered in India means income accruing and arising from India., (c) Taxable outside India: As Royalty is paid for business carried outside India means Income accruing, outside India., (d) Taxable in India: As the loan is used for business in India.
Page 451 :
Residential Status and Scope of Total Income, , 48, , Question 2(a)(ii)., Solution:, Ms. Bindu is a resident since her stay in the previous year 2021-22 is 365 days., , (4 Marks), , 2020-21 2019-20 2018-19 2017-18 2016-17 2015-16 2014-15 -, , 365, 42 (i.e. 11 + 31), Nil (since she is residing in New York), Nil (since she is residing in New York), Nil (since she is residing in New York), Nil (since she is residing in New York), Nil (since she is residing in New York), 407, Since stay in 7 years preceding the relevant previous year is less than 729 days and She is non-resident in 9, years preceding the relevant previous year, she will be considered to be NOR., , NOV – 2014, Question 2(a)., , (5 Marks), , Solution: Computation of Total Income of Mrs. X and Mrs. Y for the A.Y. 2022-23, Sl.No. Particulars, Mrs. X, NR, (i), Income from Profession in Malaysia, (set up in India) received, there, (ii), Profit from business in Delhi, but managed directly from Malaysia, 40,000, (iii), Rent (computed) from property in Malaysia deposited in a Bank at, Malaysia, later on remitted to India through approved banking, channels., (iv), Cash gift received from a friend on Mrs. Y’s 50th birthday, (v), Agricultural income from land in Maharashtra (exempt u/s 10(1)), (vi), Past foreign untaxed income brought to India, (vii), Fees for technical services rendered in India received in Malaysia, 25,000, (viii) Income from a business in Pune (Mrs. X receives 50% of the, 12,000, income in India), (ix), Interest on debentures in an Indian company (Mrs. X received the, 18,500, same in Malaysia), (x), Short-term capital gain on sale of shares of an Indian company, 15,000, (xi), Interest on fixed deposit with SBI in India, 12,000, Total Income, 1,22,500, , Mrs. Y, ROR, 51,000, 15,000, 14,000, 25,500, 8,000, 1,13,500, , MAY – 2013, Question 2(a)., Solution:, Computation of Total Income of Mr. X & Mr. Y for the A.Y. 2022-23, Sl., Particulars, No., 1., 2., 3., 4., 5., , Interest on American Development Bonds, Short term capital gains on sale of shares of an Indian company, received in India, Profit from a business in Mumbai but managed directly from America, Income from a business in Mumbai, Fees for technical services rendered in America, and received in, America, but services utilized in India, , (4 Marks), Mr. X, NR, (`), 23,000, 45,000, , Mr. Y, ROR, (`), 18,000, 75,000, , 10,000, 32,000, 1,50,000, , 28,000, -
Page 452 :
Residential Status and Scope of Total Income, 6., 7., , Interest on fixed deposit with State Bank of India, Mumbai, Income from house property at Mumbai, Total Income, , 49, , 4,500, 67,200, 3,31,700, , 12,000, 38,500, 1,71,500, , MAY – 2012, Question 2, (5 Marks), Answer: Computation of Total Income of Mr. X and Mr. Y for the A.Y. 2022-23, Sl. Particulars, Mr. X, Mr. Y, No., Non-Resident, ROR, 1., Interest on Canada Development Bond, (only 50% of interest received, 17,500, 40,000, in India), 2., Profit from a business in Nagpur, but managed directly from London, 1,00,000, 1,40,000, 3., Short term capital gain on sale of shares of an Indian company, 60,000, 90,000, received in India, 4., Income from a business in Chennai, 80,000, 70,000, 5., Fees for technical services rendered in India, but received in Canada, 1,00,000, ----6., Interest on fixed bank deposit in UCO Bank, Delhi, 7,000, 12,000, 7., Agricultural income from a land situated in Andhra Pradesh, ----------8., Income under the head house property at Bhopal, 1,00,000, 60,000, Total Income, 4,64,500, 4,12,000, Notes:, 1. Agricultural income from a land situated in Andhra Pradesh (in India), is exempted under section 10(1) of, Income tax Act, 1961 in case of both non-resident and resident assessee., , NOV – 2011, Question 5, (3 Marks), Answer:, Mr. X is a non-resident in the previous year 2021-22 as he doesn’t come to India during the year., Computation of Gross total income of Mr. X for A.Y. 2022-23, `, Income under the head Salary, Income under the head Salary {Taxable as it is deemed to be earned in India u/s 9}, 5,00,000, Income under the head Other Sources, Interest on FDR, 1,00,000, Income from agriculture in Pakistan, Nil, (assumed to be received outside India), Income under the head Other Sources, 1,00,000, Income under the head House Property, Income from house property in Pakistan, Nil, (assumed to be received outside India), Gross Total Income, 6,00,000, , MAY – 2010, Answer., Computation of Total Income of Mr. X for the A.Y. 2022-23, Particulars, Resident &, Resident but, ordinarily, not ordinarily, resident, resident, a) Profit on sale of shares of an Indian company, received in, 15,000, 15,000, Germany, c) Income from business in London deposited in a bank in, 75,000, London, , NonResident, 15,000, -
Page 453 :
Residential Status and Scope of Total Income, c) Agricultural income from land in Gujarat [See Note (ii), below], TOTAL INCOME, Note : Agricultural income is exempt under section 10(1)., , 50, , -, , -, , -, , 90,000, , 15,000, , 15,000, , MAY – 2010, Question 1, Answer:, (i) As per section 5, in case of non-resident, incomes taxable shall be:, (a) any income deemed to be accruing/arising in India., (b) any income which is received or is deemed to be received in India., , (2 Marks each), , (ii) As per Section 9, salaries payable by the Government to a citizen of India for services rendered outside, India is deemed to accrue or arise in India. Hence, salary received by Mr. X, a citizen of India, from the, Government of India for services rendered outside India is chargeable to tax under the head ‘Salaries’., , NOV – 2009, Question 1, , (4 Marks), , Answer:, Computation of Total Income of Mr. X and Mr. Y for the A.Y. 2022-23, Sl. Particulars, No., 1., Interest on U.K. Development Bonds, 2., Profit from a business in Mumbai but managed directly from London, 3., Profit on sale of shares of an Indian company received in India, 4., Income from a business in Delhi, 5., Fees for technical services rendered in India but received in London, 6., Interest on fixed Deposit in SBI Bangalore, 7., Agricultural income from a land in Rajasthan [(Exempt u/s.10(1)], 8., Income under the head House property at Bangalore, Gross Total Income, , Mr. X, Non-Resident, 12,500, 10,000, 50,000, 20,000, 1,00,000, 5,000, 50,400, 2,47,900, , Mr. Y, ROR, 20,000, 12,000, 80,000, 20,000, 15,000, 33,600, 1,80,600, , Note: Agricultural income from a land situated in the State of Rajasthan, is exempted under section 10(1) of, Income tax Act, 1961 in case of both non-resident and resident assessee., , MAY – 2007, Question 1, (2 Marks), Answer: True: A person is said to be “not-ordinarily resident” in India if he satisfies either of the, conditions given in sub-section (6) of section 6. This sub-section relates to only individuals and Hindu, Undivided Families. Therefore, only individuals and Hindu undivided families can be resident, but not, ordinarily resident in India. All other classes of assessees can be either a resident or non-resident for the, purpose of income-tax. A firm can, therefore, either be a resident or non-resident.
Page 454 :
Income Under The Head House Property, , 51, , INCOME UNDER THE HEAD, , HOUSE PROPERTY, SECTION 22 TO 27, , SOLUTIONS OF MCQS, Answer, 1.(c); 2.(b); 3. (b); 4. (c); 5. (b); 6. (a); 7. (a); 8. (b); 9. (b); 10. (c); 11.(b); 12. (b); 13. (d); 14. (a); 15. (b); 16., (b); 17. (c); 18. (c); 19. (c); 20. (a); 21. (b); 22. (b); 23. (a); 24. (a); 25. (a); 26. (d); 27. (a); 28. (a); 29. (d);, 30. (a), Hint for answer 8., Arrears of rent received, Less: Deduction under section 25A @ 30%, Income from House property, Hint for answer 9., Recovery of Unrealised Rent, Less: Deduction @ 30%, Income under the head House Property, Hint for answer 13., , 30,000.00, (9,000.00), 21,000.00, 90,000, (27,000), 63,000, `, , Gross Annual Value, Working Note:, (a) Municipal value of property, (b) Fair rent, (c) Higher of (a) and (b), (d) Standard rent, (e) Annual Letting Value / Expected Rent [lower of (c) and (d)], (f) Actual rent [20,000 x 9] + [30,000 x 3], (g) Gross Annual Value [higher of (e) and (f)], Hint for answer 16., Prior period interest, From 01.04.2017 to 31.03.2020, = 5,00,000 x 12% x 3 = 1,80,000, 1,80,000 allowed in 5 equal instalments, = 1,80,000 / 5 = ` 36,000 per annum, Current period interest, From 01.04.2021 to 31.03.2022, = 5,00,000 x 12% x 1 = `60,000, Total Interest = `60,000 + ` 36,000 = `96,000, Hint for answer 18., Computation of income under the head House Property, , 3,00,000, 2,50,000, 3,00,000, 2,90,000, 2,90,000, 2,70,000, 2,90,000, , `, 2,90,000
Page 455 :
Income Under The Head House Property, Gross Annual Value (20,000 X 12), Less: Municipal Tax, Net Annual Value, Less: Standard Deduction u/s 24(a), Less: Interest u/s 24(b), Income from House Property, Hint for answer 19., Prior period interest, Current period interest, From 01.05.2021 to 31.03.2022, = 20,00,000 x 6% x 11/12 = `1,10,000, Total Interest = `1,10,000 + ` 0 = `1,10,000, , 52, 2,40,000.00, (60,000.00), 1,80,000.00, (54,000.00), NIL, 1,26,000.00, NIL
Page 456 :
Income Under The Head House Property, , 53, , SOLUTIONS, TO, , PRACTICE PROBLEMS, Solution 1:, Computation of income under the head House Property, Gross Annual Value, Working Note:, (a) Fair Rent ( ` 80,000 x 12), (b) Municipal Valuation (` 81,000 x 12), (c) Higher of (a) or (b), (d) Standard Rent (` 1,00,000 x 12), (e) Expected Rent {Lower of (c) or (d)}, (f) Rent received (`90,000 x 12), GAV = Higher of (e) or (f), Less: Municipal Tax, Net Annual Value, Less: 30% of NAV u/s 24(a), Less: Interest on capital borrowed u/s 24(b), Income under the head House Property, Gross Total Income, Less: Deduction u/s 80C to 80U, Total Income, Computation of Tax Liability, Tax on `6,25,000 at slab rate, Add: HEC @ 4%, Total tax liability, Solution 2:, Computation of income under the head House Property, Gross Annual Value, Working Note:, (a) Fair Rent (2,50,000 x 12), (b) Municipal Value (2,75,000 x 12), (c) Higher of (a) or (b), (d) Standard Rent (2,80,000 x 12), (e) Expected Rent {Lower of (c) or (d)}, (f) Rent received /receivable (3,00,000 x 12), GAV shall be higher of (e) or (f), Less: Municipal Tax, Net Annual Value, Less: 30% of NAV u/s 24(a), Less: Interest on capital borrowed u/s 24(b), Income under the head House Property, Gross Total Income, Less: Deduction u/s 80C to 80U, , `, 10,80,000, `, 9,60,000, 9,72,000, 9,72,000, 12,00,000, 9,72,000, 10,80,000, 10,80,000, (70,000), 10,10,000, (3,03,000), (82,000), 6,25,000, 6,25,000, Nil, 6,25,000, 37,500, 1,500, 39,000, `, 36,00,000.00, `, 30,00,000, 33,00,000, 33,00,000, 33,60,000, 33,00,000, 36,00,000, 36,00,000, (6,00,000.00), 30,00,000.00, (9,00,000.00), (3,00,000.00), 18,00,000.00, 18,00,000.00, Nil
Page 457 :
Income Under The Head House Property, Total Income, Computation of Tax Liability, Tax on `18,00,000 @ 30%, Add: HEC @ 4%, Tax Liability, Solution 3., Computation of income under the head House Property, Gross Annual Value (2,00,000 x 12), Working Note:, (a) Fair Rent (1,80,000 x 12), (b) Standard Rent (2,20,000 x 12), (c) Expected Rent (lower of (a) or (b), (d) Rent Received/Receivable (2,00,000 x 12), GAV = Higher of (c) or (d), Less: Municipal Tax, Net Annual Value, Less: 30% of NAV u/s 24(a), Less: Interest on capital borrowed u/s 24(b), Income under the head House Property, Gross Total Income, Less: Deduction u/s 80C to 80U, Total Income, Computation of Tax Liability, Tax on `12,60,000 @ 30%, Add: HEC @ 4%, Tax Liability, , 18,00,000.00, 5,40,000.00, 21,600.00, 5,61,600.00, `, 24,00,000.00, `, 21,60,000, 26,40,000, 21,60,000, 24,00,000, 24,00,000, , Solution 4: Computation of Total Income & Tax Liability of Mr. X, Gross Annual Value, Working Note:, `, (a) Fair Rent (`55,000 x 12), 6,60,000, (b) Municipal Valuation (`52,000 x 12), 6,24,000, (c) Higher of (a) or (b), 6,60,000, (d) Standard Rent (`60,000 x 12), 7,20,000, (e) Expected Rent {Lower of (c) or (d)}, 6,60,000, (f) Rent received /receivable (50,000 x 9), 4,50,000, If there was no vacancy, in that case rent received/receivable would have been, (50,000 x 12) `6,00,000 which is not exceeding expected rent hence GAV shall be, expected rent i.e. `6,60,000, GAV, 6,60,000, Less: Municipal Tax, Net Annual Value, Less: 30% of NAV u/s 24(a), Less: Interest on capital borrowed u/s 24(b), Income under the head House Property, Total Income, Computation of Tax Liability, Tax on Normal Income `4,21,000 at slab rate, Less: Rebate u/s 87A, Tax Liability, , 54, , (6,00,000.00), 18,00,000.00, (5,40,000.00), Nil, 12,60,000.00, 12,60,000.00, Nil, 12,60,000.00, 3,78,000.00, 15,120.00, 3,93,120.00, `, 6,60,000.00, , (30,000.00), 6,30,000.00, (1,89,000.00), (20,000.00), 4,21,000.00, 4,21,000.00, 8,550.00, (8,550.00), Nil
Page 458 :
Income Under The Head House Property, Solution 4(b): Computation of Total Income & Tax Liability of Mr. X, Gross Annual Value, Working Note:, `, (a) Fair Rent (`55,000 x 12), 6,60,000, (b) Municipal Valuation (`52,000 x 12), 6,24,000, (c) Higher of (a) or (b), 6,60,000, (d) Standard Rent (`60,000 x 12), 7,20,000, (e) Expected Rent {Lower of (c) or (d)}, 6,60,000, (f) Rent received /receivable (60,000 x 9), 5,40,000, If there was no vacancy, in that case rent received/receivable would have been, (60,000 x 12) `7,20,000 which is exceeding expected rent hence GAV shall be Rent, received/receivable i.e. `5,40,000, GAV, 5,40,000, Less: Municipal Tax, Net Annual Value, Less: 30% of NAV u/s 24(a), Less: Interest on capital borrowed u/s 24(b), Income under the head House Property, Total Income, Computation of Tax Liability, Tax on Normal Income `3,37,000 at slab rate, Less: Rebate u/s 87A, Tax Liability, Solution 4(c): Computation of Total Income & Tax Liability of Mr. X, Gross Annual Value, Working Note:, `, (a) Fair Rent (`55,000 x 12), 6,60,000, (b) Municipal Valuation (`52,000 x 12), 6,24,000, (c) Higher of (a) or (b), 6,60,000, (d) Standard Rent (`60,000 x 12), 7,20,000, (e) Expected Rent {Lower of (c) or (d)}, 6,60,000, (f) Rent received /receivable (55,000 x 9), 4,95,000, If there was no vacancy, in that case rent received/receivable would have been, (55,000 x 12) `6,60,000 which is equal to expected rent hence GAV shall be Rent, Received/ receivable i.e. `4,95,000, GAV, 4,95,000, Less: Municipal Tax, Net Annual Value, Less: 30% of NAV u/s 24(a), Less: Interest on capital borrowed u/s 24(b), Income under the head House Property, Total Income, Computation of Tax Liability, Tax on Normal Income `3,05,500 at slab rate, Less: Rebate u/s 87A, Tax Liability, , 55, `, 5,40,000.00, , (30,000.00), 5,10,000.00, (1,53,000.00), (20,000.00), 3,37,000.00, 3,37,000.00, 4,350.00, (4,350.00), Nil, `, 4,95,000.00, , (30,000.00), 4,65,000.00, (1,39,500.00), (20,000.00), 3,05,500.00, 3,05,500.00, 2,775.00, (2,775.00), Nil
Page 459 :
Income Under The Head House Property, Solution 4(d): Computation of Total Income & Tax Liability of Mr. X, Gross Annual Value, Working Note:, `, (a) Fair Rent (`55,000 x 12), 6,60,000, (b) Municipal Valuation (`52,000 x 12), 6,24,000, (c) Higher of (a) or (b), 6,60,000, (d) Standard Rent (`60,000 x 12), 7,20,000, (e) Expected Rent {Lower of (c) or (d)}, 6,60,000, (f) Rent received /receivable (1,00,000 x 9), 9,00,000, If there was no vacancy, in that case rent received/receivable would have been, (1,00,000 x 12) `12,00,000 which is exceeding expected rent hence GAV shall be Rent, received/receivable i.e. `9,00,000, GAV, 9,00,000, Less: Municipal Tax, Net Annual Value, Less: 30% of NAV u/s 24(a), Less: Interest on capital borrowed u/s 24(b), Income under the head House Property, Total Income, Computation of Tax Liability, Tax on Normal Income `5,89,000 at slab rate, Add: HEC @ 4%, Tax Liability, Rounded off u/s 288B, Solution 5:, , 56, `, 9,00,000.00, , (30,000.00), 8,70,000.00, (2,61,000.00), (20,000.00), 5,89,000.00, 5,89,000.00, 30,300.00, 1,212.00, 31,512.00, 31,510.00, `, , Computation of Total Income & Tax Liability of Mr. X, Gross Annual Value, Working Note:, `, (a) Fair Rent (`15,50,000 x 12), 186,00,000, (b) Municipal Valuation (`15,20,000 x 12), 182,40,000, (c) Higher of (a) or (b), 186,00,000, (d) Standard Rent (`16,00,000 x 12), 192,00,000, (e) Expected Rent {Lower of (c) or (d)}, 186,00,000, (f) Rent received /receivable (15,00,000 x 9), 135,00,000, If there was no vacancy, in that case rent received/receivable would have been, (15,00,000 x 12) `180,00,000 which is not exceeding expected rent hence GAV shall, be expected rent i.e. `186,00,000, GAV, 186,00,000, Less: Municipal Tax, Net Annual Value, Less: 30% of NAV u/s 24(a), Less: Interest on capital borrowed u/s 24(b), Income under the head House Property, Total Income, Computation of Tax Liability, Tax on Normal Income `109,10,000 at slab rate, Add: Surcharge @ 15%, Tax before health & education cess, Add: HEC @ 4%, Tax Liability, Rounded off u/s 288B, , 186,00,000.00, , (13,00,000.00), 173,00,000.00, (51,90,000.00), (12,00,000.00), 109,10,000.00, 109,10,000.00, 30,85,500.00, 4,62,825.00, 35,48,325.00, 1,41,933.00, 36,90,258.00, 36,90,260.00
Page 460 :
Income Under The Head House Property, Solution 5(b):, , 57, `, , Computation of Total Income & Tax Liability of Mr. X, Gross Annual Value, Working Note:, `, (a) Fair Rent (`15,50,000 x 12), 186,00,000, (b) Municipal Valuation (`15,20,000 x 12), 182,40,000, (c) Higher of (a) or (b), 186,00,000, (d) Standard Rent (`16,00,000 x 12), 192,00,000, (e) Expected Rent {Lower of (c) or (d)}, 186,00,000, (f) Rent received /receivable (16,00,000 x 9), 144,00,000, If there was no vacancy, in that case rent received/receivable would have been, (16,00,000 x 12) `192,00,000 which is exceeding expected rent hence GAV shall be, rent received/receivable i.e. `144,00,000, GAV, 144,00,000, Less: Municipal Tax, Net Annual Value, Less: 30% of NAV u/s 24(a), Less: Interest on capital borrowed u/s 24(b), Income under the head House Property, Total Income, Computation of Tax Liability, Tax on Normal Income `79,70,000 at slab rate, Add: Surcharge @ 10%, Tax before health & education cess, Add: HEC @ 4%, Tax Liability, Rounded off u/s 288B, Solution 5(c):, , 144,00,000.00, , (13,00,000.00), 131,00,000.00, (39,30,000.00), (12,00,000.00), 79,70,000.00, 79,70,000.00, 22,03,500.00, 2,20,350.00, 24,23,850.00, 96,954.00, 25,20,804.00, 25,20,800.00, `, , Computation of Total Income & Tax Liability of Mr. X, Gross Annual Value, Working Note:, `, (a) Fair Rent (`15,50,000 x 12), 186,00,000, (b) Municipal Valuation (`15,20,000 x 12), 182,40,000, (c) Higher of (a) or (b), 186,00,000, (d) Standard Rent (`16,00,000 x 12), 192,00,000, (e) Expected Rent {Lower of (c) or (d)}, 186,00,000, (f) Rent received /receivable (15,60,000 x 9), 140,40,000, If there was no vacancy, in that case rent received/receivable would have been, (15,60,000 x 12) `187,20,000 which is exceeding expected rent hence GAV shall be, rent received/ receivable i.e. `140,40,000, GAV, 140,40,000, Less: Municipal Tax, Net Annual Value, Less: 30% of NAV u/s 24(a), Less: Interest on capital borrowed u/s 24(b), Income under the head House Property, Total Income, Computation of Tax Liability, Tax on Normal Income `77,18,000 at slab rate, Add: Surcharge @ 10%, Tax before health & education cess, , 140,40,000.00, , (13,00,000.00), 127,40,000.00, (38,22,000.00), (12,00,000.00), 77,18,000.00, 77,18,000.00, 21,27,900.00, 2,12,790.00, 23,40,690.00
Page 461 :
Income Under The Head House Property, Add: HEC @ 4%, Tax Liability, Rounded off u/s 288B, , 58, 93,627.60, 24,34,317.60, 24,34,320.00, , Solution 5(d):, , `, Computation of Total Income & Tax Liability of Mr. X, , Gross Annual Value, Working Note:, `, (a) Fair Rent (`15,50,000 x 12), 186,00,000, (b) Municipal Valuation (`15,20,000 x 12), 182,40,000, (c) Higher of (a) or (b), 186,00,000, (d) Standard Rent (`16,00,000 x 12), 192,00,000, (e) Expected Rent {Lower of (c) or (d)}, 186,00,000, (f) Rent received /receivable (20,00,000 x 9), 180,00,000, If there was no vacancy, in that case rent received/receivable would have been, (20,00,000 x 12) `240,00,000 which is exceeding expected rent hence GAV shall be, rent received/ receivable i.e. `180,00,000, GAV, 180,00,000, Less: Municipal Tax, Net Annual Value, Less: 30% of NAV u/s 24(a), Less: Interest on capital borrowed u/s 24(b), Income under the head House Property, Total Income, Computation of Tax Liability, Tax on Normal Income `104,90,000 at slab rate, Add: Surcharge @ 15%, Tax before health & education cess, Add: HEC @ 4%, Tax Liability, Rounded off u/s 288B, Solution 6:, Computation of Total Income of Mr. X for A.Y. 2022-23, Income under the head Capital Gains, Long Term Capital Gain, Less: Loss under the head House Property, Long Term Capital Gain, Income under the head other sources, Casual Income, Gross Total Income, Less: Deduction u/s 80C to 80U, Total Income, Computation of Tax Liability, Tax on LTCG Nil (`2,00,000- `2,00,000) @ 20%, Tax on casual Income `1,00,000 @ 30%, Less: Rebate u/s 87A, Tax before health & education cess, Add: HEC @ 4%, Tax Liability, , 180,00,000.00, , (13,00,000.00), 167,00,000.00, (50,10,000.00), (12,00,000.00), 104,90,000.00, 104,90,000.00, , 29,59,500.00, 4,43,925.00, 34,03,425.00, 1,36,137.00, 35,39,562.00, 35,39,560.00, , 4,00,000, (2,00,000), 2,00,000, 1,00,000, 3,00,000, Nil, 3,00,000, Nil, 30,000.00, (12,500.00), 17,500.00, 700.00, 18,200.00
Page 462 :
Income Under The Head House Property, , 59, , Note: As per section 71, Maximum loss of `2,00,000 is allowed to be set off from other heads., Solution 7:, Computation of Total Income of Mr. X for A.Y. 2022-23, Income under the head House Property, Income under the head House Property, Less: Set off of Loss of P.Y. 2014-15, Income under the head House Property, , 3,00,000, (3,00,000), Nil, , Income under the head Capital Gains, Long Term Capital Gain, , 4,00,000, , Income under the head other sources, Casual Income, , 1,00,000, , Gross Total Income, Less: Deduction u/s 80C to 80U, Total Income, Computation of Tax Liability, Tax on LTCG ` 1,50,000 (`4,00,000 - `2,50,000) @ 20% u/s 112, Tax on casual Income `1,00,000 @ 30%, Less: Rebate u/s 87A, Tax before health & education cess, Add: HEC @ 4%, Tax Liability, , 5,00,000, Nil, 5,00,000, 30,000.00, 30,000.00, (12,500.00), 47,500.00, 1,900.00, 49,400.00, , Note : Brought forward loss cannot be set off from income of other heads hence remaining loss of `1,00,000, of House Property of P.Y. 2014-15 shall be carried forward., Solution 8:, Computation of Total Income of Mr. X for A.Y. 2022-23, Income under the head House Property, Income under the head House Property, Less: Set off of Loss of P.Y. 2014-15, Income under the head House Property, , 3,00,000, (3,00,000), Nil, , Income under the head Business/Profession, , 5,00,000, , Income under the head Capital Gains, Long Term Capital Gain, , 4,00,000, , Income under the head other sources, Casual Income, , 1,00,000, , Gross Total Income, Less: Deduction u/s 80C to 80U, Total Income, Computation of Tax Liability, Tax on Normal Income `5,00,000 at slab rate, Tax on LTCG ` 4,00,000 @ 20% u/s 112, Tax on casual Income `1,00,000 @ 30%, Tax before health & education cess, , 10,00,000, Nil, 10,00,000, 12,500.00, 80,000.00, 30,000.00, 1,22,500.00
Page 463 :
Income Under The Head House Property, Add: HEC @ 4%, Tax Liability, , 60, 4,900.00, 1,27,400.00, , Note : Brought forward loss cannot be set off from income of other heads hence remaining loss of `1,00,000, of House Property of P.Y. 2014-15 shall be carried forward., Solution 9:, Computation of Total Income of Mr. X for A.Y. 2022-23, Income under the head House Property, Income from House -1, Less: Loss from House – 2, Less: Set off of Loss of P.Y. 2014-15, Income under the head House Property, , 8,00,000, (3,00,000), (4,00,000), 1,00,000, , Income under the head Capital Gains, Short Term Capital Gain 111A, , 1,50,000, , Income under the head other sources, Casual Income, Dividend Income, Income under the head other sources, , 1,00,000, 2,00,000, 3,00,000, , Gross Total Income, Less: Deduction u/s 80C to 80U, Total Income, Agriculture Income, , 5,50,000, Nil, 5,50,000, 2,00,000, , Computation of Tax Liability, Tax on STCG 111A `1,50,000 @ 15% u/s 111A, Tax on casual Income `1,00,000 @ 30%, Tax on (3,00,000 + 2,00,000) at slab rate, Tax on (`2,50,000 + 2,00,000) at slab rates, Tax before health & education cess, Add: HEC @ 4%, Tax Liability, , 22,500.00, 30,000.00, 12,500.00, (10,000.00), 55,000.00, 2,200.00, 57,200.00, , Note : 1. Current year loss of house shall be fully allowed to set off from income of another house in the, same year however such loss shall be allowed to set off from income of other heads maximum, upto `2,00,000, Solution 10:, Situation 1, Computation of Gross Annual Value, (a) Fair Rent, (10,000 x 12), (b) Municipal Valuation, (11,000 x 12), (c) Higher of (a) or (b), (d) Standard Rent, (12,000 x 12), (e) Expected Rent {Lower of (c) or (d)}, (f) Rent Received/Receivable, (7,000 x 11), GAV = Higher of (e) or (f), , `, 1,20,000, 1,32,000, 1,32,000, 1,44,000, 1,32,000, 77,000, 1,32,000
Page 464 :
Income Under The Head House Property, Gross Annual Value, Situation 2, Computation of Gross Annual Value, (a) Fair Rent, (12,000 x 12), (b) Municipal Valuation, (10,000 x 12), (c) Higher of (a) or (b), (d) Standard Rent, (11,000 x 12), (e) Expected Rent {Lower of (c) or (d)}, (f) Rent Received/Receivable, (11,500 x 10), In this case, if there was no vacancy, rent received/receivable would have been `1,38,000, hence rent received/receivable is lower in this case due to vacancy, therefore GAV, shall be the rent received/receivable., Gross Annual Value, Situation 3, Computation of Gross Annual Value, (a) Fair Rent, (13,000 x 12), (b) Municipal Valuation, (8,000 x 12), (c) Higher of (a) or (b), (d) Standard Rent, (7,000 x 12), (e) Expected Rent {Lower of (c) or (d)}, (f) Rent Received/Receivable, (20,000 x 8), In this case, rent R/R is higher than the expected rent, GAV shall be Rent R/R, Gross Annual Value, Situation 4, Computation of Gross Annual Value, (a) Fair Rent, (15,000 x 12), (b) Municipal Valuation, (17,000 x 12), (c) Higher of (a) or (b), (d) Standard Rent, (16,000 x 12), (e) Expected Rent {Lower of (c) or (d)}, (f) Rent Received/Receivable, (16,000 x 8), If there was no vacancy, in that case rent received/receivable would have been `1,76,000, and it was still less than expected rent, therefore GAV shall be expected rent., Gross Annual Value, Solution 11:, Income under the head House Property, Gross annual value, Working Note:, (a) Fair rent (60,000 x 12), (b) Municipal valuation (55,000 x 12), , 61, 1,32,000, 1,44,000, 1,20,000, 1,44,000, 1,32,000, 1,32,000, 1,15,000, , 1,15,000, 1,56,000, 96,000, 1,56,000, 84,000, 84,000, 1,60,000, 1,60,000, 1,80,000, 2,04,000, 2,04,000, 1,92,000, 1,92,000, 1,28,000, , 1,92,000, `, 7,20,000.00, , `, 7,20,000, 6,60,000
Page 465 :
Income Under The Head House Property, (c) Higher of (a) or (b), 7,20,000, (d) Standard Rent (80,000 x 12), 9,60,000, (e) Expected Rent {Lower of (c) or (d)}, 7,20,000, (f) Rent Received (70,000 x 7), 4,90,000, If there was no vacancy , then Rent Receivable shall be 70,000 x 10 =, 7,00,000, which is lower than the expected rent , hence the GAV shall, be 7,20,000, Less: Municipal taxes paid, Net Annual Value, Less: 30% of NAV u/s 24(a), Less: Interest on capital borrowed u/s 24(b), Unrealised rent recovered of 2018-19 section 25A, (1,50,000 – 45,000), Income under the head House Property, Income from other sources, Gross Total Income, Less: Deduction u/s 80C to 80U, Total Income, Computation of Tax Liability, Tax on `5,08,600 at slab rate, Add: HEC @ 4%, Tax Liability, Rounded off u/s 288B, Solution 12:, Gross Annual Value, Working Note:, `, (a) Fair Rent (20,000 x 12), 2,40,000, (b) Municipal Valuation (10,000 x 12), 1,20,000, (c) Higher of (a) or (b), 2,40,000, (d) Standard Rent (18,000 x 12), 2,16,000, (e) Expected Rent {Lower of (c) or (d)}, 2,16,000, (f) Rent Receivable = (12,000 x 8.5), 1,02,000, If there was no vacancy, in that case rent received/receivable would have been, `1,20,000 and it was still less than expected rent, therefore GAV shall be expected, rent., GAV, 2,16,000, Less: Municipal Tax, Net Annual Value, Less: 30% of NAV u/s 24(a), Less: Interest on capital borrowed u/s 24(b), Loss under the head House Property, Income under the head Other Sources, Gross Total Income, Less: Deduction u/s 80C to 80U, Total Income, Computation of Tax Liability, Tax on `5,43,500 at slab rate, Add: HEC @ 4%, Tax Liability, Rounded off u/s 288B, , 62, , (62,000.00), 6,58,000.00, (1,97,400.00), (75,000.00), 3,85,600.00, 1,05,000.00, 4,90,600.00, 18,000.00, 5,08,600.00, Nil, 5,08,600.00, 14,220.00, 568.80, 14,788.80, 14,790.00, `, 2,16,000.00, , (11,000.00), 2,05,000.00, (61,500.00), (3,00,000.00), (1,56,500.00), 7,00,000.00, 5,43,500.00, Nil, 5,43,500.00, 21,200.00, 848.00, 22,048.00, 22,050.00
Page 466 :
Income Under The Head House Property, Solution 13:, Income under the head House Property, Gross annual value, Working Note:, Fair rent (1,00,000 x 12), Rent received (80,000 x 12), Higher shall be the GAV i.e., Less: Municipal taxes paid, Net Annual Value, Less: 30% of NAV u/s 24(a), Less: Interest on capital borrowed u/s 24(b), Working Note:, Prior period interest, Current year interest 11,00,000 x 11% = 1,21,000, Income under the head House Property, Gross Total Income, Less: Deduction u/s 80C to 80U, Total Income, , 63, `, 12,00,000.00, , `, 12,00,000, 9,60,000, 12,00,000, (30,000.00), 11,70,000.00, (3,51,000.00), (1,21,000.00), Nil, , Computation of Tax Liability, Tax on `6,98,000 at slab rate, Add: HEC @ 4%, Tax Liability, Rounded off u/s 288B, , 6,98,000.00, 6,98,000.00, Nil, 6,98,000.00, 52,100.00, 2,084.00, 54,184.00, 54,180.00, , Note: For repayment of housing loan, deduction u/s 80C is allowed or not is controversial., Solution 14:, Gross Annual Value, Working Note:, `, (a) Fair Rent (10,000 x 10), 1,00,000, (b) Expected Rent, 1,00,000, (c) Received/Receivable = 9,000 x 8.5 = 76,500 – 1,000 =, 75,500, If there was no vacancy, in that case rent received/receivable would have been, `89,000 and it was still less than expected rent, therefore GAV shall be expected, rent., GAV, 1,00,000, Less: Municipal taxes, Net Annual Value, Less: 30% of NAV u/s 24(a), Less: Interest on capital borrowed u/s 24(b), Working Note:, Current Period interest, From 01.04.2021 to 31.01.2022, = 3,05,250 x 7% x 10/12 = `17,806.25, Prior period interest, From 01.09.2018 to 31.03.2021, From 01.09.2018 to 30.06.2020, = 6,10,500 x 7% x 22/12 = `78,347.5, From 01.07.2020 to 31.03.2021, = 3,05,250 x 7% x 9/12 = `16,025.63, Total = `94,373.13, Instalment = `94,373.13/5 = `18,874.63, , `, 1,00,000.00, , (3,000.00), 97,000.00, (29,100.00), (36,680.88)
Page 467 :
Income Under The Head House Property, Total interest = `17,806.25 + `18,874.63 = `36,680.88, Income under the head House Property, Income under the head Salary, Gross Total Income, Less: Deductions u/s 80C, Total Income (rounded off u/s 288A), Computation of Tax Liability, Tax on `2,96,220 at slab rate, Less: Rebate u/s 87A, Tax Liability, Solution 15:, Income under the head House Property, Gross Annual Value, Less: Municipal taxes paid, Net Annual Value, Less: 30% of NAV u/s 24(a), Less: Interest on capital borrowed u/s 24(b), Working Note:, Prior period interest, 01.07.2018 to 31.03.2020, 15, 00,000 x 11 % x 21/12= 2,88,750, Installment 2,88,750/5 =, Current year interest, 15,00,000 x 11% x 9/12 = 1,23,750, 14,30,000 x 11% x 3/12 = 39,325, , 31,219.12, 2,65,000.00, 2,96,219.12, Nil, 2,96,220.00, 2,311.00, (2,311.00), Nil, `, NIL, NIL, NIL, NIL, (2,00,000.00), `, , 57,750, 1,63,075, 2,20,825, , Subject to maximum of 2,00,000., Loss under the head House Property, Income under the head Salary, Gross Total Income, Less: Deduction u/s 80C, Total Income, Computation of Tax Liability, Tax on `3,30,000 at slab rate, Less: Rebate u/s 87A, Tax Liability, Solution 16:, Income under the head House Property, Gross Annual Value, Less: Municipal taxes paid, Net Annual Value, Less: 30% of NAV u/s 24(a), Less : Interest on capital borrowed u/s 24(b), Working Note:, Prior period interest, Current year interest, From 01.04.2021 to 31.03.2022, 8,50,000 x 10% x 9/12 = 63,750, 8,00,000 x 10% x 3/12 = 20,000, Loss under the head House Property, , 64, , (2,00,000.00), 6,00,000.00, 4,00,000.00, (70,000.00), 3,30,000.00, 4,000.00, (4,000.00), Nil, `, NIL, NIL, NIL, NIL, (83,750.00), `, Nil, , 83,750, (83,750.00)
Page 468 :
Income Under The Head House Property, Income under the head capital gains (STCG u/s 111A), Gross Total Income, Less: Deduction u/s 80C, Total Income, Computation of Tax Liability, Tax on `6,66,250 (`9,16,250 – 2,50,000) @ 15%, Add: HEC @ 4%, Tax Liability, Rounded off u/s 288B, , 10,00,000.00, 9,16,250.00, Nil, 9,16,250.00, 99,937.50, 3,997.50, 1,03,935.00, 1,03,940.00, , Solution 17:, Computation of income under the head House Property, Gross Annual Value, Working Note:, `, (a) Fair Rent (90,000 x 11), 9,90,000, (b) Expected Rent, 9,90,000, (c) Rent Received/Receivable (80,000 x 10), 8,00,000, If there was no vacancy, in that case rent received/receivable would have, been `8,80,000 and it was still less than expected rent, therefore GAV, shall be expected rent., GAV, 9,90,000, Less: Municipal Tax, Net Annual Value, Less: 30% of NAV u/s 24(a), Less: Interest on capital borrowed u/s 24(b), Working Note:, Prior period interest, From 01.07.2019 to 31.03.2021, = (15,00,000 x 10% x 1) + (15,00,000 x 10% x 9/12), = `1,50,000 + `1,12,500 = `2,62,500, Installment = `2,62,500/5 = `52,500, Current period interest, From 01.04.2021 to 31.03.2022, = (15,00,000 x 10% x 9/12) + (7,50,000 x 10% x 3/12), = `1,12,500 + `18,750 = `1,31,250, Total interest on capital borrowed, = `52,500 + ` 1,31,250 = `1,83,750, Income under the head House Property, Gross Total Income, Less: Deduction u/s 80C {Repayment of housing loan}, Total Income, Computation of Tax Liability, Tax on `3,59,250 at slab rate, Less: Rebate u/s 87A, Tax Liability, Solution 18:, Computation of income under the head House Property, Net Annual Value, Less: Interest on capital borrowed u/s 24(b), Working Note:, Current period interest, 01.04.2021 to 31.03.2022, , 65, , 9,90,000.00, , Nil, 9,90,000.00, (2,97,000.00), (1,83,750.00), , 5,09,250.00, 5,09,250.00, (1,50,000.00), 3,59,250.00, 5,462.50, (5,462.50), Nil, `, Nil, (1,48,500.00), , `
Page 469 :
Income Under The Head House Property, 11,00,000 x 10% =, Prior period interest, From 01.07.2018 to 31.03.2020, 11,00,000 x 10% x 21/12 =, Instalment = 1,92,500/5 =, Total interest = 1,10,000 + 38,500 =, Loss under the head House Property, Income under the head capital gains, Short term capital gains u/s 111A, Gross Total Income, Less: Deduction u/s 80C to 80U, Total Income, Computation of Tax Liability, Tax on `116,01,500 (`118,51,500 – 2,50,000) @ 15%, Add: Surcharge @ 15%, Tax before health & education cess, Add: HEC @ 4%, Tax liability, Rounded off u/s 288B, , 66, , 1,10,000, 1,92,500, 38,500, 1,48,500, , Solution 19:, As per the amendments now two house shall be treated as self-occupied., House I & II is self-occupied, Income from house I & II, Income under the head House Property, Add: Unrealised rent received (2,000 – 600), Loss under the head House Property, Income under the head Capital Gains (LTCG), Gross Total Income, Less: Deduction u/s 80C, Total Income, Computation of Tax Liability, Tax on `17,21,400 (`19,71,400 – `2,50,000) @ 20%, Add: HEC @ 4%, Tax Liability, Rounded off u/s 288B, Solution 20:, Gross Annual Value, Working Note:, `, (a) Fair Rent (50,000 x 12), 6,00,000, (b) Municipal Valuation (47,000 x 12), 5,64,000, (c) Higher of (a) or (b), 6,00,000, (d) Standard Rent (48,000 x 12), 5,76,000, (e) Expected rent {Lower of (c) or (d)}, 5,76,000, (f) Rent Receivable (45,000 x 7), 3,15,000, If there was no vacancy, in that case rent received/receivable would have been, `4,05,000 and it was still less than expected rent, therefore GAV shall be expected, rent, GAV, 5,76,000, Less: Municipal Tax, Net Annual Value, Less: 30% of NAV u/s 24(a), , (1,48,500.00), 120,00,000.00, 118,51,500.00, Nil, 118,51,500.00, 17,40,225.00, 2,61,033.75, 20,01,258.75, 80,050.35, 20,81,309.10, 20,81,310.00, `, (30,000.00), (30,000.00), 1,400.00, (28,600.00), 20,00,000.00, 19,71,400.00, Nil, 19,71,400.00, 3,44,280.00, 13,771.20, 3,58,051.20, 3,58,050.00, `, 5,76,000.00, , (56,400.00), 5,19,600.00, (1,55,880.00)
Page 470 :
Income Under The Head House Property, , 67, , Less: Interest on capital borrowed u/s 24(b), Income under the head House Property, Gross Total Income, Less: Deduction u/s 80C to 80U, Total Income, , (42,000.00), 3,21,720.00, 3,21,720.00, Nil, 3,21,720.00, , Computation of Tax Liability, Tax on `3,21,720 at slab rate, Less: Rebate u/s 87A, Tax Liability, , 3,586.00, (3,586.00), Nil, , Solution 21:, Income from self occupied house, Net Annual Value, Less: 30% of NAV u/s 24(a), Less: Interest on capital borrowed u/s 24(b), Income from self occupied house, Income from partly self occupied and partly let out house, Gross Annual Value, Working Note:, (a) Fair Rent (20,000 x 12), (b) Municipal Valuation (18,000 x 12), (c) Higher of (a) or (b), (d) Standard Rent (15,000 x 12), (e) Expected Rent, (f) Rent Receivable (45,000 x 8), GAV = Higher of (e) or (f), Less: Municipal taxes, Net Annual Value, Less: 30% of NAV u/s 24(a), Less: Interest on capital borrowed u/s 24(b), Income from House Property, Income under the head House Property, [`1,93,000 + `(30,000)], Solution 22:, Gross Annual Value, Working Note:, (a) Fair rent (98,000 x 12), (b) Rent receivable (1,00,000 x 12), GAV {Higher of (a) or (b)}, Less: Municipal Taxes, Net Annual Value, Less: 30% of NAV u/s 24(a), Less: Interest on capital borrowed u/s 24(b), Add: Arrears of rent (Sec 25A), Less: 30% of `3,00,000, Add: Unrealised Rent (4,00,000 – 1,20,000), Income under the head House Property, Income under the head Other Sources, Gross Total Income, Less: Deduction u/s 80C to 80U, , `, Nil, Nil, (30,000), (30,000), 3,60,000, `, 2,40,000, 2,16,000, 2,40,000, 1,80,000, 1,80,000, 3,60,000, 3,60,000, (20,000), 3,40,000, (1,02,000), (45,000), 1,93,000, 1,63,000, `, , `, 12,00,000.00, , `, 11,76,000, 12,00,000, 12,00,000, (85,000.00), 11,15,000.00, (3,34,500.00), Nil, 7,80,500.00, 3,00,000, (90,000), , 2,10,000.00, 9,90,500.00, 2,80,000.00, 12,70,500.00, 20,000.00, 12,90,500.00, Nil
Page 471 :
Income Under The Head House Property, , 68, , Total Income, Computation of Tax Liability, Tax on `12,90,500 at slab rate, Add: HEC @ 4%, Tax Liability, Rounded off u/s 288B, , 12,90,500.00, 1,99,650.00, 7,986.00, 2,07,636.00, 2,07,640.00, , Solution 23:, As per the amendments now two house shall be treated as self-occupied., Flat I & II is self-occupied, Income, Computation of Total Income, Income under the head House Property, Income under the head Business Profession, Gross Total Income, Less: Deduction u/s 80C to 80U, Total Income, , `, (30,000.00), (30,000.00), 7,00,000.00, 6,70,000.00, Nil, 6,70,000.00, , Computation of Tax Liability, Tax on `6,70,000 at slab rate, Add: HEC @ 4%, Tax Liability, Solution 24:, Computation of income from House Property of Mr. X, Net annual value is Nil, (Since house is self – occupied), Less: Deduction u/s 24(b), Interest paid on borrowed capital, 25,00,000 @ 12% = `3,00,000, As per second proviso to section 24(b), interest deduction restricted to `2,00,000, Loss under the head “House Property”, Computation of income from house property of Mr. Y, Ground Floor (Self Occupied), Net Annual Value, Less: 30% of NAV u/s 24(a), Less: Interest on capital borrowed u/s 24(b), Working Note:, Current period interest, From 01.04.2021 to 31.03.2022, = 15,00,000 x 10% x 1/2 = `75,000, Prior period interest, From 01.07.2020 to 31.03.2021, = 15,00,000 x 10% x 9 / 12 = 1,12,500, 1,12,500 allowed in 5 equal instalments, = 1,12,500 / 5 = ` 22,500 per annum, = 22,500 / 2 = `11,250, Total interest = `75,000 + ` 11,250 = `86,250, Loss from house property, First Floor (Let Out), Gross Annual Value, Working Note:, (a) Fair Rent (1,20,000 x 9/12), , 34,000.00, 1,360.00, 35,360.00, `, Nil, (2,00,000), , (2,00,000), Nil, Nil, (86,250), , (86,250), 1,20,000, `, 90,000
Page 472 :
Income Under The Head House Property, (b) Municipal Value (92,000 x 9/12), (c) Higher of (a) or (b), (d) Expected Rent, (e) Rent Received/Receivable = 20,000 x 6, GAV = Higher of (d) or (e), Less: Municipal taxes, Net Annual Value, Less: 30% of NAV u/s 24(a), Less: Interest on capital borrowed u/s 24(b), Working Note:, Current period interest, From 01.04.2021 to 31.03.2022, = 15,00,000 x 10% x 1/2 = `75,000, Prior period interest, From 01.07.2020 to 31.03.2021, = 15,00,000 x 10% x 9 / 12 = 1,12,500, 1,12,500 allowed in 5 equal instalments, = 1,12,500 / 5 = ` 22,500 per annum, = 22,500 /2 = `11,250, Total Interest = `75,000 + `11,250 = `86,250, Loss from house property, Loss under the head “income from house property” of Mr. Y, (Both ground floor and first floor), Solution 25:, Computation of income under the head House Property, Gross Annual Value, Working Note:, (a) Municipal Valuation, (b) Expected Rent, (c) Rent Received/Receivable (3,000 x 2) + (3,500 x 6), GAV = Higher of (b) or (c), Less: Municipal taxes, Net Annual Value, Less: 30% of NAV u/s 24(a), Less: Interest on capital borrowed u/s 24(b), Working Note:, = [(60,000 x 15% x 6/12) + (45,000 x 15% x 6/12)] = `7,875, Income under the head House Property, , 69, 69,000, 90,000, 90,000, 1,20,000, 1,20,000, (5,000), 1,15,000, (34,500), (86,250), , (5,750), (92,000), `, 27,000, `, 22,000, 22,000, 27,000, 27,000, Nil, 27,000, (8,100), (7,875), , 11,025, , Solution 26:, Computation of income of Unit-I, Since the unit is self-occupied throughout the year. Hence its income shall be computed under section 23(2),, accordingly there will be loss `30,000., Computation of income of Unit-II, It will be considered to be partially self-occupied and partially let out and income shall be computed under, section 23(3) in the manner given below:, `, Gross Annual Value, 82,500.00, Working Note:, `, (a) Fair Rental Value, 76,500, (b) Municipal Valuation, 65,000, (c) Higher of (a) or (b), 76,500
Page 473 :
Income Under The Head House Property, , 70, , (d) Standard Rent, 70,000, Expected Rent {Lower of (c) or (d), 70,000, (e) Expected Rent, 70,000, (f) Rent Received/Receivable (7,500 x 11), 82,500, GAV = Higher of (e) or (f), 82,500, Less: Municipal taxes, (7,800.00), Net Annual Value, 74,700.00, Less: 30% of NAV u/s 24(a), (22,410.00), Less: Interest on capital borrowed u/s 24(b), (72,900.00), Working note:, Current period interest, From 01.04.2021 to 31.03.2022, = 9,00,000 x 12% = `1,08,000, Prior period interest, From 01.07.2018 to 31.03.2020, = 9,00,000 x 12% x 21 / 12 =1,89,000, Installment = 1,89,000 / 5 = 37,800, Total interest= 1,08,000 + 37,800 = 1,45,800, Interest allowed for one unit = 1,45,800 / 2 = `72,900, Loss from house property, (20,610.00), Loss under the head House Property is, (`20,610) + (`30,000), (50,610.00), Income under the head Other Sources, 3,90,000.00, Gross Total Income, 3,39,390.00, Less: Deductions u/s 80C to 80U, Nil, Total Income, 3,39,390.00, Computation of Tax Liability, Tax on `3,39,390 at slab rate, 4,469.50, Less: Rebate u/s 87A, (4,469.50), Tax Liability, Nil, Note: Since condition regarding certificate has not been complied with hence interest has been allowed, maximum to the extent of `30,000., Solution 27:, Unit I, Gross Annual Value, Working Note:, `, (a) Fair Rental Value (1,75,000 x 70%), 1,22,500, (b) Municipal Valuation (1,55,000 x 70%), 1,08,500, (c) Higher of (a) or (b), 1,22,500, (d) Standard Rent (1,65,000 x 70%), 1,15,500, (e) Expected Rent {Lower of (c) or (d)}, 1,15,500, (f) Rent Received/Receivable (13,000 x 10.5), 1,36,500, In this case, rent R/R is higher than the expected rent, GAV shall be Rent R/R, GAV, 1,36,500, Less: Municipal taxes (35,000 x 70%), Net Annual Value, Less: 30% of NAV u/s 24(a), Less: Interest on capital borrowed u/s 24(b), Income under the head House Property, , `, 1,36,500, , (24,500), 1,12,000, (33,600), Nil, 78,400
Page 474 :
Income Under The Head House Property, Unit II, Gross Annual Value, Working Note:, (a) Fair Rental Value (1,75,000 x 30%), (b) Municipal Valuation (1,55,000 x 30%), (c) Higher of (a) or (b), (d) Standard Rent (1,65,000 x 30%), (e) Expected Rent {Lower of (c) or (d)}, (f) Rent Received/Receivable (5,000 x 11), In this case, rent R/R is higher than the expected rent, GAV shall be Rent R/R, GAV, Less: Municipal taxes (35,000 x 30%), Net Annual Value, Less: 30% of NAV u/s 24(a), Less: Interest on capital borrowed u/s 24(b), Income under the head house property, Total income from house property (78,400 + 31,150), , 71, 55,000, `, 52,500, 46,500, 52,500, 49,500, 49,500, 55,000, 55,000, , Solution 28:, Gross Annual Value, Working Note:, `, (a) Fair Rental Value, 75,000, (b) Municipal Valuation, 68,000, (c) Higher of (a) or (b), 75,000, (d) Standard Rent (7,200 x 12), 86,400, (e) Expected Rent {Lower of (c) or (d)}, 75,000, (f) Rent Received/Receivable (7,200 x 12), 86,400, GAV = Higher of (e) or (f), 86,400, Less: Municipal taxes, Net Annual Value, Less: 30% of NAV u/s 24(a), Less: Interest on capital borrowed u/s 24(b), Interest paid to non-resident without deducting tax at source is not deductible, Income from house property, Add: Recovery of unrealised rent sec 25A, (22,000 – 6,600), Income under the head House property, , (10,500), 44,500, (13,350), Nil, 31,150, 1,09,550, `, 86,400, , (14,000), 72,400, (21,720), Nil, 50,680, 15,400, 66,080, , Solution 29:, `, Option I, House I self-occupied, Income from house property I, (30,000.00), Interest on the capital borrowed = `59,583.33, (5,00,000 x 13% x 11/12 = 59,583.33)., But subject to maximum of `30,000, Interest upto `2,00,000 is allowed only if the loan is taken for purchase or construction of the house i.e. if, the loan is taken for reconstruction, higher amount is not allowed., House II Self Occupied, Income from house II, Nil, House III, Gross Annual Value, 7,20,000, Working Note:, `, (a) Fair rent (60,000 x 12), 7,20,000, (b) Expected Rent, 7,20,000
Page 475 :
Income Under The Head House Property, (c) Rent Received/Receivable (50,000 x 12), GAV = Expected Rent, Less: Municipal Taxes, Net Annual Value, Less: 30% of NAV u/s 24(a), Less: Interest on capital borrowed u/s 24(b), Income from house III, Income under the head House Property, House I and II, House III, Recovery of unrealised rent (house I), (2,000 – 600), Income under the head House Property, Computation of Total Income, Income under the head House Property, Income under the head Capital Gains (long term capital gain), Income from Other Sources, Gross Total Income, Less: Deduction u/s 80C to 80U, Total Income, Computation of Tax Liability, Tax on `4,12,950 at slab rate, Tax on `3,50,000 @ 20% u/s 112, Tax before health & education cess, Add: HEC @ 4%, Tax Liability, Rounded off u/s 288B, Solution 30:, (i) Income under the head House Property., (ii) Income under the head Capital Gains., (iii) Income under the head Business/Profession., (iv) Income under the head Other Sources., (v) Income under the head Business/Profession., (vi) Income under the head Business/Profession., (vii) Income under the head Business/Profession., (viii) Income under the head Business/Profession., (ix) Income under the head Other Sources., (x) Income under the head House Property., , 72, 6,00,000, 7,20,000, (11,000.00), 7,09,000.00, (2,12,700.00), (55,000.00), 4,41,300.00, (30,000.00), 4,41,300.00, 1,400.00, 4,12,700.00, 4,12,700.00, 3,50,000.00, 250.00, 7,62,950.00, Nil, 7,62,950.00, 8,147.50, 70,000.00, 78,147.50, 3,125.90, 81,273.40, 81,270.00
Page 476 :
Income Under The Head House Property, , 73, , SOLUTIONS OF EXAMINATION QUESTIONS, NOV – 2018 (NEW COURSE), Question 3(a), Solution: Computation of Income under the head House Property, Gross Annual Value, Working Note:, (a) Fair rent, (b) Municipal Valuation, (c) Higher of (a) or (b), (d) Standard Rent, (e) Expected Rent {Lower of (c) or (d)}, (f) Rent Received/Receivable (75,000 x 9), GAV = Higher of (e) or (f), Less: Municipal taxes, Net Annual Value, Less: 30% of NAV u/s 24(a), Less: Interest on capital borrowed u/s 24(b), Income from house property, , Marks 7, 7,20,000.00, `, 6,30,000, 7,50,000, 7,50,000, 7,20,000, 7,20,000, 6,75,000, 7,20,000, (90,000.00), 6,30,000.00, (1,89,000.00), (35,000.00), 4,06,000.00, , Note: As per explanation to section 23(1)/Rule 4, in case of unrealised rent expected rent shall be computed, for full year and while computing rent received/receivable, such unrealised rent shall be excluded and GAV, shall be higher of expected rent and rent received or receivable., In the given case conditions of rule 4 has not been complied hence rent shall not be treated as unrealised and, shall not be excluded., , NOV – 2018 (OLD COURSE), Question 2(b), Marks 5, Solution:, As per the amendments now two house shall be treated as self-occupied and after two house all house shall, be treated as deemed to be let out., House I is Self-Occupied, Loss, Nil, House II is Self-Occupied, Loss, (2,00,000), Working Note:, Prior period interest, From 01.04.2018 to 31.03.2020, = 18,00,000 x 10% x 2 = 3,60,000, 3,60,000 allowed in 5 equal instalments, = 3,60,000 / 5 = ` 72,000 per annum, Current period interest, From 01.04.2021 to 31.03.2022, = 18,00,000 x 10% x 1 = `1,80,000, Total Interest = `72,000 + ` 1,80,000 = `2,52,000 but restricted to 2,00,000, Loss under the head house property, (2,00,000)
Page 477 :
Income Under The Head House Property, , 74, , MAY – 2018 (OLD COURSE), Question 3 (b), Marks 5, Solution:, More than two house which are self-occupied (deemed to be let out property) Section 23(4), If any assessee has more than two house which are self-occupied, in such cases only two of these houses, shall be considered to be self-occupied and income shall be computed under section 23(2) and all other, houses shall be deemed to be let out and income shall be computed in the similar manner as in case of let out, house. Expected rent shall be considered to be GAV of the house., As per section 23(1)(c), if any House Property is lying vacant throughout the year, it will be considered to, be deemed to be let out and income shall be computed in the similar manner as in case of a let out house., Expected Rent shall be considered to be Gross annual value., As per section 23 (5), Where the property consisting of any building or land appurtenant thereto is held as, stock-in trade and the property or any part of the property is not let during the whole of the previous year,, the annual value of such property or part of the property, for the period up to one year from the end of the, financial year in which the certificate of completion of construction of the property is obtained from the, competent authority, shall be taken to be nil, , NOV – 2017, Question 4(a), (Marks 5), Solution:, In the given Mr. Aditya is NOR Hence Income received in India is taxable in India., Computation of Income from House Property of Mr. Aditya for the Assessment Year 2022-23, `, GAV of the house in Dubai, (20,000 p.m. x `18 per DHS x 12 months), 43,20,000.00, Less: Municipal taxes paid (1500 +2500) x `18 per DHS, (72,000.00), Net Annual Value, 42,48,000.00, Less: Statutory deduction under section 24(a) @ 30% of NAV, (12,74,400.00), Income from House property, 29,73,600.00, GAV of house at Mumbai (self occupied), Gross Annual Value, Less: Municipal taxes paid, Net Annual Value (NAV), Less: Statutory deduction under section 24(a) @ 30% of NAV, Less: Interest on capital borrowed u/s 24(b), Working Note:, Prior period interest, From 01.06.2018 to 31.03.2020, = 25,00,000 x 12% x 22 / 12 = 5,50,000, 5,50,000 allowed in 5 equal instalments, = 5,50,000 / 5 = 1,10,000 per annum, Current period interest, From 01.04.2021 to 31.03.2022, = 25,00,000 x 12% x 1 = 3,00,000, Total Interest = 1,10,000 + 3,00,000 = 4,10,000 but maximum upto ` 2,00,000, (assuming all the conditions for higher Interest is satisfied.), Loss from House property, Income from House property, , Nil, Nil, Nil, Nil, (2,00,000), , (2,00,000.00), 27,73,600.00
Page 478 :
Income Under The Head House Property, , 75, , MAY – 2017, Question 4(a) (ii), (4 Marks), Solution:, Computation of Income under the head House Property, `, Gross Annual Value (40,000 x 12), 4,80,000, Less: Municipal Taxes, (18,000), Net Annual Value, 4,62,000, Less: 30% of NAV u/s 24(a), (1,38,600), Less: Interest on capital borrowed u/s 24(b), (2,55,000), Working Note:, Prior period interest, From 01.08.2019 to 31.03.2020, = 15,00,000 x 15% x 8 / 12 = 1,50,000, 1,50,000 allowed in 5 equal instalments, = 1,50,000 / 5 = 30,000 per annum, Current period interest, From 01.04.2021 to 31.03.2022, = 15,00,000 x 15% x 1 = 2,25,000, Total Interest = 30,000 + 2,25,000 = 2,55,000, Income under the head House Property, 68,400, Note: As per proviso to section 23(1), Municipal Taxes actually paid by the owner during the previous year, is allowed to be deducted from GAV., , NOV – 2015, Question 6(a)., Solution 6(a):, Income under the head House Property, Income from shopping Complex, Gross Annual Value, Less: Municipal taxes paid, Net Annual Value, Less: 30% of NAV u/s 24(a), Less: Interest on capital borrowed u/s 24(b), Working Note:, Prior period interest, From 01.08.2019 to 31.03.2020, 25,00,000 x 10% x 8/12 = 1,66,666.67, Installment allowed = 1,66,666.67/5 = 33,333.33, Current year interest, From 01.04.2021 to 31.03.2022, 25,00,000 x 10% x 1 = 2,50,000, Total, Loss from shopping complex, Add: Arrear of Rent Received Section 25A, Less: Deduction @ 30%, Income from shopping complex, Income from self-occupied property, Gross Annual Value, Less: Municipal taxes paid, Net Annual Value, , (8 Marks), , 3,60,000.00, (8,000.00), 3,52,000.00, (1,05,600.00), (2,83,333.33), `, , 33,333.33, 2,50,000.00, 2,83,333.33, (36,933.33), 1,20,000, (36,000), , 84,000.00, 47,066.67, NIL, NIL, NIL
Page 479 :
Income Under The Head House Property, Less: 30% of NAV u/s 24(a), Less: Interest on capital borrowed u/s 24(b), Loss from self-occupied property, Loss under the head House Property (47,066.67-(2,00,000)), , 76, NIL, (2,00,000.00), (2,00,000.00), (1,52,933.33)
Page 480 :
Deduction From Gross Total Income, , 77, , DEDUCTION FROM GROSS, TOTAL INCOME, CHAPTER VI-A, SECTION 80C TO 80U, , SOLUTIONS OF MCQS, Answer, 1.(c); 2.(c); 3.(d); 4.(b); 5 (b); 6.(d); 7.(d); 8.(c); 9. (d); 10.(c); 11.(a); 12.(c); 13.(b); 14.(b); 15. (d); 16. (d);, 17. (c); 18. (e); 19. (e); 20. (e); 21.(a); 22.(c); 23.(d); 24.(a); 25.(d); 26.(a); 27.(b); 28.(c); 29.(a); 30.(b);, 31.(a); 32.(d); 33.(d); 34.(b); 35.(d), Hint for Answer 10:, Least of the following:, 1. `4,80,000 – 46,000 = 4,34,000, 2. 60,000, 3. 25% x 4,60,000 = 1,15,000, AGTI = GTI – LTCG – STCG 111A – Deduction u/s 80C to 80U, (except 80GG), = 4,60,000 – 0 –0 –0, = 4,60,000, Hint for Answer 11:, Least of the following:, 1. `1,08,000 – 59,500 = 48,500, 2. 60,000, 3. 25% x 5,95,000 = 1,15,000, AGTI = GTI – LTCG – STCG 111A – Deduction u/s 80C to 80U, (except 80GG), = 5,95,000 – 0 –0 –0, = 5,95,000
Page 481 :
Deduction From Gross Total Income, , 78, , SOLUTIONS, TO, , PRACTICE PROBLEMS, Solution 1:, Computation of income under the head House Property, Gross Annual Value, Less: Municipal Tax, Net Annual Value, Less: 30% of NAV u/s 24(a), Less: Interest on capital borrowed u/s 24(b), Working Note:, Prior period interest, From 01.07.2017 to 31.03.2019, = (10,00,000 x 10% x 9/12) + (10,00,000 x 10% x 1), = `75,000 + `1,00,000 = `1,75,000, Installment = `1,75,000/5 = `35,000, Current period interest, From 01.04.2021 to 31.03.2022, = (10,00,000 x 10% x 3/12) + (9,30,000 x 10% x 9/12), = `25,000 + `69,750 = `94,750, Total interest on capital borrowed, = `35,000 + ` 94,750 = `1,29,750, Income under the head House Property, Less: Brought forward Loss of P.Y.2013-14, Income under the head House Property, Gross Total Income, Less: Deduction u/s 80C, Repayment of Housing loan, NSC, PPF, Post Office Time Deposit, Total Income, Agricultural Income, Computation of Tax Liability, Step 1 Tax on (4,17,250 + 3,00,000) at slab rates, Step 2 Tax on (2,50,000 + 3,00,000) at slab rates, Deduct Tax at Step 2 from Step 1, Less: Rebate u/s 87A, Tax before health & education cess, Add: HEC @ 4%, Tax Liability, Rounded off u/s 288B, Solution 2:, Computation of income under the head House Property, , 9,60,000.00, (20,000.00), 9,40,000.00, (2,82,000.00), (1,29,750.00), , 5,28,250.00, (21,000.00), 5,07,250.00, 5,07,250.00, (70,000.00), (10,000.00), (5,000.00), (5,000.00), 4,17,250.00, 3,00,000.00, 55,950.00, (22,500.00), 33,450.00, (12,500.00), 20,950.00, 838.00, 21,788.00, 21,790.00, `
Page 482 :
Deduction From Gross Total Income, Gross Annual Value, Less: Municipal Tax, Net Annual Value, Less: 30% of NAV u/s 24(a), Less: Interest on capital borrowed u/s 24(b), Working Note:, Prior period interest, From 01.07.2017 to 31.03.2021, = (12,00,000 x 10% x 6/12) + (12,00,000 x 10% x 1)+ (11,60,000 x 10%, x 1)+ (11,20,000 x 10% x 1)+ (10,80,000 x 10% x 3/12), = `60,000 + `1,20,000+ `1,16,000+ `1,12,000+ `27,000 = `4,35,000, Installment = `4,35,000/5 = `87,000, Current period interest, From 01.04.2021 to 31.03.2022, = (10,80,000 x 10% x 9/12) + (10,40,000 x 10% x 3/12), = `81,000 + `26,000 = `1,07,000, Total interest on capital borrowed, = `87,000 + ` 1,07,000 = `1,94,000, Loss under the head House Property, Income under the head Capital Gains (STCG u/s 111A), Gross Total Income, Less: Deduction u/s 80C to 80U, Total Income, Computation of Tax Liability, Tax on STCG ` 7,24,000 (9,74,000-2,50,000) u/s 111A @ 15 %, Add: HEC @ 4%, Tax Liability, Rounded off u/s 288B, Note: Deduction under section 80C to 80U is not allowed from STCG u/s 111A., Solution 3:, Income under the head Business/Profession, Income under the head Other Sources, Income from subletting, Income under the head Other Sources, Gross Total Income, Less: Deduction u/s 80D, Total Income, Computation of Tax Liability, Tax on `7,65,000 at slab rate, Add: HEC @ 4%, Tax Liability, Solution 4:, Income under the head Business/Profession, Gross Total Income, Less: Deduction u/s 80DDB (40,000 – 35,000), Total Income, Computation of Tax Liability, Tax on `6,95,000 at slab rate, Add: HEC @ 4%, Tax Liability, , 79, 2,40,000, Nil, 2,40,000, (72,000), (1,94,000), , (26,000), 10,00,000, 9,74,000, NIL, 9,74,000, 1,08,600, 4,344, 1,12,944, 1,12,940, `, 4,10,000, 3,80,000, 3,80,000, 7,90,000, (25,000), 7,65,000, 65,500.00, 2,620.00, 68,120.00, `, 7,00,000, 7,00,000, (5,000), 6,95,000, 51,500, 2,060, 53,560
Page 483 :
Deduction From Gross Total Income, , 80, , Solution 4(b):, `, Income under the head Business/Profession, 7,00,000, Gross Total Income, 7,00,000, Less: Deduction u/s 80DDB, Nil, Total Income, 7,00,000, Computation of Tax Liability, Tax on `7,00,000 at slab rate, 52,500, Add: HEC @ 4%, 2,100, Tax Liability, 54,600, Note: Deduction under section 80DDB is not allowed in case assessee incurred expenditure on treatment of, his independent brother., Solution 5:, Computation of income under the head House Property, Gross Annual Value, Less: Municipal Tax, Net Annual Value, Less: 30% of NAV u/s 24(a), Less: Interest on capital borrowed u/s 24(b), Income under the head House Property, Income under the head other sources, Gross Total Income, Less: Deductions, Deduction u/s 80C for NSC, Deduction u/s 80CCC for Jeevan Suraksha Policy, Deduction u/s 80D for Mediclaim Policy, Deduction u/s 80DD for Severe Disability, Deduction u/s 80E for Education Loan Interest, Total Income, Agricultural Income, Computation of Tax Liability, Step 1 Tax on (6,17,000 + 1,00,000) at slab rates, Step 2 Tax on (2,50,000 + 1,00,000) at slab rates, Deduct Tax at Step 2 from Step 1, Tax before HEC, Add: HEC @ 4%, Tax Liability, Rounded off u/s 288B, Solution 6:, Computation of income under the head House Property, Gross Annual Value, Less: Municipal taxes, Net Annual Value, Less: 30% of NAV u/s 24(a), Less: Interest on capital borrowed u/s 24(b), Income under the head House Property, Computation of Total Income, Income under the head House Property, Income under the head Capital Gains (LTCG), Gross Total Income, Less: Deduction u/s 80C {NSC}, Less: Deduction u/s 80G, , `, 12,00,000, (1,00,000), 11,00,000, (3,30,000), Nil, 7,70,000, 30,000, 8,00,000, (30,000), Nil, (18,000), (1,25,000), (10,000), 6,17,000, 1,00,000, 55,900, (5,000), 50,900, 50,900, 2,036, 52,936, 52,940, `, 3,00,000, Nil, 3,00,000, (90,000), Nil, 2,10,000, 2,10,000, 2,00,000, 4,10,000, (6,000)
Page 484 :
Deduction From Gross Total Income, (i) National Children Fund {100% of `2,000}, (ii) Other Donations u/s 80G, Working Note:, MCD, Family planning, AGTI = GTI – LTCG – STCG u/s 111A – 80C to 80U (Except 80G), = 4,10,000 – 2,00,000 – 6,000 = 2,04,000, Qualifying amount = 10% of AGTI or donation whichever is less, = 20,400 or 7,000, = 7,000, Deduction = 50% of `5,000 + `2,000, = `4,500, Total Income, Computation of Tax Liability, Tax on LTCG `97,500 (2,00,000 – 1,02,500) @ 20% u/s 112, Tax on `1,97,500 at slab rate, Less: Rebate u/s 87A, Tax before health & education cess, Add: HEC @ 4%, Tax Liability, Solution 7 (a):, Income under the head business / profession, Income under the head Other Sources, Casual income, Income under the head other sources, Income under the head Capital Gain, Long term capital gain, Short term capital gain 111A, Income under the head capital gain, Gross Total Income, Less: Deductions, Deduction u/s 80C for NSC, Deduction u/s 80D for Mediclaim policy, Deduction u/s 80G, Prime Minister National Relief Fund, Rajiv Gandhi Foundation (50% of `8,000), Donations, Working Note:, AGTI = GTI – LTCG – STCG u/s 111A – Deduction u/s 80C to 80U, (except 80G), = 14,00,000 – 3,00,000 – 1,00,000 – 25,000, = 9,75,000, Qualifying amount = 10% of AGTI or donation whichever is less, = 97,500 or 2,30,000 whichever is less, = 97,500, 50% of qualifying amount = 48,750, Total Income, Computation of Tax Liability, Tax on casual income `2,00,000 @ 30% u/s 115BB, , 81, (2,000), (4,500), `, 5,000, 2,000, 7,000, , 3,97,500, 19,500, Nil, (12,500), 7,000, 280, 7,280, `, 8,00,000.00, 2,00,000.00, 2,00,000.00, 3,00,000.00, 1,00,000.00, 4,00,000.00, 14,00,000.00, (10,000.00), (15,000.00), (10,000.00), (4,000.00), (48,750.00), , 13,12,250.00, 60,000.00
Page 485 :
Deduction From Gross Total Income, Tax on LTCG `3,00,000 @ 20%, Tax on STCG 111A `1,00,000 @ 15%, Tax on normal income `7,12,250 at slab rate, Tax before health & education cess, Add: HEC @ 4%, Tax Liability, Rounded off u/s 288B, , 82, 60,000.00, 15,000.00, 54,950.00, 1,89,950.00, 7,598.00, 1,97,548.00, 1,97,550.00, , Solution 7(b):, Gross Total Income, Less: Deductions, Deduction u/s 80C for NSC, Deduction u/s 80D for Mediclaim policy, Deduction u/s 80G, Prime minister national relief fund, Rajiv Gandhi Foundation, Donations, Working Note:, AGTI = GTI – LTCG – STCG u/s 111A – Deduction u/s 80C to 80U, (except 80G), = 14,00,000 – 3,00,000 – 1,00,000 – 25,000, = 9,75,000, Qualifying amount = 10% of AGTI or donation whichever is less, = 97,500 or 2,50,000 whichever is less, = 97,500, 50% of qualifying amount = 77,500 x 50% + 20,000, = 58,750, Total Income, Computation of Tax Liability, Tax on casual income `2,00,000 @ 30% u/s 115BB, Tax on LTCG `3,00,000 @ 20%, Tax on STCG 111A `1,00,000 @ 15%, Tax on normal income `7,02,250 at slab rate, Tax before health & education cess, Add: HEC @ 4%, Tax Liability, Rounded off u/s 288B, , `, 14,00,000.00, , Solution 7(c):, Gross Total Income, Less: Deductions, Deduction u/s 80C for NSC, Deduction u/s 80D for Mediclaim policy, Deduction u/s 80G, Prime minister national relief fund, Rajiv Gandhi Foundation, Donations, Working Note:, AGTI = GTI – LTCG – STCG u/s 111A – Deduction u/s 80C to 80U, (except 80G), = 14,00,000 – 3,00,000 – 1,00,000 – 25,000, = 9,75,000, Qualifying amount = 10% of AGTI or donation whichever is less, , `, 14,00,000.00, , (10,000.00), (15,000.00), (10,000.00), (4,000.00), (58,750.00), , 13,02,250.00, 60,000.00, 60,000.00, 15,000.00, 52,950.00, 1,87,950.00, 7,518.00, 1,95,468.00, 1,95,470.00, , (10,000.00), (15,000.00), (10,000.00), (4,000.00), (97,500.00)
Page 486 :
Deduction From Gross Total Income, , 83, , = 97,500 or 5,30,000 whichever is less, = 97,500, 100% deduction is allowed 97,500, Total Income, Computation of Tax Liability, Tax on casual income `2,00,000 @ 30% u/s 115BB, Tax on LTCG ` 3,00,000 @ 20%, Tax on STCG 111A `1,00,000 @ 15%, Tax on normal income `6,63,500 at slab rate, Tax before health & education cess, Add: HEC @ 4%, Tax Liability, Rounded off u/s 288B, , 12,63,500.00, 60,000.00, 60,000.00, 15,000.00, 45,200.00, 1,80,200.00, 7,208.00, 1,87,408.00, 1,87,410.00, , Solution 8:, Income under the head Capital Gains (STCG), Gross Total Income, Less: Deduction u/s 80DD, Less: Deduction u/s 80G, Working Note:, AGTI = GTI – LTCG – STCG u/s 111A – 80C to 80U (Except 80G), = 6,00,000 – 75,000 = 5,25,000, Qualifying amount = 10% of AGTI or donation, whichever is less, = 52,500 or 20,000, = 20,000, Deduction = 50% of `20,000, = `10,000, Total Income, Computation of Tax Liability, Tax on `5,15,000 at slab rate, Add: HEC @ 4%, Tax Liability, Solution 9:, Income under the head Other Sources, Income under the head Business/Profession, Income under the head Capital Gains {LTCG}, Gross Total Income, Less: Deduction u/s 80G, Working Note:, MCD for family planning, Charitable institution, Adjusted GTI = GTI – LTCG – STCG u/s 111A – 80C to 80U (Except 80G), = `3,22,000 – `2,00,000 = `1,22,000, Qualifying amount = 12,200 or 13,000 whichever is less, = `12,200, Deduction = 50% of `2,200 + `10,000 = `11,100, Total Income, Computation of Tax Liability, Tax on Long term capital gain `60,900 (2,00,000 – 1,39,100) @ 20% u/s 112, , `, 6,00,000, 6,00,000, (75,000), (10,000), , 5,15,000, 15,500, 620, 16,120, `, 1,22,000.00, 2,00,000.00, 3,22,000.00, (11,100.00), `, 10,000, 3,000, 13,000, , 3,10,900.00, 12,180.00
Page 487 :
Deduction From Gross Total Income, Tax on `1,10,900 at slab rate, Less: Rebate u/s 87A, Tax before health & education cess, Add: HEC @ 4%, Tax Liability, Solution 10:, Income under the head Capital Gains {STCG}, Gross Total Income, Less: Deduction u/s 80G, (i) Prime Minister’s National Relief Fund, (ii) Other donations, Working Note:, AGTI = GTI – LTCG – STCG u/s 111A – Deduction u/s 80C to 80U (except 80G), = 5,00,000, Qualifying amount = 10% of AGTI or donation whichever is less, = 50,000 or 20,000 whichever is less, = 20,000, 50 % of the qualifying amount (i.e. 10,000), Total Income, Computation of Tax Liability, Tax on `4,83,000 at slab rate, Less: Rebate u/s 87A, Add: HEC @ 4%, Tax Liability, Solution 11:, Income under the head Business/Profession, Income under the head Business/Profession, Income under the head Other Sources, Income under the head Other Sources, Income under the head Capital gains, Long term capital gains, Income under the head Capital Gains, Gross Total Income, Less: Deduction u/s 80G, (i) National Defence Fund, (ii) Charitable institutions, Working Note:, AGTI = GTI – LTCG – STCG u/s 111A – Deduction u/s 80C to 80U (except 80G), = 4,60,000 – 2,50,000 = 2,10,000, Qualifying amount = 10% of AGTI or donation whichever is less, = 21,000 or 5,000, = 5,000, 50% of the qualifying amount = 2,500, Total Income, Computation of Tax Liability, Tax on long term capital gains `1,97,500 (`2,50,000 – `52,500) @ 20% u/s 112, Tax on normal income `1,97,500 at slab rate, Less: Rebate u/s 87A, Tax before health & education cess, Add: HEC @ 4%, , 84, Nil, (12,180.00), Nil, Nil, Nil, `, 5,00,000, 5,00,000, (7,000), (10,000), , 4,83,000, 11,650.00, (11,650.00), Nil, Nil, `, 1,00,000.00, 1,10,000.00, 2,50,000.00, 2,50,000.00, 4,60,000.00, (10,000.00), (2,500.00), , 4,47,500.00, 39,500.00, Nil, (12,500.00), 27,000.00, 1,080.00
Page 488 :
Deduction From Gross Total Income, Tax Liability, Solution 12:, Computation of income under the head Capital Gains, Short term capital gains, Long term capital gains, Income under the head Capital Gains, Computation of income under the head Other Sources, Casual income, Income under the head Other Sources, Gross Total Income, Less: Deduction u/s 80G, Working Note:, Charitable institutions, Family planning, , `, 45,000, 3,000, 48,000, AGTI = GTI – LTCG – STCG u/s 111A – Deduction u/s 80C to 80U (except 80G), = 3,65,000 – 1,00,000 = 2,65,000, Qualifying amount = 10% of AGTI or donation whichever is less, = 26,500 or 48,000, = 26,500, Deduction = 50% of 23,500 + 3,000, = 11,750 + 3,000 = 14,750, Total Income, Computation of Tax Liability, Tax on LTCG `90,250 (`1,00,000 – 9,750) @ 20% u/s 112, Tax on casual income `10,000 @ 30% u/s 115BB, Tax on normal income `2,40,250 at slab rate, Less: Rebate u/s 87A, Tax before health & education cess, Add: HEC @ 4%, Tax Liability, Rounded off u/s 288B, Solution 13., Income under the head Business/Profession, Gross Total Income, Less: Deduction u/s 80C, Less: Deduction u/s 80GG, Working Note:, Least of the following:, (i) `1,20,000 – 10% of `4,25,000 = `77,500, (ii) `60,000, (iii) 25% of `4,25,000 = `1,06,250, (AGTI = `5,00,000 – `75,000 = `4,25,000), Total Income, Computation of Tax Liability, Tax on `3,65,000 at slab rate, Less: Rebate u/s 87A, Add: HEC @ 4%, Tax Liability, , 85, 28,080.00, `, 2,55,000.00, 1,00,000.00, 3,55,000.00, 10,000.00, 10,000.00, 3,65,000.00, (14,750.00), , 3,50,250.00, 18,050.00, 3,000.00, Nil, (12,500.00), 8,550.00, 342.00, 8,892.00, 8,890.00, `, 5,00,000, 5,00,000, (75,000), (60,000), , 3,65,000, 5,750.00, (5,750.00), Nil, Nil
Page 489 :
Deduction From Gross Total Income, , 86, , Solution 14:, Income under the head House Property, Gross Total Income, Less: Deductions u/s 80GGA, Total Income, Computation of Tax Liability, Tax on `3,88,000 at slab rate, Less: Rebate u/s 87A, Add: HEC @ 4%, Tax Liability, Solution 15:, Computation of income under the head House Property, HOUSE I, Gross Annual Value, Working Note:, (a) Fair Rent, (b) Municipal valuation, (c) Higher of (a) or (b), (d) Expected Rent, (e) Rent Received or Receivable, GAV = Higher of (d) or (e), Less: Municipal taxes, Net Annual Value, Less: 30% of NAV u/s 24(a), Less: Interest on capital borrowed u/s 24(b), Income under the head House Property, HOUSE II, Gross Annual Value, Working Note:, (a) Fair Rent, (b) Municipal valuation, (c) Higher of (a) or (b), (d) Expected Rent, (e) Rent Received or Receivable, GAV = Higher of (d) or (e), Less: Municipal Taxes, Net Annual Value, Less: 30% of NAV u/s 24(a), Less: Interest on capital borrowed u/s 24(b), Income under the head House Property, HOUSE AT CHENNAI, Gross Annual Value (3,000 x 12), Less: Municipal taxes, Net Annual Value, Less: 30% of NAV u/s 24(a), Less: Interest on capital borrowed u/s 24(b), Working Note:, Current period interest, From 01.04.2021 to 30.09.2021, = 2,00,000 x 10% x 6/12 = `10,000, From 01.10.2021 to 31.03.2022, , `, 4,00,000, 4,00,000, (12,000), 3,88,000, 6,900, (6,900), Nil, Nil, `, , `, 78,000, , `, 75,000, 76,000, 76,000, 76,000, 78,000, 78,000, Nil, 78,000, (23,400), Nil, 54,600, 85,000, `, 82,000, 75,000, 82,000, 82,000, 85,000, 85,000, Nil, 85,000, (25,500), (14,000), 45,500, 36,000, Nil, 36,000, (10,800), (19,500)
Page 490 :
Deduction From Gross Total Income, , 87, , = 2,00,000 x 9.5% x 6/12 = ` 9,500, Total interest = 10,000 + 9,500 = 19,500, Income under the head House Property, 5,700, Brought forward house property loss of assessment year 2017-18, 10,100, Income under the head house property after adjusting losses, 95,700, Income under the head Other Sources, Vacant site lease rent, 4,12,000, Remunerations from Calcutta University, 3,000, Remuneration from Delhi University, 1,500, Income under the head Other Sources, 4,16,500, Income under the head Business/Profession, 9,800, Gross Total Income, 5,22,000, Less: Deduction u/s 80C, (1,50,000), Investment in bonds of NABARD, 1,000, Investment in master equity plan of UTI, 1,000, Repayment of housing loan, 2,00,000, (Whether deduction u/s 80C on repayment of the loan by, taking a fresh loan is allowed or not is not clear in the act), (But maximum upto `1,50,000), Less: Deductions u/s 80D to 80U, (1,005), Total Income (rounded off u/s 288A), 3,71,000, Computation of Tax Liability, Tax on `3,71,000 at slab rate, 6,050.00, Less: Rebate u/s 87A, (6,050.00), Add: HEC @ 4%, Nil, Tax Liability, Nil, Explanations, 1. Payments received from LIC on maturity of LIC policy is exempt under section 10(10D), 2. Investment under section 80C is allowed even from past savings and out of incomes exempt from tax., 3. Any award/reward of Central Government or State Government notified under section 10(17A) shall be, exempt from income tax., Solution 16:, Computation of Total Income, Income under the head Business/Profession, Income under the head Other Sources (Patent right), Gross Total Income, Less: Deduction u/s 80D, Less: Deduction u/s 80DD, Less: Deduction u/s 80RRB, Less: Deduction u/s 80GG, Working Note:, Least of the following:, 1. `60,000, 2. 25% x 5,27,000 = `1,31,750, 3. `3,60,000 – `52,700 = `3,07,300, (AGTI = `9,20,000 – 18,000 – 75,000 – 3,00,000 = `5,27,000), Total Income, Computation of Tax Liability, Tax on `4,67,000 at slab rate, Less: Rebate u/s 87A, Add: HEC @ 4%, Tax Liability, , `, 6,00,000.00, 3,20,000.00, 9,20,000.00, (18,000.00), (75,000.00), (3,00,000.00), (60,000.00), , 4,67,000.00, 10,850.00, (10,850.00), Nil, Nil
Page 491 :
Deduction From Gross Total Income, Solution 17:, Income under the head Other Sources, Royalty received in connection with a patent right, Gross Total Income, Less: Deduction u/s 80C, LIC premium (allowed 10% of sum assured), Repayment of housing loan to Indian Bank, Less: Deduction u/s 80CCC, LIC Pension Fund, Less: Deduction u/s 80D, Premium of medi-claim policy by cheque in the, name of his major married independent son., Less: Deduction u/s 80G, Donation to Delhi University, Family planning, Working Note:, Donation to Government for promoting family planning, 10,000, AGTI = GTI – LTCG – STCG u/s 111A – Deduction u/s 80C to 80U (Except, section 80G), = 8,00,000 – 10,000 – 50,000 – 20,000 – 30,000 – 3,00,000, = 3,90,000, Qualifying amount = 10% of AGTI or donation whichever is less, = 39,000 or 10,000, 100% of qualifying amount = `10,000, Less: Deduction u/s 80GGC, Donation to a political party, Less: Deduction u/s 80RRB, Total Income, Computation of Tax Liability, Tax on `3,70,000 at slab rate, Less: Rebate u/s 87A, Add: HEC @ 4%, Tax Liability, , 88, `, 8,00,000.00, 8,00,000.00, (10,000.00), (50,000.00), (20,000.00), Nil, (10,000.00), (10,000.00), , (30,000.00), (3,00,000.00), 3,70,000.00, 6,000.00, (6,000.00), Nil, Nil
Page 492 :
Agricultural Income, , AGRICULTURAL INCOME, SOLUTIONS OF MCQS, Answer, 1.(d); 2.(b); 3.(c); 4.(d); 5.(a); 6.(d); 7.(a); 8(d); 9(d); 10(b); 11(e); 12(c); 13(b); 14(a); 15(f), , 89
Page 493 :
Agricultural Income, , 90, , SOLUTIONS, TO, , PRACTICE PROBLEMS, Solution 1:, Income under the head Salary, Income under the head House Property, Income under the head Capital Gains (LTCG), Income under the head Other Sources (Casual Income), Gross Total Income, Less: Deduction u/s 80C, Less: Deduction u/s 80D to 80U, Total Income, Agricultural income, Computation of Tax Liability, Tax on casual income `30,000 @ 30% u/s 115BB, Tax on Long term capital gain `50,000 @ 20% u/s 112, Normal income `2,60,000, Tax on (2,60,000 + 60,000) at slab rate, Tax on (2,50,000 + 60,000) at slab rate, Tax on normal income (3,500 – 3,000), Tax before health & education cess, Add: HEC @ 4%, Tax Liability, Note: 1. Benefit of the slab rate for senior citizen is not available to non-resident assessee., 2. Rebate u/s 87A is not allowed to non-resident., , `, 3,00,000, 1,20,000, 50,000, 30,000, 5,00,000, (20,000), (1,40,000), 3,40,000, 60,000, 9,000, 10,000, 3,500, (3,000), 500, 19,500, 780, 20,280, , Solution 2:, Agricultural Income, Income from growing and manufacturing of Rubber {Rule 7A}, Agricultural income 65% and business income 35%, 1,95,000, Income from Coffee grown and cured {Rule 7B}, Agricultural income 75% and business income 25%, 1,50,000, Total, 3,45,000, Option I, House property loss can be set off from normal income, Income under the head Salary, Loss under the head House Property, Income under the head salary after adjusting house property loss, Income under the head Business/Profession, Income under the head Capital Gains, Short term capital gains, Long term capital gains, Income under the head Other Sources (Casual Income), Gross Total Income, Less: Deductions u/s 80C to 80U, Total Income, , Business Income, 1,05,000, 50,000, 1,55,000, `, 2,40,000, (1,00,000), 1,40,000, 1,55,000, 40,000, 50,000, 60,000, 4,45,000, Nil, 4,45,000
Page 494 :
Agricultural Income, , 91, , Agricultural income, 3,45,000, Computation of Tax Liability, Tax on casual income `60,000 @ 30% u/s 115BB, 18,000, Tax on Long term capital gain `50,000 @ 20% u/s 112, 10,000, Normal income `3,35,000, Tax on (3,35,000 + 3,45,000) at slab rate, 48,500, Tax on (2,50,000 + 3,45,000) at slab rate, (31,500), Tax on normal income (48,500 – 31,500), 17,000, Tax before Rebate u/s 87A, 45,000, Less: Rebate u/s 87A, (12,500), Tax before health & education cess, 32,500, Add: HEC @ 4%, 1,300, Tax Liability, 33,800, Option II, House property loss can be set off from LTCG, `, Income under the head Capital Gains (LTCG), 50,000, Loss under the head House Property, 50,000, Income under the head Capital Gains (LTCG) after adjusting house property loss, Nil, Income under the head Capital Gains (STCG), 40,000, Income under the head Salary, 2,40,000, Loss under the head House Property, 50,000, Income under the head salary after adjusting house property loss, 1,90,000, Income under the head Business/Profession, 1,55,000, Income under the head Other Sources (Casual income), 60,000, Gross Total Income, 4,45,000, Less: Deduction u/s 80C to 80U, Nil, Total Income, 4,45,000, Agricultural Income, 3,45,000, Computation of Tax Liability, Tax on casual income `60,000 @ 30% u/s 115BB, 18,000, Normal income `3,85,000, Tax on (3,85,000 + 3,45,000) at slab rate, 58,500, Tax on (2,50,000 + 3,45,000) at slab rate, (31,500), Tax on normal income (58,500 – 31,500), 27,000, Tax before Rebate u/s 87A, 45,000, Less: Rebate u/s 87A, (12,500), Tax before health & education cess, 32,500, Add: HEC @ 4%, 1,300, Tax Liability, 33,800, Tax liability is same in both the options. Therefore, house property loss can be set off either from income of, normal business or from income of long term capital gain., Solution 3:, Agricultural Income, Income from growing and manufacturing of Tea {Rule 8}, Agricultural income 60% and business income 40%, Total, Income under the head business/Profession, Income from agriculture in Nepal but received in India, Income under the head Other Sources, Gross Total Income, Less: Deductions u/s 80C to 80U, Total Income, , 6,00,000, 6,00,000, , Business Income, 4,00,000, 4,00,000, `, 4,00,000, 1,50,000, 1,50,000, 5,50,000, Nil, 5,50,000
Page 495 :
Agricultural Income, Agricultural income, Computation of Tax Liability, Tax on (5,50,000 + 6,00,000) at slab rate, Tax on (2,50,000 + 6,00,000) at slab rate, Tax on normal income (1,57,500 – 82,500), Add: HEC @ 4%, Tax Liability, Solution 4:, Income from House II, Loss from House I, Income under the head House Property, Income under the head Business/Profession, Income under the head Capital Gains (LTCG), Income under the head Other Sources, {Income from agriculture in Indonesia, received in India}, Gross Total Income, Less: Deduction u/s 80C to 80U, Total Income, Agricultural Income, Computation of Tax Liability, Tax on Long term capital gain `1,00,000 @ 20% u/s 112, Normal income `2,90,000, Tax on (2,90,000 + 2,00,000) at slab rate, Tax on (2,50,000 + 2,00,000) at slab rate, Tax on normal income (12,000 – 10,000), Tax before Rebate u/s 87A, Less: Rebate u/s 87A, Tax before health & education cess, Add: HEC @ 4%, Tax Liability, Solution 5:, Income under the head Business/Profession, Income under the head Capital Gains (LTCG), Gross Total Income, Less: Deduction u/s 80C to 80U, Total Income, Agricultural income, Computation of Tax Liability, Tax on `1,00,000 @ 30%, Tax on Long term capital gain `10,000 @ 20% u/s 112, Tax before health & education cess, Add: HEC @ 4%, Tax Liability, Note: Partial integration is not applicable in case of a partnership firm or a company., Solution 6:, Computation of Tax Liability of Partnership firm, Agricultural income, Tax liability, Computation of Tax Liability of Mr. Z, , 92, 6,00,000, 1,57,500, (82,500), 75,000, 3,000, 78,000, `, 1,00,000, (80,000), 20,000, 2,20,000, 1,00,000, 50,000, 3,90,000, Nil, 3,90,000, 2,00,000, 20,000, 12,000, (10,000), 2,000, 22,000, (12,500), 9,500, 380, 9,880, `, 1,00,000, 10,000, 1,10,000, Nil, 1,10,000, 2,00,000, 30,000, 2,000, 32,000, 1,280, 33,280, `, 20,00,000, Nil
Page 496 :
Agricultural Income, Share of profit from partnership firm {exempt u/s 10(2A)}, Income under the head House Property, Gross Total Income, Less: Deduction u/s 80C to 80U, Total Income, Tax on `2,75,000 at slab rate, Less: Rebate u/s 87A, Tax Liability, , 93, Nil, 2,75,000, 2,75,000, Nil, 2,75,000, 1,250, (1,250), Nil
Page 497 :
Clubbing of Income, , CLUBBING OF INCOME, (INCOME OF OTHER PERSONS INCLUDED, IN ASSESSEE’S TOTAL INCOME), , SECTION 60 TO 65, , SOLUTIONS OF MCQS, Answer, 1.(a); 2.(b); 3. (a); 4.(d); 5. (a); 6.(d); 7.(c); 8.(b); 9. (a); 10.(b); 11.(b); 12. (a); 13(b); 14. (c); 15. (c), , 94
Page 498 :
Clubbing of Income, , 95, , SOLUTIONS OF EXAMINATION QUESTIONS, NOV – 2018 (Old Course), Question 3(b)., (5 Marks), Solution:, Computation of Gross Total Income of Mr. Sharma, Income under the head Other Sources, `, Royalty income (6,00,000-20,000), 5,80,000, Interest from bank fixed deposits clubbed u/s 64(1A) (`5,000 -`1,500), 3,500, Value of Statue of Lord Ganesh (55,000-1,500), 53,500, Income under the head Other Sources, 6,37,000, Gross Total Income, 6,37,000, Notes:, 1. The fair market value of the statue (sculpture) received by his minor daughter as gift (not on account of, her skill) from his friends would be taxable, since its value exceeds `50,000. It would be included in the, hands of Mr. Sharma, assuming his income before considering clubbing provisions is higher than his, wife., 2. As per section 64(1A), Income of minor shall be clubbed with the income of parents having higher, income and a deduction of `1,500 shall be allowed u/s 10(32)., 3. The Fair market value of `35,000 of gold chain received on occasion of his son’s thread ceremony, would be exempt, since it is received from a relative., , NOV – 2017, Question 6(b)., (5 Marks), Answer:, As per section 64(1), If any person has transferred any asset to his or her spouse without adequate, consideration in such case Income shall be clubbed in the income of the transferor, hence Interest income of, ` 50,000 shall be clubbed in the income of Mr. Kamal., If asset received by the spouse has been invested in the proprietor business, income from the business shall, be clubbed in the income of transferor and if there is any loss, it will also be clubbed. In the given case there, is a loss of ` 15,000 from business, such loss shall be clubbed in the income of Mr. Kamal., If any person has transferred the asset to the spouse, income from the asset shall be clubbed but if same, income is invested further, any subsequent income shall not be clubbed as decided in the case of M.P., BIRLA. In the given case, Mrs. Sulochana has invested interest income in the shares and there was capital, gain on the sale of shares, such capital gain shall not be clubbed rather it will be taxable in the hands of Mrs., Sulochana., , MAY – 2012, Question 5, (5 Marks), Answer:, (a) As per Section 60 of the Income Tax Act, if any person has transferred any income without, transferring the asset in such case clubbing provision shall be applicable., In the given case, Mr. X transferred interest on fixed deposit to Mr. B (son of his brother) without, transferring the fixed deposit, such income shall be clubbed in the hands of Mr. X as per section 60., Amount to be clubbed = `5,00,000 x 9% = `45,000, (b) As per Section 64(1) of the Income Tax Act, if any person is getting salary, commission etc. from a, concern in which his or her spouse has substantial interest and further salary etc. is received without, any professional or technical qualification, in such case, salary etc. so received shall be clubbed in, the income of the spouse having substantial interest.
Page 499 :
Clubbing of Income, , 96, , In the given case Mr. X is having substantial interest in the partnership firm and Mrs. X received a, commission of `25,000 from the firm for promoting the sales of the firm without any technical or, professional qualification. So the commission shall be clubbed in the hands of Mr. X, (c) As per section 27, An individual who transfers otherwise than for adequate consideration any house, property to his or her spouse, not being a transfer in connection with an agreement to live apart shall, be deemed to be the owner of the house property so transferred., In the given case Mr. X transfers flat to Mrs. X without adequate consideration on April 1, 2021., So Mr. X shall be deemed to be the owner of the house property and income `52,000 shall be, considered as income of Mr. X., (d) As per section 64(1A), if any income accrues or arises to a minor child, such income shall be clubbed, in the income of mother or father whosoever has higher income before taking in to consideration the, income to be clubbed. So in the given case, income of ` 20,000 shall be clubbed in the income of, mother or father whosoever has higher income before taking in to consideration the income to be, clubbed. Amount to be clubbed = 20,000 – 1500 = `18,500, (e) As per section 64(1A), if any minor child has income from manual labour or through activity, involving application of his skill, talent or specialized knowledge and experience, such income shall, not be clubbed but if such income has been invested further, any new income shall be clubbed in the, income of mother or father., In the given case clubbing provision is not applicable as Mr. X’s minor son derived an income of, `20,000 through a business activity involving application of his skill and talent., Computation of Total Income of Mr. X, `, Income under the head Salary, Pension (10,000 x 12), 1,20,000.00, Less: Standard Deduction u/s 16(ia), (50,000.00), Income under the head Salary, 70,000.00, Income under the head House property, (Since asset is transferred to wife hence deeming provision shall apply), Income under the head house property, Income under the head other sources, Commission given to Mrs. X, (Since Mr. X is having substantial interest in firm), Interest on Fixed deposit transferred to Mr. B, Income from investment made by Minor son, Income under the head other sources, Gross Total Income, Less: Deduction u/s 80C to 80U, Total Income, Computation of Total Income of Mrs. X, Gross Salary, Less: Standard Deduction u/s 16(ia), Income under the head salary, Gross Total Income, Less: Deduction u/s 80C to 80U, Total Income, Computation of Total Income of Minor Child, Income from Business, Gross Total Income, Less: Deduction u/s 80C to 80U, Total Income, , 52,000.00, 25,000.00, 45,000.00, 18,500.00, 88,500.00, 2,10,500.00, Nil, 2,10,500.00, 2,40,000.00, (50,000.00), 1,90,000.00, 1,90,000.00, Nil, 1,90,000.00, 20,000.00, 20,000.00, Nil, 20,000.00
Page 500 :
Clubbing of Income, , 97, , NOV – 2011, Question 1, Answer:, Computation of total income and income tax liability of Mr. X, Income under the head Business Profession, Income under the head Capital gains, LTCG, Income under the head House property, (Since asset is transferred to son's wife hence clubbing shall be done), Gross Annual Value, Less: Municipal Taxes, Net Annual Value, Less: 30% of NAV u/s 24(a), Less: Interest on capital borrowed u/s 24(b), Income under the head house property, Income under the head other sources, Gross Total Income, Less: Deduction u/s 80C to 80U, Total Income, Computation of Tax Liability, Tax on `3,30,500 at slab rate, Tax on capital gains @ 20%, Less: Rebate u/s 87A, Tax Liability, Computation of total income and tax liability of Mrs. X, Income from house property, Loss from business profession, Income under the head Other Sources, Income under the head Capital Gains (STCG), Gross Total Income, Less: Deduction u/s 80C to 80U, Total Income, Tax Liability, , (4 Marks), `, 1,00,000.00, 50,000.00, 1,20,000.00, (5,000.00), 1,15,000.00, (34,500.00), Nil, 80,500.00, 1,50,000.00, 3,80,500.00, Nil, 3,80,500.00, 1,525.00, 10,000.00, (11,525.00), Nil, Nil, (75,000.00), 50,000.00, 2,00,000.00, 1,75,000.00, Nil, 1,75,000.00, Nil, , MAY – 2010, Question 5, (4 Marks), Answer., As per section 61, all income arising to any person by virtue of a revocable transfer of assets is to be, included in the total income of the transferor., As per section 62 the clubbing provisions are not attracted, if there is a transfer of asset which is not, revocable during the life time of the transferee, the income from the transferred asset is not includable in the, total income of the transferor provided the transferor derives no direct or indirect benefit from such income.
Page 501 :
Income under the head Other Sources, , 98, , INCOME UNDER THE HEAD, OTHER SOURCES, SECTION 56 TO 59, , SOLUTIONS OF MCQS, Answer, 1.(c); 2.(a); 3.(c); 4.(b); 5.(d); 6.(c); 7.(d); 8.(c); 9(b); 10. (b); 11. (d); 12. (d); 13. (e); 14. (c); 15. (d)
Page 502 :
Income under the head Other Sources, , 99, , SOLUTIONS, TO, , PRACTICE PROBLEMS, Solution 1:, Income under the head Business/Profession, Less: Brought forward business loss, Income under the head Business/Profession, Income under the head Other Sources (horse races), Gross Total Income, Less: Deductions u/s 80C to 80U, Total Income, Computation of tax liability, Tax on `7,000 @ 30%, Tax on `53,000 at slab rate, Less: Rebate u/s 87A, Tax Liability, Carry forward loss from owning and maintaining race horses, , `, 60,000, (7,000), 53,000, 7,000, 60,000, Nil, 60,000, 2,100, Nil, (2,100), Nil, 7,000, , Solution 2:, Income under the head Other Sources, Gross Total Income, Less: Deductions u/s 80C to 80U, Total Income, Tax on `4,00,000 at slab rate, Less: Rebate u/s 87A, Tax Liability, Note: Loss from casual income has no tax treatment and hence it is dead loss., , `, 4,00,000, 4,00,000, Nil, 4,00,000, 7,500, (7,500), Nil, , Solution 3:, Income under the head Business/Profession, Gross Total Income, Less: Deductions u/s 80C to 80U, Total Income, Tax on `4,00,000 at slab rate, Less: Rebate u/s 87A, Tax Liability, Carry forward loss from owning and maintaining race horses, , `, 4,00,000, 4,00,000, Nil, 4,00,000, 7,500, (7,500), Nil, 4,00,000, , Solution 4:, Income from House Property, Computation of income under the head Other Sources, Dividend from UTI, Pension, Working Note:, Received = `90,000, , `, 90,000, 35,000, 75,000
Page 503 :
Income under the head Other Sources, Exempt = 1/3 of pension or `15,000,whichever is less, Taxable = `75,000, Interest on bank FD, Dividend from foreign company, Winning from lottery, Income under the head Other Sources, Income under the head Capital Gains (LTCG), Gross Total Income, Less: Deduction u/s 80C, Total Income, Computation of Tax Liability, Tax on Long term capital gain `85,000 (1,20,000 – 35,000) @ 20% u/s 112, Tax on casual income `70,000 @ 30% u/s 115BB, Tax on `2,15,000 at slab rate, Less: Rebate u/s 87A, Tax before health & education cess, Add: HEC @ 4%, Tax Liability, Solution 5:, Computation of Total Income for the A.Y 2022-23, Income under head Other Source, Income from owning and maintaining race horse, Income from Royalty, Income from winning horse race (casual income), Income from winning camel race (casual income), Income from lottery income (casual income), Income under head Other Sources, Income under head Business/Profession, Income from owning and maintaining race camel, Gross Total Income, Less: Deduction u/s 80QQB (WN 1), Total Income, Computation of Tax Liability, Tax on `7,20,000 at slab rate, Tax on casual income i.e. `5,40,000 @ 30%, Tax before health & education cess, Add: HEC @ 4%, Tax Liability, Tax Liability excluding amount of casual income, Tax on ` 7,20,000 at slab rate, Add: HEC @ 4%, Total, Tax Liability including amount of casual income upto 15.12.2021, Tax on ` 7,20,000 at slab rate, Tax on casual income i.e. ` 3,40,000 @ 30%, Tax before health & education cess, Add: HEC @ 4%, Total, Interest u/s 234A, , 100, , 14,000, 36,000, 70,000, 2,30,000, 1,20,000, 4,40,000, (35,000), 4,05,000, 17,000, 21,000, Nil, (12,500), 25,500, 1,020, 26,520, `, 2,00,000, 6,00,000, 1,60,000, 1,80,000, 2,00,000, 13,40,000, 1,00,000, 14,40,000, (1,80,000), 12,60,000, 56,500, 1,62,000, 2,18,500, 8,740, 2,27,240, 56,500, 2,260, 58,760, 56,500, 1,02,000, 1,58,500, 6,340, 1,64,840, Nil, , Interest u/s 234B, 2,27,240 – 1,30,000= 97,240 = 97,200 x 1% x 3, , 2,916
Page 504 :
Income under the head Other Sources, Interest u/s 234C, Date, upto 15.06.2021, Interest = NIL, upto 15.09.2021, Interest = NIL, , Advance tax paid, 20,000, 35,000, , upto 15.12.2021, 80,000, Rounded off 119A = 43,600, Interest = 43,600 x 1% x 3 = 1,308, , 101, , Amount payable, 8,814 (58,760 x 15%), , Shortfall, NIL, , 26,442 (58,760 x 45%), , NIL, , 1,23,360 (1,64,840 x 75%), , 43,630, , upto 15.03.2022, 1,30,000, 2,27,240, Rounded off 119A = 97,200, Interest = 97,200 x 1% x 1 = 972, Interest u/s 234C, Total Interest Payable, Rounded off u/s 288B, Working Notes:, 1. Maximum deduction allowed u/s 80QQB, 15% of print price i.e. ` 600 x 15% x 2,000= `1,80,000., , 97,240, , Solution 6:, Net Distributable Amount, Share of Mr. X (10%), Share of Mr. X out of accumulated profits which is considered dividend u/s 2(22)(c), Balance to be considered full value of consideration, Less: Cost of acquisition of shares, Short term Capital Gain, Dividend u/s 2(22)(c), Computation of Total Income of Mr. X for A.Y. 2022-23, Income under the head Capital Gains, Short Term Capital Gains, Income under the head Other Sources, Interest Income of Minor Son clubbed u/s 64(1A), 2,00,000, Less: Exempt u/s 10(32), 1,500, Dividend u/s 2(22)(c), Income under the head Other Sources, Income under the head Business Profession, Income of Mrs. X clubbed u/s 64(1), Gross Total Income, Less: Deduction u/s 80C to 80U, Total Income, Agricultural Income, Computation of Tax Liability, Normal income 7,98,500, Step 1. Tax on (7,98,500 + 7,00,000) at slab rate, Step 2. Tax on (`2,50,000 + 7,00,000) at slab rates, Step 3. Deduct Tax at Step 2 from Tax at Step 1, Add: HEC @ 4%, Tax Liability, Rounded off u/s 288B, , `, 60,00,000, 6,00,000, (2,00,000), 4,00,000, (1,00,000), 3,00,000, 2,00,000, , 2,280, 5,196, 5,200, , 3,00,000, , 1,98,500, 2,00,000, 3,98,500, 1,00,000, 7,98,500, Nil, 7,98,500, 7,00,000, 2,62,050.00, (1,02,500.00), 1,59,550.00, 6,382.00, 1,65,932.00, 1,65,930.00
Page 505 :
Income under the head Other Sources, , 102, , SOLUTIONS OF EXAMINATION QUESTIONS, NOV – 2019 (NEW COURSE), Question.2. (a), , (7 Marks), , Solution:, Computation of total income of Mr. Jagdish for the A.Y. 2022-23, Stay in India for a minimum period of 182 days in the relevant previous year or, in the alternative, 60 days, in the relevant previous year and 365 days in the four immediately preceding previous years is required to, qualify as a resident. In this case, since Mr. Jagdish has not visited India at any time during the P.Y.2021-22,, he would be a non - resident for that year., Income under the head house property, Flat in pune, GAV (Rent received/receivable) (27,500 x 12), Less: Municipal tax paid, , 3,30,000, (Nil), , NAV, , 3,30,000, , Less: Standard deduction u/s 24(a) @ 30%, , (99,000), , Less: interest on loan u/s 24(b), , (84,000), , Income from flat in pune, , 1,47,000, , Arrears of rent (96,000-28,800), Income from house property, , 67,200, 2,14,200, , Income from other sources, Interest on bonds (50% received in India), , 22,500, , Gross total Income, Income under the head house property, Income under the head other sources, Gross Total income, , 2,14,200, 22,500, 2,36,700, , Less: Deductions u/s 80C to 80U, Total Income, , Nil, 2,36,700, , Notes:, 1. Rent from Bangkok house property is assumed to be received in Bangkok., 2. Municipal tax paid in 20-21 and not paid in 21-22 hence not deducted from GAV., 3. Income from apple orchid is received in Nepal as deposited in Nepal hence not taxable in India in, case of NR. Student can solve the answer by assuming received in India, Question.4. (a), Solution:, Computation of Taxable income of Mr. Mahadev for the A.Y. 2022-23, Income under the head business profession, , (5 Marks)
Page 506 :
Income under the head Other Sources, Professional income (computed), , 103, 5,65,000, , Income under the head other sources, Interest income of daughter Gudia, , 40,000, , Less: exemption u/s 10(32), , (1,500), , Income under the head other sources, , 38,500, , Gross Total Income, , 6,03,500, , Less: Deduction u/s 80C to 80U, , Nil, , Total Income, , 6,03,500, , Computation of Taxable income of Mrs. Dariya for the A.Y. 2022-23, Income under the head salary (computed), , 3,80,000, , Gross Total Income, , 3,80,000, , Less: Deduction u/s 80C to 80U, , Nil, , Total Income, , 3,80,000, , Notes:, 1. Income of Minor shall be clubbed in the income of parents who have higher income before clubbing the, income of minor. In the given case income of father is greater than mother’s income hence amount shall be, clubbed in the income of Father., 2. Loan amount would not be considered as deemed dividend u/s 2(22)(e) as company does not have, accumulated profits., 3. Income from skill and talent shall not be clubbed in the income of the parents and shall be taxable in the, hands of children itself., 4. As per section 56(2)(x), Gift shall be taxable if amount received during the year exceed 50,000 but in the, given case, cash gift received is less than 50,000 hence nothing shall be clubbed in the hands of the parent., , MAY – 2018, Question 3(b), Answer:, Tax Liability of Mr. A, Dividend received, Income under the head Other Sources, Total Income, Computation of Tax Liability, Tax on ` 17,00,000 at slab rate, Add: HEC @ 4%, Tax Liability, Tax Liability of Mr. B, Dividend received, Income under the head Other Sources, Total Income, Computation of Tax Liability, , (3 Marks), 17,00,000, 17,00,000, 17,00,000, 3,22,500, 12,900, 3,35,400, 8,50,000, 8,50,000, 8,50,000
Page 507 :
Income under the head Other Sources, , 104, , Tax on ` 8,50,000 at slab rate, Add: HEC @ 4%, Tax Liability, , 82,500, 3,300, 85,800, , NOV – 2016, Question 4(a), (2 Marks), Answer:, As per section 56, if any person has entered into an agreement to sell any capital asset and some advance, money was received but the buyer refused to purchase the capital asset and advance money was forfeited, in, such cases the amount so forfeited shall be considered to be income under the head Other Sources. So, Forfeiture of advance money shall be considered as income from other source and shall not be deducted, from cost of acquisition of the property., , MAY – 2016, Question 4(a), (2 x 2 = 4 Marks), Answer:, (i) Taxable: As per section 56(2)(viib), If any closely held company receives any consideration for issue of, shares that exceeds the face value of shares then the aggregate consideration received as exceeds the fair, market value of shares is considered as income under the other sources. In the given case, ABC Private, Ltd. issued shares at a price which exceeds the face value of shares. So the taxable amount shall be, `1,00,000 (10,000 shares x (`130-`120)), (ii) Taxable: If any person has entered into an agreement to sell any capital asset and some advance money, was received but the buyer refused to purchase the capital asset and advance money was forfeited, in, such cases the amount so forfeited shall be considered to be income under the head Other Sources. In the, given case, Mr. A forfeited `50,000 against sale of his house shall be considered as income under the, head other sources., Question 7(a)(iii), (2 Marks), Answer:, True: Dividend received by a dealer of shares is chargeable under the head “Income from Other Sources”., , NOV – 2010, Question 7, Answer., , (4 Marks), , (i), , Particulars, Head of Income, Rental income in case of a dealer engaged in business of Profit and gains of business or, letting out of house property, profession, , (ii), (iii), , Dividend on shares in case of a dealer in shares, Salary by partner from his partnership firm, , (iv), , Rental income of machinery (See Note below), , (v), , Income from other sources, Profit and gains of business or, profession, Income from other sources/ Profits and, gains of business or profession, Income from other sources, , Winnings from lotteries by a person having the same as, business activity, (vi) Salaries payable to a Member of Parliament, Income from other sources, (vii) Receipts without consideration, Income from other sources, Note: As per section 56, rental income of machinery would be chargeable to tax under the head “Income, from Other Sources”, if the same is not chargeable to income-tax under the head “Profits and gains of, business or profession”.
Page 508 :
Deduction of tax at source, , DEDUCTION OF TAX AT SOURCE, SECTION 190 TO 206AA, SOLUTIONS OF MCQS, Answer, 1.(a); 2.(a); 3.(c); 4. (c); 5. (b); 6. (d); 7. (a); 8.(b); 9.(d); 10.(d); 11. (c); 12. (c); 13(a); 14. (c); 15. (b), , 105
Page 509 :
Deduction of tax at source, , 106, , SOLUTIONS OF EXAMINATION QUESTIONS, NOV – 2019 (NEW COURSE), Question. 2. (b), (7 Marks), Answer:, (i) As per section 192A, If person responsible for making payment of recognized provident fund to any, person shall deduct tax @ 10% if amount paid/payable is taxable and amount is exceeding `50,000 but if, amount is withdrawn before continuous period of five years due to ill health then it is not taxable and TDS, shall not be deducted., In the given case amount is withdrawn from Employees provident fund before continuous period of 5 years, due to ill health hence the amount is not taxable hence TDS shall not be deducted., (ii) As per section 194A, Tax shall be deducted @ 10% if interest paid by banking company is exceeding, `50,000 for senior citizens. In the given case, interest is paid to senior citizen and amount is not exceeding, `50,000 hence banking company is not liable to deduct TDS., (iii) As per section 194B, Every person responsible for paying resident any income by way of winning and, amount paid or payable is exceeding `10,000 then tax shall be deducted @ 30%., In the given case, amount paid is exceeding `10,000 hence liable to deduct tax @ 30% on `21,000 = `6,300., (iv) As per section 2(28A), Interest means interest payable in any manner and includes service fee also., As per section 194A, TDS is not deductible in case any payment is made to a banking company., In the given case, Service fee is paid to banking company and interest includes service hence TDS is not, deductible in case of payment to a banking company., (v) As per section 194IB, any person responsible for paying rent to a resident exceeding `50,000 for a month, shall deduct tax @ 5%, in the given case rent paid is exceeding `50,000 hence Mr. Ashok is liable to deduct, tax @ 5% on `1,56,000 = `7,800, Question.4. (c), , (4 Marks), , Answer:, (i) No. As per section 206C, TCS shall be applicable on sale of motor vehicle of the value exceeding 10, lakhs., (ii) TCS @ 1% is collected on sale consideration of motor vehicle exceeding 10,00,000. It is applicable to, each sale and not on aggregate value of sale during the year., (iii) An individual who is liable to tax audit during the financial year immediately preceding the financial, year in which the motor vehicle is sold shall be liable for collection of tax at source., (iv) TCS on sale of motor vehicle is applicable on all transactions of retail sales only and not applicable on, sale by manufacturer to distributor or dealer., , MAY – 2019 (NEW COURSE), Question 2 (b), (7 Marks), Answer:, (i) As per section 194DA, in general payment on maturity of Life policy is exempt from income tax under, section 10(10D) however sometimes the amount is taxable (if premium paid has exceeded the prescribed, percentage (i.e. 10% / 15% / 20%)) and in that case tax has to deducted at source @ 5% on the amount of, income provided the amount paid or payable to any resident during a particular financial year is `1,00,000, or more. In the given case, premium paid is more than 10% and the maturity proceeds is taxable but the, amount is less than 1,00,000 hence TDS shall not be deducted.
Page 510 :
Deduction of tax at source, , 107, , (ii) As per section 194-IA, every person (including individual and HUF) making payment to a resident for, purchase of immovable property of `50 lakhs or more shall deduct tax at source @ 1% of such sum. In the, given case, though the stamp duty value is exceeding `50 lakhs but payment is not exceeding `50 lakhs, hence TDS is not applicable and Mr. Karan is not required to deduct TDS., (iii) As per section 194J, every person, who is responsible for paying to a resident any sum by way of –, (i) fees for Professional services, (ii) any Remuneration or fees or commission to a director of a company (in case salary is being paid, to a director, tax shall be deducted at source under section 192)., (iii) Royalty, (iv) Non-compete fee referred to in section 28, shall deduct tax at source at the rate of 10% if amount paid or payable is more than ` 30,000 and limit shall, be separately applicable for each of the above payments but there is no limit for payment to a director., In the given case, amount paid is less than limit prescribed above hence no TDS shall be required to be, deducted., (iv) As per section 194C, every person responsible for making payment to a resident contractor in, connection with any work shall deduct tax at source @ 2% and in case of payment to individual or Hindu, Undivided Family, the rate of TDS shall be 1%. Tax shall be deducted at source only if the amount being, paid is exceeding `30,000 or the amount paid or payable during a particular financial year to a particular, person exceeds `1,00,000., If any person making payment for purchase of goods, no tax shall be deducted at source but if such person, has supplied raw material etc. and contract is only for labour etc., tax shall be deducted at source., In the given case, material is not supplied hence TDS is not required to be deducted., (v) As per section 194J, TDS is required to be deducted if payment is made to a director and no limit of, `30,000 is applicable in case of payment to director. In the given case, payment is made to director hence, TDS is applicable and required to be deducted @ 10% on `12,000., (vi) As per section 194J, TDS is required to be deducted @ 2% instead of 10% in case of person engaged in, business of call centre . In the given case Radha Limited is engaged only in the business of operation of call, centre. TDS is required to be deducted @ 2% on ` 70,000, , MAY – 2019 (OLD COURSE), Question 2 (b), (3 Marks), Answer:, (i) As per section 192, in respect of salary payments to employees of Government deduction of tax should be, made after allowing relief under section 89. In the given case arrears of salary received in current year if the, same was not taxed earlier year then same will be taxable and TDS is required to be deducted but if the same, was considered earlier as part of salary then TDS is not required in the current year., (ii) Every person (including individual and HUF even if limit prescribed has not exceeded in the preceding, year) responsible for paying to any resident or non-resident, any income by way of winnings from any, lottery or crossword puzzle or card game and other game of any sort shall deduct tax at source @ 30%, provided the amount being paid or payable is exceeding ` 10,000., In the given case a T.V. channel pays 10 lacs as prize hence TDS is required to be deducted @ 30%., (iii) As per section 194I, TDS is required to be deducted in case of person responsible for paying to a, resident any income by way of rent shall deduct tax @ 10% on renting of immovable property provided the, amount paid or payable is more than ` 2,40,000 in a year., In the given case Nationalized bank pays ` 50,000 per month which is more than ` 2,40,000 in a year hence, TDS is required to be deducted @ 10% on ` 50,000 per month., (iv) The television company is required to deduct tax at source @ 10% u/s 194J on the professional fees, payable to the cameraman for shooting a documentary film, since such amount exceeds 30,000 during the, year.
Page 511 :
Deduction of tax at source, , 108, , NOV – 2018 (NEW COURSE), Question 2 (b), (2 Marks), Answer:, TDS in case of Payment for purchase of immovable property Section 194-IA, As per section 194IA, every person (including individual and HUF even if limit prescribed has not, exceeded in the preceding year) making payment to a resident for purchase of immovable property of `50, lakhs or more shall deduct tax at source @ 1% of such sum but no tax shall be deducted at source in case of, payment for purchase of agricultural land which is situated in the rural area., (i) As per the above provision TDS is required to be deducted on purchase of residential house situated in, Chennai., TDS = 1% of `60,00,000 = `60,000, (ii) As per the above provision TDS is not required to be deducted on purchase of agriculture land on rural, area., Question 2(c), (2 Marks), Answer:, As per section 206C, tax collection at source means the person receiving payment for selling of motor, vehicle value exceeding `10 lakhs shall collect tax at source from the buyer however TCS on sale of motor, vehicle shall be applicable in case of retail sale only accordingly it will not apply on sale of motor vehicle by, manufacturer to distributors/dealers., In the given case, TCS shall not be applicable in case of sale of car from manufacturer to dealers. So, XYZ, Ltd. is not required to collect TCS on sale of motor vehicle to dealers., In the given case, Rahil & Co. selling cars to individual buyers and value is exceeding `10 lakhs hence, Rahil & Co. shall be required to collect TCS @ 1% on sale of car., , NOV – 2018 (OLD COURSE), Question 6 (a), , (5 Marks), , Answer:, (i) As per section 194DA, In general payment on maturity of Life policy is exempt from income tax under, section 10(10D) however sometimes the amount is taxable (if premium paid has exceeded the prescribed, percentage (i.e. 10% / 15% / 20%)) and in that case tax has to deducted at source @ 5% on the amount of, income provided the amount paid or payable to any resident during a particular financial year is `1,00,000, or more., In the given case premium paid is exceeding the prescribed limit hence it is taxable but the amount paid or, payable is not exceeding `1,00,000 hence TDS is not required to be deducted., (ii) The issue of whether fees/charges taken or retained by advertising companies from media companies for, canvasing/booking advertisements (typically 15% of the billing) is 'commission' or 'discount' to attract the, provisions of tax deduction at source has been clarified by the CBDT vide its Circular No.5/2016 dated, 29.2.2016. The CBDT has clarified that no liability to deduct tax is attracted on payments made by, television channels to the advertising agency for booking or procuring of or canvassing for advertisements., Accordingly, no tax is deductible at source on the amount of `15 lakhs retained by Mudra Ltd., the, advertising company, from payment due to Cloud TV, a television channel., (iii) As per section 194IB, any person, being an individual or a Hindu undivided family, not exceeding the, specified limit in the preceding year, responsible for paying to a resident any income by way of rent, exceeding `50,000 for a month or part of a month during the previous year, shall deduct tax @ 5% but if the, person receiving payment of rent has not submitted PAN, tax shall be deducted @ 20% but maximum rent, payable for the last month. Tax shall be deducted at the time of making payment of rent for the last month of, the previous year or the last month of tenancy whichever is earlier., In the given case, TDS is required to be deducted as rent is exceeding `50,000 per month., Total Rent from June to Dec 2020 = `55,000 x 7 months = `3,85,000, he should deduct tax at source `3,85,000 x 5% = `19,250 but if person receiving payment has not submitted, PAN, amount of TDS shall be `3,85,000 x 20% = `77,000 but maximum `55,000.
Page 512 :
Deduction of tax at source, , 109, , 6 (c)., Answer:, As per Section 208, if tax payable is `10,000 or more, in that case assessee shall be required to pay advance, tax. Tax payable means total tax liability less tax deducted at source hence in the given case advance tax has, to be paid with regard to ` 2,00,000 (2,00,000 – Nil (as estimated but not deducted)), As per section 211, all assessee has to pay advance tax in the manner given below:, Due date of installment, Upto 15th June of P.Y., `30,000, Upto 15th September of P.Y., `90,000, th, Upto 15 December of P.Y., `1,50,000, Upto 15th March of P.Y., `2,00,000, , Amount payable, (15% of `2,00,000), (45% of `2,00,000), (75% of `2,00,000), (100%), , MAY – 2018, Answer:, (A) As the turnover of Mr. Bobby for F.Y.2020-21, i.e. `205 lakh, has exceeded the monetary limit, prescribed for deduction of tax by individual/HUF, he has to comply with the tax deduction provisions, during the financial year 2021-22., (i) As per Section 194-I TDS is required to be deducted @ 10% as the rent amount exceeds `2,40,000 per, annum., TDS per month = 25,000 x 10% = 2,500, (ii) As per section 194J, Limit of 30,000 per annum shall be considered separately for each services hence, no TDS is required to be deducted as payment made for technical services and royalty both is not exceeding, 30,000 separately., (iii) Tax shall not be deducted at source in case of purchase of goods., (iv) It is covered under section 194C but payment is not exceeding `30,000 hence no tax shall be deducted at, source., (B) As per section 194-IB, Any person, being an individual or a Hindu undivided family, not exceeding the, specified limit in the preceding year, responsible for paying to a resident any income by way of rent, exceeding `50,000 for a month or part of a month during the previous year, shall deduct tax @ 5%. In the, given case rent is exceeding `50,000 hence TDS shall be deducted 5% ., TDS = 60,000 x 12 x 5% = 36,000., , MAY – 2017, Question 7 (a), (4 Marks), Answer:, As per section 194A, Every person making payment of interest other than interest on securities to any, resident shall deduct tax at source @ 10% provided the amount being paid or payable during a particular, year to a particular person is exceeding `5,000 but if payment is being made by bank or post office, tax shall, be deducted only if interest being paid or payable is exceeding `40,000. Further TDS shall be only on time, deposit including recurring deposit. Limit of `40,000 shall be per branch of the bank but if the bank has core, banking solution, limit shall be per bank and not per branch., In the first case there is no core banking services in the bank then the limit shall be per branch and amount, has not exceeded `40,000 per branch hence no TDS shall be required to be deducted., In the Second case, there is a core banking services and limit has exceeded including both the branch hence, TDS shall be required to be deducted @ 10% on `57,000.
Page 513 :
Deduction of tax at source, , 110, , Question 7 (c), , (4 Marks), , Solution: As per section 234B, if advance tax paid is less than 90% of actual tax liability, assessee shall be, required to pay interest @ 1% per month or part of a month from 1 st April of assessment year upto the date, of payment. If advance tax paid is 90% or more of actual tax liability, no interest is payable., `, Income from Business, 8,10,000, Income from Other source, 2,70,000, Gross Total Income, 10,80,000, Less: Deduction u/s 80C to 80U, Nil, Total Income, 10,80,000, Computation of Tax Payable, Tax on 10,80,000 at slab rate, 1,36,500, Add: HEC @ 4%, 5,460, Tax Liability, 1,41,960, Less: TDS, (25,000), Tax Payable before adjustment of advance tax, 1,16,960, Less: Advance Tax, (1,03,000), Tax Payable, 13,960, Calculation of Interest u/s 234C, Interest under section 234C shall be computed in the manner given below:, Tax Payable, Tax Paid, Shortfall, 15.06.2021, 17,544 (1,16,960 x 15%), Nil, 17,544, Rounded off Rule 119A = 17,500, Interest u/s 234C = 17,500 x 1% x 3 = 525, 15.09.2021, 52,632 (1,16,960 x 45%), Rounded off Rule 119A = 52,600, Interest u/s 234C = 52,600 x 1% x 3 = 1,578, 15.12.2021, 87,720 (1,16,960 x 75%), Interest u/s 234C = 87,700 x 1% x 3 = 2,631, 15.03.2022, 1,16,960 (1,16,960 x 100%), Interest u/s 234C = 13,900 x 1% x 1 = 139, Total interest payable u/s 234C, , Nil, , 52,632, , Nil, , 87,720, , 1,03,000, , 13,960, 4,873, , Calculation of Interest u/s 234B (01-04-2022 to 11-12-2022), Since Advance Tax paid i.e. `1,03,000 is less than 90% of the actual tax payable i.e. `1,16,960, hence, Interest u/s 234B shall be payable and Interest shall be –, 13,900 x 1% x 9 = 1,251, Total Interest Payable (4,873 +1,251), 6,124, , NOV – 2016, Question 4(a), (2 Marks), Answer:, False: As per Section 194A, every person making payment of interest other than interest on securities to any, resident shall deduct tax at source @ 10% provided the amount being paid or payable during a particular, year to a particular person is exceeding `40,000. Further TDS shall be only on time deposit including, recurring deposit. Hence TDS shall be applicable in the given case.
Page 514 :
Deduction of tax at source, , 111, , Question 7(a) (ii), (4 Marks), Answer: As per Section 208, if tax payable is ` 10,000 or more, in that case assessee shall be required to, pay advance tax. Tax payable means total tax liability less tax deducted at source hence in the given case, advance tax has to be paid with regard to ` 60,000 (65,000 – 5,000), As per section 211, all assessee has to pay advance tax in the manner given below:, Due date of installment, Upto 15th June of P.Y., `9,000, th, Upto 15 September of P.Y., `27,000, Upto 15th December of P.Y., `45,000, Upto 15th March of P.Y., `60,000, , Amount payable, (15% of `60,000), (45% of `60,000), (75% of `60,000), , MAY – 2016, Question 7(a)(i), (4 Marks), Answer., As the turnover of Mr. Ashwin for F.Y.2020-21, i.e. `120 lakh, has exceeded the monetary limit prescribed,, he has to comply with the tax deduction provisions during the financial year 2021-22., Commission paid to Babloo – Tax has to be deducted under section 194-H as the commission exceeds, `15,000., Tax shall be deducted at source u/s 194H and shall be = 18,500 x 5% = `925, Contract payment of `23,000 to Mr. Vijay, TDS provisions under section 194C would not be attracted if the amount paid to a contractor does not, exceed `30,000 in a single payment or `1,00,000 in the aggregate during the financial year. Therefore, TDS, provisions under section 194C are not attracted in this case., Payment of fees for Technical Services, to Vivek, Tax shall be deducted at source under section 194J @ 2% because the total amount payable is exceeding, `30,000 and amount of TDS shall be = 35,000 x 2% = `700, , MAY – 2015, Question 7(a)(ii)., (4 Marks), Solution:, As per Section 194-IA every person making payment to a resident for purchase of immovable property of, `50 lakhs or more shall deduct tax at source @ 1% of such sum. No tax shall be deducted at source in case, of payment for purchase of agricultural land which is situated in the rural area., In the given case Mr. Raman is required to deduct tax at source @ 1% of `65 lakh = ` 65,000., The person deducting tax at source shall not be required to obtain Tax deduction account number as per, section 203A., , NOV – 2014, Question 7(a), (4 Marks), Solution:, (1) TDS shall be deducted u/s 194E @ 20%+HEC. TDS shall be 20.8% of `27,000 = `5,616 ., (2) As per Section 194-I TDS is not required to be deducted as the Rent amount does not exceeds `2,40,000., (3) TDS shall be deducted under section 194BB @ 30% as the amount exceeds `10,000. TDS shall be 30%, of `1,50,000 = `45,000., (4) As per Section 194LA No TDS is deductible by State of Uttar Pradesh as the amount paid does not, exceeds `2,50,000.
Page 515 :
Deduction of tax at source, , 112, , MAY – 2014, Question 7(A)., (2 x 2 = 4 Marks), Solution:, (A) Section 194J provides for deduction of tax at source @10% from any sum paid by way of any, remuneration or fees, by whatever name called, to a resident director, which is not in the nature of salary on, which tax is deductible under section 192. The threshold limit of ` 30,000 upto which the provisions of tax, deduction at source are not attracted in respect of every other payment covered under section 194J is,, however, not applicable in respect of sum paid to a director. Therefore, tax @ 10% has to be deducted at, source under section 194J in respect of the sum of ` 19,000 paid by ABC Ltd. to its director., (B) Section 194-IA, tax shall be deducted at source if the amount paid or payable is `50,00,000 or more, hence in the given case tax deducted at source amounting to `60,00,000 x 1% = 60,000., , NOV – 2012, Question No. 7(a), (4 Marks), Answer., As the turnover of Mr. X for F.Y.2020-21, i.e. ` 201 lakh, has exceeded the limit prescribed, he has to, comply with the tax deduction provisions during the financial year 2021-22., (i) TDS under section 194A is not attracted in respect of interest paid to a banking company., (ii) Advertisement =TDS on `34,000 @ 1% = `340 (Sec 194C is applicable if single payment exceeds, `30,000), (iii) Tax has to be deducted under section 194-I as the rent amount exceeds `2,40,000., TDS @ 10% on ` 2,85,000 = `28,500, (iv) Tax has to be deducted under section 194-H as the commission exceeds `15,000., TDS @ 5% on ` 16,000 = `800, , NOV – 2011, Question 7, (4 Marks), Answer., (a) Since the rent paid for hire of machinery by ABC Ltd. to Mr. X exceeds `2,40,000, the provisions of, section 194-I for deduction of tax at source are attracted., The rate applicable for deduction of tax at source under section 194-I on rent paid for hire of plant and, machinery is 2% assuming that Mr. X had furnished his permanent account number to ABC Ltd., Therefore, the amount of tax to be deducted at source: = `3,10,000 x 2% = `6,200, (b) As per the provisions of section 194M, Any person, being an individual or a Hindu undivided family, (other than those who are required to deduct income-tax as per the provisions of section 194C or section, 194J) responsible for paying any sum to any resident for carrying out any work (including supply of labour, for carrying out any work) in pursuance of a contract or by way of fees for professional services during the, financial year, shall, at the time of credit of such sum or at the time of payment of such sum in cash or by, issue of a cheque or draft or by any other mode, whichever is earlier, deduct an amount equal to five per, cent. of such sum as income -tax thereon:, Provided that no such deduction under this section shall be made if such sum or, as the case may be,, aggregate of such sums, credited or paid to a resident during a financial year does not exceed fifty lakh, rupees., Therefore, in the given case, fee paid to Dr. Y is ` 35,000, which is less than 50,00,000, the liability to, deduct tax at source is not attracted., , NOV – 2011, Question 6, (6 Marks), Answer., (i) No tax shall be deducted at source in case of payment to a transporter who has submitted his PAN., (ii) Tax shall be deducted at source under section 194J @ 2% because the total amount payable is exceeding, `30,000 and amount of TDS shall be = 45,000 x 2% = `900
Page 516 :
Deduction of tax at source, , 113, , (iii) It is covered under section 194C but payment is not exceeding `30,000 hence no tax shall be deducted, at source., (iv) Tax shall not be deducted at source in case of purchase of goods., (v) Since payment is not exceeding `2,50,000 hence no tax shall be deducted at source under section 194LA., (vi) Tax shall be deducted at source u/s 194H and shall be = 25,000 x 5% = `1,250, , MAY – 2011, Question 5, Answer., The provisions of section 194A will not apply in the following cases where the –, (1) Interest is paid by a firm to a partner of the firm;, (2) Any interest being paid to Bank/LIC or other notified financial organizations, (3) Interest on income tax refund or wealth tax refund etc., (4) Income paid in relation to a Zero Coupon Bond., (5) Interest paid in respect of deposits under any scheme notified by the government., , (2 Marks), , Question 7, (4 Marks), Answer., As the turnover of Mr. X for F.Y.2020-21, i.e. `215 lakh, has exceeded the monetary limit prescribed, he has, to comply with the tax deduction provisions during the financial year 2021-22, subject to however the, exemptions provided for under the relevant sections for applicability of TDS provisions., Interest paid to UCO Bank, TDS under section 194A is not attracted in respect of interest paid to a banking company., Contract payment of `24,000 to Mr. Y for 2 contracts of `12,000 each, TDS provisions under section 194C would not be attracted if the amount paid to a contractor does not, exceed ` 30,000 in a single payment or `1,00,000 in the aggregate during the financial year. Therefore, TDS, provisions under section 194C are not attracted in this case., Shop Rent paid to one payee – Tax has to be deducted under section 194-I as the rental payment exceeds, `2,40,000., Commission paid to Balu – Tax has to be deducted under section 194-H as the commission exceeds, `15,000., , MAY – 2010, Question 4, (2 Marks), Answer. No deduction is required to be made from any sum credited or paid or likely to be credited or paid, during the previous year to the account of a contractor, during the course of the business of plying, hiring or, leasing goods carriages, if the contractor furnishes his permanent account number (PAN) to the person, paying or crediting such sum and submitted a declaration that he don’t own more than 10 goods carriages at, any time during the year., , MAY – 2010, Question 4, (4 Marks), Answer., (i) Section 194C provides for deduction of tax at source from the payment made to resident contractors and, sub-contractors. Therefore, tax is deductible at source under section 194C for the contract payments made, for the construction of godown building. The rate of TDS under section 194C on payments made to, contractors who are individuals or HUF shall be @ 1%. Hence, X & Co. (firm) must deduct tax at source on, the contract payments made to Mr. X., (ii) In respect of lucky dip conducted by Y & Co., the provisions of Section 194B would apply. As per, Section 194B, winning from lottery or crossword puzzle or card game or other game of any sort exceeding, `10,000 payable by any person to any other person, subject to tax deduction at the rate of 30%. Since the, value of prize i.e. Maruti car would exceed `10,000 tax is deductible at source @ 30%. As the winning is in, kind, the winner must deposit 30% of the prize value to Y & Co. for remitting the same as tax. Only after, such deduction / recovery, the Maruti car is to be delivered to the prize winner.
Page 517 :
Deduction of tax at source, , 114, , (iii) As per section 194D, any person paying insurance commission in excess of `15,000 to any resident, person is liable to deduct tax at the rate of 5% in case of all assesses. Therefore, the insurance company must, deduct tax at source @ 5% in respect of the insurance commission paid to Mr. Y., (iv) Discount allowed to a customer for prompt payment is not covered by any of the tax deduction at source, provisions of the Income tax Act, 1961. Therefore, AB Ltd. need not deduct any tax at source since no, payment was involved in allowing discount to its customer viz. (namely) XY & Co., , NOV – 2009, Question 2, (6 Marks), Answer., Computation of income of Mr. X chargeable under the head “Income from other sources” for the, A.Y. 2022-23, Particulars, `, 1. Directors’ fees, 3,00,000.00, 2. Interest on bank deposit, 23,000.00, 3. Royalty on books written (2,09,000 – 10,000), 1,99,000.00, 4. Lectures in seminars, 5,000.00, 5. Interest on loan given to a relative, 7,000.00, 6. Interest on listed debentures, 36,000/ 90% x 100%, 40,000.00, 7. Interest on Post Office Savings Bank [exempt under section 10(15)], 8. Interest on Government securities, 2,200.00, 9. Interest on Post Office Monthly Income Scheme, 33,000.00, 10. Winnings from lotteries, 35,000 / 70% x 100%, 50,000.00, Income from Other Sources, 6,59,200.00, Gross Total Income, 6,59,200.00, Less: Deduction u/s 80TTA, (10,000.00), Total Income, 6,49,200.00, Computation of Tax Liability, Tax on casual income `50,000 @ 30% u/s 115BB, 15,000.00, Tax on `5,99,200 at slab rate, 32,340.00, Tax before health & education cess, 47,340.00, Add: HEC @ 4%, 1,893.60, Tax Liability, 49,233.60, Rounded off u/s 288B, 49,230.00, , NOV – 2009, Question 1, (2 Marks), Answer., False: An individual whose total sales/gross receipts/turnover from the business or profession exceeds the, monetary limits specified during the financial year immediately preceding the financial year in which such, sum is credited or paid to the account of the contractor shall be required to deduct tax at source., Question 3, (4 Marks), Answer., (1) Tax deducted at source under section 194-I would be required to be made on the amount of rent i.e. on, `12,00,000., (2) TDS shall be applicable @ 10%.
Page 518 :
Deduction of tax at source, , 115, , (3) Hence, in the given case, TDS under section 194-I @ 10% on `12,00,000 comes to `1,20,000 i.e., `10,000 will be deducted every month and cheque for net amount of (`1,00,000 – `10,000 = `90,000) will, be issued by M/s. ABC Corporation Ltd. to Mrs. X., (4) Tax deducted will be deposited within one week from the last day of the month in which deduction is, made but if the tax has been deducted in the month of March, tax should be deposited on or before 30 th, April., (5) Form No. 16-A has to be issued by M/s. ABC Corporation Ltd. to Mrs. X within 15 days from the last, date of submitting the quarterly return under section 200., (6) Assessee should file quarterly statement upto 31st of the month succeeding the relevant quarter but for, the last quarter ending March, return can be submitted upto 31 st May., , JUNE – 2009, Question 1, (2 Marks), Answer. As per section 193 of the Act, no tax is required to be deducted at source on any interest payable to, a resident on any bond or security issued by a company, where the following conditions are satisfied –, (i) where such security is in dematerialised form and, (ii) is listed on a recognised stock exchange in India., , MAY – 2008, Question 1, (2 Marks), Answer. In the case of an individual, the provisions of section 194C shall apply, where the turnover from, business has exceeded the prescribed limit during the financial year immediately preceding the financial, year in which such payment is made and payment is made for a purpose other than the personal purpose. In, the given case, since the turnover of Mrs. X has exceeded the prescribed limit for the year ended 31 st March, 2021 and the payment is relating to business activities, she shall be liable to deduct tax at source in respect, of payment made to a contractor at the applicable rate.
Page 519 :
Miscellaneous, , MISCELLANEOUS TOPICS, SOLUTIONS OF MCQS, Answer, 1.(d); 2.(d); 3.(d); 4.(c); 5.(a); 6.(a); 7.(d); 8. (d); 9.(b), , 116
Page 520 :
M.K.G, CA EDUCATION, 9811429230 / 9212011367, WWW.MKGEDUCATION.COM, , INCOME TAX, (Volume – 2), , SET OFF AND CARRY FORWARD OF LOSSES, 15-38, PROVISIONS FOR FILING OF RETURN OF, INCOME, 39-61, INCOME UNDER THE HEAD CAPITAL GAINS, 62-137, INCOME UNDER THE HEAD, BUSINESS/PROFESSION, 138-268, INCOME UNDER THE HEAD SALARY 269-372, Including, EXAMINATION QUESTIONS, After the book has been published, some error/mistake etc. may be detected/or there, may be some amendments etc., all such corrections/amendments shall be uploaded at, the end of the book given on our website. Students are requested to visit our website in, order to update the book, th, , 47, , Edition, , CA (INTER), , MAY 2022/ NOV-2022, P.Y. 2021-22, A.Y. 2022-23, F.A. – 2021, , Author, This Book is the result of combined efforts of, Chartered Accountants/ company executives /, other professionals / feedback of our thousands of students, ₹750
Page 522 :
3, PAPER – 4: TAXATION, (One paper ─ Three hours –100 Marks), Objective: To develop an understanding of the provisions of income-tax law and goods and services tax law, and to acquire the ability to apply such knowledge to make computations and address basic application, oriented issues., SECTION A: INCOME TAX LAW (60 MARKS), Contents:, 1. Basic Concepts, (i) Income-tax law: An introduction, (ii) Important definitions in the Income-tax Act, 1961, (iii) Concept of previous year and assessment year, (iv) Basis of Charge and Rates of Tax, 2. Residential status and scope of total income, (i) Residential status, (ii) Scope of total income, 3. Incomes which do not form part of total income (other than charitable trusts and institutions,, political parties and electoral trusts), (i) Incomes not included in total income, (ii) Tax holiday for newly established units in Special Economic Zones, 4. Heads of income and the provisions governing computation of income under different heads, (i) Salaries, (ii) Income from house property, (iii) Profits and gains of business or profession, (iv) Capital gains, (v) Income from other sources, 5. Income of other persons included in assessee's total income, (i) Clubbing of income: An introduction, (ii) Transfer of income without transfer of assets, (iii) Income arising from revocable transfer of assets, (iv) Clubbing of income of income arising to spouse, minor child and son’s wife in certain cases, (v) Conversion of self-acquired property into property of HUF, 6. Aggregation of income; Set-off, or carry forward and set-off of losses, (i) Aggregation of income, (ii) Concept of set-off and carry forward and set-off of losses, (iii) Provisions governing set-off and carry forward and set-off of losses under different heads of income, (iv) Order of set-off of losses, 7. Deductions from gross total income, (i) General provisions, (ii) Deductions in respect of certain payments, (iii) Specific deductions in respect of certain income, (iv) Deductions in respect of other income, (v) Other deductions, 8. Computation of total income and tax liability of individuals, (i) Income to be considered while computing total income of individuals, (ii) Procedure for computation of total income and tax liability of individuals, 9. Advance tax, tax deduction at source and introduction to tax collection at source, (i) Introduction, (ii) Direct Payment, (iii) Provisions concerning deduction of tax at source, (iv) Advance payment of tax
Page 523 :
4, (v) Interest for defaults in payment of advance tax and deferment of advance tax, (vi) Tax collection at source – Basic concept, (vii) Tax deduction and collection account number, 10. Provisions for filing return of income and self-assessment, (i) Return of Income, (ii) Compulsory filing of return of income, (iii) Fee and Interest for default in furnishing return of income, (iv) Return of loss, (v) Provisions relating to belated return, revised return etc., (vi) Permanent account number, (vii) Persons authorized to verify return of income, (viii) Self-assessment, SECTION B – INDIRECT TAXES (40 MARKS), Contents:, 1. Concept of indirect taxes, (i) Concept and features of indirect taxes, (ii) Principal indirect taxes, 2. Goods and Services Tax (GST) Laws, (i) GST Laws: An introduction including Constitutional aspects, (ii) Levy and collection of CGST and IGST, a) Application of CGST/IGST law, b) Concept of supply including composite and mixed supplies, c) Charge of tax, d) Exemption from tax, e) Composition levy, (iii) Basic concepts of time and value of supply, (iv) Input tax credit, (v) Computation of GST liability, (vi) Registration, (vii) Tax invoice; Credit and Debit Notes; Electronic way bill, (viii) Returns, (ix) Payment of tax including reverse charge, Note – If any new legislation(s) is enacted in place of an existing legislation(s), the syllabus will accordingly, include the corresponding provisions of such new legislation(s) in place of the existing legislation(s) with, effect from the date to be notified by the Institute. Similarly, if any existing legislation ceases to have effect,, the syllabus will accordingly exclude such legislation with effect from the date to be notified by the Institute., Students shall not be examined with reference to any particular State GST Law., Consequential/corresponding amendments made in the provisions of the Income-tax law and Goods and, Services Tax laws covered in the syllabus of this paper which arise out of the amendments made in the, provisions not covered in the syllabus will not form part of the syllabus. Further, the specific, inclusions/exclusions in the various topics covered in the syllabus will be effected every year by way of, Study Guidelines.
Page 524 :
5, , WEIGHTAGE, Intermediate Course Paper 4: Taxation (100 Marks), Section A: Income-tax Law (60 Marks), I. (5%-10%), 1. Basic Concepts, (i) Income-tax law: An introduction, (ii) Important definitions in the Income-tax Act, 1961, (iii) Concept of previous year and assessment year, (iv) Basis of Charge and Rates of Tax, II. (10%-15%), 2. Residential status and scope of total income, (i) Residential status, (ii) Scope of total income, III. (25%-30%), 3. Incomes which do not form part of total income (other than charitable trusts and institutions,, political parties and electoral trusts), (i) Incomes not included in total income, (ii) Tax holiday for newly established units in Special Economic Zones, 4. Heads of income and the provisions governing computation of income under different heads, (i) Salaries, (ii) Income from house property, (iii) Profits and gains of business or profession, (iv) Capital gains, (v) Income from other sources, IV (15%-20%), 5. Income of other persons included in assessee's total income, (i) Clubbing of income: An introduction, (ii) Transfer of income without transfer of assets, (iii) Income arising from revocable transfer of assets, (iv) Clubbing of income of income arising to spouse, minor child and son’s wife in certain cases, (v) Conversion of self-acquired property into property of HUF, 6. Aggregation of income; Set-off, or carry forward and set-off of losses, (i) Aggregation of income, (ii) Concept of set-off and carry forward and set-off of losses, (iii) Provisions governing set-off and carry forward and set-off of losses under different heads of income, (iv) Order of set-off of losses, 7. Deductions from gross total income, (i) General provisions, (ii) Deductions in respect of certain payments, (iii) Specific deductions in respect of certain income, (iv) Deductions in respect of other incomes, (v) Other deductions, V. (20%-25%), 8. Computation of total income and tax liability of individuals, (i) Income to be considered while computing total income of individuals, (ii) Procedure for computation of total income and tax liability of individuals
Page 525 :
6, VI. (10%-15%), 9. Advance tax, tax deduction at source, (i) Introduction, (ii) Direct Payment, (iii) Provisions concerning deduction of tax at source, (iv) Advance payment of tax, (v) Interest for defaults in payment of advance tax and deferment of advance tax, 10 Introduction to tax collection at source, (i) Tax collection at source – Basic concept, (ii) Tax deduction and collection account number, 11. Provisions for filing return of income and self-assessment, (i) Return of Income, (ii) Compulsory filing of return of income, (iii) Fee and Interest for default in furnishing return of income, (iv) Return of loss, (v) Provisions relating to belated return, revised return etc., (vi) Permanent account number, (vii) Persons authorized to verify return of income, (viii) Self-assessment, Section B: Indirect Taxes (40 Marks), I (30%-50% ), 1. Levy and collection of CGST and IGST, (a) Application of CGST/IGST law, (b) Concept of supply including composite and mixed supplies, (c) Charge of tax (including reverse charge), (d) Exemption from tax, (e) Composition levy, 2. Basic concepts of time and value of supply, 3. Input tax credit, II (20%-30%), 1. Computation of GST liability, III (25%-40%), 1. Registration, 2. Tax invoice; Credit and Debit Notes; Electronic way bill, 3. Returns, 4. Payment of tax, IV (0 - 5%), 1. Concept of indirect taxes - Concept and features of indirect taxes; Principal indirect taxes, 2. GST Laws: An introduction including Constitutional aspects
Page 526 :
7, , ETI AGARWAL, ALL INDIA TOPPER OF CA-IPC (NOV-13), ROLL NO. - 366539, MARKS IN TAXATION:89%, , (HIGHEST MARKS IN TAXATION ALL OVER INDIA), (AGGREGATE MARKS 79.71%), , (FEEDBACK), A man for whom teaching is neither a business nor a profession, rather a passion for doing good,, great and unique in the field of teaching is none other than MK Gupta Sir., Sir"s unmatchable style of teaching coupled with his patience and calmness in dealing with students, is simply excellent., The structure of learning pattern, regular mock tests, motivational cash prizes and student friendly, study material covering practical illustrations, past year questions and bare act.. all contributed to, making this journey easy and building up the confidence needed for IPCC., Moreover, the vast knowledge and experience of the faculty assisted in making the concepts crystal, clear and handling each n every doubt of students., The administration and management stands second to none., MK GUPTA classes is a place which can change the word impossible 2 I M POSSIBLE. It made, me a better person both personally n professionally., I think 4 success 4 elements are necessary-desire, dedication, direction and discipline...and all the 4 i, got from Sir.., THANK YOU so much Sir.., In the end i would just like to say MK GUPTA SIR NOT ONLY MAKES CA. HE MAKES, HUMANS!!, , ETI AGARWAL
Page 527 :
8, , AKSHAY JAIN, ALL INDIA TOPPER OF CA-IPC (NOV-13), ROLL NO.- 368162, MARKS IN TAXATION : 87%, (SECOND HIGHEST MARKS IN TAXATION ALL OVER INDIA), , (AGGREGATE MARKS 79.71%), , (FEEDBACK), Experience of those four months with M.K. GUPTA SIR was out of the world., As a teacher, M.K. GUPTA SIR is just like a sea of knowledge & you get each and everything from, very beginning to end from him., Sir is really a nice person. He is very motivational and his words of motivation can influence, anybody to work hard & make their parents proud., M.K. GUPTA CA EDUCATION is the only place where the provisions of tax laws are combined, with the practical knowledge. Study material provided is excellent and it contains numerous, problems covering all aspects and such type of problems are not available anywhere. Sir is not, giving any home work rather home work is done in the class itself and students are invited to solve, the problem before the entire class., Be honest towards your studies & Sir will show you the way of success. The way, Sir is making, students ready for the professional world is praiseworthy. Exposure given by sir to face interview of, Big four CA Firms is excellent., The test Series conducted by the Sir in all the subjects of IPC is very nice Scheme to score such, good marks and exam are conducted in the similar manner as it is conducted by ICAI., I would like to express my gratitude to Sir because it was only his efforts that helped me reach this, position., Sir its your Success., A Message to all : “COME & HAVE A TIME THAT YOU WILL CHERISH THROUGHOUT YOUR LIFE”., , AKSHAY JAIN
Page 528 :
9, , VIJENDER AGGARWAL, ALL INDIA TOPPER OF CA-IPCC (NOV-10), ROLL NO. - 174639, MARKS IN TAXATION:92%, , (HIGHEST MARKS IN TAXATION ALL OVER INDIA), (AGGREGATE MARKS 83.71%), , (FEEDBACK), A person who possesses such vast knowledge in the field of taxation, that we people can only dream, of, is none other than M. K. Gupta Sir., He possesses the rare ability to teach this procedural subject with utmost ease, enabling his students, to grasp all the provisions without any confusion., The quality of study material provided is such that a good study of it helped me score 92 marks. The, variety and complexity of practical problems covered in the books are not available anywhere else., One can find many places where taxation is being taught but it is hardly possible to find a better, place where tax laws are combined with their practical applicability to ensure that all concepts are, crystal clear., Sir is extremely generous. Money-making doesn’t appear to be his priority and it is clearly reflected, in his classes, where the infrastructure and administration stands second to none and students are, awarded handsome cash-prizes not only in classes but also in tests, which are regularly conducted., Thanking Sir for all what he has done would be an insult since it was only his efforts that helped me, reach this position. Sir, its your success. The relationship between us started in CPT only and, continued in IPCC and I hope it will continue forever., , VIJENDER AGGARWAL
Page 529 :
10, , PRACHI JAIN, ALL INDIA TOPPER OF CA-PCC (MAY-10), ROLL NO. - 66312, MARKS IN TAXATION:88%, , (HIGHEST MARKS IN TAXATION ALL OVER INDIA), (AGGREGATE MARKS 77.67%), , (FEEDBACK), M. K. Gupta Sir is an outstanding teacher. He is not only a good teacher but a good person by heart., His way of teaching is excellent. There are many provisions in tax but Sir repeats every provision, atleast two times. This helps in understanding those provisions easily., His books are very good. Everything from theory to PRACTICAL ILLUSTRATION,, EXAMINATION QUESTIONS and BARE ACT is covered in his books., Sir’s staff and management is also very good. Everything is handled in a systematic manner and on, time. Overall it was a good experience., Thanks Sir !! :-, , PRACHI JAIN
Page 530 :
11, , FEEDBACK, (CA-Intermediate), NO OTHER TEACHER OF TAXATION IN INDIA HAS BETTER RESULT THAN OURS, , OPINION OF OUR STUDENTS, 1. AKSHAY (Roll No. 685445) (Total Marks- 607) AIR-6 (Taxation-80), Firstly, I want to thanks M.K. Gupta Sir for teaching me the tax in the easiest manner many student can, think. Tax is very hard subject but the way Sir teach the Tax is excellent. I also want to say one more thing, that the staff of MKG is very supporting. They listen to the students problem very patiently and help them to, solve their problem. At last, I want to say that no one in this CA field can teach so nicely that MK Gupta Sir, teaches., Thank You, 2. DEEPANSHU GOYAL (Roll No. 625914) (Total Marks- 570) AIR-17 (Taxation-75), MK Gupta sir is powerhouse of knowledge. I can’t thank him enough for providing me with vast exposure, about taxation. His mock tests are so good that I was eager to attempt them. He is proactive in solving, doubts. This subject became so light & interesting that I started gaining knowledge in it. It was my best, experience with MKG with rank in both foundation & intermediate. Recorded classes was very helpful for, me, I used to reach at 6:30 am and continued till 11 am and studied taxation. It was a beautiful journey, altogether, and with 10 views, you can view the lectures many times. It helped me revise many concepts. It, became my interest rather than burden. I was so excited to give mock test so that I can build my confidence., 3. SHIVAM MISHRA (Roll No. 624937) (Total Marks- 560) AIR-20 (Taxation-83), MK Gupta sir has very unique style of teaching. He teaches every concept very clearly and correlates every, provision with practical life. Taxation is very vast subject you cannot learn every provision rather you can, understand them. Talking about study material, it covers all types of Question. You do not need to refer, study material as it is incorporated in sir’s books. I would recommend every one to join MK Gupta CA, Education., 4. PRASHANT YADAV (Roll No.354233), 92 Marks, M.K. Gupta Sir is an outstanding teacher. He possesses very vast knowledge of taxation. Sir repeats every, concept atleast three times which makes all concepts crystal clear. Study material provided is very good, it, covers everything from illustration to examination problem and from theory to Bare Act. Staff and, infrastructure facilities of MKG Classes is incomparable. Thank you Sir for your love and support., 5. MOHIT SHARMA (Roll No.353392), 89 Marks, A brilliant personality in my life who has motivated the student to a good path. He is very different from, others. Sir concentrates not only on the Marks but also on the overall development of the student., I am truly glad that I studied from Sir. He taught me how to compete in life. Every student get very good, marks with a little effort, if he is a student of M.K. Gupta CA Education., 6. MANISHA BHAMBRI (Roll No.456626), 89 Marks, M.K. Gupta Sir is the best teacher I have ever met. His study material being the best helped me a lot in my, exams. He is the most sincere teacher who never waste a single moment and gives his best towards his, profession. He teaches not only the theoretical portion but the practical approach too. He teaches us how to, be a good human being and how to live life happily. Thank you Sir for your support every time I needed.
Page 531 :
12, 7. ANISH SHRESTHA (Roll No.344028), 88 Marks, M.K. Gupta Sir is a very excellent teacher. The way he is dedicated towards teaching make us to be, dedicated towards our study. Every concept and every doubt of taxation whichever do I had, he has make, clear. The best thing about Sir is, he use to revise the concept more than 3 times which makes student very, easy for preparing their exam., You will have all the sufficient material for study and lots of questions with answers for practice a, systematically designed materials., Thank a lot to Sir for being so much helpful and lot of love., 8. KAPIL KHANNA (Roll No.341539), 85 Marks, Sir ‘M.K. Gupta’ is the best teacher for Taxation. I feel fortunate to be his student, the amount of knowledge, he imparts is fantastic and uncomparable. He is a person who burns himself up like a candle to light the path, of his dearest students to the road of success. I wish Sir teaches all the subjects of IPCC, since he is simply, the best. Thank you for everything Sir. It you and only you who can guide students like us to reach the, zenith., 9. JITENDRA (Roll No.337780), 85 Marks, Before joining CA, I was so much scary about the “Taxation” but after joining M.K. Gupta CA Education, for taking taxation class my scary converted into my strength now. This is just because of Sir’s knowledge, & teaching style with practicality. Study material provided by Sir is also awesome for study., 10. PUNEET WASAN (Roll No.368537), 84 Marks, M.K. Gupta Sir has a vast knowledge in the subject. The topics taken in the class are very well planned., I found the book really very good. Infact, I practiced all the previous attempts questions of each chapter and, every small question was covered in the chapter. I recommend the students to be thorough with book and, one will score undoubtedly high marks in tax. All the best!! Thank you so much Sir., 11. ISHA MALIK (Roll No.339842), 84 Marks, I do not have words to express my greatfulness for M.K. Gupta Sir. He really possesses vast knowledge and, rich experience in taxation. Study material provided by Sir is also very good which covers everything for, getting through the exam. There is no doubt that due to excellent coaching given by Sir, I have been able to, secure good marks. I pray to God for his long, happy and prospective life. I wish him to continue give, coaching to the prospective students for a longer period. I appeal to all the students who qualify CPT to take, coaching from Gupta Sir for getting sure success. Thank you Sir., 12. RUPAL GARG(Roll No.393844), 84 Marks, M.K. Gupta Sir is, as I believe, the best teacher for Taxation. He is so knowledgeable that I was totally awe, inspired by him. Every day in the class was exciting as he explains everything with real examples and full, depth. The books are superb with lots of practical questions. Thank you Sir., 13. AKANSHA GOEL (Roll No.336693), 84 Marks, It was a great experience studying from M.K. Gupta Sir. He has a vast pool of the knowledge of the subject., The book is a comprehensive one too., 14. PRASIT SHARMA (Roll No.344702), 84 Marks, Taking about the coaching, the teaching style of M.K. Gupta Sir is too much excellent. He has good dealing, with student in every situation. If anyone asked about the taxes coaching, I prefer M.K. Gupta Sir because he, is the one & only best teacher in Taxation.
Page 532 :
13, 15. MANSI BAJAJ (Roll No.354329), 83 Marks, Sir teaches so well and clarifies all our queries. He makes us understand the whole concept very clearly. He, is an amazing teacher and the best teacher in the field of Taxation., 16. RASHI GUPTA (Roll No.337864), 83 Marks, M.K. Gupta Sir is a very friendly and helping teacher. He always answered my queries well. His coaching, classes are very knowledgeable and books are also very good., 17. HARSH AGARWAL (Roll No.491097) 83 Marks, M.K. Gupta Sir is a good teacher. He teaches all aspects of Taxation whether it is practical knowledge or, theoretical knowledge. He teaches every point for 2-3 times and it gets learn in class only. His practical, knowledge about the subject is very good., 18. RAHUL ARORA (Roll No.337403), 83 Marks, M.K. Gupta Sir is a great mentor. Sir has excellent knowledge about the subject. He makes every concept, crystal clear. Every concept is explained atleast twice in the class. He connect every topic with practical life., Study material is excellent. Bare Act is covered in the study material. Three months experience with M.K., Gupta Sir is memorable moments of my life. Thank you Sir, for your guidance and encouragement., 19. SEJAL MEHTA (Roll No.353096), 83 Marks, Coaching for Taxation was an enriching experience in terms of the conceptual clarity which I gained on, each and every topic. Learning tax became so easy with the simplified notes provided., Also, the kind of knowledge that Sir shares with the students is very commendable and useful in, understanding the practical aspects of Taxation. Attending the coaching is worth the time spent., 20. ANU SETHI (Roll No.353491), 83 Marks, I have never seen teacher like M.K. Gupta Sir. His way to teaching, knowledge and experience is awesome, i.e. brilliant. Overall regards for such marks is only M.K. Gupta Sir., 21. ASHISH GUPTA (Roll No.353575), 82 Marks, M.K. Gupta Sir is a very good teacher and he has a very vast knowledge of taxation. He gives his best to, every student in a class. The atmosphere of the class when he was teaching in a class is very awesome. I am, giving all my credit to M.K. Gupta Sir for securing marks in Taxation., 22. RAGHAV GUPTA (Roll No.491122), 82 Marks, M.K. Gupta Sir is an outstanding teacher. He possesses a very vast knowledge about the subject. His way of, teaching is fabulous. Every concept is explained with help of an example. Study material is all exhaustive, that he provides. Also, queries are taken up promptly. Thank you Sir for your guidance., 23. SHREYA MALIK (Roll No.340228), 82 Marks, M.K. Gupta Sir is the best teacher I have ever come across. His level of knowledge is tremendous. The way, he teaches, with so much patience and willingness, keeps every student motivated. The marks I have scored, in tax is all because of him. Thank you so much Sir. I am a student of video class and I have never met Sir in, person. I would be grateful if I would be given a chance to meet him in person., 24. PRABHAW KUMAR AGARWALLA (Roll No.369428) 82 Marks, Teaching was excellent and queries handled were excellent. Teaching methodology was really excellent and, helped a lot to me.
Page 533 :
14, 25. PRABHAT RANJAN (Roll No.347926), 81 Marks, M.K. Gupta Sir has a very deep knowledge about the subject and his practical approach towards the subject., Sir repeats every provision atleast twice. This helps in understanding those provision easily., The books notes and all the management is done very properly and in a smooth manner. All in all the best, way to study tax., 26. ANKIT KHEMKA (Roll No.338055), 81 Marks, M.K. Gupta Sir is excellent teacher of Tax. He repeats the provision two to three times and doubts are also, taken by the faculty. His books are also very good. Bare Act is covered in his books for more understanding, about the Act. Sir also provide regular test and prize also given by him motivates the student to work hard., Environment provided by M.K. Gupta Classes is also very good to study., 27. ARTI SRIVASTAVA (Roll No.347859), 80 Marks, Sir’s unmatchable style of teaching. Regular mock test, also help in to achieve good marks in Taxation. Sir’s, books contain illustration. Past year question also help to achiever to good marks. Sir’s build confidence in, every student to achieve success in life. Thank you, so much Sir., 28. SHIVANGI GUPTA (Roll No.337956), 80 Marks, M.K. Gupta Sir is an amazing teacher. The tax subject is all about provisions so many sections but Sir makes, it simpler for us out of all the subjects, I found Taxation to be the most interesting one., Sir’s study material and notes are sufficient. Study material covers all the past year exam questions, practice, questions with solutions. His practical experiences help our understanding level to reach new heights. Thank, you Sir for everything.
Page 534 :
Set Off and Carry Forward Of Losses, , 15, , SET OFF AND CARRY FORWARD OF, LOSSES, SECTION 70 TO 80, PARTICULARS, SECTIONS, Set off of loss from one source against income from another source under the same, 70, head of income, Set off of loss from one head against income from another head, 71, Carry forward and set off of loss from house property, 71B, Carry forward and set off of business losses, 72, Losses in speculation business, 73, Losses under the head “Capital gains”, 74, Losses from certain specified sources under the head “Income from other sources”, 74A, Question 1 [V. Imp.]: Write a note on Set Off and Carry Forward of Losses under the head House, Property., Answer: As per section 70, if any person has loss from any house property, such loss can be set off from, income of any other house property and it is called inter-source adjustment or intra-head adjustment. E.g., Mr. X has two houses: there is loss of ₹34,000 from one house and income of ₹80,000 from the other house,, in this case, loss of one source (house) can be set off from income of the other source (house)., As per section 71, unadjusted loss can be set off from incomes of other heads but as per section 58(4), such, loss can not be set off from casual income and it is called inter-head adjustment. E.g. Mr. X has loss from, house property ₹2,00,000 and income from business/profession ₹5,00,000, in this case, loss is allowed to be, set off but if he has any casual income, loss can not be set off from casual income., Where in respect of any assessment year, the net result of the computation under the head “Income from, house property” is a loss and the assessee has income assessable under any other head of income, the, assessee shall not be entitled to set off such loss, to the extent the amount of the loss exceeds ₹2,00,000,, against income under the other head.’., As per section 71B, unadjusted loss is allowed to be carried forward to the subsequent years but for a, maximum period of 8 years starting from the year subsequent to the year in which the loss was incurred and, in the subsequent years, loss can be set off only from income under the head house property. E.g. Mr. X has, incurred loss under the head house property in the previous year 2021-22/assessment year 2022-23 and it, could not be set off in the same year, it can be carried forward upto previous year 2029-30/assessment year, 2030-31 (as shown below), Year 1, Previous year 2022-23, Assessment Year 2023-24, Year 2, Previous year 2023-24, Assessment Year 2024-25, Year 3, Previous year 2024-25, Assessment Year 2025-26, Year 4, Previous year 2025-26, Assessment Year 2026-27, Year 5, Previous year 2026-27, Assessment Year 2027-28, Year 6, Previous year 2027-28, Assessment Year 2028-29, Year 7, Previous year 2028-29, Assessment Year 2029-30, Year 8, Previous year 2029-30, Assessment Year 2030-31, E.g. Mr. X has loss under the head house property of the previous year 2013-14/assessment year 2014-15, ₹5,00,000 and income under the head house property ₹5,00,000 in previous year 2021-22/assessment year, 2022-23, in this case, loss shall be allowed to be set off because it will be allowed to be carried forward upto, a period of 8 years starting from previous year 2014-15/assessment year 2015-16 and is as shown below:, Year 1, Previous year 2014-15, Assessment Year 2015-16, Year 2, Previous year 2015-16, Assessment Year 2016-17
Page 535 :
Set Off and Carry Forward Of Losses, Year 3, Year 4, Year 5, Year 6, Year 7, Year 8, , Previous year 2016-17, Previous year 2017-18, Previous year 2018-19, Previous year 2019-20, Previous year 2020-21, Previous year 2021-22, , 16, , Assessment Year 2017-18, Assessment Year 2018-19, Assessment Year 2019-20, Assessment Year 2020-21, Assessment Year 2021-22, Assessment Year 2022-23, , If the loss can be set off, it has to be set off compulsorily i.e. it is not voluntary. E.g. Mr. X has loss under, the head house property ₹2,00,000 in previous year 2021-22/assessment year 2022-23 and income under the, head business/profession ₹2,50,000 in the same year, in this case loss has to be set off., Question 2 [Imp.]: Write a note on Set Off and Carry Forward of Losses under the head Business/, Profession., Answer: As per section 70, if any person has loss from any business/profession, such loss can be set off, from income of any other business/profession and it is called inter-source adjustment or intra-head, adjustment. E.g. Mr. X has two business: there is loss of ₹5,00,000 from one business and income of, ₹10,00,000 from the other business, in this case, loss of one source (business) can be set off from income of, the other source (business)., As per section 71, unadjusted loss can be set off from incomes of other heads except salary but as per, section 58(4), such loss can not be set off from casual income and it is called inter-head adjustment. E.g. Mr., X has loss from business/profession ₹3,00,000 and income from house property ₹5,00,000, in this case, loss, is allowed to be set off but if he has any casual income, loss can not be set off from casual income. Similarly, it can not be set off from income under the head salary., As per section 72, unadjusted loss is allowed to be carried forward to the subsequent years but for a, maximum period of 8 years starting from the year subsequent to the year in which the loss was incurred and, in the subsequent years, loss can be set off only from income under the head business/profession. E.g. Mr. X, has incurred loss under the head business/profession in the previous year 2021-22/assessment year 2022-23, and it could not be set off in the same year, it can be carried forward upto previous year 2029-30/assessment, year 2030-31 (as shown below), Year 1, Previous year 2022-23, Assessment Year 2023-24, Year 2, Previous year 2023-24, Assessment Year 2024-25, Year 3, Previous year 2024-25, Assessment Year 2025-26, Year 4, Previous year 2025-26, Assessment Year 2026-27, Year 5, Previous year 2026-27, Assessment Year 2027-28, Year 6, Previous year 2027-28, Assessment Year 2028-29, Year 7, Previous year 2028-29, Assessment Year 2029-30, Year 8, Previous year 2029-30, Assessment Year 2030-31, Loss of the current year shall be adjusted first and only after that brought forward loss can be adjusted. E.g., Mr. X has loss under the head house property ₹2,00,000 and income from business/profession ₹2,00,000 in, previous year 2021-22 and also brought forward loss of business/profession ₹2,00,000 of previous year, 2013-14, in this case, loss of house property shall be adjusted from income of business/profession and loss, of previous year 2013-14 shall lapse., Illustration 1: Mr. X has loss under the head House Property ₹5,00,000 and income under the head Salary, ₹3,00,000 and income under the head Business/Profession ₹10,00,000 and LTCG ₹6,00,000 and Casual, income ₹2,00,000 and unadjusted loss of Business/Profession ₹4,00,000 of Assessment Year 2017-18 and, loss of Business/Profession ₹3,00,000 of Assessment Year 2018-19., Compute his tax liability for assessment year 2022-23., Solution:, ₹, The assessee has the option to adjust loss of house property from normal income or from LTCG., Option I, Loss of house property is set off from normal income
Page 536 :
Set Off and Carry Forward Of Losses, Income under the head Salary, Less: loss of house property, Income under the head Salary, Income under the head Business/Profession, Less: Brought forward business loss of A.Y. 2017-18, Less: Brought forward business loss of A.Y. 2018-19, Income under the head Business/Profession, Long term capital gain, Casual Income, Gross Total Income, Less: Deduction u/s 80C to 80U, Total Income, Computation of Tax Liability, Tax on LTCG ₹6,00,000 @ 20%, Tax on Casual income ₹2,00,000 @ 30%, Tax on Normal income ₹4,00,000 at slab rate, Tax before health & education cess, Add: HEC @ 4%, Tax Liability, Option II, Loss of house property is set off from LTCG, Income under the head Salary, Income under the head Business/Profession, Less: Brought forward business loss of A.Y. 2017-18, Less: Brought forward business loss of A.Y. 2018-19, Income under the head Business/Profession, Long term capital gain, Less: loss of house property, Long term capital gain, Casual Income, Gross Total Income, Less: Deduction u/s 80C to 80U, Total Income, Computation of Tax Liability, Tax on LTCG ₹4,00,000 @ 20%, Tax on Casual income ₹2,00,000 @ 30%, Tax on Normal income ₹6,00,000 at slab rate, Tax before health & education cess, Add: HEC @ 4%, Tax Liability, Option II is better., Illustration 2: Mrs. X has income and losses as given below:, 1. Income under the head Salary, 2. Loss under the head House Property, 3. Income under the head Business/Profession, 4. Income from STCG, 5. Income from STCG u/s 111A, 6. Casual Income, Deduction under section 80C to 80U, Brought forward Business/Profession loss for, - Previous year 2012-13, - Previous year 2014-15, , 17, 3,00,000.00, (2,00,000.00), 1,00,000.00, 10,00,000.00, (4,00,000.00), (3,00,000.00), 3,00,000.00, 6,00,000.00, 2,00,000.00, 12,00,000.00, Nil, 12,00,000.00, 1,20,000.00, 60,000.00, 7,500.00, 1,87,500.00, 7,500.00, 1,95,000.00, 3,00,000.00, 10,00,000.00, (4,00,000.00), (3,00,000.00), 3,00,000.00, 6,00,000.00, (2,00,000.00), 4,00,000.00, 2,00,000.00, 12,00,000.00, Nil, 12,00,000.00, 80,000.00, 60,000.00, 32,500.00, 1,72,500.00, 6,900.00, 1,79,400.00, , ₹ 5,00,000, ₹10,00,000, ₹12,00,000, ₹ 2,00,000, ₹10,00,000, ₹ 3,00,000, ₹ 1,00,000, ₹ 3,00,000, ₹ 6,00,000
Page 537 :
Set Off and Carry Forward Of Losses, - Previous year 2015-16, Compute tax liability of Mrs. X for Assessment Year 2022-23., Solution:, Option I, Loss of house property is set off from normal income, Income under the head Salary, Less: Loss of house property, Income under the head Salary, Income under the head Business/Profession, Less: Brought forward business/profession loss P.Y. 2014-15, Less: Brought forward business/profession loss P.Y. 2015-16, Income under the head Business/Profession, Short term capital gain, Short term capital gain u/s 111A, Casual Income, Gross Total Income, Less: Deduction u/s 80C to 80U, Total Income, Computation of Tax Liability, Tax on STCG u/s 111A ₹10,00,000 @ 15%, Tax on Casual income ₹3,00,000 @ 30%, Tax on Normal income ₹7,00,000 at slab rate, Tax before health & education cess, Add: HEC @ 4%, Tax Liability, Option II, Loss of house property is set off from STCG u/s 111A, Income under the head Salary, Income under the head Business/Profession, Less: Brought forward business/profession loss P.Y. 2014-15, Less: Brought forward business/profession loss P.Y. 2015-16, Income under the head Business/Profession, Short term capital gain, Short term capital gain u/s 111A, Less: loss of house property, Short term capital gain u/s 111A, Casual Income, Gross Total Income, Less: Deduction u/s 80C to 80U, Total Income, Computation of Tax Liability, Tax on STCG u/s 111A ₹8,00,000 @ 15%, Tax on Casual income ₹3,00,000 @ 30%, Tax on Normal income ₹9,00,000 at slab rate, Tax before health & education cess, Add: HEC @ 4%, Tax Liability, Option I is better., , 18, ₹ 3,00,000, ₹, 5,00,000.00, (2,00,000.00), 3,00,000.00, 12,00,000.00, (6,00,000.00), (3,00,000.00), 3,00,000.00, 2,00,000.00, 10,00,000.00, 3,00,000.00, 21,00,000.00, (1,00,000.00), 20,00,000.00, 1,50,000.00, 90,000.00, 52,500.00, 2,92,500.00, 11,700.00, 3,04,200.00, 5,00,000.00, 12,00,000.00, (6,00,000.00), (3,00,000.00), 3,00,000.00, 2,00,000.00, 10,00,000.00, (2,00,000.00), 8,00,000.00, 3,00,000.00, 21,00,000.00, (1,00,000.00), 20,00,000.00, 1,20,000.00, 90,000.00, 92,500.00, 3,02,500.00, 12,100.00, 3,14,600.00, , Question 3 [V. Imp.]: Write a note on Unabsorbed Depreciation., Answer: Unabsorbed Depreciation Section 32(2), An assessee having business or profession shall debit all expenditures of business/profession before debiting, depreciation i.e. depreciation shall be debited at the end. If there is a loss by debiting other expenditure, it
Page 538 :
Set Off and Carry Forward Of Losses, , 19, , will be called loss under the head business/profession. Depreciation shall be debited only if income is, available under the head business/profession and the depreciation which can not be debited shall be called, unabsorbed depreciation and it will be allowed to be adjusted from any income under any head except casual, income and salary., If it can not be adjusted in the same year, its carry forward is allowed for unlimited period and in the, subsequent years, it can be set off from any income under any head except salary income and casual income., If any assessee has brought forward business loss as well as depreciation, business loss shall be adjusted first, and depreciation afterwards., E.g. Mr. X has income under the head business/profession ₹10,00,000 after debiting all expenditures except, depreciation of ₹13,00,000, in this case, depreciation of only ₹10,00,000 can be debited to the profit and loss, account and balance ₹3,00,000 shall be called unabsorbed depreciation and it can be set off from any income, under any head except salary income and casual income and even in the subsequent years, it can be set off, from any income under any head except salary income and casual income and such carry forward is allowed, for unlimited periods., The sequence of claiming losses and depreciation under the head business / profession shall be as, given below:, 1. Current year expenses, 2. Current year depreciation, 3. Brought forward business losses, 4. Brought forward unabsorbed depreciation, Question 4 [V. Imp.]: Write a note on Set Off and Carry Forward of loss from Speculative Business., Answer: As per section 28, income from speculative business shall be taxable under the head, business/profession and such income shall be computed in the normal manner and shall be taxable at the, normal rate i.e. it will not be considered to be causal income., Meaning of speculative business Section 43(5), "speculative transaction" means a transaction in which a contract for the purchase or sale of any commodity,, including stocks and shares, is periodically or ultimately settled otherwise than by the actual delivery or, transfer of the commodity or scrips:, e.g. Mr. X entered into a contact for purchase of one plot from Mr. A and same plot was sold by him to Mr., Y at a higher rate and he has directed Mr. Y to pay the amount directly to Mr. A and surplus amount to Mr., X and he directed Mr. A to transfer the plot directly in the name of Mr. Y, it will be called speculative, transaction but if Mr. X has transferred the plot in his name and after that plot was transferred in the name of, Mr. Y, it will be called normal business., The following shall not be deemed to be a speculative transaction:, (a) a contract in respect of raw materials or merchandise entered into by a person in the course of his, manufacturing or merchanting business to guard against loss through future price fluctuations in, respect of his contracts for actual delivery of goods manufactured by him or merchandise sold by him;, or, (b) an eligible transaction in respect of trading in commodity derivatives carried out in a recognised stock, exchange, which is chargeable to commodities transaction tax, however in respect of trading in, agricultural commodity derivatives, the requirement of chargeability of commodity transaction tax, shall not apply., Special Provision with regard to companies, Where any part of the business of a company consists in the purchase and sale of the shares of other, companies, such company shall be deemed to be carrying on speculation business to the extent to which the, business consists of the purchase and sale of such shares. However, this deeming provision does not apply to, the following companies –, (1) A company whose gross total income consists of mainly income chargeable under the heads “Income, from house property”, “Capital gains” and “Income from other sources”;
Page 539 :
Set Off and Carry Forward Of Losses, , 20, , (2) A company, the principal business of which is –, (i) the business of trading in shares; or, (ii) the business of banking; or, (iii) the granting of loans and advances., Accordingly, if these companies carry on the business of purchase and sale of shares of other companies,, they would not be deemed to be carrying on speculation business., Set Off and Carry forward of loss from Speculative Business Section 73, If any assessee has loss from speculative business, such loss can not be set off from any income under any, head however, if the assessee has two or more similar business, loss of one such business can be set off from, the income of other such business., Unadjusted loss is allowed to be carried forward but for a maximum period of 4 years starting from the year, subsequent to the year in which the loss was incurred. Even in the subsequent years, loss can be set off only, from income of speculative business., Loss under the head house property, loss from normal business, unabsorbed depreciation, loss under the, head other sources can be set off from the income of speculative business., e.g. Mr. X has loss from speculative business ₹5,00,000 and income from normal business ₹5,00,000, in this, case, loss is not allowed to be set off however its carry forward is allowed but for a maximum period of 4, years and in the subsequent years its can be set off only from income of speculation business., e.g. Mr. X has loss from house property ₹5,00,000 and income from speculative business ₹5,00,000, in this, case, loss can be set off from income of speculative business maximum upto 2,00,000 and balance 3,00,000, shall be carried forward., e.g. Mr. X has loss of speculative business ₹5,00,000 and income from some other speculative business, ₹5,00,000, in this case, loss can be set off from income of such speculative business., Question 5 [V. Imp.]: Write a note on Set Off and Carry Forward of losses under the head Capital, Gains., Answer: Set off and Carry forward of Loss under the head Capital Gain Section 70, 71 and 74, Set off of loss from one source against income from another source under the same head of income, Section 70, If any person has short term loss, it can be set off either from short term or from long term gain but if any, person has long term loss, it can be set off only from long term gains i.e. set off from short term is not, allowed., Example, Mr. X has long term loss of ₹50,00,000 and short term gain of ₹50,00,000, in this case, long term loss can, not be set off from short term gains but if he has long term gain of ₹50,00,000 and short term loss of, ₹50,00,000, in this case, set off is allowed., e.g. If Mr. X has short term loss under section 111A and has long term gain, such loss can be set off from, long term gain and also it can be set off from short term gain under section 111A or from normal short term, gain., Set off of loss from one head against income from another Section 71, Where in respect of any assessment year, the net result of the computation under the head “Capital gains” is, a loss and the assessee has income assessable under any other head of income, the assessee shall not be, entitled to have such loss set off against income under the other head., Example, Mr. X has short term loss ₹50,00,000 and business income of ₹50,00,000, in this case set off is not allowed., Carry forward of losses under the head “Capital Gains” Section 74, If any assessee has short term loss or long term loss which could not be set off, such losses shall be allowed, to be carried forward but for a maximum period of 8 years starting from the year next to the year in which, the loss was incurred., Brought forward short term loss can be set off either from short term or from long term gain but brought, forward long term loss can be set off only from long term gain and not from short term gain.
Page 540 :
Set Off and Carry Forward Of Losses, , 21, , Question 6: Write a note on Set Off and Carry Forward of Losses under the head “Income from, Other Sources”., Answer: Set off and Carry forward of Losses under the head “Income from Other Sources”, As per section 70, if the assessee has loss under the head other sources, such loss is allowed to be set off, from any income under the same head except casual income., As per section 71, unadjusted loss can be set off even from incomes of other heads., Carry forward of the loss is not allowed., Set Off and Carry Forward of losses in connection with Owning and Maintaining of Race Horses, Section 74A, If any assessee has loss from the business of owning and maintaining race horses, such loss can not be set, off from any income under any head but if the assessee has two or more business of owning and maintaining, race horses, loss of one such business can be set off from the income of other such business., Unadjusted loss is allowed to be carried forward but for a maximum period of 4 years starting from the year, subsequent to the year in which the loss was incurred and even in the subsequent year, the loss can be set off, only from income of business of owning and maintaining race horses., If the assessee has business of owning and maintaining any other race animal, income shall be taxable under, the head business/profession and loss will have the same treatment as in case of loss under the head, business/profession. E.g. If Mr. X has loss from business of owning and maintaining race camels, the loss, will have the same treatment as the loss under the head business/profession., ADDITIONAL POINTS FOR SET-OFF AND CARRY FORWARD, 1. Set off and carry forward is mandatory not voluntary. e.g. Mr. X has loss under the head house property, ₹50,000 and income under the head business/profession ₹50,000 during the assessment year 2022-23, in this, case, loss of house property has to be set off from business income of assessment year 2022-23 and Mr. X is, not allowed to carry forward the loss of house property., 2. Any loss has to be set off first within the same head and after that under some other heads and after that, carry forward is allowed., 3. Loss of current year shall be set off first and only after that brought forward losses can be adjusted, eg., Mr. X has income from one house ₹ 10,00,000 and loss from other house ₹ 10,00,000 in P.Y. 2021-22 and, also unadjusted loss of ₹ 10,00,000 under the head house property of P.Y. 2013-14, in this case loss of, current year is to be adjusted first., 4. Loss from an exempt source cannot be set-off against profits from a taxable source of income, e.g. share, of loss from a partnership firm cannot be set-off by the partner from his income rather it will be set-off only, from income of partnership firm., 5. In general losses incurred by any person are allowed to be set off and carry forward by such person but as, per section 78, if any person has inherited any business or profession, losses of such business or profession, can be carried forward by him., Question 7: Explain setoff and carry forward of loss from agriculture, Answer: Loss from Agriculture, Loss from Agriculture cannot be set off from incomes of other heads. Similarly loss of other heads cannot be, set off from Agriculture Income., Loss from agriculture can be set off only from agricultural income and carry forward is allowed for 8 years, and in subsequent years also it can be set off from agriculture income., Original Text, Set off of loss from one source against income from another source under the same head of income., 70. (1) Save as otherwise provided in this Act, where the net result for any assessment year in respect of any, source falling under any head of income, other than "Capital gains", is a loss, the assessee shall be entitled to, have the amount of such loss set off against his income from any other source under the same head., (2) Where the result of the computation made for any assessment year under sections 48 to 55 in respect of, any short-term capital asset is a loss, the assessee shall be entitled to have the amount of such loss set off, against the income, if any, as arrived at under a similar computation made for the assessment year in respect
Page 541 :
Set Off and Carry Forward Of Losses, , 22, , of any other capital asset., (3) Where the result of the computation made for any assessment year under sections 48 to 55 in respect of, any capital asset (other than a short-term capital asset) is a loss, the assessee shall be entitled to have the, amount of such loss set off against the income, if any, as arrived at under a similar computation made for the, assessment year in respect of any other capital asset not being a short-term capital asset., Set off of loss from one head against income from another., 71. (1) Where in respect of any assessment year the net result of the computation under any head of income,, other than "Capital gains", is a loss and the assessee has no income under the head "Capital gains", he shall,, subject to the provisions of this Chapter, be entitled to have the amount of such loss set off against his, income, if any, assessable for that assessment year under any other head., (2) Where in respect of any assessment year, the net result of the computation under any head of income,, other than "Capital gains", is a loss and the assessee has income assessable under the head "Capital gains",, such loss may, subject to the provisions of this Chapter, be set off against his income, if any, assessable for, that assessment year under any head of income including the head "Capital gains" (whether relating to shortterm capital assets or any other capital assets)., (2A) Notwithstanding anything contained in sub-section (1) or sub-section (2), where in respect of any, assessment year, the net result of the computation under the head "Profits and gains of business or, profession" is a loss and the assessee has income assessable under the head "Salaries", the assessee shall not, be entitled to have such loss set off against such income., (3) Where in respect of any assessment year, the net result of the computation under the head "Capital gains", is a loss and the assessee has income assessable under any other head of income, the assessee shall not be, entitled to have such loss set off against income under the other head., (3A) Notwithstanding anything contained in sub-section (1) or sub-section (2), where in respect of any, assessment year, the net result of the computation under the head "Income from house property" is a loss and, the assessee has income assessable under any other head of income, the assessee shall not be entitled to set, off such loss, to the extent the amount of the loss exceeds two lakh rupees, against income under the other, head., MTP NOV -2020, 4. The following are the details relating to Mr. Rajesh, a resident Indian, relating to the year ended, 31.03.2022, Particulars, Amount (₹), 1,40,000, Short term capital gain, 2,20,000, Loss from house property, 50,000, Loss from speculative business, 20,000, Loss from card games, 86,000, Brought forward Long term capital loss of A.Y. 2017-18, 11,00,000, Dividend from Shaiba Ltd., 96,000, Loss from tea business, Mr. Rajesh’s wife, Isha is employed with Shine Ltd., at a monthly salary of ₹ 25,000, where Mr. Rajesh, holds 21% of the shares of the company. Isha is not adequately qualified for the post held by her in Shine, Ltd., You are required to compute taxable income of Mr. Rajesh for the A.Y. 2022-23. Ascertain the amount of, losses which can be carried forward., (7 Marks), Answer:, Computation of Taxable Income of Mr. Rajesh for the A.Y. 2022-23, Particulars, , ₹, , ₹
Page 542 :
Set Off and Carry Forward Of Losses, Salaries, Isha’s salary (₹ 25,000 x 12) [See Note 1], Less: Standard deduction under section 16(ia) upto ₹50,000, , 23, 3,00,000, (50,000), 2,50,000, , Less: Loss from house property set off against salary income as per section, 71(3A) [See Note 2], Capital Gains, Short term capital gain, Less: Loss from tea business (₹ 96,000 x 40%) [See Note 3 & 4], Income from Other Sources, Dividend income, Taxable Income, , (2,00,000), , 50,000, , 1,40,000, (38,400), , 1,01,600, 11,00,000, 12,51,600, , The following losses can be carried forward for subsequent assessment years:, (i) Loss from house property to be carried forward and set-off against income from house, ₹ 20,000, property, (ii) Long-term capital loss of A.Y. 2017-18 can be carried forward and set-off against, ₹ 86,000, long-term capital gains, (iii) Loss from speculative business to be carried forward and set-off against income from, ₹ 50,000, speculative business, Notes:, (1) As per section 64(1)(ii), all the income which arises directly or indirectly, to the spouse of any, individual by way of salary, commission, fees or any other form of remuneration from a concern in, which such individual has a substantial interest shall be included in the total income of such individual., However, where spouse possesses technical or professional qualification and the income is solely, attributable to the application of such knowledge and experience, clubbing provisions will not apply., Since, Mrs. Isha is not adequately qualified for the post and Mr. Rajesh has substantial interest in Shine, Ltd by holding 21% of the shares of the Shine Ltd., the salary income of Mrs. Isha to be included in Mr., Rajesh’s income., (2) As per section 71(3A), loss from house property can be set off against any other head of income to the, extent of ₹ 2,00,000 only., (3) 60% of the losses from tea business is treated as agricultural income and therefore exempt. Loss from, an exempt source cannot be set off against profits from a taxable source., (4) As per section 71(2A), business loss cannot be set off against salary income. Hence, 40% of the losses, from tea business i.e., ₹ 38,400 set off against short term capital gains., (5) Loss from Card games can neither be set off against any other income, nor can it be carried forward., (6) Loss of ₹50,000 from speculative business can be set-off only against the income from the speculative, business. Hence, such loss has to be carried forward., As per section 74(1), brought forward Long-term capital loss can be set-off only against long-term capital, gain. Such loss can be carried forward for eight assessment years immediately succeeding the assessment, year for which the loss was first computed. Since, 8 assessment years has not expired, such loss can be, carried forward to A.Y. 2023-24 for set-off against long-term capital gains., RTP NOV -2020, 1. Mr. Krishan, residing in Indore, provides the following information for the financial year 2021-22:, Particulars, ₹, Income from textile business, 4,60,000
Page 543 :
Set Off and Carry Forward Of Losses, , 24, , 25,000, Income from speculation business, 12,000, Loss from gambling, 15,000, Loss on maintenance of race horse, 5,000, Current year depreciation of textile business not adjusted in the income given above., 10,000, Unabsorbed depreciation of assessment year 2020-21, 30,000, Speculation business loss of assessment year 2021-22, Compute the Gross Total Income of Mr. Krishan for the Assessment year 2022-23 and also state the losses, eligible for carry forward and period upto which such losses can be carried forward., Answer:, Particulars, Profits and gains of business or profession, Income from Textile business, Less: Current year depreciation allowable under section 32(1), , ₹, 4,60,000, , ₹, , (5,000), 4,55,000, , Less: Unabsorbed depreciation brought forward from A.Y.2020-21 as per section, 32(2), , 4,45,000, (10,000), , Income from speculation business, Current year income from speculation business, 25,000, Less: Speculation business loss for A.Y. 2021-22 set-off as per the provisions of, section 73(2), (30,000), Speculation business loss to be carried forward, (5,000), Gross Total Income, Losses eligible for carry forward to A.Y.2023-24, Particulars, (1) Loss from speculation business to be carried forward as per section 73, Loss from speculation business can be set off only against income from another, speculation business. The remaining loss from speculation business can be carried, forward for a maximum of four assessment years immediately succeeding the assessment, year for which the loss was first computed. Thus, such loss can be carried forward upto, A.Y.2025-26, (2) Loss on maintenance of race horses to be carried forward as per section 74A, Loss on maintenance of race horses can be set-off only against income from the activity, of owning and maintaining race horses. Such loss can be carried forward for a maximum, of four assessment years immediately succeeding the assessment year for which the loss, was first computed. Thus, such loss can be carried forward upto A.Y. 2026-27, (3) Loss from gambling can neither be set-off nor be carried forward., Illustration 3: Mr. A submits the following particulars pertaining to the A.Y.2022-23:, Particulars, Income under the head salary, Loss from self-occupied property, Business loss, Bank interest (FD) received, Compute the taxable income of Mr. A for the A.Y.2022-23., , Nil, 4,45,000, ₹, 5,000, , 15,000, , ₹, 4,00,000, (-) 70,000, (-) 1,00,000, 80,000
Page 544 :
Set Off and Carry Forward Of Losses, Solution:, Particulars, , 25, , Computation of taxable income of Mr. A for the A.Y. 2022-23, , Income under the head Salary, Less: Loss from house property, Income under the head Other Sources, Income from other sources (interest on fixed deposit with bank), Less: Loss from business, Business loss to be carried forward, Gross total income, Less: Deduction under chapter VIA, Taxable income, , Amount, (₹), 4,00,000, (70,000), 3,30,000, 80,000, (1,00,000), (20,000), 3,30,000, Nil, 3,30,000, , Illustration 4: Mr. B, a resident individual, furnishes the following particulars for the P.Y. 2021-22:, Particulars, ₹, Income under the head Salary, 45,000, Loss from house property, (24,000), Loss from business – non-speculative, (22,000), Loss from speculative business, (4,000), Short-term loss, (25,000), Long-term capital gains, 19,000, What is the total income chargeable to tax for the A.Y. 2022-23?, Solution: The total income chargeable to tax for the A.Y. 2022-23 is calculated as under:, Particulars, Amount ₹, Income under the head salaries, 45,000, Less: Loss from house property, (24,000), 21,000, Profits and gains of business and profession, Business loss to be carried forward [Note 1], (22,000), Speculative loss to be carried forward [Note 2], (4,000), Capital Gains, Long term capital gain, 19,000, Less: Short term capital loss, (25,000), Short term capital loss to be carried forward [Note 3], (6,000), Gross Total Income, 21,000, Note 1: Business loss cannot be set-off against salary income. Therefore, loss of ₹ 22,000 from the nonspeculative business cannot be set off against the income from salaries. Hence, such loss has to be carried, forward to the next year for set-off against business profits, if any., Note 2: Loss of ₹ 4,000 from the speculative business can be set off only against the income from the, speculative business. Hence, such loss has to be carried forward., Note 3: Short term capital loss can be set off against both short term capital gain and long term capital gain., Therefore, short term capital loss of ₹ 25,000 can be set-off against long-term capital gains to the extent of, ₹19,000. The balance short term capital loss of ₹ 6,000 cannot be set-off against any other income and has to, be carried forward to the next year for set-off against capital gains, if any., Illustration 5: During the P.Y. 2021-22, Mr. C has the following income and the brought forward losses:, Particulars, ₹, Short term capital gains on sale of shares, 1,50,000
Page 545 :
Set Off and Carry Forward Of Losses, Long term capital loss of A.Y. 2020-21, Short term capital loss of A.Y. 2021-22, Long term capital gain, What is the capital gain taxable in the hands of Mr. C for the A.Y.2022-23?, Solution: The capital gains taxable are as under:, Particulars, Short term capital gains on sale of shares, Less: Brought forward short term capital loss of the A.Y.2021-22, Long term capital gain, Less: Brought forward long term capital loss of A.Y.2020-21 [See Note below], , 26, (96,000), (37,000), 75,000, , ₹, 1,50,000, (37,000), 75,000, (75,000), , ₹, 1,13,000, Nil, , 1,13,000, Taxable short-term capital gains, Note: Long-term capital loss cannot be set off against short-term capital gain. Hence, the unadjusted long, term capital loss of A.Y. 2020-21 of ₹ 21,000 (i.e. ₹96,000 – ₹75,000) has to be carried forward to the next, year to be set-off against long-term capital gains of that year., Illustration 6: Mr. D has the following income for the P.Y. 2021-22 –, Particulars, ₹, 75,000, Income from the activity of owning and maintaining the race horses, 85,000, Income from textile business, 50,000, Brought forward textile business loss, Brought forward loss from the activity of owning and maintaining the race horses (relating to 96,000, A.Y.2019-20), What is the taxable income in the hands of Mr. D for the A.Y. 2022-23?, Solution: The taxable income is calculated as under:, Particulars, ₹, ₹, 75,000, Income from the activity of owning and maintaining race horses, (96,000), Less: Brought forward loss from the activity of owning and maintaining race horses, Loss from the activity of owning and maintaining race horses to be carried forward to (21,000), A.Y.2023-24, 85,000, Income from textile business, (50,000) 35,000, Less: Brought forward business loss from textile business., Taxable business income, 35,000, Note: Loss from the activity of owning and maintaining race horses cannot be set-off against any other, source/head of income., Illustration 7: Mr. E has furnished his details for the A.Y.2022-23 as under:, Particulars, ₹, Income under the head salary, 1,50,000, Income from speculation business, 60,000, Loss from non-speculation business, (40,000), Short term capital gain, 80,000, Long term capital loss of A.Y.2020-21, (30,000), Winning from lotteries, 20,000, What is the taxable income of Mr. E for the A.Y. 2022-23?, Solution:, Computation of taxable income of Mr. E for the A.Y.2022-23, Particulars, ₹, ₹, Income under the head salary, 1,50,000, 60,000, Income from speculation business, Less: Loss from non-speculation business, (40,000), 20,000, Short-term capital gain, 80,000, Winning from lotteries, 20,000, Taxable income, 2,70,000
Page 546 :
Set Off and Carry Forward Of Losses, , 27, , Note: Long term capital loss can be set off only against long term capital gain. Therefore, long term capital, loss of ₹ 30,000 has to be carried forward to the next assessment year., Illustration 8: Compute the gross total income of Mr. F for the A.Y.2022-23 from the information given, below –, Particulars, ₹, Net income from house property, 1,25,000, Income from business (before provided for depreciation), 1,35,000, Short term capital gains on sales of shares, 56,000, Long term capital loss from sale of property (brought forward from A.Y. 2021-22), (90,000), Income from growing and manufacturing of tea, 1,20,000, Current year depreciation, 26,000, Brought forward business loss (loss incurred six years ago), (45,000), Solution: The gross total income of Mr. F for the A.Y. 2022-23 is calculated as under:, Particulars, ₹, ₹, Income from house property, 1,25,000, Income from business, 1,35,000, Profits before depreciation, (26,000), Less: Current year depreciation, Less: Brought forward business loss, (45,000), 64,000, Income from growing and manufacturing of tea (40% is business income), 48,000 1,12,000, Income from the capital gains, Short term capital gains, 56,000, Long term capital loss from property (cannot be set off), 56,000, Nil, Gross Total Income, 2,93,000, Note: 60% of the income from growing and manufacturing of tea is treated as agricultural income and, therefore, exempt from tax.
Page 547 :
Set Off and Carry Forward Of Losses, , 28, , MULTIPLE CHOICE QUESTIONS, 1. Mr. X incurred long-term capital loss from sale of listed shares in recognized stock exchange and, STT is paid at the time of acquisition and sale of such shares. Such loss –, (a) can be set-off only against long-term capital gains, (b) can be set-off against both short-term capital gains and long-term capital gains, (c) can be set-off against any head of income., (d) is not allowed to be set-off, 2. The maximum period for which speculation loss can be carried forward is (a) 4 years, (b) 8 years, (c) indefinitely, (d) not allowed to be carry forward, 3. Mr. A incurred short-term capital loss of ₹10,000 on sale of shares through the National Stock, Exchange. Such loss (a) can be set-off only against short-term capital gains, (b) can be set-off against both short-term capital gains and long-term capital gains, (c) can be set-off against any head of income., (d) not allowed to be set-off, 4. Loss from house property of ₹ 3,10,000 of A.Y. 2020-21 is allowed to be set-off against income from, house property of A.Y. 2022-23 of ₹ 5,00,000 to the extent of –, (a) ₹ 2,00,000, (b) fully allowed i.e., ₹3,10,000, (c) ₹ 2,50,000, (d) ₹ 1,00,000, 5. Business loss of the current year cannot be set-off against –, (a) Any income other than business income, (b) Long-term capital gain, (c) Either long-term capital gain or short-term capital gain, (d) Salary income, 6. Brought forward loss from house property can be set-off –, (a) Against any head of income to the extent of ₹2,00,000, (b) Against income from house property to the extent of ₹2,00,000, (c) Against income from house property without any limit, (d) Against any head of income without any limit, 7. Mr. Ravi incurred loss of ₹4 lakh in the P.Y.2021-22 in retail trade business. Against which of the, following incomes earned during the same year, can he set-off such loss?, (a) profit of ₹1 lakh from wholesale cloth business, (b) long-term capital gains of ₹1.50 lakhs on sale of land, (c) speculative business income of ₹40,000, (d) All of the above, 8. Virat, runs a business of manufacturing of shoes in P.Y. 2019-20. During the P.Y. 2019-20 and P.Y., 2020-21, Virat had incurred business losses. For P.Y. 2021-22, he earned business profit (computed) of, Rs.3 lakhs. Considering he may/may not has sufficient business income to set off his earlier losses,, which of the following order of set off shall be considered:, (He does not have income from any other source), (a) First adjustment for loss of P.Y. 2019-20, then loss for P.Y. 2020-21 and then unabsorbed depreciation,, if any, income is available for adjustment., (b) First adjustment for loss of P.Y. 2020-21, then loss for P.Y. 2019-20 and then unabsorbed depreciation,, if any, income is available for adjustment., (c) First adjustment for unabsorbed depreciation, then loss of P.Y. 2020-21 and then loss for P.Y. 2019-20,, if any, income is available for adjustment, (d) First adjustment for unabsorbed depreciation, then loss of P.Y. 2019-20 and then loss for P.Y. 2020-21,, if any, income is available for adjustment, 9. Loss under the head business/profession cannot be set off from, (a) income of owning and maintaining race horses, (b) casual income, (c) income from house property, (d) income from salary, (e) (b) & (c), (f) (b) & (d)
Page 548 :
Set Off and Carry Forward Of Losses, , 29, , (g) none of these, 10. Short term loss can be set off, (a) from short term gain, (b) long term gain, (c) neither from short term nor from long term, (d) either short term or long term, (e) income under the head business/profession, (f) none of these, 11. Mr. X has unadjusted loss from house property ₹10,00,000 of the previous year 2012-13 and, income from house property of previous year 2021-22 ₹10,00,000, in this case, (a) loss cannot be set off and it will lapse, (b) loss will be set off from income from house property, (c) loss shall be carried forward to subsequent years, (d) none of these, 12. Loss from speculative business, (a) can be set off from income of owning and maintaining of race horses, (b) can be set off from income of house property, (c) can be set off from income under the head business/profession, (d) none of these, 13. Mr. X has loss from house property ₹ 5,00,000 and LTCG ₹ 5,00,000, in this case his tax liability, shall be, (a) ₹ 7,500, (b) ₹ 10,400, (c) Nil, (d) ₹ 7,800, (e) none of these, 14. Mr. X a senior citizen has loss from house property ₹ 5,00,000 and LTCG ₹ 5,00,000, in this case, his tax liability shall be, (a) ₹ 7,500, (b) ₹ 10,400, (c) Nil, (d) ₹ 7,800, (e) none of these, 15. Mr. X a senior citizen has loss from house property ₹ 5,00,000 and as income from business, ₹12,00,000, in this case his tax liability shall be, (a) ₹ 1,12,500, (b) ₹ 1,14,400, (c) ₹ 82,500, (d) none of these, , Check Detailed Solution given on our website www.mkgeducation.com, (Icon /Heading No.11: Books)
Page 549 :
Set Off and Carry Forward Of Losses, , 30, , EXAMINATION QUESTIONS, JULY – 2021 (NEW COURSE), Question.4(b), (5 Marks), Mr. X a resident individual submits the following information, relevant to the previous year ending March, 31, 2022:, Particulars, (i), , Income from Salary (Computed), , (ii), , Income from House Property, -, , House in Delhi, , -, , House in Chennai, , -, , House in Mumbai (self-occupied), , Amount (₹), 2,22,000, 22,000, 2,60,000, 20,000, , (iii) Profit and gains from business of profession, -, , Textile business, , 18,000, , -, , Cosmetics business, , 22,000, , -, , Speculative business – 1, , 74,000, , -, , Speculative business – 2, , 46,000, , (iv) Capital gains, , (v), , Short term capital loss from sale of property, , 16,000, , Long term capital gains from sale of property, , 15,400, , Income from other sources (Computed), -, , Income from betting, , 34,000, , -, , Income from Card games, , 46,000, , -, , Loss on maintenance of race horses, , 14,600, , Determine the gross total income of Mr. X for the assessment year 2022-23 and the losses to be carried, forward assuming that he does not opt to be taxed under section 115BAC, Solution is given on our website www.mkgeducation.com (Icon /Heading No.11: Books), , NOV – 2019 (NEW COURSE), Question.4. (b), Following are the details of incomes/ losses of Mr. Rishi for the F.Y. 2021-22:, (Figures in brackets represents losses), Taxable salary income (computed), , (5 Marks), ₹, 3,60,000, , Taxable income from house property (computed), - from rented house property X, , 1,20,000, , - from rented house property Y, , (3,40,000)
Page 550 :
Set Off and Carry Forward Of Losses, , 31, , Taxable profit from business (computed), - business P, , 2,30,000, , - business Q, , (12,000), , - business R (speculative business), , 15,000, , - business T (speculative business), , (25,000), , Taxable Income from other sources:, - from card games, , 16,000, , - from owning & maintenance of race horses, , (7,000), , - interest on securities, , 5,000, , You are required to determine the Gross total income of Mr. Rishi for Assessment year 2022-23., Solution:, Computation of Gross Total Income for the A.Y. 2022-23, Income under the head Salary, Taxable salary income (computed), Less: Loss from property X, Income under the head salary, , 3,60,000, (2,00,000), 1,60,000, , Income under the head House property, Income from Property X, Less: Loss from property Y, Income under the head house property, , 1,20,000, (1,20,000), Nil, , Income under the head business profession, Income from Business P, Less: Loss from Business Q, , 2,30,000, (12,000), , Income from normal business, Income from Business R (speculative), Less: Loss from Business Q (speculative), Income from speculative business, , 2,18,000, 15,000, (15,000), Nil, , Income under the head business profession, Income under the head other sources, Interest income, Income from card games, , 2,18,000, , Income under the head other sources, , 5,000, 16,000, 21,000, , Gross total income, Income under the head salary, , 1,60,000, , Income under the head business profession, Income under the head other sources, Gross total income, , 2,18,000, 21,000, 3,99,000
Page 551 :
Set Off and Carry Forward Of Losses, , 32, , Losses to be carried forward, House property, , 20,000, , Speculative business loss, 10,000, Loss from owning and maintain of race horses, 7,000, Notes:, 1. Loss from house property can be adjusted from other head income maximum upto 2,00,000 and, balance can be carried forward. (Section 71), 2. Speculative loss can be adjusted only from speculative income and balance can be carried forward., (section 73), 3. Loss from owning and maintain race horses can be adjusted only from such income and not allowed, from other income. (section 74A), 4. In the above solution HP loss is adjusted from salary income. Student can adjust such loss from, business income also., , NOV – 2018 (OLD COURSE), Question 5, Mr. Rajeev submits the following information for the previous year 2021-2022., (Amount in ₹), (i), , Income under the head salary, , 6,50,000, , (ii), , Income from House-I, , (iii), , Loss from house-II (self-occupied property), , 1,25,000, , (iv), , Loss from house-III, , 1,90,000, , (v), , Loss from leather business, , (vi), , Profit from cloth business, , (vii), , Business loss of chemical business acquired by Inheritance, , 55,000, , 68,000, 1,70,000, 45,000, , (viii) Brought forward loss of discontinued business of textile, (ix), (x), , relating to financial year 2016-2017, , 50,000, , Long term capital gain on transfer of listed equity, shares on which STT was paid, , 75,000, , Short term capital loss in equity-oriented funds on, which STT was paid, , 35,000, , (xi), , Income from crossword puzzles, , 12,000, , (xii), , Dividend from foreign company, , 8,500, , (xiii) Loss on owning and maintenance of race horses, , 7,500, , (xiv), , Income from owning and maintenance of race bulls, , 9,000, , (xv), , Mr. Rajeev had taken an education loan from XYZ Bank for, his niece who is dependent on him for pursuing full time MBA, course on 2nd April, 2021. During the year, interest on loan, was due for ₹55,000. However, he paid towards principal, and interest ₹90,000 and ₹30,000 respectively., , (10 Marks)
Page 552 :
Set Off and Carry Forward Of Losses, , 33, , Compute the gross total income and losses to be carried forward of Mr. Rajeev for. assessment year 20222023., Solution: Computation of Gross total income and losses to be carried forward, ₹, Income under the head Salary, 6,50,000.00, Less: loss of house property (maximum allowed from other head) (section 71), (2,00,000.00), Income under the head Salary, 4,50,000.00, Income under the head House Property, Income from House -I, 55,000, Loss from house -II, (1,25,000), Loss from house -III, (1,90,000), Loss under the head House property, (2,60,000), Loss upto 2,00,000 shall be adjusted with other head and balance shall be carried forward for 8 years., Income under the head Business/Profession, Profit from cloth business, Income from owning and maintaining of race bulls, Less: loss from leather business (section 70), Less: loss from chemical business (section 70), Less: Brought forward business loss of A.Y. 2017-18 (section 72), Income under the head Business/Profession, Income under the head capital gains, Long term capital gains (STT paid), Less: Short term loss (STT paid), , Nil, , 1,70,000.00, 9,000.00, (68,000.00), (45,000.00), (50,000.00), 16,000.00, 75,000, (35,000), 40,000.00, , Income under the head other sources, Income from crossword puzzles (casual income), Dividend from foreign company, Income under the head other sources, Gross Total Income, Income under the head Salary, Income under the head Business/Profession, Income under the head other sources, Income under the head capital gains, Gross Total Income, , 12,000.00, 8,500.00, 20,500.00, 4,50,000.00, 16,000.00, 20,500.00, 40,000.00, 5,26,500.00, , Losses to be carried forward, 1. Loss under the head house property F.Y. 2021-22 (for 8 years) (section 71B), 60,000.00, 2. Loss from owning and maintaining race horses F.Y. 2021-22 (for 4 years), (section 74A), 7,500.00, Notes:, 1. Short term capital loss can be set off from short term capital gains or from long term capital gains and, such loss is not allowed to be set off from other head income.(section 71), 2. Loss from owning and maintaining race horses shall not be allowed to set off from other income and, shall be carried forward for 4 years., 3. Where there is a loss under the head house property, such loss shall be adjusted from other head income, maximum upto ₹2,00,000 and balance shall be carried forward for 8 years and shall be allowed to be, adjusted from same head., 4. Dividend from foreign company is taxable under the head other sources.
Page 553 :
Set Off and Carry Forward Of Losses, , 34, , MAY – 2017, Question 6(a)(i), (4 Marks), Mr. Shyam, a resident of Chandigarh, provides the following information for the financial year 2021-22:, Particulars, Income from textile business, Income from speculation business, Loss from gambling, Loss on maintenance of race horse, Eligible current year depreciation of textile business not adjusted in the income given, above., Unabsorbed depreciation of Assessment year 2021-22 brought forward, Speculation business loss of Assessment year 2021-22, , ₹, 4,60,000, 25,000, 12,000, 15,000, 5,000, 10,000, 30,000, , Compute the Gross total Income of Mr. Shyam for the assessment year 2022-23 and any other item of, expense or loss eligible for carry forward., Solution:, Computation of Gross Total Income, ₹, Income from Textile Business, 4,60,000, Less: Current year depreciation, (5,000), Less: Unabsorbed depreciation, (10,000), Income From Textile Business, 4,45,000, Income from speculation business, Less: Brought forward speculation loss (Section 73), Income from Speculation business, Gross Total Income, , 25,000, (25,000), Nil, 4,45,000, , Note:, (1) As per Section 73, Unadjusted Brought Forward Speculation loss of A.Y. 2021-22 shall be carried, forward of ₹5,000., (2) Loss from Gambling shall not be treated as loss and have no treatment., (3) Loss on maintenance of race horse shall be allowed to be set off from income of maintenance of race, horse only and unadjusted loss of ₹ 15,000 shall be carried forward for 4 years as per section 74A., , NOV – 2015, Question 4(a)., (4 Marks), Mr. X provides the following details for the previous year ending 31.03.2022., (i) Income under the head salary from XYZ Ltd., ₹6,00,000, (ii) Interest on FD with SBI for the Financial Year 2021-22, ₹72,000 (Net of TDS), (iii) Determined long term capital loss of AY 2020-21, ₹96,000, (iv) Long term Capital gain, ₹75,000, (v) Loss of minor son ₹90,000 computed in accordance with the provisions of Income Tax Act. Mr. X, transferred his own house to his minor son without adequate consideration few years back and minor son let, it out and suffered loss., (vi) Loss of his wife’s business, ₹(2,00,000), She carried business with funds which Mr. X gifted to her., You are required to compute taxable income of Mr. X for the AY 2022-23., Solution is given on our website www.mkgeducation.com (Icon /Heading No.11: Books)
Page 554 :
Set Off and Carry Forward Of Losses, , 35, , MAY – 2014, Question 6(A)., (4 Marks), Mr. X, a resident individual, furnishes the following particulars of his income and other details for the, previous year 2021-22., ₹, (1) Income under the head salary, 15,000, (2) Income from Business, 66,000, (3) Long term capital gain on sale of Land, 10,800, (4) Loss on maintenance of Race Horses, 15,000, (5) Loss from Gambling, 9,100, The other details of unabsorbed depreciation and brought forward losses pertaining to Assessment Year, 2021-22 are as follows:, ₹, (1) Unabsorbed depreciation, 11,000, (2) Loss from Speculative business, 22,000, (3) Short term capital loss, 9,800, Compute the Gross total income of Mr. X for the Assessment Year 2022-23 and the amount of loss, if any,, that can be carried forward, or not., Solution is given on our website www.mkgeducation.com (Icon /Heading No.11: Books), , NOV – 2012, Question 5(a), (4 Marks), Mr. X an assessee aged 61 years gives the following information for the previous year 31.03.2022:, Sr., Particulars, ₹, a., Loss from profession, 1,05,000, b., Capital loss on the sale of property-short term, 55,000, c., Capital gains on sale of shares-long term, 2,05,000, d., Loss in respect of self occupied property, 15,000, e., Loss in respect of let out property, 30,000, f., Share of loss from firm, 1,60,000, g., Income from card games, 55,000, h., Winnings from lotteries, 1,00,000, i., Loss from horse races in Mumbai, 40,000, j., Medical insurance premium paid by cheque, 18,000, Compute the total income of Mr. X for the assessment year 2022-23., Solution is given on our website www.mkgeducation.com (Icon /Heading No.11: Books), , NOV – 2011, Question 1, (5 Marks), Mr. X furnishes the following details for year ended 31.03.2022., ₹, Short term capital gain, 1,40,000, Loss from speculative business, (60,000), Long term capital gain on sale of land, 30,000, Long term capital loss on sale of shares, 1,00,000, (securities transaction tax not paid), Income from business of textile (after allowing current year depreciation), 50,000, Income from activity of owning and maintaining race horses, 15,000, Income under the head salary, 1,00,000, Loss from house property, (40,000), Following are the carry forward losses:, (i) Losses from activity of owning and maintaining race horses-pertaining to A.Y. 2019-20 ₹25,000., (ii) Carry forward loss from business of textile ₹60,000- Loss pertains to A.Y. 2014-15., Compute gross total income of Mr. X for the assessment year 2022-23.
Page 555 :
Set Off and Carry Forward Of Losses, , 36, , Also state the eligible carry forward losses for the assessment Year 2022-23., Solution is given on our website www.mkgeducation.com (Icon /Heading No.11: Books), , MAY – 2011, Question 4, (4 Marks), The following are the details relating to Mr. X, a resident Indian, aged 57, relating to the year ended, 31.03.2022:, (₹), Income under the head salary, 2,20,000, Loss from house property, (1,90,000), Loss from cloth business, (2,40,000), Income from speculation business, 30,000, Long-term capital gains from sale of urban land, 2,50,000, Long-term capital loss from sale of listed shares in recognized stock exchange (STT paid), 90,000, Loss from card games, (32,000), Income from betting, 45,000, Life insurance premium paid, 1,20,000, Compute the total income and show the items eligible for carry forward., Solution is given on our website www.mkgeducation.com (Icon /Heading No.11: Books), Question 4, Mr. X furnishes the following information for the year ended 31.03.2022:, Income from business, Income from house property, Lottery winning (Gross), Speculation business income, Income under the head salary, Long term capital gain, Compute his total income, tax liability and advance tax obligations., Solution is given on our website www.mkgeducation.com (Icon /Heading No.11: Books), , (4 Marks), ₹, (1,35,000), (15,000), 3,00,000, 1,00,000, 60,000, 70,000, , NOV – 2010, Question 1, Mr. X submits the following details of his income for the assessment year 2022-23., Income under the head salary, Loss from let out house property, Income from sugar business, Loss from iron ore business b/f (discontinued in 2015-16), Short term capital loss, Long term capital gain, Dividend, Income received from lottery winning (Gross), Winning in card games (Gross), Agricultural income, Long term capital gain from shares (STT paid), Short term capital loss under section 111A, Bank interest on fixed deposit, Calculate gross total income and losses to be carried forward., Solution is given on our website www.mkgeducation.com (Icon /Heading No.11: Books), , (5 Marks), 3,00,000.00, 40,000.00, 50,000.00, 1,20,000.00, 60,000.00, 40,000.00, 5,000.00, 50,000.00, 6,000.00, 20,000.00, 10,000.00, 10,000.00, 5,000.00, , Question 1, (5 Marks), Determine the total income of Mr. X from the following information for the Assessment Year 2022-23:, ₹, (i), Interest received on enhanced compensation (It relates to transfer of land in the financial, 4,00,000
Page 556 :
Set Off and Carry Forward Of Losses, , (ii), (iii), , year 2016-17. Out of the above ₹65,000 relates to financial year 2021-22 and the balance, relate to preceding years), Business loss relating to discontinued business of the assessment year 2016-17 brought, forward and eligible for set off., Current year business income (i.e. financial year 2021-22) (Computed), , 37, , 1,50,000, 1,10,000, , Solution is given on our website www.mkgeducation.com (Icon /Heading No.11: Books), , MAY – 2010, Question 4, (6 Marks), Mr. X furnishes you the following details for the year ended 31.03.2022:, Income (loss) from house property, ₹, House – 1, 36,000, House – 2 Self occupied, (20,000), House – 3, 60,000, Profits and gains from Business or Profession, Textile Business, 2,00,000, Automobile Business, (3,00,000), Speculation Business, 2,00,000, Capital Gains, Long-term capital gain from sale of shares (STT paid), 1,50,000, Long-term capital gain from sale of vacant site, 2,00,000, Short-term capital loss from sale of building, 1,00,000, (Note: Assume that the figures given above are computed and arrived at after considering eligible, deductions)., Other sources:, Gift from a Friend (non-relative) on 05.06.2021, 60,000, Gift from Maternal Uncle on 25.02.2022, 1,00,000, Gift from Grandfather’s Younger Brother on 10.02.2022, 1,00,000, Compute the total income of Mr. X for the Assessment Year 2022-23., Solution is given on our website www.mkgeducation.com (Icon /Heading No.11: Books), , JUNE – 2009, Question 4, (4 Marks), Mrs. X, a resident individual, provides the following details of her income / losses for the year ended, 31.03.2022:, (i) Salary received as a partner from a partnership firm ₹ 7,50,000., (ii) Loss on sale of shares listed in BSE ₹1,00,000. Shares were held for 15 months and STT paid on sale., (iii) Long-term capital gain on sale of land ₹ 5,00,000., (iv) ₹ 51,000 received in cash from friends in party., (v) ₹ 55,000, received towards dividend on listed equity shares of domestic companies., (vi) Brought forward business loss of assessment year 2021-22 ₹ 12,50,000., The return for assessment year 2021-22 was filed in time., Compute gross total income of Mrs. X for the assessment year 2022-23 and ascertain the amount of loss that, can be carried forward., Solution is given on our website www.mkgeducation.com (Icon /Heading No.11: Books), , NOV – 2008, Question 4, (4 Marks), Mr. X, a resident individual, furnishes the following particulars of his income and other details for the, previous year 2021-22:, ₹, (i) Income under the head salary, 18,000
Page 557 :
Set Off and Carry Forward Of Losses, , 38, , (ii) Net annual value of house property, 70,000, (iii) Income from business, 80,000, (iv) Income from speculative business, 12,000, (v) Long term capital gain on sale of land, 15,800, (vi) Loss on maintenance of race horse, 9,000, (vii) Loss on gambling, 8,000, Depreciation allowable under the Income-tax Act comes to ₹8,000 for which no treatment is given above., The other details of unabsorbed depreciation and brought forward losses of previous year 2018-19 are:, ₹, (i) Unabsorbed depreciation, 9,000, (ii) Loss from speculative business, 16,000, (iii) Short term capital loss, 7,800, Compute the gross total income of Mr. X, for the Assessment year 2022-23, and the amount of loss that can, or cannot be carried forward., Solution is given on our website www.mkgeducation.com (Icon /Heading No.11: Books), , NOV – 2007, Question 5, (4 Marks), Discuss in brief the provisions relating to set off and carry forward of losses in speculation business., Refer answer given in the chapter, , MAY – 2007, Question 4, (4 Marks), Mr. X, engaged in various types of activities, gives the following particulars of her income for the year, ended 31.03.2022:, ₹, (a) Profit of business of consumer and house-hold products, 50,000, (b) Loss of business of readymade garments, 10,000, (c) Brought forward loss of catering business which was closed in Asst. Year 2021-22, 15,000, (d) Short-term loss on sale of securities and shares, 15,000, (e) Profit of speculative transactions entered into during the year, 12,500, (f) Loss of speculative transactions of Asst. Year 2017-18 not set off till Asst. Year 2021-22, 15,000, Compute the total income of Mr. X for the A.Y. 2022-23., Solution is given on our website www.mkgeducation.com (Icon /Heading No.11: Books)
Page 558 :
Provisions for filing of Return of Income, , 39, , PROVISIONS FOR FILING, OF RETURN OF INCOME, PARTICULARS, Submission of return of income, Return of loss, Belated return, Revised return, Certain information required in the return of income, Return of Income in the case of an Assessee engaged in Business /Profession, , Defective return, Permanent account number, Quoting of Aadhaar Number, Scheme for submission of returns through Tax Return Preparers, Return by whom to be signed, Self-Assessment, Fee for default in furnishing return of income, Fee for default to intimation of Aadhaar number, , SECTIONS, 139(1), 139(3), 139(4), 139(5), 139(6), 139 (6A), 139(9), 139A, 139AA, 139B, 140, 140A, 234F, 234H, , Question 1 [V. Imp.]: Write a note on filing of return of income. Section 139(1), Or, Write a note on filing of compulsory return of income., Answer:, Filing of return of income/ filing of voluntary return of income, Under section 139(1), a return of income is to be filed by the following persons:, (i) Every company assessee or partnership firm irrespective of their income or loss shall be required to, file return of income e.g. ABC Ltd. has total income of ₹10,000,in this case, company shall be required to, file the return., (ii) Any other person like Individual, HUF etc. shall be required to file return of income if Gross total, income, before claiming the exemption under section 54, 54B, 54D, 54EC, 54F, 54G, 54GA, and 54GB,, is exceeding exemption limit e.g. If for previous year 2021-22 gross total income of Mrs. X is ₹2,55,000 and, deductions allowed under section 80C to 80U are ₹60,000 and total income is ₹1,95,000 and tax liability, shall be nil but still Mrs. X has to file her return of income., (iii) Every person who is assessable on behalf of any other person and the person on whose behalf he is, assessable has gross total income, before claiming the exemption under section 54, 54B, 54D, 54EC, 54F,, 54G, 54GA, and 54GB, more than the income exempt from tax, in such cases also, the person is required to, file a return of income on behalf of such person. E.g. Minor son of Mr. X has gross total income from film, acting ₹5 lakhs. In this case, Mr. X has to file a return of income on behalf of his minor son., (iv) 4th Proviso to section 139(1): If any individual/HUF is resident and ordinarily resident in India, such, person shall be required to file return of income or loss if at any time during the P.Y., such person, (a) holds, as a beneficial owner or otherwise, any asset (including any financial interest in any entity)located, outside India or has signing authority in any account located outside India; or, (b) is a beneficiary of any asset (including any financial interest in any entity) located outside India,, Explanation 4.—For the purposes of this section "beneficial owner" in respect of an asset means an individual
Page 559 :
Provisions for filing of Return of Income, , 40, , who has provided, directly or indirectly, consideration for the asset for the immediate or future benefit, direct or, indirect, of himself or any other person., Explanation 5.—For the purposes of this section "beneficiary" in respect of an asset means an individual who, derives benefit from the asset during the previous year and the consideration for such asset has been provided by, any person other than such beneficiary.], , (v) 7th Proviso to section 139(1): The persons other than company and partnership firm are also required to, furnish the return of income if they have entered into any transaction given below:, (i) Persons, who has deposited an amount or aggregate of the amounts exceeding one crore rupees in one, or more current accounts maintained with a banking company or a co-operative bank; or, (ii) Persons, who has incurred expenditure of an amount or aggregate of the amounts exceeding two lakh, rupees for himself or any other person for travel to a foreign country; or, (iii) Persons, who has incurred expenditure of an amount or aggregate of the amounts exceeding one lakh, rupees towards consumption of electricity; or, (iv) fulfils such other conditions as may be prescribed., RTP NOV -2020, 3. Mr. Dinesh, a resident in India, has gross total income of ₹ 2,30,000 comprising of interest on saving A/c, and rental income during the previous year 2021-22. He incurred expenditure of ₹ 2,00,000 for his son for a, study tour to Europe. Whether he is required to file return of income for the assessment year 2022-23? If, yes, what is the due date?, (a) Yes, 31st July of A.Y, (b) Yes, 30th September of A.Y, (c) Yes, 31st October of A.Y, (d) No, he is not required to file return of income, Answer: (d), Hint: Persons, who has incurred expenditure of amount exceeding 2,00,000 for himself or for other, person for travel to a foreign country shall be required to file Income tax return even if his gross total, income is below the basic exemption limit. In the given case amount is not exceeding 2,00,000 hence he is, not required to file Income tax return, MTP NOV -2020, 1. Ms. Rashmi who is not required to furnish return u/s 139(1) as her gross total income itself is less than, basic exemption limit, has incurred expenditure of ₹3,00,000 for her daughter for travel to U.S.A. during, P.Y. 2021-22. Is she required to file return for A.Y. 2022-23? If yes, what is the due date?, (a) Yes; 31st July, 2022, (b) Yes; 30th September, 2022, (c) Yes; 31st August, 2022, (d) No, she is not required to file return of income for A.Y. 2022-23, Answer: (a), , (2 Marks), , Explanation 2, Due date for filing the return of income, Return is to be filed in general upto 31st July of the assessment year, however, in the following cases, the last, date shall be 31st October of the assessment year., 1. Every company assessee, Example, For the previous year 2021-22, ABC Ltd. has to file its return of income upto 31.10.2022., 2. Any other person who is required to get his accounts audited either under Income Tax Act or under any, other Act.
Page 560 :
Provisions for filing of Return of Income, , 41, , Example, Mr. X has his own business and his turnover for previous year 2021-22 is ₹102 lakhs. In this case, the last, date of filing the return of income shall be 31.10.2022, but if turnover is ₹97 lakhs, the last date shall be, 31.07.2022., Similarly if a partnership firm XY has turnover of its business ₹ 65 lakhs for previous year 2021-22, in this, case, the last date of filing of return of income shall be 31.07.2022., 3. Partner of a partnership firm or the spouse of such partner if the provisions of section 5A applies to such, spouse, whose accounts are required to be audited., Illustration 1: Paras aged 55 years is a resident of India. During the F.Y. 2021-22, interest of ₹2,88,000 was, credited to his Non-resident (External) Account with SBI. ₹30,000, being interest on fixed deposit with SBI,, was credited to his saving bank account during this period. He also earned ₹3,000 as interest on this saving, account. Is Paras required to file return of income?, What will be your answer, if he has incurred ₹3 lakhs as travel expenditure of self and spouse to US to stay, with his married daughter for some time?, Solution: An individual is required to furnish a return of income under section 139(1) if his total income,, before giving effect to the deductions under Chapter VI-A or exemption under section or section, 54/54B/54D/54EC or 54F, exceeds the maximum amount not chargeable to tax i.e. ₹2,50,000 (for A.Y., 2022-23)., Computation of total income of Mr. Paras for A.Y. 2022-23, Particulars, ₹, Income from other sources, Interest earned from Non-resident (External) Account, [Exempt under section 10(4)(ii), assuming that Mr. Paras has been, permitted by RBI to maintain the aforesaid account], Nil, Interest on fixed deposit with SBI, 30,000, Interest on savings bank account, 3,000, Gross Total Income, 33,000, Less: Deduction under section 80TTA (Interest on saving bank, account), (3,000), Total Income, 30,000, Since the total income of Mr. Paras for A.Y.2022-23, before giving effect, inter alia, to the deductions under, Chapter VI-A, is less than the basic exemption limit of ₹2,50,000, he is not required to file return of income, for A.Y.2022-23., Note: In the above solution, interest of ₹2,88,000 earned from Non-resident (External) account has been, taken as exempt on the assumption that Mr. Paras, a resident, has been permitted by RBI to maintain the, aforesaid account. However, in case he has not been so permitted, the said interest would be taxable. In such, a case, his total income, before giving effect, inter alia, to the deductions under Chapter VI-A, would be ₹, 3,21,000 (₹30,000 + ₹2,88,000 + ₹3,000), which is higher than the basic exemption limit of ₹2,50,000., Consequently, he would be required to file return of income for A.Y.2022-23., If he has incurred expenditure of ₹3 lakhs on foreign travel of self and spouse, he has to mandatorily file his, return of income on or before the due date under section 139(1)., Question 2 [V. Imp.]: Write a note on Return of Loss Section 139(3)., Answer: Return of Loss, Section 139(3), If any person has sustained any loss under the head Business/Profession or under the head capital gains or, the loss is from owning and maintaining of race horses and such person claims that the loss is to be carried, forward, such person has to file a return of loss and such return shall be examined by the Assessing Officer, and the loss computed by the assessee shall be confirmed by the Assessing Officer by sending an intimation, under section 157 and only after that carry forward of loss shall be allowed to the assessee., Return under section 139(3) has to be submitted within the time allowed under section 139(1).
Page 561 :
Provisions for filing of Return of Income, , 42, , Under section 80, if return of loss has been filed after the last date of filing of return of income, In that case, carry forward of losses is not allowed. E.g. For previous year 2021-22 ABC Ltd. has incurred business loss, of ₹90 lakhs. In this case, the company must file return of loss under section 139(3) maximum upto, 31.10.2022, otherwise carry forward of the loss is not allowed., The above provisions are not applicable with regard to loss under the head house property and also it is not, applicable with regard to unabsorbed depreciation., If any return is filed under section 139(3), it will be considered to be a return under section 139(1)., Question 3 [V. Imp.]: Write a note on Belated Return of Income Section 139(4)., Answer: Belated Return of Income Section 139(4), Every person is required to file a return of income within the time allowed under section 139(1) however, return of income can be filed even after the due date but upto the three months prior to the end of relevant, assessment year or before completion of assessment, whichever is earlier. E.g. For previous year 202122 ABC Ltd. has to file its return of income upto 31.10.2022. However, belated return is allowed under, section 139(4) but maximum upto 31.12.2022., Question 4: Write a note on fee for default in furnishing return of income, Answer: Fee for default in furnishing return of income. Section 234F, where a person required to furnish a return of income under section 139, fails to do so within the time, prescribed u/s 139(1), he shall pay, by way of fee, a sum of ₹ 5,000, Provided that if the total income of the person does not exceed ₹5,00,000 the fee payable under this section, shall not exceed ₹ 1,000., Example: For the P.Y.21-22, Mr. X has Total Income ₹7,00,000 and he files his return on 10th August 2022,, in this case Penalty of ₹5000 is payable but if his total income is upto ₹5,00,000, Penalty shall be ₹1000., , NOV – 2018 (NEW COURSE), Question 3(b), (Marks 3), Explain the quantum of late fees under section 234F for delay in furnishing return of income within the, prescribed time limit under section 139 (1) for A.Y. 2022-23., Answer: Refer answer given in the book, Question 5 [V. Imp.]: Write a note on Revised Return of Income Section 139(5)., Answer: Revised Return of Income Section 139(5), If any person has furnished a return under section 139(1) or under section 139(4), discovers any omission, or any wrong statement, he may furnish a revised return at any time before three months prior to the end of, the relevant assessment year e.g. If ABC Ltd. has filed its return of income on 31.10.2022 for previous year, 2021-22 and subsequently the company has detected any bonafide error, in this case, the company is allowed, to revise its return of income under section 139(5) but maximum upto 31.12.2022., If the assessment on the return has already been completed, revision is not allowed after completion of, assessment e.g. For the previous year 2021-22 ABC Ltd. has filed its return of income on 31.10.2022. This, return was checked by the Assessing Officer on 01.12.2022 and the company wish to file a revised return on, 15.12.2022. In this case, revised return shall not be accepted., An assessee is allowed to revise the return of income any number of times, however, if the earlier return has, already being assessed, revised return shall not be allowed., Revision is allowed only with regard to a return, which was filed under section 139(1) or 139(4), i.e. if the, return has been filed under any other section, its revision is not allowed., However, the return filed under section 139(3) is considered to be return under section 139(1), its revision is, allowed., , NOV – 2019 (OLD COURSE), Question.5. (b), (3 Marks), Explain with brief reasons, whether the return of income can be revised under selection 139(5) of the, Income-tax Act, 1961 in the following cases:, (i) Defective or incomplete return field under section – 139(9)
Page 562 :
Provisions for filing of Return of Income, , 43, , (ii) Return already revises once under section-139(5), (iii) Return of loss field under section-139(3), Answer: Refer answer given in the book, NOV -2020 (9 Marks), Question 2 (a), Mr. Mukesh born on 01.04.1962 furnished his original return for Assessment year 2022-23 on 30.07.2022., He has shown his salary income of ₹7.30 lakhs (computed) and interest from his savings bank of ₹12,700, and from his fixed deposits of ₹43,000. He also claimed deduction under section 80C ₹1.50 lakhs. He also, claimed deduction u/s 80D of ₹25,000. He also claimed deduction u/s 80TTA of ₹10,000. His employer has, deducted TDS of ₹33,950 from his salary, which he adjusted fully against tax payable., He paid health insurance of ₹38,000 by account payee cheque for self and wife. He paid ₹1,500 in cash for, his health check-up and ₹4,000 by cheque for preventive health check-up of his parents. He also paid, medical insurance premium of ₹33,000 during the year to insure the health of his mother, aged 80 years,, staying with his younger brother. He further incurred medical expenditure of ₹25,000 on his father, aged 81, years, who is staying with him. His father is not covered under any Mediclaim policy., He seeks your advice about possibility of revising his return and if possible file his revise return. Analyse, the above narrated facts as per applicable provisions of the Income Tax Act, 1961. Does he need to revise, his return and for what reasons? Please advise him suitably and if needed, re-compute his income and tax, payable or refund due for the assessment year 2022-23., Solution:, Computation of total income of Mr. Mukesh for A.Y.2022-23 [As per the original return filed by him], Particulars, (i), (ii), , Salaries (Computed), Income from Other Sources, Interest on savings bank account, Interest on fixed deposits, , ₹, , 7,30,000, 12,700, 43,000, , Less: Deductions under Chapter VI-A, (i), Deduction u/s 80C, 1,50,000, (ii), Deduction u/s 80D, 25,000, (iii), Deduction u/s 80TTA, 10,000, Total Income, Computation of tax liability of Mr. Mukesh for A.Y.2022-23 (As per original return), Tax on total income [20% of ₹ 1,00,700 (i.e., ₹ 6,00,700 – ₹ 5,00,000) + ₹ 12,500], Add: HEC@4%, Tax payable on total income, Tax payable on total income (rounded off u/s 288B), Less: Tax deducted at source u/s 192, Tax Payable, , ₹, , 55,700, 7,85,700, , (1,85,000), 6,00,700, ₹, 32,640, 1,306, 33,946, 33,950, (33,950), Nil, , Need for filing revised return - Analysis Since Mr. Mukesh’s birthday falls on 01.04.2022, he would be, treated as having completed 60 years of age in the P.Y.2021-22, and hence, he would be eligible for the, benefit of higher deduction u/s 80D, higher deduction of up-to ₹ 50,000 u/s 80TTB (instead of ₹ 10,000 u/s, 80TTA) while computing his total income as well as for higher basic exemption limit of ₹ 3,00,000 in the, P.Y.2021-22 itself while computing his tax liability. Also, he would be entitled to deduction in respect of, medical insurance premium paid to insure the health of his mother and medical expenses incurred on his, father who is not covered under any Mediclaim policy. Accordingly, having discovered such omissions in, the original return, he has to file his revised return of income u/s 139(5) on or before 31.3.2022 to avail these
Page 563 :
Provisions for filing of Return of Income, , 44, , benefits which he has not availed while filing his original return of income. The computation of total income, and tax liability (refund due) as per the revised return are worked out hereunder Computation of Total Income of Mr. Mukesh for the A.Y. 2022-23, [As per the Revised Return], Particulars, ₹, ₹, (i) Salaries (Computed), 7,30,000, (ii) Income from Other Sources, Interest on savings bank account, 12,700, Interest on fixed deposits, 43,000, 55,700, Gross Total Income, 7,85,700, Less: Deductions under Chapter VI-A, (i) Deduction u/s 80C, 1,50,000, (ii) Deduction u/s 80D, Medical insurance premium for self and spouse, 38,000, Preventive health check-up for self (allowable, even if paid in cash), 1,500, Fully allowed as it is within the overall limit of ₹, 50,000 for family, 39,500, Medical insurance premium for mother, 33,000, Medical expenditure for father not, covered under any policy, 25,000, Preventive health check-up for parents, (₹ 4,000, restricted to ₹ 3,500, being, ₹ 5,000 – ₹ 1,500 claimed for self and, 3,500, spouse), 61,500, , (iii), , Restricted to maximum of ₹ 50,000 for, parents, Deduction u/s 80TTB, Interest on savings bank account, Interest on fixed deposits, Restricted to maximum of ₹ 50,000, , 50,000, , 89,500, , 12,700, 43,000, 55,700, (50,000), (2,89,500), 4,96,200, , Total Income, Computation of tax liability of Mr. Mukesh for A.Y.2022-23, [As per the Revised Return], , Tax on total income [5% of ₹1,96,200 (i.e., ₹4,96,200 – ₹ 3,00,000 basic exemption limit), Less: Rebate u/s 87A (Since his total income does not exceed ₹ 5 lakh) – ₹ 12,500 or tax, on total income, whichever is lower, Tax payable on total income, Less: Tax deducted at source u/s 192, , ₹, 9,810, (9,810), Nil, (33,950)
Page 564 :
Provisions for filing of Return of Income, Refund due, , 45, 33,950, , Therefore, Mr. Mukesh has to file a revised return showing the above revised computation of total income, and tax liability on or before 31.3.2022 to claim the enhanced deductions which he had not claimed in the, original return and get refund of the entire income-tax of ₹ 33,950 deducted at source by his employer., Question 6: What are the particulars required to be furnished with the return of income in certain, specified cases, as per section 139(6)., Answer: As per Section 139(6) The prescribed form of the return shall, in such cases as may be prescribed,, require the assessee to furnish the particulars of income exempt from tax, assets of the prescribed nature and, value, held by him as a beneficial owner or otherwise or in which he is a beneficiary, his bank account and credit, card held by him, expenditure exceeding the prescribed limits incurred by him under prescribed heads and such, other outgoings as may be prescribed., Question 7: Write a note on Particulars to be Furnished with Return of Income in the case of an, Assessee engaged in Business or Profession., Answer: As per section 139(6A), Without prejudice to the provisions of sub-section (6), the prescribed form of, the returns shall, in the case of an assessee engaged in any business or profession, also require him to furnish the, report of any audit or where the report has been furnished prior to the furnishing of the return, a copy of such, report together with proof of furnishing the report, the particulars of the location and style of the principal place, where he carries on the business or profession and all the branches thereof, the names and addresses of his, partners, if any, in such business or profession and, if he is a member of an association or body of individuals, the, names of the other members of the association or the body of individuals and the extent of the share of the, assessee and the shares of all such partners or the members, as the case may be, in the profits of the business or, profession and any branches thereof., Question 8 [V. Imp.]: Write a note on Defective Return of Income Section 139(9)., Answer: Defective Return Section 139(9), (1) Under this section, the Assessing Officer has the power to call upon the assessee to rectify a defective, return., (2) Where the Assessing Officer considers that the return of income furnished by the assessee is defective,, he may intimate the defect to the assessee and give him an opportunity to rectify the defect within a period, of 15 days from the date of such intimation. The Assessing Officer has the discretion to extend the time, period beyond 15 days, on an application made by the assessee., (3) If the defect is not rectified within the period of 15 days or such further extended period, then the return, would be treated as an invalid return. The consequential effect would be the same as if the assessee had, failed to furnish the return., (4) Where, however, the assessee rectifies the defect after the expiry of the period of 15 days or the further, extended period, but before assessment is made, the Assessing Officer can condone the delay and treat the, return as a valid return., (5) A return of income shall be regarded as defective in the following cases:, (a) If annexures, statements and columns in the return of income relating to computation of income have, not been filled in properly and also the required documents have not been enclosed., (b) The columns are filled wrongly or filled incompletely, (c) Where regular books of account are not maintained by the assessee, the return should be, accompanied by –, (i) a statement indicating –, (1) the amount of turnover or gross receipts,, (2) gross profit,, (3) expenses; and, (4) net profit of the business or profession;, (ii) the basis on which such amounts mentioned in (i) above have been computed,, (iii) the amounts of total sundry debtors, sundry creditors, stock-intrade and cash balance as at the end, of the previous year.
Page 565 :
Provisions for filing of Return of Income, , 46, , Note – As per Rule 12, in general the documents are not to be enclosed with the return of income, , NOV – 2019 (NEW COURSE), Question.4. (c), (4 Marks), Elaborate the conditions, non-fulfilment of which would render a return of income filed by an assessee not, maintaining regular books of accounts, defective., Answer:, As per section 139(9) a return shall be considered to be defective if all of columns have not been filled in, properly or the required document has not been enclosed with the return. Where the Assessing Officer, considers that the return of income furnished by the assessee is defective, he may intimate the defect to the, assessee and give him an opportunity to rectify the defect within a period of 15 days from the date of such, intimation. The Assessing Officer has the discretion to extend the time period beyond 15 days, on an, application made by the assessee. If the defect is not rectified within the period of 15 days or such further, extended period, then the return would be treated as an invalid return. The consequential effect would be the, same as if the assessee had failed to furnish the return., Where regular books of accounts is not maintained by the assessee, return shall be accompanied by the, followings:, (i) amount of turnover or gross receipts, (ii) gross profit, (iii)expenses, (iv) net profit, (v) the amount of total sundry debtors, creditors, stock and cash balance at the end of the previous year., Non fulfilment of above details return shall be treated as defective., , NOV – 2011, Question 7, (4 Marks), (i) State when a return of income can be treated as defective?, (ii) An assessing officer finds a defect in the return of income and intimated the defect vide letter dated, 09.10.2021, which was received by Mr. Ram on 11.10.2021, What is the date by which Mr. Ram has to, rectify the defect, assuming that Mr. Ram has not applied for extension of time., Answer:, (i) Refer answer given in the chapter, (ii) Date by which Mr. Ram has to rectify the mistake:= 09.10.2021 + 15 days = 24.10.2021, Question 9 [V. Imp.]: Write a note on Permanent Account Number Section 139A., Answer: Permanent Account Number Section 139A, The following persons have to apply for allotment of permanent account number., Permanent account number, 139A. (1) Every person,—, (i) if his total income or the total income of any other person in respect of which he is assessable under, this Act during any previous year exceeded the maximum amount which is not chargeable to, income-tax. (As per rule 114, application should be given maximum upto 31st May of A.Y.) or, (ii) carrying on any business or profession whose total sales, turnover or gross receipts are or is likely to, exceed five lakh rupees in any previous year; (As per rule 114, application should be given, maximum upto the end of relevant F.Y.) or, (v) being a resident, other than an individual, which enters into a financial transaction of an amount, aggregating to two lakh fifty thousand rupees or more in a financial year; (As per rule 114,, application should be given maximum upto 31st May of A.Y.) or
Page 566 :
Provisions for filing of Return of Income, , 47, , (vi) who is the managing director, director, partner, trustee, author, founder, karta, chief executive, officer, principal officer or office bearer of the person referred to in clause (v) or any person, competent to act on behalf of the person referred to in clause (v); (As per rule 114, application, should be given maximum upto 31st May of A.Y.) or, (vii) who intends to enter into such transaction as may be prescribed by the Board in the interest of, revenue,, (1A/1B/2) Any person notified by Central Government or directed by the Assessing Officer., (3) Any person, may apply to the Assessing Officer for the allotment of a permanent account number., (5) Every person shall quote such number in all his returns, challans, documents, or correspondence with,, any income-tax authority and also intimate the Assessing Officer any change in his address or in the name, and nature of his business on the basis of which the permanent account number was allotted to him., (5A) Every person receiving any sum or income or amount from which tax has been deducted, shall intimate, his permanent account number to the person responsible for deducting such tax under that Chapter :, (5B) Where any sum or income or amount has been paid after deducting tax, every person deducting tax, under that Chapter shall quote the permanent account number of the person to whom such sum or income or, amount has been paid by him in all statements, certificates or returns etc., (5C) Every buyer or licensee or lessee referred to in section 206C shall intimate his permanent account, number to the person responsible for collecting tax referred to in that section., (5D) Every person collecting tax in accordance with the provisions of section 206C shall quote the, permanent account number of every buyer or licensee or lessee referred to in that section—, (i) in all certificates furnished in accordance with the provisions of sub-section (5) of section 206C;, (ii) in all returns prepared and delivered or caused to be delivered in accordance with the provisions of, sub-section (5A) or sub-section (5B) of section 206C to an income-tax authority;, (iii) in all statements prepared and delivered under section 206C(3)., (5E) Notwithstanding anything contained in this Act, every person who is required to furnish or intimate or, quote his permanent account number under this Act, and who,—, (a) has not been allotted a permanent account number but possesses the Aadhaar number, may furnish, or intimate or quote his Aadhaar number in lieu of the permanent account number, and such person, shall be allotted a permanent account number i.e. such person need not apply separately for issue of, PAN under rule 114., (b) has been allotted a permanent account number, and who has linked his Aadhaar number to his PAN, number, may furnish or intimate or quote his Aadhaar number in lieu of the permanent account, number i.e. inter-changeability of PAN and Aadhar number shall be allowed., (6A) Every person entering into such transaction, as may be prescribed, shall quote his permanent account, number or Aadhaar number, as the case may be, in the documents pertaining to such transactions and also, authenticate such permanent account number or Aadhaar number, in such manner as may be prescribed., (6B) Every person receiving any document relating to the transactions referred to in sub-section (6A), shall, ensure that permanent account number or Aadhaar number, as the case may be, has been duly quoted in such, document and also ensure that such permanent account number or Aadhaar number is so authenticated., “Authentication” means the process by which the permanent account number or Aadhaar number alongwith, demographic information or biometric information of an individual is submitted to the income-tax authority, or such other authority or agency as may be prescribed for its verification and such authority or agency, verifies the correctness, or the lack thereof, on the basis of information available with it;, Question 10: Explain Application for allotment of a permanent account number. Rule 114, Answer: Application for allotment of a permanent account number. Rule 114, (1) An application for allotment of a permanent account number shall be made in Form No. 49A or 49AA,, as the case may be:, (1A) Any person, who has not been allotted a permanent account number but possesses the Aadhaar number, and has furnished or intimated or quoted his Aadhaar number in lieu of the permanent account number,, shall be deemed to have applied for allotment of permanent account number and he shall not be required to, apply or submit any documents under this rule.
Page 567 :
Provisions for filing of Return of Income, , 48, , (1B) Any person, who has not been allotted a permanent account number but possesses the Aadhaar number, may apply for allotment of the permanent account number by intimating his Aadhaar number and he shall, not be required to apply or submit any documents under this rule., The applicant shall apply in Form no. 49A/49AA alongwith proof of identity, Proof of address, Proof of date, of birth etc., Question 11: Explain Transactions where PAN is to be mentioned Rule 114B., Answer: Transactions where PAN is to be mentioned Rule 114B, Every person shall quote his permanent account number in all documents pertaining to the transactions, specified below:—, 1. Sale or purchase of a motor vehicle other than two wheeled vehicles., , All such transactions., 2. Opening an account other than a time-deposit referred to at Sl. No.12 and a Basic Savings Bank Deposit, Account with a banking company or a co-operative bank., , All such transactions., 3. Making an application to any banking company or a co-operative bank for issue of a credit or debit card., , All such transactions., 4. Opening of a demat account., , All such transactions., 5. Payment to a hotel or restaurant against a bill or bills at any one time., , Payment in cash exceeding ₹ 50,000., 6. Payment in connection with travel to any foreign country or payment for purchase of any foreign, currency at any one time., , Cash exceeding ₹50,000., 7. Payment to a Mutual Fund for purchase of its units., , Amount exceeding ₹50,000., 8. Payment to a company or an institution for acquiring debentures or bonds issued by it., , Amount exceeding ₹50,000., 9. Payment to the Reserve Bank of India, for acquiring bonds issued by it., , Amount exceeding fifty thousand rupees., 10. Cash Deposit with banking company or a co-operative bank or post office, , exceeding ₹50,000 during any one day, 11. Purchase of bank drafts or pay orders or banker's cheques from a banking company or a co-operative, bank., , Payment in cash for an amount exceeding ₹50,000 during any one day., 12. A time deposit with, a banking company or a co-operative bank or post office or Nidhi (section 406 of, the Companies Act, 2013) or non-banking financial company., , Amount exceeding ₹50,000 or aggregating to more than ₹5,00,000 during a financial year., 13. Payment for one or more pre-paid payment instruments, as defined in the policy guidelines for issuance, and operation of pre-paid payment instruments issued by Reserve Bank of India under section 18 of the, Payment and Settlement Systems Act, 2007, to a banking company or a co-operative bank or any other, company or institution.
Page 568 :
Provisions for filing of Return of Income, , 49, , , , Payment in cash or by way of a bank draft or pay order or banker's cheque of an amount, aggregating to more than ₹50,000 in a financial year., 14. Payment as life insurance premium., , Amount aggregating to more than ₹50,000 in a financial year., 15. A contract for sale or purchase of securities (other than shares) as defined in clause (h) of section 2 of, the Securities Contracts (Regulation) Act, 1956, , Amount exceeding ₹1,00,000 per transaction., 16. Sale or purchase, by any person, of shares of a company not listed in a recognised stock exchange., Amount exceeding ₹1,00,000 per transaction., 17. Sale or purchase of any immovable property., , Amount exceeding ten lakh rupees or valued by stamp valuation authority referred to in, section 50C of the Act at an amount exceeding ten lakh rupees., 18. Sale or purchase, by any person, of goods or services of any nature other than those specified at Sl. Nos., 1 to 17 of this Table, if any., , Amount exceeding two lakh rupees per transaction:, Transaction by Minor, where a person, entering into any transaction, is a minor and who does not have any income chargeable to, income-tax, he shall quote the permanent account number of his father or mother or guardian, as the case, may be., Form 60, any person who does not have a permanent account number and who enters into any transaction specified in, this rule, he shall make a declaration in Form No.60 giving therein the particulars of such transaction., Rule not applicable, the provisions of this rule shall not apply to the following class or classes of persons, namely:—, (i) the Central Government, the State Governments and the Consular Offices;, (ii) the non-residents referred to in clause (30) of section 2 of the Act in respect of the transactions other, than a transaction referred to at Sl. No. 1 or 2 or 4 or 7 or 8 or 10 or 12 or 14 or 15 or 16 or 17 of the, Table:, , STRUCTURE OF PAN, A typical PAN is AFZPK7190K., First three characters i.e. “AFZ” in the above PAN are alphabetic series running from AAA to ZZZ, Fourth character of PAN i.e. “P” in the above PAN represents the status of the PAN holder. “P” stands for, Individual, “F” stands for Firm, “C” stands for Company, “H” stands for HUF, “A” stands for AOP, “T”, stands for TRUST etc., Fifth character i.e. “K” in the above PAN represents first character of the PAN holder’s last name/surname., Next four characters i.e. “7190” in the above PAN are sequential number running from 0001 to 9999., Last character i.e. “K” in the above PAN is an alphabetic check digit., Question 12 [V. Imp.]: Write a note on Quoting of Aadhaar number., Answer: Quoting of Aadhaar number. Section 139AA, Quoting of Aadhaar number., 139AA. (1) Every person who is eligible to obtain Aadhaar number shall, on or after the 1st day of July,, 2017, quote Aadhaar number—, (i) in the application form for allotment of permanent account number;, (ii) in the return of income (it is now compulsory w.e.f 01.04.2019 onwards)
Page 569 :
Provisions for filing of Return of Income, , 50, , Provided that where the person does not possess the Aadhaar Number, the Enrolment ID of Aadhaar, application form issued to him at the time of enrolment shall be quoted in the application for permanent, account number or, in the return of income furnished by him., (2) Every person who has been allotted permanent account number as on the 1st day of July, 2017, and who, is eligible to obtain Aadhaar number, shall intimate his Aadhaar number on or before 30th Sept 2021, otherwise his PAN shall become inoperative as per rule 114AAA., (3) The provisions of this section shall not apply to an individual who does not possess the Aadhar number, or Enrolment ID and is:, (i) residing in the States of Assam, Jammu & Kashmir and Meghalaya;, (ii) a non-resident as per Income-tax Act, 1961;, (iii) of the age of 80 years or more at any time during the previous year;, (iv) not a citizen of India, Manner of making permanent account number inoperative. Rule 114AAA, (1) Where a person, who has been allotted the permanent account number as on the 1st day of July, 2017 and, is required to intimate his Aadhaar number, has failed to intimate the same on or before the 30.09.2021, the, permanent account number of such person shall become inoperative immediately after the said date for the, purposes of furnishing, intimating or quoting under the Act., (2) Where a person, whose permanent account number has become inoperative is required to furnish,, intimate or quote his permanent account number under the Act, it shall be deemed that he has not furnished,, intimated or quoted the permanent account number, as the case may be, in accordance with the provisions of, the Act, and he shall be liable for all the consequences under the Act for not furnishing, intimating or, quoting the permanent account number., (3) Where the person has intimated his Aadhaar number after 30.09.2021, his permanent account number, shall become operative from the date of intimation of Aadhaar number for the purposes of furnishing,, intimating or quoting under the Act., Question 13: Write a note on fee for default relating to intimation of Aadhaar number., Answer: Fee for default relating to intimation of Aadhaar number Section 234H, Where a person is required to intimate his Aadhaar number under of section 139AA(2) and such person fails, to do so on or before such date, as may be prescribed, he shall be liable to pay such fee, as may be, prescribed, not exceeding one thousand rupees, at the time of making intimation under section 139AA(2), after the said date., Question 14 [V. Imp.]: Write a note on submission of returns through Tax Return Preparers., Answer: Scheme for submission of returns through Tax Return Preparers Section 139B, In order to help the persons having low income or tax liability, department has started scheme of Tax Return, Preparer who will file return for such persons. For this purpose department shall select and appoint TRPs., The tax return preparer shall hold a graduation degree from a recognised Indian university or other specified, qualifications but such persons should not be a Chartered Accountant or other specified persons., A person may approach a TRP for filing the return of income but any person who is required to get, his accounts audited shall not be allowed to file the return through the Tax Return Preparer., Similarly any non-resident shall not be allowed to file return through Tax Return Preparer., The department shall pay a commission of 3% of the tax paid on the income declared in the return or ₹1,000, whichever is less. A TRP shall be entitled for a minimum payment of ₹250 and if commission paid is less, than ₹250, he can receive the difference amount from the assessee whose return is being filed., e.g. A TRP has deposited tax of ₹60,000 on the basis of return filed by it, in this case commission payable, shall be 60,000 x 3% = 1,800 but maximum ₹1,000. If tax paid is ₹20,000, commission payable shall be, 20,000 x 3% = 600. If tax paid is ₹5,000, commission payable shall be ₹5,000 x 3% = 150 and TRP shall, allowed to charge ₹100 from the assessee.
Page 570 :
Provisions for filing of Return of Income, , 51, , Question 15: Power of Board to dispense with furnishing documents, etc., with return., Answer: As per section 139C The Board may make rules providing for a class or classes of persons who, may not be required to furnish documents, statements, receipts, certificates, reports of audit or any other, documents, which are otherwise under any other provisions of this Act, required to be furnished, along with, the return but on demand to be produced before the Assessing Officer. Accordingly rule 12 has been framed, which contain the details., Question 16: Filing of return in electronic form., Answer: As per section 139D The Board may make rules providing for—(a) the class or classes of persons, who shall be required to furnish the return in electronic form;(b) the form and the manner in which the, return in electronic form may be furnished;(c) the documents, statements, receipts, certificates or audited, reports which may not be furnished along with the return in electronic form but shall be produced before the, Assessing Officer on demand;(d) the computer resource or the electronic record to which the return in, electronic form may be transmitted., Question 17: Write a note on verifying of Return of Income., Answer: Verifying of return of income, Return by whom to be verified Section 140, 1. In the case of an individual, the return should be verified by the individual himself but if for any reason, return cannot be verified by the individual, return can be verified by his agent and the agent should enclose, copy of power of attorney with the return., If any individual is mentally incapacitated from attending to his affairs, return should be verified by his, guardian or any other person competent to act on his behalf., 2. In the case of a Hindu Undivided Family, by the karta, and, where the karta is absent from India or is, mentally incapacitated from attending to his affairs, by any other adult member (male or female) of such, family., 3. In the case of a company, by the managing director and if managing director is not available, return can, be signed by any director or any other person, as may be prescribed for this purpose., In case of non-resident company, return can be signed by its agent., If company is in liquidation, return can be signed by the official liquidator., 4. In the case of a firm, return can be signed by managing partner and if managing partner is not available,, return can be signed by any partner., 5. In the case of a limited liability partnership, return can be signed by the designated partner and if, designated partner is not available, return can be signed by any partner or any other person, as may be, prescribed for this purpose., 6. In the case of a local authority, return can be signed by the principal officer., 7. In the case of a political party, return can be signed by Chief Executive Officer., 8. In the case of any other association, return can be signed by the Principal Officer and if Principal Officer, is not available, by any member., 9. In the case of any other person, by that person or by some person competent to act on his behalf., Question 18: Write a note on Self-Assessment., Answer: Self-Assessment Section 140A, (1) Payment of tax, interest and fee before furnishing return of income, Where any tax is payable on the basis of any return required to be furnished under section 139, after taking, into account (i) the amount of tax, already paid, under any provision of the Income tax Act, 1961, (ii) the tax deducted or collected at source, the assessee shall be liable to pay such tax together with interest and fee payable before furnishing the, return. The return shall be accompanied by the proof of payment of such tax, interest and fee.
Page 571 :
Provisions for filing of Return of Income, , 52, , Consequence of failure to pay tax, interest or fee, If any assessee fails to pay the whole or any part of such of tax or interest or fee, he shall be deemed to be an, assessee in default in respect of such tax or interest or fee remaining unpaid and all the provisions of this Act, shall apply accordingly., (2) Order of adjustment of amount paid by the assessee, Where the amount paid by the assessee under section 140A falls short of the aggregate of the tax, interest, and fee as aforesaid, the amount so paid shall first be adjusted towards the fee payable and thereafter, towards interest and the balance, if any, shall be adjusted towards the tax payable., (3) Interest under section 234A, For the above purpose, interest payable under section 234A shall be computed on the amount of tax on the, total income as declared in the return, as reduced by the amount of(i) advance tax paid, if any;, (ii) any tax deducted or collected at source;, (4) Interest under section 234B, Interest payable under section 234B shall be computed on the assessed tax or on the amount by which the, advance tax paid falls short of the assessed tax., For this purpose “assessed tax” means the tax on total income declared in the return as reduced by the, amount of tax deducted or collected at source on any income which forms part of the total income;
Page 572 :
Provisions for filing of Return of Income, , 53, , PRACTICE PROBLEMS, Question 1. During the previous year 2021-22, Mr. X has income under the head house property ₹7,00,000., In this case, his last date of filing of return shall be ____________________., Question 2. A partnership firm XY has turnover of his business ₹75,00,000 and income under the head, Business/Profession ₹8,00,000 for previous year 2021-22. In this case, the last date of filing of return of, income shall be ____________., Question 3. ABC Ltd. has loss under the head Business/Profession ₹3,00,000 for previous year 2021-22. In, this case, the company has to file the return latest by ____________________., Question 4. A partnership firm XY has turnover of the business ₹205,00,000 and income from business, ₹14,00,000 for the previous year 2021-22, the last date for filing of return of income shall be, ______________., Question 5. ABC Ltd. do not have any income for the previous year 2021-22. In this case, company is not, required to file any return of income. Discuss., Question 6. ABC Ltd. has loss under the head Business/Profession ₹7,00,000 for the previous year 2021-22, and the company has filed the return of loss on 01.11.2022 under section 139(3). Discuss whether set off or, carried forward and set off of the loss is allowed or not., Question 7. For the previous year 2021-22, Mr. X an assessee shall be allowed to file belated return of, income latest upto ____________________., Question 8. For the previous year 2021-22, Mr. X has filed original return of income on 01.07.2022, he can, file revised return of income latest upto_________________., Question 9. For the previous year 2021-22, Mr. X has filed original return of income on 01.11.2022, he can, file revised return of income latest upto ____________________., Question 10. Mr. X has filed original return for previous year 2021-22 on 01.07.2022 and revised return on, 01.11.2022 and he further wants to revise the return on 01.12.2022. Discuss whether he is allowed to do so, or not., , Check Detailed Solution given on our website www.mkgeducation.com, (Icon /Heading No.11: Books)
Page 573 :
Provisions for filing of Return of Income, , 54, , MULTIPLE CHOICE QUESTIONS, 1. Mr. X has income under the head Business/Profession ₹3,00,000 and deduction under section 80C, ₹1,00,000, in this case, (a) he need not file any return of income, (b) he has to file return of income upto 31st July of assessment year, (c) he has to file return of income upto 30th Sept of assessment year, (d) none of these, 2. The due date for filing of a return of income for a company for Assessment Year 2022-23 is (a) 31st July, 2022, (b) 30th September, 2022, (c) 31st October, 2022, (d) 31st August, 2022, 3. If a minor child has income of ₹ 5,00,000 from his talent, in this case, (a) it is exempt from income tax, (b) It is taxable and income shall be clubbed in the income of father, (c) It is taxable and income shall be clubbed in the income of mother, (d) It is taxable and return shall be filed by his father as his guardian, (e) none of these, 4. The return of a company has to be verified by (a) the Managing Director or Director or any other person, as may be prescribed for this purpose, (b) the General Manager, (c) the Secretary, (d) the Manager, 5. An assessee can file a revised return of income at any time before the completion of assessment or, before expiry of the following period, whichever is earlier (a) one year from the end of the relevant assessment year, (b) two years from the end of the relevant assessment year, (c) six months from the end of the relevant assessment year, (d) three months prior to the end of the relevant assessment year, 6. As per section 139(1), filing of returns is compulsory irrespective of whether profit is earned or loss, is incurred, in case of (a) companies only, (b) firms only, (c) both companies and firms, (d) All assessees, 7. Mr. X has a total income of ₹ 7 lakhs for A.Y. 2022-23. He files his return of income for A.Y. 202223 on 13th December, 2022. He is liable to pay fee of–, (a) ₹ 1,000 under section 234F, (b) ₹ 5,000 under section 234F, (c) ₹ 10,000 under section 234F, (d) Not liable to pay any fee, 8. Mr. Y has a total income of ₹ 4,50,000 for A.Y. 2022-23. He furnishes his return of income for A.Y., 2022-23 on 2nd December, 2022. He is liable to pay fee of–, (a) ₹ 1,000 under section 234F, (b) ₹ 5,000 under section 234F, (c) ₹ 10,000 under section 234F, (d) Not liable to pay any fee, 9. Mr. Z, a salaried individual, has a total income of ₹ 8 lakhs for A.Y. 2022-23. He furnishes his, return of income for A.Y. 2022-23 on 28th August, 2022. He is liable to pay fee of–, (a) ₹ 1,000 under section 234F, (b) ₹ 5,000 under section 234F, (c) ₹ 10,000 under section 234F
Page 574 :
Provisions for filing of Return of Income, , 55, , (d) Not liable to pay any fee, 10. The due date of filing of return for a company with a business loss of ₹ 1,30,000 for A.Y. 2022-23, is–, (a) 31st July, 2022, (b) 30th September, 2022, (c) 31st October, 2022, (d) 31st August, 2022, 11. Mr. Karan filed his return of income for A.Y.2022-23 showing total income of ₹7 lakhs on, 1.12.2022. The fee payable by him under section 234F is –, (a) Nil, (b) ₹1,000, (c) ₹5,000, (d) ₹10,000, 12. Who can verify the return of income of a non-resident company?, (a) Managing Director of the company, (b) Any director of the company, (c) A person who holds a valid power of attorney from such company to do so, (d) Any of the above, 13. Which of the following returns can be revised under section 139(5)?, (a) Only a return of income filed u/s 139(1), (b) A return of income filed u/s 139(1) or a belated return filed u/s 139(4), (c) A return of income filed u/s 139(1) or a return of loss filed u/s 139(3), (d) A return filed u/s 139(1) or u/s 139(3) or u/s 139(4), 14. An assessee is allowed to file late return of income maximum, (a) Upto 31st Dec of assessment year, (b) upto three months prior to the end of assessment year, (c) Within 3 months from the last date of filing return of income, (d) None of these, 15. In case of defective return of income, assesse shall be allowed, (a) time period of 15 days from the date of intimation, (b) 20 days from the date of receiving the intimation, (c) within one month from the end of assessment year, (d) none of these, , Check Detailed Solution given on our website www.mkgeducation.com, (Icon /Heading No.11: Books)
Page 575 :
Provisions for filing of Return of Income, , 56, , EXAMINATION QUESTIONS, JULY – 2021 (NEW COURSE), Question 4(c), (4 Marks), Enumerate the cases where a return of loss has to be filed on or before the due date specifies u/s 139(1) for, carry forward of the losses. Also enumerate the case where losses can be carried forward even through the, return of loss has not been filed on or before the due date., Solution is given on our website www.mkgeducation.com (Icon /Heading No.11: Books), Question 4(c), In the following cases relating to P.Y. 2021-22, the total income of the assessee or the total income of any, other person in respect of which he/she is assessable under Income Tax Act does not exceed the basis, exemption limit. You are required to state with reasons, whether the assessee is still required to file the, return of income or loss for A.Y 2022-23 in each of the following independent situations:, (i), , Manish & Sons (HUF) sold a residential house on which there arose a long-term capital gain of ₹ 12, lakh which was invested in Capital Gain Bonds u/s 54EC so that no long-term capital gain was, taxable., , (ii), , (1½ Marks), , Mrs. Archana was born in Germany and married in India. Her residential status under section 6(6) of, the Income Tax Act, 1961 is ‘resident and ordinarily resident’. She owns a car in Germany which she, uses for her personal purposes during her visit to her parents’ place in that country., , (iii), , (1½ Marks), , Sudhakar has incurred an expenditure of ₹ 1,20,000 towards consumption of electricity, the entire, payment of which was made through banking channels., , (1 Marks), , Solution is given on our website www.mkgeducation.com (Icon /Heading No.11: Books), , JAN – 2021 (NEW COURSE), Question 3(b), (4 Marks), Mr. Hari aged 57 years is a resident of India. He provides you the following details of his incomes, pertaining to F.Y. 2021-22., - Interest on Non-Resident (External) Account maintained with State Bank of India as, per RBI stipulations, - ₹ 3,55,000, - Interest on savings bank account maintained with State Bank of India, - ₹ 8,000, - Interest on Fixed Deposits with Punjab National Bank, - ₹ 40,000, He seeks your advice on his liability to file return of income as per Income-tax Act, 1961 for the Assessment, Year 2022-23., What will be your answer, if he has incurred ₹ 4 lakhs on travel expenses of his newly married son and, daughter in law's honeymoon in Canada?, Solution is given on our website www.mkgeducation.com (Icon /Heading No.11: Books), , MAY – 2019 (NEW COURSE), Question 4 (b), (Marks 4), Discuss the provisions of Section 139A (1) which provides the persons who are compulsorily required to, apply for allotment of Permanent Account Number (PAN) with the assessing officer., Answer: Refer answer given in the book, OR, (i) What is the fee for default in furnishing return of income u/s 234F?, Answer: Refer answer given in the book
Page 576 :
Provisions for filing of Return of Income, , 57, , (ii) To whom the provision of section 139AA relating to quoting of Aadhar Number do not apply?, Answer: Refer answer given in the book, , NOV – 2018 (NEW COURSE), Question 7 (b), (Marks 4), Every person is required to file a return of income on or before due date in the prescribed form and manner, as per section 139 (1). What is the meaning of due date of filing Income Tax Returns for different categories, of assessees as per Section 139 (1) of the Income Tax Act 1961?, Answer: Refer answer given in the book, , NOV – 2018 (OLD COURSE), Question 6(b), (Marks 5), Mr. Subramaniam, due to inadvertent reasons failed to file his Income Tax return for assessment year 20212022., (i) Can he file the above return in-assessment year 2022-2023? If yes, when is the last date to file the above, return?, (ii) What are the consequences of non-filing the return within the due date under section 139(1)?, Answer: Refer answer given in the book, , MAY – 2018, Question 6(b), (Marks 5), Briefly mention the provisions of Income Tax Act with regard to Quoting Aadhaar Number u/s 139AA of, the Act., Answer: Refer answer given in the book, Question 6 (b), (Marks 5), (1) State whether quoting of PAN in the following transactions is mandatory or not, as per the, provisions of Income Tax Act 1961 for A.Y. 2022-23:, (i) Mr. A makes cash payment to a hotel Radisson Blu, Ahmedabad of ₹50,000 against the bill raised by the, hotel., (ii) Mr. Abhishek, in a single transaction, makes contract of ₹1,20,000 for sale /purchase of securities (other, than shares) as defined in section 2(h) of the Securities Contracts (Regulation ) Act, 1956., (iii) Payment to Mutual Funds of ₹70,000 for purchase of its units., Your answers must be supported with reasons., Answer:, (i) As per section 139A, quoting of PAN is mandatory in case of making cash payment of hotel or, restaurant bill exceeding ₹ 50,000. In the given case amount is not exceeding ₹50,000 hence quoting of, PAN is not mandatory., (ii) As per section 139A, quoting of PAN is mandatory in case of Sale/purchase of shares and securities, exceeding ₹1 lakh. In the given case amount is exceeding ₹1 lakh hence quoting of PAN is mandatory., (iii) As per section 139A, quoting of PAN is mandatory in case of Payment of an amount exceeding, ₹50,000 to a Mutual Fund for purchase of its units. In the given case amount is exceeding ₹50,000 hence, quoting of PAN is mandatory., (2) Briefly mention the concept of Self – Assessment tax u/s 140A of the IT Act and it components., Answer: Refer answer given in the book, Question 7 (a), (Marks 6), Indicate the three situations where the Return of Income has to be compulsorily filed u/s 139 (1) of the, Income Tax Act, 1961., Answer: Refer answer given in the book, Question 5 (b) (ii), (Marks 2), Explain the amount of fees to be paid for default in furnishing return of income under section 234F of the, Income Tax Act, 1961., Answer: Refer answer given in the book
Page 577 :
Provisions for filing of Return of Income, , 58, , Question 6 (c), (Marks 5), Pertaining to the following transactions, what is the minimum amount above which quoting of permanent, account number is mandatory?, (i), Sale or purchase of car., (ii), Payment to a hotel or restaurant against a bill or bills at any one time., (iii) Payment in connection with travel to any foreign country., (iv), Payment to the Reserve bank of India for acquiring bonds issued by it., (v), A Time Deposit with a Post Office., (vi), Payment as Life Insurance Premium to an insurer., (vii) Sale or purchase, of shares of a company not listed in a recognized stock exchange, (viii) Sale or purchase of any immovable property., Answer: Refer answer given in the book., , NOV – 2017, Question 5(b), (Marks 5), Mr. Sachin filed return on 30th September, 2022 related to Assessment Year 2022-23. In the month of, October 2022, his tax consultant found that the interest on fixed deposit was omitted in the tax return., (i) What is the time limit for filing a belated return?, (ii) Can Mr. Sachin file a revised return?, Justify the above with the relevant provisions under section 139. Assume that the due date for furnishing, return of income was 31st July, 2022 and the assessment was not completed till the month of October 2022., Answer:, (i) As per section 139(4), Any person who has not furnished a return within the time allowed to him under, section 139(1) may furnish the return for any previous year at any time three months prior to the end of the, relevant assessment year or before the completion of the assessment whichever is earlier., Therefore, in the given question, Mr. Sachin can file his belated return on or before 31st December, 2022., (ii) As per section 139(5), If any person having furnished a return u/s 139(1) or belated return u/s 139(4),, discover any omission or any wrong statement therein, he may furnish a revised return at any time three, months prior to the end of the relevant assessment year or before completion of assessment, whichever is, earlier hence, in the given case Mr. Sachin filed belated return as per Section 139(4) and he omitted his, Interest Income in the return hence he can revised his return on or before 31st December, 2022., , MAY – 2017, Question 7 (b), (4 Marks), By whom should the return of income be signed in the case of following persons:, (i) Political Party;, (ii) Company which is being wound up;, (iii) Hindu Undivided Family, when karta is unable to sign, and, (iv) Scientific research association., Answer:, As per section 140, Return should be signed by the authorised person, as given below:, (i) Political Party: In the case of a political party, Return can be signed by Chief Executive Officer., (ii) Company which is being wound up: If company is in liquidation, Return can be signed by the Official, liquidator., (iii) Hindu Undivided Family, when karta is unable to sign: In the case of a Hindu Undivided Family,, when karta is unable to sign then by any other adult member (male or female) of such family can verify, the return of Income., (iv) Scientific research association: Return can be signed by the Principal Officer and if Principal Officer, is not available, by any member.
Page 578 :
Provisions for filing of Return of Income, , 59, , NOV – 2016, Question 6(b), (2 Marks), Specify the persons who are authorized to verify u/s 140, the return of income filed u/s 139 of the Income, Tax Act, 1961 in case of a company., Answer: Refer answer given in the book, Question 7(a), (2 + 2= 4 Marks), (1) Mr. Kamal filed his Return of Income for the Assessment Year 2022-23 on 30-11-2022. Can he revise, such return of income? If so Why?, Answer: As per Section 139(5), Return filed under section 139(1) or Return filed u/s 139(4) can be revised, hence a belated return filed under section 139(4) can be revised and further return can be revised at any time, three months prior to the end of the relevant assessment year i.e. return can be revised upto 31.12.2022., (2) Mr. Atal, a super senior citizen, has reported a Gross Total Income ₹5,60,000 and the deductions eligible, under Chapter VI-A amounting to ₹70,000 for the previous year 2021-22. Is he liable to file his return of, income u/s 139(1) for the Assessment year 2022-23? If so Why?, Answer: Individual shall be required to file return of income if Gross total income, before claiming the, exemption under section 54, 54B, 54D, 54EC, 54F, 54G, 54GA, and 54GB, is exceeding exemption limit., In the given case assessee is a super senior citizen and exemption applicable is ₹5,00,000 and Gross Total, Income, before claiming the exemption under section 54, 54B, 54D, 54EC, 54F, 54G, 54GA, and 54GB,, is exceeding ₹5,00,000 So, Mr. Atal is required to file his Return of Income., , NOV – 2015, Question 7(a)., (2 Marks), Who are the persons authorized to verify return of income in the case of individual under section 140 of the, Income Tax Act, 1961?, Solution:, As per section 140 the return shall be verified—, In the case of an individual, the return should be verified by the individual himself but if for any reason, return cannot be verify by the individual, return can be verified by his agent and the agent should enclose, copy of power of attorney with the return., If any individual is mentally incapacitated from attending to his affairs, return should be verified by his, guardian or any other person competent to act on his behalf., , MAY – 2015, Question 7(a)(i)., (2 Marks), Explain the term "return of loss'' under the Income-tax Act, 1961. Can any loss be carried forward even if, return of loss has not been filed as required?, Answer: Refer answer given in the Chapter, , NOV – 2014, Question 7(a), (i) “Filing of Return of Income on or before due date is necessary for carry forward of losses.” Discuss the, correctness of this statement., (4 Marks), Answer: Refer answer given in the book, , MAY – 2014, Question ., (2 x 2 = 4 Marks), (A) Mr. X submits his return of income on 12.09.2022 for AY 2022-23 consisting of income under the head, house property and other sources. On 21.12.2022, he realized that he had not claimed deduction under, section 80TTA in respect of his interest income on the Savings Bank Account. He wants to revise his return, of income, since one year has not elapsed from the end of the relevant Previous year Year. Discuss.
Page 579 :
Provisions for filing of Return of Income, , 60, , (B) Where the Karta of an Hindu undivided family is absent from India, the return of income can be signed, by any male member of the family? Give reasoning for the statement to be true or false., Solution:, (A) Since Mr. X has income only under the heads “Income from house property” and “Income from other, sources”, he does not fall under the category of a person whose accounts are required to be audited under the, Income-tax Act, 1961 or any other law in force. Therefore, the due date of filing return for A.Y.2022-23, under section 139(1), in his case, is 31st July, 2022. Since Mr. X had submitted his return only on, 12.09.2022, the said return is a belated return under section 139(4)., As per section 139(5), a return furnished under section 139(1) or u/s 139(4) can be revised. Therefore, Mr. X, can revise the return of income filed by him under section 139(4), to claim deduction under section 80TTA., (B) The statement is false., As per section 140, where the karta of a HUF is absent from India, any other adult member of the HUF, can, sign the return of income., Thus, a male member who is not an adult cannot sign the return of income. An adult member, whether male, or female, can sign the return of income., , NOV – 2012, Question No. 7(a), (4 Marks), Enumerate the circumstances in which an individual assessee is empowered to verify his return of income, u/s 140 by himself or otherwise by an authorised signatory., Answer: Refer answer given in the Chapter, , MAY – 2011, Question 7, (4 Marks), Specify the persons who are authorized to verify under section 140, the return of income filed under section, 139 of the Income-tax Act, 1961 in the case of:, (i) Political party;, (ii) Local authority;, (iii) Association of persons, and, (iv) Limited Liability Partnership (LLP), Answer: Refer answer given in the Chapter, , MAY – 2010, Question 4, (4 Marks), What are the particulars required to be furnished with the return of income, as per section 139(6)?, Answer: Refer answer given in the book, , NOV – 2008, Question 5, (4 Marks), Explain with brief reason whether the return of income can be revised under section 139(5) of the Incometax Act, 1961 in the following cases:, (i) Defective or incomplete return filed under section 139(9)., (ii) Belated return filed under section 139(4)., (iii) Return already revised once under section 139(5)., (iv) Return of loss filed under section 139(3)., Answer., Any person who has furnished a return under section 139(1) or under section 139(4) can file a revised return, if he discovers any omission or any wrong statement in the return filed earlier. Accordingly:(i) A defective or incomplete return filed under section 139(9) cannot be revised. However, the defect can be, removed., (ii) A belated return filed under section 139(4) can be revised.
Page 580 :
Provisions for filing of Return of Income, , 61, , (iii) An assessee is allowed to revise the return any number of times but if earlier return has already been, assessed, further revision is not allowed., (iv) A return of loss filed under section 139(3) is deemed to be return filed under section 139(1), and, therefore, can be revised under section 139(5)., , NOV – 2007, Question 5, Enumerate eight transactions for which quoting of Permanent Account Number is mandatory., Answer: Refer answer given in the book, , (4 Marks), , MAY – 2007, Question 1, (2 Marks), The Assessing Officer has the power, inter alia, to allot PAN to any person by whom no tax is payable., Discuss., Answer: True: As per section 139A, The Assessing Officer may having regard to the nature of transactions, as may be prescribed also allot a PAN to any other person, whether any tax is payable by him or not, in the, manner and in accordance with the procedure as may be prescribed., Question 1, (2 Marks), Where the Karta of an HUF is absent from India, the return of income can be verified by any male member, of the family., Answer: False: Section 140 provides that where the Karta of a HUF is absent from India, the return of, income can be verified by any other adult member of the family; such member can be a male or female, member., Question 5, (4 Marks), Discuss briefly about the scheme to facilitate submission of return of income through Tax Return Preparers., Answer: Refer answer given in the book
Page 581 :
Income Under The Head Capital Gains, , 62, , INCOME UNDER THE HEAD, CAPITAL GAINS, SECTION 45 TO 55A, PARTICULARS, Meaning of capital asset, Transfer of capital assets, Capital gains in case of insurance claims, Taxability of amount received under Unit Link Insurance Policy, Transfer of capital assets into stock-in-trade, Transfer of securities by a depository, Transfer of a capital asset by way of compulsory acquisition., Transfer of capital Asset under specified agreement., Capital gains on distribution of assets by companies in liquidation, Capital Gains in case of buy back of shares., Transactions not regarded as transfer, Mode of computation, Cost with reference to certain modes of acquisition, Special provision for computation of capital gains in case of depreciable assets, Special provision for computation of capital gains in case of slump sale, Special provision for full value of consideration in certain cases, Full value of consideration for transfer of unlisted shares, Fair market value deemed to be full value of consideration in certain cases, Exemption from capital gains on transfer of property used for residence, Exemption from capital gain on transfer of land used for agricultural purposes, Exemption from capital gains on compulsory acquisition of lands and buildings, of industrial undertaking., Exemption from capital gain on transfer of any capital asset, Capital gain not to be charged on investment in units of a specified fund., Exemption from capital gain on transfer of long term capital assets except, residential house, Determination of time period in case of compulsory acquisition of capital asset, Meaning of “cost of improvement”, Meaning of “cost of acquisition”, Reference to valuation officer, Computation of tax on long term capital gains, Computation of tax on short term capital gains in certain cases, Computation of tax on long term capital gains in certain cases, What is transfer, Short-term capital asset, Capital gains on compulsory acquisition of agricultural land, , SECTIONS, 2(14), 45(1), 45(1A), 45(1B), 45(2), 45(2A), 45(5), 45 (5A), 46, 46A, 47, 48, 49, 50, 50B, 50C, 50CA, 50D, 54, 54B, 54D, 54EC, 54EE, 54F, 54H, 55(1), 55(2), 55A, 112, 111A, 112A, 2(47), 2(42A), 10(37), , Question 1: Explain Chargeability of Capital Gains, Answer: Chargeability of capital Gains Section 45(1), Any profits or gains arising from the transfer of a capital asset effected in the previous year shall be deemed, to be the income of the previous year in which the transfer took place e.g. Mr. X transferred Gold on
Page 582 :
Income Under The Head Capital Gains, , 63, , 25.03.2022 for ₹7,00,000 but payment was received on 10.04.2022, in this case capital gains shall be taxable, in the previous year 2021-22 i.e. capital gains are taxable on due basis. No books of accounts are required., Question 2: Differentiate Short Term Capital Asset and Long Term Capital Asset., Answer: If any person has transferred short term capital asset, capital gain shall be short term and if capital, asset transferred is long term, capital gain shall also be long term. As per Section 2(42A), “Short-term, capital asset” means a capital asset held by an assessee for not more than thirty-six months, however in the, following cases the period shall be twelve months instead of thirty-six months., (i) Shares Listed in Recognised Stock Exchange shall be considered to be long term after one year but, non-listed shares shall be long term after two years. E.g. Mr. X purchased unlisted equity shares on, 01.07.2020 and sold the shares on 01.08.2021, in this case shares are short term but if shares are listed,, shares shall be long term., (ii) A unit of the Unit Trust of India, (iii) A unit of an equity oriented mutual fund., (iv) A zero coupon bond. As per Section 2(48), “Zero coupon bond” means a bond issued by notified, company and in respect of which no benefit is received before maturity or redemption and which is, notified by the Central Government such bonds are issued for a minimum period of ten years and, maximum period of 20 years., (v) Any other security listed in a recognized stock exchange in India i.e. securities which are listed in, recognised stock exchange shall be long term after one year but if securities are not listed, it will be, long term after three years. e.g. Mr. X purchased non-listed debentures of ABC Ltd. on 01.10.2019 and, sold the debentures on 01.10.2021, in this case debentures shall be considered to be short term but if the, debentures are listed in stock exchange, they will be considered to be long term., Amendment previous year 2017-18: Land or Building shall be long term after 2 years., Question 3: Write a note on computation of Short term Capital Gains., Answer: Computation of Short term Capital Gains Section 48, Short term capital gain shall be computed in the manner given below:, Full Value of Consideration, xxx, Less:, - Cost of Acquisition, xxx, - Cost of Improvement, xxx, - Selling Expenses, xxx, Short Term Capital Gain, xxx, Example: Mr. X purchased one house 01-07-2019 ₹10,00,000 and constructed its first floor 01-07-2020 by, incurring ₹6,00,000 and sold the house on 01-05-2021 ₹70,00,000 and invested ₹2,00,000 in NSC. Compute, Income and Tax Liability., Answer: House is sold within 2 years from the date of purchase hence asset is a Short term capital asset and, capital gain shall be computed in the manner given below:, ₹, Full Value of Consideration, 70,00,000.00, Less:, - Cost of Acquisition, (10,00,000.00), - Cost of Improvement, (6,00,000.00), - Selling Expenses, Nil, Short Term Capital Gain, 54,00,000.00, Gross Total Income, 54,00,000.00, Less: Deduction u/s 80C, (1,50,000.00), Total Income, 52,50,000.00, Computation of Tax Liability, Tax on STCG ₹52,50,000 at slab rate, 13,87,500.00, Add: Surcharge @ 10%, 1,38,750.00, Tax before health & education cess, 15,26,250.00
Page 583 :
Income Under The Head Capital Gains, Add: HEC @ 4%, Tax liability, , 64, 61,050.00, 15,87,300.00, , Question 4: Write a note on computation of Long term Capital Gains., Answer: Computation of Long term Capital Gains Section 48, Long term capital Gain shall also be computed in the similar manner but instead of cost of acquisition and, cost of improvement, indexed cost of acquisition and indexed cost of improvement shall be taken into, consideration., Long term capital Gain shall be computed in the manner given below:, Full Value of Consideration, Less:, - Indexed Cost of Acquisition, - Indexed Cost of Improvement, - Selling Expenses, Long Term Capital Gain, , xxx, xxx, xxx, xxx, xxx, , “Indexed cost of acquisition” means the cost adjusted as per cost inflation index i.e., Indexed Cost of acquisition =, Cost of acquisition x Index of the year in which the asset was transferred, Index of the year in which the asset was purchased, “Indexed cost of any improvement” means the cost adjusted as per cost inflation index i.e., Indexed Cost of improvement =, Cost of improvement x Index of the year in which the asset was transferred, Index of the year in which cost was incurred, Example: Mr. X purchased one house 01-07-2017 ₹10,00,000 and constructed its first floor 01-07-2018 by, incurring ₹6,00,000 and sold the house on 01-05-2021 ₹80,00,000 and invested ₹2,00,000 in NSC. Compute, Income and Tax Liability., Answer: House is sold after 2 years from the date of purchase hence asset is a Long term capital asset and, capital gain shall be computed in the manner given below:, ₹, Full Value of Consideration, 80,00,000.00, Less: Indexed cost of acquisition, = 10,00,000 / Index of 17-18 x Index of 21-22, = 10,00,000 / 272 x 317 = ₹11,65,441.18, (11,65,441.18), Less: Indexed cost of improvement, Cost of constructing first floor, = 6,00,000 / Index of 18-19 x Index of 21-22, = 6,00,000 / 280 x 317 = ₹6,79,285.71, (6,79,285.71), Long Term Capital Gain, 61,55,273.11, Gross Total Income, 61,55,273.11, Less: Deduction u/s 80C to 80U, Nil, Total Income, 61,55,273.11, Rounded off u/s 288A, 61,55,270.00, Computation of Tax Liability, Tax on LTCG ₹59,05,270 (₹61,55,270 – ₹2,50,000) @ 20%, 11,81,054.00, Add: Surcharge @ 10%, 1,18,105.40, Tax before health & education cess, 12,99,159.40, Add: HEC @ 4%, 51,966.38, Tax liability, 13,51,125.78, Rounded off u/s 288B, 13,51,130.00
Page 584 :
Income Under The Head Capital Gains, , 65, , Illustration 1: Mr. X purchased one house on 01.07.2004 for ₹3,50,000. He constructed its first floor on, 01.10.2013 by incurring ₹4,00,000 and constructed its second floor on 01.10.2014 by incurring ₹6,00,000, and third floor on 01.10.2016 by incurring ₹7,00,000. Finally, sold the building on 01.01.2022 for, ₹120,00,000 and selling expenses were 2% of the sale price. He has deposited ₹1,00,000 in NSC., Compute tax liability of the assessee for the assessment year 2022-23., Solution:, ₹, Computation of Capital Gains, Full value of consideration, 120,00,000.00, Less: Indexed cost of acquisition, = 3,50,000 / Index of 04-05 x Index of 21-22, = 3,50,000 / 113 x 317 = ₹9,81,858.41, (9,81,858.41), Less: Indexed cost of improvement, Cost of constructing first floor, = 4,00,000 / Index of 13-14 x Index of 21-22, = 4,00,000 / 220 x 317 = ₹5,76,363.64, (5,76,363.64), Less: Indexed cost of improvement, Cost of constructing second floor, = 6,00,000 / Index of 14-15 x Index of 21-22, = 6,00,000 / 240 x 317 = ₹7,92,500.00, (7,92,500.00), Less: Indexed cost of improvement, Cost of constructing third floor, = 7,00,000 / Index of 16-17 x Index of 21-22, = 7,00,000 / 264 x 317 = ₹8,40,530.30, (8,40,530.30), Less: Selling Expenses, = 2% of ₹120,00,000 = ₹2,40,000, (2,40,000.00), Long Term Capital Gain, 85,68,747.65, Income under the head Capital Gain (LTCG), 85,68,747.65, Gross Total Income, 85,68,747.65, Less: Deduction u/s 80C to 80U, Nil, Total Income (Rounded off u/s 288A), 85,68,750.00, Computation of Tax Liability, Tax on LTCG ₹83,18,750 (₹85,68,750 – ₹2,50,000) @ 20%, 16,63,750.00, Add: Surcharge @ 10%, 1,66,375.00, Tax before health & education cess, 18,30,125.00, Add: HEC @ 4%, 73,205.00, Tax liability, 19,03,330.00, Question 5: Explain Computation of Capital Gains in case of Assets purchased before 01-04-2001, Answer: Asset purchased before 01.04.2001, If any capital asset has been purchased or constructed before 01.04.2001, in that case cost shall be, considered to be the cost incurred or fair market value of the asset as on 01.04.2001 whichever is higher and, further index of 2001-02 shall be used instead of the index of the earlier year. Any cost of improvement, prior to 01-04-2001 shall not be taken into consideration., In case of land or building such market value cannot exceed stamp duty value as on 01.04.2001., e.g. Mr. X purchased one house on 01.07.1998 for ₹2,00,000 and incurred ₹3,00,000 on its improvement on, 01.10.2000 and its fair market value as on 01.04.2001 is ₹7,00,000 and stamp duty value is 9,00,000, in this, case if the asset is sold, its cost of acquisition shall be taken to be ₹7,00,000 and index of 2001-02 shall be, applied. But if stamp duty value is 5,00,000, cost of acquisition shall be taken to be 5,00,000 but if stamp, duty value is 1,00,000, cost of acquisition shall be 2,00,000.
Page 585 :
Income Under The Head Capital Gains, Example:, (a) Actual cost of acquisition, FMV as on 01-04-2001, SDV as on 01-04-2001, Cost of Acquisition is higher of, (a) Actual Cost, (b) Lower of FMV or SDV as on 1-04-2001, , 66, 2,00,000, 4,00,000, 3,00,000, , 2,00,000, 3,00,000, , Higher, , 3,00,000, , (b) Actual cost of acquisition, FMV as on 01-04-2001, SDV as on 01-04-2001, Cost of Acquisition is higher of, (a) Actual Cost, (b) Lower of FMV or SDV as on 1-04-2001, , 2,00,000, 3,00,000, 4,00,000, 2,00,000, 3,00,000, , Higher, , 3,00,000, , (c) Actual cost of acquisition, FMV as on 01-04-2001, SDV as on 01-04-2001, Cost of Acquisition is higher of, (a) Actual Cost, (b) Lower of FMV or SDV as on 1-04-2001, , 4,00,000, 3,00,000, 2,00,000, 4,00,000, 2,00,000, , Higher, , 4,00,000, Cost Inflation Index, Financial year, Cost Inflation Index, 2001-2002, 100, 2002-2003, 105, 2003-2004, 109, 2004-2005, 113, 2005-2006, 117, 2006-2007, 122, 2007-2008, 129, 2008-2009, 137, 2009-2010, 148, 2010-2011, 167, 2011-2012, 184, 2012-2013, 200, 2013-2014, 220, 2014-2015, 240, 2015-2016, 254, 2016-2017, 264, 2017-2018, 272, 2018-2019, 280, 2019-2020, 289, 2020-2021, 301, 2021-2022, 317, (Students need not learn the above index rather it will be given in the question paper)
Page 586 :
Income Under The Head Capital Gains, , 67, , Illustration 2: Compute capital gains and tax liability of Mr. X in the following Individual situations for the, assessment year 2022-23:, Asset, Gold, Land, Residential House, Date of purchase, 01.07.1990, 01.04.1992, 01.07.1994, Cost price, 4,00,000, 6,00,000, 8,00,000, Cost of improvement, 1,00,000, 2,00,000, 4,00,000, Year of improvement, 1999-2000, 2000-01, 2005-06, Fair market value on 01.04.2001, 30,00,000, 60,00,000, 5,00,000, Date of Sale, 01.01.2022, 01.01.2022, 01.01.2022, Full value of consideration, 200,00,000, 300,00,000, 400,00,000, Solution:, ₹, Gold, Full value of consideration, 200,00,000.00, Less: Indexed cost of acquisition, = 30,00,000 / Index of 01-02 x Index of 21-22, = 30,00,000 / 100 x 317 = 95,10,000, (95,10,000.00), Long term capital gain, 104,90,000.00, Computation of Tax Liability, Tax on LTCG ₹102,40,000 (₹104,90,000 – ₹2,50,000) @ 20%, 20,48,000.00, Add: Surcharge @ 15%, 3,07,200.00, Tax before health & education cess, 23,55,200.00, Add: HEC @ 4%, 94,208.00, Tax liability, 24,49,408.00, Rounded off u/s 288B, 24,49,410.00, Land, Full value of consideration, 300,00,000.00, Less: Indexed cost of acquisition, = 60,00,000 / Index of 01-02 x Index of 21-22, = 60,00,000 / 100 x 317 = 190,20,000, (190,20,000.00), Long term capital gain, 109,80,000.00, Computation of Tax Liability, Tax on LTCG ₹107,30,000 (₹109,80,000 – ₹2,50,000) @ 20%, 21,46,000.00, Add: Surcharge @ 15%, 3,21,900.00, Tax before health & education cess, 24,67,900.00, Add: HEC @ 4%, 98,716.00, Tax liability, 25,66,616.00, Rounded off u/s 288B, 25,66,620.00, Residential House, Full value of consideration, Less: Indexed cost of acquisition, = 8,00,000 / Index of 01-02 x Index of 21-22, = 8,00,000 / 100 x 317 = 25,36,000.00, Less: Indexed cost of improvement, = 4,00,000 / Index of 05-06 x Index of 21-22, = 4,00,000 / 117 x 317 = 10,83,760.68, Long term capital gains, Gross Total Income, Less: Deduction u/s 80C to 80U, Total Income, Rounded off u/s 288A, , 400,00,000.00, (25,36,000.00), (10,83,760.68), 363,80,239.32, 363,80,239.32, Nil, 363,80,239.32, 363,80,240.00
Page 587 :
Income Under The Head Capital Gains, Computation of Tax Liability, Tax on LTCG ₹361,30,240 (₹363,80,240 – ₹2,50,000) @ 20%, Add: Surcharge @ 25%, Tax before health & education cess, Add: HEC @ 4%, Tax liability, Rounded off u/s 288B, , 68, 72,26,048.00, 18,06,512.00, 90,32,560.00, 3,61,302.40, 93,93,862.40, 93,93,860.00, , Illustration 3: Mr. X purchased one house property on 01.07.1992 for ₹3,00,000 and incurred ₹1,00,000 on, its improvement in 1995-96 and its market value as on 01.04.2001 was ₹32,00,000 and he incurred, ₹5,00,000 on its improvement in 2014-15 and sold the house on 01.11.2021 for ₹200,00,000., He purchased one commercial building on 01.04.2020 for ₹50,00,000 and it was let out @ ₹2,00,000 p.m. to, XYZ Ltd. and XYZ Ltd. has deducted tax at source., Mr. X has paid Municipal Tax of ₹20,000 p.m., Compute Income Tax Liability/Payable for Assessment Year 2022-23 and also amount of tax deducted at, source by XYZ Ltd. and also tax deducted at source by the person who has purchased the house property., Solution:, ₹, Computation of income from Capital Gain, Full value of consideration, 200,00,000.00, Less: Indexed Cost of acquisition, = 32,00,000/Index of 01-02 x Index of 21-22, = 32,00,000/100 x 317 = ₹101,44,000, (101,44,000.00), Less: Indexed cost of Improvement, = 5,00,000/Index of 14-15 x Index of 21-22, = 5,00,000/240 x 317 = ₹6,60,416.67, (6,60,416.67), Long Term Capital Gain, 91,95,583.33, Computation of income under head House Property, Gross Annual Value (2,00,000 x 12), 24,00,000.00, Less: Municipal Tax (20,000 x 12), (2,40,000.00), Net Annual Value, 21,60,000.00, Less: 30% of NAV u/s 24(a), (6,48,000.00), Less: Interest on capital borrowed u/s 24(b), Nil, Income under the head House Property, 15,12,000.00, Computation of Gross Total Income, Income under the head House Property, 15,12,000.00, Income from Long Term Capital Gain, 91,95,583.33, Gross Total Income, 107,07,583.33, Less: Deduction u/s 80C to 80U, Nil, Total Income (Rounded off u/s 288A), 107,07,580.00, Normal Income, LTCG, Computation of Tax Liability, Tax on LTCG ₹91,95,580 @ 20%, Tax on normal income ₹15,12,000 at slab rate, Tax before Surcharge, Add: Surcharge @ 15%, Tax before health & education cess, Add: HEC @ 4%, Tax liability, Less: TDS (10% of 24,00,000) under section 194-I, Less: TDS (1% of 200,00,000) under section 194-IA, Tax Payable, , 15,12,000.00, 91,95,580.00, 18,39,116.00, 2,66,100.00, 21,05,216.00, 3,15,782.40, 24,20,998.40, 96,839.94, 25,17,838.34, (2,40,000.00), (2,00,000.00), 20,77,838.34
Page 588 :
Income Under The Head Capital Gains, Rounded off u/s 288B, , 69, 20,77,840.00, , Question 6: Write a note on computation of capital gains in case of transfer of shares, where no STT is, paid., Answer: Capital gains in case of transfer of shares, In case of original shares, cost of acquisition shall be the amount for which the asset was purchased but if it, was purchased before 01.04.2001, cost of acquisition shall be the amount for which it was purchased or its, market value as on 01.04.2001 whichever is higher. In case of bonus shares, cost of acquisition shall be nil, but if bonus shares are issued before 01.04.2001, cost of acquisition shall be the market value as on, 01.04.2001., In case of right shares, cost of acquisition shall be the amount for which such shares have been purchased. If, right to purchase right shares has been renounced, amount received shall be considered to be short term, capital gains. Cost of acquisition for the right renouncee shall be the amount paid to the person renouncing, the right and amount paid to the company., Long term capital gain shall be taxable @ 20% but short term capital gain shall be taxable at normal rate., In case of short term equity shares or the units, capital gains shall be computed but as per section 111A, such, capital gains shall be taxed @ 15%., Illustration 4: Mr. X purchased 100 equity shares in ABC Ltd. on 01.10.1995 @ ₹10 per share. The, company has issued 100 bonus shares on 01.10.1998 and market value of the shares on 01.04.2001 was ₹7, per share. The company has again issued 100 bonus shares on 01.10.2013., The company has offered 100 right shares on 01.04.2021 @ ₹140 per share though the market value is ₹250, per share. Mr. X purchased half of the shares and remaining half were renounced by him in favour of his, friend Mr. Y. He has charged ₹20 per share from Mr. Y for renouncing the right., All the shares were sold by Mr. X and Mr. Y @ ₹300 per share on 01.01.2022, Mr. X has income under the head house property ₹2,20,000 and has causal income ₹50,000 and has invested, ₹1,00,000 in NSC., Mr. Y has income under the head house property ₹3,50,000 and has invested ₹30,000 in NSC., Compute tax liability of Mr. X and Mr. Y. (No STT was paid), Solution:, ₹, Computation of Capital Gains of Mr. X, Original Shares, Full value of consideration, 30,000.00, (100 x 300), Less: Indexed Cost of Acquisition, (3,170.00), (100 x 10)/100 x 317, Long Term Capital Gain, 26,830.00, 1st Bonus Shares, Full value of consideration, 30,000.00, (100 x 300), Less: Indexed Cost of Acquisition, (2,219.00), (100 x 7)/100 x 317, Long Term Capital Gain, 27,781.00, 2nd Bonus Shares, Full value of consideration, 30,000.00, (100 x 300), Less: Indexed Cost of Acquisition, Nil, Long Term Capital Gain, 30,000.00, Right Shares, Full value of consideration, 15,000.00, (50 x 300), Less: Cost of Acquisition, (7,000.00), (50 x 140)
Page 589 :
Income Under The Head Capital Gains, Short Term Capital Gain, Renouncing of right to purchase shares, Full value of consideration, Less: Cost of acquisition, Short Term Capital Gain, Computation of Total Income, Income under the head House Property, Income under the head Capital Gains, Long Term Capital Gain, Short Term Capital Gain, Income under the head Other Sources, Gross Total Income, Less: Deduction u/s 80C, Total Income, Rounded off u/s 288A, Computation of Tax Liability, Tax on casual income ₹50,000 @ 30%, Tax on (84,610 – 84,610) @ 20% u/s 112, Tax on ₹1,29,000 at slab rate, Less: Rebate u/s 87A, Tax before health & education cess, Add: HEC @ 4%, Tax Liability, , 70, 8,000.00, 1,000.00, Nil, 1,000.00, 2,20,000.00, 84,611.00, 9,000.00, 50,000.00, 3,63,611.00, (1,00,000.00), 2,63,611.00, 2,63,610.00, 15,000.00, Nil, Nil, (12,500.00), 2,500.00, 100.00, 2,600.00, , Mr. Y, Full value of consideration, Less: Cost of acquisition (50 x 160), Short Term Capital Gain, Computation of Total Income, Income under the head House Property, Income under the head Capital Gains, Gross Total Income, Less: Deduction u/s 80C, Total Income, Computation of Tax Liability, Tax on ₹3,27,000 at slab rate, Less: Rebate u/s 87A, Tax Liability, , 15,000.00, (8,000.00), 7,000.00, 3,50,000.00, 7,000.00, 3,57,000.00, (30,000.00), 3,27,000.00, 3,850.00, (3,850.00), Nil, , Illustration 5: Mr. X holds 500 shares of ABC Ltd. which were allotted to him on 22.04.2001 @ ₹30 per, share. On 22.07.2021 ABC Ltd. made right issue to the existing shareholders at the rate of one share for, every five shares held @ ₹20 per share. Mr. X instead of exercising his rights to obtain right shares, has, exercised his right of renouncement by renouncing the said right entitlement in favour of Mr. Y @ ₹13 per, right share entitlement on 04.08.2021., (a) Determine the nature and amount of capital gain, if any, taxable in the hands of Mr. X., (b) What will be the cost of acquisition of shares purchased by Mr. Y?, Solution:, ₹, Computation of Capital Gains in the hands of Mr. X, Full value of consideration, 1,300, (100 x 13), Less: Cost of acquisition, Nil, Short Term Capital Gain, 1,300, Cost of acquisition of shares purchased by Mr. Y = ₹33 x 100 = ₹3,300
Page 590 :
Income Under The Head Capital Gains, , 71, , Illustration 6: Mr. X is a shareholder of ABC Ltd. holding 1,000 shares of the face value of ₹10 each. The, company made a right issue in the ratio of 1:1 on 01.01.2022 at a premium of ₹50 per share. He renounced it, in favour of Mr. Y at a price of ₹10 per share., What is the capital gain chargeable in the hands of Mr. X? What will be the cost of the shares in the hands of, Mr. Y?, Solution:, ₹, Computation of Capital Gains in the hands of Mr. X, Full value of consideration, 10,000, (1,000 x 10), Less: Cost of acquisition, Nil, Short Term Capital Gain, 10,000, Cost of the shares in the hands of Mr. Y is ₹70 per share., Question 7: Explain computation of capital gains on transfer of equity shares or units of equity, oriented where STT has been paid u/s 112A, Answer: In case of long term equity shares or long term units of equity oriented mutual funds or units of, business trust, capital gains shall be computed as per section 112A provided securities transaction tax has, been paid, such capital gains shall be taxed @ 10% in excess of ₹ 1,00,000 and while computing capital, gains u/s 112A indexation is not applicable. STT paid is not considered to be selling expense, hence not, allowed to be deducted while computing capital gains., Capital gains shall be computed in the normal manner however indexation shall not be applicable even if it, is long term and also cost of acquisition shall be computed in the manner given below:, Section 55(2) (ac),, In case of equity shares or units of equity oriented mutual funds or units of business trust which have been, sold w.e.f. 01.04.2018 onwards, cost of acquisition shall be higher of, 1. Cost of acquisition, 2. Lower of, (a) Fair market value of such asset on 31.01.2018, (b) Actual sale value, If there are more than one value as on 31.01.2018, the highest of such value shall be taken in to, consideration. In case of mutual fund which is not listed, NAV i.e. Net asset value shall be taken into, consideration., Example, Mr. X purchased equity shares on 01.10.2015 for ₹ 1,00,000 and market value on 31.01.2018 is ₹ 5,00,000, and shares have been sold for ₹ 9,00,000 on 10.04.2021, in this case capital gains shall be computed in the, manner given below:, Full value of consideration, 9,00,000, Cost of acquisition, , (5,00,000), , Higher of, 1. Cost of acquisition, 2. Lower of, , 1,00,000
Page 591 :
Income Under The Head Capital Gains, , 72, , (a) Fair market value of such asset on 31.01.2018 5,00,000, (b) Actual sale value, , 9,00,000, , LTCG u/s 112A, , 4,00,000, , Tax Liability shall be, Tax on LTCG u/s 112A (4,00,000 – 1,00,000 = 3,00,000), (3,00,000 – 2,50,000) x 10%, , 5,000, , Rebate u/s 87A is not allowed, Add: HEC @ 4%, , 200, , Tax Liability, , 5,200, , The purpose is not to tax capital gains accrued upto 31.01.2018, Presume in the above question the shares have been sold for ₹ 3,00,000, in this case tax treatment shall, be, Full value of consideration, , 3,00,000, , Cost of acquisition, , (3,00,000), , Higher of, 1. Cost of acquisition, , 1,00,000, , 2. Lower of, (a) Fair market value of such asset on 31.01.2018 5,00,000, (b) Actual sale value, , 3,00,000, , LTCG u/s 112A, , Nil, , Tax Liability shall be, , Nil, , The purpose is not to compute any loss if there is decrease in value after 31.01.2018, Presume in the above case shares have been sold for ₹ 40,000, tax treatment shall be, Full value of consideration, , 40,000, , Cost of acquisition, , (1,00,000), , Higher of, 1. Cost of acquisition, , 1,00,000, , 2. Lower of, (a) Fair market value of such asset on 31.01.2018 5,00,000
Page 592 :
Income Under The Head Capital Gains, (b) Actual sale value, , 73, , 40,000, , Loss u/s 112A, Tax Liability shall be, , (60,000), Nil, , The purpose is to allow loss with regard to original cost., Illustration 7: Mr. X purchased 100 equity shares in ABC Ltd. on 01.10.1995 @ ₹10 per share. The, company has issued 100 bonus shares on 01.10.1998 and market value of the shares on 01.04.2001 was ₹7, per share. The company has again issued 100 bonus shares on 01.10.2013., The company has offered 100 right shares on 01.04.2021 @ ₹140 per share though the market value is ₹250, per share. Mr. X purchased half of the shares and remaining half were renounced by him in favour of his, friend Mr. Y. He has charged ₹20 per share from Mr. Y for renouncing the right., All the shares were sold by Mr. X and Mr. Y @ ₹300 per share on 01.01.2022 and securities transaction tax, has been paid. (market value on 31-01-2018 is ₹200 per share), Mr. X has income under the head house property ₹2,20,000 and has causal income ₹50,000 and has invested, ₹1,00,000 in NSC., Mr. Y has income under the head house property ₹3,50,000 and has invested ₹30,000 in NSC., Compute tax liability of Mr. X and Mr. Y., Solution:, ₹, Computation of Capital Gains of Mr. X, Original Shares, Full value of consideration, 30,000.00, (100 x 300), Less: Cost of Acquisition, (20,000.00), Higher of, (i) COA = 100 x 10 = 1,000, (ii) lower of, (a) FMV as on 31-01-2018 = 100 x 200 = 20,000, (b) sale value = 100 x 300 = 30,000, COA = 20,000, Long Term Capital Gain u/s 112A, 1st Bonus Shares, Full value of consideration, (100 x 300), Less: Cost of Acquisition, Higher of, (i) COA (7 x 100) = 700, (ii) lower of, (a) FMV as on 31-01-2018 = 100 x 200 = 20,000, (b) sale value = 100 x 300 = 30,000, COA = 20,000, Long Term Capital Gain u/s 112A, 2nd Bonus Shares, Full value of consideration, (100 x 300), Less: Cost of Acquisition, Higher of, (i) COA = Nil, , 10,000.00, 30,000.00, (20,000.00), , 10,000.00, 30,000.00, (20,000.00)
Page 593 :
Income Under The Head Capital Gains, (ii) lower of, (a) FMV as on 31-01-2018 = 100 x 200 = 20,000, (b) sale value = 100 x 300 = 30,000, COA = 20,000, Long Term Capital Gain u/s 112A, Right Shares, Full value of consideration, (50 x 300), Less: Cost of Acquisition, (50 x 140), Short Term Capital Gain u/s 111A, Renouncing of right to purchase shares, Full value of consideration, Less: Cost of acquisition, Short Term Capital Gain, Computation of Total Income, Income under the head House Property, Income under the head Capital Gains, Long term capital gains 112A, Short term capital gains 111A, Short term capital gains, Income under the head Capital Gains, , 74, , 10,000.00, 15,000.00, (7,000.00), 8,000.00, 1,000.00, Nil, 1,000.00, 2,20,000.00, 30,000.00, 8,000.00, 1,000.00, 9,000.00, , Income under the head Other Sources, Gross Total Income, Less: Deduction u/s 80C, Total Income, Computation of Tax Liability, Tax on casual income ₹50,000 @ 30%, Tax on (30,000 – 30,000) @ 10% u/s 112A, Tax on (8,000 – 8,000) @ 15% u/s 111A, Tax on normal income ₹1,21,000 at slab rate, Less: Rebate u/s 87A, Tax before health & education cess, Add: HEC @ 4%, Tax Liability, , 50,000.00, 3,09,000.00, (1,00,000.00), 2,09,000.00, 15,000, Nil, Nil, Nil, (12,500), 2,500, 100, 2,600, Mr. Y, , Full value of consideration, 15,000.00, Less: Cost of acquisition (50 x 160), (8,000.00), Short Term Capital Gain u/s 111A, 7,000.00, Computation of Total Income, Income under the head House Property, 3,50,000.00, Income under the head Capital Gains, 7,000.00, Gross Total Income, 3,57,000.00, Less: Deduction u/s 80C, (30,000.00), Total Income, 3,27,000.00, Computation of Tax Liability, Tax on ₹7,000 @ 15% u/s 111A, 1,050.00, Tax on ₹3,20,000 at slab rate, 3,500.00, Less: Rebate u/s 87A, (4,550.00), Tax Liability, Nil, Illustration 8: Mrs. X purchases 1,000 equity shares in X Ltd. at a cost of ₹ 15 per share (brokerage 1%) in, January 1998. She gets 100 bonus shares in August 2000. She again gets 1100 bonus shares by virtue of her
Page 594 :
Income Under The Head Capital Gains, , 75, , holding on February 2005. Fair market value of the shares of X Ltd. On April 1, 2001 is ₹ 25. In January, 2022, she transfers all her shares @ ₹ 120 per share (brokerage 2%). (market value on 31-01-2018 is ₹70 per, share), Compute the capital gains taxable in the hands of Mrs. X for the A.Y. 2022-23 assuming:, (a) X Ltd. is an unlisted company and securities transaction tax was not applicable at the time of sale., (b) X Ltd. is a listed company and the shares are sold in a recognised stock exchange and securities, transaction tax was paid at the time of sale., Solution:, (a) Computation of capital gains for the A.Y. 2022-23, Particulars, ₹, 1000 Original shares, Sale proceeds (1000 × ₹ 120), 1,20,000, Less : Indexed cost of acquisition [₹ 25 × 1000 × 317/100], (79,250), Less : Brokerage paid (2% of ₹ 1,20,000), (2,400), Long term capital gain, 38,350, 100 Bonus shares, Sale proceeds (100 × ₹ 120), 12,000, Less : Indexed cost of acquisition [₹ 25 × 100 × 317/100] [note], (7,925), Less : Brokerage paid (2% of ₹ 12,000), (240), Long term capital gain, 3,835, 1100 Bonus shares, Sale proceeds (1100 × ₹ 120), 1,32,000, Less: Cost of acquisition, NIL, Less: Brokerage paid (2% of ₹ 1,32,000), (2,640), Long term capital gain, 1,29,360, Long term capital gain, 1,71,545, Note: Cost of acquisition of bonus shares acquired before 01.04.2001 is the FMV as on 01.04.2001 (being, the higher of the cost or the FMV as on 01.04.2001)., (b) The long-term capital gains on transfer of equity shares through a recognized stock exchange on which, securities transaction tax is paid is taxable in excess of ₹1,00,000 u/s 112A @ 10% and Indexation shall not, be applicable., Particulars, ₹, 1000 Original shares, Sale proceeds (1000 × ₹ 120), 1,20,000, Less : Cost of acquisition, (70,000), Cost of Acquisition, Higher of, (i) COA = 1000 x ₹25 = ₹25,000, (ii) lower of, (a) FMV as on 31-01-2018 = 1000 x ₹70 = ₹70,000, (b) sale value = 1000 x ₹120 = ₹1,20,000, COA = ₹70,000, Less : Brokerage paid (2% of ₹ 1,20,000), Long term capital gain u/s 112A, 100 Bonus shares, Sale proceeds (100 × ₹ 120), Less : cost of acquisition, Cost of Acquisition, Higher of, , (2,400), 47,600, 12,000, (7,000)
Page 595 :
Income Under The Head Capital Gains, , 76, , (i) COA = 100 x ₹25 = 2,500, (ii) lower of, (a) FMV as on 31-01-2018 = 100 x ₹70 = ₹7,000, (b) sale value = 100 x ₹120 = ₹12,000, COA = ₹7,000, Less : Brokerage paid (2% of ₹ 12,000), Long term capital gain u/s 112A, 1100 Bonus shares, Sale proceeds (1100 × ₹ 120), Less: Cost of acquisition, Cost of Acquisition, Higher of, (i) COA = Nil, (ii) lower of, (a) FMV as on 31-01-2018 = 1100 x ₹70 = ₹77,000, (b) sale value = 1100 x ₹120 = ₹1,32,000, COA = ₹77,000, Less: Brokerage paid (2% of ₹ 1,32,000), Long term capital gain u/s 112A, Long term capital gain u/s 112A, , (240), 4,760, 1,32,000, (77,000), , (2,640), 52,360, 1,04,720, , RTP NOV 20, Mr. Arjun holding 1000 shares of X Ltd acquired on 01.07.2020 for ₹ 600 per share, sold 500 shares to Mr., Shaurya, on 01.05.2021 for ₹ 550 per share. X Ltd. declared dividend @ ₹65 per share on 20.07.2021, being, the record date for declaration of dividend. Mr. Shaurya sold 300 equity shares at ₹ 475 per share on, 28.09.2021 and the balance 200 equity shares at ₹ 450 per share on 28.10.2021. Apart from above, mentioned information, Mr. Shaurya was having only long-term capital gains from sale of unlisted shares of, ₹ 50,000. Assuming that Mr. Shaurya has no other income, his total income for A.Y. 2022-23 is –, (a) ₹ 7,500, (b) ₹ 27,000, (c) ₹ 40,000, (d) ₹ 30,000, Answer: (c), Hint: Computation of Total Income of Mr. Shaurya, Shares sold on 28.09.2021, Sales consideration (300 x 475), , 1,42,500, , Less: Cost of Acquisition (300 x 550), , 1,65,000, , Short term capital loss, , 22,500, , Shares sold on 28.10.2021, Sales consideration (200 x 450), Less: Cost of Acquisition (200 x 550), , 90,000, 1,10,000, , Short term capital loss, , 20,000, , Long term capital gain on unlisted shares, , 50,000, , Less: Setoff of short term capital loss (22,500+20,000), , (42,500)
Page 596 :
Income Under The Head Capital Gains, Long term capital Gains, Income under the head other sources (65 x 500), Total Income, , 77, 7,500, 32,500, 40,000, , Question 8: write a note on capital gains in case of Depreciable Assets., Answer: Capital gains in case of Depreciable Assets Section 50, If any person has transferred depreciable asset, gain or loss shall always be short term and indexation shall, not be applicable and capital gains shall be computed in the manner given below:, Full value of consideration, Less:, (i) Written down value of the asset in the beginning of the year, (ii) Selling expenses, Short Term Capital Gains, “Provided that in a case where goodwill of a business or profession forms part of a block of asset for the, assessment year beginning on the 1st day of April, 2020 and depreciation thereon has been obtained by the, assessee under the Act, the written down value of that block of asset and short term capital gain, if any, shall, be determined in such manner as may be prescribed.”., Example, ABC Ltd. has one plant and machinery on 01.04.2021 with w.d.v ₹6,00,000 and it was acquired by the, company on 01.04.2009 and the plant was sold on 01.01.2022 for ₹11,00,000 and selling expenses are, ₹30,000, in this case, capital gains shall be computed in the manner given below:, ₹, Full value of consideration, 11,00,000, Less:, (i) Written down value of the asset in the beginning of the year, (6,00,000), (ii) Selling expenses, (30,000), Short Term Capital Gains, 4,70,000, Question 9.[V. Imp.]: Explain the meaning of Capital Asset under Income Tax Act?, Answer: Meaning of Capital Asset, Capital assets Section 2(14), capital asset" includes all assets but the following shall not be considered to be capital asset, 1. any stock-in-trade, consumable stores or raw materials held by an assessee for the purposes of his, business or profession shall not be considered to be capital asset., Example, If Mr. X is engaged in the business of sale purchase of Jewellery, Income from such business shall be, taxable under the head business / profession., 2. Personal movable effects i.e. movable items of personal use like household furniture, utensils, TV,, fridge, sofa, personal motor car etc. shall not be considered to be Capital Assets, and no gain or loss, shall be computed on their sale., Example, (i) Mr. X purchased one motor car for his personal use and subsequently it was sold by him, in this case it, will not be considered to be capital asset., (ii) Mr. X purchased one fridge for his personal use but subsequently it was sold by him, it will not be, considered to be capital asset., The following shall be capital asset—, (a) Jewellery;, (b) archaeological collections;, (c) drawings;, (d) paintings;, (e) sculptures; or, (f) any work of art., “Jewellery” includes—
Page 597 :
Income Under The Head Capital Gains, , 78, , (a) ornaments made of gold, silver, platinum or any other precious metal or any alloy containing one or, more of such precious metals, (b) precious or semi-precious stones held in any manner., Whether capital gain arise on the sale of silver utensils., In Benarshilal v CIT, during the previous year 1976-77, the assessee sold 49.521 kgs. of silver utensils, which were in the form of thalis, katoris, tumblers, etc. The assessee contended that the silver utensils were, for personal use and they were not capital assets within the meaning of section 2(14) of the Income Tax Act,, 1961 and thus the profit on sale of these utensils was not liable to capital gains tax. The ITO rejected the, assessee’s claim that the silver utensils were ‘personal effects’. The high court held that silver utensils, constitute personal affects and no capital gains will arise on the sale of silver utensils., Whether capital gain arise on the sale of gold/silver coins., Maharaja Rana Hemant Singh v CIT, In this case the assessee sold 4825 gold sovereigns, 7,90,440 old silver rupee coins and silver bars weighing, 2,54,174 tolas and claimed that no capital gains arose as the aforesaid items fell outside the definition of, capital assets. The assessee claimed that these articles formed personal effects as they were used by the, assessee and his family for personal use as it was evident that they were used for the purpose of Mahalaxmi, Puja and other religious festivals in the family. The Supreme Court decided the case against the assessee as, according to it, these articles did not constitute ‘personal effects’., If any person has movable items in his business or profession, these items shall be considered to be, capital assets., Example, Mr. X has one motor car in the use of his business and subsequently this motor car was sold by him, it will, be considered to be capital asset and capital gains shall be computed., If personal effects are immovable, they will be considered to be capital assets. e.g. A house meant for, assessee’s own residence shall be considered to be capital asset., 3. Agricultural land, Agricultural land in India in rural area shall not be considered to be capital asset. If the land is in the urban, area, it will be considered to be capital asset., Example, Mr. X has agricultural land in the rural area which was purchased by him for ₹5,00,000 and it was sold by, him for ₹11,00,000, in this case capital gain shall not be computed, but if the land is in Delhi, in this case, capital gains shall be computed., Land in rural area shall be considered to be urban land in the following cases:, 1. If rural area is within the distance of 2 kms from the limits of urban area having population more, than 10000 but not exceeding 100000, 2. If rural area is within the distance of 6 kms from the limits of urban area having population more, than 100000 but not exceeding 1000000, 3. If rural area is within the distance of 8 kms from the limits of urban area having population more, than 1000000., If the agricultural land is in rural area outside India, it will be considered to be capital asset i.e. in other, words agricultural land situated outside India is capital asset in all cases., Example, (i) Mr. X has agricultural land in the rural area in India which was sold by him, in this case there are no, capital gains., (ii) Mr. X has one agricultural land in urban area in India which was sold by him, in this case capital gains, shall be computed., (iii) Mr. X who is resident and ordinarily resident has sold one agricultural land in rural area in Nepal, in, this case it will be considered to be capital asset because the land is not situated in India.
Page 598 :
Income Under The Head Capital Gains, , 79, , 4. Gold Deposit Bonds, Gold Deposit Bonds issued under the Gold Deposit Scheme, 1999 or deposit certificates issued under the, Gold Monetisation Scheme 2015 notified by the Central Government., Gold deposit bond means bond issued by various banks for deposit of gold with them. The assessee can sell, the bond in the market and no capital gains shall be computed. Assessee shall earn interest also and it will be, exempt from income tax. The Scheme seeks to provide depositors the opportunity to earn interest on their, idle gold holdings along with the benefits of safety and security of holding gold without any cost. Individual, banks will be free to fix the interest rates., GOLD MONETIZATION SCHEME, 2015, This scheme will replace the existing Gold Deposit Scheme, 1999. However, the deposits outstanding under, the Gold Deposit Scheme will be allowed to run till maturity unless these are withdrawn by the depositors, prematurely as per existing instructions., All designated banks will be eligible to implement the scheme., Resident Indians (Individuals, HUF, Trusts including Mutual Funds/Exchange Traded Funds registered, under SEBI (Mutual Fund) Regulations and Companies) can make deposits under the scheme. The opening, of gold deposit accounts will be subject to the same rules with regard to customer identification as are, applicable to any other deposit account., The designated banks will accept gold deposits under the Short Term (1-3 years) Bank Deposit as well as, Medium (5-7 years) and Long (12-15 years) Term Government Deposit Schemes., The designated banks may sell or lend the gold accepted under the short-term bank deposit to MMTC for, minting India Gold Coins and to jewellers, or sell it to other designated banks participating in the scheme., Gold lying in the locker appreciates in value if gold price goes up but it doesn't pay you a regular interest or, dividend. On the contrary, you incur carrying costs on it (i.e. bank locker charges)., The scheme will allow you to earn some regular interest on your gold and save your carrying costs as well., It is a gold savings account which will earn interest for the gold that you deposit in it. Your gold can be, deposited in any physical form – jewellery, coins or bars. This gold will then earn interest based on gold, weight and also the appreciation of the metal value. You get back your gold or Indian rupees as you desire, (the option to be exercised at the time of deposit), Earnings are exempt from income tax. There will be no capital gains tax on the appreciation in the value of, gold deposited. Interest on Gold deposited under this scheme has been exempted from income tax u/s, 10(15)., , RTP NOV 20, Mr. A, aged 45 years sold an agricultural land for ₹ 52 lakhs on 04.10.2021 acquired at a cost of ₹49.25, lakhs on 13.09.2020 situated at 7 kms from the jurisdiction of municipality having population of 4,00,000, and also sold another agricultural land for ₹ 53 lakhs on 12.12.2021 acquired at a cost of ₹ 46 lakhs on, 15.02.2020 situated at 1.5 kms from the jurisdiction of municipality having population of 12,000. What, would be the amount of capital gain chargeable to tax in the hands of Mr. A for the assessment year 202223? Cost inflation index for F.Y.2019-20: 289; 2020-21:301; 2021-22:317, (a) Short-term capital gain of ₹ 9.75 lakhs, (b) Short-term capital gain of ₹ 7 lakhs, (c) Long-term capital gain of ₹ 2,54,325, (d) Long-term capital gain of ₹ 4,55,482, Answer: (b)
Page 599 :
Income Under The Head Capital Gains, , 80, , As per the definition of capital asset, a land shall be considered to be urban land if it is situated within 2 kms, from the limits of urban area having population more than 10,000 but not exceeding 1,00,000 and within 6, kms from the limits of urban area having population more than 1,00,000 but not exceeding 10,00,000., In the given case, first agricultural land is situated after 6 kms having population of 4,00,000 hence such, land is considered as land in rural area and second land is situated within 2kms from the municipality having, population of 12,000 hence such land is considered as land in urban area and considered as capital asset., Land is sold within 2 years hence such asset is considered as short term and gain shall be short term capital, gains, Sales consideration, 53,00,000, Less: Cost of Acquisition, (46,00,000), Short term capital gains, 7,00,000, Question 10: Write a note on computation of capital gains in case of Insurance Claims., Answer: Capital Gains in case of Insurance Claims Section 45(1A), If any capital asset is destroyed because of fire or natural calamity etc. like flood/ earthquake etc. and, assessee has received any insurance claim for such asset, in such cases capital gains shall be computed in the, normal manner and such capital gains shall be taxable in the year in which insurance claim has been, received. Amount of insurance claim received shall be considered to be full value of consideration., Example, ABC Ltd. has one plant and machinery on 01.04.2021 with written down value ₹20,00,000 the asset is, destroyed due to natural calamity and the company has received insurance claim of ₹21,00,000, in this case, there will be short term capital gain of ₹1,00,000., Question 11: Write a note on taxability of amount received under Unit Link Insurance Policy., Answer: Section 45(1B), Notwithstanding anything contained in sub-section (1), where any person receives at any time during any, previous year any amount under a unit linked insurance policy, to which exemption under clause (10D) of, section 10 does not apply on account of the applicability of the fourth and fifth provisos thereof, including, the amount allocated by way of bonus on such policy, then, any profits or gains arising from receipt of such, amount by such person shall be chargeable to income-tax under the head "Capital gains" and shall be, deemed to be the income of such person of the previous year in which such amount was received and the, income taxable shall be calculated in such manner as may be prescribed., Question 12 [Imp.]: Write a note on computation of capital gains in case of conversion of capital assets, into Stock-In-Trade., Answer: Capital Gains in case of conversion of capital assets into Stock-In-Trade Section 45(2), If any person has converted any capital asset into stock-in-trade, it will be considered to be ‘transfer’ and, capital gains shall be computed in the year of conversion and, market value shall be considered to be full, value of consideration. Capital gains so computed shall be taxable in the year in which stock-in-trade has, been sold. (Proportionately), Illustration 9: Mr. X purchased Gold on 01.10.1991 for ₹2,00,000 and its fair market value on 01.04.2001, is ₹3,00,000 and he converted it into stock-in-trade on 01.10.2008 and market value of the gold on the date, of conversion was ₹11,00,000 and subsequently half of the stock-in-trade was sold on 01.10.2021 for, ₹6,50,000 and balance half was sold on 01.10.2022 for ₹7,50,000., Compute his total income for various years., Solution:, ₹, Previous year 2008-09, Computation of Capital Gains under section 45(2), Full value of consideration, 11,00,000, Less: Indexed cost of acquisition, = 3,00,000 / Index of 01-02 x Index of 08-09
Page 600 :
Income Under The Head Capital Gains, = 3,00,000 / 100 x 137= ₹4,11,000, Long Term Capital Gain, Previous year 2021-22, Long Term Capital Gain (1/2 of ₹6,89,000), Business Income, (₹6,50,000-₹5,50,000), Total Income, Previous year 2022-23, Long Term Capital Gain (1/2 of ₹6,89,000), Business Income, (₹7,50,000-₹5,50,000), Total Income, , 81, (4,11,000), 6,89,000, 3,44,500, 1,00,000, 4,44,500, 3,44,500, 2,00,000, 5,44,500, , Illustration 10: X converts his capital asset (acquired on June 10, 2008 for ₹ 60,000) into stock-in-trade on, March 10, 2014. The fair market value on the date of the above conversion was ₹ 3,00,000. He subsequently, sells the stock-in-trade so converted for ₹ 4,00,000 on June 10, 2021. Discuss the tax implication., Solution: In this case capital gains shall be computed in the previous year 2013-14 as given below:, FVC, 3,00,000.00, Less: Indexed cost of acquisition, = 60,000 / Index of 2008-09 x Index of 2013-14, = 60,000 / 137 x 220, (96,350.36), Long term capital gain, 2,03,649.64, Computation of income for previous year 2021-22, Income under the head Business/Profession for previous year 2021-22 shall be ₹4,00,000 – ₹3,00,000 =, ₹1,00,000, LTCG, 2,03,649.64, Question 13: Write a note on computation of capital gains in case of transfer of capital asset by a, Depository., Answer: Capital gains in case of transfer of capital asset by a depository Section 45(2A), If any person has a demat account with the depository, profits or gains from transfer of shares or securities, shall be considered to be that of beneficiary i.e. the account holder and not that of depository. The cost of, acquisition and the period of holding of any securities shall be determined on the basis of the first-in-firstout method., Question 14 [V. Imp.]: Write a note on computation of capital gains on compulsory acquisition of a, Capital Asset., Answer:, Computation of capital gains on compulsory acquisition of a capital asset Section 45(5), If any capital asset has been acquired compulsorily by the Government or other similar agency, capital gains, shall be computed in the year in which the asset was acquired but capital gains so computed shall be taxable, in the year in which the compensation or the part of compensation is first received., Enhanced Compensation, If the compensation is enhanced by the Court, Tribunal etc., such enhanced compensation shall be the, capital gains of the year in which the enhanced compensation is received. The cost of acquisition and the, cost of improvement shall be taken to be nil., Death of the transferor- It is possible that the transferor may die before he receives the enhanced, compensation. In that case, the enhanced compensation will be chargeable to tax in the hands of the person, who receives the same.
Page 601 :
Income Under The Head Capital Gains, , 82, , Illustration 11: Mr. X (Date of birth 01.10.1946) has purchased one house on 01.04.1995 for ₹4,00,000 and, incurred ₹2,00,000 on its improvement on 01.10.1998. Its market value on 01.04.2001 was ₹3,00,000. This, house was acquired by the Government on 01.10.2013 and the compensation fixed was ₹50,00,000 and the, Government has paid half of the compensation on 01.10.2021 and balance half on 01.10.2022., The assessee has filed an appeal for increasing the compensation and the court has given decision on, 31.03.2023 directing the Government to pay additional compensation of ₹5,00,000., The Government has paid half of the amount on 01.04.2024 and balance half on 01.04.2025., He has invested ₹72,000 in NSC in previous year 2021-22., Compute assessee’s tax liability for the assessment year 2022-23 and also capital gains for various years., Solution:, Computation of Capital Gains under section 45(5), Capital gain shall be computed in the year in which the asset was acquired by the Government i.e. in the, previous year 2013-14 and shall be taxed in the year in which the first payment has been received by the, assessee i.e. in the previous year 2021-22, ₹, Full value of consideration, 50,00,000.00, Less: Indexed cost of acquisition, = 4,00,000 / Index of 01-02 x Index of 13-14, = 4,00,000 / 100 x 220 = ₹8,80,000, (8,80,000.00), Long Term Capital Gain, 41,20,000.00, Income under the head Capital Gain (LTCG), 41,20,000.00, Gross Total Income, 41,20,000.00, Less: Deduction u/s 80C, Nil, {Deduction under section 80C is not allowed from LTCG}, Total Income, 41,20,000.00, Computation of Tax Liability, {Since normal income is nil, as per section 112 deficiency of ₹3,00,000 shall be allowed from long term, capital gains and balance income shall be taxed at flat rate of 20%}, Tax on ₹38,20,000 (₹41,20,000 – ₹3,00,000) @ 20%, 7,64,000.00, Add: HEC @ 4%, 30,560.00, Tax Liability, 7,94,560.00, Computation of Capital Gain for the previous year 2024-25, Long Term Capital Gain, 2,50,000.00, Computation of Capital Gain for the previous year 2025-26, Long Term Capital Gain, 2,50,000.00, Illustration 12: Mrs. X purchased one house on 01.07.1997 for ₹2,00,000 and incurred ₹1,00,000 on its, improvement in 1998-99 and its market value as on 01.04.2001 is ₹2,50,000. She incurred ₹2,00,000 on its, improvement in 2011-12 and the house was acquired by the Government on 01.07.2014 and compensation, fixed is ₹60,00,000 and half of the amount was paid by the Government on 01.01.2022 and balance half on, 01.01.2023. She has also received interest of ₹ 2,00,000 in previous year 2021-22 from the Government for, delay in payment of compensation., Income under the head Business/Profession ₹20,03,990., Compute tax liability of Mrs. X for the Assessment Year 2022-23., Solution:, Computation of income under the head Capital Gains, Capital gain shall be computed in the year in which the asset was acquired by the Government i.e. in the, previous year 2014-15 and shall be taxed in the year in which the first payment has been received by the, assessee i.e. in the previous year 2021-22, ₹, Full value of consideration, 60,00,000.00, Less: Indexed cost of acquisition, = 2,50,000/Index of 01-02 x Index of 14-15, = 2,50,000/100 x 240, (6,00,000.00)
Page 602 :
Income Under The Head Capital Gains, Less: Indexed cost of improvement, = 2,00,000/ Index of 11-12 x Index of 14-15, = 2,00,000/184 x 240, Long Term Capital Gain, Computation of income under the head Other Sources, Interest income, Less: Deduction u/s 57 @ 50%, Income under the head Other Sources, Computation of income under the head Business/Profession, Income under the head Business Profession, Computation of Gross Total Income, Income under the head Business Profession, Income from long term capital gains, Income under the head Other Sources, Gross Total Income, Less: Deduction u/s 80C to 80U, Total Income, LTCG, Rounded off u/s 288A, Normal income, Computation of Tax Liability, Tax on LTCG ₹51,39,130 @ 20% u/s 112, Tax on ₹21,03,990 at slab rate, Tax before surcharge, Add: Surcharge @ 10%, Tax before health & education cess, Add: HEC @ 4%, Tax Liability, Rounded off u/s 288B, , 83, , (2,60,869.57), 51,39,130.43, 2,00,000.00, (1,00,000.00), 1,00,000.00, 20,03,990.00, 20,03,990.00, 51,39,130.43, 1,00,000.00, 72,43,120.43, Nil, 72,43,120.43, 51,39,130.43, 51,39,130.00, 21,03,990.00, 10,27,826.00, 4,43,697.00, 14,71,523.00, 1,47,152.30, 16,18,675.30, 64,747.01, 16,83,422.31, 16,83,420.00, , Question 15: Write a note on computation of capital gains on Transfer of Land or Building under, specified agreement., Answer: Computation of capital gains on Transfer of Land or Building under specified agreement, Section 45 (5A), If any Individual or HUF has transferred Land or Building under a specified agreement, In such cases, capital gain shall be computed in the year in which certificate of completion has been issued and full value, of consideration shall be the Stamp duty value on the date of issue of certificate. If such individual or HUF, has transferred any part of the building before completion to any other person, in that case capital gains shall, be computed in the year of transfer for such part., "specified agreement" means a registered agreement in which a person owning land or building or both,, agrees to allow another person to develop a real estate project on such land or building or both, in, consideration of a share, being land or building or both in such project, whether with or without payment of, part of the consideration in cash., Question 16: Write a note on capital gains on distribution of assets by a company on Liquidation., Answer: Capital Gains on distribution of assets by companies in Liquidation Section 46, If any company is in liquidation and the company has distributed its assets to the shareholders in connection, with liquidation, it will be exempt from capital gains., If the same asset has been sold by the shareholder subsequently, its cost of acquisition shall be the amount, for which the shareholder has received the asset from the company and capital gains shall be computed, accordingly.
Page 603 :
Income Under The Head Capital Gains, , 84, , The amount received by the shareholder out of accumulated profits of the company shall be considered to be, dividend under section 2(22)(c) and excess over it shall be considered to be full value of consideration for, computing capital gains., (Already discussed under the head Other Sources under section 2(22)(c)), Illustration 13: ABC Ltd. has issued one-lakh shares of ₹10 each and the company goes into liquidation on, 01.10.2021 and distributable asset of the company are valued at ₹8 lakh. The company’s accumulated profits, on the date of liquidation are ₹3.5 lakhs which are included in ₹8 lakhs. Mr. X has purchased 100 shares in, this company on 01.10.1998 for ₹ 10 each and market value of the shares on 01.04.2001 is ₹12 per share., Compute dividends in the hands of Mr. X and also capital gains., Solution:, ₹, Share of Mr. X in the distributable profits, 8,00,000 x 100/1,00,000, 800, Accumulated profits, 3,50,000, Proportionate share of Mr. X in accumulated profits, 350, Dividends in the hands of Mr. X as per sec 2(22)(c). Taxable in the hands of Mr. X, 350, Computation of capital gains as per section 46, Full value of consideration, 450, Less: Indexed cost of acquisition, = (12 x 100) / Index of 01-02 x Index of 21-22, = 1,200/100 x 317 = 3,804, (3,804), Long-term capital loss, (3,354), Question 17: Explain capital gains in case of buy back of shares., Answer:, Capital Gains on Buyback of shares or Specified Securities [Section 46A], In case of buy back of securities and also buy back of shares of a company other than domestic company,, capital gains shall be computed in the hands of its holder. Mr. X purchased 100 shares of ABC Ltd. on, 01/10/2012 for ₹ 10,000 and these shares were bought back by the company on 01/10/2021 for ₹ 3,00,000,, in this case capital gains shall be computed in the manner given below, FVC, Less: Indexed cost of acquisition 10,000/200 X 317, Long term capital gain, , 3,00,000, (15,850), 2,84,150, , In case of buyback of shares (whether listed or unlisted) by domestic companies, no capital gain shall be, computed in the hands of shareholder and shareholder shall be exempt u/s 10(34A) rather the company has, to pay additional income tax @ 20% (plus surcharge @12% and cess@4%) is leviable in the hands of the, company., Taxation provisions in respect of buyback, (1), (2), Taxability in the, Buyback of shares by, hands of, domestic companies, Company, Shareholder /, holder of specified, securities, , Subject to additional, income-tax @ 23.296%., Income arising to, shareholders exempt, under section 10(34A), , (3), Buyback of shares by a, company, other than a, domestic company, Not subject to tax in the, hands of the company., Income arising to, shareholder taxable as, capital gains u/s 46A., , (4), Buyback of specified, securities by any company, Not subject to tax in the, hands of the company., Income arising to holder of, specified securities taxable as, capital gains u/s 46A., , Question 18 [V. Imp.]: Write a note on transactions not regarded as transfer., Answer: Transactions not regarded as transfer Section 47, The following transactions will not be considered as transfer and therefore, no capital gains will arise:-
Page 604 :
Income Under The Head Capital Gains, , 85, , (1) No capital gain shall be computed in case of transfer of any capital asset through gift or will or, inheritance etc., Illustration 14: Mr. X purchased one house on 01.10.1998 for ₹2,00,000 and incurred ₹1,00,000 on its, improvement on 01.10.1999. Its fair market value on 01.04.2001 is ₹4,50,000., Mr. X expired on 01.05.2006 and the house was inherited by his son Mr. Y and value for the purpose of, charging stamp duty was ₹10,00,000., Mr. Y has sold the house on 01.11.2021 for ₹72,00,000., Compute tax liability of Mr. Y for the assessment year 2022-23., Solution:, ₹, Computation of Capital Gains, Full value of consideration, 72,00,000.00, Less: Indexed cost of acquisition, = 4,50,000 / 100 x 317 = ₹14,26,500, (14,26,500.00), Long Term Capital Gain, 57,73,500.00, Income under the head Capital Gain, 57,73,500.00, Gross Total Income, 57,73,500.00, Less: Deduction u/s 80C to 80U, Nil, Total Income, 57,73,500.00, Computation of Tax Liability, {Since normal income is nil, as per section 112 deficiency of ₹2,50,000 shall be allowed from long term, capital gain and balance income shall be taxed at flat rate of 20%}, Tax on ₹55,23,500 (₹57,73,500 – ₹2,50,000) @ 20%, 11,04,700.00, Add: Surcharge @ 10%, 1,10,470.00, Tax before health & education cess, 12,15,170.00, Add: HEC @ 4%, 48,606.80, Tax Liability, 12,63,776.80, Rounded off u/s 288B, 12,63,780.00, (2) Any distribution of capital assets on the partition of a Hindu Undivided Family., Illustration 15: Mr. X & sons, HUF, purchased a land for ₹ 40,000 in 2001-02. In 2005-06, a partition takes, place when Mr. A, a coparcener, is allotted this plot valued at ₹ 2,00,000. In 2006-07, he had incurred, expenses of ₹ 1,85,000 towards fencing of the plot. Mr. A sells this plot of land for ₹ 15,00,000 in 2021-22, after incurring expenses of ₹ 20,000. You are required to compute the capital gain for the A.Y. 2022-23., Solution:, Computation of taxable capital gains for the A.Y. 2022-23, Particulars, ₹, Sale consideration, 15,00,000.00, Less: Indexed cost of acquisition, = 40,000 / Index of 01-02 x Index of 21-22, = ₹ 40,000 /100 x 317, (1,26,800.00), Less: Indexed cost of improvement, = 1,85,000 / Index of 06-07 x Index of 21-22, (4,80,696.72), = ₹ 1,85,000 /122 × 317, Less: Expenses incurred for transfer, (20,000.00), Long term capital gains, 8,72,503.28, (3) Transfer of capital asset by holding company to subsidiary company or by subsidiary company to, holding company provided company receiving capital asset is an Indian company and also 100% share, capital of subsidiary company is held by holding company or its nominees., (4) Transfer of any capital asset by the amalgamating company to the amalgamated company if the, amalgamated company is an Indian company., (5) Transfer of a capital asset by the demerged company to the resulting company, if the resulting
Page 605 :
Income Under The Head Capital Gains, , 86, , company is an Indian company., (6) Receiving of shares from an amalgamated company in lieu of shares held in amalgamating company, provided the amalgamated company is an Indian company. E.g. Mr. X purchased 2000 shares in ABC, Ltd. on 01.07.2021 @ ₹10 per share and ABC Ltd. was amalgamated with XYZ Ltd. on 01.12.2021 and, Mr. X received 1000 shares in XYZ Ltd. and market value is ₹50 per share, in this case no capital gains, shall be computed but if Mr. X has sold the shares, capital gains shall be computed and cost will be, ₹20,000., (7) Transfer or issue of shares by a resulting company in case of demerger., (8) In case of Conversion of bonds or debentures etc. into shares or conversion of preference shares, into equity shares, no capital gains shall be computed., (9) Redemption by an individual of Sovereign Gold Bonds issued by RBI under the Sovereign Gold, Bond Scheme, 2015., Sovereign Gold Bond Scheme, SGBs are government securities denominated in grams of gold. They are substitutes for holding physical, gold. Investors have to pay the issue price in cash and the bonds will be redeemed in cash on maturity., The Bond is issued by Reserve Bank on behalf of Government of India., The quantity of gold for which the investor pays is protected, since he receives the ongoing market price, at the time of redemption/ premature redemption. The SGB offers a superior alternative to holding gold, in physical form. The risks and costs of storage are eliminated. Investors are assured of the market value, of gold at the time of maturity and periodical interest., (10) Any transfer of a capital asset in a transaction of reverse mortgage., (11) Any transfer of any of the following capital asset to the Government or to the University or the National, Museum, National Art Gallery, National Archives or any other public museum or institution notified by the, Central Government to be of national importance or to be of renown throughout any State:, (i) work of art, (ii) archaeological, scientific or art collection, (iii) book, (iv) manuscript, (v) drawing, (vi) painting, (vii) photograph or, (viii) print., (12) Any other transaction listed under section 47., Illustration 16: In which of the following situations capital gains tax liability does not arise?, (i), Mr. A purchased gold in 2004 for ₹ 25,000. In the P.Y. 2021-22, he gifted it to his son at the time of, marriage. Fair market value (FMV) of the gold on the day the gift was made was ₹ 1,00,000., (ii), A house property is purchased by a Hindu undivided family in 1985 for ₹ 20,000. It is given to one, of the family members in the P.Y. 2021-22 at the time of partition of the family. FMV on the day of, partition was ₹ 12,00,000., (iii) Mr. B purchased 50 convertible debentures for ₹ 40,000 in 2005 which are converted into 500 shares, worth ₹ 85,000 in November 2021 by the company., Answer:, The liability of capital gains tax in the situations given above is discussed as follows:, (i), As per the provisions of section 47, transfer of a capital asset under a gift is not regarded as transfer, for the purpose of capital gains. Therefore, capital gains tax liability does not arise in the given, situation., (ii), As per the provisions of section 47, transfer of a capital asset on partition of Hindu undivided family, is not regarded as transfer for the purpose of capital gains. Therefore, capital gains tax liability does, not arise in the given situation.
Page 606 :
Income Under The Head Capital Gains, (iii), , 87, , As per the provisions of section 47, transfer by way of conversion of bonds into shares is not, regarded as transfer for the purpose of capital gains. Therefore, capital gains tax liability does not, arise in the given situation., , Question 19. Write a note on cost with reference to certain modes of Acquisition., Answer: Cost with reference to certain modes of acquisition Section 49(1), If any person has received an asset through the transaction section 47 and subsequently asset was sold by, him, in such cases cost of acquisition and cost of improvement of previous owner shall be considered to be, cost of acquisition/improvement of the assessee and also cost of improvement by assessee shall be taken into, consideration., As per section 2(42A), time period of previous owner shall also be taken into consideration., E.g. Mr. X purchased house 01.04.2001 ₹2,00,000 and incurred ₹3,00,000 on improvement on 01.07.2002, and it was received by his son Mr. Y on 01.07.2011 and Mr. Y incurred ₹4,00,000 on improvement, 01.07.2013 and house was sold by him on 01.07.2021 ₹100,00,000, in this case tax liability of Mr. Y shall, be, Full value of consideration, 100,00,000.00, Less: Indexed cost of acquisition, = 2,00,000 / 100 x 317, (6,34,000.00), Less: Indexed Cost of improvement, = 3,00,000 / 105 x 317, (9,05,714.29), Less: Indexed Cost of improvement, = 4,00,000 / 220 x 317, (5,76,363.63), Long term capital gains, 78,83,922.08, Gross Total Income, 78,83,922.08, Less: Deduction u/s 80C to 80U, Nil, Total Income (rounded off u/s 288A), 78,83,920.00, Computation of Tax Liability, Tax on LTCG ₹76,33,920 (78,83,920 – 2,50,000) @ 20%, 15,26,784.00, Add: Surcharge @ 10%, 1,52,678.40, Tax before health & education cess, 16,79,462.40, Add: HEC @ 4%, 67,178.50, Tax Liability, 17,46,640.90, Rounded off u/s 288B, 17,46,640.00, Cost of acquisition in case of assets received as gift Section 49(4) (applicable w.e.f 01.10.2009), If any individual or HUF has received gift in kind and it was taxable under section 56, in such cases, at the, time of sale, cost of acquisition of such asset shall be the value which has been taken into consideration for, the purpose of computing taxable amount of gift., Example, Mr. X purchased one house property on 01.07.2002 for ₹ 2,00,000 and it was gifted to Mr. Y on 01.11.2021, and value for the purpose of charging stamp duty was ₹5,00,000 and subsequently the house property was, sold by Mr. Y on 01.01.2022 for ₹25,00,000, in this case tax liability of Mr. Y shall be computed in the, manner given below:, ₹, Income under the head Other Sources, 5,00,000.00, (Being the amount of gift under section 56), Income under the head capital gain, Full value of consideration, Less: Cost of acquisition, Short term capital gain, Gross Total Income, Less: Deduction u/s 80C to 80U, , 25,00,000.00, (5,00,000.00), 20,00,000.00, 25,00,000.00, Nil
Page 607 :
Income Under The Head Capital Gains, Total Income, Computation of Tax Liability, Tax on ₹25,00,000 at slab rate, Add: HEC @ 4%, Tax Liability, , 88, 25,00,000.00, 5,62,500.00, 22,500.00, 5,85,000.00, , Illustration 17: Mr. X purchased one house on 01.10.2002 for ₹2,00,000 and incurred ₹ 5,00,000 on its, improvement in F.Y. 2009-2010 and Mr. X gifted the house on 01.10.2012 to his friend Mr. Y when its, value for the purpose of charging stamp duty was ₹10,00,000., Mr. Y sold the house on 01.01.2022 for ₹42,00,000., Compute his tax liability., Solution:, ₹, Computation of Capital Gains, Full value of consideration, 42,00,000.00, Less: Indexed cost of acquisition, = 10,00,000 / 200 x 317 = ₹15,85,000.00, (15,85,000.00), Long Term Capital Gain, 26,15,000.00, Income under the head Capital Gain, 26,15,000.00, Gross Total Income, 26,15,000.00, Less: Deduction u/s 80C to 80U, Nil, Total Income, 26,15,000.00, Computation of Tax Liability, {Since normal income is nil, as per section 112 deficiency of ₹2,50,000 shall be allowed from long term, capital gain and balance income shall be taxed at flat rate of 20%}, Tax on ₹23,65,000 (₹26,15,000 – ₹ 2,50,000) @ 20% u/s 112, 4,73,000.00, Add: HEC @ 4%, 18,920.00, Tax Liability, 4,91,920.00, (b) Presume Mr. Y is son of Mr. X., Solution:, ₹, Computation of Capital Gains, Full value of consideration, 42,00,000.00, Less: Indexed cost of acquisition, = 2,00,000 / 105 x 317 = ₹6,03,809.52, (6,03,809.52), Less: Indexed cost of improvement, = 5,00,000 / 148 x 317 = ₹10,70,945.95, (10,70,945.95), Long Term Capital Gain, 25,25,244.53, Income under the head Capital Gain, 25,25,244.53, Gross Total Income, 25,25,244.53, Less: Deduction u/s 80C to 80U, Nil, Total Income (Rounded off u/s 288A), 25,25,240.00, Computation of Tax Liability, {Since normal income is nil, as per section 112 deficiency of ₹2,50,000 shall be allowed from long term, capital gain and balance income shall be taxed at flat rate of 20%}, Tax on ₹22,75,240 (₹25,25,240 – ₹2,50,000) @ 20%, 4,55,048.00, Add: HEC @ 4%, 18,201.92, Tax Liability, 4,73,249.92, Rounded off u/s 288B, 4,73,250.00, Question 20: Explain Reverse Mortgage., Answer: As per section 47, reverse mortgage shall not be considered to be transfer for the purpose of capital
Page 608 :
Income Under The Head Capital Gains, , 89, , gain., Under reverse mortgage, an individual can mortgage his house property to the bank and the bank shall grant, a loan against the security of house property and such loan shall be given in monthly/quarterly installments, and the amount so received shall not be considered to be income of the mortgagor under section 10(43)., After the death of the mortgagor the bank shall have right to sell off the property and shall adjust loan and, interest and shall compute capital gains for the deceased person and shall pay tax to the government., The purpose of the scheme is to make available regular amount to the persons who do not have regular, income but are the owners of the house property., In general, the mortgagor repay the loan in installments but in this case mortgagee i.e. bank is paying, installment to the mortgagor and hence it is called reverse mortgage., Question 21: Write a note on computation of capital gain in case of slump sale covered under section, 50B., Answer: Special provision for computation of capital gains in case of Slump Sale Section 50B, If any person has transferred by any means, any unit/division for a lump sum consideration, it is called, slump sale and capital gain shall be computed for the entire unit instead of individual asset and capital gains, shall be computed in the manner given below:, • Net worth of the unit on the date of sale shall be deducted from full value of consideration to compute, Capital Gains. Also expenses in connection with transfer shall be deducted, • Indexation is not applicable., If unit is sold within 3 years, a capital gain is Short term otherwise Capital Gain is Long Term., While computing net worth, revaluation of asset shall be ignored., “Net worth” shall be the aggregate value of total assets of the undertaking or division as reduced by the, value of liabilities of such undertaking or division as appearing in its books of account:, • Fair market value of the capital assets as on the date of transfer, calculated in the prescribed manner,, shall be deemed to be the full value of the consideration received or accruing as a result of the, transfer of such capital asset., • For computing the net worth, the aggregate value of total assets shall be,—, (a) in the case of depreciable assets, the written down value of the block of assets;, (aa) in the case of capital asset being goodwill of a business or profession, which has not been acquired, by the assessee by purchase from a previous owner, nil;, (b) in the case of capital assets in respect of which the whole of the expenditure has been allowed or is, allowable as a deduction under section 35AD, nil; and, (c) in the case of other assets, the book value of such assets., Example, ABC Ltd. has sold one of its division on 01.10.2021 for ₹35,00,000 and its net worth on 01.10.2021 was, ₹20,00,000 and it was setup in 2003, in this case there is long term capital gain of ₹15,00,000., Illustration 18: Mr. A is a proprietor of ABC Enterprises having 2 units started on 01.04.2013. He, transferred on 01.04.2021 his unit 1 by way of slump sale for a total consideration of ₹45 Lacs. The, expenses is incurred for this transfer were ₹65,000/-. His Balance Sheet as on 31.03.2021 is as under:, Liabilities, Total, Assets, Unit 1, Unit 2, Total, ₹, ₹, ₹, ₹, Own Capital, 21,00,000 Building, 15,00,000, 4,00,000, 19,00,000, Revaluation Reserve, 6,00,000 Machinery, 5,00,000, 2,00,000, 7,00,000, (for building of unit 1), Bank Loan, 4,00,000 Debtors, 3,00,000, 70,000, 3,70,000, (70% for unit 1), Trade creditors, 3,10,000 Other assets, 3,50,000, 90,000, 4,40,000, (25% for unit 1), Total, 34,10,000 Total, 26,50,000, 7,60,000, 34,10,000, Other information:, (i), Revaluation reserve is created by revising upward the value of the building of unit 1.
Page 609 :
Income Under The Head Capital Gains, (ii) No individual value of any asset is considered in the transfer deed., Compute the capital gain for the assessment year 2022-23., Solution: Computation of capital gains on slump sale of Unit 1, Particulars, Sale value, Less: Expenses on sale, Less: Net worth (See Note (i) below), Long term capital gain, Note (i) : Computation of net worth of Unit 1 of ABC Enterprises, Particulars, Building (excluding ₹6 lakhs on account of revaluation), Machinery, Debtors, Other assets, Total assets, Less:, Bank Loan, Creditors, Net worth, , 90, , ₹, 45,00,000, (65,000), (16,92,500), 27,42,500, ₹, 9,00,000, 5,00,000, 3,00,000, 3,50,000, 20,50,000, (2,80,000), (77,500), 16,92,500, , Question 22 [Imp.]: Write a note on full value of consideration in certain cases., Answer: Special provision for full value of consideration in certain cases Section 50C, If any person has transferred land or building and stamp duty value is upto 110% of the FVC claimed by the, assessee, in such cases FVC shall be the consideration claimed by the assessee but if stamp duty value is, more than 110% of the consideration claimed by the assessee, in that case FVC shall be the Stamp duty, value., If the assessee has disputed such amount, assessing officer may refer the matter to the Valuation Officer and, value determined by Valuation Officer shall be taken into consideration but if the value determined by, Valuation Officer is more than the stamp duty value, in that case stamp duty value shall be considered to be, FVC and capital gains shall be computed accordingly. Valuation Officer means an expert employed by, Income Tax Department to determine the value., If the date of agreement and date of registration are different, in that case value on the date of agreement, shall be taken into consideration provided some advance was given by account payee cheque, an account, payee bank draft or by use of electronic clearing system through a bank account or through such other, electronic modes as may be prescribed. (Other electronic mode means Credit Card, Debit Card, Net, Banking, IMPS (Immediate Payment Service), UPI (Unified Payment Interface), RTGS (Real Time Gross, Settlement), NEFT (National Electronic Funds Transfer), and BHIM (Bharat Interface for Money) Aadhaar, Pay) on or before the date of agreement., Example, Mr. X sold one house property for ₹ 60,00,000 but stamp duty value is ₹ 70,00,000, in this case FVC shall, be taken to be ₹ 70,00,000. In case of dispute matter shall be referred to the Valuation Officer. If value, determined by Valuation Officer is ₹ 65,00,000, FVC shall be ₹ 65,00,000 but if value determined is, ₹75,00,000, FVC shall be ₹ 70,00,000., Illustration 19: Mr. X who transferred his land and building on 10.02.2022, furnishes the following, information:, (i) Net consideration received ₹35,00,000., (ii) Value adopted by stamp valuation authority, which was contested by Mr. X ₹50,00,000., (iii) value ascertained by Valuation Officer on reference by the Assessing Officer ₹52,00,000., (iv) This land was distributed to Mr. X on the partial partition of his HUF on 01.04.2001. Fair market value, of the land as on 01.04.2001 was ₹1,60,000., (v) A residential building was constructed on the above land by Mr. X at a cost of ₹3,50,000 (construction, completed on 01.12.2012) during the financial year 2012-13.
Page 610 :
Income Under The Head Capital Gains, , 91, , (vi) Brought forward short-term capital loss (incurred on sale of shares during the financial year 2016-17), ₹80,000., What should be the maximum amount to be invested by Mr. X in NHAI / RECL bonds so as to be exempt, from clutches of capital gain tax?, Solution:, Computation of Capital Gains of Mr. X for the Assessment Year 2022-23, ₹, Full value of consideration, 50,00,000.00, Less: Indexed cost of acquisition, Indexed cost of land (1,60,000 / 100 x 317), (5,07,200.00), Indexed cost of building (3,50,000 / 200 x 317), (5,54,750.00), Long term capital gain, 39,38,050.00, Less: Brought forward short term capital loss set off, (80,000.00), Long term capital gain, 38,58,050.00, Amount to be invested in NHAI / RECL bonds (38,58,050 – 2,50,000), 36,08,050.00, Since income upto ₹2,50,000 is exempt from income tax hence amount can be invested upto ₹36,08,050, instead of ₹38,58,050., Illustration 20: Mr. X sold his house property in Bangalore as well as his rural agricultural land for a, consideration of ₹ 60 lakh and ₹ 15 lakh, respectively, to Mr. Y on 01.08.2021. He has purchased the house, property and the land in the year 2021 for ₹ 40 lakh and ₹ 10 lakh, respectively. The stamp duty value on the, date of transfer, i.e., 01.08.2021, is ₹ 85 lakh and ₹ 20 lakh for the house property and rural agricultural land,, respectively. Determine the tax implications in the hands of Mr. X and Mr. Y and the TDS implications, if, any, in the hands of Mr. Y, assuming that both Mr. X and Mr. Y are resident Indians., Solution:, (i), Tax implications in the hands of Mr. X, As per section 50C, the stamp duty value of house property (i.e. ₹ 85 lakh) would be deemed to be the, full value of consideration., Therefore, ₹ 45 lakh (i.e., ₹ 85 lakh – ₹ 40 lakh), would be taxable as short-term capital gains., Since rural agricultural land is not a capital asset, capital gains shall not be computed., (ii) Tax implications in the hands of Mr. Y, In case immovable property is received for inadequate consideration, the difference between the stamp, value and actual consideration would be taxable as gift and amount of gift shall be 85 lakh – 60 lakh =, 25 lakh., Since agricultural land is not a capital asset, the provisions of section 56(2)(x) are not attracted in, respect of receipt of agricultural land for inadequate consideration. The definition of “property” under, section 56(2)(x) does not include agricultural land., (iii) TDS implications in the hands of Mr. Y, Since the sale consideration of house property exceeds ₹ 50 lakh, Mr. Y is required to deduct tax at, source under section 194-IA. The tax to be deducted under section 194-IA would be ₹ 60,000, being, 1% of ₹ 60 lakh., TDS provisions under section 194-IA are not attracted in respect of transfer of rural agricultural land., , RTP NOV 20, Ms. Chanchal, aged 45, provides the following data of her gross receipts for the financial year 2020-21 and, 2021-22. She is engaged in agency business along with providing services as tarot card reader., F.Y., Receipts from business, Receipts from profession, Total Gross Receipts, (₹), (₹ ), (₹ ), 2020-21, 78,00,000, 43,00,000, 1,21,00,000, 2021-22, 85,00,000, 47,00,000, 1,32,00,000, During the F.Y. 2021-22, she paid an amount of ₹ 1,20,000 to a contractor for polishing her old furniture., She has taken services from renowned interior designers for her self- occupied residential house property for, which she paid ₹ 2,50,000.
Page 611 :
Income Under The Head Capital Gains, , 92, , Further, on 28.05.2021 she sold one commercial property for ₹ 50,00,000. The value adopted for stamp duty, was ₹ 52,00,000. It was purchased for ₹ 40,00,000 on 28.04.2019. (Cost Inflation Index for F.Y. 2021-22:, 317, F.Y. 2019-20: 289)., The brought forward long-term capital loss from unlisted shares of F.Y. 2020-21 is ₹ 7,80,000., During the year, Ms. Chanchal incurred a loss of ₹ 70,00,000 while trading in the agricultural commodity, derivatives (no CTT paid)., From the details given above, choose the most appropriate option to the questions given below:, (i) Is Ms. Chanchal liable to tax audit under the Income-tax Act, 1961 for the P.Y. 2021- 22?, (a) Yes, as the total gross receipts exceeds ₹ 1,00,00,000, (b) No, as the gross receipts from business or profession are below the specified threshold limits., (c) Yes, as the gross receipts from business exceed ₹ 50,00,000, (d) Yes, as the gross receipts from profession exceed ₹ 25,00,000, Answer: (b), Hint: Since receipt in business is not exceeding 100 lakhs and also receipts in profession is not exceeding, 50 lakhs, in the current year, audit is not required., (ii) What is the total amount of tax to be deducted by Ms. Chanchal for P.Y. 2021-22?, (a)₹ 1,200, (b) ₹ 26,200, (c) Nil, (d) ₹ 27,400, Answer: (c), Hint: Since tax audit is not applicable hence she is not liable for deduction of tax at source., (iii) What is the amount and nature of Capital gain chargeable to tax in the hands of Ms. Chanchal?, (a) ₹ 10,00,000 and Short-term capital gain., (b) ₹ 12,00,000 and Short-term capital gain., (c) ₹ 6,12,457 and Long-term capital gain., (d) ₹ 9,50,000 and Long-term capital gain., Answer: (c), Hint: Since Asset is sold after 2 years, it is long term. Section 50C shall not apply because stamp, duty value is not exceeding actual sales value by more than 10% of sales value., Computation of Long term capital gains, Sales consideration, Less: ICOA (40,00,000/289 x 317), Long term capital gains, , 50,00,000, (43,87,543), 6,12,457, , (iv) What is the amount of losses which can be carried forward to A.Y. 2023-24, assuming that business, income is ₹ 45,00,000 and income from profession is ₹ 25,00,000 for the P.Y. 2021-22?, (a) ₹ 7,80,000 under section 74, (b) ₹ 70,00,000 under section 73, (c) ₹ 1,67,543 under section 74, (d) ₹ 30,000 under section 74 and ₹ 70,00,000 under section 73, Answer: (c), Hint: Amount of loss to be carried forward, Income under the business profession, Income from business, 45,00,000, Income from profession, 25,00,000, Loss from business, (70,00,000), Income from business, Nil
Page 612 :
Income Under The Head Capital Gains, Income under the capital gains, LTCG, Less: Brought forward long term Loss, Income under the head capital gains, Loss to be carried forward, , 93, 6,12,457, (7,80,000), Nil, 1,67,543, , Note: Losses in agricultural commodity derivative shall be normal business as per section 43(5)., Question 23. Write a note on full value of consideration for transfer of unlisted shares, Answer: full value of consideration for transfer of unlisted shares. Section 50CA, In order to ensure the full consideration is not understated in case of transfer of unlisted shares, a new, section 50CA has been inserted to provide that where the consideration received or accruing as a result of, transfer of a capital asset, being share of a company other than a quoted share, is less than the fair market, value of such share determined in such manner as may be prescribed (given under rule 11UA which is not, covered in syllabus), such fair market value shall be deemed to be the full value of consideration received or, accruing as a result of such transfer., The provisions of this section shall not apply to any consideration received or accruing as a result of, transfer by such class of persons and subject to such conditions as may be prescribed., Question 24. Write a note on fair market value deemed to be full value of consideration in certain, cases covered under section 50D, Answer: Fair market value deemed to be full value of consideration in certain cases Section 50D, Where the consideration received or accruing as a result of the transfer of a capital asset by an assessee is, not ascertainable or cannot be determined, then, for the purpose of computing income chargeable to tax as, capital gains, the fair market value of the said asset on the date of transfer shall be deemed to be the full, value of the consideration received or accruing as a result of such transfer., Question 25 [V. Imp.]: Write a note on exemption under section 54., Answer: Profit on sale of property used for residence Section 54, 1. Assessee: The assessee should be individual or a Hindu Undivided Family. (i.e. exemption is not, allowed to firm, company, association of person or body of individual etc.), 2. Asset: Capital asset transferred should be buildings or lands appurtenant thereto, being a residential, house, the income of which is chargeable under the head “Income from house property”., 3. Type of capital gain: Capital gain should be long term., 4. Investment: The assessee has within a period of one year before or two years after the date on which, the transfer took place purchased, or has within a period of three years after that date constructed, one, residential house in India (no exemption for house outside India)., Exemption of two houses are allowed provided capital gains is upto ₹2 crores. further such option is, allowed only once in the life time of the assessee i.e. afterwards benefit of only one house shall be, allowed., 5. Amount of exemption: Exemption shall be allowed to be the extent of investment., 6. Withdrawal of exemption: The house so purchased/constructed must not be transferred within a, period of three years otherwise exemption given shall be withdrawn and for this purpose while computing, capital gains, its cost of acquisition shall be reduced by the amount of the exemption earlier allowed., 7. Capital gains account Scheme 1988: The amount of capital gain has to be utilised till the last date of, furnishing of return of income otherwise amount should be deposited in capital gains account scheme 1988, and proof of such deposit should be enclosed with the return of income. Subsequently the amount should be, withdrawn from this scheme and should be utilised for the specified purpose otherwise it will be considered, to be long term capital gain of the year in which the prescribed period has expired., 8. Extension of time for acquiring new asset or depositing or investing amount of capital gain section, 54H: If the asset has been acquired compulsorily by the Government, period of investment shall be
Page 613 :
Income Under The Head Capital Gains, , 94, , determined from the date of payment instead of the date of compulsory acquisition., 9. If any person has purchased a house and has deposited some amount in capital gain account scheme for, construction on the same house, In that case exemption shall be allowed even for the amount so deposited as, decided in B.B. Sarkar vs Commissioner Of Income-Tax (CALCUTTA HC), Illustration 21: Mr. X purchased one residential house on 01-07-2001 for ₹2,00,000 and it was sold by him, on 01-07-2021 for ₹100 lakhs and he purchased one house in 01-07-2022 for ₹20,00,000. He sold this house, on 01-07-2023 for ₹22,00,000. Compute his Tax Liability for A.Y. 2022-23 and also capital gains for, various years., Solution:, ₹, Full value of consideration, 1,00,00,000.00, Less: Indexed cost of acquisition, = 2,00,000 / 100 x 317 = ₹6,34,000, (6,34,000.00), Long Term Capital Gains, 93,66,000.00, Less: Exemption u/s 54, (20,00,000.00), Long Term Capital Gains, 73,66,000.00, Income under the head Capital Gain (LTCG), 73,66,000.00, Gross Total Income, 73,66,000.00, Less: Deduction u/s 80C to 80U, Nil, Total Income, 73,66,000.00, Computation of Tax Liability, {Since normal income is nil, as per section 112 deficiency of ₹2,50,000 shall be allowed from long term, capital gain and balance income shall be taxed at flat rate of 20%}, Tax on ₹71,16,000 (₹73,66,000 – ₹2,50,000) @ 20%, 14,23,200.00, Add: Surcharge @ 10%, 1,42,320.00, Tax before health & education cess, 15,65,520.00, Add: HEC @ 4%, 62,620.80, Tax Liability, 16,28,140.80, Rounded off u/s 288B, 16,28,140.00, Computation of Capital Gain for the assessment year 2024-25, Capital gain on sale of House, Full value of consideration, 22,00,000.00, Less: Cost of acquisition (₹20,00,000 – ₹20,00,000), (Nil), Short Term Capital Gain, 22,00,000.00, Illustration 22: Mr. X purchased one residential house on 01.04.2002 for ₹5,00,000. This house was, acquired compulsorily by the Government on 01.10.2013 and compensation of ₹50,00,000 was fixed by the, government but the amount was paid by the Government on 01.03.2022. The assessee has purchased one, residential house on 01.01.2022 for ₹2,00,000 and the house was sold by him on 01.01.2023 for ₹4,00,000., Compute his tax liability for the assessment year 2022-23 and also capital gains for the various years., Solution:, ₹, Computation of capital gains under section 45(5), Capital gain shall be computed in the year in which the asset was acquired i.e. in the previous year 201314 and shall be taxed in the year in which the first payment has been received i.e. in the previous year, 2021-22, Full value of consideration, 50,00,000.00, Less: Indexed cost of acquisition, = 5,00,000 / 105 x 220 = ₹10,47,619.05, (10,47,619.05), Long Term Capital Gains, 39,52,380.95, Less: Exemption u/s 54, (2,00,000.00), Long Term Capital Gains, 37,52,380.95, Income under the head Capital Gain (LTCG), 37,52,380.95, Gross Total Income, 37,52,380.95
Page 614 :
Income Under The Head Capital Gains, , 95, , Less: Deduction u/s 80C to 80U, Nil, Total Income (rounded off u/s 288A), 37,52,380.00, Computation of Tax Liability, {Since normal income is nil, as per section 112 deficiency of ₹2,50,000 shall be allowed from long term, capital gain and balance income shall be taxed at flat rate of 20%}, Tax on ₹35,02,380 (₹37,52,380 – ₹2,50,000) @ 20%, 7,00,476.00, Add: HEC @ 4%, 28,019.04, Tax Liability, 7,28,495.04, Rounded off u/s 288B, 7,28,500.00, Computation of Capital Gain for the assessment year 2023-24, Capital gain on sale of House, Full value of consideration, 4,00,000.00, Less: Cost of acquisition (₹2,00,000 – ₹2,00,000), Nil, Short Term Capital Gain, 4,00,000.00, Hence Short Term Capital Gain for assessment year 2023-24, 4,00,000.00, (b) Presume the house was purchased on 01.09.2022 instead of 01.01.2022., Solution:, ₹, Computation of capital gains under section 45(5), Capital gain shall be computed in the year in which the asset was acquired i.e. in the previous year 201314 and shall be taxed in the year in which the first payment has been received i.e. in the previous year, 2021-22, Full value of consideration, 50,00,000.00, Less: Indexed cost of acquisition, = 5,00,000 / 105 x 220 = ₹10,47,619.05, (10,47,619.05), Long Term Capital Gains, 39,52,380.95, Income under the head Capital Gain (LTCG), 39,52,380.95, Gross Total Income, 39,52,380.95, Less: Deduction u/s 80C to 80U, Nil, Total Income (rounded off u/s 288A), 39,52,380.00, Computation of Tax Liability, {Since normal income is nil, as per section 112 deficiency of ₹2,50,000 shall be allowed from long term, capital gain and balance income shall be taxed at flat rate of 20%}, Tax on ₹37,02,380 (₹39,52,380 – ₹2,50,000) @ 20%, 7,40,476.00, Add: HEC @ 4%, 29,619.04, Tax Liability, 7,70,095.04, Rounded off u/s 288B, 7,70,100.00, Note: Exemption under section 54 is not allowed as the house was purchased after the last date of filing of, return of income (i.e. 31st July 2022), Computation of Capital Gain for the assessment year 2023-24, Capital gain on sale of House, Full value of consideration, 4,00,000.00, Less: Cost of acquisition, (2,00,000.00), Short Term Capital Gain, 2,00,000.00, Hence Short Term Capital Gain for assessment year 2023-24, 2,00,000.00, Illustration 23: Mr. X purchased one house on 01.04.2001 for ₹2,00,000 and sold the house on 01.07.2021, for ₹70,00,000 and purchased one house on 01.09.2021 for ₹12,00,000 and it was sold by him on, 01.01.2022 for ₹15,00,000., He is aged 82 years., Compute his income and tax liability for assessment year 2022-23., Solution:, ₹, Computation of income under the head Capital Gains, Full value of consideration, 70,00,000.00
Page 615 :
Income Under The Head Capital Gains, , 96, , Less: Indexed cost of acquisition, = 2,00,000 / Index of 01-02 x Index of 21-22, = 2,00,000 / 100 x 317 = ₹6,34,000, (6,34,000.00), Long Term Capital Gains, 63,66,000.00, The assessee has the option either not to avail exemption under section 54 or to avail exemption under, section 54 and also it will be withdrawn, Option I, Exemption is not availed:, Long Term Capital Gain, 63,66,000.00, Sale of house purchased on 01.09.2021, Full value of consideration, 15,00,000.00, Less: Cost of acquisition, (12,00,000.00), Short term capital gain, 3,00,000.00, Income under the head Capital Gains, 66,66,000.00, Gross Total Income, 66,66,000.00, Less: Deduction u/s 80C to 80U, Nil, Total Income, 66,66,000.00, Computation of Tax Liability, Tax on LTCG ₹61,66,000 (₹63,66,000 – ₹2,00,000) @ 20%, 12,33,200.00, Tax on ₹3,00,000 at slab rate, Nil, Add: Surcharge @10%, 1,23,320.00, Tax before health & education cess, 13,56,520.00, Add: HEC @ 4%, 54,260.80, Tax Liability, 14,10,780.80, Rounded off u/s 288B, 14,10,780.00, Option II, Exemption is availed, Long Term Capital Gain, 63,66,000.00, Less: Exemption u/s 54, (12,00,000.00), Long Term Capital Gain, 51,66,000.00, Sale of house purchased on 01.09.2021, Full value of consideration, 15,00,000.00, Less: Cost of acquisition (12,00,000 – 12,00,000), Nil, Short term capital gain, 15,00,000.00, Income under the head Capital Gains, 66,66,000.00, Gross Total Income, 66,66,000.00, Less: Deduction u/s 80C to 80U, Nil, Total Income, 66,66,000.00, Computation of Tax Liability, Tax on LTCG ₹51,66,000 @ 20%, 10,33,200.00, Tax on ₹15,00,000 at slab rate, 2,50,000.00, Tax before surcharge, 12,83,200.00, Add: Surcharge @10%, 1,28,320.00, Tax before health & education cess, 14,11,520.00, Add: HEC @ 4%, 56,460.80, Tax Liability, 14,67,980.80, Rounded off u/s 288B, 14,67,980.00, Hence assessee should opt Option-I., Illustration 24: Mr. X Purchased one residential house on 01.04.2001 for ₹ 2,00,000 and it was sold by him, on 01.07.2021 for ₹ 50,00,000 and he purchased a new house on 01.09.2021 for ₹ 55,00,000 and this house, was sold by him on 01.07.2024 for ₹ 56,00,000. Compute his tax liability for A.Y. 2022-23 and also capital, gains for all the years.
Page 616 :
Income Under The Head Capital Gains, Solution:, Computation of Capital Gains, Full value of consideration, Less: Indexed cost of acquisition, = 2,00,000 / 100 x 317 = ₹6,34,000, Long Term Capital Gain, Less: Exemption u/s 54, Long Term Capital Gain, Income under the head Capital Gain (LTCG), Gross Total Income, Less: Deduction u/s 80C, Total Income, Tax Liability, Previous year 2024-25, Full value of consideration, Less: Cost of acquisition 55,00,000, Less: Exemption allowed (43,66,000), Balance, 11,34,000, ICOA = 11,34,000 x index of 2024-25/index of 2021-22, Since index of 2024-25 is not available hence computation is not possible., , 97, , 50,00,000.00, (6,34,000.00), 43,66,000.00, (43,66,000.00), Nil, Nil, Nil, Nil, Nil, Nil, 56,00,000.00, , Illustration 25: Mr. X purchased a residential house on July, 2019 for ₹ 10,00,000 and made some additions, to the house incurring ₹ 2,00,000 in August 2019. He sold the house property in April 2021 for ₹20,00,000., Out of the sale proceeds, he spent ₹ 5,00,000 to purchase another house property in September 2021., What is the amount of capital gains taxable in the hands of Mr. X for the A.Y.2022-23?, Solution:, The house is sold before 24 months from the date of purchase. Hence, the house is a short term capital asset, and no benefit of indexation would be available., Particulars, ₹, Sale consideration, 20,00,000, Less: Cost of acquisition, (10,00,000), Less: Cost of improvement, (2,00,000), Short-term capital gains, 8,00,000, Note: The exemption of capital gains under section 54 is available only in case of long-term capital asset., As the house is short-term capital asset, Mr. X cannot claim exemption under section 54. Thus, the amount, of taxable short-term capital gains is ₹ 8,00,000., Question 26 [V. Imp.]: Write a note on exemption under section 54B., Answer: Capital gain on transfer of land used for agricultural purposes not to be charged in certain, cases Section 54B, 1. Assessee: The assessee should be individual or a Hindu Undivided Family. (i.e. exemption is not, allowed to firm, company, association of person or body of individual etc.), 2. Asset: The asset transferred should be land which, in the two years immediately preceding the date on, which the transfer took place, was being used by the assessee or a parent of his for agricultural purposes., 3. Type of capital gain: It may be short term or long term., 4. Investment: The assessee has, within a period of two years after that date, purchased any other land for, being used for agricultural purposes., 5. Amount of exemption: Exemption allowed shall be equal to the amount invested., 6. Withdrawal of exemption: The land so purchased must not be transferred within a period of three, years otherwise exemption given shall be withdrawn and for this purpose while computing capital gains on, the transfer of new asset, its cost of acquisition shall be reduced by the amount of the exemption earlier, allowed., 7. Capital gains account Scheme 1988: The amount of capital gain has to be utilised till the last date of
Page 617 :
Income Under The Head Capital Gains, , 98, , furnishing of return of income otherwise amount should be deposited in capital gains account scheme 1988, and proof of such deposit should be enclosed with the return of income and subsequently the amount should, be withdrawn from this scheme and should be utilised for the specified purpose otherwise it will be, considered to be capital gain of the year in which the prescribed period has expired., Capital gains in case of compulsory acquisition of agricultural land Section 10(37), If any individual or Hindu Undivided Family has agricultural land and this land was being used by him for, agricultural purposes for a period of at least 2 years when it was acquired by the government, in this case, capital gains shall be exempt from income tax., Illustration 26: Mr. X purchased agricultural land in urban area on 01.10.2002 for ₹3,00,000 and it was, being used for agricultural purposes by him. It was sold on 01.01.2022 for ₹50,00,000. The assessee has, purchased one agricultural land in the rural area on 10.01.2022 for ₹10,00,000 and this land was sold by him, on 11.02.2022 for ₹11,00,000 and has invested ₹30,000 in National Saving Certificate., He is aged about 86 years., Compute his tax liability for assessment year 2022-23., (b) Presume the land was purchased in the urban area instead of rural area., Solution (a):, ₹, Computation of Capital Gains, Full value of consideration, 50,00,000.00, Less: Indexed cost of acquisition, = 3,00,000 / 105 x 317 = ₹9,05,714.29, (9,05,714.29), Long Term Capital Gain, 40,94,285.71, Less: Exemption u/s 54B, (10,00,000.00), Long Term Capital Gain, 30,94,285.71, Income under the head Capital Gain (LTCG), 30,94,285.71, Gross Total Income, 30,94,285.71, Less: Deduction u/s 80C, Nil, {Deduction u/s 80C is not allowed from LTCG}, Total Income, 30,94,290.00, Computation of Tax Liability, {Since normal income is nil, as per section 112 deficiency of ₹5,00,000 shall be allowed from long term, capital gain and balance income shall be taxed at flat rate of 20%}, Tax on ₹25,94,290 (₹30,94,290.00 – ₹5,00,000) @ 20%, 5,18,858.00, Add: HEC @ 4%, 20,754.32, Tax Liability, 5,39,612.32, Tax Liability (Rounded off u/s 288B), 5,39,610.00, Note: If land is purchased in rural area, exemption is allowed under section 54B but on its sale exemption is, not withdrawn., Solution (b):, ₹, Computation of Capital Gains, Full value of consideration, 50,00,000.00, Less: Indexed cost of acquisition, = 3,00,000 / 105 x 317 = ₹9,05,714.29, (9,05,714.29), Long Term Capital Gain, 40,94,285.71, The assessee has the option either not to avail exemption under section 54B or to avail exemption under, section 54B., Option I, Exemption is not availed:, Long Term Capital Gain, 40,94,285.71, Urban agricultural land, Full value of consideration, 11,00,000.00, Less: Cost of acquisition, (10,00,000.00), Short Term Capital Gain, 1,00,000.00, Income under the head Capital Gains, 41,94,285.71, Gross Total Income, 41,94,285.71
Page 618 :
Income Under The Head Capital Gains, Less: Deduction u/s 80C {NSC}, Total Income, Total Income (Rounded Off u/s 288A), Computation of tax liability, Tax on long term capital gain ₹36,64,290 (40,94,290 – 4,30,000) @ 20%, Tax on ₹70,000 at slab rate, Add: HEC @ 4%, Tax Liability, Tax Liability (Rounded Off u/s 288B), Option II, Exemption is availed:, Long Term Capital Gain, Less: Exemption u/s 54B, Long Term Capital Gain, Urban agricultural land, Full value of consideration, Less: Cost of acquisition (10,00,000 – 10,00,000), Short Term Capital Gain, Income under the head Capital Gains, Gross Total Income, Less: Deduction u/s 80C {NSC}, Total Income, Total Income (Rounded Off u/s 288A), Computation of tax liability, Tax on long term capital gain ₹30,94,290 @ 20%, Tax on ₹10,70,000 at slab rate, Tax before health & education cess, Add: HEC @ 4%, Tax Liability, Tax Liability (Rounded off u/s 288B), Hence the assessee should opt for option–I and his tax liability shall be 7,62,170., , 99, (30,000.00), 41,64,285.71, 41,64,290.00, 7,32,858.00, Nil, 29,314.32, 7,62,172.32, 7,62,170.00, 40,94,285.71, (10,00,000.00), 30,94,285.71, 11,00,000.00, Nil, 11,00,000.00, 41,94,285.71, 41,94,285.71, (30,000.00), 41,64,285.71, 41,64,290.00, 6,18,858.00, 1,21,000.00, 7,39,858.00, 29,594.32, 7,69,452.32, 7,69,450.00, , Illustration 27: Mr. X has an agricultural land (costing ₹ 6 lakh) in Lucknow and has been using it for, agricultural purposes since 01.04.2003 till 01.08.2013 when the Government took over compulsory, acquisition of this land. A compensation of ₹ 10 lakh was settled. The compensation was received by Mr. X, on 01.07.2021. Compute the amount of capital gains taxable in the hands of Mr. X., Solution: In the given problem, compulsory acquisition of an urban agricultural land has taken place. This, land had also been used for at least 2 years by the assessee himself for agricultural purposes. Thus, as per, section 10(37), entire capital gains arising on such compulsory acquisition will be fully exempt and nothing, is taxable in the hands of Mr. X in the year of receipt of compensation i.e. A.Y.2022-23., Illustration 28: Will your answer be any different if Mr. X had by his own will sold this land to his friend, Mr. Y? Explain., Solution: As per section 10(37), exemption is available if compulsory acquisition of urban agricultural land, takes place. Since the sale is out of own will and desire, the provisions of this section are not attracted and, the capital gains arising on such sale will be taxable in the hands of Mr. X., Illustration 29: Will your answer be different if Mr. X had not used this land for agricultural activities?, Explain., Solution: As per section 10(37), exemption is available only when such land has been used for agricultural, purposes during the preceding two years by such individual or a parent of his or by such HUF. Since the, assessee has not used it for agricultural activities, the provisions of this section are not attracted and the, capital gains arising on such compulsory acquisition will be taxable in the hands of Mr. X.
Page 619 :
Income Under The Head Capital Gains, , 100, , Illustration 30: Will your answer be different if the land belonged to ABC Ltd. and not Mr. X and, compensation on compulsory acquisition was received by the company? Explain., Solution: Section 10(37) exempts capital gains arising to an individual or a HUF from transfer of, agricultural land by way of compulsory acquisition. Since the land belongs to ABC Ltd., a company, the, provisions of this section are not attracted and the capital gains arising on such compulsory acquisition will, be taxable in the hands of ABC Ltd., Question 27: Write a note on exemption under section 54D., Answer: Capital gain on compulsory acquisition of lands and buildings not to be charged in certain, cases Section 54D, 1. Assessee: Exemption is allowed to all the assessee., 2. Asset: The asset should be land or building forming part of an industrial undertaking belonging to the, assessee which, in the two years immediately preceding the date on which the transfer took place, was, being used by the assessee for the purposes of the business of the said undertaking and further there should, be compulsory acquisition., 3. Type of capital gain: It can be short term or long term., 4. Investment: The assessee can invest the amount in land or building for the purpose of industrial, undertaking within a period of three years after the date of payment by the Govt., 5. Amount of exemption: Exemption allowed is equal to investment., 6. Withdrawal of exemption: The land or building so purchased/constructed must not be transferred, within a period of three years otherwise exemption given shall be withdrawn and for this purpose while, computing capital gains on the transfer of new asset, its cost of acquisition shall be reduced by the amount of, the exemption earlier allowed., 7. Capital gains account Scheme 1988: The amount of capital gain has to be utilised till the last date of, furnishing of return of income otherwise amount should be deposited in capital gains account scheme 1988, and proof of such deposit should be enclosed with the return of income. Subsequently the amount should be, withdrawn from this scheme and should be utilised for the specified purpose otherwise it will be considered, to be capital gain of the year in which the prescribed period has expired., Illustration 31: Mr. X has one industrial undertaking in Wazirpur industrial area and the building which is, being used for industrial purposes was purchased on 01.10.2007. Since then it was being used for industrial, purpose and was purchased for ₹23,00,000 and its w.d.v. as on 01.04.2014 is ₹10,38,000. This building was, acquired by the Government on 01.01.2015 and compensation fixed was ₹25,00,000. Entire payment was, released by the Government on 01.07.2021. The assessee has purchased one building for the purpose of, industrial undertaking in Bawana Industrial Area on 01.01.2022 for ₹6,00,000., Compute his tax liability for assessment year 2022-23., Solution:, ₹, Computation of Capital Gains under section 45(5), Capital gains shall be computed in the year of compulsory acquisition i.e. in the previous year 2014-15, Full value of consideration, 25,00,000.00, Less: W.d.v of the building, (10,38,000.00), Short Term Capital Gain, 14,62,000.00, Computation of capital gains and tax liability for the assessment year 2022-23, Capital gain shall be taxed in the year in which payment has been given by the Government i.e. in the, previous year 2021-22, Short Term Capital Gain, 14,62,000.00, Less: Exemption u/s 54D, (6,00,000.00), Short Term Capital Gain, 8,62,000.00, Income under the head Capital Gain (STCG), 8,62,000.00, Gross Total Income, 8,62,000.00, Less: Deduction u/s 80C to 80U, Nil, Total Income, 8,62,000.00
Page 620 :
Income Under The Head Capital Gains, Computation of Tax Liability, Tax on ₹8,62,000 at slab rate, Add: HEC @ 4%, Tax Liability, Rounded off u/s 288B, , 101, , 84,900.00, 3,396.00, 88,296.00, 88,300.00, , Question 28 [V. Imp.]: Write a note on exemption under section 54EC., Answer: Capital gain not to be charged on investment in certain bonds Section 54EC, 1. Assessee: Exemption is allowed to all the assessee., 2. Asset: The assessee can transfer any land or building., 3. Type of capital gain: It should be only long term capital gain., 4. Investment: The assessee has, at any time within a period of six months after the date of such transfer,, invested the whole or any part of capital gains in the long-term specified asset., “Long-term specified asset” means any bond redeemable after five years, issued by,—, (i) National Highways Authority of India, (ii) Rural Electrification Corporation Limited, (iii) Power Finance Corporation Limited., (iv) Indian Railway Finance Corporation Limited., 5. Amount of exemption: Maximum exemption allowed in a particular previous year shall be ₹50 lakh., 6. Withdrawal of exemption: If the long term specified asset is transferred or converted into cash within a, period of 5 years, exemption earlier allowed shall be considered to be long term capital gains of the year in, which such asset was transferred or converted into cash., Converting into cash means taking a loan on the security of the specified asset., 7. Capital gains account scheme 1988: Capital gain account scheme shall not apply., 8. Extension of time for acquiring new asset or depositing or investing amount of capital gain section, 54H: If the asset has been acquired compulsorily by the Government, period of investment shall be, determined from the date of payment instead of the date of compulsory acquisition., Illustration 32: Mr. X purchased agricultural land in the urban area on 01.04.2001 for ₹2,00,000. It was, being used for agricultural purposes since then and was sold by the assessee on 01.07.2021 for ₹123,00,000., He made following investments:, (i) Bonds of National Bank for Agriculture and Rural Development on 01.06.2021 for ₹1,50,000 which, are redeemable after 5 years., (ii) He purchased agricultural land on 01.09.2021 for ₹2,00,000., (iii)He has invested ₹75,000 on 01.10.2021 in the bonds of National Highway Authority of India, redeemable after five years., He sold the bonds of National Highway Authority of India on 15.04.2022 for ₹3,00,000., Compute his capital gains for various years and also tax liability for assessment year 2022-23., Solution:, ₹, Previous year 2021-22, Computation of Capital gains, Full value of consideration, 123,00,000.00, Less: Indexed cost of acquisition, = 2,00,000 / 100 x 317 = ₹6,34,000, (6,34,000.00), Long Term Capital Gain, 116,66,000.00, Less: Exemption u/s 54B, (2,00,000.00), Less: Exemption u/s 54EC, (75,000.00), Long Term Capital Gain, 113,91,000.00, Income under the head Capital Gain (LTCG), 113,91,000.00, Gross Total Income, 113,91,000.00, Less: Deduction u/s 80C to 80U, Nil, Total Income, 113,91,000.00
Page 621 :
Income Under The Head Capital Gains, , 102, , Computation of Tax Liability, {Since normal income is nil, as per section 112 deficiency of ₹2,50,000 shall be allowed from long term, capital gain and balance income shall be taxed at flat rate of 20%}, Tax on ₹111,41,000 (₹113,91,000 – ₹2,50,000) @ 20%, 22,28,200.00, Tax before Surcharge, 22,28,200.00, Add: Surcharge @ 15%, 3,34,230.00, Tax before health & education cess, 25,62,430.00, Add: HEC @ 4%, 1,02,497.20, Tax Liability, 26,64,927.20, Rounded off u/s 288B, 26,64,930.00, Previous year 2022-23, Full value of consideration, Less: Cost of acquisition, Short Term Capital Gain, Long Term Capital Gain (withdrawal of exemption), , 3,00,000.00, (75,000.00), 2,25,000.00, 75,000.00, , Illustration 33: ABC Ltd. purchased one commercial building on 01.07.1995 ₹2,00,000 and paid brokerage, ₹20,000 and its market value as on 01.04.2001 ₹2,10,000. The company sold the building on 01.07.2021 for, ₹500,00,000 and invested ₹60,00,000 in bond of NHAI redeemable after five years., Compute tax liability of the company for Assessment Year 2022-23., (b) Presume building was sold for ₹11,72,00,000., Solution:, ₹, Computation of Capital gains, Full value of consideration, 500,00,000.00, Less: Indexed cost of acquisition, = 2,20,000 / 100 x 317 = ₹6,97,400, (6,97,400.00), Long Term Capital Gain, 493,02,600.00, Less: Exemption u/s 54EC, (50,00,000.00), (exemption under section 54EC cannot exceed ₹50,00,000), Long Term Capital Gain, 443,02,600.00, Income under the head Capital Gain (LTCG), 443,02,600.00, Gross Total Income, 443,02,600.00, Less: Deduction u/s 80C to 80U, Nil, Total Income, 443,02,600.00, Computation of Tax Liability, Tax on ₹443,02,600 @ 20%, 88,60,520.00, Add: Surcharge @ 7%, 6,20,236.40, Tax before health & education cess, 94,80,756.40, Add: HEC @ 4%, 3,79,230.26, Tax Liability, 98,59,986.66, Rounded off u/s 288B, 98,59,990.00, (b), Solution:, ₹, Computation of Capital gains, Full value of consideration, 11,72,00,000.00, Less: Indexed cost of acquisition, = 2,20,000 / 100 x 317 = ₹6,97,400, (6,97,400.00), Long Term Capital Gain, 11,65,02,600.00, Less: Exemption u/s 54EC, (50,00,000.00), (exemption under section 54EC cannot exceed ₹50,00,000), Long Term Capital Gain, 11,15,02,600.00, Income under the head Capital Gain (LTCG), 11,15,02,600.00, Gross Total Income, 11,15,02,600.00
Page 622 :
Income Under The Head Capital Gains, Less: Deduction u/s 80C to 80U, Total Income, Computation of Tax Liability, Tax on ₹11,15,02,600 @ 20%, Add: Surcharge @ 12%, Tax before health & education cess, Add: HEC @ 4%, Tax Liability, Rounded off u/s 288B, , 103, Nil, 11,15,02,600.00, 223,00,520.00, 26,76,062.40, 249,76,582.40, 9,99,063.30, 259,75,645.70, 259,75,650.00, , Question 29: Write a note on exemption under section 54EE., Answer: Exemption in case of investment in units of mutual fund Section 54EE, 1. Assessee: Exemption is allowed to all the assessee., 2. Asset: The assessee can transfer any capital asset., 3. Type of capital gain: It should be only long term capital gain., 4. Investment: Assessee can make investment within a period of six months after the date of transfer of, original asset and investment should be in Long-term specified asset., “Long-term specified asset” means units of such fund as may be notified by the Central Government., 5. Amount of exemption: Maximum exemption allowed in a particular previous year shall be ₹50 lakh., 6. Withdrawal of exemption: If the long term specified asset is transferred or converted into cash within a, period of 3 years, exemption earlier allowed shall be considered to be long term capital gains of the year in, which such asset was transferred or converted into cash., Converting into cash means taking a loan on the security of the specified asset., 7. Capital gains account scheme 1988: Capital gain account scheme shall not apply., Question 30 [V. Imp.]: Write a note on exemption under Section 54F., Answer: Exemption from capital gains on transfer of any capital assets other than a Residential, House Section 54F, 1. Assessee: The assessee should be individual or Hindu Undivided Family., 2. Asset: Capital asset transferred can be any asset but it should not be a residential house., 3. Type of capital gain: Capital gain should be long term., 4. Investment: The assessee has within a period of one year before or two years after the date on which, the transfer took place purchased, or has within a period of three years after that date constructed, one, residential house and further the assessee should either purchase or construct only one house and also, assessee should not have more than one house in his name at the time of transfer of the asset besides the, house which is being purchased or constructed for availing exemption., 5. Amount of exemption: Exemption allowed shall be that percentage of the capital gain as the amount, invested bears to net consideration. i.e. exemption = capital gain x investment / net consideration., Net consideration is equal to full value of consideration less selling expenses. i.e. full value of, consideration – selling expenses., 6. Withdrawal of exemption: The house so purchased or constructed must not be transferred for a, minimum period of three years otherwise exemption earlier allowed shall be considered to be the long term, capital gain of the year in which the asset has been transferred (i.e. exemption shall be withdrawn in the, similar manner as given under section 54EC)., Similarly if the assessee has purchased any other house within one year before or two years after or the, assessee has constructed any other house within three years after the date of transfer of original asset,, exemption given shall be withdrawn in that case also., 7. Capital gains account Scheme 1988: The assessee should invest the amount till the last date of, furnishing of return of income otherwise amount should be deposited in capital gains account scheme 1988, and proof of such deposit should be enclosed with the return of income and subsequently the amount should, be withdrawn from this scheme and should be utilised for the specified purpose otherwise exemption earlier, allowed will be considered to be long term capital gain of the year in which the prescribed period has, expired.
Page 623 :
Income Under The Head Capital Gains, , 104, , 8. Extension of time for acquiring new asset or depositing or investing amount of capital gain section, 54H: If the asset has been acquired compulsorily by the Government, period of investment shall be, determined from the date of payment instead of the date of compulsory acquisition., Illustration 34: Mr. X purchased gold on 01.04.1991 for ₹3,00,000 and its market value on 01.04.2001 is, ₹2,00,000. This gold was sold by him on 01.01.2022 for ₹35,00,000 and selling expenses are ₹37,000. He, has purchased one house on 01.05.2022 for ₹4,00,000 because he did not have any house in his name and he, deposited ₹3,00,000 in capital gain account scheme on 30.09.2022., Mr. X is also engaged in a business and he has turnover of his business ₹105,00,000 and cost of goods sold, ₹100,00,000 and other expenses ₹5,10,000., He has withdrawn ₹2,00,000 from capital gain account scheme on 01.01.2023 and constructed 1st floor of, the house which was purchased by him on 01.05.2022., Remaining amount in the capital gain account scheme was unutilized., Compute assessee’s tax liability for assessment year 2022-23 and capital gains for various years., Solution:, ₹, Previous year 2021-22, Computation of capital gain, Full value of consideration, 35,00,000.00, Less: Indexed cost of acquisition, = 3,00,000 / 100 x 317 = ₹9,51,000, (9,51,000.00), Less: Selling expenses, (37,000.00), Long Term Capital Gain, 25,12,000.00, Less: Exemption u/s 54F, = 25,12,000 / 34,63,000 x 7,00,000, (5,07,767.83), Long Term Capital Gain, 20,04,232.17, Income under the head Capital Gain (LTCG), 20,04,232.17, Loss under the head Business/Profession, (10,000.00), Gross Total Income, 19,94,232.17, Less: Deduction u/s 80C to 80U, Nil, Total Income (rounded off u/s 288A), 19,94,230.00, Computation of tax liability, {Since normal income is nil, as per section 112 deficiency of ₹2,50,000 shall be allowed from long term, capital gain and balance income shall be taxed at flat rate of 20%}, Tax on ₹17,44,230 (19,94,230 – 2,50,000) @ 20%, 3,48,846.00, Add: HEC @ 4%, 13,953.84, Tax Liability, 3,62,799.84, Tax Liability (rounded off u/s 288B), 3,62,800.00, Previous year 2024-25, Amount deposited in capital gain a/c scheme, 3,00,000.00, Less: Amount withdrawn, (2,00,000.00), Balance amount, 1,00,000.00, Long Term Capital Gain, = 25,12,000 x 1,00,000 = ₹72,538.26, 72,538.26, 34,63,000, (Proportionate exemption with regard to the unutilized amount lying in the capital gain account scheme is, chargeable to tax after expiry of period of three years.), Illustration 35: Mr. X sold gold for ₹5,50,000 on 01.10.2021 which had been acquired by him in October,, 2004 for ₹55,000. He wants to utilize the said amount of sale consideration for purchase or construction of a, new residential house. He already owns one residential house at the time of sale of the gold on 01.10.2021., He has deposited ₹4,00,000 under the capital gains deposit scheme with a specified bank on 30.04.2022., Ascertain the capital gains taxable in Mr. X’s hands for assessment year 2022-23 and advise him as to what, further action he has to take to avail the exemption.
Page 624 :
Income Under The Head Capital Gains, , 105, , Solution:, ₹, Computation of Capital Gains, Full value of consideration, 5,50,000.00, Less: Indexed cost of acquisition, = 55,000 / 113 x 317 = ₹1,54,292.04, (1,54,292.04), Long Term Capital Gain, 3,95,707.96, Less: Exemption u/s 54F, = 3,95,707.96/5,50,000 x 4,00,000 = ₹2,87,787.61, (2,87,787.61), Long Term Capital Gain, 1,07,920.35, X has to fulfill the following conditions so as to avail exemption of section 54F, • He should acquire a residential house property by withdrawing from the deposit account. The new, house can be purchased at any time upto 30.09.2023 or it can be constructed upto 30.09.2024. If, the amount utilised is lower than ₹4,00,000 then the following amount will become chargeable to, tax as long term capital gain for the assessment year 2024-25, = [₹4,00,000 – Amount utilised]/ 5,50,000 x 3,95,707.96, • He should not transfer the new house within 3 years, • He should not purchase another residential house upto 30.09.2023 and he should not complete, construction of another residential house property upto 30.09.2024., , MTP NOV-2020 (7 Marks), 3. (b) Mr. Yusuf bought a vacant land for ₹ 80 lakhs in March 2005. Registration and other expenses, were 10% of the cost of land. He constructed a residential building on the said land for ₹ 100 lakhs, during the financial year 2006-07., He entered into an agreement for sale of the above said residential house with Mr. John (not a, relative) in July 2021. The sale consideration was fixed at ₹ 600 lakhs and on the date of, agreement, Mr. Yusuf received ₹ 20 lakhs as advance in cash. The stamp duty value on that date, was ₹ 620 lakhs., The sale deed was executed and registered on 10-2-2022 for the agreed consideration. However,, the State stamp valuation authority had revised the values, hence, the value of property for stamp, duty purposes was ₹ 670 lakhs. Mr. Yusuf paid 1% as brokerage on sale consideration received., Subsequent to sale, Mr. Yusuf made following investments:, (i), Acquired a residential house at Delhi for ₹ 80 lakhs., (ii), Acquired a residential house at London for ₹ 40 lakhs., (iii) Subscribed to NHAI bond: ₹ 45 lakhs on 29-5-2022 and ₹ 15 lakhs on 12-7-2022., Compute the income chargeable under the head “Capital Gains” for A.Y. 2022-23. The choice of, exemption must be in the manner most beneficial to the assessee., Cost Inflation Index:, , F.Y 2004-05, F.Y 2006-07, F.Y 2021-22, , 113, 122, 317, , Answer:, Computation of income chargeable under the head “Capital Gains” for A.Y.2022-23, Particulars, Capital Gains on sale of residential building, Actual sale consideration ₹ 600 lakhs, Value adopted by Stamp Valuation Authority ₹670 lakhs, Gross Sale consideration, [Where the actual sale consideration is less than the value adopted by the Stamp, , ₹, (in lakhs), , ₹, (in lakhs), , 670.00
Page 625 :
Income Under The Head Capital Gains, Valuation Authority for the purpose of charging stamp duty, and such stamp duty, value exceeds 110% of the actual sale consideration, then, the value adopted by, the Stamp Valuation Authority shall be taken to be the full value of consideration, as per section 50C., In a case where the date of agreement is different from the date of registration,, stamp duty value on the date of agreement can be considered provided the whole, or part of the consideration is paid by way of account payee cheque/bank draft or, by way of ECS through bank account or through any other prescribed electronics, mode on or before the date of agreement., In this case, since advance of ₹ 20 lakh is received in cash on the date of, agreement, stamp duty value on the date of registration has to be considered., Since stamp duty value on the date of registration exceeds 110% of the actual, consideration, such stamp duty value on the date of registration would be taken, as full value of consideration], Less: Brokerage@1% of sale consideration (1% of ₹600 lakhs), Net Sale consideration, Less: Indexed cost of acquisition, - Cost of vacant land, ₹ 80 lakhs, plus registration and other expenses i.e.,, ₹ 8 lakhs, being 10% of cost of land [₹ 88 lakhs /113 × 317], - Construction cost of residential building (₹ 100 lakhs/122 x 317), Long-term capital gains before exemption, Less: Exemption under section 54, Since the amount of capital gain does not exceed ₹ 2 crore, the capital gain, arising on transfer of a long-term residential property shall not be, chargeable to tax to the extent such capital gain is invested in the purchase, of two residential house property in India one year before or two years, after the date of transfer of original asset, at the option of the assessee., , 106, , (6.00), 664.00, 246.867, 259.836, , 506.703, 157.297, 80.00, , However, in the present case, the exemption would be available only in, respect of the residential house acquired at Delhi and not in respect of the, residential house in London since the residential house property should be, purchased or constructed in India, Less: Exemption under section 54EC, Amount deposited in capital gains bonds of NHAI within six months from, the date of transfer (i.e., on or before 09.08.2022) would qualify for, exemption, to the maximum extent of ₹ 50 lakhs., , (50.00), , Therefore, in the present case, exemption can be availed only to the extent, of ₹ 50 lakh out of ₹ 60 lakhs, even if the both the investments are made, on or before 09.08.2022 (i.e., within six months from the date of transfer)., Long term capital gains chargeable to tax, , 27.297, , Note: Since the residential house property was held by Mr. Yusuf for more than 24 months immediately, preceding the date of its transfer, the resultant gain is a long-term capital gain., Question 31: Write a note on computation of capital gains on conversion of debentures etc. into, shares., Answer: Capital gains on conversion of debentures etc. into shares, As per section 47, no capital gain shall be computed in case of conversion of debenture etc. into shares,, however if subsequently these shares have been sold, capital gains shall be computed in the manner given, below:, 1. As per section 49(2A), the cost of acquisition of the shares shall be the cost of acquisition of the, debentures etc.
Page 626 :
Income Under The Head Capital Gains, , 107, , 2. Period of holding shall start from the date of purchasing the debentures etc., Illustration 36: Mr. X has purchased 100 debentures in ABC Ltd. on 01.10.2002 @ ₹300 per debentures, and subsequently these debentures were converted into shares on 01.10.2017 and 3 shares were issued for, each debenture., The assessee has sold all the shares on 01.04.2021 @ ₹750 per share and market value as on 31-01-2018, ₹500 per share . Compute capital gains for the assessment year 2022-23 in the following situations:, (a) STT not paid, (b) STT Paid, Solution:, (a) Computation of Capital Gains, Full value of consideration (300 x 750), Less: Indexed Cost of acquisition, = (100 x 300) / Index of 02-03 x Index of 21-22, = 30,000 / 105 x 317 = ₹90,571.43, Long Term Capital Gain, , ₹, 2,25,000.00, (90,571.43), 1,34,428.57, , (b) Computation of Capital Gains, Full value of consideration (300 x 750), 2,25,000.00, Less: Cost of acquisition, (1,50,000.00), Higher of, (i) COA = 100 debentures x 300 = 30,000, (ii) lower of, (a) FMV as on 31-01-2018 = 300 shares x 500 = 1,50,000, (b) sale value = 300 shares x 750 = 2,25,000, COA = 1,50,000, Long Term Capital Gain u/s 112A, 75,000.00, Illustration 37: Mr. X purchased 100 Debentures of ABC Ltd on 01.07.2005 @ ₹ 1000 per debenture and, company has converted debentures into shares on 01.10.2017 and issued 4 shares for each debenture and, market value of shares ₹ 500 per share. The assessee has sold half of the shares on 01.05.2021 @ ₹700 per, share and remaining half on 01.12.2021 @ ₹800 per share. Market value as on 31-01-2018 @ ₹500 per share, Compute Capital Gains and Income tax liability in two situations., (i) (a) Securities Transaction Tax (STT) is not paid, (b) Securities Transaction Tax (STT) is paid, (ii) Presume in the above question, there is no Conversion rather all the Debentures were sold on 01.07.2021, @ ₹ 6,000 per Debenture, Solution (i):, No Capital Gain shall be computed at the time of conversion as per Section 47. However Capital Gain shall, be computed at the time of sale of shares., (a) Securities Transaction Tax (STT) is not paid, Computation of Capital gains of Mr. X, Full value of consideration (200 x 700), 1,40,000.00, Less: Indexed Cost of acquisition, = (50 x 1,000) / Index of 05-06 x Index of 21-22, = 50,000 / 117 x 317 = ₹1,35,470.09, (1,35,470.09), Long Term Capital Gain, 4,529.91, Full value of consideration (200 x 800), 1,60,000.00, Less: Indexed Cost of acquisition, = (50 x 1,000) / Index of 05-06 x Index of 20-21, = 50,000 / 117 x 317 = ₹1,35,470.09, (1,35,470.09), Long Term Capital Gain, 24,529.91, Gross Total Income, 29,059.82, Less: Deduction u/s 80C to 80U, Nil
Page 627 :
Income Under The Head Capital Gains, Total Income, Rounded off u/s 288A, Computation of Tax Liability, Tax on Nil (₹29,060 - ₹29,060) @ 20%, Tax Liability, (b) Securities Transaction Tax (STT) paid, Computation of Capital gains of Mr. X, Full value of consideration (200 x 700), Less: Cost of acquisition, Higher of, (i) COA = 50 debentures x 1000 = 50,000, (ii) lower of, (a) FMV as on 31-01-2018 = 200 shares x 500 = 1,00,000, (b) sale value = 200 shares x 700 = 1,40,000, COA = 1,00,000, Long Term Capital Gain u/s 112A, Full value of consideration (200 x 800), Less: Cost of acquisition, Higher of, (i) COA = 50 debentures x 1000 = 50,000, (ii) lower of, (a) FMV as on 31-01-2018 = 200 shares x 500 = 1,00,000, (b) sale value = 200 shares x 800 = 1,60,000, COA = 1,00,000, Long Term Capital Gain u/s 112A, Gross Total Income, Less: Deduction u/s 80C to 80U, Total Income, Tax Liability, Solution (ii):, Full Value of consideration (100 debentures x ₹6,000), Less: Cost of acquisition (100 x ₹1,000), Long Term Capital Gain, Gross Total Income, Less: Deduction u/s 80C to 80U, Total Income, Computation of Tax Liability, Tax on LTCG ₹2,50,000 (₹5,00,000 – ₹2,50,000) @ 10%, Less: Rebate u/s 87A, Tax before HEC, Add: HEC @ 4%, Tax Liability, , 108, 29,059.82, 29,060.00, Nil, Nil, , 1,40,000.00, (1,00,000.00), , 40,000.00, 1,60,000.00, (1,00,000.00), , 60,000.00, 1,00,000.00, Nil, 1,00,000.00, Nil, 6,00,000.00, (1,00,000.00), 5,00,000.00, 5,00,000.00, Nil, 5,00,000.00, 25,000.00, (12,500.00), 12,500.00, 500.00, 13,000.00, , Question 32: Explain computation of capital gains on transfer of debentures or bonds., Answer: While computing capital gains on transfer of debenture or bond, indexation shall not be applicable, even if it is long term capital asset and LTCG shall be taxable at the rate of 10% instead of 20% e.g. Mr. X, purchased listed debentures of ABC Ltd. on 01.07.2002 for ₹5,00,000 and sold it on 01.07.2021 for, ₹15,00,000, in this case tax treatment shall be, Full Value of consideration, 15,00,000.00, Less: Cost of acquisition, (5,00,000.00), Long Term Capital Gain, 10,00,000.00
Page 628 :
Income Under The Head Capital Gains, Total Income, Tax Liability (10,00,000 – 2,50,000) x 10%, Add: HEC @ 4%, Tax Liability, , 109, 10,00,000.00, 75,000.00, 3,000.00, 78,000.00, , Question 33 [Imp.]: Write a note on reference to Valuation Officer., Answer: Reference to Valuation Officer Section 55A Rule 111AA, If the Assessing Officer is of the view that the fair market value of a capital asset computed by the assessee, is not correct, Assessing Officer may refer the valuation to the Valuation Officer in the following, circumstances:, (i) If the value in the opinion of the Assessing Officer is exceeding by more than 15% of the value computed, by the assessee or it is exceeding by more than ₹25,000 of the value computed by the assessee., Example, Mr. X has converted one capital asset into stock in trade and its market value computed by the assessee is, ₹1,00,000 but in the opinion of the Assessing Officer, value should be ₹1,10,000, in this case valuation can, not be referred to the Valuation Officer. But if the value in the opinion of the Assessing Officer is ₹1,20,000,, in this case matter can be referred to the Valuation Officer., Similarly, if the value computed by the assessee is ₹2,00,000 but in the opinion of the Assessing Officer, value should be ₹2,27,000, matter can be referred to the Valuation Officer., (ii) in a case where the value of the asset has been estimated by a registered valuer, if the Assessing Officer, is of opinion that the value so claimed is at variance with its fair market value., “Valuation Officer” means an expert employed by Income Tax Department to determine the value of, various assets., Question 34: Write a note on Cost of Improvement., Answer: Cost of Improvement Section 55(1), Cost of improvement means expenditure of capital nature incurred in connection with capital asset i.e., if any expenditure is of revenue nature and has been claimed as an expenditure while computing income, under any head, it will not to be considered to be cost of improvement., Example, If an additional floor has been constructed in an existing house, it will be considered to be cost of, improvement but if it is a case of minor repairs or white washing, painting etc., it will not be considered to, be cost of improvement., Cost of improvement in different cases is determined in the manner given below:, 1. If expenditure is incurred before 01.04.2001, it will not be taken into consideration., 2. If expenditure is incurred from 01.04.2001, actual expenditure incurred shall be taken into, consideration., 3. In case of following intangible assets, cost of improvement shall be nil, (i) goodwill., (ii) right to manufacture, produce or process any article or thing (patent right)., (iii) right to carry on any business or profession (Franchisee)., Question 35: Write a note on Cost of Acquisition., Answer: Cost of Acquisition Section 55(2), Cost of acquisition means the actual expenditure incurred for acquiring an asset and it will be, determined in the manner given below:, 1. If the asset is acquired before 01.04.2001, cost of acquisition shall be the expenditure incurred by the, assessee for acquiring the asset or its fair market value as on 01.04.2001, whichever is higher., However, In case of land or building or both, the fair market value of such asset on the 1st day of April,, 2001, shall not exceed the stamp duty value, wherever available, of such asset as on the 1st day of April,, 2001., 2. If the asset has been acquired with effect from 01.04.2001 onwards, the cost of acquisition shall be the
Page 629 :
Income Under The Head Capital Gains, , 110, , expenditure incurred by the assessee for acquiring the asset., 3. In case of original shares/right shares, cost of acquisition shall be the actual amount paid for purchasing, the shares., 4. In case of bonus shares, cost of acquisition shall be nil, but if the bonus shares have been issued prior to, 01.04.2001, their cost of acquisition shall be the market value on 01.04.2001., 5. If the right to purchase the rights shares has been renounced, cost of acquisition of such right shall be, taken to be nil., 6. In case of rights renouncee, cost shall be the amount paid to the company and to the right holder who has, renounced the right., 7. “Cost of acquisition” in relation to a capital asset, being goodwill of a business or profession, or a trade, mark or brand name associated with a business or profession, or a right to manufacture, produce or, process any article or thing, or right to carry on any business or profession, or tenancy rights, or stage, carriage permits, or loom hours,(i) in the case of acquisition of such asset by the assessee by purchase from a previous owner, means the, amount of the purchase price; and, (ii) in the case falling under section 49(1) and where such asset was acquired by the previous owner (as, defined in that section) by purchase, means the amount of the purchase price for such previous owner;, and, (iii) in any other case, shall be taken to be nil:, Provided that where the capital asset, being goodwill of a business or profession, in respect of which a, deduction on account of depreciation under section 32 has been obtained by the assessee in any previous, year preceding the previous year relevant to the assessment year commencing on or after the 1st day of, April, 2021, the provisions of sub-clauses (i) and (ii) shall apply with the modification that the total, amount of depreciation obtained by the assessee under sub-section (1) of section 32 before the assessment, year commencing on the 1st day of April, 2021 shall be reduced from the amount of purchase price;, Illustration 38: Mr. X started business on 01-04-2001 and Mr. Y started profession on 01-04-2001. Each, one of them sold goodwill on 01-07-2021 for ₹ 50,00,000, in this case tax treatment shall be as given below:, 1. Capital Gains of Mr. X, Full value of consideration, 50,00,000.00, Less: Indexed cost of acquisition, Nil, Long Term Capital Gain, 50,00,000.00, Cost of acquisition of self - generated goodwill of business shall be taken to be Nil., 2. Capital Gains of Mr. Y, Full value of consideration, Less: Indexed cost of acquisition, Long Term Capital Gain, Cost of acquisition of self - generated goodwill of Profession shall be taken to be Nil., , 50,00,000.00, Nil, 50,00,000.00, , Question 36: What types of transactions are included in the term ‘transfer’ in relation to a capital, asset?, Answer: Meaning of ‘transfer’ in relation to a capital asset, Transfer Section 2(47), Capital gains shall be computed in case of transfer of capital asset and term transfer shall include, 1. Sale of the asset, 2. Compulsory acquisition of land or building, 3. Conversion of capital asset into stock-in trade, 4. The relinquishment of the asset e.g. Mr. X has received the right to purchase the right shares but he has, relinquished his right to purchase the share in favour of some other person by charging ₹1,00,000, in this, case, he has capital gain of ₹1,00,000., 5. The extinguishment of any rights/asset e.g. Mr. X was holding shares in ABC Ltd. The company has, gone into liquidation and Mr. X has received ₹2,00,000 being the full value of consideration and the cost of
Page 630 :
Income Under The Head Capital Gains, , 111, , acquisition was ₹1,50,000, in this case there is a capital gain of ₹50,000., 6. The maturity or redemption of a zero coupon bond., 7. If any person has given possession of immovable property and has taken full payment but ownership in, documents has not yet been transferred. It will also be considered to be transfer and capital gains shall be, computed e.g. Mr. X enters into an agreement for the sale of his house. The purchaser gives the entire sale, consideration to Mr. X. Mr. X hands over complete rights of possession to the purchaser since he has, realised the entire sales consideration. The above transaction is considered as transfer., Illustration 39: Mr. X owns a plot of land acquired on 01.06.2002 for a consideration of ₹ 2 lakhs. He, enters into an agreement to sell the property on 15.03.2022 for a consideration of ₹ 20 lakhs. In part, performance of the contract, he handed over the possession of land on 21.03.2022 on which date he received, the full consideration. As on 31st March 2022 the sale was not registered., Discuss the liability to capital gain for the assessment year 2022-23., Solution:, ₹, Computation of Capital Gains, Full value of consideration, 20,00,000.00, Less: Indexed cost of acquisition, = 2,00,000 / Index of 02-03 x Index of 21-22, = 2,00,000 / 105 x 317 = ₹6,03,809.52, (6,03,809.52), Long Term Capital Gain, 13,96,190.48, Illustration 40: Mr. X purchased 2,000 equity shares of ABC Ltd. (a listed company) on 01.04.2014 at ₹20, per share. He sold all the shares on 01.06.2021 at ₹50 per share. He also had to pay securities transaction tax, (STT) on the same. Explain the taxability in the hands of Mr. X in the year of transfer i.e. A.Y. 2022-23., Solution: In the given problem, since the listed equity shares of ABC Ltd. are being sold after 12 months, and also STT has been paid, it is exempt upto ₹1,00,000 u/s 112A and excess over ₹1,00,000 is taxable @, 10%., Illustration 41: Will your answer be different if these shares were preference shares and not equity shares?, Explain., Solution: Since section 112A is not applicable to preference shares, capital gains shall be taxable., Illustration 42: Will you answer be different if these shares were not listed in a recognised stock exchange?, Explain., Solution: Since the shares are not listed, section 112A is not applicable, capital gains shall be taxable., Question 37: Write a note on taxability of Long Term Capital Gains., Answer:, Tax on Long-Term Capital Gains Section 112(1), If total income of an assessee includes long term capital gains, such capital gains shall be taxed @ 20%., Example, If ABC Ltd. has business income of ₹5,00,000 and long term capital gain ₹4,00,000, company tax liability, shall be ₹2,39,200., Special provision in case of resident individual and resident Hindu Undivided Family, Where the total income as reduced by such long-term capital gains is below the maximum amount which is, not chargeable to income-tax (2,50,000/3,00,000/5,00,000), then, such long-term capital gains shall be, reduced by the amount by which the total income as so reduced falls short of the maximum amount which is, not chargeable to income-tax and the tax on the balance of such long-term capital gains shall be computed at, the rate of 20%., Deductions under section 80C to 80U shall not be allowed from long term capital gain Section 112, If any assessee has long term capital gains, no deduction shall be allowed from such long term capital gains, under chapter VI-A i.e. deductions under section 80C to 80U are not allowed., Example
Page 631 :
Income Under The Head Capital Gains, , 112, , Mr. X has long term capital gains of ₹6,00,000. He has donated ₹1,00,000 in Prime Minister’s National, Relief Fund eligible for deduction under section 80G, his tax liability shall be (3,50,000 x 20%) + 4% =, ₹72,800 (deduction under section 80G is not been allowed but deficiency of the normal income has been, allowed.), Question 38: Write short note on special provisions for Full Value of Consideration in certain cases, in, the context of capital gains liability., Answer: Special provisions for full value of consideration in certain cases, Full value of consideration means the sale price in connection with a capital asset which has been, transferred, however in the following cases, full value of consideration shall be computed on notional basis:, 1. As per section 45(1A), in case of destruction of a capital asset, full value of consideration shall be claim, received from insurance company., 2. As per section 45(2), in case of conversion of capital asset into stock in trade, market value of the asset on, the date of conversion., 3. As per section 50C, if the full value of consideration in connection with land or building claimed by an, assessee is less than the value adopted by the stamp valuation authority, the value adopted by the stamp, valuation authority., Illustration 43: Mr. A is an individual carrying on business. His stock and machinery were damaged and, destroyed in a fire accident. The value of stock lost (total damaged) was ₹ 6,50,000. Certain portion of the, machinery could be salvaged. The opening WDV of the block as on 1-4-2021 was ₹10,80,000. During the, process of safeguarding machinery and in the firefighting operations, Mr. A lost his gold chain and a, diamond ring, which he had purchased in April, 2005 for ₹1,20,000. The market value of these two items as, on the date of fire accident was ₹1,80,000., Mr. A received the following amounts from the insurance company:, (i) Towards loss of stock ₹4,80,000, (ii) Towards damage of machinery ₹6,00,000, (iii)Towards gold chain and diamond ring ₹1,80,000, You are requested to briefly comment on the tax treatment of the above three items under the provisions of, the Income-tax Act, 1961., Answer:, (i) Compensation towards loss of stock: Any compensation received from the insurance company towards, loss/damage to stock in trade is to be construed as a trading receipt. Hence, ₹4,80,000 received as insurance, claim for loss of stock has to be assessed under the head “Profit and gains of business or profession”., Note - The assessee can claim the value of stock destroyed by fire as revenue loss, eligible for deduction, while computing income under the head “Profits and gains of business or profession”., (ii) Compensation towards damage to machinery: The question does not mention whether the salvaged, machinery is taken over by the Insurance company or whether there was any replacement of machinery, during the year. Assuming that the salvaged machinery is taken over by the Insurance company, and there, was no fresh addition of machinery during the year, the block of machinery will cease to exist. Therefore,, ₹4,80,000 being the excess of written down value (i.e ₹10,80,000) over the insurance compensation (i.e., ₹6,00,000) will be assessable as a short-term capital loss., Note – If new machinery is purchased in the next year, it will constitute the new block of machinery, on, which depreciation can be claimed for that year., (iii) Compensation towards loss of gold chain and diamond ring: Gold chain and diamond ring are, capital assets as envisaged by section 2(14). They are not “personal effects”, which alone are to be excluded., As per section 45(1A), if any profit or gain arises in a previous year owing to receipt of insurance claim, the, same shall be chargeable to tax as capital gains. The capital gains has to be computed by reducing the, indexed cost of acquisition of jewellery from the insurance compensation of ₹1,80,000.
Page 632 :
Income Under The Head Capital Gains, , 113, , MULTIPLE CHOICE QUESTIONS, 1. Which of the following would be regarded as transfer (a) transfer of a capital asset in a scheme of reverse mortgage, (b) transfer of a capital asset under a gift or will or an irrevocable trust, (c) transfer by way of conversion of equity shares from preference shares, (d) Redemption of Zero coupon bond, 2. Short-term capital gains arising on transfer of listed shares on which STT is paid at the time of, transfer, would be chargeable to tax –, (a) at the rate of 10%, (b) at the rate of 20%, (c) at the rate of 15%, (d) at the rate of 5%, 3. Distribution of assets at the time of liquidation of a company(a) is not a transfer in the hands of the company or the shareholders, (b) is not a transfer in the hands of the company but capital gains is chargeable to tax on such distribution in, the hands of the shareholders, (c) is not a transfer in the hands of the shareholders but capital gains is chargeable to tax on such distribution, in the hands of the company, (d) is a transfer both in the hands of shareholders and company, 4. Land or building would be long term capital asset only if it is, (a) held for more than 12 months immediately preceding the date of transfer, (b) held for more than 24 months immediately preceding the date of transfer, (c) held for more than 30 months immediately preceding the date of transfer, (d) held for more than 36 months immediately preceding the date of transfer, 5. Capital gain on transfer of depreciable asset would be(a) long term capital gain, if held for more than 36 months, (b) long term capital gain, if held for more than 24 months, (c) long term capital gain, if held for more than 12 months, (d) short term capital gain, irrespective of the period of holding, 6. For an assessee, who is a salaried employee who invests in equity shares, what is the benefit, available in respect of securities transaction tax paid by him on sale and acquisition of 100 listed, shares of X Ltd. which has been held by him for 14 months before sale?, (a) Rebate under section 88E is allowable in respect of securities transaction tax paid, (b) Securities transaction tax paid is treated as expenses of transfer and deducted from sale consideration., (c) Capital gains is taxable at a concessional rate of 10% on such capital gains exceeding ₹1 lakh, (d) Capital gains is taxable at concessional rate of 15%., 7. Under section 50C, the guideline value (Stamp duty value) for stamp duty is taken as the full value, of consideration only if (a) the asset transferred is building and the actual consideration is less than the guideline value, (b) the asset transferred is either land or building or both and guideline value exceeds the actual, consideration, (c) the asset transferred is either land or building or both and the guideline value exceeds 110% of the actual, consideration., (d) the asset transferred is land and the actual consideration is less than the guideline value, 8. Where there is a transfer of a capital asset by a partner to the firm by way of capital contribution, or otherwise, the consideration would be taken as (a) The market value of the capital asset on the date of transfer, (b) The cost less notional depreciation of the capital asset, (c) The value of the asset recorded in the books of the firm., (d) Any of the above, at the option of the assessee, 9. Under section 54F, capital gains are exempted if, (a) long-term capital gain arising on transfer of residential house is invested in acquisition of one residential, house situated in or outside India, (b) long-term capital gain arising on transfer of a capital asset other than a residential house is invested in
Page 633 :
Income Under The Head Capital Gains, , 114, , acquisition of one residential house situated in or outside India, (c) net sale consideration on transfer of a capital asset other than a residential house is invested in acquisition, of one residential house situated in India, (d) short term or long-term capital gain arising on transfer of a capital asset other than a residential house, is invested in acquisition of one residential house situated in India, 10. Under section 54EC, capital gains on transfer of land or building or both are exempted if invested, in the bonds issued by NHAI & RECL or other notified bond(a) within a period of 6 months from the date of transfer of the asset, (b) within a period of 6 months from the end of the relevant previous year, (c) within a period of 6 months from the end of the previous year or the due date for filing the return of, income under section 139(1), whichever is earlier, (d) At any time before the end of the relevant previous year., 11. Non listed equity shares shall be long term after, (a) One year, (b) Two year, (c) Three year, (d) None of these, 12. Which of the following is not correct for long term capital gains under section 112A, (a) rebate u/s 87A is not allowed, (b) deductions u/s 80C to 80U not allowed, (c) it is taxable in excess of ₹ 2,00,000, (d) while computing capital gains indexation is not applicable, (e) none of these, 13. Mr. X purchased one motor car for ₹3,00,000 and it was sold for ₹5,00,000 after one year, in this, case, (a) there is short term capital gains of ₹ 2,00,000, (b) there is long term capital gains of ₹ 2,00,000, (c) income under the head other sources ₹ 2,00,000, (d) none of these, 14. Mr. X purchased agricultural land in Delhi and it was sold after 2 years and there is a profit of, ₹10,00,000, in this case, (a) it will be considered to be short term capital gains, (b) it will be considered to be long term capital gains, (c) it will be exempt from income tax, (d) it will be considered to be income under the head other sources, (e) none of these, 15. In case of conversion of capital asset into stock in trade, capital gains shall be computed, (a) in the year of conversion, (b) in the year of sale of stock in trade, (c) subsequent to be year of conversion, (d) none of these, 16. In case of compulsory acquisition of land or building by the Govt.,, (a) capital gains shall be taxable in the year of acquisition, (b) capital gains shall be taxable in the year in which Govt. has made full payment, (c) capital gains shall be taxable in the year in which Govt. has made the first payment, (d) none of these, 17. If any person has received any capital asset as gift and it was not taxable as gift, in that case, (a) cost of acquisition for such person shall be nil, (b) cost of acquisition for such person shall be the market value on the date on which such gift has been, received, (c) cost of acquisition of the previous owner shall be considered to be the cost of acquisition of the assessee, (d) none of these, 18. Mr. X sold one house property for ₹ 70,00,000 and its stamp duty value is ₹ 73,00,000, in this case, FVC shall be taken to be, (a) ₹ 70,00,000, (b) ₹ 73,00,000, (c) ₹ 73,50,000, (d) none of these, 19. Mr. X sold one house property for ₹ 70,00,000 and its stamp duty value is ₹ 84,00,000, in this case, FVC shall be taken to be, (a) ₹ 70,00,000, (b) ₹ 84,00,000, (c) ₹ 73,50,000, (d) none of these, 20. Exemption under section 54, (a) is allowed to an individual for short term capital gains on transfer of residential house
Page 634 :
Income Under The Head Capital Gains, , 115, , (b) is allowed to an individual for long term capital gains on transfer of any house property, (c) is allowed to a company for long term capital gains on transfer of residential house, (d) none of these, 21. Exemption under section 54B is allowed, (a) to an individual if he has invested within 3 years in agricultural land in rural area, (b) to an individual if he has invested within 1 year in agricultural land in rural area, (c) to an individual if he has invested within 2 year in agricultural land in rural area or urban area, (d) none of these, 22. Exemption under section 54D is allowed, (a) only to a company in case of compulsory acquisition of land or building of an industrial undertaking, (b) to any assessee in case of compulsory acquisition of land or building of an industrial undertaking, (c) to any assessee in case of sale of land or building of an industrial undertaking, (d) none of these, 23. Exemption under section 54EC is allowed to, (a) any assessee on the transfer of any land or building which is long term, (b) any assessee on the transfer of any asset which is long term, (c) any assessee on the transfer of any asset which is long term or short term, (d) none of these, 24. Under section 54EE, (a) exemption allowed to all the assessee but maximum ₹ 50,00,000 in a particular previous year, (b) exemption allowed to an individual only but maximum ₹ 60,00,000 in a particular previous year, (c) exemption allowed to an individual or HUF only but maximum ₹ 70,00,000 in a particular previous year, (d) none of these, 25. Under section 54F, (a) investment should be made within 2 years after the date of transfer of original asset, (b) investment should be made within 1 years before and 4 years after the date of transfer of original asset, for construction of house, (c) investment should be made within 1 years before and 2 years after the date of transfer of original asset, for purchase of house, (d) none of these, , Check Detailed Solution given on our website www.mkgeducation.com, (Icon /Heading No.11: Books)
Page 635 :
Income Under The Head Capital Gains, , 116, , PRACTICE PROBLEMS, TOTAL PROBLEMS 18, Problem 1., Mr. X acquired a residential house in January, 1999 for ₹2,00,000 and its market value on 01.04.2001 is, ₹1,80,000 and he constructed its 1st floor in September’ 2007 by incurring ₹3,00,000 and constructed second, floor in October’ 2011 by incurring ₹4,00,000 and constructed its third floor in February’ 2013 by incurring, ₹5,00,000 and sold the house on 01.01.2022 for ₹100,00,000 and paid brokerage @ 1% and he invested, ₹20,000 in equity shares of infrastructure development company notified under section 80C., Compute his tax liability for assessment year 2022-23., Answer = Tax Liability: ₹15,55,190, Problem 2., Compute capital gains and tax liability of Mr. X in the following Independent situations for the assessment, year 2022-23:, Asset, Gold, Land, Residential house, Date of purchase, 01.07.1998, 01.04.1996, 01.07.1993, Cost price, 3,00,000, 5,00,000, 7,00,000, Cost of improvement, 20,000, 1,00,000, 3,00,000, Year of improvement, 1999-00, 2000-01, 2016-17, Fair market value on 01.04.2001, 35,00,000, 45,00,000, 55,00,000, Date of Sale, 01.01.2022, 01.01.2022, 01.01.2022, Full value of consideration, 150,00,000, 320,00,000, 400,00,000, Answer = Tax Liability: Gold: ₹7,60,240; Land: ₹41,82,410; Residential House: ₹57,08,240, Problem 3., Mr. X purchased 100 equity shares in ABC Ltd. (listed) on 01.10.1996 @ ₹10 per share. The company had, issued 100 bonus shares on 01.10.2000 and market value of the share as on 01.04.2001 is ₹8 per share., Company has again issued 100 bonus shares on 01.10.2006., The company has further offered 100 right shares on 01.05.2021 @ ₹150 per share and Mr. X has purchased, half of the shares and balance half was renounced in favour of Mr. Y by charging ₹5 per share., Mr. X and Mr. Y both have transferred all the shares on 01.01.2022 @ ₹200 per share and securities, transaction tax has been paid. Market value as on 31-01-2018 ₹100 per share, Mr. X has income under the head business/profession ₹20,00,000 and he has invested ₹70,000 in public, provident fund., Mr. Y has income under the head business/profession ₹10,00,000 and he has invested ₹50,000 in public, provident fund., Compute tax liability of Mr. X and Mr. Y., Answer = Mr. X: Tax Liability: ₹4,07,630; Mr. Y: Tax Liability: ₹1,06,950, Problem 4., Mr. X purchased one house on 01.10.2002 for ₹5,00,000 and this house was acquired compulsorily by the, Government on 01.07.2014. Compensation fixed by the Government was ₹55,00,000. Government has paid, half of the amount on 01.10.2021 and balance half on 01.10.2022., The assessee was not satisfied with the compensation and he has filed an appeal in the High Court. The High, Court has given decision on 31.03.2024 directing the Government to pay additional compensation of, ₹5,00,000 and the Government has paid ₹3,00,000 on 10.04.2024 and balance ₹2,00,000 on 10.04.2025., Compute capital gains for the various years and tax liability for assessment year 2022-23., Answer = Assessment Year 2022-23: Long term capital gains: ₹43,57,140; Tax Liability: ₹8,54,290;, Assessment Year 2025-26: Long term capital gains: ₹3,00,000;, Assessment Year 2026-27: Long term capital gains: ₹2,00,000
Page 636 :
Income Under The Head Capital Gains, , 117, , Problem 5., Discuss whether the following are capital assets or not:, (i), Household furniture, (ii), Personal Motor car, (iii) Residential house, (iv), Urban land, (v), Agricultural land in rural area in India, (vi), Agricultural land in rural area in Nepal, (vii) Stock in trade, (viii) Gold ornaments, (ix), Music system for personal use, (x), Music system in business use, (xi), Motor car in business use, (xii) Plant and machinery in business use, (xiii) Silver utensils for personal use, (xiv) Precious stones in personal use, Answer = (i) Not a capital asset; (ii) Not a capital asset; (iii) Capital asset; (iv) Capital asset; (v) Not a, capital asset; (vi) Capital asset; (vii) Not a capital asset; (viii) Capital asset; (ix) Not a capital asset; (x), Capital asset; (xi) Capital asset; (xii) Capital asset; (xiii) Not a capital asset; (xiv) Capital asset, Problem 6., Mr. X (aged 55 years) sold the following assets during the previous year 2021-22:, 1. He purchased one house in rural area on 01.10.1991 for ₹2,00,000 and incurred ₹50,000 on its, improvement on 01.07.2000. Its market value on 01.04.2001 is ₹2,30,000. It was sold on 01.04.2021, for ₹5,00,000., 2. He purchased agricultural land in the rural area for ₹2,00,000 on 01.07.2002 and sold it on 01.07.2021, for ₹3,00,000., 3. He purchased one T.V. for his personal use on 01.01.2003 for ₹25,000 and sold it on 30.12.2021 for, ₹20,000., 4. He purchased gold on 01.07.2018 for ₹3,00,000 and sold it on 01.04.2021 for ₹4,50,000., 5. He has one motor car in his business with written down value as on 01.04.2021 ₹2,00,000 and it was, sold by him on 01.07.2021 for ₹2,50,000., 6. He purchased one house on 01.10.2002 for ₹7,00,000 and incurred ₹4,50,000 on 01.10.2012 to, construct its first floor and subsequently the house was sold on 01.01.2022 for ₹90,00,000 and selling, expenses were 2% of the sale price., Compute tax liability of Mr. X for the assessment year 2022-23., Answer = Tax Liability: ₹13,07,440, Problem 7., Mr. X purchased one residential house on 01.10.2002 for ₹5,00,000 and sold the house on 01.07.2021 for, ₹100,00,000 and purchased one house on 01.01.2022 for ₹20,00,000 and this house was sold by him on, 01.01.2023 for ₹25,00,000., Compute his income tax liability for assessment year 2022-23 and also capital gains for all the years., Answer = Income Tax Liability : ₹14,27,820, Assessment Year 2023-24: Short term capital gains: ₹25,00,000, (b) Presume the house purchased on 01.01.2022 was sold on 31.01.2022, Answer = Income Tax Liability : ₹19,56,920, (c) Presume the house purchased on 01.01.2022 was purchased on 01.10.2022 and was not sold upto, 01.09.2025., Answer = Income Tax Liability : ₹18,85,420, (d) Presume no house was purchased but the amount was deposited in capital gains account scheme on, 31.07.2022 and the amount remained unutilized., Answer = Income Tax Liability : ₹14,27,820
Page 637 :
Income Under The Head Capital Gains, , 118, , Assessment Year 2025-26: Long term capital gains: ₹20,00,000, Problem 8., Mr. X purchased agricultural land in urban area for ₹3,00,000 on 01.10.2005 and this land was transferred, by him on 01.07.2021 for ₹32,00,000 (this agricultural land is used for agricultural purpose since its, purchase). Mr. X purchased one agricultural land on 30.09.2022 in the urban area for ₹6,00,000. The, agricultural land were sold on 01.01.2023 for ₹10,00,000., He has one business also with turnover ₹105,00,000 and has income from business ₹1,10,000., Compute capital gains for various years and also tax liability for assessment year 2022-23., Answer = Assessment Year 2022-23: Long Term Capital Gains: ₹17,87,179.49; Tax Liability: 3,42,610, Assessment Year 2023-24: Short Term Capital Gains: ₹10,00,000, Problem 9., Mr. X sells a commercial house property on 15th December 2021 for ₹22,00,000 (cost of acquisition on 23rd, April 2001 ₹1,50,000). On 14th March 2022 he purchases a residential house for ₹3,00,000 for availing, exemption under section 54F and bonds of NHAI which are redeemable after 5 years for ₹3,80,000 for, claiming exemptions under section 54EC. He does not own any other house. He also incurred a short term, capital loss amounting to ₹50,000 during the previous year 2021-22., During the previous year his only other income was from business amounting to ₹50,00,000., He deposited ₹20,000 in public provident fund., Compute the total income and tax liability of Mr. X for the assessment year 2022-23., Answer = Total Income: ₹60,39,340; Tax Liability: ₹17,37,010, Problem 10., Mr. X purchased agricultural land on 01.05.1991 for ₹3,00,000 in urban area and its market value on, 01.04.2001 is ₹2,00,000. This land was gifted by him to his son Mr. Y on 01.01.2013 when its market value, was ₹15 lakhs. Mr. Y has transferred this land on 01.01.2022 for ₹40 lakhs (this agricultural land is used for, agricultural purposes since its purchase) and he purchased one more agricultural land in rural area on 10th, January 2022 for ₹2,50,000 and purchased one residential house on 31.01.2022 because he didn’t have any, house in his name, for ₹7,00,000., He invested ₹1,00,000 in bonds of National Highways Authority of India redeemable after 5 years. The, amount was invested on 30.06.2022., Compute his tax liability for the assessment year 2022-23., Answer = Tax Liability: ₹ 3,98,410, Problem 11., On 25.04.2021 Mr. X sold an urban agricultural land for ₹60,00,000 which he had been using for, agricultural purposes for several years. He acquired that land in 2000 for ₹2,50,000. The market value of, such land as on 01.04.2001 was ₹5,00,000. He purchased rural agricultural land for ₹8,00,000 on 25.06.2021, which was sold for ₹12,50,000 on 18.01.2022. A sum of ₹12,50,000 was also invested by him in purchase of, residential property on 25.07.2021. He did not own any house property before this date. The new house, property was sold on 28.03.2022 for ₹15,00,000., Compute tax liability for assessment year 2022-23., Answer = Long term capital gain: ₹36,15,000; Short term capital gain: ₹2,50,000; Tax Liability: ₹7,51,920, Problem 12., Mr. X is the owner of the following assets:, 1. He purchased gold in 1997-98 for ₹90,000 and its market value as on 01.04.2001 is ₹1,01,000., 2. He purchased equity shares in A Ltd (listed) in 1993-94 for ₹1,92,000 and its market value on, 01.04.2001 is ₹2,00,000 and market value as on 31-01-2018 ₹2,83,000, Mr. X died on 16.08.2008 and as per his will these assets were transferred to his son Y. Mr. Y now sells, these assets on 10.06.2021 for ₹20,00,000 and ₹3,00,000 respectively and securities transaction tax has been, paid on sale of equity shares., Find out the amount of capital gains chargeable to tax and also tax liability for the assessment year 2022-23., Answer = Income under the head Capital Gains: ₹16,96,830; Tax Liability: ₹2,97,400
Page 638 :
Income Under The Head Capital Gains, , 119, , Problem 13., Mr. X purchased a house property for ₹36,000 on 10.05.1993. He gets the first floor of the house, constructed in 1997-98 by spending ₹80,000. He dies on 12.09.2003. The property is transferred to Mrs. X, by his will. Mrs. X spends ₹40,000 during 2004-05 for renewals/reconstruction of the property. Mrs. X sells, the house property for ₹14,50,000 on 15.03.2022 (brokerage paid by Mrs. X is ₹14,500). The fair market, value of the house on 01.04.2001 is ₹1,10,000., Find out the amount of capital gain chargeable to tax for the assessment year 2022-23., Answer = Long Term Capital Gain: ₹9,74,587.61, Problem 14., Mr. X a senior citizen (aged 65 years) sold residential building at Alwar for ₹40,00,000 on October 1st,, 2021. This building was acquired by his father on 01.01.1999 for ₹1,00,000. On the death of his father on, July 5th, 2006, he inherited this building. Fair market value of this property on 01.04.2001 was ₹1,50,000., He paid brokerage @ 1% to the real estate agent at the time of sale of the building. He purchased a, residential building at Bangalore on March 7th, 2022 for ₹8,00,000 and deposited ₹3,00,000 on April 20th,, 2022 in the bonds of National Highways authority of India redeemable after one year., His other incomes are ₹ 50,000. He deposited ₹ 10,000 in public provident fund., Compute total income and tax liability of Mr. X for the assessment year 2022-23., Answer = Total Income: ₹27,24,500; Tax Liability: ₹5,04,300, Problem 15., Mr. X purchased 100 debentures in ABC Ltd. on 01.10.2004 @ ₹300 per debenture and subsequently the, company has converted the debentures into shares on 01.10.2017 and for each debenture 3 shares were, issued and market value of the shares on the date of conversion was ₹250 per share and market value as on, 31-01-2018 is ₹300 and subsequently assessee has sold all these shares on 01.04.2021 @ ₹500 per share and, has paid brokerage @ 1% of the sale price. Compute capital gains in the hands of Mr. X in the following, cases:, (a) STT not paid, (b) STT paid, Answer = (a) Long Term Capital Gains: ₹64,340.71 (b) Long Term Capital Gains u/s 112A: ₹58,500, Problem 16., Mr. X purchased 500 debentures on 01.07.2001 of ABC Ltd. @ ₹ 390 per debenture and paid brokerage @, 1.5%. The debentures were converted into share @ 3 share for each debenture on 01.07.2011. Market value, on the date of conversion was ₹ 170 per share. All the shares were sold on 01.07.2021 @ ₹900 per share and, no securities transaction tax has been paid and paid brokerage @ 1.5%., A sum of ₹1,00,000 was invested in purchasing a house on 28.06.2022 because the assessee did not have, any house and ₹1,00,000 was deposited in capital gain account scheme on 30.06.2022 for availing, exemption under section 54F and ₹ 50,000 was withdrawn on 02.07.2022 to construct first floor of the house, purchased on 28.06.2022., Compute total income and tax liability for assessment year 2022-23 and capital gains for various years., Answer = Total Income: ₹5,96,690; Tax Liability: 72,110;, Assessment Year 2025-26: Long Term Capital Gain: ₹26,408.26, Problem 17., Mr. X has submitted information regarding sale of certain assets as given below:, 1. He purchased one house on 01.10.1998 for ₹5,00,000 and paid brokerage ₹25,000. He entered into an, agreement to sell this house on 01.04.2001 for ₹5,10,000 but the buyer backed out. He constructed its, first floor on 01.01.2014 by incurring ₹4,00,000 and subsequently this house was sold on 01.01.2022, for ₹160,00,000 and selling expenses were ₹85,000., 2. He purchased Preference shares in ABC Ltd. on 01.07.2013 for ₹1,50,000 and sold these shares on, 31.03.2022 for ₹1,00,000., 3. He purchased one motor car for personal use on 28.02.2003 for ₹2,00,000 and sold it on 01.04.2021
Page 639 :
Income Under The Head Capital Gains, , 120, , for ₹2,10,000., 4. He purchased gold ornaments on 01.10.2000 for ₹2,10,000. Its market value on 01.04.2001 is, ₹2,00,000 and it was sold by him on 01.07.2021 for ₹8,00,000., 5. He purchased silver utensils on 01.07.2002 for ₹30,000 and these utensils were sold by him on, 01.01.2022 for ₹23,000., 6. He has invested ₹35,000 in the units of UTI., Compute his income tax liability for assessment year 2022-23., Answer = Tax Liability: ₹32,15,460, Problem 18., Mr. X owns several assets but does not own any residential house. He sells the following assets and requests, you to compute his tax liability for the assessment year 2022-23., 1. Shares (non-listed) purchased in April 2007 for ₹1,30,000 sold on 19.07.2021 for ₹12,00,000., 2. On 01.04.2001, he had agreed to sell the jewellery to Mr. Y for ₹3,50,000 which was purchased in, 1996 for ₹1,80,000. However, the sale could not be effected as Mr. Y backed out. He now sold the, jewellery on 15.08.2021 for ₹18,00,000 and incurred ₹30,000 incidental selling expenses on account of, brokerage and commission., In December 2021, he also purchased a small residential house for ₹2,00,000., He has deposited ₹1,60,000 on 20.01.2022 in deposit account with a public sector bank under capital, gains deposit scheme for construction on the house which he has purchased in December 2021., On 15.01.2022, he invested ₹2,50,000 in the bonds issued by National Highway Authority of India, which are redeemable after 5 years., 3. Debentures (unlisted) purchased in April 2019 for ₹80,000 sold on 31.12.2021 for ₹1,40,000., 4. Sold his motor car purchased in August 2007 for ₹1,50,000 on 15.03.2022 for ₹18,000., 5. He purchased equity shares of ABC Limited on 01.11.2020 for ₹2,00,000 and sold all the shares on, 01.06.2021 for ₹10,00,000 and has paid STT @ 0.25% of sale price., Compute his income tax liability., Answer = Tax Liability: ₹3,50,870., , Check Detailed Solution given on our website www.mkgeducation.com, (Icon /Heading No.11: Books)
Page 640 :
Income Under The Head Capital Gains, , 121, , EXAMINATION QUESTIONS, JULY – 2021 (NEW COURSE), Question 3(b), (2 x 2 = 4 Marks), Examine the taxability of Capital gains in the following scenarios for the Assessment Year 2022-23,, determine the taxable amount and rate of tax applicable:, (i), , On 28th February, 2022 10,000 shares of XY Ltd., a listed company are sold by Mr. B @ 550 per, share and STT was paid at the time of sale of shares. These shares were acquired by him on 5th April,, 2017 @ 395 per share by paying STT at the time of purchase. On 31st January, 2018, the share of XY, Ltd. were traded on a recognized stock exchange at the Fair Market Value of ₹ 390 per Share., , (ii), , Mr. A is the owner of residential house which was purchased on 1st September, 2016 for ₹9,00,000., He sold the said house on 4th September, 2021 for ₹ 19,00,000. Valuation as per stamp valuation, authorities was ₹ 45,00,000. He invested ₹ 19,00,000 in NHAI Bonds on 21st March 2022., , The Cost Inflation index forF.Y. 2016-2017, 264, F.Y. 2021-2022, 311, Solution is given on our website www.mkgeducation.com (Icon /Heading No.11: Books), Question 3(c), (4 Marks), Mr. Patel is a proprietor of Star Stores since 20-05-2019. He has transferred his shop by way of slump sale, for a total consideration of ₹ 40 Lakh. The professional fees & brokerage paid for this sale are ₹80,000. His, Balance Sheet as on 31.03.2022 is as under, Liabilities, Own Capital, , ₹, , Assets, , ₹, , 10,50,000 Building, , 5,00,000, , Bank Loan, , 5,00,000 Furniture, , 5,00,000, , Trade Creditors, , 2,50,000 Debtors, , 2,00,000, , Unsecured Loan, , 2,00,000 Other Assets, , 8,00,000, , Total, , 20,00,000 Total, , 20,00,000, , Other Information:, 1. No individual value of any asset is considered in the transfer deed., 2. Other assets include trademarks valuing ₹ 2,00,000 as on 01.04.2021 on which no depreciation, has been provided., 3. Furniture of ₹ 1,50,000 purchased on 05.11.2021 on which no depreciation has been provided., 4. Unsecured loan includes ₹ 50,000 as advance received from his wife, which she has agreed to, waive off., (i) Computer the capital gain for AY 2022-23., Solution is given on our website www.mkgeducation.com (Icon /Heading No.11: Books)
Page 641 :
Income Under The Head Capital Gains, , 122, , NOV – 2020 (NEW COURSE), Question 4 (b), (5 Marks), Mr. Govind purchased 600 shares of "Y" limited at ₹130 per share on 26.02.1979. "Y" limited issued him,, 1,200 bonus shares on 20.02.1984. The fair market value of these share at Mumbai Stock Exchange as on, 1.04.2001 was ₹900 per share and ₹2,000 per share as on 31.01.2018. On 31.01.2020 he converted 1000, shares as his stock in trade. The shares was traded at Mumbai Stock Exchange on that date at a high of, ₹2,200 per share and closed for the day at ₹2,100 per share., On 07.07.2021 Mr. Govind sold all 1800 shares @ ₹2,400 per share at Mumbai Stock Exchange and, securities transaction tax was paid., Compute total income of Mr. Govind for the assessment year 2022-23, Solution is given on our website www.mkgeducation.com (Icon /Heading No.11: Books), , NOV – 2019 (NEW COURSE), Question.3. (b), (6 Marks), Mr. Rajan provides you the following details with regard to sale of certain securities by him during F.Y., 2021-22:, (i) Sold 10000 shares of A Ltd. on 05-04-2021 @ ₹ 650 per share, A Ltd. is a listed company. These shares were acquired by Mr. Rajan on 05-04-2016 @ ₹ 100 per share., STT was paid both at the time of acquisition as well as at the time of transfer of such shares which was, affected through a recognized stock exchange. On 31-01-2018, the shares of A Ltd. were traded on a, recognized stock exchange as under:, Highest price - ₹ 300 per share, Average price - ₹ 290 per share, Lowest price - ₹ 280 per share, (ii) Sold 1000 units of B Mutual Fund on 20-04-2021 @ ₹ 50 unit, B Mutual Fund is an equity oriented fund. These units were acquired by Mr. Rajan on 15-04-2017 @ ₹ 10, per unit. STT was paid only at the time of transfer of such units. On 31-01-2018, the Net Asset Value of the, units of B Mutual Fund was ₹ 55 per unit., (iii) Sold 100 shares of C Ltd. on 25-04-2021 @ ₹ 200 per share, C Ltd. is an un-listed company. These shares were issued by the company as bonus shares on 30-09-1997., The Fair Value of these shares as on 01-04-2001 was ₹ 50 per share., Cost Inflation Index for various financial year are as under:, 2001-02 - 100, 2016-17, 2017-18, 2021-22, Calculate, , - 264, - 272, - 317, the amount chargeable to tax under the head ‘Capital Gains’ and also calculate tax on such gains, , for A.Y. 2022-23 assuming that the other incomes of Mr. Rajan exceeds the maximum amount not, chargeable to tax. (Ignore surcharges and cess)., Solution: Computation of amount chargeable to tax under the head capital gains, (i) Sold 10000 shares of A Ltd. on 05-04-2021 @ ₹ 650 per share, Full value of consideration (10,000 x 650), 65,00,000
Page 642 :
Income Under The Head Capital Gains, Cost of acquisition, Higher of, 1. Cost of acquisition (10,000 x 100), 2. Lower of, (a) Fair market value of such asset on 31.01.2018 (10,000 x 300), (b) Actual sale value, LTCG u/s 112A, , 123, (30,00,000), 10,00,000, 30,00,000, 65,00,000, 35,00,000, , (ii) Sold 1000 units of B Mutual Fund on 20-04-2021 @ ₹ 50 unit (Assumed units are not listed as on, 31.01.2018), Full value of consideration (1,000 x 50), 50,000, Cost of acquisition, (50,000), Higher of, 1. Cost of acquisition (1,000 x 10), 10,000, 2. Lower of, (a) Fair market value of such asset on 31.01.2018 (1,000 x 55), 55,000, (b) Actual sale value (1,000 x 50), 50,000, LTCG u/s 112A, Nil, (iii) Sold 100 shares of C Ltd. on 25-04-2021 @ ₹ 200 per share, Full value of consideration, (100 x 200), Less: Indexed Cost of Acquisition, (100 x 50 (being FMV on 01.04.2001))/100 x 317, Long Term Capital Gain, Income under the head capital gains, , 20,000, (15,850), 4,150, 35,04,150, , Computation of Tax before health and education cess, Tax on (35,00,000 – 1,00,000) @ 10% u/s 112A, Tax on ₹4,150 @ 20% u/s 112, Tax before health & education cess, , 3,40,000, 830, 3,40,830, , MAY – 2019 (OLD COURSE), Question 4(a), Marks 4, Mr. Deepak has a residential house property taxable u/s 22. Such property was acquired on 12-08-2005 for, ₹ 2,00,000. The property is sold for ₹ 23,00,000. The sub-registrar refused to register the documents for the, said value, as according to him, stamp value valuation, based on State Government guidelines was, 28,00,000. Mr. Deepak preferred an appeal to the revenue divisional officer who fixed the value of the house, ₹ 25,00,000. He acquired another residential house on 31-03-2022 for ₹ 17,00,000 for self-occupation. On, 01-03-2023, he sold such new residential house for ₹ 30,00,000. Compute his capital gain for the A.Y. 202223 and 2023-24. (Cost of indexation : 2001-02; 2005-06 and 2021-22 are, 100; 117 and 317), Solution:, Computation of Capital Gains for the A.Y. 2022-2023 & 2023-24, As per section 50C, in case the actual sale consideration (i.e., ₹23 lakhs, in this case) is less than the stamp, duty value (i.e., ₹28 lakhs, in this case) assessed by the stamp valuation authority (Sub-registrar, in this, case), the stamp duty value shall be deemed as the full value of consideration if it exceeds 110% of the sale, consideration However, if assessee has preferred an appeal to the Valuation Officer (i.e., revenue divisional, officer, in this case) and the Valuation Officer has fixed the value of the house (i.e., ₹25 lakh, in this case)
Page 643 :
Income Under The Head Capital Gains, , 124, , less than stamp duty value (i.e., ₹28 lakh, in this case), such value determined by the Valuation Officer shall, be deemed as the full value of consideration., Sale consideration, ₹ 23,00,000, Valuation made by Sub Register for stamp duty, ₹ 28,00,000, Valuation made by the Divisional Revenue officer on a reference, ₹ 25,00,000, A.Y. 2022-2023, Computation of Capital Gains, Full value of consideration (section 50C), 25,00,000.00, Less: Indexed cost of acquisition, = 2,00,000 / 117 x 317 = ₹ 5,41,880.34, (5,41,880.34), Long Term Capital Gain, 19,58,119.66, Less: Exemption u/s 54, (17,00,000.00), Purchase of new residential house property on 31.3.2022, (i.e., within two years from the date of transfer of residential house, Long Term Capital Gain, 2,58,119.66, A.Y. 2023-2024, Since new house is sold within one year hence gain shall be treated as short term., Computation of Capital Gains, Full value of consideration, 30,00,000.00, Less: cost of acquisition, = (17,00,000-17,00,000), Nil, Short Term Capital Gain, 30,00,000.00, [Since the residential house is held for a period less than 24 months], Note: As per section 54, if the new residential house purchased (i.e., on 31.3.2022, in this case) is transferred, within 3 years of its purchase (i.e., on 1.3.2023, in this case), and the cost of acquisition of the new house, (i.e., ₹ 17 lakhs, in this case) is lower than the long-term capital gain (i.e., ₹ 19,58,119.66, in this case), the, cost of acquisition of such new residential house shall be taken as Nil, while computing capital gains on sale, of the new residential house, , MAY – 2019 (NEW COURSE), Question 3 (b), , Marks 6, , Mr. Roy owned a residential house in Noida. It was acquired on 09.09.2009 for ₹ 30,00,000. He sold it for ₹, 1,57,00,000 on 07.01.2019., Mr. Roy utilized the sale proceeds of the above property to acquire a residential house in Panchkula for ₹, 2,05,00,000 on 20.07.2019. The said house property was sold on 01.08.2021 and he purchased another, residential house in Delhi for ₹ 2,57,00,000 on 02.03.2022. The property at Panchkula was sold for ₹, 3,25,00,000., Calculate capital gains chargeable to tax for the assessment year 2019-20 and 2022-23. All workings should, form part of your answer., Cost inflation index for various financial years are as under:, 2009-10, , –, , 148, , 2018-19, , –, , 280, , 2019-20, , –, , 289, , 2021-22, , –, , 317, , Solution:, Computation of Capital Gains for the A.Y. 2019-2020 & 2022-23, A.Y 2019-2020
Page 644 :
Income Under The Head Capital Gains, Computation of Capital Gains, Full value of consideration, Less: Indexed cost of acquisition, = 30,00,000 / 148 x 280 = ₹56,75,675.68, Long Term Capital Gain, Less: Exemption u/s 54, Long Term Capital Gain, , 125, 1,57,00,000.00, (56,75,675.68), 1,00,24,324.32, (1,00,24,324.32), Nil, , Note: Purchase of new residential house property at Panchkula for 2,05,00,000 on 20.7.2019 i.e., within two, years from the date of transfer of residential house in Noida; exemption restricted to long term capital gain,, since cost of new house exceeds long-term capital gain, A.Y 2022-2023, Computation of Capital Gains, Full value of consideration, Less: Indexed cost of acquisition, = (205,00,000-1,00,24,324.32) / 289 x 317 = ₹114,90,620.04, Long Term Capital Gain [Since the residential house is held for more than 24 months], Less: Exemption u/s 54, Long Term Capital Gain, , 3,25,00,000.00, (114,90,620.04), 2,10,09,379.96, (2,10,09,379.96), Nil, , Note:, 1. As per section 54, if the new residential house purchased (i.e., on 20.7.2019, in this case) is transferred, within 3 years of its purchase (i.e., on 01.08.2021, in this case), and the cost of acquisition of the new house, (i.e.,2,05,00,000) is higher than the long-term capital gain (i.e., 1,00,24,324.32,) then, the cost of acquisition, of such new residential house shall be reduced by long term capital gain exempted earlier, while computing, capital gains on sale of the new residential house] [114,90,620.04 (2,05,00,000-1,00,24,324.32) / 289 x 317], 2. Purchase of new residential house property in Delhi for 2,57,00,000 on 2.3.2022 i.e., within two years, from 01.08.2021, being the date of transfer of residential house at Panchkula; exemption restricted to long, term capital gain, since cost of new house exceeds long-term capital gains., , NOV – 2018 (NEW COURSE), Question 4(a), Marks 6, Mr. Subramani sold a house plot to Mrs. Vimala for ₹45 lakhs on 12-5-2021. The valuation determined by, the stamp valuation authority was ₹53 lakhs. Discuss the tax consequences of above, in the hands of each, one of them, viz, Mr. Subramani & Mrs. Vimala., Mrs. Vimala has sold this plot to Ms. Padmaja on 21-3-2022 for ₹55 lakhs., The valuation as per stamp valuation authority remains the same at ₹53 lakhs., Compute the capital gains arising on sale of the house plot by Mrs. Vimala., Note: None of the parties viz Mr. Subramani, Mrs. Vimala & Ms. Padmaja are related to each other; the, transactions are between outsiders., Solution:, (i) Tax consequences in the hands of Mr. Subramani, As per section 50C, the stamp duty value of immovable property, being land or building or both, would be, deemed to be the full value of consideration arising on transfer of such property, if the same is higher than, actual consideration. Accordingly, in this case, capital gains would be computed in the hands of Mr., Subramani, for A.Y.2022-23, taking the stamp duty value of 53 lakh of house plot as the full value of, consideration arising on transfer of such house plot, since the same is higher than the actual consideration of, 45 lakh., Note – If it is assumed that Mr. Subramani is a property dealer, the income would be taxable as his business, income under section 43CA
Page 645 :
Income Under The Head Capital Gains, , 126, , (ii) Tax consequences in the hands of Mrs. Vimala, In case immovable property is received for inadequate consideration, the difference between the stamp duty, value and actual consideration would be taxable under section 56(2)(x) in the hands of the recipient, if such, difference exceeds ₹50,000. Therefore, in this case, ₹8 lakh (₹53 lakh – ₹45 lakh) would be taxable in the, hands of Mrs. Vimala under the head “Income from Other Sources” in A.Y.2022-23. At the time of, subsequent sale of property by Mrs. Vimala to Mrs. Padmaja (on 21.3.2022), short-term capital gains would, arise in the hands of Mrs. Vimala in A.Y.2022-23, since the property is held by her for less than 24 months., Computation of capital gains in the hands of Mrs. Vimala, Full Value of consideration, 55,00,000, Less: Cost of Acquisition, (53,00,000), Short term capital Gains, 2,00,000, Note: As per section 49(4), in case where capital gains arises from subsequent sale of property, which was, subject to tax under section 56(2)(x), the value taken into account for the purpose of section 56(2)(x) would, be deemed to be the cost of acquisition., , MAY – 2018 (NEW COURSE), Question 2, (10 Marks), Star Enterprises has transferred its unit R to A Ltd. by way of Slump Sale on January 23, 2022. The, summarized Balance Sheet of Star Enterprises as on that date is given below:, Amount, Amount, (₹ in Lacs), Liabilities, (₹ in Lacs), Assets, Own Capital, Accumulated P & L balance, Liabilities :, Unit P, Unit Q, Unit R, , 1,1750, 670, 90, 160, 140, , Fixed Assets:, Unit P, Unit Q, Unit R, Other Assets:, Unit P, Unit Q, Unit R, , 200, 150, 600, 570, 850, 440, , TOTAL, 2,810, TOTAL, 2,810, Using the further information below, compute the Capital Gains arising from slump of unit R for, Assessment year 2022-23., (i) Slump sale consideration on transfer of Unit R was ₹930 lacs., (ii) Fixed Assets of Unit R includes land which was purchased at ₹110 lacs in the year 2008 and was, revalued at ₹140 lacs., (iii) Other fixed assets are reflected at ₹460 lacs. (i.e. ₹600 lacs less value of land) which represents, written down value of those assets as per books. The written down value of these asset is ₹430 lacs., (iv) Unit R was set up by Star Enterprises in Oct, 2006., NOTE: Cost of Inflation Indices for the financial year 2006-07 and financial year 2021-22 are 122 and 317, respectively., Solution:, Computation of capital gains on slump sale of Unit R for A.Y. 2022-23, Particulars, ₹, Full Value of consideration, 930,00,000, Less: Deemed cost of acquisition (Net worth is deemed to be the cost of acquisition), [Refer Working Note below], (840,00,000), Long term capital gain [Since the Unit is held for more than 36 months], 90,00,000, Note: Computation of net worth of Unit R of Star Enterprises
Page 646 :
Income Under The Head Capital Gains, Particulars, Cost of Land (Revaluation not to be considered), WDV of other depreciable fixed assets as per the Income-tax Act, 1961, Other assets (book value), Total assets, Less:, Liabilities, Net worth, , 127, ₹, 110,00,000, 430,00,000, 440,00,000, 980,00,000, (140,00,000), 840,00,000, , Note:, (1) In case of slump sale, net worth of the undertaking transferred shall be deemed to be the cost of, acquisition and cost of improvement as per section 50B., (2) “Net worth” of the undertaking shall be the aggregate value of total assets of the undertaking or division, as reduced by the value of liabilities of such undertaking or division as appearing in the books of accounts, However, any change in the value of assets on account of revaluation shall not be considered for this, purpose, (3) For calculating aggregate value of total assets of the undertaking or division in case of slump sale in case, of depreciable assets, the written down value of block of assets determined in accordance with the, provisions contained in section 43(6) of Income-tax Act, 1961 is to be considered and for all other assets,, book value is to be considered., (4) Since Unit R is held by the assessee for more than 36 months, the capital gain arising from slump sale is, a long-term capital gain., (5) Indexation benefit is not available in case of slump sale., Question 6 (a), (5 Marks), Mr. Madhav made a gift of ₹2,50,000 to his handicapped son, Master Tapan who was aged 12 years as on, 31st March 2019, which he deposited in a fixed deposit account in a Nationalised bank at 10% interest p.a., compounded annually. The balance in this account as on 1st April, 2021 was ₹2,75,000 and the bank credited, a sum of ₹27,500 as interest on 31st March, 2022., Madhav’s father gifted equity shares worth ₹50,000 of an Indian company to Master Manan, another son of, Mr. Madhav (Date of birth 10th April , 2010 ) in July 2010 which were purchased by him on 8th December ,, 2004 for ₹80,000 Manan received a dividend of ₹5,000 on these shares in October 2021. He sold these, shares on 1st November, 2021 for ₹5,00,000 and deposited ₹3,00,000 in a company at 15% interest per, annum., Cost Inflation Index:, Financial Year, Cost Inflation Index, 2004-05, 113, 2010-11, 167, 2021-22, 317, Mr. Madhav has a taxable income of ₹3,50,000 from his profession during the financial year 2021-22., Compute his Gross Total Income for the A.Y. 2022-23., Solution:, Computation of Gross Total Income for the A.Y. 2022-23, ₹, Income from Profession, 3,50,000.00, Income of Minor Son Manan, Income under the head other sources, Interest Income (3,00,000 x 15% x 5/12), 18,750.00, Dividend Income, 5,000.00, Income under the head capital Gains, Full Value of consideration, 5,00,000.00, Less: Indexed cost of acquisition (80,000/113 x 317), (2,24,424.78), Long term capital Gains, 2,75,575.22, (It is assumed that STT is not paid on sale of shares of an Indian company)
Page 647 :
Income Under The Head Capital Gains, , 128, , Less: Exempt u/s 10 (32), (1,500.00), Income to be clubbed, 2,74,075.22, Gross Total Income, 6,47,825.22, Rounded off u/s 288A, 6,47,830.00, Notes:, (1) As per section 64(1A), in computing the total income of an individual, all such income accruing or, arising to a minor child shall be included. However, income of a minor child suffering from disability, specified under section 80U would not be included in the income of the parent but would be taxable in the, hands of the minor child. Therefore, in this case, interest income of ₹27,500 arising to handicapped son,, Master Tapan, would not be clubbed with the income of Mr. Madhav, (2) Income of the other minor child, Master Manan, is includible in the hands of Mr. Madhav, assuming that, Mr. Madhav’s income is higher than that of his wife., (3) If the income of minor child is to be clubbed, exemption shall be allowed under section 10(32) upto, 1,500 per annum per child. Exemption can be given from LTCG or from Other sources., , MAY – 2018 (OLD COURSE), Question 2 (b), (5 Marks), Mrs. Mahalakshmi an individual, aged 68 years, mortgaged her Residential Property, purchased for ₹3 lakhs, on 01-10-2002, with a bank, under a notified reverse mortgage scheme and was sanctioned a loan of ₹20, lakh. As per the said scheme she was receiving the loan amount in equal monthly installments of ₹30, thousand per month from the bank. Mrs. Mahalakshmi was not able to repay the loan on maturity and in lieu, of settlement of the loan surrenders the residential property to the bank. Bank sold the property for ₹25 lakhs, on 22-02-2022. She had no other income during the year., Discuss the Tax consequences and compute tax for the Asst. Year 2022-23., Cost inflation index, 2002-03 - 105, 2021-22 - 317, Solution:, Any transfer of a capital asset in a transaction of reverse mortgage under a scheme made and notified by the, Central Government shall not be considered as a “transfer” for the purpose of capital gain as per section, 47(xvi). Accordingly, the mortgaging of residential house with bank by Mrs. Mahalakshmi will not be, regarded as a transfer for levy of capital gains. Therefore, no capital gain will be attracted on such mortgage., Any amount received by the senior citizen as a loan, either in lump sum or in installments, in a transaction of, reverse mortgage is exempt from income-tax under section 10(43). Therefore, the monthly installment of ₹, 30,000 received by Mrs. Mahalakshmi is exempt from income-tax under section 10(43). However, capital, gains tax liability would be attracted in the P.Y. 2021-22 when the bank sells the mortgaged property for the, purposes of recovering the loan, Computation of Capital Gains in the hands of Mrs. Mahalakshmi AY 22-23, Long-term Capital gains [Since the residential house property was held by Mrs. Mahalakshmi for more than, 24 months immediately preceding the date of its transfer], Full Value of Consideration, Less: Indexed Cost of Acquisition (3,00,000/105 x 317), Long term Capital Gains, Gross Total Income, Less: Deduction u/s 80C to 80U, Total Income, Computation of Tax Liability, Tax on LTCG u/s 112 on 12,94,290 (15,94,290-3,00,000) @ 20%, Add: HEC @ 4%, , 25,00,000.00, (9,05,714.29), 15,94,285.71, 15,94,285.71, Nil, 15,94,290.00, 2,58,858.00, 10,354.32
Page 648 :
Income Under The Head Capital Gains, Tax Liability, Tax Liability (Rounded off u/s 288B), , 129, 2,69,212.32, 2,69,210.00, , NOV – 2017, Question 4(b), (Marks 5), Mr. Sunil entered into an agreement with Mr. Dhaval to sell his residential house located at Navi Mumbai, on 16.08.2021 for ₹80,00,000. The sale proceeds was to be paid in the following manner;, (i) 20% through account payee bank draft on the date of agreement., (ii) 60% on the date of the possession of the property., (iii) Balance after the completion of the registration of the title of the property., Mr. Dhaval was handed over the possession of the property on 15.12.2021 and the registration process, was completed on 14.01.2022. He paid the sale proceeds as per the sale agreement., The value determined by the Stamp Duty Authority on 16.08.2021 was ₹90,00,000 whereas, on, 14.01.2022 it was ₹91,50,000., Mr. Sunil had acquired the property on 01.04.2001 for ₹10,00,000. After recovering the sale proceeds, from Dhaval, he purchased another residential house property for ₹35,00,000., Compute the income under the head “Capital Gains” for the Assessment Year 2022-23 and also compute, tax liability., Cost Inflation Index for Financial Year(s), 2001-02 – 100, 2021-22 – 317, Solution is given on our website www.mkgeducation.com (Icon /Heading No.11: Books), , MAY– 2017, Question 5(a), (8 Marks), Mr. Y bought a vacant Land for ₹80 lakhs in May 2004. Registration and other expenses were 10% of the, cost of land. He constructed a residential building on the said land for ₹100 lakhs during the financial year, 2006-07., He entered into an agreement for sale of the above said residential house with Mr. John (not a relative) in, April 2016. The sale consideration was fixed at ₹700 lakhs and on 23-4-2016, Mr. Y received ₹20 lakhs as, advance in cash by executing an agreement., The sale deed was executed and registered on 14-1-2022 for the agreed consideration. However, the State, stamp valuation authority had revised the values; hence the value of property for stamp duty purposes was, ₹780 lakhs. Mr. Y, paid 1% as brokerage on sale consideration received., Subsequent to sale, Mr. Y made following investments:, (i) Acquired a residential house at Delhi for ₹110 lakhs., (ii) Acquired a residential house at London for ₹190 lakhs., (iii) Subscribed to NHAI capital gains bond (approved under section 54EC) for ₹45 lakhs on 29-3-2022 and, for 50 lakhs on 12-5-2022., Compute the income chargeable under the head ' Capital Gains '. The choice of exemption must beneficial to, the assessee., Cost Inflation Index: F.Y.2004-05 = 113, F.Y.2006-07 = 122, F.Y.2021-22 = 317, Solution is given on our website www.mkgeducation.com (Icon /Heading No.11: Books)
Page 649 :
Income Under The Head Capital Gains, , 130, , NOV– 2016, Question 2(a), (8 Marks), Mr. X, a resident individual, aged 55 years, purchased 10 Plots in the financial year 2003-04 for ₹12 Lakh., On 1stApril 2004, he started a business of property dealing and converted all 10 plots as stock in trade of his, business and recorded the cost at ₹40 Lakh in his books being the Fair market value on 1stApril 2004., On 31st March 2011, he sold all 10 Plots for ₹55 Lakh and purchased a residential house property for ₹50, Lakh. He has constructed 2 rooms in this residential house in June 2011 and has spent ₹8 Lakh., He sold the above residential house on 5th Feb 2022, for ₹85 Lakh. The valuation adopted by Stamp, valuation authority for the payment of stamp duty was ₹112 Lakh. On the request of Mr. X, A.O. made a, reference to the valuation officer. The Valuation Officer determined the value at ₹115 Lakh. Mr. X paid, brokerage 1% of sale consideration., Compute the total Income and total Tax liability of Mr. X for the Assessment year 2022-23., (Cost Inflation Index: 2003-04-109; 2004-05-113; 2010-11-167; 2011-12-184; 2021-22-317.), Solution is given on our website www.mkgeducation.com (Icon /Heading No.11: Books), , NOV– 2015, Question 3(a)., (8 Marks), Mr. X sold his residential house property on 08.06.2021 for ₹70 lakhs which was purchased by him for ₹20, lakhs on 05.05.2006., He paid ₹1 lakh as brokerage for the sale of said property. The stamp duty valuation assessed by sub, registrar was ₹83 lakhs., He bought another house property on 25.12.2021 for 11 lakhs., He deposited ₹8 lakhs on 10.11.2021 in the capital gain bond of National Highway Authority of India, (NHAI)., He deposited another ₹10 lakhs on 10.07.2022 in the capital gain deposit scheme with SBI for construction, of additional floor of house property., Compute income under the head “Capital Gains” for A.Y. 2022-23 as per Income Tax Act 1961 and also, Income tax payable on the assumption that he has no other income chargeable to tax., Cost inflation index for Financial Year 2006-07 = 122 and 2021-22 = 317, Solution is given on our website www.mkgeducation.com (Icon /Heading No.11: Books), , MAY – 2015, Question 5(a)., (8 Marks), Mrs. X transferred a house to her friend Mrs. Y for ₹37,00,000 on 01.10.2021. The Sub Registrar valued the, land at ₹50,00,000. Mrs. X contested the valuation and the matter was referred to Divisional Revenue, Officer, who valued the house at ₹46,00,000. Accepting the said value, differential stamp duty was also paid, and the transferred was completed., The total income of Mrs. X and Mrs. Y for the assessment year 2022-23, before considering the transfer of, said house are ₹2,80,000 and ₹3,45,000, respectively. Mrs. X had purchased the house on 15th May 2011, for ₹25,00,000 and registration expenses were ₹1,50,000., You are required to explain provisions of Income-tax Act, 1961 applicable to present case and also, determine the total income of both Mrs. X and Mrs. Y taking into account the above said transactions. Cost, inflation indices for :, (i) Financial Year 2011-12: 184 and, (ii) Financial Year 2021-22: 317, Solution is given on our website www.mkgeducation.com (Icon /Heading No.11: Books), , MAY – 2014, Question 5(a)., (8 Marks), Mr. X, aged 55 years owned a Residential House in Ghaziabad. It was acquired by Mr. X on 10.10.2006 for, ₹6,00,000. He sold it for ₹53,00,000 on 04.11.2021. The stamp valuation authority of the state fixed value of
Page 650 :
Income Under The Head Capital Gains, , 131, , the property at ₹70,00,000. The Assessee paid 2% of the sale consideration as brokerage on the sale of the, said property., Mr. X Acquired a Residential House property at Kolkata on 10.12.2021 for ₹10,00,000 and deposited, ₹4,00,000 on 10.04.2022 and ₹5,00,000 on 15.06.2022 in the capital gains bonds of Rural Electrification, Corporation Ltd. He deposited ₹4,00,000 on 06.07.2022 and ₹3,00,000 on 01.11.2022 in the capital gain, deposit scheme in a Nationalized Bank for construction of an additional floor on the residential house, property in Kolkata., Compute the Capital Gain chargeable to Tax for the Assessment Year 2022-23 and Income Tax chargeable, thereon assuming Mr. X has no other income., Cost inflation index for Financial Year 2006-07 = 122 and for Financial Year 2021-22 = 317., Solution is given on our website www.mkgeducation.com (Icon /Heading No.11: Books), , NOV – 2013, Question 4(a)., (4 Marks), Mr. X sold a house, held as a capital asset, to his friend Mr. Y on 1st December, 2021 for a consideration of, ₹25,00,000. The Sub-Registrar refused to register the document for the said value, as according to him,, stamp duty valuation based on State Government guidelines was ₹45,00,000. Mr. X preferred an appeal to, the Revenue Divisional Officer, who fixed the value of the house as ₹35,00,000 (₹22,00,000 for land and the, balance for building portion). The differential stamp duty was paid, accepting the said value determined. Mr., X had purchased the land on 1st June, 2006 for ₹5,19,000 and completed the construction of the house on, January, 2020 for ₹14,00,000., Cost inflation indices may be taken as 122 for the financial year 2006-07 and 317 for the financial year, 2021-22., Briefly discuss the tax implications in the hands of Mr. X for the assessment year 2022-23 and compute the, capital gains chargeable to tax., Solution is given on our website www.mkgeducation.com (Icon /Heading No.11: Books), Question 6(a)., (5 Marks), Compute the total income of Mr. X for the assessment year 2022-23 from the following particulars:, Particulars, Amount (₹), Income from business before adjusting the following items:, 1,75,000, (a) Business loss brought forward from assessment year 2016-17, 70,000, (b) Current depreciation, 40,000, (c) Unabsorbed depreciation of earlier year, 1,55,000, Income from house property (Gross annual value), 4,32,000, Municipal taxes paid, 32,000, Mr. X sold a plot at Noida on 12th Sep., 2021 for a consideration of ₹6,40,000, which had been purchased by, him on 20th Dec., 2019 at a cost of ₹4,10,000., Long-term capital gain on sale of land, 60,000, During the previous year 2021-22, Mr. X has repaid ₹1,67,000 towards housing loan from a scheduled bank., Out of ₹1,67,000, ₹97,000 was towards payment of interest and rest towards principal payments. Cost, inflation indices are as under:, Financial Year, Index, 2018-19, 280, 2021-22, 317, Solution is given on our website www.mkgeducation.com (Icon /Heading No.11: Books), , MAY – 2013, Question 6(a), (5 Marks), Mrs. X had purchased 500 equity shares in A Ltd. At a cost of ₹30 per share (brokerage 1%) in February, 1999., She got 50 bonus shares in September 2000.
Page 651 :
Income Under The Head Capital Gains, , 132, , She again got 550 bonus shares by virtue of her holding on March, 2005. Fair market value of the shares of, A Ltd. on April, 2001 is ₹ 50. Market value as on 31-01-2018 ₹200 per share, In January 2022, she transferred all her shares @ 300 per share (brokerage 2%), Compute the capital gains taxable in the hands of Mrs. X for the Assessment Year 2022-23 assuming., (a) A Ltd. is an unlisted company and securities transaction tax was not applicable at the time of sale., (b) A Ltd. is a listed company and the shares are sold in a recognized stock exchange and securities, transaction tax was paid at the time of sale., Cost of inflation index., F.Y. 2001-02, 100, F.Y. 2004-05, 113, F.Y. 2021-22, 317, Solution is given on our website www.mkgeducation.com (Icon /Heading No.11: Books), , NOV – 2012, Question No. 3(a), (4 Marks), Mr. X inherited from his father 8 plots of land in 2000. His father had purchased the plots in 1990 for ₹ 5, lakhs. The fair market value of the plots as on 01.04.2001 was ₹8 lakhs. (₹1 lakh for each plot), On 1st June 2010, Mr. X started a business of dealer in plots and converted the 8 plots as stock in trade of his, business. He recorded the plots in his books and ₹30 lakhs being the fair market value of that date. In June, 2021, Mr. X sold the 8 plots for ₹70 lakhs., On 01-07-2021 he acquired a residential house property for ₹15 lakhs. He invested an amount of ₹10 lakhs, in construction of one more floor in his house in January 2022. The house was sold by him in March 2022, for ₹63,50,000., The valuation adopted by the registration authorities for charge of stamp duty was ₹85,50,000. As per the, assessee’s request the assessing officer made a reference to a valuation officer. The value determined by the, valuation officer was ₹87,20,000. Brokerage of 1% of sale consideration was paid by Mr. X., The relevant Cost Inflation Indices are, F.Y. 2001-02, 100, F.Y. 2010-11, 167, F.Y. 2021-22, 317, Give the tax computation for the relevant assessment years with reasoning., Solution is given on our website www.mkgeducation.com (Icon /Heading No.11: Books), Question No. 7(a), (4 Marks), Discuss the tax implication arising consequent to conversion of a Capital Asset into stock-in-trade of, business and its subsequent sale., Answer: Refer answer given in the Chapter, , MAY – 2012, Question 3, Mr. X transfers land and building on 02.01.2022 and furnishes the following informations., Particulars, (i) Net consideration received, (ii) Value adopted by Stamp Valuation Authority, (iii) Value ascertained by Valuation Officer on reference by the Assessing Officer, (iv) This land was acquired by Mr. X on 01.04.2001. Fair Market value of the land as on, 01.04.2001 was, (v) A Residential building was constructed on land by Mr. X at cost of ₹3,20,000, (construction completed on 01.12.2002 during financial year 2002-03.), Short term capital loss incurred on sale of shares during financial year 2014-15 b/f of, ₹50,000, , (4 Marks), ₹, 30,00,000, 34,00,000, 35,00,000, 1,10,000
Page 652 :
Income Under The Head Capital Gains, , 133, , Mr. X seeks your advice to the amount to be invested in NHAI bonds so as to be exempt from capital gain, tax under Income Tax Act., Cost inflation index of FY 2001-2002 = 100, Cost inflation index of FY 2002-2003 = 105, Cost inflation index of FY 2021-2022 = 317, Solution is given on our website www.mkgeducation.com (Icon /Heading No.11: Books), Question 6, (5 Marks), Mr. X received a vacant site as gift from his friend in November 2008. The site was acquired by his friend, for ₹3,00,000 in April 2002. Mr. X constructed a residential building during the year 2009-10 in the said site, for ₹15,00,000. He carried out some further extension of a construction in the year 2011-12 for ₹5,00,000., Mr. X sold the residential building for ₹55,00,000 in January 2022 but the state stamp valuation authority, adopted ₹65,00,000 as value for the purpose of stamp duty., Compute his long term capital gain, for the assessment year 2022-23 based on the above information. The, cost inflation index are as follows:, Financial year, Cost inflation index, 2002-03, 105, 2008-09, 137, 2009-10, 148, 2011-12, 184, 2021-22, 301, Solution is given on our website www.mkgeducation.com (Icon /Heading No.11: Books), Question 6, (3 Marks), Mr. X (62 years old), pledged his residential house to a bank under a notified reverse mortgage scheme. He, was getting loan from bank in monthly instalments. Mr. X did not repay the loan on maturity and hence gave, possession of the house to the bank to discharge his loan. How will the treatment of long-term capital gain, be on such reverse mortgage transaction?, Solution is given on our website www.mkgeducation.com (Icon /Heading No.11: Books), , NOV – 2011, Question 3, (4 Marks), Mr. X, acquired a residential house in January, 2002 for ₹10,00,000 and made some improvements by way, of additional construction to the house, incurring expenditure of ₹2,00,000 in October, 2004. He sold the, house property in October, 2021 for ₹70,00,000. The value of property was adopted as ₹80,00,000 by the, State stamp valuation authority for registration purpose. He acquired a residential house in January, 2021 for, ₹25,00,000. He deposited ₹20,00,000 in capital gains bonds issued by National Highways Authority of India, (NHAI) in June 2022., Compute the capital gain chargeable to tax for the assessment year 2022-23., What would be the tax consequence and in which assessment year it would be taxable, if the house property, acquired in January, 2021 is sold for ₹40,00,000 in December, 2022?, Cost inflation index: F.Y. 2001-2002 = 100, F.Y. 2004-2005 = 113, F.Y. 2021-2022 = 317, Solution is given on our website www.mkgeducation.com (Icon /Heading No.11: Books), Question 3, (4 Marks), Mr. X purchased 100 shares of ABC Ltd. on 01.04.2005 at rate of ₹1,000 per shares in public issue of the, company., Company allotted bonus shares in the ratio of 1:1 on 01.12.2020., He has sold all the shares on 01.10.2021 at the rate of ₹3,000 per share through a recognized stock exchange, and paid brokerage of 1% and securities transaction tax of 0.1% to celebrate his 55th birthday. Market value, as on 31-01-2018 ₹2,000 per share. The cost inflation Index are as follows:
Page 653 :
Income Under The Head Capital Gains, , 134, , Financial Year, Cost Inflation Index, 2005-06, 117, 2021-22, 317, Compute his total income and tax liability for Assessment Year 2022-23 assuming that he is having no, income other than given above., Solution is given on our website www.mkgeducation.com (Icon /Heading No.11: Books), , MAY – 2011, Question 1, (5 Marks), Mr. X transferred a vacant site on 28.10.2021 for ₹100 lakhs. The site was acquired for ₹9,99,300 on, 30.06.2005. He deposited ₹50 lakhs in eligible bonds issued by Rural Electrification Corporation (REC) on, 20.03.2022., Again, he deposited ₹20 lakhs in eligible bonds issued by National Highways Authority of India (NHAI) on, 16.04.2022., Compute total income and tax liability of Mr. X for the assessment year 2022-23., Financial year, Cost Inflation Index, 2005-06, 117, 2021-22, 317, Solution is given on our website www.mkgeducation.com (Icon /Heading No.11: Books), Question 4, (4 Marks), Decide the following transactions in the context of Income-tax Act, 1961:, (i), Mrs. X transferred a vacant site to Mrs. Y for ₹4,25,000. The stamp valuation authority fixed the, value of vacant site for stamp duty purpose at ₹6,00,000. The total income of Mrs. X and Mrs. Y, before considering the transfer of vacant site are ₹50,000 and ₹2,05,000 respectively. The indexed, cost of acquisition for Mrs. X in respect of vacant site is ₹4,00,000 (computed)., Determine the total income of both Mrs. X and Mrs. Y taking into account the above said, transaction., (ii), Mr. X is employed in a company with taxable salary income of ₹5,00,000. He received a cash gift of, ₹1,00,000 from ABC Charitable Trust (registered under section 12AA) in December 2021 for, meeting his medical expenses., Is the cash gift so received from the trust chargeable to tax in the hands of Mr. X?, Solution is given on our website www.mkgeducation.com (Icon /Heading No.11: Books), , NOV – 2010, Question 1, (5 Marks), Mr. A is a proprietor of ABC Enterprises having 2 units which were set up on 01.07.2012. He transferred on, 01.04.2021 his unit 1 by way of slump sale for a total consideration of ₹25 Lacs. The expenses is incurred, for this transfer were ₹28,000/-. His Balance Sheet as on 31.03.2021 is as under:, Liabilities, Total, Assets, Unit 1, Unit 2, Total, ₹, ₹, ₹, ₹, Own Capital, 15,00,000 Building, 12,00,000, 2,00,000, 14,00,000, Revaluation Reserve, 3,00,000 Machinery, 3,00,000, 1,00,000, 4,00,000, (for building of unit 1), Bank Loan, 2,00,000 Debtors, 1,00,000, 40,000, 1,40,000, (70% for unit 1), Trade creditors, 1,50,000 Other assets, 1,50,000, 60,000, 2,10,000, (25% for unit 1), Total, 21,50,000 Total, 17,50,000, 4,00,000, 21,50,000, Other information:, (i), Revaluation reserve is created by revising upward the value of the building of unit 1., (ii) No individual value of any asset is considered in the transfer deed.
Page 654 :
Income Under The Head Capital Gains, , 135, , Compute the capital gain for the assessment year 2022-23., Solution is given on our website www.mkgeducation.com (Icon /Heading No.11: Books), Question 4, (7 Marks), Mr. X (aged 55 years) owned a residential house at Nagpur. It was acquired by Mr. X on 10.10.2004 for, ₹4,00,000. It was sold for ₹55,00,000 on 04.11.2021. The State stamp valuation authority fixed the value of, the property at ₹70,00,000. The assessee paid 2% of the sale consideration as brokerage for the sale of said, property., Mr. X acquired a residential house at Chennai on 10.12.2021 for ₹13,00,000 and deposited ₹10,00,000 on, 10.04.2022 in the capital gain bond or Rural Electrification Corporation Ltd (RECL). He deposited, ₹5,00,000 on 06.07.2022 in the Capital Gain Deposit Scheme in a nationalized bank for construction of, additional floor on the residential house property acquired at Chennai., Compute the capital gain chargeable to tax in the hands of Mr. X for the assessment year 2022-23. Calculate, the Income Tax Payable on the assumption that he has no other income chargeable to tax., Cost inflation index: Financial year 2004-05 = 113, Financial year 2021-22 = 317, Solution is given on our website www.mkgeducation.com (Icon /Heading No.11: Books), Question 7, (4 Marks), Mr. X received ₹15,00,000 on 23.01.2022 on transfer of his residential building in a transaction of reverse, mortgage under a scheme notified by the Central Government. The building was acquired in March 2004 for, ₹8,00,000., Is the amount received on reverse mortgage chargeable to tax in the hands of Mr. X under the head ‘Capital, Gains’?, Cost inflation index for the Financial year 2003-04 = 109, Financial year 2021-22 = 317, Solution is given on our website www.mkgeducation.com (Icon /Heading No.11: Books), , MAY – 2010, Question 2, (4 Marks), Mr. X sold a house to his friend Mr. Y on 1st November, 2021 for a consideration of ₹25,00,000. The SubRegistrar refused to register the document for the said value, as according to him, stamp duty had to be paid, on ₹45,00,000, which was the Government guideline value. Mr. X preferred an appeal to the Revenue, Divisional Officer, who fixed the value of the house as ₹32,00,000 (₹22,00,000 for land balance for building, portion). The differential stamp duty was paid, accepting the said value determined. Assuming that the fair, market value is ₹32,00,000, what are the tax implications in the hands of Mr. X and Mr. Y for the, assessment year 2022-23? Mr. X had purchased the land on the 1st June, 2008 for ₹5,19,000 and completed, the construction of house on 1st March, 2020 for ₹14,00,000., Cost inflation indices may be taken as 137 for the financial year 2008-09, 272 for the financial year 2017-18, and 317 for the financial year 2021-22., Solution is given on our website www.mkgeducation.com (Icon /Heading No.11: Books), , NOV – 2009, Question 2, (4 Marks), Compute the net taxable capital gains and tax liability of Smt. X on the basis of the following information :, A house was purchased on 01.05.2008 for ₹4,50,000 and was used as a residence by the owner. The owner, had contracted to sell this property in June, 2021 for ₹15 lacs and had received an advance of ₹70,000, towards sale. The intending purchaser did not proceed with the transaction and the advance was forfeited by, the owner. The property was sold in December, 2021 for ₹19,00,000. The owner, from out of sale proceeds,, invested ₹2 lacs in a new residential house in January, 2022., Cost inflation index: Financial year 2008-09 = 137
Page 655 :
Income Under The Head Capital Gains, , 136, , Financial year 2021-22 = 317, Solution is given on our website www.mkgeducation.com (Icon /Heading No.11: Books), Question 4, (4 Marks), Explain the concept of reverse mortgage and discuss its tax implications., Answer. Refer answer given in the Chapter, Question 4, (4 Marks), What are the circumstances under which the Assessing Officer can make reference to the Valuation Officer, u/s 55A of the Income Tax Act, 1961?, Answer. Refer answer given in the Chapter, , NOV – 2009, Question 4, (4 Marks), Mr. X, a resident individual had purchased a plot of land at a cost of ₹75,000 in June, 2008. He constructed a, house for his residence on that land at a cost of ₹1,25,000 in August, 2011. He sold that house in May, 2021, at ₹16,00,000 and purchased another residential house in June, 2021 for ₹8,00,000. He furnishes other, income and investment as follows:, ₹, Interest on fixed deposit with a Bank (after TDS), 45,000, TDS made by bank, 5,000, Investment in NSC, 20,000, You are required to compute total income and tax payable by Mr. X for the assessment year 2022-23., Cost inflation index: Financial year 2008-09 = 137, Financial year 2011-12 = 184, Financial year 2021-22 = 317, Solution is given on our website www.mkgeducation.com (Icon /Heading No.11: Books), , JUNE – 2009, Question 3, (4 Marks), Mr. X is the owner of a residential house which was purchased in September, 2012 for ₹30,00,000. He sold, the said house on 5th August, 2021 for ₹ 58,00,000. Valuation as per stamp valuation authority of the said, plot of land was ₹ 73,00,000. He invested ₹ 8,00,000 in NHAI Bonds on 12th January, 2022. He purchased a, residential house on 8th September, 2021 for ₹ 12,00,000. He gives other particulars as follows:, Interest on fixed Bank Deposit, ₹ 32,000, Investment in public provident fund, ₹ 12,000, You are requested to calculate the total income for the assessment year 2022-2023 and the tax liability, if, any., Cost inflation index for F.Y. 2012-13 and 2021-22 are 200 and 317 respectively., Solution is given on our website www.mkgeducation.com (Icon /Heading No.11: Books), Question 4, (3 Marks), Mr. X’s father, who is 62 years old had pledged his residential house to a bank under a notified reverse, mortgage scheme. He was getting loan from bank in monthly installments. Mr. X's father did not repay the, loan on maturity and gave possession of the house to the bank to discharge his loan. How will the treatment, of long-term capital gain be made on such reverse mortgage transaction?, Solution is given on our website www.mkgeducation.com (Icon /Heading No.11: Books), , NOV – 2008, Question 1, (2 Marks), Capital gain of ₹75 lakh arising from transfer of long term capital assets will be exempt from tax if such, capital gain is invested in the bonds redeemable after three years, issued by NHAI under section 54EC of the, Act., Solution is given on our website www.mkgeducation.com (Icon /Heading No.11: Books)
Page 656 :
Income Under The Head Capital Gains, , 137, , NOV – 2007, Question 4., (4 Marks), Mr. X inherited a house in Jaipur under will of his father in May, 2003. The house was purchased by his, father in January, 2001 for ₹ 2,50,000. The market value of house as on 01.04.2001 was ₹2,70,000., He invested an amount of ₹ 7,00,000 in construction of one more floor in this house in June, 2005., The house was sold by him in November, 2021 for ₹ 37,50,000. The valuation adopted by the registration, authorities for charge of stamp duty was ₹ 47,25,000, but as per assessee’s request, the Assessing Officer, made a reference to Valuation Officer. The value determined by the Valuation Officer was ₹47,50,000., Brokerage @ 1% of sale consideration was paid by Mr. X to Mr. Y., You are required to compute the amount of capital gain chargeable to tax for A.Y. 2022-23 with the help of, given information 1and by taking CII for the F.Y. 2021-22 as 317, F.Y. 2003-04 as 109 and for F.Y. 200506 as 117., Solution is given on our website www.mkgeducation.com (Icon /Heading No.11: Books)
Page 657 :
Income Under The Head Business/Profession, , 138, , PROFITS AND GAINS, OF, , BUSINESS OR PROFESSION, , “PGBP”, SECTION 28 TO 44DB, PARTICULARS, Profits and gains of business or profession chargeability/ scope of income under this, head, Income from profits and gains of business or profession, how computed?, Rent, rates, taxes, repairs and insurance for buildings, Repairs and insurance of machinery, plant and furniture, Depreciation, Treatment of unabsorbed depreciation, Expenditure on scientific research, Deduction in respect of expenditure on specified business, Amortisation of certain preliminary expenses, Amortisation of expenditure incurred under voluntary retirement scheme, Other deductions, Interest on borrowed capital, Employer’s contribution to a recognised provident fund or approved, superannuation fund, Employer’s contribution to an approved gratuity fund, Sums received from employees towards certain welfare schemes if credited to their, accounts before the due date, Bad debts, Expenditure on promoting family planning amongst the employees, Securities transaction tax, Commodities transaction tax, General deductions, Advertisement to political parties, Building, etc., partly used for business and partly for personal purpose., Amounts not deductible, Payments to relatives/related persons, Disallowance out of cash expenditure exceeding ₹20,000, , SECTIONS, 28, 29, 30, 31, 32, 32(2), 35, 35AD, 35D, 35DDA, 36, 36(1)(iii), 36(1)(iv), 36(1)(v), , 36(1)(va), 36(1)(vii), 36(1)(ix), 36(1)(xv), 36(1)(xvi), 37(1), 37(2B), 38, 40, 40A(2), 40A(3)/, Rule 6DD, Disallowance in respect of provision for gratuity, 40A(7), Disallowance in respect of contribution to non-statutory funds, 40A(9), Deemed profits chargeable to tax, 41, Actual cost, 43(1), Written down value, 43(6), Certain deductions to be only on actual payment, 43B, Special provisions for full value of consideration for transfer of assets other than 43CA
Page 658 :
Income Under The Head Business/Profession, capital assets in certain cases, Maintenance of accounts by certain persons carrying on business or profession, Compulsory audit of accounts, Special provisions for computing profits and gains of business of civil construction, Special provision for computing profits and gains of profession on presumptive, basis, Special provisions for computing profits and gains of business of plying, hiring or, leasing goods carriage, Method of accounting, Block of Assets, , 139, 44AA/Rule 6F, 44AB/Rule 6G, 44AD, 44ADA, 44AE, 145, 2(11), , Question 1 [Imp.]: What are the incomes chargeable to tax under the head Business/Profession?, Answer: Incomes chargeable to tax under the head Business/ Profession Section 28, As per section 28, income from any business/profession shall be taxable under the head Business/Profession, and income shall be computed in the similar manner as in case of general practice of accountancy but, incomes and expenditures shall be such as are given under Income Tax Act. The following incomes shall, also be taxable under the head Business/Profession., (i) Income from Speculation Business shall be taxable under the head business/profession., (ii) Gift in connection with business/profession, Any gift or perquisite or benefit received in connection with business/profession. If any gift has been, received from any client, it will be considered to be income under the head Business/Profession e.g. If a, Chartered Accountant has received gift of ₹30,000 from one of his client, it will be considered to be his, income under the head business/profession., Example, ABC Ltd. has engaged one Advocate with regard to its legal proceedings. The company has provided him, facilities of free travelling, boarding/lodging and has incurred ₹25,000, it will be considered to be, professional receipt of the Advocate., (iii) Payments for not pursuing any business activity or profession/non-compete fee, If any person has received any payment from any other person for not pursuing any business activity or, profession i.e. payment has been received for closing down the business or profession, it will also be, considered to be income under the head business/profession. Similarly if payment has been received for not, using any patent right or technical know-how or other similar right, it will also be considered to be income, under the head business/profession., It is also called non-compete fee. The person making payment should deduct tax at source @ 10% as per, section 194J., Example, ABC Ltd. has received ₹30,00,000 for not carrying out a particular business activity, in this case, the amount, so received shall be considered to be income of the assessee., Example, ABC Ltd. has received ₹ 10,00,000 for not sharing a particular patent, in this case, it will be considered to be, income under the head business/profession., (iv) Payment under Keyman Insurance Policy, Sometimes employer may take a life policy in the name of any of his employees who are considered to be, very important for business or profession and such policy is called keyman insurance policy and premium is, paid by employer and employer is allowed to debit it to profit and loss account and amount received on, maturity shall be considered to be income of employer as per section 28., If any payment has been received by the employee, it will be considered to be income under the head salary., Similarly a policy may be taken in the name of any other person who is considered to be very important for, the business of the employer, such policy is also called keyman insurance policy. If payment has been, received by such other person, it will be considered to be his income under the head other sources as per, section 56., (v) Export Incentives, If any manufacturer is exporting the goods manufactured by him, in such cases he may be given certain
Page 659 :
Income Under The Head Business/Profession, , 140, , incentives by the Govt. and such incentives are called export incentives and shall be considered to be income, of the assessee under the head business/ profession and in general there are two types of incentives:, (i) GST paid by the assessee on inputs or other goods shall be refunded to assessee as an incentive and it will, be called duty drawback i.e. drawing back the duty paid by the assessee., (ii) The exporters are issued special licenses for importing goods without payment of custom duty and such, licenses are called import entitlement licenses and an exporter is allowed to sell it in the market and profit on, sale of import entitlement license shall be considered to be income under the head business/profession., Example, ABC Ltd. has computed its income to be ₹20,00,000 and some of the entries noted from profit and loss, account are as given below:, 1. Company has debited the amount of opening stock ₹33,00,000 which is overvalued by 10%., 2. Company has received duty drawbacks of ₹7,00,000 but the amount has not been credited to the profit, and loss account., 3. The company has received import entitlement license from the Government and it was sold it at a profit of, ₹3,00,000. The amount has not been credited to the profit and loss account., Solution:, In this case company’s tax liability shall be:, ₹, Net profit as per profit and loss account, 20,00,000, Add:, • Opening stock overvalued (33,00,000 /110 x 10), 3,00,000, • Duty drawback received, 7,00,000, • Sale of import entitlement license, 3,00,000, Income under the head business/profession, 33,00,000, Total Income, 33,00,000, Computation of Tax Liability, Tax on ₹33,00,000 @ 30%, 9,90,000, Add: HEC @ 4%, 39,600, Tax Liability, 10,29,600, (vi) If any person has received any amount in connection with termination or modification of terms and, conditions of any contracts relating to his business, amount so received shall be considered to be income, under the head business/profession., (vii) If any person has converted any inventory or stock in trade in to a capital asset, in such cases business, income shall be computed and for this purpose fair market value of the inventory on the date of conversion, shall be taken into consideration, e.g. ABC limited is engaged in business of sale /purchase of generators but, company has used one generator in its business premises which was purchased for ₹ 10,00,000 but market, value is ₹ 11,00,000, in this case there will be business income of ₹1,00,000 and for the purpose of charging, depreciation, value shall be taken to be ₹ 11,00,000., (viii) Income from specific services performed for its members by a trade, professional or business:, Income derived by any trade, professional or similar associations from specific services rendered by them to, their members. It may be noted that this forms an exception to the general principle governing the, assessment of income of mutual associations such as chambers of commerce, stock brokers’ associations etc., As a result, a trade, professional or similar association performing specific services for its members is to be, deemed as carrying on business in respect of these services and on that assumption the income arising, therefrom is to be subjected to tax. For this purpose, it is not necessary that the income received by the, association should definitely or directly be related to these services., Income from profits and gains of business or profession, how computed Section 29, The income referred to in section 28 shall be computed in accordance with the provisions contained in, sections 30 to 43D., Question 2: Write a note on deductibility of expenditures relating to Buildings., Answer: Rent, Rates, Taxes, Repairs and Insurance for Buildings Section 30, If any assessee has any building in the use of business/profession, all expenses relating to the building shall, be allowed to be debited to the profit and loss account and such expenses may be:, (i) Repairs expenses
Page 660 :
Income Under The Head Business/Profession, , 141, , (ii) Municipal tax or local tax or land revenue (but on payment basis as per section 43B), (iii) Premium for insurance of house, (iv) Any other expenditure like depreciation etc., If the building is owned by the assessee, he is not allowed to debit rent on notional basis, (No income shall be computed with regard to this house property under the head house property)., Question 3: Write a note on deductibility of expenditures relating to Plant and Machinery and, Furniture and Fixtures., Answer: As per section 31 if any assessee has any plant and machinery or furniture/fixture in his, business/profession, assessee is allowed to debit all the expenses to the profit and loss account and such, expenses may be like repairs or insurance or rent etc. If plant & machinery etc. are owned by the assessee,, its notional rent is not allowed to be debited., Question 4: Write a note on rates of Depreciation., Answer: Depreciation Section 32, Depreciation under Income Tax Act is allowed on the basis of written down value method but in case of, power generating unit, the assessee has the option to compute depreciation either on the basis of written, down value method or on the basis of Straight Line Method. Rate of depreciation shall be as given below:, PART A TANGIBLE ASSETS, I, Buildings, Block 1. Buildings which are used mainly for residential purposes except hotels and boarding houses, , 5%, , Block 2. Buildings which are not used mainly for residential purposes and not covered by Block (1), above and (3) below, , 10%, , Block 3. Buildings acquired on or after 1st September, 2002 for installing machinery and plant, forming part of water supply project or water treatment system and which is put to use for, the purpose of business of providing infrastructure facilities, , 40%, , Block 4. Purely temporary erections such as wooden structures, , 40%, , II, Furniture and Fittings, Block 1. Furniture and fittings including electrical fittings, 10%, [“Electrical fittings” include electrical wiring, switches, sockets, other fittings and fans, etc.], III, Plant & Machinery, Block 1. (i) Motor cars other than those used in a business of running them on hire, acquired, during the period from 23.8.2019 to 31.03.2020 and put to use on or before 31.03.2020, (ii) Motor cars other than those used in a business of running them on hire, acquired or put, to use on or after 1-4-1990 [Other than motor cars mentioned in (i) above], , 30%, 15%, , Block 2. (i) Motors buses, motor lorries, motor taxis used in a business of running them on, hire, acquired during the period from 23.8.2019 to 31.03.2020 and put to use on or, before 31.03.2020, (ii) Motors buses, motor lorries, motor taxis used in the business of running them on hire, [Other than mentioned in (i) above], , 45%, , Block 3. Moulds used in rubber and plastic goods factories, , 30%, , Block 4. Aeroplanes, Aeroengines, , 40%, , Block 5. Specified air pollution control equipments, water pollution control equipments, solid waste, control equipment and solidwaste recycling and resource recovery systems, , 40%, , Block 6. Plant & Machinery used in semi-conductor industry covering all Integrated Circuits (Ics), , 30%, , Block 7. Life saving medical equipment, , 40%, , Block 8. Machinery and plant, acquired and installed on or after the 1st day of September, 2002, in a water supply project or a water treatment system and which is put to use for the, , 40%, , 30%
Page 661 :
Income Under The Head Business/Profession, , 142, , purpose of business of providing infrastructure facility, Block 9. Oil wells, , 15%, , Block 10. Renewable Energy Saving Devices (as specified), , 40%, , (i) Windmills and any specially designed devices which run on windmills installed on, or after 1.4.2014, , 40%, , (ii) Any special devices including electric generators and pumps running on wind, energy installed on or after 1.4.2014 would be eligible for depreciation, , 40%, , (iii) Windmills and any specially designed devices running on windmills installed on, or before 31.3.2014 and any special devices including electric generators and pumps, running on wind energy installed on or before 31.3.2014, , 15%, , Block 11. Computers including computer software, , 40%, , Block 12. Books (annual publications or other than annual publications) owned by assessees carrying, on a profession, , 40%, , Block 13. Books owned by assessees carrying on business in running lending libraries, , 40%, , Block 14. Plant & machinery (General rate), , 15%, , IV, Ships, Block 1. Ocean-going ships, , 20%, , Block 2. Vessels ordinarily operating on inland waters not covered by Block (3) below, , 20%, , Block 3. Speed boats operating on inland water, , 20%, , PART B INTANGIBLE ASSETS, Know-how, patents, copyrights, trademarks, licences, franchises or any other business or commercial, rights of similar nature (no depreciation on goodwill), , 25%, , Question 5 [V. Imp.]: Write a note on computation of Depreciation., Answer: Computation of Depreciation, If any particular asset is purchased during the year and it has been put to use for less than 180 days during, the year, in that case, depreciation is allowed at half the normal rate. If it is purchased during the year and is, not at all put to use, depreciation shall not be allowed. But in the subsequent year whenever the asset is put, to use, full depreciation shall be allowed irrespective of period of use., “Put to use” do not mean putting the asset to actual use rather it means making an asset ready for use., Example, ABC Ltd. has purchased one plant and machinery on 01.07.2021 for ₹30,00,000, it was installed on, 01.10.2021, but it was brought into actual use w.e.f. 01.03.2022, in this case, depreciation allowed shall be, ₹4,50,000, because the asset was put to use for 180 days or more, but if the asset was installed on, 10.10.2021, depreciation allowed shall be ₹2,25,000. If the asset was not at all installed in the year 2021-22,, depreciation allowed in 2021-22 shall be nil. If the asset was installed on 31.03.2023, depreciation allowed, in 2021-22 shall be nil, but the depreciation allowed in the year 2022-23, shall be ₹4,50,000., If any asset has been sold at any time during the year, in that case, depreciation is not allowed for that year., Example, ABC Ltd. purchased one plant and machinery on 01.10.2016, its written down value on 01.04.2021 is, ₹20,00,000 but it was sold on 31.03.2022, in this case, no depreciation is allowed in the previous year 202122., Question 6: Write a note on Computation of depreciation on the basis of block of assets., Answer:, Under section 32, depreciation under income tax is allowed on the basis of written down value method. It is, not computed on the basis of individual assets rather on the basis of a group of assets called Block of Assets, which means a group of similar type of assets having same rate of depreciation and shall be computed in the
Page 662 :
Income Under The Head Business/Profession, , 143, , manner given below:, 1. Take opening written down value of the particular block of asset as on 1st day of April of the relevant, year., 2. Add purchases during the year., 3. Deduct sale value in case of sale and amount of insurance claim in case of fire or theft etc. or scrap, value in case of discarded assets., 4. Apply depreciation on the balance amount as on the last day of the year., 5. If any asset was put to use for less than 180 days, depreciation shall be allowed at half the normal rate, and for this purpose its actual cost shall be separated from the total written down value of the block, and if total written down value is less than the actual cost, depreciation shall be applied on the written, down value of the block at half the normal rate., 6. If there is a negative balance at the end of the year, it will be considered to be short term capital gain as, per section 50 and no depreciation is allowed., 7. If there is no asset at the end of the year but still there is some balance, it will be considered to be short, term loss as per section 50 and no depreciation is allowed., Block of Assets: A “block of assets” is defined in section 2(11), as a group of assets falling within a class of, assets comprising—, (a) tangible assets, being buildings, machinery, plant or furniture;, (b) intangible assets, being know-how, patents, copyrights, trademarks, licenses, franchises or any other, business or commercial rights of similar nature, not being goodwill of a business profession,, in respect of which the same percentage of depreciation is prescribed., Computation of depreciation can be illustrated in the manner given below:, Situation 1:, Written down value of plants A, B, C on 01.04.2021, Add: purchased plant D on 05.05.2021 and put to use on 05.05.2021, Less: sale of plant A, Less: depreciation for the year @ 15%, Written down value of plant B, C, D as on 01.04.2022, Situation 2:, Written down value of plants A, B, C on 01.04.2021, Add: purchased plant D on 05.05.2021 and put to use on 01.12.2021, Less: sale of plant A, Less: depreciation [7.5% on 20 Lakhs & 15% on 52 Lakhs], Written down value of plants B, C, D as on 01.04.2022, Situation 3:, Written down value of plants A, B, C on 01.04.2021, Add: purchased plant D on 05.05.2021 and put to use on 01.12.2021, Less: sale of plant A, Less: depreciation [7.5% on 15 Lakhs], Written down value of plants B, C, D as on 01.04.2022, Situation 4:, Written down value of plants A, B, C on 01.04.2021, Less: Sale of plant A, Short term capital gain under section 50, Depreciation for the year, Written down value of plants B, C as on 01.04.2022, , ₹, 70,00,000, 20,00,000, (18,00,000), 72,00,000, (10,80,000), 61,20,000, ₹, 70,00,000, 20,00,000, (18,00,000), 72,00,000, (9,30,000), 62,70,000, ₹, 70,00,000, 20,00,000, (75,00,000), 15,00,000, (1,12,500), 13,87,500, ₹, 70,00,000, (75,00,000), 5,00,000, Nil, Nil
Page 663 :
Income Under The Head Business/Profession, Situation 5:, Written down value of plants A, B, C on 01.04.2021, Less: Sale of plant A, B, C, Short term capital loss under section 50, Depreciation for the year, , 144, ₹, 70,00,000, (20,00,000), 50,00,000, Nil, , Illustration 1: Mr. X started his business on 01.04.2018 and purchased various plants and machinery as, given below:, He has purchased plant P1 on 01.04.2018 which was put to use on 01.06.2018 for ₹20,00,000., He has purchased plant P2 on 01.05.2018 which was put to use on 01.07.2018 for ₹25,00,000., He has purchased plant P3 on 01.06.2018 which was put to use on 01.09.2018 for ₹25,00,000., He has purchased plant P4 on 01.07.2018 which was put to use on 01.09.2018 for ₹35,00,000., He sold plant P1 on 01.01.2019 for ₹ 11,00,000., He purchased plant P5 on 01.05.2019 and was put to use on 01.11.2019 for ₹26,00,000., He purchased plant P6 on 01.12.2019 and was put to use on 31.03.2020 for ₹20,00,000., He purchased plant P7 on 01.06.2020 and put to use on 10.12.2020 for ₹10,00,000., He sold plant P2 on 31.03.2021 for ₹ 9,00,000., He purchased plant P8 on 01.07.2021 and was put to use on 01.01.2022 for ₹27,00,000., Determine depreciation for various years. (Ignore additional depreciation), Solution:, ₹, Plant and Machinery, Depreciation @ 15%, Previous Year 2018-19, Purchased P1 on 01.04.2018, put to use on 01.06.2018, 20,00,000.00, Purchased P2 on 01.05.2018, put to use on 01.07.2018, 25,00,000.00, Purchased P3 on 01.06.2018, put to use on 01.09.2018, 25,00,000.00, Purchased P4 on 01.07.2018, put to use on 01.09.2018, 35,00,000.00, Sale P1 on 01.01.2019, (11,00,000.00), Written down value P2, P3 and P4 on 31.03.2019, 94,00,000.00, Depreciation @ 15%, 14,10,000.00, Previous Year 2019-20, Written down value P2, P3 and P4 on 01.04.2019, 79,90,000.00, Purchased P5 on 01.05.2019, put to use on 01.11.2019, 26,00,000.00, Purchased P6 on 01.12.2019, put to use on 31.03.2020, 20,00,000.00, Written down value P2, P3, P4, P5 and P6 on 31.03.2020, 1,25,90,000.00, Depreciation @ 7.5% on ₹46,00,000, 3,45,000.00, Depreciation @ 15% on ₹79,90,000, 11,98,500.00, Previous Year 2020-21, Written down value P2, P3, P4, P5 and P6 on 01.04.2020, 1,10,46,500.00, Purchased P7 on 01.06.2020, put to use on 10.12.2020, 10,00,000.00, Sale P2 on 31.03.2021, (9,00,000.00), Written down value P3, P4, P5, P6 and P7 on 31.03.2021, 1,11,46,500.00, Depreciation @ 7.5% on ₹10,00,000, 75,000.00, Depreciation @ 15% on ₹1,01,46,500, 15,21,975.00, Previous Year 2021-22, Written down value P3, P4, P5, P6 and P7 on 01.04.2021, 95,49,525.00, Purchased P8 on 01.07.2021, put to use on 01.01.2022, 27,00,000.00, Written down value P3, P4, P5, P6, P7 and P8 on 31.03.2022, 1,22,49,525.00, Depreciation @ 7.5% on ₹27,00,000, 2,02,500.00, Depreciation @ 15% on ₹95,49,525, 14,32,428.75, MAY-2007 (4 Marks), A newly qualified Chartered Accountant Mr. X, commenced practice and has acquired the following assets, in his office during F.Y. 2021-22 at the cost shown against each item. Calculate the amount of depreciation, that can be claimed from his professional income for A.Y. 2022-23:
Page 664 :
Income Under The Head Business/Profession, Sl. No., 1., 2., 3., 4., 5., 6., 7., , Description, Computer, Computer software, Computer printer, Books, Office furniture (Acquired from practising C.A.), Laptop, Fire extinguisher, , Date of, acquisition, 27 Sept., 2021, 1 Oct., 2021, 1 Oct., 2021, 1 Apr., 2021, 1 Apr., 2021, 26 Sept.,2021, 1 Apr., 2021, , 145, Date when, put to use, 2 Oct., 2021, 1 Oct., 2021, 3 Oct., 2021, 1 Apr., 2021, 1 Apr., 2021, 1 Oct., 2021, 1 Apr., 2021, (No instance, arose to use, during, F.Y., 2021-22), , Amount, ₹, 35,000, 8,500, 12,500, 13,000, 3,00,000, 43,000, 2,500, , 8., , Purchased practising CA’s office in April’ 2021, who had run it for 4 years, for ₹5 lacs which, includes ₹2 lacs for goodwill and ₹3 lacs for cost, of furniture (included in 5 above), Note: Depreciation is to be provided at the applicable rates., Answer., Computation of depreciation allowable for A.Y. 2022-23, Computation of depreciation, Block of Assets, ₹, Block 1: Furniture – rate 10%, ₹3,00,000 @ 10%, 30,000, Block 2: Computer – rate 40%, Computer, 35,000, Computer software, 8,500, Laptop, 43,000, Total, 86,500 @ 40%, 34,600, Block 3: Books – rate 40%, Books, 13,000 @ 40%, 5,200, Block 4: Plant – Rate 15%, Computer printer, 12,500, Fire extinguisher, 2,500, Total, 15,000 @ 15%, 2,250, (Whether computer printer is computer or plant and machinery is controversial), MAY-2010 (4 Marks), Mr. X has the following Assets which are eligible for depreciation at 15% on Written Down Value (WDV), basis:, 01.04.2018, WDV of plant ‘X’ and Plant ‘Y’, ₹2,00,000, 10.12.2021, Acquired a new plant ‘Z’ for, ₹2,00,000, 22.01.2022, Sold plant ‘Y’ for, ₹4,00,000, Expenditure incurred in connection with transfer, ₹ 10,000, Compute eligible depreciation claim/chargeable capital gain if any, for the Assessment Year 2022-23., Solution:, Computation of depreciation and capital gains of Mr. X for the A.Y. 2022-23, ₹, W.D.V. of Plant ‘X’ & Plant ‘Y’ as on 01.04.2018, 2,00,000, Less: Depreciation @ 15% for the assessment year 2019-20, (30,000), W.D.V. of Plant ‘X’ & Plant ‘Y’ as on 01.04.2019, 1,70,000, Less: Depreciation @ 15% for the assessment year 2020-21, (25,500), W.D.V. of Plant ‘X’ & Plant ‘Y’ as on 01.04.2020, Less: Depreciation @ 15% for the assessment year 2021-22, , 1,44,500, (21,675)
Page 665 :
Income Under The Head Business/Profession, W.D.V. of Plant ‘X’ & Plant ‘Y’ as on 01.04.2021, Add : Cost of new Plant ‘Z’ acquired during the previous year ending on 31.03.2022, Less: Sale consideration of Plant ‘Y’ ₹4,00,000 (restricted to), W.D.V. of Plant ‘X’ and ‘Z’ as on 01.04.2022, Depreciation, Sale proceeds of Plant ‘Y’, Less: Deduction under section 50, W.D.V of the block, Expenditure incurred in connection with transfer, Short term Capital Gains, , 146, 1,22,825, 2,00,000, 3,22,825, (3,22,825), Nil, Nil, 4,00,000, (3,22,825), (10,000), 67,175, , Question 7: Write a note on Additional Depreciation., Answer: Additional Depreciation Section 32, Additional depreciation shall be allowed @ 20% to all the assessee in connection with plant and machinery, for the purpose of manufacturing and also to the assessee engaged in generation, transmission or, distribution of electricity. Additional depreciation shall be allowed only in the year in which asset has been, put to use. It is allowed only once i.e. it is not allowed every year., Additional depreciation is not allowed in the following cases:, (i) Second hand plant and machinery i.e. plant and machinery should be brand new, (ii) Any machinery or plant installed in any office premises or any residential accommodation,, including accommodation in the nature of a guest-house or, (iii)Any office appliances or road transport vehicles or ships and aircraft, (iv) Any machinery or plant, the actual cost of which has been debited to profit and loss account., If the asset is purchased and put to use for less than 180 days, additional depreciation shall be allowed at, 10% and remaining additional depreciation shall be allowed in the subsequent year., Illustration 2: ABC Ltd. is engaged in manufacturing and has submitted information as given below:, 1. Factory Building – Written down value on 01.04.2021 was ₹12,00,000., 2. Plant and Machinery (Rate 15%) – Written down value on 01.04.2021 is ₹8,70,000., 3. Purchase of new plant (eligible for additional depreciation) on 30.06.2021 (Put to use on 01.07.2021), ₹1,20,000., 4. Purchase of new plant (eligible for additional depreciation) on 31.12.2021 (Put to use on 01.01.2022), ₹1,10,000., 5. Sale of old Plant on 01.12.2021 ₹6,40,000., 6. Motor Car (Rate 15%) – Written down value on 01.04.2021 was ₹1,20,000., 7. Sale of Car on 30.09.2021 ₹1,50,000., Compute depreciation allowed., Solution:, ₹, Factory Building, Depreciation @ 10%, Written down value on 01.04.2021, 12,00,000, Depreciation @ 10%, 1,20,000, Plant and Machinery, Depreciation @ 15%, Written down value on 01.04.2021, Purchase on 30.06.2021, put to use on 01.07.2021, Purchase on 31.12.2021, put to use on 01.01.2022, Sale of old plant on 01.12.2021, Written down value on 31.03.2022, Depreciation @ 15% on ₹3,50,000, Depreciation @ 7.5% on ₹1,10,000, , 8,70,000, 1,20,000, 1,10,000, (6,40,000), 4,60,000, 52,500, 8,250
Page 666 :
Income Under The Head Business/Profession, , 147, , Additional depreciation, 1,20,000 x 20%, 24,000, 1,10,000 x 10%, 11,000, Motor Car, Depreciation @ 15%, Written down value on 01.04.2021, 1,20,000, Sale on 30.09.2021, (1,50,000), Short term capital gain, 30,000, Illustration 3: ABC Ltd. an industrial undertaking has started manufacturing on 01.05.2021 and the, company has purchased the following asset:, 1. Plant and machinery for use in the factory ₹30 lakhs, purchased on 01.07.2021 and put to use on, 15.07.2021., 2. Air-conditioner and generator for ₹2,00,000, purchased on 01.08.2021 and put to use on 10.08.2021, for use in office premises., 3. One motor car for ₹10 lakhs for use of business, purchased on 01.09.2021 and put to use on, 10.09.2021., 4. One T.V. and one fridge for ₹50,000, purchased and put to use on 01.05.2021., 5. Furniture and fixture for use in factory ₹5,00,000, purchased and put to use on 01.06.2021., Depreciation and additional depreciation shall be allowed in the manner given below:, Solution:, ₹, Computation of Depreciation, Block –I, Plant and machinery, depreciation @ 15%, Plant and Machinery purchased on 01.07.2021, put to use on 15.07.2021, 30,00,000, Air-conditioner and generator purchased on 01.08.2021, put to use on 10.08.2021, 2,00,000, One T.V. and one fridge purchased and put to use on 01.05.2021, 50,000, 32,50,000, Depreciation @ 15%, 4,87,500, Additional depreciation on plant and machinery for use in factory [30,00,000 x 20%], 6,00,000, Block –II, Motor Car, depreciation @ 15%, Motor Car purchased on 01.09.2021 and put to use on 10.09.2021, 10,00,000, Depreciation @ 15%, 1,50,000, Block –III, Furniture and Fixtures, depreciation @ 10%, Furniture and fixture for use in factory, purchased and put to use on 01.06.2021, 5,00,000, Depreciation @ 10%, 50,000, NOV-2013 (4 Marks), Mr. X is engaged in the business of generation and distribution of electric power. He always claims, depreciation on written down value. From the following details, compute the depreciation allowable as per, the provisions of the Income-tax Act, 1961 for the assessment year 2022-23:, ( ₹ in lacs), (i) Opening WDV of block (15% rate), 42, (ii) New machinery purchased on 12.10.2021, 10, (iii) Machinery imported from Colombo on 12.04.2021. This machine had been, 9, used only in Colombo earlier and the assessee is the first user in India., (iv) New computer installed in generation wing of the unit on 15.07.2021, 2, Solution:, Computation of depreciation under section 32 for A.Y.2022-23, Particulars, Block 15%, Opening w.d.v. as on 01.04.2021, Add: New Machinery purchased and put to use on 12.10.2021, , 15%, 42,00,000, 10,00,000
Page 667 :
Income Under The Head Business/Profession, Add: Machinery imported from Colombo purchased and put to use on 12.04.2021, W.d.v. as on 31.03.2022, Normal Depreciation @ 7.5% on ₹10,00,000, Normal Depreciation @ 15% on ₹51,00,000, Additional depreciation @ 10% on ₹10,00,000, Block 40%, New Computer in Generation wing purchased and put to use on 15.07.2021, Normal Depreciation @ 40% on ₹2,00,000, Additional depreciation @ 20% on ₹2,00,000, , 148, 9,00,000, 61,00,000, 75,000, 7,65,000, 1,00,000, 2,00,000, 80,000, 40,000, , Illustration 4: ABC Ltd. is engaged in manufacturing and company has purchased new plant and, machinery during the previous year 2021-22, 1. ₹ 20.00 crore (purchased and put to use on 01.07.2021), 2. ₹30.00 crore (purchased and put to use on 01.11.2021), Compute depreciation / additional depreciation and also w.d.v as on 01.04.2022., Solution:, Crore (₹), Computation of depreciation / additional depreciation, Plant and machinery purchased and put to use on 01.07.2021, 20.00, Plant and machinery purchased and put to use on 01.11.2021, 30.00, Less: depreciation @ 15% on ₹20.00, (3.00), Less: depreciation @ 7.5% on ₹30.00, (2.25), Less: Additional depreciation @ 20% on ₹20.00, (4.00), Less: Additional depreciation @ 10% on ₹30.00, (3.00), w.d.v as on 01.04.2022, 37.75, Illustration 5: ABC Ltd. is engaged in manufacturing and company has purchased plant and machinery, during the previous year 2021-22 for ₹26 crores (purchased and put to use on 10.11.2021) and it includes, second hand plant and machinery for ₹5 crores., Compute depreciation / additional depreciation and also w.d.v as on 01.04.2022., Solution:, Computation of depreciation / additional depreciation, Plant and machinery purchased and put to use on 10.11.2021, 26 crore, Less: depreciation @ 7.5% on ₹26 crore, (1.95 crore), Less: Additional depreciation @ 10% on ₹21 crore, (2.10 crore), w.d.v as on 01.04.2022, 21.95 crore, MAY – 2014 (4 Marks), JK Ltd., a manufacturing company purchased the following new Plant and Machinery., Date of Acquisition and Installation, Actual Cost (in Crores), 25.05.2021, 10.00, 31.10.2021, 22.00, From the above information compute the amount of depreciation available u/s 32, additional depreciation, if, any for the Assessment Years 2022-23., Solution:, Computation of depreciation allowance under section 32 for the A.Y. 2022-23, Particulars, New Plant and Machinery acquired and installed during the P.Y.2021-22, Depreciation for P.Y.2021-22 @ 15% on ₹10.00 crore, Depreciation for P.Y.2021-22 @ 7.5% on ₹22.00 crore, Additional Depreciation @ 20% on ₹10.00 crore, Additional Depreciation @ 10% on ₹22.00 crore, WDV as on 01.04.2022, , ₹ in crores, 32.00, (1.50), (1.65), (2.00), (2.20), 24.65
Page 668 :
Income Under The Head Business/Profession, , 149, , Question 8: Write a note on depreciation in case of Amalgamation, Demerger, Conversion of, Proprietary Concern or Partnership Firm into a Company or Conversion of Private Limited, Company or Unlisted Public Company into Limited Liability Partnership Firm., Answer: As per section 32, depreciation shall be computed considering that no such amalgamation etc. has, taken place and the depreciation so computed shall be apportioned between the predecessor and successor in, the ratio of number of days the asset was used by each one of them, Example, M/s XY & Co., a sole proprietary concern is converted into a company, XY Co. Ltd. with effect from, December 29, 2021. The written down value of assets as on April 1st, 2021 is as follows:, Items, Rate of Dep., WDV as on 1st April, 2021, Building, 10%, ₹3,50,000, Furniture, 10%, ₹ 50,000, Plant and Machinery, 15%, ₹2,00,000, Further, on October 15, 2021, M/s XY & Co. purchased a plant for ₹1,00,000 (rate of depreciation 15%) and, it was put to use on the same date. After conversion, the company added another plant worth ₹50,000 (rate, of depreciation 15%) on 01.01.2022 and put to use on the same date., Compute the depreciation available to (i) M/s XY & Co. and (ii) XY Co. Ltd. for Assessment Year 2022-23., Solution:, As per section 32, while determining depreciation, if there is change of ownership of assets because of, conversion of sole proprietary concern into company, depreciation will be calculated in the manner given, below:, ₹, Building, Depreciated value on April 1st, 2021, 3,50,000, Depreciation @ 10%, 35,000, Furniture, Depreciated value on April 1st, 2021, 50,000, Depreciation @ 10%, 5,000, Plant and Machinery, Depreciated value on April 1st, 2021, 2,00,000, Add: Cost of new plant and machinery, 1,00,000, Written down value, 3,00,000, Depreciation @ 7.5% on ₹1,00,000, 7,500, Depreciation @ 15% on ₹2,00,000, 30,000, Number of days when assets are used by, Sole Proprietors, 272 days, Company, 93 days, Depreciation available to the sole proprietary Concern, 70,000 / 365 x 272, 52,164.38, 7,500 / 168 x 75, 3,348.21, Depreciation available to the company, 70,000 / 365 x 93, 17,835.62, 7,500 / 168 x 93, 4,151.79, Depreciation to the company on plant purchased for ₹50,000, 50,000 x 7.5%, 3,750.00, Total depreciation allowed to the company, 25,737.41, NOV-2014 (4 Marks), Mr. X carrying on business as proprietor converted the same into a limited company by name X Pipes (P), Ltd. from 01.07.2021 The details of the assets are given below:, ₹, Block — I WDV of plant & machinery (rate of depreciation @ 15%), 12,00,000, Block - II WDV of building (rate of depreciation @ 10%), 25,00,000, The company X Pipes (P) Ltd. acquired plant and machinery in December 2021 for 10,00,000. It has been, doing the business from 01.07.2021.
Page 669 :
Income Under The Head Business/Profession, , 150, , Compute the quantum of depreciation to be claimed by Mr. X and successor X Pipes (P) Ltd. for the, assessment year 2022-23., Note : Ignore additional depreciation., Solution:, Section 32 is applicable while determining depreciation, if there is change of ownership of assets because of, conversion of sole proprietary concern into company. In respect of assets transferred to the company,, depreciation will be calculated asunder:, ₹, Block — I, Depreciated value on April 1st, 2021, 12,00,000, Depreciation @ 15%, 1,80,000, Block - II, Depreciated value on April 1st, 2021, 25,00,000, Depreciation @ 10%, 2,50,000, Plant and Machinery, Cost of new plant and machinery, 10,00,000, Depreciation @ 7.5% on ₹10,00,000, 75,000, Number of days when assets are used by, Sole Proprietors, 91 days, Company, 274 days, Depreciation available to the sole proprietary Concern, (i.e. 91 / 365 of ₹4,30,000), 1,07,205.48, Depreciation available to the company, (i.e. 274 / 365 of ₹4,30,000), 3,22,794.52, Depreciation to the company on plant purchased, 75,000.00, Total depreciation allowed to the company, 3,97,794.52, Note: The depreciation on the plant which was purchased after conversion shall be allowed to the company, and further it is presumed that the plant was put to use on the date of purchase., NOV-2010 (4 Marks), X Ltd. has a block of assets carrying 15% rate of depreciation, whose written down value on 01.04.2021 was, ₹40 lacs. It purchased another asset of the same block on 01.11.2021 for ₹14.40 lacs and put to use on the, same day. X Ltd. was amalgamated with Y Ltd. with effect from 01.01.2022., You are required to compute the depreciation allowable to X Ltd. & Y Ltd. for the previous year ended on, 31.03.2022., Solution:, Statement showing computation of depreciation allowable to X Ltd. & Y Ltd. for P.Y. 2021-22, Particulars, Amount, ₹, Written down value (WDV) as on 01.04.2021, 40,00,000, Addition during the year (used for less than 180 days), 14,40,000, Total, 54,40,000, Depreciation on ₹40,00,000 @ 15%, 6,00,000, Depreciation on ₹14,40,000 @ 7.5%, 1,08,000, Total depreciation for the year, 7,08,000, Apportionment between two companies:, (a), Amalgamating company, X Ltd., ₹6,00,000 / 365 × 275, 4,52,054.79, ₹1,08,000 / 151 × 61, 43,629.14, 4,95,683.93, (b), Amalgamated company, Y Ltd., ₹6,00,000 / 365 × 90, 1,47,945.21, ₹1,08,000 / 151 × 90, 64,370.86
Page 670 :
Income Under The Head Business/Profession, , 151, , Question 9: Write a note on depreciation in case of Power Generating Units., Answer: Depreciation in case of Power Generating Units, A power generating unit shall have the option to claim depreciation either on the basis of SLM or WDV and, any option taken cannot be changed subsequently., If the assessee has claimed depreciation on the basis of WDV, depreciation shall be allowed on the basis of, block of the assets and if depreciation is claimed on SLM, depreciation shall be computed on the basis of, individual asset however concept of 180 days shall be applicable. Rate of depreciation for SLM shall be as, prescribed under Income Tax Act., In case of sale of asset, tax treatment shall be as given below:, Sale of asset, Terminal depreciation, If the asset is sold, any loss on their sale shall be considered to be terminal depreciation and shall be allowed, to be debited to the profit and loss account., Balancing Charge Section 41(2), If any asset has been sold or destroyed etc. and depreciation was claimed on SLM basis, any profit on sale, shall be considered to be income under the head business/profession and shall be called balancing charge but, only to the extent depreciation was debited to the profit and loss account., If the amount is received after closing down of the business, still it will be considered to be income under, the head business/profession i.e. it will be a case of having income under the head business/profession but, without any business/ profession., The excess over it shall be taxable as capital gains under section 50A., Example, ABC Ltd. is a power generating unit and the company has purchased one plant and machinery on, 01.07.2018 for ₹20 lakhs (not eligible for additional depreciation) and it was put to use on 01.11.2018 and, rate of depreciation is 7.8%, in this case depreciation allowed shall be, 2018-19 20,00,000 x 7.8% x ½ = ₹ 78,000, 2019-20 20,00,000 x 7.8%, = ₹1,56,000, 2020-21 20,00,000 x 7.8%, = ₹1,56,000., If this plant is sold on 01.10.2021, 1. For ₹ 7,00,000, 2. For ₹19,00,000, 3. For ₹23,00,000, The tax treatment shall be as given below:, 1. Written down value of the asset as on 01.04.2021 is ₹16,10,000 but it was sold for ₹7,00,000, in this, case terminal depreciation is 7,00,000 – 16,10,000 = ₹9,10,000 and it will be allowed to be debited to, profit and loss account., 2. If the asset is sold for ₹19,00,000, there will be profit of 19,00,000 – 16,10,000 = ₹2,90,000 and it will, be called ‘balancing charge’ under section 41(2) and shall be considered to be deemed income under, the head business/profession., 3. There will be gain of 23,00,000 – 16,10,000 = ₹6,90,000. There will be balancing charge to the extent, depreciation has been debited i.e. 3,90,000 and balance amount i.e. ₹3,00,000 shall be short term, capital gain as per section 50A., Illustration 6: Lights and Power Ltd. engaged in the business of generation of power, furnishes the, following particulars pertaining to P.Y. 2021-22. Compute the depreciation allowable under section 32 for, A.Y.2022-23. The company has opted for the depreciation allowance on the basis of written down value., Particulars, ₹, 1., Opening Written down value of Plant and Machinery (15% block) as on 01.04.2021, 5,78,000, 2., Purchase of second hand machinery (15% block) on 29.12.2021 for business purpose, and put to use on the same date, 2,00,000, 3., Machinery Y (15% block) purchased and installed on 12.07.2021 for the purpose of, power generation, 8,00,000, 4., Acquired and installed for use a new air pollution control equipment (40% block), on 31.07.2021, 2,50,000
Page 671 :
Income Under The Head Business/Profession, , 152, , 5., 6., , New air conditioner purchased and installed in office premises on 08.09.2021, 3,00,000, New machinery Z (15% block) acquired and installed on 23.11.2021 for the purpose, of generation of power, 3,25,000, 7., Sale value of an old machinery X, sold during the year, 3,10,000, Solution: Computation of depreciation allowance under section 32 for the A.Y. 2022-23, Particulars, Plant and Machinery Plant and Machinery, (15%), (40%), Opening WDV as on 01.04.2021, 5,78,000, Add: Plant and Machinery acquired during the year, - Second hand machinery, 2,00,000, - Machinery Y, 8,00,000, - Air conditioner for office, 3,00,000, - Machinery Z, 3,25,000, - Air pollution control equipment, 2,50,000, Less: Asset sold during the year, (3,10,000), Nil, Written down value before charging depreciation, 18,93,000, 2,50,000, Normal depreciation, 40% on air pollution control equipment, 1,00,000, Depreciation on plant and machinery put to use for less, than 180 days@ 7.5% (2,00,000 + 3,25,000), 39,375, 15% on ₹ 13,68,000, 2,05,200, Additional depreciation, - Machinery Y (₹ 8,00,000 × 20%), 1,60,000, - Machinery Z (₹ 3,25,000 × 10%), 32,500, Total depreciation, 4,37,075, 1,00,000, Hire purchase - In the case of assets under the hire purchase system the allowance for depreciation would, under Circular No. 9 of 1943 R. Dis. No. 27(4) I.T. 43 dated 23-3-1943, be granted as follows:, • In every case of payment purporting to be for hire purchase, production of the agreement under which, the payment is made would be insisted upon by the department., •, , Where the effect of an agreement is that the ownership of the asset is at once transferred on the lessee the, transaction should be regarded as one of purchase by instalments and consequently no deduction in, respect of the hire amount should be made. This principle will be applicable in a case where the lessor, obtains a right to sue for arrears of installments but has no right to recover the asset back from the lessee., Depreciation in such cases should be allowed to the lessee on the hire purchase price determined in, accordance with the terms of hire purchase agreement., , •, , Where the terms of an agreement provide that the asset shall eventually become the property of the hirer, or confer on the hirer an option to purchase an asset, the transaction should be regarded as one of hire, purchase. In such case, periodical payments made by the hirer should for all tax purposes be regarded as, made up of, (i) the consideration for hirer which will be allowed as a deduction in assessment, and, (ii) payment on account of the purchase price, to be treated as capital outlay and depreciation being, allowed to the lessee on the initial value namely, the amount for which the hired assets would have, been sold for cash at the date of the agreement., The allowance to be made in respect of the hire should be the amount of the difference between the, aggregate amount of the periodical payments under the agreement and the initial value as stated above., The amount of this allowance should be spread over the duration of the agreement evenly. If, however,, agreement is terminated either by outright purchase of the asset or by its return to the seller, the, deduction should cease as from the date of termination of agreement.
Page 672 :
Income Under The Head Business/Profession, , 153, , Question 10: Is it Mandatory to Claim Depreciation?, Answer: As per section 32, depreciation has to be claimed compulsorily i.e. it is not voluntary e.g. Mr. X, has income u/h house property ₹ 2.5 lakh and his depreciation in business profession is ₹ 2.5 lakh and Mr. X, refuses to claim depreciation, in this case he can not do so and he has to claim depreciation and also it has to, be set off from income of house property., Question 11 [V. Imp.]: Write a note on expenditure on Scientific Research., Answer: Expenditure on Scientific Research Section 35 (1)(i)/(iv), If any person has incurred expenditure whether revenue or capital in connection with scientific research, relating to business, such expenditure is allowed to be debited without any restriction however expenditure, incurred on land is not allowed. If the assessee has incurred expenditure on purchase/construction of, building, expenditure is allowed excluding the value of land., Example, ABC Ltd. engaged in manufacturing of cement has incurred ₹3 lakhs on scientific research, in this case,, expenditure is allowed, but if the research is not related to the business of the assessee, expenditure is not, allowed., Example, ABC Ltd. has purchased one plant and machinery on 01.07.2021 for the purpose of scientific research for, ₹30 lakhs, in this case, entire amount is allowed to be debited to the profit and loss account in the year 202122. But if the company has purchased land for the purpose of scientific research, expenditure is not allowed., Similarly if a building has been purchased for ₹40,00,000 and cost of land is ₹25,00,000, expenditure, allowed shall be ₹15,00,000., Expenditure before commencement of business, If expenditure is incurred before commencement of business but within 3 years prior to commencement,, capital expenditure is allowed without any limit in the year of commencement of business but revenue, expenditure is allowed only to the extent permitted by prescribed authority. Similarly payment of salary, except perquisite (facilities) are allowed only to the extent permitted by the prescribed authority., Example, ABC Ltd. has commenced its business on 01.07.2021, but before commencement, the company has incurred, revenue expenditure of ₹2 lakhs on scientific research from 01.07.2018 onwards and the prescribed authority, has certified expenditure of ₹1.5 lakhs, in this case ₹ 1.5 lakhs shall be allowed in the previous year 202122, but if any expenditure has been incurred prior to 01.07.2018, expenditure is not allowed., Donation/contribution to research association, If any assessee has given donation to the notified research association, assessee shall be allowed to debit the, amount to the profit and loss account in the manner given below:, (i) As per section 35(1)(ii), an amount equal to the amount of donation given to an approved scientific, research association or approved university, college etc. shall be allowed to be debited to profit and loss, account. E.g. ABC Ltd. has donated ₹10,00,000 to an approved research association for scientific research,, company is allowed to debit ₹10,00,000 to profit and loss account., (ii) As per section 35(1)(iia), if donation is given to an Indian company approved by prescribed authority, for the purpose of scientific research, deduction allowed shall be equal to the donation., (iii) As per section 35(1)(iii), deduction allowed shall be equal to the donation if donation is given to any, approved institution for the purpose of research in social science or statistical research., (iv) As per section 35(2AA), an amount equal to the amount of donation given to National Laboratory,, University or Indian Institute of Technology with a specific direction that the said sum shall be used, for scientific research undertaken. shall be allowed to be debited to profit and loss account., Further there is no condition that the research should be related to the business or profession of the, assessee., (If any assessee do not have business/profession, such assessee can claim deduction under section 80GGA.)
Page 673 :
Income Under The Head Business/Profession, , 154, , The Institution covered under section 35(1)(ii)/(iia)/(iii) shall give an intimation in prescribed manner to, prescribed Income Tax Authority within 3 months from the date when FA,2020 comes into force otherwise, their approval shall stand cancelled. Also any approval granted shall be valid for 5 years., MAY – 2011 (4 Marks), Mr. X has furnished the following particulars relating to payments made towards scientific research for the, year ended 31.03.2022:, (₹ in lacs), (i) Payments made to K Research Ltd., 20, (ii) Payment made to LMN College, 15, (iii) Payment made to OPQ College, 10, Note: K Research Ltd. and LMN College are approved research institutions and these, payments are to be used for the purposes of scientific research., (iv) Payment made to National Laboratory, 8, (v) Machinery purchased for in-house scientific research, 25, (vi) Salaries to research staff engaged in in-house scientific research, 12, Compute the amount of deduction available under section 35 of the Income-tax Act, 1961 while arriving at, the business income of the assessee., Answer., Computation of deduction allowable under section 35, Particulars, Amount, % of, Amount of, (₹in lacs), weighted, deduction, deduction, (₹ in lacs), Payment for scientific research, K Research Ltd., 20, 100%, 20.00, LMN College, 15, 100%, 15.00, OPQ College, 10, Nil, Nil, National Laboratory, 8, 100%, 8.00, In-house research, Capital expenditure, 25, 100%, 25.00, Revenue expenditure, 12, 100%, 12.00, Deduction allowable under section 35, 80.00, Sale of assets used for scientific research, Section 41(3), If any assessee has acquired any capital asset for scientific research and amount was debited to profit and, loss account and subsequently the asset was sold, amount received shall be considered to be income under, the head business/profession but only to the extent amount was debited to profit and loss account. If the, assessee has closed down his business/profession at that time, still it is income under the head, business/profession., Example, ABC Ltd. purchased one plant and machinery for ₹ 20 lakhs on 01.10.2020 for scientific research and entire, amount was debited to the Profit and loss account, subsequently the asset was sold for ₹ 23 lakhs in the year, 2021-22, in this case deemed income under section 41(3), shall be ₹20 lakhs i.e. the amount recovered on, sale maximum to the extent of the amount debited and excess over it shall be capital gain., Transfer of asset to the normal business, If any asset was used for scientific research and subsequently it was transferred to the normal business, in, such cases, it will be entered in the respective block of assets and its w.d.v shall be taken to be nil., NOV – 2007 (4 Marks), XY Bio-medicals Ltd. is engaged in the business of manufacture of bio-medical items. The following, expenses were incurred in respect of activities connected with scientific research:, Year ended, Item, Amount (₹)
Page 674 :
Income Under The Head Business/Profession, , 155, , 31.03.2019, Land, 10,00,000, (Incurred after 01.09.2018), Building, 25,00,000, 31.03.2020, Plant and machinery, 5,00,000, 31.03.2021, Raw materials, 2,20,000, The business was commenced on 01.09.2021 and expenditure incurred on raw materials and salaries is, ₹1,80,000, In view of availability of better model of plant and machinery, the existing plant and machinery were sold, for ₹ 8,00,000 on 01.03.2022., Discuss the implications of the above for the Assessment Year 2022-23 along with brief computation of, deduction permissible under section 35 assuming that necessary conditions have been fulfilled., Answer., The eligible expenditure and quantum of deduction will be:, (a) Current year capital or revenue expenditure incurred for scientific research., (b) Any expenditure incurred during earlier 3 years immediately preceding the date of commencement of, business on payment of salary or purchase of materials, or capital expenditure incurred other than, expenditure on acquisition of land (actual expenditure qualifies for deduction)., The deduction available under section 35 for scientific research will, therefore, be:, Particulars, ₹, (a), Land, Nil, (b), Building, 25,00,000, (c), Revenue expenses of last 3 years, 2,20,000, (d), Capital expenditure of last 3 years: Plant and machinery, 5,00,000, (e), Current year revenue expenditure ₹ 1,80,000, 1,80,000, Deduction under section 35, 34,00,000, Tax treatment on sale of Plant and Machinery, Section 41(3) provides that where a capital asset used for scientific research is sold, without having been, used for other purposes, the sale proceeds shall be considered to be income under the head, business/profession but only to the extent amount was debited to the profit and loss account hence there will, income under the head business/profession ₹5,00,000 and balance amount of ₹3,00,000 shall be considered, to be short term capital gain., Carry forward of unadjusted capital expenditure of scientific research, Un-adjusted capital expenditure of scientific research shall be allowed to be carried forward just like, unabsorbed depreciation i.e. carry forward shall be allowed for unlimited period and brought forward, expenditure can be adjusted from any income under any head except casual income., Illustration 7: X Ltd. was incorporated on 01.01.2021 for manufacture of tyres and tubes for motor, vehicles. The manufacturing unit was set up on 01.05.2021. The company commenced its manufacturing, operations on 01.06.2021. The total cost of the plant and machinery installed in the unit is ₹55 crore. The, said plant and machinery included second hand plant and machinery bought for ₹20 crore and new plant and, machinery for scientific research relating to the business of the assessee acquired at a cost of ₹ 15 crore., Compute the amount of depreciation allowable under section 32 of the Income-tax Act, 1961 in respect of, the assessment year 2022-23., Solution: Computation of depreciation allowable for the A.Y. 2022-23 in the hands of X Ltd., Particulars, ₹ in crore, Total cost of plant and machinery, 55.00, Less: Used for Scientific Research, (15.00), 40.00, Normal Depreciation at 15% on ₹ 40 crore, 6.00, Additional Depreciation: 20.00 x 20%, 4.00, No additional depreciation allowed on second hand machinery and also machinery used for, scientific research., Illustration 8: On 01.04.2021 ABC Ltd. owns plants A, B, C and D (rate of depreciation 15%) depreciated
Page 675 :
Income Under The Head Business/Profession, , 156, , value of the block as on 01.04.2021 is ₹5,40,000. On 14.06.2021 plant E which was initially purchased for, ₹96,000 for conducting scientific research is transferred from laboratory to the factory. No other asset is, purchased or sold. Find out depreciation allowed for the previous year 2021-22., Solution:, ₹, Computation of written down value as on 31st March 2022, Written down value as on April 1st, 2021, 5,40,000, th, Add: Cost of plant E transferred from laboratory to factory on June 14 , 2021, [i.e. ₹ 96,000–Deduction of ₹ 96,000 claimed under section 35], Nil, Written down value as on 31.03.2022 before charging depreciation, 5,40,000, Depreciation @ 15%, 81,000, Example, (i) ABC Ltd. has incurred ₹2,00,000 on purchase of plant and machinery for the purpose of scientific, research relating to his business, in this case entire expenditure can be debited to the profit and loss account, instead of permitting depreciation but if the research is not related to the business of the assessee,, expenditure is not allowed., (ii) ABC Ltd. has purchased one building for ₹50,00,000 out of which value of land is ₹40,00,000. The, building shall be used for the purpose of setting up a laboratory for the purpose of scientific research relating, to the business of the assessee, in this case company can debit ₹10,00,000 to the profit and loss account, being the cost of building. (Cost of land is not allowed), (iii) ABC Ltd. has commenced its business on 01.06.2021 and the company has incurred expenses before, commencement of business as given below:, (a) ₹5,00,000 during May 2018, being capital expenditure in connection with scientific research., (b) ₹3,00,000 during May 2020, being capital expenditure in connection with scientific research., (c) ₹1,00,000 during April 2018 on raw materials for scientific research., (d) ₹1,00,000 during June 2018 on raw materials for scientific research. (amount permitted by the prescribed, authority ₹75,000), (e) ₹40,000 in connection with perquisites given to the staff engaged in the scientific research., In this case, amount allowed to be debited shall be ₹3,75,000 (₹3,00,000 + ₹75,000) Expenditure incurred, before the period of 3 years is not allowed., (iv) ABC Ltd. has donated ₹1,00,000 to an approved scientific research association which is conducting, research not connecting to the business of the company, in this case, amount allowed to be debited shall be, ₹1,00,000., (v) ABC Ltd. has income under the head Business/Profession ₹3,00,000 before debiting capital expenditure, of ₹5,00,000 relating to scientific research, in this case, amount allowed to be debited shall be ₹3,00,000 and, unadjusted capital expenditure on scientific research shall be allowed to be set off and carried forward just, like unabsorbed depreciation., (vi) ABC Ltd. is engaged in manufacturing chemicals and its research programme has been approved by the, prescribed authority and the company has incurred the following expenses in connection with scientific, research., (a) Expenditure on purchasing a land ₹10,00,000., (b) Expenditure on construction of building on land ₹5,00,000., (c) Plant and machinery ₹10,00,000., (d) Raw materials ₹2,00,000., In this case, company shall be allowed weighted deduction of 100% of 17,00,000 i.e. ₹17,00,000., Illustration 9: A Ltd. which is engaged in manufacturing, furnishes the following particulars for the, P.Y.2021-22. Compute the deduction allowable under section 35 for A.Y.2022-23, while computing its, income under the head “Profits and gains of business or profession”., Particulars, ₹, 1., Amount paid to Indian Institute of Science, Bangalore, a notified research, 1,00,000, organisation for scientific research, 2., Amount paid to IIT, Delhi for an approved scientific research programme, 2,50,000, 3., Amount paid to X Ltd., a company registered in India which has as its main object, 4,00,000
Page 676 :
Income Under The Head Business/Profession, , 157, , scientific research and development, as is approved by the prescribed authority, 4., Expenditure incurred on in-house research and development facility as approved by, the prescribed authority, (a), Revenue expenditure on scientific research, (b), Capital expenditure (including cost of acquisition of land ₹ 5,00,000) on scientific, research, Solution: Computation of deduction under section 35 for the A.Y.2022-23, Particulars, ₹, Section, % of, deduction, Payment for scientific research, Indian Institute of Science, IIT, Delhi, X Ltd., Expenditure incurred on in-house research and, development facility, Revenue expenditure, Capital expenditure (excluding cost of acquisition, of land ₹ 5,00,000), Deduction allowable under section 35, , 3,00,000, 7,50,000, , Amount of, deduction, (₹), , 1,00,000, 2,50,000, 4,00,000, , 35(1)(ii), 35(2AA), 35(1)(iia), , 100%, 100%, 100%, , 1,00,000, 2,50,000, 4,00,000, , 3,00,000, 2,50,000, , 35(2AB), 35(2AB), , 100%, 100%, , 3,00,000, 2,50,000, 13,00,000, , MTP NOV – 2020 (4 Marks), Mr. Abhimanyu has furnished the following particulars relating to payments made and expenditure incurred, towards scientific research for the year ended 31.3.2022:, S.NO, Particulars, ₹ (in lakhs), (i), Payment made to AB University, an approved University, 15, (ii), Payment made to Siya College, 17, (iii), Payment made to IIT, Madras (under an approved programme for scientific, 12, research), (iv), Machinery purchased for in-house scientific research, 25, Compute the deduction available under section 35 of the Income-tax Act, 1961 for A.Y. 2022-23, while, computing his income under the head “Profits and gains of business or profession”., Answer:, Computation of deduction allowable under section 35, Particulars, Amount, Section, % of, Amount of, (₹ in lakhs), deduction, deduction, (₹ in lakhs), Payment for scientific research, AB University, an approved University, 15, 35(1)(ii), 100%, 15, Siya College [See Note 1], 17, –, NIL, NIL, IIT Madras (under an approved Programme for, 12, 35(2AA), 100%, 12, scientific research), In-house research, Capital expenditure – Purchase of Machinery, 25, 35(1)(iv), 100%, 25, r.w. 35(2), Deduction allowable under section 35, 52.00, Notes:-
Page 677 :
Income Under The Head Business/Profession, , 158, , 1. Payment to Siya College: Since the question clearly mentions that AB University (mentioned in item, (i)) is approved research institutions, it is logical to conclude that Siya College mentioned in item (ii) is, not an approved research institution. Therefore, payment to Siya College would not qualify for deduction, under section 35., Question 12: Write a note on specified business., Answer: Deduction in respect of expenditure on Specified Business Section 35AD, In case of certain business, the assessee shall, if he opts, be allowed to debit even the capital expenditure to, the profit and loss account and such business shall be called specified business and further amount allowed, to be debited shall be equal to the capital expenditure incurred and such business are as given below:, 01. Cold chain facility for storing agricultural produce, meat and meat products, poultry and dairy products, etc., 02. Warehousing facility for storage of agricultural produce., 03. Hospitals with at least one hundred beds for patients., 04. Housing project under a scheme for affordable housing., 05. Production of fertilizer including increase in installed capacity of an existing plant., 06. Pipeline network for distribution of natural gas or petroleum products., 07. Pipeline network for the transportation of iron ore., 08. Hotel of two star or above category., 09. Housing project for slum development., 10. Inland container depot or a container freight station., 11. Bee-keeping and production of honey., 12. Warehousing facility for storage of sugar., 13. Semi-conductor wafer fabrication manufacturing unit., 14. Developing or maintaining or operating a new infrastructure facility., The capital expenditure incurred before commencement of business shall also be allowed to be debited in, the year in which the business has commenced., The following capital expenditure shall not be allowed, • Acquisition of any land; or, • Goodwill; or, • Financial instrument, If any capital asset which was debited to profit and loss account, has been sold, amount received on sale, shall be considered to be income under the head business/profession as per section 28., If any capital asset was acquired for the said business and amount was debited to profit and loss account, it, must be used for the said business for a period of atleast 8 years otherwise the amount debited shall be, considered to be income of the assessee of the year in which the asset has been used for other purpose,, however normal depreciation shall be deducted and only balance amount shall be considered to be income., As per section 73A, loss of specified business can be set off only from profits and gains of any other, specified business and carried forward is allowed for unlimited periods and in the subsequent years also, the, loss can be set off only from income of specified business., Loss from normal business and unadjusted depreciation can be adjusted from income of specified business., Similarly loss under the head house property or loss under the head other sources can be setoff from income, of specified business., Capital Expenditure shall not include any expenditure in respect of which the payment or aggregate of, payments made to a person in a day, otherwise than by an account payee cheque drawn on a bank or an, account payee bank draft or use of electronic clearing system through a bank account or Credit Card,, Debit Card, Net Banking, IMPS (Immediate Payment Service), UPI (Unified Payment Interface), RTGS, (Real Time Gross Settlement), NEFT (National Electronic Funds Transfer), and BHIM (Bharat, Interface for Money) Aadhaar Pay, exceeds ₹10,000.
Page 678 :
Income Under The Head Business/Profession, , 159, , No deduction shall be allowed to the assessee under any other section in any previous year or under this, section if the deduction has been claimed or opted by the assessee and allowed to him under this section., Illustration 10: An Assessee starts business of setting up and operating a warehousing facility for, agricultural produce on 01.06.2021. Following information is given to you:, (i) Profits from operating warehousing facility, ₹50,00,000, The following assets have been purchased for warehousing facility and the profit of ₹50,00,000 is computed, without giving effect to the following:, (i) Machinery purchased on 30.06.2020, ₹ 9,00,000, (ii) Land purchased on 30.06.2020, ₹15,00,000, (iii) Machinery purchased on 31.12.2021, ₹12,00,000, (iv) Building purchased on 31.12.2020, ₹10,00,000, (v) Building Constructed on 31.05.2021, ₹29,00,000, Compute income or loss for the assessment year 2022-23., Solution:, Profit of specified business, 50,00,000, Less: Deduction allowable under section 35AD, Machinery purchased on 30.06.2020, (9,00,000), Machinery purchased on 31.12.2021, (12,00,000), Building purchased on 31.12.2020, (10,00,000), Building constructed on 31.05.2021, (29,00,000), Loss, (10,00,000), The loss of ₹10,00,000 can be set-off only against the profits of specified business in current year and next, years. Such loss can be carried forward indefinitely., Illustration 11: Mr. X commenced operations of the businesses of setting up a warehousing facility for, storage of food grains, sugar and edible oil on 01.04.2021. He incurred capital expenditure of ₹80 lakh, ₹60, lakh and ₹50 lakh, respectively, on purchase of land and building during the period January, 2021 to March,, 2021 exclusively for the above businesses, and capitalized the same in its books of account as on 1st April,, 2021. The cost of land included in the above figures are ₹50 lakh, ₹40 lakh and ₹30 lakh, respectively., Further, during the P.Y.2021-22, it incurred capital expenditure of ₹20 lakh, ₹15 lakh & ₹10 lakh,, respectively, for extension/ reconstruction of the building purchased and used exclusively for the above, businesses. Compute the income under the head “Profits and gains of business or profession” for the, A.Y.2022-23 and the loss to be carried forward, assuming that Mr. X has fulfilled all the conditions, specified for claim of deduction under section 35AD. The profits from the business of setting up a, warehousing facility (before claiming deduction under section 35AD and section 32) for the A.Y. 2022-23 is, ₹16 lakhs, ₹14 lakhs and ₹31 lakhs, respectively., Solution:, Particulars, ₹ (in lakhs), 1. Profit from business of setting up of warehouse for storage of food grains, 16, Less: 80 lakhs – 50 lakhs + 20 lakhs = 50 lakhs, (50), Loss from business, (34), 2. Profit from business of setting up of warehouse for storage of sugar, 14, Less: 60 lakhs – 40 lakhs + 15 lakhs = 35 lakhs, (35), Loss from business, (21), 3. Profit from business of setting up of warehouse for storage of edible oil, 31, Less: Depreciation under section 32, 10% of ₹ 30 lakh, being (₹ 50 lakh – ₹ 30 lakh + ₹ 10 lakh), (3), Income from business, 28, Loss from specified business shall not be allowed to set off from incomes of other business, Illustration 12: XYZ Ltd. commenced operations of the business of a new three-star hotel in Madurai,, Tamil Nadu on 01.04.2021. The company incurred capital expenditure of ₹50 lakh during the period, January, 2021 to March, 2021 exclusively for the above business, and capitalized the same in its books of, account as on 1st April, 2021. Further, during the P.Y.2021-22, it incurred capital expenditure of ₹2 crore
Page 679 :
Income Under The Head Business/Profession, , 160, , (out of which ₹1.50 crore was for acquisition of land) exclusively for the above business. Compute the, income under the head “Profits and gains of business or profession” for the A.Y.2022-23, assuming that, XYZ Ltd. has fulfilled all the conditions specified for claim of deduction under section 35AD. The profits, from the business of running this hotel (before claiming deduction under section 35AD) for the A.Y.202223 is ₹25 lakhs., Assume that the company also has another existing business (specified business) of running a four-star hotel, in Coimbatore, which commenced operations 2 years back, the profits from which are ₹ 120 lakhs for the, A.Y.2022-23., Solution:, Particulars, ₹, Profits from the specified business of new hotel in Madurai, 25 lakh, Less:, 1. Capital expenditure incurred during the P.Y.2021-22 (excluding the, expenditure incurred on acquisition of land) = ₹ 200 lakh –₹ 150 lakh, (50 lakh), 2. Capital expenditure incurred prior to 01.04.2021, (50 lakh), Loss from the specified business of new hotel in Madurai, (75 lakh), Profit from the existing business of running a hotel in Coimbatore, 120 lakh, Net profit from business after set-off of loss of specified business against profits of another specified, business under section 73A, 45 lakh, NOV – 2020 (8 Marks), Question 3 (a), Ms. Pooja a resident individual provides the following information of her income / losses for the year, ended on 31st March, 2022:, S. No., Particulars, (₹), 1, Income from salary (computed), 2,20,000, 2, Income from house property (let out) (Net Annual Value), 1,50,000, 3, Share of loss from firm in which she is a partner, 10,000, 4, Loss from specified business covered under section 35AD, 20,000, 3,00,000, 5, Income from textile business before adjusting the following items:, 60,000, (a) Current year depreciation, 2,25,000, (b) Unabsorbed depreciation of earlier year, 90,000, (c) Brought forward loss of textile business of the A.Y. 2020-21, 6, Long term capital gains on sale of debentures, 75,000, 7, Long term capital gains on sale of equity shares (STT not paid), 1,00,000, 8, Long term capital gains on sale of equity shares listed in a recognized stock, 1,50,000, exchange (STT paid at the time of acquisition and sale), 9, Dividend from units of UTI, 5,000, During the previous year 2021-22, Ms. Pooja has repaid ₹5,25,000 towards housing loan from a scheduled, bank. Out of this ₹3,16,000 was towards payment of interest and rest towards principal., Compute Gross total income of Ms. Pooja and ascertain the amount of loss that can be carried forward. Ms., Pooja has always filed her return within the due date specified under section 139(1) of the Income tax, Act,1961., Solution:, Computation of Gross Total Income of Ms. Pooja, Income under the head salary, Income from salary (computed), 2,20,000, Less: Setoff of House property loss section 71, (2,00,000), Income under the head salary, 20,000
Page 680 :
Income Under The Head Business/Profession, Income under the head House property, Net Annual Value (NAV given in question), Less: Standard deductions @ 30% u/s 24(a), Less: Interest on capital borrowed u/s 24(b), Loss under the head house property, (as per section 71, Maximum setoff allowed from other heads is 2,00,000, and balance 11,000 shall be carried forward as per section 71B), Income under the head business/ profession, Income from textile business, Less: current year depreciation, Less: brought forward loss of textile business, Less: unabsorbed depreciation section 32(2), (balance 75,000 shall be adjusted from capital gains), Income under the head business/profession, Income under the head capital gains, Long term capital gains on sale of debentures, Long term capital gains on sale of equity shares (STT paid), Less: Setoff of long term capital loss on sale of shares (STT not paid) (section 70), Less: Unabsorbed depreciation section 32(2), Income under the head capital gains, Income under the head other sources, Dividend from units of UTI, , 161, , 1,50,000, (45,000), (3,16,000), (2,11,000), , 3,00,000, (60,000), (90,000), (1,50,000), Nil, 75,000, 1,50,000, (1,00,000), (75,000), 50,000, 5,000, , Gross Total Income, , 75,000, , Carried forward of losses, 1. Loss of house property, 2. Loss from specified business u/s 35AD, (Shall be allowed to be adjusted from specified business income only), , 11,000, 20,000, , MAY – 2011 (4 Marks), ABC Limited commenced the business of operating a three star hotel in Tirupati on 01.04.2021. It furnishes, you the following information:, (i) Cost of land (acquired in June 2019), ₹60 lakhs, (ii) Cost of construction of hotel building, Financial year 2019-20, ₹30 lakhs, Financial year 2020-21, ₹150 lakhs, (iii) Plant and machineries (all new), Acquired during financial year 2020-21, ₹30 lakhs, [All the above expenditures were capitalized in the books of the company], Net profit before depreciation for the financial year 2021-22, ₹80 lakhs, Determine the amount eligible for the deduction under section 35AD of the Income-tax Act, 1961, for the, Assessment Year 2022-23., Solution:, In this case business of operating of three star hotel is a specified business and 100% of capital expenditure, shall be allowed to be debited to the profit and loss account including expenditure incurred before, commencement of business but expenditure on land is not allowed and computation of income shall be as, given below:, Profits before debiting the expenses, ₹80,00,000
Page 681 :
Income Under The Head Business/Profession, , 162, , Cost of land (not eligible for deduction under section 35AD), Nil, Cost of construction of hotel building (₹30 lakhs + ₹150 lakhs), (180,00,000), Cost of plant and machinery, (30,00,000), Loss from specified business, (130,00,000), As per section 73A such loss can be set off from income of any other specified business and its carry, forward is allowed for unlimited period and even in the subsequent years it can be set off from income of, specified business., NOV – 2011 (4 Marks), State any four of the specified business eligible for deduction under section 35AD of the Income-tax Act,, 1961., Solution: Refer answer given in the chapter, MAY – 2012 (4 Marks), MNP Ltd. Commenced operations of the business of a new four-star hotel in Chennai on 01.04.2021. The, company incurred capital expenditure of ₹40 lakh during the period January, 2021 to March, 2021, exclusively for the above business, and capitalized the same in its books of account as on 1st April, 2021., Further, during the Previous Year 2021-22, it incurred capital expenditure of ₹ 2.5 crore (out of which ₹ 1, crore was for acquisition of land) exclusively for the above business. Compute the income under the heading, “profit and gains of business or profession” for the assessment year 2022-23, assuming that MNP Ltd. has, fulfilled all the conditions specified for claim of deduction under section 35AD. The profits from the, business of running this hotel (before claiming deduction under section 35AD) for the assessment year, 2022-23 is ₹ 80 lakhs., Assume that the company also has another existing business (specified business) of running a four-star hotel, in Kanpur, which commenced operations 2 years back, the profits from which was ₹130 lakhs for, Assessment Year 2022-23., Answer:, Deduction allowable under section 35AD, ₹, Profit of specified business, 80,00,000, Less: Capital Expenditure before commencement, (40,00,000), Less: Capital Expenditure during the year (250 lakhs – 100 lakhs), (150,00,000), Loss from specified business of new hotel in Chennai, (110,00,000), Loss can be set off from income of other specified business and net income shall be, 130 lakhs – 110 lakhs, 20,00,000, NOV – 2014 (4 Marks), Name any four specified businesses covered under section 35AD and state the fiscal incentives available to, such businesses., Refer answer given in the Chapter, Question 13 [Imp.]: Explain briefly the provisions of amortisation of Preliminary Expenses., Answer: Amortisation of certain Preliminary Expenses Section 35D, Expenditure incurred before commencement of business shall be called preliminary expenses and shall be, allowed to be debited in 5 annual equal installments after commencement of business and such expenses are, allowed to an Indian company and also to resident assessee i.e. it is not allowed to non-residents and to, foreign company., Only the notified expenditure incurred before commencement of business shall be allowed and such, expenses may be, 1. Expenditure in connection with—, (i) preparation of feasibility report., (ii) preparation of project report., (iii) conducting market survey or any other survey necessary for the business of the assessee., (iv) engineering services relating to the business of the assessee., Provided that the work in connection with the above is carried out by the assessee himself or by a concern, which is approved by the Central Board of Direct Taxes.
Page 682 :
Income Under The Head Business/Profession, , 163, , 2. Legal charges for drafting any agreement between the assessee and any other person for purpose of the, business of the assessee., 3. Where the assessee is a company, also expenditure—, (i) by way of legal charges for drafting the Memorandum and Articles of Association of the company., (ii) on printing of the Memorandum and Articles of Association., (iii)by way of fees for registering the company under the provisions of the Companies Act., (iv) in connection with the issue of shares or debentures of the company, being underwriting commission,, brokerage and charges for drafting, typing, printing and advertisement of the prospectus., Maximum expenditure allowed shall be upto 5% of the project cost but an Indian company has the option, to take 5% of the capital employed., Example, ABC Ltd. has incurred expenditure of ₹30,00,000 and its project cost is ₹100,00,000 and capital employed is, ₹120,00,000, instalment allowed to the company shall be, ₹30,00,000 but subject to a maximum of (120,00,000 x 5%) i.e. ₹6,00,000, Instalment allowed shall be = 6,00,000 / 5 = ₹1,20,000, “Cost of the project” means, in a case of new business, the actual cost of the fixed assets, being land, buildings, plant, machinery,, furniture, fittings etc. as on the last day of the year in which the assessee has commenced the business., “Capital employed” means, in a case of new business, the aggregate of the issued share capital, debentures and long-term borrowings as, on the last day of the previous year in which the business of the company commences., In case of an existing business if there is extension of such business, expenses incurred in connection with, such extension shall also be allowed in the similar manner as in case of new business and project cost and, capital employed shall be taken into consideration relating to extension of business., Illustration 13: ABC Ltd. an Indian company has incurred expenditure before the commencement of, business asunder:, 1. Expenditure on advertisements ₹3 lakhs., 2. Expenditure on preparation of project report and the report was prepared by a concern which is, approved by the Board ₹85,000., 3. Expenditure in connection with travelling and stay in hotels ₹45,000., 4. Expenditure on drafting and printing of memorandum and articles of associations ₹4 lakhs., All the above expenditures have been debited to the profit and loss account and the company has computed, income to be ₹ 7 lakh., The company has commenced its business on 01.06.2021., Company’s project cost is ₹50 lakhs and capital employed is 57 lakhs., Compute company’s Tax Liability for Assessment Year 2022-23., Solution:, ₹, Net profit as per profit and loss account, 7,00,000, Add:, • Expenditure on advertisement, 3,00,000, • Expenditure in connection with travelling and stay in hotels, 45,000, • Excessive expenditure under section 35D, 4,28,000, Working Note:, ₹, Eligible expenditure under section 35D, 1. Expenditure on preparation of project report, 85,000, 2. Expenditure on drafting and printing of memorandum and articles, of associations, 4,00,000, Total =, 4,85,000, Expenditure allowed under section 35D can not exceed 5% of the capital employed, 57,00,000 x 5% = 2,85,000, Instalment allowed 2,85,000/5 = 57,000, Expenditure disallowed = 4,85,000 – 57,000 = 4,28,000, Income under the head Business/Profession, 14,73,000
Page 683 :
Income Under The Head Business/Profession, , 164, , Gross Total Income, 14,73,000, Less: Deduction u/s 80C to 80U, Nil, Total Income, 14,73,000, Computation of Tax Liability, Tax on ₹14,73,000 @ 30%, 4,41,900, Add: HEC @ 4%, 17,636, Tax Liability, 4,59,576, Rounded off u/s 288B, 4,59,580, Question 14: Write a note on amortization of expenditure under Voluntary Retirement Scheme., Answer: Amortisation of expenditure incurred under Voluntary Retirement Scheme Section 35DDA, If any employer has given voluntary retirement to the employees and has paid any amount in connection, with such voluntary retirement, it will be allowed to be debited in 5 annual equal installments e.g. ABC Ltd., has given voluntary retirement to 500 employees and has paid ₹4,00,000 to each of the employee and total, payment made is ₹2000 lakhs, in this case expenditure is allowed in 5 annual equal installments., MAY – 2011 (4 Marks), X Co. Ltd. paid ₹120 lakhs as compensation as per approved Voluntary Retirement Scheme (VRS) during, the financial year 2022-23., How much is deductible under section 35DDA for the assessment year 2022-23?, Answer., It is deductible in 5 equal annual instalments commencing from the previous year of payment. ₹24 lakhs,, being 1/5th of ₹ 120 lakhs, is deductible under section 35DDA for the A.Y.2022-23., Question 15: Explain deductibility of Insurance Premium., Answer: Deductibility of insurance premium, Payment of premium for the insurance of stocks Section 36(1)(i), If any assessee has paid premium for insurance of raw material or finished goods etc., such premium is, allowed to be debited to profit and loss account., Payment of premium in connection with medi claim policy Section 36(1)(ib), If any assessee has paid premium for medi claim policy taken in the name of employees, such premium is, allowed to be debited to profit and loss account provided premium was paid otherwise than in cash., Question 16 [Imp.]: Write a note on payment of interest., Answer: Payment of Interest Section 36(1)(iii), If any assessee has taken a loan for the purpose of business/profession, interest on such loan is allowed., No interest is allowed to the proprietor on his capital (similarly no salary or any other payment is allowed to, the proprietor)., Question 17: Explain deductibility of employer’s contribution towards Recognised Provident Fund, etc., Answer: Employer’s contribution to Recognised Provident Fund or Approved Superannuation Fund, Section 36(1)(iv), Employer contribution to Recognised Provident Fund and Approved Superannuation Fund shall be, allowed to be debited only to the extent it has been permitted in the relevant Act / Rule. E.g. Employer, contribution to recognized provident fund is allowed maximum @ 12% of employees salary., Employer’s contribution towards a Pension Scheme Section 36(1)(iva), Employer contribution to notified pension scheme as per section 80CCD shall be allowed maximum to the, extent of 10% of salary of employee. E.g. If salary of employee is ₹2,00,000, employer can contribute, maximum ₹20,000., (shall be discussed under the head Salary), Employer’s contribution towards approved Gratuity Fund Section 36(1)(v), Employer contribution to approved gratuity fund shall be allowed to be debited to the extent allowed in the
Page 684 :
Income Under The Head Business/Profession, , 165, , relevant Act / Rule., Question 18: Explain deductibility of employee’s contribution received by the employer., Answer: As per section 36(1)(va), employees contribution shall be allowed to be debited only if employer, has deposited the amount in the relevant account upto the time allowed in the relevant Act., As per paragraph 38 of The Employees’ Provident Funds Scheme, 1952, the employer should pay, within 15 days of the subsequent month., As per section 31 of Employees' State Insurance (General) Regulations, 1950, ESI contribution should, be deposited maximum upto 15th of subsequent month., Question 19 [Imp.]: Write note on deduction of Bad Debts of a Business., Answer: Deduction for Bad Debts of a Business Section 36(1)(vii), If any assessee has written off bad debts as irrecoverable in the books of accounts, he will be allowed to, debit such bad debts to the profit and loss account. However provision for bad debts is not allowed (in, general provision or reserve for any purpose is not allowed.), Recovery of bad debts Section 41(4), If any amount was debited as bad debts and subsequently it was recovered by the assessee, it will be, considered to the income of the assessee under the head business /profession of the year in which it has been, recovered and if the assessee do not have business or profession in that particular year, even than it will be, considered to be income under the head business/profession. If debt was incurred by a person but it was, recovered by his successor, in that case it will not be considered to be income of successor., Example, Mr. X debited bad debts ₹ 4,00,000 in previous year 17-18 but recovered ₹ 1,00,000 in previous year 21-22,, in this case as per section 41(4) it will be considered to be income under the head business profession of, previous year 21-22. If amount was debited by his father and after his death, his son has inherited the, business and has recovered ₹ 1,00,000, it will not be considered to be income of son i.e. successor., Question 20 [Imp.]: Write a brief note on deductibility of Family Planning Expenditure under section, 36(1)(ix) of the Income Tax Act, 1961., Answer: If any company has incurred expenditure in connection with promotion of family planning norms, among the employees, the assessee shall be allowed to debit the expenditure to profit and loss account., Revenue expenditure can be debited in the same year and capital expenditure shall be allowed in 5 annual, equal installment. Any other assessee is not allowed to debit any expenditure in connection with promotion, of family planning., Expenditure are allowed to debited only to the extent income is available under the head business/profession, and unadjusted expenditure shall be allowed to be set off and carried forward just like unabsorbed, depreciation., Question 21: Write a note on Securities Transaction Tax., Answer: Securities Transaction Tax Section 36(1)(xv), If the assessee has paid securities transaction tax in connection with taxable securities transaction which are, part of his business, STT shall be allowed to be debited to the profit and loss account. If it is a case of capital, gain, it will not be allowed to be deducted e.g. Mr. X purchased shares of ₹ 4 lakh and sold for ₹ 10 lakh, after 6 months and paid STT ₹ 1000 in this case capital gains u/s 111A shall be ₹ 6 lakh and shall be taxable, @ 15% and if shares were sold after 1 year it will be long term capital gain u/s 112A and shall be taxable @, 10% and amount of capital gains shall be ₹ 6 lakh. If he has business of sale purchase of shares, STT shall, be deducted and capital gain shall be ₹ 5,99,000 and shall be taxable at the normal rate., Question 22: Write a note on Commodities Transaction Tax., Answer: Commodities Transaction Tax Section 36(1)(xvi), If the assessee has paid commodities transaction tax in connection with taxable commodities transaction, which are part of his business, CTT shall be allowed to be debited to the profit and loss account. Such, business is considered to be speculative business.
Page 685 :
Income Under The Head Business/Profession, , 166, , Question 23 [V. Imp.]: State the conditions to be satisfied for claiming deduction under section 37(1), of the Act., Answer: As per section 37(1), if any expenditure is neither allowed nor disallowed specifically under any, particular section, such expenditure is allowed to be debited if it is related to business or profession and is, revenue in nature. If it is capital expenditure, depreciation is allowed. Personal expenditure is never allowed., Illegal expense is not allowed. Any fine or penalty for an offence is not allowed., Various expenditure which may be allowed under section 37(1) are as given below:, 1. Expenditure in connection with advertisement e.g. ABC Ltd. has incurred ₹20,000 on printing of diaries, and calendars, the expenditure is allowed. Similarly if expenditure has been incurred on advertisement in, newspaper/magazine/ radio / TV / Internet etc., it will be allowed. If the expenditure incurred is capital, nature, depreciation is allowed., 2. Expenditure on travelling including the expenses of boarding and lodging in connection with, business/profession., 3. Salary paid to the employees., 4. Expenditure in connection with entertainment/amusement of the employees or the customers., 5. Expenditure in connection with opening ceremony (Mahurat) of the business/profession. E.g. ABC Ltd., has incurred ₹50,000 in connection with ‘shamiana’ and refreshments on occasion of opening ceremony., 6. Expenditure on the occasion of various festivals like Diwali etc. for employees or customers., 7. Incentives given to the articled assistant by a Chartered Accountant., 8. Interest on late payment of GST., 9. Expenditure in connection with legal proceedings., 10. Professional tax paid by a person carrying on business or profession., 11. Expenditure on the filing of return of income, filing of appeal or audit fee etc. is allowed., 12. Any other expenditure which is revenue in nature and it is related to business or profession., Any expenditure incurred by an assessee on the activities relating to corporate social responsibility, referred to in section 135 of the Companies Act, 2013 shall not be deemed to be an expenditure, incurred by the assessee for the purposes of the business or profession., NOV – 2007 (2 Marks), Mr. X, a hotelier, claimed expenditure on replacement of Linen and carpets in his hotel as revenue, expenditure., Answer. The expenditure on replacement of linen and carpets in a hotel are in the nature of expenses, incurred for the business and are allowable as revenue expenses under section 37(1)., Question 24: Write a note on deductibility of expenditures in connection with advertisement in the, newspaper etc. of a Political Party., Answer: Expenditure in connection with advertisement in the newspaper etc. of a Political Party, Section 37(2B), No allowance shall be made in respect of expenditure incurred by an assessee on advertisement in any, souvenir, brochure, tract, pamphlet or the like published by a political party., Illustration 14: ABC limited is a company engaged in the business of biotechnology. The net profit of the, company for the financial year ended 31.03.2022 is ₹15,25,890 after debiting the following items:, 1. Purchase price of raw material used for the purpose of in-house research and development, 1,80,000, 2. Purchase price of asset used for in-house research and development, (1) Land, 5,00,000, (2) Building, 3,00,000, 3. Expenditure incurred on advertisement in the souvenir published by a political party, 75,000, Compute the income under the head “Profits and gains of business or profession” for the A.Y. 2022-23 of, ABC Ltd., Solution: Computation of income under the head “Profits and gains of business or profession” for the, A.Y.2022-23, Net profit as per profit and loss account, 15,25,890, Add: Purchase price of Land used in in-house research and development, 5,00,000
Page 686 :
Income Under The Head Business/Profession, , 167, , Add: Expenditure incurred on advertisement in the souvenir published by a political party, not allowed as deduction as per section 37(2B), 75,000, Profits and gains of business/profession, 21,00,890, Note: The expenditure incurred on advertisement in the souvenir published by a political party is disallowed, as per section 37(2B) while computing income under the head “Profit and Gains of Business or Profession”., Question 25 [Imp.]: Explain deductibility of expenditure in connection with assets which are partly in, business use and partly in personal use section 38., Answer: Expenditure in connection with assets which are partly in business use and partly in personal, use Section 38, If any person has any asset in business or profession as well as in personal use, expenditure is allowed only, to the extent the asset is in the use of the business or profession., Example, If Mr. X has one motor car which is used to the extent of 60% in business and 40% for personal use, all, expenditures shall be allowed to be debited to the extent of 60%., Question 26 [V. Imp.]: Write a note on deductibility of expenditure on which tax has not been, deducted at source., Answer: Tax deduction at source for payment of interest, royalty etc. outside India Section 40(a)(i), If any person has paid any interest, royalty or technical fee or other sum chargeable to income tax and the, amount is being paid outside India or it has been paid in India to a non-resident or to any foreign company,, amount shall be allowed to be debited only if tax has been deducted at source upto the end of the year and, also tax has been paid to the government upto the last date of filing of return of income otherwise, expenditure is disallowed however it is allowed in the year in which tax has been paid to the government., (As per section 195, every person has to deduct tax at source on every payment made outside India), Example: ABC Ltd. has paid ₹20 lakhs as interest outside India on 03.01.2022 and tax was deducted at, source on 10.03.2022 and it was paid to the Government on 31.10.2022, expenditure is allowed in previous, year 2021-22 but if tax was deducted at source on 01.04.2022, expenditure is not allowed in previous year, 2021-22 but it is allowed in previous year 2022-23. If tax was deducted at source on 10.03.2022 but it was, paid to the Government on 01.11.2022, expenditure is not allowed in previous year 2021-22 however it is, allowed in previous year 2022-23., Tax deduction at source for payment of interest, commission, brokerage etc. in India, Section, 40(a)(ia), If any person has to pay any sum to a resident on which tax is to be deducted at source, in that case such, person must deduct tax at source upto the end of the relevant previous year and also payment should be, made upto the last date of ROI otherwise 30% of such expenditure is disallowed however it is allowed in the, year in which tax has been paid to the government., Example: ABC Ltd. has paid rent of ₹10 lakhs to XYZ Ltd. in India on 31.12.2021 and tax was deducted at, source on 31.03.2022 and was paid to the Government on 31.10.2022, expenditure is allowed in previous, year 2021-22 but if tax is deducted at source after 31.03.2022 or payment is made to the Government after, 31.10.2022, expenditure allowed in previous year 2021-22 shall be 10,00,000 x 70% = 7,00,000 and balance, ₹3,00,000 is disallowed however it will be allowed in the year in which tax has been paid to the, Government., If any person has not deducted tax at source but person who received the payment has paid tax and filed, return and it is confirmed by the Chartered Accountant, in that case it will be presumed that such person has, deducted and paid tax on the date of filing of ROI by the person receiving payment. (applicable in section, 40(a)(i)/40(a)(ia)), Example: ABC Ltd. has paid rent of ₹10 lakhs to XYZ Ltd. in India on 31.12.2021 and company has not, deducted tax at source but XYZ Ltd. has deposited the tax and filed return on 31.10.2022, in this case it will, be presumed that tax has been deducted on 31.10.2022 and paid to the Government on 31.10.2022 and 70%, expenditure shall be allowed to ABC Ltd. in previous year 2021-22 and balance 30% in previous year 202223., e.g. ABC Ltd. has paid ₹1,00,000 to XYZ Ltd. being the amount of rent and no tax has been deducted at
Page 687 :
Income Under The Head Business/Profession, , 168, , source, in this case expenditure is allowed because as per section 194-I, TDS is not applicable if rent payable, is upto ₹2,40,000., MAY – 2011 (4 Marks), During the financial year 2021-22, the following payments/expenditure were made/incurred by Mr. X, a, resident individual (whose turnover during the year ended 31.03.2021 was ₹99 lacs):, (i), Interest of ₹12,000 was paid to ABC & Co., a resident partnership firm, without deduction of tax at, source;, (ii), Interest of ₹4,000 was paid as interest to Mr. Y, a non-resident, without deduction of tax at source;, (iii), ₹3,00,000 was paid as salary to a resident individual without deduction of tax at source;, (iv), He had sold goods worth ₹5 lacs to Mr. D. He paid Commission of ₹15,000 to Mr. Z on 02.07.2021., In none of these transactions, tax was deducted at source., Briefly discuss whether any disallowance arises under the provisions of section 40(a) of the Income tax Act,, 1961., Answer., (i) Since Turnover of Mr. X is less than 100 lakhs in the preceding year hence expenditure is allowed even if, tax has not been deducted at source., (ii) In the case of interest paid to a non-resident, there is obligation to deduct tax at source under section 195,, hence non-deduction of tax at source will attract disallowance., (iii) Disallowance under section 40(a) is attracted for failure to deduct tax at source under section 192 from, salaries. 70% of salary i.e. ₹3,00,000 x 70% = ₹2,10,000 shall be allowed in previous year 2021-22 and, balance i.e. ₹90,000 is disallowed., (iv) Since Turnover of Mr. X is less than 100 lakhs in the preceding year hence expenditure is allowed even, if tax has not been deducted at source., NOV – 2010 (5 Marks), M/s ABC Ltd., submits the following details of expenditures pertaining to the financial year 2021-22:, (i), Payment of professional fees to Mr. M ₹50,000. Tax not deducted at source., (ii), Interior works done by Mr. H for ₹2,00,000 on a contract basis. Payment made in the month of, March 2022. Tax deducted in March 2022, was paid on 30.06.2022., (iii) Factory Rent paid to Mrs. R ₹15,00,000. Tax deducted at source and paid on 01.11.2022., (iv) Interest paid on Fixed Deposits ₹2,00,000. Tax deducted on 31.12.2021 and paid on 28.09.2022., (v), Payment made to M/s G & Co. towards import of Raw Materials ₹25,00,000. No tax was deducted at, source. The supplier G & Co. is located in London., Examine the above with reference to allowability of the same in the Assessment Year 2022-23 under the, Income Tax Act, 1961. Your answer must be with reference to Section 40(a) read with relevant tax, deduction at source provisions., Solution:, Allowability of expenses of M/s. ABC Ltd for the A.Y. 2022-23, (i) Payment of professional fees is subject to TDS under section 194J. Since no tax is deducted at source, the, expenditure of ₹15,000 (i.e. 30% of ₹50,000) is disallowed under section 40(a) and balance 70% is allowed., (ii) Since the tax was deducted in March, 2022 and paid on or before the due date of filing the return (i.e., on, or before October 31st, 2022), the expenditure on interior works will be allowed as deduction., (iii) The maximum time allowable for deposit of tax deducted at source is upto the due date of filing of, return i.e., October 31st, 2022. In this case, since tax deducted under section 194-I was paid after the due, date of filing the return, the expenditure of ₹4,50,000 (30% of ₹15,00,000) is disallowed in previous year, 2020-21 and balance 70% is allowed in previous year 2021-22., (iv) Since the tax was deducted in December, 2021 and paid on or before the due date of filing the return, (i.e., on or before October 31st, 2022), the interest paid on fixed deposits will be allowed as deduction., (v) Since payment towards import of raw materials does not attract the provisions of deduction of tax at, source, the expenditure will be allowed as deduction.
Page 688 :
Income Under The Head Business/Profession, , 169, , Question 27 [V. Imp.]: Explain deductibility of Income Tax or Securities Transaction Tax., Answer: Deductibility of Income Tax or Securities Transaction Tax, Under section 40(a)(ii) and (iia), payment of income tax is not allowed because it is not considered to be, liability of business/profession, however as per section 43B, GST, composition tax under GST, Municipal, tax, professional tax, licence fee, etc. is allowed., If interest has been paid for late payment of income tax or additional income tax, such interest is not allowed, but if interest has been paid for late payment of GST etc., interest is allowed., If any interest has been paid for the loan taken for payment of income tax, interest is not allowed., If interest has been paid for the loan taken for the payment of GST etc., interest is allowed under section, 36(1)(iii)., If there is any income tax refund, it will not to be considered income but if there is refund of GST etc., it, will be considered to be income as per section 41(1)., If any interest has been received in connection with income tax refund or GST refund etc., it will be, considered to be income. Interest on Income Tax Refund shall be taxable under the head other sources but, interest on refund of GST etc. shall be taxable under the head business/profession., If any person has paid any fine or penalty in connection with income tax, GST etc., it will not be allowed., Under section 36(1)(xv), securities transaction tax shall be allowed to be debited., If any person has sold equity shares or units of equity oriented mutual fund and has paid STT and the asset is, long term, capital gain shall be taxable in excess of ₹1,00,000 u/s 112A and if it is short term, capital gains, shall be taxable @ 15% under section 111A and assessee shall not be allowed to deduct STT paid by him as, selling expense. If any person has business of sale purchase of shares or units and has paid STT, while, computing income under the head Business/Profession, assessee shall be allowed to debit STT while, computing income and such income shall be taxable at the normal rate., Under section 36(1)(xvi), commodities transaction tax shall be allowed to be debited., If any assessee has income from commodities transaction, assessee shall be allowed to debit CTT to Profit, and loss account and income shall be taxable at the normal rate., If employer has paid income tax on behalf of the employee, employer is allowed to debit such amount to, profit and loss account and it will also be considered to be income of the employee., e.g. Mr. X is employed in ABC Ltd. and is getting salary of ₹10,00,000 p.a. and employer has paid income, tax of ₹1,00,000 on behalf of the employee besides salary of ₹10,00,000, in this case employer is allowed to, debit ₹11,00,000 to profit and loss account and tax liability of the employee shall be computed in the manner, given below:, Income from salary, 11,00,000, Less: Standard deduction u/s 16(ia), (50,000), Income under the head salary, 10,50,000, Tax at slab rate on ₹10,50,000, 1,27,500, Add: HEC @ 4%, 5,100, Tax Liability, 1,32,600, Less: Tax paid by employer, (1,00,000), Tax Payable, 32,600, If income tax has been paid by the employer on behalf of the employee in connection with nonmonetary perquisites, employer shall not be allowed to debit such amount to profit and loss account, and also it will not be considered to be income of employee under the head salary under section, 10(10CC)., Question 28: Write a note on payment of salary or interest to the partners., Answer:, As per section 40(b), interest to the partner is allowed but maximum @ 12% p.a. simple interest., Payment of salary, bonus, commission or any other remuneration is allowed but only to the working partner., Maximum amount of salary, bonus, commission etc. allowed to a partner shall be computed in the manner, given below:, Maximum amount of remuneration allowed shall be as given below:, * First ₹3,00,000 of the book profits, 90% of the book profit or ₹1,50,000 whichever is more, * On balance amount of book profit, 60% of book profit
Page 689 :
Income Under The Head Business/Profession, , 170, , Example, A partnership firm has book profits of ₹ 5 lakhs, in this case maximum amount of salary etc. allowed to all, the partners shall be, Upto ₹3,00,000, 90% of 3,00,000 or ₹1,50,000 whichever is more, 2,70,000, Next ₹2,00,000, 60% of 2,00,000, 1,20,000, 3,90,000, Example, There is a partnership firm engaged in business and its book profits are ₹1,35,000, in this case maximum, amount of remuneration allowed to all the partners shall be ₹1,50,000., Meaning of Book Profits, Book profit means profit and gains of business profession but before charging any salary, bonus,, commission etc. to the partners and further if the assessee has any brought forward depreciation, it will be, adjusted while computing the book profits but if there are brought forward business loss, such business loss, shall not be adjusted., Share received by a partner from income of Partnership Firm Section 10(2A), If any partner has received share out of the profits of the partnership firm, such share shall be exempt from, income tax., As per section 28, interest or salary received by a partner shall be taxable under the head, business/profession., Example, XY Partnership firm has computed net profit of ₹ 5,00,000 and some of the amount debited & credited, given below :, Debited ₹1,00,000 being salary paid to one of the employees by crossed cheque, Debited ₹ 4,00,000 being the cost of motor car purchased and put to use 31.03.2022, Debited ₹3,00,000 being interest paid to Mr. X @ 15 % p.a., Debited ₹ 4,50,000 being interest paid to Mr. Y @ 15 % p.a., Debited salary of ₹10,00,000 Paid to Mr. X, Debited salary of ₹ 500,000 Paid to Mr. Y, Debited ₹80,000 being advance income tax, Credited ₹ 6,00,000 being long term capital gain (LTCG), The firm has b/f business loss of ₹30,000, The firm has donated ₹ 40,000 by cheque to charitable institution notified u/s 80G, Compute tax liability of partners & partnership firm A.Y. 2022-23., Solution:, ₹, Net profit as per profit and loss account, 5,00,000, Add : Salary paid by crossed cheque to employee, 1,00,000, Add: Capital exp. Debited to P/L a/c, 4,00,000, Less: Depreciation on motor car 400,000 x 7.5%, (30,000), Add: Interest in excess to Mr. X 3,00,000 x 3%/15%, 60,000, Add: Interest to Mr. Y 4,50,000 x 3% /15%, 90,000, Add: Salary to Mr. X, 10,00,000, Add: Salary to Mr. Y, 500,000, Add: Advance income tax, 80,000, Less: LTCG credited to P/L a/c, (6,00,000), Books Profits, 21,00,000, Salary allowed, First 3,00,000 x 90 % = 2,70,000, Bal. 18,00,000 x 60 % = 10,80,000, Total Salary allowed = 13,50,000, Salary to X (13,50,000 / 3 x 2), (9,00,000), Salary to Y (13,50,000 / 3 x 1), (4,50,000), Income under the head Business/Profession, 7,50,000
Page 690 :
Income Under The Head Business/Profession, Less: B/F business Loss, LTCG, Gross Total Income, Less: Deduction u/s 80G, Adjusted GTI = GTI -LTCG –STCG 111A- ALL deduction u/s 80C to 80U (except 80G), = 13,20,000- 6,00,000 = 7,20,000, 10% of ₹7,20,000 or 40,000 whichever is less, Qualifying amount 50% of ₹40,000, Total Income, Computation of Tax Liability, Tax on normal income ₹7,00,000 @ 30%, Tax on LTCG ₹6,00,000 @ 20%, Tax before health & education cess, Add: HEC @ 4%, Tax Liability, Mr. X, Business/Profession – Interest, Business/Profession – Salary, Gross Total Income/ Total Income, Computation of Tax Liability, Tax on normal income ₹11,40,000 @ slab rate, Add: HEC @ 4%, Tax Liability, Mr. Y, Business/Profession – Interest, Business/Profession – Salary, Gross Total Income/Total Income, Computation of Tax Liability, Tax on normal income ₹8,10,000 @ slab rate, Add: HEC @ 4%, Tax Liability, , 171, (30,000), 6,00,000, 13,20,000, (20,000), , 13,00,000, 2,10,000, 1,20,000, 3,30,000, 13,200, 3,43,200, 2,40,000, 9,00,000, 11,40,000, 1,54,500, 6,180, 1,60,680, 3,60,000, 4,50,000, 8,10,000, 74,500, 2,980, 77,480, , Illustration 15: XYZ are the partners in a firm with profit sharing ratio 5:3:2 and profit and loss account of, the partnership firm is as given below:, Particulars, Amount, Particulars, Amount, ₹, ₹, Purchases, 90,00,000 Sales, 102,00,000, Discount, 10,000, Salary and bonus to partners, 3,00,000, X, 2,50,000, Y, 1,50,000, Z, Municipal tax payable, 30,000 Interest from Indian company, 60,000, General expenses, 1,00,000 Interest on drawings, 10,000, Expenditure on technical know-how, 40,000 Income tax refund, 5,000, (purchased and put to use on 01.01.2022), Advance Income Tax, 70,000, Expenses on GST proceedings, 10,000, Expenses on income tax proceedings, 8,000, Advertisements, 50,000, Interest on capital to partners @ 13% p.a.
Page 691 :
Income Under The Head Business/Profession, X, Y, Z, Rent of building owned by partnership, firm, Net Profit, , 172, , 65,000, 39,000, 26,000, 1,20,000, 27,000, 102,85,000, , 102,85,000, , Additional information:, 1. Capital contributed by Mr. X is ₹5,00,000 and by Mr. Y ₹3,00,000 and by Mr. Z ₹2,00,000., 2. Salary paid to Mr. X is ₹3,00,000 and to Mr. Y ₹2,50,000 and to Mr. Z ₹1,50,000., 3. The partnership firm has brought forward business loss for assessment year 2019-20 amounting to, ₹1,00,000., 4. Municipal tax was paid on 01.11.2022., Personal incomes of partners:, (i) Mr. X has income from house property ₹5,00,000 and amount invested in National Saving Certificate, ₹80,000., (ii) Mr. Y has income from house property ₹2,00,000 and amount invested in National Saving Certificate, ₹1,00,000., (iii) Mr. Z has loss from house property ₹2,00,000., Compute Tax Liability of the partnership firm and also that of its partners for the Assessment Year 2022-23., Solution:, ₹, Net Profit as per profit and loss account, 27,000.00, Add:, • Salary and bonus to partners, X, ₹3,00,000, Y, ₹2,50,000, Z, ₹1,50,000, 7,00,000.00, • Municipal tax payable (Sec 43B), 30,000.00, • Technical Know-how, 40,000.00, • Income Tax (Sec 40(a)), 70,000.00, • Interest on capital, X (65,000 x 1/13), 5,000.00, Y (39,000 x 1/13), 3,000.00, Z (26,000 x 1/13), 2,000.00, • Rent of own building (Sec 30), 1,20,000.00, Less:, • Depreciation on technical know-how, (5,000.00), (40,000 x 25% x ½), • Interest from Indian company, (60,000.00), • Income tax refund, (5,000.00), Book Profit, 9,27,000.00, • Salary and bonus allowed to partners, X (6,46,200 x 3/7), (2,76,942.86), Y (6,46,200 x 2.5/7), (2,30,785.71), Z (6,46,200 x 1.5/7), (1,38,471.43), Working note:, Computation of remuneration allowed to partners, 3,00,000 x 90% = ₹2,70,000, 6,27,000 x 60% = ₹3,76,200, Total remuneration = ₹6,46,200, Salary allowed to partners maximum to ₹6,46,200, Income under the head Business/Profession, 2,80,800.00
Page 692 :
Income Under The Head Business/Profession, , 173, , Less: Brought forward business loss, (1,00,000.00), Income under the head Business/Profession, 1,80,800.00, Income under the head Other Sources, Interest from Indian company, 60,000.00, Income under the head Other Sources, 60,000.00, Gross Total Income, 2,40,800.00, Less: Deductions u/s 80C to 80U, Nil, Total Income, 2,40,800.00, Computation of Tax Payable, Tax on ₹2,40,800 @ 30%, 72,240.00, Add: HEC @ 4%, 2,889.60, Tax Liability, 75,129.60, Less: Income tax paid, (70,000.00), Tax Payable, 5,129.60, Rounded off u/s 288B, 5,130.00, Computation of Total Income and Tax Liability of Mr. X, Salary from partnership firm, 2,76,942.86, Interest from partnership firm, 60,000.00, Income under the head Business/Profession, 3,36,942.86, Income from house property, 5,00,000.00, Gross Total Income, 8,36,942.86, Less: Deduction u/s 80C {National Saving Certificate}, (80,000.00), Total Income (rounded off u/s 288A), 7,56,940.00, Tax on ₹7,56,940 at slab rate, 63,888.00, Add: HEC @ 4%, 2,555.52, Tax Liability, 66,443.52, Rounded off u/s 288B, 66,440.00, Computation of Total Income and Tax Liability of Mr. Y, Salary from partnership firm, 2,30,785.71, Interest from partnership firm, 36,000.00, Income under the head business/profession, 2,66,785.71, Income from house property, 2,00,000.00, Gross Total Income, 4,66,785.71, Less: Deduction u/s 80C {National Saving Certificate}, (1,00,000.00), Total Income (Rounded off u/s 288A), 3,66,790.00, Tax on ₹3,66,790 at slab rate, 5,839.50, Less: Rebate u/s 87A, (5,839.50), Tax Liability, Nil, Computation of Total Income and Tax Liability of Mr. Z, Salary from partnership firm, 1,38,471.43, Interest from partnership firm, 24,000.00, Income under the head Business/Profession, 1,62,471.43, Loss from house property, (2,00,000.00), Carry forward house property loss, (37,528.57), Illustration 16: If a firm has paid ₹7,50,000 as remuneration to its partners for the P.Y.2021-22, in, accordance with its partnership deed, and it has a book profit of ₹ 10 lakh, then, the allowable remuneration, calculated as per the limits specified in section 40(b) would be –, Particulars, ₹, On first ₹ 3 lakh of book profit [₹ 3,00,000 × 90%], 2,70,000, On balance ₹ 7 lakh of book profit [₹ 7,00,000 × 60%], 4,20,000, 6,90,000, The excess amount of ₹ 60,000 (i.e., ₹ 7,50,000 – ₹ 6,90,000) would be disallowed as per section 40(b)(v).
Page 693 :
Income Under The Head Business/Profession, , 174, , NOV-2011 (4 Marks), X & Y, a partnership firm consisting of two partners, reports a net profit of ₹7,00,000 before deduction of, the following items:, (1) Salary of ₹20,000 each per month payable to two working partners of the firm (as authorized by the deed, of partnership)., (2) Depreciation on plant and machinery under section 32 (computed) ₹1,50,000., (3) Interest on capital at 15% per annum (as per the deed of partnership). The amount of capital eligible for, interest ₹5,00,000, Compute:, (i) Book-profit of the firm under section 40(b) of the Income-Tax Act 1961., (ii) Allowable working partner salary for the Assessment Year 2022-23 as per section 40(b) of the Incometax Act, 1961., Answer:, (i) Computation of Book- Profits under section 40(b) of Income Tax Act 1961, ₹, Net Profit, 7,00,000.00, Less: Depreciation u/s 32, (1,50,000.00), Less: Interest on capital, (60,000.00), (5,00,000 x 12%), Book Profit as per section 40(b), 4,90,000.00, (ii) Calculation of allowable salary to partners, Book Profit, 4,90,000.00, Allowable Salary, On first 3,00,000 of book profit, 90% of book profits or 1,50,000 whichever is higher, 2,70,000.00, On balance book profit, 60% on balance book profit (1,90,000 x 60/100), 1,14,000.00, Hence, salary as per section 40(b) would be, 3,84,000.00, Question 29 [V. Imp.]: Write a note on payment to Relative/Related person., Answer: Payment to Relative/Related Person Section 40A(2), If the assessee incurs any expenditure and payment has been given to any person mentioned below and such, expenditure is excessive or unreasonable having regard to the fair market value of the goods, services or, facilities, so much of the expenditure as is so considered to be excessive or unreasonable shall not be, allowed as a deduction. E.g. Mr. X has purchased raw material for his business from his brother and has paid, ₹5,00,000 but market value is ₹3,00,000, in this case expenditure disallowed shall be ₹2,00,000., The persons covered in this category are –, 1. If any individual has made any payment to his relative. As per section 2(41) Relative, in relation to an, individual, means the husband, wife, brother or sister or any lineal ascendant or descendant of that, individual., 2. If the assessee is a company, firm, association of persons or Hindu Undivided Family etc. and it has made, payment to any director of the company, partner of the firm, or member of the association or family, or any, relative of such director, partner or member etc., 3. If any person made payment to any other person who has substantial interest in the business of the, assessee e.g. ABC Ltd. has paid ₹5,00,000 to XYZ Ltd. and XYZ Ltd. is holding 20% shares of ABC Ltd.,, in this case excessive payment is disallowed., 4. Any other person covered under section 40A(2)., Question 30 [V. Imp.]. Discuss provisions relating to payments in excess of ₹10,000., Answer: Payment in excess of ₹10,000 Section 40A(3) Rule 6DD, If an assessee has incurred any expenditure and the payment or the aggregate of the payments made to a, person with regard to such expenditure on any single day exceeds ₹10,000 and payment was made otherwise, than through account payee cheque or account payee bank draft or use of electronic clearing system, through a bank account or Credit Card, Debit Card, Net Banking, IMPS (Immediate Payment Service),, UPI (Unified Payment Interface), RTGS (Real Time Gross Settlement), NEFT (National Electronic, Funds Transfer), and BHIM (Bharat Interface for Money) Aadhaar Pay, in such cases entire expenditure
Page 694 :
Income Under The Head Business/Profession, , 175, , is disallowed., In case of payment made for plying, hiring or leasing goods carriages, the ceiling of ten thousand, rupees shall be enhanced to thirty-five thousand rupees., Example, Mr. X has incurred an expenditure of ₹29,000. Mr. X makes separate payments of ₹9,000, ₹8,000 and, ₹12,000 all by cash, to the person concerned in a single day. The aggregate amount of payment made to a, person in a day, in this case, is ₹29,000. Since, the aggregate payment by cash exceeds ₹10,000, ₹29,000, will not be allowed as a deduction in computing the total income of Mr. X., Example, (i) If ABC Ltd. has paid ₹65,000 in cash, expenditure disallowed shall be ₹65,000., (ii) If Mr. X has paid ₹11,000 by a bearer cheque, amount disallowed shall be ₹11,000., (iii) If ABC Ltd. has paid ₹10,050 by a crossed cheque, amount disallowed shall be ₹10,050., (iv) ABC Ltd. has paid ₹35,000 by an account payee cheque, entire amount is allowed., (v) Mr. X pays a salary to his employee ₹15,000 by crossed cheque, in this case entire expenditure is, disallowed., (vi) ABC Ltd. has paid ₹32,000 in cash to a goods transport agency for transportation of goods, expenditure, is allowed., (vii) Mr. X purchases goods worth ₹75,000 on 01.01.2022 and payment was made ₹60,000 on 03.01.2022, by account payee cheque and ₹8,000 in cash on 03.01.2022 and ₹7,000 in cash on 05.01.2022, in this case, expenditure is allowed., (viii) Mr. X purchases goods worth ₹8,000 and ₹5,000 against two bills from Mr. Y and makes the payment, ₹13,000 in cash in a single day, in that case entire expenditure is allowed., (ix) Mr. X purchases goods worth ₹15,000 from Mr. Y against one bill but makes payment of ₹7,500 and, ₹7,500 at different times on the same date, in that case entire expenditure is disallowed., Exceptions under rule 6DD, As per rule 6DD the above provisions are not applicable with regard to following payments:, 1. Payment made to Reserve Bank of India, State Bank of India or other banking institutions, LIC, UTI /, Central/State Government etc., 2. If the payment is made in a village or town and there is no bank at such place on the date of making the, payment and payment is being given to any person who ordinarily resides at that place or has his, business or profession at that place., 3. Where the payment is made for the purchase of, (i) agricultural or forest produce ; or, (ii) the produce of animal husbandry or dairy or poultry farming ; or, (iii) fish or fish products ; or, (iv) the products of horticulture or apiculture (Honey making),, to the cultivator, grower or producer of such articles, produce or products., 4. Where the payment is made for the purchase of the products manufactured in a cottage industry, to, the producer of such products., 5. Where the payment is made by transferring funds from one bank account to the other or payment is, being made by any credit card/ a debit card/ letter of credit etc., payment is allowed., 6. If payment is being made to an employee after retirement or to his family member after the death of, the employee and payment is in connection with gratuity etc. and payment is not exceeding ₹50,000., 7. Any other situation given under Rule 6DD., Question 31: Write a note on deductibility in respect of provision for Gratuity Fund., Answer: Deductibility in respect of provision for Gratuity Fund Section 40A(7), In general no provision is allowed under Income Tax Act however as a special case, provision for gratuity is, allowed. The assessee can make provision for contribution towards approved gratuity fund but such, provision should be actuarial provision i.e. it should not be hypothetical., Question 32: Write a note on employer’s contribution to various funds., Answer: Employer’s contribution to various funds Section 40A(9), Employer’s contribution to various funds is allowed only if such funds are notified under any Act. If the, employer has contributed to the recognised provident fund, approved superannuation fund, approved
Page 695 :
Income Under The Head Business/Profession, , 176, , gratuity fund or any other similar fund required under any other Act, such contribution is allowed, but, payment has to be made upto the last date of filing of return of income as per section 43B., If the employer has contributed to any other fund like unrecognised provident fund, unapproved gratuity, fund, unapproved superannuation fund etc., expenditure shall not be allowed., Question 33: Explain incomes under section 41., Answer: In general a person cannot have income under the head business/profession without, business/profession but as per section 41(1), 41(2), 41(3) and 41(4), such incomes shall be taxable under the, head business/profession even if the assessee do not have any business/profession and are as under below:, As per section 41(1), if any assessee has debited any amount to the profit and loss account and subsequently, it was recovered by him, it will be considered to be his income under the head business/profession of the, year in which it has been recovered even if the assessee do not have any business or profession in that year., e.g. Mr. X has debited ₹20,000 to profit and loss account being the municipal tax paid but in the subsequent, year there was refund of ₹3,000, in this case it will be considered to be income under the head, business/profession in the year of recovery. If amount has been recovered by the successor of business, in, that case, it will be considered to be income of such successor, “Successor in business” means, the amalgamated company / the resulting company / where a firm is, succeeded by another firm, the other firm etc., (Section 41(2), 41(3) and 41(4) have been discussed in the relevant questions.), Question 34 [Imp.]: Write a note on actual cost. Sec 43(1), Answer: Actual Cost Section 43(1), In case of depreciable assets, depreciation is allowed on actual cost and as per section 43(1), actual cost, means total expenses incurred upto the date of putting the asset to use., If ABC Ltd. has taken a loan of ₹ 40 lakhs @ 10% p.a. on 01.04.2021 for purchasing a particular plant and, machinery and the company has made additional payment asunder:, 1. Transportation charges ₹2 lakh., 2. Loading and unloading expenses ₹25,000, 3. Payments for the expert staff to install the plant and machinery ₹3 lakh., 4. Company has incurred ₹ 4 lakh for construction of a platform for installing the plant and machinery., The asset was put to use on 01.01.2022, in this case actual cost of the asset shall be ₹52,25,000 (40,00,000 +, interest 3,00,000 (40,00,000 x 10% x 9/12) + 2,00,000 + 25,000 + 3,00,000 + 4,00,000), If the assessee has received any subsidy from the Government or other similar agency, the subsidy so, received shall be deducted and only the balance amount shall be considered to be the actual cost., Example, If in the above case, the assessee has received a subsidy of ₹ 2 lakh in connection with plant and machinery, because it was a non-polluting plant, in this case, actual cost of asset shall be ₹50,25,000., Treatment of interest subsequent to putting the asset to use, Any interest in connection with the acquisition of an asset relating to any period after such asset is first put, to use shall not be included, in the actual cost of such asset, rather it is revenue expenditure and is allowed to, be debited to profit and loss account but interest prior to putting the asset to use shall be added in the actual, cost., Buildings in personal use subsequently used in business/profession, Where a building previously the property of the assessee is brought into use for the purpose of the business, or profession, the actual cost to the assessee shall be the actual cost of the building to the assessee, as, reduced by an amount equal to the depreciation that would have been allowable had the building been used, for the business/profession since the date of its acquisition., Where the assessee incurs any expenditure for acquisition of any asset or part thereof in respect of which a, payment or aggregate of payments made to a person in a day, otherwise than by an account payee cheque, drawn on a bank or an account payee bank draft or use of electronic clearing system through a bank account, or Credit Card, Debit Card, Net Banking, IMPS (Immediate Payment Service), UPI (Unified Payment, Interface), RTGS (Real Time Gross Settlement), NEFT (National Electronic Funds Transfer), and BHIM, (Bharat Interface for Money) Aadhaar Pay, exceeds ten thousand rupees, such expenditure shall be ignored, for the purposes of determination of actual cost.
Page 696 :
Income Under The Head Business/Profession, , 177, , Where a stock in trade is converted into capital assets then fair market value of the stock shall be treated, as actual cost of the asset., Illustration 17: Dr. Sagar purchased a residential building on 01.12.2019 for ₹12,00,000 and it was put to, use on the same date. Till 01.12.2021 the same was self-occupied as residence. On this date, the building, was brought into use for the purpose of his medical profession (it was used as residential building). What, would be the depreciation allowable for the Assessment Year 2022-23?, Solution:, ₹, Computation of depreciation for the Assessment Year 2022-23, In this case notional depreciation shall be allowed as per section 43(1) and depreciation allowable for the, Assessment Year 2022-23 shall be computed in the manner given below:, 12,00,000, Cost of building as on December 1st, 2019, Less: depreciation for the previous year 2019-20, (30,000), (2.5% of ₹ 12,00,000), (As building purchased during the year 2019-20 is put to use for, less than 180 days during the year), Written down value as on 01.04.2020, 11,70,000, Less: depreciation for previous year 2020-21 @ 5%, (58,500), Written down value as on 01.04.2021, 11,11,500, Depreciation for the previous year 2021-22 @ 5%, 55,575, MAY – 2012 (4 Marks), A car purchased by Dr. Ramesh on 10.08.2019 for ₹5,25,000 for the personal use is brought into, professional use on 01.07.2021 by him, when its market value was ₹2,50,000., Compute the actual cost of the car and amount of depreciation for the assessment year 2022-23 assuming the, rate of depreciation to be 15%., Answer:, Computation of Actual Cost of the Car and Depreciation for the A.Y.2022-23, Actual cost of the car =, ₹5,25,000, Depreciation [₹5,25,000 x 15%], ₹78,750, Question 35: Write a note on the Method of Accounting as per Section 145., Answer: Method of Accounting Section 145, Income chargeable under the head “Profits and gains of business or profession” or “Income from other, sources” shall, be computed in accordance with either cash or mercantile system of accounting regularly, employed by the assessee however any system of accounting once adopted has to be followed consistently, but it can be changed with the permission of assessing officer., Question 36 [V. Imp.]: Write a note on Section 43B., Answer: Certain deductions to be only on actual payment Section 43B, If any assessee has maintained books of accounts on the basis of mercantile system of accounting, all the, expenditures are allowed on due basis. But the expenditures listed under section 43B are allowed only on, actual payment basis., These expenditures are: (a) any sum payable by the assessee by way of tax, duty, cess or fee, by whatever name called, under any, law like Municipal Tax, Professional Tax , Composition Tax etc., (b) Employer’s contribution to any provident fund or superannuation fund or gratuity fund,, Employees State Insurance (ESI) or any other fund for the welfare of employees., (c) Bonus or commission or leave salary to the employee., (d) Interest on any loan or borrowing from any Public Financial Institution or a State Financial, Corporation or a State Industrial Investment Corporation or scheduled bank., (e) Any sum payable by the assessee to the Indian Railways for the use of railway assets,, (f) any sum payable by the assessee as interest on any loan or borrowing from a deposit taking nonbanking financial company or systemically important non-deposit taking non-banking financial
Page 697 :
Income Under The Head Business/Profession, , 178, , company, in accordance with the terms and conditions of the agreement governing such loan or, borrowing,, The assessee is allowed to make the payment till the last date of filing of return of income relating to the, previous year in which the expenditure was incurred., If the payment is made after the last date of filing of return of income, expenditure is allowed in the year in, which the assessee has made the payment., Example, ABC Ltd has debited bonus of ₹3,00,000 to the Profit/Loss A/c for the previous year 2021-22 and the, company paid the bonus on 07.12.2022, in this case expenditure is not allowed in the previous year 2021-22., Rather expenditure is allowed in the previous year 2022-23. Similarly if the payment is made by the, company on 07.05.2023, expenditure shall be allowed in the previous year 2023 - 24., Illustration 18: Mr. X (age 82 years) has one house which is 50% in business/profession and 50% is let out, @ 10,000 p.m. and municipal taxes for the entire house are ₹7,000 which were paid on 10.04.2022 and, business income of Mr. X before debiting any expense of house property is ₹7,80,000. Compute tax liability, for the Assessment Year 2022-23., Solution:, ₹, Income under the head business/profession shall be, Less: Municipal taxes, ₹7,80,000, Net Profit as per P/L accounts, (₹3,500), 7,76,500.00, (Municipal tax allowed because payment has been made upto the last date of filing of return of income), Income under the head House Property, Gross annual value, 1,20,000.00, Less: Municipal taxes (allowed only on actual payment basis), Nil, Net annual value, 1,20,000.00, Less: 30% of NAV u/s 24(a), (36,000.00), Less: Interest on capital borrowed u/s 24(b), Nil, Income under the head House Property, 84,000.00, Gross Total Income, 8,60,500.00, Less: Deduction u/s 80C to 80U, Nil, Total Income, 8,60,500.00, Computation of Tax Liability, Tax on ₹8,60,500 at slab rate, 72,100.00, Add: HEC @ 4%, 2,884.00, Tax Liability, 74,984.00, Rounded off u/s 288B, 74,980.00, (Municipal tax not allowed because payment has not been made upto the end of previous year), Illustration 19: Mr. X has computed his income ₹3,50,000 and some of the amounts debited to the profit, and loss account are as given below:, 1. Household expense ₹5,000, 2. Rent for own building ₹1,20,000 (half of the building is self occupied and balance half in business use)., 3. Municipal tax of the building ₹3,000 (amount was paid on 01.04.2022), 4. Expenditure on repairs of the building ₹4,000., 5. Premium paid for insurance of the building ₹2,000., 6. Mr. X has purchased one motor car for ₹3,00,000 on 01.01.2022 and it was put to use on the same date., The car was used for personal purpose as well as official use (50% official and 50% personal). Assessee, has also debited petrol expenses of ₹5,000., 7. He has debited ₹20,000 being the amount invested in public provident fund., His tax liability for the Assessment Year 2022-23 shall be computed in the manner given below:, Solution:, ₹, Net income as per profit and loss account, 3,50,000.00, Add:
Page 698 :
Income Under The Head Business/Profession, , 179, , 1. Household expenses, 5,000.00, 2. Rent of the own building, 1,20,000.00, 3. Municipal tax, 1,500.00, 4. Repairs, 2,000.00, 5. Insurance, 1,000.00, 6. Capital expenditure on motor car, 3,00,000.00, 7. Petrol, 2,500.00, 8. Public Provident Fund, 20,000.00, Less:, Depreciation of motor car, 3,00,000 x 15% x ½ x ½, (11,250.00), Income under the head Business/Profession, 7,90,750.00, Income under the head house property, GAV of self occupied house, Nil, Municipal tax, Nil, NAV, Nil, 24(a), Nil, Income under the head house property, Nil, Gross Total Income, 7,90,750.00, Less: Deduction u/s 80C, (20,000.00), Total Income, 7,70,750.00, Computation of Tax Liability, Tax on ₹7,70,750 at slab rate, 66,650.00, Add: HEC @ 4%, 2,666.00, Tax Liability, 69,316.00, Rounded off u/s 288B, 69,320.00, Question 37: Write a note on full value of consideration for transfer of assets other capital asset in, certain cases., Answer: Special provision for full value of consideration for transfer of assets other than capital assets, in certain cases Section 43CA, If any person has sold land or building which was held as stock-in-trade and it was sold at a value less than, the stamp duty value, in such cases sale value shall be considered to be stamp duty value but if the seller has, entered into a agreement to sell the property at an earlier date and some advance was taken through account, payee cheque or account payee draft or through electronic clearing system through a bank account, Credit, Card, Debit Card, Net Banking, IMPS (Immediate Payment Service), UPI (Unified Payment Interface),, RTGS (Real Time Gross Settlement), NEFT (National Electronic Funds Transfer), and BHIM (Bharat, Interface for Money) Aadhaar Pay, in such cases stamp duty value on the date of agreement shall be taken, into consideration., Provided that where the value adopted or assessed or assessable by the authority for the purpose of payment, of stamp duty does not exceed one hundred and ten per cent of the consideration received or accruing as a, result of the transfer, the consideration so received or accruing as a result of the transfer shall, for the, purposes of computing profits and gains from transfer of such asset, be deemed to be the full value of the, consideration., it can be shown in the manner given below:, (₹ in lakhs), Case, Actual, Stamp duty value on, Stamp duty value on, Consideration, Consideration, the date of agreement, the date of registration, which will be, applicable, 210, 120, 1, 100, 120, (1/5/2020), (31/8/2019), (₹10 lakhs received by, Account payee cheque on, 31/8/2019)
Page 699 :
Income Under The Head Business/Profession, 2, , 100, , 3, , 100, , 120, (31/8/2019), (₹10 lakhs received by cash, on 31/8/2019), 120, (1/5/2020), , 180, , 210, (1/5/2020), , 210, , 210, (31/3/2021), (Full amount received on, the date of registration), , 210, , Provided further that in case of transfer of an asset, being a residential unit, the provisions of this proviso, shall have the effect as if for the words “one hundred and ten per cent.”, the words “one hundred and, twenty per cent.” had been substituted, if the following conditions are satisfied, namely: ––, (i) the transfer of such residential unit takes place during the period beginning from the 12th day of, November, 2020 and ending on the 30th day of June, 2021;, (ii) such transfer is by way of first time allotment of the residential unit to any person; and, (iii) the consideration received or accruing as a result of such transfer does not exceed two crore, rupees., Explanation.––For the purposes of this section, “residential unit” means an independent housing unit, with separate facilities for living, cooking and sanitary requirement, distinctly separated from other, residential units within the building, which is directly accessible from an outer door or through an, interior door in a shared hallway and not by walking through the living space of another household., Question 38 [V. Imp.]: Discuss the provisions of the Income Tax Act 1961, regarding compulsory, maintenance of accounts., Answer: Compulsory maintenance of accounts Section 44AA Rule 6F, Provisions regarding maintaining of books of accounts shall be as given below:, 1. Persons having specified profession, The person having specified profession have to maintain any books of accounts as may enable the Assessing, Officer to compute his total income, however they have to maintain prescribed books of accounts if gross, receipt exceeds ₹1,50,000 in all the three years immediately preceding the previous year., Example, Mr. X is engaged in medical profession and his gross receipt during the various years is asunder:, 1. 2020-21, 1,40,000, 2. 2019-20, 1,70,000, 3. 2018-19, 1,25,000, In this case, during the previous year 2021-22, Mr. X is not required to maintain prescribed books of, accounts because gross receipt has not exceeded ₹1,50,000 during all the three years immediately preceding, the relevant previous year. But if receipt during 2020-21 is ₹1,75,000 and during 2018-19 it is ₹1,55,000, he, has to maintain prescribed books of accounts during 2021-22., If profession has been newly setup in the previous year and gross receipt are likely to exceed ₹1,50,000, he, should maintain prescribed books of accounts., Specified Profession shall include, 1. Legal profession 2. Medical profession 3. Engineering profession 4. Architectural profession 5. Profession, of accountancy 6. Technical consultancy 7. Interior decoration 8. Authorised representatives 9. Film artists, 10. Company Secretary 11. Information Technology., Preservation of the books of accounts, The books of accounts are to be kept and maintained for the period of atleast 6 years from the end of the, relevant assessment year., 2. Persons carrying on business or any profession, not specified above, Such persons are not required to maintain accounts in general, however if their income from business or, profession exceeds one lakh twenty thousand rupees or their total sales turnover or gross receipts as, the case may be, in business or profession exceeds ₹ 10 lakhs in any one of the three years immediately, preceding the previous year, they will be required to maintain any books of accounts.
Page 700 :
Income Under The Head Business/Profession, , 181, , In case of business or profession newly set up in any previous year, obligation to maintain accounts will, arise if the income is likely to exceed ₹1,20,000 or total sales turnover or gross receipts as the case may be, in business or profession are likely to exceed ₹10 lakhs during such previous year., For Individual and HUF Limit of ₹2,50,000 instead of ₹1,20,000 and Limit of ₹25,00,000 instead of, ₹10,00,000 shall be applicable., 3. Persons whose business income is to be computed on presumptive basis under section, 44AD/44ADA/44AE, If income of any person is to be computed under section 44AD or 44ADA or 44AE on presumptive basis but, such person has rejected presumptive income and his income is exceeding the maximum amount which is, exempt from income tax, in such cases such person shall be required to maintain any books of accounts (also, audit is required as per section 44AB), e.g. Mr. X has turnover of his business ₹20,00,000 but he has, rejected presumptive income, books/ audit not required but if turnover is ₹50,00,000 and person has rejected, presumptive income, books as well as audit is required., Question 39 [V. Imp.]: Write short note on Compulsory Tax Audit., Answer: Compulsory Tax Audit, Audit of accounts of certain persons carrying on business or profession Section 44AB, The following persons have to get their accounts audited., 1. Every person carrying on business, if his total sales turnover or gross receipts, in business exceeds ₹100, lakh during the previous year., 2. Every person carrying on profession if his gross receipts in profession exceed ₹50 lakh during the, previous year., 3. If income of any person is to be computed under section 44AD or 44ADA or 44AE on presumptive basis, but such person has rejected presumptive income, in such cases such person shall be required to get the, accounts audited., 4. Every person carrying on business, if his total sales turnover or gross receipts, in business exceeds, ₹1000 lakh during the previous year provided that in the case of a person whose––, (a) aggregate of all amounts received including amount received for sales, turnover or gross receipts, during the previous year, in cash, does not exceed five per cent. of the said amount. and, (b) aggregate of all payments made including amount incurred for expenditure, in cash, during the, previous year does not exceed five per cent. of the said payment., Provided further that for the purposes of this clause, the payment or receipt, as the case may be, by a, cheque drawn on a bank or by a bank draft, which is not account payee, shall be deemed to be the, payment or receipt, as the case may be, in cash., The accounts should be audited by a Chartered Accountant and audit report should be submitted latest by, one month prior to the last date of filing of return of income, This section shall not apply to the person, who declares profits and gains for the previous year in, accordance with the provisions of section 44AD and his total sales, turnover or gross receipts, as the case, may be, in business does not exceed two crore rupees in such previous year., Penalty for violating provisions of Section 44AB Section 271B, If any person fails to get his accounts audited or fails to submit audit report in time, penalties may be, imposed under section 271B equal to ½% of total turnover or gross receipt subject to a maximum of, ₹1,50,000., Example, Mr. X has turnover of his business ₹105 lakhs but he has failed to get his accounts audited, in this case, penalties may be imposed amounting to ₹52,500 but if his turnover was ₹400 lakhs, penalties imposable, shall be ₹2,00,000 but maximum ₹1,50,000.
Page 701 :
Income Under The Head Business/Profession, , 182, , Question 40 [V. Imp.]. Briefly describe provisions of income tax act for computing profit and gains of, business on Presumptive Basis., Answer: Special provision for computing profits and gains of business on presumptive basis. Section, 44AD, 1. If any assessee has turnover of his business upto ₹200 lakhs, such assessee is allowed to compute income, on presumptive basis and income under the head business/profession shall be presumed to be 8% of the, turnover and no further deduction is allowed under section 30 to 38., 2. Such option is allowed only to an Individual/ HUF / Firm who are resident but not to LLP or Company., 3. Section 44AD is applicable only to business and not to specified profession and also it is not applicable, for the persons having earning as commission or brokerage., 4. Such assessee shall be required to pay advance tax to the extent of 100% of tax liability on or before 15th, March of the relevant previous year otherwise interest shall be charged @ 1% for one month on the amount, of default., 5. Brought forward business loss is allowed to be adjusted from such income but brought forward, depreciation is not allowed to be adjusted from such income., 6. The assessee shall be exempt from maintaining books of accounts or audit., 7. If an assessee has opted for presumptive income under section 44AD and in the subsequent 5 years he, has rejected presumptive income, in that case he will not be allowed to opt for presumptive income for next, 5 years. If assessee has rejected the presumptive income, he will be required to maintain any books of, accounts and also audit is required. e.g. Mr. X has opted for presumptive income under section 44AD in the, previous year 2021-22, in this case he cannot reject 44AD during the subsequent 5 previous years i.e., previous year 2022-23, 2023-24, 2024-25, 2025-26, 2026-27. If he has rejected 44AD in any of these 5, years, he will not be allowed to opt for 44AD in next 5 years. If he has rejected 44AD in previous year, 2023-24, he cannot opt for 44AD during the previous year 2024-25, 2025-26, 2026-27, 2027-28,2028-29., Rate of 6% shall be applied instead of 8% if the amount of total turnover or gross receipts which is, received by an account payee cheque or an account payee bank draft or use of electronic clearing system, through a bank account or Credit Card, Debit Card, Net Banking, IMPS (Immediate Payment Service),, UPI (Unified Payment Interface), RTGS (Real Time Gross Settlement), NEFT (National Electronic, Funds Transfer), and BHIM (Bharat Interface for Money) Aadhaar Pay during the previous year or, before the due date specified in subsection (1) of section 139 in respect of that previous year., Illustration 20: Mr. X is engaged in a business with turnover ₹170,00,000 (all payments received by, account payee cheque, bank draft or through electronic clearing) and expenses incurred in connection with, earning of income are ₹160,00,000. He has LTCG ₹5,00,000. He has donated ₹4,00,000 to Rajiv Gandhi, Foundation by cheque. He has brought forward loss of business ₹1,00,000 of previous year 2017-18., Compute his Income and Tax Liability for previous year 2021-22, in two situations (i) He has opted for section 44AD., (ii) He has not opted for section 44AD., Solution:, (i) As per section 44AD, Presumptive Income shall be, ₹, 170,00,000 x 6% =, 10,20,000, Less: Brought forward loss of P.Y. 2017-18, (1,00,000), Income under the head Business or Profession, 9,20,000, Income under the head Capital Gains, Long Term Capital Gains, 5,00,000, Gross Total Income, 14,20,000, Less: Deduction u/s 80G (50% of ₹4,00,000), (2,00,000), Total Income, 12,20,000
Page 702 :
Income Under The Head Business/Profession, Computation of Tax Liability, Tax on ₹7,20,000 at slab rate, Tax on LTCG ₹5,00,000 @ 20% u/s 112, Add: HEC @ 4%, Tax Liability, Note:, 1. The Assessee shall be exempt from maintaining books of accounts and also from Audit., (ii), Income under the head Business or Profession, Gross Receipts, Less: Expenses Incurred, Less: Brought forward loss of P.Y. 2017-18, Income under the head Business or Profession, Income under the head Capital Gains, Long Term Capital Gains, Gross Total Income, Less: Deduction u/s 80G (50% of 4,00,000), Total Income, Computation of Tax Liability, Tax on ₹7,00,000 at slab rate, Tax on LTCG ₹5,00,000 @ 20% u/s 112, Add: HEC @ 4%, Tax Liability, Note: The Assessee shall be liable to maintain books of accounts and also liable to Audit., , 183, 56,500.00, 1,00,000.00, 6,260.00, 1,62,760.00, , 170,00,000, (160,00,000), (1,00,000), 9,00,000, 5,00,000, 14,00,000, (2,00,000), 12,00,000, 52,500.00, 1,00,000.00, 6,100.00, 1,58,600.00, , MAY – 2011 (5 Marks), Mr. X engaged in Retails Trade, reports a turnover of ₹58,50,000 (all payments received in account payee, cheque) for the financial year 2021-22. His income from the said business as per books of account is, computed at ₹2,90,000. Retail trade is the only source of income for Mr. X., (i) Is Mr. X eligible to opt for presumptive determination of his income chargeable to tax for the, Assessment Year 2022-23?, (ii) Is so, determine his income from retail trade as per the applicable presumptive provision., (iii) In case, Mr. X has not opted for presumptive taxation of income from retail trade, what are his, obligations under the Income-tax Act, 1961?, (iv) What is the ‘due date’ for filing his return of income, under both the options?, Solution:, (i) Yes. Since his total turnover for the F.Y.2021-22 is below ₹200 lakhs, he is eligible to opt for, presumptive taxation scheme under section 44AD in respect of his retail trade business., (ii) His income from retail trade, applying the presumptive tax provisions under section 44AD, would be, ₹3,51,000, being 6% of ₹ 58,50,000., (iii) In case he has not opted for the presumptive taxation scheme under section 44AD, and claims that his, income is ₹2,90,000 (which is lower than the presumptive business income of ₹3,51,000), he has to maintain, books of account as required under section 44AA and also get them audited and furnish a report of such, audit under section 44AB, since his total income exceeds the basic exemption limit of ₹2,50,000. And he is, not eligible to claim the benefit of presumptive taxation for the five assessment year., (iv) In case he opts for the presumptive taxation scheme under section 44AD, the due date would be 31st, July, 2022.
Page 703 :
Income Under The Head Business/Profession, , 184, , In case he has not opted for the presumptive taxation scheme and claims that his income is ₹2,90,000 as per, books of account, then he has to get his books of account audited under section 44AB, in that case the due, date for filing of return would be 31st October, 2022., MTP NOV – 2020 (3 Marks), Mr. Karan is engaged in the business of producing and selling toys. During the previous year 2021-22, his, turnover was ₹ 1.75 crores. He opted for paying tax as per presumptive taxation scheme laid down in section, 44AD. He has no other income during the previous year. Is he liable to pay advance tax and if so, what is the, minimum amount of advance tax to be paid and the due date for payment of such advance tax assuming that, whole of the turnover represents cash receipts?, Answer:, Computation of advance tax liability in the hands of Mr. Karan opting for presumptive taxation, scheme under section 44AD, Particulars, ₹, As per section 211, an eligible assessee, opting for computation of profits or gains of business, on presumptive basis in respect of an eligible business referred to in section 44AD, shall be, required to pay advance tax of the whole amount on or before 15th March of the financial year., Thus, Mr. Karan is required to pay advance tax for F.Y.2021-22 on or before 15th March, 2022., However, any amount paid by way of advance tax on or before 31st March shall also be treated, as advance tax paid during that financial year on or before 15th March., The advance tax liability is computed as follows –, Total Income being 8% of ₹1,75,00,000, since Mr. Karan is an eligible assessee opting for, 14,00,000, presumptive taxation scheme under section 44AD, (Total income comprises of only income under the head “Profits and gains of business or, profession”, since Mr. Karan is not having any other income during the previous year), Tax Liability, Upto ₹ 2,50,000, Nil, ₹ 2,50,001 to ₹ 5,00,000@5%, 12,500, ₹ 5,00,001 to ₹ 10,00,000@20%, 1,00,000, Above ₹ 10,00,000@30% (4,00,000 x 30%), 1,20,000, 2,32,500, Add: Health and Education cess @4%, 9,300, Total Tax Payable, 2,41,800, Accordingly, Mr. Karan is required pay ₹ 2,41,800 as minimum amount of advance tax by 31st March 2022., Question 41. Explain Special provision for computing profits and gains of profession on presumptive, basis. Section 44ADA, Answer: Special provision for computing profits and gains of profession on presumptive basis Section, 44ADA, (1) An Assessee, being an individual or a partnership firm other than a limited liability partnership who is, a resident in India, having specified profession shall be allowed to have option to compute income on, presumptive basis provided gross receipt is not exceeding ₹ 50 lakh during that year and income under the, head Business/Profession shall be presumed to be 50% of gross receipt and no further deduction shall be, allowed under the head Business/Profession., (2) The assessee shall be exempt from maintaining books of accounts., (3) Such Assessee has the option to reject presumptive income but in that case the assessee shall be required, to maintain any books of accounts and also audit is required., (4) Assessee can change the option on year to year basis.
Page 704 :
Income Under The Head Business/Profession, , 185, , (5) Brought forward business loss is allowed to be adjusted from such income but brought forward, depreciation is not allowed to be adjusted from such income., (6) Such assessee shall be required to pay advance tax to the extent of 100% of tax liability on or before 15th, March of the relevant previous year otherwise interest shall be charged @ 1% for one month on the amount, of deposit default., Illustration 21: Mr. X is engaged in specified profession and has gross receipt ₹42,00,000. He has Long, term Capital Gain ₹7,00,000 and brought forward business loss ₹30,000 of A.Y. 2018-19. He donated ₹, 20,000 to Prime Minister National Relief Fund (PMNRF) by cheque. Compute his Tax Liability for the, Assessment Year 2022-23. He has opted for Section 44ADA., Solution:, ₹, Gross Receipt, 42,00,000, Presumptive Income u/s 44ADA (50% of 42,00,000), 21,00,000, Income under the head Business Profession, 21,00,000, Less: B/F business loss, (30,000), Income under the head Business Profession, 20,70,000, Income under the head Capital Gains, 7,00,000, Gross Total Income, 27,70,000, Less: Deduction u/s 80G, (20,000), Total Income, 27,50,000, Computation of Tax Liability, Tax on ₹20,50,000 at slab rate, 4,27,500, Tax on LTCG ₹7,00,000 @ 20%, 1,40,000, Tax before health & education cess, 5,67,500, Add: HEC @ 4%, 22,700, Tax Liability, 5,90,200, Question 42 [V. Imp.]. Describe the provisions for computing profit and gains of business of Plying,, Hiring or Leasing Goods Carriages., Answer: Special provision for computing profits and gains of business of plying, hiring or leasing, goods carriages Section 44AE, 1. If any person is engaged in the business of plying, hiring or leasing goods carriages, he will have the, option to compute income under the head business/profession on presumptive basis and it will be ₹7,500 per, month or part of the month per goods carriage provided it is not a heavy goods vehicle. Heavy goods, vehicle means goods vehicle having gross weight more than 12 ton (12000 kg.) If it is a heavy goods vehicle, income shall be presume to be ₹1000 per ton of gross weight, e.g. if weight of vehicle is 14 ton (14000 kg),, income shall be ₹ 14,000 per month., If actual income is more than the presumptive income, actual income shall be taken into consideration., Assessee should not have more than 10 goods carriages at any time during the year otherwise such option, is not allowed., 2. No further deduction is allowed under section 30 to 38 but in case of a firm interest and salary to, partners is allowed as per section 40(b)., 3. The assessee shall be exempt from maintaining books of accounts or audit., 4. The assessee has the option to reject presumptive income but in that case assessee should maintain any, books of accounts and also audit is required., 5. An assessee, who is in possession of a goods carriage, whether taken on hire purchase or on instalments,, shall be deemed to be the owner of such goods carriage., 6. Assessee can change the option on year to year basis., 7. Brought forward depreciation shall not be allowed to be adjusted but brought forward business loss shall, be allowed to be adjusted., Illustration 22: Mr. X retired from Govt. service in March 2021. He got ₹20,00,000 on account of, retirement benefits. Out of the aforesaid sum, he purchased on 23rd April 2021 a few motor vehicles and got
Page 705 :
Income Under The Head Business/Profession, , 186, , their delivery on that date., The particulars of the vehicles are given below–, Vehicle, Number, Cost of the vehicle, Heavy goods vehicle (15 ton), 2, ₹9,00,000, Medium goods vehicle (8 ton), 4, ₹4,50,000, Light commercial Vehicle (4 ton), 3, ₹3,20,000, He started plying the vehicles from 04.06.2021. On an average every vehicle remains off the road for about, a week for repairs and maintenance. He maintains a rough record of the receipts and outgoings which is, given below –, Receipts, ₹3,70,000, Less: Expenses (Excluding depreciation and salaries to Mr. Y) (₹ 60,000), ₹3,10,000, You are required to compute the Total Income of Mr. X from the business of goods carriage for the previous, year 2021-22., Solution:, Computation of Business Income, As per section 44AE, Goods vehicle, (₹7,500 x 7 x 12), Heavy goods vehicle, (₹1,000 x 15 x 12 x 2), Business Income, Gross Total Income, Less: Deduction u/s 80C to 80U, Total Income, , ₹, , 6,30,000, 3,60,000, 9,90,000, 9,90,000, Nil, 9,90,000, , Illustration 23: An assessee owns a heavy commercial vehicle having gross vehicle weight of 15 ton each, for 9 months 15 days, a medium goods vehicle having gross vehicle weight of 8 ton for 9 months and a light, goods vehicle having gross vehicle weight of 5 ton for 12 months during the previous year. Compute his, income applying the provisions of section 44AE., Solution:, His profits and gains from the 3 trucks shall be deemed to be (15 x 1,000 x 10) + (₹7,500 × 9) + (₹7,500 ×, 12) = ₹3,07,500, MAY-2011 (4 Marks), Mr. X had 4 heavy goods vehicles having gross vehicle weight of 15 ton each as on 01.04.2021. He sold 2, heavy goods vehicles having gross vehicle weight of 15 ton each on 31.05.2021., He acquired 7 heavy goods vehicles having gross vehicle weight of 14 ton each on 27.06.2021., He has brought forward business loss of ₹ 50,000 relating to Assessment Year 2018-19 of a discontinued, business. Assuming that he opts for presumptive taxation of income as per section 44AE, compute his Total, Income chargeable to tax for the Assessment Year 2022-23., Solution:, Computation of Total Income of Mr. X for A.Y.2022-23, Particulars, ₹, Presumptive business income under section 44AE, 60,000, 2 heavy goods vehicles for 2 months (2 x 15 x ₹1,000 x 2), 3,60,000, Balance 2 heavy goods vehicles for 12 months (2 x 15 x ₹1,000 x 12), 9,80,000, 7 heavy goods vehicles for 10 months (7 x 14 x 1,000 x10), Business Income, 14,00,000, Less: Brought forward business loss of discontinued business, (50,000), Gross Total Income, 13,50,000, Less: Deduction u/s 80C to 80U, Nil
Page 706 :
Income Under The Head Business/Profession, Total Income, , 187, 13,50,000, , NOV-2011 (4 Marks), Mr. X (aged 38) owned 6 heavy goods vehicles having gross vehicle weight of 16 ton (16000 Kg) each as on, 01.04.2021. He acquired 2 more light goods vehicles having gross vehicle weight of 8 ton (8000 Kg) each, on 01.07.2021. He is solely engaged in the business of plying goods vehicles on hire since financial year, 2017-18., He did not opt for presumptive provision contained in section 44AE for the financial year 2020-21. His, books were audited under section 44AB and the return of income was filed on 05.08.2021. He has, unabsorbed depreciation of ₹70,000 and Business loss of ₹1,00,000 for the financial year 2020-21., Following further information is provided to you:, (i) Paid medical insurance premium of ₹23,000 for his parents (both aged above 70) by means of bank, demand draft., (ii) Paid premium on life insurance policy of his married daughter ₹25,000., (iii) Repaid principal of ₹40,000 and interest of ₹15,000 to Canara Bank towards education loan of his, daughter, who completed B.E. two years ago. She is employed after completion of her studies., Assuming that Mr. X has opted for presumptive provision contained in section 44AE of the Income-tax Act,, 1961, compute the Total Income of Mr. X for the Assessment Year 2022-23., Solution:, Computation of total income Mr. X, ₹, Presumptive income u/s 44 AE, Heavy Goods Vehicle (6 x 16 x 1,000 x 12), 11,52,000, Light Goods Vehicle (7,500 x 2 x 9), 1,35,000, Less: Business loss to be adjusted, (1,00,000), Less: Unabsorbed Depreciation, Nil, (Not allowed) (since covered u/s 32 (2)), Gross Total Income, 11,87,000, Less: Deduction u/s 80C, (25,000), Less: Deduction u/s 80D, (23,000), Less: Deduction u/s 80E, (15,000), Total Income, 11,24,000, ORDER OF SET-OFF OF LOSSES / DEPRECIATION, Order of set of losses and depreciation shall be as given below, (a) Current year expenses, (b) Current year depreciation / Current year capital expenditure on scientific research and current year, expenditure on family planning, to the extent allowed., (c) Brought forward loss from business/profession [Section 72(1)], (d) Unabsorbed depreciation [Section 32(2)], (e) Unabsorbed capital expenditure on scientific research [Section 35(4)]., (f) Unabsorbed expenditure on family planning [Section 36(1)(ix)]
Page 707 :
Income Under The Head Business/Profession, , 188, , ALTERNATE MINIMUM TAX, Section 115JC to 115JF, Alternate Minimum Tax provisions shall be applicable in case of all the assesses except a company (in case, of company minimum alternate tax is applicable)., Further such provisions shall not be applicable for individual, HUF, AOP, BOI and Artificial Juridical, persons if adjusted total income does not exceed 20 lakhs., "Alternate minimum tax" means the amount of tax computed on adjusted total income at a rate of, eighteen and one-half per cent;, "Regular income-tax" means the income-tax payable for a previous year by a person on his total income, computed in the normal manner., Adjusted total income shall be the total income as increased by—, (i) deductions claimed, if any, under any section (other than section 80P) included in Chapter VI-A, under the heading "C.—Deductions in respect of certain incomes" i.e. 80JJAA,80QQB, 80RRB etc., (ii) deduction claimed, if any, under section 10AA and, (iii) deduction claimed, if any, under section 35AD as reduced by the amount of depreciation allowable, in accordance with the provisions of section 32 as if no deduction under section 35AD was allowed, in respect of the assets on which the deduction under that section is claimed., If AMT is payable, tax credit is allowed which is AMT minus regular tax and it can be adjusted in the, subsequent years whenever regular tax is more than AMT but to the extent excess of regular tax over AMT, and carry forward is allowed for maximum 15 years starting from the year subsequent to the year to which, tax credit relates., Example: XY partnership firm has total income of ₹48,00,000 after claiming deduction u/s 10AA, ₹32,00,000, in this case tax treatment shall be as given below:, Regular income tax, Add: HEC @ 4%, Tax Payable, , (₹48,00,000 x 30%), , Alternate minimum tax, (₹48,00,000+₹32,00,000) x 18.5%, Add: HEC @ 4%, Tax payable, , 14,40,000, 57,600, 14,97,600, 14,80,000, 59,200, 15,39,200, , In the given case regular income tax payable is lower than alternate minimum tax payable hence tax shall be, payable based on alternate minimum tax., Tax credit allowed 15,39,200-14,97,600 = 41,600, If in previous year 2021-22, normal tax liability is 10,00,000 and AMT is 9,00,000, tax payable shall be, 10,00,000-41,600 = 9,58,400, But if AMT is 11,00,000, tax payable shall be 11,00,000 and tax credit allowed shall be 1,00,000 + 41,600 =, 1,41,600
Page 708 :
Income Under The Head Business/Profession, , 189, , Example: Mr X has total income of ₹300,00,000 after claiming deduction u/s 35AD ₹160,00,000, (depreciation ₹10,00,000 not claimed u/s 32), in this case applying provision of AMT, tax payable shall be, as given below:, Regular income tax (₹300,00,000 at slab rate), Add: Surcharge @ 25%, Tax before Health and education cess, Add: HEC @ 4%, Tax Payable, , 88,12,500, 22,03,125, 110,15,625, 4,40,625, 114,56,250, , Alternate minimum tax, (₹300,00,000+₹160,00,000-₹10,00,000) x 18.5%, Add: Surcharge @ 25%, Tax before cess, Add: HEC @ 4%, Tax payable, , 83,25,000, 20,81,250, 104,06,250, 4,16,250, 108,22,500, , In this case regular income tax shall be paid 114,56,250 but if there is brought forward tax credit 10,00,000,, tax payable shall be, Tax Payable, 114,56,250, Less: Tax Credit (114,56,250-108,22,500), (6,33,750), Tax Payable, 108,22,500, Example: Compute tax payable and tax credit in the following cases, Previous year, , AMT, , Tax Payable, , Tax Credit, , 2021-2022, 2022-2023, 2023-2024, , Regular Income, Tax, 10,00,000, 7,00,000, 9,20,000, , 9,00,000, 8,50,000, 8,40,000, , 2024-2025, , 15,00,000, , 14,00,000, , 2025-2026, , 12,00,000, , 14,00,000, , 10,00,000, 8,50,000, 8,40,000, (9,20,000-80,000), 14,30,000, (15,00,000-70,000), 14,00,000, , 1,50,000, 70,000, (1,50,000-80,000), (70,000-70,000), 2,00,000, , If assessee has opted for section 115BAC, provisions of AMT shall not apply., AMT provisions are not applicable if the person has not claimed any deduction under Chapter VI-A, section, 10AA and section 35AD. Also the provisions are not applicable to an individual, HUF, AOP, BOI and, artificial Juridical Person provided adjusted total income of such person does not exceed ₹20,00,000., However even in such cases, tax credit shall be allowed to be adjusted., Example: An individual for pervious year 31.03.2022 has business income of ₹30,00,000. For Previous, Year 31.03.2021 he was subject to AMT as he was claiming deduction under section 35AD. He has an AMT, credit of ₹4,00,000. During Previous Year 31.03.2022, he is not entitled to deductions under Chapter VI-A/, 10AA / 35AD., Answer:, Although AMT is not applicable to the assessee in Previous Year 31.03.2022, yet he can claim AMT credit, as per section 115JEE., Normal tax on ₹30,00,000, ₹7,41,000, Alternate Minimum Tax @ 19.24% on ₹30,00,000, ₹5,77,200, AMT credit available for set-off, ₹1,63,800, Therefore, tax payable by assessee shall be ₹5,77,200 after taking credit of AMT of ₹1,63,800. Assessee will, carry forward balance AMT of ₹2,36,200.
Page 709 :
Income Under The Head Business/Profession, , 190, , Question: Mr. X, an individual set up a unit in Special Economic Zone (SEZ) in the financial year 2017-18, for production of washing machines. The unit fulfills all the conditions of section 10AA of the Income-tax, Act, 1961. During the financial year 2020-21, he has also set up a warehousing facility in a district of Tamil, Nadu for storage of agricultural produce. It fulfills all the conditions of section 35AD. Capital expenditure in, respect of warehouse amounted to ₹75 lakhs (including cost of land ₹10 lakhs). The warehouse became, operational with effect from 1st April, 2021 and the expenditure of ₹75 lakhs was capitalized in the books, on that date., Relevant details for the financial year 2021-22 are as follows:, Particulars, ₹, Profit of unit located in SEZ, 40,00,000, Export sales of above unit, 80,00,000, Domestic sales of above unit, 20,00,000, Profit from operation of warehousing facility, 1,05,00,000, (before considering deduction under Section 35AD), Compute income-tax (including AMT under Section 115JC) liability of Mr. X for Assessment Year 2022-23, both as per regular provisions of the Income-tax Act and as per section 115BAC for Assessment Year 202223. Advise Mr. X whether he should opt for section 115BAC., Answer:, Computation of total income and tax liability of Mr. X for A.Y.2022-23, (under the regular provisions of the Income-tax Act, 1961), ₹, ₹, Profits and gains of business or profession, Profit from unit in SEZ, 40,00,000, Less: Deduction u/s 10AA [See Note (1) below], 32,00,000, Business income of SEZ unit chargeable to tax, 8,00,000, Profit from operation of warehousing facility, 1,05,00,000, Less: Deduction u/s 35AD [See Note (2) below], 65,00,000, Business income of warehousing facility chargeable to tax, 40,00,000, Total Income, 48,00,000, Computation of tax liability (under the normal/regular provisions), Tax on ₹48,00,000, 12,52,500, Add: Health and Education cess @ 4%, 50,100, Total tax liability, 13,02,600, Computation of adjusted total income of Mr. X for levy of Alternate, Minimum Tax, Particulars, ₹, ₹, Total Income (computed above as per regular provisions of income tax), 48,00,000, Add: Deduction under section 10AA, 32,00,000, 80,00,000, Add: Deduction under section 35AD, 65,00,000, Less: Depreciation under section 32, On building @10% of ₹65 lakhs, 6,50,000, 58,50,000, Adjusted Total Income, 1,38,50,000, Alternate Minimum Tax @ 18.5%, 25,62,250, Add: Surcharge@15% (since adjusted total income > ₹1 crore), 3,84,338, 29,46,588, Add: Health and Education cess@4%, 1,17,863, 30,64,451, Tax liability u/s 115JC (rounded off), 30,64,450, Since the regular income-tax payable is less than the alternate minimum tax payable, the adjusted total, income shall be deemed to be the total income and tax is leviable @18.5% thereof plus surcharge@15% and, cess@4%. Therefore, tax liability as per section 115JC is ₹30,64,450.
Page 710 :
Income Under The Head Business/Profession, , 191, , Computation of total income and tax liability of Mr. X for A.Y.2022-23, (under the provisions of section 115BAC of the Income-tax Act, 1961), Particulars, ₹, Total Income (as computed above as per regular provisions of income tax), Add: Deduction under section 10AA (not allowable), Add: Deduction under section 35AD, Less: Depreciation under section 32, On building @10% of ₹65 lakhs (normal depreciation under, section 32 is allowable), Total Income, Computation of tax liability as per section 115BAC, Tax on ₹1,38,50,000, Add: Surcharge@15%, , ₹, 48,00,000, 32,00,000, 80,00,000, , 65,00,000, 6,50,000, , 58,50,000, 1,38,50,000, 38,92,500, 5,83,875, 44,76,375, 1,79,055, 46,55,430, , Add: Health and Education cess@4%, Total tax liability, Notes:, (1) Deductions u/s 10AA and 35AD are not allowable as per section 115BAC(2). However, normal, depreciation u/s 32 is allowable., (2) Individuals or HUFs exercising option u/s 115BAC are not liable to alternate minimum tax u/s 115JC., Since the tax liability of Mr. X under section 115JC is lower than the tax liability as computed u/s 115BAC,, it would be beneficial for him not to opt for section 115BAC for A.Y. 2022-23. Moreover, benefit of, alternate minimum tax credit is also available to the extent of tax paid in excess over regular tax., AMT Credit to be carried forward under section 115JEE, ₹, Tax liability under section 115JC, 30,64,450, Less: Tax liability under the regular provisions of the Income tax Act, 1961, 13,02,600, 17,61,850, Notes:, (1) Deduction under section 10AA in respect of Unit in SEZ =, Profit of the Unit in SEZ × Export turnover of the Unit in SEZ / Total turnover of the Unit in SEZ, ₹ 40,00,000 × ₹ 80,00,000 / ₹ 1,00,00,000 = ₹ 32,00,000, (2) Deduction@100% of the capital expenditure is available under section 35AD for A.Y.2022-23 in respect, of specified business of setting up and operating a warehousing facility for storage of agricultural produce, which commences operation on or after 01.04.2009., Further, the expenditure incurred, wholly and exclusively, for the purposes of such specified business, shall, be allowed as deduction during the previous year in which he commences operations of his specified, business if the expenditure is incurred prior to the commencement of its operations and the amount is, capitalized in the books of account of the assessee on the date of commencement of its operations., Deduction under section 35AD would, however, not be available on expenditure incurred on acquisition of, land., In this case, since the capital expenditure of ₹65 lakhs (i.e., ₹75 lakhs – ₹10 lakhs, being expenditure on, acquisition of land) has been incurred in the F.Y.2020-21 and capitalized in the books of account on, 1.4.2021, being the date when the warehouse became operational, ₹65,00,000, being 100% of ₹65 lakhs, would qualify for deduction under section 35AD.
Page 711 :
Income Under The Head Business/Profession, , 192, , NOV 2020 (NEW COURSE), Question 1, (14 Marks), From the following particulars furnished by Mr. Ganesh, aged 58 years, a resident Indian for the previous, year ended 31.03.2022, you are requested to compute his total income and tax liability under normal as well, as special provisions (AMT), if any, applicable to him for the Assessment Year 2022-23., (i) He occupies ground floor of his residential building and has let out first floor for residential use at an, annual rent of ₹2,28,000. He has paid municipal taxes of ₹60,000 for the current financial year., (ii) He owns an industrial undertaking established in a SEZ and which has commenced operation during the, financial year 2019-20. Total turnover of the undertaking was ₹200 lakhs, which includes ₹140 lakhs from, the export turnover. This industrial undertaking fulfills all the conditions of section 10AA of the Income-Tax, Act,1961. Profit from this industry is ₹25 lakhs., (iii) He received royalty of ₹2,88,000 from abroad for a book authored by him on the nature of artistic. The, rate of royalty as 18% of value of books and expenditure made for earning this royalty was ₹40,000. The, amount remitted to India till 30th September, 2022 is ₹2,30,000, (iv) Received 40,000 as interest on saving bank deposits., (v) Received 47,000 as share of profit from an AOP where all the members are individual and which had, paid the tax by normal rates of income tax., (vi) He also sold his vacant land on 10.11.2021 for ₹10 lakhs. The stamp duty value of land as on, 01.04.2001 was ₹4 lakhs. This land was acquired by him on 05.08.1995 for ₹1.80 lakhs. He had incurred, registration expenses of ₹10,000 at that time., The cost of inflation index for the year 2021-22 and 2001-02 are 317 and 100 respectively., (vii) He paid the following amounts, out of his taxable income:, (a) Insurance premium of ₹39,000 paid on life insurance policy of son, who is not dependent on him., (b) Insurance premium of ₹48,000 on policy of his dependent father,, (c) Tuition fees of ₹42,000 for his three children to a school. The fees being ₹14,000 p.a. per child., Solution:, Income under the head house property, Let out House, Gross annual value (Rent Received is taken as GAV), 2,28,000, Less: Municipal Taxes (60,000/2), (30,000), Net Annual value, 1,98,000, Less: standard deduction @ 30% u/s 24(a), (59,400), Less: Interest on capital borrowed u/s 24(b), Nil, Income under the head house property, 1,38,600, Self - occupied house, Gross annual value, Less: Municipal Taxes (not allowed in case of self-occupied house), Net Annual value, Less: standard deduction @ 30% u/s 24(a), Less: Interest on capital borrowed u/s 24(b), Income under the head house property, Income under the head Business/Profession, Profit from Industry, , Nil, Nil, Nil, Nil, Nil, Nil, , 25,00,000
Page 712 :
Income Under The Head Business/Profession, , 193, , Income under the head capital gains, Full value of consideration (as per section 50C), 14,00,000, (Since stamp duty value is exceeding 5% of actual consideration hence Stamp duty value, shall be actual consideration), Less: Indexed cost of Acquisition, (12,68,000), (4,00,000/100 X 317) (higher of actual cost and FMV as on 01.04.2001 shall be taken as COA), Long term capital gains (held for more than 24 months), 1,32,000, Income under the head other sources, Royalty income, Less: Expense on royalty, Royalty income, Interest on saving bank, Income under the head other sources, Gross Total Income, Less: Deduction u/s 10AA, (25,00,000/200,00,000 X 1,40,00,000) X 100%, Less: Deductions u/s 80C - Insurance premium of son, Tuition fee Paid (Maximum of 2 children allowed), Less: Deductions u/s 80QQB, Working Note:, (i) 15% of value of books, 2,40,000, (2,88,000/18% x 15%), but cannot exceed amount received within 6 months, from the end of the previous year i.e. 2,30,000, Allowed, 2,30,000, Less: Expenses, (40,000), Deduction allowed, 1,90,000, Less: Deductions u/s 80TTA, Total Income, Computation of Tax Liability, Tax on 1,32,000 @ 20% u/s 112, Tax on balance 9,09,600 at slab rate, Tax before Health and education cess, Add: HEC @4%, Tax Liability, Rounded off u/s 288B, , 2,88,000, (40,000), 2,48,000, 40,000, 2,88,000, 30,58,600, (17,50,000), (39,000), (28,000), (1,90,000), , (10,000), 10,41,600, , 26,400.00, 94,420.00, 1,20,820.00, 4,832.80, 1,25,652.80, 1,25,650.00, , Notes:, 1. Taxability of BOI/AOP is not covered in syllabus, hence student can give any treatment. The above, solution is given without taking into consideration the share from AOP. The provisions of AOP/BOI, are given under section 67A/86/110/167B
Page 713 :
Income Under The Head Business/Profession, , 194, , 2. As per section 80C, Insurance Premium of son is allowed but for father is not allowed. It is not, specified whether policy of father is life policy or Mediclaim policy. It is presumed that it is life, policy., 3. As per section 80C, Tuition fee shall be allowed for maximum of 2 children., 4. Municipal tax paid for self-occupied portion is not allowed., 5. As per section 10AA, Industrial undertaking was started in 2019-20 and for first five year assessee, can claim 100% of export profit as exemption u/s 10AA. Export profit means Total Profit/Total, Turnover x Export turnover., AMT Provisions Section 115JC, Calculation of Adjusted GTI, Total income, Add: Deduction u/s 10AA, Add: Deduction u/s 80QQB, Adjusted GTI, , 11,05,600.00, 17,50,000.00, 1,90,000.00, 30,45,600.00, , Since Adjusted GTI is exceeding 20,00,000 hence AMT provisions shall be applicable., Calculation of Tax as per AMT, Adjusted GTI, 30,45,600.00, Tax @ 18.5%, 5,63,436.00, HEC @ 4%, 22,537.44, Tax Liability, 5,85,973.44, Rounded off u/s 288B, 5,85,970.00, Since regular income tax payable is less than the alternate tax, hence assessee have to pay tax as per, alternate minimum tax i.e. 5,85,970., , RTP NOV 20, Mr. Suraj aged 50 years, a resident individual, engaged in a wholesale business of health products. He is also, a partner in XYZ & Co., a partnership firm. The following details are made available for the year ended, 31.3.2022:, Sl. No., Particulars, (i), Interest on capital received from XYZ & Co., at 15% [in accordance, with the partnership deed], (ii), Share of profit from the firm, (iii), Salary as working partner (fully allowed in the hands of the firm), (iv), Interest from bank on fixed deposit (Net of TDS), (v), Interest on saving bank account, (vi), Income-tax refund received relating to assessment year 2021-22, including interest of ₹ 2,300, (vii), Net profit from wholesale business, Amounts debited include the following:, - Depreciation as per books, - Motor car expenses, - Municipal taxes for the shop, (For two half years; payment for one half year made on 12.7.2022 and, , ₹, , ₹, 1,50,000, 35,000, 1,00,000, 40,500, 12,300, 34,500, 5,60,000, , 34,000, 40,000, 7,000
Page 714 :
Income Under The Head Business/Profession, , 195, , for the other on 31.12.2022), Salary to manager by way of a single cash payment, 21,000, (viii) The WDV of the assets (as on 1.4.2021) used in above wholesale, business is as under:, - Computers, 2,40,000, - Computer printer, 1,50,000, (ix), Motor car acquired on 31.12.2021 (20% used for personal use), 6,80,000, (x), He owned a house property in Mumbai which was sold in January,, 1,15,000, 2016. He received arrears of rent in respect of the said property in, October, 2021., (xi), LIP paid for independent son, 60,000, (xii), PPF of his wife, 70,000, (xiii) Health insurance premium paid towards a policy covering her mother, 35,000, aged 75 by way of cheque. She is not dependents on him., (xiv) Contribution toward Prime Minister National Relief Fund, 50,000, You are required to compute the total income of the Mr. Suraj for the assessment year 2022-23 and the, closing WDV of each block of assets., Answer:, Computation of total income of Mr. Suraj for the A.Y.2022-23, Particulars, ₹, ₹, Income from house property, Arrears of rent, 1,15,000, (taxable under section 25A even if Mr. Suraj is not the owner of the house, property in the P.Y.2020-21), 34,500, 80,500, Income chargeable under this head, Profits and gains of business or profession, Income from wholesale business, 5,60,000, Net profit as per books, Add: Amount debited to P & L A/c, not allowable as deduction, 34,000, - Depreciation as per books, - Disallowance of municipal taxes paid for the second half- year under section, 3,500, 43B, since the same was paid after the due date of filing of return (₹ 7,000/2), - Disallowance under section 40A(3) in respect of salary paid in cash since the, 21,000, same exceeds ₹ 10,000, 8,000, - 20% of car expenses for personal use, 6,26,500, Less: Depreciation allowable (Note 1), 1,96,800, 4,29,700, Income from firm, Share of profit from the firm is exempt under section 10(2A), –, Interest on capital from partnership firm (Note 2), 1,20,000, Salary as working as working partner full taxable, 1,00,000, 2,20,000 6,49,700, Income from other sources, Interest on bank fixed deposit (Gross), 45,000, Interest on saving bank account, 12,300, Interest on income-tax refund, 2,300, 59,600
Page 715 :
Income Under The Head Business/Profession, , 196, , Gross Total Income, Less: Deduction under Chapter VIA (Note 3), , 7,89,800, 2,25,000, 5,64,500, , Notes:, (1) Depreciation allowable under the Income-tax Rules, 1962, Opening Rate, WDV, Block 1 Computers, 2,40,000 40%, Computer Printer, 1,50,000 40%, Block 2 Motor Car, 6,80,000 15% 51,000 [50% of 15% is, allowable, since it is, put to use for less than, 180 days], , Less: 20% disallowance for personal use, , Depreciation, 96,000, 60,000, 40,800, , Closing, WDV, 1,44,000, 90,000, 6,39,200, , 1,96,800, , 8,73,200, , 10,200, , (2) Only to the extent the interest is allowed as deduction in the hands of the firm, the same is includible as, business income in the hands of the partner. Since interest is paid in accordance with partnership deed,, maximum interest allowable as deduction in the hands of the firm is 12% p.a. Therefore, interest @12% p.a., amounting to ₹ 1,20,000 would be treated as the business income of Mr. Suraj., (3) Deduction under Chapter VI-A, Particulars, Under section 80C, LIP for independent son, PPF paid in wife’s name, Since the maximum deduction under section 80C and 80CCE is ₹ 1,50,000,, the entire sum of ₹ 1,30,000 would be allowed as deduction, Under Section 80D, Health insurance premium taken for mother is fully allowable as deduction,, even though she is not dependent on him. Since she is senior citizen whole of, amount is allowable as deduction as it is within overall limit of ₹ 50,000, Under Section 80G, Contribution towards PM National Relief Fund eligible for 100% deduction, without any qualifying limit, Under section 80TTA, Interest on saving bank account, restricted to, Total deduction, , ₹, , ₹, , 60,000, 70,000, 1,30,000, 1,30,000, 35,000, , 50,000, , 10,000, 2,25,000
Page 716 :
Income Under The Head Business/Profession, , 197, , MULTIPLE CHOICE QUESTIONS, 1. An assessee uses plant and machinery for the purpose of carrying on his business. Under section 31,, he shall be eligible for deduction on account of(a) both capital and revenue expenditure on repairs, (b) current repairs, (c) current repairs plus 1/5th of capital expenditure on repairs., (d) both (a) & (b), 2. An electricity company charging depreciation on straight line method on each asset separately, sells, one of its machinery in April, 2021 at ₹1,20,000. The WDV of the machinery at the beginning of the, year i.e. on 1st April, 2021 is ₹1,35,000. No new machinery was purchased during the year. The, shortfall of ₹15,000 is treated as (a) Terminal depreciation, (b) Short-term capital loss, (c) Normal depreciation., (d) Any of the above, at the option of the assessee, 3. Mr. X, acquires an asset which was previously used for scientific research for ₹2,75,000. Deduction, under section 35(1)(iv) was claimed in the previous year 2016-17. The asset was brought into use for, the business of Mr. X, after the research was completed. The actual cost of the asset to be included in, the block of assets is (a) Nil, (b) Market value of the asset on the date of transfer to business, (c) ₹2,75,000 less notional depreciation under section 32 upto the date of transfer., (d) Actual cost of the asset i.e., ₹2,75,000, 4. A Ltd. has unabsorbed depreciation of ₹4,50,000 for the P.Y.2021-22. This can be carried forward(a) for a maximum period of 8 years and set-off against business income., (b) Indefinitely and set-off against business income., (c) Indefinitely and set-off against any head of income, (d) Indefinitely and set-off against any head of income except casual income., 5. Mr. X, a retailer acquired furniture on 10th May 2021 for ₹10,000 in cash and on 15th May 2021,, for ₹15,000 and ₹20,000 by a bearer cheque and account payee cheque, respectively. Depreciation, allowable for A.Y. 2022-23 would be –, (a) ₹2,000, (b) ₹3,000, (c) ₹3,500, (d) ₹4,500, 6. XYZ Ltd. incurred capital expenditure of ₹1,50,000 on 1.4.2021 for acquisition of patents and, copyrights. Such expenditure is (a) Eligible for deduction in 14 years from A.Y.2022-23, (b) Eligible for deduction in 5 years from A.Y. 2022-23, (c) Subject to depreciation @ 25% under section 32, (d) Subject to depreciation @ 15% under section 32, 7. Under section 44AE, presumptive taxation is applicable at a particular rate provided the assessee is, the owner of a maximum of certain number of goods carriages. The rate per month or part of the, month relevant for A.Y.2022- 23 and the maximum number specified under the section are (a) ₹7,500 for each goods carriage in the case of an assessee owning not more than 10 goods carriages at any, time during the year, (b) ₹7,500 for each goods carriage in the case of an assessee owning less than 10 goods carriages at any time, during the year, (c) ₹1,000 per ton of gross vehicle weight for per month or part of a month for a goods carriage for an, assessee owning not more than 10 goods carriages at the end of the previous year
Page 717 :
Income Under The Head Business/Profession, , 198, , (d) ₹1,000 per ton of gross vehicle weight or unladen weight, as the case may be, for per month or part of a, month for a heavy goods carriage and ₹7,500 per month or part of a month for other goods carriages in the, case of an assessee owning not more than 10 goods carriages at any time during the previous year, 8. Where the total turnover of an assessee, eligible for presumptive taxation u/s 44AD, is received, entirely by account payee cheque during the previous year 2021-22, the specified rate of presumptive, business income is (a) 5% of total turnover, (b) 6% of total turnover, (c) 7% of total turnover, (d) 8% of total turnover, 9. The W.D.V. of a block (Plant and Machinery, rate of depreciation 15%) as on 1.4.2021 is ₹3,20,000., A second hand ‘machinery costing ₹50,000 was acquired on 1.9.2021 but put to use on 1.11.2021., During Jan 2022, part of this block was sold for ₹2,00,000. The depreciation for A.Y.2022-23 would be, (a) ₹21,750, (b) ₹25,500, (c) ₹21,125, (d) ₹12,750, 10. Employer’s contribution to provident fund/superannuation fund/gratuity fund is allowed as, deduction in computing income under the head “Profits and gains of business or profession”,, provided it has been paid (a) before the end of the previous year, (b) on or before the due date by which the employer is required to credit an employee’s contribution to the, employee’s account in the relevant fund., (c) on or before the due date for filing the return of income under section 139(1)., (d) before the end of the relevant assessment year, 11. The benefit of payment of advance tax in one installment on or before 15th March is available to, assesses computing profits on presumptive basis –, (a) under section 44AD, (b) under section 44AD and 44ADA, (c) under section 44AD and 44AE, (d) under section 44AD, 44ADA and 44AE, 12. M/S Thakural & Sons, paid ₹11,00,000 as remuneration to its partner. The same was in, accordance with partnership deed. Partners are also entitled to interest on capital @ 11% as per, partnership deed. Total interest paid during the year is ₹1,30,000. The book profit before interest on, capital and remuneration is ₹37,00,000. The salary allowable as deduction to M/S Thakural & Sons is:, (a) ₹22,62,000, (b) ₹11,00,000, (c) ₹23,10,000, (d) ₹22,32,000, 13. An interior decorator has opted for presumptive taxation scheme under section 44ADA for A.Y., 2022-23. He is liable to pay advance tax (a) In one instalment, (b) In two instalments, (c) In three instalments, (d) In four instalments, 14. Any loss from the specified business referred to in section 35AD can be set off against (a) only profit and gains of same specified business of the assessee, (b) profits and gains of any business of the assessee, (c) profit and gains of any other specified business of the assessee, (d) income from any other head, 15. According to section 80, no loss which has not been determined in pursuance of a return filed in, accordance with the provisions of section 139(3), shall be carried forward. The exceptions to this are -
Page 718 :
Income Under The Head Business/Profession, , 199, , (a) Loss from specified business under section 73A, (b) Loss under the head “Capital Gains” and unabsorbed depreciation carried forward under section 32(2), (c) Loss from house property and unabsorbed depreciation carried forward under section 32(2), (d) Loss from speculation business under section 73, 16. Section 70 enables set off of losses under one source of income against income from any other, source under the same head. The exceptions to this section are (a) Loss under the head “Capital Gains”, Loss from speculative business, Loss from house property and loss, from the activity of owning and maintaining race horses, (b) Long-term capital loss, Loss from speculative business, Loss from specified business and loss from the, activity of owning and maintaining race horses, (c) Short-term capital loss and loss from speculative business, (d) Loss from specified business and short-term capital loss, 17. The maximum period for which loss from specified business can be carried forward is(a) 4 years, (b) 8 years, (c) indefinitely, (d) not allowed to be carry forward, 18. Which of the following is not taxable under the head business profession(i) Income from Speculation Business, (ii) Payments for not pursuing any business activity or profession/non-compete fee, (iii) Gift in connection with business/profession, (iv) Gift received by member of HUF from HUF, (a) all the above, (b) only (iii) & (iv), (c) only (iii), (d) None of the above, (e) only (ii), (iii) & (iv), (f) only (iv), 19. Additional depreciation shall be allowed @ _______% to all the assessee in connection with plant, and machinery for the purpose of manufacturing and also to the assessee engaged in generation,, transmission or distribution of electricity., (a) 15%, (b) 10%, (c) 30%, (d) 20%, 20. If the asset is purchased and put to use for less than _______ days, additional depreciation shall be, allowed at ______% and remaining additional depreciation shall be allowed in the subsequent year., (a) 180,15%, (b) 180,10%, (c) 182,10%, (d) 180,20%, 21. Additional depreciation is not allowed in the following cases:, (i) Second hand plant and machinery i.e. plant and machinery should be brand new, (ii) Any machinery or plant installed in any office premises or any residential accommodation,, including accommodation in the nature of a guest-house or, (iii) Any office appliances or road transport vehicles or ships and aircraft, (iv) Any machinery or plant, the actual cost of which has not been debited to profit and loss account., (a) all the above, (b) only (i),(ii) & (iv), (c) only (i), (ii) & (iii), (d) None of the above, (e) only (ii), (iii) & (iv), (f) only (i), (iii) & (iv), (g) only (iv)
Page 719 :
Income Under The Head Business/Profession, , 200, , 22. Which of the following is correct, (a) As per section 32, set off of depreciation or loss is compulsory and not voluntary., (b) As per section 32, set off of depreciation or loss is not compulsory but voluntary., (c) As per section 32, set off of depreciation or loss is compulsory and voluntary., (d) As per section 32, set off of depreciation or loss is not compulsory., (e) None of the above is correct, 23. Which of the following is not covered under section 35AD, (i) Cold chain facility for storing agricultural produce, meat and meat products, poultry and dairy products, etc., (ii) Warehousing facility for storage of agricultural produce., (iii) Hospitals with at least one hundred beds for patients., (iv) Housing project under a scheme for affordable housing., (v) Warehousing facility for storage of non - agricultural produce., (vi) Hospitals with at least fifty beds for patients., (a) all the above, (b) only (i),(ii) & (iv), (c) only (i), (ii) & (iii), (d) None of the above, (e) only (ii), (iii) & (iv), (f) only (v) & (vi), (g) only (v), (h) only (vi), 24. Which Statement from the following is correct(a) As per section 40(b), interest to the partner is allowed but maximum @ 12% p.a. simple interest., (b) As per section 40(b), interest to the partner is allowed but maximum @ 18% p.a. simple interest., (c) As per section 40(b), interest to the partner is allowed but maximum @ 12% per month simple interest., (d) As per section 40(b), interest to the partner is allowed but maximum @ 15% p.a. simple interest., 25. A partnership firm has book profits of ₹5 lakhs, in this case maximum amount of salary etc., allowed to all the partners shall be, (a) 5,00,000, (b) 3,00,000, (c) 3,90,000, (d) 4,50,000, 26. Which Statement from the following is correct(a) If any partner has received share out of the profits of the partnership firm, such share shall be exempt, from income tax., (b) If any partner has received share out of the profits of the partnership firm, such share shall not be exempt, from income tax., (c) interest or salary received by a partner shall be taxable under the head salary, (d) There is a partnership firm engaged in business and its book profits are ₹1,35,000, in this case maximum, amount of remuneration allowed to all the partners shall be ₹1,35,000., 27. Mr. X purchases goods worth ₹75,000 on 01.01.2022 and payment was made ₹60,000 on 03.01.2022, by account payee cheque and ₹8,000 in cash on 03.01.2022 and ₹7,000 in cash on 05.01.2022, in this, case expenditure allowed shall be., (a) ₹75,000, (b) ₹60,000, (c) ₹68,000, (d) Nil, 28. Which Statement from the following is correct(a) Where a stock in trade is converted into capital assets then fair market value of the stock shall be treated, as actual cost of the asset., (b) Where a stock in trade is converted into capital assets then actual purchase cost of the stock shall be, treated as actual cost of the asset.
Page 720 :
Income Under The Head Business/Profession, , 201, , (c) Where a stock in trade is converted into capital assets then written down value of the stock shall be, treated as actual cost of the asset on the date of conversion., (d) None of the above., 29. A car purchased by Dr. Ramesh on 10.08.2019 for ₹5,25,000 for the personal use is brought into, professional use on 01.07.2021 by him, when its market value was ₹2,50,000. Depreciation for the, assessment year 2022-23 shall be, (a) ₹37,500, (b) ₹78,750, (c) Nil, (d) 52,500, 30. Mr. X is engaged in a business with turnover ₹150,00,000 (all payments received by account payee, cheque, bank draft or through electronic clearing) and expenses incurred shall be ₹1,40,00,000,, presumptive income shall be, (a) ₹10,00,000, (b) ₹9,00,000, (c) ₹12,00,000, (d) ₹11,00,000, Answer, 1. (b); 2. (a); 3.(a); 4. (d); 5. (b); 6. (c); 7. (d); 8. (b); 9. (a); 10. (c); 11. (b); 12. (b); 13. (a); 14.(c); 15. (c);, 16. (b); 17. (c); 18. (f); 19. (d); 20. (b); 21. (c); 22. (a); 23. (f); 24. (a); 25. (c); 26. (a); 27. (a); 28. (a); 29., (b); 30. (b)
Page 721 :
Income Under The Head Business/Profession, , 202, , PRACTICE PROBLEMS, TOTAL PROBLEMS 40, Problem 1., G Ltd. furnishes you the following information:, Block I: Plant and machinery (consisting of 3 plants), rate of depreciation 15%., w.d.v. on April 1st, 2021: ₹2,70,000., Block II: Buildings (two buildings), rate of depreciation 10%, w.d.v. on April 1st, 2021 ₹6,50,000., Acquired on June 2nd, 2021, 2 plants for ₹ 2,10,000 and put to use on the same date., Sold on November 30th, 2021 all the five plants for ₹ 5,00,000., Acquired on December 15th, 2021 two plants for ₹ 1,60,000 and put to use on the same date., Admissible rate of depreciation in relation to all acquired plants is 15%., Compute the amount of depreciation admissible to G Ltd. for the Assessment Year 2022-23., Answer = ₹75,500, Problem 2., X Ltd. is a manufacturing company. On April 1st, 2021, it owns plant A and plant B (depreciation rate: 15, per cent; depreciated value of block being ₹2,40,000). Plant C (depreciation rate: 15 per cent) is purchased, by the company on June 10th, 2021 for ₹60,000 and it was used in the office premises. It is put to use on the, same day., Find out the tax consequences in the following different situations:, 1. Plant B is destroyed by fire on January 25th, 2022. ₹ 10,000, being the compensation, is paid by the, insurance company on February 10th, 2022;, 2. If the insurance compensation in situation (1) is ₹ 3,70,000;, 3. Plant A, B And C is destroyed by fire on January 25th, 2022. Compensation paid by insurance, company on February 10th, 2022 is ₹ 20,000;, 4. If the insurance compensation in situation (3) is ₹ 4 lakhs., Answer =, Situation 1: Depreciation: ₹43,500 and Short term capital gain/loss: Nil;, Situation 2: Depreciation: Nil and Short term capital gain: ₹70,000;, Situation 3: Depreciation: Nil and Short term capital Loss: ₹2,80,000;, Situation 4: Depreciation: Nil and Short term capital gain: ₹1,00,000, Problem 3., The following is the receipts and payments account of a medical practitioner for the year ending 31.03.2022., Receipts, Amount, Payments, Amount, ₹, ₹, Balance b/d, 1,30,000 Clinic expenses, 1,24,000, Visiting fees, 45,75,000 Medical books purchased and put to, 15,000, use on 01.07.2021, Consultation fees, 9,15,000 Surgical equipment, 90,000, Sale of medicines, 28,000 Motor car expenses, 36,000, Payment received for using Operation, 18,000 Indian, Medical, Association, 7,000, Theatre, membership fees, Dividend from domestic company, 22,000 Payment to C.A. firm for filing, 4,000, return of income, Bank loan for purchasing a flat, 2,00,000 Entertainment expenses, 24,000, Life insurance policy (maturity proceeds), 1,00,000 Medical purchases, 33,000, Rental income from flat, 60,500 Purchase of flats, 2,80,000, Bank interest on loan, 30,000
Page 722 :
Income Under The Head Business/Profession, Balance c/d, 60,48,500, , 203, 54,05,500, 60,48,500, , Additional information:, 1. A cash payment of ₹75,000 was given to him by a patient in appreciation of his medical services but was, not recorded in books., 2. Flat was purchased on 01.04.2021 and was self occupied for residence for a month from the date of its, purchase. Thereafter it was let out @ ₹5,500 p.m., the municipal value of the flats is ₹66,000 p.a. and, municipal taxes assessed, though not paid, is ₹4,500., 3. One–third of motor car expenses relate to his personal use. Depreciation on car allowable under Income, Tax Act for professional use is ₹12,000., 4. The rate of depreciation on surgical equipment is 15%. The written down value of equipment on, 01.04.2021 is ₹60,000. He sold some of the equipment for ₹30,000 during the year. New equipment was, purchased on 01.11.2021 for ₹90,000 and was put to use on the same date., Compute his Total Income and Tax Liability for the Assessment Year 2022-23., Answer = Total Income: ₹54,03,950; Tax Liability: ₹16,40,140, Problem 4., Mr. X is an advocate in Delhi High Court. He keeps his books on cash basis. His receipts and payments, account for the financial year 2021-22 is given below:, Receipts, Amount, Payments, Amount, ₹, ₹, Balance b/d, 49,200 Rent Paid for Building, 1,44,000, Consultancy fee, 55,35,500 Office expenses, 46,000, Remuneration from university as, 7,000 New car purchased and put to use on, 3,00,000, evaluator of LLB exams, 01.05.2021, 5,00,000 Computer purchased and put to use on, 50,000, Sale proceeds of residential house (it, 01.04.2021, was purchased on 01.07.2020 for, ₹3,00,000), Salary from law faculty for working, 45,000 Legal books purchased, 30,000, as part time lecturer, Car expenses, 42,000, Advance Income tax paid, 22,000, Electricity and water charges for the, 16,000, entire house, Son’s college tuition fee paid, 54,000, Gift to daughter, 25,000, Life insurance premium paid on own life, 12,000, (sum assured ₹50,000), Balance c/f, 53,95,700, 61,36,700, 61,36,700, Additional information:, 1. On 31.03.2022 legal fees outstanding amounted to ₹22,000, 2. Rent is payable @ ₹12,000 p.m., 3. 70% of the use of the car is for official purpose and 30% for personal purpose., 4. Legal books for ₹12,000 was purchased on 01.05.2021 and put to use on the same date and for ₹18,000, on 01.11.2021 and put to use on the same date., 5. Half of the house taken on rent is being used for residential purposes., Compute the Total Income and Tax Payable of Mr. X for the Assessment Year 2022-23., Answer = Total Income: ₹54,68,200; Tax payable: ₹16,40,190;
Page 723 :
Income Under The Head Business/Profession, , 204, , Problem 5., Mr. X is a Chartered Accountant and has prepared the following income and expenditure account as on, 31.03.2022., Income and Expenditure Account, Expenditure, Amount, Income, Amount, ₹, ₹, Office expenses, 12,000 Professional fee, 65,00,000, Employee’s salary, 20,000 Consultancy Fee, 55,000, Magazines and newspapers, 800 Dividend from Indian co., 8,500, Entertainment Expenses (Personal), 17,500 Profit on sale of debentures (STCG), 8,450, Donation for a charity show, 600 Gift from father in-law, 6,050, Interest on loan for professional purpose, 800, Income Tax (advance tax), 5,000, Car Expenses, 2,500, Purchase of books, 2,000, Stationery, 21,000, Diwali gift to employees, 1,000, Rent of own building, 60,000, Municipal tax, 1,000, White washing and Painting of building, 2,000, Expenses incurred on the Opening, 3,000, ceremony (refreshments), Net profit, 64,28,800, 65,78,000, 65,78,000, You are required to compute his Total Income and Tax Liability for the Assessment Year 2022-23, considering the following points –, 1. The car is used equally for official and personal purposes., 2. ₹ 1,500 for domestic servant’s salary is included in employee’s salary., 3. Books were purchased on 01.09.2021 and were put to use on the same date., 4. Payment of stationery ₹20,500 was made by a bearer cheque and ₹ 500 was paid in cash., 5. Mr. X is owner of a building. Its written down value is ₹ 90,000 on 01.04.2021. The building is used for, official purposes. No depreciation is claimed., 6. Furniture having written down value of ₹ 30,000 as on 01.04.2021 is also used for profession. Office, chairs and tables were purchased and put to use on 30.03.2022 for the purpose of a new office which has, been inaugurated on 31.03.2022. No depreciation has been debited to the profit and loss account. Actual, cost ₹20,000, 7. Employee’s salary includes bonus of ₹5,000 which was paid to one of the employees on 01.07.2022., Answer = Total Income: ₹65,17,800; Tax Liability: ₹20,22,410, Problem 6., The following is the profit and loss account of Mr. X for the Assessment Year 2022-23., Particulars, Amount, Particulars, (Debits), ₹, (Credits), Opening stock, 25,000 Sales, Purchases, 100,60,000 Closing Stock, Wages, 12,000 Gift from brother, Rent, 7,000 Income tax refund, Repairs of car, 500, Income tax paid, 4,000, Medical expenses, 1,500, , Amount, ₹, 104,96,000, 30,000, 10,000, 3,000
Page 724 :
Income Under The Head Business/Profession, Depreciation of car, Net Profit, , 205, , 3,000, 4,26,000, 105,39,000, , 105,39,000, Following further information is given:, 1. Mr. X bought one air conditioner for ₹ 25,000 on 01.05.2021 and it was put to use on the same date and, no depreciation was claimed by him., 2. Medical expenses were incurred for treatment of Mrs. X and also it includes premium of ₹ 300 of medi, claim policy taken in the name of one of employees and the payment was made by a cheque., 3. Wages include ₹2,500 on account of Mr. X’s salary., 4. Opening and closing stock are overvalued by 5%., 5. Sales include a sale of ₹50,000 being goods withdrawn by Mr. X (Cost price ₹45,000, market price, ₹47,000)., Mr. X was employed in a private firm upto 30.06.2021 and was getting a salary of ₹6,000 p.m. and his, employer has not yet paid salary for the month of June 2021., He has paid tuition fees of ₹200 p.m. per child for his two children to a public school., Mr. X has not opted for presumptive taxation of Income u/s 44AD. Compute his Total Income and also his, Tax Liability for Assessment Year 2022-23., Answer = Total Income: ₹4,06,910; Tax Liability: Nil, Problem 7., Mr. X (age 79 years) is running a shop at Chandni Chowk and has submitted the following profit and loss, account for the Assessment Year 2022-23., Particulars, Amount, Particulars, Amount, (Debits), ₹, (Credits), ₹, Opening stock, 10,00,000 Sales, 110,00,000, Purchases, 95,70,000 Closing stock, 4,00,000, Salaries, 1,50,000, Market rent (building is owned by the, 1,00,000, assessee himself), Municipal taxes of the building (due), 5,000, Loss by theft, 19,000, Donation for Ram Lila celebration, 1,500, Provision for bad debts, 11,000, Gifts to relatives, 400, Presents to clients for advertisements, 300, Public provident fund, 12,000, Interest on loan for business, 13,000, Interest on capital, 4,000, Addition to business premises, 2,00,000, Repairs of business premises, 600, Income tax, 2,000, Fine for violation of traffic rules, 100, Net Profit, 3,11,100, 114,00,000, 114,00,000, Additional information:, 1. Purchases includes purchase of ₹1,00,000 from a relative and it is excessive by ₹20,000 and payment, was made in cash., 2. Salary includes ₹14,000 paid outside India without deducting tax at source and ₹7,000 were paid to one, of the relatives which is more than the market rate by ₹1,000., 3. Business is being run in a commercial building which is owned by the assessee and its written down, value on 01.04.2021 is ₹10 lakhs and addition was made to the building on 01.01.2022 and brought into
Page 725 :
Income Under The Head Business/Profession, , 206, , immediate use and no depreciation has been debited to profit and loss account., Mr. X has not opted for presumptive taxation of Income u/s 44AD. Compute his Total Income and Tax, Liability for the Assessment Year 2022-23., Answer = Total Income: ₹6,40,100; Tax Liability: ₹39,540, Problem 8., Mrs. X submitted the following profit & loss account for the Assessment Year 2022-23., Particulars, Amount, Particulars, (Debits), ₹, (Credits), Salary (including proprietor’s salary of, 46,000 Gross Profit, ₹15,000), General Expenses, 6,000 Bad debts recovered (not allowed earlier, due to lack of evidence), Advertisements, 39,000 Interest on company deposit, Interest on proprietor’s capital, 2,000 Long term capital gains, Provision for bad debts, 2,000, Depreciation, 4,000, Reserve for GST, 10,000, Advance income tax, 9,500, Donation to scientific research, 1,000, institution, Motor car expenses, 1,000, Stationery, 1,900, Net Profit, 5,06,600, 6,29,000, Other information:, 1. General expenses include ₹ 300 given to a poor student to enable him to pursue his studies., 2. Motor car expenses include ₹300 for personal purposes., 3. Scientific institution is an approved institution., Compute her Tax Liability and Tax Payable for the Assessment Year 2022-23., Answer = Tax Liability: ₹61,089.60 Tax Payable: ₹51,590, , Amount, ₹, 1,22,000, 2,000, 5,000, 5,00,000, , 6,29,000, , Problem 9., Mr. X a Chartered Accountant submits his receipt and payment account for assessment year 2022-23., Receipts, Amount, Payments, Amount, ₹, ₹, Balance b/d, 1,00,000 Stipend to articled clerks, 12,000, Audit Fees, 4,40,000 Office Expenses, 24,000, Payment received for appearing before, 25,000 Office Rent, 18,000, Income Tax Appellate Tribunal, Misc. receipts, 20,000 Salaries and Wages, 20,500, Rent received for house property, 24,000 Printing and Stationery, 4,000, Present from clients, 10,000 Subscription to ICAI, 1,500, 15,000, Purchased books for professional, purposes on 01.07.2021 and put to use, on the same date, Travelling Expenses, 5,000, Interest on loan for payment of income, 12,000, tax, Donation to poor persons, 5,000
Page 726 :
Income Under The Head Business/Profession, , 207, , Drawings for personal use, Balance c/f, , 1,02,000, 4,00,000, 6,19,000, 6,19,000, Mr. X has not opted for presumptive taxation of Income u/s 44ADA. Compute his Tax Liability for the, Assessment Year 2022-23., Answer = Tax Liability: Nil, Problem 10., ABC Ltd. submits the profit & loss account for the year ending 31st March 2022., Particulars, Amount, Particulars, (Debits), ₹, (Credits), Salary to staff, 3,00,000 Gross Profit, Capital expenditure for promotion of, 14,000 Rent of flats given to staff, family planning amongst employee, GST (paid on 01.11.2022), 24,000 Sundry receipts, Gratuity paid to staff, 24,000 Short term Capital gains on sale of land, Reserve for future losses, 30,000, Reserve for bad debts, 14,000, Payment of advance income-tax, 17,000, Car expenses, 20,000, Depreciation, 30,000, Office expenses, 12,000, Repair of flats given to staff, 24,000, Sundry expenses, 46,000, Net Profit, 63,000, 6,18,000, Determine the Total Income and Tax Liability of company for the Assessment year 2022-23., Answer = Total Income: ₹1,59,200; Tax Liability ₹49,670, , Amount, ₹, 5,27,000, 24,000, 7,000, 60,000, , 6,18,000, , Problem 11., From the following profit and loss account of Mr. X for the year ending March 31st, 2022, compute his Total, Income and Tax Liability for the Assessment Year 2022-23., Particulars, Amount, Particulars, Amount, (Debits), ₹, (Credits), ₹, Opening stock, 4,62,000 Sales, 103,00,000, Purchases, 90,35,000 Closing stock, 4,97,000, Salaries, 8,50,000 Rental income from house property, 84,000, Rent rate & taxes, 1,25,000 Dividends from an Indian company, 12,000, Legal charges, 45,000 Income from owning and maintaining, 20,000, of race camels, Miscellaneous expenses, 22,000, Provision for gratuity, 22,000, Provision for Income Tax, 53,000, Salary to Mrs. X, 36,000, Depreciation, 40,000, Net Profit, 2,23,000, 109,13,000, 109,13,000, Additional information:, (i) Purchases include ₹1,10,000 paid in cash to a cultivator for purchase of an agricultural produce.
Page 727 :
Income Under The Head Business/Profession, , 208, , (ii), , Purchases also include ₹ 15,000 paid by way of compensation to a supplier as the assessee was unable, to take the delivery of goods due to lack of storage space and finance., (iii) Opening stock was overvalued by 25% and closing stock was undervalued by 25%., (iv) Salary includes ₹ 25,000 paid as customary bonus on the occasion of Diwali over and above the bonus, payable under the Payment of Bonus Act 1965., (v) Rent, rates and taxes include, (a) ₹3,000 on account of municipal taxes for property let out and payment was made on 31.03.2022., (b) Penalty imposed by GST department ₹25,000., (vi) Provision for Gratuity is on actuarial basis., (vii) Mrs. X is a law graduate and actively working in the assessee firm and salary paid is reasonable., (viii) He has invested ₹1,00,000 in equity shares of infrastructure development companies notified u/s 80C., (ix) He has loss from owning and maintaining of race horses ₹20,000., Mr. X has not opted for presumptive taxation of Income u/s 44AD., Answer = Total Income: ₹4,34,770; Tax Liability: Nilf, Problem 12., Net profit as per the profit and loss account of Mr. X is ₹ 7,70,000 for the year ending 31st March, 2022., The following information is noted from the accounts:, (a) Advertisement expenditure debited to profit and loss account includes the following:, (i) Expenditure incurred outside India: ₹ 56,000 (Tax has been deducted at source and paid during the, year), (ii) Articles presented by way of advertisement (60 articles cost of each being ₹700, and 36 articles cost, of each being ₹1,500);, (iii)₹20,000 being the cost of advertisement which appeared in a newspaper owned by a political party;, (iv) ₹14,400 being capital expenditure on advertisement; (eligible for dep. @ 25%), (v) ₹9,000 paid in cash, (vi) ₹9,000 paid to a concern in which X has substantial interest (amount is excessive to the extent of, ₹1,800), (b) Out of salary to the employees debited to the profit and loss account:, (i) ₹60,000 is employee’s contribution to the recognized provident fund, ₹47,500 of which is credited in, the employee’s account in the relevant fund before the due date for provident fund;, (ii) ₹58,000 is bonus which is paid on 13th November, 2022;, (iii)₹44,000 is commission which is paid on 1st December, 2022;, (iv) ₹25,000 is incentive to workers, which is paid on 10th December, 2022., (v) ₹46,000 is paid outside India in respect of which tax is not deducted at source;, (vi) ₹6,000 being capital expenditure for promoting family planning amongst employees; and, (vii) ₹55,000 being entertainment allowance given to employees., (c) Entertainment expenses debited to profit and loss account is ₹ 12,000., Determine the Total Income and Tax Liability of Mr. X for the Assessment Year 2022-23., Answer = Total Income: ₹9,69,100; Tax Liability ₹1,10,570, Problem 13., The profit and loss account of Mr. X for the year ending 31st March, 2022 discloses net profit of ₹3,90,000., Travelling expenses debited to the profit and loss account include the following:, (i) ₹1,80,000 being expenditure incurred on a foreign tour, out of which ₹15,000 is incurred in Indian, currency and ₹1,65,000 in foreign currency for a visit of 8 days to Germany; out of 8 days, 2 days are, utilized by Mr. X for attending personal work., (ii) ₹45,000 being expenditure on air–fare in India by a sales manager., (iii) ₹6,500 incurred for purchasing a machine for factory. (Put to use for more than 180 days), (iv) ₹66,000 being hotel expenses as follows:, (a) 4 days visit to Madras, :, ₹ 18,000, (b) 3 days visit to Bombay, :, ₹ 8,000, (c) 17 days visit to Bangalore, :, ₹ 40,000, Salary to employees include the following:
Page 728 :
Income Under The Head Business/Profession, , 209, , (1) Own salary of Mr. X :, ₹ 26,000, (2) Commission on purchases to employees, (which is actually paid on 1st November, 2022):, ₹42,000, Find out the Total Income and Tax Liability of Mr. X for the Assessment Year 2022-23., Answer = Total Income: ₹5,08,530; Tax Liability: ₹14,770, Problem 14., From the following profit and loss account of Mr. X for the year ended 31st March, 2022, compute his Total, Income and Tax Liability for the Assessment Year 2022-23:, Particulars, Amount, Particulars, Amount, (Debits), ₹, (Credits), ₹, Opening Stock, 9,50,000 Sales, 101,06,000, Purchases, 80,50,000 Closing Stock, 3,60,000, Salaries, 7,00,000 Long term capital gain on sale of house, 36,000, property, Rent, rates and taxes, 1,25,000 Dividends from foreign company, 12,000, Deposit in National Saving Certificate, 42,000 Winnings of a lottery (gross), 5,00,000, Miscellaneous Expenses, 21,000, Provision for Income Tax, 31,000, Provision for gratuity, 24,000, Provision for GST, 45,000, Salary to Mrs. X, 48,000, Purchased one computer on 01.11.2021, 40,000, and put to use on the same date, Net Profit, 9,38,000, 110,14,000, 110,14,000, Additional information:, (i) Purchases include, (a) Purchase of ₹ 1,00,000 from a relative (market price ₹80,000) and payment was made in cash., (b) Purchase of ₹25,000 being the products manufactured without aid of power in a cottage industry and, the payment was made to its producer and payment was made in cash., (c) Purchases of ₹35,000 from a person who is residing in a village having no bank and payment was, made in cash., (ii) Opening and closing stock were overvalued by 10%., (iii) Salary includes ₹ 25,000 being bonus paid to the staff on 01.11.2022 on the occasion of Diwali., (iv) Rent, rates and taxes include Municipal tax paid on 01.11.2022 ₹30,000, (v) Provision for Gratuity is on actuarial basis., (vi) Mrs. X is a housewife and payment is excessive by ₹48,000., Mr. X has not opted for presumptive taxation of Income u/s 44AD., Answer = Total Income: ₹13,02,640; Tax Liability ₹2,31,950, Problem 15., The profit and loss account of ABC Ltd. for the year ended 31st March, 2022 showed a net profit of, ₹8,00,000 and some of the debits and credits are as given below:, (A) Debit side of profit and loss account included the following:, (i) The depreciation provided in the books ₹60,000, however the amount computed under the Income Tax, Act ₹1,20,000., (ii) ₹30,000 was paid to the company’s lawyer for arguing appeals of the company before the Income Tax, Appellate Tribunal against levy of penalty for some earlier cases where appeals have been dismissed, by the tribunal., (iii) ₹2,000 being fine imposed by the municipality for violating their regulations.
Page 729 :
Income Under The Head Business/Profession, , 210, , (iv) Provision for Income Tax ₹35,000., (B) The credit side of the profit and loss account included the following:, (i) Income from units of UTI ₹35,000, (ii) Dividend from Indian company ₹20,000, (C) It is also observed that both the opening stock of ₹90,000 and closing stock of ₹1,08,000 are undervalued, by 10% on cost., Compute the Total Income and Tax Liability of the company for the Assessment Year 2022-23., Answer = Total Income: ₹7,79,000; Tax Liability: ₹2,43,050, Problem 16., Mr. X is a leading lawyer of Mumbai. He deposits in the bank all the receipts and always pays all the, expenses by cheque. The analysis of his bank account for the year ended 31st March, 2022 is asunder:, Receipts, Amount Payments, Amount, ₹, ₹, Balance b/f, 15,000 Salaries, 5,00,000, Professional Fees, 16,75,000 Rent of chamber, 2,55,000, Dividend from Indian Co., 8,000 Telephone Expenses, 26,000, Rent from house property which is let, 60,000 Magazine Subscription, 3,000, out, Dividend from UTI, 10,000 Motor car expenses, 10,000, Interest from a company (gross), 8,000 Motor car (purchased and put to use on, 3,00,000, 01.12.2021), Gift from his son from outside India, 6,000 Misc. office expenses, 5,500, Honorarium for delivering lectures in, 5,000 Advance payment of income-tax, 38,000, C.A. institute, Honorarium for writing articles in, 1,000 Personal expenses, 48,500, Hindustan Times, House property expenses:, Municipal taxes, 6,000, Repairs, 1,000, Insurance, 2,000, Collection Charges, 1,000, 10,000, Subscription to Bar Association, 1,500, Balance c/f, 5,90,500, 17,88,000, 17,88,000, Mr. X has not opted for presumptive taxation of Income u/s 44ADA. Compute his Total Income, Tax, Liability and Tax Payable after taking into account the following information:, (i) 10% of the motor car expenses relate to personal use., (ii) Salaries include employer’s contribution to Recognised Provident Fund of ₹18,000 which was credited, on 01.07.2022., (iii) Mr. X stays in his house, the gross annual value of which is ₹ 16,800., Following are the expenses which have been included in the above account in respect of this house:, (a) Municipal taxes: ₹ 2,000., (b) Repairs: ₹500, (c) Insurance premium: ₹500, (iv) He has loss under the head house property ₹31,200 and the loss can be set off as per section 71B., Answer = Total Income: ₹8,94,750; Tax Liability: ₹95,108; Tax Payable: ₹57,110, Problem 17., ABC Ltd., a manufacturing company, which maintains accounts under mercantile system has disclosed a net, profit of ₹12.50 lakhs for the year ending 31st March, 2022. You are required to compute the total Income, and Tax Liability of the company for the Assessment Year 2022-23, after considering the following
Page 730 :
Income Under The Head Business/Profession, , 211, , information, duly explaining the reasons for each item of adjustment:, (i) Advertisement expenditure includes the sum of ₹60,000 paid in cash to the sister concern of a director,, the market value of which is ₹52,000., (ii) Repairs of plant and machinery includes ₹1.80 lakhs towards replacement of worn out parts of, machineries., (iii)A sum of ₹6,000 on account of liability foregone by a creditor has been taken to general reserve. The, same was charged to the revenue account in the Assessment Year 2019-20., (iv) Sale proceeds of import entitlements amounting to ₹ 1 lakh has been credited to profit and loss account,, which the company claims as capital receipt not chargeable to income tax., (v) The company has donated ₹2,00,000 to National Urban Poverty Eradication Fund. The amount has been, debited to the profit and loss account., (vi) Being also engaged in the biotechnology business, the company incurred the following expenditure on, in-house research and development as approved by the prescribed authority:, (a) Research equipment purchased ₹1,50,000., (b) Remuneration paid to scientists ₹ 50,000., The total amount of ₹2,00,000 is debited to the profit and loss account., Answer = Total Income: ₹13,16,000; Tax Liability: ₹4,10,590, Problem 18., (i) Gross total income of Mrs. X, aged 60, a resident of Delhi for the financial year 2021-22 is ₹4,00,000. It, includes an income of ₹20,000 from the business of dealing in shares on which she has paid securities, transaction tax of ₹1,800 and it has not been debited to the profit and loss account. She has also deposited, ₹10,000 in her public provident fund account with the State Bank of India., Compute her Tax Liability for the Assessment Year 2022-23., Answer = Tax Liability: Nil, (ii) ABC Ltd. is engaged in the business of sale/purchase of shares and the company has computed its, income ₹11,00,000 after debiting securities transaction tax of ₹1,85,000., Compute Tax Payable by the company for the Assessment Year 2022-23., Answer = ₹3,43,200, (iii) Mr. X is engaged in the business of sale/purchase of shares and he has computed its income ₹18,00,000, after debiting securities transaction tax of ₹2,10,000., Compute Tax Payable by Mr. X., Answer = ₹3,66,600, Problem 19., Determine the previous year in which the expenditure is allowable in the following cases (TDS is supposed, to be deducted with regard to all the payments and all the payments are in India):, (i) ABC Ltd. has made payment of interest on 10th, June 2021 and has deducted tax at source on the same, date and has deposited the amount on 08.07.2021., (ii) The company has paid commission on 10.03.2022 and has deducted tax on the same date but it was paid, on 05.04.2022., (iii) The company has paid fees for professional services on 31.03.2022 and deducted tax at source on the, same date but the tax was paid on 07.04.2022., (iv) The company has paid to a contractor on 31.03.2022 and tax was deducted on the same date but it was, paid on 01.06.2022., (v) The company has paid technical fees on 01.01.2022 and no tax has been deducted at source., (vi) The company has paid brokerage on 01.04.2022 and has deducted the tax on the same date and has paid, it on 07.04.2022., Problem 20., Following is the profit & Loss account of Mr. A, a dealer in shares and securities for the year ended on 31st, March, 2022:
Page 731 :
Income Under The Head Business/Profession, Particulars, (Debits), To Trading Expenses, To Administrative Expenses, To Financial Expenses, To Demat and Delivery charges, , Amount, ₹, 103,60,000, 1,15,000, 50,265, 5,350, , Particulars, (Credits), By Sales, By Interest on fixed deposit with bank, By Dividend from Indian company, By Interest on GST refund, (Assessment Year 2020-21), , 6,500, 9,02,075, 114,39,190, Compute Total Income and Tax Liability of Mr. A for Assessment Year 2022-23., Answer = Total Income: ₹9,02,080; Tax Liability: ₹96,630, , 212, Amount, ₹, 113,54,000, 18,500, 66,360, 330, , To Securities Transaction Tax, To Net profit before depreciation, , 114,39,190, , Problem 21., Mr. X is engaged in the business of plying goods carriages. On 1st April, 2021, he owns 10 trucks (out of, which 6 are heavy goods vehicles having capacity of 18 ton and balance 4 trucks having capacity of 8 ton )., On 2nd May, 2021, he sold two of the heavy goods vehicle and purchased two light goods vehicles having, capacity of 8 ton on 6th May, 2021. Those new vehicles could however be put to use only on 15th June, 2021., Compute the Total Income and Tax Liability of Mr. X for the Assessment Year 2022-23, taking note of the, following data in two situations i.e. presumptive basis and normal basis., ₹, ₹, Freight charges collected, 9,90,000, Less: operational expenses, 5,25,000, Depreciation as per sec 32, 1,85,000, Other office expenses, 15,000, (7,25,000), Net Profit, 2,65,000, Other business and non-business income, 1,00,000, Answer = Presumptive Basis: Total Income: ₹15,61,000; Tax Liability: ₹2,92,030, Non-presumptive Basis: Total Income: ₹3,65,000; Tax Liability: Nil, Problem 22., Profit and loss account of Mr. A for the previous year 2021-22 is asunder:, Particulars, Amount, Particulars, (Debits), ₹, (Credits), Salaries & Wages, 6,00,000 Gross Profit, Advertisement, 1,00,000 Long term capital gains, Travelling Expenses, 2,00,000 Recovery of bad debts (earlier it was, allowed), Depreciation on business assets, 50,000, Donation to an approved institution for, 70,000, rural development, Interest paid, 2,30,000, General Expenses, 3,50,000, Net Profit, 3,00,000, 19,00,000, Additional informations:, (i) Salaries and wages include the sum of ₹1,60,000 paid to Mr. A, (ii) Payment of interest includes:, (a) Interest to his major son ‘X’ amounting to ₹30,000 @ 15% on a deposit of ₹2,00,000, (b) Interest to Mr. A amounting to ₹30,000 @ 12% p.a., (c) Interest of ₹20,000 paid on loan taken for the payment of income tax liability., , Amount, ₹, 14,50,000, 4,00,000, 50,000, , 19,00,000
Page 732 :
Income Under The Head Business/Profession, , 213, , (iii) The amount of depreciation allowable is ₹40,000., (iv) Mr. A has purchased National Saving Certificate VIII issue on 31.03.2022 for ₹40,000 and has, deposited ₹60,000 in public provident fund account during the year 2021-22., Compute Tax Liability of Mr. A for the Assessment Year 2022-23., Answer = Tax Liability: ₹22,360, Problem 23., Profit and loss account of Mr. A for the Previous Year 2021-22 is as given below:, Profit and Loss Account, Particulars (Debits), Amount, Particulars, Amount, ₹, (Credits), ₹, To Purchases, 95,00,000 By Sales, 102,71,000, To Salaries and bonus, 2,70,000 By Discount, 10,000, To GST payable, 30,000 By Interest (gross) from Indian, 70,000, companies, To General expenses, 1,00,000, To Expenditure on technical know-how, 36,000, To Expenses on GST proceedings, 15,000, To Advertisements, 50,000, To Interest on capital, 20,000, To Rent of buildings owned by Mr. A, 30,000, To Net profit, 3,00,000, 103,51,000, 103,51,000, Additional information is given below:, 1. Purchases include a cash purchase of ₹20,000 from a farmer who lives in a village which is not served, by any bank., 2. Advertisements include presentation of 50 VIP bags in Jan 2022 costing ₹350 each., 3. Salaries and bonus includes the following payments., Salary paid to Mr. A ₹52,000, Bonus paid to Mr. A ₹30,000, Commission paid to Mr. A ₹23,000, 4. The proprietor has carried forward business loss from the assessment year 2020-21 amounting to, ₹1,00,000., 5. The written down value of buildings on 01.04.2021 was ₹2,50,000., 6. Technical know-how was purchased and put to use on 01.07.2021., Compute the Total Income and Tax Liability of Mr. A for the Assessment Year 2022-23., Answer = Total Income: ₹3,87,000; Tax Liability: Nil, Problem 24., The Profit & Loss account of Mr. X for the previous year ending 31.03.2022 is as given below:, Particulars, Amount, Particulars, (Debits), ₹, (Credits), To Purchases, 90,00,000 By Sales, To Business expenses, 6,00,000 By Stock, To Depreciation, 50,000 By Dividend from foreign company, (gross), To Salary to Mr. X, 3,60,000 By Sundry receipts, To Interest on capital, 1,89,000 By Long term capital gain, To Sundry expenses, 1,01,000, To Net profit, 2,00,000, 105,00,000, , Amount, ₹, 102,00,000, 1,50,000, 30,000, 20,000, 1,00,000, , 105,00,000
Page 733 :
Income Under The Head Business/Profession, , 214, , You are further informed that –, 1. Purchases include cash purchases of ₹1,00,000 (payment made on a particular date to a particular, person), 2. Bonus of ₹1,07,000 for the previous year 2020-21 was paid on 31.12.2021 but not included in the profit, and loss account., 3. Recovery of bad debts during the year from a discontinued business of ₹1,00,000 but not included in the, profit and loss account. Deduction was allowed in respect of bad debts., 4. Written down value of machinery as on 01.04.2021 was ₹5,00,000. Rate of depreciation being 15%., a. Machinery sold during the year for ₹1,00,000, b. Machinery acquired and put to use in December 2021 for ₹4,00,000, 5. Loss and allowances carried forward, • Business loss – Assessment Year 2018-19 = ₹3,00,000, • Depreciation – Assessment Year 2019-20 = ₹2,00,000, Mr. X has not opted for presumptive taxation of Income u/s 44AD. Compute Total Income and Tax Liability, for the Assessment Year 2022-23., Answer = Total Income: ₹3,02,000; Tax Liability: Nil, Problem 25., Mr. X is exporting computer software outside India. The profit & loss account for the previous year ending, 31.03.2022., Particulars, Amount, Particulars, Amount, (Debits), ₹, (Credits), ₹, To Purchases, 80,00,000 By Total sales, 101,60,000, To Salaries, 4,10,000 By Profit on sale of land held for the, 6,00,000, last 1 ½ years, To Interest to Mr. X, 1,50,000, To Salary to Mr. X, 6,00,000, To Depreciation on plant and machinery, 2,00,000, To Purchase of trade marks, 2,00,000, To Net profit, 12,00,000, 107,60,000, 107,60,000, You are further informed that –, 1. Trade marks purchased and put to use on 01.07.2021., 2. Written down value of plant and machinery as on 01.04.2021 is ₹12,00,000. Rate of depreciation being, 15%., 3. Employer contribution to recognized provident fund of ₹4,00,000 for the previous year 2020-21 was, paid on 31.12.2021 but not included in the above profit and loss account., 4. The proprietor has brought forward depreciation and long term capital loss amounting ₹2,36,000 and, ₹2,00,000 from the Assessment Year 2019-20 respectively., Mr. X has not opted for presumptive taxation of Income u/s 44AD. You are required to compute the Total, Income and Tax Liability for the Assessment Year 2022-23., Answer = Total Income: ₹14,84,000; Tax Liability: ₹2,68,010, Problem 26., Mr. X is engaged in the business of civil construction including repairs of dams and supply of labour for, civil construction. The Profit and Loss account of Mr. X for the year ending 31.03.2022 is as follows:, Particulars, Amount, Particulars, Amount, (Debits), ₹, (Credits), ₹, Opening stock of raw material, 21,700 Receipt from business of civil, 63,70,700, construction, Depreciation, 2,39,430 Rental income from Godown, 48,000
Page 734 :
Income Under The Head Business/Profession, Salary to employees, Purchase of raw material, Interest on loan taken to make deposit in, companies, Interest on loan taken for business, purposes, Travelling, entertainment and, advertisement expenses, Other expenses, Municipal tax (6,000) and insurance, (1,200) of Godown, Salary to Mr. X, Interest to Mr. X, Net profit, , 1,30,000 Interest on company deposits (gross), 44,10,210 Closing Stock of raw material, 13,800, , 215, 2,60,000, 1,31,600, , 74,400, 57,800, 7,42,000, 7,200, 1,80,000, 69,000, 8,64,760, 68,10,300, , 68,10,300, , Other information:, (i) Out of other expenses debited to profit and loss account ₹ 20,000 is not deductible under section 37(1)., (ii) Out of travelling, advertisement and entertainment expenses ₹ 25,000 is not deductible under section, 37(1)., (iii) On 01.04.2021, Mr. X owns the following depreciable assets:, Plants A, B and C, depreciated value: ₹ 3,70,000, rate of depreciation 15%., Plants D and E, depreciated value: ₹ 1,98,000, rate of depreciation 40%., On 01.01.2022 Mr. X sells plant D for ₹9,10,000 and purchases plant F and put to use on the same date (rate, of depreciation 15%) for ₹ 4,86,000., (iv) Mr. X wants to set-off the following losses brought forward from earlier years:, Assessment Year, 2020-21, 2021-22, ₹, ₹, Business loss, 20,000, xxxxx, Capital loss (short-term), 2,000, 1,000, Find out Total Income and Tax Liability of Mr. X for the Assessment Year 2022-23., Answer = Total Income: ₹19,83,840; Tax Liability: ₹4,23,960, Problem 27., Mr. X furnishes the following information relevant for the Assessment Year 2022-23., Particulars, Amount, Particulars, (Debits), ₹, (Credits), General expenses, 23,400 Gross profit, Bad debts, 8,000 Commission, Provision for GST, 10,000 Brokerage, Advance tax, 11,000 Sundry receipts, 4,000 Dividend from Indian companies, Legal expenses (paid to a Chartered, Accountant for preparation of income, tax return), Insurance of stocks, 1,600 Income from Unit Trust of India, Salary to staff, 25,000, Leave encashment to staff (paid during, 5,000, the year), Salary to Mr. X, 24,000, Interest on overdraft, 6,000, Interest on loan to Mrs. X, 32,000, , Amount, ₹, 8,21,300, 9,500, 1,200, 2,500, 12,500, 6,500
Page 735 :
Income Under The Head Business/Profession, Interest on capital to Mr. X, Expenditure on account of Deepawali, (being gift given to customers), Depreciation, Advertisement expenses, Contribution to employees recognised, provident fund (credited within due date), Net profit, , 216, , 22,000, 8,000, 45,000, 7,000, 12,000, , 6,09,500, 8,53,500, 8,53,500, 1. The amount of depreciation allowable is ₹52,500 as per the Income Tax Act, 1961., 2. Income of ₹25,000 received on January 20th, 2022 on units of mutual fund specified under section, 10(23D) is not recorded in profit and loss account., 3. Loan from Mrs. X was taken for payment of arrears of income tax., 4. Salary to staff includes payment of ₹9,000 to a relative, which is unreasonable to the extent of ₹4,000., 5. General expenses include ₹5,000 incurred for payment of school fees of Mr. X’s son., 6. Mr. X has paid insurance premium of ₹70,000 on the life policy (sum assured ₹3,40,000) of his major, son on March 25th, 2022 and he invested ₹20,000 in Industrial Development Bank of India (IDBI), Infrastructure Bonds notified under section 80C on March 30th, 2022., Determine the Total Income and Tax Liability of Mr. X for the Assessment Year 2022-23., Answer = Total Income: ₹ 6,76,000; Tax Liability: 49,610, Problem 28., Mrs. X is a company secretary in practice. She was born on 01.02.1959 in India. Her income and, expenditure account for the year ended 31.03.2022 is as follows:, Expenditure, Amount, Income, Amount, ₹, ₹, Salaries paid to staff, 17,10,000 Fees earned:, Secretarial audit, 6,30,800, Taxation services, 6,20,000, Consultancy Services, 17,50,000 30,00,800, Stipends to apprenticeship trainees, 22,500 Dividend on shares of Indian companies, 10,500, Incentives to apprenticeship trainees, 10,000 Income from Unit Trust of India, 6,500, Rent for the building in the use of, 20,000 Profit on sale of equity shares on which, 20,800, profession, STT has been paid (short term), Printing and stationery, 9,600 Honorarium received from various, 6,600, institutions for evaluation work, Contribution to recognised provident, 35,000 Rent received from residential flat let, 62,000, fund, out, Meeting, seminar and conference, 40,000, expenses, Interest on loan, 60,000, Journals, magazines, newspaper, income, 15,000, tax report, Courier, telephone and fax, 2,90,400, Repairs, maintenance and petrol for car, 16,500, Depreciation:, Car, 8,500, Computer, 14,000, Typewriter, 5,000, Furniture, 3,000, 30,500, Travelling Expenses, 57,000
Page 736 :
Income Under The Head Business/Profession, Municipal tax paid in respect of house, property, Net Profit, , 217, , 2,000, 7,88,700, 31,07,200, , 31,07,200, Other information:, (i) A sum of ₹15,000 incurred for entertaining various clients in hotels and clubs is already included in the, meeting, seminar and conference expenses., (ii) Incentives to apprentices represent amount paid to two apprentices for passing the company secretaries, intermediate examination at first attempt., (iii) One fifth of use of car is attributable to personal purposes., (iv) 50% of loan was used for the purpose of construction of house property and 50% of loan was used for, other business purpose., (v) Mrs. X follows accrual basis of accounting, (a) cost of stationery items for ₹3,000 purchased in accounting year 2020-21 which was not provided, for in that year due to oversight, has been included in printing and stationery for the year 2021-22;, and, (b) amount of ₹5,000 for the month of March, 2022 of provident fund contribution could not be paid, upto 05.11.2022., (vi) The written down value of various assets as on 01.04.2021 as follows:, (a) Car (Acquired on 01.04.2017), ₹85,840, (b) Typewriter (Acquired on 01.04.2017), ₹15,000, (c) Furniture (Acquired on 01.04.2017), ₹25,000, (d) Computer (Acquired on 15.12.2021 at cost of ₹1,50,000 and put to use on the same date), (vii) Salaries include ₹30,000 paid to a computer specialist in cash for assisting Mrs. X in one professional, assignment., (viii) Mrs. X has deposited ₹60,000 in her public provident account and invested ₹20,000 in infrastructure, bonds of the ICICI Ltd. notified under section 80C., Mrs. X not opted for presumptive taxation of Income u/s 44ADA. Compute Total Income and Tax Liability, of Mrs. X for the Assessment Year 2022-23., Answer = Total income: ₹ 7,17,450; Tax Liability: ₹54,550, Problem 29., Mr. X has computed his income under the head business/profession ₹10,00,000 and he has debited the, following amount., (1) Cost of goods sold ₹ 7,00,000, out of which ₹ 4,00,000 paid to a relative for purchasing stock and its, market value is ₹ 3,00,000 and Mr. X has paid ₹ 2,00,000 by account payee cheque and ₹ 2,00,000 in, cash., (2) He has debited ₹45,000 in connection with purchase of a computer which was purchased on, 27.10.2021 and was put to use on the same date and payment was made in cash., (3) He has purchased one generator from his relative for ₹ 45,000 and payment was made in cash and, market value was ₹ 40,000 and it was purchased on 01.10.2021 and was put to use on 07.10.2021., (4) He has paid advance tax being income tax ₹ 45,000 on 01.10.2021., (5) He has paid ₹ 21,000 to a Chartered Accountant for filing a return of income, out of which ₹9,500 was, paid in cash and balance by an account payee cheque., (6) He has donated ₹ 20,000 to an approved research association and research work taken up by such, association is not related to the business/profession of assessee., (7) He has purchased household furniture for ₹12,000 for personal use., (8) He has paid ₹ 20,000 in cash in connection with his medical treatment., (9) Salary paid to the proprietor is ₹ 36,000., (10) Interest on capital ₹ 9,000., (11) He has invested ₹ 25,000 in National Saving Certificate., (12) He has invested ₹ 10,000 in public provident fund in the name of his minor child., (13) He has debited rent of ₹ 35,000 in connection with his own building which is being used in his
Page 737 :
Income Under The Head Business/Profession, , 218, , business/profession., (14) Opening stock debited is ₹ 4,50,000 which is overvalued by 10%., (15) He has incurred ₹ 7,000 on printing and distribution of diaries and calendars., Compute his Tax Liability for the Assessment Year 2022-23., Answer = Tax Liability: ₹2,69,230, Problem 30., Mr. X furnishes the following trading, profit and loss account for the previous year ending on 31.03.2022., Particulars, Amount, Particulars, Amount, (Debits), ₹, (Credits), ₹, To Stocks, 14,000 By Sales, 100,84,500, 19,500, To Purchases, 94,80,000 By Maturity proceeds of National, Saving Certificate, To Freight and duty, 5,000 By Maturity proceeds of Bank Fixed, 24,000, Deposit, To Manufacturing wages, 25,000 By Maturity proceeds of Public, 13,000, provident fund, To Factory, rent, rates and taxes, 30,000 By Rent of staff quarters built in, 19,000, 2015, To Office salaries, 27,000 By Refund of income tax penalty, 1,100, To Establishment expenses, 6,100 By Sale of an old machinery, 25,000, To Cost of computer, 24,000 By Recovery of bad debts (Not, allowed earlier), 6,000, To Interest on capital, 3,300 By Income tax refund, 2,400, (it includes interest- ₹400), To Donation to an orphan, 1,000 By Gift from friends and relatives, 3,600, To Fire insurance, 200 By Sundry receipts, 5,000, To Bad debts, 6,000 By Maturity proceeds of LIC policy, 24,000, 1,00,000, To Income Tax, 6,000 By Refund of deposit from a, supplier who could not supply the, machine in time, To National Urban Poverty Eradication, 2,000 By Closing stocks, 26,400, Fund, To Employer’s contribution to, 8,000, Recognised provident fund, To Service charge for air-conditioner, 11,000, To Expenses on GST proceedings, 12,000, To Expenses on income tax proceedings, 3,000, To Diwali expenses, 4,000, To Legal Expenses, 4,000, To Medical expenses of proprietor, 3,000, To Staff welfare fund, 2,000, To Repairs of staff quarters, 4,000, To Bonus payable to employees, 5,000, To Provision for GST, 25,000, To Municipal taxes for staff quarters, 4,000, To General reserve, 5,000, To Entertainment expenses, 6,000, To Net Profit, 6,27,900, 103,53,500, 103,53,500
Page 738 :
Income Under The Head Business/Profession, , 219, , Mr. X has not opted for presumptive taxation of Income u/s 44AD. You are required to compute Tax, Liability after taking the following into consideration:, 1. Purchases include a purchase of ₹ 20,100. Its payment was made by a bearer cheque and also includes a, purchase from a relative of ₹23,000 and the payment was made in cash and market price of the, purchases is ₹22,000., 2. Factory rent, rates and taxes includes municipal tax of the factory building, which was paid on, 31.07.2022., 3. Assessee has always valued the stocks at cost price but since 2021-22 he has valued it at market price,, which was in excess of the cost price by 10%., 4. Office salaries paid include ₹12,400 to the proprietor of the business., 5. Diwali expenses include gifts of ₹1,000 made to the relatives., 6. The written down value of the block consisting of machinery as on 01.04.2021 is ₹ 59,000, 7. The written down value of the block consisting of factory building as on 01.04.2021 is ₹ 85,000. An, addition was made to building on 01.08.2021 at a cost of ₹12,000., 8. Service charge for air-conditioner were paid in two instalment of ₹10,000 and ₹1,000 on 10.01.2022 and, 11.01.2022 in cash., 9. Employer’s contribution was made through an account payee cheque on 10.04.2022 and the cheque, realised on 20.04.2022 and the due date for the purpose of provident fund was 15.04.2022., 10. Computer was purchased on 31.03.2021 and it was put to use on 31.03.2022., Answer = Tax Liability: ₹15,850, Problem 31: Mr. X, aged 75 years, has submitted his profit and loss account for the year ending 31.03.2022, as given below:, Particulars, Amount, Particulars, Amount, (₹), (₹), Opening Stock, 13,50,000 Sales, 105,00,000, Purchases, 75,00,000 Gift from friend, 1,200, Franchises, 1,00,000 Bad debts recovered, 2,900, Advertisement, 9,000 Rental income from House Property, 1,40,000, Income Tax of previous year 2020-21, 7,000 Income tax refund, 700, Income tax (advance), 1,200 Dividends from a foreign company, 3,000, Addition to the office building, 45,000 Closing stock, 1,80,000, Investment in public provident fund, 70,000, Net Profit, 17,45,600, 108,27,800, 108,27,800, Additional information:, 1. Opening and closing stocks are undervalued by 10%., 2. Franchises were purchased on 01.07.2021 and were put to use on 03.10.2021., 3. Advertisement expenditure relates to a neon sign board which was purchased and put to use on, 01.08.2021., 4. Office building has written down value of ₹56,00,000 as on 01.04.2021 and addition was made to the, building by constructing additional room on the roof. Construction was completed on 01.11.2021 and, it was put to use on the same date. The expenditure of ₹45,000 includes cost of wiring and switches of, ₹4,500. No depreciation has been debited with regard to the building., 5. Sale includes sale of ₹1,20,000 to the proprietor and the cost of these goods was ₹1,00,000 and market, price ₹1,25,000., 6. Bad debts recovered were allowed earlier., Mr. X has not opted for presumptive taxation of Income u/s 44AD. Compute his Tax Liability for the, Assessment Year 2022-23., Answer = Tax Liability: ₹1,53,630
Page 739 :
Income Under The Head Business/Profession, , 220, , Problem 32: Mr. X is an Advocate in the Supreme Court and he keeps his books of accounts on cash basis, and his receipts and payments a/c for the year 2021-22 is as given below:, Receipts, Amount, Payments, Amount, ₹, ₹, Balance b/f, 4,50,000 Rent of building in the use of profession, 2,20,000, Legal consultancy fee, 9,20,000 Office expenses, 30,000, 35,000, Interest on units of UTI, 12,000 New computer purchased by account, payee cheque on 01.11.2021 and put to, use on the same date, 25,000, Remuneration from Delhi University, 4,000 Computer purchased by account payee, for setting one paper for LLB exams, cheque on 10.11.2021 and put to use on, the same date, Honorarium for delivering lectures as, 3,000 Motor car purchased by account payee 4,00,000, guest speaker, cheque on 01.12.2021 and put to use on, the same date, Sales proceeds of an old computer with, 7,000 Legal books purchased on 01.01.2022, 9,000, w.d.v as on 01.04.2021 ₹2,300, and put to use on the same date, Sale proceeds of one house which was, 9,80,000 Income tax paid being advance tax under, 12,000, purchased on 01.04.2020 for ₹6,70,000, section 207 to 219, Subscription to Bar Association, 3,000, Deposit in public provident fund in the, 12,000, name of major son, Balance carried forward, 16,30,000, 23,76,000, 23,76,000, Mr. X has not opted for presumptive taxation of Income u/s 44ADA. Compute Tax Liability/Tax Payable for, the Assessment Year 2022-23., Answer = Tax Liability: ₹1,04,757.12; Tax Payable: ₹92,760, Problem 33: Mr. X submits the profit and loss account for the year ending 31.03.2022 asunder–, Particulars, Amount, Particulars, (Debits), ₹, (Credits), Household expense, 20,000 Gross Profit, Interest on loan taken from Mrs. X, 2,000 Income tax refund, Income tax, 12,000 Interest on income tax refund, Interest on loan for payment of income tax, 1,200 GST refund, Contribution to Unrecognised Provident Fund, 4,000 Interest on GST refund, Expenditure on advertisement (revenue), 25,000 Bad debts recovered, Public provident fund contribution, 7,000 Dividends from foreign company, Investment in post- office saving bank account, 12,000, Purchase of car (paid by account payee cheque), 2,45,000, Purchase of computer (paid by account payee, 35,000, cheque), Purchase of plant (paid by account payee, 23,000, cheque), Net Profit, 1,55,000, 5,41,200, , Amount, ₹, 5,28,500, 3,000, 300, 1,000, 400, 5,000, 3,000, , 5,41,200
Page 740 :
Income Under The Head Business/Profession, , 221, , Addition Information:, Car, computer and plant and machinery were purchased on 01.10.2021 and were put to use on the same date., Compute Tax Liability of Mr. X for Assessment Year 2022-23., Answer = Tax Liability: Nil, Problem 34: Mr. X submits his profit & loss account for year ending 31st March 2022., ₹, Computed net profit after debiting the following amounts to, 87,000, 1. Provisions for doubtful debts, 16,000, 2. Depreciation reserve, 21,000, 3. Household expenses, 20,000, 4. Donations to poor persons and, 10,000, 5. Other charitable donations, 20,000, 6. Cash payment for purchases, 80,000, 7. Advertisement expenses ₹ 5,000 spent on a neon sign board purchased and put to use on 01.07.2021 and, advertisement gifts to 50 customers at a cost of ₹ 100 each., 8. Audit fee charged ₹5,000, including expenses on income-tax proceedings ₹3,000., 9. Patents purchased for ₹70,000 (paid by account payee cheque) on 01.10.2021 and put to use on, 07.10.2021., 10. Preliminary expenses covered under section 35D: Market survey expenses ₹5,000; feasibility report, expenses ₹10,000. Project cost ₹10,00,000., Incomes credited to profit and loss account were:, (i) Interest on company deposit ₹ 50,000., (ii) Opening stock is valued at cost plus 15% basis, whereas closing stock was valued at cost minus 15%, basis. Opening stock valued was ₹1,15,000; closing stock valued was ₹1,70,000., Compute his Tax Liability for the Assessment Year 2022-23., Answer = Tax Liability: Nil, Problem 35: The profit and loss account of Mr. X for the previous year ending 31st March, 2022 is as, follows:, Particulars, Amount, Particulars, Amount, ₹, ₹, Cost of Goods Sold, 105,45,000 Sales, 109,70,000, Remuneration to Prop., 3,00,000 Dividend from Indian Company, 30,000, Remuneration to Employees, 1,70,000 Long term capital gain, 1,90,000, Interest to proprietary, 40,000, Other Expenses, 1,00,000, Income Tax Paid, 10,000, Net Profit, 25,000, 111,90,000, 111,90,000, Additional information is given below:, (1) Other expenses include the following:, (i) Entertainment expenses incurred for business purpose ₹20,000, (ii) V.I.P bags, Costing ₹1,500 each, given to ten dealers who exceeded the sales target under the sales, promotion scheme., (iii) Employer’s contribution to recognized provident fund amounting to ₹10,000 was paid on 20.04.2021., (iv) ₹30,000 paid in cash to a supplier who refused to accept payment by a cheque., (2) Other income of Mr. X is under the head house property of ₹90,000., Mr. X has not opted for presumptive taxation of Income u/s 44AD. You are required to compute Tax, Liability for the Assessment Year 2022-23.
Page 741 :
Income Under The Head Business/Profession, , 222, , Answer = Tax Liability: ₹29,380, Problem 36: ABC Ltd. has net profits of ₹7,00,000 after debiting municipal taxes of ₹12,000 relating to the, previous year 2021-22, which were paid on 20.09.2022. Municipal taxes are related to a building which is, owned by the company, the ground floor and first floor (which is 2/3rd of the complete building) was being, used by company. The company has debited market rent of ₹ 20,000 p.m. to the profit & loss account for, using the building and credited rent of ₹10,000 p.m. to the profit & loss account for the second floor of the, building which has been let out to some person during the previous year 2021-22., Compute Tax Liability of the company for the Assessment Year 2022-23., Answer = Tax Liability: ₹ 2,83,300, Problem 37: Dr. Sagar furnishes you the following information:, Income and Expenditure account for the year ended 31st March, 2022., Expenditure, Amount, Income, Amount, ₹, ₹, To Medicines Consumed, 2,52,000 By Fee Receipts, 8,49,500, To Staff Salary, 1,55,000 By Rental income from house property, 29,000, To Hospital Consumables, 48,500 By Dividend from Indian companies, 15,000, To Rent Paid, 60,000, To Administrative Expenses, 1,28,000, To Net Income, 2,50,000, 8,93,500, 8,93,500, (i) Rent paid includes rent for his residential accommodation of ₹ 38,000 (paid in cash)., (ii) Hospital equipments (eligible for depreciation @ 15%), 01.04.2021 opening written down value, ₹5,50,000, 07.12.2021 acquired paid by account payee cheque (cost), put to use on the same date, ₹2,50,000, (iii)Medicines consumed include medicines (cost) ₹12,000 used for Dr. Sagar’s family., (iv) Rent received–relates to a property situated at Mysore. The municipal tax of ₹ 3,500 paid in December,, 2021 has been included in the “administrative expenses.”, (v) He received ₹10,000 per month as salary from ‘Full Cure Hospital’. This has not been included in the, “Fee Receipts” credited to income and expenditure account., Dr. Sagar has not opted for presumptive taxation of Income u/s 44ADA. Compute Dr. Sagar’s Taxable, Income for the year ended 31.03.2022 and also his Tax Liability., Answer = Taxable Income: 2,61,100; Tax Liability: Nil, Problem 38: ABC Ltd. presents the following information to you pertaining to the year ending March 31st,, 2022:, 1. Having regard to the vast purchase of a particular chemical by the company, the supplier of the, chemical presents a car worth ₹ 2,50,000, which is used for business purposes by the company., 2. Expenditure towards acquisition of technical know-how paid to a foreign company in a lump sum ₹6, lakhs by account payee cheque., 3. The company has paid income–tax of ₹ 60,000 being the tax in respect to non–monetary perquisites of, an employee., 4. The company wanted to start a new plant for manufacturing of a new product. Y Ltd., paid to the, company ₹ 10 lakh in order not to start the same and not to compete with it., 5. The company has paid ₹ 20 lakh to four employees at the time of their voluntary retirement, in, accordance with the approved scheme of voluntary retirement., 6. The company has borrowed ₹ 15 lakh for acquiring a machinery. Interest paid is ₹90,000. The, machinery is not put to use during the year., 7. Payment of ₹ 40,000 is made to a Don for ensuring that the employees will not indulge in strike.
Page 742 :
Income Under The Head Business/Profession, , 223, , 8. The company has incurred expenditure of ₹ 34,000 in respect of exempt income. This forms part of, administrative expenses., You are requested to briefly state with reasons as to how the above are to be dealt with in computing the, total income of the company for the assessment year 2022-23. The total income need not be computed., Problem 39: State with reasons, the deductibility or otherwise of the following expenses/payments under, the Income-tax Act, 1961, while computing income under the head “Profits and gains of business or, profession” for the Assessment Year 2022-23:, (i) Mr. M paid ₹ 75,000 as commodity transaction tax in respect of sale of commodity during the previous, year 2021-22., (ii) D & Co. has set up a warehousing facility for storage of food grains. It commenced operations on, 01.04.2021. For this purpose, D & Co. incurred capital expenditure of ₹ 50 lakhs on purchase of building in, March 2020. (Through an account payee cheque.), (iii) ABC (P) Ltd. incurred an amount of ₹ 2,50,000 on a notified project to enhance skill development of its, employees., Problem 40: Compute tax liability for A.Y. 2022-23 in the following cases:, (i) Mr. A has LTCG 112A ₹300,00,000, (ii) Mr. B has STCG 111A ₹300,00,000, (iii) Mr. C has Dividend Income ₹300,00,000, (iv) Mr. D has LTCG ₹300,00,000, (v) Mr. E has casual income ₹300,00,000, (vi) Mr. F has business income ₹300,00,000, Answer = (i) Tax Liability: ₹35,46,140; (ii) Tax Liability: ₹53,37,150; (iii) Tax Liability: ₹105,39,750; (iv), Tax Liability: ₹77,35,000; (v) Tax Liability: 117,00,000; (vi) Tax Liability: ₹114,56,250, , Check Detailed Solution given on our website www.mkgeducation.com, (Icon /Heading No.11: Books)
Page 743 :
Income Under The Head Business/Profession, , 224, , EXAMINATION QUESTIONS, JULY – 2021 (NEW COURSE), Question 1, (14 Marks), Mr. Ashish, a resident individual, aged 43 years, provides professional services in the field of interior, decoration. His Income & Expenditure A/c for the ended 31st March ,2022 is as under:, Expenditure, ₹, Income, ₹, To Employees’ Remuneration &, , 13,66,000 By Consultancy Charges, , 58,80,000, , Benefits, To Office & Administrative Exp., , 3,14,000 By Interest on Public Provident, Fund (PPF) Account, , To General Expenses, , 75,000 By Interest on Savings Bank, , To Electricity Expenses, , 65,000 By Interest on National Saving, Certificates VIII Issue (for 3rd, , 60,000, 20,000, 21,000, , year), To Medical Expenses, , 80,000, , To Purchase of Furniture, , 48,000, , To Depreciation, , 90,000, , To Excess of income over exp., , 39,43,000, 59,81,000, , 59,81,000, , The following other information related to financial year 2021-22, (i) The expenses on Employees’ Remuneration & Benefits includes:, (a) Family Planning expenditure of ₹ 20,000 incurred for the employees which was revenue in, nature. The same was paid through account payee cheque., (b) Payment of salary of ₹ 25,000 per month to sister-in-law of Mr. Ashish, who was in-charge, of the Accounts & Receivables department. However, in comparison to similar work profile,, the reasonable salary at market rates is ₹ 20,000 per month., (ii) Amount received by Mr. Ashish as Employees’ Contribution to EPF for the month of February, 2022, - ₹10,000 was also deposited after the due date under the relevant Act relating to EPF., (iii)Medical Expenses of ₹ 80,000 as appearing in the Income & Expenditure A/c was expensed for the, treatment of father of Mr. Ashish. His father was 72 years old and was not covered by any health, insurance policy. The said payment of ₹ 80,000 was made through account payee cheque, (iv) General expenses as appearing in the Income & Expenditure A/c, includes a sum of ₹ 25,000 paid to, Ms. Anjaleen on 5th January 2022 as commission for securing work from new clients. This payment, was made to her without deduction of tax at source., (v) Written down value of the depreciable assets as on 1st April, 2021 were as follows:, Professional, , ₹ 90,000
Page 744 :
Income Under The Head Business/Profession, Computers, , 225, , ₹ 35,000, , (vi) The new Furniture as appearing in the Income & Expenditure A/c was purchased on 31st August,, 2021 and was put to use on the same day., The payment was made as under:, -, , ₹ 18,000 paid in cash at the time of purchase of new furniture on 31.08.2021., , -, , ₹ 19,000 paid by account payee cheque on 05.09.2021 as balance cost of new furniture and, , -, , ₹ 11,000 paid in cash on 31.08.21 to the transporter as freight charges for the new furniture., , (vii) Mr. Ashish purchased a car on 02.04.2020 for ₹ 3,35,000 for personal use. However, on 30.04.2021, he brought the said car for use in his profession. The fair market value of the car as on 30.04.2021, was ₹ 2,50,000., (viii) Mr. Ashish made a contribution of ₹ 1,00,000 in his PPF A/c on 31.01.2022, (ix) The Gross Professional Receipts of Mr. Ashish for P.Y. 2020-21 was ₹ 52,00,000., Compute the total income and tax liability of Mr. Ashish for A.Y. 2022-23 assuming that he has not, opted for payment of tax under section 115BAC., Ignore provisions relating to AMT and under section 14A relating to disallowance of expenditure incurred in, relation to income not includible in total income., Solution is given on our website www.mkgeducation.com (Icon /Heading No.11: Books), JAN – 2021 (NEW COURSE), Question 1, (14 Marks), Mr. Krishna (aged 65 years), a furniture manufacturer, reported a profit of ₹ 5,64,44,700 for the previous, year 2021-22 after debiting/crediting the following items:, Debits:, 1. ₹ 20,000 paid to a Gurudwara registered u/s 80G of the Income-tax Act, in cash where no cheques are, accepted., 2. ₹ 48,000 contributed to a university approved and notified u/s 35(1)(ii) to be used for scientific research., 3. Interest paid ₹ 1,67,000 on loan taken for purchase of E-vehicle on 15-05-2021 from a bank. The Evehicle was purchased for the personal use of his wife., 4. His firm has purchased timber under a forest lease of ₹ 20,00,000 for the purpose of business., Credits:, 1. Income of ₹ 4,00,000 from royalty on patent registered under the Patent Act received from different, resident clients. No TDS was needed to be deducted by any of the clients., 2. He received ₹ 3,00,000 from a debtor which was written off as bad in the year 2017-18., Amount due from the debtor (which was written off as bad) was ₹ 5,00,000, out of which tax officer had, only allowed ₹ 3,00,000 as deduction in computing the total income for assessment year 2018-19., 3. He sold some furniture to his brother for ₹ 7,00,000. The fair market value of such furniture was, ₹9,00,000., Other information :, 1. Depreciation in books of accounts is computed by applying the rates prescribed under the Income tax, laws., 2. Mr. Krishna purchased a new car of ₹ 12,00,000 on 1st September, 2021 and the same was put to use in, the business on the same day. No depreciation for the same has been taken on car in the books of account., 3. Mr. Krishna had sold a house on 30th March, 2019 and deposited the long term capital gains of, ₹25,00,000 in capital gain account scheme by the due date of filing return of income for that year. On 1st, March, 2022, he sold another house property in which he resided for ₹ 1 crore. He earned a long term capital
Page 745 :
Income Under The Head Business/Profession, , 226, , gain of ₹ 50,00,000 on sale of this property. On 25th March, 2022, he withdrew money out of his capital, gain account and invested ₹ 1 crore on construction of one house., 4. Mr. Krishna also made the following payments during the previous year 2021-22, - Lump-sum premium of ₹ 30,000 paid on 30th March, 2022 for the medical policy taken for self and, spouse. The policy shall be effective for five years i.e. from 30th March, 2022 to 29th March, 2027., - ₹ 8,000 paid in cash for preventive health check-up of self and spouse., Compute the total income and tax payable by Mr. Krishna for the assessment year 2022-23., Solution is given on our website www.mkgeducation.com (Icon /Heading No.11: Books), Question 2(b), (6 Marks), Mr. Xavier, an Indian resident individual, set up an unit in Special Economic Zone (SEZ) in the financial, year 2017-18 for production of Mobile Phones. The unit fulfills all the conditions of section 10AA of the, Income-tax Act, 1961., During the financial year 2020-21, he has also set up a warehousing facility in a district of Tamil Nadu for, storage of agricultural produce. It fulfills all the conditions of section 35AD., Capital expenditure in respect of warehouse amounted to ₹ 93 lakhs (including cost of land ₹ 13 lakhs). The, warehouse became operational with effect from 1st April, 2021 and the expenditure of ₹ 63 lakhs was, capitalized in the books on that date., Further details relevant for the financial year 2021-22 are as follows :, Particulars, ₹, Profit from operation of warehousing facility before claiming deduction under section, 1,10,00,000, 35AD, Net Profit of SEZ (Mobile Phone) Unit, 50,00,000, Export sales of SEZ (Mobile Phone) Unit, 90,00,000, Domestic Sales of SEZ (Mobile Phone) Unit, 60,00,000, Compute income tax (including AMT under 115JC) payable by Mr. Xavier for Assessment Year 2022-23., Solution is given on our website www.mkgeducation.com (Icon /Heading No.11: Books), Question.4(a)., (8 Marks), During the previous year 2021-22, following transactions took place in respect of Mr. Raghav who is 56, years old., (i), Mr. Raghav owns two house properties in Mumbai. The details in respect of these properties are as, under–, House 1, House 2, Self-occupied, Let-out, Rent received per month, Not applicable, ₹ 60,000, Municipal taxes paid, ₹ 7,500, Nil, Interest on loan (taken for purchase of property), ₹ 3,50,000, ₹ 5,00,000, Principal repayment of loan (taken from HDFC bank), (ii), , (iii), , ₹ 2,00,000, , ₹ 3,00,000, , Mr. Raghav had a house in Delhi. During financial year 2012-13, he had transferred the house to Ms., Vamika, daughter of his sister without any consideration. House would go back to Mr. Raghav after, the life time of Ms. Vamika. The transfer was made with a condition that 10% of rental income, from such house shall be paid to Mrs. Raghav. Rent received by Ms. Vamika during the previous, year 2021-22 from such house property is ₹ 5,50,000., Mr. Raghav receives following income from M/s M Pvt. Ltd. during P.Y. 2021-22:, • Interest on Debentures of ₹ 7,50,000; and, • Salary of ₹ 3,75,000. He does not possess the adequate professional qualification commensurate, with the salary received by him., Shareholding of M/s M Pvt. Ltd. as on 31.3.2022 is as under -
Page 746 :
Income Under The Head Business/Profession, , 227, , Equity shares, Preference shares, Mr. Raghav, Nil, Nil, Mrs. Raghav, 2%, 25%, Mr. Jai Kishan, 98%, 75%, (brother of Mrs. Raghav), (iv), Mr. and Mrs. Raghav forms a partnership firm with equal share in profits. Mr. Raghav, transferred a fixed deposit of ₹ 1 crore to such firm. Firm had no income or expense other than the, interest of ₹ 9,00,000 received from such fixed deposit. Firm distributed the entire surplus to Mr. and, Mrs. Raghav at the end of the year., (v), Mr. Raghav holds preference shares in M/s K Pvt. Ltd. He instructed the company to pay dividends, to Ms. Geetanshi, daughter of his servant. The transfer is irrevocable for the life time of Geetanshi., Dividend received by Ms. Geetanshi during the previous year 2021-22 is ₹ 13,00,000., (vi), Other income of Mr. Raghav includes, - Interest from saving bank account of ₹ 2,00,000, - Cash gift of ₹ 75,000 received from daughter of his sister on his birthday., Compute the total income of Mr. Raghav for the Assessment Year 2022-23., Solution is given on our website www.mkgeducation.com (Icon /Heading No.11: Books), , NOV – 2019 (NEW COURSE), Question.3. (a), , (4 Marks), st, , Mr. Prakash is in the business of operating goods vehicles. As on 1 April, 2021, he had the following, vehicles:, Vehicle, , Gross Vehicle, Weighted (in Kgs.), , Date of Purchase, , Put to use during F.Y., 2020-21, , A, , 8500 02-04-2020, , Yes, , B, , 13000 15-05-2020, , Yes, , C, , 12000 04-08-2020, , No (as under repairs), , During P.Y. 2021-22, he purchased the following vehicles:, Vehicle, , Gross Vehicle Weight, (in Kgs.), , Date of Purchase, , Date on which put to, use, , D, , 11000 30-4-2021, , 10-5-2021, , E, , 15000 15-5-2021, , 18-5-2021, , Compute his income under Section 44AE of the Income Tax Act, 1961 for A.Y. 2022-23., Solution:, Since Mr. Prakash does not own more than 10 vehicles at any time during the previous year 2021-22, he is, eligible to opt for presumptive taxation scheme under section 44AE. As per section 44AE, ₹1,000 per ton of, gross vehicle weight or unladen weight, as the case may be, per month or part of the month for each heavy, goods vehicle and ₹7,500 per month or part of month for each goods carriage other than heavy goods, vehicle, owned by him would be deemed as his profits and gains from such goods carriage. Heavy goods, vehicle means any goods carriage, the gross vehicle weight of which exceeds 12,000 kg., Computation of Presumptive Income as per section 44AE, Type of carriage, No. of Months, Rate per ton per, Ton, Amount, the vehicle is, month, owned by Mr., Prakash, (1), (2), (3), (4), (5)
Page 747 :
Income Under The Head Business/Profession, , 228, [(2) x (3) x (4), , Heavy goods vehicle, Vehicle B (13,000, kgs) held throughout, the year, Vehicle E (15,000, kgs) purchased on, 15.5.2021, Goods vehicles other, than heavy goods, vehicle, Vehicle A held, throughout the year, Vehicle C held, throughout the year, Vehicle D purchased, on 30.4.2021, , 12, , 1,000, , 13 (13,000/1,000), , 1,56,000, , 11, , 1,000, , 15 (15,000/1,000), , 1,65,000, , 12, , 7,500, , -, , 90,000, , 12, , 7,500, , -, , 90,000, , 12, , 7,500, , -, , 90,000, , Total, 5,91,000, The “put to use” date of the vehicle is not relevant for the purpose of computation of presumptive income, under section 44AE, since the presumptive income has to be calculated per month or part of the month for, which the vehicle is owned by Mr. Prakash., , MAY – 2019 (OLD COURSE), Question 3(b), , (4 Marks), , M/s. Keshav Enterprises, a sole proprietorship own four machines, put in use for business in March, 2020., The depreciation on these machines is charged @ 15%. The written down value of these machines as on 1st, April, 2021 was ₹ 7,70,000. Two of the old machines were sold on 15th July, 2021 for ₹ 10,00,000. A, second hand plant was bought for ₹ 6,10,000 on 30th December, 2021. You are required to:, (i) Determine the claim of depreciation for Assessment Year 2022-23., (ii) Compute the capital gains liable to tax for Assessment Year 2022-23., (iii) If Keshav Enterprises has sold the two machines in July, 2021 for ₹ 15,00,000, explain, will there, be any difference in your above workings?, Solution:, (i) Computation of Depreciation for the A.Y. 2022-23, WDV of Machines as on 01st April 2021, 7,70,000, Add: Addition during the year (on 30th December 2021), 6,10,000, Less: Sold during the year (on 15th July 2021), (10,00,000), WDV as on 31st March 2022, 3,80,000, Depreciation during the year (3,80,000 x 7.5%), 28,500, (ii) Computation of Capital Gains for the A.Y. 2022-23, Since block of asset exists at the end of the year and WDV is also there at the end of the year hence no, capital gains shall be computed., (iii) If asset sold for 15,00,000, WDV of Machines as on 01st April 2021, 7,70,000, Add: Addition during the year (on 30th December 2021), 6,10,000, Less: Sold during the year (on 15th July 2021), (15,00,000), Nil, WDV as on 31st March 2022, Depreciation during the year, Nil, If there is a negative balance at the end of the year, it will be considered to be short term capital gains as per, section 50., Short term Capital Gains, Sale Consideration, 15,00,000, Less: Cost of Asset, (13,80,000)
Page 748 :
Income Under The Head Business/Profession, Short term capital gains, , 229, 1,20,000, , NOV – 2018 (NEW COURSE), Question 1, (10 Marks), Miss Sudha, a resident individual, aged 32 years, furnishes the following particulars relating to the year, ended 31-3-2022:, (a) Analysis of her bank account in her ledger reveals the under-mentioned data:, (i) Winnings from a TV Game show (Net), 70,000, (ii) Gift received from mother's father, 80,000, (iii) Gift received from Ramya, her close friend, 60,000, (iv) Interest on capital received from Vidyut & Co., a, 3,00,000, partnership firm in which she is a partner (@ l5% p.a.), (v) Rent received for a vacant plot of land, 2,00,000, (vi) Amount received from Sharks Pvt. Ltd., for a house at Salem for which, 1,50,000, she had been in negotiation for enhanced rent three years back. This has not been taxed in any, earlier year. The house was, however, sold off in March, 2021., (vii) Amount received under KeyMan Insurance Policy, 2,20,000, (viii) Amount forfeited by a buyer of her vacant plot, since, 3,10,000, the buyer could not finalize the deal as per agreement., (b) Donation given in cash to a charitable trust registered u/s 12AA, 12,000, (c) She owns agricultural lands at Colombo, Sri Lanka. She has derived, 1,80,000, agricultural income therefrom, (d) (i) Public Provident Fund paid in the name of her minor daughter, 75,000, (ii) Interest credited in the said PPF account during the year, 8,900, (e) Share of profits received from Vidyut & Co.,, 1,90,000, You are required to compute the total income of the assessee and the tax payable for the assessment year, 2022-23., Computation should be made under proper heads of income., Solution: Computation of Total Income and Tax Payable, Income under the head House Property, Arrears of rent u/s 25A [Taxable, even if Miss. Sudha is no longer the, owner of house property], Less: Deduction @ 30%, , ₹, 1,50,000, (45,000), 1,05,000, , Income under the head business profession, Interest on capital @12%, being the maximum allowable interest [₹3,00,000/15% x 12%] assuming, interest@12% is authorized by the partnership deed and has been allowed as deduction while computing the, income of the firm, 2,40,000, Share of profit from Vidyut & Co., a firm [Exempt], Nil, Income under the head other sources, Winnings from a TV Game show (70,000/70%) (section 56), Gift received from mother's father (section 56 - received from relative so not taxable), Gift received from Ramya, her close friend (section 56 - received from friend so taxable), Rent received for a vacant plot of land (section 56), Amount received under KeyMan Insurance policy (section 56), Forfeiture of advance money on sale of vacant plot (section 56), Agriculture Income from outside India is taxable u/s 56, Interest from PPF account exempt u/s 10(11), Share of Profits from Vidyut & co. exempt u/s 10(2A), Income under the head other sources, , 1,00,000, Nil, 60,000, 2,00,000, 2,20,000, 3,10,000, 1,80,000, Nil, Nil, 10,70,000
Page 749 :
Income Under The Head Business/Profession, Gross Total Income, Less: Deduction u/s 80C- PPF, Total Income, Computation of Tax Payable, Tax on 12,40,000 at slab rate, Tax on winning from TV show @ 30% on 1,00,000 u/s 115BB, Tax before HEC, Add: HEC @ 4%, Tax Liability, Less: TDS, Tax Payable, , 230, 14,15,000, (75,000), 13,40,000, 1,84,500, 30,000, 2,14,500, 8,580, 2,23,080, (30,000), 1,93,080, , Note:, 1. Deduction under section 80G is not allowed because payment is made by cash., 2. Income from agricultural land in India is exempt but income from agricultural land outside India, shall be taxable under head other sources., 3. Winning from a TV game show is liable to TDS @ 30% and net amount is given in question and, gross amount is taxable so for gross income amount will be divided by 70%., 4. Interest on capital received from partnership firm shall be taxable upto 12%., 5. Amount received from Keyman is taxable under income from other sources alternatively it can be, taxed under the head PGBP., Alternate Solution:, Maternal grandparents are covered under lineal ascendant or not is controversial. Student can take, any view. In the above solution gift from mother’s father is assumed to be covered under the, definition of relative under lineal ascendant and it is exempt u/s 56 but if it is assumed that maternal, grandparents is not covered under relative then such amount shall be taxable and treatment shall be, Solution: Computation of Total Income and Tax Payable, ₹, Income under the head House Property, Arrears of rent u/s 25A [Taxable, even if Miss. Sudha is no longer the, owner of house property], 1,50,000, Less: Deduction @ 30%, (45,000), 1,05,000, Income under the head business profession, Interest on capital @12%, being the maximum allowable interest [₹3,00,000/15% x 12%] assuming, interest@12% is authorized by the partnership deed and has been allowed as deduction while computing the, income of the firm, 2,40,000, Share of profit from Vidyut & Co., a firm [Exempt], Nil, Income under the head other sources, Winnings from a TV Game show (70,000/70%) (section 56), Gift received from mother's father (section 56), Gift received from Ramya, her close friend (section 56 - received from friend so taxable), Rent received for a vacant plot of land (section 56), Amount received under KeyMan Insurance policy (section 56), Forfeiture of advance money on sale of vacant plot (section 56), Agriculture Income from outside India is taxable u/s 56, Interest from PPF account exempt u/s 10(11), Share of Profits from Vidyut & co. exempt u/s 10(2A), Income under the head other sources, , 1,00,000, 80,000, 60,000, 2,00,000, 2,20,000, 3,10,000, 1,80,000, Nil, Nil, 11,50,000, , Gross Total Income, , 14,95,000
Page 750 :
Income Under The Head Business/Profession, Less: Deduction u/s 80C- PPF, Total Income, Computation of Tax Payable, Tax on ₹13,20,000 at slab rate, Tax on winning from TV show @ 30% on 1,00,000 u/s 115BB, Tax before HEC, Add: HEC @ 4%, Tax Liability, Less: TDS, Tax Payable, , 231, (75,000), 14,20,000, 2,08,500, 30,000, 2,38,500, 9,540, 2,48,040, (30,000), 2,18,040, , Note:, 1. Deduction under section 80G is not allowed because payment is made by cash., 2. Income from agricultural land in India is exempt but income from agricultural land outside India, shall be taxable under head other sources., 3. Winning from a TV game show is liable to TDS @ 30% and net amount is given in question and, gross amount is taxable so for gross income amount will be divided by 70%., 4. Interest on capital received from partnership firm shall be taxable upto 12%., Amount received from Keyman is taxable under income from other sources alternatively it can be taxed, under the head PGBP., Question 4(b), (4 Marks), Mr. Rangamannar resides in Delhi. As per new rule in the city, private cars can be plied in the city only on, alternate days., He has purchased a car on 21-09-2021, for the purpose of his business as per following details:, Cost of car (excluding GST) 12,00,000, Add: Delhi GST at 14%, 1,68,000, Add: Central GST at 14%, 1,68,000, Total price of car, 15,36,000, He estimates the usage of the car for personal purposes will be 25%. He is advised that since the car has run, only on alternate days, half the depreciation, which is otherwise allowable, will be actually allowed., He has started using the car immediately after purchase., Determine the depreciation allowable on car for the AY 2022-23, if this is the only asset in the block., Rate of depreciation may be taken at 15%, If this car were to be used in the subsequent Assessment Year 2023-24 on the same terms and conditions, above, what will be the depreciation allowable? Assume that there is no change in the legal position under, the Income-Tax Act, 1961., Solution:, Computation of Depreciation Allowable, Since the car was put to use for more than 180 days in the P.Y.2021-22, full depreciation@15% is allowable, on the actual cost of ₹15,36,000, which is the total price (inclusive of GST). However, the depreciation, actually allowed would be restricted to 75%, since 25% of usage is estimated for personal use, on which, depreciation is not allowable, Block of Assets, ₹, Block 1: Plant and machinery – rate 15%, ₹15,36,000 @ 15% (D.O.P – 21-09-2021), 2,30,400, Depreciation allowed for FY 21-22 (75% for official use) (2,30,400 x 75%), 1,72,800, WDV as on 31.03.2022 (15,36,000 – 2,30,400), 13,05,600, ₹13,05,600 @ 15%, 1,95,840, Depreciation allowed for FY 22-23 (75% for official use) (1,95,840 x 75%), 1,46,880, WDV as on 31.03.2023 (13,05,600 – 1,95,840), 11,09,760
Page 751 :
Income Under The Head Business/Profession, , 232, , Note: Since ITC for Motor car is not allowed under GST, GST paid shall be considered to be part of the, cost., Alternative Solution: In the above solution WDV is calculated deducting full depreciation (including, personal use) alternatively student can solve the solution by deducting only allowed depreciation (i.e., official use depreciation allowed under income tax), Block of Assets, ₹, Block 1: Plant and machinery – rate 15%, ₹15,36,000 @ 15% (D.O.P – 21-09-2021), 2,30,400, Depreciation allowed for FY 21-22 (75% for official use) (2,30,400 x 75%), 1,72,800, WDV as on 31.03.2022 (15,36,000 – 1,72,800), 13,63,200, ₹13,63,200 @ 15%, 2,04,480, Depreciation allowed for FY 22-23 (75% for official use) (2,04,480 x 75%), 1,53,360, WDV as on 31.03.2023 (13,63,200 – 1,53,360), 12,09,840, Note: Since ITC for Motor car is not allowed under GST, GST paid shall be considered to be part of the, cost., Question 6, (5+5 Marks), Answer any two of the following three sub-parts., (a) Mr. Rakesh Gupta has derived the following income/loss, as computed below, for the previous year, 2021-22:, Loss from let out house property, 2,50,000, Loss from non-speculation business, 3,20,000, Income from speculation business, 12,45,000, Loss from specified business covered u/s 35AD, 4,10,000, Winnings from lotteries (Gross), 1,50,000, Winnings from bettings, 90,000, Loss from card games, 3,40,000, You are required to compute the total income of the assessee for the assessment year 2022-23, showing, clearly the manner of set-off and the items eligible for carry forward. The return of income has been filed on, 30-7-2022, Solution:, Computation of Gross Total Income, ₹, Income from speculation business, 12,45,000, Less: loss from non - speculation business (section 70), (3,20,000), Less: Loss from house property (section 71), (2,00,000), Income from Speculation business, 7,25,000, Income under the head other sources, Income from lotteries, Income from bettings, Income under the head other sources, , 1,50,000, 90,000, 2,40,000, , Gross Total Income, 9,65,000, Less: Deduction u/s 80C to 80U, Nil, Total Income, 9,65,000, Note:, (1) As per Section 71, loss from house property shall be allowed to be adjusted from other head income, maximum upto ₹2,00,000 and balance amount i.e. ₹50,000 shall be carried forward (section 71B) for, a maximum period of 8 years and shall be set off from the income under the head house property, only., (2) Loss from card games shall not be treated as loss and have no treatment.
Page 752 :
Income Under The Head Business/Profession, , 233, , (3) Income from betting and lotteries is chargeable at a flat rate of 30% under section 115BB and no, expenditure or allowance can be allowed as deduction from such income, nor any loss be set-off, against such income., As per section 73A, loss of specified business can be set off only from profits and gains of any other, specified business and carried forward is allowed for unlimited periods and in the subsequent years also, the, loss can be set off only from income of specified business., , NOV – 2018 (OLD COURSE), Question 1, (10 Marks), Mr. Murari, a resident individual, age 48 years provides consultancy services in the field of accountancy., His Income and Expenditure account for the year ended 3lst March , 2022 is as follows :, Income and Expenditure account for the year ending 31st March, 2022, Expenditure, Income, Amount (₹), Amount (₹), 8,00,000, By Consulting fees, To Salary, 3,00,000, 25,000, By Share of Profit from, To Motor car expenses, 58,000, HUF, To Depreciation, 47,500, 15,000, By Interest on saving, To Medical expenses, 70,000, bank deposits, To Purchase of computer, 80,000, 8,000, By Interest on income, To Bonus, 10,000, tax refund, To General expenses, 55,000, To Office & administrative, 75,000, To Excess of income over, 1,52,500, expenditure, 8,48,000, 8,48,000, The following other information relates to the financial year 2021-2022:, (1) Salary includes a payment of ₹12,000 per month to his brother-in-law who is in-charge of the marketing, department. However, in comparison to similar business, the reasonable salary of a marketing supervisor is, ₹10,000 per month., (2) Interest on saving bank deposit belongs to his wife who has deposited the money out of the pocket, money given to her every month., (3) Written down value of the assets as on 1st April, 2021 are as follows :, Motor Car (40% used for personal use), ₹2,00,000, Furniture and Fittings, ₹50,000, (4) Medical expenses includes:, • Family planning expenditure ₹15,000 incurred for the employees which was revenue in nature., • Medical expenses for his father ₹35,000. (Father's age is 65 years), (5) The computer was purchased on 5th June, 2021 on credit. The total invoice was paid in the following, manner:, • ₹18,000 paid in cash as down payment on the date of purchase., • Remaining amount was paid through account payee cheque on 10th August, 2021., (6) Bonus was paid on 31st October, 2022., (7) General expenses include commission payment of ₹22,000 to Mr. Sridhar for the promotion of business, on 17th September, 2021 without deduction of tax at source., (8) He also received gold coins from a family friend on the occasion of marriage anniversary on 5th, December, 2021. The market value of the coins on the said date was ₹55,000., Compute the total income and the tax liability of Mr. Murari for the assessment year 2022-2023, Solution: Computation of Total Income and Tax Liability, Excess of Income over expenditure, , ₹, 1,52,500.00
Page 753 :
Income Under The Head Business/Profession, Add: inadmissible expenses, Motor car expense (personal use 40% x 58,000), Purchase of computer, Depreciation as per books, Family planning expenditure on employees, Medical expense for his father is a personal expenditure, Bonus being paid to employee after the due date of filing the return (section 43B), Less:, Share of Profit from HUF, Interest on saving bank deposits, Interest on Income-tax refund, Depreciation on motor car (₹2,00,000 @ 15% x 60%) (section 32), Depreciation on computer (₹62,000 @ 40%) (section 32), Depreciation on furniture (₹50,000 @ 10%) (section 32), Income under the head Business/profession, Income under the head other sources, Gift from friend on anniversary is taxable (section 56), Share of profit from HUF is exempt (section 10 (2)), Interest from saving bank account (clubbed u/s 64), Interest on income tax refund, Income under the head other sources, Gross Total Income, Less: Deduction u/s 80D (Medical expenditure on his father), Less: Deduction u/s 80TTA, Total Income, Computation of Tax Liability, Tax on ₹3,00,400 at slab rate, Less: Rebate u/s 87A, Tax Liability, , 234, 23,200.00, 80,000.00, 47,500.00, 15,000.00, 35,000.00, 10,000.00, (25,000.00), (15,000.00), (8,000.00), (18,000.00), (24,800.00), (5,000.00), 2,67,400.00, 55,000.00, Nil, 15,000.00, 8,000.00, 78,000.00, 3,45,400.00, (35,000.00), (10,000.00), 3,00,400.00, 2,520.00, (2,520.00), Nil, , Notes:, 1. As per section 36(1)(ix), family planning expenditure is allowed to companies in 5 annual equal, instalments. In the given case assessee is an individual hence it is not allowed., 2. As per section 43 payment of capital expenditure made in cash in excess of ₹10,000 shall not be, included in cost of the asset., Question 4(a), (5 Marks), Mr. Dheeraj, a resident individual, is a dealer of food grains. During the previous year 2021-2022, total, turnover of his business was ₹80 lakhs. (out of which ₹15 lakhs was received in account payee cheques and, balance in cash). He estimates similar turnover in the previous year 2022-2023. As suggested by his tax, consultant, Mr. Dheeraj wants to opt for computation of profit and gains of business on presumption basis, under section 44AD for the previous year 2022-2023., Guide Mr. Dheeraj relating to the provisions of advance tax with its due date along with the amount payable,, if he opts for the above mentioned presumptive taxation., Solution:, Special provision for computing profits and gains of business on presumptive basis. Section 44AD, 1. If any assessee has turnover of his business upto ₹200 lakhs, such assessee is allowed to compute income, on presumptive basis and income under the head business/profession shall be presumed to be 8% of the, turnover and no further deduction is allowed under section 30 to 38., 2. Such option is allowed only to an Individual/ HUF / Firm who are resident but not to LLP or Company.
Page 754 :
Income Under The Head Business/Profession, , 235, , 3. Section 44AD is applicable only to business and not to specified profession and also it is not applicable, for the persons having earning as commission or brokerage., 4. Such assessee shall be required to pay advance tax to the extent of 100% of tax liability on or before 15th, March of the relevant previous year otherwise interest shall be charged @ 1% for one month on the amount, of deposit default., 5. The assessee shall be exempt from maintaining books of accounts or audit., Rate of 6% shall be applied instead of 8% if the amount of total turnover or gross receipts which is received, by an account payee cheque or an account payee bank draft or use of electronic clearing system through a, bank account during the previous year or before the due date specified in subsection (1) of section 139 in, respect of that previous year., As per the above provision, Mr. Dheeraj can opt for presumptive taxation u/s 44AD and 100% of his tax, liability shall be payable as advance tax on or before 15th March of the relevant previous year otherwise 1%, interest shall be charged for one month on the amount of default., As per above provision, 8% of turnover shall be considered as business income if payment is received in, cash and 6% of turnover is payable in case payment is received in account payee cheque., Income under the head business/profession, 8% of 65,00,000, 5,20,000, 6% of 15,00,000, 90,000, Income under the head business/profession, 6,10,000, , MAY – 2018 (NEW COURSE), Question 1, (10 Marks), Mr. Hari, aged 55 years, a resident individual and practicing Chartered Accountant, furnishes you the, receipts and payments account for the financial year 2021-22., Receipts and Payments Account, Receipts, ₹, Payments, ₹, Staff salary, bonus and stipend to articled, 20,50,000, Opening Balances, Clerks, (01-04-2021), 20,000, Other general and administrative expenses, 12,00,000, Cash & Bank, Fee from professional, Services, , 39,60,000, , Office rent, Life insurance premium, , Motor car loan from SBI, @10% interest per annum, , 2,00,000, , Motor car, (Acquired in January 2022 by way of online, payment), Books bought (annual publication by credit, card), Computer acquired on 1-11-2021 for, professional use, Domestic drawings, Motor car maintenance, Public Provident fund subscription, Closing balances (31-03-2022), Cash & Bank, , 41,80,000, , 48,000, 23,000, 4,00,000, 22,000, 25,000, 2,50,000, 12,000, 1,40,000, 10,000, 41,80,000, , Other information:, (i) Motor car was put to use for both official and personal purposes. 1/4th of the motor car is for, personal. No interest on car loan was paid during the year., (ii) Mr. Hari purchased a flat in Jaipur for ₹15,00,000 in July 2018 cost of which was partly financed, by a loan from State Bank of India of ₹10,00,000 @ 10% interest, his own savings ₹1,00,000 and a
Page 755 :
Income Under The Head Business/Profession, , 236, , deposit from Bank of Baroda for ₹4,00,000 on lease for 10 years @ ₹40,000 per month. The, following particulars are relevant:, (a) Municipal taxes paid by Mr. Hari, ₹4,200 per annum, (b) House insurance, ₹1,000, (iii) He earned ₹1,00,000 in share speculation business and lost ₹1,50,000 in commodity speculation, business., (iv) Mr. Hari received a gift of ₹15,000 each from four of his family friends., (v) He contributed ₹1,11,000 to Prime Minister’s Draught Relief Fund by way of bank draft., (vi) He donated to a registered political party ₹3,00,000 by way of cheque., Compute the total income of Mr. Hari and the tax payable for the assessment year 2022-23., Solution:, Computation of income from profession, Gross receipts:, Fee from Professional Service, 39,60,000.00, Payments:, Salary, bonus and Stipend, (20,50,000.00), Other general and administrative expenses, (12,00,000.00), Office rent, (48,000.00), Depreciation on Books @ 40% (22,000 x 40%), (8,800.00), Motor car maintenance (12,000 x ¾), (9,000.00), Depreciation on car @ 15% (4,00,000 x 15%/2 x ¾), (22,500.00), Depreciation on computer @ 40% (25,000 x 40%/2 ), (5,000.00), Income from profession, 6,16,700.00, Computation of income from house property, Gross Annual Value, Less: Municipal Tax, Net Annual Value, Less: 30% of NAV u/s 24(a), Less: Interest on capital borrowed u/s 24(b) (10,00,000 x 10%), Income from house property, Income from Other Sources, Gift received from friends (15,000 x 4) u/s 56(2), Computation of Total Income, Income from profession, Income from house property, Income from other sources, Gross total income, Less: Deduction u/s 80C – LIC, 23,000, PPF 1,40,000 1,63,000 restricted to 1,50,000, Less: Deduction u/s 80G – PMDRF @ 50% (1,11,000 x 50%), Less: Deduction u/s 80GGC, Total Income, Computation of Tax Liability, Tax on ₹4,04,260 at slab rate, Less: Rebate u/s 87A, Tax Liability, Note:, , 4,80,000.00, (4,200.00), 4,75,800.00, (1,42,740.00), (1,00,000.00), 2,33,060.00, 60,000.00, 6,16,700.00, 2,33,060.00, 60,000.00, 9,09,760.00, (1,50,000.00), (55,500.00), (3,00,000.00), 4,04,260.00, 7,713.00, (7,713.00), Nil
Page 756 :
Income Under The Head Business/Profession, , 237, , 1. Date of Purchase of books is not mentioned in question hence full year depreciation is allowed., 2. Speculation loss of 50,000 (1,50,000 – 1,00,000) shall be carried forward for next year., , MAY – 2018 (OLD COURSE), Question 4, (10 Marks), Mr. Querashi is a business man. During the year ended 31-03-2022 he was engaged in the business of, Hypermarket and Super Market. He maintains proper books of accounts for both businesses in mercantile, system. Sales from Hypermarket achieved a turnover of ₹75 lakhs and all receipts were in cash. However,, Supermarket business is through online and entire receipts of ₹50 lakhs during the year were received, through online in his bank account. The expenses were incurred in ratio 65:35., Following additional information is furnished, ₹, To salary, 10,00,000, To repairs on building, 1,81,000, To Interest, 1,10,000, To Travelling, 1,30,550, To depreciation, 8,12,000, Net profit, 3,93,950, (a) In addition to the above, repairs of ₹1,00,000 was incurred for building a new room which was debited to, P & L a/c., (b) Depreciation as per income tax Act is ₹7,17,000., (c) ₹75,000 was paid in cash on 30-09-21 to Mrs. Ann, accountant for preparation of the accounts for the, year ended 31-03-2021 and adjusted under the head “expenses payable” account., (d) He was forced to shutdown his furniture business in the year 2018 as his accountant absconded business, loss of furniture business is ₹3 lakhs. ₹4 lakhs was received as insurance compensation on 31-03-2022 for, the cash theft., Mr. Querashi wants to declare income under “Presumptive income” basis., Compute the income chargeable under the head profits and gains of business or profession of Mr. Querashi, under Presumptive Income scheme under section 44AD and his Total Income for the year ended 31-032022., Solution:, As per section 44AD, If any assessee has turnover of his business upto ₹200 lakhs, such assessee is allowed, to compute income on presumptive basis and income under the head business/profession shall be presumed, to be 8% of the turnover and no further deduction is allowed under section 30 to 38. Brought forward, business loss is allowed to be adjusted from such income but brought forward depreciation is not allowed to, be adjusted from such income., Rate of 6% shall be applied instead of 8% if the amount of total turnover or gross receipts which is received, by an account payee cheque or an account payee bank draft or use of electronic clearing system through a, bank account during the previous year., Computation of Total Income, Sales from Hyper Market in cash, Presumptive Income u/s 44AD (8% of 75,00,000), Sales from Super Market online, Presumptive Income u/s 44AD (6% of 50,00,000), Income under the head Business Profession, Insurance compensation received section 41(1), Less: B/F business loss section 72, Income under the head Business Profession, Gross Total Income, Less: Deduction u/s 80C to 80U, Total Income, , ₹, 75,00,000, 6,00,000, 50,00,000, 3,00,000, 9,00,000, 4,00,000, (3,00,000), 10,00,000, 10,00,000, Nil, 10,00,000
Page 757 :
Income Under The Head Business/Profession, , 238, , NOV – 2017, Question 1, (10 Marks), Mr. Pandey, a resident individual, aged 45 years, is a Chartered Accountant in practice. He maintains his, accounts on cash basis. His profit & Loss Account for the year ended 31st March, 2022 is as follows:, Profit & Loss Account for the year ending March 31, 2022, Expenditure, Amount, Income, Amount, (₹), (₹ ), Fees earned, 18,25,000, Staff salary, 43,00,000, Audit, 6,00,000, Rent of the office, 34,50,000, Taxation, Premises, Consultancy services, 5,75,000, Administrative expenses, relating to syndication of, 1,85,000, Stipend to Articled clerks, loan from financial, Meeting, seminars and, 10,00,000, institution, 36,500, conferences, 87,50,000, 55,000, Depreciation, Gifts, 8,75,000, Printing and Stationery, 1,00,000, Dividends from Indian, 59,13,500, Net Profit, 12,00,000, companies, Interest on deposit, 15,000, certificates issued under, Gold Monetization Scheme,, 2015, 1,00,65,000, 1,00,65,000, Other Information:, (1) Depreciation allowable under Income-Tax Act ₹1,25,000., (2) Administrative expenses include ₹55,000 paid to a tax consultant in cash for assisting Mr. Pandey in, one of the professional assignments., (3) Gifts represent fair market value of a LED TV which was given by one of the clients for successful, presentation of case in the Income Tax Appellate Tribunal., (4) Last month’s rent of ₹50,000 was paid without deduction of tax at source., (5) Mr. Pandey had taken a loan of ₹32,00,000 for the purchase of a house property valuing ₹45,00,000, from a recognized financial institution on 1st May, 2021. He repaid ₹1,50,000 on 31st March, 2022, out of which ₹1,00,000 is towards principal payment and the balance is for interest on loan . The, possession of the property will be handed over to him in October 2022., (6) Mr. Pandey paid medical insurance premium of his parents (senior citizens and not dependent on, him) by cheque amounting to ₹27,000. He also paid ₹8,500 by cash towards preventive health, checkup for himself and his spouse., Compute the total income of Mr. Pandey and tax payable by him for Assessment Year 2022-23., Solution:, Income under the head Business/Profession, Net Profit as per profit and loss account, 59,13,500, Add: Expenses disallowable, Depreciation as per books of accounts, 55,000, Administrative expense paid in cash in excess of ₹ 10,000 section 40A (3), 55,000, Rent Paid without deduction of TDS Section 40(a)(ia) (Disallowed 30%), 15,000, (assumed Mr. Pandey is liable to deduct TDS), Less: Expenses allowable/ Income considered in separate head etc., Dividends, Interest on deposit certificate issued under Gold Monetization scheme, , (12,00,000), (15,000)
Page 758 :
Income Under The Head Business/Profession, , 239, , Depreciation as per Income Tax Act, Income under the head Business/Profession, , (1,25,000), 46,98,500, , Income under the head other sources, Dividend Income, Interest on deposit certificate issued under Gold Monetization scheme, Income under the head other sources, , 12,00,000, Nil, 12,00,000, , Total Income, Income under the head Business/Profession, Income under the head other sources, Gross Total Income, Less: Deduction u/s 80D, Medical Insurance of Parents, PHC of himself limited to ₹ 5,000, Total Income, Computation of Tax Liability, Tax on ₹ 58,66,500 at slab rate, Add: Surcharge @ 10%, Tax before health & education cess, Add: HEC @ 4%, Tax Liability/Tax Payable, Rounded off u/s 288B, , 46,98,500, 12,00,000, 58,98,500, (27,000), (5,000), 58,66,500, 15,72,450.00, 1,57,245.00, 17,29,695.00, 69,187.80, 17,98,882.80, 17,98,880.00, , Notes:, 1. Deduction u/s 80C is not allowed for repayment of loan as the possession of property will be handed over, in next year., 2. Deduction of payment made in cash for PHC is allowed u/s 80D., 3. Gift received from Client shall be considered as Professional Income as per section 28., 4. Interest income from certificate issued under Gold Monetization scheme is exempt u/s 10(15)., Question 5(a)(ii), (3 Marks), Mr. S is engaged in the business of producing and selling toys. During the previous year 2021-22 his, turnover was ₹1.80 crores. He opted for paying tax as per presumptive taxation scheme laid down in section, 44AD., He has no other income during the previous year. Is he liable to pay advance tax and if so,, what is the minimum amount of advance tax to be paid and the due date for payment of such advance tax ?, Answer:, As per section 211, An assessee who declares income under section 44AD, 100% Advance tax shall be paid, on or before 15th March of the financial year., Yes Mr. Subramany is liable to pay 100% advance tax and should be paid upto 15th March 2022., Income under the head business/profession, As per section 44AD, 8% of Turnover shall be considered to be business income., 8% x 1,80,00,000, , 14,40,000, , Computation of Tax Liability, Tax on ₹ 14,40,000 at slab rate, Add: HEC @ 4%, Tax Liability/Tax Payable, , 2,44,500, 9,780, 2,54,280, , Advance Tax Payable before 15th March 2022, , 2,54,280
Page 759 :
Income Under The Head Business/Profession, , 240, , Question 6.(a), (5 Marks), Mr. Prakash furnishes you the following details in respect of the Financial Year 2021-22., (i) Loss from the business carried on by him as a proprietor: ₹11,20,000, (ii) Unabsorbed Depreciation: ₹4,80,000, (iii) Loss from House Property: ₹2,50,000, The due date for filing the return for Mr. Prakash was 31st July, 2022 under section 139(1). However, he, filed the return on 29.9.2022. Discuss with reference to the relevant provisions of Income-Tax Act, 1961 if, the losses could be carried forward by Mr. Prakash., Answer:, In general unadjusted losses are allowed to be carried forward but as per section 139(3), the return of loss, has to be filed further as per section 80 it should be filed in time otherwise carry forward is not allowed. In, the given case return is filed after the due date hence carry forward of losses is not allowed. Section 139(3), do not cover loss from house property and also unabsorbed depreciation, hence carry forward of HP loss, and Depreciation is allowed. As per section 71B, Loss of House property is allowed to be carried forward, for a period of 8 years. As per section 32(2), Carry forward of unabsorbed depreciation shall be allowed, for unlimited period., Losses/Depreciation could be carried forward shall be as given below:, Loss from House Property, ₹ 2,50,000, Unabsorbed Depreciation, ₹ 4,80,000, , MAY – 2017, Question 1(a), (10 Marks), Ms. Rekha a resident individual aged 50, provides the following information for the financial year 2021-22:, (i) She is a partner in AK & CO. and received the following amounts from the firm:, Share of profit from the firm, ₹35,000, Interest on capital @ 15% p.a. ₹3,00,000, Salary as working partner, ₹1,00,000, (fully allowed in the hands of the firm), (ii) She is running a rice mill as proprietor. The Net profit as per Profit & Loss Account is 4,50,000., The following items are debited to Profit and Loss account:, • Advance Income-tax paid, ₹1,00,000, • Personal drawings, ₹ 50,000, The following items are credited to Profit & Loss Account:, • Interest on savings bank account with SBI, ₹12,000, • Interest on savings account with post office ₹ 5,000, • Dividend from listed Indian Company, ₹80,000, (iii) She owned a house property in Mumbai which was sold in January, 2020. She received ₹90,000 by, way of arrear rent in respect of the said property in October, 2021., (iv) She made the following investments:, Life insurance premium on a policy in the name of her daughter ₹60,000. (The policy was taken on, 01.10.2020 sum assured being ₹5,00,000)., Health insurance premium on a policy covering her mother aged 75. She is not dependant on Ms. Rekha., Premium paid by cheque ₹35,000. Compute the Total Income and the tax liability of Ms. Rekha for the, Assessment year 2022-23., Solution:, Computation of Total Income of Ms. Rekha for the A.Y.2022-23, Income under the head Business Profession, ₹, Share of Profit from Partnership Firm (Exempt u/s 10(2A)), Nil, Interest on capital (3,00,000 x 12%/15%), 2,40,000, Salary from Partnership firm, 1,00,000, Net Profit as per Profit and loss Account, 4,50,000, Add: Income Tax Paid, 1,00,000
Page 760 :
Income Under The Head Business/Profession, Add: Personal Drawings, Less: Interest on saving bank account with SBI, Less: Interest on saving bank account with Post Office, Less: Dividend from Indian Company, Income from Rice Mill, Income under the head Business Profession, Income under the head other Sources, Interest on saving bank account with SBI, Interest on saving bank account with Post Office, Less: Exempt u/s 10(15), Interest on saving bank account with Post Office, Dividend from Indian Company, Income under the head other Sources, Income under the head House Property, Arrear of Rent Section 25A, Less: 30%, Income under the head House Property, , 241, 50,000, (12,000), (5,000), (80,000), 5,03,000, 8,43,000, 12,000, 5,000, (3,500), 1,500, 80,000, 93,500, , 90,000, (27,000), 63,000, , Gross Total Income, Less: Deduction u/s 80C, For life Insurance of married daughter (10% of 5,00,000), Less: Deduction u/s 80D, For health Insurance of mother, Less: Deduction u/s 80TTA, For Interest on saving account, Total Income, Computation of Tax Liability, Total Income, Tax on ₹9,04,500 at slab rate, Add: HEC @ 4%, Tax Liability, Rounded off u/s 288B, , 9,99,500, (50,000), (35,000), (10,000), 9,04,500, 9,04,500, 93,400, 3,736, 97,136, 97,140, , Notes:, 1. As per section 40(b) Interest is allowed to the partner’s maximum at a rate of 12% p.a., 2. Interest from post office savings bank account is exempt upto ₹3500., 3. Arrears of rent shall be taxable under the head house property even if the assessee donot have any, house property in his name., 4. If life policy is taken w.e.f. 1st April 2012 onwards, deduction is allowed maximum @ 10% of sum, assured., 5. As per section 80D, maximum deduction allowed for senior citizen shall be ₹ 30,000., , NOV – 2016, Question 1(a), (10 Marks), Mr. X, a resident individual aged 35 years, furnished the following information from his Profit and Loss, Account for the year ended 31st March 2022:, (i) The net profit was ₹6,50,000., (ii) The following incomes were credited in the Profit & Loss Account:, (a) Interest on government securities ₹25,000, (b) Dividend from a foreign company ₹18,000
Page 761 :
Income Under The Head Business/Profession, , 242, , (c) Gold coins worth ₹55,000 received as gift from his father., (iii) Depreciation debited in the books of account was ₹85,000. Depreciation allowed as per Income-tax, Act, 1961 was ₹96,000., (iv) Interest on loan amounting to ₹68,000 was paid in respect of capital borrowed for the purchase of, the new asset which has not been put to use till 31st March 2022., (v) General expenses included:, (a) An expenditure of ₹20,500 which was paid by a bearer cheque., (b) Compensation of ₹4,500 paid to an employee while terminating his services in business unit., He contributed the following amounts by cheque:, (a) ₹45,000 in Sukanya Samridhi Scheme in the name of his minor daughter Alpa., (b) ₹20,000 to the Swachh Bharat Kosh set up by the Central Government., (c) ₹28,000 towards premium for health insurance and ₹2,500 on account of preventive health, check up for self and his wife., (d) ₹30,000 on account of medical expenses of his father aged 82 years (no insurance scheme had, been availed on the health of his father)., You are required to compute the total income of Mr. X for the Assessment Year 2022-23., Solution:, Income under the head business/profession, ₹, Net Profit as per profit and loss account, 6,50,000.00, Less:, • Interest on Govt. Securities, (25,000.00), • Dividend from foreign company, (18,000.00), • Gold received as gift, (55,000.00), • Depreciation allowed as per income tax, (96,000.00), Add:, • Depreciation allowed as per income tax, 85,000.00, • Interest on Loan to be capitalised, 68,000.00, • Expenditure paid in bearer cheque exceeding ₹10,000 Section 40A(3), 20,500.00, Income under the head business/profession, 6,29,500.00, Income under the head other sources, Interest on Govt. Securities, 25,000.00, Dividend from foreign company, 18,000.00, Gold received as gift exempt (Received from Relative), Nil, Income under the head Other Sources, 43,000.00, Gross Total Income, 6,72,500.00, Less: Deduction u/s 80C - Sukanya Samridhi Scheme, (45,000.00), Less: Deduction u/s 80D (₹25,000+30,000), (55,000.00), Less: Deduction u/s 80G - Swachh Bharat Kosh, (20,000.00), Total Income, 5,52,500.00, Note:, 1. Dividend received from foreign company is taxable., 2. Payment of Interest on loan taken for purchase of new asset from the date of purchase till the date of, put to use shall not be allowed as deduction rather it shall be capitalized., , MAY – 2016, Question 5(a), (8 Marks), Venus Ltd., engaged in manufacture of pesticides, furnishes the following particulars relating to its, manufacturing unit at Chennai (for the year ending 31.03.2022):, (₹ in lacs), Opening WDV of Plant and Machinery, 20, New machinery purchased on 01.09.2021, 10, New car purchased on 01.12.2021, 8
Page 762 :
Income Under The Head Business/Profession, , 243, , Computer purchased on 03.01.2022, 4, Additional information:, • All assets were put to use immediately., • Computer has been installed in the office., • During the year ended 31.03.2021, a new machinery had been purchased on 05.10.2020, for ₹10 lacs., Additional depreciation, besides normal depreciation, had been claimed thereon., • Depreciation rate for machinery may be taken as 15%., Compute the depreciation available to the assessee as per the provisions of the Income-tax Act, 1961 and the, WDV of different blocks of assets as on 31.03.2022., Solution: Computation of depreciation allowable for A.Y. 2022-23, Block of Assets, ₹, Block 1: Plant – rate 40%, Computer, 4,00,000, Less: depreciation 4,00,000 @ 40% x1/2, (80,000), Wdv as on 31.03.2022, 3,20,000, Block 2: Plant & Machinery – rate 15%, Opening WDV of the block, 20,00,000, Add: Machinery purchased on 01.09.2021, 10,00,000, Less: depreciation 30,00,000 @ 15%, (4,50,000), Less: additional depreciation 10,00,000 @ 20%, (2,00,000), Less: additional depreciation 10,00,000 @ 10%, (1,00,000), Wdv as on 31.03.2022, 22,50,000, Block 3: Motor Car – rate 15%, Opening WDV of the block, Nil, Add: Car purchased on 01.12.2021, 8,00,000, Less: depreciation 8,00,000 @ 15% x ½, (60,000), Wdv as on 31.03.2022, 7,40,000, Note:, 1. If asset is used for less than 180 days then depreciation shall be allowed at half of the normal rate, 2. If asset is put to use for less than 180 days then additional depreciation is allowed at half of the, normal rate in the year of purchase and balance shall be allowed in the subsequent year., 3. Additional depreciation shall not be allowed on any machinery installed in office and road transport, vehicles., Question 6(a), Mr. Aditya furnishes the following details for year ended 31.03.2022., Loss from speculative business A, Income from speculative business B, Loss from specified business covered under section 35AD, Net Income from salary, Loss from house property, Income from Trading Business, Long term capital gain on sale of urban land, Long term capital loss on sale of shares (STT not paid), Following are the brought forward losses:, (1) Losses from owning and maintaining race horses pertaining to A.Y. 2021-22 ₹2,000., (2) Brought forward loss from trading business ₹5,000 relating to A.Y. 2018-19., Compute the total income of Mr. Aditya and show the items eligible for carry forward., Solution: Calculation of Total Income of Mr. Aditya for the Assessment Year 2022-23, Income under the head Business/Profession, Income from Business, , (8 Marks), ₹, (25,000), 5,000, (20,000), 2,50,000, (1,50,000), 45,000, 2,00,000, (75,000), , ₹, 45,000
Page 763 :
Income Under The Head Business/Profession, , 244, , Less: Loss Brought forward from business, (5,000), Income from Business, 40,000, Income from speculative business B, 5,000, Less: Loss from speculative business A, (5,000), Income from speculative business, Nil, Income under the head Business/Profession, 40,000, Income under the head Salary, Salary, 2,50,000, Less: Loss from house property, (1,50,000), Income under the head Salary after set off, 1,00,000, Income under the head Capital Gains, Long Term Capital Gains, 2,00,000, Less: Long term loss, (75,000), Income under the head Capital Gains, 1,25,000, Gross Total Income, 2,65,000, Less: Deduction u/s 80C to 80U, Nil, Total Income, 2,65,000, Items eligible for carry forward, • Loss from speculative business A of A.Y. 2022-23 ₹20,000 to be carried forward for 4 years starting, from assessment year 2023-24., • Loss from specified business covered u/s 35AD of A.Y. 2022-23 ₹20,000 to be carried forward, starting from assessment year 2023-24., • Losses from owning and maintaining race horses pertaining to A.Y. 2021-22 ₹2,000 shall be carried, forward., , NOV – 2015, Question 5(a)., (8 Marks), State with reasons, the allowability of the following expenses incurred by XYZ Limited, a wholesale dealer, of commodities, under the Income Tax Act, 1961 while computing Profit & Gains from business or, profession for the Assessment Year 2022-2023., (i) Construction of school building in compliance with CSR activities amounting to ₹5,60,000., (ii) Purchase of building for setting up a warehousing facility for storage of food grains amounting to, ₹4,50,000., (iii) Interest on loan paid to Mr. X (a resident) ₹50,000 on which tax has not been deducted., (iv) Commodity transaction tax paid ₹20,000 on sale of bullion., Solution 5 (a):, (i) Any expenditure incurred by an assessee on the activities relating to corporate social responsibility, referred to in section 135 of the Companies Act, 2013 shall not be deemed to be an expenditure incurred by, the assessee for the purposes of the business or profession., (ii) As per section 35AD, assessee shall be allowed to debit 100% of the expenditure incurred in connection, with warehousing facility or agricultural produce hence assessee shall be allowed to debit 4.5 lakhs x 100%, = 4.5 lakh., (iii) Payment of Interest is subject to TDS. Since no tax is deducted at source, the expenditure of ₹15,000, (i.e. 30% of ₹50,000) is disallowed under section 40(a) and balance 70% is allowed., (iv) An amount equal to commodity transaction tax paid by the assessee shall be allowable as deduction,, under section 36(1)(xvi), if the income arising from taxable commodities transactions is included in the, income computed under the head “Profits and gains of business or profession”. In the given case, MN, Limited, is entitled to claim deduction in respect of commodity transaction tax of ₹ 20,000 paid by him., , MAY – 2015, Question 1(a)., (10 Marks), Dr. Shashank is a noted child specialist of Mumbai. His Income and Expenditure account for the financial, year ended 31.03.2022 is given below:
Page 764 :
Income Under The Head Business/Profession, Expenditure, , 245, , Amount, Income, Amount, (₹), (₹), To Staff salary, 2,78,000 By Fee receipts, 16,76,000, To Administrative expenses, 1,64,000 By Winning at TV game, 35,000, show (Net of TDS), To Medicine consumed, 3,95,800 By LIC policy matured, 1,15,000, To Consumables, 57,500 By Honorarium for, 24,000, giving, lectures, at, seminars, To Depreciation, 1,25,000, To Rent of clinic, 1,20,000, To Donation to National Children's Fund, 51,000, To Net Profit, 6,58,700, Total, 18,50,000, Total, 18,50,000, (1) Depreciation computed as per Income Tax Rules, 1962 has been ascertained at ₹75,000., (2) Medicines consumed include cost of medicine for self and family of ₹18,000 and for treating poor, patients of ₹24,000 from whom he did not charged any fee either., (3) Salary includes ₹30,000 paid in cash to a computer specialist who computerized his patient’s data on 30th, September, 2021 at 3 p.m., (4) Donation to National Children’s Fund has been made by way of an account payee cheque., (5) He has paid a sum of ₹25,000 for a life insurance policy (sum assured ₹2,00,000) of himself, which was, taken on 01.07.2021., (6) He also contributed ₹1,20,000 towards Public Provident Fund., (7) Dr. Shashank also paid interest of ₹10,000 on loan taken for higher education of his daughter., (8) Dr. Shashank paid Medical Insurance Premium by cheque of ₹25,000., (9) Dr. Shashank also made donation of ₹1,00,000 by cheque to a charitable trust registered & eligible for, deduction under Income Tax Act, 1961., Dr. Shashank has not opted for presumptive taxation of Income u/s 44ADA. You are required to compute, the total income and tax payable by Dr. Shashank for the assessment year 2022-23., Solution: Computation of Total Income of Dr. Shashank for A.Y. 2022-23, ₹, Income from profession, Net profit as per Income and Expenditure account, 6,58,700, Less :, LIC policy matured, (1,15,000), Honorarium for giving lectures at seminars, (24,000), Winning at TV game show (net of TDS), (35,000), Depreciation as per income tax rules, (75,000), Add:, Donation to National Children's Fund, 51,000, Depreciation, 1,25,000, Medicine Consumed for self & Family, 18,000, Salary Paid in cash disallowed u/s 40A (3), 30,000, Income under the head Business/Profession, 6,33,700, Income from other sources, LIC policy matured Exempt u/s 10(10D), Nil, Honorarium for giving lectures at seminars, 24,000, Winning at TV game show (₹35,000 + ₹15,000), 50,000, Income under the head Other Sources, 74,000, Gross Total Income, 7,07,700, Less: Deductions under Chapter VI A, Deduction under section 80C, Premium of Life Insurance Policy (10% of sum assured i.e. 10% x 2,00,000), (20,000)
Page 765 :
Income Under The Head Business/Profession, , 246, , Contribution towards PPF, (1,20,000), Deduction under section 80D, (25,000), Deduction under section 80E, Interest on loan taken for higher education is deductible, (10,000), Less: Deduction u/s 80G, (i) National Children Fund, (51,000), (ii) Charitable trust, (26,635), Working Note:, Charitable trust, 1,00,000, AGTI = GTI – Deduction u/s 80C to 80U (except 80G), = 7,07,700 – 1,40,000 – 25,000 – 10,000, = 5,32,700, Qualifying amount = 10% of AGTI or donation whichever is less, = 53,270 or 1,00,000 whichever is less, = 53,270, 50% of the qualifying amount = 26,635, Total Income, 4,55,065, Rounded off u/s 288A, 4,55,070, Computation of Tax Liability, Tax on ₹50,000 @ 30% u/s 115BB, 15,000.00, Tax on normal income ₹4,05,070, 7,753.50, Less: Rebate u/s 87A, (12,500.00), Tax before health & education cess, 10,253.50, Add: HEC @ 4%, 410.14, Tax Liability, 10,663.64, Less: TDS, (15,000.00), Tax Refund, (4,336.36), Rounded off u/s 288B, (4,340.00), Note: Winnings from Lottery should be grossed up for the chargeability under the head “Income from other, sources” (₹35,000 + ₹15,000). Thereafter, while computing tax liability, TDS of ₹15,000 should be deducted, to arrive at the tax payable. Winnings from lottery are subject to tax @ 30% as per section 115BB., , NOV – 2014, Question 1(a)., (10 Marks), Mr. X an Indian Resident aged 38 years carries on his own business. He has prepared following Profit &, Loss A/c for the year ending 3l.03.2022:, Particulars, ₹, Particulars, ₹, Salary, 48,000 Gross Profit, 4,30,400, Advertisement, 24,000 Cash Gift (on the occasion of, 1,20,000, Marriage), 5,400, Sundry Expenses, 54,500 Interest on Debentures (Listed in, recognised stock Exchange) Net of, Taxes, Fire Insurance (₹10,000 relates to, 30,000, House Property), Income Tax, 27,000, Household expenses, 42,500, Depreciation (Allowable), 23,800, Contribution to a University approved, 1,00,000, and notified U/s. 35(1)(ii), Municipal taxes paid for House, 36,000, property
Page 766 :
Income Under The Head Business/Profession, Printing & Stationary, Repairs & Maintenance, Net Profit, , 12,000, 24,000, 1,34,000, 5,55,800, , 247, , 5,55,800, , Other information:, (i) Mr. X owns a House Property which is being used by him for the following purposes:, - 25% of the property for own business, - 25% of the property for self-residence, - 50% let out for Residential purpose, (ii) Rent received from 50% let out portion during the year was ₹1,65,000., (iii) On 0l.12.2021 he acquired a vacant site from his friend for ₹1,05,000. The State Stamp Valuation, Authority fixed the value of the site at ₹2,80,000 for stamp duty purpose., (iv) He received interest on Post office Savings bank Account amounting to ₹500, (v) Cash gift on the occasion of marriage includes gift of ₹20,000 from Non-relatives., (vi) LIC premium paid (Policy value ₹3,00,000 taken on 01.06.2021) ₹60,000 for his handicapped son who, is not dependant on Mr. X., (vii) He purchased 10000 shares of X Company Ltd on 01.01.2016 for ₹1,00,000 and received a 1 : 1 bonus, on 0l.01.2016. He sold 5000 bonus shares in September 2021 for ₹ 2,20,000. (Shares are not listed and STT, not Paid)., Compute Total Income and Net Tax payable by Mr. X for the Assessment Year 2022-23., Solution:, Computation of total income of Mr. X, ₹, Net profits as per profit and loss account, 1,34,000, Add: Inadmissible Expenses, • Fire Insurance, 7,500, • Income Tax, 27,000, • Household expenses, 42,500, • Municipal taxes paid for House property, 27,000, Less:, • Cash Gift (on the occasion of Marriage), (1,20,000), • Interest on Debentures (Listed in recognised stock Exchange), (5,400), Income under the head Business/Profession, 1,12,600, Income under the head House Property, Gross annual value, 1,65,000, Less: municipal taxes, (18,000), Net annual value, 1,47,000, Less: Deduction u/s 24(a) @ 30% of NAV, (44,100), Less: Interest on capital borrowed u/s 24(b), Nil, Income from house property, 1,02,900, Income from Capital Gains (Long Term Capital Gains on shares), 2,20,000, Income under the head Other Sources, Cash Gift (on the occasion of Marriage), Nil, Interest on Debentures (Listed in recognised stock Exchange), 6,000, Interest on post office saving account (exempt u/s 10(15)), Nil, Acquired vacant site (2,80,000-1,05,000), 1,75,000, Income under the head Other Sources, 1,81,000, Gross Total Income, 6,16,500, Less: Deduction u/s 80C (15% of 3,00,000), (45,000), Total Income, 5,71,500, Computation of Tax Liability, Tax on Long term capital gain ₹2,20,000 @ 20%, 44,000.00, Tax on normal income ₹3,51,500 at slab rate, 5,075.00, Tax before health & education cess, 49,075.00, Add: HEC @ 4%, 1,963.00
Page 767 :
Income Under The Head Business/Profession, Tax Liability, Less: TDS, Net Tax Payable, Rounded off u/s 288B, , 248, 51,038.00, (600.00), 50,438.00, 50,440.00, , NOV – 2013, Question 1(a)., (10 Marks), The following is the Profit and Loss Account of Mr. X, aged 58 years, a resident, for the year ended, 31.03.2022:, Particulars, ₹, Particulars, ₹, Rent, 60,000 Gross Profit, 5,85,000, Repair of car, 3,000 Gift of cash from a friend (received on, 25,000, 15.09.2021), Medical expenses, 4,500 Sale of car, 17,000, Salary, 18,000 Interest on income-tax refund, 3,000, Depreciation on car, 3,000, Advance income-tax, 6,500, Net Profit, 5,35,000, 6,30,000, 6,30,000, Other information:, (1), Mr. X bought a car during the year for ₹ 20,000. He charged depreciation @ 15% on the value of the, car. The above car was sold during the year for ₹17,000. The use of the car was 3/4th for business and, 1/4th for personal use., (2), Medical expenses were incurred for the treatment of Mrs. X., (3), Salary had been paid on account of car driver., (4), Mr. X had also let out a house property at a monthly rent of ₹ 25,000. The expected rent is, considered to be ₹2,50,000. The municipal taxes are ₹6,000, out of which ₹3,000 are paid by the, tenant and ₹3,000 are yet to be paid by Mr. X. Interest on loan taken for the house property is, ₹20,000., (5), Mr. X’s minor daughter received ₹75,000 from stage acting. Interest on company deposits of Mr. X’s, daughter (deposit was made out of income from stage acting) was ₹10,000., (6), Mr. X incurred an expense of ₹50,000 on the medical treatment of his dependant son, who is, suffering from severe disability., (7), Mr. X had taken a loan during the year 2021-22 for the education of his son, from a notified, organization and his son is pursuing B.Com. in Delhi University. Interest paid on the same during the, year was ₹10,000., Compute the total income and tax liability of Mr. X for the assessment year 2022-23., Solution:, Income from house property, ₹, ₹, Gross Annual Value (Higher of Actual Rent and expected rent), Actual Rent (₹ 25,000 × 12), 3,00,000, Expected rent, 2,50,000, 3,00,000, Less: Municipal taxes, Nil, Net Annual Value (NAV), 3,00,000, Less: Deductions under section 24, (a) 30% of NAV, 90,000, (b) Interest on loan, 20,000, (1,10,000), Income from house property, 1,90,000, Profits and gains of business or profession, Net Profit as per profit and loss account, 5,35,000, Add: Expenses disallowed:, Advance income-tax, 6,500
Page 768 :
Income Under The Head Business/Profession, Depreciation on car, Medical expenses of wife, Driver’s salary (¼th being for personal use), Repair of car (¼th being for personal use), , 249, 3,000, 4,500, 4,500, 750, 5,54,250, , Less:, Cash gift from friend, (25,000), Sale of car, (17,000), Interest on income-tax refund, (3,000), Profits and gains of business or profession, 5,09,250, The transaction of purchase and sale of motor car during the year would result in a short-term capital loss to, be carried forward for set-off against capital gains of the subsequent year., Income from other sources, Interest on income-tax refund, 3,000, Interest on company deposits accruing to Mr. X’s minor daughter, 10,000, Less: Exempt under section 10(32), (1,500), 8,500, Income from other sources, 11,500, Gross Total Income, 7,10,750, Under section 80DD, (1,25,000), Under section 80E - Interest on loan for higher education of son, (10,000), Total Income, 5,75,750, Computation of Tax Liability, Tax on ₹5,75,750 at slab rate, 27,650.00, Add: HEC @ 4%, 1,106.00, Tax Liability, 28,756.00, Rounded off u/s 288B, 28,760.00, Note: Income received by Mr. X’s minor daughter from stage acting is not includible in the income of Mr., X, since the income has been earned by her on account of her special talent. However, interest on company, deposits is includible in Mr. X’s income as per section 64(1A), even though the deposit was made out of, income derived from special talent., , MAY – 2013, Question 1(a)., (10 Marks), Mrs. X a resident aged 50 years is running an acupuncture clinic. Her Income and Expenditure, Account and other relevant information for the year ending 31st March, 2022 are given below:, Expenditure, Amount, Income, Amount, ₹, ₹, To Staff Salary, 2,40,000 By Fees receipts, 10,00,000, To Clinic rent, 1,20,000 By Dividend from Indian Companies, 10,500, To Medicines and needles, 1,05,000 By Winning from Lotteries, 7,000, net of TDS (TDS ₹3,000), To Depreciation, 81,000 By Income-tax Refund, 1,750, To Administrative expenses, 1,52,000, To Donation to Prime Minister’s, 20,000, National Relief Fund, To Excess of Income over Expenditure, 3,01,250, 10,19,250, 10,19,250, (i), Depreciation in respect of all assets has been ascertained at ₹ 60,000 as per Income-tax rules., (ii), Medicines & needles of ₹ 22,000 have been used for her family., (iii) Fees Receipts include ₹24,000 being honorarium for valuing acupuncture examination answer, books.
Page 769 :
Income Under The Head Business/Profession, , 250, , (iv), , She has also received ₹57,860 on maturity of one LIC Policy, not included in the above, Income and Expenditure Account., (v), She has paid an LIC premium of ₹12,000 for self (Sum Assured ₹50,000 policy taken on, 15.06.2021)., (vi), She has paid ₹2,500 for purchase of lottery tickets., Mrs. X has not opted for presumptive taxation of Income u/s 44ADA. From the above compute the total, Income and tax payable thereon of Mrs. X for the Assessment year 2022-23., Solution: Computation of total income and tax liability of Mrs. X for the A.Y. 2022-23, Income under the head “Profits and gains of business or profession”, Net Income as per Income and Expenditure Account, Add: Expenses disallowed:, Depreciation (81,000 - 60,000), Cost of medicines etc. for personal use, Donation to Prime Minister’s Relief Fund, , ₹, 3,01,250, 21,000, 22,000, 20,000, 3,64,250, , Less:, Dividend from Indian companies, (10,500), Income-tax refund, (1,750), Winning from Lotteries, (7,000), Honorarium for valuing answer books, (24,000), Income under the head “Profits and gains of business or profession”, 3,21,000, Computation of income under the head “Income from other sources”, Dividend from Indian Companies, 10,500, Honorarium for valuing answer books, 24,000, Winning from Lotteries (Net), 7,000, Add: TDS, 3,000, Income from other sources, 44,500, Gross Total Income, 3,65,500, Less: Deduction under Chapter VI-A, U/s 80C Life Insurance Premium (maximum 10% of sum assured), (5,000), U/s 80G Donation to Prime Minister’s Relief Fund [100% deduction without qualifying limit], (20,000), Total Income, 3,40,500, Computation of tax on total income, Tax on winnings from lotteries [ ₹10,000 @ 30%], 3,000, Tax on balance income of ₹ 3,30,500 [5% of (₹3,30,500 –₹ 2,50,000,, representing the basic exemption limit)], 4,025, Tax on total income, 7,025, Less: Rebate u/s 87A, (7,025), Total tax liability, Nil, Less: Tax deducted at source (TDS), (3,000), Tax Refundable, (3,000), Note – 1. As per section 58(4), no deduction in respect of any expenditure or allowance is allowable in, respect of winnings from lotteries. Hence, ₹2,500 paid for purchase of lottery tickets is not allowable as, deduction., 2. Maturity proceeds of life insurance policy, Any sum received under a life insurance policy is wholly exempt from tax under section 10(10D)., 3. The maturity proceeds of LIC have been taken as exempt under section 10(10D) presuming that the, premium paid during any of the years of the policy does not exceed the specified percentage of the actual, capital sum assured.
Page 770 :
Income Under The Head Business/Profession, , 251, , Question 4(a), (8 Marks), From the following information of Mr. X for the financial year 2021-22 you are required to compute his, total income for the financial year 2021-22 and ascertain the amount of losses which will be carried forward, to next year., ₹, (i) He owns two houses:, House No. 1 – Income after all statutory deductions, 80,000, House No. 2 – Current year loss, (38,000), (ii) He has three proprietary business concerns :, (a) Textile Business :, (i) Discontinued from 30th September 2021 – Current year’s loss, 40,000, (ii) Brought forward loss of Asst. year 2018-19, 95,000, (b), (i) Chemical Business – since discontinued :, Nil, (ii) Bad debts allowed in earlier years recovered during the year, 35,000, (iii) Brought forwarded business loss of Asst. year 2020-21, 50,000, (c) Leather Business – Profit for the current year, 1,00,000, (d) Share of Profit in a firm in which he is partner since 2016, 16,550, (iii) (a) Short Term Capital Gain, 60,000, (b) Long Term Capital Loss, 35,000, (iv), Contribution to L.I.C. towards premium, 10,000, Solution: Computation of Total Income of Mr. X for the A.Y. 2022-23, Particulars, ₹, ₹, 1. Income from house property, House No.1, House No.2, , 80,000, (-) 38,000, , 2. Profits and gains of business or profession, Profit from leather business, Less: Current year loss of textile business, Bad debts (allowed as deduction in earlier year) recovered during the year is, taxable under section 41(4), Less: Brought forward business loss of textile business for A.Y.2018-19 set off, against the business income of current year, , 42,000, , 1,00,000, (40,000), 60,000, 35,000, 95,000, (95,000), , 3. Capital Gains, Short-term capital gains, Gross Total Income, Less: Deduction under Chapter VI-A, Under section 80C – LIC premium paid, Total Income, , Nil, 60,000, 1,02,000, (10,000), 92,000, , Losses to be carried forward to A.Y. 2022-23, Particulars, Business loss of discontinued chemical business of A.Y. 2020-21 to be carried forward under, section 72, Long term capital loss of A.Y. 2022-23 to be carried forward under section 74 (Note 2), , ₹, 50,000, 35,000, , Notes:, (1) Share of profit from firm of ₹ 16,550 is exempt under section 10(2A) in the hands of the partner, Mr. X., (2) Long-term capital loss cannot be set-off against short-term capital gains. Therefore, it has to be carried, forward to the next year to be set-off against long-term capital gains of that year. It can be carried forward, for a maximum of 8 assessment years.
Page 771 :
Income Under The Head Business/Profession, , 252, , Note: The above solution has been worked out on the basis of the following assumptions:, (1) The current year loss of ₹38,000 is respect of House No. 2 is the computed loss after deduction of, municipal tax, interest and 30% of Net Annual Value., (2) Life insurance premium paid is fully allowable as deduction under section 80C on the assumption that, the same is within the limit of 10% or 20% of actual capital sum assured, as the case may be (20% if the, policy is issued before 01.04.2012 and 10% if the policy is issued on or after 01.04.2012)., Question 7(a), (4 Marks), (i) Explain the provision regarding the taxability of limited liability partnership under the Income-Tax Act,, 1961., Answer: Taxability of limited liability partnerships (LLPs) under the Income-tax Act, 1961, (i) The taxation scheme of LLPs in the Income-tax Act, 1961 is on the same lines as applicable for general, partnerships, i.e. tax liability would be attracted in the hands of the LLP and tax exemption would be, available to the partners., Therefore, the same tax treatment would be applicable for both general partnerships and LLPs., (ii) The rate of income-tax applicable to LLPs is the same as the rate applicable for firms i.e. 30% of total, income., (iii) The provisions of section 40(b) requiring payment of remuneration only to working partner in, accordance with the terms of the partnership deed for a period commencing on or after the date of the, partnership deed, would apply to LLPs as well. Further, disallowance of interest in excess of 12% per annum, and salary exceeding the prescribed limits would also be applicable in the case of LLPs., (iv) However, whereas a partnership firm can opt for presumptive taxation scheme under section 44AD, an, LLP cannot opt for such scheme., Question 7(a), (4 Marks), (ii) State the head of income in the following cases under which the receipt is to be assessed and comment., (A) A uses his property for his own business. Can he claim depreciation?, (B) B Lets out his property to X. X sublets it. How is sub-letting to be assessed in the hands of X?, (C) C has built a house on a leasehold land. He has let out the property and claims the rent as income from, house property and deducted expenses on repairs, security charges, insurance and collection charges, totaling to 40% of receipts., Answer:, (A) Yes, Mr. A can claim depreciation, since the property is an asset used for business purposes. Section 22,, which is the charging section for “Income from house property” specifically excludes from its scope,, property which an assessee, as an owner, occupies for the purpose of any business or profession carried on, by him., (B) In the hands of Mr. B, income from letting out of property to Mr. X is chargeable under the head, “Income from house property”, since Mr. B is the owner of the property., However, since Mr. X is not the owner of the house property, the income from sub-letting will not be, chargeable under the head “Income from house property”. It will be assessed as “Income from other, sources” in the hands of Mr. X., In the alternative, it would be assessed as “Profits and gains of business or profession”, if X is engaged in, the business of sub-letting., (C) Income from letting out of a house built on leasehold land is taxable under the head “Income from house, property” in the hands of Mr. C. Ownership of land is not a pre-requisite for charge of income under the, head “Income from house property”., The annual letting value (higher of municipal value and fair rent, but restricted to standard rent) or actual, rent, whichever is higher, would be the gross annual value of the house property. The municipal taxes paid, by Mr. C during the year is to be deducted from gross annual value to arrive at the net annual value., Deduction of 30% of net annual value is allowable under section 24 to arrive at the income chargeable under, the head “Income from house property”. No other deduction is allowable in respect of repairs, insurance,, security and collection charges., Note – Interest on loan borrowed for construction of house is deductible under section 24(b) from Net, Annual Value to arrive at the Income from house property, in addition to the statutory deduction of 30%.
Page 772 :
Income Under The Head Business/Profession, , 253, , NOV – 2012, Question No. 1 (a), (6 Marks), Mr. X provides the following information for the year ending 31.03.2022:, ₹, (i) Rent from vacant site let on lease, 1,12,000, (ii) Rent from house property at Delhi, 20,000 p.m., (iii) Turnover from retail trade in grains (No books of account maintained), 24,37,500, (iv) Arrears of salary received from ex-employer, 40,000, (v) Purchase of 10,000 shares (non listed) of X Co. Ltd., on 01.01.2019, 1,00,000, He received a 1:1 bonus on 01.01.2020. Sale of 5,000 bonus shares in September, 2021, 2,20,000, (vi) Received ₹ 1,50,000 on 12.02.2022 being amount due from Mr. A relating to goods supplied by Mr. X’s, father, which was written off as bad debt by his father in Assessment Year 2020-21 and allowed as, deduction. Mr. X’s father died in July 2020., (vii) Brought forward business loss relating to discontinued textile business of Mr. X relating, to the Assessment Year 2020-21., 1,97,500, (viii) Brought forward depreciation relating to discontinued textile business of Mr. X., 1,50,000, (ix) Mr. X contributed ₹ 30,000 to Prime Minister’s National Relief Fund and ₹ 40,000 to Charitable Trust, enjoying exemption u/s 80G by cheque., Compute the Total Income and the Tax thereon of Mr. X for the Assessment Year 2022-23., Answer: Computation of total income and tax liability of Mr. X for the A.Y. 2022-23, ₹, Income from Salaries, Arrears of salary received from ex-employer, 40,000.00, Less: Standard deductions u/s 16(ia), (40,000.00), Income under the head salaries, Nil, Income from house property, Income from House Property at Delhi, Gross Annual Value (GAV), 2,40,000.00, Less: Municipal taxes paid, Nil, Net Annual Value (NAV), 2,40,000.00, Less: Deduction under section 24 @ 30% of NAV, (72,000.00), Income from house property, 1,68,000.00, Profit and gains of business or profession, Income from business of retail trade in grains (24,37,500 x 8%), 1,95,000.00, Less: Set-off of brought forward business loss relating to, A.Y.2020-21 of discontinued textile business, (1,95,000.00), (Balance loss of 2,500 shall be carried forward), Income from business, Nil, Capital gains, Full value of consideration, 2,20,000.00, Less: Cost of acquisition, Nil, Short term capital gains, 2,20,000.00, Less: Depreciation for textile business, (1,50,000.00), Balance short term capital gain, 70,000.00, Income from other sources, Rent from vacant site let on lease, 1,12,000.00, Gross Total Income (1,68,000 + 70,000 + 1,12,000), 3,50,000.00, Less: Contribution to Prime Minister National Relief Fund section 80G, (30,000.00), Less: Contribution to Charitable trust notified u/s 80G ₹40,000, (17,500.00), Adjusted GTI = 3,50,000, Q.A = 10% of ₹3,50,000 or ₹40,000 whichever is less, = 35,000, Deduction 50% of ₹35,000 = 17,500, Total Income, 3,02,500.00, Tax on ₹3,40,500 at slab rate, 2,625.00
Page 773 :
Income Under The Head Business/Profession, , 254, , Less: Rebate u/s 87A, (2,625.00), Tax Liability, Nil, Note: ₹1,50,000 represents the amount due from Mr. A relating to goods supplied by Mr. X’s father, which, was written off as a bad debt by his father in the A.Y.2020-21 and allowed as deduction to him. The said, sum recovered by Mr. X, in the A.Y.2022-23, would not be treated as his income since there is no such, provision under section 41(4) to treat the sum recovered by the successor in business as his income., Question No. 4(a), (4 Marks), Following is the profit and loss account of Mr. X for the year ended 31.03.2022:, Particulars, ₹, Particulars, ₹, To Repair on Building, 1,81,000 By Gross Profit, 6,01,000, To Amount paid to IIT Mumbai for an 1,00,000 By Income Tax Refund, 8,100, approved Scientific research programme, u/s 35(2AA), To Interest, 1,10,000 By Interest on Company Deposits, 6,400, To Travelling, 1,30,550, To Net Profit, 93,950, 6,15,500, 6,15,500, Following additional informations are furnished:, (1) Repairs on building includes ₹1,00,000 being cost of laying a toilet roof., (2) Interest payment include ₹50,000 on which TDS has not been deducted and penalty for, contravention of GST Act of ₹24,000., Mr. X is required to gets his accounts audited in the preceding year 2020-21., Compute the income chargeable under the head “Profits and gains of Business or Profession” of Mr. X for, the year ended 31.03.2022 ignoring depreciation., Answer: Computation of Income under the head Business/ Profession for the A.Y. 2022-23, ₹, Net profit as per P & L A/c, 93,950, Add: Expenses Disallowed, (i) Repair of building(capital expenditure), 1,00,000, (ii) Interest paid without TDS (50,000 x 30%), 15,000, (iii)GST Penalty, 24,000, 1,39,000, 2,32,950, Less: Incomes not taxable, (i) IT Refund ( not an income), 8,100, (ii) Interest on deposit (taxable u/h other sources), 6,400, (14,500), Income under the head Business/Profession, 2,18,450, , MAY – 2012, Question 1., (10 Marks), Mrs. X is a Chartered Accountant in practice. She maintains her accounts on cash basis. Her income and, Expenditure account for the year ended March 31, 2022 reads as follows:, Expenditure, ₹, Income, ₹, ₹, Salary to Staff, Stipend to articled Assistants, Incentive to Articled Assistants, Office rent, Printing and Stationery, Meeting, Seminar and, Conference, , 5,50,000 Fees earned:, 37,000 Audit, 3,000 Taxation Services, 24,000 Consultancy, 22,000 Dividend on shares of Indian, Companies (Gross), 31,600 Income from unit Trust of India, , 7,88,000, 5,40,300, 2,70,000, , 15,98,300, 10,524, 7,600
Page 774 :
Income Under The Head Business/Profession, Purchase of car, , Repair, Maintenance And petrol, of car, Travelling expenses, Municipal Tax paid in respect, of house Property, Net profit, , 80,000 Honorarium received from, Various, Institutions, for, valuation of answer papers, Rent Received from Residential, 4,000 flat let out, 35,000, 3,000, 9,28,224, 17,17,824, , 255, , 15,800, 85,600, , 17,17,824, , Other information:, (i), Allowable rate of depreciation on motor car is 15%, (ii), Value of benefits received from clients during the course of profession is ₹10,500., (iii) Incentives to articled assistants represent amount paid to two articled assistants for the passing, CA Inter Examination at first attempt., (iv), Repairs and maintenances of car include ₹2,000 for the period from 01.10.2021 to 30.09.2022., (payment was made on 31.03.2022), (v), Salary include ₹30,000 to a computer specialist in cash for assisting Mrs. X in one professional, assignment., (vi), The total travelling expenses incurred on foreign tour was ₹32,000 which was within the RBI, norms., (vii) Medical Insurance premium on the health of dependent brother and major son dependent on her, amounts to ₹5,000 and ₹10,000 respectively paid in cash., (viii) She invested an amount of ₹10,000 in National Saving Certificate., Mrs. X has not opted for presumptive taxation of Income u/s 44ADA. Compute the Total Income and Tax, Payable of Mrs. X for the Assessment Year 2022-2023., Answer:, Computation of Total Income of Mrs. X, ₹, Income under the head business/profession, Net profit as per profit and loss account, 9,28,224.00, Add: Value of benefit received, 10,500.00, Add: inadmissible items, • Purchase of car, 80,000.00, • Municipal tax, 3,000.00, • Payment made in cash in excess of ₹10,000 {Sec 40A(3)}, 30,000.00, Less:, • Dividend on shares, (10,524.00), • Income from Unit Trust of India, (7,600.00), • Honorarium received, (15,800.00), • Rent received, (85,600.00), • Depreciation on Car (80,000 x 15% ), (12,000.00), Income under the head business/profession, 9,20,200.00, Income under the head House Property, Gross annual value, Less: Municipal Taxes, Net annual value, Less: 30% of NAV u/s 24(a), Less: Interest on capital borrowed u/s 24(b), Income from house property, , 85,600.00, (3,000.00), 82,600.00, (24,780.00), Nil, 57,820.00
Page 775 :
Income Under The Head Business/Profession, Income under the head Other Sources, Dividend from Indian company, Income from UTI, Honorarium received, Income under the head Other Sources, , 256, 10,524.00, 7,600.00, 15,800.00, 33,924.00, , Gross Total Income, 10,11,944.00, Less: Deduction u/s 80C NSC, (10,000.00), Total Income, 10,01,944.00, Rounded off u/s 288A, 10,01,940.00, Computation of Tax Payable, Tax on ₹10,01,940 at slab rate, 1,13,082.00, Add: HEC @ 4%, 4,523.28, Tax Liability, 1,17,605.28, Less: TDS, Nil, Tax Payable, 1,17,605.28, Rounded off u/s 288B, 1,17,610.00, Question 4, (8 Marks), Mr. Y carries on his own business. An analysis of his trading and profit & loss for the year ended, 31.03.2022 revealed the following information:, (1) The net profit was ₹11,20,000., (2) The following incomes were credited in the profit and loss account:, (a) Dividend from UTI ₹ 22,000., (b) Interest on debentures ₹17,500, (c) Winnings from races ₹15,000., (3) It was found that some stocks were omitted to be included in both the opening and closing stocks,, the value of which were:, Opening stock ₹8,000., Closing stock ₹12,000, (4) ₹1,00,000 was debited in the profit and loss account being contribution to a University approved, and notified under section 35(1)(ii)., (5) Salary includes ₹20,000 paid to his brother which is unreasonable to the extent of ₹2,500., (6) Advertisement expenses include 15 gift packets of dry fruits costing ₹1,000 per packet presented to, important customers., (7) Total expenses on car was ₹78,000. The car was used both for business and personal purposes., 3/4th is for business purposes., (8) Miscellaneous expenses included ₹30,000 paid to A & Co., a good transport operator in cash on, 31.01.2022 for distribution of the company’s product to the warehouses., (9) Depreciation debited in the books was ₹55,000. Depreciation allowed as per IT rules was ₹50,000., (10) Drawing ₹10,000, (11) Investment in NSC ₹15,000., Compute the Total Income of Mr. Y for the Assessment Year 2022-23., Solution: Computation of Total Income of Mr. Y for the Assessment Year 2022-23, Income under the head Business/Profession, Net profit as per profit and loss account, Add: inadmissible expenses, • Car used for personal purpose (78,000 x 1/4 ), • Under valuation of closing stock, • Salary disallowed u/s 40A(2), • Depreciation (55,000 – 50,000), • Drawing, • Investment in NSC, , ₹, 11,20,000, 19,500, 12,000, 2,500, 5,000, 10,000, 15,000
Page 776 :
Income Under The Head Business/Profession, , 257, , • Total, 11,84,000, Less:, • Dividend from units of UTI, (22,000), • Interest on debentures, (17,500), • Winning From Races, (15,000), • Under valuation of opening stock, (8,000), Business income, 11,21,500, Income under the head Other Sources, Dividend from units of UTI, 22,000, Interest on debentures, 17,500, Winning From Races, 15,000, Income under the head Other Sources, 54,500, Gross Total Income, 11,76,000, Less: Deduction u/s 80C, (15,000), Total Income, 11,61,000, Question 5, (10 Marks), Dr. Sagar, a resident individual at Madurai, aged 50 years is running a clinic. His Income and Expenditure, Account for the year ending March 31st 2022 is as under:, ₹, Income, ₹, Expenditure, To Medicine consumed, 8,40,000 By Consultation and Medical charges, 21,00,000, To Staff salary, 4,25,000 By Income tax refund, 16,500, (Principal ₹15,000, interest ₹1,500), To Clinic consumables, 1,55,000 By Dividend from Indian companies, 27,000, To Rent paid, 1,20,000 By Wining from lottery Net of TDS, 35,000, To Administrative expenses, 3,00,000 By Rent, 54,000, To Donation (to IIT, Delhi for, Research approved under, section 35(2AA), To Net profit, , 1,00,000, 2,92,500, , 22,32,500, 22,32,500, (i) Rent paid includes ₹ 36,000 paid by cheque towards rent for his residence., (ii) Clinic equipments are:, 01.04.2021, Opening WDV, 4,50,000, 07.02.2022, Acquired (cost- put to use on the same date), 1,00,000, (iii) Rent received relates to property let out at Madurai. Gross annual value ₹54,000. The municipal tax of, ₹9,000, paid in January 2022 has been included in “administrative expenses”., (iv) Dr. Sagar availed loan of ₹5,50,000 from a bank for higher education of his daughter. He repaid, principal of ₹50,000, and interest thereon ₹65,000 during the year 2021-22., (v) He paid ₹60,000 as tuition fee to the university for full time education of his son., Dr. Sagar has not opted for presumptive taxation of Income u/s 44ADA. From the above, compute the Total, Income and Tax Liability of Dr. Sagar for the Assessment Year 2022-2023., Solution: Computation of Total Income of Dr. Sagar for A.Y. 2022-23, Income from house property, Gross Annual Value (GAV), Less : Municipal taxes paid, Net Annual Value (NAV), Less : Deduction under section 24(a) (30% of ₹ 45,000), Income under the head House Property, Income from profession, Net profit as per Income and Expenditure account, , ₹, 54,000, (9,000), 45,000, (13,500), 31,500, 2,92,500
Page 777 :
Income Under The Head Business/Profession, , 258, , Less :, Rent received, (54,000), Dividend from Indian Companies, (27,000), Winning from lottery (net of TDS), (35,000), Income tax refund, (16,500), Depreciation on Clinic equipments, on ₹4,50,000 @ 15%, (67,500), on ₹1,00,000 @ 7.5%, (7,500), Add:, Rent for his residential accommodation included in Income and Expenditure A/c, 36,000, Municipal tax paid relating to residential house at Surat included in administrative expenses, 9,000, Income under the head Business/Profession, 1,30,000, Income from other sources, Interest on income-tax refund, 1,500, Dividend from Indian Company, 27,000, Winnings from lottery, (₹35,000 + ₹15,000), 50,000, Income under the head Other Sources, 78,500, Gross Total Income, 2,40,000, Less: Deductions under Chapter VI A, Deduction under section 80C, Tuition fee paid to university for full time education of his daughter, (60,000), Deduction under section 80E, Interest on loan taken for higher education is deductible, (65,000), Total Income, 1,15,000, Computation of Tax Liability, Tax on normal income, Nil, Tax on ₹50,000 @ 30% u/s 115BB, 15,000, Less: Rebate u/s 87A, (12,500), Tax before health & education cess, 2,500, Add: HEC @ 4%, 100, Tax Liability, 2,600, Notes:, (i) The principal amount received towards income-tax refund will be excluded from computation of total, income. Interest received will be taxed under the head “Income from other sources”., (ii) Winnings from Lottery should be grossed up for the chargeability under the head “Income from other, sources” (₹35,000 + ₹15,000). Thereafter, while computing tax liability, TDS of ₹15,000 should be deducted, to arrive at the tax payable. Winnings from lottery are subject to tax @ 30% as per section 115BB., (iii) As per section 58(4) deduction under Chapter VIA is not allowed from casual income., Question 7, (4 Marks), Mr. X engaged in various types of activities gives the following information for the year ended 31.03.2022., ₹, Loss from automobile business (Total turnover ₹6,00,000), 1,10,000, Profit from wholesale trade in furniture items at the prescribed percentage of turnover, as per section 44AD, 4,00,000, Brought forward loss relating to discontinued textile business, (discontinued w.e.f. 01.06.2014), 2,00,000, Short term capital loss on sale of vacant site during the year, 70,000, Profit from speculation business related to oil seeds, 1,10,000, Loss from speculation business brought forward and related to cotton, (brought forward from assessment year 2021-22), 50,000, Brought forward unabsorbed depreciation of trade in furniture items related to, assessment year 2021-22, 60,000
Page 778 :
Income Under The Head Business/Profession, , 259, , Note: Aggregate total business turnover of Mr. X to be assumed as below limit prescribed under section, 44AB., Compute the Total Income of Mr. X for the Assessment Year 2022-23., Solution: Computation of Total Income of Mr. X for the assessment year 2022-23, ₹, Income under the head Business/Profession, Income from wholesale trade in furniture, 4,00,000, Less: Loss from automobile business (current year), (1,10,000), Less: Brought forward loss relating to discontinued textile business (P.Y. 2014-15), (2,00,000), Income from wholesale trade in furniture, 90,000, Income from Speculation Business, 1,10,000, Less: Loss from speculation business brought forward and related to cotton, (50,000), Income from Speculation Business, 60,000, Income under the head Business/Profession, 1,50,000, Gross Total Income, 1,50,000, Less: Deduction u/s 80C to 80U, Nil, Total Income, 1,50,000, Note:, (i) Brought forward unabsorbed depreciation of trade in furniture items related to Assessment Year 2021-22, is not allowed to be adjusted., (ii) Short Term Capital Loss on sale of vacant site A.Y. 2022-23 ₹70,000 to be carried forward for 8 years, starting from Assessment Year 2023-24., , NOV – 2011, Question 4, (8 Marks), ABC Ltd., engaged in manufacture of medicines (pharmaceuticals) furnishes the following information for, the year ended 31.03.2022:, (i) Municipal tax relating to office building ₹51,000 not paid till 31.10.2022., (ii) Patent acquired for ₹20,00,000 on 01.09.2021 and used from the same month., (iii) Capital expenditure on scientific research ₹10,00,000 which includes cost of land ₹2,00,000., (iv) Amount due from customer X outstanding for more than 3 years written off as bad debt in the books, ₹5,00,000., (v) Income tax paid ₹90,000 by the company in respect of non-monetary perquisites provided to its, employees., (vi) Provident fund contribution of employees ₹5,50,000 relating to salary of July 2020 was deposited on, 27th August 2021., (vii) Expenditure towards advertisement in souvenir of a political party ₹1,50,000., (viii) Refund of GST ₹75,000 received during the year, which was claimed as expenditure in an earlier year., State with reasons the taxability or deductibility of the items given above under the Income-tax Act, 1961., Solution:, (i) Municipal taxes relating to office building not paid till the last date of filing of the return is not allowed, as deduction as per sec 43B., (ii) Patent acquired for 20,00,000, Depreciation shall be allowed @ 25% of 20,00,000 = ₹5,00,000.00, (iii) Capital expenditure on scientific research shall be allowed ₹8,00,000 u/s 35., Further deduction allowed shall be 100%. No deduction shall be allowed for land., (iv) Amount written off as bad debts shall be allowed as deduction as per section 36(1)., (v) Income Tax paid ₹90,000 shall not be allowed as deduction u/s 40(a)., (vi) Provident fund paid on 27th August 2021 is not allowed because as per paragraph 38 of The Employees’, Provident Funds Scheme, 1952, the employer should pay such contribution within 15 days of the, subsequent month. In this case, amount is being remitted on 27th August 2021, hence expenditure is not, allowed., (vii) Expenditure towards advertisement in souvenir of political party shall not be allowed as deduction as, per section u/s 37(2B).
Page 779 :
Income Under The Head Business/Profession, , 260, , (viii) As per section 41(1), refund of GST shall be treated as income since it was claimed as an expenditure, in an earlier year., Question 5, (7 Marks), State with reasons the allowability of the following expenses under Income-Tax Act, 1961 while computing, income from business or profession for the Assessment Year 2022-23:, (i) Provision made on the basis of actuarial valuation for payment of gratuity ₹5,00,000. However, no, payment on account of gratuity was made before due date of filing return., (ii) Purchase of oil seeds of ₹50,000 in cash from a farmer on a banking day., (iii) Tax on non monetary perquisite provided to an employee ₹20,000., (iv) Payment of ₹50,000 by using credit card for fire insurance., (v) Salary payment of ₹2,00,000 by a company outside India without deduction of tax., (vi) GST deposited in cash ₹50,000 with State Bank of India., (vii) Payment made in cash ₹30,000 to a transporter in a day for carriage of goods., Solution:, (i) Allowed, provision made on the basis of actuarial valuation is allowed as business expense., (ii) Allowed, cash payment of ₹50,000 for purchase of oil seeds is allowed as business expense., (iii) Not allowed, tax on non-monetary perquisite is not deductible u/s 40(a), (iv) Allowed, payment through credit card is allowed., (v) If tax is neither deducted nor paid, it is not allowed. In the given question it is not mentioned that tax is, paid hence it is presumed that tax is not paid and in that case it is disallowed., (vi) Allowed, payment of GST in cash shall be allowed and deductible as per Rule 6DD., (vii) Allowed, as per section 40A(3), payment in cash to a transporter upto ₹35,000 is allowed, Question 6, (6 Marks), Mr. X is having a trading business and his Trading and Profit & Loss Account for the financial year 2021-22, is asunder:, Particulars, Amount, Particulars, Amount, (₹), (₹), To Opening stock, 1,00,000 By Sales, 110,00,000, To Purchases, 89,00,000 By Closing stock, 50,000, To Gross profit, 20,50,000, Total, 110,50,000 Total, 110,50,000, Salary to employees (Including, 5,00,000 By Gross Profit b/d, 20,50,000, Contribution to PF), 1,00,000, Donation to Prime Minister Relief, Fund by cheque, Provision for bad debts, 50,000, Bonus to employees, 50,000, Interest of bank loan, 50,000, Family planning expenditure incurred, 20,000, on employees, Depreciation, 30,000, Income Tax, 1,00,000, To Net profit, 11,50,000, Total, 20,50,000 Total, 20,50,000, Other information:, (i) Depreciation allowable ₹40,000 as per Income Tax Rules., (ii) Payment of Interest on bank loan has been made in August 2022., (iii) Payment of bonus to workers made in September 2022., (iv) Out of salary, ₹25,000 pertains to his contributions to recognized provident fund which was deposited, after the due date. Further, employees contribution of ₹25,000 was also deposited after the due date, Mr. X has not opted for presumptive taxation of Income u/s 44AD. Computation Total Income and Tax, Liability of Mr. X for the Assessment Year 2022-23.
Page 780 :
Income Under The Head Business/Profession, , 261, , Solution: Computation of Total Income of Mr. X, ₹, Net profit as per profit and loss account, 11,50,000, Add: inadmissible expenses, • Employees’ contribution deposited after due date of provident fund, 25,000, • Employer’s contribution deposited after due date of filing of return of income, 25,000, • Donation to Prime Minister Relief Fund, 1,00,000, • Provision for bad debt, 50,000, • Family planning expenses, 20,000, • Depreciation as per books, 30,000, • Income-tax, 1,00,000, Less:, • Depreciation as per Income Tax Act, (40,000), Gross Total Income, 14,60,000, Less: Deduction u/s 80G, (1,00,000), Total Income, 13,60,000, Computation of Tax Liability, Tax on ₹13,60,000 at slab rate, 2,20,500, Add: HEC @ 4%, 8,820, Tax Liability, 2,29,320, Question 7, (4 Marks), M/s ABC Ltd. a manufacturing concern furnishes the following particulars:₹, (i) Opening writing down value under Income Tax of block plant and machinery, 5,00,000, (ii) Purchase of plant and machinery, 2,00,000, (put to use before 01.10.2021), (iii) Sale proceeds of plant and machinery which became obsolete- the plant and, 5,000, machinery was purchased on 01.04.2019 for ₹5,00,000., Further, out of purchase of plant and machinery:, (a) Plant and machinery of ₹20,000 has been installed in office., (b) Plant and machinery of ₹20,000 was used previously for the purpose of business by the seller., Compute depreciation and additional depreciation as per Income-Tax Act for the Assessment Year 2022-23., Solution:, Block – Plant and Machinery, ₹, Opening writing down value, 5,00,000, Add: Purchases, 2,00,000, Less: Sales, (5,000), W.d.v, 6,95,000, Depreciation @ 15%, 1,04,250, Additional depreciation (2,00,000 – 20,000 – 20,000) x 20%, 32,000, (full depreciation because put to use for 180 days or more), , MAY – 2011, Question 3, (8 Marks), Mr. X, resident individual aged 64, is a partner in XYZ & Co., a partnership firm. He also runs a wholesale, business in medical products. The following details are made available for the year ended 31.03.2022:, ₹, ₹, (i) Interest on capital received from XYZ & Co., at 15%, 1,50,000, (ii) Interest from bank on fixed deposit (Net of TDS ₹1,500), 13,500, (iii) I.T. refund received relating to asst. year 2020-21 including interest of ₹2,300, 34,500, (iv) Net profit from wholesale business, 5,60,000, Amounts debited include the following:, Depreciation as per books, 34,000, Motor car expenses, 40,000, Municipal taxes for the shop, 7,000
Page 781 :
Income Under The Head Business/Profession, , 262, , (For two half years; payment for one half year made on 12.06.2022, and for the other on 14.11.2022), Salary to manager for whom single cash payment was made for, 21,000, (v) The WDV of the assets (as on 01.04.2021) used in above wholesale, Business is as under:, Computers, 1,20,000, Motor car, 3,20,000, (20% used for personal use), (vi) LIP paid for major son, 60,000, PPF of his wife, 70,000, Compute the Total Income of the assessee for the Assessment Year 2022-23. The computation should show, the proper heads of income. Also compute the WDV of the different blocks of assets as on 31.03.2022., Solution: Computation of Total Income of Mr. X for the A.Y. 2022-23, Particulars, ₹, ₹, Profits and gains of business or profession, Income from own business, 5,60,000, Net profit as per books, Add:, 34,000, Depreciation as per books, Disallowance of municipal taxes paid for the second half-year under section, 43B, since the same was paid after the due date of filing of return (₹ 7,000/2), , 3,500, , Disallowance under section 40A(3) in respect of salary paid in cash since the, same exceeds ₹10,000, , 21,000, 8,000, , 20% of car expenses for personal use, Less:, Depreciation allowable (Note 1), Income from firm, Interest on capital from partnership firm (Note 2), Income from other sources, Interest on bank fixed deposit (Gross), Interest on income-tax refund, Gross Total Income, Less: Deductions under 80C (Note 3), Less: Deduction under 80TTB, Total Income, Notes:, (1) Depreciation allowable under the Income-Tax Act, 1961, Opening, Rate, WDV, Block 1 Computers, 1,20,000, 40%, Block 2 Motor Car, 3,20,000, 15%, Less: 20% Disallowance for, personal use, , 66,500, 6,26,500, (86,400), 5,40,100, 1,20,000, 6,60,100, , 15,000, 2,300, , 17,300, 6,77,400, (1,30,000), (15,000), 5,32,400, , Depreciation, , Closing, WDV, 48,000, 72,000, , 48,000, 9,600, , 38,400, 2,81,600, 86,400, (2) Only to the extent the interest is allowed as deduction in the hands of the firm, the same is includible as, business income in the hands of the partner. Maximum interest allowable as deduction in the hands of the, firm is 12% p.a. Therefore, interest @12% p.a. amounting to ₹1,20,000 would be treated as the business, income of Mr. X.
Page 782 :
Income Under The Head Business/Profession, , 263, , (3) Deduction u/s 80C, Particulars, ₹, Under section 80C, 60,000, LIP for major son, 70,000, PPF paid in wife’s name, Total deduction, 1,30,000, Question 2, (12 Marks), Mr. X a resident individual and practicing Chartered Accountant furnishes you the receipts and payments, account for the Financial Year 2021-22., Receipts and Payments Account, Receipts, ₹, Payments, ₹, Opening balance (01.04.2021), 2,000 Staff salary, bonus and stipend to, 1,50,000, Cash on hand and at Bank, articled clerks, Fee from professional services, 9,38,000 Other administrative expenses, 48,000, Rent, 60,000 Office rent, 30,000, Motor car loan from Canara Bank (@, 2,50,000 Housing loan repaid to SBI (includes, 1,88,000, 9% per annum), interest of ₹88,000), Life insurance premium, 24,000, Motor car (acquired in Jan. 2022), 4,25,000, 18,000, Medical insurance premium (for self, and wife) paid by cheque, Books bought on 20.04.2021, 20,000, 30,000, Computer acquired on 01.11.2021 and, put to use on the same date (for, professional use), Domestic drawings, 2,72,000, Public provident fund subscription, 20,000, Motor car maintenance, 10,000, Closing balance (31.03.2022), 15,000, Cash on hand and at Bank, 12,50,000, 12,50,000, Following further information is given to you:, (1) He occupies 50% of the building for own residence and let out the balance for residential use at a, monthly rent of ₹5,000. The building was constructed during the year 1998-99., (2) Motor car was put to use both for official and personal purpose. One-fifth of the motor car use is for, personal purpose. No car loan interest was paid during the year., (3) The written down value of assets as on 01.04.2021 are given below:, Furniture & fittings, ₹60,000, Plant & Machinery, ₹80,000, (Air-conditioners, Photocopiers, etc.), Computers, ₹50,000, Note: Mr. X follows regularly the cash system of accounting., Mr. X has not opted for presumptive taxation of Income u/s 44ADA. Compute the Total Income and Tax, Liability of Mr. X for the Assessment Year 2022-23., Solution:, Computation of Total Income of Mr. X for the Assessment Year 2022-23, ₹, Income from house property, Self-occupied, Annual value, Nil, Less: Deduction under section 24(b), Interest on housing loan
Page 783 :
Income Under The Head Business/Profession, 50% of ₹ 88,000 = 44,000 but limited to, Loss from self occupied property, Let out property, Gross Annual value, Less: Municipal Tax, Net Annual Value, Less: Deductions under section 24, (a) 30% of Net Annual Value, (b) Interest on housing loan, (50% of ₹ 88,000), Loss from let out house property, Total loss under the head House Property, Profits and gains of business or profession, Fees from professional services, Less: Expenses, Staff salary, bonus and stipend, Other administrative expenses, Office rent, Motor car maintenance (10,000 x 4/5), Car loan interest – not allowable (since the same has, not been paid and the assessee follows cash system, of accounting), Depreciation on Motor car ₹ 4,25,000 x 7.5% x 4/5, Depreciation on Books @ 40% on ₹ 20,000, Depreciation on Furniture and fittings @ 10% of ₹ 60,000, Depreciation on Plant and machinery @ 15% of ₹80,000, Depreciation on Computer @ 40% of ₹50,000, Depreciation on Computer (New) ₹30,000 @ 40% x ½ thereon, Income under the head Business/Profession, Gross Total Income, Less: Deduction under Chapter VI-A, Section 80C, Housing loan principal repayment, PPF subscription, Life insurance premium, Section 80D, Medical insurance premium paid, Total Income, Computation of Tax Liability, Tax on ₹4,30,500 at slab rate, Less: Rebate u/s 87A, Tax Liability, , 264, (30,000.00), (30,000.00), 60,000.00, Nil, 60,000.00, (18,000.00), (44,000.00), (2,000.00), (32,000.00), 9,38,000.00, (1,50,000.00), (48,000.00), (30,000.00), (8,000.00), Nil, (25,500.00), (8,000.00), (6,000.00), (12,000.00), (20,000.00), (6,000.00), 6,24,500.00, 5,92,500.00, (1,00,000.00), (20,000.00), (24,000.00), (18,000.00), 4,30,500.00, 9,025.00, (9,025.00), Nil, , NOV – 2010, Question 3, (8 Marks), Dr. Mrs. X is medical practitioner. Her age is 64 as on 1st Jan 2022. Her Receipts and Payments account of, 2021-22 is asunder:, Receipt, ₹, Payment, ₹, Balance B/f:, 10,000 Purchase of motor car on, 4,00,000, 01.07.2021, Receipts from sale of Medicine, 2,50,000 Drawing, 2,50,000, Consultation fee, 50,000 Deposit in bank for 5 years, 1,50,000, Visiting fee, 2,00,000 Surgical instrument purchased, 50,000
Page 784 :
Income Under The Head Business/Profession, Lecture Fees, Family pension, Saving bank interest, Loan from bank for purchase of, motor car @ 8% p.a., Share from HUF, Agricultural income, Income from lottery (net after, deduction of TDS @ 30%), , before 30 Sep. 2021, 5,000 Instalment of motor car loan paid, (including interest ₹18,000), 2,80,000 Medical insurance premium, 1,000 Instalment of housing loan, (Principal component ₹48,000 and, interest ₹60,000), 3,00,000 Advance tax paid, 50,000 Purchase of medicine, 1,00,000 Payment for medical journal, 35,000 Motor car expenses, Balance C/f:, , 265, 1,18,000, 32,000, 1,08,000, 20,000, 47,000, 5,000, 50,000, 51,000, 12,81,000, , Total, 12,81,000, Total, Other relevant information is as under:, (i), She resides in her own house which was constructed in 1998 with a loan from LIC Housing of, ₹10,00,000 out of which 6,00,000 was still due. She got a loan from SBI of ₹6,00,000 on, 01.04.2021 at the rate of 10% and repaid the original loan., One fourth portion of the house is used for clinic purposes., (ii), She invested in term deposit ₹1,50,000 in Bank of Baroda on 01.07.2021 for a period of 5 years in, the name of her minor daughter at 9% interest p.a., (iii), She purchased a motor car on 1st July 2021 and put to use on the same date at ₹4,00,000. A loan of, ₹3,00,000 was taken to buy the motor car at 8% p.a. interest., One fourth use of vehicle is estimated to be personal., (iv), She paid medical insurance premium for herself of ₹16,000 and for mother ₹16,000. Her mother is, dependent on her., (v), She got her share from HUF’s income of ₹50,000., Mrs. X has not opted for presumptive taxation of Income u/s 44ADA. Compute her Total Income and Tax, Liability for Assessment Year 2022-23., Solution:, Computation of total income of Dr. X for A.Y. 2022-23, ₹, ₹, Income from house property:, Net Annual value of self-occupied house, Nil, Less: Interest on loan [₹45,000, being 3/4th of ₹60,000], (Restricted to ₹30,000), (30,000.00), Loss under the head House Property, (30,000.00), Income from profession:, Sale of medicine, 2,50,000.00, Consultation fees, 50,000.00, Visiting fee, 2,00,000.00, Less: Expenses, Medicine purchases, (47,000.00), Medical journal, (5,000.00), Depreciation on Surgical instrument(15% of ₹50,000), (7,500.00), Depreciation on Vehicle (15% of ₹4,00,000) x ¾, (45,000.00), Vehicle expenses (3/4th), (37,500.00), Interest on loan (18,000 x 3/4), (13,500.00), Interest on housing loan (60,000 x 1/4), (15,000.00), Income under the head Business/Profession, 3,29,500.00, Income from other sources
Page 785 :
Income Under The Head Business/Profession, , 266, , Family Pension, 2,80,000, Less : 33⅓% or ₹15,000, whichever is lower, (15,000), 2,65,000.00, Lecture fees, 5,000.00, Savings bank interest, 1,000.00, Interest on bank FD in the name, of minor daughter [1,50,000 × 9% × 9/12], 10,125, Less: Exempt u/s 10(32), (1,500), 8,625.00, Winnings from lottery, 50,000.00, Income under the head Other Sources, 3,29,625.00, Gross Total Income, 6,29,125.00, Less: Deductions under Chapter VI-A, Section 80C, Repayment of housing loan (48,000 × ¾), (36,000.00), Section 80D Medical Insurance Premium, Own, (16,000.00), Mother, (16,000.00), Section 80TTB, (9,625.00), Total income, 5,51,500.00, Agricultural Income, 1,00,000.00, Computation of Tax Liability, Tax on agricultural income + non-agricultural income, Tax on ₹1,00,000 + 5,01,500 at slab rate, 30,300.00, Tax on agricultural income + ₹3,00,000, Tax on ₹1,00,000 + ₹3,00,000 at slab rate, (5,000.00), Tax on normal income (30,300 – 5,000), 25,300.00, Tax on ₹50,000 @ 30%, 15,000.00, Tax before health & education cess, 40,300.00, Add: HEC @ 4%, 1,612.00, Tax Liability, 41,912.00, Less: TDS, (15,000.00), Less: Advance Tax, (20,000.00), Tax Payable, 6,912.00, Rounded off u/s 288B, 6,910.00, Notes:, (i), Since the residential house was constructed before 01.04.1999, the deduction for interest is, restricted to ₹30,000., (ii), Since ¼th portion of house is used for business purposes, therefore, ¼th share of interest paid is, deductible while computing business income., (iii) Agricultural income is exempt under section 10(1) and share of income from HUF is exempt, under section 10(2)., (iv), Term deposit of ₹1,50,000 in the name of minor daughter does not qualify for deduction under, section 80C. However, principal repayment of housing loan (3/4th) would qualify for deduction, under section 80C. Therefore, the deduction under section 80C would be ₹36,000 (i.e. 3/4th of, ₹48,000)., (v), Depreciation @15% has been provided on surgical instruments., Question 7, (4 Marks), State under which heads the following incomes are taxable:, (i) Rental income in case of dealer in property, (ii) Dividend on shares in case of a dealer in shares, (iii) Salary by a partner from his partnership firm, (iv) Rental income of machinery, (v) Winnings from lotteries by a person having the same as business activity, (vi) Salaries payable to a Member of Parliament
Page 786 :
Income Under The Head Business/Profession, , 267, , (vii) Receipts without consideration, (viii) In case of retirement, interest on employee’s contribution if provident fund is unrecognized., Solution:, (i), (ii), (iii), , Particulars, Rental income in case of dealer in property, Dividend on shares in case of a dealer in shares, Salary by partner from his partnership firm, , (iv), , Rental income of machinery (See Note below), , (v), , Head of Income, Income from house property, Income from other sources, Profit and gains of business or, profession, Income from other sources/ Profits, and gains of business or profession, Income from other sources, , Winnings from lotteries by a person having the same as, business activity, (vi) Salaries payable to a Member of Parliament, Income from other sources, (vii) Receipts without consideration, Income from other sources, (viii) In case of retirement, interest on employee’s contribution if, Income from other sources, provident fund is unrecognized, Note –, As per section 56, rental income of machinery would be chargeable to tax under the head “Income from, Other Sources”, if the same is not chargeable to income-tax under the head “Profits and gains of business or, profession”., Question 2, Mr. X, a manufacturer at Chennai, gives the following, for the year ended 31.03.2022., ₹, To Opening Stock, 71,000, To Purchase of Raw materials, 16,99,000, To Manufacturing Wages &, 5,70,000, Expenses, To Gross profit, 10,60,000, 34,00,000, To Administrative charges, 3,26,000, To GST penalty paid, 5,000, To General Expenses, To Interest to Bank (On machinery, term loan), To Depreciation, To Net Profit, , (12 Marks), Manufacturing, Trading and Profit & Loss Account, , By Sales, By Closing Stock, , By Gross profit, By Dividend from domestic, companies, 1,64,000 By Income from agriculture (net), 60,000, , ₹, 32,00,000, 2,00,000, , 34,00,000, 10,60,000, 15,000, 1,80,000, , 2,00,000, 5,00,000, 12,55,000, 12,55,000, Following are the further information relating to the financial year 2021-22:, (i), Administrative charges include ₹46,000 paid as commission to brother of the assessee. The, commission amount at the market rate is ₹36,000., (ii), The assessee paid ₹33,000 in cash to a transport carrier on 29.12.2021. This amount is included in, manufacturing expenses. (Assume that the provisions relating to TDS are not applicable to this, payment.), (iii), A sum of ₹4,000 per month was paid as salary to a staff throughout the year and this has not been, debited in the books of account., (iv), Bank term loan interest actually paid upto 31.03.2022 was ₹20,000 and the balance was paid in, October 2022., (v), Housing loan principal repaid during the year was ₹50,000 and it relates to residential property
Page 787 :
Income Under The Head Business/Profession, , (vi), , 268, , occupied by him. Interest on housing loan was ₹23,000. Housing loan was taken from Canara Bank., These amounts were not dealt with in the profit and loss account given above., Depreciation allowable under the Act is to be computed on the basis of following information:, Plant & Machinery (Depreciation rate @ 15%), ₹, 12,00,000, Opening WDV (as on 01.04.2021), Additions during the year (put to use for more than 180 days), 2,00,000, 4,00,000, Total additions during the year, Note : Ignore additional depreciation, Compute the Total Income of Mr. X for the Assessment Year 2022-23., He has rejected presumptive income under section 44AD and his accounts were audited., , Solution:, , Computation of Total Income of Mr. X for the A.Y. 2022-23, Particulars, Profits and gains of business or profession, Net profit as per profit and loss account, Add:, • Excess commission paid to brother disallowed under section 40A(2), • Bank term loan interest paid after the due date of filing of return under, section 139(1) – disallowed as per section 43B, • GST penalty paid disallowed, • Depreciation debited to profit and loss account, Less:, •, •, •, •, , ₹, , Salary paid to the staff not debited to profit and loss account, Dividend from domestic companies, Income from agriculture [Exempt under section 10(1)], Depreciation under the Income-tax Act, 1961 (As per working note), , Income under the head other sources, • Dividend from domestic companies, Income from house property, Net Annual value of self-occupied property, Nil, Less: Deduction under section 24(b) – interest on housing loan, 23,000, Gross Total Income, Less: Deduction under section 80C in respect of Principal repayment of housing loan, Total Income, Working Note:, Computation of depreciation under the Income-tax Act, 1961, Depreciation @15% on ₹14 lakh (Opening WDV of ₹12 lakh plus assets, purchased during the year and put to use for more than 180 days ₹2 lakh), Depreciation @7.5% on ₹2 lakh (Cost of assets put to use for less than 180 days), , ₹, 5,00,000, 10,000, 40,000, 5,000, 2,00,000, 7,55,000, (48,000), (15,000), (1,80,000), (2,25,000), 2,87,000, 15,000, (23,000), 2,79,000, (50,000), 2,29,000, ₹, 2,10,000, 15,000, 2,25,000
Page 788 :
Income Under The Head Salary, , 269, , INCOME UNDER THE HEAD, SALARY, SECTION 15 TO 17, PARTICULARS, Meaning of salary, Basis of charge/chargeability of salaries, House rent allowance, Special allowances, Foreign allowances, Standard Deduction, Deduction on account of tax on employment/ professional tax, ‘Salary’, ‘Perquisite’ and ‘Profits in lieu of salary’, Rent free accommodation, Accommodation at concessional rent, Motor Car Facility, Gardener/Watchmen/Sweeper or any other servant, Gas/Electricity/Water, Education facility, Transport facility, Obligation of the employee met by employer, Payment of insurance premium on behalf of the employee, Specified securities or Sweat equity allotted/transferred to employees by an, employer, Any contribution to an approved superannuation fund / recognized, provident fund / national pension scheme by the employer in respect of an, employee and annual accretion, Any other fringe benefit –, Loan to the employee either at concessional rate or free of interest, Expenditure in connection with travelling, touring or accommodation to, the employee, Free refreshment or foods to the employees, Any gift, voucher or token, Expenses on credit cards, Club Membership and expenses incurred in a club, Use of moveable assets, Transfer of any moveable assets, Any other benefit, Leave travel concession, Death–cum–retirement gratuity, Treatment of commuted pension, Deduction in respect of contribution to New Pension System, Treatment of leave salary, Provident Fund, Retrenchment compensation, Compensation received on voluntary retirement, Profits in lieu of salary, , SECTIONS, 17(1), 15, 10(13A) Rule 2A, 10(14) Rule 2BB, 10(7), 16(ia), 16(iii), 17, 17(2)(i) Rule 3(1), 17(2)(ii) Rule 3(1), 17(2)(iii) Rule 3(2), 17(2)(iii) Rule 3(3), 17(2)(iii) Rule 3(4), 17(2)(iii) Rule 3(5), 17(2)(iii) Rule 3(6), 17(2)(iv), 17(2)(v), 17(2)(vi), 17(2)(vii)/17(2)(viia), , 17(2)(viii)Rule 3(7)(i), 17(2)(viii)Rule 3(7)(ii), 17(2)(viii)Rule 3(7)(iii), 17(2)(viii)Rule 3(7)(iv), 17(2)(viii)Rule 3(7)(v), 17(2)(viii)Rule 3(7)(vi), 17(2)(viii)Rule 3(7)(vii), 17(2)(viii)Rule 3(7)(viii), 17(2)(viii)Rule 3(7)(ix), 10(5) Rule 2B, 10(10), 10(10A), 80CCD, 10(10AA), 10(11), 10(12), 10(13), 10(10B), 10(10C) Rule 2BA, 17(3)
Page 789 :
Income Under The Head Salary, , 270, , COMPONENTS OF SALARY, Part – A, (i) Basic pay, (ii) Dearness allowance, (iii) Bonus, (iv) Commission/Fees etc., Part – B Allowances, Allowance means a fixed sum paid to an employee for a specific purpose without confirming the end, utilization and allowance can be divided into four categories., 1. House Rent Allowance Sec. 10(13A) Rule 2A, 2. Special Allowance Sec. 10(14) Rule 2BB, Special allowances can be divided into two categories, (i) Personal Allowance, (a) Children Education Allowance, (b) Hostel Allowance, (c) Transport Allowance, (d) Outstation Allowance, (e) Tribal Area Allowance, (f) Underground Allowance, (g) Composite Hill and fuel Allowance/High Altitude Allowance/Uncongenial climate Allowance/Snow, Bound Area Allowance, (h) Border Area Allowance/Remote Area Allowance/Difficult Area Allowance/Disturbed Area, Allowance., (i) Other allowance like Compensatory Field Area Allowance, Compensatory Highly Active Field Area, Allowance etc., (ii) Allowance for official Duty, (a) Any allowance granted to meet the cost of travel on tour or on transfer., (b) Daily allowance, (c) Conveyance allowance, (d) Helper allowance, (e) Academic allowance/research allowance, (f) Uniform allowance, 3. Foreign Allowance Sec. 10(7), 4. Any other Allowance, - City Compensatory Allowance., - Cash Allowance., - Split Duty Allowance., - Overtime Allowance., - Medical Allowance., - Servant Allowance., - Tiffin Allowance., - Entertainment Allowance (Subject to deduction u/s 16(ii))., - Similarly there may be any number of other personal allowances., Part – C Perquisites (Facilities), - Rent Free Accommodation Sec. 17(2)(i) Rule 3(1), - Accommodation at concessional rent Sec. 17(2)(ii) Rule 3(1), - Motor Car Facility Sec. 17(2)(iii) Rule 3(2), - Gardener/Watchmen/Sweeper or any other servant Sec.17(2)(iii) Rule 3(3), - Gas/Electricity/Water Sec. 17(2)(iii) Rule 3(4), - Education Facility Sec. 17(2)(iii) Rule 3(5), - Free transport Sec. 17(2)(iii) Rule 3(6), - Payment by the employer on behalf of the employee Sec. 17(2)(iv), - Payment of insurance premium on behalf of the employee Sec. 17(2)(v)
Page 790 :
Income Under The Head Salary, , 271, , - Specified securities or Sweat equity allotted/transferred to employees by an employer Sec. 17(2)(vi), - Any contribution to an approved superannuation fund / recognized provident fund / national pension, scheme by the employer in respect of an employee Sec. 17(2)(vii), - Any other fringe benefit Sec. 17(2)(viii), - Loan to the employee either at concessional rate or free of interest Rule 3(7)(i), - Expenditure in connection with travelling, touring or accommodation to the employee Rule 3(7)(ii), - Free refreshment or foods to the employees Rule 3(7)(iii), - Any gift, voucher or token Rule 3(7)(iv), - Expenses on credit cards Rule 3(7)(v), - Club Membership and expenses incurred in a club Rule 3(7)(vi), - Use of moveable assets Rule 3(7)(vii), - Transfer of any moveable asset Rule 3(7)(viii), - Any other benefit Rule 3(7)(ix), - Leave travel Concession Sec. 10(5) Rule 2B, Part – D, Superannuation Benefits, (a) Gratuity Sec. 10(10), (b) Pension Sec. 10(10A), (c) Leave Salary Sec. 10(10AA), (d) Provident Fund Sec. 10(11), 10(12), 10(13),, Part A of Fourth Schedule to Income Tax Act, (e) Retrenchment Compensation Sec. 10(10B), (f) Voluntary Retirement Sec. 10(10C) Rule 2BA, Part – E, Any Other Payment, Profits in lieu of salary Sec. 17(3), Question 1: Explain Basic Pay/Dearness Allowance/ Bonus/Commission/Fees., Answer: Basic Pay/Dearness Allowance/Bonus/Commission/Fees, Basic Pay: Basic Pay is the essential component of salary. It is given by employer to employee for his basic, qualities like qualification, experience and expertise in particular field and it is generally given in the form, of a pay scale 2,000 – 100 – 2,500 – 200 – 3,500 – 300 – 5,000 – 400 – 7,000. The pay scale has in general, 20 increments. Basic pay is always fully taxable., Dearness Allowance: Dearness Allowance is given to an employee to compensate him for increase in prices, and it is generally allowed as certain percentage of basic pay and it is linked to consumer price index and it, is revised on quarterly basis. Dearness allowance of an employee is always fully chargeable to tax., As per Section 16 (ia), Standard Deduction of `50,000 or salary whichever is lower shall be allowed from, the salary., Illustration 1: Mr. X is employed in ABC Ltd. since 01.07.2009 in the pay scale of 11,000 – 500 – 14,500, – 1200 – 20,500 – 1,500 – 28,000. The employer has paid dearness allowance @ 20% of his basic pay from, 01.04.2021 to 30.09.2021 and thereafter dearness allowance was allowed @ 25% of basic pay., Compute employee’s Tax Liability for Assessment Year 2022-23., Solution:, `, Basic Pay [(19,300 x 3) + (20,500 x 9)], 2,42,400.00, Working Note:, `, 01.07.2009 – 30.06.2010 =, 11,000 p.m., 01.07.2010 – 30.06.2011 =, 11,500 p.m., 01.07.2011 – 30.06.2012 =, 12,000 p.m., 01.07.2012 – 30.06.2013 =, 12,500 p.m., 01.07.2013 – 30.06.2014 =, 13,000 p.m., 01.07.2014 – 30.06.2015 =, 13,500 p.m., 01.07.2015 – 30.06.2016 =, 14,000 p.m., 01.07.2016 – 30.06.2017 =, 14,500 p.m.
Page 791 :
Income Under The Head Salary, 01.07.2017 – 30.06.2018 =, 01.07.2018 – 30.06.2019 =, 01.07.2019 – 30.06.2020 =, 01.07.2020 – 30.06.2021 =, 01.07.2021 – 30.06.2022 =, Dearness Allowance, Working Note:, From April to September, (19,300 x 3) x 20% =, (20,500 x 3) x 20% =, From October to March, (20,500 x 6) x 25% =, Gross Salary, Less: Standard Deduction u/s 16 (ia), Income under the head Salary, Gross Total Income, Less: Deduction u/s 80C to 80U, Total Income, Computation of Tax Liability, Tax on `2,47,030 at slab rate, Tax Liability, , 272, , 15,700 p.m., 16,900 p.m., 18,100 p.m., 19,300 p.m., 20,500 p.m., 54,630.00, `, 11,580, 12,300, 30,750, 2,97,030.00, (50,000.00), 2,47,030.00, 2,47,030.00, Nil, 2,47,030.00, Nil, Nil, , Illustration 2: Mr. X joins ABC Ltd. on 1st July’ 2012 in the pay scale of 20,000 – 500 – 25,000 – 700 –, 30,600 – 1,000 – 37,600. The company has allowed him dearness allowance @ 5% of the basic pay from, 01.04.2021 upto 30.06.2021 and thereafter dearness allowance was allowed @ 11% of the basic pay but, upto 31.12.2021 and after that dearness allowance was allowed @ 18% of the basic pay., Compute employee’s Tax Liability for the Assessment Year 2022-23., Solution:, `, Basic Pay [(24,000 x 3) + (24,500 x 9)], 2,92,500.00, Working Note:, `, 01.07.2012 – 30.06.2013 =, 20,000 p.m., 01.07.2013 – 30.06.2014 =, 20,500 p.m., 01.07.2014 – 30.06.2015 =, 21,000 p.m., 01.07.2015 – 30.06.2016 =, 21,500 p.m., 01.07.2016 – 30.06.2017 =, 22,000 p.m., 01.07.2017 – 30.06.2018 =, 22,500 p.m., 01.07.2018 – 30.06.2019 =, 23,000 p.m., 01.07.2019 – 30.06.2020 =, 23,500 p.m., 01.07.2020 – 30.06.2021 =, 24,000 p.m., 01.07.2021 – 30.06.2022 =, 24,500 p.m., Dearness Allowance, 33,000.00, Working Note:, `, From April to June, (24,000 x 3) x 5% =, 3,600, From July to December, (24,500 x 6) x 11% =, 16,170, From January to March, (24,500 x 3) x 18% =, 13,230, Gross Salary, 3,25,500.00, Less: Standard Deduction u/s 16 (ia), (50,000.00), Income under the head Salary, 2,75,500.00, Gross Total Income, 2,75,500.00
Page 792 :
Income Under The Head Salary, Less: Deduction u/s 80C to 80U, Total Income, Computation of Tax Liability, Tax on `2,75,500 at slab rate, Less: Rebate u/s 87A, Tax Liability, , 273, Nil, 2,75,500.00, 1,275.00, (1,275.00), Nil, , Bonus: It is the part of the profits of the employer, which is given to an employee and it is fully taxable., Fees/Commission: Extra payment for extra work is called commission or fees and it is always fully taxable., Question 2 [V. Imp.]: Discuss the provisions relating to taxability of recognised Provident Funds?, Answer: Recognised provident fund Part A of fourth schedule/ Section 10(12), Employer’s contribution upto 12% of the employee’s retirement benefit salary shall be exempt from, income tax., Interest credited to the provident fund account upto 9.5% p.a. shall be exempt from income tax., ‘Provided that the provisions of this clause shall not apply to the income by way of interest accrued, during the previous year in the account of a person to the extent it relates to the amount or the aggregate, of amounts of contribution made by that person exceeding two lakh and fifty thousand rupees in any, previous year in that fund, on or after the 1st day of April, 2021 and computed in such manner as may be, prescribed., Provided further that if the contribution by such person is in a fund in which there is no contribution by, the employer of such person, the provisions of the first proviso shall have the effect as if for the words, “two lakh and fifty thousand rupees”, the words “five lakh rupees” had been substituted., Deduction shall be allowed under section 80C for employee contribution., Meaning of Retirement Benefit Salary, Retirement Benefit Salary shall include:, (i) Basic pay, (ii) Dearness allowance if the terms of employment so provided, (iii) Commission if it is paid as a fixed percentage of the turnover as decided in Gestetner Duplicators Pvt., Ltd v CIT, (1979)(SC)., Illustration 3: Mr. X is employed in ABC Ltd. getting basic pay `60,000 p.m. and dearness allowance, `10,000 p.m. (forming part of salary). Employer has paid bonus `20,000 during the year. Commission was, allowed @ 2% of sales turnover of `50,00,000. The employer and employee both are contributing `11,000, p.m. (each) to the recognised provident fund. During the year interest of `1,00,000 was credited to the RPF, @ 10% p.a. Compute tax liability of Mr. X for A.Y. 2022-23., Solution:, `, Basic Pay (60,000 x 12), 7,20,000, Dearness allowance (10,000 x 12), 1,20,000, Bonus, 20,000, Commission (50,00,000 x 2%), 1,00,000, Employer’s contribution to recognised provident fund in excess of 12% of retirement benefit salary 19,200, Working Note:, RBS = 7,20,000 + 1,20,000 + 1,00,000, = 9,40,000, 12% of RBS = 1,12,800, Employer’s contribution = 11,000 x 12 = 1,32,000, 1,32,000 – 1,12,800 = 19,200, Interest credited in excess of 9.5% p.a., 2,500, (1,00,000 / 10% x 0.5%)/2, Gross Salary, 9,81,700, Less: Standard Deduction u/s 16 (ia), (50,000), Income under the head Salary, 9,31,700, Income under the head other sources, Interest credited in excess of 9.5% p.a., 2,500
Page 793 :
Income Under The Head Salary, , 274, , (1,00,000 / 10% x 0.5%)/2, Gross Total Income, 9,34,200, Less: Deduction u/s 80C, (1,32,000), Total Income, 8,02,200, Computation of Tax Liability, Tax on `8,02,200 at slab rate, 72,940.00, Add: HEC @ 4%, 2,917.60, Tax Liability, 75,857.60, Rounded off u/s 288B, 75,860.00, Note: Interest from employees contribution has been taxed under the head other sources but ICAI has taxed, it under the head salary. Students may tax it under the head salary., Payments From Recognised Provident Funds, Payments received from recognised provident fund shall be exempt from income tax if the employee has, complied with any of the conditions given below:, (i) If the employee has rendered continuous service for a period of 5 years or more, or, (ii) If he has not rendered such continuous service, the service has been terminated by reason of the, employee’s ill-health, or by the contraction or discontinuance of the employer’s business or other cause, beyond the control of the employee, or, (iii) If the employee obtains employment with any other employer and the provident fund has been, transferred to such employer and the total service with the former employer and the current employer is, of 5 years or more., If the employee has not complied with even a single condition, in that case amount received shall be taxable, but only that part which was exempt earlier. Employer contribution and interest shall be taxable under the, head Salary., Example, Mr. X retired from ABC Ltd. and received RPF balance as given belowEmployer Contribution, 5,00,000, - 15%, Employee contribution, 5,00,000, - 15%, Interest, 2,00,000, - 10% p.a., The employee has not complied with even a single condition prescribed for this purpose, taxable amount, shall be., Employer contribution = 5,00,000 / 15% x 12% = 4,00,000 (Taxable under the head Salary), Interest on employer contribution = (1,00,000 /10% x 9.5%) = 95,000 (Taxable under the head Salary), Interest on employee contribution = (1,00,000 /10% x 9.5%)= 95,000 (Taxable under the head other sources), Employee contribution 5,00,000 Exempt., Question 3: Explain provisions relating to Unrecognised Provident Fund., Answer: Employer contribution and interest on employee and employer contribution shall be exempt from, income tax so long as the employee is in employment but at the time of leaving the job, employer, contribution and interest on employer and employee contribution shall be taxable, however amount of, employee contribution shall not be taxed at the time of receipt because it has already been taxed when the, employee was in employment., The employer’s contribution and interest thereon is taxable under the head salary but interest on employee, contribution shall be taxable under the head Other Sources., No deduction is allowed under section 80C for employee contribution., Illustration 4: Mr. X is employed in ABC limited basic pay `70,000 per month, D.A. `30,000 per month ,, Bonus `50,000 and commission @ 1% on sales turnover of `100 Lakh. Employer has contributed `18,000, per month to Unrecognized provident fund and employee has also contributed an equal amount. During the, year Interest of `2,00,000 was credited on employee plus employer contribution @ 10% per annum., Compute his Income and Tax Liability.
Page 794 :
Income Under The Head Salary, Solution:, Basic Pay (70,000 x 12), Dearness allowance (30,000 x 12), Bonus, Commission (100,00,000 x 1%), Employer’s contribution to unrecognised provident fund, Interest credited in unrecognised provident fund, Gross Salary, Less: Standard Deduction u/s 16 (ia), Income under the head Salary, Gross Total Income, Less: Deduction u/s 80C to 80U, Total Income, Computation of Tax Liability, Tax on `13,00,000 at slab rate, Add: HEC @ 4%, Tax Liability, , 275, `, 8,40,000, 3,60,000, 50,000, 1,00,000, Nil, Nil, 13,50,000, (50,000), 13,00,000, 13,00,000, Nil, 13,00,000, 2,02,500, 8,100, 2,10,600, , Illustration 5: Mr. X retires from service on December 31, 2021, after 25 years of service. Following are, the particulars of his income/investments for the previous year 2021-22:, Particulars, `, Basic pay @ ` 16,000 per month for 9 months, 1,44,000, Dearness pay (50% forms part of the retirement benefits) ` 8,000 per month for 9 months, 72,000, Lumpsum payment received from the Unrecognised Provident Fund, 6,00,000, Deposits in the PPF account, 40,000, Out of the amount received from the provident fund, the employer’s share was ` 2,20,000 and the interest, thereon `50,000. The employee’s share was `2,70,000 and the interest thereon `60,000. What is the taxable, portion of the amount received from the unrecognized provident fund in the hands of Mr. A for the, assessment year 2022-23?, Solution:, Taxable portion of the amount received from the URPF in the hands of Mr. A for the A.Y. 2022-23 is, computed hereunder:, `, Amount taxable under the head “Salaries”:, Employer’s share in the payment received from the URPF, 2,20,000, Interest on the employer’s share, 50,000, Total, 2,70,000, Amount taxable under the head “Income from Other Sources” :, Interest on the employee’s share, 60,000, Total amount taxable from the amount received from the fund, 3,30,000, Illustration 6: Will your answer be any different if the fund mentioned above was a recognised provident, fund?, Solution: Since the fund is a recognised one, and the maturity is taking place after a service of 25 years, the, entire amount received on the maturity of the RPF will be fully exempt from tax., Question 4: Explain taxability of Statutory provident fund. Section 10(11) (Provident Fund Act 1925), Answer: Statutory provident fund (also called Government Provident Fund) is applicable in case of, Government employees and is regulated through Provident Fund Act, 1925. The Employer donot contribute, to this fund hence there is no tax treatment for employer contribution and interest on employer contribution., Interest on employee contribution is exempt from Income Tax., ‘Provided that the provisions of this clause shall not apply to the income by way of interest accrued, during the previous year in the account of a person to the extent it relates to the amount or the, aggregate of amounts of contribution made by that person exceeding two lakh and fifty thousand, rupees in any previous year in that fund, on or after the 1st day of April, 2021 and computed in such
Page 795 :
Income Under The Head Salary, , 276, , manner as may be prescribed:, Provided further that if the contribution by such person is in a fund in which there is no contribution by, the employer of such person, the provisions of the first proviso shall have the effect as if for the words, “two lakh and fifty thousand rupees”, the words “five lakh rupees” had been substituted;’, Further, the lump sum payment from such provident fund at the time of retirement or termination of service, is also exempt from tax., Deduction shall be allowed under section 80C for employee contribution., Question 5 [V. Imp.]: Write a note on Taxability of Gratuity., Answer: Taxability of Gratuity, Gratuity means a gratuitous payment made by the employer to the employee at the time of his leaving the, job in recognition of the meritorious services and the association of the employee with the institution. With, the enactment of Payment of Gratuity Act 1972, gratuity has become a statutory obligation on the part of, the employer., Gratuity is fully chargeable to tax if it is given during continuity of the job., Death cum retirement gratuity Section 10(10), Tax treatment of gratuity is asunder:, A – Employees of State Government/Central Government/Local Authority, B – Employees covered under payment of Gratuity Act 1972, C – Any other employee., A – Employees of State Government/Central Government/Local Authority:, Any death cum retirement gratuity received by Central or State Government employees including employees, of a local authority is fully exempt from tax., B – Employees covered under payment of Gratuity Act 1972:, Any gratuity received by the employees covered under payment of Gratuity Act 1972, shall be exempt to the, extent of the least of the following:, (i) Gratuity received, (ii) ` 20,00,000, (iii) 15 days salary for each completed year of service or part thereof in excess of six month., In case of employees of a seasonal establishment, in place of 15 days, only 7 days salary will be taken., 15 days or 7 days wages shall be calculated by considering number of days in a month to be 26., Salary here means last drawn salary and includes only Basic Pay and Dearness Allowance, However, in case of piece rated employees, salary shall be computed on the basis of average of the total, wages received by them for a period of three months immediately preceding the termination of their, employment., Example, Mr. X is the piece rated employee who is retired on 10.03.2022 and wages received by him from 11.12.2021, to 10.03.2022 are `33,000. In this case, one month salary shall be `33,000/3 = `11,000 and 15 days salary, shall be = 11,000/26 x 15 = `6,346.15, C – Any other employee:, The least of the following will be exempt, (i) Gratuity received, (ii) ` 20,00,000, (iii) Half month’s salary for each completed year of service., Salary here means average salary for ten months immediately preceding the month of retirement and, will consist of Basis pay + Dearness allowance (if provided) + Commission on sales turnover achieved by, the employee and paid at fixed rate., If an employee was retired earlier and has received gratuity and some exemption was allowed and same, employee has taken up some other employment and is retired from the other employer also, in this case, exemption shall be allowed again but maximum exemption allowed from all the employers cannot exceed, `20 lakh., If any employee is expired and gratuity has been received by the family members, exemption shall be, allowed in the normal manner and balance amount shall be taxable as income of such member under the
Page 796 :
Income Under The Head Salary, , 277, , head other sources., No exemption from gratuity is allowed if the relationship of employer and employee does not exist. e.g., Gratuity paid by LIC to its insurance agents is chargeable to tax., Illustration 7: Mr. X was employed in ABC Ltd. getting basic pay `18,000 p.m. but it was increased to, `24,000 p.m. w.e.f. 01-07-2021, dearness allowance `6,000 p.m. but it was increased to `9,000 p.m. w.e.f., 01-07-2021 (50% of DA forms part of salary). The employee was retired on 10.01.2022 after serving the, employer for 20 years and 10 months. The employer has paid him gratuity of `9,10,000 and the employee, was covered under Payment of Gratuity Act, 1972., Compute taxable portion of gratuity and also Tax Liability., Solution:, Basic Pay [(18,000 x 3) + (24,000 x 6)+ (24,000 x 10/30)], Dearness Allowance [(6,000 x 3) + (9,000 x 6) + (9,000 x 10/30)], Gratuity {Sec 10(10)}, Working Note:, Least of the following is exempt:, 1. `9,10,000, 2. `20,00,000, 3. 15/26 x 33,000 x 21 = `3,99,807.69, Received = `9,10,000.00, Exempt = (`3,99,807.69), Taxable = `5,10,192.31, Gross Salary, Less: Standard Deduction u/s 16 (ia), Income under the head Salary, Gross Total Income, Less: Deduction u/s 80C to 80U, Total Income, Rounded off u/s 288A, Computation of Tax Liability, Tax on `7,41,190 at slab rate, Add: HEC @ 4%, Tax Liability, Rounded off u/s 288B, (b) Presume Mr. X is not covered in Payment of Gratuity Act 1972., Solution:, Basic Pay [(18,000 x 3) + (24,000 x 6)+ (24,000 x 10/30)], Dearness Allowance [(6,000 x 3) + (9,000 x 6) + (9,000 x 10/30)], Gratuity {Sec 10(10)}, Working Note:, Least of the following is exempt:, 1. `9,10,000, 2. `20,00,000, 3. 1/2 x (21,000 x 4 + 28,500 x 6)/10 x 20 = `2,55,000, Received = `9,10,000, Exempt = (`2,55,000), Taxable = `6,55,000, Gross Salary, Less: Standard Deduction u/s 16 (ia), Income under the head Salary, Gross Total Income, Less: Deduction u/s 80C to 80U, , `, 2,06,000.00, 75,000.00, 5,10,192.31, , 7,91,192.31, (50,000.00), 7,41,192.31, 7,41,192.31, Nil, 7,41,192.31, 7,41,190.00, 60,738.00, 2,429.52, 63,167.52, 63,170.00, `, 2,06,000.00, 75,000.00, 6,55,000.00, , 9,36,000.00, (50,000.00), 8,86,000.00, 8,86,000.00, Nil
Page 797 :
Income Under The Head Salary, Total Income, Computation of Tax Liability, Tax on `8,86,000 at slab rate, Add: HEC @ 4%, Tax Liability, Rounded off u/s 288B, , 278, 8,86,000.00, 89,700.00, 3,588.00, 93,288.00, 93,290.00, , Illustration 8: Mr. X is retired from ABC Ltd. w.e.f. 11.11.2021 after serving the employer for 20 years and, 11 months. The employer has paid him gratuity of `5,75,000. At the time of retirement, employee’s basic, pay was `18,000 p.m. However upto 31.07.2021 it was `11,000 p.m. and the employee was getting dearness, allowance `5,000 p.m. but it was `3000 p.m. upto 31.07.2021. 30% of dearness allowance forms part of, salary., Compute taxable part of gratuity and employee’s Tax Liability., Solution:, `, Basic Pay, 1,04,000.00, [(11,000 x 4) + (18,000 x 3) + (18,000 x 10/30)], Dearness Allowance [(3,000 x 4) + (5,000 x 3) + (5,000 x 10/30)], 28,666.67, Gratuity {Sec 10(10)}, 4,33,200.00, Working Note:, Least of the following is exempt:, 1. `5,75,000, 2. `20,00,000, 3. ½ x 14,180 x 20 = `1,41,800, Received = `5,75,000, Exempt = (`1,41,800), Taxable = `4,33,200, Calculation of Average Salary, Basic Pay, From January to July `11,000 x 7 = ` 77,000, From August to October `18,000 x 3 = ` 54,000, Dearness Allowance, From January to July, ` 900 x 7 = ` 6,300, From August to October `1,500 x 3 = ` 4,500, Total = 1,41,800, Average Salary = 1,41,800 ÷ 10 = `14,180, Gross Salary, 5,65,866.67, Less: Standard Deduction u/s 16 (ia), (50,000.00), Income under the head Salary, 5,15,866.67, Gross Total Income, 5,15,866.67, Less: Deduction u/s 80C to 80U, Nil, Total Income, 5,15,866.67, Rounded off u/s 288A, 5,15,870.00, Computation of Tax Liability, Tax on `5,15,870 at slab rate, Add: HEC @ 4%, Tax Liability, Rounded off u/s 288B, (b) Presume employee is covered under Payment of Gratuity Act 1972., Solution:, Basic Pay, [(11,000 x 4) + (18,000 x 3) + (18,000 x 10/30)], Dearness Allowance [(3,000 x 4) + (5,000 x 3) + (5,000 x 10/30)], , 15,674.00, 626.96, 16,300.96, 16,300.00, `, 1,04,000.00, 28,666.67
Page 798 :
Income Under The Head Salary, Gratuity {Sec 10(10)}, Working Note:, Least of the following is exempt:, 1. `5,75,000, 2. `20,00,000, 3. 15/26 x 23,000 x 21 = `2,78,653.85, Received = `5,75,000.00, Exempt = (`2,78,653.85), Taxable = `2,96,346.15, Gross Salary, Less: Standard Deduction u/s 16 (ia), Income under the head Salary, Gross Total Income, Less: Deduction u/s 80C to 80U, Total Income, Rounded off u/s 288A, Computation of Tax Liability, Tax on `3,79,010 at slab rate, Less: Rebate u/s 87A, Tax Liability, , 279, 2,96,346.15, , 4,29,012.82, (50,000.00), 3,79,012.82, 3,79,012.82, Nil, 3,79,012.82, 3,79,010.00, 6,450.50, (6,450.50), Nil, , Illustration 9: Mr. X retired on 15.06.2021 after completion of 26 years 8 months of service and received, gratuity of `6,00,000. At the time of retirement his salary was:, Basic Salary, : ` 5,000 p.m., Dearness Allowance, : ` 3,000 p.m. (60% of which is for retirement benefits), Commission, : 1% of turnover (turnover in the last 12 months was ` 12,00,000), Bonus, : ` 12,000 p.a., Compute his taxable gratuity assuming:, (a) He is non-government employee and covered by the Payment of Gratuity Act 1972., (b) He is non-government employee and not covered by Payment of Gratuity Act 1972., (c) He is a Government employee., Solution:, (a) He is covered by the Payment of Gratuity Act 1972., Least of the following shall be exempt:, i., Gratuity received: ` 6,00,000, ii., Statutory limit : ` 20,00,000, iii., 15 days salary based on last drawn salary for each, completed year of service or part thereof in excess, of 6 months, 15/26 x last drawn salary x years of service, 15/26 x (`5,000 + `3,000) × 27 = ` 1,24,615, Taxable Gratuity, (6,00,000 – 1,24,615), ` 4,75,385, (b), He is not covered by the Payment of Gratuity Act 1972., Least of the following is exempt:, i., Gratuity received: ` 6,00,000, ii., Statutory limit : ` 20,00,000, , 10 , , (5,000 × 10 ) + (3,000 × 60% × 10 ) + 1% × 12,00,000 × 12 , 1 , , , x 26, iii., =, 2, 10, = ` 1,01,400, Taxable Gratuity (6,00,000 – 1,01,400), ` 4,98,600
Page 799 :
Income Under The Head Salary, , (c), , 280, , He is a government employee, Entire amount of gratuity is exempt, , Question 6 [V. Imp.]: Write a note on taxability of pension., Answer: Taxability of Pension, Uncommuted pension, Pension is a periodical payment received by an employee after his retirement and is taxable as salary in case, of all categories of employees., Family pension Section 56, If any employee is expired and pension is being received by his family members, such pension shall be, called family pension and as per section 56, it is taxable under the head other sources and the assessee, shall be allowed deduction under section 57 equal to 1/3 of gross pension or `15,000, whichever is less., Example, Mrs. X is getting family pension of `4,000 p.m. after the death of Mr. X. In this case, her taxable income, shall be `33,000., Commuted pension Section 10(10A), 1. Commuted Pension received by employees of Central Government, State Government, Local, Authority or Statutory Corporation., It is wholly exempt from tax under section 10(10A)., 2. Commuted pension received by any other employee, (a) In case where any other employee receives gratuity, the commuted value of 1/3rd of the pension is, exempt from tax., (b) If the employee has not received gratuity, the commuted value of ½ of such pension is exempt from, tax., Illustration 10: Mr. X is employed in ABC Ltd. getting basic pay `22,000 p.m., dearness allowance `5,000, p.m. He was retired on 21.12.2021. The employer has allowed him pension of `9,000 p.m. and the employee, has requested for commutation of 52% of his pension. The employer has allowed him such commutation on, 01.02.2022 and has paid `5,61,600. The employer has paid him gratuity of `6,95,000 and employee has, completed service of 20 years and 11 months., Compute Tax Liability for the Assessment Year 2022-23., Solution:, `, Basic Pay [(22,000 x 8) + (22,000 x 21/30)], 1,91,400.00, Dearness Allowance [(5,000 x 8) + (5,000 x 21/30)], 43,500.00, Gratuity {Sec 10(10)}, 4,75,000.00, Working Note:, Least of the following is exempt:, 1. Gratuity received `6,95,000, 2. `20,00,000, 3. ½ x 22,000 x 20 = `2,20,000, Received = `6,95,000, Exempt = (`2,20,000), Taxable = `4,75,000, Uncommuted Pension {Sec 17(1)(ii)}, 20,340.00, Working Note:, `, For December’ 2021, 9,000 x 9/30 =, 2,700, For January’ 2022, 9,000 x 1 =, 9,000, From February to March’ 2022, 9,000 x 48% x 2 =, 8,640, Total = `2,700 + `9,000 + `8,640 =, 20,340, Commuted Pension {Sec 10(10A)}, 2,01,600.00
Page 800 :
Income Under The Head Salary, , Working Note:, Received =, Exempt = 5,61,600 / 52% x 1/3, Taxable =, Gross Salary, Less: Standard Deduction u/s 16 (ia), Income under the head Salary, Gross Total Income, Less: Deduction u/s 80C to 80U, Total Income, Computation of Tax Liability, Tax on `8,81,840 at slab rate, Add: HEC @ 4%, Tax Liability, Rounded off u/s 288B, , 281, , `, 5,61,600.00, (3,60,000.00), 2,01,600.00, 9,31,840.00, (50,000.00), 8,81,840.00, 8,81,840.00, Nil, 8,81,840.00, 88,868.00, 3,554.72, 92,422.72, 92,420.00, , Illustration 11: Mr. X retired w.e.f 01.10.2021 receiving `5,000 p.m. as pension. On 01.02.2022, he, commuted 60% of his pension and received `3,00,000 as commuted pension. You are required to compute, his taxable pension assuming:, a. He is a government employee., b. He is a non-government employee, receiving gratuity of ` 5,00,000 at the time of retirement., c. He is a non-government employee and is in receipt of no gratuity at the time of retirement., Solution:, (a) He is a government employee., Uncommuted pension received (October – March), ` 24,000, [(` 5,000 × 4 months) + (40% of ` 5,000 × 2 months)], ` 3,00,000, Commuted pension received, `(3,00,000), NIL, Less : Exempt u/s 10(10A), Taxable pension, ` 24,000, (b) He is a non-government employee, receiving gratuity ` 5,00,000 at the time of retirement., Uncommuted pension received (October – March), `24,000.00, [(` 5,000 × 4 months) + (40% of ` 5,000 × 2 months)], ` 3,00,000.00, Commuted pension received, Less: Exempt u/s 10(10A), 1 `3,00,000, , × 100% , `(1,66,666.67), `1,33,333.33, ×, 60%, 3, , Taxable pension, `1,57,333.33, (c) He is a non-government employee and is not in receipt of gratuity at the time of retirement., Uncommuted pension received (October – March), ` 24,000, [ (` 5,000 × 4 months) + (40% of ` 5,000 × 2 months)], Commuted pension received, ` 3,00,000, Less: Exempt u/s 10(10A), 1 `3,00,000, , × 100% , ` (2,50,000), ` 50,000, ×, 60%, 2, , Taxable pension, ` 74,000, Pension by winners of gallantry award Section 10(18), Pension received by individuals who are winners of Param Vir Chakra, Maha Vir Chakra or Vir, Chakra or such other gallantry awards shall be fully exempt from income tax., Similarly, pension received by the family members of an individual mentioned above shall be exempt from, income tax., Family pension received by family members of the persons who died in the course of operational, duties Section 10(19)
Page 801 :
Income Under The Head Salary, , 282, , Any family pension received by the widow or children etc. of the members of armed forces including paramilitary forces of the union shall be exempt from income tax provided death of such member has occurred in, the course of operational duties., , Question 7: Write a note on deduction in case of contribution to Pension Scheme of Central, Government. (NPS), Answer: Deduction in respect of contribution to Pension Scheme of Central Government Section, 80CCD, (1) Where an assessee, being an individual has in the previous year paid or deposited any amount in his, account under a pension scheme notified by the Central Government, he shall be allowed a deduction in the, computation of his total income, of the whole of the amount so paid or deposited as does not exceed,—, (a) in the case of an employee, ten per cent of his salary in the previous year; and, (b) in any other case, twenty per cent of his gross total income in the previous year, (1A) Omitted., (1B) An assessee referred to in sub-section (1), shall be allowed a deduction in computation of his total, income, whether or not any deductions is allowed under sub-section (1), of the whole of the amount paid or, deposited in the previous year in his account under a pension scheme notified by the Central Government,, which shall not exceed fifty thousand rupees:, Provided that no deduction under this sub-section shall be allowed in respect of the amount on which a, deduction has been claimed and allowed under sub-section (1)., (2) Where, in the case of an assessee referred to in sub-section (1), the Central Government or any other, employer makes any contribution to his account referred to in that sub-section, the assessee shall be, allowed a deduction in the computation of his total income, of the whole of the amount contributed by the, Central Government or any other employer as does not exceed—, , (a) fourteen per cent, where such contribution is made by the Central Government;, (b) ten per cent, where such contribution is made by any other employer,, of his salary in the previous year., (3) Where any amount standing to the credit of the assessee in his account in respect of which a deduction, has been allowed, together with the amount accrued thereon, if any, is received by the assessee or his, nominee, in whole or in part, in any previous year,—, (a) on account of closure or his opting out of the pension scheme., (b) as pension received from the annuity plan purchased or taken on such closure or opting out,, the whole of the amount shall be deemed to be the income of the assessee or his nominee, as the case may, be, in the previous year in which such amount is received, and shall accordingly be charged to tax as, income of that previous year:, Provided that the amount received by the nominee, on the death of the assessee, under the circumstances, referred to in clause (a), shall not be deemed to be the income of the nominee., (5) For the purposes of this section, the assessee shall be deemed not to have received any amount in the, previous year if such amount is used for purchasing an annuity plan in the same previous year., Explanation.—For the purposes of this section, "salary" includes dearness allowance, if the terms of, employment so provide, but excludes all other allowances and perquisites., Deduction shall be allowed for contribution to ATAL PENSION YOJANA, , Payments from NPS, As per Section 10 (12A) any payment from the National Pension System Trust to an employee on closure of, his account or on his opting out of the pension scheme referred to in section 80CCD, to the extent it does not
Page 802 :
Income Under The Head Salary, , 283, , exceed Sixty per cent of the total amount payable to him at the time of such closure or his opting out of the, scheme shall be exempt from Income Tax., As per Section 10 (12B), any payment from the National Pension System Trust to an employee under the, pension scheme referred to in section 80CCD, on partial withdrawal made out of his account in accordance, with the terms and conditions, specified under the Pension Fund Regulatory and Development Authority, Act, 2013 and the regulations made thereunder, to the extent it does not exceed twenty-five per cent of the, amount of contributions made by him shall be exempt from Income Tax., , ATAL PENSION YOJANA, Atal Pension Yojana (APY) is the Government Of India’s social benefit pension program. Earlier when it, was launched, there were no tax benefits. However, now APY is treated like NPS for tax benefits and, eligible for deduction u/s 80CCD. Features of the scheme is as follows:, 1. Any citizen of India whose age is between 18 years to 40 years can join this scheme., 2. You will start to receive the pension when you turn 60 years of age., 3. If the subscriber dies before the age of 60 years, his/her spouse would be given an option to continue, contributing as usual, for the remaining period, till the original subscriber would have attained the age of 60, years., 4. If the spouse of the deceased not interested to continue the APY account, then he or she can close the, account there itself and can claim the amount., 5. In case of death of subscriber, the same pension would be available to the spouse and on the death of both, of them (subscriber and spouse), the pension wealth accumulated till age 60 of the subscriber would be, returned to the nominee, 6. Exit before age 60 would be permitted only in exceptional circumstances, i.e., in the event of the death of, the beneficiary or terminal disease., , Illustration 12: Mrs. X is employed in Central Government since 01.01.2021 and is getting basic pay of, `1,00,000 p.m. She has contributed `15,000 p.m. to the notified pension scheme of Central Government and, employer has also contributed an equal amount. She has paid premium of Jeevan Suraksha Policy `3,000, and invested `1,00,000 in NSC., Compute her tax liability for the assessment year 2022-23., Solution:, `, Basic Pay, 12,00,000.00, (1,00,000 x 12), Contribution to the pension fund by Central Government, 1,80,000.00, (15,000 x 12), Gross Salary, 13,80,000.00, Less: Standard Deduction u/s 16 (ia), (50,000.00), Income under the head Salary, 13,30,000.00, Gross Total Income, 13,30,000.00, Less: Deduction u/s 80C, 1,00,000.00, Less: Deduction u/s 80CCC, 3,000.00, Less: Deduction u/s 80CCD (1), 47,000.00, Additional Deduction u/s 80CCD (1B), 50,000.00, Employee contribution 1,80,000 but maximum ₹1,20,000, Employer contribution (14% of 12,00,000), 1,68,000.00, (3,68,000.00), Total Income, 9,62,000.00, Computation of Tax Liability, Tax on `9,62,000 at slab rate, 1,04,900.00, Add: HEC @ 4%, 4,196.00, Tax Liability, 1,09,096.00, Rounded off u/s 288B, 1,09,100.00
Page 803 :
Income Under The Head Salary, , 284, , Illustration 13: Mr. X has income under the head Business/Profession `20,00,000 and income under the, head house property `2,00,000 and he has deposited `2,20,000 in notified pension scheme, in this case his, income and tax liability shall be, Income under the head Business/Profession, 20,00,000, Income under the head House Property, 2,00,000, Gross Total Income, 22,00,000, Less: Deduction u/s 80CCD, (2,00,000), Total Income, 20,00,000, Computation of Tax Liability, Tax on `20,00,000 at slab rate, 4,12,500, Add: HEC @ 4%, 16,500, Tax Liability, 4,29,000, Illustration 14: The basic salary of Mr. X is ` 80,000 p.m. Both Mr. A and his employer contribute 10% of, basic salary to the pension scheme referred to in section 80CCD. Explain the tax treatment in respect of such, contribution in the hands of Mr. X., Solution:, Basic Pay, 9,60,000, (80,000 x 12), Employer’s contribution to NPS, 96,000, Gross Salary, 10,56,000, Less: Standard Deduction u/s 16 (ia), (50,000), Income under the head Salary, 10,06,000, Gross Total Income, Less: Deduction u/s 80CCD, Employer’s contribution (9,60,000 x 10%), Employee contribution (9,60,000 x 10%), Total Income, , 10,06,000, (96,000), (96,000), 8,14,000, , Computation of Tax Liability, Tax on `8,14,000 at slab rate, Add: HEC @ 4%, Tax Liability, Rounded off u/s 288B, , 75,300, 3,012, 78,312, 78,310, , RTP NOV 2020, 2. Mr. A (aged 52 years), is a CEO of XYZ Enterprise Limited. During the previous year 2021-22, he, earned salary of ₹ 1,65,00,000 and long-term capital gain on sale of listed equity shares amounting to, ₹1,06,500. He earned interest of ₹ 4,82,778 on saving account., Further, he has provided the following other information for filing his return of income:, He does not receive house rent allowance from his employer. Mr. A took a loan from State Bank of India, on 27th October 2019 for repairing his house (self-occupied) at Delhi and paid interest on such, borrowings of ₹ 80,000 and ₹ 1,50,000 towards principal amount during the previous year 2021-22., Mr. A has made the following payments towards medical insurance premium for health policies taken, for his family members:, Medical premium for his brother: ₹ 13,500 (by cheque), Medical premium for his parents: ₹ 17,670 (by cheque), Medical premium for self and his wife: ₹ 21,000 (by cheque)., He also incurred ₹ 6,400 towards preventive health check-up of his wife in cash. He deposited ₹1,00,000, towards PPF. He also deposited ₹ 50,000 and 2,50,000 towards Tier I and Tier II NPS A/c, respectively.
Page 804 :
Income Under The Head Salary, , 285, , He has paid ₹ 5,30,000 as advance tax. His employer has deducted tax at source of ₹ 51,89,000. He is of, the opinion the balance amount of tax, if any he will pay on 27 July 2022 (i.e. before the due date for, filing of return of income)., From the details given above, choose the most appropriate option to the questions given below:, (i) Compute the amount of deduction available to Mr. A under Chapter VI-A for the assessment year 202223:, (a) ₹ 2,04,070, (b) ₹ 2,42,670, (c) ₹ 2,52,670, (d) ₹ 2,02,670, , Answer: (d), Hint:, Deductions available to Mr. A under Chapter VI-A for the Assessment year 2022-23, Section 80C - Deposits in PPF, Section 80CCD - Deposits in NPS (Tier-1), Section 80D, Medical Premium paid for self and his wife, 21,000, Preventive health check- up of his wife, 4,000, Medical Premium of his parents, Section 80TTA- Interest on saving account, Total, , 1,00,000, 50,000, 25,000, 17,670, 10,000, 2,02,670, , Notes:, 1. Repayment of Principal Amount for loan taken for repairing of his house is not eligible under section, 80C., 2. Medical premium paid for his brother is not eligible under section 80D., 3. Tier I is NPS account/80CCD. Tier II is a name of a deposit scheme u/s 80C. Deduction u/s 80C for Tier, II account is given only to Government employees and further investment should be for three years or more., (ii) Assuming Mr. A pays rent of ₹ 65,000 per month for his rented house at Mumbai to Mr. C, a resident, individual, is Mr. A liable to deduct TDS on such rent. If so, what would be the rate and amount of TDS?, (a) Yes, Mr. A is liable to deduct TDS @5% amounting to ₹ 3,250 every month i.e., at the time of payment, of such rent, (b) Yes, Mr. A is liable to deduct TDS @10% amounting to ₹ 6,500 every month i.e., at the time of payment, of such rent, (c) Yes, Mr. A is liable to deduct TDS @5% amounting to ₹ 39,000 in the month of March 2022, (d) No, Mr. A is not liable to deduct TDS, since he is not required to get his books of accounts audited under, section 44AB, , Answer: (c), Hint: As per section 194IB,, `65,000 x 12 = 7,80,000 x 5% = 39,000, (iii) What would be the amount of net tax payable for the assessment year 2022-23 in the hands of Mr. A?, (a) Tax payable of ₹ 78,230, (b) Tax payable of ₹ 60,290, (c) Tax payable of ₹ 49,530, (d) Tax payable of ₹ 67,470, , Answer: (c)
Page 805 :
Income Under The Head Salary, , 286, , Hint:, Computation of Total Income and tax payable, Income under the head salary, Less: Standard deduction u/s 16(ia), Income under the head salary, Less: Loss from house property, Income under the head salary, , 1,65,00,000, (50,000), 1,64,50,000, (30,000), 1,64,20,000, , Income under the head capital gains, LTCG 112A, , 1,06,500, , Income under the head other sources, Saving Interest, , 4,82,778, , Gross Total Income, Less: Deductions u/s 80C to 80U, Total Income (Rounded off u/s 288A), , 1,70,09,278, (2,02,670), 1,68,06,610, , Computation of Tax payable, Tax on LTCG 112A 6,500 (1,06,500-1,00,000) @ 10%, Tax on 1,67,00,110 at slab rate, Tax before surcharge and cess, Add: Surcharge @ 15%, Tax before cess, Add: Health and education Cess @ 4%, Tax Liability, Less: Advance Tax, Less: TDS, Tax Payable, Rounded off u/s 288B, , 650.00, 48,22,533.00, 48,23,183.00, 7,23,477.45, 55,46,660.45, 2,21,866.42, 57,68,526.87, 5,30,000.00, 51,89,000.00, 49,526.87, 49,530.00, , (iv) Compute the amount of interest chargeable under section 234B on account of short payment of advance, tax:, (a) ₹ 1,980, (b) Nil, (c) ₹ 3,130, (d) ₹ 2,410, , Answer: (b), Hint:, Tax Liability, , 57,68,526.87, , Less: TDS, , 51,89,000.00, , Tax Payable, , 5,79,526.87, , Advance tax paid, , 5,30,000.00, , Since 5,30,000 is 90% or more of 5,79,526.87, hence no interest shall be payable u/s 234B, , Question 8 [V. Imp.]: Write a note on taxability of Leave Salary/Encashment of Leave., Answer: Taxability of Leave Salary/ Encashment of Leave
Page 806 :
Income Under The Head Salary, , 287, , Sometimes an employee may surrender his leave and may get equivalent payment in cash, it is called leave, salary., , Exemption in respect of encashment of leave salary Section 10(10AA), (1) Any leave salary received by an employee while he is in service is fully taxable under section 17(1)., (2) If he gets encashment at the time of leaving the service (including resignation) he can avail the, exemption under section 10(10AA)., The provisions of the exemption are as follows: (i) In the case of Government employees: Any amount received as leave salary at the time of retirement, whether on superannuation or otherwise, is exempt from tax, in case of employees of State, Government/Central Government. E.g. Mr. X is retired from Central Govt. and has received leave salary of, `2,00,000, in this case it will be exempt from income tax., (ii) In case of other employees including the employees of local authority and public sector undertakings:, Leave salary is exempt from tax to the extent of the least of the following:, (a) Leave salary received, (b) ` 3,00,000, (c) 10 months x average salary, (d) average salary x Leave at the credit, Leave at the credit = Leave entitlement, Less:, (i) Leave availed during entire service, Less:, (ii) Leave encashed during entire service, While computing leave entitlement, maximum leave allowed shall be 30 days for each completed year of, service (part of the year shall not be taken into consideration), ‘Salary’ includes basic salary plus dearness allowance to the extent the terms of employment so provide, plus fixed percentage of commission on the turnover achieved by the assessee., Average salary is to be calculated on the basis of the average salary drawn by the employee during the, period of 10 months immediately preceding his retirement., If any employee has received leave salary from two or more employers, exemption for each of the, employers shall be computed separately, however, total exemption allowed can not exceed `3,00,000., Salary paid to legal heirs of the deceased employee in respect of privilege leave standing to the credit of, such employee at the time of his/her death is not taxable., Illustration 15: Mr. X is retired from ABC Ltd. on 10.11.2021 after serving the employer for 20 years and, 10 months. The employer has paid him leave salary of `5,00,000. The employee was entitled for 2 month, leave per year of service. During entire service, he has availed 6 month leave and has encashed 7 month, leave. The employee was getting basic pay `27,000 p.m. but it was increased to ` 33,000 p.m. w.e.f. 01-072021. He was getting DA ` 9,000 per month but it was increased to 12,000 per month w.e.f. 01-07-2021., 50% of DA forms part of salary., Compute his Tax Liability for the Assessment Year 2022-23., Solution:, Basic Pay [(27,000 x 3)+(33,000 x 4)+(33,000 /30 x 10)], DA [(9,000 x 3)+(12,000 x 4)+(12,000 /30 x 10)], Leave Salary {Sec 10(10AA)}, Working Note:, Least of the following is exempt:, 1. `5,00,000, 2. 10 x 34,750 = `3,47,500, 3. `3,00,000, 4. 7 x 34,750 = `2,43,250, Received = `5,00,000, Exempt = (`2,43,250), Taxable = `2,56,750, Computation of leave at the credit, , `, 2,24,000.00, 79,000.00, 2,56,750.00
Page 807 :
Income Under The Head Salary, Leave Entitlement = 1 month x 20 = 20 month, Less: Leave availed, =, (6) month, Less: Leave Encashed, =, (7 ) month, Leave at the credit, =, 7 month, Calculation of average salary 11-01-2021 to 10-11-2021, Basic Pay, [(27000/30 x 20) + (27,000 x 5) + (33,000 x 4)+ (33,000/30x10)] = 2,96,000, Dearness Allowance, [(4,500/30 x 20) + (4,500 x 5) + (6,000 x 4)+ (6,000/30x10)] =, 51,500, Total, 3,47,500, Average Salary, = 3,47,500/10 =, 34,750, Gross Salary, Less: Standard Deduction u/s 16 (ia), Income under the head Salary, Gross Total Income, Less: Deduction u/s 80C to 80U, Total Income, Computation of Tax Liability, Tax on `5,09,750 at slab rate, Add: HEC @ 4%, Tax Liability, Rounded off u/s 288B, , 288, , 5,59,750.00, (50,000.00), 5,09,750.00, 5,09,750.00, Nil, 5,09,750.00, 14,450.00, 578.00, 15,028.00, 15,030.00, , Illustration 16: Mr. X is retired from ABC Ltd. w.e.f. 01.12.2021 after serving the employer for 20 years, and 10 months. The employer has paid him leave salary of `3,75,000. The employee was entitled for 20, days leave per year of service. During entire service, he has availed 35 days of leave and has encashed 10, days of leave. The employee was getting basic pay `27,000 p.m. but it was increased to ` 30,000 p.m. w.e.f., 01-07-2021. The employer has allowed him pension of `6,000 p.m. and employee was allowed commutation, of 1/3rd of his pension on 01.03.2022 amounting to `2,40,000., Compute his Tax Liability for the Assessment Year 2022-23., Solution:, `, Basic Pay [(27,000 x 3)+(30,000 x 5)], 2,31,000.00, Uncommuted Pension {Sec 17(1)(ii)}, 22,000.00, Working Note:, `, From December to February, 6,000 x 3 =, 18,000, For March, 6,000 x 2/3 =, 4,000, Total = 18,000 + 4,000 =, 22,000, Commuted Pension {Sec 10(10A)}, Nil, Leave Salary {Sec 10(10AA)}, 90,000.00, Working Note:, Least of the following is exempt:, 1. `3,75,000, 2. 10 x 28,500 = `2,85,000, 3. `3,00,000, 4. 355/30 x 28,500 = `3,37,250, Received = `3,75,000, Exempt = (`2,85,000), Taxable = `90,000, Computation of leave at the credit, Leave Entitlement = 20 x 20 = 400 days, Less: Leave availed, = (35 days)
Page 808 :
Income Under The Head Salary, Less: Leave Encashed, Leave at the credit, , 289, , = (10 days), = 355 days, , Calculation of average salary 01-02-2021 to 30-11-2021, Basic Pay, [(27,000 x 5) + (30,000 x 5) = 2,85,000, Average Salary = 2,85,000/10 = 28,500, Gross Salary, Less: Standard Deduction u/s 16 (ia), Income under the head Salary, Gross Total Income, Less: Deduction u/s 80C to 80U, Total Income, Computation of Tax Liability, Tax on `2,93,000 at slab rate, Less: Rebate u/s 87A, Tax Liability, , 3,43,000.00, (50,000.00), 2,93,000.00, 2,93,000.00, Nil, 2,93,000.00, 2,150.00, (2,150.00), Nil, , Illustration 17: Mr. X retired w.e.f 01.12.2021 after 20 years 10 months of service, receiving leave salary of, ` 5,00,000. Other details of his salary income are:, Basic Salary, : ` 5,000 p.m. (` 1,000 was increased w.e.f. 01.04.2021), Dearness Allowance, : ` 3,000 p.m. (60% of which is for retirement benefits), Commission, : ` 500 p.m., Bonus, : ` 1,000 p.m., Leave availed during service, : 480 days, He was entitled to 30 days leave every year., You are required to compute his taxable leave salary assuming:, (a) He is a government employee., (b) He is a non government employee., Solution:, (a) He is a government employee., Leave Salary received at the time of retirement, ` 5,00,000, Less: Exemption under section 10(10AA), `(5,00,000), Taxable Leave salary, Nil, (b) He is a non-government employee, Leave Salary received at the time of retirement, ` 5,00,000, Less: Exempt under section 10(10AA) [note 1], ` (26,400), Taxable Leave Salary, ` 4,73,600, Note 1 : Exemption under section 10(10AA) is least of the following:, (i), Leave salary received, ` 5,00,000, (ii), Statutory limit, ` 3,00,000, (iii) 10 months salary based on average salary of last 10 months, Salary of last 10 months i.e. Feb - Nov , , i.e. 10 ×, , 10 months, , (5000 × 8) + (4000 × 2) + (60% × 3000 × 10), , ` 66,000, = 10 ×, , 10 months, , (iv), Cash equivalent of leave standing at the credit of, the employee based on the average salary of last, 10 months (max. 30 days per year of service), Leave Due, = Leave allowed – Leave taken, = ( 30 days per year × 20 years ) – 480 days, = 120 days
Page 809 :
Income Under The Head Salary, Leave due (in days), , i.e. , × Average salary p.m., 30 days, , , 120 days 66,000 , 30 days × 10 , , , , 290, , ` 26,400, , Question 9: Are receipts in the nature of retrenchment compensation received by a person at the time, of retrenchment of his service taxable? Discuss., Answer: Retrenchment Compensation Section 10(10B), Retrenchment in general means termination of employees because the employer is closing down his business, or profession or there is substantial decline in business of employer and in such cases the employer has to, pay compensation to the employees and it is called retrenchment compensation and it will be exempt to the, extent of the least of the following:, (a) Retrenchment compensation received, (b) An amount calculated in accordance with the provisions of section 25F(b) of the Industrial Disputes Act,, 1947 or, (c) The amount as specified by the Government (i.e. `5,00,000)., , Section 25F(b) of the Industrial Disputes Act provides for payment of retrenchment compensation, equivalent to 15 days’ average pay for every year of continuous service or any part thereof in excess of six, months. Average shall be taken for 3 months and salary shall include only basic pay and dearness allowance, MAY – 2013 (4 Marks), Mr. X received retrenchment compensation of `10,00,000 after 30 years 4 months of service. At the time of, retrenchment, he was receiving basic salary of `20,000 p.m.; dearness allowance of ` 5,000 p.m. Compute, his taxable retrenchment compensation., Solution:, As per section 10(10B), exemption available to Mr. X in respect of retrenchment compensation, in this case,, will be the least of the following limits:, Compensation actually received, = `10,00,000, Statutory limit, = ` 5,00,000, Amount calculated in accordance with the provisions of section 25F of the Industrial Disputes Act, 1947, 15/26 x (20,000+5,000) x 30, = ` 4,32,692, Therefore, ` 4,32,692, being the least of the above limits, would be exempt under section 10(10B)., The taxable retrenchment compensation will be :, Retrenchment compensation received, ` 10,00,000, Less: Exemption under section 10(10B), `(4,32,692), Taxable Retrenchment Compensation, ` 5,67,308, Question 10: Write a note on voluntary retirement scheme., Answer: Voluntary Retirement Scheme Section 10(10C) Rule 2BA, Sometimes the employer may offer some amount to the employee so that the employee himself submits his, resignation and it is called voluntary retirement and in such cases amount paid by employer shall be exempt, from income tax to the extent of the least of the following, (i) The amount receivable on voluntary retirement, (ii) `5,00,000, (iii) three months’ retirement benefit salary for each completed year of service. (part of the year shall be, ignored), (iv) retirement benefit salary at the time of retirement x the balance months of service left before the date, of his retirement on superannuation., (As per section 35DDA, employer shall be allowed to debit the amount to the profit and loss account in five, annual equal installments), e.g. Mr. X has taken voluntary retirement after completion of 18 years of service and at that time remaining, service was 7 years and employer paid `6,00,000 on voluntary retirement and his retirement benefit salary at
Page 810 :
Income Under The Head Salary, , 291, , the time of voluntary retirement was `5,000 p.m., in this case taxable amount shall be, (i) `6,00,000, (ii) `5,00,000, (iii) 5,000 x 3 x 18 = 2,70,000, (iv) 5,000 x 12 x 7 = 4,20,000, Received = `6,00,000, Exempt = (`2,70,000), Taxable = `3,30,000, If in the above case employee has taken voluntary retirement after completion of 22 years of service, taxable, amount shall be, (i) `6,00,000, (ii) `5,00,000, (iii) 5,000 x 3 x 22 = 3,30,000, (iv) 5,000 x 12 x 3 = 1,80,000, Received = `6,00,000, Exempt = (`1,80,000), Taxable = `4,20,000, Illustration 18: Mr. X received voluntary retirement compensation of ` 7,00,000 after 30 years 4 months of, service. He still has 6 years of service left. At the time of voluntary retirement, he was drawing basic salary, ` 20,000 p.m.; Dearness allowance (which forms part of pay) ` 5,000 p.m. Compute his taxable voluntary, retirement compensation., Solution:, Voluntary retirement compensation received, ` 7,00,000, Less: Exemption under section 10(10C) [Note 1], `(5,00,000), Taxable voluntary retirement compensation, ` 2,00,000, Note 1: Exemption is to the extent of least of the following:, (i) Compensation actually received, = ` 7,00,000, (ii) Statutory limit, = ` 5,00,000, (iii) Last drawn salary × 3 × completed years of service, = (20,000 + 5,000) × 3 × 30 years, = ` 22,50,000, (iv) Last drawn salary × remaining months of service, = (20,000 + 5,000) × 6 × 12 months, = ` 18,00,000, Question 11 [V. Imp.]: Discuss the taxability of house rent allowance given by the employer to his, employee?, Answer: House Rent Allowance Section 10(13A) Rule 2A, Payment in cash by the employer to the employee for a specific purpose is called allowance e.g. If Mr. X is, employed in ABC Ltd. and the employer has paid him `5,000 p.m. for taking a house on rent, it will be, called house rent allowance., House rent allowance is exempt to the extent of the least of the following:, (i) Rent paid over 10% of retirement benefits salary due to the assessee for the relevant period., (ii) 50% of retirement benefit salary in case of Bombay, Calcutta, Madras or Delhi., 40% of retirement benefit salary in case of any other place., (iii) House rent allowance received., If there is any change in house rent allowance, rent paid, retirement benefits salary or the place of, posting during the year, there will be separate calculation for each of such change., Illustration 19: Mr. X is employed in ABC Ltd. getting basic pay `20,000 p.m., dearness allowance `7,000, p.m. and half of the dearness allowance forms the part of salary for the purpose of retirement benefits., The employer has paid bonus @ `500 p.m., commission @ 1% on the sales turnover of `20 lakhs. The, employer paid him house rent allowance `6,000 p.m. Employee has paid rent `7,000 p.m. and was posted at, Agra. Compute his Tax Liability for the Assessment Year 2022-23.
Page 811 :
Income Under The Head Salary, Solution:, Computation of Gross Salary, Basic Pay (20,000 x 12), Dearness Allowance (7,000 x 12), Bonus (500 x 12), Commission (1% of `20,00,000), House rent allowance {Sec 10(13A) Rule 2A}, Working Note:, Least of the following is exempt:, 1. `84,000 – `30,200 = 53,800, 2. 40% of retirement benefit salary = `1,20,800, (Retirement benefit salary = 2,40,000 + 42,000 + 20,000 = 3,02,000), 3. `72,000, Received = `72,000, Exempt = (`53,800), Taxable = `18,200, Gross Salary, Less: Standard Deduction u/s 16 (ia), Income under the head Salary, Gross Total Income, Less: Deduction u/s 80C to 80U, Total Income, Computation of Tax Liability, Tax on `3,18,200 at slab rate, Less: Rebate u/s 87A, Tax Liability, Illustration 20: Compute taxable amount of house rent allowance in the following cases:, Name of the employee, Mr. A, Mr. B, Mr. C, Basic Pay, 20,000 p.m., 20,000 p.m., 20,000 p.m., House rent allowance, 5,000 p.m., 5,000 p.m., 5,000 p.m., Rent paid, 1,500 p.m., 12,000 p.m., 14,000 p.m., Place of residence, Delhi, Delhi, Delhi, Solution:, Situation 1: (Mr. A), Computation of taxable amount of House rent allowance, 1. Rent paid over 10% of retirement benefit salary ( `18,000 – `24,000), 2. 50% of retirement benefit salary, (Retirement benefit salary = 2,40,000), 3. House Rent allowance received (5,000 x 12), Received, Exempt, Taxable, Situation 2: (Mr. B), 1. Rent paid over 10% of retirement benefit salary ( `1,44,000 – `24,000), 2. 50% of retirement benefit salary, (Retirement benefit salary = 2,40,000), 3. House Rent allowance received (5,000 x 12), Received, Exempt, Taxable, Situation 3: (Mr. C), 1. Rent paid over 10% of retirement benefit salary ( `1,68,000 – `24,000), , 292, `, , 2,40,000.00, 84,000.00, 6,000.00, 20,000.00, 18,200.00, , 3,68,200.00, (50,000.00), 3,18,200.00, 3,18,200.00, Nil, 3,18,200.00, 3,410.00, (3,410.00), Nil, Mr. D, 20,000 p.m., 5,000 p.m., Nil, Delhi, , `, Nil, 1,20,000, , 60,000, 60,000, Nil, 60,000, 1,20,000, 1,20,000, 60,000, 60,000, (60,000), Nil, 1,44,000
Page 812 :
Income Under The Head Salary, 2. 50% of retirement benefit salary, (Retirement benefit salary = 2,40,000), 3. House Rent allowance received (5,000 x 12), Received, Exempt, Taxable, Situation 4: (Mr. D), 1. Rent paid over 10% of retirement benefit salary, 2. 50% of retirement benefit salary, (Retirement benefit salary = 2,40,000), 3. House Rent allowance received (5,000 x 12), Received, Exempt, Taxable, , 293, 1,20,000, 60,000, 60,000, (60,000), Nil, Nil, 1,20,000, 60,000, 60,000, Nil, 60,000, , Illustration 21: Mr. X is employed in ABC Ltd. getting basic pay `20,000 p.m. but it was increased to, `30,000 p.m. w.e.f. 01.09.2021. The employer was paying him house rent allowance `6,000 p.m. but it was, decreased to `3,000 p.m. w.e.f. 01.11.2021. The employee was paying rent `4,000 p.m. but it was increased, to `7,000 p.m. w.e.f. 01.12.2021. The employee was posted at Amritsar but was transferred to Calcutta, w.e.f. 01.02.2022. The employee has resigned w.e.f. 01.03.2022., Compute his Tax Liability for the Assessment Year 2022-23., Solution:, `, Basic Pay [(20,000 x 5) + (30,000 x 6)], 2,80,000, House rent allowance {Sec 10(13A) Rule 2A}, 32,000, Working Note:, From April to August, Least of the following is exempt:, 1. `20,000 – `10,000 = `10,000, 2. 40% of retirement benefit salary = `40,000, (Retirement benefit salary = `1,00,000), 3. `30,000, Received = `30,000, Exempt = (`10,000), Taxable = `20,000, From September to October, Least of the following is exempt:, 1. `8,000 – `6,000 = `2,000, 2. 40% of retirement benefit salary = `24,000, (Retirement benefit salary = `60,000), 3. `12,000, Received = `12,000, Exempt = ` (2,000), Taxable = ` 10,000, For November, Least of the following is exempt:, 1. `4,000 – `3,000 = `1,000, 2. 40% of retirement benefit salary = `12,000, (Retirement benefit salary = `30,000), 3. `3,000, Received = `3,000, Exempt = (`1,000), Taxable = `2,000
Page 813 :
Income Under The Head Salary, From December to January, Least of the following is exempt:, 1. `14,000 – `6,000 = `8,000, 2. 40% of retirement benefit salary = `24,000, (Retirement benefit salary = `60,000), 3. `6,000, Received = `6,000, Exempt = (`6,000), Taxable = Nil, For February, Least of the following is exempt:, 1. `7,000 – `3,000 = `4,000, 2. 50% of retirement benefit salary = `15,000, (Retirement benefit salary = `30,000), 3. `3,000, Received = `3,000, Exempt = (`3,000), Taxable =, Nil, Total = 20,000 + 10,000 + 2,000 + Nil + Nil = `32,000, Gross Salary, Less: Standard Deduction u/s 16 (ia), Income under the head Salary, Gross Total Income, Less: Deduction u/s 80C to 80U, Total Income, Computation of Tax Liability, Tax on `2,62,000 at slab rate, Less: Rebate u/s 87A, Tax Liability, , 294, , 3,12,000, (50,000), 2,62,000, 2,62,000, Nil, 2,62,000, 600.00, (600.00), Nil, , MAY-2012 (8 Marks), Mr. X is employed with XY Ltd. on a basic salary of `10,000 p.m. He is also entitled to Dearness allowance, @ 100% of basic salary, 50% of which is included in salary as per terms of employment. The company, gives him house rent allowance of ` 6,000 p.m. which was increased to ` 7,000 p.m. with effect from, 01.01.2022. He also got an increment of ` 1,000 p.m. in his basic salary with effect from 01.02.2022. Rent, paid by him during the previous year 2021-22 is as under:, April and May, 2021, Nil, as he stayed with his parents., June to October, 2021, ` 6,000 p.m. for an accommodation in Ghaziabad., November, 2021 to March, 2022, ` 8,000 p.m. for an accommodation in Delhi., Compute the gross salary for Assessment Year 2022-23., Answer:, Computation of gross salary of Mr. X for A.Y. 2022-23, `, Particulars, Basic salary [(` 10,000 × 10) + (` 11,000 × 2)], 1,22,000, Dearness Allowance (100% of basic salary), 1,22,000, House Rent Allowance (See Note below), 21,300, Gross Salary, 2,65,300, Note: Computation of Taxable House Rent Allowance (HRA), 1. April to May, Least of the following is exempt:, 1. Nil – `3,000 = Nil, 2. 40% of retirement benefit salary = `12,000, (Retirement benefit salary =(`10,000 + 5,000) x 2
Page 814 :
Income Under The Head Salary, = 30,000, 3. (6,000 x 2)= `12,000, Received = `12,000, Exempt = Nil, Taxable = `12,000, 2. June to October, Least of the following is exempt:, 1. (6,000 x 5) = 30,000 – `7,500 = 22,500, 2. 40% of retirement benefit salary = `30,000, (Retirement benefit salary =(`10,000 + 5,000) x 5, = 75,000, 3. (6,000 x 5) = `30,000, Received = `30,000, Exempt = `22,500, Taxable = ` 7,500, 3. November to December, Least of the following is exempt:, 1. (8,000 x 2) = 16,000 – `3,000 = 13,000, 2. 50% of retirement benefit salary = `15,000, (Retirement benefit salary =(`10,000 + 5,000) x 2, = 30,000, 3. (6,000 x 2) = `12,000, Received = `12,000, Exempt = `12,000, Taxable =, Nil, 4. January, Least of the following is exempt:, 1. (8,000 x 1) = 8,000 – `1,500 = 6,500, 2. 50% of retirement benefit salary = `7,500, (Retirement benefit salary =(`10,000 + 5,000) x 1, = 15,000, 3. (7,000 x 1) = `7,000, Received = `7,000, Exempt = `6,500, Taxable = ` 500, 5. February to March, Least of the following is exempt:, 1. (8,000 x 2) = 16,000 – `3,300 = 12,700, 2. 50% of retirement benefit salary = `16,500, (Retirement benefit salary =(`11,000 + 5,500) x 2, = 33,000, 3. (7,000 x 2) = `14,000, Received = `14,000, Exempt = `12,700, Taxable = ` 1,300, Question 12 [V. Imp.]: Write a note on special allowances Section 10(14) Rule 2BB., Answer: Special Allowance Section 10(14) Rule 2BB, Special allowances are divided into 2 parts:, (A) Personal allowance Section 10(14), (B) Official allowance Section 10(14), Personal allowance Section 10(14) Rule 2BB, Personal allowances are as given below:, , 295
Page 815 :
Income Under The Head Salary, , 296, , (1) Children Education Allowance, Children education allowance is exempt upto `100 p.m. per child upto two child., Example, Mr. X is employed in ABC Ltd. and the employer has paid him children education allowance of `175 p.m., per child for three children. In this case, taxable amount shall be, I, II, III, 175, 175, 175, 100, 100, 75, 75, 175 = 325 x 12 = 3,900, Similarly, if the employer has paid children education allowance of `45 p.m. per child for three children,, taxable amount shall be, I, II, III, 45, 45, 45, 45, 45, Nil, Nil, 45 = 45 x 12 = 540, , • Exemption is allowed irrespective of the expenditure incurred., Example, Mr. X is employed in ABC Ltd. and is getting children education allowance of `400 p.m. per child for three, children and Mr. X has incurred `500 p.m. on the education of each of the child, in this case taxable amount, shall be, I, II, III, 400, 400, 400, 100, 100, Nil, 300, 300, 400 = 1000 x 12 = 12,000, (2) Hostel Allowance, Any allowance granted to an employee to meet the hostel expenditure on his child is exempt upto `300 p.m., per child upto two children. Remaining provisions are similar to children education allowance., (3) Transport Allowance, Allowance given to an employee to meet his expenditure for the purpose of commuting between the place of, his residence and the place of his duty is called transport allowance. Transport allowance granted to an, employee, who is blind or orthopaedically handicapped with disability of lower extremities, to meet his, expenditure for the purpose of commuting between the place of his residence and the place of his duty is, exempt upto `3,200 p.m., (4) Outstation Allowance, Any allowance granted to an employee working in any transport system to meet his personal expenditure, during his duty performed in the course of running of such transport from one place to another place is, called outstation allowance. Such allowance is given in lieu of daily allowance. It is exempt to the extent of, least of the following:, (i) 70% of the allowance, (ii) `10,000 p.m., Example, Mr. X is employed in Indian Airlines and is getting outstation allowance of `10,000 p.m. In this case,, exemption allowed shall be (10,000 x 12 x 70%) or (10,000 x 12) whichever is less i.e. `84,000 and taxable, amount shall be `36,000, If the transport system has provided daily allowance as well as outstation allowance, in such cases, daily, allowance is fully exempt and outstation allowance is fully taxable., Transport system shall include Railways, Roadways, Shipping company etc. It will also include any other, private transporter., (5) Underground Allowance, Sometimes the employer may pay some allowance to the employees who are working in the mines and such, allowance is called underground allowance and it is exempt upto `800 p.m.
Page 816 :
Income Under The Head Salary, , 297, , (6) Tribal Area Allowance, Sometimes an employee may be posted in the tribal area and employer may pay him some allowance, it is, called tribal area allowance and it is exempt upto `200 p.m., Official allowance Section 10(14) Rule 2BB, The allowances given by the employer for official purpose are called official allowance and are exempt from, income tax however saving is taxable and are as given below:, (1) Travelling Allowance means allowance given by the employer to meet the cost of traveling when the, employee is on official tour., (2) Daily Allowance means allowance given by the employer to meet the cost of boarding and lodging, when the employee is on official tour. Since it is given on per day basis, it is called daily allowance., (3) Conveyance Allowance: Any allowance granted to meet the expenditure incurred on conveyance is, called conveyance allowance., (4) Helper Allowance: Any allowance granted to meet the expenditure incurred on a helper where such a, helper is engaged for the performance of duties of office or employment., (5) Academic Allowance: Any allowance granted for encouraging academic research and training, pursuits in educational and research institutions., (6) Uniform Allowance: Any allowance granted to meet the expenditure incurred on the purchase or, maintenance of uniforms for wear during the performance of the duties of an office or employment., , If the above allowances are given for personal purpose, it will be taxable e.g. If travelling allowance or, conveyance allowance is given for personal purpose, it will be taxable., Illustration 22: Mr. X is employed in Central Government getting basic pay `18,000 p.m., dearness, allowance `6,000 p.m. Employer has paid children education allowance `700 p.m. per child w.e.f., 01.09.2021 and hostel allowance of `1,000 p.m. for one child w.e.f. 01.10.2021., Employer has paid transport allowance `1,700 p.m. w.e.f. 01.11.2021. Employer has paid house rent, allowance `5,000 p.m. w.e.f 01.01.2022., The employee has resigned from 01.02.2022 and has taken up a new job w.e.f. 01.03.2022. He is getting, basic pay `27,000 p.m. and house rent allowance `4,000 p.m., Compute his Tax Liability for the Assessment Year 2022-23., Solution:, `, Basic Pay [(18,000 x 10) + (27,000 x 1)], 2,07,000.00, Dearness Allowance (6,000 x 10), 60,000.00, House rent allowance {Sec 10(13A) Rule 2A}, 9,000.00, Working Note:, For January, Least of the following is exempt:, 1. Nil, 2. 40% of retirement benefit salary = `7,200, (Retirement benefit salary = `18,000), 3. `5,000, Received = `5,000, Exempt =, Nil, Taxable = `5,000, For March, Least of the following is exempt:, 1. Nil, 2. 40% of retirement benefit salary = `10,800, (Retirement benefit salary = `27,000), 3. `4,000, Received = `4,000, Exempt =, Nil, Taxable = `4,000
Page 817 :
Income Under The Head Salary, Total = `5,000 + `4,000 = `9,000, Children Education Allowance {Sec 10(14) Rule 2BB}, Working Note:, Received = `700 x 5 x 1 =, Exempt = `100 x 5 x 1 =, Taxable, =, Hostel Allowance {Sec 10(14) Rule 2BB}, Working Note:, Received = `1,000 x 4 x 1 =, Exempt = ` 300 x 4 x 1 =, Taxable, =, Transport Allowance, Gross Salary, Less: Standard Deduction u/s 16(ia), Income under the head salary, Gross Total Income, Less: Deduction u/s 80C to 80U, Total Income, Computation of Tax Liability, Tax on `2,36,900 at slab rate, Tax Liability, , 298, 3,000.00, `, 3,500, (500), 3,000, , 2,800.00, `, 4,000, (1,200), 2,800, , 5,100.00, 2,86,900.00, (50,000.00), 2,36,900.00, 2,36,900.00, Nil, 2,36,900.00, Nil, Nil, , Illustration 23: Mr. X has two sons. He is in receipt of children education allowance of ` 150 p.m. for his, elder son and ` 70 p.m. for his younger son. He also receives the following allowances:, Tribal area allowance: ` 500 p.m., Compute his taxable allowances., Solution:, Taxable allowance in the hands of Mr. X is computed as under Children Education Allowance:, Elder son [(` 150 – ` 100) p.m. × 12 months], = ` 600, Younger son [(` 70 – ` 70) p.m. × 12 months], = Nil, ` 600, Tribal area allowance [(` 500 – ` 200)p.m. × 12 months], ` 3,600, Taxable allowances, ` 4,200, Question 13: What is tax incidence on allowance and perquisites provided by the Govt. to its, employees posted abroad? (Foreign allowance), Answer: Allowances and perquisites provided by the government to its employees posted abroad, Section 10(7), Any allowances or perquisites paid or allowed as such outside India by the Government to a citizen of India, for rendering service outside India are exempt from income tax., Example, Mr. X is an I.F.S. and is a Citizen of India and is getting Medical Allowance, Servant Allowance, Education, Allowance and some other allowance outside India, and is posted in U.S.A in Indian Embassy. All these, allowances are fully exempt from income tax under section 10(7)., Similarly, any perquisite allowed to such employees shall be fully exempt from tax., Question 14: Explain other allowances., Answer: Other Allowances Section 17(1), Any other allowance is fully chargeable to tax and such allowances may be:, - City Compensatory Allowance., - Cash Allowance, - Overtime Allowance., - Medical Allowance.
Page 818 :
Income Under The Head Salary, -, , 299, , Servant Allowance., Tiffin Allowance., Entertainment Allowance (Subject to deduction under section 16(ii)), Similarly there may be any number of other personal allowances., , Illustration 24: Mr. X is employed in ABC Ltd. getting basic pay `30,000 p.m. and employer has paid him, following allowances., 1. Servant allowance `500 p.m. but the employee has saved `100 p.m., 2. Lunch allowance `300 p.m., 3. Cash allowance `500 p.m., 4. Overtime allowance `400 p.m., 5. Outstation allowance `700 p.m., 6. Special allowance `1,700 p.m., 7. Conveyance allowance (personal use) `200 p.m., Compute employee’s Gross Salary and Tax Liability for the Assessment Year 2022-23., Solution:, `, Basic Salary (30,000 x 12), 3,60,000, Servant Allowance (500 x 12), 6,000, Lunch Allowance (300 x 12), 3,600, Cash Allowance (500 x 12), 6,000, Overtime Allowance (400 x 12), 4,800, Outstation Allowance (700 x 12), 8,400, Special Allowance [1,700 x 12], 20,400, Conveyance Allowance (200 x 12), 2,400, Gross Salary, 4,11,600, Less: Standard Deduction u/s 16(ia), (50,000), Income under the head salary, 3,61,600, Gross Total Income, 3,61,600, Less: Deduction u/s 80C to 80U, Nil, Total Income, 3,61,600, Computation of Tax Liability, Tax on `3,61,600 at slab rate, Less: Rebate u/s 87A, Tax Liability, , 5,580.00, (5,580.00), Nil, , Question 15: Write a note on Standard Deduction., Answer: As per Section 16 (ia) a deduction of fifty thousand rupees or the amount of the salary, whichever, is less., Question 16: Write a note on entertainment allowance., Answer: Entertainment allowance Section 16(ii) – Sometimes the employer may pay some amount to the, employee to entertain the customers of the employer and it is called entertainment allowance and entire, amount shall be added to the gross salary of the employee however deduction shall be allowed in case of, government employees under section 16(ii) to the extent of the least of the following:, (i) 20% of basic salary, (ii) ` 5,000, (iii)The actual allowance received by the employee, Deduction is allowed only if the employee is State Government or Central Government employee i.e. in case, of employees of Local Authority, Statutory Corporation, Public Sector Undertaking etc, deduction is not, allowed., If the employee has saved any amount, it will not be taken into consideration.
Page 819 :
Income Under The Head Salary, , 300, , Illustration 25: Mr. X is employed in central Government getting basic pay `30,000 p.m., dearness, allowance `7,000 p.m., servant allowance `2,000 p.m., entertainment allowance `1,000 p.m., city, compensatory allowance `600 p.m. In this case tax liability of Mr. X shall be, Basic Salary (30,000 x 12), 3,60,000, Dearness Allowance (7,000 x 12), 84,000, Servant Allowance (2,000 x 12), 24,000, Entertainment Allowance ( 1,000 x 12), 12,000, City Compensatory Allowance (600 x 12), 7,200, Gross Salary, 4,87,200, Less: Standard Deduction u/s 16(ia), (50,000), Less: Entertainment Allowance u/s 16(ii), (5,000), Working Note:, Least of the following:, (i) 20% of basic salary (3,60,000 x 20% = 72,000), (ii) ` 5,000, (iii) The actual allowance received by the employee `12,000, Income under the head salary, 4,32,200, Gross Total Income, 4,32,200, Less: Deduction u/s 80C to 80U, Nil, Total Income, 4,32,200, Computation of Tax Liability, Tax on `4,32,200 at slab rate, 9,110.00, Less: Rebate u/s 87A, (9,110.00), Tax Liability, Nil, Question 17: Write a note on Professional Tax/Employment Tax., Answer: Professional Tax/Employment Tax, Section 16(iii), As per article 276 of Indian constitution, state government is empowered to levy a tax on profession,, business or employment and such tax shall be called professional tax or employment tax. If the person has, business or profession, such tax can be debited to profit and loss account on actual payment basis and if the, assessee is the employee he will be allowed to claim deduction from gross salary under section 16(iii) to, compute income under the head salary. If the amount has been paid by the employer on behalf of the, employee, it will be first included in gross salary under section 17(2)(iv) and subsequently deduction is, allowed under section 16(iii). If the amount is due but not paid, deduction is not allowed., Illustration 26: Mr. X is employed in Central Government getting basic pay `14,000 p.m., dearness, allowance `5,000 p.m., House rent allowance `4,000 p.m. w.e.f. 01.07.2021. However, employee is residing, in the house of his parents. Employer has paid cash allowance `300 p.m., medical allowance `250 p.m. and, entertainment allowance `400 p.m. Employer has paid professional tax `75 p.m. on behalf of the employee., Employee has saved `35 p.m. out of entertainment allowance., Compute employee’s income under the head Salary and Tax Liability for the Assessment Year 2022-23., Solution:, `, Basic Pay (14,000 x 12), 1,68,000.00, Dearness allowance (5,000 x 12), 60,000.00, House rent allowance {Sec 10(13A) Rule 2A}, 36,000.00, Working Note:, Least of the following is exempt:, 1. Nil, 2. 40% of retirement benefit salary = `50,400, (Retirement benefit salary = `1,26,000), 3. `36,000, Received = `36,000, Exempt =, Nil
Page 820 :
Income Under The Head Salary, Taxable = `36,000, Cash Allowance (300 x 12), Medical Allowance (250 x 12), Entertainment Allowance (400 x 12), Professional tax paid by employer (75 x 12), Gross Salary, Less: Standard Deduction u/s 16(ia), Less: 16(ii) Entertainment Allowance, Working Note:, Least of the following is deductible:, 1. Entertainment allowance received `4,800, 2. `5,000, 3. 20% of `1,68,000 = `33,600, Less: 16(iii) Professional Tax, Income under the head salary, Gross Total Income, Less: Deduction u/s 80C to 80U, Total Income, Computation of Tax Liability, Tax on `2,20,600 at slab rate, Tax Liability, , 301, 3,600.00, 3,000.00, 4,800.00, 900.00, 2,76,300.00, (50,000.00), (4,800.00), , (900.00), 2,20,600.00, 2,20,600.00, Nil, 2,20,600.00, Nil, Nil, , Illustration 27: Mr. X receives the following emoluments during the previous year ending 31.03.2022., Basic pay, ` 40,000, Dearness Allowance, ` 15,000, Commission, ` 10,000, Entertainment allowance, ` 4,000, Medical allowance, ` 10,000, Professional tax paid, ` 3,000 (` 2,000 was paid by his employer), He has no other income. Determine the income from salary for A.Y. 2022-23, if Mr. X is a State, Government employee., Solution:, Computation of Salary of Mr. X for the A.Y. 2022-23, Particulars, `, `, Basic Salary, 40,000, Dearness Allowance, 15,000, Commission, 10,000, Entertainment Allowance received, 4,000, Medical allowance, 10,000, Professional tax paid by the employer, 2,000, Gross Salary, 81,000, - Standard Deduction u/s 16(ia), (50,000), Less: Deductions, under section 16(ii) Entertainment allowance being lower of :, (a) Allowance received, 4,000, (b) One fifth of basic salary [1/5 × 40,000], 8,000, (c) Statutory amount, 5,000, (4,000), - under section 16(iii) Professional tax paid, (3,000), Income from Salary, 24,000, Question 18 [V. Imp.]: Write a note on chargeability of Salary., Answer: Chargeability of Salary Section 15, Salary is taxable on due basis or receipt basis whichever is earlier i.e. if salary is due but not received, it is
Page 821 :
Income Under The Head Salary, , 302, , taxable in the year in which it is due. Similarly if salary has been received in advance, it is taxable in the, year in which it has been received., Example, Mr. X is employed in ABC Ltd. and is getting basic pay of `50,000 p.m. during the previous year 2021-22, and the employer has made the payment in time upto February 2022 but salary for the month of March 2022, was paid in April 2022, in this case his salary taxable in previous year 2021-22 shall be `6,00,000., Example, Mr. X is employed in ABC Ltd. and is getting basic pay of `50,000 p.m. during the previous year 2021-22, and he has taken salary in advance for the month of April and May 2022 in the month of March 2022, in this, case his salary taxable in previous year 2021-22 shall be `7,00,000., Arrears of Salary, Sometimes salary of employee may be increased from retrospective effect i.e. from back date and employee, may receive arrear of salary, such arrears are taxable in the year in which arrears have been received, however relief shall be allowed under section 89., Example, Mr. X is employed in ABC Ltd. getting basic pay `50,000 p.m. but the employer has increased his basic pay, to `60,000 p.m. on 01.07.2021 but the increase is w.e.f 01.01.2021, in this case arrears of salary amounting, to `30,000 shall be taxable in the year 2021-22 and accordingly his gross salary shall be considered to be, `7,50,000., Question 19: Write a note on relief under section 89., Answer: Relief under Section 89, If any person has received arrears of salary or advance of salary and because of this reason his tax liability, has increased, he may claim relief under section 89 in the manner given below:, 1. Compute tax liability for the previous year in which the arrear or advance of salary has been received, including the amount of such arrear or advance., 2. Compute tax liability for the previous year in which the arrear or advance has been received excluding, such arrear or advance., 3. Tax at step no. 1 minus tax at step no. 2 shall be the tax on such arrear or advance., 4. Compute tax liability of the previous year to which the arrear or advance relates including such arrear, or advance., 5. Compute tax liability of the previous year to which arrear or advance relates excluding such arrear or, advance., 6. Tax at step no. 4 minus tax at step no. 5 shall be tax on the arrears or advance in the year to which such, arrear or advance relates., 7. Tax at step no. 3 minus tax at step no. 6 shall be the relief under section 89., If there is no excess, no relief is admissible., JUNE – 2009 (5 Marks), Mr. X, an employee of a PSU, furnishes the following particulars for the previous year ending 31.03.2022:, `, i. Salary for the year, 6,00,000, ii. Arrears of Salary for Previous Year 2020-21 received during the year, 2,00,000, iii. Salary for the Previous Year 2020-21, 4,00,000, You are requested by the assessee to compute relief under section 89 of the Income-tax Act, 1961, in terms, of tax payable for previous year 2021-22., The rates of Income-tax for the Previous Year 2020-21 are:, Tax Rate (%), On first ` 2,50,000, Nil, On ` 2,50,000 - ` 5,00,000, 5, On ` 5,00,000 - ` 10,00,000, 20, Above ` 10,00,000, 30
Page 822 :
Income Under The Head Salary, , 303, , Health & Education cess @ 4%, Standard deduction u/s 16(ia) was ` 50,000 in year 2020-21, Answer., Computation of Relief under section 89 for the Previous Year 2021-22, Step 1., Previous Year 2021-22, Salary for the year excluding the arrears, Add: Arrears relating to Previous Year 2020-21, Gross Salary, Less: Standard deduction u/s 16(ia), Income under the head salary, Tax before health & education cess, Add: HEC @ 4%, Tax Liability, Step 2., Previous Year 2021–22, Salary, Gross Salary, Less: Standard deduction u/s 16(ia), Income under the head Salary, Tax before health & education cess, Add: HEC @ 4%, Tax Liability, Step 3. Difference between Step 1 and Step 2 (65,000 – 23,400), Step 4., Previous Year 2020–21, Salary, Add: Arrears, Gross Salary, Less: Standard deduction u/s 16(ia), Income under the head Salary, Tax before HEC, Add: HEC @ 4%, Tax Liability, Step 5., Previous Year 2020–21, Salary, Gross Salary, Less: Standard deduction u/s 16(ia), Income under the head Salary, Tax before HEC, Less: Rebate u/s 87A, Tax Liability, Step 6. Difference between Step 4 and Step 5 (23,400 – Nil), Step 7. Relief under section 89 Step 3 – Step 6 (41,600 – 23,400), Tax after adjusting relief u/s 89 [65,000 – 18,200], , `, , 6,00,000, 2,00,000, 8,00,000, (50,000), 7,50,000, 62,500, 2,500, 65,000, 6,00,000, 6,00,000, (50,000), 5,50,000, 22,500, 900, 23,400, 41,600, 4,00,000, 2,00,000, 6,00,000, (50,000), 5,50,000, 22,500, 900, 23,400, 4,00,000, 4,00,000, (50,000), 3,50,000, 5,000, (5,000), Nil, 23,400, 18,200, 46,800, , MAY-2012 (4 Marks), Mr. X, an employee of a PSU, furnishes the following particulars for the previous year ending 31.03.2022, `, (i) Salary for the year, 10,00,000, (ii) Arrear of Salary for previous year 2020-21 received during the year, 2,00,000, (iii) Salary for the previous year 2020-21, 8,00,000, You are requested by the assessee to compute relief under section 89 of the Income-tax Act, 1961 in terms, of tax payable for assessment year 2022-23:, The rates of income tax for the previous year 2020-21 are, Tax rate (%)
Page 823 :
Income Under The Head Salary, On first `2,50,000, Nil, On `2,50,000- `5,00,000, 5, On `5,00,000-`10,00,000, 20, Above `10,00,000, 30, Health & Education cess @ 4%, Standard deduction u/s 16(ia) was ` 50,000 in year 2020-21, Answer., Computation of Relief under section 89 for the Previous Year 2021-22, Step 1., Previous Year 2021-22, Salary for the year excluding the arrears, Add: Arrears relating to Previous Year 2020-21, Gross Salary, Less: Standard deduction u/s 16(ia), Income under the head salary, Tax before health & education cess, Add: HEC @ 4%, Tax Liability, Step 2., Previous Year 2021–22, Salary for the year excluding the arrears, Gross Salary, Less: Standard deduction u/s 16(ia), Income under the head salary, Tax before health & education cess, Add: HEC @ 4%, Tax Liability, Step 3. Difference between Step 1 and Step 2 (1,63,800 – 1,06,600), Step 4., Previous Year 2020–21, Salary, Add: Arrears, Gross Salary, Less: Standard deduction u/s 16(ia), Income under the head Salary, Tax before health & education cess, Add: HEC @ 4%, Tax Liability, Step 5., Previous Year 2020–21, Salary, Gross Salary, Less: Standard deduction u/s 16(ia), Income under the head Salary, Tax before health & education cess, Add: HEC @ 4%, Tax Liability, Step 6. Difference between Step 4 and Step 5 (1,06,600 – 65,000), Step 7. Relief under section 89 Step 3 – Step 6 (57,200 – 41,600), Tax after adjusting relief u/s 89 [1,63,800– 15,600], , 304, , `, , 10,00,000, 2,00,000, 12,00,000, (50,000), 11,50,000, 1,57,500, 6,300, 1,63,800, 10,00,000, 10,00,000, (50,000), 9,50,000, 1,02,500, 4,100, 1,06,600, 57,200, 8,00,000, 2,00,000, 10,00,000, (50,000), 9,50,000, 1,02,500, 4,100, 1,06,600, 8,00,000, 8,00,000, (50,000), 7,50,000, 62,500, 2,500, 65,000, 41,600, 15,600, 1,48,200, , Question 20: Explain when the salary becomes due., Answer: When is salary due :, Last day of the month/first of the next month, Salary is due on the last day of the month or on the first day of the next month depending upon the, agreement between the employee and the employer., If salary is due on the last day of the month, salary from April to March shall be due during the particular, year but if salary is due on the first of next month, salary from March to February shall be due during the, relevant previous year.
Page 824 :
Income Under The Head Salary, , 305, , e.g. Mr. X is employed in ABC Ltd. getting basic pay `50,000 p.m. but it was increased to `70,000 p.m., w.e.f. 1st July 2021. Compute his total income and tax liability in two situations, (i) Salary is due on last day of the month, (ii) Salary is due on first of next month, Solution:, (i) Salary is due on last day of the month, In this case salary shall be taxable from April to March and shall be as given below:, April to June 2021 (50,000 x 3), 1,50,000, July to March 2022 (70,000 x 9), 6,30,000, Gross Salary, 7,80,000, Less: Standard Deduction u/s 16(ia), (50,000), Income under the head salary, 7,30,000, Gross Total Income/Total Income, 7,30,000, Computation of Tax Liability, Tax on `7,30,000 at slab rate, Add: HEC @ 4%, Tax Liability, , (ii) Salary is due on first of next month, In this case salary shall be taxable from March to February and shall be as given below:, March to June 2021 (50,000 x 4), July to February 2022 (70,000 x 8), Gross Salary, Less: Standard Deduction u/s 16(ia), Income under the head salary, Gross Total Income/Total Income, Computation of Tax Liability, Tax on `7,10,000 at slab rate, Add: HEC @ 4%, Tax Liability, , 58,500, 2,340, 60,840, , 2,00,000, 5,60,000, 7,60,000, (50,000), 7,10,000, 7,10,000, 54,500, 2,180, 56,680, , Illustration 28: Mr. X joined ABC Ltd. on 01.07.2014 in the pay scale of 21,000 – 1,200 – 28,200 – 1,400 –, 39,400 – 1,600 – 49,000. The employer has allowed him 3 increments in advance at the time of joining. The, employee’s salary is due on the 1st of next month., Employee was allowed dearness allowance @ `7,000 p.m., during the previous year 2020-21 and @ `9,000, p.m. in 2021-22., Compute Tax Liability for the Assessment Year 2022-23., Solution:, `, Computation of income under the head Salary, Basic Pay [(32,400 x 4) + (33,800 x 8)], 4,00,000, Working Note:, `, 01.07.2014 – 30.06.2015 =, 24,600 p.m., 01.07.2015 – 30.06.2016 =, 25,800 p.m., 01.07.2016 – 30.06.2017 =, 27,000 p.m., 01.07.2017 – 30.06.2018 =, 28,200 p.m., 01.07.2018 – 30.06.2019 =, 29,600 p.m., 01.07.2019 – 30.06.2020 =, 31,000 p.m., 01.07.2020 – 30.06.2021 =, 32,400 p.m., 01.07.2021 – 30.06.2022 =, 33,800 p.m., Dearness allowance [(7,000 x 1) + (9,000 x 11)], 1,06,000, Gross Salary, 5,06,000, Less: Standard Deduction u/s 16 (ia), (50,000)
Page 825 :
Income Under The Head Salary, Income under the head Salary, Gross Total Income, Less: Deduction u/s 80C to 80U, Total Income, Computation of Tax Liability, Tax on `4,56,000 at slab rate, Less: Rebate u/s 87A, Tax Liability, , 306, 4,56,000, 4,56,000, Nil, 4,56,000, 10,300, (10,300), Nil, , Taxability of Perquisites, • Rent Free Accommodation Sec. 17(2)(i) Rule 3(1), • Accommodation at concessional rent Sec. 17(2)(ii) Rule 3(1), • Motor Car facility Sec. 17(2)(iii) Rule 3(2), • Gardener/Watchmen/Sweeper or any other servant Sec.17(2)(iii)Rule 3(3), • Gas/Electricity/Water Sec. 17(2)(iii) Rule 3(4), • Education Facility Sec. 17(2)(iii) Rule 3(5), • Transport facility Sec. 17(2)(iii) Rule 3(6), • Payment by the employer on behalf of the employee Sec. 17(2)(iv), • Payment of insurance premium on behalf of the employee Sec. 17(2)(v), • Specified securities or sweat equity shares allotted/ transferred to employees by an employer Sec., 17(2)(vi), • Any contribution to an approved superannuation fund / recognized provident fund / national pension, scheme by the employer in respect of an employee and annual accretion Sec. 17(2)(vii)/17(2)(viia), • Any other fringe benefit Sec. 17(2)(viii), - Loan to the employee either at concessional rate or free of interest Rule 3(7)(i), - Expenditure in connection with travelling, touring or accommodation to the employee Rule 3(7)(ii), - Free refreshment or foods to the employees Rule 3(7)(iii), - Any gift, voucher or token Rule 3(7)(iv), - Expenses on credit cards Rule 3(7)(v), - Club Membership and expenses incurred in a club Rule 3(7)(vi), - Use of moveable assets Rule 3(7)(vii), - Transfer of any moveable asset Rule 3(7)(viii), - Any other benefit Rule 3(7)(ix), • Medical Facility Proviso to Sec. 17(2), • Leave travel Concession Sec. 10(5) Rule 2B, Meaning of Perquisite, Perquisite means facilities or perks given by employer and are as given below:, Question 21 [V. Imp.]: Discuss the taxability of rent free accommodation given to an employee by his, employer?, Answer: Taxability of rent free accommodation given to an employee by his employer Section 17(2)(i), Rule 3(1), If the employer has taken the accommodation on rent and given it to the employee free of rent, taxable, amount shall be 15% of salary or rent paid or payable by the employer whichever is less e.g. If salary is, `5,00,000 and rent paid by employer is `60,000, taxable amount shall be `60,000 but if rent paid by, employer is `1,00,000, taxable amount shall be `75,000 i.e. 15% of `5,00,000, If furniture is also provided alongwith house, If employer has also provided furniture (including T.V., refrigerators, other household appliances, air, conditioning plant or equipment), Perquisite value shall be 10% p.a. of original cost of furniture and not, WDV. If the employer has taken furniture on rent, rent paid by employer shall be considered to be perquisite, value e.g. Mr. X is employed in ABC Ltd. and the employer has provided him facility of air-conditioner for, 3 months and rent paid by employer is `3,000, in this case taxable amount shall be `3,000.
Page 826 :
Income Under The Head Salary, , 307, , If the accommodation is owned by the employer, taxable amount shall be as given below:, (i) If the population is upto 10 lakhs, 7.5% of rent free accommodation salary, (ii) If the population is more than 10 lakhs but upto 25 lakhs, 10% of rent free accommodation salary, (iii) If the population is more than 25 lakhs, 15% of rent free accommodation salary, Salary shall have the same meaning as given above, Example, Compute perquisite value in the following situations:, (i) Mr. X is employed in ABC Ltd. getting basic pay `50,000 p.m. and the employer has provided him a rent, free accommodation which is owned by the employer himself at a place with population of 2,00,000., (ii) Mr. X is employed in ABC Ltd. getting basic pay `50,000 p.m. and the employer has provided him a, rent free accommodation which is owned by the employer himself at a place with population of 10,00,000., (iii) Mr. X is employed in ABC Ltd. getting basic pay `50,000 p.m. and the employer has provided him a, rent free accommodation which is owned by the employer himself at a place with population of 22,00,000., (iv) Mr. X is employed in ABC Ltd. getting basic pay `50,000 p.m. and the employer has provided him a, rent free accommodation which is owned by the employer himself at a place with population of 32,00,000, (accommodation was provided only for three months)., Solution:, (i) Computation of Perquisite value, 7.5% of rent free accommodation salary, Rent free accommodation salary = 50,000 x 12 = 6,00,000, 7.5% of `6,00,000 = 45,000, Perquisite value of rent free accommodation = `45,000, (ii) Computation of Perquisite value, 7.5% of rent free accommodation salary, Rent free accommodation salary = 50,000 x 12 = 6,00,000, 7.5% of `6,00,000 = 45,000, Perquisite value of rent free accommodation = `45,000, (iii) Computation of Perquisite value, 10% of rent free accommodation salary, Rent free accommodation salary = 50,000 x 12 = 6,00,000, 10% of `6,00,000 = 60,000, Perquisite value of rent free accommodation = `60,000, (iv) Computation of Perquisite value, 15% of rent free accommodation salary, Rent free accommodation salary = 50,000 x 3 = 1,50,000, 15% of `1,50,000 = 22,500, Perquisite value of rent free accommodation = `22,500, Meaning of Salary, Rent free accommodation salary shall include:, (i) Basic pay, (ii) Dearness Allowance/Dearness Pay. If it forms part of salary for retirement benefits as per service, agreement., (iii) Taxable portion of all allowances., (iv) Bonus /Commission /Fees etc., (v) Leave salary (when the employee is in employment), It will not include, (i) Taxable portion of perquisites whether monetary or non-monetary, (ii) Taxable portion of provident fund, (iii) Any payment after retirement like gratuity/ commuted pension or provident fund etc., (iv) Arrear of salary or advance salary, Salary only for the period for which rent free accommodation is provided shall be taken into consideration., Example, If rent free accommodation is provided from 01.05.2021 to 31.12.2021, Salary only for this period shall, taken into consideration.
Page 827 :
Income Under The Head Salary, , 308, , Meaning of Monetary and Non- Monetary Perquisites, If any facility has been arranged by the employer for the employee and payments are also made by the, employer, it will be called Non - monetary perquisite, eg. If employer has made arrangements of one servant, for the employee and also payment is being made by the employer, it will be called non-monetary perquisite., If facility has been taken by the employee himself and employer has made the payments or employer has, given reimbursement to the employee, it will be called monetary perquisite u/s 17(2)(iv). Similarly if, employer has paid any amount on behalf of the employee, it will be called monetary perquisite, eg., Employer paid professional tax on behalf of the employee, it will be called monetary perquisite and is, covered u/s 17 (2) (iv)., Illustration 29: Mr. X employed in ABC Ltd. and getting basic pay `20,000 p.m., dearness allowance, `10,000 p.m. and 50% of DA forms part of salary. Employer has paid bonus `1,000 p.m. commission, `2,000 p.m. children education allowance `150 p.m. per child for 3 children and hostel allowance `500 p.m., for one child and entertainment allowance `500 p.m., transport allowance `1,800 p.m. Employer has paid, professional tax `200 p.m. on behalf of the employee., Employer has provided him club facility and has paid membership fee `1,000 p.m., Employer has provided him rent free accommodation for which rent paid by employer is `11,000 p.m., Compute his income and tax liability A.Y. 2022-23., Solution:, `, Basic Salary (20,000 x 12), 2,40,000.00, Dearness Allowance (10,000 x 12), 1,20,000.00, Bonus (1,000 x 12), 12,000.00, Commission (2,000 x 12), 24,000.00, Children Education Allowance {Sec 10(14), Rule 2BB}, 3,000.00, Working Note:, `, Received = `150 x 3 x 12 =, 5,400, Exempt = `100 x 2 x 12 =, (2,400), Taxable, =, 3,000, Hostel Allowance {Sec 10(14), Rule 2BB}, 2,400.00, Working Note:, `, Received = 500 x 1 x 12 =, 6,000, Exempt = 300 x 1 x 12 =, (3,600), Taxable, =, 2,400, , Entertainment Allowance (500 x 12), Transport Allowance, Professional Tax (200 x 12), Club Facility {Sec 17(2)(viii) Rule 3(7)(vi)}, (1,000 x 12), Rent free Accommodation {Sec 17(2)(i), Rule 3(1)}, Working Note:, 15% of rent free accommodation salary or rent paid whichever is less, Rent free accommodation Salary, = Basic Pay + Dearness Allowance (forming part of salary) + Bonus +, Commission + Children Education allowance + Hostel Allowance +, Entertainment Allowance + Transport Allowance, = 2,40,000 + 60,000 + 12,000 + 24,000 + 3,000 + 2,400 + 6,000 + 21,600, = `3,69,000, 15% of rent free accommodation Salary = `55,350, Rent Paid = `11,000 x 12 = `1,32,000, Perquisite value of unfurnished house = `55,350, , 6,000.00, 21,600.00, 2,400.00, 12,000.00, 55,350.00
Page 828 :
Income Under The Head Salary, Gross Salary, Less: Standard Deduction u/s 16(ia), Less: 16(iii) Professional Tax, Income under the head Salary, Gross Total Income, Less: Deduction u/s 80C to 80U, Total Income, Computation of Tax Liability, Tax on `4,46,350 at slab rate, Less: Rebate u/s 87A, Tax Liability, , 309, 4,98,750.00, (50,000.00), (2,400.00), 4,46,350.00, 4,46,350.00, Nil, 4,46,350.00, 9,817.50, (9,817.50), Nil, , Government Employees: In case Central or State Government provides the accommodation to their, employees, perquisite value shall be the licence fee determined as per Government rules., If furniture is also provided alongwith house, If employer has also provided furniture (including T.V., refrigerators, other household appliances, air, conditioning plant or equipment), Perquisite value shall be 10% p.a. of original cost of furniture and not, WDV. If the employer has taken furniture on rent, rent paid by employer shall be considered to be perquisite, value e.g. Mr. X is employed in Central Govt. and the employer has provided him facility of air-conditioner, for 3 months and rent paid by employer is `3,000, in this case taxable amount shall be `3,000. (taxability of, furniture in case of other employee is also the same)., Illustration 30: Mr. X is employed in Central Government getting basic pay `73,000 p.m. Employer has, provided him rent free accommodation and the rent determined as per Government rules is `6,000 p.m. The, employer has provided him furniture with actual cost `1,00,000 and written down value `65,000. The, employer has provided one air-conditioner also during April and May’ 2021. Rent paid by the employer for, the air-conditioner is `1,000 p.m., Compute employee’s Tax Liability for the Assessment Year 2022-23., Solution:, `, Basic Pay (73,000 x 12), 8,76,000, Rent free accommodation {Sec 17(2)(i) Rule 3(1)}, 84,000, Working Note:, `, Perquisite value of unfurnished house (6,000 x 12), 72,000, Add: 10% of cost of furniture (1,00,000 x 10%), 10,000, Add: Rent of air-conditioner (1,000 x 2), 2,000, Perquisite value of furnished house, 84,000, Gross Salary, 9,60,000, Less: Standard Deduction u/s 16(ia), (50,000), Income under the head Salary, 9,10,000, Gross Total Income, 9,10,000, Less: Deduction u/s 80C to 80U, Nil, Total Income, 9,10,000, Computation of Tax Liability, Tax on `9,10,000 at slab rate, 94,500, Add: HEC @ 4%, 3,780, Tax Liability, 98,280, If accommodation has been given to the employees from a particular date but it is occupied by the employee, from some other date, in such cases perquisite value shall be computed from the date when accommodation, was occupied by the employee., Accommodation provided at two places, If any employee has been transferred and employer has provided him accommodation at the new place also,
Page 829 :
Income Under The Head Salary, , 310, , in such cases only one of the accommodation shall be taxable having lower perquisite value but only for a, period of 90 days (three months) and thereafter both of the accommodations shall be taxable., Example, Mr. X is employed in ABC Ltd. and is getting a basic pay of `30,000 p.m., dearness allowance `10,000 p.m., Employer has provided him a rent free accommodation at a place with population of 4,00,000 which is, owned by the employer himself. The employee was transferred to some other place having population, 13,00,000 w.e.f. 01.11.2021 and the employer has provided him accommodation at the new place also and, for which rent paid by the employer is `2,000 p.m. Compute his Tax Liability for Assessment Year 2022-23., Solution:, `, Basic Pay (30,000 x 12), 3,60,000.00, Dearness Allowance (10,000 x 12), 1,20,000.00, Rent Free Accommodation {Sec 17(2)(i) Rule 3(1)}, 30,250.00, Working Note:, `, From 01.04.2021 to 31.10.2021, 7.5% of rent free accommodation salary, Rent free accommodation salary, 2,10,000, 7.5% of rent free accommodation salary, 15,750, Perquisite value of rent free accommodation, 15,750, Computation of perquisite value for rent free accommodation in old, place, From 01.11.2021 to 31.01.2022, 7.5% of rent free accommodation salary, Rent free accommodation salary, 90,000, 7.5% of rent free accommodation salary, 6,750, (a) Perquisite value of rent free accommodation, 6,750, Computation of perquisite value for rent free accommodation in new, place, From 01.11.2021 to 31.01.2022, 15% of rent free accommodation salary or rent paid whichever is less, Rent free accommodation salary, 90,000, 15% of rent free accommodation salary, 13,500, Rent paid (2,000 x 3), 6,000, (b) Perquisite value of rent free accommodation, 6,000, Perquisite value of rent free accommodation from 01.11.2021 to, 31.01.2022, = Least of (a) or (b), 6,000, Computation of perquisite value for rent free accommodation in old, place, From 01.02.2022 to 31.03.2022, 7.5% of rent free accommodation salary, Rent free accommodation salary, 60,000, 7.5% of rent free accommodation salary, 4,500, Computation of perquisite value for rent free accommodation in new, place, From 01.02.2022 to 31.03.2022, 15% of rent free accommodation salary or rent paid whichever is less, Rent free accommodation salary, 60,000, 15% of rent free accommodation salary, 9,000, Rent paid (2,000 x 2), 4,000, Perquisite value of rent free accommodation, 4,000, Total taxable amount = 15,750 + 6,000 + 4,500 + 4,000 = 30,250, Gross Salary, , 5,10,250.00
Page 830 :
Income Under The Head Salary, Less: Standard Deduction u/s 16(ia), Income under the head Salary, Gross Total Income, Less: Deduction u/s 80C to 80U, Total Income, Computation of Tax Liability, Tax on `4,60,250 at slab rate, Less: Rebate u/s 87A, Tax Liability, , 311, (50,000.00), 4,60,250.00, 4,60,250.00, Nil, 4,60,250.00, 10,512.50, (10,512.50), Nil, , Accommodation provided in a hotel, Where accommodation is provided by the employer in a hotel except where the employee is provided such, accommodation for a period not exceeding in aggregate 15 days on the transfer from one place to another,, in such a case perquisite value shall be 24% of salary or actual expenditure incurred whichever is less., Example, Mr. X is employed in ABC Ltd. and is getting basic pay of `50,000 p.m. and the employer has provided him, accommodation in a hotel and expenditure incurred during the year is `1,60,000, in this case taxable amount, shall be `1,44,000., Illustration 31: Mr. X is employed in ABC Ltd. getting basic pay `37,000 p.m., dearness allowance, `32,000 p.m. The employer has provided him rent free accommodation at a place with population of 13, lakhs and the rent paid by the employer is `10,000 p.m. The employer has provided him furniture with, original cost `1,50,000. However, the employer has discontinued the facility of rent free accommodation, and furniture both w.e.f. 1st March, 2022. He has paid him house rent allowance `7,000 p.m. The employee, has shifted in his own house w.e.f. 01.03.2022., Compute Tax Liability for the Assessment Year 2022-23., (b) Presume in the above illustration the accommodation is owned by the employer., Solution:, `, Basic Pay (37,000 x 12), 4,44,000.00, Dearness allowance (32,000 x 12), 3,84,000.00, Rent free accommodation {Sec 17(2)(i) Rule 3(1)}, 74,800.00, Working Note:, `, 15% of rent free accommodation salary or rent paid whichever is less., Rent free accommodation Salary (37,000 x 11), 4,07,000, 15% of rent free accommodation salary, 61,050, Rent Paid = 10,000 x 11, 1,10,000, Value of unfurnished house, 61,050, Add: 10% of cost of furniture (1,50,000 x 10% x 11/12), 13,750, Perquisite value of furnished house, 74,800, House rent allowance {Sec 10(13A) Rule 2A}, 7,000.00, Working Note:, Least of the following is exempt:, 1. Nil, 2. 40% of retirement benefit salary = `14,800, (Retirement benefit salary = `37,000), 3. `7,000, Received = `7,000, Exempt =, Nil, Taxable = `7,000, Gross Salary, 9,09,800.00, Less: Standard Deduction u/s 16(ia), (50,000.00), Income under the head Salary, 8,59,800.00, Gross Total Income, 8,59,800.00
Page 831 :
Income Under The Head Salary, , 312, , Less: Deduction u/s 80C to 80U, Total Income, Computation of Tax Liability, Tax on `8,59,800 at slab rate, Add: HEC @ 4%, Tax Liability, Rounded off u/s 288B, Solution (b):, Basic Pay (37,000 x 12), Dearness allowance (32,000 x 12), Rent free accommodation {Sec 17(2)(i) Rule 3(1)}, Working Note:, 10% of rent free accommodation salary, Rent free accommodation Salary (37,000 x 11), 10% of rent free accommodation salary, Value of unfurnished house, Add: 10% of cost of furniture (1,50,000 x 10% x 11/12), Perquisite value of furnished house, House rent allowance {Sec 10(13A) Rule 2A}, Working Note:, Least of the following is exempt:, 1. Nil, 2. 40% of retirement benefit salary = `14,800, (Retirement benefit salary = `37,000), 3. `7,000, Received = `7,000, Exempt =, Nil, Taxable = `7,000, Gross Salary, Less: Standard Deduction u/s 16(ia), Income under the head Salary, Gross Total Income, Less: Deduction u/s 80C to 80U, Total Income, Computation of Tax Liability, Tax on `8,39,450 at slab rate, Add: HEC @ 4%, Tax Liability, Rounded off u/s 288B, , Nil, 8,59,800.00, 84,460.00, 3,378.40, 87,838.40, 87,840.00, `, 4,44,000.00, 3,84,000.00, 54,450.00, `, , 4,07,000, 40,700, 40,700, 13,750, 54,450, 7,000.00, , 8,89,450.00, (50,000.00), 8,39,450.00, 8,39,450.00, Nil, 8,39,450.00, 80,390.00, 3,215.60, 83,605.60, 83,610.00, , Question 22: Explain taxability of accommodation at concessional rent., Answer: Accommodation at concessional rent Section 17(2)(ii) Rule 3(1), If the employer has provided accommodation to the employee and has recovered some amount from the, employee for the accommodation, in such cases perquisite value shall be computed in the similar manner as, in case of rent free accommodation however amount recovered from the employee shall be deducted and, only balance amount shall be taxable., Illustration 32: Compute tax liability in the following situations:, Name of the Employee, Mr. A, Mr. B, Mr. C, Mr. D, Basic Pay, 25,000 p.m., 25,000 p.m., 25,000 p.m., 25,000 p.m., Accommodation provided by Owned by the Owned by the Owned by the Hired by the, the employer, employer, employer, employer, employer
Page 832 :
Income Under The Head Salary, Rent paid by the employer, Nil, Nil, Population of the place, 10 lakhs, 25 lakhs, Rent recovered from the 500 p.m., 500 p.m., employee, Solution:, Computation of Tax Liability, Situation 1 (Mr. A):, Basic Salary (25,000 x 12), Accommodation at concessional rent {Sec 17(2)(ii) Rule 3(1)}, Working Note:, Rent free accommodation Salary, 7.5% of rent free accommodation salary, Value of unfurnished house, Less: Amount recovered from the employee (500 x 12), Perquisite value of accommodation at concessional rent, Gross Salary, Less: Standard Deduction u/s 16(ia), Income under the head Salary, Tax on `2,66,500 at slab rate, Less: Rebate u/s 87A, Tax Liability, Situation 2 (Mr. B):, Basic Salary (25,000 x 12), Accommodation at concessional rent {Sec 17(2)(ii) Rule 3(1)}, Working Note:, Rent free accommodation Salary, 10% of rent free accommodation salary, Value of unfurnished house, Less: Amount recovered from the employee (500 x 12), Perquisite value of accommodation at concessional rent, Gross Salary, Less: Standard Deduction u/s 16(ia), Income under the head Salary, Tax on `2,74,000 at slab rate, Less: Rebate u/s 87A, Tax Liability, Situation 3 (Mr. C):, Basic Salary (25,000 x 12), Accommodation at concessional rent {Sec 17(2)(ii) Rule 3(1)}, Working Note:, Rent free accommodation Salary, 15% of rent free accommodation salary, Value of unfurnished house, Less: Amount recovered from the employee (500 x 12), Perquisite value of accommodation at concessional rent, Gross Salary, Less: Standard Deduction u/s 16(ia), Income under the head Salary, Tax on `2,89,000 at slab rate, Less: Rebate u/s 87A, Tax Liability, , 313, Nil, 35 lakhs, 500 p.m., , 4,000 p.m., 10 lakhs, 500 p.m., , `, 3,00,000.00, 16,500.00, `, 3,00,000, 22,500, 22,500, (6,000), 16,500, , 3,16,500.00, (50,000.00), 2,66,500.00, 825.00, (825.00), Nil, `, 3,00,000, 24,000, `, 3,00,000, 30,000, 30,000, (6,000), 24,000, , 3,24,000, (50,000), 2,74,000, 1,200, (1,200), Nil, `, 3,00,000, 39,000, `, 3,00,000, 45,000, 45,000, (6,000), 39,000, , 3,39,000, (50,000), 2,89,000, 1,950.00, (1,950.00), Nil
Page 833 :
Income Under The Head Salary, Situation 4 (Mr. D):, Basic Salary (25,000 x 12), Accommodation at concessional rent {Sec 17(2)(ii) Rule 3(1)}, Working Note:, `, Rent paid or 15% of rent free accommodation salary whichever is less, Rent free accommodation Salary, 3,00,000, 15% of rent free accommodation salary, 45,000, Rent paid (4,000 x 12), 48,000, Value of unfurnished house, 45,000, Less: Amount recovered from the employee (500 x 12), (6,000), Perquisite value of accommodation at concessional rent, 39,000, Gross Salary, Less: Standard Deduction u/s 16(ia), Income under the head Salary, Tax on `2,89,000 at slab rate, Less: Rebate u/s 87A, Tax Liability, , 314, `, 3,00,000, 39,000, , 3,39,000, (50,000), 2,89,000, 1,950.00, (1,950.00), Nil, , Illustration 33: Mr. X is a Finance Manager in ABC Ltd. The company has provided him with rent-free, unfurnished accommodation in Mumbai which is owned by the company. He gives you the following, particulars:, Basic salary, ` 6,000 p.m., Advance salary for April 2022, ` 5,000, Dearness Allowance, ` 2,000 p.m., (30% is for retirement benefits), Bonus, ` 1,500 p.m., Even though the company allotted the house to him on 01.04.2021, he occupied the same only from, 01.11.2021. Calculate the taxable value of the perquisite for A.Y. 2022-23., Solution:, Value of the rent free unfurnished accommodation, = 15% of salary for the relevant period i.e. 01.11.2021 to 31.03.2022, = 15% of [(` 6,000 × 5) + (` 2,000 × 30% × 5) + (` 1,500 × 5)], = 15% of ` 40,500 = ` 6,075., Illustration 34: Presume in the above question the employer has recovered `1,000 p.m. from the employee, in connection with the accommodation provided., Solution:, In this case, it will be called accommodation at concessional rent under section 17(2)(ii) Rule 3(1) and, amount recovered from the employee shall be deducted and balance amount shall be taxable., Value of the rent free unfurnished accommodation, = ` 6,075, Less: Rent paid by the employee (` 1,000 × 5), = ` (5,000), Perquisite value of unfurnished accommodation given at concessional rent, = ` 1,075, Question 23: Write a short note on perquisites in case of Specified Employee., Answer: Perquisite in case of Specified Employee Section 17(2)(iii), The perquisites covered under section 17(2)(iii) shall be taxable only in case of specified employees and, such perquisites are, 1. Motor car facility 17(2)(iii) Rule 3(2), 2. Gardener, Watchmen, Sweeper or Any other personal attendant [Sec. 17(2)(iii) Rule 3(3)]., 3. Gas, Electricity or Water facility [Sec. 17(2)(iii) Rule3(4)]., 4. Educational facility [Sec. 17(2)(iii) Rule 3(5)]., 5. Transport facility [Sec. 17(2)(iii) Rule 3(6)]., Specified employee means any employee who has complied with atleast one the conditions given below:
Page 834 :
Income Under The Head Salary, , 315, , (i) Any employee who is a director in a company whether full time or part time, (ii) Any employee who has a substantial interest in the company i.e. he is holding 20% or more of the voting, power of the company., (iii)Any employee whose monetary income under the head ‘Salaries’ exceeds `50,000., If any employee is not a specified employee, he will be considered to be non-specified employee and the, above perquisites shall be exempt from income tax., Monetary income means income under the head salary but it will not include perquisite value of non, monetary perquisites i.e. it will include:, (i) Basic Pay, (ii) Dearness Allowance/Dearness Pay, (iii) Bonus/commission/fees etc., (iv) Taxable portion of all allowances, (v) Monetary perquisites, (vi) Any other payment in cash like gratuity, pension, leave salary etc. but it will not include contribution of, employer to provident fund or interest of provident fund., • Any arrears of salary or advance salary shall also be taken into consideration., • Where salary is received from more than one employer during the relevant previous year, the aggregate, salary from these employers is to be taken into account in determining the above ceiling limit of ` 50,000., • Deduction under section 16(ia), 16(ii) and 16(iii) shall also be allowed., Example, Mr. X is employed in ABC Ltd. getting basic pay `4,500 p.m., dearness allowance `500 p.m. Employer has, paid children education allowance `300 p.m. for one child, entertainment allowance `35 p.m. Employer has, also paid professional tax of `100 p.m. on behalf of the employee. Employer has provided him rent free, accommodation with perquisite value `35,000 p.a., The employee has received arrears of salary `4,000 and advance salary `3,000., Compute employee’s monetary income to determine whether he is the specified employee or non-specified, employee., Solution:, `, Computation of Monetary Income, Basic Pay, 54,000, (4,500 x 12), Dearness Allowance, 6,000, (500 x 12), Children Education Allowance {Sec 10(14), Rule 2BB}, 2,400, Working Note:, `, Received = `300 x 12 =, 3,600, Exempt = `100 x 12 =, (1,200), Taxable, =, 2,400, Entertainment Allowance, 420, (35 x 12), Professional Tax, 1,200, (100 x 12), Arrears of Salary {Sec 15}, 4,000, Advance Salary {Sec 15}, 3,000, Gross Salary, 71,020, Less: Standard Deduction u/s 16(ia), (50,000), Less: 16(iii) Professional Tax, (1,200), Monetary Income, 19,820, So, he is non-specified employee.
Page 835 :
Income Under The Head Salary, , 316, , Question 24: Discuss the taxability of motor car facility given to an employee by his employer?, Answer:, As per Section 17(2)(iii), Rule 3(2), if the employer has provided motor car facility for personal use, it will, be taxable and perquisite value shall be 10% per annum of actual cost plus expenses incurred by employer, but facility shall be taxable only in case of specified employees and if employer has recovered any amount, from the employee, it will be deducted and only balance amount shall be taxable. e.g. Mr. X is employed in, ABC Ltd. and getting basic pay `50,000 p.m., dearness allowance `10,000 p.m. and employer has provided, him one motor car for personal use with original cost `6,00,000 and expenditure on petrol `20,000, repairs, `11,000, driver salary `10,000 p.m., Employer has provided him rent free accommodation for which rent paid by employer is `15,000 p.m., in, this case his income and tax liability shall be:, Basic Pay (50,000 x 12), 6,00,000, Dearness Allowance (10,000 x 12), 1,20,000, RFA (Sec 17(2)(i) Rule 3(1)), RFA Salary = `6,00,000, 6,00,000 x 15% or 15,000 x 12 whichever is less, 90,000, Motor car (Sec 17(2)(iii) Rule 3(2)), 2,11,000, (6,00,000 x 10%) + 20,000 + 11,000 + 1,20,000, Gross Salary, 10,21,000, Less: Standard Deduction u/s 16(ia), (50,000), Income under the head salary, 9,71,000, Total Income, 9,71,000, Tax on `9,71,000 at slab rate, 1,06,700, Add: HEC @ 4%, 4,268, Tax Liability, 1,10,968, Rounded off u/s 288B, 1,10,970, If in the above case employer has recovered `2,000 p.m. from the employee for use of motor car, taxable, amount shall be 2,11,000 – 24,000 = 1,87,000, Car used partly for official and partly for personal purposes, If the employer has provided motor car facility for official as well as personal use, perquisite value shall be, 600 p.m. if engine capacity of motor car is upto 1.6 litres otherwise it is `900 p.m., If employer is incurring other expenses also except driver, taxable amount shall be 1,800 p.m. and 2,400, p.m. respectively. If employer has provided chauffeur (driver) also, there will be additional perquisite value, of `900 p.m. Part of the month shall be ignored while computing perquisite value. E.g. Mr. X is employed in, ABC Ltd. and employer has provided him one motor car for official/personal purpose with engine capacity, 1.6 litres and all expenses are met by employer and driver is also provided, in this case taxable amount shall, be (1,800 + 900) x 12 = 32,400 but if driver is not provided, taxable amount shall be 1,800 x 12 = 21,600., Illustration 35: Mr. X is employed in ABC Ltd. getting basic pay `24,000 p.m. and the employee has paid, professional tax `200 p.m. and the employer has provided him motor car for official as well as personal use, and its engine capacity is 1.2 litres and it is a chauffeur driven car and all expenses are met by the employer, himself., Compute Tax Liability for the Assessment Year 2022-23., Solution:, `, Basic Pay (24,000 x 12), 2,88,000, Motor Car {Sec 17(2)(iii) Rule 3(2)} [(1,800 + 900) x 12], 32,400, Gross Salary, 3,20,400, Less: Standard Deduction u/s 16(ia), (50,000), Less: 16(iii) Professional Tax, (2,400), Income under the head Salary, 2,68,000, Computation of Tax Liability, Tax on `2,68,000 at slab rate, 900, Less: Rebate u/s 87A, (900), Tax Liability, Nil
Page 836 :
Income Under The Head Salary, , 317, , More than one motor car is provided to the employee for official/personal use, If the employer has provided more than one motor car for official/personal use, in that case only one of the, motor cars shall be considered to be for official/personal use and all other motor cars shall be considered to, be for personal use and perquisite value shall be computed accordingly., Motor car for going to office and coming back to residence is exempt from income tax., Illustration 36: Mr. X is employed in ABC Ltd. getting basic pay `22,000 p.m. Employer has paid, professional tax of `75 p.m. on behalf of the employee and employee himself has paid professional tax of, `25 p.m. The employer has provided him rent free accommodation which is owned by the employer himself, and it is provided at a place with population of 5,00,000., The employer has provided him three motor cars for official as well as personal use with particulars as given, below:, I, II, III, Actual cost, 4,00,000, 3,00,000, 2,50,000, Engine capacity, 1.8 litres, 1.6 litres, 1.4 litres, Petrol expenses, 3,000, 10,000, 15,000, Repairs, 5,000, 4,000, 3,000, Driver, 4,000 p.m., 3,000 p.m., no driver, All the expenses met by the employer. Employee has income from gambling activities `11,000 and also he, has long-term capital gains `5,000. Compute his Tax Liability for the Assessment Year 2022-23., Solution:, `, Basic Pay (22,000 x 12), 2,64,000.00, Professional Tax paid by employer (75 x 12), 900.00, Rent free accommodation {Sec 17(2)(i) Rule 3(1)}, 19,800.00, Working Note:, `, 7.5% of rent free accommodation salary, Rent free accommodation salary, 2,64,000, 7.5% of rent free accommodation salary, 19,800, Perquisite value of rent free accommodation, 19,800, Motor Car {Sec 17(2)(iii) Rule 3(2)}, 1,62,600.00, Working Note:, Option I, {Presuming Car I is for official and personal purposes and Car II and Car III, for personal purposes, perquisite value shall be:, Car I = (`2,400 + `900) x 12 = `39,600, Car II = `30,000 + `10,000 + `4,000 + `36,000 = `80,000, Car III = `25,000 + `15,000 + `3,000 = `43,000, Perquisite value = `1,62,600, Option II, Presuming Car II is for official and personal purpose and Car I and Car III is, for personal use, perquisite value shall be:, Car I = `40,000 + `3,000 + `5,000 + `48,000 = `96,000, Car II = (`1,800 + `900) x 12 = `32,400, Car III = `43,000, Perquisite Value = `1,71,400, Option III, Presuming Car III is for official and personal purpose and Car I and Car II is, for personal use, perquisite value shall be:, Car I = `96,000, Car II = `80,000, Car III = `1,800 x 12 = `21,600, Perquisite Value = `1,97,600, So, 1st Option is better., Perquisite value of motor car = `1,62,600
Page 837 :
Income Under The Head Salary, , 318, , Gross Salary, 4,47,300.00, Less: Standard Deduction u/s 16(ia), (50,000.00), Less: 16(iii) Professional Tax, (1,200.00), Income under the head Salary, 3,96,100.00, Income under the head Capital Gains (LTCG), 5,000.00, Income under the head Other Sources (Gambling activities), 11,000.00, Gross Total Income, 4,12,100.00, Less: Deduction u/s 80C to 80U, Nil, Total Income, 4,12,100.00, Computation of Tax Liability, Tax on LTCG `5,000 @ 20%, 1,000.00, Tax on Gambling activities income of `11,000 @ 30%, 3,300.00, Tax on normal income `3,96,100 at slab rate, 7,305.00, Tax before rebate, 11,605.00, Less: Rebate u/s 87A, (11,605.00), Tax Liability, Nil, Motor car owned by the employee and expenses are met or reimbursed by the employer Section, 17(2)(iii) Rule 3(2), Car used partly for official and partly for personal purposes – Value shall be amount paid by employer, less `1,800 p.m. where cubic capacity of engine does not exceed 1.6 litres or `2,400 p.m. where cubic, capacity of the engine exceeds 1.6 litres. It will be further reduced by `900 p.m. if the employee has, provided driver also. E.g. Mr. X is employed in ABC Ltd. and is getting basic pay `20,000 p.m. and he has, one motor car with engine capacity 1.8 litres and it is being used for official purpose also and employee is, getting `6,000 p.m. from the employer, in this case taxable amount shall be 6,000 – 2,400 = 3,600 p.m., If the assessee has maintained records, deduction shall be allowed on the basis of such records., If employee has any other automotive conveyance (two wheeler), amount to be deducted shall be `900 p.m., If the assessee has maintained records, deduction shall be allowed on the basis of such records., Personal purpose includes the benefit of employee’s household, which means spouse(s), children and their, spouses, parents, servants and dependants., Obligation of the employee met by the employer Section 17(2)(iv), If motor car is owned by employee and it is used for his personal purpose and payment or reimbursement is, given by employer, it is covered u/s 17(2)(iv) and entire amount paid or reimbursed is taxable in case of, specified as well as non specified employees., Question 25: Discuss the taxability of Gardener/ Watchman/Sweeper or Any Other Servant., Answer: Taxability of gardener/watchman/ sweeper or any other servant Section 17(2)(iii) Rule 3(3), If the employer has provided facilities of gardener/watchman / sweeper or any other servant, entire amount, shall be taxable but only in case of specified employees. Any amount recovered from the employee shall be, deducted and only balance amount shall be taxable., Obligation of the employee met by the employer Section 17(2)(iv), If the facility has been availed by the employee himself and payment or reimbursement has been given by, the employer, entire amount paid or reimbursed shall be taxable in case of specified as well as non-specified, employees, as per section 17(2)(iv)., Question 26: Write a note on taxability of Gas/ Electricity or Water Facility., Answer: Taxability of Gas/Electricity or Water Facility Section 17(2)(iii) Rule 3(4), If the employer has provided facilities of gas, electricity or water, it will be taxable but only in case of, specified employee and if the employer has his own business, perquisite value shall be the manufacturing, cost to the employer. E.g. Mr. X is employed in Bisleri and the company has provided him free water, facility for which manufacturing cost of the company is `1,000 and its market value is `1,100, in this case,, perquisite value shall be `1,000., If any amount has been recovered from the employee, it will be deducted and only balance amount shall be, taxable.
Page 838 :
Income Under The Head Salary, , 319, , Obligation of the employee met by the employer Section 17(2)(iv), If the facility has been availed by the employee himself and payment or reimbursement has been given by, the employer, entire amount paid or reimbursed shall be taxable in case of specified as well as non-specified, employees, as per section 17(2)(iv)., Question 27: [V. Imp.] Write a note on taxability of educational facility., Answer: Taxability of Educational Facility Section 17(2)(iii) Rule 3(5), • If the employer has provided free education or training facility to the employee, there is no perquisite, value., • If education facility is provided to the children of the employee, it is exempt to the extent of `1,000, p.m. per child. (irrespective of the number of children), • If education facility is provided in employer’s own institution, Still it is taxable however normal, exemption of `1,000 p.m. per child shall be allowed. Value for this purpose shall be the cost of similar, type of education in a similar type of institution in the same locality., • If any amount has been recovered from the employee, it will be deducted and only the balance amount, shall be taxed., • Since the facility is covered under section 17(2)(iii), it is taxable only in case of specified employees., Note: It is controversial that amount in excess of 1,000 p.m. shall be taxable or if amount exceeds 1,000, p.m. then entire amount is taxable., Obligation of the employee met by the employer Section 17(2)(iv), If the facility has been availed by the employee for education of members of his household and payment or, reimbursement has been given by the employer, entire amount paid or reimbursed shall be taxable in case of, specified as well as non-specified employees as per section 17(2)(iv)., • If the employer has given children education allowance, it is exempt upto `100 p.m. per child for two, children., Example, Mr. X is employed in ABC Ltd. and is a specified employee. Compute perquisite value of educational, facilities in the following situations:, (i), The employer has deputed him on one day seminar on Industrial Finance and Corporate Taxation, and has paid participation fees of `3,000., (ii), The employer has made arrangements for the education of his three childrens in his own school, and has incurred `1,500 per month per child and has recovered `300 per month per child from the, employee., (iii) If the employee himself has made arrangements of the education of his three children in a public, school and the employer has reimbursed `1,500 per month per child., Solution:, (i), There is no perquisite value., (ii), Perquisite value shall be `7,200., (iii) Perquisite value shall be `54,000., Example, Compute perquisite value in the following situations:, 1. Mr. X is employed in ABC Ltd. and is getting salary of `4,000 p.m. The employer has incurred `1,500, p.m. on the education of his one son, perquisite value shall be __________., 2. Mr. X is employed in ABC Ltd. and is getting a salary of `7,600 p.m. and the employer has incurred, `1,500 p.m. on the education of his one adopted son, perquisite value shall be ___________., 3. Mr. X is employed in ABC Ltd. and is getting a salary of `10,000 p.m. and employer has spent `500 p.m., on the education of his daughter in law, perquisite value shall be __________., 4. Mr. X is employed in ABC Ltd. and is getting a salary of `4,000 p.m. and he has incurred `700 p.m. on, the education of his one son, in this case perquisite value shall be ___________.
Page 839 :
Income Under The Head Salary, , 320, , Solution:, 1. Nil, 2. Nil, 3. `6,000, 4. Nil, Question 28: Write a note on taxability of Transport Facility., Answer: Transport Facilities Section 17(2)(iii), Rule 3(6), If the employer is engaged in the business of carriage of goods or passengers and the employer has, provided facilities of free transport to the employee or to the members of his household, perquisite value, shall be the amount charged by the employer from other person for similar facility., If the employer has recovered any amount from the employee for such facility, it will be deducted and only, balance amount shall be taxable., The facility is exempt in case of employees of Airlines and Railways., Since the facility is covered u/s 17(2)(iii), it is taxable only in case of specified employees., Question 29: Write a note on obligation of the employee met by employer., Answer: Obligation of the employee met by employer, Section 17(2)(iv), Any sum paid by the employer in respect of any obligation which, but for such payment, would have been, payable by the assessee shall be taxable in the hands of the employee and shall be called monetary, perquisite., If the employer has paid any amount on behalf of the employee or has given any reimbursement to the, employee i.e. obligation of the employee has been met by the employer, it will also be called monetary, perquisites as per section 17(2)(iv) and entire amount paid or reimbursed by the employer shall be, chargeable to tax., Example, Mr. X is employed in ABC Ltd. and he has taken one electricity connection in his name and the electricity, bill amounting to ` 5,000 has been paid by the employer on behalf of the employee, it will be called, monetary perquisite and the amount so paid shall be added to the gross salary of the employee., Similarly, if the employee has paid the bill but subsequently the employer has reimbursed the amount to, him, it will also be called monetary perquisite., Question 30: Explain taxability of payments of insurance premium by the employer on behalf of the, employee., Answer: Taxability of payments of insurance premium by the employer on behalf of the employee, Section 17(2)(v), If any employee has taken any policy in his name but premium has been paid by employer, the premium so, paid shall be taxable however premium paid for personal accident policy, staff group insurance scheme shall, be exempt., Question 31: Explain taxability of issue of shares or securities by employer to employee., Answer: As per section 17(2)(vi), if the employer has issued shares or securities to the employees,, perquisite value shall be market value of such shares or securities and if employer has recovered some, amount from the employee, it will be deducted and only difference amount shall be taxed. E.g. Mr. X is, employed in ABC Ltd. and employer has issued 100 equity shares to the employee free of cost on, 01.07.2021 and market value is `150 per share, in this case, taxable amount shall be 100 x 150 = `15,000, and if the shares have been sold by the employee, as per section 49(2AA), cost of acquisition shall be the, market value which was taken into consideration under the head salary i.e. `15,000., If the employee sold the above shares on 01.11.2021 for `270 per share, capital gains shall be computed in, the manner given below:, Full value of consideration (100 x 270), 27,000, Less: Cost of acquisition, (15,000), Short term capital gain, 12,000
Page 840 :
Income Under The Head Salary, , 321, , If shares were issued by the employer @ `40 per share, perquisite value under the head salary shall be (150 40 = 110) x 100 = 11,000 and capital gains shall be, Full value of consideration (100 x 270), 27,000, Less: Cost of acquisition, (15,000), Short term capital gain, 12,000, As per section 192(1C), A person, being an eligible start-up referred to in section 80-IAC, responsible for, paying any income to the assessee being perquisite covered under section 17(2)(vi), shall deduct tax on such, income within fourteen days—, (i) after the expiry of forty-eight months from the end of the assessment year in which shares were, allotted; or, (ii) from the date of the sale of such specified security or sweat equity share by the assessee; or, (iii) from the date of the assessee ceasing to be the employee of the person,, whichever is the earliest, on the basis of rates in force for the financial year in which the said specified, security or sweat equity share is allotted or transferred., As per Rule 3(8), the fair market value of any specified security or sweat equity share, shall be determined, in the manner given below:, In a case where, on the date of the exercising of the option, the share in the company is listed on a, recognized stock exchange, the fair market value shall be the average of the opening price and closing price, of the share on that date on the said stock exchange :, Provided that where, on the date of exercising of the option, the share is listed on more than one recognized, stock exchanges, the fair market value shall be the average of opening price and closing price of the share on, the recognised stock exchange which records the highest volume of trading in the share :, Provided further that where, on the date of exercising of the option, there is no trading in the share on any, recognized stock exchange, the fair market value shall be—, (a) the closing price of the share on any recognised stock exchange on a date closest to the date of, exercising of the option and immediately preceding such date; or, (b) the closing price of the share on a recognised stock exchange, which records the highest volume of, trading in such share, if the closing price, as on the date closest to the date of exercising of the, option and immediately preceding such date, is recorded on more than one recognized stock, exchange., In a case where, on the date of exercising of the option, the share in the company is not listed on a, recognised stock exchange, the fair market value shall be such value of the share in the company as, determined by a merchant banker on the specified date., Question 32: Explain contribution to Recognised Provident fund or approved superannuation fund or, notified pension fund by the employer in respect of an employee., Any contribution to an approved superannuation fund by the employer in respect of an employee, Section 17(2)(vii), Employers contribution to recognized provident fund to the extent of 12% of salary shall be exempt., Employers contribution to NPS shall be eligible for deduction u/s 80CCD to the extent of 10% / 14% of, salary. As per the new provision maximum exemption allowed for aggregate contribution to recognized, provident fund and NPS and approved superannuation fund shall be ₹ 7,50,000 and excess over it shall be, taxable. E.g. Mr. X employed ABC ltd. getting salary ₹ 100 lakhs and employer contributed 12% i.e. ₹ 12, lakh in this case ₹ 7.5 lakh shall be exempt and balance ₹ 4.5 lakh shall be taxable., As per section 17(2)(viia), If any amount is taxable u/s 17(2)(vii), interest dividend etc. on such amount, shall also be taxable. In the above case entire interest on ₹ 4.5 lakh shall be taxable., Example: Suppose X Pvt. Ltd. contributed ₹8,50,000 during the previous year 2021-22 towards recognised, provident fund to the account of Mr. A. Mr. A had also made an equivalent contribution. Balance in his RPF, A/c as on 1.4.2021 is ₹32,00,000. Interest accrued in his RPF during the previous year 2021-22 is ₹3,44,250.
Page 841 :
Income Under The Head Salary, , 322, , The taxable perquisite under section 17(2)(viia) for P.Y.2021-22 would be computed in the following, manner:, Take the opening balance + closing balance and take the average, Opening balance, 32,00,000, Closing balance = 32,00,000 + 17,00,000 + 3,44,250 =, 52,44,250, Average balance = 32,00,000 + 52,44,250 = 84,44,250 / 2 =, 42,22,125, Interest on average balance, 3,44,250, Interest on Employer contribution 3,44,250 / 2, 1,72,125, Interest on ₹1,00,000 = 1,72,125 / 42,22,125 x 1,00,000 =, 4,076.73, Rounded off, 4,077, Question 33 [V. Imp.]: Write a note on fringe benefits under Section 17(2)(viii) Rule 3(7)., Answer: Fringe Benefits under Section 17(2)(viii), Fringe Benefits covered under section 17(2)(viii) are asunder:, (1) Interest free or concessional loans: Section 17(2)(viii) Rule 3(7)(i), If the employer has given any loan to the employee or to the members of his household, it will be taxable, and perquisite value shall be computed on the basis of interest rate charged by State Bank and interest rate, taken by employer e.g. If employer has given a loan of `10 lakh to an employee on 01.04.2021 @ 4% p.a., and interest rate charged by State Bank is 10% p.a., perquisite value shall be 10,00,000 x (10% – 4%) =, 60,000. Further while computing perquisite value, balance outstanding at the end of each month shall be, taken into consideration i.e. there is no calculation for part of the month., Example, Mr. X is employed in ABC Ltd. and he has taken a loan of `10 lakh from employer on 20.04.2021 at a rate, of 4% p.a. but SBI rate is 10% p.a. and loan was repaid in monthly installment of `2 lakh each starting from, 10.07.2021, in this case, taxable amount shall be:, April’ 2021, 10,00,000 x 6% x 1/12, =, 5,000, May’ 2021, 10,00,000 x 6% x 1/12, =, 5,000, June’ 2021, 10,00,000 x 6% x 1/12, =, 5,000, July’ 2021, 8,00,000 x 6% x 1/12, =, 4,000, August’ 2021, 6,00,000 x 6% x 1/12, =, 3,000, September’ 2021, 4,00,000 x 6% x 1/12, =, 2,000, October’ 2021, 2,00,000 x 6% x 1/12, =, 1,000, Taxable amount, 25,000, If employer has given a petty loan, there is no perquisite value. Petty loan means one or more loan given by, the employer where aggregate amount of all such loan during a particular year is upto `20,000., Example, Mr. ‘X’ is employed in ABC Ltd. and employer has given following interest free loan as given below: (SBI, rate 10% p.a.), 01.04.2021, `10,000 x 10% x 12/12 =, 1,000.00, 01.12.2021, `8,000 x 10% x 4/12 =, 266.67, 01.03.2022, `3,000 x 10% x 1/12 =, 25.00, 1291.67, Since original amount of all the 3 loans is exceeding `20,000 hence it is taxable but if employer has not, given loan of 01.03.2022, it would have been exempt from income tax., If employer has given loan for treatment of specified disease given under rule 3A, there is no perquisite, value., Example, Mr. X is employed in ABC Ltd. and he has taken a loan of `5 Lakh on 01.07.2021 interest free from, employer SBI rate 10% p.a. for treatment of specified disease. in this case, perquisite value shall be nil, If any employee has taken advance salary, it will not be considered to be loan or advance and no perquisite, value shall be computed with regard to such advance salary., Example, (i) Mr. X is employed in ABC Ltd. and he has taken interest free loan of `3,00,000 on 10.07.2021 for
Page 842 :
Income Under The Head Salary, , 323, , personal purpose for a period of 3 years and the loan is to be repaid in monthly instalments of `10,000 and, repayment shall start with effect from 21.09.2021, in this case perquisite value for the loan shall be, computed in the manner given below (Presume SBI Rate 12%):, Amount of interest, Months, Outstanding balance at the end, (in `), (in `), July, 3,00,000, 3,00,000 x 12% x 1/12 = 3,000, August, 3,00,000, 3,00,000 x 12% x 1/12 = 3,000, September, 2,90,000, 2,90,000 x 12% x 1/12 = 2,900, October, 2,80,000, 2,80,000 x 12% x 1/12 = 2,800, November, 2,70,000, 2,70,000 x 12% x 1/12 = 2,700, December, 2,60,000, 2,60,000 x 12% x 1/12 = 2,600, January, 2,50,000, 2,50,000 x 12% x 1/12 = 2,500, February, 2,40,000, 2,40,000 x 12% x 1/12 = 2,400, March, 2,30,000, 2,30,000 x 12% x 1/12 = 2,300, Total, 24,200, So, perquisite value of interest free loan = `24,200, (ii) Mr. X is employed in ABC Ltd. and he has taken interest free loan of `3,00,000 on 10.07.2021 for, purchasing a new motor car in Delhi and the loan is to be repaid in monthly instalments of `10,000 and, repayment shall start with effect from 21.09.2021, in this case perquisite value for the loan shall be, computed in the manner given below (Presume SBI Rate 10%):, Amount of interest, Months, Outstanding balance at the end, (in `), (in `), July, 3,00,000, 3,00,000 x 10% x 1/12 = 2,500.00, August, 3,00,000, 3,00,000 x 10% x 1/12 = 2,500.00, September, 2,90,000, 2,90,000 x 10% x 1/12 = 2,416.67, October, 2,80,000, 2,80,000 x 10% x 1/12 = 2,333.33, November, 2,70,000, 2,70,000 x 10% x 1/12 = 2,250.00, December, 2,60,000, 2,60,000 x 10% x 1/12 = 2,166.67, January, 2,50,000, 2,50,000 x 10% x 1/12 = 2,083.33, February, 2,40,000, 2,40,000 x 10% x 1/12 = 2,000.00, March, 2,30,000, 2,30,000 x 10% x 1/12 = 1,916.67, Total, 20,166.67, So, perquisite value of interest free loan = `20,166.67, (iii) Mr. X is employed in ABC Ltd. and he has taken interest free loan of `13,000 on 10.07.2021 for, personal purpose and the loan is to be repaid in monthly instalments of `1,300 and repayment shall start, with effect from 21.09.2021, in this case perquisite value for the loan shall be computed in the manner given, below:, Perquisite value of interest free loan = Nil (because it is a petty loan), (2) Facility of travelling, touring, accommodation (holiday home) etc. Section 17(2)(viii) Rule 3(7)(ii), If the employer has provided facilities of travelling, touring or accommodation, it is taxable but it will not, include leave travel concession under section 10(5)Rule 2B., Perquisite value shall be actual expenditure incurred by the employer less amount recovered from the, employee., If the facility is maintained by the employer, perquisite value shall be the market value of the such facility., If the employee is on official tour and any member of his household has accompanied him and the employer, has incurred expenditure for such member, the amount so incurred shall be taxable., If the employee is on official tour and the tour was extended for personal purpose, expenditure for the, extended part of the tour shall be taxable., (3) Free food or refreshment Section 17(2)(viii) Rule 3(7)(iii), If the employer has provided free refreshments to the employees at the place of work during office hours, it, will be exempt.
Page 843 :
Income Under The Head Salary, , 324, , If the employer has provided free meals to the employees at the place of work during office hours, it will be, exempt if the value per meal is upto `50. Excess over `50 shall be taxable e.g. Mr. X is employed in the, office of Chartered Accountant and during the year he was given free lunch on many occasions and value, per lunch is `125, in this case `75 per lunch is taxable., (4) Gifts to the employees Section 17(2)(viii) Rule 3(7)(iv), Gift given by the employer in kind upto `5,000 in aggregate during a particular year is exempt and excess, over it is taxable. If the employer has given any voucher or token in lieu of which such gift may be received,, it will also be exempt in the similar manner., Gifts in cash or gifts convertible into cash i.e. gift cheques etc. shall be fully chargeable to tax., E.g. Mr. X is employed in ABC Ltd. and employer has gifted him one mobile phone of value `26,000, in, this case, taxable amount shall be `21,000 but if employer has given gift of `26,000 in cash, entire amount, shall be taxable under the head salary., Alternate View: The value of any gift or voucher received by the employee or by member of his household, upto `5,000 in aggregate during the previous year would be exempt and if aggregate amount is exceeding, `5,000 then entire amount is taxable., (5) Credit card facility Section 17(2)(viii) Rule 3(7)(v), The amount of expenses including membership fees and annual fees incurred by the employee or any, member of his household, which is charged to a credit card (including any add-on-card) provided by the, employer, or paid or reimbursed by such employer shall be taxable as reduced by the amount recovered from, the employee., However, expenses incurred wholly and exclusively for official purposes would not be treated as a, perquisite if the following conditions are fulfilled., (1) complete details in respect of such expenditure are maintained by the employer which may, inter alia,, include the date of expenditure and the nature of expenditure;, (2) the employer gives a certificate for such expenditure to the effect that the same was incurred wholly, and exclusively for the performance of official duties., (6) Club facilities Section 17(2)(viii) Rule 3(7)(vi), , The value of benefit to the employee resulting from the payment or reimbursement by the employer of any, expenditure incurred (including the amount of annual or periodical fee) in a club by him or by a member of, his household shall be taxable. The amount so determined shall be reduced by the amount recovered from, the employee. However, where the employer has obtained corporate membership of the club and the facility, is enjoyed by the employee or any member of his household, the value of perquisite shall not include the, initial fee paid for acquiring such corporate membership., Further, if such expenditure is incurred wholly and exclusively for business purposes, it would not be, taxable provided the following conditions are fulfilled:(1) complete details in respect of such expenditure are maintained by the employer which may, inter alia,, include the date of expenditure, the nature of expenditure and its business expediency;, (2) the employer gives a certificate for such expenditure to the effect that the same was incurred wholly and, exclusively for the performance of official duties., There would be no perquisite for use of health club, sports and similar facilities provided uniformly to all, employees by the employer, Facilities of health club, sports and similar facilities shall be exempt., Example, ABC Ltd. has allowed membership of health club and sports club to their employees, in this case, expenditure incurred by the company shall not be taxable in the hands of employees., (7) Use of employer’s assets by the employees Section 17(2)(viii) Rule 3(7)(vii)
Page 844 :
Income Under The Head Salary, , 325, , If the employer has given any movable asset to the employee or to the members of his household for, personal use, in such cases it will be taxable and perquisite value shall be 10% p.a. of actual cost of such, asset less any amount recovered from the employee e.g. Mr. X is employed in ABC Ltd. and employer has, given him one video camera with original cost `2,00,000 for 25 days, in this case taxable amount shall be, 2,00,000 x 10% x 25/365 = 1,369.86. If employer has recovered `1,000 for such use, taxable amount shall, be 1,369.86 – 1,000 = 369.86, If the employer has given any laptop or computer to the employee for personal use, it will not be taxable., If the employer has provided a two wheeler to the employee, it will be taxable in the similar manner., (8) Sale of assets by employer to the employee Section 17(2)(viii) Rule 3(7)(viii), If employer has sold any movable asset to the employee, taxable amount shall be actual cost of such asset, less 10% of actual cost per completed year of use of asset by the employer less amount paid by employee. In, case of motor car, amount to be deducted shall be 20% of w.d.v. instead of 10% of actual cost and in case of, computers, laptops, data storage devices, digital diaries and printers, it will be 50% of w.d.v., Example, Asset, Furniture, Microwave Motor car, Washing, Computer, oven, machine, Original cost, 75,000, 25,000, 2,40,000, 20,000, 55,000, Date of purchase by the employer, 07.03.2017 01.06.2019, 10.07.2018 01.10.2011 01.01.2019, Date of putting to use, 31.03.2017 01.06.2019, 11.07.2018 01.11.2011 10.01.2019, Date of sale of asset to the, 01.07.2021 01.04.2021, 01.07.2021 31.12.2021 09.01.2022, employee, Payment made by the employee, 25,000, Gift to the, 95,000, 1,000, 30,000, employee, Solution:, `, Computation of perquisite value of Furniture, Cost of the furniture, 75,000, Less: Depreciation on straight line method @ 10% from 31.03.2017 to 30.03.2018, (7,500), Less: Depreciation on straight line method @ 10% from 31.03.2018 to 30.03.2019, (7,500), Less: Depreciation on straight line method @ 10% from 31.03.2019 to 30.03.2020, (7,500), Less: Depreciation on straight line method @ 10% from 31.03.2020 to 30.03.2021, (7,500), Written down value, 45,000, Less: Amount paid by the assessee, (25,000), Perquisite value of Furniture, 20,000, Computation of perquisite value of Microwave oven, Cost of Microwave oven, 25,000, Less: Depreciation on straight line method @ 10% from 01.06.2019 to 31.05.2020, (2,500), Written down value, 22,500, Less: Gift to the employee, (5,000), Perquisite value of microwave oven, 17,500, Computation of perquisite value of Motor car, Cost of the motor, 2,40,000, Less: Depreciation on reducing balance method @ 20% from 11.07.2018 to 10.07.2019, (48,000), Written down value, 1,92,000, Less: Depreciation on reducing balance method @ 20% from 11.07.2019 to 10.07.2020, (38,400), Written down value, 1,53,600, Less: Amount paid by the assessee, (95,000), Perquisite value of motor car, 58,600, Computation of perquisite value of Washing Machine, Cost of the washing machine, 20,000, Less: Depreciation on straight line method @ 10% from 01.11.2011 to 31.10.2012, (2,000), Written down value, 18,000, Less: Depreciation on straight line method @ 10% from 01.11.2012 to 31.10.2013, (2,000), Written down value, 16,000
Page 845 :
Income Under The Head Salary, , 326, , Less: Depreciation on straight line method @ 10% from 01.11.2013 to 31.10.2014, (2,000), Written down value, 14,000, Less: Depreciation on straight line method @ 10% from 01.11.2014 to 31.10.2015, (2,000), Written down value, 12,000, Less: Depreciation on straight line method @ 10% from 01.11.2015 to 31.10.2016, (2,000), Written down value, 10,000, Less: Depreciation on straight line method @ 10% from 01.11.2016 to 31.10.2017, (2,000), Written down value, 8,000, Less: Depreciation on straight line method @ 10% from 01.11.2017 to 31.10.2018, (2,000), Written down value, 6,000, Less: Depreciation on straight line method @ 10% from 01.11.2018 to 31.10.2019, (2,000), Written down value, 4,000, Less: Depreciation on straight line method @ 10% from 01.11.2019 to 31.10.2020, (2,000), Written down value, 2,000, Less: Depreciation on straight line method @ 10% from 01.11.2020 to 31.10.2021, (2,000), Written down value, Nil, Less: Amount paid by the assessee, (1,000), Perquisite value of washing machine, Nil, Computation of perquisite value of Computer, Cost of the Computer, 55,000, Less: Depreciation on reducing balance method @ 50% from 10.01.2019 to 09.01.2020, (27,500), Written down value, 27,500, Less: Depreciation on reducing balance method @ 50% from 10.01.2020 to 09.01.2021, (13,750), Written down value, 13,750, Less: Depreciation on reducing balance method @ 50% from 10.01.2021 to 09.01.2022, (6,875), Written down value, 6,875, Less: Amount paid by the assessee, (30,000), Perquisite value of computer, Nil, (9) Any other benefit Section 17(2)(viii) Rule 3(7)(ix), The value of any other benefit provided by the employer to the employee is chargeable to tax and its value, shall be determined on the basis of cost to the employer., If the employer has provided telephone facility including the mobile phone, it will be exempt. If the facility, has been taken by the employee himself and the employer has made payment or reimbursement, still it is, exempt from tax., If the employer has given telephone allowance, it will be chargeable to tax., Illustration 37: X Ltd. provided the following perquisites to its employee Mr. Y for the P.Y.2021-22 –, (1) Accommodation taken on lease by X Ltd. for ` 15,000 p.m. ` 5,000 p.m. is recovered from the salary of, Mr. Y., (2) Furniture, for which the hire charges paid by X Ltd. is ` 3,000 p.m. No amount is recovered from the, employee in respect of the same., (3) A Santro Car (engine capacity 1.6 litres) which is owned by X Ltd. and given to Mr. Y to be used both, for official and personal purposes. All running and maintenance expenses are fully met by the employer. He, is also provided with a chauffeur., (4) A gift voucher of ` 10,000 on his birthday., Compute the value of perquisites chargeable to tax for the A.Y.2022-23, assuming his salary for perquisite, valuation to be ` 10 lakh., Solution:, Computation of the value of perquisites chargeable to tax in the hands of Mr. Y for the A.Y.2022-23, Particulars, Amount in `, (1), Value of concessional accommodation, Actual amount of lease rental paid by X Ltd., 1,80,000, 1,50,000, 15% of salary i.e., 15% of ` 10,00,000
Page 846 :
Income Under The Head Salary, Lower of the above, Less: Rent paid by Mr. Y (` 5,000 × 12), , (2), (3), , Add: Hire charges paid by X Ltd. for furniture, provided for the use of Mr. Y, Perquisite value of Santro car owned by X Ltd., And provided to Mr. Y for his personal and, official use [(` 1,800 + ` 900) × 12], Value of gift voucher (10,000 – 5,000), Value of perquisites chargeable to tax, , 327, 1,50,000, (60,000), 90,000, 36,000, , 1,26,000, 32,400, 5,000, 1,63,400, , Illustration 38: Mr. X is employed with ABC Ltd. on a monthly salary of ` 25,000 per month and an, entertainment allowance and commission of ` 1,000 p.m. each. The company provides him with the, following benefits:, 1., A company owned accommodation is provided to him in Delhi. Furniture costing ` 2,40,000 was, provided on 01.08.2021., 2., A personal loan of ` 5,00,000 on 01.07.2021 on which it charges interest @ 6.75% p.a. The entire, loan is still outstanding. (Assume SBI rate of interest to be 12.75% p.a.), 3., His son is allowed to use a motor cycle belonging to the company. The company had purchased this, motor cycle for ` 60,000 on 01.05.2018. The motor cycle was finally sold to him on 01.08.2021 for, `30,000., 4., Professional tax paid by Mr. X is ` 2,000., Compute Total income and tax liability of Mr. X for the A.Y.2022-23., Solution:, Computation of Income from Salary of Mr. X for the A.Y.2022-23, Particulars, `, `, Basic salary [` 25,000 × 12], 3,00,000, Commission [` 1,000 × 12], 12,000, 12,000, Entertainment allowance [` 1,000 × 12], Rent free accommodation Sec 17(2)(i) Rule 3(1), Working Note:, = 15% of salary for the relevant period, = 15% of (` 3,00,000 + ` 12,000 + ` 12,000) = ` 48,600, 48,600, Add : Value of furniture [` 2,40,000 × 10% p.a. for 8 months], 16,000, 64,600, Interest on personal loan Sec 17(2)(viii) Rule 3(7)(i), Working Note:, = [` 5,00,000 × (12.75% - 6.75%) for 9 months] = ` 22,500, 22,500, Use of motor cycle Sec 17(2)(viii) Rule 3(7)(vii), [` 60,000 × 10% p.a. for 4 months], 2,000, Transfer of motor cycle Sec 17(2)(viii) Rule 3(7)(viii), Working Note:, Depreciated value of the motor cycle, = Original cost – Depreciation @ 10% p.a. for 3 completed years, = ` 60,000 – (` 60,000 × 10% p.a. × 3 years) = ` 42,000., Perquisite = ` 42,000 – ` 30,000 = ` 12,000., 12,000, Gross Salary, 4,25,100, Less: Standard Deduction u/s 16(ia), (50,000), Less : Deduction under section 16(iii), Professional tax paid, (2,000), Income from Salary, 3,73,100, Tax on `3,73,100 at slab rate, 6,155.00, Less: Rebate u/s 87A, (6,155.00), Tax Liability, Nil
Page 847 :
Income Under The Head Salary, , 328, , Question 34. Write a note on Medical Facility., Answer: Medical Facility Proviso to Section 17(2), If the employer has provided medical facility to the employee or to his family members it will be exempt, provided facility is given in any hospital owned by the Government / local authority / employer himself or it, is any other private hospital approved by the Income Tax Department. E.g. Mr. X is employed in ABC Ltd., and employer has incurred `3,00,000 on his treatment in a private hospital approved by Income Tax, Department, in this case it will be exempt from income tax. If treatment was taken by the employee himself, in any such hospital (only above four hospitals) and employer has given reimbursement, in that case also it, will be exempt from income tax., , If employer has incurred expenditure for treatment at any other place like private clinic or hospital etc., it, will be taxable., If the employer has paid premium for medi-claim policy taken in the name of employee or his family, it will, be exempt from income tax., If employer has paid medical allowance, it is always taxable., “Family”, shall include, (i) The spouse and children of the individual and, (ii) The parents, brothers and sisters of the individual provided they are dependent on the individual., , , If expenditure has been incurred by the employer on the treatment of any other person like mother in, law, father in law, independent father etc., facility is taxable., , Medical facilities outside India, (i) If the employer has incurred expenditure on the treatment of the employee or any member of his family, outside India, it is exempt to the extent permitted by Reserve Bank of India., (ii) If the employer has incurred expenditure on the stay abroad of the patient including one attendant, it is, exempt to the extent permitted by Reserve Bank of India., (iii) If the employer has incurred expenditure on the travelling of the patient including one attendant, it is, exempt provided gross total income of the employee do not exceed `2,00,000 before taking into, consideration the expenditure incurred on travelling., Illustration 39: Mr. X is employed in ABC Ltd. getting basic pay `12,000 p.m. dearness allowance `2,000, p.m. medical allowance `200 p.m. entertainment allowance `300 p.m. Employer has paid professional tax of, `100 p.m. on behalf of the employee. Employer incurred `3,00,000 on his treatment outside India but RBI, permitted `2,80,000. Amount incurred on stay is `1,00,000 but permitted by RBI is `90,000. Expenditure, incurred on travelling is `1,10,000. He has income under the head house property `50,000 and deduction, allowed under section 80C to 80U is `10,000. In this case his total income shall be computed in the manner, given below:, Solution:, Basic Pay (12,000 x 12), 1,44,000, Dearness Allowance (2,000 x 12), 24,000, Medical Allowance (200 x 12), 2,400, Entertainment Allowance (300 x 12), 3,600, Professional Tax (100 x 12), 1,200, Treatment outside India (3,00,000 – 2,80,000), 20,000, Stay aboard (1,00,000 – 90,000), 10,000, Gross Salary, 2,05,200, Less: Standard Deduction u/s 16(ia), (50,000), Less: Deduction u/s 16(ii), Nil, Less: Deduction u/s 16(iii), (1,200), Income under the head Salary, 1,54,000, Income under the head house property, 50,000
Page 848 :
Income Under The Head Salary, , 329, , Gross Total Income, 2,04,000, Since gross total income is exceeding `2,00,000 hence expenditure on travelling shall be taxable and shall, be as given below:, Basic Pay (12,000 x 12), 1,44,000, Dearness Allowance (2,000 x 12), 24,000, Medical Allowance (200 x 12), 2,400, Entertainment Allowance (300 x 12), 3,600, Professional Tax (100 x 12), 1,200, Treatment outside India (3,00,000 – 2,80,000), 20,000, Stay aboard (1,00,000 – 90,000), 10,000, Travelling, 1,10,000, Gross Salary, 3,15,200, Less: Standard Deduction u/s 16(ia), (50,000), Less: Deduction u/s 16(ii), Nil, Less: Deduction u/s 16(iii), (1,200), Income under the head Salary, 2,64,000, Income under the head house property, 50,000, Gross Total Income, 3,14,000, Less: Deduction u/s 80C to 80U, (10,000), Total Income, 3,04,000, Computation of Tax Liability, Tax on `3,04,000 at slab rate, 2,700, Less: Rebate u/s 87A, (2,700), Tax Liability, Nil, , RTP NOV 2020, Mr. Neeraj, a salaried employee, furnishes the following details for the financial year 2021-22:, Particulars, Basic salary, Dearness allowance, Commission, Entertainment allowance, Medical expenses reimbursed by the employer, Profession tax (of this, 50% paid by employer), Health insurance premium paid by employer, Gift voucher given by employer on his birthday, Life insurance premium of Neeraj paid by employer, Laptop provided for use at home. Actual cost of Laptop to employer Children of the, assessee are also using the Laptop at home], Employer company owns a Maruti Suzuki Swift car (Engine cubic capacity more than 1.6, litres), which was provided to the assessee, both for official and personal use. No driver, was provided. All expenses are met by the employer, Annual credit card fees paid by employer [Credit card is not exclusively used for official, purposes; details of usage are not available], , ₹, 5,40,000, 3,60,000, 50,000, 7,500, 21,000, 4,000, 9,000, 12,000, 34,000, 30,000, , 5,000, , You are required to compute the income chargeable under the head Salaries for the assessment year 202223.
Page 849 :
Income Under The Head Salary, , 330, , Answer:, Computation of Income chargeable under the head salaries of Mr. Neeraj for A.Y. 2022-23, Particulars, ₹, Basic Salary, 5,40,000, Dearness allowance, 3,60,000, Commission, 50,000, Entertainment allowance, 7,500, Medical expenses reimbursed by the employer is fully taxable, 21,000, Professional tax paid by the employer is a taxable perquisite as per section 17(2)(iv), since it, 2,000, is an obligation of the employee which is paid by the employer, Health insurance premium of ₹9,000 paid by the employer is an exempt perquisite [Clause, Nil, (iii) of proviso to section 17(2)], Gift voucher given by employer on Mr. Neeraj birthday [entire amount is taxable since the, 12,000, perquisite value exceeds ₹ 5,000, as per Rule 3(7)(iv)], Life insurance premium of Mr. Neeraj paid by employer is a taxable perquisite as per, 34,000, section 17(2)(v), Laptop provided for use at home is an exempt perquisite as per Rule 3(7)(vii), Nil, Provision of motor car (engine cubic capacity more than 1.6 litres) owned by employer, 28,800, provided to employee, the perquisite value would be ₹28,800 [₹ 2,400 ×12] as per Rule 3(2), Annual credit card fees paid by employer is a taxable perquisite as per Rule 3(7)(v) since the, credit card is not exclusively used for official purposes and details of usage are not available, 5,000, Gross Salary, 10,60,300, Less: Deductions under section 16, - Standard Deduction as per section 16(ia), lower of gross salary and ₹ 50,000, 50,000, - Entertainment allowance (deduction not allowable since Mr. Neeraj is not a, Nil, Government employee), - Professional tax paid allowable as deduction as per section 16(iii), 4,000, Income chargeable under the head “Salaries”, 10,06,300, Note: As per Rule 3(7)(iv), the value of any gift or voucher received by the employee or by member of his, household on ceremonial occasions or otherwise from the employer shall be determined as the sum equal to, the amount of such gift. However, the value of any gift or voucher received by the employee or by member, of his household below ₹ 5,000 in aggregate during the previous year would be exempt as per the proviso to, Rule 3(7)(iv). In this case, the gift voucher of ₹ 12,000 was received by Mr. Neeraj from his employer on the, occasion of his birthday. Since the value of the gift voucher exceeds the limit of ₹ 5,000, the entire amount, of ₹ 12,000 is liable to tax as perquisite. The above solution has been worked out accordingly., , An alternate view possible is that only the sum in excess of ₹ 5,000 is taxable in view of the language of, Circular No.15/2001 dated 12.12.2001, which states that such gifts upto ₹,000 in the aggregate per annum, would be exempt, beyond which it would be taxed as a perquisite. As per this view, the value of perquisite, would be ₹7,000. Accordingly, the gross salary and net salary would be 10,55,300 and 10,01,300,, respectively, Question 35 [V. Imp.]: Write a note on Leave Travel Concession., Answer:, Leave Travel Concession, Section 10(5) Rule 2B, Sometimes the employer may permit the employee or his family member to go to any place in India and, travelling expenditure are incurred by the employer, such facility is called leave travel concession or leave, fare concession.
Page 850 :
Income Under The Head Salary, , 331, , Example, Mr. X is employed in ABC Ltd. and is posted in Delhi and the employer has allowed him to travel from, Delhi to Goa and travelling expenditure has been incurred by the employer, in this case it will be called, leave travel concession., Taxability shall be as given below:, 1. If the employee has travelled by air, exemption shall be allowed upto air economy fare of the national, carrier. E.g. Mr. X travelled from place A to place B by air business class and employer paid `12,000 per, ticket but economy class air fare is `7,000 per ticket, in this case taxable amount shall be `5,000., 2. If employee has not travelled by air, exemption shall be allowed upto first class AC fare of railways e.g., Mr. X travelled from place A to place B by railway first class AC and employer paid `6,000 per ticket, in, this case entire amount is exempt but if employee has travelled by private tourist bus and employer paid, `7,000 per ticket, taxable amount shall be `1,000 per ticket., 3. If the places are not connected by rail but there is some recognised transport system, exemption shall be, allowed upto first class or deluxe class of such recognised transport system. Recognised transport system, means govt. transport or private transport recognised by Govt., 4. If there is no rail connection and also there is no recognised transport system, exemption shall be, allowed upto first class AC fare of railways (fare shall be determined on the basis of distance between the, place travelled) E.g. Mr. X has travelled from place A to place B by a private bus (distance 50 kms) and, employer has paid ` 600. There is no rail connection and also no recognised transport. First class AC fare of, railway is `500, taxable amount shall be `100., Ceiling on number of journeys, The exemption shall be available to an individual two times in each block of four calendar years., Carry forward of leave travel concession, If the employee has not availed any leave travel concession or has availed only one leave travel concession, during a particular block, carry forward shall be allowed but only for one leave travel concession and such, LTC must be availed during very first year of the next block otherwise the LTC shall lapse., Example: An employee does not avail any LTC for the block 2016-19. He avails one LTC during 2020. In, this case, he will be eligible for exemption and two more journeys can be further availed., “Family”, shall include—, (i) the spouse and children of the individual however exemption shall be allowed maximum 2 children but, in case of multiple birth after the birth of one child, exemption is allowed for all the children e.g. Mr. X has, one son aged 10 years and twin daughters aged 5 years, in this case exemption is allowed for all the 3, children., (ii) the parents, brothers and sisters of the individual or any of them, wholly or mainly dependent on the, individual., If the employer has paid leave travel allowance and employee has not travelled to any place, entire, amount paid by the employer is taxable., The exemption is allowed only in respect of fare i.e. expenses incurred on conveyance from residence to, the railway station/airport/ bus stand etc. and back shall be taxable., Example, (i) Mr. X is employed in ABC Ltd. and the employer has allowed him to travel from Delhi to Bombay by air, (business class) and has reimbursed `12,000 per ticket but economy class air fare is `7,000 per ticket, in this, case taxable amount per ticket shall be `5,000., If in the above case the employee has travelled alongwith Mrs. X, two children and one independent brother, and the employer has reimbursed five tickets, taxable amount shall be `32,000., (ii) Mr. Y is employed in ABC Ltd. and he has travelled from place ‘A’ to place ‘B’ by a private bus, because there is no rail link and also there is no recognised transport and the employer has reimbursed him, `3,000 per ticket but first class A/C fare of the railways is `2,800 per ticket, in this case taxable amount, shall be `200 per ticket., (iii) Mr. X has travelled from Delhi to Bombay by second class railways and the employer has paid him, `5,000 per ticket being first class A/C fare. The employee has incurred `1,000 per ticket, in this case taxable, amount shall be `4,000 per ticket.
Page 851 :
Income Under The Head Salary, , 332, , Illustration 40: Mr. D went on a holiday on 25.12.2021 to Delhi with his wife and three children (one son –, age 5 years; twin daughters – age 2 years). They went by flight (economy class) and the total cost of tickets, reimbursed by his employer was ` 60,000 (` 45,000 for adults and ` 15,000 for the three minor children)., Compute the amount of LTC exempt., Solution: Since the son’s age is more than the twin daughters, Mr. D can avail exemption for all his three, children. The restriction of two children is not applicable to multiple births after one child. The holiday, being in India and the journey being performed by air (economy class), the entire reimbursement met by the, employer is fully exempt., Illustration 41: In the above illustration, will be there be any difference if among his three children the, twins were 5 years old and the son 3 years old? Discuss., Solution: Since the twins’ age is more than the son, Mr. D cannot avail for exemption for all his three, children. LTC exemption can be availed in respect of only two children., Taxable LTC, =15,000 × 1/3 = ` 5,000., LTC exempt is only ` 55,000 (i.e. ` 60,000 – ` 5,000), Illustration 42: Mrs. X, an employee of XYZ Ltd., submits the following information for the assessment, year 2022-23:, Salary: `2,56,000; City compensatory allowance: `8,000; Bonus: `10,200; Education allowance: `4,000 (for, her grand children); Income tax penalty paid by the employer: `2,000; Leave travel concession: `1,000, (expenditure incurred by the employee nil); Free residential telephone: `4,000; Free refreshment during, office hours `4,000; reimbursement of electricity bill by the employer: ` 1,060; reimbursement of gas bills:, `1,000; Professional tax paid by the employer: `300 on behalf of Mrs. X; Professional tax paid by Mrs. X:, `150., Determine the Total Income and Tax Liability of Mrs. X for the Assessment Year 2022-23., Solution:, `, Computation of Income under the head Salary, Salary, 2,56,000.00, City Compensatory Allowance, 8,000.00, Bonus, 10,200.00, Education Allowance {Sec 10(14) Rule 2BB}, 4,000.00, Income tax penalty paid by employer {Sec 17(2)(iv)}, 2,000.00, Leave Travel Concession {Sec 10(5) Rule 2B}, 1,000.00, Free Refreshment, Nil, Free Residential Telephone, Nil, Payment of electricity bills by employer {Sec 17(2)(iv)}, 1,060.00, Reimbursement of gas bills {Sec 17(2)(iv)}, 1,000.00, Professional tax paid by employer {Sec 17(2)(iv)}, 300.00, Gross Salary, 2,83,560.00, Less: Standard Deduction u/s 16(ia), (50,000.00), Less: Entertainment Allowance u/s 16(ii), Nil, Less: Professional Tax u/s 16(iii), (450.00), Income under the head Salary, 2,33,110.00, Gross Total Income, 2,33,110.00, Less: Deductions u/s 80C to 80U, Nil, Total Income, 2,33,110.00, Computation of Tax Liability, Tax on `2,33,110 at slab rate, Nil, Tax Liability, Nil, MAY-2013 (4 Marks), Mr. X went to Shrinagar on a holiday on 15.11.2021 with his wife and three children (one son – age 6 years;, twin daughters – age 3 years). They went by aeroplane (economy class) and the total cost of tickets by his
Page 852 :
Income Under The Head Salary, , 333, , employer was `58,000 (`43,000 for adults and `15,000 for the three minor children). Compute the amount, of Leave Travel Concession exempt., Will the answer be different if among his three children the twins are 6 years old and son 3 years old?, Discuss., Solution: Section 10(5) exempts the leave travel concession received by an employee from his employer for, himself and his family which includes, inter alia, his spouse and children, in connection with proceeding on, leave to any place in India. The exemption is not available to more than two surviving children of an, individual. However, this restriction shall not apply in case of multiple births on the second occasion (i.e.,, after the first child)., In the present case, Mr. X can avail exemption for all his three children since the son’s age is more than the, age of his twin daughters. The holiday being in India and the journey being performed by air (economy, class), the entire reimbursement of `58,000 towards leave travel concession met by the employer is fully, exempt under section 10(5)., However, if the twins’ age is more than the age of the son, Mr. X cannot avail exemption for all his three, children. The exemption in respect of leave travel concession under section 10(5) can be availed in respect, of only two children., The taxable leave travel concession, in this case, will be `5,000, being one-third of `15,000., The leave travel concession exempt under section 10(5), in such a case, would be ` 53,000 (`58,000 –, `5,000), Question 36: Define ‘Members of Employee’s Household’ and ‘Family’., Answer: ‘Members of employee’s Household’ and ‘Family’, “Member of household” shall include—, (a) spouse(s), (b) children and their spouses, (c) parents, (d) any person dependant on the employee, (e) servants, “Family”, in relation to an individual, means—, (i) the spouse and children of the individual and, (ii) the parents, brothers and sisters of the individual or any of them, wholly or mainly dependent on the, individual., Question 37: Explain the taxability of income tax paid by the employer on behalf of the employee in, connection with non-monetary perquisites., Answer:, Payment of income tax in connection with non-monetary perquisites Section 10(10CC), If employer has paid income tax on behalf of the employee in connection with non- monetary perquisites,, employer shall not be allowed to debit the amount so paid to the profit and loss account and also it will not, be considered to be income of the employee as per section 10(10CC)., If income tax so paid is not in connection with non-monetary perquisites, employer shall be allowed to debit, the amount to the profit and loss account and as per section 17(2)(iv), it will be considered to be income of, the employee under the head salary., Illustration 43: During the previous year 2021-22, ABC Ltd. pays `60,000 p.m. as salary to Mr. X and, provides a rent free unfurnished house (lease rent being `15,000 p.m.). ABC Ltd. has also paid income tax, of `9,000 on behalf of Mr. X in connection with rent free accommodation provided to Mr. X., Compute Tax Liability of Mr. X for the Assessment Year 2022-23. Also discuss whether income tax paid by, the company shall be considered to be income of Mr. X., Solution:, `, Salary, 7,20,000, (60,000 x 12), Rent free accommodation {Sec 17 (2)(i) Rule 3(1)}, 1,08,000
Page 853 :
Income Under The Head Salary, , 334, , Working Note:, 15% of rent free accommodation salary or rent paid whichever is less, Rent free accommodation salary = 7,20,000, 15% of rent free accommodation salary = 1,08,000, Rent paid = 15,000 x 12 = 1,80,000, Perquisite value of rent free accommodation = 1,08,000, Gross salary, 8,28,000, Less: Standard Deduction u/s 16(ia), (50,000), Income under the head salary, 7,78,000, Gross Total Income, 7,78,000, Less: Deduction u/s 80C to 80U, Nil, Total Income, 7,78,000, Computation of Tax Liability, Tax on `7,78,000 at slab rate, 68,100, Add: HEC @ 4%, 2,724, Tax Liability, 70,824, Less: Tax Paid by the company, (9,000), Tax Payable, 61,824, Rounded off u/s 288B, 61,820, Since tax has been paid by the company in connection with non-monetary perquisite, it will not be, considered to be income of Mr. X. If the company has paid income tax upto `9,832 it will not be considered, to be income of Mr. X but any excess over it shall be considered to be income of Mr. X. The tax liability, with regard to rent free accommodation shall be `9,832, as shown below:, 70,824 / 7,78,000 x 1,08,000 = 9,831.61 = 9,832, (b) Presume in the above question ABC Ltd. has paid income tax of `20,000 instead of `9,000. Tax Liability, and Tax Payable shall be computed in the manner given below:, Solution:, `, Salary, 7,20,000.00, (60,000 x 12), Rent free accommodation {Sec 17 (2)(i) Rule 3(1)}, 1,08,000.00, Working Note:, 15% of rent free accommodation salary or rent paid whichever is less, Rent free accommodation salary = 7,20,000, 15% of rent free accommodation salary = 1,08,000, Rent paid = 15,000 x 12 = 1,80,000, Perquisite value of rent free accommodation = 1,08,000, Payment of income tax on behalf of employee {Sec 17(2)(iv)} (20,000 – 9,832), 10,168.00, Gross salary, 8,38,168.00, Less: Standard Deduction u/s 16(ia), (50,000.00), Income under the head Salary, 7,88,168.00, Gross Total Income, 7,88,168.00, Less: Deduction u/s 80C to 80U, Nil, Total Income, 7,88,168.00, Rounded off u/s 288A, 7,88,170.00, Computation of Tax Liability, Tax on `7,88,170 at slab rate, 70,134.00, Add: HEC @ 4%, 2,805.36, Tax Liability, 72,939.36, Less: Tax Paid by the company, (20,000.00), Tax Payable, 52,939.36, Rounded off u/s 288B, 52,940.00
Page 854 :
Income Under The Head Salary, , 335, , Illustration 44: Mr. X, finance manager of KLM Ltd. Mumbai, furnishes the following particulars for the, financial year 2021-22:, (i) Salary `46,000 per month, (ii) Rent free accommodation owned by the company, (iii) Housing loan of `6,00,000 at the interest rate of 5% p.a. (No repayment made during the year, but the, loan is repayable in tenth year) (Presume SBI Rate 10.5% p.a.), (iv) Gifts made by the company in kind on the occasion of wedding anniversary of Mr. X `4,750, (v) A wooden table and 4 chairs were provided to Mr. X at his residence. These were purchased on, 01.05.2018 for ` 60,000 and put to use on 01.06.2018 and sold to Mr. X on 01.08.2021 for `30,000, (vi) Personal purchases through credit card provided by the company amounting to `10,000 was paid by, the company. No part of the amount was recovered from Mr. X., (vii) An ambassador car which was purchased by the company on 16.07.2018 for `2,50,000 and put to use, on the same date. It was sold to the assessee on 14.07.2021 for `80,000., Compute the Total Income of Mr. X and the tax thereon for the Assessment Year 2022-23., Solution:, `, Computation of Taxable Income of Mr. X for the Assessment Year 2022-23, Salary (46,000 x 12), 5,52,000.00, Rent Free Accommodation {Sec 17(2)(i) Rule 3(1)}, 84,800.00, Working Note:, `, 15% of rent free accommodation salary, Rent Free Accommodation salary = `5,52,000, Value of unfurnished house, 82,800, Add: 10% of cost of furniture (60,000 x 10% x 4/12), 2,000, Perquisite value of furnished house, 84,800, Perquisite of interest on loan {Sec 17(2)(viii) Rule 3(7)(i)}, 33,000.00, Working Note:, 10.5% is taxable which is to be reduced by actual rate of interest charged, i.e.[10.5%-5% = 5.5%], (6,00,000 x 5.5%) = `33,000, Gift given on the occasion of wedding anniversary, Nil, Sale of Table and Chairs {Sec 17(2)(viii) Rule 3(7)(viii)}, 12,000.00, Working Note:, `, Perquisite on sale of table and chairs, Cost, 60,000, Less: Dep. on straight line method @ 10% for 3 years, (18,000), Written down value, 42,000, Less: Amount paid by the assessee, (30,000), Perquisite value of Table and chairs, 12,000, Credit Card Facility, 10,000.00, Sale of Ambassador Car {Sec 17(2)(viii) Rule 3(7)(viii)}, 80,000.00, Working Note:, `, Original cost of Car, 2,50,000, Less: Dep. from 16.07.2018 to 15.07.2019, (50,000), Less: Dep. from 16.07.2019 to 15.07.2020, (40,000), Written down value, 1,60,000, Less: amount received from the assessee, (80,000), Perquisite value of Ambassador car, 80,000, Gross Salary, 7,71,800.00, Less: Standard Deduction u/s 16(ia), (50,000.00), Income under the head Salary, 7,21,800.00, Gross Total Income, 7,21,800.00, Less: Deduction u/s 80C to 80U, Nil
Page 855 :
Income Under The Head Salary, , 336, , Total Income, 7,21,800.00, Computation of tax liability, Tax on `7,21,800 at slab rate, 56,860.00, Add: HEC @ 4%, 2,274.40, Tax Liability, 59,134.40, Rounded off u/s 288B, 59,130.00, Note: It is assumed that furniture (wooden table and 4 chairs) were provided to Mr. X at his residence on, April 1st, 2021 or earlier., Illustration 45: Mr. X, a resident individual, retires from C Ltd. Delhi w.e.f. 1st February, 2022 after 25, years of service. He joined T Ltd. on the same day i.e. 1st February, 2022., The following information is provided by him about his incomes/outgoing during the Previous Year 202122:, (a) Salary/allowances/perquisites/other payment from 01.04.2021 to 31.01.2022 from C Ltd., `, (i) Basic salary, 12,000 p.m., (ii) Dearness allowance (One-half includible for superannuation benefits), 3,000 p.m., (iii) Commission, 5% on turnover achieved by him, 6,000, (iv) House accommodation, rent paid by company, 5,000 p.m., (v) Best suggestion award for total quality management scheme (in kind), 12,000, (vi) Lunch Facility (cost per meal is upto `50), 500 p.m., (vii) Gratuity under Gratuity Act, 1972, 3,35,000, (viii) Pension, 3,000 p.m., (ix) Commuted value of one-half pension w.e.f. 01.02.2022, 2,25,000, (x) Refund of employer contribution from unrecognised provident fund, (Including interest of `1,00,000), 2,50,000, (xi) Refund of employee contribution from unrecognised provident fund, (Including interest of `1,00,000), 2,50,000, (b) Salary/allowances/perquisites etc. from 01.02.2022 to 31.03.2022 from T Ltd., (i) Salary, 8,000 p.m., (ii) House rent allowance, 1,500 p.m., (iii) Free use of motorcar (exceeding 1.6 litres engine capacity), (expenses met by employer), (iv) Rent paid by assessee, 2,000 p.m., You are required to compute his income under the head Salary and Tax Liability for the Assessment Year, 2022-23., Solution:, `, Computation of income under the head Salary in C Ltd., Basic Pay (12,000 x 10), 1,20,000.00, Dearness Allowance (3,000 x 10), 30,000.00, Commission, 6,000.00, Rent free accommodation {Sec 17(2)(i), Rule 3(1)}, 21,150.00, Working Note:, `, 15% of rent free accommodation salary or rent paid whichever is less, Rent free accommodation salary, = Basic Pay + Dearness Allowance + Commission, = 1,20,000 + 15,000 + 6,000 = 1,41,000, 15% of rent free accommodation salary, 21,150, Rent paid = 5,000 x 10, 50,000, Perquisite value of rent free accommodation, 21,150, Best suggestion award (Gift) (12,000 – 5,000), 7,000.00, Lunch Facility, Nil, Gratuity {Sec 10(10)}, 1,18,653.85
Page 856 :
Income Under The Head Salary, Working Note:, Least of the following is exempt:, 1. `3,35,000, 2. `20,00,000, 3. 15/26 x 25 x 15,000 = `2,16,346.15, Received = `3,35,000.00, Exempt = (`2,16,346.15), Taxable = `1,18,653.85, Uncommuted Pension {Sec 17(1)(ii)} {3,000 x 50% x 2}, Commuted Pension {Sec 10(10A)}, Working Note:, Received, =, Exempt = 1/3 x 4,50,000 =, Taxable, =, Refund of employer’s contribution (including interest), , 337, , 3,000.00, 75,000.00, `, 2,25,000, (1,50,000), 75,000, , In T Ltd., Basic Pay (8,000 x 2), House Rent Allowance {Sec 10(13A) Rule 2A}, Working Note:, Least of the following is exempt:, 1. `4,000 – `1,600 = `2,400, 2. 40% of retirement benefit salary = `6,400, (Retirement benefit salary = `16,000), 3. `3,000, Received = `3,000, Exempt = (`2,400), Taxable = ` 600, Motor Car (2,400 x 2), Gross Salary, Less: Standard Deduction u/s 16(ia), Income under the head Salary, Income under the head Other Sources, {interest on employee’s contribution to unrecognised provident fund}, Gross Total Income, Less: Deduction u/s 80C to 80U, Total Income {Rounded off u/s 288A}, Computation of Tax Liability, Tax on `7,02,200 at slab rate, Add: HEC @ 4%, Tax Liability, Rounded off u/s 288B, , 2,50,000.00, 16,000.00, 600.00, , 4,800.00, 6,52,203.85, (50,000.00), 6,02,203.85, 1,00,000.00, 7,02,203.85, Nil, 7,02,200.00, 52,940.00, 2,117.60, 55,057.60, 55,060.00, , Illustration 46: Mr. X has taken voluntary retirement from ABC Ltd. on 31.10.2021 after serving the, employer for 23 years and 2 months. The employer has paid him `2,10,000 in connection with voluntary, retirement, a gratuity of `1,80,000 and leave salary of `1,50,000., The employee was getting the basic pay `15,000 p.m. at the time of retirement. The employer has, unrecognised provident fund., After retirement the employer has paid him provident fund balance of `5,00,000, out of which employee’s, contribution is `2,00,000. The employer’s contribution is also `2,00,000 and balance is the interest on, employee’s and employer’s contribution., The employee has taken voluntary retirement after completion of the age 50 years though he was to be
Page 857 :
Income Under The Head Salary, , 338, , retired at the age of 58., The employer has allowed him one month leave per year of service. The employee has availed seven months, leave throughout his service and has encashed six months leave., Compute employee’s Tax Liability for the Assessment Year 2022-23., Solution:, `, Basic Pay (15,000 x 7), 1,05,000.00, Employer’s contribution to unrecognised provident fund, 2,00,000.00, Interest on employer’s contribution to unrecognised provident fund, 50,000.00, Gratuity {Sec 10(10)}, 7,500.00, Working Note:, Least of the following is exempt:, 1. `1,80,000, 2. `20,00,000, 3. ½ x 15,000 x 23 = `1,72,500, Received = `1,80,000, Exempt = (`1,72,500), Taxable = ` 7,500, Leave Salary {Sec 10(10AA)}, Nil, Working Note:, Least of the following is exempt:, 1. `1,50,000, 2. `3,00,000, 3. 10 x 15,000 = `1,50,000, 4. 10 x 15,000 = `1,50,000, Received = `1,50,000, Exempt = (`1,50,000), Taxable =, Nil, Computation of leave at the credit, Leave Entitlement, = 23 months, Less: Leave availed = (7 months), Less: Leave Encashed = (6 months), Leave at the credit, = 10 months, Voluntary Retirement {Sec 10(10C) Rule 2BA}, Nil, Working Note:, Least of the following is exempt:, 1. 15,000 x 3 x 23 = `10,35,000, 2. 15,000 x 12 x 8 = `14,40,000, 3. `2,10,000, Received = `2,10,000, Exempt = (`2,10,000), Taxable =, Nil, Gross Salary, 3,62,500.00, Less: Standard Deduction u/s 16(ia), (50,000.00), Income under the head Salary, 3,12,500.00, Income under the head Other Sources, 50,000.00, {Interest on employee’s contribution to unrecognised provident fund }, Gross Total Income, 3,62,500.00, Less: Deduction u/s 80C to 80U, Nil, Total Income, 3,62,500.00, Computation of Tax Liability, Tax on `3,62,500 at slab rate, 5,625.00, Less: Rebate u/s 87A, (5,625.00)
Page 858 :
Income Under The Head Salary, Tax Liability, , 339, Nil, , Illustration 47: Mr. X is employed in ABC Ltd. and is getting Basic Pay (BP) `80,000 p.m. Employer, Contribution `10,000 p.m. to Unrecognised Provident Fund (URPF) and employee also contributed equal, amount and interest of `r80,000 was credited to URPF in P.Y. 2021-22 at a rate of 10% p.a. He has taken a, medi claim insurance in his name and paid ` 25,000 by cheque. He has taken a loan of ` 20 lakh from SBI, on 01.07.2021 at a rate of 15% p.a. for purchase of one house and at the time of taking the loan, the assessee, do not have any house in his name. Value of house is `30 lakh., He has submitted a certificate confirming the amount of interest and house is self-occupied. He has repaid, `1,50,000 on 01.03.2022., Compute his tax liability for A.Y. 2022-2023., Solution:, `, Income under the head Salary, Basic Pay (80,000 x 12), 9,60,000.00, Less: Standard Deduction u/s 16(ia), (50,000.00), Income under the head Salary, 9,10,000.00, , Income under the head House property, Gross Annual Value, Less: Municipal Tax, Net Annual Value, Less: Deduction of 30% u/s 24 (a), Less: Interest on capital borrowed u/s 24(b), Working Note:, Current period interest, From 01.07.2021 to 28.02.2022, = 20,00,000 x 15% x 8/12 = `2,00,000, From 01.03.2022 to 31.03.2022, 18,50,000 x 15% x 1/12 = `23,125, Total interest = `2,23,125 but limited to `2,00,000, Income from house property, Gross Total Income, Less: Deductions u/s 80C, Less: Deductions u/s 80D, Less: Deductions u/s 80EEA (2,23,125-2,00,000), Total Income, Rounded off u/s 288A, Computation of Tax Liability, Tax on `5,11,880 at slab rate, Add: HEC @ 4%, Tax Liability, Rounded off u/s 288B, , Nil, Nil, Nil, Nil, (2,00,000.00), , (2,00,000.00), 7,10,000.00, (1,50,000.00), (25,000.00), (23,125.00), 5,11,875.00, 5,11,880.00, 14,876.00, 595.04, 15,471.04, 15,470.00, , Illustration 48: Mr. X is employed in ABC Ltd. getting basic pay `12,000 p.m. and dearness allowance, `5,000 p.m. forming part of salary. He has contributed `3,000 p.m. to the recognised provident fund and, employer has also contributed an equal amount. During the year interest of `25,000 was credited @ 8.5%, p.a., Employer has provided rent free accommodation to the employee for which rent paid by the employer is, `5,000 p.m., The employee has encashed one month leave and was allowed leave salary of `17,000., Compute his income under the head salary for the previous year 2021-22., Solution:, `, Basic Pay (12,000 x 12), 1,44,000, Dearness Allowance (5,000 x 12), 60,000
Page 859 :
Income Under The Head Salary, Leave Salary, Rent free accommodation {Sec 17(2)(i) Rule 3(1)}, Working Note:, `, 15% of rent free accommodation salary or rent paid whichever is less, Rent free accommodation salary, = Basic Pay + Dearness Allowance + Leave Salary, = 1,44,000 + 60,000 + 17,000 = 2,21,000, 15% of rent free accommodation salary, 33,150, Rent paid = 5,000 x 12, 60,000, Perquisite value of rent free accommodation, 33,150, Employer’s contribution to recognised provident fund in excess of, 12% of retirement benefit salary {Rule 6 of Part A of schedule IV}, (36,000 – 24,480), Gross Salary, Less: Standard Deduction u/s 16(ia), Income under the head Salary, , 340, 17,000, 33,150, , 11,520, 2,65,670, (50,000), 2,15,670, , Illustration 49: Mr. X is working in XYZ Ltd. and has given the details of his income for the P.Y. 2021-22., You are required to compute his tax liability from the details given below:, Basic Salary, ` 30,000 p.m., D.A. (50% is for retirement benefits), ` 8,000 p.m., Commission as a percentage of turnover, 1%, Turnover during the year, ` 5,00,000, Bonus, ` 40,000, Gratuity, ` 25,000, His own contribution in the RPF, ` 20,000, Employer’s contribution to RPF, 20% of his basic salary, Interest accrued in the RPF @ 9.5% p.a. (on employer’s contribution), ` 13,000, Solution:, Computation of Tax Liability of Mr. X for the A.Y.2022-23, Particulars, `, `, 3,60,000, Basic Salary [ ` 30,000 × 12], 96,000, Dearness Allowance [` 8,000 × 12], 5,000, Commission on turnover [1% × ` 5,00,000], Bonus, 40,000, Gratuity, 25,000, 72,000, Employers contribution to RPF [20% of ` 3,60,000], Less : Exempt, (49,560), 22,440, Gross Salary, 5,48,440, , Less: Standard Deduction u/s 16(ia), Gross Total Income, Less: Deduction u/s 80C, Total Income, Tax, Less: Rebate u/s 87A, Tax Liability, Note:, Gratuity received during service is fully taxable., Employers contribution in the RPF is exempt up to 12% of the salary., i.e. 12% of [B.S + D.A. for retirement benefits + Commission based on turnover], = 12% of [` 3,60,000 + (50% × ` 96,000) + ` 5,000] = 12% of ` 4,13,000 = ` 49,560, , (50,000), 4,98,440, (20,000), 4,78,440, 11,422.00, (11,422.00), Nil
Page 860 :
Income Under The Head Salary, , 341, , MTP NOV 2020, Question 1, (5 x 2 Marks), Mr. Rajesh Sharma, aged 54 years, an Indian citizen, is working as Assistant Manager in ABC India Ltd. He, is getting basic salary of ₹58,000 per month. He used to travel frequently out of India for his office work. He, left India from Delhi Airport on 5th Oct, 2021 and returned to India on 2nd April, 2022., For previous year 2021-22, following information are relevant;, , (a) Dearness Allowance - 10% of Basic Pay (considered for retirement purposes), (b) Bonus - ₹ 98,000, (c) Medical allowance paid during P.Y. 2021-22 amounting to ₹60,000, (d) He was also reimbursed medical bill of his mother amounting to ₹15,000., (e) He was also transferred a laptop by company for ₹15,000 on 31st Dec 2021. The laptop was acquired, by company on 1st Oct,2018 for ₹1,00,000. Company was charging depreciation at 31.666%, assuming useful life of laptop as 3 years., (f) He was also reimbursed salary of house servant of ₹ 4,000 per month during P.Y. 2021-22., (g) Professional Tax paid by employer during P.Y. 2021-22 amounting to ₹ 2,400., (h) 400 equity shares allotted by ABC India Ltd. during P.Y. 2021-22 at the rate of ₹250 per share, against fair market value of share of ₹ 350 on the date of exercise of option., (i) Short-term capital gain on sale of shares of listed company on which STT is paid amounting to, ₹94,000., (j) Mr. Rajesh was also found owner of ₹5 lakh worth jewellery, of which he could not provide any, satisfactory explanation., Based on the above information, choose the most appropriate option of the following Multiple Choice, Questions (MCQs) for A.Y. 2022-23:, (i) What is Mr. Rajesh Sharma’s residential status for the A.Y. 2022 -23?, (a) Resident but can’t determine resident and ordinarily resident or resident but not ordinarily, resident from the given information, (b) Non-Resident, (c) Resident but not ordinarily resident, (d) Resident and ordinarily resident, Answer: (a), Since stay of Mr. Rajesh is 188 days in the current years hence he is resident in India. From the given data, it, cannot be determined whether he is ROR/NOR because the period of stay of earlier years is not given., (ii) What is his taxable perquisite for A.Y. 2022-23?, (a) ₹ 55,000, (b) ₹ 90,400, (c) ₹1,05,400, (d) ₹1,90,400, Answer: (c), Computation of taxable perquisite, Medical reimbursement fully taxable, Reimbursement of servant salary, Laptop (not taxable since value of laptop is less than the sale value), Professional tax, Equity shares (400 x (350-250)), , 15,000, 48,000, Nil, 2,400, 40,000
Page 861 :
Income Under The Head Salary, Taxable perquisite, , 342, 1,05,400, , (iii) What is the income chargeable under the head “Salaries” in the hands of Mr. Rajesh Sharma for A.Y., 2022-23?, (a) ₹ 9,76,600, (b) ₹ 9,86,600, (c) ₹ 9,71,600, (d) ₹ 9,61,600, Answer: (a), Computation of income chargeable under the head salaries, Basic salary (58,000 x 12), Dearness Allowance (10%), Bonus, Medical allowance, Medical reimbursement fully taxable, Reimbursement of servant salary, Laptop (not taxable since value of laptop is less than the sale value), Professional tax, Equity shares (400 x (350-250)), Gross salary, Less: Standard deduction u/s 16(ia), Less: Professional tax u/s 16(iii), Income under the head salary, , 6,96,000, 69,600, 98,000, 60,000, 15,000, 48,000, Nil, 2,400, 40,000, 10,29,000, (50,000), (2,400), 9,76,600, , (iv) The tax liability (without considering surcharge and Health and education cess, if any) of Mr. Rajesh, Sharma towards unexplained jewellery would be ____, (a) ₹ 1,00,000, (b) ₹ 1,50,000, (c) ₹ 3,00,000, (d) ₹ 2,50,000, Answer: (c), Computation of tax without surcharge and cess on unexplained jewellery, Unexplained income, , 5,00,000, , Tax @ 60%, , 3,00,000, , (v) The total tax liability of Mr. Rajesh Sharma for A.Y. 2022-23 is _______, (a) ₹ 5,16,800, (b) ₹ 5,18,880, (c) ₹ 4,38,800, (d) ₹ 4,40,880, Answer: (a), Computation of total tax liability, Tax on 9,76,600 at slab rate, Tax on STCG 111A 94,000 @ 15%, Tax on unexplained jewellery (60% + surcharge @ 25%), Tax before cess, Health and education cess @ 4%, Total tax liability, Rounded off u/s 288B, , 1,07,820, 14,100, 3,75,000, 4,96,920, 19,876.80, 5,16,796.80, 5,16,800.00
Page 862 :
Income Under The Head Salary, , 343, , Question 38: What is profit in lieu of salary and under what head it is chargeable to tax?, Answer:, Profit in lieu of Salary, As per section 17(3), certain payments given by the employer to the employee are called profit in lieu of, salary because it is not appropriate to call such payments as salary and are as given below:, 1. Taxable portion of Retrenchment Compensation or Voluntary Retirement., 2. Taxable portion of Gratuity, Commuted Pension and Provident Fund., 3. Amount received by the employee under Keymen Insurance Policy., 4. Amount received before taking up the employment or after termination of the employment., 5. Any other payment notified for this purpose., Question 39: Define salary under section 17(1)., Answer: Meaning of Salary Section 17(1), “Salary” includes— (i) wages; (ii) Bonus; (iii) Commission; (iv) Perquisites; (v) gratuity; (vi) pension;, (vii) Profits in lieu of salary; (viii) leave salary; (ix) employer contribution to recognised provident fund in, excess of 12% of salary of the employee; (x) amount contributed by employer to new pension system, covered under section 80CCD., Question 40: Distinguish between Foregoing of Salary and Surrender of Salary?, Answer: Foregoing of Salary / Surrender of Salary, Foregoing of Salary, If any salary has accrued to an employee, it is chargeable to tax even if he foregoes his salary. Waiver by an, employee of his salary is foregoing of salary. Once salary accrues, subsequent waiver does not absolve him, from liability to income tax., Surrender of Salary, If any employee surrenders his salary to the Central Government under the Voluntary Surrender of, Salaries (Exemption from Taxation) Act, 1961, the surrendered salary would not be included in, computing his taxable income, whether he is a private sector/public sector or Government employee., Question 41: The question whether a particular income is “Income from Salary”. or is “Income from, Business” depends upon whether the contracts is a ‘Contract of Service’ or is a ‘Contract for Service’., Discuss., Answer: Contract of Service / Contract for Service, Contract of service, Income is taxable under the head salary, if there is a ‘contract of service’ i.e. the relationship is that of, employer–employee. In other words, the employee does the work for his master. Control and supervision, vests in the master., Contract for service, A ‘contract for service’, on the other hand, is one, in which a person offers his services to any person who is, willing to pay the prescribed charges. He has discretion to do the work in his own way. He is entitled to the, fruits of his labour and liable for its losses. Such receipts constitute income from business in his hands., Question 42: What are the incomes taxable under the head Salary?, Answer: Incomes chargeable under the head Salary, Payments must be out of employer/employee relationship, The amount received by an individual shall be treated as salary only if the relationship between payer and, payee is that of an employer and employee or master and servant. The employee may be a full time, employee or part-time employee., The important point is that payment received by an individual from a person other than his employer cannot, be termed as salary. e.g. Commission received by a director from a company is salary if the director is an, employee of the company and if the director is not an employee of the company, commission will be, taxable under the head “Profits and gains of business or profession” or “Income from other sources.”, Payments received by a college lecturer from a university
Page 863 :
Income Under The Head Salary, , 344, , Emoluments received by a college lecturer from his college are salary, irrespective of the fact whether it is, received for academic work or otherwise. If lecturer is paid for setting question paper by university, the, remuneration is not salary, as it is not received from the employer and is taxable under the head “Income, from other sources”. The deciding factor is that what is not received from employer cannot be treated as, salary., A Member of Parliament or State Legislature is not treated as an employee of the Government, hence salary, and allowances received by him are, not chargeable to tax under the head “Salaries” but are chargeable to, tax under head “Income from other sources”., Question 43: Explain Treatment of Payments received to MPs & MLAs, Answer: Payments to MPs & MLAs [Section 10(17)], The following incomes of Members of Parliament or State Legislatures will be exempt:, (i) Daily Allowance - Daily allowance received by any Member of Parliament or of State Legislatures or, any Committee thereof., (ii) Constituency Allowance of MPs - In the case of a Member of Parliament or of any Committee thereof,, any allowance received under Members of Parliament (Constituency Allowance) Rules, 1986; and, (iii) Constituency allowance of MLAs - Any constituency allowance received by any person by reason of, his membership of any State Legislature under any Act or rules made by that State Legislature.
Page 864 :
Income Under The Head Salary, , 345, , MULTIPLE CHOICE QUESTIONS, 1. The maximum ceiling limit for exemption under section 10(10) in respect of gratuity for employees, covered by the Payment of Gratuity Act, 1972 is (a) `10,00,000, (b) `5,00,000, (c) `3,50,000, (d) `20,00,000, 2. The maximum ceiling limit for exemption under section 10(10C) with respect to compensation, received on voluntary retirement is (a) `2,50,000, (b) `3,00,000, (c) `3,50,000, (d) `5,00,000, 3. The HRA paid to an employee residing in Patna is exempt up to the lower of actual HRA, excess of, rent paid over 10% of salary and (a) 30% of salary, (b) 40% of salary, (c) 50% of salary, (d) 60% of salary, 4. Anirudh stays in New Delhi. His basic salary is `10,000 p.m., D.A. (60% of which forms part of pay), is `6,000 p.m., HRA is `5,000 p.m. and he is entitled to a commission of 1% on the turnover achieved, by him. Anirudh pays a rent of `5,500 p.m. The turnover achieved by him during the current year is, `12 lakhs. The amount of HRA exempt under section 10(13A) is –, (a) `48,480, (b) `45,600, (c) `49,680, (d) `46,800, 5. Where there is a decision to increase the D.A. in March, 2022 with retrospective effect from, 1.4.2020, and the increased D.A. is received in April, 2022, the increase is taxable (a) in the previous year 2020-21, (b) in the previous year 2021-22, (c) in the previous year 2022-23, (d) in the respective years to which they relate, 6. Rajesh is provided with a rent free unfurnished accommodation, which is owned by his employer,, XY Pvt. Ltd., in New Delhi. The value of perquisite in the hands of Rajesh is (a) 20% of salary, (b) 15% of salary, (c) 10% of salary, (d) 7.5% of salary, 7. Anand is provided with furniture to the value of `70,000 along with house from February, 2021., The actual hire charges paid by his employer for hire of furniture is `5,000 p.a.. The value of, furniture to be included along with value of unfurnished house for A.Y.2022- 23 is(a) `5,000, (b) `7,000, (c) `10,500, (d) `14,000, 8. For the purpose of determining the perquisite value of loan at concessional rate given to the, employee, the lending rate of State Bank of India as on __________ is required;, (a) 1st day of the relevant previous year, (b) Last day of the relevant previous year, (c) the day the loan is given, (d) 1st day of the relevant assessment year
Page 865 :
Income Under The Head Salary, , 346, , 9. Mr. Kashyap received basic salary of `20,000 p.m. from his employer. He also received children, education allowance of `3,000 for three children and transport allowance of `1,800 p.m. The amount, of salary chargeable to tax for P.Y. 2021-22 is (a) `2,62,600, (b) `2,12,600, (c) `2,22,200, (d) `2,07,800, 10. The entertainment allowance received by a Government employee is exempt up to the lower of the, actual entertainment allowance received, 1/5th of basic salary and(a) `4,000, (b) `6,000, (c) `5,000, (d) `10,000., 11. For the purposes of computing exemption under section 10(10), in case of Mr. Anand, an employee, of ABC Ltd., who is covered by the Payment of Gratuity Act, 1972, "salary" includes –, (a) only basic pay, (b) basic pay and dearness allowance, if provided in the terms of employment, (c) basic pay and dearness allowance, (d) basic pay, dearness allowance and commission as a fixed percentage of turnover, 12. Provision of rent free accommodation and motor car owned by Beta Ltd. to its employee Mr., Anand, where motor car is allowed to be used by Mr. Anand both for official and personal purposes,, is a –, (a) perquisite taxable in case of all employees, (b) perquisite taxable only in case of specified employees, (c) perquisite of rent free accommodation is taxable in case of all employees whereas perquisite of motor car, is taxable only in case of specified employees, (d) perquisite of rent free accommodation is taxable only in case of specified employees whereas perquisite, of motor car is taxable in case of all employees, 13. Retirement Benefit Salary shall include:, (a) Basic pay plus commission, (b) Basic pay plus Dearness allowance, (c) Basic pay plus Dearness allowance (forming part of salary) plus commission (fixed on turnover), (d) Basic pay plus Dearness allowance plus any commission, (e) taxable salary, 14. Payments received from recognised provident fund shall be exempt from income tax if the, (a) If the employee has rendered continuous service for a period of 10 years or more, (b) If the employee has rendered service for a period of 5 years or more, (c) If the employee has rendered continuous service for a period of 5 years or more, (d) If the employee has rendered service for a period of 10 years or more, 15. Any gratuity received by the employees covered under payment of Gratuity Act 1972, shall be, exempt, (a) Lower of Gratuity received or ` 20,00,000 or 15 days salary for each completed year of service or part, thereof in excess of six month., (b) Lower of Gratuity received or ` 10,00,000 or 15 days salary for each completed year of service or part, thereof in excess of six month., (c) Lower of Gratuity received or ` 5,00,000 or 30 days salary for each completed year of service or part, thereof in excess of six month., (d) Lower of Gratuity received or ` 20,00,000 or 30 days salary for each completed year of service or part, thereof in excess of six month., 16. Salary for the purpose of Gratuity (employees not covered under gratuity act) means, (a) Basic pay plus commission, (b) Basic pay plus Dearness allowance, (c) Basic pay plus Dearness allowance (forming part of salary) plus commission (fixed on turnover), (d) Basic pay plus any Dearness allowance plus any commission
Page 866 :
Income Under The Head Salary, , 347, , (e) taxable salary, 17. Mr. X retired on 15.06.2021 after completion of 26 years 8 months of service and received gratuity, of `6,00,000. At the time of retirement his salary was:, Basic Salary, : ` 5,000 p.m., Dearness Allowance, : ` 3,000 p.m. (60% of which is for retirement benefits), Commission, : 1% of turnover (turnover in the last 12 months was ` 12,00,000), Bonus, : ` 12,000 p.a., Taxable Gratuity shall be if he is non-government employee and covered by the Payment of Gratuity, Act 1972., (a) 6,00,000, (b) 3,75,385, (c) 4,75,385, (d) 4,55,385, 18. Mr. X retired on 15.06.2021 after completion of 26 years 8 months of service and received gratuity, of `6,00,000. At the time of retirement his salary was:, Basic Salary, : ` 5,000 p.m., Dearness Allowance, : ` 3,000 p.m. (60% of which is for retirement benefits), Commission, : 1% of turnover (turnover in the last 12 months was ` 12,00,000), Bonus, : ` 12,000 p.a., Taxable Gratuity shall be if he is non-government employee and not covered by the Payment of, Gratuity Act 1972., (a) 6,00,000, (b) 4,98,600, (c) 4,75,600, (d) 4,55,600, 19. Which of the statements is correct, (a) If the employee has not received gratuity, the commuted value of 1/2 of such pension is exempt from tax., (b) If the employee has not received gratuity, the commuted value of 1/3 of such pension is exempt from tax., (c) If the employee has not received gratuity, the commuted value of 2/3 of such pension is exempt from tax., (d) If the employee has not received gratuity, the commuted value of 1/4 of such pension is exempt from tax., 20. Which of the statements is correct, (a) If the employee has received gratuity, the commuted value of 1/2 of such pension is exempt from tax., (b) If the employee has received gratuity, the commuted value of 1/3 of such pension is exempt from tax., (c) If the employee has received gratuity, the commuted value of 2/3 of such pension is exempt from tax., (d) If the employee has received gratuity, the commuted value of 1/4 of such pension is exempt from tax., 21. Which of the statements is correct, (a) If the employee has received pension from Local Authority, the commuted value of 100% of such, pension is exempt from tax., (b) If the employee has received pension from Local Authority, the commuted value of 50% of such pension, is exempt from tax., (a) If the employee has received pension from Local Authority, the commuted value of 33.33% of such, pension is exempt from tax., (a) If the employee has received pension from Local Authority, the commuted value of 60% of such pension, is exempt from tax., 22. Which of the statements is correct, (a) If the employee has contributed to the pension scheme, deduction u/s 80CCD shall be allowed for such, contribution but maximum to the extent of 10% of retirement benefit salary, (b) If the employee has contributed to the pension scheme, deduction u/s 80CCD shall be allowed for such, contribution but maximum to the extent of 10% of taxable salary, (c) If the employee has contributed to the pension scheme, deduction u/s 80CCD shall be allowed for such, contribution but maximum to the extent of 20% of retirement benefit salary, (d) If the employee has contributed to the pension scheme, deduction u/s 80CCD shall be allowed for such, contribution but maximum to the extent of 20% of salary.
Page 867 :
Income Under The Head Salary, , 348, , 23. Which of the statements is correct, (a) Mr. X engaged in business has contributed to the pension scheme, deduction u/s 80CCD shall be allowed, for such contribution but maximum to the extent of 10% of contribution, (b) Mr. X engaged in business has contributed to the pension scheme, deduction u/s 80CCD shall be allowed, for such contribution but maximum to the extent of 10% of gross total income, (c) Mr. X engaged in business has contributed to the pension scheme, deduction u/s 80CCD shall be allowed, for such contribution but maximum to the extent of 20% of contribution, (d) Mr. X engaged in business has contributed to the pension scheme, deduction u/s 80CCD shall be allowed, for such contribution but maximum to the extent of 20% of gross total income., 24. Mr. X (Non- Govt. Employee) retired w.e.f 01.12.2021 after 20 years 10 months of service,, receiving leave salary of ` 5,00,000. Other details of his salary income are:, Basic Salary, : ` 5,000 p.m. (` 1,000 was increased w.e.f. 01.04.2021), Dearness Allowance, : ` 3,000 p.m. (60% of which is for retirement benefits), Commission, : ` 500 p.m., Bonus, : ` 1,000 p.m., Leave availed during service, : 480 days, He was entitled to 30 days leave every year. Taxable leave salary shall be, (a) 5,00,000, (b) Nil, (c) 4,73,600, (d) 4,55,600, 25. Mr. X received retrenchment compensation of `10,00,000 after 30 years 4 months of service. At, the time of retrenchment, he was receiving basic salary of `20,000 p.m.; dearness allowance of ` 5,000, p.m., taxable retrenchment compensation shall be, (a) 5,67,308, (b) Nil, (c) 6,25,000, (d) 7,00,000, (e) 10,00,000, 26. Standard deduction u/s section 16(ia) from gross salary shall be allowed maximum, (a) 50,000, (b) 50% of gross salary, (c) 40,000, (d) 1,00,000, (e) no deduction shall be allowed, 27. which of the statement is correct from the following, (a) Children education allowance is exempt upto `100 p.m. per child upto two child., (b) Children education allowance is exempt upto `100 p.m. per child upto three child., (c) Children education allowance is exempt upto `100 per child upto two child., (d) Children education allowance is exempt upto `100 per child for many child., 28. which of the statement is correct from the following, (a) Any allowance granted to an employee to meet the hostel expenditure on his child is exempt upto `300, p.m. per child upto two children., (b) Any allowance granted to an employee to meet the hostel expenditure on his child is exempt upto `300, per child upto two children., (c) Any allowance granted to an employee to meet the hostel expenditure on his child is exempt upto `100, p.m. per child upto two children., (d) Any allowance granted to an employee to meet the hostel expenditure on his child is exempt upto `400, p.m. per child upto two children., 29. Mr. X is employed in central Government getting basic pay `30,000 p.m., dearness allowance, `7,000 p.m., servant allowance `2,000 p.m., entertainment allowance `1,000 p.m., In this case, entertainment allowance exempt u/s 16(ii) for the F.Y. 2021-22 shall be, (a) Nil, (b) 5,000
Page 868 :
Income Under The Head Salary, , 349, , (c) 12,000, (d) 7,000, 30. which of the statement is not correct from the following, (a) If the population is upto 10 lakhs, taxable amount shall be 7.5% of rent free accommodation salary, (b) If the population is more than 10 lakhs but upto 25 lakhs, taxable amount shall be 10% of rent free, accommodation salary, (c) If the population is more than 25 lakhs, taxable amount shall be 15% of rent free accommodation salary, (d) If the population is more than 10 lakhs, taxable amount shall be 15% of rent free accommodation salary, Answer, 1. (d); 2. (d); 3. (b); 4. (a); 5. (b); 6. (b); 7. (a); 8. (a); 9. (b); 10. (c); 11.(c); 12.(c); 13.(c); 14.(c); 15.(a);, 16.(c); 17.(c); 18.(b); 19.(a); 20.(b); 21.(a); 22.(a); 23.(d); 24.(c); 25.(a); 26.(a); 27.(a); 28.(a); 29.(b); 30.(d)
Page 869 :
Income Under The Head Salary, , 350, , PRACTICE PROBLEMS, TOTAL PROBLEMS 28, Problem 1., Mr. X has joined ABC Ltd. on 01.07.2011 in the pay scale of 40,000 – 500 – 50,000 – 1,000 – 60,000 –, 2,000 – 80,000. The employer has allowed him dearness allowance @ 7% of his basic pay from 01.04.2021, to 30.06.2021 and thereafter dearness allowance was allowed @ 10% of the basic pay., Compute employee’s Salary, Total Income and Tax Liability for the Assessment Year 2022-23., Answer = Salary: `5,38,345; Total Income: `5,38,350; Tax Liability: `20,980, Problem 2., Mr. X has joined ABC Ltd. on 01.10.2008 in the pay scale of 25,000 – 900 – 30,400 – 1,100 – 38,100 –, 1,500 – 50,100. The employer has allowed him dearness allowance @ 4.35% of the basic pay from, 01.04.2021 to 30.09.2021, @ 7.5% upto 31.12.2021. Thereafter it was allowed @ 10.5% of the basic pay., Compute employee’s Gross Salary, Total Income and Tax Liability for the Assessment Year 2022-23., Answer = Gross Salary: `4,30,831; Total Income: `4,30,830; Tax Liability: Nil, Problem 3., Mr. X is employed in ABC Ltd. getting basic pay `40,000 p.m. and dearness allowance ` 10,000 p.m. (40%, of DA forming part of salary for retirement benefit). Employer has paid Bonus `1,000 p.m., Commission @, 2% on sales turnover of `60,00,000. Employer and employee each has contributed `8,000 p.m. to, recognized provident fund. During the year interest of ` 1,00,000 was credited to recognized provident fund, @ 10% p.a. on employer plus employee contribution. Compute his Income and Tax Liability for the, Assessment Year 2022-23., Answer = Total Income: `6,09,240; Tax Liability: `35,720, Problem 4., Mr. X is employed in ABC Ltd. getting basic pay `80,000 p.m. and dearness allowance ` 20,000 p.m., Employer has paid Bonus `2,000 p.m., Commission @ 2% on sales turnover of `80,00,000. Employer and, employee each has contributed `14,000 p.m. to recognized provident fund. During the year interest of, `2,00,000 was credited to recognized provident fund @ 9% p.a. on employer plus employee contribution., Income under the head House Property ` 3,00,000, Compute his Income and Tax Liability for the Assessment Year 2022-23., Answer = Total Income: `15,17,600; Tax Liability: `2,78,490, Problem 5., Mr. X is retired from ABC Ltd. on 11.09.2021 after serving the employer for 11 years 10 months and 20, days. At the time of retirement his basic pay was `37,000 p.m. but it was `33,000 p.m. upto 31.05.2021. The, employee was getting dearness allowance ` 4,000 p.m. but upto 31.05.2021 it was `3,000 p.m. The, employer has paid him gratuity of `5,10,000. Half of the dearness allowance forms part of the salary for, retirement benefits., Compute his tax liability in two situations –, (a) He is covered under Payment of Gratuity Act 1972;, (b) He is not covered under Payment of Gratuity Act 1972., Answer = Tax Liability: (a) Nil; (b) Nil, Problem 6., Mr. X is retired from ABC Ltd. with effect from 18.09.2021 after serving the employer for 20 years and 6, months. At the time of his retirement his basic pay was `39,000 p.m. and dearness allowance `3,000 p.m., The employee was covered under Payment of Gratuity Act 1972. The employer has paid him gratuity of, `5,70,000 and has allowed him pension of `5,000 p.m. The employer has also allowed him commutation of, pension on 01.01.2022 for 48% of the pension and has paid `2,88,000., Compute employee’s Tax Liability for the Assessment Year 2022-23., Answer = Tax Liability: Nil
Page 870 :
Income Under The Head Salary, , 351, , Problem 7., Mr. X is retired from ABC Ltd. on 27.11.2021 after serving the employer for 11 years 11 months and 11, days. The employer has paid him gratuity of `2,50,000. At the time of his retirement his basic pay was, `12,500 p.m., The employer has allowed him pension of `6,200 p.m., the employee has requested for commutation of, pension on 01.02.2022 and employer has allowed him commutation @ 52% of his pension and has paid, `3,86,880., Compute his Tax Liability for the Assessment Year 2022-23., Answer = Tax Liability: Nil, Problem 8., Mr. X joined ABC Ltd. in the pay scale of ` 10,800 – 400 – 16,400 – 500 – 19,400 on 01.07.2003 and he, resigned on 15.09.2021. He was allowed dearness allowance @ 50% of his basic pay, forming part of salary, for retirement benefits., On retirement, he received gratuity of ` 2,60,000. He was allowed pension of `6,000 per month with effect, from 16.09.2021. He was allowed commutation of 75% of his pension on 01.01.2022 and received a sum of, `6,00,000 as commuted pension., Compute his Tax Liability for Assessment Year 2022-23., Answer = Tax Liability: Nil, Problem 9., Mr. X is retired from ABC Ltd. on 31.03.2022 after serving the employer for 30 years and 11 months and, the employer has paid him leave salary of `5,00,000. At the time of retirement, he was getting basic pay, `25,000 p.m. but it was `22,000 p.m. upto 31.07.2021. Further, the employee was getting dearness, allowance `6,000 but it was `4,000 p.m. upto 31.07.2021 and 50% of the dearness allowance forms the part, of salary for retirement benefits., The employee was entitled for 3 months leave for every year of service, but the employee has availed 7, months leave throughout the service and has encashed 4 months leave., Compute employee’s Tax Liability for the Assessment Year 2022-23., Answer = Tax Liability: `19,240, Problem 10., Mr. X is retired from ABC Ltd. on 28.02.2022 after serving the employer for 21 years and 10 months. At the, time of his retirement his basic pay was `13,000 p.m. but upto 30.09.2021 it was `9,500 p.m. The employer, has allowed him dearness allowance @ 10% of his basic pay., The employee was entitled for 45 days leave per year of service. During entire service the employee has, availed 65 days leave and has encashed 45 days leave. The employer has paid him leave salary of `3,10,000, at the time of retirement., Employer has also paid him gratuity of `2,50,000, pension of `6,000 p.m. and the employee was allowed, commutation of 40% of his pension amounting to ` 2,88,000., Compute his Tax Liability for the Assessment Year 2022-23., Answer = Tax Liability: Nil, Problem 11., Mr. X is retired from ABC Ltd. with effect from 01.12.2021 after serving the employer for 16 years. At the, time of his retirement his basic pay was `43,000 p.m. The employee was entitled for 65 days leave per year, of service. The employee has 780 days leave at his credit at the time of retirement (as per employer’s record), which were encashed by the employer. An amount of `5,12,000 was paid by the employer., Compute his Tax Liability for the Assessment Year 2022-23., Answer = Tax Liability: `14,250, Problem 12., Mr. X is employed in ABC Ltd. getting basic pay `45,000 p.m., dearness allowance `7,000 p.m. The
Page 871 :
Income Under The Head Salary, , 352, , employer has provided him rent free accommodation for which rent paid by the employer is `3,000 p.m. The, employer has contributed `6,000 p.m. to recognised provident fund and the employee has also contributed, equal amount. The interest of `16,000 @ 8.5% was credited to the provident fund account on 30.06.2021 for, the period 01.07.2020 upto 30.06.2021., Compute employee’s Total Income and tax liability for the Assessment Year 2022-23., Answer = Total Income: `5,45,200; Tax Liability: `22,400, Problem 13., Mr. X is employed in ABC Ltd. getting basic pay `20,000 p.m., dearness allowance `7,000 p.m. The, employer has contributed `3,500 to the unrecognised provident fund and the employee has also contributed, equal amount. The employee was retired on 31.10.2021 after serving the employer for 20 years and 6, months and employer has credited interest `21,000 to the provident fund account on 31.10.2021 and interest, rate is 12% p.a., The employer has paid provident fund balance `10,00,000 to the employee on 01.11.2021 out of which, employee’s contribution is `4,00,000 and employer’s contribution is also `4,00,000 and balance is interest., Employer has paid gratuity `2,60,000 and allowed him pension `5,000 p.m. The employee was allowed, commutation of pension on 01.01.2022 for 40% of the pension and has paid `2,40,000., Compute employee’s Tax Liability for the Assessment Year 2022-23., Answer = Tax Liability: `87,460, Problem 14., Mr. X is employed in ABC Ltd., Calcutta and is getting basic pay `40,000 p.m., dearness allowance `16,000, p.m. (50% of dearness allowance forms part of salary for the purpose of retirement benefits)., The employee was allowed bonus `1,000 p.m. and commission @ 2.5% on the sales turnover of `60,00,000., The employer has paid him house rent allowance `6,000 p.m. The employee has paid rent `6,500 p.m., Compute his Gross Salary and Tax Liability for the Assessment Year 2022-23., Answer = Gross Salary: `8,50,600; Tax Liability: `85,920, Problem 15., Mr. X is employed in ABC Ltd. getting basic pay `45,000 p.m., dearness allowance `19,000 p.m. (half of it, is taken into consideration for retirement benefit)., Employer has allowed him house rent allowance with effect from 01.10.2021 @ 10,000 p.m. and the, employee has paid rent `12,000 p.m. throughout the year. Employer has paid him children education, allowance `75 per month per child for four children and has also paid him hostel allowance `500 per month, for one child, the actual expenses incurred by the employee is `1,000 per month per child., Employer has paid him transport allowance `1,700 per month with effect from 01.07.2021 and the employee, has incurred `1,500 p.m., Compute his Gross Salary and Tax Liability for the Assessment Year 2022-23., Answer = Gross Salary: `7,58,200; Tax Liability: `66,710, Problem 16., Mr. X is employed in Indian Airlines as pilot and is getting basic pay of `55,000 p.m. and dearness, allowance @ 10% of basic pay., Employer has paid him children education allowance of `750 per month for one of his adopted child. The, employer has also paid transport allowance of `1,800 per month. The employee has incurred `2,000 per, month. The employer has paid him flight allowance in lieu of daily allowance `10,000 p.m., Compute his Gross Salary and Tax Liability for the Assessment Year 2022-23., Answer = Gross Salary: `7,41,400; Tax Liability: `63,210, Problem 17., Mr. X is employed in Central Government getting basic pay `39,000 p.m. and dearness allowance @ 60% of, basic pay. Employer has paid children education allowance `600 per month per child for 3 children and has, paid hostel allowance `1,000 per month per child for one child. Employer has paid professional tax of `175
Page 872 :
Income Under The Head Salary, , 353, , p.m. on behalf of the employee and has allowed him entertainment allowance `200 p.m. out of which he has, saved ` 100 p.m. The employer has paid medical allowance `300 p.m. but employee’s expenditure is `500, p.m., Compute his income under the head Salary and Tax Liability for the Assessment Year 2022-23., Answer = Income under the head Salary: `7,30,000; Tax Liability: `60,840, Problem 18., Mr. X is employed in ABC Ltd. since 01.07.2005 in the pay scale of 35,000 – 300 – 36,500 – 500 – 40,000 –, 750 – 47,500 – 1000 – 57,500. The employer has given him two increments in advance at the time of his, taking up the job., During the previous year 2021-22, he was allowed dearness allowance @ 11% of the basic pay. The, employer has allowed him house rent allowance @ `3,000 p.m. Entertainment allowance `600 p.m. but the, employee has saved `100 p.m. which was donated by him to a charitable institution., The employer has paid Professional tax of `2,400 on his behalf on 02.04.2021 though it was due on, 31.03.2021. The employer has paid conveyance allowance of `500 p.m. The employee has incurred `100, p.m. for official purpose, `150 p.m. for personal purpose and balance has been saved by the employee., Compute his income under the head Salary and Tax Liability for the Assessment Year 2022-23., Answer = Income under the head salary: `5,88,240; Tax Liability: `31,350, Problem 19., Mr. X joined ABC Ltd. on 01.07.2014 in the pay scale of 25,000 – 1,500 – 31,000 – 1,600 – 39,000 – 1,800, – 49,800. The employer has allowed him 3 increments in advance at the time of taking up the job. The, employee’s salary is due on the 1st of next month., Employee was allowed dearness allowance @ `10,000 p.m., during the previous year 2020-21 and @, `12,000 p.m. in 2021-22. The employee has resigned w.e.f. 01.03.2022. The employee was allowed pension, @ `10,000 p.m. and his pension is due on the last day of the month., Compute Tax Liability for the Assessment Year 2022-23., Answer = `30,560, Problem 20., Mrs. X is employed in ABC Ltd. and her salary is `6,00,000, but it is increased to `6,60,000 in previous, year 2021-22 w.e.f. previous year 2020-21. Compute Tax Liability and relief under section 89., Tax Rate of Previous Year 2020-21 for resident woman, If total income upto `2,50,000, Nil, On Next 2,50,000, 5%, On Next 5,00,000, 20%, On Balance amount, 30%, Heath & education cess @ 4%, rebate u/s 87A will be allowed up to ` 12,500 if total income of the person is, up to ` 5,00,000., Standard deduction u/s 16(ia) was ` 50,000 in year 2020-21., Answer = Tax Liability: `48,360; Relief: Nil, Problem 21., Mr. X is employed in ABC Ltd. getting basic pay `40,000 p.m. and dearness allowance `5,000 p.m. (half of, the dearness allowance forms part of salary for retirement benefit salary). The employee was working in, sales deptt. and employer has allowed him commission @ 1.5% on the sales turnover of `20,00,000., Employee has contributed `6,000 p.m. to the recognized provident fund. The employer has also contributed, an equal amount. During the year interest of `20,000 was credited on 30.06.2021 @ 8.5% p.a., The employer has provided him rent free accommodation which is owned by the employer himself and the, population of the place is 14,00,000., Compute his total income and tax liability for A.Y. 2022-23., Answer = Total Income : `5,09,200; Tax Liability: `14,910
Page 873 :
Income Under The Head Salary, , 354, , Problem 22., Mr. X is employed in ABC Ltd. getting basic pay `41,000 p.m., dearness allowance `7,000 p.m. (10% of the, dearness allowance forms part of salary for retirement benefits)., The employer has paid commission of `3,000 p.m. and has allowed him medical allowance `400 p.m. The, employee was paid house rent allowance `6,000 p.m. The employee has paid rent of `5,000 p.m., The employer has discontinued payment of house rent allowance with effect from 01.09.2021 and has, provided him rent free accommodation with effect from 01.11.2021. The accommodation was owned by the, employer and the population of the place is 4,00,000. The employee was allowed arrears of salary `10,000, and advance salary `20,000. The employee was also provided furniture with effect from 01.01.2022. Its, original cost is `1,00,000 and written down value is `35,000., Compute employee’s Tax Liability for the Assessment Year 2022-23., Answer = Tax Liability: `42,550, Problem 23., Mr. X is employed in ABC Ltd. getting basic pay `50,000 p.m. and dearness allowance 5,000 p.m., Commission `3,500 p.m. Employer has paid overtime allowance with effect from 01.05.2021 @ `1,000, p.m. and has allowed him house rent allowance `2,000 p.m. The employee has paid rent `500 p.m., The employer has discontinued payment of house rent allowance with effect from 01.06.2021 and has, allowed him rent free accommodation with effect from 01.09.2021. The accommodation was owned by the, employer itself at Calcutta. Employer has also provided him furniture from the same date with original cost, `1,50,000 and has also paid professional tax `200 p.m. on behalf of the employee., The employee has received arrears of salary `35,000., Compute his Tax Liability for the Assessment Year 2022-23., Answer = Tax Liability: `68,740, Problem 24., Mr. X is employed in ABC Ltd. getting basic pay `45,000 p.m., dearness allowance `5,000 p.m. and 30% of, it forms part of salary., The employee is also getting dearness pay `1,000 p.m. and 10% of it forms part of salary. He is getting, bonus `1,200 p.m. The employer has provided him one accommodation in Delhi for which rent paid by the, employer is `1,200 p.m., The employee was transferred to Bombay with effect from 01.01.2022 and the employer has provided him, rent free accommodation at Bombay also which is owned by the employer himself., The employee has received arrears of salary `32,000 and advance salary of `11,000., Compute employee’s Tax Liability for the Assessment Year 2022-23., Answer = Tax Liability: `40,830, Problem 25., Mr. X is employed in ABC Ltd getting basic pay `40,000 p.m. Salary for the month of March 2021 was paid, to the employee on 03.04.2021., The employer has paid house rent allowance `5,000 p.m. with effect from 01.11.2021 and the employee has, paid rent `6,000 p.m., Prior to 01.11.2021 the employer has provided him an accommodation and rent paid by the employer was, `7,000 p.m., Employee has also received advance salary `20,000., Compute his Tax Liability for the Assessment Year 2022-23., Answer = Tax Liability: `14,460, Problem 26., Mr. X has taken four loans from his employer –, (i) `7,00,000 (term loan) on 23.09.2021 @ 3% p.a. for the purpose of purchasing a new motor car (in, Delhi). The loan was repaid in monthly instalments of ` 25,000 each starting from 10.12.2021., (Presume SBI Rate 10%)
Page 874 :
Income Under The Head Salary, , 355, , (ii), , `10,00,000 on 11.05.2021 @ 5% p.a. for purchasing a house. The loan was repaid in annual, instalments of ` 55,000 each starting from 31.03.2022. (Presume SBI Rate 10.5%), (iii) The employee has taken a loan of `3,00,000 for the treatment of specified disease on 28.11.2021, (Presume SBI Rate 12%), (iv) He has taken a personal loan of `18,000 on 30.03.2022 for a period of 2 years. (Presume SBI Rate, 12%), Compute perquisite value of the loan given to the employee., Answer = Total Perquisite Value: `77,469.57, , Problem 27., Find out the perquisite value in the following cases:, Asset, Furniture Air-conditioner Video camera Motor car, Original cost, 1,00,000, 45,000, 50,000, 3,40,000, Date of purchase by the, 07.03.2018, 01.07.2020, 10.07.2019, 01.10.2017, employer, Date of putting to use by, 31.03.2018, 01.07.2020, 11.07.2019, 01.10.2017, employer, Date of sale of asset to the, 01.09.2021, 01.08.2021, 01.08.2021, 01.01.2022, employee, Payment made by the, 40,000, 15,000, 20,000, 1,50,000, employee, Answer: Furniture: `30,000; Air-Conditioner: `25,500; Video Camera: `20,000; Motor, Computer: Nil, , Computer, 55,000, 01.01.2019, , 10.01.2019, 09.01.2022, 25,000, Car : Nil;, , Problem 28., Mr. X is employed in ABC Ltd. getting basic pay of `8,000 p.m., Employer has paid medical allowance of `10,000 during the year and has incurred `7,000 on the treatment, of father in law of Mr. X in India. The treatment was provided in a Government hospital and father in law of, Mr. X is dependent on him., The employee has been provided with a motor car of 1.8 litre engine capacity for official as well as personal, use and all expenses are met by the employee himself but driver has been provided by the employer., Mr. X has income under the head house property `1,00,000 and income under the head business profession, `1,50,000 and deductions allowed under section 80C to 80U are `3,500., Compute his Tax Liability for the Assessment Year 2022-23., Answer = Tax Liability: Nil, , Check Detailed Solution given on our website www.mkgeducation.com, (Icon /Heading No.11: Books)
Page 875 :
Income Under The Head Salary, , 356, , EXAMINATION QUESTIONS, JULY – 2021 (NEW COURSE), Question 2(a), (6 Marks), Mrs. Rohini, aged 62 years, was born and brought up in New Delhi. She got married in Russia in 1996 and, , settled there since then. Since her marriage, she visits India for 60 days each year during her summer break., The following are the details of her income for the previous year ended 31.03.2022:, S. No., , Particulars, , Amount, (in ₹), , 1., , Pension received from Russian Government, , 65,000, , 2., , Long-term capital gain on sale of land at New Delhi (computed), , 3., , Short-term capital gain on sale of share of Indian listed companies in respect, , 3,00,000, 60,000, , of which STT was paid both at the time acquisition as well as at the time of, sale (computed), 4., , Premium paid to Russian Life Insurance Corporation at Russia, , 75,000, , 5., , Rent received (equivalent to Annual Value) in respect of house property in, , 90,000, , New Delhi, You are required to ascertain the residential status of Mrs. Rohini and compute her total income and tax, liability in India for Assessment year 2022-23., Solution is given on our website www.mkgeducation.com (Icon /Heading No.11: Books), , JAN – 2021 (NEW COURSE), Question 3(a), (7 Marks), (a) Rajesh was employed in Axis Ltd., Mumbai. He received a salary of ₹ 45,000 p.m. from 1.04.2021 to, 20.09.2021. He resigned and left for Dubai for the first time on 28.09.2021 and got monthly salary of rupee, equivalent of ₹ 90,000 from 1.10.2021 to 31.03.2022., His salary for October to December was credited in his Mumbai bank account directly and the salary for, January to March 2022 was credited in his Dubai bank account., The cost of his air tickets to Dubai costing ₹ 1,50,000 was funded by her sister staying in London. The cost, of his initial stay at Dubai costing ₹ 40,000 was funded by one of his friends staying in Delhi., He further received interest of ₹ 10,500 on his fixed deposits and ₹ 7,500 on his savings a/c with his, Mumbai bank. He also paid LIC Premiums of ₹ 15,000 for self, ₹ 10,000 for spouse and ₹ 25,000 for, dependent mother aged 71 years., Compute taxable income of Mr. Rajesh for the Assessment Year 2022-23., Solution is given on our website www.mkgeducation.com (Icon /Heading No.11: Books), , NOV – 2019 (NEW COURSE), Question.1, (14 Marks), Mrs. Mitul, a resident individual, aged 63 years, is a qualified medical practitioner. She runs her own clinic., Income & Expenditure A/c of Mrs. Mitul for the year ending March 31st 2022 is as under:, Expenditure, , ₹, , Income, , To Salary to Staff, , 1,20,000 By Consultation Fees, , To Administrative Exp., , 2,90,000 By Salary received from True Care, , ₹, , 12,00,000, 1,80,000
Page 876 :
Income Under The Head Salary, , 357, , Hospitals (P) Ltd., To Conveyance Expenses, , 24,000 By Rental Income from House Property, , 78,000, , To Power & Fuel, , 24,000 By Dividend from Foreign Companies, , 10,000, , To Interest on Housing Loan, , 1,00,000, , To Interest on Education Loan for son, , 26,000, , To Amount paid to scientific research, association approved & Notified under, , 25,000, , Section 35, To net profit, Total, , 8,59,000, 14,68,000 Total, , 14,68,000, , Explanatory Information:, (i) She is working part-time with True Care Hospitals (P) Ltd. Her salary details are as under:, , Basic Pay, , ₹13,000 p.m., , Transport Allowance, , ₹ 2,000 p.m., , Total, , ₹15,000 p.m., , Further, during P.Y. 2021-22, her son had undergone a medical treatment in True Care Hospitals (P) Ltd., free of cost. The hospital would have charged a sum of ₹ 60,000 for a similar treatment to un-related, patients., (ii) She owns a residential house. Ground floor of the house is self-occupied by her while first floor has been, rented out since 01/10/2021. The reconstruction of the house was started on 01-04-2021 and was completed, on 30-09-2021. The monthly rent is ₹ 10,000. The tenant also pays ₹ 3,000 p.m. as power back-up charges., She took a housing loan of ₹ 12 lakhs on 01-04-2021. Interest on housing loan for the period 01-04-2021 to, 30-09-2021 was 60,000 and for the period 01-10-2021 to 31-03-2022 was ₹40,000. During the year, she also, paid municipal taxes for the F.Y. 2020-21 ₹5,000 and for F.Y. 2021-22 ₹5,000., (iii) Other informations:, (a) Conveyance expenses include a sum of ₹12,000 incurred for conveyance from house to True Care, Hospital (P) Ltd. and vice-versa in relation to her employment., (b) Power & fuel expenses include a sum of ₹6,000 incurred for generator fuel for providing power back-up, to the tenant., (c) Administrative expenses include a sum of ₹10,000 paid as Municipal Taxes for her house., (d) Clinic equipment’ details are:, Opening W.D.V. of clinic equipments as on 01-04-2021 was ₹1,00,000 and fresh purchase made on 2808-2021 is ₹25,000 which was paid in cash., (e) She also paid tuition fee of ₹40,000 for her grand-daughter, which has been debited to her Capital A/c., (f) She availed a loan of ₹8,00,000 from bank for higher education of her son. She repaid principal of ₹, 50,000 and interest of ₹ 26,000 during P.Y. 2021-22., You are required to compute her net taxable income and net tax liability for the Assessment Year 2022-23., Solution is given on our website www.mkgeducation.com (Icon /Heading No.11: Books)
Page 877 :
Income Under The Head Salary, , 358, , MAY – 2019 (NEW COURSE), Question 1, (14 Marks), From the following particulars of Shri Jagdish (Aged 59 Years) for the Assessment Year 2022-23, you are, required to find out his taxable income and net tax liability:, (i) Basic Salary @ ` 51,000 per month, Dearness allowance @ ` 10,000 per month (Part of salary for, , retirement benefits), House rent allowance ` 4,000 per month and rent paid for house in Mumbai is, `7,000 per month., (ii) He owns a commercial building at New Delhi, which is let out on 1/7/2021 at a monthly rent of, `46,000. He paid for municipal taxes of ` 27,000 and ` 25,000 for the financial year 2020-21 and, 2021-22 on 31-3-2022 and 20-4-2022 respectively., (iii) He deals in shares. During financial year 2021-22 he earned ` 1,70,000 from his share business and, paid ` 30,000 as security transaction tax., (iv) He purchased 4000 unlisted shares of Shyam Limited on 16-1-2010 for ` 80,000. Company declared, bonus in the ratio of 1:1 on 1st February, 2010. Shri Jagdish sold 3000 Bonus Shares on 28/12/2021, for ` 2,00,000 to his friend Mr. Mehul through unrecognized stock exchange.(Cost Inflation Index:, 2009-10: 148, 2021-22 : 317), (v) He received dividend of ` 13,00,000 as dividend income from listed domestic company, Interest, from saving bank account deposits with IDBI Bank ` 15,000 and lottery winnings (Net of TDS @, 30%) is ` 21,000., He paid the following amount out of his taxable income :, (a) Deposits in Public Provident Fund ` 2,00,000., (b) Medical insurance premium paid for health of his wife ` 19,000 and for health of dependent son, `12,000 through cheque., Solution is given on our website www.mkgeducation.com (Icon /Heading No.11: Books), Question 3 (a), (8 Marks), Mr. Madhvan is a finance manager in Star Private Limited. He gets a salary of ` 30,000 per month. He owns, two houses, one of which has been let out to his employer and which is in-turn provided to him as rent free, accommodation. Following details (annual) are furnished in respect of two house properties for the Financial, Year 2021-22., , House 1, , House 2, , Fair rent, , 75,000, , 1,95,000, , Actual rent, , 65,000, , 2,85,000, , Municipal Valuation, , 74,000, , 1,90,000, , Municipal taxes paid, , 18,000, , 70,000, , Repairs, , 15,000, , 35,000, , Insurance premium on building, , 12,000, , 17,000, , Ground rent, , 7,000, , 9,000, , Let-out to Star Private Limited, , Let-out to Ms. puja, , Nature of occupation, , ` 17,000 were paid as Interest on loan taken by mortgaging House 1 for construction of House 2., During the Previous year 2021-22, Mr. Madhvan purchased a rural agricultural land for ` 2,50,000., Stamp valuation of such property is ` 3,00,000., Determine the taxable income of Mr. Madhvan for the assessment year 2022-23. All workings, should from part of your answer., Solution is given on our website www.mkgeducation.com (Icon /Heading No.11: Books)
Page 878 :
Income Under The Head Salary, , 359, , Question 4 (a), (10 Marks), Ms. Geeta, a resident individual, provides following details of her income/losses for the year ended, 31.03.2022:, Particulars, Amount, (`), (i), Income from salary (computed), 41,20,000, , (ii), , Rent received from house property situated in Delhi, , 5,00,000, , (iii), , Interest on loan taken for purchase of above property. Loan was taken from a, , 7,50,000, , friend, (iv), , Rent received from house property situated in Jaipur, , 3,20,000, , (v), , Interest on loan taken for house property in Mumbai which is self-occupied., , 1,57,000, , Loan was taken from PNB on 01.01.1999 for purchase of this property, (vi), , Interest on loan taken for repair of house properties situated in Mumbai and, , 1,50,000, , Delhi. Loan was taken on 01.04.19 and was utilized in 50:50 ratio for house, properties situated in Mumbai and Delhi, respectively., (vii), , Long term capital gains on sale of equity shares computed in accordance with, , 8,95,000, , Section 112A, (viii), , Interest on fixed deposit, , (ix), , Loss from textile business, , 7,50,000, , (x), , Speculation profit, , 2,30,000, , (xi), , Lottery income, , (xii), , Loss incurred by the firm in which she is a partner, , (xiii), , Salary received as a partner from partnership firm. The same was allowed to, , 73,000, , 75,000, 1,60,000, 50,000, , firm, (xiv), , Brought forward short-term capital loss on sale of gold., , (xv), , Brought forward loss on sale of equity shares of the nature specified u/s 111A, , 25,000, , (xvi), , Life insurance premium paid for her son who is 30 years of age and is, , 15,000, , 2,75,000, , working in USA, Compute total income of Ms. Geeta for the assessment year 2022-23 and the amount of loss that can, be carried forward., For the above solution, you may assume principal repayment of loan as under:, (1) Loan taken for purchase of house property in – ` 2,50,000, Delhi, (2) Loan taken for purchase of house property in – ` 50,000, Mumbai, (3) Loan taken for repair of house properties in, – ` 75,000, Delhi and Mumbai, Workings notes should form part of your answer. Wherever necessary, suitable assumptions may be made, by the candidates and disclosed by way of note., Solution is given on our website www.mkgeducation.com (Icon /Heading No.11: Books)
Page 879 :
Income Under The Head Salary, , 360, , MAY – 2019 (OLD COURSE), Question 1, (14 Marks), Mr. X working in a private company from last 10 years. His salary details for the financial year 2021-22 are:, `, , i., , Basic Salary, , 1,50,000 p.m., , ii., , Dearness Allowance, , 55,000 p.m., , iii., , Commission, , 35,000 p.m., , iv., , Transport Allowance, , v., , Medical Reimbursement, , 5,000 p.m., 20,000 paid during the year, , Mr. X resigned from the services on 30th September, 2021. He was paid gratuity of ` 20 lakhs on his, retirement. A lumpsum amount of ` 36 lakhs was also paid from unrecognized provident fund. The, provident fund amount consisted of employer contribution ` 13.20 lakhs and interest thereon ` 3 lakhs. The, employee contribution was ` 16.20 lakhs and interest thereon was ` 3.60 lakhs., He had taken the possession of house on 28th February, 2022 after making payment of final installment of, housing loan to bank. Loan was taken on 01-04-2020. The accumulated interest as on 31st march, 2021 was, ` 1.5 lakh. He made payment of ` 2,20,000 during the year which included interest ` 1,10,000 for 11, months., He started business of hiring of goods vehicle, purchased 3 small goods vehicle on 15th November, 2021 and, 3 heavy vehicles having gross weight of 15 MTs each on 1st December, 2021. He did not maintain books of, accounts for income and expenditure of hiring of goods vehicle. One of his friend gifted him ` 6 lakhs to, purchase the vehicles., He was holding 25% equity shares in CMF Ltd., an Indian company. The paid up share capital of company, as on 31st March, 2021 was ` 20 lakh divided into 2 lakh shares of ` 10 each which were issued at a, premium of ` 30 each. Company allotted shares to shareholders on 1st October, 2015. Company bought back, 30% of its share on 30th April, 2021 under the provisions of Companies Act, 2013 on making payment of, `60 per share., He paid insurance premium of ` 20,000 on his life policy during the financial year 2021-22. The policy was, taken in April 2011 and sum assured was ` 1,50,000. He also made payment of ` 25,000 L.I.C pension fund, and premium of ` 40,000 towards Mediclaim policy for self and wife., Compute total income and tax payable thereon for the Assessment year 2022-23. There was no change in, salary of Mr. X from last two years. Cost inflation Index is:, Financial Year, , Cost Inflation Index, 2015-16, 254, 2021-22, 317, Solution is given on our website www.mkgeducation.com (Icon /Heading No.11: Books), , NOV – 2018 (NEW COURSE), Question 5(a), (6 Marks), Mr. Janakaraj, employed as General Manager in Rajus Refractories Pvt. Ltd., furnishes you the undermentioned information for the year ended 31-03-2022:, (i) Basic salary upto 30-11-2021, `70,000 p.m., `80,000 p.m., Basic salary from 01-12-2021, Note : Salary is due and paid on the last day of every month.
Page 880 :
Income Under The Head Salary, , 361, , (ii) Dearness allowance @ 50% of basic salary (not forming part of salary for retirement benefits)., (iii) Bonus equal to one month salary. This was paid in November, 2021 on basic salary plus dearness, allowance applicable for that month., (iv) Contribution of employer to recognized provident fund account of the employee @ 18% of basic, salary, employee also contributing an equivalent amount., (v) Profession tax paid `6,000 of which `3,000 was paid by the employer., (vi) Facility of laptop was provided to Janakaraj for both official and personal use. Cost of laptop `65,000, and was purchased by the company on 11-10-2021., (vii) Leave travel concession given to Janakaraj, his wife and three children (one daughter aged 6 and twin, sons aged 4). Cost of air tickets (economy class) reimbursed by the employer `20,000 for adults and, lumpsum of `25,000 for three children. Janakaraj is eligible for availing exemption this year to the extent it, is permissible under the Income-tax Act, 1961., Compute the taxable salary of Mr. Janakaraj., Solution is given on our website www.mkgeducation.com (Icon /Heading No.11: Books), Question 5(b), (2 Marks), Examine with brief reasons, whether the following are chargeable to income tax and· the amount liable to, tax with reference to the provisions of the Income Tax Act,1961:, (i) Allowance received by an employee Mr. Ram working in a transport system at `12,000 p.m. which has, been granted to meet his personal expenditure while on duty. He is not in receipt of any daily allowance, from his employer., Solution is given on our website www.mkgeducation.com (Icon /Heading No.11: Books), , NOV – 2018 (OLD COURSE), Question 3(a), , (5 Marks), , Ms. Nandini, a resident individual, aged 48 years, is an assistant manager of Dye Hard Ltd. She was, appointed on 1st June, 2019 at a salary of ` 32,000 per month. During the previous year 2021-2022, she, received the following amounts from her employer., (i) Dearness allowance (10% of basic pay which forms part of salary for retirement benefits)., (ii) Bonus for the previous year 2020-2021 amounting to `32,000 was received on 01st October, 2021., (iii) Fixed Medical allowance of ` 20,000 for meeting medical expenditure., (iv) She was also reimbursed the medical bill of her father-in-law dependent on her amounting to, ` 3,000., (v) Ms. Nandini was provided;, • a laptop both for official and personal use. Laptop was acquired by the company on 1st June, 2019 at, ` 15,000., • a domestic servant at a monthly salary of ` 1,000 which was reimbursed by her employer., (vi) Dye Hard Ltd. allotted 500 equity shares in the month of December 2021 @ ` 150 per share against, the fair market value of ` 250 per share on the date of exercise of option by Ms. Nandini. The fair market, value was computed in accordance with the method prescribed under the Act., (vii) Professional tax ` 2,500 (out of which ` 1,800 was paid by the employer)., Compute the total Income of Ms. Nandini for the assessment year 2022-2023. (Assume that Ms., Nandini pays tax on the receipt basis), Solution is given on our website www.mkgeducation.com (Icon /Heading No.11: Books), , MAY – 2018 (NEW COURSE), Question 4, (10 Marks), Mr. Honey is working with a domestic company having a production unit in the U.S.A. for last 15 years. He, has been regularly visiting India for export promotion of company’s product. he has been staying in India for, at least 184 days every year., He submits the following information:
Page 881 :
Income Under The Head Salary, , 362, , Salary received outside India (for 6 months) `50,000 P.M., Salary received in India (for 6 months) `50,000 P.M., He has been given rent free accommodation in U.S.A. for which company pays `15,000 per month as rent,, but when he comes in India, he stays in the guest house of the company. During this period he is given free, lunch facility. During the previous year company incurred an expenditure of `48,000 on this facility., He has been provided a car of 2000 cc capacity in U.S.A. which is used by him for both office and private, purposes. The actual cost of the car is `8,00,000. But when he is in India, the car is used by him and the, members of his family only for personal purpose. The monthly expenditure of car is `5,000. His elder son is, studying in India for which his employer spends `12,000 per year where as his younger son is studying is, U.S.A. and stays in a hostel for which Mr. Honey gets `3,000 per month as combined allowance., The company has taken an accident insurance policy and a life insurance policy. During the previous year, the company paid premium of `5,000 and `10,000 respectively., Compute Mr. Honey’s taxable income from salary for the Assessment Year 2022-23., Solution is given on our website www.mkgeducation.com (Icon /Heading No.11: Books), , MAY – 2018 (OLD COURSE), Question 1, (10 Marks), Mrs. Babu, working as journalist with ABC Limited provides the following information for the year ended, 31-03-2022., Basic salary, `25,000 p.m., DA (50% of it is meant for retirement benefits), 50% Basic Pay, Own contribution to Recognized Provident fund (R.P.F.), `30,000, Employer’s contribution to R.P.F, 20% of Basic Salary, Interest credited in the R.P.F. @ 15%, `15,000, Arrears of rent received from ABC Limited, `69,000, Received interest `10,000 from Axis Bank Savings account during the year, and interest of `12,040 from the, debentures of M/s. Coal India ltd., She made payment through cheque `12,500 for Mediclaim Insurance Policy for her major daughter., , She had contributed `1,196 pm towards Atal Pension Yojana and `5,000 pm towards Sukanya Samridhi, account., M/s. ABC Limited has taken residential house of Mrs. Babu as Company’s guest house and later purchased, from her in the 2018 at market value for `75 lakhs. Purchased cost was only `10 lakhs in April, 2006., During August, 2021 Mrs. Babu had lost her gold chain and a diamond ring which she had purchased in, April, 2005 for `1,17,000 and market value of these two items were `2,50,000 and she has received, insurance compensation of `3,20,000 during Feb.,2022, Compute Total Income for the Asst. year 2022-23., (CII FOR 2005- 06 = 117, 2018 -19 = 280, and 2021-22=317), Solution is given on our website www.mkgeducation.com (Icon /Heading No.11: Books), , NOV – 2017, Question 2(b), (5 Marks), Mr. Srivastava, aged 40 years, a salaried employee of Nirja Ltd. was contributing to National Pension, Scheme `50,000 every year since 2018 and was claiming deduction under section 80CCD. In December, 2021, he opted out of the pension scheme and withdrew a lump sum amount of `2,00,000. Is the amount so, withdrawn taxable? If yes, how much is the taxable amount?, Solution is given on our website www.mkgeducation.com (Icon /Heading No.11: Books), Question 3, Mrs. Jaya is the marketing manager in XYZ limited. She gives you the following particulars:, Basic Salary, `65,000 p.m., , (10 Marks)
Page 882 :
Income Under The Head Salary, , 363, , Dearness Allowance, `22,000 p.m. (30% is for retirement benefits), Bonus, `17,000 p.m., Her employer has provided her with an accommodation on 1st April, 2021 at a concessional rent. The house, was taken on lease by XYZ Ltd. for `12,000 p.m. Mrs. Jaya occupied the house from 1st November, 2021., `4,800 p.m. is recovered from the salary of Mrs. Jaya., The employer gave her a gift voucher of `8,000 on her birthday. She contributes 18% of her salary (Basic, Pay+ 30% of DA) towards recognized provident fund and the company contributes the same amount., The company pays medical insurance premium to effect insurance on the health of Mrs. Jaya `18,000., Motor car owned by the employer (cubic capacity of engine 1.4 litres) provided to Mrs. Jaya from 1st, November, 2021 which is used for both official and personal purposes. Repair and running expenses of, `50,000 were fully met by the company. The motor car was self- driven by the employee., Compute the income chargeable to tax under the head “Salaries” in the hands of Mrs. Jaya for the, Assessment Year 2022-23. Also compute her tax liability for A.Y. 2022-23., Solution is given on our website www.mkgeducation.com (Icon /Heading No.11: Books), , MAY – 2017, Question 3(a), (8 Marks), Mr. Nambi, a salaried employee, furnishes the following details for the financial year 2021-22:, Particulars, `, Basic salary, 6,00,000, Dearness allowance, 3,20,000, Commission, 50,000, Entertainment allowance, 7,500, Profession Tax (of this, 50% paid by employer), 7,000, Health insurance premium paid by employer, 9,000, Gift voucher given by employer on his birthday, 12,000, Life insurance premium of Nambi Paid by employer, 34,000, Laptop provided for use at home . Actual cost of Laptop to employer, 30,000, [Children of the assessee are also using the Laptop at home], Employer -Company owns a Tata Nano car, which was provided to the assessee,, Both for official and personal use. No driver was provided. (Engine cubic capacity less than 1.6 litres), Annual credit card fees paid by employer [Credit card is not exclusively used for, 2,000, Official purposes; details of usage are not available], You are required to compute the income chargeable under the head "Salaries" for the assessment year 202223., Solution is given on our website www.mkgeducation.com (Icon /Heading No.11: Books), , NOV – 2016, Question 3(a), (4 Marks), (i) Mr. X, a citizen of India, serving in the Ministry of Finance in India and transferred to High Commission, of Australia on 15th March 2021. He did not come to India during the financial year 2021-22. His income, during the financial year 2021-22 is given here under:, Particulars, `, Salary from Govt. of India, 7,20,000, Foreign Allowances from Govt. of India, 6,00,000, Rent from a house situated at London, received in London, 3,60,000, Interest accrued on National Saving Certificate during the year 2021-22, 45,000, Compute The Gross Total Income of Mr. X for the Assessment year 2022-23., Solution is given on our website www.mkgeducation.com (Icon /Heading No.11: Books)
Page 883 :
Income Under The Head Salary, , 364, , Question 5(a), (4 Marks), (ii) Compute the amount of LTC Exemption in the following cases with reference to the provision under, Income Tax Act, 1961:, (a) Mr. X went on a holiday on 09.09.2021 to Mysore with his wife and 3 children - one daughter born, on 02.02.2014 and twin sons born on 05.05.2016. The total cost of travel was `80,000. The ticket, cost for Mr. X and his wife was `50,000 and for all three children was `30,000.The Employer, reimbursed total ticket cost `80,000., (b) In the above case (a), if among his 3 children the twin sons born on 02.02.2014 and the daughter was, born on 05.05.2016, what shall be the exemption?, Solution is given on our website www.mkgeducation.com (Icon /Heading No.11: Books), , MAY – 2016, Question 1(a), (10 Marks), Mr. Vinod Kumar, resident, aged 62, furnishes the following information pertaining to the year ended, 31.03.2022:, (`), (i) Pension receives (Net of TDS), 6,27,000, (ii) Short-term capital gains (from sale of listed shares), 65,000, (iii) Long-term capital gains (from sale of listed shares), 1,24,000, (iv) Interest on fixed deposit from bank, 1,60,000, (v) Pertaining to consultancy services provided by him :, Gross receipts, 12,60,000, Expenses:, Rent for premises, 1,44,000, Salary of P.A., 1,20,000, Stenographer's salary, 1,00,000, Business Development expenditure, 91,000, Conveyance, 3,00,000, (vi) Contribution to PPF, 1,10,000, (vii) Premium on life insurance policy taken on 10.01.2022 (sum assured `5,00,000), 60,000, (viii) Mediclaim Insurance Premium for self(paid otherwise than by cash), 27,000, Preventive health checkup expenses (in cash), 6,000, (ix) Donation given in cash to a charitable trust registered under Section 12AA, 14,000, (eligible for deduction u/s 80G) of the Income-Tax Act, 1961, (x) Interest received from Post Office Savings A/c., 18,000, Additional information:, • TDS from pension, 25,000, • 1/4th of conveyance expenses is estimated for personal use., Compute the total income of the assessee for the assessment year 2022-23, under proper heads of income., Listed share were sold in recognized stock exchange., Solution is given on our website www.mkgeducation.com (Icon /Heading No.11: Books), , NOV – 2015, Question 1(a)., (10 Marks), Mrs. X provides the following information for the financial year ending 31.03.2022. Compute her total, income and tax payable thereon for AY 2022-2023 as per Income Tax Act 1961., Income / Receipts:, (1) Salary from M/s. XYZ, - `60,000 per month (joined from 1st March, 2021)., (2) She is in receipt of HRA, `15,000 per month and also educational allowance of `1,500 per month for all, the three of her children., (3) She bought a truck on 01.08.2021 and has been letting it on hire. She does not maintain books of account, for this business. But she declares for income tax purpose, that she is earning net income of `11,000 per, month from this business.
Page 884 :
Income Under The Head Salary, , 365, , (4) She received `8,500 as interest on Post Office Savings Bank Account., (5) She received `25,000 as interest from Company Deposits., (6) Amounts withdrawn from National Savings Scheme (Principal `10,000 & Interest `25,000), Expenses / Payments:, (1) Interest payable to bank `1,000 per month on loan for the purchase of truck., (2) Total interest paid to bank for loan borrowed for investing in company deposits is `5,000., (3) Rent paid for residence is `18,000 per month., (4) Tuition fees paid for the year 2021-22 for her three children is `50,000, `30,000 and `20,000, respectively, to educational institution situated in India., (5) Medical insurance premium for her and for her husband is `30,000 (paid by cheque) and `25,000 (paid, by cash) respectively., (6) She has deposited during the year, in 5 year Post Office Recurring Deposit Scheme `20,000., Solution is given on our website www.mkgeducation.com (Icon /Heading No.11: Books), , MAY – 2015, Question 4(a)., (8 Marks), From the following details furnished by Mr. Dinesh, a marketing manager of XL Corporation Ltd., Delhi., Compute the gross total income for the Assessment Year 2022-23., Particulars, Amount, (`), Salary including Dearness Allowance, 6,50,000, Conveyance allowance of 900 p.m., 10,800, Bonus, 50,000, Salary of servant provided by the employer, 48,000, Bills paid by the employer for gas, electricity and water provided free of cost at the, residence of Mr. Dinesh, 82,000, Dinesh purchased a flat in a co-operative housing society in Dwarka, Delhi for self occupation for, `35,00,000 in April 2017, which was finance by a loan from Bank of India of `20,00,000 @ 11% interest, and his own savings of `5,00,000 and a deposit of `10,00,000 from Bank of Baroda, to whom he let out his, another house in Rohini, Delhi on lease for ten years. The rent payable by Bank of Baroda is `35,000 per, month. Assessee has submitted certificate of interest. Other relevant particulars are given below:, (i) Municipal taxes paid by Dinesh for his flat in Dwarka are `18,000 per annum and for his house in Rohini, are `12,000 per annum., (ii) Principal loan amount outstanding as on 01.04.2021 was `18,50,000., (iii) He also paid `8,000 towards insurance of both the houses., (iv) In the financial year 2020-21, he had gifted `40,000 each to his wife and minor son. The gifted amounts, were advanced to Mr. Sandeep, who is paying interest @ 18% per annum., (v) Mr. Dinesh's son is studying in a school run by the employer company throughout the financial year, 2021-22 The education facility was provided free of cost. The cost of such education in similar school is, `2,500 per month., (vi) Dinesh also received gifts of `45,000 each from his two friends during the previous year 2021-22., Solution is given on our website www.mkgeducation.com (Icon /Heading No.11: Books), , NOV – 2014, Question 4(a), (8 Marks), Mr. X an employee of XYZ Co. Ltd. at Mumbai and covered by Payment of Gratuity Act, retires at the age, of 64 years on 31.12.2021 after completing 33 years and 7 months of service. At the time of retirement, his, employer pays `20,51,640 as Gratuity and `6,00,000 as accumulated balance of Recognised Provident fund., He is also entitled for monthly pension of `8,000. He gets 75% of pension Commuted for `4,50,000 on 1st, February, 2022., Determine the salary chargeable to tax for Mr. X for the Assessment Year 2022-23 with the help of, following information:, `
Page 885 :
Income Under The Head Salary, Basic Salary (`80,000 x 9), Bonus, House Rent Allowance (`15,000 x 9), Rent paid by Mr. X (`10,000 x 12), Employer contribution towards Recognized Provident Fund, Professional Tax paid by Mr. X, Note: Salary and Pension falls due on the last day of each month., Solution is given on our website www.mkgeducation.com (Icon /Heading No.11: Books), , 366, 7,20,000, 36,000, 1,35,000, 1,20,000, 1,10,000, 2,000, , MAY – 2014, Question 1(a)., (10 Marks), From the following details compute the total income of Mr. X, A resident individual aged 54 years for the, `, year ended 31.03.2022. Tax payable need not be calculated., 1. Salary including Dearness Allowance, 5,00,000, 2. Bonus, 15,000, 3. Salary to servant provided by Employer, 12,000, 4. Free gas, electricity and water provided by employer, 14,500, 5 Cost of Laptop provided by the employer, 40,000, (Used both for official and personal purposes), Following additional information is provided:, (1) Mr. X purchased a flat in a Cooperative Housing Society in Delhi for `10,75,000 in April, 2017 by, taking loan from State Bank of India amounting to `5,00,000 @ 15% per annum interest, ` 65,000 from his, own savings and a deposit from a Nationalized Bank to whom this flat was given on lease for 10 years at a, monthly lease rental of ` 5,500. The outstanding amount of loan is `1,60,000., (2) Municipal Taxes paid by Mr. X ` 4,500 p.a., (3) Insurance in respect of the said flat `1,275, (4) Mr. X earned a profit of `15,000 in shares speculation business and incurred a loss of `20,200 in, speculation business of cotton., (5) In the year 2019-20, he had gifted ` 50,000 to his wife and ` 30,000 to his son who was aged 11 years, then. These amounts were advanced to Mr. Mohan @ 15% per annum interest., (6) Mr. X received a gift of ` 25,000 each from his four friends on the occasion of his birthday., (7) He contributed ` 10,500 to Public Provident Fund and `6,000 to Unit Linked Insurance plan., (8) He deposited ` 60,000 in tax saver deposit with a Nationalised Bank in the name of his married son., (9) He has taken a policy on life for his married daughter on 01.04.2021 and paid a premium of ` 25,000., The sum assured for policy is `2,00,000., Solution is given on our website www.mkgeducation.com (Icon /Heading No.11: Books), , NOV – 2013, Question 3(a)., (8 Marks), From the following details, find out the salary chargeable to tax of Mr. X for the assessment year 2022-23:, Mr. X is a regular employee of ABC Ltd. in Mumbai. He was appointed on 01.03.2021 in the scale of, 25,000-2,500-35,000. He is paid dearness allowance (which forms part of salary for retirement benefits) @, 15% of basic pay and bonus equivalent to one and a half month’s basic pay as at the end of the year. He, contributes 18% of his salary (basic pay plus dearness allowance) towards recognized provident fund and, the Company contributes the same amount., He is provided free housing facility which has been taken on rent by the Company at `15,000 per month. He, is also provided with following facilities:, (i), The monthly salary of `2,000 of a house keeper is reimbursed by the Company., (ii), He is getting telephone allowance @ `1,000 per month., (iii) A gift voucher of `4,700 was given on the occasion of his marriage anniversary., (iv), The Company pays medical insurance premium to effect an insurance on the health of Mr. X, `12,000.
Page 886 :
Income Under The Head Salary, , 367, , (v), , Motor car running and maintenance charges fully paid by employer of `36,600. (The motor car is, owned and driven by Mr. X. The engine cubic capacity is below 1.60 litres. The motor car is used for, both official and personal purpose by the employee.), (vi) Value of free lunch provided during office hours is `2,200., Solution is given on our website www.mkgeducation.com (Icon /Heading No.11: Books), , NOV – 2012, Question No. 6(a), (8 Marks), Discuss whether the following receipts are taxable and also indicate the head of income under which the, same is taxable:, (i), Bonus shares received by equity shareholder and preference shareholder., (ii), Loan advanced by a company in which public are not substantially interested to a person holding, 15% of the beneficial ownership of the share capital of the company., (iii) Medical allowance received by an employee, the entire amount of which has been spent by him, for medical treatment., (iv), Receipt of cash gift of `60,000/- from a friend on the occasion of wedding anniversary., (v), Gift of a plot of land given to a chartered accountant by one of his clients. The chartered, accountant has been fully compensated for his services and this gift has been given in, appreciation of his personal qualities., (vi), A lawyer closed down his profession. Subsequently he accepted a case on the insistence of his, friend but advised his friend to pay the fee payable to him directly to a charitable trust., (vii) Payment from unrecognised provident fund at the time of retirement which consists of, employee’s contribution, employer’s contribution and interest on both contributions., Solution is given on our website www.mkgeducation.com (Icon /Heading No.11: Books), , MAY – 2012, Question 1 (a), (4 Marks), Ms. X, employed in a private sector company, furnishes following information for the year ended, 31.03.2022, `, Income from salary (computed), 3,45,000, Bank interest on savings bank account, 15,000, Tax on non-monetary perquisite paid by employer, 20,000, Amount contributed by her during the year of given below:, Contribution to Recognized Provident Fund, 60,000, Health Insurance Premium –on self (paid by crossed cheque), 7,000, Medical expenditure for dependent sister with disability, 20,000, Compute the total income of Ms. X for the Assessment Year 2022-23., Solution is given on our website www.mkgeducation.com (Icon /Heading No.11: Books), Question 3, (4 Marks), Mr. X is Production Manager of XYZ Ltd. From the following details, compute the Total Income for the, Assessment Year 2022-23., Basic salary, `50,000 per month, Dearness allowance, 40% of basic salary, Transport allowance(for commuting between, `3,000 per month, Place of residence and office), Motor car running and maintenance charges fully paid by employer, ` 60,000, The motor car is owned by the company and driven by the employee. The engine cubic capacity is above, 1.60 litres. The motor car is used for both official and personal purpose by the employee., Expenditure on accommodation in hotels while touring on official, `80,000, duties met by the employer, Loan from recognized provident fund (maintained by the employer), `60,000
Page 887 :
Income Under The Head Salary, , 368, , Lunch provided by the employer during office hours., Cost to the employer, `24,000, Computer (cost `35,000) kept by the employer in the residence of, Mr. X from 01.06.2021, Mr. X made the following payments:, Medical insurance premium: Paid in cash, ` 4,800, Paid by account payee crossed cheque, `15,200, Solution is given on our website www.mkgeducation.com (Icon /Heading No.11: Books), , NOV – 2011, Question 2, (8 Marks), Mr. X, employed as Production Manager in B Ltd., furnishes you the following information for the year, ended 31.03.2022:, (i) Basic salary upto 31.10.2021., `50,000 p.m., Basic salary from 01.11.2021., `60,000 p.m., Note: Salary is due and paid on the last day of every month., (ii) Dearness allowance @ 40% of basic salary., (iii) Bonus equal to one month salary. Paid in October 2021 on basic salary plus dearness allowance, applicable for that month., (iv) Contribution of employer to recognized provident fund account of the employee @ 16% of basic salary., (v) Professional tax paid `3,000 of which `2,000 was paid by the employer., (vi) Facility of laptop and computer was provided to Mr. X for both official and personal use. Cost of laptop, ` 45,000 and computer ` 35,000 were acquired by the company on 01.12.2021., (vii) Motor car owned by the employer (cubic capacity of engine exceeds 1.60 litres) provided to the, employee from 01.11.2021 meant for both official and personal use. Repair and running expenses of, `45,000 from 01.11.2021 to 31.03.2022 were fully met by the employer. The motor car was self-driven by, the employee., (viii) Leave travel concession given to employee, his wife and three children (one daughter aged 7 and twin, sons- aged 3). Cost of air tickets (economy class) reimbursed by the employer `30,000 for adults and, `45,000 for three children. Mr. X is eligible for availing exemption this year to the extent it is permissible in, law., Compute the salary income and also tax liability of Mr. X for the assessment year 2022-23., Solution is given on our website www.mkgeducation.com (Icon /Heading No.11: Books), Question 3, (8 Marks), Mrs. X (aged 40 years) is working with ABC Company Ltd., a manufacturer of tyres based at Mumbai, has, received the following payments during the financial year 2021-22 from her employer:, Basic salary, :, `60,000 per month., Dearness allowance :, 40% of basic salary., Her employer has taken on rent her own house on a monthly rent of `15,000 and the same has been provided, for residence of Mrs. X. Company is recovering `2,000 per month as rent of house., Mrs. X has further furnished the following details:, (i) She has paid professional tax of `6,000 during financial year 2021-22., (ii) She is owning only one house and payment of interest of `1,75,000 and principal of `1,00,000 was, made for housing loan taken from SBI for purchase of house., (iii) She has also taken a loan of `2,00,000 from her employer for study of her son. SBI rate for such loan is, 10%. Her employer has recovered `10,000 as interest from her salary for such loan during the year., Compute Total Income and Tax Liability for Assessment Year 2022-23., Solution is given on our website www.mkgeducation.com (Icon /Heading No.11: Books), Question 4, (8 Marks), Mr. X a senior citizen retired from the services of M/s Y Ltd. on 31.01.2022 after completing service of 30, years and one month. He received the following on his retirement:
Page 888 :
Income Under The Head Salary, , 369, , (i) Gratuity `6,00,000. He was covered under the Payment of Gratuity Act, 1972., (ii) Leave encashment of `3,30,000 for 330 days leave balance in his account. He was credited 30 days leave, for each completed year of service., (iii) He purchased one motor car from the company on 31.03.2022. This car was purchased on 01.07.2018, by the company for `5,00,000. It was put use by the company on the same date. The car was sold by the, company to Mr. X for `2,00,000. Company depreciates the vehicles at the rate of 15% on Straight Line, Method., (iv) An amount of `3,00,000 as commutation of 2/3 of his pension., (v) Company presented him a gift voucher worth `6,000 on his retirement., (vi) His colleagues also gifted him a Television (LCD) worth `1,50,000 from their own contribution., Following are the other particulars:, (i) He has drawn a Basic Salary of `20,000 and 50% Dearness allowance per month for the period from, 01.04.2021 to 31.01.2022., (ii) Received pension of `5,000 per month for the period 01.02.2022 to 31.03.2022 after commutation of, pension., Compute his total income and tax liability from the above for Assessment Year 2022-23., Solution is given on our website www.mkgeducation.com (Icon /Heading No.11: Books), , MAY – 2011, Question 3, (5 Marks), Mr. X employed in ABC Co. Ltd. as Finance Manager gives you the list of perquisites provided by the, company to him for the entire financial year 2021-22:, (i), Domestic servant was provided at the residence of Mr. X. Salary of domestic servant is `1,500 per, month. The servant was engaged by him and the salary is reimbursed by the company (employer)., In case, the company has employed the domestic servant, what is the value of perquisite?, (ii), Free education was provided to his two children Mr. S and Mr. A in a school maintained and owned, by the company. The cost of such education for Mr. S is computed at `900 per month and for Mr. A, at `1,200 per month. No amount was recovered by the company for such education facility from Mr., X., (iii) The employer has provided movable assets such as television refrigerator and air conditioner at the, residence of Mr. X. The actual cost of such assets provided to the employee is `1,10,000., (iv), A gift voucher worth `10,000 was given on the occasion of his marriage anniversary. It is given by, the company to all employee above certain grade., State the taxability or otherwise of the above said perquisites and compute the total value of taxable, perquisites., Solution is given on our website www.mkgeducation.com (Icon /Heading No.11: Books), , NOV – 2010, Question 5, (5 Marks), From the following details find out the salary chargeable to tax for the assessment year 2022-23., Mr. X is a regular employee of ABC & Co. in Gurgaon. He was appointed on 01.01.2021 in the scale of, 20000-1000-30000. He is paid 10% D.A. (forms part for retirement benefits salary) & Bonus equivalent to, one month pay. He contributes 15% of his pay and D.A. towards his recognized provident fund and the, company contributes the same amount., He is provided free housing facility which has been taken on rent by the company at `10,000 per month. He, is also provided with following facilities., (i), Facility of laptop costing `50,000., (ii), The monthly salary of `1,000 of a house keeper is reimbursed by the company., (iii) A gift voucher of `10,000 on the occasion of his marriage anniversary., (iv), Conveyance allowance of `1,000 per month is given by the company towards actual reimbursement., (v), He is provided personal accident policy for which premium of `5,000 is paid by the company., (vi), He is getting telephone allowance @ `500 per month.
Page 889 :
Income Under The Head Salary, , 370, , (vii) Company pays medical insurance premium of his family of `10,000., Solution is given on our website www.mkgeducation.com (Icon /Heading No.11: Books), Question 7, (4 Marks), Mr. X, an Accounts Manager, has retired from JK Ltd. on 15.01.2022 after rendering services for 30 years 7, months. His salary is `25,000/- p.m. upto 30.09.2021 and ` 27,000 p.m. thereafter. He also gets `2,000/p.m. as dearness allowance (55% of it is a part of salary for computing retirement benefits)., He is not covered by the payments of Gratuity Act, 1972., He has received ` 8 Lacs as gratuity from the employer company., Solution is given on our website www.mkgeducation.com (Icon /Heading No.11: Books), Question 5, (4 Marks), Mr. X, Marketing Manager of KL Ltd. based at Mumbai furnishes you the following information for the, year ended 31.03.2022:, Basic salary, `1,00,000 per month, Dearness allowance `50,000 per month (forming part for retirement benefit salary), Bonus, 2 Months basic salary, Contribution of employer to Recognized Provident Fund @ 15% of basic salary plus Dearness allowance, Rent free unfurnished accommodation was provided by the company at Mumbai (accommodation owned by, the company)., `, (i), Recognized Provident Fund contribution made by Mr. X., 1,50,000, (ii) Health insurance premium for his family paid by cheque., 20,000, (iii) Health insurance premium in respect of parents (senior citizens) paid by cheque., 28,000, (iv) Medical expenses of dependent brother with ‘severe disability’ (covered by Section, 60,000, 2(o) of National Trust of Welfare of Persons with Autism, Cerebral Palsy, Mental, Retardation and Multiple Disabilities Act, 1999)., (v), Interest on loan taken for education of his son studying B.com (full-time) in a, 24,000, recognized college., (vi) Interest on loan taken for education of a student for whom Mr. X is the legal, 20,000, guardian for pursuing B.Sc. (Physics) (full-time) in a recognized university., Compute the Total Income of Mr. X for the Assessment Year 2022-23., Solution is given on our website www.mkgeducation.com (Icon /Heading No.11: Books), Question 6, (2 Marks), Allowance received by an employee working in a transport system at `10,000 per month to meet his, personal expenditure while in duty. He is not receiving any daily allowance., Solution is given on our website www.mkgeducation.com (Icon /Heading No.11: Books), Question 6, Amount withdrawn from Public Provident Fund as per relevant rules., Solution is given on our website www.mkgeducation.com (Icon /Heading No.11: Books), , (2 Marks), , Question 6, (2 Marks), Telephone provided at the residence of employee and the bill aggregating to `25,000 paid by the employer., Determine the perquisite value taxable in the hands of employee., Solution is given on our website www.mkgeducation.com (Icon /Heading No.11: Books), Question 7, (4 Marks), AB Co. Ltd., allotted 1000 sweat equity shares to Mr. X in June 2021. The shares were allotted at `200 per, share as against the fair market value of `300 per share on the date of exercise of option by the allottee viz., Mr. X. The fair market value was computed in accordance with the method prescribed under the Act., (i) What is the perquisite value of sweat equity shares allotted to Mr. X?, (ii) In the case of subsequent sale of those shares by Mr. X, what would be the cost of acquisition of those, sweat equity shares?
Page 890 :
Income Under The Head Salary, , 371, , Solution is given on our website www.mkgeducation.com (Icon /Heading No.11: Books), , MAY – 2010, Question 3, (6 Marks), Following benefits have been granted by ABC Software Ltd. to one of its employees Mr. X:, (i) Housing loan @ 6% per annum. Amount outstanding on 01.04.2021 is ` 6,00,000. Mr. X pays `12,000, per month, on 5th of each month., (ii) Air-conditioners purchased 4 years back for ` 2,00,000 have been given to Mr. X for ` 90,000., Compute the chargeable perquisite in the hands of Mr. X for the Assessment Year 2022-23., The lending rate of State Bank of India as on 01.04.2021 for housing loan may be taken as 10.50%., Solution is given on our website www.mkgeducation.com (Icon /Heading No.11: Books), , NOV – 2009, Question 1, (10 Marks), From the following details compute the total income of Mr. X of Delhi and tax liability for the Assessment, Year 2022-23:, `, Salary including dearness allowance, 3,35,000, Bonus, 11,000, Salary of servant provided by the employer, 12,000, Rent paid by Mr. X for his accommodation, 49,600, Bills paid by the employer for gas, electricity and water provided free of cost, 11,000, at the above flat, Mr. X was provided with company’s car engine capacity 1.6 litre (self driven) also for personal use and it, is not possible to determine expenditure on personal use and all expenses were borne by the employer., Mr. X purchased a flat in a Co-operative Housing Society for `4,75,000 in April, 2017, in Delhi, which, was financed by a loan from Life Insurance Corporation of India of `1,60,000 @ 15% interest, his own, savings of `65,000 and a deposit from a nationalised bank for `2,50,000 to whom this flat was given on, lease for ten years. The rent payable was `20,000 per month. The following particulars are relevant:, `, (a) Municipal taxes paid (per annum), 4,300, (b) Society charges for passage lights, watchman’s salary ( per annum), 1,900, (c) Insurance, 860, (d) He earned `2,700 in share speculation business and lost `4,200 in cotton speculation business., (e) In the year 2019-20 he had gifted `30,000 to his wife and `20,000 to his son who was aged 11. The, gifted amounts were advanced to Mr. Rajesh, who was paying interest @ 19% per annum., (f) Mr. X received a gift of `25,000 each from four friends., (g) He contributed `5,600 to public provident fund and `4,000 to Unit Linked Insurance Plan., (h) He received national award for humanitarian work from the Central Government in the form of a, land whose fair market value is `5,00,000 as on 31st March, 2022., Solution is given on our website www.mkgeducation.com (Icon /Heading No.11: Books), , NOV – 2008, Question 3, (6 Marks), Mr. X is an area manager of M/s N. Steels Co. Ltd. During the financial year 2021-22, he gets the following, emoluments from his employer:, Basic Salary, Up to 31.08.2021, ` 20,000 p.m., From 01.09.2021, ` 25,000 p.m., Transport allowance, ` 2,000 p.m., Contribution to Recognised Provident Fund by employer and employee 15% of basic salary (each)
Page 891 :
Income Under The Head Salary, Children education allowance, ` 500 p.m. for two children, City compensatory allowance, ` 300 p.m., Hostel expenses allowance, ` 380 p.m. for two children, Tiffin allowance (actual expenses ` 3,700), ` 5,000 p.a., Tax paid on employment, ` 2,500, Compute Tax Liability of Mr. X for the Assessment Year 2022-23., Solution is given on our website www.mkgeducation.com (Icon /Heading No.11: Books), , 372
Page 892 :
Section 115BAC, , 373, , TAXABILITY OF INDIVIDUAL AND HUF, AS PER SECTION 115BAC, Question 1: Write a note on taxability as per section 115BAC., Answer: An individual or a Hindu undivided family shall at the option of such person be computed at, the rate of tax given below:, If total Income upto `2,50,000, Nil, On next `2,50,000, 5%, On next `2,50,000, 10%, On next `2,50,000, 15%, 20%, On next `2,50,000, On next `2,50,000, 25%, On Balance amount, 30%, , Option has been given only to individual and HUF to compute tax at the above rates but it is subject to many, conditions given under section 115BAC i.e. the assessee has to forgo many of the exemptions and, deductions., In brief as per section 115BAC, the assessee has to forgo the following exemptions and deductions., 1. Higher Slab for senior citizen / very senior citizen is not allowed., 2. Rebate under section 87A is allowed., 3. Tax rate under section 111A, 112, 112A shall be same as in case of normal category., 4. Section 24 (b) in case of property under section 23(2) i.e. no interest shall be allowed in case of a self, occupied house., 5. Deductions under chapter VIA i.e. 80C to 80U. However Deduction shall be allowed u/s 80CCD (2) &, 80JJAA, 6. Section 10(32) – Exemption of 1,500 in case of minor children., 7. Section 57(iia) – Deductions of Family Pension, 8. Section 32(1) (iia) – Additional depreciation., 9. Section 35 (1) (ii) – Donation to Notified scientific research association (100%), 10. Section 35 (1) (iia) – Donation to Indian companies for scientific research (100%), 11. Section 35 (1) (iii) – Donation for social science or statistical research (100%), 12. Section 35 (2AA) – Donation to National laboratory or Indian Institute of Technology or other notified, Institution (100%), 13. Section 35AD – Specified business, 14. Section 10(5) – Leave travel concession (LTC), 15. Section 10(13A) – House Rent allowance (HRA), 16. Section 10(14) – Special allowances other than those prescribed*, *(However the following allowances shall be allowed), Special Allowances, Personal Allowances, (a) Transport Allowance granted to an employee who is blind or deaf and dumb or orthopedically, handicapped to meet expenditure for the purpose of commuting between place of residence and, place of duty., Official Allowances, (a)Any Allowance granted to meet the cost of travel on tour or on transfer., (b) Daily Allowance to meet the ordinary daily charges incurred by an employee on account of absence, from his normal place of duty, (c) Conveyance Allowance granted to meet the expenditure on conveyance in performance of duties of, an office., 17. Section 10AA – Special Economic Zone, 18. Section 16(ia),16(ii),16(iii), 19. Section 10(17) – Allowances to MPs and MLAs (Not Covered in syllabus)
Page 893 :
Section 115BAC, , 374, , 20.Carried forward loss or depreciation relating to the above shall not be allowed and shall lapse., 21. Interest u/s 24(b) shall be allowed for all other category except self occupied house. Loss of house, property shall not be allowed to be set off from incomes from other heads and shall lapse., 22. Exemption or deduction for allowances or perquisites under any other law shall not be allowed., 23. Current year depreciation except additional depreciation shall be allowed., As per section 115BAC (5). An individual and HUF shall exercise their option at the time of filing of return, of income on year to year basis but a person having income under business/profession who has opted for this, scheme shall be allowed to come out only once and after that he can never opt for this scheme. If, subsequently such person donot have income under the head business/profession, again such person can, exercise such option on year to year basis., If assessee has opted for section 115BAC, provisions of AMT shall not apply. Also tax credit relating to, earlier years shall lapse., Original section 115BAC, 115BAC. (1) Notwithstanding anything contained in this Act but subject to the provisions of this Chapter,, the income-tax payable in respect of the total income of a person, being an individual or a Hindu undivided, family, for any previous year relevant to the assessment year beginning on or after the 1st day of April,, 2021, shall, at the option of such person, be computed at the rate of tax given in the following Table, if the, conditions contained in sub-section (2) are satisfied, namely:—, TABLE, , Sl. No., , Total income, , Rate of tax, , (1), , (2), , (3), , 1., , Upto Rs. 2,50,000, , Nil, , 2., , From Rs. 2,50,001 to Rs. 5,00,000, , 5 per cent, , 3., , From Rs. 5,00,001 to Rs. 7,50,000, , 10 per cent, , 4., , From Rs. 7,50,001 to Rs. 10,00,000, , 15 per cent, , 5., , From Rs. 10,00,001 to Rs. 12,50,000, , 20 per cent, , 6., , From Rs. 12,50,001 to Rs. 15,00,000, , 25 per cent, , 7., , Above Rs. 15,00,000, , 30 per cent:, , Provided that where the person fails to satisfy the conditions contained in sub-section (2) in any previous, year, the option shall become invalid in respect of the assessment year relevant to that previous year and, other provisions of this Act shall apply, as if the option had not been exercised for the assessment year, relevant to that previous year:, Provided further that where the option is exercised under clause (i) of sub-section (5), in the event of, failure to satisfy the conditions contained in sub-section (2), it shall become invalid for subsequent, assessment years also and other provisions of this Act shall apply for those years accordingly., (2) For the purposes of sub-section (1), the total income of the individual or Hindu undivided family shall, be computed,—, (i) without any exemption or deduction under the provisions of clause (5) or clause (13A) or prescribed, under clause (14) (other than those as may be prescribed for this purpose) or clause (17) or clause (32),, of section 10 or section 10AA or section 16 or clause (b) of section 24 (in respect of the property referred, to in sub-section (2) of section 23) or clause (iia) of sub-section (1) of section 32 or section, 32AD or section 33AB or section 33ABA or sub-clause (ii) or sub-clause (iia) or sub-clause (iii) of sub-
Page 894 :
Section 115BAC, , 375, , section (1) or sub-section (2AA) of section 35 or section 35AD or section 35CCC or clause (iia) of section, 57 or under any of the provisions of Chapter VI-A other than the provisions of sub-section (2) of section, 80CCD or section 80JJAA;, (ii) without set off of any loss,—, (a) carried forward or depreciation from any earlier assessment year, if such loss or depreciation is, attributable to any of the deductions referred to in clause (i);, (b) under the head "Income from house property" with any other head of income;, (iii) by claiming the depreciation, if any, under any provision of section 32, except clause (iia) of subsection (1) of the said section, determined in such manner as may be prescribed; and, (iv) without any exemption or deduction for allowances or perquisite, by whatever name called, provided, under any other law for the time being in force., (3) The loss and depreciation referred to in clause (ii) of sub-section (2) shall be deemed to have been, given full effect to and no further deduction for such loss or depreciation shall be allowed for any, subsequent year:, Provided that where there is a depreciation allowance in respect of a block of assets which has not been, given full effect to prior to the assessment year beginning on the 1st day of April, 2021, corresponding, adjustment shall be made to the written down value of such block of assets as on the 1st day of April, 2020, in the prescribed manner, if the option under sub-section (5) is exercised for a previous year relevant to the, assessment year beginning on the 1st day of April, 2021., (4) In case of a person, having a Unit in the International Financial Services Centre, as referred to in subsection (1A) of section 80LA, which has exercised option under sub-section (5), the conditions contained, in sub-section (2) shall be modified to the extent that the deduction under section 80LA shall be available, to such Unit subject to fulfilment of the conditions contained in the said section., Explanation.—For the purposes of this sub-section, the term "Unit" shall have the meaning assigned to it in, clause (zc) of section 2 of the Special Economic Zones Act, 2005 (28 of 2005)., (5) Nothing contained in this section shall apply unless option is exercised in the prescribed manner by the, person,—, (i) having income from business or profession, on or before the due date specified under sub-section (1), of section 139 for furnishing the returns of income for any previous year relevant to the assessment year, commencing on or after the 1st day of April, 2021, and such option once exercised shall apply to, subsequent assessment years;, (ii) having income other than the income referred to in clause (i), alongwith the return of income to be, furnished under sub-section (1) of section 139 for a previous year relevant to the assessment year:, Provided that the option under clause (i), once exercised for any previous year can be withdrawn only once, for a previous year other than the year in which it was exercised and thereafter, the person shall never be, eligible to exercise option under this section, except where such person ceases to have any income from, business or profession in which case, option under clause (ii) shall be available., Illustration 1: Mr. X is employed in Central Government., Basic Pay, `1,00,000 per month, House Rent allowance, `20,000 per month, Entertainment allowance received, `1,000 per month, Employer allowed him leave travel concession from Delhi to Goa and paid `7,000 per ticket for 4 members, of family including Mr. X but economy class fare is `3,000 per ticket., He has one house which is self-occupied. He has taken loan of `35,00,000 @ 12% p.a. from State Bank on, 01-05-2021 for purchase of this house. He has deposited the certificate of interest., He invested `1,00,000 in NSC and paid `20,000 medi-claim premium by cheque. Donated `40,000 to, PMCARES Funds by cheque.
Page 895 :
Section 115BAC, Compute his tax liability in the following situations, (i) Not opted for section 115BAC, (ii) Opted for section 115BAC, Solution:, (i) Not opted for section 115BAC, Computation of Tax Liability of Mr. X, Income under the head salary, Basic Salary (1,00,000 x 12 ), House Rent allowance (fully taxable since he owns his own house) (20,000 x 12 ), Leave Travel allowance (4,000 x 4), Entertainment allowance (1,000 x 12), Gross salary, Less: Deduction u/s 16(ia), Less: Deduction u/s 16(ii), Least of the followings:, (i) 20% of basic salary i.e. 12,00,000 x 20% = 2,40,000, (ii) 5,000, (iii)The actual allowance received = 12,000, Income under the head salary, , 376, , 12,00,000.00, 2,40,000.00, 16,000.00, 12,000.00, 14,68,000.00, (50,000.00), (5,000.00), , 14,13,000.00, , Income under the head house property (Self occupied house), Gross Annual Value, Less: Municipal Taxes paid, Net Annual Value, Less: Standard deduction, Less: Interest on capital borrowed, (35,00,000 x 12% x 11/12) = 3,85,000 but restricted to 2,00,000, Loss under the head house property, , (2,00,000.00), , Gross Total Income (14,13,000 -2,00,000), Less: Deductions u/s 80C, Less: Deductions u/s 80D, Less: Deductions u/s 80EEA (3,85,000-2,00,000=1,85,000 but restricted to), Less: Deductions u/s 80G (100%), Total Income, , 12,13,000.00, (1,00,000.00), (20,000.00), (1,50,000.00), (40,000.00), 9,03,000.00, , Calculation of Tax Liability, Tax on 9,03,000 at slab rate, Add: HEC @ 4%, Tax Liability, Rounded off u/s 288B, , Nil, Nil, Nil, Nil, (2,00,000.00), , 93,100.00, 3,724.00, 96,824.00, 96,820.00, , (ii) Opted for section 115BAC, Computation of Tax Liability of Mr. X, Income under the head salary, Basic Salary (1,00,000 x 12 ), House Rent allowance, Leave Travel allowance (7,000 x 4), Entertainment allowance (1,000 x 12), Gross salary, Income under the head salary, , 12,00,000.00, 2,40,000.00, 28,000.00, 12,000.00, 14,80,000.00, 14,80,000.00
Page 896 :
Section 115BAC, Income under the head house property (Self occupied house), Gross Annual Value, Less: Municipal Taxes paid, Net Annual Value, Less: Standard deduction, Less: Interest on capital borrowed, Income under the head house property, Gross Total Income, Less: Deductions u/s 80C to 80U, Total Income, Calculation of Tax Liability as per section 115BAC, Tax on `14,80,000 as per section 115BAC, Add: HEC @ 4%, Tax Liability, , 377, Nil, Nil, Nil, Nil, Nil, Nil, 14,80,000.00, Nil, 14,80,000.00, 1,82,500.00, 7,300.00, 1,89,800.00, , Illustration 2: Mr. X has income under the business profession 20,00,000 but no adjustment has been made, for the followings:, 1. Donation given `2,00,000 to approved scientific research association., 2. Unabsorbed depreciation of earlier years `1,75,000, 3. Unabsorbed depreciation relating to additional depreciation of earlier year `1,40,000, 4. Brought Forward loss of house property `3,00,000 of A.Y. 2020-21, His minor son has interest income `1,00,000, He has invested `40,000 in PPF., Compute his tax liability in the following situations, (i) Not opted for section 115BAC, (ii) Opted for section 115BAC, Solution: (i) Not opted for section 115BAC, Computation of Tax Liability of Mr. X, Income under the head Business/Profession, Business Income, 20,00,000.00, Less: Donation for approved scientific research association, (2,00,000.00), Less: unabsorbed depreciation, (1,75,000.00), Less: unabsorbed depreciation relating to additional depreciation, (1,40,000.00), Income under the head business profession, 14,85,000.00, Income under the head other sources, Interest Income of minor son, Less: exemption u/s 10(32), Income under the head other sources, Gross Total Income, Less: Deductions u/s 80C, Total Income, Calculation of Tax Liability, Tax on 15,43,500 at slab rate, Add: HEC @ 4%, Tax Liability, Rounded off u/s 288B, (ii) Opted for section 115BAC, Computation of Tax Liability of Mr. X, Income under the head Business/Profession, Business Income, Less: Donation for approved scientific research association, Less: unabsorbed depreciation, , 1,00,000.00, (1,500.00), 98,500.00, 15,83,500.00, (40,000.00), 15,43,500.00, 2,75,550.00, 11,022.00, 2,86,572.00, 2,86,570.00, , 20,00,000.00, Nil, (1,75,000)
Page 897 :
Section 115BAC, , 378, , Less: unabsorbed depreciation relating to additional depreciation, Income under the head business profession, Income under the head other sources, Interest Income of minor son, Less: exemption u/s 10(32), Income under the head other sources, Gross Total Income, Less: Deductions u/s 80C, Total Income, , Nil, 18,25,000.00, , Calculation of Tax Liability as per section 115BAC, Tax on 19,25,000 as per section 115BAC, 2,50,000, Nil, 2,50,000, 5%, 12,500, 2,50,000, 10%, 25,000, 2,50,000, 15%, 37,500, 2,50,000, 20%, 50,000, 2,50,000, 25%, 62,500, 4,25,000, 30%, 1,27,500, Add: HEC @ 4%, Tax Liability, , 1,00,000.00, Nil, 1,00,000.00, 19,25,000.00, Nil, 19,25,000.00, 3,15,000.00, , 12,600.00, 3,27,600.00, , Illustration 3: Mr. X is employed in ABC Ltd. and is getting basic pay ₹1,00,000 per month. Employer and, employee both have contributed @ 10% of salary to NPS. Mr. X has one house property which is let out @, ₹20,000 per month. He paid Municipal Tax ₹ 40,000 and interest on loan u/s 24(b) is ₹ 3,00,000. Compute, his tax liability in two situations, , (i), , Not opted for 115BAC, , (ii), , Opted for 115BAC, , Solution:, , Basic Pay 1,00,000 X 12, Employers Contribution to NPS, Gross Salary, Less: Standard Deduction 16(ia), Income under the head salary, , 12,00,000, 1,20,000, 13,20,000, (50,000), 12,70,000, , Income under the head house property, Gross Annual Value 20,000 X 12, , 2,40,000, , Less: Municipal Tax, , (40,000), , Net Annual Value, , 2,00,000, , Less: Standard Deduction @ 30% u/s 24(a), , (60,000), , Less: Interest u/s 24(b), , (3,00,000), , Loss from house property, , (1,60,000), , Gross Total Income, , 11,10,000, , Less: Deduction u/s 80CCD (1), , (1,20,000), , Less: Deduction u/s 80CCD (2), , (1,20,000), , Total Income, , 8,70,000
Page 898 :
Section 115BAC, , 379, , Computation of Tax Liability, , 2,50,000, , Nil, , 2,50,000, , 12,500, , 3,70,000, , 74,000, , 86,500, , Add: HEC @ 4%, , 3460, , Total, , 89,960, , (ii) Opted for 115BAC, Solution:, , Basic Pay 1,00,000 X 12, , 12,00,000, , Employers Contribution to NPS, Gross Salary, , 1,20,000, 13,20,000, , Less: Standard Deduction 16(ia) (standard deduction not allowed), Income under the head salary, , Nil, 13,20,000, , Income under the head house property, Gross Annual Value 20,000 X 12, , 2,40,000, , Less: Municipal Tax, , (40,000), , Net Annual Value, , 2,00,000, , Standard Deduction u/s 24(a), , (60,000), , Interest u/s 24(b) (Interest is allowed but no interest is allowed for self occupied house), , (3,00,000), , Loss from house property (Loss can be set off only from house property and not from other heads), (1,60,000), Gross Total Income (HP Loss not allowed to be set off), Less: Deduction u/s 80CCD(1) (Deduction u/s 80CCD(1) not allowed), , 13,20,000, Nil, , Less: Deduction u/s 80CCD(2), , (1,20,000), , Total Income, , 12,00,000, , Computation of Tax Liability u/s 115BAC, , 2,50,000, , Nil, , 2,50,000, , 5%, , 12,500, , 2,50,000, , 10%, , 25,000, , 2,50,000, , 15%, , 37,500, , 2,00,000, , 20%, , 40,000, , Add: HEC 4%, Total, , 1,15,000, 4,600, 1,19,600, , Section 115BAC, , If any person has opted section 115BAC, in that case highest rate of depreciation allowed shall be 40%, If any person has opted section 115BAC, no exemptions shall be allowed for free food or refreshments u/s, 17(2)(viii)/ Rule 3(7)(ii), Illustration 4: Mr. X, an individual set up a unit in Special Economic Zone (SEZ) in the financial year
Page 899 :
Section 115BAC, , 380, , 2017-18 for production of washing machines. The unit fulfills all the conditions of section 10AA of the, Income-tax Act, 1961. During the financial year 2019-20, he has also set up a warehousing facility in a, district of Tamil Nadu for storage of agricultural produce. It fulfills all the conditions of section 35AD., Capital expenditure in respect of warehouse amounted to ₹ 75 lakhs (including cost of land ₹ 10 lakhs)., The warehouse became operational with effect from 1st April, 2021 and the expenditure of ₹ 75 lakhs was, capitalized in the books on that date. Relevant details for the financial year 2021-22 are as follows:, Particulars, Profit of unit located in SEZ, , ₹, 40,00,000, , Export sales of above unit, , 80,00,000, , Domestic sales of above unit, , 20,00,000, , Profit from operation of warehousing facility (before considering deduction under, Section 35AD), , 1,05,00,000, , Compute income-tax (including AMT under Section 115JC) liability of Mr. X for Assessment Year 202223 both as per regular provisions of the Income-tax Act and as per section 115BAC for Assessment Year, 2022-23. Advise Mr. X whether he should opt for section 115BAC., Answer:, Computation of total income and tax liability of Mr. X for A.Y.2022-23, (under the regular provisions of the Income-tax Act, 1961), Particulars, Profits and gains of business or profession, , ₹, , Profit from unit in SEZ, , 40,00,000, , Less: Deduction u/s 10AA [See Note (1) below], , 32,00,000, , Business income of SEZ unit chargeable to tax, Profit from operation of warehousing facility, Less: Deduction u/s 35AD [See Note (2) below], , 8,00,000, 1,05,00,000, (65,00,000), , Business income of warehousing facility chargeable to tax, , 40,00,000, , Total Income, , 48,00,000, , Computation of tax liability (under the normal/regular provisions), Tax on ₹ 48,00,000, , 12,52,500, , Add: Health and Education cess@4%, , 50,100, , Total tax liability, , 13,02,600, , Computation of adjusted total income of Mr. X for levy of Alternate, Minimum Tax, Particulars, Total Income (computed above as per regular provisions of income tax), , ₹, 48,00,000, , Add: Deduction under section 10AA, , 32,00,000, 80,00,000, , Add: Deduction under section 35AD, , 65,00,000, , Less: Depreciation under section 32 On building @10% of ₹ 65 lakhs, , (6,50,000), , Adjusted Total Income, , Alternate Minimum Tax @ 18.5%, , 58,50,000, 1,38,50,000, , 25,62,250
Page 900 :
Section 115BAC, , 381, , Add: Surcharge@15% (since adjusted total income > ₹ 1 crore), , 3,84,338, 29,46,588, , Add: Health and Education cess@4%, , 1,17,863, 30,64,451, , Tax liability u/s 115JC (rounded off), , 30,64,450, , Since the regular income-tax payable is less than the alternate minimum tax payable, the adjusted total, income shall be deemed to be the total income and tax is leviable @18.5% thereof plus surcharge@15%, and cess@4%. Therefore, tax liability as per section 115JC is ₹ 30,64,450., Computation of total income and tax liability of Mr. X for A.Y.2022-23, (under the provisions of section 115BAC of the Income-tax Act, 1961), Particulars, , ₹, , Total Income (as computed above as per regular provisions of income tax), , 48,00,000, , Add: Deduction under section 10AA (not allowable), , 32,00,000, 80,00,000, , Add: Deduction under section 35AD, , 65,00,000, , Less: Depreciation under section 32 On building @10% of ₹ 65 lakhs, (normal depreciation under section 32 is allowable), , (6,50,000), , Total Income, , 58,50,000, 1,38,50,000, , Computation of tax liability as per section 115BAC, , Tax on ₹ 1,38,50,000, , 38,92,500, , Add: Surcharge@15%, , 5,83,875, 44,76,375, , Add: Health and Education cess@4%, Total tax liability, , 1,79,055, 46,55,430, , Notes:, (1) Deductions u/s 10AA and 35AD are not allowable as per section 115BAC. However, normal, depreciation u/s 32 is allowable., , (2) Individuals or HUFs exercising option u/s 115BAC are not liable to alternate minimum tax u/s 115JC., Since the tax liability of Mr. X under section 115JC is lower than the tax liability as computed u/s, 115BAC, it would be beneficial for him not to opt for section 115BAC for A.Y. 2022-23. Moreover,, benefit of alternate minimum tax credit is also available to the extent of tax paid in excess over regular, tax., AMT Credit to be carried forward under section 115JEE, , Tax liability under section 115JC, Less: Tax liability under the regular provisions of the Income tax Act, 1961, , ₹, , 30,64,450, (13,02,600), 17,61,850, , Notes:, (1) Deduction under section 10AA in respect of Unit in SEZ =, , Profit of the Unit in SEZ × Export turnover of the Unit in SEZ / Total turnover of the Unit in SEZ, ₹ 40,00,000 × ₹ 80,00,000 / ₹ 1,00,00,000 = ₹ 32,00,000
Page 901 :
Section 115BAC, , 382, , (2) Deduction@100% of the capital expenditure is available under section 35AD, for A.Y.2022-23 in respect of specified business of setting up and operating a warehousing facility for, storage of agricultural produce which commences operation on or after 01.04.2009., Further, the expenditure incurred, wholly and exclusively, for the purposes of such specified business,, shall be allowed as deduction during the previous year in which he commences operations of his specified, business if the expenditure is incurred prior to the commencement of its operations and the amount is, capitalized in the books of account of the assessee on the date of commencement of its operations., Deduction under section 35AD would, however, not be available on expenditure incurred on acquisition, of land., In this case, since the capital expenditure of ₹ 65 lakhs (i.e., ₹ 75 lakhs – ₹ 10 lakhs, being expenditure on, acquisition of land) has been incurred in the F.Y.2019-20 and capitalized in the books of account on, 1.4.2021, being the date when the warehouse became operational, ₹ 65,00,000, being 100% of ₹ 65 lakhs, would qualify for deduction under section 35AD.
Page 902 :
M.K.G, CA EDUCATION, 9811429230 / 9212011367, WWW.MKGEDUCATION.COM, , SOLUTIONS INCOME TAX, (Volume – 2), , SETOFF AND CARRY FORWARD OF LOSSES, 02-10, PROVISIONS FOR FILING OF RETURN OF, INCOME, 11-14, INCOME UNDER THE HEAD CAPITAL GAINS, 15-48, INCOME UNDER THE HEAD, BUSINESS/PROFESSION, 49-85, INCOME UNDER THE HEAD SALARY, 86-139, Including, EXAMINATION QUESTIONS, After the book has been published, some error/mistake etc may be detected/or there may be some, amendments etc, all such corrections/amendments shall be uploaded on our website. Students are, requested to visit our website and also they should subscribe our youtube channel in order to update, the book., , 47th, , Edition, , CA (INTER), , MAY 2022/ NOV-2022, P.Y. 2021-22, A.Y. 2022-23, F.A. – 2021, , Author, This Book is the result of combined efforts of, Chartered Accountants/ company executives /, other professionals / feedback of our thousands of students, , FOR ONLINE PURCHASE OF OUR BOOK PLEASE SEE DETAILS, GIVEN ON OUR WEBSITE :WWW.MKGEDUCATION.COM
Page 903 :
Set Off and Carry Forward Of Losses, , 2, , SET OFF AND CARRY FORWARD OF, LOSSES, SECTION 70 TO 80, , SOLUTIONS OF MCQS, Answer, 1. (a); 2. (a); 3.(b); 4. (b); 5. (d); 6. (c); 7.(d); 8.(a); 9. (f); 10. (d); 11. (a); 12(d); 13. (c); 14. (c); 15. (b), Hint Q. No. 13:, Income under the head Capital Gains, LTCG, Less: Loss from House Property, Income under the head Capital Gains, Total Income, Tax on LTCG (3,00,000-2,50,000)*20%, Less: Rebate u/s 87A, Tax Liability, , 5,00,000, (2,00,000), 3,00,000, 3,00,000, 10,000, (10,000), NIL, , Hint Q. No. 14:, Income under the head Capital Gains, LTCG, Less: Loss from House Property, Income under the head Capital Gains, Total Income, Tax on LTCG (3,00,000-3,00,000)*20%, Tax Liability, , 5,00,000, (2,00,000), 3,00,000, 3,00,000, NIL, NIL, , Hint Q. No. 15:, Income under the head PGBP, Income from Business, Less: Loss from House Property, , 12,00,000, (2,00,000), , Income under the head PGBP, Total Income, , 10,00,000, 10,00,000, , Tax on 10,00,000 at slab rate, Add: HEC @ 4%, Tax Liability, , 1,10,000, 4,400, 1,14,400
Page 904 :
Set Off and Carry Forward Of Losses, , 3, , SOLUTIONS OF EXAMINATIONS QUESTIONS, JULY – 2021 (NEW COURSE), Solution 4(b):, Computation of Gross Total Income of Mr. X for A.Y. 2022-23, Particulars, , Amount, , Amount, , Salaries, Income from salary (computed), , 2,22,000, , Less: Set-off of loss from house property of ₹ 2,58,000 to the extent of ₹ 2, lakhs by virtue of section 71(3A), , 2,00,000, , 22,000, , Income from house property, - House in Delhi, - House in Chennai, - House in Mumbai (self-occupied), , 22,000, (2,60,000), (20,000), (2,58,000), , Loss upto ₹ 2 lakhs can be set off against income from salary., Balance loss of ₹ 58,000 from house property has to be carried forward to, A.Y.2022-23., Profits and gains from business or profession, Profits from Speculative business – 2, Less: Loss of ₹ 74,000 from speculation business - 1 set off to the extent, , 46,000, (46,000), , of profits of ₹ 46,000 as per section 73(1) from another speculation, business. Loss from speculation business cannot be set-off against any, income other than profit and gains of another speculation business., -, , Hence, the balance loss of ₹ 28,000 from speculative business has to be, carried forward to A.Y.2023-24., Profits from textile business, , 18,000, , Less: Loss from cosmetic business of ₹ 22,000 set off against profits from, textile business to the extent of ₹ 18,000 as per section 70(1)., , (18,000), , Balance loss of ₹ 4,000 from cosmetic business has to be carried forward, to A.Y.2023-24, since the same cannot be set-off against salary income., Capital Gains, Long term capital gain from sale of property, , 15,400, , -
Page 905 :
Set Off and Carry Forward Of Losses, , 4, , Less: Short-term capital loss can be set-off against both short-term capital, gains and long-term capital gains. Short term capital loss of ₹ 16,000 set, off against long- term capital gains to the extent of ₹ 15,400 as per section, 74(1)., , (15,400), , -, , Balance short term capital loss of ₹ 600 has to be carry forward to, A.Y.2023-24, Income from Other Sources, Income from betting [No loss is allowed to be set off against such income], , 34,000, , Income from card games [No loss is allowed to be set off against such, 46,000, , income], Loss on activity of owning and maintenance of race horses [Loss incurred, on activity of owning and maintenance of race horses cannot be set-off, , Nil, , against income from any source other than the activity of owning and, maintaining race horses. Hence, such loss of ₹ 14,600 has to be carried, forward to A.Y.2023-24], , 80,000, , Gross Total Income, , 1,02,000, , NOV – 2015, Solution 4(a):, Computation of taxable income of Mr. X for the A.Y. 2022-23, `, Income under the head Salary, 6,00,000.00, Less: Loss under the head house property (Loss of minor son), (90,000.00), Income under the head Salary, 5,10,000.00, Income under the head capital Gains, Long Term Capital Gain, 75,000.00, Less: Loss from Business of his wife, (75,000.00), Income under the head capital Gains, Nil, Income under the head other sources, Interest Income from Fixed Deposit, 80,000.00, Less: Loss from Business of his wife, (80,000.00), Income under the head other sources, Nil, (Balance amount of `45,000 to be carried forward), Gross Total Income, 5,10,000.00, Less: Deduction from 80C to 80U, Nil, Total Income, 5,10,000.00, Note: 1. Mr. X shall be the deemed owner of the house property transferred to minor son hence it will be, considered to be loss of Mr. X., 2. Loss from business of Mrs. X shall also be clubbed, 3. Brought Forward Long term capital loss of AY 2020-21 to be carried forward `96,000., , MAY – 2014, Solution 6(A):, Computation of Gross Total Income of Mr. X for the A.Y. 2022-23, Particulars, , `
Page 906 :
Set Off and Carry Forward Of Losses, Income under the head Salary, Profits and gains of business or profession, Less : Unabsorbed depreciation brought forward from A.Y.2021-22, Capital gains, Long term capital gain on sale of land, Less : Brought forward short term capital loss, , 5, 15,000, 66,000, (11,000), 55,000, 10,800, (9,800), 1,000, 71,000, , Gross Total Income, Amount of loss to be carried forward to A.Y.2023-24, , Particulars, `, (1) Loss from speculative business shall be carried forward as per section 73, 22,000, (2) Loss on maintenance of race horses shall be carried forward as per section 74A, 15,000, (3) Loss from gambling can neither be set-off nor be carried forward, Working Note:, Short-term capital loss can be set-off against both short-term capital gains and long-term capital gains as per, section 74, , NOV – 2012, Solution 5(a):, Income under the head Capital Gains, Long term capital Gain, Less: Short term capital loss on sale of property, Less: Loss from profession, Less: Loss from House Property, Income under the head Capital Gains, Income under the head Other Sources, Winning from lottery, Income from card game, Income under the head Other Sources, Gross Total Income, Less: Deduction u/s 80D (Deductions are not allowed from casual incomes), Total Income, Working Notes:, 1. Share of loss from firm is not allowed to be set off by the partner., 2. Loss from races can neither be set off nor be carried forward., , `, 2,05,000, (55,000), (1,05,000), (45,000), Nil, 1,00,000, 55,000, 1,55,000, 1,55,000, Nil, 1,55,000, , NOV – 2011, Answer 1:, Calculation of Gross Total Income of Mr. X for the assessment year 2022-23, Income under the head Salary, Salary, Less: Loss from house property, Income under the head Salary after set off, Income under the head Business/Profession, Income from Business of textile, Less: Loss Carried forward from textile business (A.Y. 2014-15), Balance loss of `10,000 shall lapse, Income under the head Capital Gains, Short Term Capital Gains, Long Term Capital Gains, Less: Long term loss, , `, 1,00,000, (40,000), 60,000, 50,000, (50,000), 1,40,000, 30,000, (30,000)
Page 907 :
Set Off and Carry Forward Of Losses, , 6, , (Balance of loss of 70,000 shall be carried forward), Income under the head Other Sources, Income from owning and maintaining race horses, 15,000, Less: Loss carried forward to be adjusted (A.Y. 2019-20), (15,000), (Balance brought forward loss of `10,000 to be carried forward to next year), Gross Total Income, 2,00,000, Loss from speculative business of A.Y. 2022-23 `60,000 to be carried forward for 4 years starting from, assessment year 2023-24., , MAY – 2011, Answer 4., Computation of total income of Mr. X for the A.Y. 2022-23, Particulars, `, , `, Salaries, 2,20,000, Income under the head salary, (1,90,000), Less: Loss from house property, 30,000, Profits and gains of business or profession, Income from speculation business, 30,000, Less: Loss from cloth business set off, Nil, (30,000), Capital gains, Long-term capital gains from sale of urban land, 2,50,000, Less: Loss on sale of listed shares, (90,000), Less: Loss from cloth business set off, (1,60,000), Nil, Income from other sources, Income from betting, 45,000, Gross total income, 75,000, Less: Deduction under section 80C (life insurance premium paid), (30,000), Total income, 45,000, Losses to be carried forward, (1) Loss from cloth business (2,40,000-30,000-1,60,000), 50,000, Notes, (i) Business loss cannot be set off against salary income., (ii) Loss from card games can neither be set off against any other income, nor can it be carried forward., (iii)Income from betting is chargeable at a flat rate of 30% under section 115BB and no expenditure or, allowance can be allowed as deduction from such income, nor can any loss be set-off against such income., Solution 4: Computation of total income of Mr. X for the year ended 31.03.2022, Option 1: Loss of House property is set off from Normal Income, Particulars, Income under the head salary, Less: Loss from house property, Net Salary (after set off of loss from house property), Profits and gains of business or profession, Speculation business income, Less: Business loss set-off, Net business loss to be set-off against long-term capital gain, Capital Gains, Long term capital gain, Less: Business loss set-off, Long term capital gain after set off of business loss, , `, 60,000, (15,000), , `, 45,000, , 1,00,000, (1,35,000), (35,000), 70,000, (35,000), 35,000
Page 908 :
Set Off and Carry Forward Of Losses, , 7, , Income from other sources, Lottery winnings (Gross), Total Income, Computation of tax liability, Particulars, On Normal Income Nil (`45,000 - `45,000) at slab rate, On LTCG Nil (`35,000 - `35,000) u/s 112 @ 20%, On lottery winnings of `3,00,000 @ 30%, Less: Rebate u/s 87A, Tax Liability after rebate, Add: HEC @ 4%, Total tax liability, , 3,00,000, 3,80,000, `, Nil, Nil, 90,000, (12,500), 77,500, 3,100, 80,600, , The assessee need not pay advance tax since the total income (excluding lottery income) liable to tax is, below the basic exemption limit. Further, in respect of lottery income, tax would have been deducted at, source @ 30% under section 194B. Since no tax is payable, advance tax liability is not attracted., Option 2: Loss of House property is set off from LTCG, Particulars, Income under the head salary, Profits and gains of business or profession, Speculation business income, Less: Business loss set-off, Net business loss to be set-off against long-term capital gain, Capital Gains, Long term capital gain, Less: Business loss set-off, Less: Loss from House Property, Long term capital gain after set off of loss, Income from other sources, Lottery winnings (Gross), Total Income, , `, , `, 60,000, , 1,00,000, (1,35,000), (35,000), 70,000, (35,000), (15,000), 20,000, 3,00,000, 3,80,000, , Computation of tax liability, Particulars, On Normal Income Nil (`60,000 - `60,000) at slab rate, On LTCG Nil (`20,000 - `20,000) u/s 112 @ 20%, On lottery winnings of `3,00,000 @ 30%, Less: Rebate u/s 87A, Tax Liability after rebate, Add: HEC @ 4%, Total tax liability, , `, Nil, Nil, 90,000, (12,500), 77,500, 3,100, 80,600, , The assessee need not pay advance tax since the total income (excluding lottery income) liable to tax is, below the basic exemption limit. Further, in respect of lottery income, tax would have been deducted at, source @ 30% under section 194B. Since no tax is payable, advance tax liability is not attracted., Since Tax Liability in both the options is same hence assessee can take any option., , NOV – 2010, Answer 1., , Computation of gross total income of Mr. X for the A.Y. 2022-23, Particulars, `, , Salaries, Income under the head salary, Less: Loss from house property set-off against salary income, , 3,00,000, (40,000), , `, 2,60,000
Page 909 :
Set Off and Carry Forward Of Losses, , 8, , as per section 71, Profits and gains of business or profession, Income from sugar business, 50,000, Less: Brought forward loss from iron-ore business set-off as per, (50,000), Nil, section 72, Balance business loss of `70,000 of P.Y.2015-16 carried, forward to A.Y.2023-24, Capital gains, Long term capital gain, 40,000, Less: Short term capital loss set-off, (40,000), Nil, Long term capital gain 112A, 10,000, Less: Short term capital loss set-off, (10,000), Nil, Balance short-term capital loss of `10,000 to be carried forward, Short-term capital loss of `10,000 under section 111A to be carried forward, Income from other sources, Dividend, 5,000, Winnings from lottery, 50,000, Winnings from card games, 6,000, Bank interest on fixed deposit, 5,000, 66,000, Gross Total Income, 3,26,000, Losses to be carried forward to A.Y. 2023-24, Loss of iron-ore business, 70,000, Short term capital loss, 10,000, Short term capital loss u/s 111A, 10,000, Notes:, 1. The following income are exempt under section 10 –, (i) Agricultural income [Exempt under section 10(1)], 2. It is presumed that loss from iron-ore business relates to P.Y.2015-16, the year in which the business was, discontinued., Answer 1., Computation of total income of Mr. X for A.Y. 2022-23, Particulars, , Amount, (`), , Amount, (`), , Profits and gains of business or profession, Current year business income, 1,10,000, Less: Brought forward business loss of discontinued business, `1,50,000 set-off to the extent of current year business, income as per section 72, (1,10,000), Nil, Income from other sources, Interest on enhanced compensation taxable on receipt basis, under section 56(2), 4,00,000, Less: Deduction under section 57 @ 50%, (2,00,000), 2,00,000, Total Income, 2,00,000, The unabsorbed business loss of `40,000 (`1,50,000 – `1,10,000) of A.Y. 2016-17 relating to discontinued, business will be carried forward for set-off against income from any business in the next year i.e. A.Y. 202324., , MAY – 2010, Answer 4. Computation of total income of Mr. X for the Assessment year 2022-23, `, Income (loss) House property, House –I, 36,000, , `
Page 910 :
Set Off and Carry Forward Of Losses, House-2 –Self occupied, House-3, Income from House Property, Profits and gains of business and profession, Textile business, Automobile business, Speculation business, Income from business or profession representing speculation, business profit (after set off of loss of automobile business), Capital Gains, Long term capital gain from sale of shares (STT paid) u/s 112A, Long term capital gain from sale of vacant site, Short term capital loss from sale of building, Long term capital gain-after set off of short term loss against, long term capital gain, Income from Other sources, Gift from a friend (non relative) on 05.06.2021, Gift from maternal uncle (on 25.02.2022) `1,00,000, not taxable, since maternal uncle is covered by the definition of the term’ relative’, given in explanation to section 56(2), Gift from grand father’s younger brother on 10.02.2022. This amount, is taxable as grandfather’s younger brother is not covered by the, definition of ‘relative’., Gross Total Income, Less: Deduction u/s 80C to 80U, Total income, , 9, (20,000), 60,000, 76,000, 2,00,000, (3,00,000), 2,00,000, 1,00,000, 1,50,000, 2,00,000, (1,00,000), 1,00,000, 2,50,000, 60,000, Nil, , 1,00,000, , 1,60,000, 5,86,000, Nil, 5,86,000, , JUNE – 2009, Answer 4., , Computation of Gross Total Income of Mrs. X for the Assessment Year 2022-23, Particulars, `, Profits and gains of business and profession, Salary received as a partner from a partnership firm is taxable under the head, “Profits and gains of business and profession”, 7,50,000, Less: brought forward business loss of assessment year 2021-22 to be, (7,50,000), set-off against business income, Nil, Capital Gains, Long term capital gain on sale of land –, 5,00,000, Less: Long term loss on sale of shares, (1,00,000), Income under the head Capital gains, 4,00,000, Income from other sources, Cash gift received from friends - since the value of cash gift exceeds, `50,000, the entire sum is taxable, 51,000, Dividend received from a domestic company, 55,000, 1,06,000, Gross Total Income, 5,06,000, Note: Balance brought forward business loss of assessment year 2021-22 of `5,00,000 has to be carried, forward to the next year.
Page 911 :
Set Off and Carry Forward Of Losses, , 10, , NOV – 2008, Answer 4., , Computation of Gross Total Income of Mr. X for the A.Y. 2022-23, `, (i) Income under the head salary, (ii) Income from House Property, Net annual value, 70,000, Less: Deduction under section 24(a) (30% of `70,000), (21,000), (iii) Income from business and profession, (a) Profit from business, 80,000, Less: Current year depreciation, (8,000), 72,000, Less: Unabsorbed depreciation, (9,000), , `, 18,000, 49,000, , 63,000, , (b) Income from speculative business, 12,000, Less: Brought forward loss from speculative business, (12,000), Nil, (Balance loss of `4,000 (i.e. `16,000 – `12,000), can be carried forward to the next year), (iv) Income from capital gain, Long term capital gain on sale of land, 15,800, Less: Brought forward short term capital loss, (7,800), 8,000, Gross total income, 1,38,000, Amount of loss to be carried forward to the next year, Particulars, `, Loss from speculative business (to be carried forward as per section 73), 4,000, Loss on maintenance of race horses (to be carried forward as per section 74A), 9,000, Notes:, (i) Loss on gambling can neither be set-off nor be carried forward., (ii) It has been assumed that the brought forward losses relate to P.Y. 2018-19 or thereafter. Only then, speculative business loss can set off against income from speculative business of the current year and the, balance loss can be carried forward to A.Y. 2023-24. It may be noted that speculative business loss can be, carried forward for a maximum of four years as per section 73., , MAY – 2007, Answer 4., , Computation of total income of Mr. X for the A.Y. 2022-23, Particulars, `, `, Profit of business of consumer and house-hold products, 50,000, Less: Loss of business of readymade garments for the year adjusted, under section 70, (10,000), 40,000, Less: Brought forward loss of catering business closed in A.Y. 2021-22, set off against business income for the year as per section 72, (15,000), 25,000, Profit of speculative transaction, 12,500, Total Income, 37,500, Notes: (i) Loss of speculative transaction of A.Y. 2017-18 is not allowed to be set off against the profit of, speculative transaction of the A.Y. 2022-23, since, as per the provisions of section 73, such loss can be, carried forward for set-off for a maximum period of 4 years only i.e. up to A.Y. 2021-22., (ii) Short term capital loss of `15,000 on sale of securities and shares has to be carried forward as per section, 74 since there is no income under the head Capital Gains for the A.Y. 2022-23. The loss is to be carried, forward for set off in future years against income chargeable under the head Capital Gains. Such loss can be, carried forward for a maximum period of 8 assessment years.
Page 912 :
Provisions for filing of Return of Income, , 11, , PROVISIONS FOR FILING, OF RETURN OF INCOME, , SOLUTIONS, TO, , PRACTICE PROBLEMS, Ans 1. 31.07.2022, Ans 2. 31.07.2022, Ans 3. 31.10.2022, Ans 4. 31.10.2022, Ans 5. As per section 139(1), every company has to file return of income in every case., Ans 6. The company is allowed to set off the loss during the previous year 2021-22 but its carried forward is, not allowed because return of loss has to be filed within the time allowed under section 139(1) i.e., 31.10.2022 in the above case., Ans 7. 31.12.2022, Ans 8. 31.12.2022, Ans 9. 31.12.2022, Ans 10. A revised return can also be revised further any number of times, however, if the earlier return has, already been assessed, revised return is not allowed subsequently. In the given case, revised return can be, filed on 01.12.2022.
Page 913 :
Provisions for filing of Return of Income, , SOLUTIONS OF MCQS, Answer, 1.(b); 2. (c); 3. (d); 4. (a); 5. (d); 6. (c); 7. (b); 8. (a); 9. (b); 10. (c); 11.(c); 12.(d); 13. (d); 14 (b); 15(a), , 12
Page 914 :
Provisions for filing of Return of Income, , 13, , EXAMINATION QUESTIONS, JULY – 2021 (NEW COURSE), Answer 4(c):, [First Alternative], As per section 139(3), an assessee is required to file a return of loss within the due date specified u/s 139(1), for filing return of income., As per section 80, certain losses which have not been determined in pursuance of a return filed under section, 139(3) on or before the due date specified under section 139(1) cannot be carried forward and set-off. Thus,, the assessee has to file a return of loss under section 139(3) within the time allowed u/s 139(1) in order to, carry forward and set off of following losses:, - loss under the head “Capital Gains”,, - loss from activity of owning and maintaining race horses., - business loss,, - speculation business loss and, - loss from specified business., However, following can be carried forward for set-off even if the return of loss has not been filed before the, due date:, - Loss under the head “Income from house property” and, - Unabsorbed depreciation., [Second Alternative], (i) A HUF whose total income without giving effect to, inter alia, section 54EC, exceeds the basic, exemption limit of ₹ 2,50,000, is required to file a return of its income on or before the due date under, section 139(1). In this case, since the total income without giving effect to exemption under section 54EC is, ₹ 12 lakhs, exceeds the basic exemption limit, the HUF is required to file its return of income for A.Y. 202223 on or before the due date under section 139(1)., (ii) Every person, being a resident other than not ordinarily resident in India would be required to file a, return of income or loss for the previous year on or before the due date, even if his or her total income does, not exceed the basic exemption limit, if such person, at any time during the previous year, inter alia, holds, any asset located outside India., In this case, though Mrs. Archana owns a car in Germany, the same does not fall within the ambit of “capital, asset” as it is a personal effect. Hence, Mrs. Archana is not required to file her return of income for A.Y., 2022-23 on account of owning a car for personal purposes in Germany., Note – “Asset” for the purpose of the fourth proviso to section 139(1) has not been specifically defined in, the said section or elsewhere in the Act. Schedule FA of the income-tax return forms, however, requires, details of foreign assets for the purpose of filing of return of income under this provision. The foreign assets, listed in the said Schedule does not include car. It, however, includes “any other capital assets outside, India”. Car used for personal purposes is not a capital asset as it is a “personal effect”. Hence, it is not, included in the meaning of “asset” for the purpose of the fourth proviso to section 139(1). The above answer, is based on the view taken regarding the ambit of the term “asset”, based on the list of assets detailed in the, relevant schedule of the income-tax return forms., Alternative view - On the plain reading of the fourth proviso to section 139(1) and the general meaning, attributable to the word “asset”, it is possible to take a view that Mrs. Archana is required to file her return
Page 915 :
Provisions for filing of Return of Income, , 14, , of income as she owns an asset, i.e., a car in Germany. Accordingly, due credit may also be given to the, candidates who have answered on this basis., (iii) If an individual has incurred expenditure exceeding ₹ 1 lakh towards consumption of electricity during, the previous year, he would be required to file a return of income, even if his total income does not exceed, the basic exemption limit. Since Mr. Sudhakar has incurred expenditure of ₹ 1,20,000 in the P.Y.2021-22, towards consumption of electricity, he has to file his return of income for A.Y. 2022-23 on or before the due, date under section 139(1)., , JAN – 2021 (NEW COURSE), Answer 3(b), An individual is required to furnish a return of income under section 139(1) if his total income, before, giving effect to the deductions under Chapter VI-A or exemption under section 54/54B/54D/54EC or 54F,, exceeds the maximum amount not chargeable to tax i.e., ₹ 2,50,000., Computation of total income of Mr. Hari for A.Y. 2022-23, Particulars, Income from other sources, Interest earned from Non-resident (External) Account ₹ 3,55,000 [Exempt, u/s 10(4)(ii), since he is maintaining the said account as per RBI stipulations], Interest on savings bank account, Interest on fixed deposit with Punjab National Bank, Gross Total Income, Less: Deduction u/s 80TTA (Interest on saving bank account), Total Income, , ₹, , NIL, 8,000, 40,000, 48,000, (8,000), 40,000, , Since the total income of Mr. Hari for A.Y.2022-23, before giving effect, inter alia, to the deductions under, Chapter VI-A, is less than the basic exemption limit of ₹ 2,50,000, he is not required to file return of income, for A.Y.2022-23., However, if he has incurred expenditure exceeding ₹ 2 lakhs for himself or any other person for travel to a, foreign country, he would be required to file a return of income,, even if his total income does not exceed the basic exemption limit. Since he has incurred expenditure of ₹ 4, lakhs on foreign travel of his newly married son and daughter in law in the F.Y. 2021-22, he has to, mandatorily file his return of income for A.Y. 2022-23 on or before the due date under section 139(1).
Page 916 :
Income Under The Head Capital Gains, , 15, , INCOME UNDER THE HEAD, CAPITAL GAINS, SECTION 45 TO 55A, , SOLUTIONS OF MCQS, Answer, 1. (d); 2. (c); 3. (b); 4. (b); 5. (d); 6. (c); 7.(c); 8.(c); 9.(c); 10.(a); 11(b); 12(c); 13. (d); 14. (b); 15. (a); 16(c);, 17. (c); 18. (a); 19. (b); 20. (d); 21(c); 22. (b); 23. (a); 24. (a); 25. (c)
Page 917 :
Income Under The Head Capital Gains, , 16, , SOLUTIONS, TO, , PRACTICE PROBLEMS, Solution 1:, Computation of income under the head Capital Gain, Full value of consideration, Less: Indexed cost of acquisition, = 2,00,000 / Index of 01-02 x Index of 21-22, = 2,00,000 / 100 x 317 = `6,34,000, Less: Indexed cost of improvement, Cost of constructing first floor, = 3,00,000 / Index of 07-08 x Index of 21-22, = 3,00,000 / 129 x 317 = `7,37,209.30, Less: Indexed cost of improvement, Cost of constructing second floor, = 4,00,000 / Index of 11-12 x Index of 21-22, = 4,00,000 / 184 x 317 = `6,89,130.43, Less: Indexed cost of improvement, Cost of constructing third floor, = 5,00,000 / Index of 12-13 x Index of 21-22, = 5,00,000 / 200 x 317 = `7,92,500, Less: Brokerage @ 1%, = 1% of `100,00,000 = `1,00,000, Long Term Capital Gain, Income under the head Capital Gain (LTCG), Gross Total Income, Less: Deduction u/s 80C to 80U, Total Income, Rounded off u/s 288A, Computation of Tax Liability, Tax on ` 67,97,160 (`70,47,160 – `2,50,000) @ 20%, Add: Surcharge @10%, Tax before health & education cess, Add: HEC @ 4%, Tax Liability, Rounded off u/s 288B, Solution 2:, Gold, Full value of consideration, Less: Indexed cost of acquisition, = 35,00,000 / Index of 01-02 x Index of 21-22, = 35,00,000 / 100 x 317 = 110,95,000, Long term capital gain, Gross Total Income, , `, 100,00,000.00, (6,34,000.00), , (7,37,209.30), , (6,89,130.43), , (7,92,500.00), (1,00,000.00), 70,47,160.27, 70,47,160.27, 70,47,160.27, Nil, 70,47,160.27, 70,47,160.00, 13,59,432.00, 1,35,943.20, 14,95,375.20, 59,815.00, 15,55,190.20, 15,55,190.00, `, 150,00,000, (110,95,000), 39,05,000, 39,05,000
Page 918 :
Income Under The Head Capital Gains, Less: Deduction u/s 80C, Total Income, Computation of Tax Liability, Tax on `36,55,000 (`39,05,000 – 2,50,000) @ 20% u/s 112, Add: HEC @ 4%, Tax Liability, Land, Full value of consideration, Less: Indexed cost of acquisition, = 45,00,000 / Index of 01-02 x Index of 21-22, = 45,00,000 / 100 x 317 = 142,65,000, Long term capital gain, Gross Total Income, Less: Deduction u/s 80C, Total Income, Computation of Tax Liability, Tax on `174,85,000 (`177,35,000 – 2,50,000) @ 20% u/s 112, Add: Surcharge @ 15%, Tax before health & education cess, Add: HEC @ 4%, Tax Liability, Rounded off u/s 288B, Residential House, Full value of consideration, Less: Indexed cost of acquisition, = 55,00,000 / Index of 01-02 x Index of 21-22, = 55,00,000 / 100 x 317 = 174,35,000, Less: Indexed cost of improvement, = 3,00,000 / Index of 16-17 x Index of 21-22, = 3,00,000 / 264 x 317 = 3,60,227.27, Long term capital gain, Gross Total Income, Less: Deduction u/s 80C, Total Income, Rounded off u/s 288A, Computation of Tax Liability, Tax on `219,54,770 (`222,04,770 – 2,50,000) @ 20% u/s 112, Add: Surcharge @ 25%, Tax before health & education cess, Add: HEC @ 4%, Tax Liability, Rounded off u/s 288B, Solution 3:, Computation of Capital Gains in the hands of Mr. X, Original shares, Full value of consideration, (100 x 200), Less: Cost of acquisition, Higher of, (i) COA = 100 x 10 = 1,000, , 17, Nil, 39,05,000, 7,31,000.00, 29,240.00, 7,60,240.00, 320,00,000.00, (142,65,000.00), 177,35,000.00, 177,35,000.00, Nil, 177,35,000.00, 34,97,000.00, 5,24,550.00, 40,21,550.00, 1,60,862.00, 41,82,412.00, 41,82,410.00, 400,00,000, (174,35,000), (3,60,227.27), 222,04,772.73, 222,04,772.73, Nil, 222,04,772.73, 222,04,770.00, 43,90,954.00, 10,97,738.50, 54,88,692.50, 2,19,547.70, 57,08,240.20, 57,08,240.00, , 20,000, (10,000)
Page 919 :
Income Under The Head Capital Gains, (ii) lower of, (a) FMV as on 31-01-2018 = 100 shares x 100 = 10,000, (b) sale value = 100 shares x 200 = 20,000, COA = 10,000, Long term capital gain u/s 112A, 1st bonus shares, Full value of consideration, (100 x 200), Less: Cost of acquisition, Higher of, (i) COA = 100 x 8 = 800, (ii) lower of, (a) FMV as on 31-01-2018 = 100 shares x 100 = 10,000, (b) sale value = 100 shares x 200 = 20,000, COA = 10,000, Long term capital gain u/s 112A, 2nd bonus shares, Full value of consideration, (100 x 200), Less: Cost of acquisition, Higher of, (i) COA = Nil, (ii) lower of, (a) FMV as on 31-01-2018 = 100 shares x 100 = 10,000, (b) sale value = 100 shares x 200 = 20,000, COA = 10,000, Long term capital gain u/s 112A, , 18, , 10,000, 20,000, (10,000), , 10,000, 20,000, (10,000), , 10,000, , Computation of capital gains in case of right shares, `, Full value of consideration, 10,000, (50 x 200), Less: Cost of acquisition, (7,500), (50 x 150), Short term capital gain u/s 111A, 2,500, Computation of capital gains in case of shares renounced, Full value of consideration, 250, (50 x 5), Less: Cost of acquisition, Nil, Short term capital gain, 250, Short term capital gain of Mr. X, 2,750, Working Note:, Period of holding in case of renouncing of right to purchase a right shares section 2(42A), In the case of a capital asset, being the right to subscribe to any financial asset, which is renounced in favour, of any person, the period shall be reckoned from the date of the offer of such right by the company or, institution, as the case may be, making such offer., Computation of Total Income, Income under the head Business/Profession, Income under the head Capital Gains, Long term capital gains 112A, , 20,00,000.00, 30,000.00
Page 920 :
Income Under The Head Capital Gains, Short term capital gains, Gross Total Income, Less: Deduction u/s 80C, Total Income, Computation of Tax Liability, Tax on `2,500 @ 15% u/s 111A, Tax on `19,30,250 at slab rate, Tax on LTCG 112A (30,000-30,000), Tax before health & education cess, Add: HEC @ 4%, Tax Liability, Rounded off u/s 288B, Computation of capital gains in case of Mr. Y, Full value of consideration, (200 x 50), Less: Cost of acquisition, (50 x 155), Short term capital gain u/s 111A, , 19, 2,750.00, 20,32,750.00, (70,000.00), 19,62,750.00, 375.00, 3,91,575.00, Nil, 3,91,950.00, 15,678.00, 4,07,628.00, 4,07,630.00, 10,000, (7,750), 2,250, , Working Note:, Cost of acquisition of right renouncee section 55, In relation to any financial asset purchased by any person in whose favour the right to subscribe to such, asset has been renounced, means the aggregate of the amount of the purchase price paid by him to the person, renouncing such right and the amount paid by him to the company or institution, as the case may be, for, acquiring such financial asset., Period of holding in case of right renouncee section 2(42A), In the case of a capital asset, being a share or any other security subscribed to by the assessee on the basis of, his right to subscribe to such financial asset or subscribed to by the person in whose favour the assessee has, renounced his right to subscribe to such financial asset, the period shall be reckoned from the date of, allotment of such financial asset., Computation of Total Income, Income under the head Business/Profession, 10,00,000.00, Income under the head Capital Gains, 2,250.00, Gross Total Income, 10,02,250.00, Less: Deduction u/s 80C, (50,000.00), Total Income, 9,52,250.00, Computation of Tax Liability, Tax on `2,250 @ 15% u/s 111A, 337.50, Tax on `9,50,000 at slab rate, 1,02,500.00, Tax before health & education cess, 1,02,837.50, Add: HEC @ 4%, 4,113.50, Tax Liability, 1,06,951.00, Rounded off u/s 288B, 1,06,950.00, (Deduction under section 80C is not allowed from short term capital gain on the transfer of equity shares, on which STT has been paid.), Solution 4:, Computation of Tax Liability for the previous year 2021-22 under section 45(5), Since the Government has made the first payment in the previous year 2021-22, Long term capital gain shall, be taxed in the previous year 2021-22. However, Long term capital gain shall be computed in the year in, which the asset has been acquired i.e. in the year 2014-15., `, Computation of capital gains, Full value consideration, 55,00,000.00, Less: Indexed cost of acquisition
Page 921 :
Income Under The Head Capital Gains, , 20, , = 5,00,000 / Index of 02-03 x Index of 14-15, = 5,00,000/105 x 240 = `11,42,857.14, (11,42,857.14), Long Term Capital Gain, 43,57,142.86, Income under the head Capital Gains (LTCG), 43,57,142.86, Gross Total Income, 43,57,142.86, Less: Deductions u/s 80C to 80U, Nil, Total Income {Rounded off u/s 288A}, 43,57,140.00, Computation of Tax Liability, {Since there is no income under any other head so as per section 112 deficiency of `2,50,000 shall be, allowed from LTCG and the balance income shall be taxed at flat rate of 20%}, Tax on `41,07,140 (`43,57,140 – 2,50,000) @ 20%, 8,21,428.00, Add: HEC @ 4%, 32,857.12, Tax liability of Mr. X, 8,54,285.12, Rounded off u/s 288B, 8,54,290.00, Computation of Capital Gains, Capital gains for the previous year 2024-25 i.e. the year in which additional compensation has been, received., Long term capital gain for 2024-25, 3,00,000.00, Capital gain for the previous year 2025-26 in which balance amount of additional compensation has been, received., Long term capital gain for the year 2025-26, 2,00,000.00, Solution 5:, (i), Household furniture is not a capital asset., (ii), Personal motor car is not a capital asset., (iii) Residential house is a capital asset., (iv), Urban land is a capital asset., (v), Agricultural land in rural area in India is not a capital asset., (vi), Agricultural land in rural area in Nepal is a capital asset., (vii) Stock in trade is not a capital asset., (viii) Gold ornaments are a capital asset., (ix), Music system for personal use is not a capital asset., (x), Music system for business use is a capital asset., (xi), Motor car in business use is a capital asset., (xii) Plant and machinery in business use is a capital asset., (xiii) Silver utensils for personal use is not a capital asset., (xiv) Precious stones in personal use is a capital asset., Solution 6:, Computation of Capital Gains, 1. House, Full value of consideration, Less: Indexed cost of acquisition (2,30,000 / 100 x 317), Long term capital loss, 2. Agricultural Land in rural area not an asset as per section 2(14)., 3. T.V. is not an asset as per section 2(14)., 4. Gold, Full value of consideration, Less: Cost of acquisition, Short term capital gain, 5. Motor car, Full value of consideration, Less: written down value, , `, 5,00,000.00, (7,29,100.00), (2,29,100.00), , 4,50,000.00, (3,00,000.00), 1,50,000.00, 2,50,000.00, (2,00,000.00)
Page 922 :
Income Under The Head Capital Gains, Short term capital gain as per section 50, 6. House, Full value of consideration, Less: Indexed cost of acquisition (7,00,000 / 105 x 317), Less: Indexed cost of improvement (4,50,000 / 200 x 317), Less: Selling expenses @ 2%, Long term capital gains, Less: Long term capital loss on sale of first house, Long term capital gain, Short Term Capital Gain, Income under the head Capital Gains, Gross Total Income, Less: Deduction u/s 80C to 80U, Total Income, Rounded off u/s 288A, Computation of Tax Liability, Tax on `2,00,000 at slab rate, Tax on LTCG `57,14,320 (`57,64,320 – `50,000) @ 20%, Tax before Surcharge, Add: Surcharge @ 10%, Tax before health & education cess, Add: HEC @ 4%, Tax Liability, Rounded off u/s 288B, Solution 7:, Computation of Capital Gains, Full value of consideration, Less: Indexed cost of acquisition, = 5,00,000 / Index of 02-03 x Index of 21-22, = 5,00,000 / 105 x 317 = 15,09,523.81, Long Term Capital Gains, Less: Exemption u/s 54, Long Term Capital Gains, Income under the head Capital Gains, Gross Total Income, Less: Deduction u/s 80C to 80U, Total Income, Rounded off u/s 288A, Computation of Tax Liability, Tax on LTCG `62,40,480 (`64,90,480 – `2,50,000) @ 20%, Add: Surcharge @ 10%, Tax before health & education cess, Add: HEC @ 4%, Tax Liability, Rounded off u/s 288B, Computation of Capital Gain for the assessment year 2023-24, Full value of consideration, Less: Cost of acquisition (20,00,000- 20,00,000), Short Term Capital Gain, , 21, 50,000.00, 90,00,000.00, (21,13,333.33), (7,13,250.00), (1,80,000.00), 59,93,416.67, (2,29,100.00), 57,64,316.67, 2,00,000.00, 59,64,316.67, 59,64,316.67, Nil, 59,64,316.67, 59,64,320.00, Nil, 11,42,864.00, 11,42,864.00, 1,14,286.40, 12,57,150.40, 50,286.02, 13,07,436.42, 13,07,440.00, `, 100,00,000.00, (15,09,523.81), 84,90,476.19, (20,00,000.00), 64,90,476.19, 64,90,476.19, 64,90,476.19, Nil, 64,90,476.19, 64,90,480.00, 12,48,096.00, 1,24,809.60, 13,72,905.60, 54,916.22, 14,27,821.82, 14,27,820.00, 25,00,000.00, Nil, 25,00,000.00
Page 923 :
Income Under The Head Capital Gains, , 22, , Solution 7(b):, Computation of Capital Gains, Full value of consideration, 100,00,000.00, Less: Indexed cost of acquisition, = 5,00,000 / Index of 02-03 x Index of 21-22, = 5,00,000 / 105 x 317 = 15,09,523.81, (15,09,523.81), Long Term Capital Gains, 84,90,476.19, The assessee has the option either not to avail exemption under section 54 or to avail exemption under, section 54 and also it will be withdrawn, Option I, Exemption is not availed:, Long Term Capital Gain, 84,90,476.19, Sale of house purchased on 01.01.2022, Full value of consideration, 25,00,000.00, Less: Cost of acquisition, (20,00,000.00), Short Term Capital Gain, 5,00,000.00, Income under the head Capital Gains, 89,90,476.19, Gross Total Income, 89,90,476.19, Less: Deduction u/s 80C to 80U, Nil, Total Income, 89,90,476.19, Rounded off u/s 288A, 89,90,480.00, Computation of Tax Liability, Tax on `5,00,000 at slab rate, 12,500.00, Tax on LTCG `84,90,480 @ 20%, 16,98,096.00, Tax before Surcharge, 17,10,596.00, Add: Surcharge @ 10%, 1,71,059.60, Tax before health & education cess, 18,81,655.60, Add: HEC @ 4%, 75,266.22, Tax Liability, 19,56,921.82, Rounded off u/s 288B, 19,56,920.00, Option II, Exemption is availed, Long Term Capital Gain, 84,90,476.19, Less: Exemption u/s 54, (20,00,000.00), Long Term Capital Gains, 64,90,476.19, Sale of house purchased on 01.01.2022, Full value of consideration, 25,00,000.00, Less: Cost of acquisition (20,00,000 – 20,00,000), Nil, Short Term Capital Gain, 25,00,000.00, Income under the head Capital Gains, 89,90,476.19, Gross Total Income, 89,90,476.19, Less: Deduction u/s 80C to 80U, Nil, Total Income, 89,90,476.19, Rounded off u/s 288A, 89,90,480.00, Computation of Tax Liability, Tax on `25,00,000 at slab rate, 5,62,500.00, Tax on LTCG `64,90,480 @ 20%, 12,98,096.00, Tax before Surcharge, 18,60,596.00, Add: Surcharge @ 10%, 1,86,059.60, Tax before health & education cess, 20,46,655.60, Add: HEC @ 4%, 81,866.22, Tax Liability, 21,28,521.82, Rounded off u/s 288B, 21,28,520.00, Hence the assessee should opt for option I and his tax liability shall be `19,56,920.
Page 924 :
Income Under The Head Capital Gains, , 23, , Solution 7(c):, Computation of Capital Gains, Full value of consideration, 100,00,000.00, Less: Indexed cost of acquisition, = 5,00,000 / Index of 02-03 x Index of 21-22, = 5,00,000 / 105 x 317 = 15,09,523.81, (15,09,523.81), Long Term Capital Gains, 84,90,476.19, (Exemption is not allowed because house was purchased after the last date of filing of return of income), Income under the head Capital Gains, 84,90,476.19, Gross Total Income, 84,90,476.19, Less: Deduction u/s 80C to 80U, Nil, Total Income, 84,90,476.19, Rounded off u/s 288A, 84,90,480.00, Computation of Tax Liability, Tax on LTCG `82,40,480 (`84,90,480 – `2,50,000) @ 20%, 16,48,096.00, Add: Surcharge @ 10%, 1,64,809.60, Tax before health & education cess, 18,12,905.60, Add: HEC @ 4%, 72,516.22, Tax Liability, 18,85,421.82, Rounded off u/s 288B, 18,85,420.00, Solution 7(d):, Computation of Capital Gains, Full value of consideration, Less: Indexed cost of acquisition, = 5,00,000 / Index of 02-03 x Index of 21-22, = 5,00,000 / 105 x 317 = 15,09,523.81, Long Term Capital Gains, Less: Exemption u/s 54, Long Term Capital Gains, Income under the head Capital Gains, Gross Total Income, Less: Deduction u/s 80C to 80U, Total Income, Rounded off u/s 288A, Computation of Tax Liability, Tax on LTCG `62,40,480 (`64,90,480 – `2,50,000) @ 20%, Add: Surcharge @ 10%, Tax before health & education cess, Add: HEC @ 4%, Tax Liability, Rounded off u/s 288B, Computation of Capital Gain for the assessment year 2025-26, Unutilized amount in capital gain account scheme after expiry of three years, Long Term Capital Gain, Solution 8:, Computation of Capital Gains, Previous year 2021-22, Full value of consideration, , 100,00,000.00, (15,09,523.81), 84,90,476.19, (20,00,000.00), 64,90,476.19, 64,90,476.19, 64,90,476.19, Nil, 64,90,476.19, 64,90,480.00, 12,48,096.00, 1,24,809.60, 13,72,905.60, 54,916.22, 14,27,821.82, 14,27,820.00, , 20,00,000.00, `, 32,00,000.00
Page 925 :
Income Under The Head Capital Gains, Less: Indexed cost of acquisition, = `3,00,000 / Index of 05-06 x Index of 21-22, = `3,00,000 / 117 x 317 = `8,12,820.51, Long term capital gain, Less: Exemption u/s 54B, Long term capital gain, Income under the head Business/Profession, Gross Total Income, Less: Deduction u/s 80C to 80U, Total Income, Rounded off u/s 288A, Computation of Tax Liability, Tax on LTCG `16,47,180 (`17,87,180 – `1,40,000) @ 20%, Tax on `1,10,000 at slab rate, Add: HEC @ 4%, Tax Liability, Rounded off u/s 288B, Previous year 2022-23, Sale of land, Full value of consideration, Less: Cost of acquisition = `6,00,000, Less: Exemption earlier allowed = `6,00,000, So, Cost of acquisition = Nil, Short term capital gain, Solution 9:, Computation of Capital Gains, Full value of consideration, Less: Indexed cost of acquisition, = 1,50,000 / Index of 01-02 x Index of 21-22, = 1,50,000 / 100 x 317 = `4,75,500, Long term capital gain, Less: Exemption u/s 54EC, Less: Exemption u/s 54F, = 17,24,500/22,00,000 x 3,00,000, Long term capital gain, Less: Short Term Capital Loss, Income under the head Capital Gains (LTCG), Income under the head Business/Profession, Gross Total Income, Less: Deductions u/s 80C, Total Income, Rounded off u/s 288A, Computation of Tax Liability, Tax on `49,80,000 at slab rate, Tax on `10,59,340 @ 20% u/s 112, Tax before Surcharge, Add: Surcharge @ 10%, Tax before health & education cess, Add: HEC @ 4%, Tax Liability, Rounded off u/s 288B, , 24, , (8,12,820.51), 23,87,179.49, (6,00,000.00), 17,87,179.49, 1,10,000.00, 18,97,179.49, Nil, 18,97,179.49, 18,97,180.00, 3,29,436.00, Nil, 13,177.44, 3,42,613.44, 3,42,610.00, , 10,00,000.00, Nil, 10,00,000.00, `, 22,00,000.00, (4,75,500.00), 17,24,500.00, (3,80,000.00), (2,35,159.09), 11,09,340.91, (50,000.00), 10,59,340.91, 50,00,000.00, 60,59,340.91, (20,000.00), 60,39,340.91, 60,39,340.00, 13,06,500.00, 2,11,868.00, 15,18,368.00, 1,51,836.80, 16,70,204.80, 66,808.19, 17,37,012.99, 17,37,010.00
Page 926 :
Income Under The Head Capital Gains, , Solution 10:, Computation of Capital Gains, Full value of consideration, Less: Indexed cost of acquisition, = 3,00,000 / Index of 01-02 x Index of 21-22, = 3,00,000 / 100 x 317 = `9,51,000, Long term capital gain, Less: Exemption u/s 54B, Purchased on 10.01.2022, Less: Exemption u/s 54F, = Capital Gains / Net Consideration x Amount of investment, = `30,49,000/ 40,00,000 x 7,00,000, Less: Exemption u/s 54EC, Long term capital gain after various deductions, Income under the head Capital Gains (LTCG), Gross Total Income, Less: Deductions u/s 80C to 80U, Total Income, Rounded off u/s 288A, Computation of Tax Liability, Tax `19,15,430 (`21,65,430– `2,50,000) @ 20% u/s 112, Add: HEC @ 4%, Tax Liability, Rounded off u/s 288B, , 25, , `, 40,00,000.00, (9,51,000.00), 30,49,000.00, (2,50,000.00), (5,33,575.00), (1,00,000.00), 21,65,425.00, 21,65,425.00, 21,65,425.00, Nil, 21,65,425.00, 21,65,430.00, 3,83,086.00, 15,323.44, 3,98,409.44, 3,98,410.00, , Solution 11:, `, Computation of Capital Gains, 1. Sale of Land, Full value of consideration, 60,00,000, Less: Indexed cost of acquisition, = 5,00,000 / Index of 01-02 x Index of 21-22, = 5,00,000 / 100 x 317 = `15,85,000, (15,85,000), Long term capital gain, 44,15,000, Less: Exemption u/s 54B, (8,00,000), Long term capital gain, 36,15,000, 2. House, Full value of consideration, 15,00,000, Less: Cost of acquisition, (12,50,000), Short term capital gain, 2,50,000, Income under the head capital Gain, 38,65,000, Gross Total Income, 38,65,000, Less: Deduction u/s 80C to 80U, Nil, Total Income, 38,65,000, Computation of Tax Liability, Tax on `2,50,000 at slab rate, Nil, Tax on LTCG `36,15,000 @ 20% u/s 112, 7,23,000, Add: HEC @ 4%, 28,920, Tax Liability, 7,51,920, Note: Assessee will be allowed exemption under section 54F but exemption shall be withdrawn because the, house has been sold hence exemption allowed and exemption withdrawn will be the same amount and it will
Page 927 :
Income Under The Head Capital Gains, , 26, , give the same tax liability., Solution 12:, Computation of Capital Gains, Gold, Full value of consideration, Less: Indexed cost of acquisition, = 1,01,000 / Index of 01-02 x Index of 21-22, = 1,01,000 / 100 x 317 = `3,20,170, Long term capital gain, Shares in A Ltd, Full value of consideration, Less: cost of acquisition, Higher of, (i) COA = 2,00,000, (ii) lower of, (a) FMV as on 31-01-2018 = 2,83,000, (b) sale value = 3,00,000, COA = 2,83,000, Long term capital gain u/s 112A, Income under the head Capital Gains, Gross Total Income, Less: Deduction u/s 80C to 80U, Total Income, Computation of Tax Liability, Tax on LTCG `14,29,830 (`16,79,830 – `2,50,000) @ 20%, Tax on LTCG Nil (17,000-17,000) u/s 112A, Add: HEC @ 4%, Tax Liability, Rounded off u/s 288B, Solution 13:, Computation of Capital Gains, Full value of consideration, Less: Indexed cost of acquisition, = 1,10,000 / Index of 01-02 x Index of 21-22, = 1,10,000 / 100 x 317 = `3,48,700, Less: Indexed cost of improvement, = 40,000 / Index of 04-05 x Index of 21-22, = 40,000 / 113 x 317 = `1,12,212.39, Less: Brokerage, Long term capital Gain, Solution 14:, Computation of income under the head Capital Gains, Full value of consideration, Less: Indexed cost of acquisition, = 1,50,000 / Index of 01-02 x Index of 21-22, = 1,50,000 / 100 x 317 = `4,75,500, Less: Brokerage, 1% of `40,00,000 = `40,000, Less: Exemption u/s 54, , `, 20,00,000.00, (3,20,170.00), 16,79,830.00, 3,00,000.00, (2,83,000.00), , 17,000.00, 16,96,830.00, 16,96,830.00, Nil, 16,96,830.00, 2,85,966.00, Nil, 11,438.64, 2,97,404.64, 2,97,400.00, `, 14,50,000.00, (3,48,700.00), (1,12,212.39), (14,500.00), 9,74,587.61, `, 40,00,000.00, (4,75,500.00), (40,000.00), (8,00,000.00)
Page 928 :
Income Under The Head Capital Gains, Long term capital gain, Income under the head Capital Gains (LTCG), Income under the head Other Sources, Gross Total Income, Less: Deduction u/s 80C, Total Income, Computation of Tax Liability, Tax on LTCG on `24,24,500 (`26,84,500 – `2,60,000) @ 20% u/s 112, Tax on `40,000 at slab rate, Add: HEC @ 4%, Tax Liability, Rounded off u/s 288B, Solution 15:, (a) STT not paid, Computation of Capital Gains, Full value of consideration (300 x 500), Less: Indexed Cost of acquisition, = (100 x 300) / Index of 04-05 x Index of 21-22, = 30,000 / 113 x 317 = `84,159.29, Less: Brokerage @ 1% on 1,50,000, Long Term Capital Gain, (b) STT Paid, Computation of Capital Gains, Full value of consideration (300 x 500), Less: Cost of acquisition, Higher of, (i) COA = 100 debentures x 300 = 30,000, (ii) lower of, (a) FMV as on 31-01-2018 = 300 shares x 300 = 90,000, (b) sale value = 300 shares x 500 = 1,50,000, COA = 90,000, Less: Brokerage @ 1% on 1,50,000, Long Term Capital Gain u/s 112A, Solution 16:, Computation of Capital Gains, Previous Year 2021-22 Assessment Year 2022-23, Full value of consideration, (1,500 x 900), Less: Indexed cost of acquisition, = (500 x 395.85) / Index of 01-02 x Index of 21-22, = 1,97,925 / 100 x 317 = `6,27,422.25, Less: Brokerage @ 1.5%, = 1.5 % of `13,50,000 = `20,250, Long Term Capital Gains, Less: Exemption u/s 54F, = 7,02,327.75 x 2,00,000 = `1,05,633.05, 13,29,750, Long Term Capital Gain, Gross Total Income, Less: Deduction u/s 80C to 80U, , 27, 26,84,500.00, 26,84,500.00, 50,000.00, 27,34,500.00, (10,000.00), 27,24,500.00, 4,84,900.00, Nil, 19,396.00, 5,04,296.00, 5,04,300.00, `, 1,50,000.00, (84,159.29), (1,500.00), 64,340.71, , 1,50,000.00, (90,000.00), , (1,500.00), 58,500.00, `, 13,50,000.00, , (6,27,422.25), (20,250.00), 7,02,327.75, (1,05,633.05), 5,96,694.70, 5,96,694.70, Nil
Page 929 :
Income Under The Head Capital Gains, Total Income, Rounded off u/s 288A, Computation of Tax Liability, Tax on `3,46,690 (`5,96,690 – `2,50,000) @ 20%, Add: HEC @ 4%, Tax Liability, Rounded off u/s 288B, , 28, 5,96,694.70, 5,96,690.00, 69,338.00, 2,773.52, 72,111.52, 72,110.00, , Assessment Year 2025-26, Proportionate capital gains for unutilized amount shall be as given below:, = 7,02,327.75 x 50,000 = `26,408.26, Long Term Capital Gain, 26,408.26, 13,29,750, (Proportionate exemption with regard to the unutilised amount lying in the capital gain account scheme is, Chargeable to tax after expiry of period of three years.), Solution 17:, Computation of income under the head Capital Gain, 1. House, Full value of consideration, Less: Indexed cost of acquisition (5,25,000/ 100 x 317), Less: Indexed cost of improvement of First floor (4,00,000/ 220 x 317), Less: Selling Expenses, Long term capital gain, 2. Shares, Full value of Consideration, Less: Indexed cost of acquisition (1,50,000 / 220 x 317), Long term capital loss, 3. Motor car for personal use is not an asset as per section 2(14)., 4. Gold, Full value of consideration, Less: Indexed cost of acquisition (2,10,000 / 100 x 317), Long term capital gains, 5. Silver Utensils not an asset as per section 2(14), Long term capital gain, Long term capital gains on sale of Gold, Less: Long term capital loss on sale of shares, Long term capital gain after adjusting long term loss, Gross Total Income, Less: Deduction u/s 80C to 80U, Total Income, Computation of Tax Liability, Tax on `1,34,42,550 (`1,36,92,550 – `2,50,000) @ 20%, Add: Surcharge @ 15%, Tax before health & education cess, Add: HEC @ 4%, Tax Liability, Rounded off u/s 288B, Solution 18:, Computation of capital gains, Option 1:Section 54F Exemption taken from Shares, 1. Shares, , `, 160,00,000.00, (16,64,250.00), (5,76,363.63), (85,000.00), 1,36,74,386.37, 1,00,000.00, (2,16,136.36), (1,16,136.36), 8,00,000.00, (6,65,700.00), 1,34,300.00, 1,36,74,386.37, 1,34,300.00, (1,16,136.36), 1,36,92,550.01, 1,36,92,550.01, Nil, 1,36,92,550.00, 26,88,510.00, 4,03,276.50, 30,91,786.50, 1,23,671.46, 32,15,457.96, 32,15,460.00, `
Page 930 :
Income Under The Head Capital Gains, , 29, , Full value of consideration, 12,00,000.00, Less: Indexed cost of acquisition, = 1,30,000 / Index of 07-08 x Index of 21-22, = 1,30,000 / 129 x 317 = `3,19,457.36, (3,19,457.36), Long term capital gain, 8,80,542.64, Less: Exemption u/s 54F, = 8,80,542.64 / 12,00,000 x 3,60,000 = 2,64,162.79, (2,64,162.79), Long term capital gain, 6,16,379.85, 2. Jewellery, Full value of consideration, 18,00,000.00, Less: Indexed cost of acquisition, = 3,50,000 / Index of 01-02 x Index of 21-22, = 3,50,000 / 100 x 317 = `11,09,500, (11,09,500.00), Less: Selling Expenses, (30,000.00), Long term capital gain, 6,60,500.00, 3. Debentures, Full value of consideration, 1,40,000.00, Less: Cost of acquisition, (80,000.00), Short term capital gain, 60,000.00, 4. Motor car: is covered under the personal movable effects, hence, no capital gains shall be computed, 5. Equity Shares, Full value of consideration, 10,00,000.00, Less: Cost of acquisition, (2,00,000.00), Short term capital gain u/s section 111A, 8,00,000.00, Income under the head Capital gain, Long Term Capital Gains, 12,76,879.85, Short Term Capital Gains, 8,60,000.00, Gross Total Income, 21,36,879.85, Less: Deduction u/s 80C to 80U, Nil, Total Income, 21,36,879.85, Rounded off u/s 288A, 21,36,880.00, Computation of Tax Liability, Tax on `10,86,880 (`12,76,880 – ` 1,90,000) @ 20% u/s 112, Tax on `8,00,000 @ 15% u/s 111A, Tax on `60,000 at slab rate, Tax before health & education cess, Add: HEC @ 4%, Tax Liability, Rounded off u/s 288B, Option 2:Section 54F Exemption taken from Jewellery, 1. Shares, Full value of consideration, Less: Indexed cost of acquisition, = 1,30,000 / Index of 07-08 x Index of 21-22, = 1,30,000 / 129 x 317 = `3,19,457.36, Long term capital gain, 2. Jewellery, Full value of consideration, Less: Indexed cost of acquisition, = 3,50,000 / Index of 01-02 x Index of 21-22, = 3,50,000 / 100 x 317 = `11,09,500, Less: Selling Expenses, , 2,17,376.00, 1,20,000.00, Nil, 3,37,376.00, 13,495.04, 3,50,871.04, 3,50,870.00, , 12,00,000.00, (3,19,457.36), 8,80,542.64, 18,00,000.00, (11,09,500.00), (30,000.00)
Page 931 :
Income Under The Head Capital Gains, , 30, , Long term capital gain, 6,60,500.00, Less: Exemption u/s 54F, = 6,60,500.00 / 17,70,000 x 3,60,000 = 1,34,338.98, (1,34,338.98), Long term capital gain, 5,26,161.02, 3. Debentures, Full value of consideration, 1,40,000.00, Less: Cost of acquisition, (80,000.00), Short term capital gain, 60,000.00, 4. Motor car: is covered under the personal movable effects, hence, no capital gains shall be computed, 5. Equity Shares, Full value of consideration, 10,00,000.00, Less: Cost of acquisition, (2,00,000.00), Short term capital gain u/s section 111A, 8,00,000.00, Income under the head Capital gain, Long Term Capital Gains, 14,06,703.66, Short Term Capital Gains, 8,60,000.00, Gross Total Income, 22,66,703.66, Less: Deduction u/s 80C to 80U, Nil, Total Income, 22,66,703.66, Rounded off u/s 288A, 22,66,700.00, Computation of Tax Liability, Tax on `12,16,700 (`14,06,700 – ` 1,90,000) @ 20% u/s 112, Tax on `8,00,000 @ 15% u/s 111A, Tax on `60,000 at slab rate, Tax before health & education cess, Add: HEC @ 4%, Tax Liability, Rounded off u/s 288B, Option 1 is better., , 2,43,340.00, 1,20,000.00, Nil, 3,63,340.00, 14,533.60, 3,77,873.60, 3,77,870.00
Page 932 :
Income Under The Head Capital Gains, , 31, , EXAMINATION QUESTIONS, JULY – 2021 (NEW COURSE), Answer 3(b):, Particulars, , Amount, ₹, , (i), , (ii), , Long-term capital gain on transfer of 10,000 shares of XY Ltd. [taxable u/s, 112A @10% on amount exceeding ₹ 1,00,000], Full value of consideration [10,000 x ₹ 550], Less: Cost of acquisition, Higher of, 39,50,000, Cost of acquisition [10,000 x ₹ 395], 39,00,000, Lower of fair market value per share as on 31.1.2018 i.e.,, ₹390 per share and sale consideration i.e., ₹ 550 per share, [10,000 x ₹ 390], , 55,00,000, , (39,50,000), , Long term capital gain taxable u/s 112A, 15,50,000, Long-term capital gain exceeding ₹ 1 lakh i.e., ₹ 14,50,000 would be taxable, @10%, Sale of residential house [long-term capital asset, since held for more than 24 months], Full value of consideration [stamp duty value, since it exceeds 110% of actual, sale consideration], Less: Indexed cost of acquisition [₹ 9,00,000 x 317/264], Less: Deduction under section 54EC, No deduction under section 54EC would be allowed on investment of ₹ 19,00,000, in NHAI bonds, since such investment is made on 21st March 2022 i.e., after six, months from the date of transfer i.e., 4th September, 2022, Long-term capital gain taxable u/s 112 @ 20%, , 45,00,000, (10,80,682), 34,19,318, Nil, , 34,19,318, , Answer 3(c):, Computation of capital gains on slump sale of shop, Particulars, Sale value, Less: Expenses on sale [professional fees & brokerage], Net sale consideration, Less: Net worth (See Working Note below), Short-term capital gain [Since shop is held for not more than 36 months immediately, , ₹, 40,00,000, 80,000, 39,20,000, 10,42,500, 28,77,500, , preceding the date of transfer], Working Note:, Computation of net worth of shop, Building, , 5,00,000
Page 933 :
Income Under The Head Capital Gains, Furniture, , 32, 5,00,000, , Less: Deprecation on ₹1,50,000 @ 5%, being 50% of 10% since furniture is, put to use for less than 180 days during the previous year, , 7,500, , Debtors, , 4,92,500, 2,00,000, , Other assets, , 8,00,000, , Less: Deprecation on ₹ 2,00,000, being intangible asset @ 25%, , 50,000, , Total assets, , 7,50,000, 19,42,500, , Less: Bank loan, , 5,00,000, , Trade creditors, , 2,50,000, , Unsecured loan ₹ 2,00,000 less ₹ 50,000, being the amount waived off by, his wife, , 1,50,000, , Net worth, , 9,00,000, 10,42,500, , NOV – 2020 (NEW COURSE), Solution 4(b), Computation of total income of Mr. Govind for the A.Y.2022-23, Particulars, I, , II, , Profits and gains of business and profession, Full value of consideration [1000 shares x ₹ 2,400 per share], Less: FMV on the date of conversion (₹ 2,100 x 1000 shares], [See Note below], Capital Gains, In respect of 800 shares held as capital asset up- to the date of, sale, Full value of consideration [800 shares x ₹ 2,400 per share], , Less: Cost of acquisition [800 shares x ₹ 2,000], (See Working Note below), In respect of 1,000 shares converted into stock in trade on, 31.1.2020 (Capital gains is taxable in the P.Y.2021-22, when the, stock in trade is sold), Full value of consideration [1000 shares x ₹ 2,100, being FMV on, the date of conversion], Less: Cost of acquisition [1000 shares x ₹ 2,000], (See Working Note below), Total Income, Working Note - Cost of acquisition (per share), Higher of (i) and (ii), below, (i), ₹ 900 per share, being, In case of shares purchased - Original cost of acquisition (₹ 130), or FMV as on 1.4.2001 (₹ 900), at the option of the assessee, , ₹, , ₹, , 24,00,000, (21,00,000), , 3,00,000, , 19,20,000, (16,00,000), , 3,20,000, , 21,00,000, (20,00,000), , 1,00,000, 7,20,000, , 2,000
Page 934 :
Income Under The Head Capital Gains, , (ii), , 33, , In case of bonus shares - FMV as on 1.4.2001 (Nil or ₹ 900, at the, option of the assessee), ₹ 2,000 per share, being the lower of, FMV as on 31.1.2018 - ₹ 2,000 per share, Sale consideration – ₹ 2,400 per share, , Note – Explanation to section 55(2)(ac) defines “fair market value” as the highest price of capital asset, quoted on the stock exchange only for the purpose of the said clause (ac) i.e., to arrive at the FMV as on, 31.1.2018 for computing cost of acquisition of shares., However, the question states two prices on 31.1.2020, being the date of conversion of capital asset into stock, in trade for which we have to consider the definition of “fair market value” as per section 2(22B). As per, this definition, FMV refers to the price that the capital asset would ordinarily fetch on sale in the open, market on the relevant date. In the question, two prices are given on the relevant date i.e., the date of, conversion of capital asset into stock in trade, namely, the highest price and the closing price. The above, solution is given considering the closing price as the FMV as on 31.1.2020., Alternatively, highest price can also be considered as the FMV as on 31.1.2020. In such case, the total, income of Mr. Govind would be computed in the following manner:, Alternate Answer, Computation of total income of Mr. Govind for the A.Y.2022-23, Particulars, ₹, ₹, I, Profits and gains of business and profession, Full value of consideration [1000 shares x ₹ 2,400 per share], 24,00,000, Less: FMV on the date of conversion (₹ 2,200 x 1000 shares], [See Note above], II, , (22,00,000), , 2,00,000, , Capital Gains, In respect of 800 shares held as capital asset upto the date of, sale, Full value of consideration [800 shares x ₹ 2,400 per share], Less: Cost of acquisition [800 shares x ₹ 2,000], (See Working Note below), , In respect of 1,000 shares converted into stock in trade on, 31.1.2020 (Capital gains is taxable in the P.Y.2021-22, when the, stock in trade is sold), Full value of consideration [1000 shares x ₹ 2,200, being FMV on, the date of conversion] [See Note above], Less: Cost of acquisition [1000 shares x ₹ 2,000], (See Working Note below), Total Income, Working Note - Cost of acquisition (per share), Higher of (i) and (ii), below, (i) ₹ 900 per share, being, In case of shares purchased - Original cost of acquisition (₹ 130) or, FMV as on 1.4.2001 (₹ 900), at the option of the assessee, In case of bonus shares - FMV as on 1.4.2001 (Nil or ₹ 900, at the, option of the assessee), (ii) ₹2,000 per share, being the lower of, FMV as on 31.1.2018 - ₹ 2,000 per share, Sale consideration – ₹ 2,400 per share, , 19,20,000, (16,00,000), , 3,20,000, , 22,00,000, (20,00,000), , 2,000, , 2,00,000, 7,20,000
Page 935 :
Income Under The Head Capital Gains, , 34, , Note - It is possible to take a view that since no STT was paid on the date of conversion of capital asset,, being listed shares into stock in trade, capital gains has to be computed u/s 112 and not 112A. If this view is, taken, the total income of Mr. Govind would, accordingly, be computed in the following manner:, Computation of total income of Mr. Govind for the A.Y.2022-23, Particulars, ₹, I, , II, , Profits and gains of business and profession, Full value of consideration [1000 shares x ₹ 2,400 per share], Less: FMV on the date of conversion (₹ 2,100 x 1000 shares], [See Note below], Capital Gains, In respect of 800 shares held as capital asset up- to the date of, sale, Full value of consideration [800 shares x ₹ 2,400 per share], Less: Indexed cost of acquisition [800 shares x ₹ 900 x 317/100], (See Working Notes 1 and 2 below), In respect of 1,000 shares converted into stock in trade on, 31.1.2020 (Capital gains is taxable in the P.Y.2021-22, when the, stock in trade is sold), Full value of consideration [1000 shares x ₹ 2,100, being FMV on, the date of conversion], Less: Cost of acquisition [1000 shares x ₹ 900 x 289/100] (See, Working Notes 1 and 2 below)], , Long-term capital loss to be carried forward = (3,62,400) +, (5,01,000) =, Total Income, Working Note –, 1. Cost of acquisition (per share), Higher of (i) and (ii), below i.e., ₹ 900 per share, (i) ₹ 900 per share, being the FMV as on 1.4.2001, (ii) In case of shares purchased - Original cost of acquisition (₹ 130), In case of bonus shares – Nil, 2. In case of 800 shares sold during the year, the CII of F.Y.2021-22, i.e., 317 has to be considered to calculate indexed cost of, acquisition. In case of 1000 shares converted into stock in trade on, 31.1.2020, the CII of the year of conversion, i.e., F.Y.2019-20 i.e.,, 289 has to be considered to compute the indexed cost of acquisition., , ₹, , 24,00,000, (21,00,000), , 3,00,000, , 19,20,000, (22,82,400), (3,62,400), , 21,00,000, (26,01,000), (5,01,000), (8,63,400), ___ ____, 3,00,000, , Note – Explanation to section 55(2)(ac) defines “fair market value” as the highest price of capital asset, quoted on the stock exchange only for the purpose of the said clause (ac) i.e., to arrive at the FMV as on, 31.1.2018 for computing cost of acquisition of shares., However, the question states two prices on 31.1.2020, being the date of conversion of capital asset into stock, in trade for which we have to consider the definition of “fair market value” as per section 2(22B). As per, this definition, FMV refers to the price that the capital asset would ordinarily fetch on sale in the open, market on the relevant date. In the question, two prices are given on the relevant date i.e., the date of
Page 936 :
Income Under The Head Capital Gains, , 35, , conversion of capital asset into stock in trade, namely, the highest price and the closing price. The above, solution is given considering the closing price as the FMV as on 31.1.2020., Alternatively, highest price can also be considered as the FMV as on 31.1.2020. In such case, the total, income of Mr. Govind would be computed in the following manner:, Alternate Answer, Particulars, ₹, ₹, I, , Profits and gains of business and profession, Full value of consideration [1000 shares x ₹ 2,400 per share], Less: FMV on the date of conversion (₹ 2,200 x 1000 shares], [See Note above], , 24,00,000, (22,00,000), 2,00,000, , II, , Capital Gains, In respect of 800 shares held as capital asset up- to the date of, sale, Full value of consideration [800 shares x ₹ 2,400 per share], Less: Indexed cost of acquisition [800 shares x, ₹ 900 x 317/100] (See Working Notes 1 and 2 below), , 19,20,000, (22,82,400), (3,62,400), , In respect of 1,000 shares converted into stock in trade on, 31.1.2020 (Capital gains is taxable in the P.Y.2021-22, when the, stock in tradse is sold), Full value of consideration [1000 shares x ₹ 2,200, being FMV on, the date of conversion], Less: Cost of acquisition [1000 shares x ₹ 900 x 289/100] (See, Working Notes 1 and 2 below)], Long-term capital loss to be carried forward = (3,62,400) +, (4,01,000) =, Total Income, , 22,00,000, , 26,01,000, (4,01,000), (7,63,400), , _______, 2,00,000, , Working Note –, 1. Cost of acquisition (per share), Higher of (i) and (ii), below i.e., ₹ 900 per share, (i), (ii), 2., , ₹ 900 per share, being the FMV as on 1.4.2001, In case of shares purchased - Original cost of acquisition (₹130), In case of bonus shares – Nil, In case of 800 shares sold during the year, the CII of F.Y.2021-22, i.e., 317 has to be considered to calculate indexed cost of, acquisition. In case of 1000 shares converted into stock in trade on, 31.1.2020, the CII of the year of conversion, i.e., F.Y.2019-20 i.e.,, 289 has to be considered to compute the indexed cost of, acquisition., , NOV – 2017, Solution 4(b):, As per section 50C, If Full Value of consideration claimed by an Assessee is less than the Stamp Duty, Value, in such cases FVC shall be taken to be Stamp duty value.
Page 937 :
Income Under The Head Capital Gains, , 36, , If the date of agreement and date of registration are different in that case value on the agreement shall be, taken into consideration provided some advance was given otherwise than in cash on or before the, agreement., In the given case 20% amount was paid on the date of agreement through account payee bank draft hence, value as on the date of agreement shall be considered., Since Full Value of consideration is less than the Stamp duty Value on the date of agreement hence Stamp, Duty Value shall be considered as full value of consideration i.e. ₹90,00,000., Computation of Capital Gains u/s 48, Full value of consideration, 90,00,000.00, Less: Indexed cost of acquisition, (10,00,000 / 100 x 317), (31,70,000.00), Long term capital gain, 58,30,000.00, Less: Investment in house property section 54, (35,00,000.00), Long Term Capital Gains, 23,30,000.00, Gross Total Income, 23,30,000.00, Less: Deduction u/s 80C-80U, Nil, Total Income, 23,30,000.00, Computation of Tax Liability, Tax on LTCG ₹20,80,000 (23,30,000- 2,50,000) @ 20%, Add: HEC @ 4%, Tax Liability, , 4,16,000.00, 16,640.00, 4,32,640.00, , MAY– 2017, Solution 5(a):, Computation of Capital Gains of Mr. Y for the Assessment Year 2022-23, Full value of consideration, Less: Indexed cost of acquisition, Indexed cost of land (88,00,000 / 113 x 317), Indexed cost of building (100,00,000 / 122 x 317), Less: Brokerage, Long term capital gain, Less: Investment in house property section 54, Less: Investment in NHAI section 54EC (assumed redeemable after 5 years), Long Term Capital Gains, , ₹, 780,00,000.00, (246,86,725.66), (259,83,606.56), (7,00,000.00), 266,29,667.78, (110,00,000.00), (50,00,000.00), 106,29,667.78, , Note:, 1. Registration and other expenses paid at the time of purchase shall be part of the cost., 2. Stamp duty value on the date of actual sale shall be taken in to consideration as per section 50C, because advance was paid in cash., 3. Maximum Deduction allowed u/s 54EC during a particular previous year shall be ₹50,00,000., 4. Residential house purchased in India is eligible for exemption u/s 54. (Residential house purchased, outside India is not eligible for exemption u/s 54.), , NOV– 2016, Solution 2(a):, Computation of total income and tax liability of Mr. X for A.Y. 2022-23, ₹
Page 938 :
Income Under The Head Capital Gains, Capital Gains on sale of residential house property, Value declared by Mr. X, ₹85,00,000, Value adopted by Stamp Valuation Authority, ₹112,00,000, Valuation as per Valuation Officer, ₹115,00,000, Full value of consideration, Less: Brokerage @ 1% of sale consideration, Less: Indexed cost of acquisition (₹50,00,000 /167× 317), Indexed cost of improvement (₹8,00,000 /184× 317), Long-term capital gain, Total Income, Rounded off u/s 288A, Computation of Tax Liability, Tax on LTCG ₹ Nil (2,45,720- 2,45,720) @ 20%, , 37, , 112,00,000.00, (85,000.00), (94,91,017.96), (13,78,260.87), 2,45,721.17, 2,45,721.17, 2,45,720.00, NIL, , NOV– 2015, Solution 3(a):, Computation of Capital Gains of Mr. X for the Assessment Year 2022-23, Full value of consideration, Less: Indexed cost of acquisition, Indexed cost of building (20,00,000 / 122 x 317), Less: Brokerage, Long term capital gain, Less: Investment in house property section 54, Less: Investment in NHAI section 54EC (assumed redeemable after 5 years), Less: Deposited in Capital Gain Account Scheme, Long Term Capital Gains, Gross Total Income, Less: Deduction from 80C to 80U, Total Income, Rounded off u/s 288A, Computation of Tax Liability, Tax on LTCG ₹ 0 (₹ 1,03,280 – 1,03,280) @ 20%, Tax Liability, , ₹, 83,00,000.00, (51,96,721.31), (1,00,000.00), 30,03,278.69, (11,00,000.00), (8,00,000.00), (10,00,000.00), 1,03,278.69, 1,03,278.69, Nil, 1,03,278.69, 1,03,280.00, Nil, Nil, , MAY – 2015, Solution 5(a):, Computation of Total Income of Mrs. X for A.Y. 2022-23, Computation of Long term Capital Gain, Sale consideration, ₹ 37,00,000, Valuation made by Sub Registrar for stamp duty, ₹ 50,00,000, Valuation made by the Divisional Revenue officer on a reference, ₹ 46,00,000, Applying the provisions of section 50C to the present case, ₹46,00,000, being, the value adopted by the, Divisional Revenue officer for stamp duty, shall be taken as the sale consideration for the purpose of charge, of capital gain., Sale consideration as per section 50C of the Act, 46,00,000.00, Less: Indexed cost of acquisition = 26,50,000/184 x 317, (45,65,489.13), Long term capital gain, 34,510.87, Other Income, 2,80,000.00, Gross Total Income, 3,14,510.87, Less: Deduction u/s 80C to 80U, Nil, Total Income, 3,14,510.87
Page 939 :
Income Under The Head Capital Gains, Rounded off u/s 288A, Computation of Total Income of Mrs. Y for A.Y. 2022-23, Income under the head other sources, Gift (44,00,000 – 37,00,000), Other Income, Gross Total Income, Less: Deduction u/s 80C to 80U, Total Income, , 38, 3,14,510.00, 7,00,000.00, 3,45,000.00, 10,45,000.00, Nil, 10,45,000.00, , MAY – 2014, Solution 5(a):, Computation of Capital Gains chargeable to tax in the hands of Mr. X for the A.Y. 2022-23, ₹, Full value of Consideration, 70,00,000.00, Less: Indexed cost of acquisition [₹ 6,00,000 /122 x 317], (15,59,016.39), Less: Brokerage @ 2%, (1,06,000.00), Long-term capital gain, 53,34,983.61, Less: Exemption under section 54, - Acquisition of residential house property at Kolkata, (10,00,000.00), - Amount deposited in capital gains accounts scheme, (4,00,000.00), Exemption under section 54EC, Amount deposited in capital gains bonds of RECL on 10.04.2022, (4,00,000.00), Long-term capital gain, 35,34,983.61, Total Income (rounded off u/s 288A), 35,34,980.00, Computation of Tax Liability, Tax on LTCG ₹32,84,980 (₹35,34,980 – 2,50,000) @ 20%, 6,56,996.00, Add: HEC @ 4%, 26,279.84, Tax Liability, 6,83,275.84, Rounded off u/s 288B, 6,83,280.00, Note: As per the decision of Gauhati High Court in CIT vs Rajesh Kumar Jalan (2006) and Punjab &, Haryana High Court in CIT vs Jagriti Aggarwal (2011), exemption under section 54 is allowable even if the, amount of capital gain is deposited in Capital Gains Accounts Scheme after the due date specified under, section 139(1) but before the period specified for filing a belated return under section 139(4)., If we apply the above interpretation in this case, Mr. X would be eligible for exemption under section 54 in, respect of ₹ 3,00,000 deposited in Capital Gains Accounts Scheme on 01.11.2022 also, since the said date, falls within the time specified under section 139(4). On the basis of this interpretation, the long term capital, gain chargeable to tax in the hands of Mr. X would be ₹ 32,34,980 and the consequent tax liability would, also be ₹ 6,20,880., , NOV – 2013, Solution 4(a): As per section 50C, FVC shall be taken to be ₹22,00,000 for land and ₹13,00,000 for the, building and capital gains shall be computed separately for land and building., In the given problem, land has been held for a period exceeding 24 months and building for a period less, than 24 months. Therefore, land is a long-term capital asset, whereas building is a short-term capital asset., Computation of Capital Gains chargeable to tax, Particulars, ₹, ₹, Long term capital gain on sale of land, 22,00,000.00, Consideration received or accruing as a result of transfer of land, Less: Indexed cost of acquisition ₹ 5,19,000 /122 x 317, (13,48,549.18) 8,51,450.82, Long-term capital gain (A), Short-term capital loss on sale of building
Page 940 :
Income Under The Head Capital Gains, Consideration received or accruing from transfer of building, Less: Cost of construction, Short term capital loss (B), Long-term capital gain (A-B), , 39, 13,00,000, (14,00,000), , Solution 6(a):, Computation of total income of Mr. X for the A.Y 2022-23, Particulars, I., Income from house property, Gross Annual Value, Less: Municipal taxes paid, Net Annual Value (NAV), Less: Deductions under section 24, (a) 30% of NAV u/s 24(a), (b) Interest on housing loan u/s 24(b), Income under the head House Property, II. Income from business, Income from business, Less : Current year depreciation under section 32, Less: Set-off of brought forward business loss of A.Y.2016-17 under section 72, Less: Unabsorbed depreciation, Income under the head Business/Profession, III. Capital gains, Long term capital gain on sale of land, Short terms capital gains on sale of land (6,40,000 – 4,10,000), Assessee has the option to set off remaining depreciation of ₹90,000 (1,55,000 – ₹65,000), from normal income or from LTCG, Option I, Normal income shall be 4,13,000 (₹1,83,000 + 2,30,000), If depreciation is set off from normal income, balance amount of normal income shall be, (4,13,000 – 90,000), Long term capital gain, Gross Total Income, Less : Chapter VI-A deduction section 80C [Principal repayment of housing loan], Total income, Computation of Tax Liability, Tax on ₹2,53,000 at slab rate, Tax on LTCG ₹60,000 @ 20%, Tax before rebate, Less: rebate u/s 87A, Tax Liability, Option II, If depreciation ₹60,000 is set off from LTCG and balance from normal income (house, property), tax liability shall be, Long term capital gains (60,000 – 60,000), Normal income (4,13,000 – 30,000), Gross Total Income, Less : Chapter VI-A deduction section 80C [Principal repayment of housing loan], , (1,00,000), 7,51,450.82, , ₹, 4,32,000, (32,000), 4,00,000, (1,20,000), (97,000), 1,83,000, 1,75,000, (40,000), 1,35,000, (70,000), 65,000, (65,000), Nil, 60,000, 2,30,000, , 3,23,000, 60,000, 3,83,000, (70,000), 3,13,000, 150.00, 12,000.00, 12,150.00, (12,150.00), Nil, , Nil, 3,83,000, 3,83,000, (70,000)
Page 941 :
Income Under The Head Capital Gains, Total Income, Computation of Tax Liability, Tax on ₹3,13,000 at slab rate, Less: Rebate u/s 87A, Tax Liability, Tax liability in both the Option is same. Assessee can choose any 1., , 40, 3,13,000, 3,150.00, (3,150.00), Nil, , MAY – 2013, Solution 6(a):, (a) In case A Ltd. is an unlisted company and STT was not applicable at the time of sale, Computation of capital gains of Mrs. X for the A.Y. 2022-23, Particulars, 500 equity shares, Sale proceeds (500 × ₹ 300), Less : Indexed cost of acquisition [₹ 50 × 500/100 × 317], Less : Brokerage paid (2% of ₹ 1,50,000), Long term capital Gain (A), 50 Bonus shares (Allotted in September, 2000), Sale proceeds (50 × ₹ 300), Less : Indexed cost of acquisition [₹ 50 × 50 /100 × 317] [See Note below], Less : Brokerage paid (2% of ₹ 15,000), Long term capital Gain (B), , ₹, 1,50,000, (79,250), (3,000), 67,750, 15,000, (7,925), (300), 6,775, , 550 Bonus shares (Allotted in March, 2005), Sale proceeds (550 × ₹ 300), 1,65,000, Less: Cost of acquisition, NIL, Less: Brokerage paid (2% of ₹ 1,65,000), (3,300), Long term capital gain (C), 1,61,700, Taxable long term capital Gain (A+B+C), 2,36,225, Note: In case of bonus shares allotted before 01.04.2001, the fair market value as on 01.04.2001 is taken as, the cost of acquisition. In case of bonus shares allotted after 01.04.2001, the cost of acquisition will be Nil., (b) In case A Ltd. is a listed company and the shares are sold in a recognized stock exchange and STT, was paid at the time of sale., Computation of capital gains of Mrs. X for the A.Y. 2022-23, Particulars, ₹, 500 equity shares, Sale proceeds (500 × ₹ 300), 1,50,000, Less : cost of acquisition, (1,00,000), Higher of, (i) COA = 500 x 50 = 25,000, (ii) lower of, (a) FMV as on 31-01-2018 = 500 shares x 200 = 1,00,000, (b) sale value = 500 shares x 300 = 1,50,000, COA = 1,00,000, Less : Brokerage paid (2% of ₹ 1,50,000), Long term capital Gain (A), 50 Bonus shares (Allotted in September, 2000), Sale proceeds (50 × ₹ 300), Less : cost of acquisition, , (3,000), 47,000, 15,000, (10,000)
Page 942 :
Income Under The Head Capital Gains, Higher of, (i) COA = 50 x 50 = 2,500, (ii) lower of, (a) FMV as on 31-01-2018 = 50 shares x 200 = 10,000, (b) sale value = 50 shares x 300 = 15,000, COA = 10,000, Less : Brokerage paid (2% of ₹ 15,000), Long term capital Gain (B), 550 Bonus shares (Allotted in March, 2005), Sale proceeds (550 × ₹ 300), Less: Cost of acquisition, Higher of, (i) COA = Nil, (ii) lower of, (a) FMV as on 31-01-2018 = 550 shares x 200 = 1,10,000, (b) sale value = 550 shares x 300 = 1,65,000, COA = 1,10,000, Less: Brokerage paid (2% of ₹ 1,65,000), Long term capital gain (C), Taxable long term capital Gain (A+B+C) u/s 112A, , 41, , (300), 4,700, 1,65,000, (1,10,000), , (3,300), 51,700, 1,03,400, , Note: In case of bonus shares allotted before 01.04.2001, the fair market value as on 01.04.2001 is taken as, the cost of acquisition. In case of bonus shares allotted after 01.04.2001, the cost of acquisition will be Nil., , NOV – 2012, Solution 3(a):, Computation of total income and tax liability of Mr. X for A.Y. 2022-23, Previous year 2010-11, Computation of Capital Gains under section 45(2), Full value of consideration, Less: Indexed cost of acquisition, = 8,00,000 / Index of 01-02 x Index of 10-11, = 8,00,000 / 100 x 167 = ₹13,36,000, Long Term Capital Gain, Previous year 2021-22, Long Term Capital Gain, Business Income, (₹70,00,000-₹30,00,000), Capital Gains on sale of residential house property, Value declared by Mr. X, ₹ 63,50,000, Value adopted by Stamp Valuation Authority, ₹ 85,50,000, Valuation as per Valuation Officer, ₹ 87,20,000, Gross Sale consideration, Less: Brokerage @ 1% of sale consideration, Net Sale consideration, Less: Cost of acquisition, Less: Cost of Improvement, Short-term capital gain, Total Income, , ₹, 30,00,000.00, (13,36,000.00), 16,64,000.00, 16,64,000.00, 40,00,000.00, , 85,50,000.00, (63,500.00), 84,86,500.00, (15,00,000.00), (10,00,000.00), 59,86,500.00
Page 943 :
Income Under The Head Capital Gains, Long Term Capital Gain, Business Income, Short-term capital gain, Gross Total Income, Less: Deduction u/s 80C to 80U, Total Income, , 42, 16,64,000.00, 40,00,000.00, 59,86,500.00, 1,16,50,500.00, Nil, 1,16,50,500.00, , Computation of Tax Liability, Tax on LTCG ₹16,64,000 @ 20% u/s 112, 3,32,800.00, Tax on ₹99,86,500 @ slab rate, 28,08,450.00, Tax before surcharge, 31,41,250.00, Add: Surcharge @ 15%, 4,71,187.50, Tax before health & education cess, 36,12,437.50, Add: HEC @ 4%, 1,44,497.50, Tax Liability, 37,56,935.00, Rounded off u/s 288B, 37,56,940.00, Note: In case of conversion of capital assets in to stock in trade, time period for investment shall be, determined from the date of sale of stock in trade, hence in the given case exemption u/s 54F shall be, allowed but at the same time exemption shall be withdrawn because house has been sold in the same year., , MAY – 2012, Solution 3:, Computation of Long term Capital Gain for A.Y. 2022-23, Sale consideration, ₹ 30,00,000, Valuation made by registration authority for stamp duty, ₹ 34,00,000, Valuation made by the valuation officer on a reference, ₹ 35,00,000, Applying the provisions of section 50C to the present case, ₹ 34,00,000, being, the value adopted by the, registration authority for stamp duty, shall be taken as the sale consideration for the purpose of charge of, capital gain., Sale consideration as per section 50C of the Act, 34,00,000.00, Less: Indexed cost of acquisition = 1,10,000 /100 x 317, (3,48,700.00), Indexed cost of improvement = 3,20,000 /105 x 317, (9,66,095.24), Long term capital gain, 20,85,204.76, Less: Short term capital loss 2014-15, (50,000.00), Long term capital gains, 20,35,204.76, Since there is general exemption of ₹2,50,000, assessee should invest in the bonds of NHAI ₹20,35,204.76 –, 2,50,000 = 17,85,204.76, Solution 6:, , Computation of capital gain for the Assessment Year 2022-23, , Full value of consideration, Less: Indexed cost of acquisition, = ₹3,00,000/ Index of 02-03 x Index of 21-22, = ₹ 3,00,000 / 105 x 317 = ₹8,60,000.00, Less: Indexed cost of Improvement, = ₹15,00,000/ Index of 09-10 x Index of 21-22, = ₹ 15,00,000 / 148 x 317 = ₹32,12,837.84, Less: Indexed cost of Improvement, = ₹5,00,000/ Index of 11-12 x Index of 21-22, = ₹ 5,00,000 / 184 x 317 = ₹8,61,413.04, Long Term Capital Gain, Answer 6:, , ₹, 65,00,000.00, (9,05,714.29), (32,12,837.84), (8,61,413.04), 15,20,034.83
Page 944 :
Income Under The Head Capital Gains, , 43, , As per section 47, reverse mortgage shall not be considered to be transfer for the purpose of capital gain., Under reverse mortgage, a senior citizen can mortgage his house property to the bank and the bank shall, grant a loan against the security of house property and such loan shall be given in monthly installments and, the amount so received shall not be considered to be income of the mortgagor under section 10(43)., After the death of the mortgagor the bank shall have right to sell off the property and shall adjust loan and, interest and shall compute capital gains for the deceased person and shall pay tax to the government., The purpose of the scheme is to make available regular amount to the persons who do not have regular, income but are the owners of the house property., In general, the mortgagors repay the loan in installments but in this case mortgagee i.e. bank is paying, installment to the mortgagor and hence it is called reverse mortgage., , NOV – 2011, Solution 3:, Computation of capital gains for A.Y. 2022-23, ₹, Full value of consideration, 80,00,000.00, (See Note-1), Less: Indexed cost of acquisition, = 10,00,000 / Index of 2001-2002 x Index of 2021-2022, = 10,00,000 / 100 x 317 = ₹31,70,000, (31,70,000.00), Less: Indexed cost of improvement, = 2,00,000 / Index of 2004-2005 x Index of 2021-2022, = 2,00,000 / 113 x 317 = ₹5,61,061.95, (5,61,061.95), Long term capital gain, 42,68,938.05, Less: Exempted u/s 54, -Purchase of new house, (25,00,000.00), Long term capital gain, 17,68,938.05, Gross Total Income, 17,68,938.05, Less: Deduction u/s 80C to 80U, Nil, Total Income (rounded off u/s 288A), 17,68,940.00, Note:1- Since the value adopted by stamp valuation authority is higher than the sale value, hence, the value, determined by stamp valuation authority shall be the sale consideration., Note:-2 No exemption u/s 54EC is available since capital gain bonds are purchased after 6 months, from the date of sale, Tax consequences in case the property purchased in January 2021 sold on December 2022, Tax consequences shall be for the Assessment Year 2023-24, Full value of consideration, 40,00,000.00, Less: Cost of Acquisition, NIL, ( 25,00,000-25,00,000), Short term capital gain, 40,00,000.00, Solution 3:, Computation of Total Income and Tax Liability for A.Y. 2022-23, Original Shares, Full value of consideration, (100 x 3000), Less: Cost of Acquisition, Higher of, (i) COA = 100 x 1,000 = 1,00,000, (ii) lower of, (a) FMV as on 31-01-2018 = 100 shares x 2000 = 2,00,000, (b) sale value = 100 shares x 3000 = 3,00,000, COA = 2,00,000, Less: Expenses (3,00,000 x 1%), , ₹, 3,00,000.00, (2,00,000.00), , (3,000.00)
Page 945 :
Income Under The Head Capital Gains, , 44, , Long term capital gain u/s 112A, Bonus shares, Full value of consideration, (100 x 3000), Less: Cost of Acquisition, Less: Expenses (3,00,000 x 1%), Short term capital gain u/s 111A, Gross Total Income, Less: Deduction u/s 80C to 80U, Total income, Computation of Tax Liability, Tax on LTCG Nil (97,000-97,000) @ 10% u/s 112A, Tax on STCG ₹47,000 (2,97,000-2,50,000) @ 15% u/s 111A, (Since normal income is below exemption limit hence will be reduced from STCG u/s 111A), Less: Rebate u/s 87A, Tax Liability, Solution 1:, , 97,000.00, 3,00,000.00, Nil, (3,000.00), 2,97,000.00, 3,94,000.00, Nil, 3,94,000.00, Nil, 7,050.00, (7,050.00), Nil, , Computation of chargeable capital gain of Mr. X for the A.Y. 2022-23, , Particulars, Sale consideration, Less: Indexed cost of acquisition (9,99,300 /117 x 317), Less: Deduction under section 54EC, 20.03.2022 RECL bonds, 16.04.2022 NHAI bonds, restricted to ₹50,00,000, Long term capital gain, Gross Total Income, Less: Deduction u/s 80C to 80U, Total Income (rounded off u/s 288A), Computation of Tax Liability, Tax on ₹20,42,490 (22,92,490 – 2,50,000) @ 20%, Add: HEC @ 4%, Tax Liability, Rounded off u/s 288B, , ₹, , (50,00,000), (20,00,000), (70,00,000), , ₹, 100,00,000.00, (27,07,505.13), 72,92,494.87, , (50,00,000.00), 22,92,494.87, 22,92,494.87, Nil, 22,92,490.00, 4,08,498.00, 16,339.92, 4,24,837.92, 4,24,840.00, , Answer 4., (i) Transfer of immovable property for inadequate consideration will (6,00,000 – 4,25,000) and difference of, 1,75,000 will be taxable in the hands of Mrs. Y have tax implication in the hands of transferee under section, 56. Therefore, in the hands of transferee, i.e., Mrs. Y, the provisions of section 56 would be attracted., However, for the transferor, Mrs. X, the value adopted for stamp duty purpose will be taken as the deemed, sale consideration under section 50C for computation of capital gains., Mrs. Y, Particulars, Mrs. X, (Transferee), (Transferor), ₹, ₹, Capital gains, 6,00,000, Deemed sale consideration under section 50C, (4,00,000), Less: Indexed cost of acquisition, 2,00,000, Other Sources, 1,75,000, Other income (computed), 50,000, 2,05,000, Total income, 2,50,000, 3,80,000
Page 946 :
Income Under The Head Capital Gains, , 45, , (ii) The provisions of section 56 would not apply to any sum of money or any property received from any, trust or institution registered under section 12AA. Therefore, the cash gift of ₹1 lakh received from ABC, Charitable Trust, being a trust registered under section 12AA, for meeting medical expenses would not be, chargeable to tax under section 56 in the hands of Mr. X., , NOV – 2010, Solution 1:, Computation of capital gains on slump sale of Unit 1, Sale value, Less: Expenses on sale, Less: Net worth (See Note (i) below), Long term capital gain, Note (i) : Computation of net worth of Unit 1 of ABC Enterprises, Building (excluding ₹3 lakhs on account of revaluation), Machinery, Debtors, Other assets, Total assets, Less:, Bank Loan, Creditors, Net worth, , ₹, 25,00,000, (28,000), (12,72,500), 11,99,500, ₹, 9,00,000, 3,00,000, 1,00,000, 1,50,000, 14,50,000, (1,40,000), (37,500), 12,72,500, , Solution 4:, , Computation of capital gains in the hands of Mr. X for the A.Y. 2022-23, Particulars, ₹, ₹, Deemed sale consideration (under section 50C), 70,00,000.00, Less: Brokerage @ 2% of ₹55,00,000, (1,10,000.00), Less: Indexed cost of acquisition 4,00,000 / 113 x 317, (11,22,123.89), 57,67,876.11, Less: Exemption under sections 54 and 54EC, Under section 54:, (i) Residential house acquired at Chennai on 10.12.2021, (13,00,000), (ii) Amount deposited in Capital Gains Accounts Scheme, on 06.07.2022 (before the due date of filing of return), for construction of additional floor on the residential house, property acquired at Chennai, (5,00,000), (18,00,000), Under section 54EC:, Amount deposited in RECL bonds on 10.04.2022 (within six, months from the date of transfer), (10,00,000) (28,00,000.00), Taxable long-term capital gain, 29,67,876.11, Total Income (Rounded off u/s 288A), 29,67,880.00, Computation of tax liability of Mr. X for A.Y. 2022-23, Tax on ₹27,17,880 (₹29,67,880 – ₹2,50,000) @ 20%, Add: HEC @ 4%, Tax liability, Rounded off u/s 288B, , 5,43,576.00, 21,743.04, 5,65,319.04, 5,65,320.00, , Answer 7., As per section 47, any transfer of a capital asset in a transaction of Reverse Mortgage under a scheme made, and notified by the Central Government will not be regarded as a transfer. Therefore, capital gains tax, liability is not attracted.
Page 947 :
Income Under The Head Capital Gains, , 46, , Section 10(43) provides that the amount received by an individual as a loan, either in lump sum or in, installments, in a transaction of Reverse Mortgage would be exempt from income-tax. Therefore, the, amount received by Mr. X in a transaction of Reverse Mortgage of his residential building is exempt under, section 10(43)., , MAY – 2010, Answer 2., As per section 50C, FVC shall be taken to be ₹22,00,000 for land and ₹10,00,000 for the building and, capital gains shall be computed separately for land and building., In the given problem, land has been held for a period exceeding 24 months and building for a period less, than 24 months. Therefore, land is a long-term capital asset, whereas building is a short-term capital asset., Particulars, ₹, Long term capital gain on sale of land, Full value of consideration, 22,00,000.00, Less: Indexed cost of acquisition 5,19,000 /137 x 317, (12,00,897.81), Long-term capital gain, 9,99,102.19, Short-term capital loss on sale of building, Full value of consideration, 10,00,000, Less: Cost of acquisition, (14,00,000), Short term capital loss, (4,00,000), As per section 70, short-term capital loss can be set-off against long-term capital gains. Therefore, the net, taxable long-term capital gains would be ₹5,99,102.19 (i.e., ₹9,99,102.19 – ₹4,00,000)., Income of Mr. Y, Income under the head other sources, (32,00,000- 25,00,000), , 7,00,000, , NOV – 2009, Solution 2:, , Computation of net taxable capital gains of Smt. X for the A.Y. 2022-23, Particulars, ₹, Full value of consideration, 19,00,000.00, Less: Indexed cost of acquisition, (10,41,240.88), = ₹4,50,000 / 137 x 317 = 10,41,240.88, Long term capital gain, 8,58,759.12, Less: Exemption under section 54, (2,00,000.00), Long term capital gain, 6,58,759.12, Income under the head other sources, 70,000.00, Gross Total Income, 7,28,759.12, Less: Deduction u/s 80C to 80U, Nil, Total Income (Rounded off u/s 288A), 7,28,760.00, Computation of Tax Liability, Tax on ₹70,000 at slab rate, Nil, Tax on ₹4,78,760 (₹6,58,760 – 1,80,000) @ 20%, 95,752.00, Add: HEC @ 4%, 3,830.08, Tax Liability, 99,582.08, Rounded off u/s 288B, 99,580.00, , NOV – 2009, Solution 4:, , Computation of Total Income and tax payable by Mr. X for the A.Y. 2022-23, ₹, ₹, 1. Income from Capital Gains, Full value of consideration, 16,00,000.00
Page 948 :
Income Under The Head Capital Gains, Less : Indexed cost of acquisition of land, ₹ 75,000 / 137 x 317, Less : Indexed cost of improvement i.e. house, ₹ 1,25,000 / 184 x 317, Less : Exemption under section 54, Cost of new residential house, Long term capital gains, 2. Income from other sources, Interest on Bank deposit, Add: Tax deducted at source, Gross Total Income, Less: Deduction under Chapter VIA :, Deduction under section 80C, Investment in NSC, Total Income, Rounded off u/s 288A, , 47, (1,73,540.15), (2,15,353.26), (8,00,000.00), 4,11,106.59, 45,000, 5,000, , 50,000.00, 4,61,106.59, (20,000.00), 4,41,106.59, 4,41,110.00, , Computation of Tax Liability, Tax on normal income of ₹30,000, Tax on LTCG {₹ 4,11,110 – 2,20,000} x 20% u/s 112, Less: Rebate u/s 87A, Tax before HEC, Add: HEC @4%, Tax Liability, Less: TDS, Tax Payable, Rounded off u/s 288B, , Nil, 38,222.00, (12,500.00), 25,722.00, 1,028.88, 26,750.88, (5,000.00), 21,750.88, 21,750.00, , JUNE – 2009, Solution 3: Computation of total income and tax liability of Mr. X for the A.Y. 2022-23, Particulars, ₹, Capital Gains:, Sale price of the residential house, 58,00,000.00, Valuation as per Stamp Valuation authority, 73,00,000.00, (Value to be taken is the higher of actual sale price or valuation, adopted for stamp duty purpose as per section 50C), Therefore, Consideration for the purpose of Capital Gains, 73,00,000.00, Less: Indexed Cost of Acquisition = 30,00,000 / 200 x 317, (47,55,000.00), Long-term Capital gain, 25,45,000.00, Less: Exemption u/s 54, (12,00,000.00), Less: Exemption u/s 54EC, (8,00,000.00), Long-term Capital gain, 5,45,000.00, Income from other sources:, Interest on fixed bank deposits, 32,000.00, Gross Total Income, 5,77,000.00, Less: Deduction under Chapter VI-A, Section 80C – Investment in PPF, (12,000.00), Total Income, 5,65,000.00, Computation of Tax Liability, Tax on normal income of ₹20,000, Nil, Tax on LTCG {₹ 5,45,000 – 2,30,000} x 20% u/s 112, 63,000.00, Add : HEC @ 4%, 2,520.00, Tax Liability, 65,520.00
Page 949 :
Income Under The Head Capital Gains, , 48, , Answer 4. As per section 47, any transfer of a capital asset in a transaction of reverse mortgage under a, scheme made and notified by the Central Government shall not be considered as a transfer for the purpose of, capital gain., Accordingly, the transaction made by Mr. X's father will not be regarded as a transfer. Therefore, no capital, gain will be charged on such transaction., Further, section 10(43) provides that the amount received as a loan, either in lump sum or in installment, in a, transaction of reverse mortgage would be exempt from income-tax., However, capital gains tax liability would be attracted at the stage of alienation of the mortgaged property, by the bank for the purposes of recovering the loan., , NOV – 2008, Answer 1. The statement is false., The exemption under section 54EC has been restricted, by limiting the maximum investment in long term, specified assets (i.e. bonds of NHAI or RECL, redeemable after 5 years) to ₹50 lakh during any financial, year. Therefore, in this case, the exemption under section 54EC can be availed only to the extent of ₹50, lakh., , NOV – 2007, Solution 4:, Computation of Long term Capital Gain for A.Y. 2022-23, Sale consideration as per section 50C of the Act, Less: Indexed cost of acquisition = 2,70,000 /100 x 317, Less: Indexed cost of improvement = 7,00,000 /117 x 317, Less: @ 1% of sale consideration of ₹ 37.50 lacs, Long term capital gain, , 47,25,000.00, (8,55,900.00), (18,96,581.20), (37,500.00), 19,35,018.80
Page 950 :
Income Under The Head Business/Profession, , 49, , PROFITS AND GAINS, OF, , BUSINESS OR PROFESSION, , “PGBP”, SECTION 28 TO 44DB, , SOLUTIONS OF MCQS, Answer, 1. (b); 2. (a); 3.(a); 4. (d); 5. (b); 6. (c); 7. (d); 8. (b); 9. (a); 10. (c); 11. (b); 12. (b); 13. (a); 14.(c); 15. (c);, 16. (b); 17. (c); 18. (f); 19. (d); 20. (b); 21. (c); 22. (a); 23. (f); 24. (a); 25. (c); 26. (a); 27. (a); 28. (a); 29., (b); 30. (b)
Page 951 :
Income Under The Head Business/Profession, , 50, , SOLUTIONS, TO, , PRACTICE PROBLEMS, Solution 1:, Block I, Plant and machinery, depreciation @ 15%, Written down value as on 1st, April 2021, Add: Purchases of 2 Plants on June 2nd, 2021 and put to use on the same date, Less: Sale of Plants on 30.11.2021, Add: Purchase of 2 Plants on Dec 15th, 2021, Written down value as on 31.03.2022, Dep. @ 7.5% on `1,40,000, Block II, Building, depreciation @ 10%, Written down value as on 1st, April 2021, Dep. @ 10% on `6,50,000, Total depreciation for Assessment Year 2022-23, Solution 2:, Situation 1, Written down value of Plant A and Plant B as on 01.04.2021, Add: Plant C purchased on 10.06.2021 and put to use on the same date, Less: Insurance claim of plant B, Written down value as on 31.03.2022, Depreciation @ 15% on `2,90,000, Situation 2, Written down value of Plant A and Plant B as on 01.04.2021, Add: Plant C purchased on 10.06.2021 and put to use on the same date, Less: Insurance claim of Plant B, Short term capital gain as per section 50, No depreciation is allowed, Situation 3, Written down value of plant A and Plant B as on 01.04.2021, Add: Plant C purchased on 10.06.2021 and put to use on the same date, Less: Insurance claim of Plant A, B and C, Short term loss as per section 50, No depreciation is allowed, Situation 4, Written down value of plant A and Plant B as on 01.04.2021, Add: Plant C purchased on 10.06.2021 and put to use on the same date, Less: Insurance claim of plant A, B and C, Short term capital gain as per section 50, No depreciation is allowed, , `, 2,70,000, 2,10,000, (5,00,000), 1,60,000, 1,40,000, 10,500, 6,50,000, 65,000, 75,500, `, 2,40,000, 60,000, (10,000), 2,90,000, 43,500, 2,40,000, 60,000, (3,70,000), 70,000, 2,40,000, 60,000, (20,000), (2,80,000), 2,40,000, 60,000, (4,00,000), 1,00,000
Page 952 :
Income Under The Head Business/Profession, Solution 3:, Computation of Total Income, • Visiting fees, • Consultation fees, • Sale of medicines, • Operation theatre rent, • Cash payment not recorded in books, Less:, • Clinic expenses, • Depreciation on medical books @ 40%, • Depreciation on surgical equipment, Working Note:, `, w.d.v as on 01.04.2021, 60,000, Sale of surgical instruments, (30,000), Purchase on 01.11.2021, 90,000, Balance, 1,20,000, Depreciation @ 7.5% on 90,000, 6,750, Depreciation @ 15% on 30,000, 4,500, rd, • Motor car expenses (2/3 of `36,000), • Depreciation on car, • Indian Medical Association membership fees, • Payment to C.A. for filing return of income, • Entertainment expenses, • Medical purchases, Income under the head Business/Profession, Income under the head House Property, Gross Annual Value, Working Note:, `, (a) Municipal valuation, 66,000, (b) Expected Rent, 66,000, (c) Rent received/receivable = 5,500 x 11 =, 60,500, GAV = Higher of (b) or (c), 66,000, Less: municipal tax, Net Annual Value, Less: 30% of NAV u/s 24(a), Less: Interest on capital borrowed u/s 24(b), Income under the head House Property, Income under the head Business/Profession, Income under the head Other Sources (Dividend), Gross Total Income, Less: Deduction u/s 80C to 80U, Total Income, Computation of Tax Liability, Tax on `54,03,950 at slab rate, Add: Surcharge @ 10%, Tax Before health & education cess, Add: HEC @ 4%, Tax Liability, Rounded off u/s 288B, Note:, 1. Maturity proceeds of life policy is exempt from tax under section 10(10D)., 2. Membership fees given to the professional bodies is allowed u/s 37(1)., , 51, `, 45,75,000.00, 9,15,000.00, 28,000.00, 18,000.00, 75,000.00, (1,24,000.00), (6,000.00), (11,250.00), , (24,000.00), (12,000.00), (7,000.00), (4,000.00), (24,000.00), (33,000.00), 53,65,750.00, 66,000.00, , Nil, 66,000.00, (19,800.00), (30,000.00), 16,200.00, 53,65,750.00, 22,000.00, 54,03,950.00, Nil, 54,03,950.00, 14,33,685.00, 1,43,368.50, 15,77,053.50, 63,082.14, 16,40,135.64, 16,40,140.00
Page 953 :
Income Under The Head Business/Profession, Solution 4:, Computation of income under the head Business/Profession, Consultancy fee, Less:, Rent (50%), Office expenses, Depreciation on car, {3,00,000 x 15% x 70% = 31,500}, Depreciation on computer @ 40% (50,000 x 40%), Depreciation on legal books, {(12,000 x 40%) + (18,000 x 20%)}, Car expenses {70% x 42,000}, Electricity and water charges (50%), Income under the head Business/Profession, Computation of income under the head Other Sources, Payment from university as an evaluator, Income under the head other sources, Computation of income under the head salary, Gross Salary, Less: Standard deduction u/s 16(ia), Income under the head salary, Computation of Total Income, Income under the head Salary, Income under the head Business/Profession, Income under the head Capital Gains (STCG), Income under the head Other Sources, Gross Total Income, Less: Deduction u/s 80C (54,000 + 5,000), Total Income, Computation of Tax Payable, Tax on `54,68,200 at slab rate, Add: Surcharge @ 10%, Tax before health & education cess, Add: HEC @ 4%, Tax Liability, Less: Tax Paid in Advance, Tax Payable, Rounded off u/s 288B, Note: It is assumed that life insurance policy has been taken on or after 01.04.2012., Solution 5:, Computation of professional income as per income & expenditure account, Net profit as per profit and loss account, Add: inadmissible expenses, Domestic servant salary, Entertainment expenses, Donation for charity show, Income tax, Car expenses, Books purchased, Stationery, Rent of own building, , 52, `, 55,35,500, (72,000), (46,000), (31,500), (20,000), (8,400), (29,400), (8,000), 53,20,200, 7,000, 7,000, 45,000, (45,000), Nil, Nil, 53,20,200, 2,00,000, 7,000, 55,27,200, (59,000), 54,68,200, 14,52,960.00, 1,45,296.00, 15,98,256.00, 63,930.24, 16,62,186.24, (22,000.00), 16,40,186.24, 16,40,190.00, , `, 64,28,800.00, 1,500.00, 17,500.00, 600.00, 5,000.00, 1,250.00, 2,000.00, 21,000.00, 60,000.00
Page 954 :
Income Under The Head Business/Profession, Less:, Dividend, Profit on sale of debentures, Gift from father in law, Depreciation on building (`90,000 @ 10%), Depreciation on books (2,000 @ 40%), Depreciation on furniture, (`30,000 @ 10%), (`20,000 @ 5%), Income under the head Business/profession, Income under the head Capital Gains (STCG), Income under the head Other sources (Dividend), Gross Total Income, Less: Deduction u/s 80C to 80U, Total Income, Computation of Tax Liability, Tax on `65,17,800 at slab rate, Add: Surcharge @ 10%, Tax before health & education cess, Add: HEC @ 4%, Tax Liability, Rounded off u/s 288B, Note:, Expenses on opening ceremony are allowed under section 37(1)., Solution 6:, Computation of income under the head business/profession, Net Profit as per profit & loss account, Add: inadmissible expenses, Income tax paid, Medical expenses, (Payment of medi claim insurance is allowed u/s 36(1)(ib)), Opening stock adjustment (25,000 x 5/105), Mr. X’s salary, Total, Less:, Gift from brother, Income tax refund, Depreciation on Air conditioner @ 15%, Adjustment for sales (50,000 – 45,000), Closing stock adjustment (30,000 x 5/105), Business income, Computation of income under the head Salary, Basic Salary, (6,000 x 3), Gross Salary, Less: Standard deduction u/s 16(ia), Income under the head Salary, Gross Total Income, Less: Deduction u/s 80C, Total Income (Rounded off u/s 288A), , 53, (8,500.00), (8,450.00), (6,050.00), (9,000.00), (800.00), (3,000.00), (1,000.00), 65,00,850.00, 8,450.00, 8,500.00, 65,17,800.00, Nil, 65,17,800.00, 17,67,840.00, 1,76,784.00, 19,44,624.00, 77,784.96, 20,22,408.96, 20,22,410.00, , `, 4,26,000.00, 4,000.00, 1,200.00, 1,190.48, 2,500.00, 4,34,890.48, (10,000.00), (3,000.00), (3,750.00), (5,000.00), (1,428.57), 4,11,711.91, 18,000.00, 18,000.00, (18,000.00), Nil, 4,11,711.91, (4,800.00), 4,06,910.00
Page 955 :
Income Under The Head Business/Profession, Computation of Tax Liability, Tax on `4,06,910 at slab rate, Less: Rebate u/s 87A, Tax Liability, Solution 7:, Computation of Total Income, Income from business, Net Profit as per profit and loss account, Add: inadmissible expenses, Excessive payment to relatives {Sec. 40A(2)}, Payment disallowed {Sec. 40A(3)}, Salary paid outside India without TDS {Sec. 40(a)}, Salary paid to relatives {Sec. 40A(2)}, Market rent (rent for own building is not allowed) Sec. 30, Municipal taxes of building (due), Donation for Ram Lila, Provision for bad debts, Gift to relatives, Public provident fund, Interest on capital, Addition to business premises, Income tax, Fine for violation of traffic rules, Less:, Depreciation for building @ 10%, 10,00,000 x 10% = 1,00,000, 2,00,000 x 5% =, 10,000, Income from business, Gross Total Income, Less: Deduction u/s 80C {Public Provident Fund}, Total Income, Computation of Tax Liability, Tax on `6,40,100 at slab rate, Add: HEC @ 4%, Tax Liability, Rounded off u/s 288B, Solution 8:, Computation of Total Income, Income under the head Business/Profession, Profit as per profit and loss account, Add: Inadmissible expenses, Proprietor’s salary, Charity to poor student, Interest on proprietor capital, Provision for bad debts, Reserve for GST, Advance income-tax, Out of motor car expenses, Total, Less:, Bad debts recovered, , 54, 7,845.50, (7,845.50), Nil, `, 3,11,100.00, 20,000.00, 80,000.00, 14,000.00, 1,000.00, 1,00,000.00, 5,000.00, 1,500.00, 11,000.00, 400.00, 12,000.00, 4,000.00, 2,00,000.00, 2,000.00, 100.00, (1,10,000.00), 6,52,100.00, 6,52,100.00, (12,000.00), 6,40,100.00, 38,020.00, 1,520.80, 39,540.80, 39,540.00, `, 5,06,600.00, 15,000.00, 300.00, 2,000.00, 2,000.00, 10,000.00, 9,500.00, 300.00, 5,45,700.00, (2,000.00)
Page 956 :
Income Under The Head Business/Profession, Interest on company’s deposit, Long term capital gains, Income from business, Income under the head Other Sources, Income under the head Capital Gains (LTCG), Gross Total Income, Less: Deduction u/s 80C to 80U, Total Income, Computation of Tax Payable, Tax on LTCG `2,93,700 (`5,00,000 – 2,06,300) @ 20% u/s 112, Tax on ` 43,700 at slab rate, Add: HEC @ 4%, Tax Liability, Less: Income tax paid, Tax Payable, Rounded off u/s 288B, Solution 9:, Computation of income from profession, Gross receipts:, Audit fee, Appellate tribunal appearance, Misc. Receipt, Presents from client, Total, Payments:, Stipend, Office expenses, Office rent, Salary and wages, Printing and stationery, Subscription to C.A. institute, Depreciation on books @ 40%, Travelling expenses, Income from profession, Computation of income from house property, Gross Annual Value, Less: Municipal Tax, Net Annual Value, Less: 30% of NAV u/s 24(a), Less: Interest on capital borrowed u/s 24(b), Income from house property, Computation of Total Income, Income from profession, Income from house property, Gross total income, Less: Deduction u/s 80C to 80U, Total Income, Computation of Tax Liability, Tax on `4,20,800 at slab rate, Less: Rebate u/s 87A, Tax Liability, , 55, (5,000.00), (5,00,000.00), 38,700.00, 5,000.00, 5,00,000.00, 5,43,700.00, Nil, 5,43,700.00, 58,740.00, Nil, 2,349.60, 61,089.60, (9,500.00), 51,589.60, 51,590.00, `, 4,40,000.00, 25,000.00, 20,000.00, 10,000.00, 4,95,000.00, (12,000.00), (24,000.00), (18,000.00), (20,500.00), (4,000.00), (1,500.00), (6,000.00), (5,000.00), 4,04,000.00, 24,000.00, Nil, 24,000.00, (7,200.00), Nil, 16,800.00, 4,04,000.00, 16,800.00, 4,20,800.00, Nil, 4,20,800.00, 8,540.00, (8,540.00), Nil
Page 957 :
Income Under The Head Business/Profession, Solution 10:, Computation of Total Income, Income under the head Business/Profession, Net profit as per profit and loss account, Add: inadmissible expenses, Out of capital expenditure on promotion of family planning, amounting to `14,000 one fifth is allowed, GST, Reserve for future losses, Reserve for bad debts, Payment of advance Income tax, Less:, Capital gain, Income under the head Business/Profession, Income under the head Capital Gains (STCG), Gross Total Income, Less: Deduction u/s 80C to 80U, Total Income, Computation of Tax Liability, Tax on `1,59,200 @ 30%, Add: HEC @ 4%, Tax Liability, Rounded off u/s 288B, , 56, `, 63,000.00, 11,200.00, 24,000.00, 30,000.00, 14,000.00, 17,000.00, 1,59,200.00, (60,000.00), 99,200.00, 60,000.00, 1,59,200.00, Nil, 1,59,200.00, 47,760.00, 1,910.40, 49,670.40, 49,670.00, , Solution 11:, Computation of Total Income, Income from Business, Net profit as per profit & loss account, Add: Inadmissible expenses, Provision for income tax, Under valuation of closing stock [25/75 of ` 4,97,000], Overvaluation of opening stock [25/125 of ` 4,62,000], Municipal tax, Penalty, Total, Less:, Rental income from house property, Dividends received from companies, Income from Business, , (84,000.00), (12,000.00), 4,66,066.67, , Income from house property, Gross Annual Value, Less: municipal taxes, Net Annual Value, Less: 30% of NAV u/s 24(a), Less: Interest on capital borrowed u/s 24(b), Income from house property, Income under the head other sources (Dividend), , 84,000.00, (3,000.00), 81,000.00, (24,300.00), Nil, 56,700.00, 12,000.00, , Gross Total Income, Less: Deduction u/s 80C, , `, 2,23,000.00, 53,000.00, 1,65,666.67, 92,400.00, 3,000.00, 25,000.00, 5,62,066.67, , 5,34,766.67, (1,00,000.00)
Page 958 :
Income Under The Head Business/Profession, Total Income (rounded off u/s 288A), Computation of Tax Liability, Tax on `4,34,770 at slab rate, Less: Rebate u/s 87A, Tax Liability, Solution 12:, Computation of Total Income, Net profit as per profit and loss account, Add: inadmissible items, Advertisement in a newspaper owned by a political party (Sec 37(2B)), Capital expenditure on advertisement, Excess amount paid to a concern in which ‘X’ has substantial interest, Employee contribution to recognised provident fund (to the extent not deposited, before the due date), Bonus being paid to employee after the due date of filing the return, Commission to employee after the due date of filing the return, Salary paid outside India in respect of which tax is not deducted at source, Capital expenditure for promoting family planning amongst employees, (allowed only to a company assessee), Total, Less:, Depreciation on capital expenditure on advertisement @ 25% of `14,400, (assuming put to use for 180 days or more), Income under the head Business/Profession, Gross Total Income, Less: Deduction u/s 80C to 80U, Total Income, Computation of Tax Liability, Tax on `9,69,100 at slab rate, Add: HEC @ 4%, Tax Liability, Rounded off u/s 288B, Solution 13:, Computation of Total Income, Net profit as per profit & loss account, Add: inadmissible items, Travelling expenses incurred on foreign tour to the extent not admissible, [1,80,000 – (6/8 of ` 1,80,000)], Travelling expenses incurred for purchasing a machine for factory, Own salary of Mr. X, Commission to employee’s paid after the due date of furnishing return of income, under section 139(1) of the Act; hence not allowed u/s 43B, Less:, Depreciation on machinery @ 15%, Income under the head Business/Profession, Gross Total Income, Less: Deduction u/s 80C to 80U, Total Income (rounded off u/s 288A), Computation of Tax Liability, Tax on `5,08,530 at slab rate, Add: HEC @ 4%, , 57, 4,34,770.00, 9,238.50, (9,238.50), Nil, `, 7,70,000.00, 20,000.00, 14,400.00, 1,800.00, 12,500.00, 58,000.00, 44,000.00, 46,000.00, 6,000.00, 9,72,700.00, (3,600.00), 9,69,100.00, 9,69,100.00, Nil, 9,69,100.00, 1,06,320.00, 4,252.80, 1,10,572.80, 1,10,570.00, `, 3,90,000.00, 45,000.00, 6,500.00, 26,000.00, 42,000.00, (975.00), 5,08,525.00, 5,08,525.00, Nil, 5,08,530.00, 14,206.00, 568.24
Page 959 :
Income Under The Head Business/Profession, , 58, , Tax Liability, 14,774.24, Rounded off u/s 288B, 14,770.00, Note: It is assumed that machinery is not used for manufacturing purpose, so additional depreciation is not, allowed., Solution 14:, Net profit as per profit & loss account, Add: expenses debited to profit & loss account but not allowable, Deposit in NSC (not an expenditure), Provision for income tax, Provision for GST, Salary to Mrs. X (Sec 40A(2)), Purchase of computer (capital expenditure), Purchase from relative (Sec 40A(2)), Payment in cash (Sec 40A(3)), Adjustment for opening stock (9,50,000 x 10 / 110), Bonus paid after due date (Sec 43B), Municipal tax paid after due date (Sec 43B), Total, Less:, Depreciation on computer (40,000 x 40% x ½), Closing stock overvalued (3,60,000 x 10/110), Long term capital gain, Dividend from foreign company, Winnings of lottery, Business income, Income from Other Sources, Dividend from foreign company, Winnings from lottery, Income from Other Sources, Income under the head Capital Gains (LTCG), Gross Total Income, Less: Deduction u/s 80C, {Deposit in NSC}, Total Income (rounded off u/s 288A), Computation of Tax Liability, Tax on Long term capital gain `36,000 @ 20% u/s 112, Tax on `5,00,000 @ 30% u/s 115BB, Tax on `7,66,640 at slab rate, Tax before health & education cess, Add: HEC @ 4%, Tax Liability, Rounded off u/s 288B, Solution 15:, Computation of Total Income of ABC Ltd. for the Assessment Year 2022-23, Income under the head Business/Profession, Net profit as per profit and loss account, Add: inadmissible expenses, • Fine imposed by the municipality for violation of regulation, • Provision for Income Tax, • Under valuation of closing stock (1,08,000 x 1/9), Total, , `, 9,38,000.00, 42,000.00, 31,000.00, 45,000.00, 48,000.00, 40,000.00, 20,000.00, 80,000.00, 86,363.64, 25,000.00, 30,000.00, 13,85,363.64, (8,000.00), (32,727.27), (36,000.00), (12,000.00), (5,00,000.00), 7,96,636.37, 12,000.00, 5,00,000.00, 5,12,000.00, 36,000.00, 13,44,636.37, (42,000.00), 13,02,640.00, 7,200.00, 1,50,000.00, 65,828.00, 2,23,028.00, 8,921.12, 2,31,949.12, 2,31,950.00, , `, 8,00,000, 2,000, 35,000, 12,000, 8,49,000
Page 960 :
Income Under The Head Business/Profession, , 59, , Less:, • Income from units of UTI, • Dividend from Indian company, • Under valuation of opening stock, • Depreciation (1,20,000 – 60,000), Business income, Income under the head Other Sources, Dividend from Indian company, Income from UTI, Income under the head Other Sources, Gross Total Income, Less: Deduction u/s 80C to 80U, Total Income, Computation of Tax Liability, Tax on `7,79,000 @ 30%, Add: HEC @ 4%, Tax Liability, Rounded off u/s 288B, Note: Amount paid to the lawyer of `30,000 for arguing appeals before the Tribunal is an, expense., Solution 16:, Computation of income from profession of Mr. X for the Assessment Year 2022-23, Professional incomes, Professional fees, Less:, Salaries, Rent of chamber, Telephone expenses, Magazines subscription, 9/10 of motor car expenses, Dep. on motor car (3,00,000 x 7.5% x 90%), Misc. office expenses, Subscription to Bar Association, Income from profession, Computation of income from house property, Let out house, Gross Annual Value, Less: municipal taxes (6,000 – 2,000), Net annual value, Less: 30% of NAV u/s 24(a), Less: Interest on capital borrowed u/s 24(b), Income from house property, Self occupied house, Annual value, Income from house property, Loss under the head House Property, Income under the head House Property, Computation of income under the head Other Sources, Honorarium for delivering lectures in C.A. Institute, , (35,000), (20,000), (10,000), (60,000), 7,24,000, 20,000, 35,000, 55,000, 7,79,000, Nil, 7,79,000, 2,33,700, 9,348, 2,43,048, 2,43,050, allowable, , `, 16,75,000.00, (5,00,000.00), (2,55,000.00), (26,000.00), (3,000.00), (9,000.00), (20,250.00), (5,500.00), (1,500.00), 8,54,750.00, , 60,000.00, (4,000.00), 56,000.00, (16,800.00), Nil, 39,200.00, Nil, 39,200.00, (31,200.00), 8,000.00, , 5,000.00
Page 961 :
Income Under The Head Business/Profession, , 60, , Honorarium for writing articles in Hindustan Times, 1,000.00, Dividend from Indian company, 8,000.00, Dividend from UTI, 10,000.00, Interest from a company, 8,000.00, Income under the head Other Sources, 32,000.00, Computation of Total Income of Mr. X, Income under the head House Property, 8,000.00, Income under the head Business/Profession, 8,54,750.00, Income under the head Other Sources, 32,000.00, Gross Total Income, 8,94,750.00, Less: Deduction u/s 80C to 80U, Nil, Total Income, 8,94,750.00, Computation of Tax Payable, Tax on `8,94,750 at slab rate, 91,450.00, Add: HEC @ 4%, 3,658.00, Tax Liability, 95,108.00, Less: Advance Income Tax Paid, (38,000.00), Tax Payable, 57,108.00, Rounded off u/s 288B, 57,110.00, Note:, Annual value of self - occupied house is taken as nil and no deduction except interest on loan is permissible, Solution 17:, `, Computation of Total Income of ABC Ltd., Net profit as per profit and loss account, 12,50,000, Add:, (i) Payment of advertisement expenditure of ` 60,000, (a) `8,000, being the excess payment to a related disallowed under section 40A(2), (b) As the payment is made in cash and since the remaining amount of `52,000 exceeds, `10,000, shall be disallowed under section 40A(3), 60,000, (ii) Under Section 31, expenditure relatable to repairs of plant, machinery or furniture is allowed. -------(iii)Liability foregone by creditor [Taxable under section 41(1)], 6,000, (iv) Sale proceeds of import entitlement licence. The sale of the rights gives rise to, profits or gains taxable under section 28. As the amount has already been credited, to profit and loss a/c, no further adjustment is necessary., ------(v) Donation to National Urban Poverty Eradication Fund is allowed u/s 35CCA, ------Income under the head business/profession, 13,16,000, Gross Total Income, 13,16,000, Less: Deduction u/s 80C to 80U, Nil, Total Income, 13,16,000, Computation of Tax Liability, Tax on `13,16,000 @ 30%, 3,94,800, Add: HEC @ 4%, 15,792, Tax Liability, 4,10,592, Rounded off u/s 288B, 4,10,590, Solution 18:, (i), Income under the head Business/Profession, Less: Securities transaction tax, Income under the head Business/Profession, Gross Total Income, Less: Deduction u/s 80C, , `, 4,00,000.00, (1,800.00), 3,98,200.00, 3,98,200.00, (10,000.00)
Page 962 :
Income Under The Head Business/Profession, Total Income, Tax on `3,88,200 at slab rate, Less: Rebate u/s 87A, Tax Liability, (ii), Income under the head Business/Profession, Gross Total Income, Less: Deduction u/s 80C to 80U, Total Income, Tax on `11,00,000 @ 30%, Add: HEC @ 4%, Tax Liability, (iii), Income under the head Business/Profession, Total Income, Tax on `18,00,000 at slab rate, Add: HEC @ 4%, Tax Liability, , 61, 3,88,200.00, 4,410.00, (4,410.00), Nil, 11,00,000, 11,00,000, Nil, 11,00,000, 3,30,000, 13,200, 3,43,200, 18,00,000, 18,00,000, 3,52,500, 14,100, 3,66,600, , Solution 19:, (i) Previous year 2021-22;, (ii) Previous year 2021-22;, (iii) Previous year 2021-22;, (iv) Previous year 2021-22;, (v) 30% of the amount disallowed in P.Y. 2021-22, (vi) Previous year 2022-23, Solution 20:, Income under the head business/profession, Net Profit as per profit and loss account, Less:, • Interest on Fixed deposit with bank, • Dividend from Indian company, Income under the head business/profession, Income under the head other sources, Interest on Fixed deposit, Dividend from Indian company, Income under the head Other Sources, Gross Total Income, Less: Deduction u/s 80C to 80U, Total Income (rounded off u/s 288A), Computation of Tax Liability, Tax on `9,02,080 at slab rate, Add: HEC @ 4%, Tax Liability, Rounded off u/s 288B, Solution 21:, Computation of Total Income, As per section 44AE, Heavy goods vehicle, [(`18,000 x 4 x 12) + (`18,000 x 2 x 2)], Light goods vehicle, , `, 9,02,075.00, (18,500.00), (66,360.00), 8,17,215.00, 18,500.00, 66,360.00, 84,860.00, 9,02,075.00, Nil, 9,02,080.00, 92,916.00, 3,716.64, 96,632.64, 96,630.00, `, 9,36,000.00, 5,25,000.00
Page 963 :
Income Under The Head Business/Profession, , 62, , [(`7,500 x 4 x 12) + (`7,500 x 2 x 11)], Other business and non business income, 1,00,000.00, Income under the head Business/Profession, 15,61,000.00, Gross Total Income, 15,61,000.00, Less: Deduction u/s 80C to 80U, Nil, Total Income, 15,61,000.00, Computation of Tax Liability, Tax on `15,61,000 at slab rate, 2,80,800.00, Add: HEC @ 4%, 11,232.00, Tax Liability, 2,92,032.00, Rounded off u/s 288B, 2,92,030.00, The assessee has the option to reject the presumptive taxation and can compute the income in the normal, manner but the assessee has to maintain books of accounts and also he should get his accounts audited as, per section 44AB. In such case tax liability of the assessee shall be as given below:, Gross Receipt, 9,90,000, Less: Operational expenses, (5,25,000), Less: Depreciation as per section 32, (1,85,000), Less: Other office expenses, (15,000), 2,65,000, Add: Other business and non- business income, 1,00,000, Total Income, 3,65,000, Computation of Tax Liability, Tax on `3,65,000 at slab rate, 5,750.00, Less: Rebate u/s 87A, (5,750.00), Tax Liability, Nil, Solution 22:, Computation of Income under the head Business/profession, Net Profit as per profit and loss account, Add:, • Salary to the proprietor, • Interest to Mr. A, • Interest for income tax liability, • Depreciation, Less:, • Long term capital gains, Income under the head Business/Profession, Income under the head Capital Gains (LTCG), Gross Total Income, Less: Deduction u/s 80C, National Saving Certificate, Public provident fund, Total Income, Computation of Tax Liability, Tax on `1,70,000 (`4,00,000 – `2,30,000) @ 20% u/s 112, Tax on `20,000 at slab rate, Tax before HEC, Add: HEC @ 4%, Tax Liability, , `, , `, 3,00,000, 1,60,000, 30,000, 20,000, 10,000, (4,00,000), 1,20,000, 4,00,000, 5,20,000, (1,00,000), , 40,000, 60,000, 4,20,000, 34,000, (12,500), 21,500, 860, 22,360
Page 964 :
Income Under The Head Business/Profession, Solution 23:, Computation of Income under the head Business/profession, Net Profit as per profit and loss account, Add:, • Salaries and bonus, • GST payable, • Expenditure on technical know-how, • Interest on capital, • Rent of own building, Less:, • Depreciation on technical know-how {u/s 32}, (36,000 x 25%), • Depreciation on building, (2,50,000 x 10%), • Interest from Indian companies, Income under the head Business/Profession, Less: Brought forward business loss of assessment year 2020-21, Income under the head Business/Profession, Income under the head Other Sources, {Interest from Indian companies}, Gross Total Income, Less: Deductions u/s 80C to 80U, Total Income, Computation of Tax Liability, Tax on `3,87,000 at slab rate, Less: Rebate u/s 87A, Tax Liability, Solution 24:, Computation of Income under the head Business/profession, Net Profit as per profit and loss account, Add:, Cash purchases {u/s 40A(3)}, Recovery of bad debts {as per sec 41(4)}, Salary of Mr. X, Interest on capital, Less:, Bonus paid, Depreciation on machinery, Working Note:, `, Written down value, 5,00,000, Less: Sale, (1,00,000), Add: Purchase, 4,00,000, 8,00,000, Depreciation, 7.5% on `4,00,000 =, 30,000, 15% on `4,00,000 =, 60,000, Total, 90,000, Already provided in profit & loss A/c, 50,000, Balance, 40,000, Dividend from foreign company, Long term capital gains, , 63, `, 3,00,000, 1,05,000, 30,000, 36,000, 20,000, 30,000, (9,000), (25,000), (70,000), 4,17,000, (1,00,000), 3,17,000, 70,000, 3,87,000, Nil, 3,87,000, 6,850.00, (6,850.00), Nil, , `, 2,00,000, 1,00,000, 1,00,000, 3,60,000, 1,89,000, (1,07,000), (40,000), , (30,000), (1,00,000)
Page 965 :
Income Under The Head Business/Profession, , 64, , Income under the head Business/Profession, Less: b/f Business Loss, Less: Unabsorbed depreciation, Income under the head Business/Profession, Income under the head Capital Gains (LTCG), Income under the head Other Sources, {Dividend from foreign company}, Gross Total Income, Less: Deduction u/s 80C to 80U, Total Income, Computation of Tax Liability, Tax on LTCG `52,000 (`1,00,000 – `48,000) @ 20% u/s 112, Tax on `2,02,000 at slab rate, Less: Rebate u/s 87A, Tax Liability, , 6,72,000, (3,00,000), (2,00,000), 1,72,000, 1,00,000, 30,000, 3,02,000, Nil, 3,02,000, 10,400, Nil, (10,400), Nil, , Solution 25:, Computation of Income under the head Business/profession, Net Profit as per profit and loss account, Add:, Interest to Proprietor, Salary to Proprietor, Purchase of trademark, Depreciation on plant and machinery, Less:, Short term capital gains, Depreciation on trade mark, Employer contribution to recognized provident fund, Income under the head Business/Profession, Less: Brought forward depreciation, Income under the head Business/Profession, Income under the head Capital Gains (STCG), Gross Total Income, Less: Deduction u/s 80C to 80U, Total Income, Computation of Tax Liability, Tax on `14,84,000 at slab rate, Add: HEC @ 4%, Tax Liability, Rounded off u/s 288B, Solution 26:, Net profit as per profit and loss account, Add:, Other expenses, Travelling, advertisement and entertainment expenses, Depreciation, Working Note:, Written down value as on 01.04.2021, Add: Purchased of plant F on 01.01.2022, Depreciation @ 15% on `3,70,000, Depreciation @ 7.5% on `4,86,000, , `, 12,00,000.00, 1,50,000.00, 6,00,000.00, 2,00,000.00, 20,000.00, (6,00,000.00), (50,000.00), (4,00,000.00), 11,20,000.00, (2,36,000.00), 8,84,000.00, 6,00,000.00, 14,84,000.00, Nil, 14,84,000.00, 2,57,700.00, 10,308.00, 2,68,008.00, 2,68,010.00, `, 8,64,760.00, 20,000.00, 25,000.00, 1,47,480.00, `, 3,70,000, 4,86,000, 8,56,000, 55,500, 36,450
Page 966 :
Income Under The Head Business/Profession, , 65, , Total, 91,950, Excessive depreciation (2,39,430 – 91,950), Interest on loan taken to make deposit in companies, Municipal tax and insurance of godown, Salary to Proprietor, Interest to Proprietor, Less:, Rental income from Godown, Interest on company deposits, Income under the head Business/Profession, Less: Brought forward business loss, Income under the head Business/Profession, Computation of income under the head house property, Gross Annual Value, Less: Municipal Taxes, Net Annual Value, Less: 30% of NAV u/s 24(a), Less: Interest on capital borrowed u/s 24(b), Income under the head House Property, Income under the head Other Sources (2,60,000 – 13,800), Computation of income under the head Capital Gains, Income under the head Capital Gains (STCG), (9,10,000 – 1,98,000), Less: Brought forward short term capital loss for the assessment year 2020-21, Less: Brought forward short term capital loss for the assessment year 2021-22, Income under the head Capital Gains (STCG), Computation of Total Income, Gross Total Income, Less: Deduction u/s 80C to 80U, Total Income, Computation of Tax Liability, Tax on `19,83,840 at slab rate, Add: HEC @ 4%, Tax Liability, Rounded off u/s 288B, Solution 27:, Computation of Total Income of Mr. X, Net profit as per profit and loss account, Add: inadmissible expenses, Provision for GST, Advance tax, Salary to Mr. X, Interest to Mr. X, Interest to Mrs. X, Salary to relative {Sec 40A(2)}, School fees for Mr. X’s son, Total, Less:, Dividend from Indian companies, Income from UTI, Depreciation [52,500-45,000], Income under the head Business/Profession, , `, , 13,800.00, 7,200.00, 1,80,000.00, 69,000.00, (48,000.00), (2,60,000.00), 10,19,240.00, (20,000.00), 9,99,240.00, 48,000.00, (6,000.00), 42,000.00, (12,600.00), Nil, 29,400.00, 2,46,200.00, 7,12,000.00, (2,000.00), (1,000.00), 7,09,000.00, 19,83,840.00, Nil, 19,83,840.00, 4,07,652.00, 16,306.08, 4,23,958.08, 4,23,960.00, `, 6,09,500, 10,000, 11,000, 24,000, 22,000, 32,000, 4,000, 5,000, 7,17,500, (12,500), (6,500), (7,500), 6,91,000
Page 967 :
Income Under The Head Business/Profession, , 66, , Income under the head Other Sources, Dividend from Indian company, Income from mutual fund, Income from units of mutual fund, Income under the head Other Sources, Gross Total Income, Less: deduction u/s 80C, Life insurance premium, (Maximum 10% of sum assured), Tuition fees of Mr. X’s son, Investment in infrastructure bonds, Total Income, Computation of Tax Liability, Tax on `6,76,000 at slab rate, Add: HEC @ 4%, Tax Liability, Rounded off u/s 288B, Solution 28:, Computation of Total Income of Mrs. X, Income under the head Business/Profession, Net profit as per profit and loss account, Add: inadmissible items, • One fifth of car maintenance, • Interest on loan (50%), • Municipal tax, • Depreciation as per books, • (8,500 + 14,000 + 5,000 + 3,000), • Printing bill for 2020-21, • Provident fund (Section 43B), • Payment made in cash in excess of `10,000 {Sec 40A(3)}, Less:, • Honorarium received from various institutions, • Dividend on shares, • Income from Unit Trust of India, • Profit on sale of equity shares, • Rent received, • Depreciation as per Income Tax Act, • Car (85,840 x 15% x 4/5), • Computer (1,50,000 x 40% x 1/2), • Typewriter (15,000 x 15%), • Furniture (25,000 x 10%), Income under the head business/profession, Income under the head House Property, Gross annual value, Less: Municipal Taxes, Net annual value, Less: 30% of NAV u/s 24(a), Less: Interest on capital borrowed u/s 24(b), Income from house property, Income from capital gain (STCG), Income under the head Other Sources, , 12,500, 6,500, 25,000, 44,000, 7,35,000, (59,000), 34,000, 5,000, 20,000, 6,76,000, 47,700, 1,908, 49,608, 49,610, `, , `, 7,88,700.00, 3,300.00, 30,000.00, 2,000.00, 30,500.00, 3,000.00, 5,000.00, 30,000.00, (6,600.00), (10,500.00), (6,500.00), (20,800.00), (62,000.00), , 10,300.80, 30,000.00, 2,250.00, 2,500.00, , (45,050.80), 7,41,049.20, 62,000.00, (2,000.00), 60,000.00, (18,000.00), (30,000.00), 12,000.00, 20,800.00
Page 968 :
Income Under The Head Business/Profession, Honorarium received from various institutions, Dividend from Indian company, Income from UTI, Income under the head Other Sources, Gross Total Income, Less: Deduction u/s 80C, Public provident fund contribution, Infrastructure bonds of ICICI, Total Income (rounded off u/s 288A), Computation of Tax Liability, Tax on STCG `20,800 @ 15% u/s 111A, Tax on `6,96,650 at slab rate, Tax before health & education cess, Add: HEC @ 4%, Tax Liability, Rounded off u/s 288B, Solution 29:, Computation of Total Income, Net profit as per profit & loss account, Add: inadmissible expenses, Excess payment to relative u/s 40A(1) & (2), Excess payment in cash u/s 40A(3), Capital expenditure debited in the profit and loss account (computer), Capital expenditure debited in the profit and loss account (generator), Income tax u/s 40(a), Household furniture, Medical treatment, Salary paid to proprietor, Interest on capital, Amount invested in National Saving Certificate, Amount invested in public provident fund, Rent paid, Opening balance (4,50,000 x 10/110), Business Income, Gross Total Income, Less: Deduction u/s 80C, Total Income (rounded off u/s 288A), Computation of Tax Liability, Tax on `14,87,910 at slab rate, Add: HEC @ 4%, Tax Liability, Rounded off u/s 288B, Solution 30:, Computation of income under the head Business/Profession, Net profit as per profit & loss account, Add: inadmissible expenses, Payment of purchases in excess of `10,000 {Sec 40A(3)}, Payment of purchases in excess of `10,000 {Sec 40A(3)}, Payment of purchases {Sec 40A(2)}, Salary to proprietor, Cost of computer, , 67, 6,600.00, 10,500.00, 6,500.00, 23,600.00, 7,97,449.20, (80,000.00), 60,000, 20,000, 7,17,450.00, 3,120.00, 49,330.00, 52,450.00, 2,098.00, 54,548.00, 54,550.00, `, 10,00,000.00, 1,00,000.00, 1,00,000.00, 45,000.00, 45,000.00, 45,000.00, 12,000.00, 20,000.00, 36,000.00, 9,000.00, 25,000.00, 10,000.00, 35,000.00, 40,909.09, 15,22,909.09, 15,22,909.09, (35,000.00), 14,87,910.00, 2,58,873.00, 10,354.92, 2,69,227.92, 2,69,230.00, `, 6,27,900.00, 20,100.00, 22,000.00, 1,000.00, 12,400.00, 24,000.00
Page 969 :
Income Under The Head Business/Profession, Interest on capital, Donation to orphan, Income tax, Gift to relative, Medical expenses of proprietor, Staff welfare fund, Bonus payable {Sec 43B}, Provision for GST, General reserve, Less:, Maturity proceeds of National Saving Certificate, Maturity proceeds of bank Fixed Deposit, Maturity proceeds of public provident fund, Income tax penalty refund, Sale of machinery, Recovery of bad debts, Income tax refund, Gift from relatives, Maturity proceeds of LIC, Closing Stock{`26,400 x 10 / 110}, Refund of deposit from supplier, Depreciation:, Computer = 40% on `24,000, Machinery - w.d.v = `59,000, Less: sale, = (`25,000), Written down value `34,000, Dep. @ 15%, Factory building – w.d.v = `85,000, Add: purchase = `12,000, Dep. @ 10% on `97,000, Income under the head business/profession, Computation of income under the head Other Sources, Interest on income tax refund, Income under the head other sources, Income under the head business/profession, Gross Total Income, Deductions u/s 80C to 80U, Total Income, Computation of Tax Liability, Tax on `5,13,700 at slab rates, Add: HEC @ 4%, Tax liability, Rounded off u/s 288B, Solution 31:, Net Profit as per profit & loss account, Add: inadmissible expenses, 1. Franchises, being capital expenditure, 2. Advertisement, being capital expenditure, 3. Income tax (income tax is not allowed as per sec 40(a)), 4 Addition to office building, being capital expenditure, 5. Investment in public provident fund, (not a revenue expenditure), , 68, 3,300.00, 1,000.00, 6,000.00, 1,000.00, 3,000.00, 2,000.00, 5,000.00, 25,000.00, 5,000.00, (19,500.00), (24,000.00), (13,000.00), (1,100.00), (25,000.00), (6,000.00), (2,400.00), (3,600.00), (24,000.00), (2,400.00), (1,00,000.00), (9,600.00), , (5,100.00), (9,700.00), 5,13,300.00, 400.00, 400.00, 5,13,300.00, 5,13,700.00, Nil, 5,13,700.00, 15,240.00, 609.60, 15,849.60, 15,850.00, ₹, 17,45,600.00, 1,00,000.00, 9,000.00, 8,200.00, 45,000.00, 70,000.00
Page 970 :
Income Under The Head Business/Profession, , 69, , Add: incomes not credited to profit & loss account, Closing stock undervalued by 10% (1,80,000 x 10/90), 20,000.00, Deduct expenditures not debited to profit & loss account, Opening stock undervalued by 10% (13,50,000 x 10/90), (1,50,000.00), Depreciation, (5,88,150.00), Working Note:, ₹, 1. Franchises 1,00,000 x 25%, 25,000, 2. Furniture/fixture @ 10%, - Neon sign board 9,000 x 10%, 900, - Wirings etc. in the building 4,500 x 5%, 225, 3. Office building 56,00,000 x 10%, 5,60,000, Addition 40,500 x 5%, 2,025, Total Depreciation, 5,88,150, Deduct amounts credited to profit & loss, but not considered to be incomes, 1. Gift from friend (any gift is not considered to be income except gift received from client or, gift received from any person in excess of ₹50,000), (1,200.00), 2. Rental income from House Property, (1,40,000.00), 3. Income tax refund, (700.00), 4. Dividends from a foreign company (to be taxed under the head other sources), (3,000.00), 5. Sale to the proprietor should be at cost price, (20,000.00), Income under the head business/profession, 10,94,750.00, Income under the head other sources, 3,000.00, Income under the head house property, Gross Annual Value, Less: Municipal taxes, Net Annual Value, Less: 30% of NAV u/s 24(a), Less: Interest on capital borrowed u/s 24(b), Income under the head House Property, Gross Total Income, Less: Deduction u/s 80C, Total Income, , 1,40,000.00, Nil, 1,40,000.00, (42,000.00), Nil, 98,000.00, 11,95,750.00, (70,000.00), 11,25,750.00, , Computation of Tax liability, Tax on ₹11,25,750 at slab rate, Add: HEC @ 4%, Tax Liability, Rounded off u/s 288B, Solution 32:, Computation of income under the head Business/Profession, Legal consultancy fees, Less:, • Rent of building, • Office expenses, • Depreciation on computer, Working Note:, Computer – w.d.v, Less: Sale of computer, Add: New computer purchased and put to use on 01.11.2021, Add: Computer purchased and put to use on 10.11.2021, Balance, , 1,47,725.00, 5,909.00, 1,53,634.00, 1,53,630.00, ₹, 9,20,000.00, (2,20,000.00), (30,000.00), (11,060.00), ₹, 2,300, (7,000), 35,000, 25,000, 55,300
Page 971 :
Income Under The Head Business/Profession, Depreciation @ 20%, • Depreciation on car, Working Note:, Motor car – Purchase and put to use on 01.12.2021, Depreciation @ 7.5% on ₹4,00,000, • Depreciation on books, Working Note:, Books purchased and put to use on 01.01.2022, Depreciation @ 20% on ₹9,000, • Subscription to bar association (allowed u/s 37(1)), Income under the head Business/Profession, Computation of income under the head Capital Gains on sale of house, Full Value of Consideration, Less: Cost of acquisition, Income under the head Capital Gains (STCG), Computation of income under the head Other Sources, Remuneration from Delhi University, Honorarium for delivering lectures, Interest from UTI, Income under the head Other Sources, Computation of Total Income, Income under the head Business/Profession, Income under the head Capital Gains (STCG), Income under the head Other Sources, Gross Total Income, Less: Deduction u/s 80C, Total Income, Computation of Tax Liability, Tax on ₹9,41,140 at slab rate, Add: HEC @ 4%, Tax Liability, Less: Advance Tax Paid, Tax Payable, Rounded off u/s 288B, Solution 33:, Computation of income under the head Business/Profession, Net Profit as per profit and loss account, Add:, • Household expenses, • Income tax, • Interest on loan for payment of income tax, • Contribution to Unrecognised provident fund, • Contribution to public provident fund, • Investment in post office saving bank account, • Purchase of car, • Purchase of computer, • Purchase of plant, Less:, • Income tax refund, • Interest on refund, , 70, 11,060, (30,000.00), ₹, 4,00,000, 30,000, (1,800.00), ₹, 9,000, 1,800, (3,000.00), 6,24,140.00, 9,80,000.00, (6,70,000.00), 3,10,000.00, 4,000.00, 3,000.00, 12,000.00, 19,000.00, 6,24,140.00, 3,10,000.00, 19,000.00, 9,53,140.00, (12,000.00), 9,41,140.00, 1,00,728.00, 4,029.12, 1,04,757.12, (12,000.00), 92,757.12, 92,760.00, ₹, 1,55,000, 20,000, 12,000, 1,200, 4,000, 7,000, 12,000, 2,45,000, 35,000, 23,000, (3,000), (300)
Page 972 :
Income Under The Head Business/Profession, • Dividends, • Depreciation @ 15% on car, (2,45,000 x 15%), • Depreciation @ 40% on computer, (35,000 x 40%), • Depreciation @ 15% on plant, (23,000 x 15%), Income under the head Business/Profession, Computation of income under the head Other Sources, Interest on income tax refund, Dividends from foreign company, Income under the head Other Sources, Income under the head Business/Profession, Gross Total Income, Less: Deductions u/s 80C, Total Income, Computation of Tax Liability, Tax on ₹4,50,000 at slab rate, Less: Rebate u/s 87A, Tax Liability, Solution 34:, Computation of Business Income, Net Profit as per profit and loss account, Add: inadmissible expenses, • Provision for doubtful debts, • Depreciation Reserve, • Household Expenses, • Donations to poor persons, • Other charitable donations, • Cash purchases in excess ₹10,000, • Cost of neon sign board (capital expenditure), • Patents purchased, • Installment for preliminary expenses under section 35D, (15,000 – 3,000), Working Note:, ₹15,000 but subject to a maximum of ₹10,00,000 x 5% = ₹50,000,, installment allowed ₹15,000/5 = ₹3,000, • Opening stock overvalued 1,15,000 x 15/115, • Closing stock undervalued 1,70,000 x 15/85, Less:, • Interest on company deposit, • Depreciation on neon sign @ 10% on ₹5,000, • Depreciation on patents @ 12.5% on ₹70,000, Income under the head Business/Profession, Income under the head Other Sources, {Interest on company deposit}, Gross Total Income, Less: Deduction u/s 80C to 80U, Total Income, , 71, (3,000), (36,750), (14,000), (3,450), 4,53,700, 300, 3,000, 3,300, 4,53,700, 4,57,000, (7,000), 4,50,000, 10,000.00, (10,000.00), Nil, ₹, 87,000.00, 16,000.00, 21,000.00, 20,000.00, 10,000.00, 20,000.00, 80,000.00, 5,000.00, 70,000.00, 12,000.00, , 15,000.00, 30,000.00, (50,000.00), (500.00), (8,750.00), 3,26,750.00, 50,000.00, 3,76,750.00, Nil, 3,76,750.00
Page 973 :
Income Under The Head Business/Profession, Computation of Tax Liability, Tax on ₹ 3,76,750 at slab rate, Less: Rebate u/s87A, Tax Liability, Solution 35:, Computation of total income of Mr. X, Net profits as per profit and loss account, Add: Inadmissible Expenses, • Remuneration given to proprietor, not allowed, • Interest given to proprietor, not allowed, • Cash payment to a supplier, • Income Tax Paid, Total, Less:, • Dividend from Indian company, • Long Term Capital Gains, Income under the head Business/Profession, Income under the head House Property, Income from Capital Gains (Long Term Capital Gains), Income from Other Sources, Dividend from Indian company, Gross Total Income, Less: Deduction u/s 80C to 80U, Total Income, Computation of Tax Liability, Tax on Long term capital gain ₹1,90,000 @ 20%, Tax on normal income ₹3,05,000 at slab rate, Less: Rebate u/s 87A, Tax before health & education cess, Add: HEC @ 4%, Tax Liability, Solution 36:, Computation of Business Income, Net Profit, Add: Inadmissible Expenses, • Municipal Taxes (12,000 x 1/3), • Market Rent, Total, Less:, • Rent Received, Business Income, Computation of Income under the head House Property, Gross Annual Value, (10,000 x 12), Less: Municipal Taxes, (Not paid during the year), Net Annual Value, Less: 30% of NAV u/s 24(a), Less: Interest on capital borrowed u/s 24(b), Income under the head House Property, , 72, 6,337.50, (6,337.50), Nil, ₹, 25,000, 3,00,000, 40,000, 30,000, 10,000, 4,05,000, (30,000), (1,90,000), 1,85,000, 90,000, 1,90,000, 30,000, 4,95,000, Nil, 4,95,000, 38,000.00, 2,750.00, (12,500.00), 28,250.00, 1,130.00, 29,380.00, ₹, 7,00,000.00, 4,000.00, 2,40,000.00, 9,44,000.00, (1,20,000.00), 8,24,000.00, 1,20,000.00, Nil, 1,20,000.00, (36,000.00), Nil, 84,000.00
Page 974 :
Income Under The Head Business/Profession, , 73, , Computation of Total Income, Income under the head Business, Income under the head House Property, Gross Total Income, Less: Deduction u/s 80C to 80U, Total Income, , 8,24,000.00, 84,000.00, 9,08,000.00, Nil, 9,08,000.00, , Computation of Tax Liability, Tax on ₹9,08,000 @ 30%, Add: HEC @ 4%, Tax Liability, Rounded off u/s 288B, , 2,72,400.00, 10,896.00, 2,83,296.00, 2,83,300.00, , Solution 37:, Computation of Income of Dr. Sagar, Net profit as per profit and loss account, Add: Inadmissible expenses, • Rent for residential accommodation, • Medicines for personal use, • Municipal taxes, Less:, • Depreciation on hospital equipment, Working Note:, Depreciation on ₹5,50,000 @ 15%, Depreciation on ₹2,50,000 @ 7.5%, • Rental income from house property, • Dividend from Indian companies, Income under the head Business/Profession, Income from Salary, Salary, (10,000 x 12), Gross Salary, Less: Standard deduction u/s 16(ia), Income under the head Salary, Income from House Property, Gross Annual Value, Less: Municipal Taxes, Net Annual Value, Less: 30% of NAV u/s 24(a), Less: Interest on capital borrowed u/s 24(b), Income under the head House Property, Income under the head Other Sources, Dividend from Indian company, Gross Total Income, Less: Deduction u/s 80C to 80U, Total Income, Computation of Tax Liability, Tax on normal income ₹2,61,100 at slab rate, Less: Rebate u/s 87A, Tax Liability, , ₹, 2,50,000.00, 38,000.00, 12,000.00, 3,500.00, (1,01,250.00), ₹, 82,500, 18,750, (29,000.00), (15,000.00), 1,58,250.00, 1,20,000.00, 1,20,000.00, (50,000.00), 70,000.00, 29,000.00, (3,500.00), 25,500.00, (7,650.00), Nil, 17,850.00, 15,000.00, 2,61,100.00, Nil, 2,61,100.00, 555.00, (555.00), Nil
Page 975 :
Income Under The Head Business/Profession, , 74, , Solution 38:, 1. As per section 28, Any gift received in connection with business/profession shall be considered to be, income under the head business/ profession hence ₹ 2,50,000 being value of the motor car shall be, considered to be income under the head business/profession. Since car is being used for the purpose of, business, depreciation shall be allowed as per section 32., 2. As per section 32, depreciation shall be allowed even for intangible assets, hence ₹ 6 lakh qualifies for, depreciation @ 25%., 3. As per section 40(a), while calculating income of the employer, the tax paid by the employer on nonmonetary perquisites to employees is not deductible., 4. As per section 28, any sum received for not carrying out any activity in relation to any business is, chargeable to tax as business income. Thus, ₹ 10 lakh is taxable as business income being noncompete fee., 5. Section 35DDA provides that where an assessee incurs any expenditure in any previous year by way of, payment of any sum to an employee at the time of his voluntary retirement under any scheme of, voluntary retirement, one fifth of the amount so paid shall be deducted in computing the profits and, gains of the business for that previous year, and the balance shall be deducted in equal instalments for, each of the four immediately succeeding previous years. In view of the aforesaid provisions, ₹ 4 lakh, shall be allowable as deduction in the assessment year 2021-22., 6. As per section 43(1), all expenses upto the date of putting the asset to use shall be capitalized i.e. it will, be added to the actual cost but in the given case asset has not been put to use till the end of the year, hence neither the amount can be debited to profit and loss account nor depreciation is allowed., 7. As per section 37(1), in order to claim deduction the expenditure should not have been incurred for any, purpose, which is an offence or is prohibited by any law. Since the payment of ₹40,000 to Don is, unlawful, it is not allowable as deduction., 8. As per section 14A, no deduction shall be made in respect of expenditure incurred by the assessee in, relation to income which does not form part of the total income. ₹34,000 is, therefore, not allowable as, deduction., Solution 39:, (i) An amount equal to commodity transaction tax paid by the assessee shall be allowable as deduction,, under section 36(1)(xvi), if the income arising from taxable commodities transactions is included in the, income computed under the head “Profits and gains of business or profession”. In the given case, Mr. M, is, entitled to claim deduction in respect of commodity transaction tax of ₹ 75000 paid by him., (ii) As per section 35AD, assessee shall be allowed to debit 100% of the expenditure incurred in connection, with warehousing facility for agricultural produce hence assessee shall be allowed to debit 50 lakhs x 100%, = 50 lakh., (iii) ABC (P) Ltd. is entitled to a weighted deduction of a sum equal to 150% of the expenditure incurred by, it on notified skill development project, under section 35CCD. Therefore, it can claim ₹ 3,75,000 (i.e., 150%, of ₹ 2,50,000) as deduction under section 35CCD for the P.Y.2021-22., Solution 40:, (i), Computation of Tax Liability, Total Income (LTCG 112A), Tax on ₹296,50,000 (300,00,000-1,00,000-2,50,000) @ 10% u/s 112A, Add: Surcharge @ 15%, Tax before HEC, Add: HEC @ 4%, Tax Liability, , 300,00,000.00, 29,65,000.00, 4,44,750.00, 34,09,750.00, 1,36,390.00, 35,46,140.00
Page 976 :
Income Under The Head Business/Profession, , 75, , (ii), Computation of Tax Liability, Total Income (STCG 111A), Tax on ₹297,50,000 (300,00,000-2,50,000) @ 15% u/s 111A, Add: Surcharge @ 15%, Tax before HEC, Add: HEC @ 4%, Tax Liability, , 300,00,000.00, 44,62,500.00, 6,69,375.00, 51,31,875.00, 2,05,275.00, 53,37,150.00, , (iii), Computation of Tax Liability, Total Income (Dividend Income), Tax on ₹300,00,000 at slab rate, Add: Surcharge @ 15%, Tax before HEC, Add: HEC @ 4%, Tax Liability, , 300,00,000.00, 88,12,500.00, 13,21,875.00, 101,34,375.00, 4,05,375.00, 105,39,750.00, , (iv), Computation of Tax Liability, Total Income (LTCG), Tax on ₹297,50,000 (300,00,000-2,50,000) @ 20% u/s 112, Add: Surcharge @ 25%, Tax before HEC, Add: HEC @ 4%, Tax Liability, , 300,00,000.00, 59,50,000.00, 14,87,500.00, 74,37,500.00, 2,97,500.00, 77,35,000.00, , (v), Computation of Tax Liability, Total Income (casual income), Tax on ₹300,00,000 @ 30% u/s 115BB, Add: Surcharge @ 25%, Tax before HEC, Add: HEC @ 4%, Tax Liability, , 300,00,000.00, 90,00,000.00, 22,50,000.00, 112,50,000.00, 4,50,000.00, 117,00,000.00, , (vi), Computation of Tax Liability, Total Income (Business Income), Tax on ₹300,00,000 at slab rate, Add: Surcharge @ 25%, Tax before HEC, Add: HEC @ 4%, Tax Liability, , 300,00,000.00, 88,12,500.00, 22,03,125.00, 110,15,625.00, 4,40,625.00, 114,56,250.00
Page 977 :
Income Under The Head Business/Profession, , 76, , EXAMINATION QUESTIONS, JULY – 2021 (NEW COURSE), Answer 1:, Computation of total income of Mr. Ashish for A.Y. 2022-23, Particulars, I Income from business or profession, , ₹, , ₹, 39,43,000, , Excess of income over expenditure, Add: Items debited but not allowable while computing, business income, - Family planning expenditure incurred for employees, [not allowable as deduction since expenditure on, family planning for employees is allowed only to a, company assesse/not allowed in case of individuals., Since the amount is debited to Income and Expenditure, Account, the same has to be added back for computing, business income], - Salary payment to sister-in-law in excess of market rate, [Any expenditure incurred for which payment is made, to a relative, to the extent it is considered unreasonable, is disallowed. However, sister-in-law is not included in, the definition of “relative” for the purpose of section, 40A(2)., Therefore, no adjustment is required for excess salary, paid to Mr. Ashish’s sister-in- law], - Employees’ Contribution to EPF [Sum received by the, assessee from his employees as contribution to EPF is, income of the employer. Deduction in respect of such, sum is allowed only if such amount is credited to the, employee’s account on or before due date under the, relevant Act. Since, the employees contribution to EPF, for February 2021 is deposited after the due date under, the relevant Act, deduction would not be available], -, , Medical expenses for the treatment of father [Not, allowed as deduction since it is a personal, expenditure/not an expenditure incurred for the, purpose of business of Mr. Ashish. Since the amount, is debited to Income and Expenditure Account, the, same has to be added back for computing business, income], - Commission to Ms. Anjaleen without deduction of tax, at source – [Mr. Ashish would be liable to deduct tax, at source on commission since his gross receipts, from profession exceeded ₹ 50 lakhs during, F.Y.2019-20. Since commission has been paid, without deduction of tax at source, hence 30% of ₹, 25,000, being commission paid without deducting, , 20,000, , Nil, , 10,000, , 80,000, , 7,500, , ₹
Page 978 :
Income Under The Head Business/Profession, , 77, , tax at source, would be disallowed under section, 40(a)(ia) while computing the business income of, A.Y.2021-22], -, , Depreciation as per books of account, , 90,000, , -, , Purchase of Furniture [not allowable, since it is a, capital expenditure], , 48,000, , 2,55,500, 41,98,500, , Less: Depreciation as per Income-tax Rules, - On Professional Books [₹ 90,000 x 40%], - On Computers [₹ 35,000 x 40%], - On Furniture [₹19,000 x 10%, since it has been put to, use for more than 180 days during the year] [Any, expenditure for acquisition of any asset in respect of, which payment or aggregate of payment made to a, person, otherwise than by an a/c payee cheque/ bank, draft or use of ECS or through prescribed electronic, mode, exceeds ₹ 10,000 in a day, such expenditure, would not form part of actual cost of such asset., Hence, ₹ 18,000 and ₹ 11,000 paid on 31.8.2020 in, cash would not be included in the actual cost of, furniture], - On Car [₹ 3,35,000 x 15%] [Actual cost of car would, be the purchase price of the car to Mr. Ashish, i.e.,, ₹3,35,000], , 36,000, 14,000, 1,900, , 50,250, , 1,02,150, 40,96,350, , Less: Items of income credited but not taxable or, taxable under any other head of income, - Interest on Public Provident Fund [Exempt], Interest on savings bank account, [Taxable under the head “Income from other, sources”], Interest on National Savings Certificates VIII, Issue (3rd Year) [Taxable under the head “Income, from other sources”], II Income from other sources, Interest on savings bank account, Interest on National Savings Certificates VIII Issue (3rd, Year), Gross Total Income, Less: Deduction under Chapter VI-A, Deduction under section 80C, Contribution to PPF, Interest on NSC (3rd Year) (Reinvested), , 60,000, 20,000, , 21,000, 1,01,000, 39,95,350, 20,000, 21,000, 41,000, 40,36,350, , 1,00,000, 21,000, , 1,21,000
Page 979 :
Income Under The Head Business/Profession, Deduction under section 80D, Medical expenses for the treatment of father [Since Mr., Ashish’s father is a senior citizen and not covered by any, health insurance policy, payment for medical, expenditure by a mode other than cash would be allowed, as deduction to the extent of ₹ 50,000], Deduction under section 80TTA, Interest on savings bank account to the extent of ₹10,000, , 78, , 50,000, , 10,000, (1,81,000), 38,55,350, , Total income, Computation of tax liability of Mr. Ashish for A.Y.2022-23, ₹, Particulars, Tax on total income of ₹ 38,55,350, Upto ₹ 2,50,000, Nil, ₹ 2,50,001 – ₹ 5,00,000 [@5% of ₹ 2.50 lakh], 12,500, ₹ 5,00,001 – ₹ 10,00,000 [@20% of ₹ 5 lakh], 1,00,000, ₹ 10,00,001- ₹ 38,55,350 [@30% of ₹ 28,55,350], 8,56,605, , ₹, , 9,69,105, 38,764, 10,07,869, 10,07,870, , Add: Health and education cess@4%, Tax liability, Tax liability (rounded off), , JAN – 2021 (NEW COURSE), Answer 1, , I, , Computation of total income of Mr. Krishna for A.Y. 2022-23, Particulars, ₹, ₹, Income from business or profession, Net profit as per profit and loss account, 5,64,44,700, Add: Items of expenditure debited but not, allowable while computing business income, 1. Donation to Gurudwara in cash [not allowable as, deduction since it is not incurred wholly and, exclusively for business purpose. Since the, amount is already debited, the same has to be, added back while computing business income], 20,000, 2. Interest on loan taken for purchase of e-vehicle, [Interest on loan for purchase of e- vehicle for, personal purpose is not allowed as, deduction, from business income since the same is not, incurred wholly and exclusively for, business, purpose. Since it is already debited, the same has, to be added back while computing business, income], 1,67,000, 3. Sale of furniture to brother at less than FMV [The, provisions of section 40A(2) are not applicable in, case of sale transaction, even if the same is to a, related party. Therefore, no adjustment is, necessary in respect of difference of ₹ 2 lakh], 1,87,000, 5,66,31,700, , ₹
Page 980 :
Income Under The Head Business/Profession, Less: Items of income credited but not taxable or, taxable under any other head of income, 4. Royalty on patent [Not taxable as business, income since Mr. Krishna is engaged in, manufacturing business. Since the amount is, already credited to profit and loss account, the, same has to be reduced while computing business, income], 5. Bad debt recovered [Actual bad debt is₹ 2 lakhs, i.e., ₹ 5 lakhs less ₹ 3 lakh, being the amount of, bad debt recovered. Bad debt written off is ₹ 3, lakhs. Bad debt recovered to the extent of ₹ 1 lakh, being excess of bad debt recovered over actual, bad debt would be deemed to be business income., Since the entire ₹ 3 lakhs is credited to the profit, and loss account, ₹ 2 lakhs has to be reduced], , 79, , 4,00,000, , 2,00,000, , 6,00,000, 5,60,31,700, , Less: Allowable expenditure, 6. Depreciation on car [₹ 12 lakh x 15%, since car is, put to use for more than 180 days in the, P.Y.2021-22], (1,80,000), II, , III, , Capital Gain, Long term capital gain on sale of house property, Less: Exemption under section 54 [Since whole, amount of long term capital gain is invested in, construction of house within the stipulated time, limit.], [Capital gain of ₹ 25 lakhs in capital gain account, scheme is not taxable in P.Y. 2021-22, since the same, is withdrawn and invested in construction of house, within the stipulated time limit. The remaining, amount of ₹75 lakhs invested in construction of, house is eligible for exemption u/s 54, subject to a, maximum of ₹50 lakhs being long-term capital gain, on sale of house property during the P.Y.2021-22], Income from Other Sources, Royalty on patent [Taxable as “income from other, sources”, since he is engaged in business of, manufacturing furniture], Gross Total Income, Less: Deduction under Chapter VI-A, Deduction under section 80D, - Mediclaim premium for self and spouse [In case of, lump sum premium for medical policy, deduction is, allowed for equally for each relevant, previous, years. [₹ 30,000/6 years, being relevant previous, years in which the insurance is in force], - Preventive health check up of self and spouse, [Preventive health check up paid in cash allowed to, the extent of ₹ 5,000], , 5,58,51,700, , 50,00,000, (50,00,000), , 4,00,000, 5,62,51,700, , 5,000, , 5,000, , 10,000
Page 981 :
Income Under The Head Business/Profession, Deduction under section 80EEB, , 80, 1,50,000, , [Deduction of interest on loan taken for purchase of, e-vehicle is allowed to the extent of ₹ 1,50,000], Deduction under section 80G, [Donation of ₹ 20,000 to Gurudwara not allowable as, deduction since amount exceeding ₹ 2,000 paid in, cash], Deduction under section 80RRB [Deduction in, respect of royalty on patent registered under the, Patent Act subject to a maximum of ₹ 3 lakh], Total income, , -, , 3,00,000, , (4,60,000), 5,57,91,700, , Computation of tax liability of Mr. Krishna for A.Y.2022-23, Particulars, Tax on total income of ₹ 5,57,91,700, Upto ₹ 3,00,000, ₹ 3,00,001 – ₹ 5,00,000 [@5% of ₹ 2 lakh], ₹ 5,00,001 – ₹ 10,00,000 [@20% of ₹ 5,00,000], ₹ 10,00,001- ₹ 5,57,91,700 [@30% of ₹ 5,47,91,700], Add: Surcharge @ 37%, since total income exceeds, ₹ 5,00,00,000, Add: Health and education cess@4%, Total tax liability, Less: TCS u/s 206C(1) @ 2.5% on ₹ 20 lakh i.e., timber, TCS u/s 206C(1F)@1% of ₹ 12 lakh i.e., sale of, motor car where consideration exceeds ₹ 10 lakh, TDS u/s 194-IA@1% of ₹ 1 crore i.e., sale of, immovable property where consideration is ₹ 50, lakh or more, Tax payable, Tax payable (rounded off), , ₹, , ₹, , Nil, 10,000, 1,00,000, 1,64,37,510, , 1,65,47,510, 61,22,578.70, 2,26,70,088.70, 9,06,803.55, 2,35,76,892.25, , 50,000, 12,000, , 1,00,000, , (1,62,000), 2,34,14,892.25, 2,34,14,890, , Answer 2(b), Computation of total income and tax liability of Mr. Xavier for A.Y. 2022-23 (under the regular, provisions of the Act), Particulars, ₹, Profits and gains of business or profession, Profit from unit in SEZ, Less: Deduction under section 10AA, [50,00,000 x 90,00,000/1,50,00,000 x 100%, since it is 5th year of manufacturing], Business income of SEZ unit chargeable to tax, Profit from operation of warehousing facility, , 50,00,000, (30,00,000), 20,00,000, 1,10,00,000
Page 982 : Income Under The Head Business/Profession, , 81, , Less: Deduction u/s 35AD [Deduction@100% in respect of the expenditure incurred, prior to the commencement of its operations and capitalized in the books of account, on 1.4.2021. Deduction is not available on expenditure incurred on acquisition of, land] [₹ 93 lakhs – ₹ 13 lakhs], Business income of warehousing facility chargeable to tax, Total Income, Computation of tax liability, Tax on ₹ 50,00,000, Add: Health and Education cess@4%, Total tax liability, , (80,00,000), , 30,00,000, 50,00,000, 13,12,500, 52,500, 13,65,000, , Computation of adjusted total income and AMT of Mr. Xavier for A.Y. 2022-23, Particulars, , ₹, , Total Income (as computed above), Add: Deduction under section 10AA, Add: Deduction under section 35AD, Less: Depreciation u/s 32 [On building@10% of ₹ 80 lakhs], Adjusted Total Income, Alternate Minimum
[email protected]%, Add: Surcharge @15% (since adjusted total income >₹ 1 crore), Add: Health and Education cess@4%, Total tax liability, Tax Liability (Rounded off), , ₹, 50,00,000, 30,00,000, 80,00,000, , 80,00,000, (8,00,000), , 72,00,000, 1,52,00,000, 28,12,000, 4,21,800, 32,33,800, 1,29,352, 33,63,152, 33,63,150, , Since the regular income-tax payable is less than the alternate minimum tax payable, the adjusted total, income shall be deemed to be the total income and tax is leviable @18.5% thereof plus surcharge@15% and, cess@4%. Therefore, the tax liability is ₹ 33,63,150., AMT Credit to be carried forward under section 115JEE, , Tax liability under section 115JC, Less: Tax liability under the regular provisions of the Income-tax, Act, 1961, , ₹, 33,63,150, (13,65,000), 19,98,150, , Note:, 1. Assuming the capital expenditure of ₹ 80 lakhs is incurred entirely on building, 2. In the third para of the question, there is a difference between the figure of capital expenditure incurred in, respect of warehouse i.e., ₹ 93 lakhs (including cost of land ₹ 13 lakhs) and the figure of capital expenditure, capitalised in the books on 1.4.2020 i.e., ₹ 63 lakhs. It appears to be a typographical error, due to which the, main solution has been worked out considering ₹ 93 lakhs as the amount capitalised in the books on, 1.4.2020., However, alternative answers have been worked out below considering ₹ 63 lakhs (being the figure as, printed in the question paper) as the amount capitalised in the books on 1.4.2020. In Alternative 1, it has, been assumed that the amount of ₹63 lakhs capitalised on 1.4.2020 does not include cost of land. In
Page 983 : Income Under The Head Business/Profession, , 82, , Alternative 2, it has been assumed that the amount of ₹ 63 lakhs capitalised on 1.4.2020 includes cost of, land., Alternative 1 (The amount of ₹ 63 lakhs capitalized on 1.4.2021 does not include cost of land), Computation of total income and tax liability of Mr. Xavier for A.Y. 2022-23 (under the regular, provisions of the Act), Particulars, Profits and gains of business or profession, Profit from unit in SEZ, Less: Deduction u/s 10AA, [50,00,000 x 90,00,000/1,50,00,000 x 100%, since it is 5th year of manufacturing], Business income of SEZ unit chargeable to tax, Profit from operation of warehousing facility, Less: Deduction u/s 35AD [Deduction@100% in respect of the expenditure incurred, prior to the commencement of its operations and capitalized in the books of account on, 1.4.2021. It is assumed that the capitalized expenditure of ₹ 63 lakhs does not include, cost of land], Business income of warehousing facility chargeable to tax, Total Income, Computation of tax liability, Tax on ₹ 67,00,000, Add: Surcharge @10%, , ₹, 50,00,000, (30,00,000), , 20,00,000, 1,10,00,000, (63,00,000), , 47,00,000, 67,00,000, 18,22,500, 1,82,250, 20,04,750, 80,190, 20,84,940, , Add: Health and Education cess@4%, Total tax liability, Computation of adjusted total income and AMT of Mr. Xavier for A.Y. 2022-23, Particulars, Total Income (as computed above), Add: Deduction under section 10AA, Add: Deduction under section 35AD, Less: Depreciation u/s 32 [On building @10% of ₹63, Lakhs], Adjusted Total Income, Alternate Minimum
[email protected]%, Add: Surcharge@15% (since adjusted total income >₹ 1 crore), Add: Health and Education cess@4%, Total tax liability, Tax Liability (Rounded off), , ₹, , 63,00,000, (6,30,000), , ₹, 67,00,000, 30,00,000, 97,00,000, 56,70,000, 1,53,70,000, 28,43,450, 4,26,518, 32,69,968, 1,30,799, 34,00,767, 34,00,770, , Since the regular income-tax payable is less than the alternate minimum tax payable, the adjusted total, income shall be deemed to be the total income and tax is leviable @18.5% thereof plus surcharge@15% and, cess@4%. Therefore, the tax liability is ₹ 34,00,770.
Page 984 :
Income Under The Head Business/Profession, , 83, , AMT Credit to be carried forward under section 115JEE, ₹, 34,00,770, (20,84,940), , Tax liability under section 115JC, Less: Tax liability under the regular provisions of the Income-tax Act, 1961, , 13,15,830, Note:, 1. Since the question mentions ₹ 1,10,00,000 as the profit from operation of warehousing facility before, claiming deduction u/s 35AD, it is assumed that said figure of profit is after providing depreciation u/s 32 on, ₹ 17 lakhs, being the amount of capital expenditure not capitalized as on 1.4.2020 less cost of land (i.e., ₹ 93, lakhs – ₹ 63 lakhs = ₹ 30 lakhs– ₹ 13 lakhs (cost of land) = ₹ 17 lakhs), 2. Assuming the capital expenditure of ₹ 63 lakhs is incurred entirely on building, Alternative 2 (The amount of ₹63 lakh capitalized includes cost of land), Computation of total income and tax liability of Mr. Xavier for A.Y. 2022-23 (under the regular, provisions of the Act), Particulars, , ₹, , Profits and gains of business or profession, Profit from unit in SEZ, Less: Deduction u/s 10AA, [50,00,000 x 90,00,000/1,50,00,000 x 100%, since it is 5th year of manufacturing], Business income of SEZ unit chargeable to tax, Profit from operation of warehousing facility, Less: Deduction u/s 35AD [Deduction@100% in respect of the expenditure incurred, prior to the commencement of its operations, and capitalized in the books of account, on 1.4.2021. Deduction is not available on expenditure incurred on acquisition of, land. It is assumed that the capitalized expenditure includes ₹ 13 lakhs of land] [₹ 63, lakhs – ₹ 13 lakhs], Business income of warehousing facility chargeable to tax, Total Income, Computation of tax liability, Tax on ₹ 80,00,000, Add: Surcharge @10%, , 50,00,000, (30,00,000), 20,00,000, 1,10,00,000, (50,00,000), , 60,00,000, 80,00,000, 22,12,500, 2,21,250, 24,33,750, , Add: Health and Education cess@4%, Total tax liability, , 97,350, 25,31,100, , Computation of adjusted total income and AMT of Mr. Xavier for A.Y. 2022-23, Particulars, , ₹, , Total Income (as computed above), Add: Deduction under section 10AA, Add: Deduction under section 35AD, Less: Depreciation u/s 32 [On building @10% of ₹ 50 lakhs], , ₹, 80,00,000, 30,00,000, 1,10,00,000, , 50,00,000, (5,00,000), , 45,00,000
Page 985 : Income Under The Head Business/Profession, , 84, , Adjusted Total Income, Alternate Minimum
[email protected]%, Add: Surcharge@15% (since adjusted total income, > ₹ 1 crore), , 1,55,00,000, 28,67,500, 4,30,125, , 32,97,625, Add: Health and Education cess@4%, 1,31,905, Total tax liability, 34,29,530, Since the regular income-tax payable is less than the alternate minimum tax payable, the adjusted total, income shall be deemed to be the total income and tax is leviable @18.5% thereof plus surcharge@15% and, cess@4%. Therefore, the tax liability is ₹ 34,29,530., AMT Credit to be carried forward under section 115JEE, ₹, Tax liability under section 115JC, Less: Tax liability under the regular provisions of the Income-tax Act, 1961, , 34,29,530, (25,31,100), 8,98,430, , Note:, 1. Since the question mentions ₹ 1,10,00,000 as the profit from operation of warehousing facility before, claiming deduction u/s 35AD, it is assumed that said figure of profit is after providing depreciation u/s 32 on, ₹ 30 lakhs, being the amount of capital expenditure not capitalized as on 1.4.2020 (₹ 93 lakhs – ₹ 63 lakhs)., 2. Assuming the capital expenditure of ₹ 50 lakhs is incurred entirely on building, Answer 4(a), Computation of Total Income of Mr. Raghav for A.Y. 2022-23, Particulars, Amount, (₹), Salary, [Since Mrs. Raghav along with her brother holds shares carrying 100%, voting power in M/s M Pvt. Ltd., they have a substantial interest in the, company. Since Mr. Raghav is working in the same company without, any professional qualifications commensurate with his salary, the salary, of ₹ 3,75,000 received by him would be included in the hands of Mrs., Raghav., Income from house property, House 1 [Self-occupied], Net annual value, Less: Interest on loan [upto ₹2,00,000], House 2 [Let out], Gross annual value [₹60,000 x 12], Less: Municipal taxes, Net annual value, Less: Deductions from Net Annual Value, (a) 30% of Net Annual Value, (b) Interest on loan, House in Delhi [Since Mr. Raghav receives direct or indirect benefit, from income arising to his sister’s daughter, Ms. Vamika, from the, transfer of house to her without consideration, such income is to be, included in the total income of Mr. Raghav as per proviso to section, , Amount, (₹), Nil, , (2,00,000), , (2,00,000), , 7,20,000, 7,20,000, (2,16,000), (5,00,000), , 4,000
Page 986 :
Income Under The Head Business/Profession, 62(1), even though the transfer may not be revocable during lifetime of, Ms. Vamika’s], Gross Annual Value, Less: Municipal taxes, Net Annual Value, Less: Deductions from Net Annual Value, (a) 30% of Net Annual Value, (b) Interest on loan, Profits and gains from business or profession, Share of profit from firm [Exempt u/s 10(2A)], Exempt income cannot be clubbed, Income from other sources, Dividend on preference shares [Taxable in the hands of Mr. Raghav, as per section 60, since he transferred the income, i.e.,, dividend, without transferring the asset, i.e., preference shares], Interest on debentures, Interest from saving bank account, Cash gift [Taxable, since sum of money exceeding, ₹ 50,000 is received from his niece, who is not a relative, as per section 56(2)], Gross Total Income, Less: Deduction under Chapter VI-A, Deduction under section 80C [Principal repayment of, loan ₹ 5 lakh, restricted to ₹ 1,50,000], Deduction under section 80TTA [Interest from, savings bank account], Total Income, , 85, , 5,50,000, 5,50,000, (1,65,000), -, , 3,85,000, 1,89,000, , -, , 13,00,000, , 7,50,000, 2,00,000, , 75,000, , 23,25,000, 25,14,000, , 1,50,000, , 10,000, , 1,60,000, 23,54,000, , Note:, 1. Rent receivable has been taken as the gross annual value in the absence of other information
Page 987 :
Income Under The Head Salary, , 86, , INCOME UNDER THE HEAD, SALARY, SECTION 15 TO 17, , SOLUTIONS OF MCQS, Answer, 1. (d); 2. (d); 3. (b); 4. (a); 5. (b); 6. (b); 7. (a); 8. (a); 9. (b); 10. (c); 11.(c); 12.(c); 13.(c); 14.(c); 15.(a);, 16.(c); 17.(c); 18.(b); 19.(a); 20.(b); 21.(a); 22.(a); 23.(d); 24.(c); 25.(a); 26.(a); 27.(a); 28.(a); 29.(b); 30.(d)
Page 988 :
Income Under The Head Salary, , 87, , SOLUTIONS, TO, , PRACTICE PROBLEMS, Solution 1:, Computation of Gross Salary, Basic Pay, [(44,500 x 3) + (45,000 x 9)], Working Note:, 01.07.2011 – 30.06.2012 =, 01.07.2012 – 30.06.2013 =, 01.07.2013 – 30.06.2014 =, 01.07.2014 – 30.06.2015 =, 01.07.2015 – 30 06.2016 =, 01.07.2016 – 30.06.2017 =, 01.07.2017 – 30.06.2018 =, 01.07.2018 – 30.06.2019 =, 01.07.2019 – 30.06.2020 =, 01.07.2020 – 30.06.2021 =, 01.07.2021 – 30.06.2022 =, Dearness Allowance, Working Note:, From April to June, 7% of (44,500 x 3) =, From July to March, 10% of (45,000 x 9) =, Total = ` (9,345 + 40,500) =, Gross Salary, Less: Standard Deduction u/s 16(ia), Income under the head Salary, Gross Total Income, Less: Deduction u/s 80C to 80U, Total Income (Rounded off u/s 288A), Computation of Tax Liability, Tax on `5,38,350 at slab rate, Add: HEC @ 4%, Tax Liability, Rounded off u/s 288B, Solution 2:, Computation of Gross Salary, Basic Pay, [(37,000 x 6) + (38,100 x 6)], Working Note:, 01.10.2008 – 30.09.2009 =, 01.10.2009 – 30.09.2010 =, , `, 5,38,500.00, `, 40,000 p.m., 40,500 p.m., 41,000 p.m., 41,500 p.m., 42,000 p.m., 42,500 p.m., 43,000 p.m., 43,500 p.m., 44,000 p.m., 44,500 p.m., 45,000 p.m., 49,845.00, `, 9,345, 40,500, 49,845, 5,88,345.00, (50,000.00), 5,38,345.00, 5,38,345.00, Nil, 5,38,350.00, 20,170.00, 806.80, 20,976.80, 20,980.00, `, 4,50,600.00, `, 25,000 p.m., 25,900 p.m.
Page 989 :
Income Under The Head Salary, 01.10.2010 – 30.09.2011 =, 01.10.2011 – 30.09.2012 =, 01.10.2012 – 30.09.2013 =, 01.10.2013 – 30.09.2014 =, 01.10.2014 – 30.09.2015 =, 01.10.2015 – 30.09.2016 =, 01.10.2016 – 30.09.2017 =, 01.10.2017 – 30.09.2018 =, 01.10.2018 – 30.09.2019 =, 01.10.2019 – 30.09.2020 =, 01.10.2020 – 30.09.2021 =, 01.10.2021 – 30.09.2022 =, Dearness Allowance, Working Note:, From April to September, 4.35% of (37,000 x 6) =, From October to December, 7.5% of (38,100 x 3) =, From January to March, 10.5% of (38,100 x 3) =, Total, Gross Salary, Less: Standard Deduction u/s 16(ia), Income under the head Salary, Gross Total Income, Less: Deduction u/s 80C to 80U, Total Income (rounded off u/s 288A), Computation of Tax Liability, Tax on `4,30,830 at slab rate, Less: Rebate u/s 87A, Tax Liability, , 26,800 p.m., 27,700 p.m., 28,600 p.m., 29,500 p.m., 30,400 p.m., 31,500 p.m., 32,600 p.m., 33,700 p.m., 34,800 p.m., 35,900 p.m., 37,000 p.m., 38,100 p.m., , Solution 3:, Basic Pay (40,000 x 12), Dearness allowance (10,000 x 12), Bonus, Commission (60,00,000 x 2%), Employer’s contribution to recognised provident fund in excess of, 12% of retirement benefit salary, Working Note:, RBS = 4,80,000 + 48,000 + 1,20,000, = 6,48,000, 12% of RBS = 77,760, Employer’s contribution = 8,000 x 12 = 96,000, 96,000 – 77,760 = 18,240, Interest credited in excess of 9.5% p.a., (1,00,000 / 10% x 0.5%)/2, Gross Salary, Less: Standard Deduction u/s 16(ia), Income under the head Salary, Income under the head other sources, Interest credited in excess of 9.5% p.a., (1,00,000 / 10% x 0.5%)/2, , 88, , 30,231.00, `, 9,657.00, 8,572.50, 12,001.50, 30,231.00, 4,80,831.00, (50,000.00), 4,30,831.00, 4,30,831.00, Nil, 4,30,830.00, 9,041.50, (9,041.50), Nil, `, 4,80,000.00, 1,20,000.00, 12,000.00, 1,20,000.00, 18,240.00, , 2,500.00, 7,52,740.00, (50,000.00), 7,02,740.00, 2,500.00
Page 990 :
Income Under The Head Salary, , 89, , Gross Total Income, Less: Deduction u/s 80C, Total Income, Computation of Tax Liability, Tax on `6,09,240 at slab rate, Add: HEC @ 4%, Tax Liability, Rounded off u/s 288B, , 7,05,240.00, (96,000.00), 6,09,240.00, , Solution 4:, Basic Pay (80,000 x 12), Dearness allowance (20,000 x 12), Bonus, Commission (80,00,000 x 2%), Employer’s contribution to recognised provident fund in excess of, 12% of retirement benefit salary, Working Note:, RBS = 9,60,000 + 1,60,000, = 11,20,000, 12% of RBS = 1,34,400, Employer’s contribution = 14,000 x 12 = 1,68,000, 1,68,000 – 1,34,400 = 33,600, Gross Salary, Less: Standard Deduction u/s 16(ia), Income under the head Salary, Income under the head House Property, Gross Total Income, Less: Deduction u/s 80C (` 1,68,000 but limited to ` 1,50,000), Total Income, Computation of Tax Liability, Tax on `15,17,600 at slab rate, Add: HEC @ 4%, Tax Liability, Rounded off u/s 288B, , `, 9,60,000.00, 2,40,000.00, 24,000.00, 1,60,000.00, 33,600.00, , Solution 5:, (a) He is covered under Payment of Gratuity Act, 1972, Basic Salary, Working Note:, From April to May, 33,000 x 2 =, From June to August, 37,000 x 3 =, For September, 11/30 x 37,000 =, Total = `66,000 + 1,11,000 + 13,566.67 =, Dearness Allowance, Working Note:, From April to May, 3,000 x 2 =, From June to August, 4,000 x 3 =, For September, , 34,348.00, 1,373.92, 35,721.92, 35,720.00, , 14,17,600.00, (50,000.00), 13,67,600.00, 3,00,000.00, 16,67,600.00, (1,50,000.00), 15,17,600.00, 2,67,780.00, 10,711.20, 2,78,491.20, 2,78,490.00, `, 1,90,566.67, `, 66,000, 1,11,000, , 13,566.67, 1,90,566.67, 19,466.67, `, 6,000.00, 12,000.00
Page 991 :
Income Under The Head Salary, 11/30 x 4,000 =, Total = `6,000 +12,000 + 1,466.67 =, Gratuity {Sec 10(10)}, Working Note:, Least of the following is exempt:, 1. `5,10,000, 2. `20,00,000, 3. 15/26 x (37,000 + 4,000) x 12 = `2,83,846.15, Received = `5,10,000.00, Exempt = (`2,83,846.15), Taxable = `2,26,153.85, Gross Salary, Less: Standard Deduction u/s 16(ia), Income under the head Salary, Gross Total Income / Total Income, (Rounded off u/s 288A), Computation of Tax Liability, Tax on `3,86,190 at slab rate, Less: Rebate u/s 87A, Tax Liability, (b) He is not covered under Payment of Gratuity Act, 1972, Basic Salary, Working Note:, From April to May, 33,000 x 2 =, From June to August, 37,000 x 3 =, For September, 11/30 x 37,000 =, Total = `66,000 + 1,11,000 + 13,566.67 =, Dearness Allowance, Working Note:, From April to May, 3,000 x 2 =, From June to August, 4,000 x 3 =, For September, 11/30 x 4,000 =, Total = `6,000 +12,000 + 1,466.67 =, Gratuity {Sec 10(10)}, Working Note:, Least of the following is exempt, 1. `5,10,000, 2. `20,00,000, 3. ½ x 35,850 x 11 = `1,97,175, Received = `5,10,000, Exempt = (`1,97,175), Taxable = `3,12,825, Calculation of Average Salary, Basic Pay, From November to May `33,000 x 7 =, , 90, 1,466.67, 19,466.67, 2,26,153.85, , 4,36,187.19, (50,000.00), 3,86,187.19, 3,86,190.00, 6,809.50, (6,809.50), Nil, 1,90,566.67, `, 66,000, 1,11,000, 13,566.67, 1,90,566.67, 19,466.67, `, 6,000.00, 12,000.00, 1,466.67, 19,466.67, 3,12,825.00, `, , 2,31,000
Page 992 :
Income Under The Head Salary, From June to August `37,000 x 3 =, Total, =, D.A., From November to May `1,500 x 7 =, From June to August `2,000 x 3 =, Total, =, Average Salary = (3,42,000 + 16,500)/10 =, Gross Salary, Less: Standard Deduction u/s 16(ia), Income under the head Salary, Gross Total Income / Total Income, (Rounded off u/s 288A), Computation of Tax Liability, Tax on `4,72,860 at slab rate, Less: Rebate u/s 87A, Tax Liability, Solution 6:, Computation of income under the head Salary, Basic Pay, [(39,000 x 5) + (39,000 x 17 /30)], Dearness Allowance, [(3,000 x 5) + (3,000 x 17/30)], Gratuity {Sec 10(10)}, Working Note:, 1. `5,70,000, 2. `20,00,000, 3.15/26 x 42,000 x 20 = `4,84,615.38, Received = `5,70,000.00, Exempt = (`4,84,615.38), Taxable = `85,384.62, Uncommuted Pension {Sec 17(1)(ii)}, Working Note:, From September, 5,000 x 13/30 =, From October to December, 5,000 x 3 =, From January to March, 5,000 x 52% x 3 =, Total = `2,166.67 + `15,000 + ` 7,800 =, Commuted Pension {Sec 10(10A)}, Working Note:, Received, =, Exempt = 2,88,000 / 48% x 100% x 1/3 =, Taxable, =, Gross Salary, Less: Standard Deduction u/s 16(ia), Income under the head Salary, Gross Total Income, Less: Deductions u/s 80C to 80U, Total Income, Rounded Off u/s 288A, , 91, 1,11,000, 3,42,000, 10,500, 6,000, 16,500, 35,850, 5,22,858.34, (50,000.00), 4,72,858.34, 4,72,860.00, 11,143.00, (11,143.00), Nil, `, 2,17,100.00, 16,700.00, 85,384.62, , 24,966.67, `, 2,166.67, 15,000.00, 7,800.00, 24,966.67, 88,000.00, `, 2,88,000, (2,00,000), 88,000, 4,32,151.29, (50,000.00), 3,82,151.29, 3,82,151.29, Nil, 3,82,151.29, 3,82,150.00
Page 993 :
Income Under The Head Salary, , 92, , Computation of Tax Liability, Tax on `3,82,150 at slab rate, Less: Rebate u/s 87A, Tax Liability, Solution 7:, Computation of income under the head Salary, Basic Pay, [(12,500 x 7) + (12,500 x 27/30)], Gratuity {Sec 10(10)}, Working Note:, Least of the following is exempt:, 1. `2,50,000, 2. `20,00,000, 3. ½ x 1,25,000/10 x 11 = `68,750, Received = `2,50,000, Exempt = (` 68,750), Taxable = `1,81,250, Uncommuted Pension {Sec 17(1)(ii)}, Working Note:, From November, 6,200 x 3/30 =, From December and January, 6,200 x 2 =, From February to March, 6,200 x 2 x 48% =, Total = `620 +12,400 + 5,952 =, Commuted Pension {Sec 10(10A)}, Working Note:, Received =, Exempt = 3,86,880 / 52% x 1/3 =, Taxable =, Gross Salary, Less: Standard Deduction u/s 16(ia), Income under the head Salary, Gross Total Income, Less: Deductions u/s 80C to 80U, Total Income, Rounded off u/s 288A, Computation of Tax Liability, Tax on `3,87,850 at slab rate, Less: Rebate u/s 87A, Tax Liability, Solution 8:, Computation of income under the head Salary, Basic Pay, [(17,900 x 3) + (18,400 x 2) + (18,400 x 15/30)], Working Note:, 01.07.2003 – 30.06.2004 =, 01.07.2004 – 30.06.2005 =, 01.07.2005 – 30.06.2006 =, , 6,607.50, (6,607.50), Nil, `, 98,750.00, 1,81,250.00, , 18,972.00, `, 620, 12,400, 5,952, 18,972, 1,38,880.00, `, 3,86,880.00, (2,48,000.00), 1,38,880.00, 4,37,852.00, (50,000.00), 3,87,852.00, 3,87,852.00, Nil, 3,87,852.00, 3,87,850.00, 6,892.50, (6,892.50), Nil, `, 99,700.00, `, 10,800 p.m., 11,200 p.m., 11,600 p.m.
Page 994 :
Income Under The Head Salary, 01.07.2006 – 30.06.2007 =, 01.07.2007 – 30.06.2008 =, 01.07.2008 – 30.06.2009 =, 01.07.2009 – 30.06.2010 =, 01.07.2010 – 30.06.2011 =, 01.07.2011 – 30.06.2012 =, 01.07.2012 – 30.06.2013 =, 01.07.2013 – 30.06.2014 =, 01.07.2014 – 30.06.2015 =, 01.07.2015 – 30.06.2016 =, 01.07.2016 – 30.06.2017 =, 01.07.2017 – 30.06.2018 =, 01.07.2018 – 30.06.2019 =, 01.07.2019 – 30.06.2020 =, 01.07.2020 – 30.06.2021 =, 01.07.2021 – 30.06.2022 =, Dearness Allowance, Gratuity {Sec 10(10)}, Working Note:, Least of the following is exempt:, 1. `2,60,000, 2. `20,00,000, 3. ½ x 27,000 x 18 = `2,43,000, Calculation of average salary, Basic Pay, [(17,900 x 8) + (18,400 x 2)] = 1,80,000, Dearness Allowance, 50% of `1,80,000, = 90,000, Average Salary, = 2,70,000/10, = 27,000, Received = `2,60,000, Exempt = (`2,43,000), Taxable = ` 17,000, Uncommuted Pension {Sec 17(1)(ii)}, Working Note:, From September, 6,000 x 15/30 =, From October to December, 6,000 x 3 =, From January to March, 6,000 x 3 x 25% =, Total = `3,000 +18,000 + 4,500 =, Commuted Pension {Sec 10(10A)}, Working Note:, Received =, Exempt = (6,00,000 x 4/3 x 1/3) =, Taxable =, Gross Salary, Less: Standard Deduction u/s 16(ia), Income under the head Salary, Gross Total Income, Less: Deductions u/s 80C to 80U, , 93, , 12,000 p.m., 12,400 p.m., 12,800 p.m., 13,200 p.m., 13,600 p.m., 14,000 p.m., 14,400 p.m., 14,800 p.m., 15,200 p.m., 15,600 p.m., 16,000 p.m., 16,400 p.m., 16,900 p.m., 17,400 p.m., 17,900 p.m., 18,400 p.m., 49,850.00, 17,000.00, , 25,500.00, `, 3,000, 18,000, 4,500, 25,500, 3,33,333.33, `, 6,00,000.00, (2,66,666.67), 3,33,333.33, 5,25,383.33, (50,000.00), 4,75,383.33, 4,75,383.33, Nil
Page 995 :
Income Under The Head Salary, , 94, , Total Income, Rounded off u/s 288A, Computation of Tax Liability, Tax on `4,75,380 at slab rate, Less: Rebate u/s 87A, Tax Liability, , 4,75,383.33, 4,75,380.00, , Solution 9:, Basic Salary, [(22,000 x 4) + (25,000 x 8)], Dearness Allowance, [(4,000 x 4) + (6,000 x 8)], Leave Salary {Sec 10(10AA)}, Working Note:, 1. ` 5,00,000, 2. ` 3,00,000, 3. ` 10 x 2,72,000/10 = ` 2,72,000, 4. 2,72,000 /10 x 19 = ` 5,16,800, Received = `5,00,000, Exempt = (`2,72,000), Taxable = `2,28,000, Calculation of average salary, Basic Pay, [(22,000 x 2) + (25,000 x 8)] = 2,44,000, Dearness Allowance, [(2,000 x 2) + (3,000 x 8)] = 28,000, Average Salary, = 2,72,000/10, = 27,200, Computation of leave at credit, Leave Entitlement, =, 30, Less: Leave Availed, =, (7), Less: Leave Encashed, =, (4), Leave at Credit, =, 19, Gross Salary, Less: Standard Deduction u/s 16(ia), Income under the head Salary, Gross Total Income, Less: Deduction u/s 80C to 80U, Total Income, Computation of Tax Liability, Tax on `5,30,000 at slab rate, Add: HEC @ 4%, Tax Liability, Solution 10:, Computation of income under the head Salary, Basic Pay, Working Note:, [(9,500 x 6) + (13,000 x 5)], Dearness Allowance, (10% of basic pay), Gratuity {Sec 10(10)}, Working Note:, Least of the following is exempt:, , `, 2,88,000.00, , 11,269.00, (11,269.00), Nil, , 64,000.00, 2,28,000.00, , 5,80,000.00, (50,000.00), 5,30,000.00, 5,30,000.00, Nil, 5,30,000.00, 18,500.00, 740.00, 19,240.00, `, 1,22,000.00, , 12,200.00, 1,35,550.00
Page 996 :
Income Under The Head Salary, 1. `2,50,000, 2. `20,00,000, 3. ½ x 1,09,000/10 x 21 = `1,14,450, Received = `2,50,000, Exempt = (`1,14,450), Taxable = `1,35,550, Calculation of Average Salary, Computation of Basic Pay, [(9,500 x 6) + (13,000 x 4)] = 1,09,000, Average Salary = 1,09,000/10 = 10,900, Commuted Pension {Sec 10(10A)}, Working Note:, Received, =, Exempt = 2,88,000 / 40% x 100% x 1/3 =, Taxable, =, Uncommuted Pension {Sec 17(1)(ii)}, Working Note:, (6,000 x 60%) x 1 = `3,600, Leave Salary {Sec 10(10AA)}, Working Note:, Least of the following is exempt:, 1. `3,10,000, 2. `3,00,000, 3.10 x 1,12,500/10 = `1,12,500, 4. `1,12,500/10 x 520/30 = `1,95,000, Received = `3,10,000, Exempt = (`1,12,500), Taxable = `1,97,500, Computation of leave at credit, Leave Entitlement = 30 x 21 = 630 days, Less: Leave Encashed = (45 days), Less: Leave Availed = (65 days), Leave at Credit = 520 days, Calculation of Average Salary, Computation of Basic Pay, [(9,500 x 5) + (13,000 x 5)] = 1,12,500, Average Salary = 1,12,500/10 = 11,250, Gross Salary, Less: Standard Deduction u/s 16(ia), Income under the head Salary, Gross Total Income, Less: Deductions u/s 80C to 80U, Total Income, Computation of Tax Liability, Tax on `4,68,850 at slab rate, Less: Rebate u/s 87A, Tax Liability, Solution 11:, Computation of income under the head Salary, Basic Pay, (43,000 x 8), , 95, , 48,000.00, `, 2,88,000, (2,40,000), 48,000, 3,600.00, , 1,97,500.00, , 5,18,850.00, (50,000.00), 4,68,850.00, 4,68,850.00, Nil, 4,68,850.00, 10,942.50, (10,942.50), Nil, `, 3,44,000.00
Page 997 :
Income Under The Head Salary, Leave Salary {Sec 10(10AA)}, Working Note:, Computation of leave availed and encashed by the employee, Leave entitlement at the rate of 30 days (30 x 16), = 480 days, Leave availed and encashed by the employee, = 260 days, Leave at the credit, = 220 days, Leave allowed by employer (65 x 16), = 1040 days, Less: Leave encashed by the employee at the time of retirement = (780 days), Hence leave availed/encashed while in service, = 260 days, Average salary of 10 months ending November 30, 2021, = 43,000, Least of the following is exempt: –, (1) Cash equivalent of leave at the credit of the employee at the time of, retirement (i.e. 43,000 x 220/30), = 3,15,333.33, (2) 10 Months Average Salary = 43,000 x 10 = 4,30,000, (3) ` 3,00,000, (4) ` 5,12,000, Received = ` 5,12,000, Exempt = (` 3,00,000), Taxable = ` 2,12,000, Gross Salary, Less: Standard Deduction u/s 16(ia), Income under the head Salary, Gross Total Income, Less: Deductions u/s 80C to 80U, Total Income, Computation of Tax Liability, Tax on `5,06,000 at slab rate, Add: HEC @ 4%, Tax Liability, Rounded off u/s 288B, Solution 12:, Computation of Taxable Income, Basic Pay, (45,000 x 12), Dearness Allowance, (7,000 x 12), Employer’s contribution in excess of 12% of salary {Rule 6 of Part A of schedule IV}, (72,000-64,800), Rent Free Accommodation {Sec 17(2)(i) Rule 3(1)}, Working Note:, 15% of rent free accommodation salary or rent paid whichever is less, Rent free accommodation salary = Basic Pay = `5,40,000, 15% of rent free accommodation salary = `81,000, Rent Paid = `36,000, Perquisite value = `36,000, Gross Salary, Less: Standard Deduction u/s 16(ia), Income under the head Salary, Gross Total Income, Less: Deduction u/s 80C, {Employee’s contribution in recognised provident fund}, Total Income, , 96, 2,12,000.00, , 5,56,000.00, (50,000.00), 5,06,000.00, 5,06,000.00, Nil, 5,06,000.00, 13,700.00, 548.00, 14,248.00, 14,250.00, `, 5,40,000, 84,000, 7,200, 36,000, , 6,67,200, (50,000), 6,17,200, 6,17,200, (72,000), 5,45,200
Page 998 :
Income Under The Head Salary, Computation of Tax Liability, Tax on `5,45,200 at slab rate, Add: HEC @ 4%, Tax Liability, Rounded off u/s 288B, Solution 13:, Computation of income under the head Salary, Basic Pay, (20,000 x 7), Dearness Allowance, (7,000 x 7), Refund of employer’s contribution in unrecognised provident fund, Refund of Interest on employer’s contribution in unrecognised provident fund, Gratuity {Sec 10(10A)}, Working Note:, Least of the following is exempt:, 1. `2,60,000, 2. `20,00,000, 3. ½ x 20,000 x 20 = ` 2,00,000, Received = `2,60,000, Exempt = (`2,00,000), Taxable = ` 60,000, Uncommuted Pension {Sec 17(1)(ii)}, Working Note:, `, For November to December, 5,000 x 2 =, 10,000, For January to March, 5,000 x 60% x 3 =, 9,000, Total = `10,000 + `9,000 =, 19,000, Commuted Pension {Sec 10(10A)}, Working Note:, `, Received =, 2,40,000, Exempt = 2,40,000 / 40% x 100% x 1/3 =, (2,00,000), Taxable, =, 40,000, Gross Salary, Less: Standard Deduction u/s 16(ia), Income under the head Salary, Income under the head Other Sources, (Interest on employee’s contribution), Gross Total Income, Less: Deduction u/s 80C to 80U, Total Income, Computation of Tax Liability, Tax on `8,58,000 at slab rate, Add: HEC @ 4%, Tax Liability, Rounded off u/s 288B, Solution 14:, Computation of Gross Salary, Basic Pay, , 97, 21,540.00, 861.60, 22,401.60, 22,400.00, `, 1,40,000, 49,000, 4,00,000, 1,00,000, 60,000, , 19,000, , 40,000, , 8,08,000, (50,000), 7,58,000, 1,00,000, 8,58,000, Nil, 8,58,000, 84,100, 3,364, 87,464, 87,460, `, 4,80,000.00
Page 999 :
Income Under The Head Salary, , 98, , (40,000 x 12), Dearness Allowance, (16,000 x 12), Bonus, (1,000 x 12), Commission, (2.5% of 60,00,000), House Rent Allowance {Sec 10 (13A), Rule 2A}, Working Note:, Least of the following is exempt:, 1. `78,000 – 72,600 = `5,400, 2. 50% of retirement benefit salary = `3,63,000, (Retirement benefit salary = `7,26,000), 3. `72,000, Received = `72,000, Exempt = (` 5,400), Taxable = `66,600, Gross Salary, Less: Standard Deduction u/s 16(ia), Income under the head Salary, Gross Total Income, Less: Deduction u/s 80C to 80U, Total Income, Computation of Tax Liability, Tax on `8,50,600 at slab rate, Add: HEC @ 4%, Tax Liability, Rounded off u/s 288B, Solution 15:, Computation of Gross Salary, Basic Pay, (45,000 x 12), Dearness Allowance, (19,000 x 12), House Rent Allowance {Sec 10(13A), Rule 2A}, Working Note:, From October to March, Least of the following is exempt, 1. `72,000 – `32,700 = `39,300, 2. 40% of retirement benefit salary = `1,30,800, (Retirement benefit salary = `3,27,000), 3. `60,000, Received = `60,000, Exempt = (`39,300), Taxable = ` 20,700, Children Education Allowance {Sec 10(14), Rule 2BB}, Working Note:, Received = `75 x 4 x 12 =, Exempt = `75 x 2 x 12 =, Taxable, =, Hostel Allowance {Sec 10(14), Rule 2BB}, , 1,92,000.00, 12,000.00, 1,50,000.00, 66,600.00, , 9,00,600.00, (50,000.00), 8,50,600.00, 8,50,600.00, Nil, 8,50,600.00, 82,620.00, 3,304.40, 85,924.80, 85,920.00, `, 5,40,000.00, 2,28,000.00, 20,700.00, , 1,800.00, `, 3,600, (1,800), 1,800, 2,400.00
Page 1000 :
Income Under The Head Salary, Working Note:, Received = `500 x 12 =, Exempt = `300 x 12 =, Taxable, =, Transport Allowance, Gross Salary, Less: Standard Deduction u/s 16(ia), Income under the head Salary, Gross Total Income, Less: Deduction u/s 80C to 80U, Total Income, Computation of Tax Liability, Tax on `7,58,200 at slab rate, Add: HEC @ 4%, Tax Liability, Rounded off u/s 288B, Solution 16:, Computation of Gross Salary, Basic Pay, (55,000 x 12), Dearness Allowance, (10% of Basic pay), Children Education Allowance {Sec 10(14), Rule 2BB}, Working Note:, Received = `750 x 12 =, Exempt = `100 x 12 =, Taxable, =, Transport Allowance, Flight Allowance {Sec 10(14), Rule 2BB}, Working Note:, Least of the following is exempt:, 1. 70% of allowance received, = 70% of (10,000 x 12) = `84,000, 2. 10,000 x 12 = `1,20,000, Received = `1,20,000, Exempt = (` 84,000), Taxable = ` 36,000, Gross Salary, Less: Standard Deduction u/s 16(ia), Income under the head Salary, Gross Total Income, Less: Deduction u/s 80C to 80U, Total Income, Computation of Tax Liability, Tax on `7,41,400 at slab rate, Add: HEC @ 4%, Tax Liability, Rounded off u/s 288B, , 99, `, 6,000, (3,600), 2,400, 15,300.00, 8,08,200.00, (50,000.00), 7,58,200.00, 7,58,200.00, Nil, 7,58,200.00, 64,140.00, 2,565.60, 66,705.60, 66,710.00, `, 6,60,000.00, 66,000.00, 7,800.00, `, 9,000, (1,200), 7,800, 21,600.00, 36,000.00, , 7,91,400.00, (50,000.00), 7,41,400.00, 7,41,400.00, Nil, 7,41,400.00, 60,780.00, 2,431.20, 63,211.20, 63,210.00
Page 1001 :
Income Under The Head Salary, Solution 17:, Computation of income under the head Salary, Basic Pay, (39,000 x 12), Dearness Allowance, (60% of 4,68,000), Children Education Allowance {Sec 10(14), Rule 2BB}, Working Note:, Received = `600 x 3 x 12 =, Exempt = `100 x 2 x 12 =, Taxable, =, Hostel Allowance {Sec 10(14), Rule 2BB}, Working Note:, Received = 1,000 x 1 x 12 =, Exempt = 300 x 1 x 12 =, Taxable, =, Entertainment Allowance, (200 x 12), Professional Tax, (175 x 12), Medical Allowance, (300 x 12), Gross Salary, Less: Standard Deduction u/s 16(ia), Less: 16(ii) Entertainment Allowance, Working Note:, Least of the following is deductible:, 1. `2,400, 2. `5,000, 3. 20% of `4,68,000 = `93,600, So, Deductible = `2,400, Less: 16(iii) Professional Tax, Income under the head Salary, Gross Total Income, Less: Deduction u/s 80C to 80U, Total Income, Computation of Tax Liability, Tax on `7,30,000 at slab rate, Add: HEC @ 4%, Tax Liability, Solution 18:, Computation of income under the head Salary, Basic Pay, [(43,750 x 3) + (44,500 x 9)], Working Note:, 01.07.2005 – 30.06.2006 =, 01.07.2006 – 30.06.2007 =, 01.07.2007 – 30.06.2008 =, 01.07.2008 – 30.06.2009 =, 01.07.2009 – 30.06.2010 =, 01.07.2010 – 30.06.2011 =, , 100, `, 4,68,000.00, 2,80,800.00, 19,200.00, `, 21,600, (2,400), 19,200, 8,400.00, `, 12,000, (3,600), 8,400, 2,400.00, 2,100.00, 3,600.00, 7,84,500.00, (50,000.00), (2,400.00), , (2,100.00), 7,30,000.00, 7,30,000.00, Nil, 7,30,000.00, 58,500.00, 2,340.00, 60,840.00, `, 5,31,750.00, `, 35,600 p.m., 35,900 p.m., 36,200 p.m., 36,500 p.m., 37,000 p.m., 37,500 p.m.
Page 1002 :
Income Under The Head Salary, 01.07.2011 – 30.06.2012 =, 01.07.2012 – 30.06.2013 =, 01.07.2013 – 30.06.2014 =, 01.07.2014 – 30.06.2015 =, 01.07.2015 – 30.06.2016 =, 01.07.2016 – 30.06.2017 =, 01.07.2017 – 30.06.2018 =, 01.07.2018 – 30.06.2019 =, 01.07.2019 – 30.06.2020 =, 01.07.2020 – 30.06.2021 =, 01.07.2021 – 30.06.2022 =, Dearness Allowance {11% of Basic Pay}, House Rent Allowance {Sec 10(13A), Rule 2A}, Working Note:, From 01.04.2021 To 30.06.2021, Least of the following is exempt:, 1. Nil, 2. 40% of retirement benefit salary = `52,500, (Retirement benefit salary = `1,31,250), 3. `9,000, Received = `9,000, Exempt =, Nil, Taxable = `9,000, From 01.07.2021 To 31.03.2022, Least of the following is exempt:, 1. Nil, 2. 40% of retirement benefit salary = `1,60,200, (Retirement benefit salary = 4,00,500), 3. `27,000, Received = `27,000, Exempt =, Nil, Taxable = `27,000, Total = 9,000 + 27,000 = `36,000, Entertainment Allowance, (600 x 12), Professional Tax, Conveyance Allowance, Working Note:, Conveyance incurred for official purpose is only, `100 p.m. Hence taxable is 400 x 12 = `4,800, Gross Salary, Less: Standard Deduction u/s 16(ia), Less: 16(iii) Professional Tax, Income under the head Salary, Gross Total Income, Less: Deduction u/s 80C to 80U, Total Income, Rounded off u/s 288A, Computation of Tax Liability, Tax on `5,88,240 at slab rate, Add: HEC @ 4%, Tax Liability, Rounded off u/s 288B, , 101, , 38,000 p.m., 38,500 p.m., 39,000 p.m., 39,500 p.m., 40,000 p.m., 40,750 p.m., 41,500 p.m., 42,250 p.m., 43,000 p.m., 43,750 p.m., 44,500 p.m., 58,492.50, 36,000.00, , 7,200.00, 2,400.00, 4,800.00, , 6,40,642.50, (50,000.00), (2,400.00), 5,88,242.50, 5,88,242.50, Nil, 5,88,242.50, 5,88,240.00, 30,148.00, 1,205.92, 31,353.92, 31,350.00
Page 1003 :
Income Under The Head Salary, Solution 19:, Computation of income under the head Salary, Basic Pay [(39,000 x 4) + (40,800 x 8)], Working Note:, 01.07.2014 – 30.06.2015 =, 01.07.2015 – 30.06.2016 =, 01.07.2016 – 30.06.2017 =, 01.07.2017 – 30.06.2018 =, 01.07.2018 – 30.06.2019 =, 01.07.2019 – 30.06.2020 =, 01.07.2020 – 30.06.2021 =, 01.07.2021 – 30.06.2022 =, Dearness allowance [(10,000 x 1) + (12,000 x 11)], Pension, Gross Salary, Less: Standard Deduction u/s 16(ia), Income under the head Salary, Gross Total Income, Less: Deduction u/s 80C to 80U, Total Income, Computation of Tax Liability, Tax on `5,84,400 at slab rate, Add: HEC @ 4%, Tax Liability, Rounded off u/s 288B, Solution 20:, Step 1., Previous Year 2021–22, Salary, Add: Arrears for previous year 2020-21, Gross Salary, Less: Standard deduction u/s 16(ia), Income under the head Salary, Tax before health & education cess, Add: HEC @ 4%, Tax Liability, Step 2., Previous Year 2021–22, Salary, Gross Salary, Less: Standard deduction u/s 16(ia), Income under the head Salary, Tax before health & education cess, Add: HEC @ 4%, Tax Liability, Step 3. Difference between Step 1 and Step 2, Step 4., Previous Year 2020–21, Salary, Add: Arrears, Gross Salary, Less: Standard deduction u/s 16(ia), Income under the head Salary, Tax before health & education cess, Add: HEC @ 4%, , 102, `, 4,82,400.00, , `, 29,500 p.m., 31,000 p.m., 32,600 p.m., 34,200 p.m., 35,800 p.m., 37,400 p.m., 39,000 p.m., 40,800 p.m., 1,42,000.00, 10,000.00, 6,34,400.00, (50,000.00), 5,84,400.00, 5,84,400.00, Nil, 5,84,400.00, 29,380.00, 1,175.20, 30,555.20, 30,560.00, `, 6,60,000, 60,000, 7,20,000, (50,000), 6,70,000, 46,500, 1,860, 48,360, 6,60,000, 6,60,000, (50,000), 6,10,000, 34,500, 1,380, 35,880, 12,480, 6,00,000, 60,000, 6,60,000, (50,000), 6,10,000, 34,500, 1,380
Page 1004 :
Income Under The Head Salary, Tax Liability, Step 5., Previous Year 2020–21, Salary, Gross Salary, Less: Standard deduction u/s 16(ia), Income under the head Salary, Tax before health & education cess, Add: HEC @ 4%, Tax Liability, Step 6. Difference between Step 4 and Step 5 ( 35,880-23,400), Step 7. Relief under section 89 Step 3 – Step 6 (12,480-12,480), Tax after adjusting relief u/s 89 [48,360 – Nil], Solution 21:, Computation of income under the head Salary, Basic Pay, (40,000 x 12), Dearness Allowance, (5,000 x 12), Commission, (1.5% of 20,00,000), Employer’s contribution to provident fund {Rule 6 of Part A of schedule IV}, Working Note:, Retirement benefit salary = `4,80,000 + 30,000 + 30,000 = `5,40,000, 12% of retirement benefit salary = `64,800, Employer contribution = `72,000, Allowed = 12% of retirement benefit salary = `64,800, Taxable = `7,200, Rent Free Accommodation {Sec 17(2)(i), Rule 3(1)}, Working Note:, 10% of rent free accommodation salary = `54,000, Rent free accommodation Salary = `5,40,000, Gross Salary, Less: Standard deduction u/s 16(ia), Income under the head Salary, Gross Total Income, Less: Deduction u/s 80C, Total Income, Computation of Tax Liability, Tax on `5,09,200 at slab rate, Add: HEC @ 4%, Tax Liability, Rounded off u/s 288B, Solution 22:, Computation of income under the head Salary, Basic Pay, (41,000 x 12), Dearness Allowance, (7,000 x 12), Commission, (3,000 x 12), Medical Allowance, , 103, 35,880, 6,00,000, 6,00,000, (50,000), 5,50,000, 22,500, 900, 23,400, 12,480, Nil, 48,360, `, 4,80,000.00, 60,000.00, 30,000.00, 7,200.00, , 54,000.00, , 6,31,200.00, (50,000.00), 5,81,200.00, 5,81,200.00, (72,000.00), 5,09,200.00, 14,340.00, 573.60, 14,913.60, 14,910.00, `, 4,92,000.00, 84,000.00, 36,000.00, 4,800.00
Page 1005 :
Income Under The Head Salary, (400 x 12), House Rent Allowance {Sec 10(13A), Rule 2A}, Working Note:, From April to August, 1. `25,000 – `20,850 = `4,150, 2. 40% of retirement benefit salary = `83,400, (Retirement benefit salary = (41,000 + 700) x 5 = 2,08,500), 3. `30,000, Received = `30,000, Exempt = (` 4,150), Taxable = `25,850, Rent Free Accommodation {Sec 17(2)(i), Rule 3(1)}, Working Note:, From November to March, Perquisite value = 7.5% of rent free accommodation salary = `16,912.50, Rent free accommodation Salary, = Basic Pay + Dearness Allowance + Commission + Medical Allowance, = `2,05,000 + `3,500 + `15,000 + `2,000 = `2,25,500, Perquisite value of furniture = 10% of (1,00,000 x 3/12) = `2,500, Taxable Amount = `16,912.50 + `2,500 = `19,412.50, Arrears of Salary, Advance Salary, Gross Salary, Less: Standard deduction u/s 16(ia), Income under the head Salary, Gross Total Income, Less: Deductions u/s 80C to 80U, Total Income, Rounded off u/s 288A, Computation of Tax Liability, Tax on `6,42,060 at slab rate, Add: HEC @ 4%, Tax Liability, Rounded off u/s 288B, Solution 23:, Computation of income under the head Salary, Basic Pay, (50,000 x 12), Dearness Allowance, (5,000 x 12), Commission, (3,500 x 12), Overtime Allowance, (1,000 x 11), House Rent Allowance {Sec 10(13A), Rule 2A}, Working Note:, From April to May, 1. `1,000 – `10,000 = Nil, 2. 50% of retirement benefit salary = `50,000, (Retirement benefit salary = 50,000 x 2 = `1,00,000), 3. `4,000, Received = `4,000, Exempt =, Nil, , 104, 25,850.00, , 19,412.50, , 10,000.00, 20,000.00, 6,92,062.50, (50,000.00), 6,42,062.50, 6,42,062.50, Nil, 6,42,062.50, 6,42,060.00, 40,912.00, 1,636.48, 42,548.48, 42,550.00, `, 6,00,000.00, 60,000.00, 42,000.00, 11,000.00, 4,000.00
Page 1006 :
Income Under The Head Salary, Taxable = `4,000, Rent Free Accommodation {Sec 17(2)(i), Rule 3(1)}, Working Note:, From September to March, 15% of rent free accommodation salary = `57,225, Rent free accommodation Salary, = Basic Pay + Commission + Overtime Allowance, = `3,50,000 + 24,500 + 7,000 =`3,81,500, Add: cost of furniture = `1,50,000 x 7/12 x 10% = `8,750, Perquisite value of furnished house = `57,225 + `8,750 = `65,975, Professional Tax, (200 x 12), Arrears of Salary {Sec 15}, Gross Salary, Less: Standard Deduction u/s 16(ia), Less: 16(iii) Professional Tax, Income under the head Salary, Gross Total Income, Less: Deductions u/s 80C to 80U, Total Income, Rounded off u/s 288A, Computation of Tax Liability, Tax on `7,67,980 at slab rate, Add: HEC @ 4%, Tax Liability, Rounded off u/s 288B, Solution 24:, Computation of income under the head Salary, Basic Pay, (45,000 x 12), Dearness Allowance, (5,000 x 12), Dearness Pay, (1,000 x 12), Bonus, (1,200 x 12), Rent Free Accommodation {Sec 17(2)(i), Rule 3(1)}, Working Note:, From April To December, 15% of Rent free accommodation Salary or rent paid whichever is less, Rent free accommodation Salary, = Basic Pay + Dearness Allowance + Dearness Pay + Bonus, = 4,05,000 + 13,500 + 900 + 10,800 = `4,30,200, 15% of rent free accommodation Salary = `64,530, Rent Paid = `1,200 x 9 = `10,800, (A) Perquisite value of unfurnished house = `10,800, From January To March, Rent free accommodation Salary of Delhi, = Basic Pay + Dearness Allowance + Dearness Pay + Bonus, = 1,35,000 + 4,500 + 300 + 3,600 = `1,43,400, 15% of Rent free accommodation Salary = `21,510, Rent paid = `3,600, , 105, 65,975.00, , 2,400.00, 35,000.00, 8,20,375.00, (50,000.00), (2,400.00), 7,67,975.00, 7,67,975.00, Nil, 7,67,975.00, 7,67,980.00, 66,096.00, 2,643.84, 68,739.84, 68,740.00, `, 5,40,000.00, 60,000.00, 12,000.00, 14,400.00, 14,400.00
Page 1007 :
Income Under The Head Salary, Perquisite value of Rent free accommodation of Delhi = `3,600, Rent free accommodation of Bombay, Rent free accommodation Salary, = Basic Pay + Dearness Allowance + Dearness Pay + Bonus, = 1,35,000 + 4,500 + 300 + 3,600 = `1,43,400, 15% of Rent free accommodation Salary = `21,510, Perquisite value of rent free accommodation of Bombay = `21,510, (B) Perquisite value of unfurnished house {least is in Delhi} = `3,600, Total Amount = A + B = `10,800 + 3,600 = `14,400, Arrears of Salary {Sec 15}, Advance of Salary {Sec 15}, Gross Salary, Less: Standard Deduction u/s 16(ia), Income under the head Salary, Gross Total Income, Less: Deductions u/s 80C to 80U, Total Income, Computation of Tax Liability, Tax on `6,33,800 at slab rate, Add: HEC @ 4%, Tax Liability, Rounded off u/s 288B, Solution 25:, Computation of income under the head Salary, Basic Pay, (40,000 x 12), House rent allowance {Sec 10(13A), Rule 2A}, Working Note:, From November to March, 1. `30,000 – `20,000 = `10,000, 2. 40% of retirement benefit salary = `80,000, (Retirement Benefit Salary = 40,000 x 5 = `2,00,000), 3. `25,000, Received = `25,000, Exempt = (`10,000), Taxable = `15,000, Advance Salary {Sec 15}, Rent Free Accommodation {Sec 17(2) (ii), Rule 3(1)}, Working Note:, 15% of Rent free accommodation salary or Rent paid whichever is less, Rent free accommodation salary = Basic Pay = `2,80,000, 15% of Rent free accommodation Salary = `42,000, Rent paid = `7,000 x 7 = `49,000, Perquisite value of unfurnished house = `42,000, Gross Salary, Less: Standard Deduction u/s 16(ia), Income under the head Salary, Gross Total Income, Less: Deductions u/s 80C to 80U, Total Income, , 106, , 32,000.00, 11,000.00, 6,83,800.00, (50,000.00), 6,33,800.00, 6,33,800.00, Nil, 6,33,800.00, 39,260.00, 1,570.40, 40,830.40, 40,830.00, `, 4,80,000, 15,000, , 20,000, 42,000, , 5,57,000, (50,000), 5,07,000, 5,07,000, Nil, 5,07,000
Page 1008 :
Income Under The Head Salary, , 107, , Computation of Tax Liability, Tax on `5,07,000 at slab rate, Add: HEC @ 4%, Tax liability, Rounded off u/s 288B, Solution 26:, (i) Computation of perquisite value of the loan, Months, Outstanding balance at the end, (in `), , 13,900, 556, 14,456, 14,460, , Amount of interest, (in `), Rate = 10% - 3% = 7%, 7,00,000 x 7% x 1/12 = 4,083.33, 7,00,000 x 7% x 1/12 = 4,083.33, 7,00,000 x 7% x 1/12 = 4,083.33, 6,75,000 x 7% x 1/12 = 3,937.50, 6,50,000 x 7% x 1/12 = 3,791.67, 6,25,000 x 7% x 1/12 = 3,645.83, 6,00,000 x 7% x 1/12 = 3,500.00, 27,124.99, , September, 7,00,000, October, 7,00,000, November, 7,00,000, December, 6,75,000, January, 6,50,000, February, 6,25,000, March, 6,00,000, Total, So, perquisite value of interest free loan = `27,124.99, (ii) Computation of perquisite value of the loan, `10,00,000 x 5.5% x 10/12, ` 9,45,000 x 5.5% x 1/12, Perquisite value of interest free loan, (iii) Computation of perquisite value of the loan, Perquisite value shall be Nil in case of loan for specified disease, (iv) Computation of perquisite value of the loan, `18,000 x 12% x 1/12, Total perquisite value, , Solution 27:, Computation of perquisite value of Furniture, Cost of the furniture, Less: Depreciation on straight line method @ 10% from 31.03.2018 to 30.03.2019, Less: Depreciation on straight line method @ 10% from 31.03.2019 to 30.03.2020, Less: Depreciation on straight line method @ 10% from 31.03.2020 to 30.03.2021, Written down value, Less: Amount paid by the assessee, Perquisite value of Furniture, Computation of perquisite value of Air-conditioner, Cost of the Air-conditioner, Less: Depreciation on straight line method @ 10% from 01.07.2020 to 30.06.2021, Written down value, Less: Amount paid by the assessee, Perquisite value of Air-conditioner, Computation of perquisite value of Video Camera, Cost of the Video Camera, Less: Depreciation on straight line method @ 10% from 11.07.2019 to 10.07.2020, Written down value, Less: Depreciation on straight line method @ 10% from 11.07.2020 to 10.07.2021, Written down value, Less: Amount paid by the assessee, Perquisite value of Video Camera, , `, 45,833.33, 4,331.25, 50,164.58, , 180.00, 77,469.57, `, 1,00,000, (10,000), (10,000), (10,000), 70,000, (40,000), 30,000, 45,000, (4,500), 40,500, (15,000), 25,500, 50,000, (5,000), 45,000, (5,000), 40,000, (20,000), 20,000
Page 1009 :
Income Under The Head Salary, Computation of perquisite value of Motor car, Cost of the motor, Less: Depreciation on reducing balance method @ 20% from 01.10.2017 to 30.09.2018, Written down value, Less: Depreciation on reducing balance method @ 20% from 01.10.2018 to 30.09.2019, Written down value, Less: Depreciation on reducing balance method @ 20% from 01.10.2019 to 30.09.2020, Written down value, Less: Depreciation on reducing balance method @ 20% from 01.10.2020 to 30.09.2021, Written down value, Less: Amount paid by the assessee, Perquisite value of motor car, Computation of perquisite value of Computer, Cost of the Computer, Less: Depreciation on reducing balance method @ 50% from 10.01.2019 to 09.01.2020, Written down value, Less: Depreciation on reducing balance method @ 50% from 10.01.2020 to 09.01.2021, Written down value, Less: Depreciation on reducing balance method @ 50% from 10.01.2021 to 09.01.2022, Written down value, Less: Amount paid by the assessee, Perquisite value of computer, Solution 28:, Computation of income under the head Salary, Basic Pay, (8,000 x 12), Medical Allowance, Medical Facility, Motor Car {Sec 17(2) (iii), Rule 3(2)}, Working Note:, Since basic pay is `96,000 so monetary income is more than `50,000 hence,, he is a specified employee, (1,800 x 12), Gross Salary, Less: Standard Deduction u/s 16(ia), Income under the head Salary, Income under the head House Property, Income under the head Business/Profession, Gross Total Income, Less: Deductions u/s 80C to 80U, Total Income, Computation of Tax Liability, Tax on `3,31,100 at slab rate, Less: Rebate u/s 87A, Tax Liability, , 108, 3,40,000, (68,000), 2,72,000, (54,400), 2,17,600, (43,520), 1,74,080, (34,816), 1,39,264, (1,50,000), Nil, 55,000, (27,500), 27,500, (13,750), 13,750, (6,875), 6,875, (25,000), Nil, `, 96,000.00, 10,000.00, 7,000.00, 21,600.00, , 1,34,600.00, (50,000.00), 84,600.00, 1,00,000.00, 1,50,000.00, 3,34,600.00, (3,500.00), 3,31,100.00, 4,055.00, (4,055.00), Nil
Page 1010 :
Income Under The Head Salary, , 109, , SOLUTIONS OF EXAMINATION QUESTIONS, JULY – 2021 (NEW COURSE), Answer 2(a):, An Indian citizen or a person of Indian origin who, being outside India, comes on a visit to India (and whose, total income, other than from foreign sources, does not exceed ₹ 15,00,000) would be resident in India only, if he or she stays in India for a period of 182 days or more during the previous year., Since Mrs. Rohini is a person of Indian origin who comes on a visit to India only for 60 days in the, P.Y.2021-22 and her income other than from foreign sources does not exceed ₹ 15,00,000, she is nonresident for the A.Y. 2022-23., A non-resident is chargeable to tax in respect of income received or deemed to be received in India and, income which accrues or arises or is deemed to accrue or arise to her in India. Accordingly, her total income, and tax liability would be determined in the following manner:, Computation of total income and tax liability of Mrs. Rohini for A.Y. 2022-23, Amt (₹), , Particulars, Salaries, Pension received from Russian Government [Not taxable, since it neither accrues or arises, , Nil, , in India nor is it received in India], Income from House Property, Annual Value [Rental Income from house property in New Delhi is taxable,, , 90,000, , since it is deemed to accrue or arise in India, as it accrues or arises from a, property situated in India], (27,000), , 63,000, , Long-term capital gains on sale of land at New Delhi [Taxable, since it is deemed to accrue, , 3,00,000, , Less: Deduction u/s 24(a) @ 30%, Capital Gains, or arise in India as it is arising from transfer of land situated in India], , Short-term capital gains on sale of shares of Indian listed companies in respect of which, , 60,000, , STT was paid [Taxable, since it is deemed to accrue or arise in India, as such income arises, on transfer of shares of Indian listed companies], Gross Total Income, , 4,23,000, , Less: Deduction under Chapter VI-A, Deduction under section 80C, , (63,000), , - Life insurance premium of ₹ 75,000 [Premium paid to Russian Life Insurance, Corporation allowable as deduction. However, the same has to be restricted to gross total, income excluding LTCG and STCG, as Chapter VI-A deductions are not allowable, against such income chargeable to tax u/s 112 and 111A,, respectively], Total Income, , 3,60,000
Page 1011 :
Income Under The Head Salary, , 110, , Computation of Tax Liability, Long-term capital gains taxable @20% u/s 112 [3,00,000 x 20%], , 60,000, , Short-term capital gains taxable @15% u/s 111A [60,000 x 15%], , 9,000, 69,000, , Add: Health and Education Cess @4%, , 2,760, , Tax Liability, , 71,760, , Note –, (i) Even if her total income exceeds ₹ 15 lakh, still, she would be non-resident since the minimum, period of stay required in the current year for being a resident is 120 days., (ii) The benefit of adjustment of unexhausted basic exemption limit against long-term capital gains, taxable u/s 112 and short-term capital gains taxable u/s 111A is not available in case of non-resident., Further, rebate u/s 87A is not allowable to a non-resident, even if his income does not exceed ₹ 5, lakh., (iii)It is assumed that such premium is paid for self or spouse or any child of Mrs. Rohini., , JAN – 2021 (NEW COURSE), Solution 3(a):, (a) In case of an Indian citizens leaving India for employment during the relevant previous year, the period, of their stay during that previous year for being treated as a resident of India must be 182 days or more., During the previous year 2021-22, Mr. Rajesh, an Indian citizen, was in India for 181 days only (i.e.,, 30+31+30+31+31+28 days). Thereafter, he left India for employment purposes., Since he does not satisfy the minimum criteria of 182 days, he is a non-resident for the A.Y. 2022-23., A non-resident is chargeable to tax in respect of income received or deemed to be received in India and, income which accrues or arises or is deemed to accrue or arise to him in India. Hence, salary for January to, March 2022, which was credited in his Dubai bank account for services rendered in Dubai, would not be, taxable in the hands of Mr. Rajesh., Computation of taxable income of Mr. Rajesh for A.Y. 2022-23, Particulars, Amount (₹), Salary, Salary from 1.4.2021 to 20.9.2021 [45,000 x 5 + 45,000 x 20/30], , 2,55,000, , Salary from 1.10.2021 to 31.12.2021 [90,000 x 3], , 2,70,000, , Gross Salary, Less: Standard deduction u/s 16(ia), , 5,25,000, (50,000), , Net Salary, Income from Other Sources, , 4,75,000, , Interest on fixed deposits, Interest on Savings account, , 10,500, 7,500, , 18,000
Page 1012 :
Income Under The Head Salary, Gross Total Income, Less: Deduction under Chapter VI-A, - Deduction under section 80C, LIC premium for self and spouse [LIC premium for mother is not allowed for, deduction], - Deduction under section 80TTA, [Interest on savings account with Mumbai bank], Total Income, , 111, 4,93,000, (25,000), (7,500), 4,60,500, , Working Notes –, 1. Cost of his air tickets to Dubai costing ₹ 1,50,000 funded by his sister is not taxable under section, 56(2)(x) in the hands of Mr. Rajesh, since “sister” is a relative., 2. Cost of initial stay at Dubai costing ₹ 40,000 funded by his friend is also not taxable under section, 56(2)(x), since the amount does not exceed ₹50,000., , NOV – 2019 (NEW COURSE), Solution 1:, Computation of taxable income and tax liability of Mrs. Mitul for the Assessment year 2022-23, Income under the head salary, Basic (13,000 x 12), Transportation allowance (2,000 x 12), Cost of treatment for son in True Care Hospitals (P) Ltd. [Exempt, since value of medical, , 1,56,000, 24,000, Nil, , treatment provided to an employee’s family member in any hospital maintained by the employer, is excluded from the definition of perquisite (clause (i) of proviso to section 17(2))], Gross salary, Less: standard deduction u/s 16(ia) [Actual salary or 50,000, whichever is less], , 1,80,000, (50,000), , Taxable salary, Less: Loss under the head house property, Income under the head salary, Income under the head house property, First floor (let out), Gross Annual Value [Rent received is taken as GAV = 10,000 p.m. x 6 months], Less: Municipal taxes paid by her in the P.Y.2020- 21 pertaining to let out portion, [(`5,000 + `5,000)/2], allowable since it is paid during the year, even if it relates to earlier years, Net Annual Value (NAV), Less: 30% of 55,000 standard deduction u/s 24(a), Less: Interest u/s 24(b) (60,000 + 40,000/2), Loss from first floor, Ground floor (Self occupied), Net Annual Value, Less: 30% standard deduction u/s 24(a), Less: Interest on housing loan for reconstruction u/s 24(b) (1,00,000/2) but limited to 30,000, , 1,30,000, (41,500), 88,500, , 60,000, (5,000), 55,000, (16,500), (50,000), (11,500), , Nil, Nil, (30,000)
Page 1013 :
Income Under The Head Salary, , 112, , Loss from first floor, Loss under the head house property, Income under the business profession, , (30,000), (41,500), , Net Profit, Less: Items of income to be treated separately under the respective head of income, Salary received taxable under other head, Rent from house property, , 8,59,000, , Dividend from foreign company, Less: Allowable expenditure, Depreciation on clinical equipments (1,00,000 x 15%), On additions during the year 25,000, no depreciation is allowable,, , (1,80,000), (78,000), (10,000), (15,000), Nil, , since payment was made in cash and hence, it will not form part of actual cost, Add: Items of expenditure not allowable while computing business income, Interest on housing loan for reconstruction of residential house, Interest on education loan for son, , 1,00,000, 26,000, , Conveyance expense in relation to her employment, Power & Fuel expenses incurred for power back up to tenant not deductible, Municipal taxes paid relating to residential house, Income under the head business profession, , 12,000, 6,000, 10,000, 7,30,000, , Income under the head other sources, Income from power back up facility, Power backup charges from tenant (3,000 x 6), Less: Actual expenditure incurred for providing power back up, Dividend from foreign companies, Income under the head other sources, Calculation of Total Income, Income under the head salary, Income under the head business profession, , 18,000, (6,000), 12,000, 10,000, 22,000, , 88,500, 7,30,000, , Income under the head other sources, Gross total income, Deduction under section 80C – Tuition fee paid for grand child is not allowable, Less: Deduction under section 80E - Interest on loan taken for higher education of, , 22,000, 8,40,500, Nil, , her son is deductible [principal repayment is not deductible], Total income, , (26,000), 8,14,500, , Computation of tax payable, Tax on 8,14,500 at slab rate, Add: HEC @ 4%, , 75,400, 3,016
Page 1014 :
Income Under The Head Salary, Tax liability, Rounded off u/s 288B, Note:, , 113, 78,416, 78,420, , Loss from house property can also be set-off against business income. In such a case, salary income would, be ` 1,30,000 and business income would be `6,88,500 (i.e., `7,30,000- `41,500). Gross total income, total, income and tax liability would remain the same., Alternative solution: Student can solve business income by taking 44ADA because Mrs. Mitul is a medical, practitioner and gross receipts is below 50 lakhs. In such case business income shall be 50% of gross, receipts i.e. 50% of 12,00,000 = 6,00,000., Computation of taxable income and tax liability of Mrs. Mitul for the Assessment year 2022-23, Income under the head salary, Basic (13,000 x 12), 1,56,000, Transportation allowance (2,000 x 12), 24,000, Cost of treatment for son in True Care Hospitals (P) Ltd. [Exempt, since value of medical, treatment provided to an employee’s family member in any hospital maintained by the employer, is excluded from the definition of perquisite (clause (i) of proviso to section 17(2))], Gross salary, , 1,80,000, , Less: standard deduction u/s 16(ia) [Actual salary or 50,000, whichever is less], Taxable salary, Less: Loss under the head house property, Income under the head salary, , (50,000), 1,30,000, (41,500), 88,500, , Income under the head house property, First floor (let out), Gross Annual Value [Rent received is taken as GAV = 10,000 p.m. x 6 months], Less: Municipal taxes paid by her in the P.Y.2019- 20 pertaining to let out portion, , Nil, , 60,000, , [(`5,000 + `5,000)/2], allowable since it is paid during the year, even if it relates to earlier years, Net Annual Value (NAV), Less: 30% of 55,000 standard deduction u/s 24(a), Less: Interest u/s 24(b) (60,000 + 40,000/2), , (5,000), 55,000, (16,500), (50,000), , Loss from first floor, Ground floor (Self occupied), Net Annual Value, Less: 30% standard deduction u/s 24(a), , (11,500), , Less: Interest on housing loan for reconstruction u/s 24(b) (1,00,000/2) but limited to 30,000, Loss from first floor, Loss under the head house property, Income under the business profession, , (30,000), (30,000), (41,500), , Consultation fees, Income under the head business profession, , Nil, Nil, , 12,00,000, 6,00,000
Page 1015 :
Income Under The Head Salary, , 114, , (50% of 12,00,000 u/s 44ADA), Income under the head other sources, Income from power back up facility, Power backup charges from tenant (3,000 x 6), Less: Actual expenditure incurred for providing power back up, Dividend from foreign companies, , 18,000, (6,000), 12,000, 10,000, , Income under the head other sources, , 22,000, , Calculation of Total Income, Income under the head salary, , 88,500, , Income under the head business profession, Income under the head other sources, Gross total income, Less: Deductions u/s 80E, , 6,00,000, 22,000, 7,10,500, (26,000), , Total income, , 6,84,500, , Computation of tax payable, Tax on 6,84,500 at slab rate, , 46,900, , Add: HEC @ 4%, Tax liability, Rounded off u/s 288B, Notes:, , 1,876, 48,776, 48,780, , Loss from house property can also be set-off against business income. In such a case, salary income would, be `1,30,000 and business income would be `5,58,500 (i.e., `6,00,000- `41,500). Gross total income, total, income and tax liability would remain the same., , MAY – 2019 (NEW COURSE), Solution 1:, Computation of Total Income and Net Tax Liability of Mr. Jagdish of A.Y. 22-23, Computation of Income under the head Salary, Basic Pay, (51,000 x 12), Dearness Allowance, (10,000 x 12), House Rent Allowance {Sec 10 (13A), Rule 2A}, Working Note:, Least of the following is exempt:, 1. `84,000 – 73,200 = `10,800, 2. 50% of retirement benefit salary = `3,66,000, (Retirement benefit salary (`51,000+10,000) x 12 = `7,32,000), 3. `48,000, Received = `48,000, , 6,12,000, 1,20,000, 37,200
Page 1016 :
Income Under The Head Salary, Exempt = (`10,800), Taxable = `37,200, Gross Salary, Less: Standard Deduction u/s 16(ia), Income under the head Salary, Income from house property, Gross Annual Value (GAV) (Rent receivable is taken as GAV in the, absence of other information) (46,000 × 9), Less: Municipal taxes paid, Net Annual Value (NAV), Less: Deductions u/s 24 (a) 30% of NAV, Less: Deductions u/s 24 (b) Interest, Income from House Property, , 115, , 7,69,200, (50,000), 7,19,200, , 4,14,000, (27,000), 3,87,000, (1,16,100), Nil, 2,70,900, , Income under the head business/profession, Earned from his share business, Less: Securities Transaction tax paid (section 36(1)(xv)), Income under the head business/profession, , 1,70,000, (30,000), 1,40,000, , Income under the head capital Gains, Full value of Consideration on Sale of Bonus shares, Less: Indexed cost of Acquisition, Long Term capital gains, , 2,00,000, Nil, 2,00,000, , Income under the head other sources, Dividend Income, Interest from saving bank account, Lottery Winning (21,000/70%), Income under the head other sources, , 13,00,000, 15,000, 30,000, 13,45,000, , Gross Total Income, Income under the head Salary, Income from House Property, Income under the head business/profession, Income under the head capital Gains, Income under the head other sources, Gross Total Income, Less: Deduction u/s 80C – PPF, Less: Deduction u/s 80D (19,000+12,000 = 31,000 but limited to 25,000), Less: Deduction u/s 80TTA, Total Income, , 7,19,200, 2,70,900, 1,40,000, 2,00,000, 13,45,000, 26,75,100, (1,50,000), (25,000), (10,000), 24,90,100, , Computation of Tax Liability, Tax on `30,000 @ 30% u/s 115BB, Tax on `2,00,000 @ 20% u/s 112, Tax on normal income `22,60,100, Tax before health & education cess, Add: HEC @ 4%, Tax Liability, Less: TDS, Tax Payable, , 9,000.00, 40,000.00, 4,90,530.00, 5,39,530.00, 21,581.20, 5,61,111.20, (9,000.00), 5,52,111.20
Page 1017 :
Income Under The Head Salary, Rounded off u/s 288B, , 5,52,110.00, , Solution 3(a):, Computation of Taxable Income of Mr. Madhvan for the A.Y. 2022-23, Computation of income under the head Salary, Basic salary, (30,000 x 12), Rent free accommodation {Sec 17(2)(i) Rule 3(1)}, Working Note:, 15% of rent free accommodation salary or rent paid whichever is less, Rent free accommodation salary = 3,60,000, 15% of `3,60,000 = `54,000, Rent Paid = `65,000, Value of unfurnished house, 54,000, Gross Salary, Less: Standard Deduction u/s 16(ia) [Actual salary or 50,000, whichever is less], Income under the head Salary, Computation of income under the head House Property, House 1, Gross Annual Value, Working Note:, (a) Fair Rent, (b) Municipal Value, (c) Expected Rent (higher of (a) or (b), (d) Rent Received/Receivable, GAV = Higher of (c) or (d), Less : Municipal Tax, Net Annual Value, Less: 30% of NAV u/s 24(a), Less: Interest on capital borrowed u/s 24(b), Income under the head House Property, House 2, Gross Annual Value, Working Note:, (a) Fair Rent, (b) Municipal Value, (c) Expected Rent (higher of (a) or (b), (d) Rent Received/Receivable, GAV = Higher of (c) or (d), Less: Municipal Tax, Net Annual Value, Less: 30% of NAV u/s 24(a), Less: Interest on capital borrowed u/s 24(b), Income under the head House Property, , 116, , `, 3,60,000, 54,000, , 4,14,000, (50,000), 3,64,000, , 75,000, `, 75,000, 74,000, 75,000, 65,000, 75,000, (18,000), 57,000, (17,100), Nil, 39,900, 2,85,000, `, 1,95,000, 1,90,000, 1,95,000, 2,85,000, 2,85,000, (70,000), 2,15,000, (64,500), (17,000), 1,33,500, , Income under the house property (1,33,500+39,900), , 1,73,400, , Gross Total Income, Less: Deduction u/s, Total Income, , 5,37,400, Nil, 5,37,400
Page 1018 :
Income Under The Head Salary, , 117, , Note:, 1. Purchase of rural agricultural land for a consideration less than stamp duty value [Not taxable under, section 56(2)(x), since rural agricultural land is not a capital asset., 2. Expenditure on repairs, insurance premium on building and ground rent are not allowable under the head, house property., Solution 4(a):, Computation of Total Income of Ms. Geeta for the A.Y. 2022-23, Income under the head salary, Income from salary (computed), Less: Loss from House Property maximum allowed 2,00,000, Income from salary, Income under the head House Property, House 1 (House in Delhi), GAV, (Actual receipt of rent shall be treated as GAV in the absence of other information), Less: Municipal Tax, Net Annual Value, Less: 30% of NAV u/s 24(a), Less: Interest on capital borrowed u/s 24(b) (7,50,000+75,000), Loss from House 1, House 2 (House in Jaipur), GAV, (Actual receipt of rent shall be treated as GAV in the absence of other information), Less: Municipal Tax, Net Annual Value, Less: 30% of NAV u/s 24(a), Less: Interest on capital borrowed u/s 24(b), Income from House 2, House 3 (House in Mumbai Self occupied), GAV, Less: Municipal Tax, Net Annual Value, Less: 30% of NAV u/s 24(a), Less: Interest on capital borrowed u/s 24(b), Interest on loan for purchase and repairs (to be restricted to 30,000, since loan, for purchase was taken prior to 1.4.1999), Loss from House 3, Loss under the head house property (2,24,000-4,75,000-30,000), As per section 71(3A), loss from house property to be set-off against salary income, to the extent of 2,00,000, Income under the head capital Gains, Long-term capital gains on sale of equity shares computed in accordance with section 112A, Less: Current year loss under the head business profession, Less: Set-off of brought forward short-term capital loss as per section 74 on sale of gold, Less: Brought forward loss Short term loss u/s 111A, LTCG 112A, Income under the head business/profession, Speculation profit (assumed as business income), , 41,20,000, (2,00,000), 39,20,000, 5,00,000, Nil, 5,00,000, (1,50,000), (8,25,000), (4,75,000), 3,20,000, Nil, 3,20,000, (96,000), Nil, 2,24,000, Nil, Nil, Nil, Nil, (30,000), (30,000), (2,81,000), , 8,95,000, (4,70,000), (2,75,000), (25,000), 1,25,000, 2,30,000
Page 1019 :
Income Under The Head Salary, , 118, , Salary received as partner of firm is taxable in her hands since the entire salary, was allowed as deduction in the hands of the firm, Less: Loss from textile business, Income under the head business/profession, Income under the head other sources, Interest on Fixed Deposit, Lottery Income, Income under the head other sources, Computation of Total Income, Income under the head salary, Income under the head capital gains, Income under the head house property, Income under the head business/profession, Income under the head other sources, Gross Total Income, Less: Deductions u/s 80C, – Repayment of Principal amount (Mumbai House), – LIC Paid, Total Income, , 50,000, (2,80,000), Nil, 73,000, 75,000, 1,48,000, 39,20,000, 1,25,000, Nil, Nil, 1,48,000, 41,93,000, , 50,000, 15,000, , Loss to be carried forward, Loss from House Property, , (65,000), 41,28,000, 81,000, , Note –, 1. Share of loss of 1,60,000 incurred by the firm in which she is partner cannot be set-off against salary, received as partner of firm or any other income, since loss from an exempt source cannot be set-off against, profit from a taxable source., 2. As per section 71(3A), loss from house property can be set-off against any other head of income to the, extent of 2,00,000 only. As per section 71B, balance loss not set-off can be carried forward to the next year, for set-off against income from house property of that year. Such loss can be carried forward for a maximum, of eight assessment years., 3. 2,50,000, for house property in Delhi, not allowable since loan is taken from a friend., 4. 75,000, for house properties in Mumbai and Delhi, not allowable since loan is taken for repairs of, properties., Alternate solution: Student can setoff business loss from other source income and balance from, capital gains, in such case also total income will remain same., , MAY – 2019 (OLD COURSE), Solution 1:, Computation of Total Income and Net Tax Liability of Mr. X of A.Y. 22-23, Computation of Income under the head Salary, Basic Pay (1,50,000 x 6), Dearness Allowance (55,000 x 6), Commission (35,000 x 6), Transport Allowance (5,000 x 6), Medical Reimbursement, Gratuity {Sec10(10)}, Working Note:, Least of the following is exempt:, 1. ` 20,00,000, , `, 9,00,000, 3,30,000, 2,10,000, 30,000, 20,000, 12,50,000
Page 1020 :
Income Under The Head Salary, , 119, , 2. ` 20,00,000, 3. ` 1/2 x (1,50,000 x 10)/10 x 10 = ` 7,50,000, Received = ` 20,00,000, Exempt = (` 7,50,000), Taxable = (` 12,50,000), Employer contribution from URPF (lumpsum), Interest on Employer’s contribution, Gross Salary, Less: Standard Deduction u/s 16(ia) [Actual salary or 50,000, whichever is less], Income under the head Salary, Less: Loss under the head house property, Income under the head Salary, , 13,20,000, 3,00,000, 43,60,000, (50,000), 43,10,000, (1,40,000), 41,70,000, , Income from house property, Gross Annual Value (GAV), Less: Municipal taxes paid, Net Annual Value (NAV), Less: Deductions u/s 24 (a) 30% of NAV, Less: Deductions u/s 24 (b) Interest, Current Period interest ` 1,10,000, Prior period installment ` 1,50,000 / 5 = ` 30,000, (assumed all the conditions has been satisfied), Loss from House Property, Income under the head business/profession, As per section 44AE, Small goods vehicle (3 x 7500 x 5), Heavy goods vehicle (3 x 15000 x 4), Income under the head business/profession, , Nil, Nil, Nil, Nil, (1,40,000), , (1,40,000), 1,12,500, 1,80,000, 2,92,500, , Income under the head other sources, Interest on Employee contribution, Gift from friend (section 56(2)(x)), Income under the head other sources, Gross Total Income, Income under the head salary, Income under the head business/profession, Income under the head other sources, Gross Total Income, Less: Deduction u/s 80C – LIC, Less: Deduction u/s 80C – Repayment of Housing Loan (2,20,000-1,10,000), Less: Deduction u/s 80CCC – LIC Pension fund, Less: Deduction u/s 80D (40,000 but limited to 25,000), (assumed payment made through cheque), Total Income, Computation of Tax Liability, Tax on normal income ` 52,47,500, Add: Surcharge @ 10%, Tax before health & education cess, Add: HEC @ 4%, Tax Liability, Round off u/s 288B, , 3,60,000, 6,00,000, 9,60,000, 41,70,000, 2,92,500, 9,60,000, 54,22,500, (20,000), (1,10,000), (25,000), 1,55,000, , (1,50,000), (25,000), 52,47,500, 13,86,750.00, 1,38,675.00, 15,25,425.00, 61,017.00, 15,86,442.00, 15,86,440.00
Page 1021 :
Income Under The Head Salary, , 120, , Note:, 1. As per section 10 (34A), any income arising to an assessee being a shareholder on account of buy, back of shares by the company shall be exempt from income tax., 2. Dearness allowance is not forming part of salary hence not taken in salary computation., Solution 5(a): Computation of Taxable salary of Mr. Janakaraj, `, Basic Salary, 8,80,000.00, (70,000 x 8) + (80,000 x 4), Dearness Allowance, 4,40,000.00, (50% x 8,80,000), Bonus, 1,05,000.00, (70,000 + 35,000), Contribution to recognized provident fund, 52,800.00, (8,80,000 x 6%) (in excess of 12% shall be taxable), (Salary includes only basic salary, since dearness allowance, in this case, does, not form part of salary for retirement benefits), Professional Tax paid by the employer Section 17 (2)(iv), 3,000.00, (Perquisite includes any sum paid by the employer in respect of any obligation, which would have been payable by the employee), Facility of Laptop/computer (Section 17 (2)(viii)/Rule 3(7)(vii)), Nil, (Facility of laptop is an exempt perquisite, whether used for official or personal purpose or both), Leave Travel concession (section 10(5)/Rule 2B), Nil, Gross Salary, 14,80,800.00, Less: Standard Deduction u/s 16(ia), (50,000.00), Less: Deduction of professional tax u/s 16(iii), (6,000.00), Income under the head Salary, 14,24,800.00, Note:, Mr. Janakaraj can avail exemption on the entire amount of 45,000 reimbursed by the employer towards, leave travel concession since the leave travel concession was availed for himself, wife and three children and, the journey was undertaken by economy class airfare. The restriction imposed for two children is not, applicable in case of multiple birth which take place after the first child., Answer 5(b): Partly taxable, Any allowance granted to an employee working in a transport system to meet his personal expenditure, during his duty is exempt provided he is not in receipt of any daily allowance. The exemption is 70% of, such allowance (i.e., `8,400 per month being, 70% of `12,000) or `10,000 per month, whichever is less., Hence, 1,00,800 (i.e., `8,400 x 12) is exempt. Balance `43,200 (`1,44,000 – `1,00,800) is taxable in the, hands of Mr. Ram., , NOV – 2018 (OLD COURSE), Solution 3(a):, Basic Pay (32,000 x 12), Dearness Allowance (3,200 x 12), Medical Allowance (section 17(1)), Medical Facility taxable, Laptop facility (section 17(2)(viii) /Rule 3(7)(vii)), Servant reimbursement (1,000 x 12) (section 17(2)(iii) /Rule 3(3)), Equity Shares (500 x (` 250-` 150)) (section 17(2)(vi)), Professional Tax, Gross Salary, Less: Deduction u/s 16(ia), Less: Deduction u/s 16(iii), Income under the head Salary, , `, 3,84,000, 38,400, 20,000, 3,000, Nil, 12,000, 50,000, 1,800, 5,09,200, (50,000), (2,500), 4,56,700
Page 1022 :
Income Under The Head Salary, Gross Total Income, Less: Deduction u/s 80C to 80U, Total Income, , 121, 4,56,700, Nil, 4,56,700, , Notes:, 1. Bonus received on 01-10-2021 was for the previous year 2020-21 hence taxable in the year 2020-21., 2. Reimbursement of medical bill of father in law is fully taxable., 3. Domestic servant was employed by the employee and the salary of such domestic servant was, paid/reimbursed by the employer. It is taxable as perquisite for all categories of employees., 4. As per section 17(2)(vi), equity shares received at less than market price then difference of market, price and amount recovered shall be taxable., , MAY – 2018 (NEW COURSE), Solution 4:, Determination of Residential Status, As per section 6(1), Stay in India is 182 or more during the previous year, hence he is resident and his stay, during the 7 years is more than 729 days and he is resident in India for more than a year in 10 years, preceding the relevant previous year and not able to comply even a single condition of section 6(6)(a), hence, he is ROR., Computation of Income under the head salary for the A.Y. 2022-23, `, Basic Salary (50,000 x 12), 6,00,000.00, Rent free Accommodation {Sec 17(2)(i), Rule 3(1)}, 94,680.00, Working Note:, 15% of rent free accommodation salary or rent paid whichever is less, Rent free accommodation Salary, = Basic Pay + Children Education allowance + Hostel Allowance, = 6,00,000 + 31,200, = `6,31,200, 15% of rent free accommodation Salary = `94,680, Rent Paid = `15,000 x 12 = `1,80,000, Perquisite value of unfurnished house = `94,680, Guest House facility (official purpose), Exempt, Free lunch Facility Rule 3(7) (iii), Total Incurred `48,000, Total Stay 184 days, Value of each lunch `260.87, Less exemption, `50.00, Taxable, `210.87, Total Taxable `210.87 x 184 =, 38,800.08, Motor car facility (used for personal and official) (2,400 x 6) Section 17(2) (iii)/Rule 3(2), 14,400.00, Motor car facility (used for personal purpose only) Section 17(2) (iii)/ Rule 3(2), Taxable value (8,00,000 x 10% x 6/12), 40,000.00, Expenditure on car (5,000 x 6), 30,000.00, Education Facility Section 17(2) (iii)/Rule 3(5), Elder son (12,000-12,000), Nil, Younger Son Children Education allowance and Hostel allowance combined, 36,000, Less: Exempt Children Education allowance (100 x 12) Section 10(14)/Rule 2BB (1,200), Less: Exempt Hostel allowance (300 x 12) Section 10(14)/ Rule 2BB, (3,600), 31,200.00, Accident Insurance Policy section 17(2)(v), Nil, Life Insurance Policy section 17(2)(v), 10,000.00, Gross Salary, 8,59,080.08, Less: Standard deduction u/s 16(ia), (50,000.00)
Page 1023 :
Income Under The Head Salary, Income under the head salary, , 122, 8,09,080.08, , MAY – 2018 (OLD COURSE), Solution 1:, Basic Pay (25,000 x 12), 3,00,000, Dearness allowance (12,500 x 12), 1,50,000, Employer’s contribution to recognised provident fund in excess of 12% of retirement benefit salary 15,000, Working Note:, RBS = 3,00,000 + 75,000, = 3,75,000, 12% of RBS = 45,000, Employer’s contribution = 3,00,000 x 20% = 60,000, 60,000 – 45,000 = 15,000, Interest credited in recognised provident fund in excess of 9.5%, 3,667, (15,000/15% x 5.5%) x 2/3, Gross Salary, 4,68,667, Less: Standard deduction u/s 16(ia), (50,000), Income under the head salary, 4,18,667, Income under the head other sources, Interest credited in recognised provident fund in excess of 9.5%, (15,000/15% x 5.5%) x 1/3, Interest from axis bank saving account, Interest from debentures, Income under the head other sources, Income from house property, Arrears of rent, Less: 30% u/s 25A, Income from house property, Income under the head Capital Gains, Gold chain and diamond ring lost, Full Value of Consideration, Less: Indexed cost of acquisition (1,17,000/117 x 317), Long term capital gains, Gross Total Income, Income under the head Salary, Income under the head other sources, Income from house property, Long term capital gains, Gross Total Income, Less: Deduction u/s 80C, Employee contribution to RPF, 30,000, Sukanya Samridhi account, 60,000, Less: Deduction u/s 80D for major daughter, Less: Deduction u/s 80CCD Contribution to Pension fund (1,196 x 12), Less: Deduction u/s 80TTA, Total Income, Rounded off u/s 288A, Notes:, 1. Interest on debentures is presumed to be gross receipts., , 1,833, 10,000, 12,040, 23,873, 69,000, (20,700), 48,300, 3,20,000.00, (3,17,000.00), 3,000.00, 4,18,667, 23,873, 48,300, 3,000, 4,93,840, (90,000), (12,500), (14,352), (10,000), 3,66,988, 3,66,990
Page 1024 :
Income Under The Head Salary, , 123, , 2. Residential house is sold in 2018 hence tax on such income is already given in 2018 hence no, treatment is done in current year., 3. It is assumed in section 80D that Daughter is dependent., , NOV – 2017, Answer 2(b):, As per section 80CCD, If an assessee has received any amount from the accumulated balance under, National Pension Scheme, the amount so received is taxable but w.e.f. assessment year 18-19 some, exemption has been granted u/s 10(12A) and is as given below:, Any payment from the National Pension System Trust to an employee on closure of his account or on his, opting out of the pension scheme referred to in section 80CCD shall be exempt to the extent of 60% of the, total amount payable to him at the time of such closure or his opting out of the scheme., Further as per section 80CCD, Lumpsum amount received by the nominee on the death of the assessee shall, be fully exempt from Income Tax., Gross Value, Less: Exempt u/s 10(12A) (60% x 2,00,000), Taxable Value, Solution 3:, Computation of Salary chargeable to tax of Mrs. Jaya for A.Y. 2022-23, Basic Salary (65,000 x 12), Dearness Allowance (22,000 x 12), Bonus (17,000 x 12), Health Insurance by Employer proviso to Section 17(2), Gift Voucher (Rule 3(7)(iv)) (8,000-5,000), Accommodation at concessional rent {Sec 17(2)(ii) Rule 3(1)}, Working Note:, `, In the given question accommodation is given on 01-04-21 but occupied, on 01-11-2021, in such cases it will be taxable from 01-11-21 and taxable, value shall be as given below:, Rent paid or 15% of rent free accommodation salary whichever is less, Rent free accommodation Salary, 4,43,000, (65,000 +6,600+17,000) x 5, 15% of rent free accommodation salary, 66,450, Rent paid (12,000 x 5), 60,000, Value of unfurnished house, 60,000, Less: Amount recovered from the employee (4,800 x 5), (24,000), Perquisite value of accommodation at concessional rent, 36,000, Employer’s contribution to provident fund {Part A of schedule IV}, Working Note:, Retirement benefit salary = `65,000 + (30% of 22,000) x 12 =, `8,59,200, Taxable Amount of Employer contribution, `51,552, (8,59,200 x 6%), Perquisite value of use of motor car (section 17 (2) (iii)/Rule 3(2)), (1,800 x 5), Gross Salary, Less: Standard Deduction u/s 16(ia), Gross Total Income, Less: Deduction u/s 80C (8,59,200 x 18%) but maximum ` 1,50,000, , 2,00,000, (1,20,000), 80,000, , `, 7,80,000.00, 2,64,000.00, 2,04,000.00, Nil, 3,000.00, 36,000.00, , 51,552.00, , 9,000.00, 13,47,552.00, (50,000.00), 12,97,552.00, (1,50,000.00)
Page 1025 :
Income Under The Head Salary, Total Income, Rounded off u/s 288A, Computation of Tax Liability, Tax on `11,47,550 at slab rate, Add: HEC @ 4%, Tax Liability, Rounded off u/s 288B, , 124, 11,47,552.00, 11,47,550.00, 1,56,765.00, 6,270.60, 1,63,035.60, 1,63,040.00, , MAY – 2017, Solution 3(a):, Computation of Salary chargeable to tax of Mr. Nambi for A.Y. 2022-23, Basic Salary, Dearness Allowance, Commission, Entertainment allowance, Professional Tax paid by the employer Section 17 (2)(iv), Facility of Laptop/computer (Rule 3(7)(vii)), Health Insurance by Employer proviso to Section 17(2), Gift Voucher (Rule 3(7)(iv)) (12,000-5,000), Life Insurance paid by Employer (section 17 (2) (v)), Perquisite value of use of motor car (section 17 (2) (iii)/Rule 3(2)), (1,800 x 12), Annual Credit card fees (Rule 3 (7)(v)), Gross Salary, Less: Standard Deduction u/s 16(ia), Less: Deduction of professional tax u/s 16(iii), Income under the head Salary, , `, 6,00,000.00, 3,20,000.00, 50,000.00, 7,500.00, 3,500.00, Nil, Nil, 7,000.00, 34,000.00, 21,600.00, 2,000.00, 10,45,600.00, (50,000.00), (7,000.00), 9,88,600.00, , Notes:, 1. As per section 16(ii) Deduction of entertainment allowance is allowed to Government employees and not, to other employees., 2. Professional tax paid by employer shall be added to the gross salary of the employee then deduction u/s, 16 (iii) shall be allowed for professional tax., 3. Health insurance premium paid by the employer to effect an insurance on the health of the employee is, fully exempt., 4. As per Rule 3 (7)(iv), The value of any gift or voucher or token in lieu of which gift is received by the, employee or by member of his household upto `5,000 in aggregate during the previous year is exempt. In, the given case Gift voucher received on birthday exceeds `5000. Hence, excess amount is taxable., 5. As per section 17 (2) (v), Life Insurance premium of employee paid by employer shall be included in his, income as it is a perquisite for an employee., 6. Credit card facility is exempt only if it is exclusively for official purpose and employer has maintained, complete records., , NOV – 2016, Solution 3(a):, Mr. X is a Non- Resident as he did not come to India during the current financial year., Computation of Gross Total Income of Mr. X for the Assessment year 2022-23, Income under the head Salary, Salary from Govt. of India, Foreign Allowances from Govt. of India (exempt), Gross Salary, Less: Standard Deduction u/s 16(ia), , 7,20,000, Nil, 7,20,000, (50,000)
Page 1026 :
Income Under The Head Salary, , 125, , Income under the head Salary, 6,70,000, Income under the head other sources, Interest accrued on National Saving Certificate during the year 2021-22, 45,000, Income under the head other sources, 45,000, Gross Total Income, 7,15,000, Note:, 1. Rent from a house situated at London, received in London is a Income accruing arising outside India, and received outside India hence not taxable in case of Non- Resident., 2. Salary Received from Govt. of India is taxable in India as Income is accruing arising from India., 3. Foreign allowances received from Govt. of India are exempt from Tax as per section 10(7)., 4. Interest accrued on NSC is deemed to accrue arise in India and taxable in India., Solution 5(a):, As per Section 10(5), Leave Travel concession shall be allowed to employee and his family. Exemption, shall be allowed for the expenditure incurred during the trip., Family shall include spouse and children of the employee however exemption shall be allowed for, maximum of 2 children but in case of multiple birth after the birth of one child, exemption is allowed for all, the children., (i) As per the above provision Exemption shall be allowed for all the children as the case is of multiple, birth after the birth of one child. Hence cost of travel of all shall be exempt., (ii) As per the above provision Exemption shall be allowed for only 2 children., Hence Exemption shall be `80,000 – `30,000/3 = `70,000., Taxable Amount shall be `80,000 - `70,000 = `10,000., , MAY – 2016, Solution 1(a):, Computation of Total Income of Mr. Vinod Kumar for the A.Y. 2022-23, Income from Salaries, Pension, Gross Salary, Less: Standard Deduction u/s 16(ia), Income under the head Salary, Profits and gains of business or profession, Gross Receipts, Less: Allowable expenses, Rent, Salary to P.A., Stenographer’s salary, Business Development expenditure, Conveyance (3/4th of `3,00,000 allowed), Income under the head Profits and gains of business or profession, Income from Other Sources, Interest from Post Office Saving Bank, Less: Exempt u/s 10(15), Interest income from fixed deposits, Income under the head Other Sources, Income under the head Capital Gains, Short Term Capital Gains u/s 111A, Long Term Capital Gains u/s 112A, Income under the head Capital Gains, Gross Total Income, Less: Deduction under section 80C, , `, 6,52,000.00, 6,52,000.00, (50,000.00), 6,02,000.00, 12,60,000.00, (1,44,000.00), (1,20,000.00), (1,00,000.00), (91,000.00), (2,25,000.00), 5,80,000.00, 18,000.00, (3,500.00), 14,500.00, 1,60,000.00, 1,74,500.00, 65,000.00, 1,24,000.00, 89,000.00, 15,45,500.00
Page 1027 :
Income Under The Head Salary, , 126, , - PPF, 1,10,000.00, - LIC (lower of 10% of 5,00,000 or 60,000), 50,000.00, Restricted to 1,50,000, (1,50,000.00), Less: Deduction under section 80D (27,000+ 5,000= 32,000), (32,000.00), Less: Deduction under section 80TTB, (50,000.00), Total Income, 13,13,500.00, Note: 1. Deduction under section 80G is not allowed because payment is made by cash., 2. Business development expenditure are assumed as revenue in nature and wholly incurred for the purpose, of business/profession., , NOV – 2015, Solution 1(a):, Computation of total income of Mrs. X for the A.Y. 2022-23, Income from Salaries, Salary including dearness allowance, House rent allowance {Sec 10(13A) Rule 2A}, Working Note:, Least of the following is exempt:, 1. `2,16,000-`72,000 =1,44,000, 2. 40% of retirement benefit salary = `2,88,000, (Retirement benefit salary = `7,20,000), 3. `1,80,000, Received = `1,80,000, Exempt = (1,44,000), Taxable = `36,000, Children Education Allowance {Sec 10(14), Rule 2BB}, Working Note:, Received = `1500 x 12 =, Exempt = `100 x 12 x 2 =, Taxable, =, Gross Salary, Less: Standard Deduction u/s 16(ia), Income under the head Salary, Profits and gains of business or profession, Income from truck business (11,000 x 8), (Taken 11,000 P.m. Because assessee is declaring higher income), Income under the head Profits and gains of business or profession, Income from Other Sources, Interest from Post Office Saving Bank (8,500-3,500), Interest income from company deposits [`25,000 – 5,000], Withdrawn from National Saving Scheme (Note), Income under the head Other Sources, Gross Total Income, Less: Deduction under section 80C - Tuition Fees, Less: Deduction under section 80D, Less: Deduction under section 80TTA, Total Income, Computation of Tax Liability, Tax on `7,49,600 at slab rate, Add: HEC @ 4%, Tax Liability, , `, 7,20,000.00, 36,000.00, , 15,600.00, `, 18,000, (2,400), 15,600, 7,71,600.00, (50,000.00), 7,21,600.00, 88,000.00, 88,000.00, 5,000.00, 20,000.00, 25,000.00, 50,000.00, 8,59,600.00, (80,000.00), (25,000.00), (5,000.00), 7,49,600.00, 62,420.00, 2,496.80, 64,916.80
Page 1028 :
Income Under The Head Salary, Rounded off u/s 288B, , 127, 64,920.00, , Note: The taxability of withdrawals from National Savings Scheme depends on whether the withdrawal was, from National Savings Scheme, 1992 or National Savings Scheme, 1987. In the above solution, the interest, component alone has been brought to tax on the assumption that the withdrawal is from National Savings, Scheme, 1992. However, if the withdrawal is in respect of National Savings Scheme, 1987, both the interest, and the principal would be chargeable to tax and in such case, amount of `35,000 shall be taxable., , MAY – 2015, Solution 4(a):, Computation of Gross Total Income of Mr. Dinesh for the A.Y. 2022-23, Income under the head Salary, Salary including dearness allowance, Conveyance allowance (see Note 1 below), Bonus, Value of perquisites:, (i) Salary of servant, (ii) Free gas, electricity and water, (iii)Education facility (2,500 – 1,000) x 12 (see Note 2 below), Gross Salary, Less: Standard Deduction u/s 16(ia), Income under the head Salary, Income from house property, Letout house, Gross Annual Value (GAV) (Rent receivable is taken as GAV in the, Absence of other information) (35,000 × 12), Less: Municipal taxes paid, Net Annual Value (NAV), Less: Deductions under section 24, (i) 30% of NAV, (ii) Interest on loan, Income from House 1, Self occupied house, Gross Annual Value (GAV), Less: Municipal taxes paid, Net Annual Value (NAV), Less: Deductions under section 24, (i) 30% of NAV, (ii) Interest on loan (18,50,000 x 11 % but restricted to `2,00,000), Loss from House 2, Income under the head house property, Income from Other Sources, (i) Income on account of interest earned from advancing money, gifted to his minor son is includible in the hands of Mr. Dinesh as per section 64(1A), Less: Exempt under section 10(32), (ii) Interest income earned from advancing money gifted to wife has, to be clubbed with the income of the assessee as per section 64(1), (iii) Gift received from two friends (taxable under section 56(2) as the aggregate amount, received during the year exceeds `50,000), Income under the head other sources, Gross Total Income, , `, 6,50,000, Nil, 50,000, 48,000, 82,000, 18,000, 8,48,000, (50,000), 7,98,000, , 4,20,000, (12,000), 4,08,000, (1,22,400), Nil, 2,85,600, Nil, Nil, Nil, Nil, (2,00,000), (2,00,000), 85,600, 7,200, (1,500), 5,700, 7,200, 90,000, 1,02,900, 9,86,500
Page 1029 :
Income Under The Head Salary, , 128, , Note:, 1. The allowances given by the employer for official purpose are called official allowance and are exempt, from income tax however saving is taxable and assuming conveyance allowance entire for official purpose., It is exempt., 2. If education facility is provided in employer’s own institution, it is taxable in case of members of, household but in case of children, it is exempt upto `1,000 p.m. per child. Value for this purpose shall be the, cost of similar type of education in a similar type of institution in the same locality., , NOV – 2014, Solution 4(a):, Computation of income under the head Salary, Basic Pay, (80,000 x 9), Bonus, House rent allowance {Sec 10(13A), Rule 2A}, Working Note:, 1. `90,000 – `72,000 = `18,000, 2. 50% of retirement benefit salary = `3,60,000, (Retirement Benefit Salary = 80,000 x 9 = `7,20,000), 3. `1,35,000, Received = `1,35,000, Exempt = (`18,000), Taxable = `1,17,000, Employer’s contribution to provident fund {Part A of schedule IV}, Working Note:, Retirement benefit salary = `7,20,000, Employer contribution = `1,10,000, Allowed = 12% of retirement benefit salary = `86,400, Taxable = `23,600, Gratuity {Sec 10(10)}, Working Note:, Least of the following is exempt:, 1. `20,51,640, 2. `20,00,000, 3. 15/26 x 80,000 x 34 = `15,69,230.77, Received = `20,51,640.00, Exempt = (`15,69,230.77), Taxable = `4,82,409.23, Uncommuted Pension {Sec 17(1)}, Working Note:, `, January, 8,000 x 1 =, 8,000.00, February & March, 8,000 x 25% x 2 =, 4,000.00, Total =, 12,000.00, Commuted Pension {Sec 10(10A)}, Working Note:, `, Received, =, 4,50,000, Exempt = 4,50,000 / 75% x 100% x 1/3 =, (2,00,000), Taxable, =, 2,50,000, Gross Salary, , `, 7,20,000, 36,000, 1,17,000, , 23,600, , 4,82,409.23, , 12,000.00, , 2,50,000.00, , 16,41,009.23
Page 1030 :
Income Under The Head Salary, , 129, , Less: Standard Deduction u/s 16(ia), (50,000.00), Less: Deduction u/s 16(iii) - Professional Tax, (2,000.00), Income under the head Salary, 15,89,009.23, Note: `6,00,000 as accumulated balance of Recognised Provident fund received from employer is exempt as, Mr. X has rendered continuous service of more than 5 Years with the employer., , MAY – 2014, Solution 1(a): Computation of total income of Mr. X for the A.Y. 2022-23, Particulars, `, Income from Salaries, Salary including dearness allowance, 5,00,000, Bonus, 15,000, Salary of servant provided by employer Sec 17(2)(iii)Rule 3(3), 12,000, Free gas, electricity and water provided by employer Sec 17(2)(iii) Rule 3(4), 14,500, Laptop Sec 17(2)(viii) Rule 3(7)(vii), Nil, Gross Salary, 5,41,500, Less: Standard Deduction u/s 16(ia), (50,000), Income under the head Salary, 4,91,500, Income from house property, Gross Annual Value (GAV) (` 5,500 × 12), 66,000, Less: Municipal taxes paid, (4,500), Net Annual Value (NAV), 61,500, Less: Deductions, 30% of NAV 24(a), (18,450), Less: Interest on loan from State Bank of India@15% of ` 1,60,000 u/s 24(b), (24,000), Income under the head House Property, 19,050, Profits and gains of business or profession, Income from share speculation business, 15,000, Less: Loss from cotton speculation business, (20,200), Net loss from speculation business to be carried forward, (5,200), Income from Other Sources, Income of minor child (30,000 x 15%) – 1,500, 3,000, Interest income of spouse [` 50,000 x 15%], 7,500, Gift (` 25,000 x 4), 1,00,000, Income under the head Other Sources, 1,10,500, Gross Total Income, 6,21,050, Less: Deduction under section 80C, - Public Provident Fund, (10,500), - Unit Linked Insurance Plan, (6,000), - Tax saver deposit with Nationalized bank in the name of his married son does not, qualify for deduction under section 80C. The deposit has to be in Mr. X’s own name., Nil, - Life Insurance Premium [paid to insure life of married daughter is allowable], [In respect of policies taken on or after 01.04.2012, the deduction is restricted to 10%, (20,000), of minimum capital sum assured. Hence, in this case, deduction is restricted to 10% of ` 2,00,000], Total Income, 5,84,550, Notes:, (1) No separate deduction is available for insurance of `1,275, while computing income under the head, “Income from house property”., (2) It is assumed that `1,60,000 is the loan outstanding at the beginning of the year and there is no principal, repayment of housing loan during the year qualifying for deduction under section 80C. Interest under section, 24 has, accordingly, been calculated at the rate 15% of ` 1,60,000., (3) It is assumed that Mr. X’s total income, before including minor’s income, is higher than that of his, spouse.
Page 1031 :
Income Under The Head Salary, , 130, , NOV – 2013, Solution 3(a):, Computation of taxable salary of Mr. X for A.Y. 2022-23, Particulars, `, 3,02,500, Basic pay [(` 25,000×11) + (` 27,500×1)] = ` 2,75,000 + ` 27,500, 45,375, Dearness allowance [15% of basic pay], 41,250, Bonus [` 27,500 × 1.5], Employer’s contribution to Recognized Provident Fund in excess of 12%, 20,873, (18% - 12% = 6% of ` 3,47,875), 12,000, Telephone allowance, Rent-free accommodation Sec 17(2)(i)/Rule 3(1), 60,169, 15% of salary (3,02,500 + 45,375 + 41,250 +12,000) or `1,80,000 whichever is less., 24,000, Reimbursement of salary of housekeeper [` 2,000 × 12] Sec 17(2)(iv), Gift voucher Sec 17(2)(viii)Rule 3(7)(iv), 15,000, Motor car owned and driven by employee, Sec 17(2)(iii) Rule 3(2), [` 36,600 – ` 21,600 (i.e., ` 1,800 × 12)], Value of free lunch facility Sec 17(2)(viii) Rule 3(7)(iii), (Presuming that value per lunch was upto `50), 5,21,167, Gross Salary, (50,000), Less: Standard Deduction u/s 16(ia), 4,71,167, Income under the head Salary, 1. Medical insurance premium paid by the employer to effect an insurance on the health of the employee is, fully exempt., , NOV – 2012, Answer 6(a):, (i) Bonus shares received by equity shareholders is not taxable. Bonus share is deemed dividend in the hands, of preference shareholder only and it is covered under the head Other Sources, (ii) Such loan is deemed dividend in the hand of shareholder u/s 2(22)(e). He is liable to tax thereon under, the head other sources., (iii) Fully taxable under the head salary., (iv) `60,000 taxable as gift under the head other Sources., (v) Perquisites under section 28 taxable as PGBP, (vi) Taxable as income under the head Business/Profession., (vii) Employer’s contribution & interest is taxable as salary. Employee’s contribution is not taxable., However, interest on his contribution is taxable as Income from other Sources., , MAY – 2012, Answer 1(a):, Computation of Total Income of Ms. X for the A.Y. 2022-23, Income under the head Salary, Income under the head Other Sources, (Bank Interest), Gross Total Income, Less: Deduction u/s 80C – Contribution to Recognized Provident Fund, Less: Deduction u/s 80D – Health Insurance Premium, Less: Deduction u/s 80DD – Medical expenditure for dependent sister with disability, Less: Deduction u/s 80TTA (10,000 or 15,000 whichever is less), Total Income, Note: Tax on non-monetary perquisite paid by employer is exempt u/s 10(10CC), , `, 3,45,000, 15,000, 3,60,000, (60,000), (7,000), (75,000), (10,000), 2,08,000
Page 1032 :
Income Under The Head Salary, , 131, , Answer 3:, `, Basic Pay [(50,000 x 12), 6,00,000.00, Dearness Allowance (50,000 x 40% x 12), 2,40,000.00, Transport Allowance, 36,000.00, Motor Car Facility {Section 17(2)(iii) Rule 3(2)} [2,400 x 12], 28,800.00, Gross Salary, 9,04,800.00, Less: Standard Deduction u/s 16(ia), (50,000.00), Income under the head salary, 8,54,800.00, Gross Total Income, 8,54,800.00, Less: Deduction u/s 80D-Medical Insurance Premium, (15,200.00), Total Income, 8,39,600.00, Notes:, 1. Expenditure on accommodation in hotels while touring on official duties met by the employer is not, taxable., 2. Lunch provided by the employer during office hours is not taxable as per Section 17(2)(viii), Rule 3(7)(iii). It is assumed that expenditure per meal is upto `50., 3. Computer provided at the residence of Mr. X is not taxable as per section 17(2)(viii)Rule 3(7)(vii), , NOV – 2011, Solution 2:, Computation of Salary chargeable to tax of Mr. X for A.Y. 2022-23, Basic Salary, (50,000 x 7)+ (60,000 x 5), Dearness Allowance, (40% x 6,50,000), Bonus, (50,000 + 20,000), Contribution to recognized provident fund, (6,50,000 x 4%), Professional Tax paid by the employer, Facility of Laptop/computer, Perquisite value of use of motor car, (2,400 x 5), Leave Travel concession, Gross Salary, Less: Standard Deduction u/s 16(ia), Less: Deduction of professional tax u/s 16(iii), Income under the head Salary, Gross Total Income, Less: Deduction u/s 80C to 80U, Total Income, Computation of Tax Liability, Tax on `9,67,000 at slab rate, Add: HEC @ 4%, Tax Liability, Rounded off u/s 288B, Answer 3:, Computation of Taxable Income of Mrs. X for the A.Y. 2022-23, Computation of income under the head Salary, Basic salary, , `, 6,50,000.00, 2,60,000.00, 70,000.00, 26,000.00, 2,000.00, NIL, 12,000.00, NIL, 10,20,000.00, (50,000.00), (3,000.00), 9,67,000.00, 9,67,000.00, Nil, 9,67,000.00, 1,05,900.00, 4,236.00, 1,10,136.00, 1,10,140.00, `, 7,20,000
Page 1033 :
Income Under The Head Salary, (60,000 x 12), Dearness Allowance, (7,20,000 x 40%), Accommodation at concessional rent {Sec 17(2)(ii) Rule 3(1)}, Working Note:, 15% of rent free accommodation salary or rent paid whichever is less, Rent free accommodation salary = 7,20,000, 15% of `7,20,000 = `1,08,000, (assuming that dearness allowance does not form part of pay for, retirement benefits), Rent Paid = `15,000 x 12 = `1,80,000, Value of unfurnished house, 1,08,000, Less: Amount recovered from the employee (2,000 x 12), (24,000), Perquisite value of accommodation at concessional rent, 84,000, Perquisite of Interest on loan {Sec 17(2)(viii) Rule 3(7)(i)}, [(2,00,000 x 10%) – 10,000], Gross Salary, Less: Standard Deduction u/s 16(ia), Less: Professional Tax u/s 16(iii), Income under the head Salary, Computation of income under the head House Property, Gross Annual Value, (15,000 x 12), Less: Municipal Tax, Net Annual Value, Less: 30% of NAV u/s 24(a), Less: Interest on capital borrowed u/s 24(b), Loss under the head House Property, Gross Total Income, Less: Deduction u/s, 80C-Repayment of principal, Total Income, Computation of Tax Liability, Tax on `8,97,000 at Slab rate, Add: HEC @ 4%, Tax Liability, Rounded off u/s 288B, Answer 4:, Computation of Total Income, Basic Salary, (20,000 x 10), DA, (2,00,000 x 50%), Gift voucher, (6,000-5,000), Motor Car (Sec 17(2)(viii) Rule 3(7)(viii)), Working Note:, Cost, Less: Depreciation @ 20%, 01.07.2018-30.06.2019, 01.07.2019-30.06.2020, 01.07.2020-30.06.2021, , 132, 2,88,000, 84,000, , 10,000, 11,02,000, (50,000), (6,000), 10,46,000, 1,80,000, Nil, 1,80,000, (54,000), (1,75,000), (49,000), 9,97,000, (1,00,000), 8,97,000, 91,900, 3,676, 95,576, 95,580, `, 2,00,000, 1,00,000, 1,000, 56,000, , 5,00,000, (1,00,000), (80,000), (64,000)
Page 1034 :
Income Under The Head Salary, WDV, 2,56,000, Less: Amount Recovered, (2,00,000), Perquisite value of car, 56,000, Uncommuted Pension {Sec 17(1)}, (5,000 x 2), Commuted pension {Sec 10(10A)}, Working Note:, Amount received, 3,00,000, Less: exempted, (1,50,000), (3,00,000 x 3/2 x 1/3), Taxable, 1,50,000, Gratuity {Sec 10(10)}, Working Note:, Least of the following is exempt, 1. Gratuity received `6,00,000, 2. `20,00,000, 3. 15/26 x 30,000 x 30 = 5,19,231, Received = `6,00,000, Exempt = (`5,19,231), Taxable = ` 80,769, Leave Salary {Sec 10(10A)}, Working Note:, Least of the following is exempt, 1. `3,30,000, 2. `10 x 20,000 = `2,00,000, 3. `3,00,000, 3. 330/30 x 20,000 = 2,20,000, Received = `3,30,000, Exempt = (`2,00,000), Taxable = `1,30,000, Gross Salary, Less: Standard Deduction u/s 16(ia), Income under the head Salary, Income under the head Other Sources, (Since LCD is not covered under the definition of kind as given under section 56), Gross Total Income, Less: Deduction u/s 80C to 80U, Total Income, Rounded off u/s 288A, Computation of Tax Liability, Tax on `6,77,770 at slab rate, Add: HEC @ 4%, Tax Liability, Rounded off u/s 288B, , 133, , 10,000, 1,50,000, , 80,769, , 1,30,000, , 7,27,769, (50,000), 6,77,769, Nil, 6,77,769, Nil, 6,77,769, 6,77,770, 45,554.00, 1,822.16, 47,376.16, 47,380.00, , MAY – 2011, Answer 3., Taxability of perquisites provided by ABC Co. Ltd. to Mr. X, (i) Domestic servant was employed by the employee and the salary of such domestic servant was, paid/reimbursed by the employer. It is taxable as perquisite for all categories of employees., Taxable perquisite value = `1,500 × 12 = ` 18,000.
Page 1035 :
Income Under The Head Salary, , 134, , If the company had employed the domestic servant and the facility of such servant is given to the employee,, then the perquisite is taxable only in the case of specified employees. The value of the taxable perquisite in, such a case also would be `18,000., (ii) Where the educational institution is owned by the employer, the value of perquisite in respect of free, education facility shall be determined with reference to the reasonable cost of such education in a similar, institution in or near the locality., However, there would be no perquisite if the cost of such education per child does not exceed `1,000 per, month., Therefore, there would be no perquisite in respect of cost of free education provided to his child Mr. S, since, the cost does not exceed `1,000 per month., However, the cost of free education provided to his child Mr. A would be taxable, since the cost exceeds, `1,000 per month., Only the sum in excess of ` 1,000 per month is taxable. The value of perquisite would be `2,400., (iii) Where the employer has provided movable assets to the employee or any member of his household,, 10% per annum of the actual cost of such asset owned or the amount of hire charges incurred by the, employer shall be the value of perquisite. However, this will not apply to laptops and computers. In this, case, the movable assets are television, refrigerator and air conditioner and actual cost of such assets is, `1,10,000., The perquisite value would be 10% of the actual cost i.e., `11,000, being 10% of `1,10,000., (iv) Only the sum in excess of ` 5,000 is taxable in view of the language of Circular No.15/2001 dated, 12.12.2001 that such gifts upto ` 5,000 in the aggregate per annum would be exempt, beyond which it would, be taxed as a perquisite., Total value of taxable perquisite = ` 36,400 [i.e. `18,000 + 2,400 + 11,000 + 5,000]., , NOV – 2010, Answer 5., , Computation of taxable salary of Mr. X for A.Y. 2022-23, Particulars, `, Basic pay [(20,000×9) + (21,000×3)] = 1,80,000 + 63,000, 2,43,000, Dearness allowance [10% of basic pay], 24,300, Bonus [See Note (1) below], 21,000, Employer’s contribution to RPF in excess of 12%, (15%-12% =3% of `2,67,300), 8,019, Taxable allowances, Telephone allowance, 6,000, Taxable perquisites, Rent-free accommodation [See Note (2) below], 44,145, Reimbursement of salary of housekeeper, 12,000, Gift voucher (10,000 – 5,000), 5,000, Salary income chargeable to tax, 3,63,464, Less: Standard deduction u/s 16 (ia), (50,000), Income under the head Salary, 3,13,464, Notes:, (1) Bonus has been taken as one month’s basic pay as at the end of the year i.e. `21,000. In the alternative,, the problem can also be worked out by taking bonus as `20,000, being one month’s basic pay upto, 31.12.2021., (2) Where the accommodation is taken on lease or rent by the employer, the value of rent-free, accommodation provided to employee would be actual amount of lease rental paid or payable by the, employer or 15% of salary, whichever is lower., For the purposes of valuation of rent free house, salary includes:, (i) Basic salary i.e., `2,43,000, (ii) Dearness allowance i.e. `24,300, (iii) Bonus i.e., `21,000, (iv) Telephone allowance i.e., `6,000
Page 1036 :
Income Under The Head Salary, , 135, , Therefore, salary works out to, 2,43,000 + 24,300 + 21,000 + 6,000 = 2,94,300., 15% of salary = 2,94,300 × 15/100 = 44,145, Value of rent-free house = Lower of rent paid by the employer (i.e. `1,20,000) or 15% of salary (i.e.,, `44,145)., Therefore, the perquisite value is `44,145., (3) Facility of laptop is not a taxable perquisite., (4) Conveyance allowance is exempt since it is based on actual reimbursement for official purposes., (5) Premium of `5,000 paid by the company for personal accident policy is not liable to tax., (6) As per Circular No.15/2001, dated: 12.12.2001, Gift, voucher or token in lieu of gift - It is customary in, India, as it is in other parts of the world, to provide presents directly or indirectly in the form of vouchers or, tokens to employees on social and religious occasions like Diwali, Christmas, New Year, the anniversary of, the organization etc. Such gifts upto ` 5,000 in the aggregate per annum would be exempt, beyond which it, would be taxed as a perquisite. However, gifts made in cash or convertible into cash, like gift cheques etc., do not fall in the purview of this sub-rule., Answer 7., Computation of gratuity taxable in the hands of Mr. X for the P.Y. 2021-22, As per section 10(10), gratuity received by an employee would be exempt upto the least of the following, limits –, `, (i) Gratuity received = ` 8,00,000, (ii) Half-month’s salary for every year of completed service, = ½ x 26,700 x 30 = `4,00,500, (iii) Monetary limit = `20,00,000, Received, 8,00,000, Less: Exempt, (4,00,500), Taxable, 3,99,500, Note:, (1) One of the limits for calculation of gratuity exempt under section 10(10) is one-half-month’s salary for, each year of completed service (fraction of a year to be ignored), calculated on the basis of average salary, for the ten months immediately preceding the month of retirement. In this case, the month of retirement is, January, 2022. Therefore, average salary for the months of March 2021 to December 2021 have to be, considered. The salary is `25,000 p.m. upto 30.09.2021 and `27,000 p.m. from 01.10.2021. Hence, average, salary would be `26,700 [(`25,000 × 7) + (`27,000 × 3) + (2000× 55%×10)]/10., Further, half-month’s salary should be multiplied by the number of years of completed service and any, fraction of a year has to be ignored. Therefore, in this case, half-month’s salary should be multiplied by 30, and the fraction of 7 months should be ignored., (2) PS – The requirement of the question has not been specified. Having regard to the information given in, the question, the taxable gratuity has been computed., Answer 5., Computation of Total Income of Mr. X for the Assessment Year 2022-23, Particulars, Basic salary, Dearness allowance, Bonus, Employer contribution to RPF in excess of 12% is taxable, (3% of 18,00,000) [See Note below], Rent free accommodation @ 15% of `20 lakh (basic salary +, dearness allowance + bonus), Gross Salary, Less: Standard deduction u/s 16 (ia), Less: Deductions under Chapter VI-A, , `, , `, 12,00,000, 6,00,000, 2,00,000, 54,000, 3,00,000, 23,54,000, (50,000), 23,04,000
Page 1037 :
Income Under The Head Salary, Section 80C, Contribution to RPF, Section 80D – Health insurance premium, Family, 20,000, Parents (Senior Citizens), 28,000, Section 80DD, Medical treatment of dependent brother with severe disability, (flat deduction irrespective of expenditure incurred), Section 80E – Interest on loan taken for full-time education of, - his son studying B.Com., 24,000, - a student studying B.Sc. for whom he is the legal guardian 20,000, , 136, 1,50,000, 48,000, 1,25,000, , 44,000, Total income, , (3,67,000), 19,37,000, , Answer 6., Under section 10(14), any allowance granted to an employee working in a transport system to meet his, personal expenditure during his duty is exempt provided he is not in receipt of daily allowance. The, exemption is 70% of such allowance (i.e., `7,000 per month, being 70% of `10,000) or `10,000 per month,, whichever is less. Hence, `84,000 (i.e., 7,000 × 12) is allowable as deduction under section 10(14)., , Answer 6., Any amount withdrawn from public provident fund as per relevant rules is not exigible to tax. Such, exemption is provided in section 10(11)., Answer 6., Telephone provided at the residence of the employee and payment of bill by the employer is a tax free, perquisite as per section 17(2)(viii) Rule 3(7)(ix)., Answer 7., (i) As per section 17(2)(vi), the value of sweat equity shares chargeable to tax as perquisite shall be the fair, market value of such shares on the date on which the option is exercised by the assessee as reduced by the, amount actually paid by, or recovered from, the assessee in respect of such shares., Particulars, `, Fair market value of 1,000 sweat equity shares @ `300 each, 3,00,000, Less: Amount recovered from Mr. X 1,000 shares @ `200 each, (2,00,000), Value of perquisite of sweat equity shares allotted to Mr. X, 1,00,000, (ii) As per section 49(2AA), where capital gain arises from transfer of sweat equity shares, the cost of, acquisition of such shares shall be the fair market value which has been taken into account for perquisite, valuation under section 17(2)(vi)., Therefore, in case of subsequent sale of sweat equity shares by Mr. X, the cost of acquisition would be, `s3,00,000., , MAY – 2010, Answer 3. Perquisite value for housing loan, The value of the benefit to the assessee resulting from the provision of interest-free or concessional loan, made available to the employee or any member of his household during the relevant previous year by the, employer or any person on his behalf shall be determined as the sum equal to the interest computed at the, rate charged per annum by the State Bank of India (SBI) as on the 1st day of the relevant previous year in, respect of loans for the same purpose advanced by it. This rate should be applied on the maximum, outstanding monthly balance and the resulting amount should be reduced by the interest, if any, actually paid, by him.
Page 1038 :
Income Under The Head Salary, , 137, , “Maximum outstanding monthly balance” means the aggregate outstanding balance for loan as on the last, day of each month., The perquisite value for computation is 10.50% - 6% = 4.5%, Month, Maximum outstanding balance as on, Perquisite value at 4.5%, last date of month, for the month, April, 2021, 5,88,000, 5,88,000 x 4.5% x 1/12 = 2,205, May, 2021, 5,76,000, 5,76,000 x 4.5% x 1/12 = 2,160, June, 2021, 5,64,000, 5,64,000 x 4.5% x 1/12 = 2,115, July, 2021, 5,52,000, 5,52,000 x 4.5% x 1/12 = 2,070, August, 2021, 5,40,000, 5,40,000 x 4.5% x 1/12 = 2,025, September, 2021, 5,28,000, 5,28,000 x 4.5% x 1/12 = 1,980, October, 2021, 5,16,000, 5,16,000 x 4.5% x 1/12 = 1,935, November, 2021, 5,04,000, 5,04,000 x 4.5% x 1/12 = 1,890, December, 2021, 4,92,000, 4,92,000 x 4.5% x 1/12 = 1,845, January, 2022, 4,80,000, 4,80,000 x 4.5% x 1/12 = 1,800, February, 2022, 4,68,000, 4,68,000 x 4.5% x 1/12 = 1,755, March, 2022, 4,56,000, 4,56,000 x 4.5% x 1/12 = 1,710, 23,490, Total value of this perquisite, Perquisite Value of Air Conditioners:, `, Original cost, 2,00,000, Depreciation on SLM basis for 4 years @10% i.e. `2,00,000 x 10% x 4, 80,000, Written down value, 1,20,000, Amount recovered from the employee, 90,000, Perquisite value, 30,000, Chargeable perquisite in the hands of Mr. X for the assessment year 2022-23, `, Housing loan, 23,490, Air Conditioner, 30,000, Total, 53,490, Note: It is assumed that payment of `12,000 is excluding interest., , NOV – 2009, Answer 1: Computation of Total Income and Tax Liability of Mr. X for the A.Y. 2022-23, `, Salary Income, Salary including dearness allowance, Bonus, Value of perquisites:, (i) Salary of servant, 12,000, (ii) Car (1,800 x 12), 21,600, (iii) Free gas, electricity and water, 11,000, Less: Standard deduction u/s 16 (ia), Income under the head salary, Income from house property, Gross Annual Value (GAV) (Rent receivable is taken as GAV in the, absence of other information) (20,000 × 12), Less: Municipal taxes paid [See Note (ii)(a)], Net Annual Value (NAV), Less: Deductions under section 24, (i) 30% of NAV, 70,710, (ii) Interest on loan from LIC @ 15% of 1,60,000, 24,000, [See Note (ii)(b)], Income from speculative business, , `, 3,35,000, 11,000, , 44,600, 3,90,600, (50,000), 3,40,600, , 2,40,000, (4,300), 2,35,700, (94,710), , 1,40,990
Page 1039 :
Income Under The Head Salary, Income from share speculation business, Less: Loss from cotton speculation business, Net Loss, Net loss from speculative business has to be carried forward as it, cannot be set off against any other head of income., Income from Other Sources, (i) Income on account of interest earned from advancing money, gifted to his minor son is includible in the hands of Mr. X as, per section 64(1A), Less: Exempt under section 10(32), (ii) Interest income earned from advancing money gifted to wife has, to be clubbed with the income of the assessee as per section 64(1), (iii) Gift received from four friends (taxable under section 56(2) as the, aggregate amount received during the year exceeds `50,000), , 138, 2,700, (4,200), 1,500, , 3,800, (1,500), 2,300, 5,700, 1,00,000, 1,08,000, 5,89,590, , Gross Total Income, Less: Deduction under section 80C, Contribution to Public Provident Fund, 5,600, Unit Linked Insurance Plan, 4,000, (9,600), Total Income, 5,79,990, Particulars, `, Tax on total income, 28,498.00, Add: HEC @ 4%, 1,139.92, Tax Liability, 29,637.92, Rounded off u/s 288B, 29,640.00, Notes:, (i) National Award for humanitarian work given by the Central Government is exempt under section, 10(17A) of the Income-tax Act, 1961., (ii) The following assumptions have been made while computing income under the head “Income from, house property” –, (a) It is the owner, namely, Mr. X, who has paid the municipal taxes;, (b) The entire loan of `1,60,000 is outstanding as on 31.03.2022; and, (c) Society charges of `1,900 p.a. is not included in the rent of `20,000 p.m. payable by the tenant., Such charges have either been paid directly by Mr. X or recovered separately from the tenant., , NOV – 2008, Answer 3., , Computation of Taxable Salary of Mr. X for the Assessment Year 2022-23, Particulars, Amount, Amount, (`), (`), Basic Salary (`20,000 x 5) + (`25,000 x 7), 2,75,000.00, Transport allowance ( `2,000 x 12), 24,000.00, Children education allowance (`500 x 12), 6,000, Less: Exempt under section 10(14) ( `100 x 2 x 12), (2,400), 3,600.00, City Compensatory Allowance (`300 x 12), 3,600.00, Hostel Expenses Allowance (`380 x 12), 4,560, Less: Exempt under section 10(14) ( `300 x 2 x 12 i.e. `7,200, but restricted to the actual allowance of `4,560), (4,560), Nil, Tiffin allowance (fully taxable), 5,000.00, Tax paid on employment, 2,500.00, Employer’s contribution to Recognised Provident Fund in excess of 12% of salary, (i.e. 3% of `2,75,000), 8,250.00, Gross Salary, 3,21,950.00
Page 1040 :
Income Under The Head Salary, , 139, , Less: Standard deduction u/s 16 (ia), (50,000.00), Less : Tax on employment under section 16(iii), (2,500.00), Taxable salary, 2,69,450.00, Computation of Total Income, Gross Total Income, 2,69,450.00, Less: Deduction u/s 80C, Employee’s contribution in Recognised Provident Fund, (41,250.00), Total Income, 2,28,200.00, Computation of Tax Liability, Tax on `2,28,200 at slab rate, Nil, Tax Liability, Nil, Notes:, Professional tax paid by employer should be included in the salary of Mr. X as a perquisite since it is, discharge of monetary obligation of the employee by the employer. Thereafter, deduction of professional tax, paid is allowed to the employee from his gross salary.